76
Study Material of 29/6/2020 For Class Nursery to XII Class Nursery Subject Topic Execution English Practice Book – Pg: 22 Do in the book Math Practice Book Pg: 46 Do in the book Bengali সসসসসসস সসসসসস – Pg ২৬ সসস সসসসস সসসস সসস সসসসস সসস সসস Do in the book Hindi Letter স – 4 Lines Write in the copy Class KG I Subject Topic Execution English Reading Book – Pg 39 Rhyme: Ten little fingers Listen and repeat Math Before, after Numbers Worksheet Bengali সসসসসসস সসসসসস Pg ১২ সসস সসসস সসসস Read and Learn Hindi সসসসসস (সসস সসস Pg 5) Check WhatsApp for Audio – listen, repeat and learn Computer Pg: 18, 19 Look at the pictures, then listen and understand. Class KG II Subject Topic Execution English Cursive writing Book Pg 25 Write in the book Math Numbers names (Revision) Write the numbers names of 50– 80 in the copy Science(Ter m I) Pg 128 (Peacock) Our national Bird is peacock. Join the dots and colour the picture of PEACOCK Computer Chapter – 4: Use of Computer Look at the pictures then listen and understand (try to read) Art/Craft Pg 8 Do in the book Class I Subjec t Topic Summary Execution Englis h 1 Sounds of animals Hens –cackle Horses –neigh Lions –roar Owls –hoots Snake –hiss Englis h 2 Mother’s love Done Page 31 A. Encircle the correct answer 1. Every Mouse and bumble bee loves to feel her

 · Web viewSubject . Topic . Summary . Execution . English 1 . Sounds of animals . Hens –cackle Horses –neigh Lions –roar Owls –hoots Snake –hiss. English 2 . Mother’s

  • Upload
    others

  • View
    4

  • Download
    0

Embed Size (px)

Citation preview

Page 1:  · Web viewSubject . Topic . Summary . Execution . English 1 . Sounds of animals . Hens –cackle Horses –neigh Lions –roar Owls –hoots Snake –hiss. English 2 . Mother’s

Study Material of 2962020For Class Nursery to XII

Class NurserySubject Topic ExecutionEnglish Practice Book ndash Pg 22 Do in the bookMath Practice Book Pg 46 Do in the book

Bengali সাহিতয কহিঃ ndash Pg ২৬ বাম হিকেকর সাকে ডান হিকেকর হিমল ককেরা

Do in the book

Hindi Letter ञ ndash 4 Lines Write in the copyClass KG I

Subject Topic ExecutionEnglish Reading Book ndash Pg 39

Rhyme Ten little fingers Listen and repeat

Math Before after Numbers WorksheetBengali সাহিতয মকলঃ Pg ১২ ঈ কার

যাকে শবদ Read and Learn

Hindi डाकिया (बाल गीत Pg 5) Check WhatsApp for Audio ndash listen repeat and learn

Computer Pg 18 19 Look at the pictures then listen and understand

Class KG IISubject Topic ExecutionEnglish Cursive writing Book Pg 25 Write in the bookMath Numbers names (Revision) Write the numbers names of 50ndash80 in the copy

Science(Term I) Pg 128 (Peacock) Our national Bird is peacockJoin the dots and colour the picture of PEACOCK

Computer Chapter ndash 4 Use of Computer Look at the pictures then listen and understand (try to read)

ArtCraft Pg 8 Do in the book

Class ISubject Topic Summary Execution English 1 Sounds of animals Hens ndashcackle Horses ndashneigh Lions ndashroar Owls ndashhoots Snake ndashhissEnglish 2 Motherrsquos love Done Page 31

A Encircle the correct answer1 Every Mouse and bumble bee loves to feel her motherrsquos love2Every bear loves his mother more than Honey3 The poet loves her mother just as the forest creatures do

Maths Chapter 5 Done Do no 5 and 6EVS Chapter 9 Done Page 141

1I am rude to animals-kind2 I throw garbage in the open-dustbin3 I have bad study habits-good

Hindi Chapter 8 ऋ ी मातरा स शबद वकष किवकष गह अमत घणा पापथवीBengali বইndashবাংলা

সাহিতয পহিরচয়৩ সঠিক উততর যবকেনাও -ক) সব যাকাকেনর সামকেন ( ফাকা হি- )

পাঠndash২৩লপndashযমলার মজাঅনশীলনীর পরকে4াততর

খ) রকের হিন সবাই( পাপ যবগহিন ) যখকেত চায়) ( কাকেঠর কাকেজর যতকেল -াজার ) যাকাকেনর সামকেনই যবহিশ হি-ঘ) নারকোলা ( পাচবার শবার ) যঘারার পর যকেম যকেলাঙ) ( -াই যবান ) নার যালা চার বায়না ধরকেলা৪ হিবপরীত শবদ -কের timesকাকে যবাকা timesচালাক উপকেরtimes হিনকেচ রাহিজ timesঅরাহিজ য-তকের timesবাইকের৫ একেলাকেমকেলা শবদ সাহিজকেয় যলকেখা -যল-াকেতজা = যতকেল-াজা যয়ানআজ = আকেয়াজন কাকোন =

যাকান হিলহিজহিপ = হিজহিলহিপ যানারলা = নারকোলা কমারহির = রকমাহির

Class IISubject Topic Summary Execution Eng literature

Ch 2 FriendshipBook Orchid

Refer to pg 15Once there lived a crow a tortoise and a mouse They were very good friends and lived in the forest Once they saw a deer running and coming towards them They understood that the deer was afraid of the hunters so the tortoise told him that the hunters donrsquot come inside this deep forest They wanted the deer to be their friend and they four became good friends Once when they didnrsquot find the deer anywhere they became worried and started looking for him Suddenly the crow saw the deer trapped in the net near river bank and he at once went to inform his other friends

Read pg 15

Eng language

Ch 13 Adverbs Adverbs are words that tells us something more about the verb That is how an action happenedWe add ly to form adverbs For eg A tortoise walks slowly[The word walks is a verb and the word slowly is an adverb which is telling how the tortoise is walking]

Read pg 61

EVS Ch 8 Safety First Refer to pg 139Safety in swimming pool and playgroundChildren the things that we should keep in mind while we are in the swimming pool are that as you are very small never go alone near ituse swimming tube no running near the pool as you might slip and fall into the water wait for your turn and never push each other in the pool If you follow these small safety rules then you can avoid many accidents to happen

Read pg 139 and try to understand

Mathematics

CHAP-6SHAPES

SOLID SHAPES ARE CALLED 3-D SHAPESSOLID SHAPES INCLUDES FACESEDGES AND CORNERS

READ AND UNDERSTAND THE SUMMARY PART

FACE- PART OF THE SHAPE THAT IS FLAT (OR CURVED) IS KNOWN AS FACE OF THE SOLID SHAPE A CUBE HAS 6 FACES

EDGE- THE LINE WHERE TWO FACES MEET IS KNOWN AS EDGEA CUBE HAS 12 EDGES

VERTEX-THE POINT WHERE TWO OR MORE EDGES MEET IS KNOWN AS VERTEXA CUBE HAS 8 VERTICES

General knowledge

Ch 21 part of plants

Refer to pg 25Do in the book and learn [ practise drawing a plant and label the different parts of the plant]

Qs 1 Which part of the plant grow under the groundAns Roots Qs2 Which part of the plant looks colourfulAns FlowerQs 3 Which part of the plant look greenAns Leaves Qs 4 Which part of the plant has leaves flowers and fruitsAns StemQs 5 Which part of a plant do you eatAns Fruit

Bengali বইndashবাংলা সাহিতয পহিরচয়

পাঠndash৭লপndashবকোহিনঅনশীলনীর পরকে4াততর

৩ সঠিক উততর যবকেনাও -ক) ( হিসটানকের বাঙাহিলকের ) সবকেচকেয় বকো উৎসব বহিনখ) এই হিনটি ( জরকেতর হিজশহিকেসটর জনমহিন) এইহিনটি পরম ( আনকেFর ঃকেখর ) হিনঘ) হিজGায় ( বাইকেবল যকারান ) পাঠকরায়ঙ) বহিকেনর হিপরয় খাবার কেলা ( যকক হিখচহি ) ৪ হিবপরীতশবদ -বকো timesযাকেIা জনম timesমতয আনFtimes ঃখ নতন timesপকেরাকেনা হিপরয় timesঅহিপরয় হিশশ times বদধপহিবতর times অপহিবতর আকেলা times অনধকার সমাহিNtimes সচনা৫ একেলাকেমকেলা শবদ সাহিজকেয় যলকেখা -যাহিবন = বকোহিন ইকেববাল = বাইকেবল নমজনহি = জনমহিন পারউ = উপার হিনয়মহিব = হিবহিনময় পাসউনা = উপাসনা

Hindi सवनाम अपन लिलए म हममरा हमारा मझ आदिद शबदो ा परयोग रत ह जि0सस बात र रह हउस लिलएmdashतम तमहारा तमह आप आपा आदिद शबदो ा परयोग रत ह जि0स बार म बात र रह ह या किसी अनय लिलएmdashवहव उस उना उनह उनहोनआदिद शबदो ा परयोग किया 0ाता ह ौन ोईकया आदिद शबद भी सवनाम ह

Read the chapter

Class IIISubject Topic Summary Execution COMPUTER

DRAWING IN MS PAINT

Q3) WHAT TOOL DO YOU USE TO INSERT TEXT IN MS PAINTANS) THE TEXT TOOL IS USED TO ADD TEXT IN MS PAINTQ4) WRITE THE STEPS TO USE THE ERASER TOOLANS) THE STEPS ARE

CLICK ON THE ERASER TOOL IN THE TOOLS GROUP DRAG THE MOUSE POINTER OVER THE AREA YOU WANT TO ERASE

বই ndash বাংলা সাহিতয পহিরচয়

পাঠ ndash ৮লপ ndash IনIহিন ও রাজার কাযলখক ndash উকেপনদরহিককেশার রায়কেচৌধরীঅনশীলনীর পরকে4াততর

৯ বাকয রচনা ককেরা -জবদ ndash সবাই হিমকেল যচারকেক পহিলকেশর াকেত তকেল হিকেয় জবদ করকেলা হিখলহিখল ndash নতন যখলনা যপকেয় হিশশটি হিখলহিখল ককের যকেস উঠকেলা কাতর ndash কাতর ককেS বদধ যলাকটি জল চাইহিল IফI ndash আঘাত লাায় ককরIা IফI করহিল -য়ানক ndash যকারনাকে -য়ানক পরলয় কেয়হিল

Hindi सवनाम अपन लिलए म हममरा हमारा मझ आदिद शबदो ा परयोग रत ह जि0सस बात र रह हउस लिलएmdashतम तमहारा तमह आप आपा आदिद शबदो ा परयोग रत ह mdashजि0स बार म बात र रह ह या किसी अनय लिलए वह व उस उना उनह उनहोन आदिद

शबदो ा परयोग किया 0ाता ह ौन ोई कया आदिद शबद भी सवनाम ह Science

Ch ndash Food we get from Plants

We see many plants around us They are of different shapes and sizes Some plants grow tall some stay short Some plants grow along the ground some plants need support to grow

Trees ndash big and tall plants are called trees Trees have thick hard and woody stem called trunk They have thin stems called branches Most trees live for many years A banyan tree can live for hundreds of years Mango banyan neem are examples of trees

Fill in the blanks (pg no- 63)

The main stem of a tree is called trunk

Cotton and hibiscus are examples of shrubs

Rice and mint are examples of herbs

Social studies

Indian literature Jataka talesThe jataka tales are short educational tales They have stories concerning the previous births of Gautam Buddha in both animal and human form These moral tales were written in fourth century and are based on the life of Gautama Buddha These are a collection of about 550 fables Every fable exhibits some virtue where

Buddha may appear in it as a king God or an elephant There are several translations of these Jataka tales in different languages

Read the portion which has been explainedEng language

Ch 19 Adverbs Adverbs are words that tells us more about an action In other words adding something more to the verb For eg The cat ran quicklyhere the word ran is the verb as it is showing an action done by the cat and quickly is the adverb as it is telling how the cat ran

Refer to pg 68Do in the bookWarm upUnderline the words that tell you how an action happened

The cat ran quicklyThe man ran swiftlyIt hurriedly went into its hole The mouse sat safely in its houseThe cat squeaked loudly

MAT

HEM

ATIC

S

Ch 7

Frac

tions

A fraction is a number that stands for parts of a whole object or a collection of objects

Each fraction has two numbers separated by a rule(a) The number above the rule is called numerator(b) The number below the rule is called denominator

Example 7minusminusminusminusrarr Numerator

13minusminusminusminusrarr Denominator

Notes

(1) 05 is a fraction in which numerator is 0

(2) There is no fraction whose denominator is zero

(3) When the numerator and denominator are equal the fraction represents the whole or 1

Example 33=1and2

2 = 1

Exercise ndash 421 Read the following fractional numbers

(c) 411 (d)

815 (e)

2337

Solution

(c) 411

four by eleven

(d)8

15Eight by fifteen

(e) 2337

twenty three by thirty seven

2 Write the following fractional numbers(d) Three over eleven (e) Eighteen by twenty one (f) Four-seventhsSolution

(d) Three over eleven 311

(e) Eighteen by twenty one 1821

(f) Four-sevenths 47

3 Write the numerator of each of the following

(e) 09 (f)

56 (g)

311 (h)

617

Solution (e) 0 (f) 5 (g) 3 (h) 64 Write the denominator of each of the following

(e) 08 (f)

813 (g)

719 (h)

417

Solution (e) 8 (f) 13 (g) 19 (h) 17

5 Write the fractions whose numerators and denominators are given below

(a) Numerator = 4 and Denominator = 13 (b) Numerator = 6 and Denominator = 17(c) Numerator = 11 and Denominator = 16(d) Numerator = 0 and Denominator = 9

Numerator Denominator

Fraction

(a) 4 13 413

(b) 6 17 617

(c) 11 16 1116

(d) 0 9 09

6 Find the fractions in which the denominator is 3 more than the numerator69

74

811

49

1113

711

1411

1613

2023

Solution 69

811

2023

8 A book seller had 15 books He sold 8 books What fraction of the books did he sell

Solution Out of 15 books the book seller sold 8 books

there4 The fraction of the books he sold =8

15

10 A class has 35 students On Monday 34 students were present What fraction of the students was absent

Solution Total number of student = 35On Monday 34 students were presentthere4 The number of student absent on Monday = (35 ndash 34) = 1

So fraction of the students was = 135

Class IVSubject Topic Summary Execution English language

Verbs the -ing form

Meaning of verb A word used to describe an action is known as verb

The ndash ing form of a verb is a very important form for it is used in many different waysExampleJack is playing(Present continuous)Jack was playing with his friends yesterday ( Past continuous)Jack will be playing in a match tomorrow ( Future continuous)So we see that the ndash ing form of verb used to form the continuous tenses

The ndash ing form of verb can also be used as an adjectiveExampleYou should not disturb a sleeping dogThe kettle is full of boiling water

Say which of the ndash ingforms of verbs in the sentences given below have been used as adjectives and which to form continuous tenses ( solved exercises please follow this)

3Interesting ndash adjective

4Was eating ndash past continuous tense

5terrifying ndashadjective

6are helping is spreading ndash present continuous tense

7will be visiting ndashfuture continuous tense

8twittering ndash adjective

Social studies

Map reading A Map helps us to see the whole World continents countries cities and neighborhood They are of different sizes handy and can be rolled up folded or carried easily However the maps do not show the surface accurately This is because the map is flat whereas the Earth is round in shape Cartography or a map making is a study and practice representation of the

One word answers1 It helps us to see the whole world countries and cities ndash Map

2 A study and practice representation of the Earth on a flat surface ndash Cartography

Earth on a flat surface

Elements of a mapTo be able to read a map we must understand the elements of a map

DirectionsDirections are the basic guidelines that help us to locate places It is very important for us to know the correct directions East West North and South are the four Cardinal directions These directions help us to locate the places Beside these there are four sub directionsndash Northeast Southeast North West and South West Compass is an instrument that helps to locate the directions

3 The basic guidelines to help us to locate places ndash directions

4These four directions help us to locate the places ndash Cardinal directions

COMPUTER

Formatting in ms word

Q3) What do you mean by formattingAns) Formatting is a general arrangement of text in a document We can change the appearance of a document by using the features available in ms word We can use different fonts colours and styles in the textQ4) What is alignmentAns) Alignment of text is the way in which it is placed between the margins of a page Text can be aligned to the left side in the centre or to the right side of a pageQ5) what is fontAns) A font is a style of writing and typing A font provides specific textual appearance to the document You can change the size style or give various effects to a font

Hindi 2ndlang

ाला किहरण सभी धमlt ा मल धम ह दया और रणा मरी हर 0ीव अपन ही दकिनया म रहत ह हम

पराणी कयो उनो परशान रत ह ऐसा हम नही रना चाकिहए

0ानवरो स हम ए सीख मिमलती ह किस तरह हम एता म रहना चाकिहए

हर पराणी पयार ी भाा समझता ह अगर हम पयार ी उममीद रत ह तो कया 0त

0ानवर हमस उममीद नही रत कि हम भी उनह पयार द यह हानी ए ाल किहरण ी ह 0ो अपन

समह ा नततव रता था हमशा कया उलिचत ह यह सोचता था अचान

उन पर लिशारिरयो न हमला बोल दिदया जि0सम ाला किहरण परा गया कयोकि वह

दलभ ह और लिशारिरयो ी न0र उसी पर थी परत 0ानवरो ी एता ो दखर

रा0मार ो दया आ गई और उसन उनो छोड दिदया 0ानवर भी बईमान नही थी वह उनस अकसर मिमलन उन बाद म 0ाता रहता रा0ा ो धनयवाद रता अबोध म

पराणिणयो म एता तजञता अभी भी ह

शबदाथ तराई ndash पहाड आसपास नीच

ी भमिमवश- ल या परिरवारअदवभत- अनोखाचौननाndash सत सावधानओझलndash गायबकिवपणिRndash मसीबतसगठन- एताउममीदndash आशाातर दमिU ndashकिववशता

हिहस ndash हानी पहचान वालातवयndash फ0

दल ndash समहनततवndash सचालन रना

ओट ndash आडआतमसमपणndash अपन ो सौप दनाम- चप

हम उनस सीख लनी चाकिहएবইndashবাংলা সাহিতয পহিরচয়

পাঠndash১০লপndashহিবকেবকানকেFর যকেলকেবলাযলখকndashশশী-ষণ াশগNঅনশীলনীর পরকে4াততর

৯প পহিরবতG ন ককেরা -শরীর = শারীহিরক -ত = য-ৌহিতক সG ার = সG াহির সথায়ী = সথাহিয়তবহিবশবাস = হিবশবাসী া = যকো১০ বাকয রচনা ককেরা -আডডা = পাার বদধরা াতলায় আডডা যয়হিডবাহিজ = রাম া যকেক হিডবাহিজ যখকেয় পকেলাসG ার = বকেলহিল যকেল যর সG ারবকহিন = অঙক -ল করায় রীনা মাকেয়র কাকে বকহিন যখকেলাবহিদধ = বহিদধ াককেল উপায় য়হিনশহিত = হিনশহিত রাকেত রাজবাহিকেত ডাকাত পকেলা-য়ানক = পাাহি রাসতা বষটির পকের -য়ানক হিবপ জনক য়হিবশবাস = মানকেষর পরহিত হিবশবাস ারাকেনা পাপ

Science Ch ndash The Food We Eat

Living things need food to live to grow to stay strong and fit When we need food we feel hungry Food gives us energy to do work It also protects us from diseases and helps us to stay healthy Nutrients in food The food we eat contains many substances that are necessary for our body These substances are called nutrients that help us to grow well and stay healthy

Nutrients give us energy to study work and play

They help our body to grow and repair the damaged parts of our body

They also help our body to fight against diseases and remain healthy

Write T for True or F for False (pg no- 11)

1 Food contains nutrients that help us to grow True

2 Foods rich in carbohydrates are called body-building foods False

MAT

HEM

ATIC

S

Ch 9

Com

mon

Fra

ction

s

A fraction is a number that stands for parts of a whole object or a collection of objects

Each fraction has two numbers One is written above the other separated by a line The one above the line is called numerator and the one below the line is called denominator

Example 5minusminusminusminusrarr Numerator

11minusminusminusminusrarr Denominator

Exercise ndash 362 In following fill in the blanks

(b) 37 ___ is denominator ___ is

numerator

(c) 89 ___ is denominator ___ is

numerator

Solution

(b) 37 7 is denominator 3 is numerator

(c) 89 9 is denominator 8 is numerator

3 Write down the fractions whose numerators and denominators are given below in the bracket The first number stands for numerator and the second number standing for denominator

(25) 25

(311) 311

(416) 416

(712) 712

Class VSubject Topic Summary Execution

Science Ch ndash PlantReproduction

In pollination chapter we have learnt that the flowers change into fruits and the fruits bear seeds Now we are going to learn how this process takes place

FertilizationThe process of fusion of the male reproductive cell (male gamete) and female reproductive cell (female gamete) is known as fertilizationWhen a pollen grain reaches from the anther to a stigma it begins to grow and forms apollentube The Pollen tube then travels down through the style to enter an ovule inside the ovary On reaching the ovule male reproductive cell in the pollen grain unites with the egg cell present in the ovule

Books exercise

A) Tick the correct answer

1Which of the following do ovules change into after fertilization ndashseeds

2Which of the following is not a part of the seed ndash flower

3Which of the following condition is needed for germination of a seed ndash all ofthese

English language

Sentences phrases and

Solved exercisesSay which of the underlined groups of words are phrases and which clauses

clauses8 In her new clothes ndashphrasesAs pretty as a doll ndash phrases

9 looking sad and upset ndash phrasesHe had lost all the tickets for the Test Match ndash clauses

10 During the vacation now only a month away ndashphrases

11 too sweet and too hot ndashphrases

12 At the Olympic Games ndashPhrasesOf Laurel leaves ndash phrases

13 Who played the role of Hamlet ndashclauses

14 However fast ndash phrases

15 When the men fell asleep ndash clausesSocial studies

Indian Government

Lok Sabha (lower house) ndash It has 552 members Of these 530 membersrepresent States 20 members represent the union territories and two members represent the Anglo- Indian community All except the representatives of the Anglo-Indian community are elected by Indian citizens A person above the age of 25 can contest in the elections for Lok Sabha One term of Lok Sabha is for 5 yearsRajya Sabha (upper house) ndash Its members are elected by the MLAs or members of the legislative Assembly There are 250 members in the Rajya Sabha of which 12 are nominated by the President One term of Rajya Sabha is for 6 years Anyone above the age of 30 can be elected as a member of Rajya Sabha

ExecutivePresidentThe President is the head of the country in India He is elected by the MPs and the MLAs for a tenure of 5 years He appoints the Prime Minister and the Council of Ministers Prime MinisterThe party which wins the election forms the government and its leader is elected as the Prime Minister He is the chief advisor to the President The Council of Ministers assists the Prime Minister and is accountable for their roles For example the Education minister is responsible for the education system in our country

1 How many members are there in Lok SabhaAns 552 members

2 What is the term for Lok SabhaAns 5 years

3 How many members are there in the Rajya SabhaAns 250 members

4 Who is the head of our countryAns President

5 Who is the chief advisor to the PresidentAns Prime Minister

Book ndash GK

Ch ndash 1First in space

1 First living being into space in 1957 Ans Laika

Times 2 First person to go into space in 1961 Ans Yuri Gagarin

3 First woman to go into space in 1963 Ans ValentinaTereshkova4 First person ever to walk in space in 1965Ans alexei Leonov5 First person to land on the moon in 1959 Ans Neil Armstrong6 First Indian to go into space in 1984 Ans Rakesh Sharma 7 First Indian woman to go into space in 19978 Ans Kalpana Chawla9 First woman tourist in space in 2006

Ans Anusheh AnsariCOMPUTER

ALGORITHM AND FLOWCHART

Q) DRAW THE SYMBOLS USED IN A FLOWCHART WITH THEIR DESCRIPTIONS(IN EXAM IT CAN COME AS SHORT QUESTIONS ASKING INDIVIDUAL SYMBOLS FUNCTION)ANS)

MAT

HEM

ATIC

S

Ch 6

Com

mon

Fra

ction

s

Multiplication of FractionsA Multiply a fractional number by whole numberTo multiply a fractional number by whole number we multiply the numerator of the fractional number by the whole number and denominator of the fractional number by 1 The first product thus obtained is the numerator and the second product is the denominator of the required product

Exercise ndash 30Multiply

7 2027

times 9

Solution 2027

times 9 = 203 = 6

23

8 611

times11

Solution 611

times11 = 6

15 71

20times16

Solution 71

20times16 =

14120

times16

= 1415

times 4 = 141times 4

5 = 564

5 = 11245

B Multiplication of a fractional number by a fractional number To multiply a fractional number by a fractional number we multiply the numerator of the first fractional number by the numerator of the second fractional number and the denominator of the first fractional number by the denominator of the second fractional number The first product thus obtained is the numerator and the second product is the denominator of the required product

16 2712

times24

Solution 2712

times24 = 3112

times24

= 31times2 = 62

Exercise ndash 31

11 83

times 34

2

Solution 83

times 34 = 2

14 723

times2 25

4

Solution 723

times2 25 =

233

times 125 =

23times 45

= 925 = 18

25

15 1212

times1 13

2

Solution 1212

times1 13 =

252

times 43 =

25times 23

= 503 = 16

23

State the following statements are true or false

17 1912

times 239 = 1

Solution LHS = 1912

times 239

= 392

times 239 = 1 = RHS

[LHS = Left hand side amp RHS = Right hand side]

there4 1912

times 239 = 1 [True]

21 213

times2 13 = 4

19

Solution LHS = 213

times2 13 =

73

times 73

= 7times73times3 =

499 = 5

49

there4 LHS ne RHS

So 213

times2 13 = 4

19 [False]

23 23

times 45 =

2times 5+3 times43times 5

Solution

LHS = 23

times 45 =

2times 43 times5 =

815 again

RHS = 2times 5+3 times4

3times 5 = 10+12

15 = 2215

there4 LHS ne RHS So 23

times 45 =

2times 5+3times43times 5

[False]

25 23 of

13 =

29

Solution

LHS= 23 of

13 =

23 times

13 =

29 = RHS]

there4 23 of

13 =

29 [True]

Practice at HomeExercise ndash 31State the following statements are true or false

24 12 of 4 =

18

Class VISubject Topic Summary Execution

HISTORY AND CIVICS

Chapter 5The Mauryan Empire

DECLINE OF MAURYAN EMPIREDecline of Mauryan empire started after the death of Ashoka at around 232 BCThere are several reasons for break up of the empire1 Weak successor Emperors after Ashoka were

capable of handling vast and mighty Mauryan empire In 185BC the last Mauryan ruler Bri-hadrath was murdered by his Commander-in-Chief Pushyamitra Sunga

2 Provincial Revolts Due to weak central author-ity provincial chiefs of Kalinga and southern provinces revolted against emperor and freed themselves from Mauryan empire

3 Weakness of Economy Prosperity of Mauryan was based on solid economic activities which

ExercisesI Multiple choice questions-1 Chandragupta defeated Seleucus in the year ndashc) 305 BC2 Who killed the last Mauryan ruler Brihadrath b) Pushyamitra3 Which of the following was not a reason for the decline of the Mauryan empirec) Chandraguptarsquos weakness4 Ashoka invaded Kalinga in the year c) 261 BC

II Fill in the blanks1Chandragupta ascended the throne in 324

was taken care by early monarchs Later kings had neither ability nor interest in economic af-fairs That led to failure in tax collection As a result they failed to maintain a large army that were essential to keep empire intact

4 Greek Invasion Greeks freed north-western provinces from weak Mauryan monarchs and reestablished their authority

5 Ashokarsquos Policy some scholar opined that after Kalinga war Ashoka embraced Buddhism re-nounced the policy of war and disbanded the Army But this is partially true as there is no proper evidence of disbanding the army

Based on above points we can conclude that main reason for decline of Mauryan empire is weakness of Ashokarsquos successors Kunal Samprati Dasharath Salisuk all were weak kingsAt last in 185 BCPushyamitra Sunga killed king Brihadrath and established the Sunga dynasty

BC2 Bindusara was the son of Chandragupta and father of Ashoka

3 Pataliputra was administered by City Magistrate committess of 5 members each4 The Greek General Seleucus sent his ambassador Megasthenes to Chandraguptarsquos court5 Ashoka sent his son Prince Mahendra and daughter Sanghamitra to spread his Dhamma6 The Indian Rebublic has adopted the Lion Capital of Saranath Pillar as its national emblem 7 Pushyamitra killed the last Mauryan ruler Brihadrath and founded the Sunga dynasty

III Name the following

1The author of Arthashastra-Kautilya2 The ruler who founded the Mauryan dynasty-Chandragupta3 The author of Indika-Megasthenes 4 The officers who were appointed by Ashoka to spread Dhamma-Dhamma Mahamatras5 The general of Alexander whom Chandragupta defeated-Seleucus

V Match the columns1 Kautilya (c)2 Megasthenes (d)3 Pushyamitra (e)4 Brihadrath (b)5 Bindusara (a)

BENGALI(2ND

LANGUAGE)

পশপাহিখর -াষাসহিবনয় রায়কেচৌধরী

যলখক পহিরহিচহিত- পরখযাত সাহিহিতযক উকেপনদরহিককেশার রায়কেচৌধরীর পতর সহিবনয় রায়কেচৌধরী lsquoসকেFশrsquo পহিতরকার সকেb হিতহিন কত হিকেলন তার উকেdখকোয বই lsquoসহিবনয় রায়কেচৌধরীর রচনা সংগরrsquo

পরম হিকেনর পাঠ- lsquoপশপাহিখর হিক -াষাhelliphellip helliphelliphelliphellipপরসপরকেক জানাবার উপায়ও পশপাহিখরা যবশ জাকেনrsquoপরকেমই আমারা জাহিন -াষা হিক -াষা ল আমাকের মকেনর -াব পরকাশ করার জনয আমরা নানান ধরকেনর -হিb বা হিবকেশষ ধরকেনর আওয়াজ মকেখর মাধযকেম কহির অনযকেক যবাঝাকেনার জনয তাকেল এবার আমরা জাহিন পশপাহিখর -াষা হিক পশ পাহিখরা হিক কা বকেল যা পশপাহিখকেরও -াষা আকে তারা তাকের হিনজসব -াষায় কা বকেল মকেনর -াব পরকাশ ককের পশ পাহিখরা মানকেষর হিক হিক -াষা যবাকেঝ হিকনত তারা বলকেত পাকেরনা পরসপরকেক বহিঝকেয় যবার উপায় তারা জাকেননা তকেব তারা হিবকেশষ ককেয়কটি শকেবদর মাধযকেম তাকের মকেনর -াব বহিঝকেয় যয় হিক বহিদধ মান জীব ndashককর হিবাল বন মানষ যঘাা পর-হিত এরা মানকেষর যওয়া নাম শনকেল কান খাা ককের ndash নাম ধকের ডাককেল কাকে আকেস যমন - মরহিরা lsquoহিত ndashহিতrsquo ডাক শকেন আকেস াল lsquoঅ ndashর -র ডাক শকেন কাকে আকেস াহিত মাহকেতর কা শকেন চকেল ককররা মাহিলকেকর হকম পালন ককের সবসময় তাইকেতা ককরকেক পর- -কত পরানী বলা য় ককর আর হিবাল একের আওয়াজ তহিম লকষয করকেল বঝকেব ককররা যরকে যকেল lsquoযঘউ যঘউrsquo করকেত াকেক আবার কাকেল lsquoযকউ যকউrsquo ককের হিবাল সাধারণ lsquoমযাওrsquo বা lsquoহিমউrsquo ককের রা কেল lsquoওয়াওrsquo আওয়াকেজর মাধযকেম মকেনর -াব পরকাশ ককের একেতা যল পশকের কা পাহিখরাও -য় রা পরকাশ করার জনয হিবকেশষ ধরকেনর শবদ ককের হিবপকের সময় পশ পাহিখরা সবার আকে পরসপরকেক জানাবার উপায় তারা জাকেন বহকাল

১) পশপাহিখর -াষা কেলপর যলখক সমপকেকG হিক জাকেনা

উঃ- পরখযাত সাহিহিতযক উকেপনদরহিককেশার রায়কেচৌধরীর পতর সহিবনয় রায়কেচৌধরী lsquoসকেFশrsquo পহিতরকার সকেb হিতহিন কত হিকেলন তার উকেdখকোয বই lsquoসহিবনয় রায়কেচৌধরীর রচনা সংগরrsquo হিতহিন ারকেমাহিনয়াম এসরাজ পর-হিত বাযনতর বাজাকেত পারকেতন ানও জানকেতন হিতহিন যাকেIাকের জনয মজাার লপ কহিবতা হিলখকেতন

২) পশপাহিখ কেলপর মল-াব হিকউঃ- পশপাহিখকেরও -াষা আকে তারা তাকের হিনজসব -াষায় কা বকেল মকেনর -াব পরকাশ ককের পশ পাহিখরা মানকেষর হিক হিক -াষা যবাকেঝ হিকনত তারা বলকেত পাকেরনা পরসপরকেক বহিঝকেয় যবার উপায় তারা জাকেননা তকেব তারা হিবকেশষ ককেয়কটি শকেবদর মাধযকেম তাকের মকেনর -াব বহিঝকেয় যয় হিরউকেবন কযাসটং সাকেব হিতহিন চহিdশ বর বনযজনত যর সকেb যকেককেন হিতহিন বকেলকেন আমরা হি তাকের -াষা তাকের আব কায়া যমকেন চহিল তাকেল আর -কেয়র যকান কারণ াকেকনা আমরা একI -াকেলাকেবকেস যচষটা করকেল পশপাহিখকের সকেb -াব পাতাকেত পাহির

ধকের মানষ এই পশ পাহিখর -াষা হিনকেয় নানা রককেমর পরীকষা ককের আসকে এইরকম একজন হিরউকেবন কযাসটাং সাকেকেবর কা আমরা জানকেবাhelliphellip

Hindi 2nd

langमतर किनमनलिलखिखतपरशनोउRरदीजि0ए

) बढ वयलि` बचच ो कया हआ था ख) डॉकटर साहबन पाटc किस उददशय स रखी थी ग) ाल साप ो हाथ म लर लाश न कया किया घ) डॉकटर चडढा न बढ पतरो दखन स कयो मना र दिदया था ङ) भगत न लाश ो दखर कया हा

उRर ndash) उस बहत बखार थी और 4 दिदनो स आख भी नही खोला थाख) उन बट ी सालकिगरह थीग) ाल सापो हाथ म लर लाश न उसी गदन 0ोर स दबार पडी थीघ) डॉकटर चडढा न बढ वयलि` पतरो दखन स मना र दिदया कयोकि उनह गोलफ खलन 0ाना थाङ) लाश ो दखर हा कि नारायण चाहग तो आध घट म भया उठ 0ाएग

English literature

In the bazaars of Hyderabad- Sarojini Naidu

Through the poem In The Bazaars of Hyderabad Sarojini wanted to convey the message that India is rich in tradition and they donrsquot need the foreign products So she goes on to give a picture of a bazaar where traditional Indian products are rulingThe poem is in the form of questions and answers The poet asks the questions and the merchants answer them Through this technique she make the picture of the bazaar visible to us

Read the poem

PHYSICS FORCE Types of FrictionThere are three types of friction static sliding rolling Static sliding and rolling friction occur between solid surfaces

1 Static Friction The frictional force that acts between the surfaces when they are at rest with respect to each other is called Static FrictionStatic Friction Examples

Skiing against the snow Creating heat by rubbing both the hands

together Table lamp resting on the table

2 Sliding Friction The resistance that is created between any two objects when they are sliding against each other is called Sliding FrictionExamples Of Sliding Friction

Sliding of the block across the floor Two cards sliding against each other in a

deck

3 Rolling Friction The force which resists the motion of a ball or wheel is called Rolling Friction Is the weakest types of frictionExamples Of Rolling Friction

Rolling of the log on the ground Wheels of the moving vehicles

6What effect can a force produce on a body which is not allowed to move Ans - When a force is applied on a body which is not free to move it gets deformed i e the shape or size of the body changes7Give one example each to indicate that the application of a force

1 produces motion2 stops motion3 slows down motion4 changes the direction of motion5 deforms a body

Ans- 1 A car originally at rest when pushed

begins to move2 A moving bicycle is stopped by

applying the brakes3 The speed of a moving vehicle is

slowed down by applying the brakes4 A player kicks a moving football to

change its direction of motion5 On stretching a rubber string its

length increases

8State the effect produced by a force in the following cases (a) The sling of a rubber catapult is stretched(b) A man pushes a heavy cart(c) A player uses his stick to deflect the ball (d) A cyclist applies brakes(e) A spring is compressedAns- (a) The shape and size of catapult changes ie its length increases(b) The heavy cart begins to move(c) The direction of the ball changes(d) The speed of the moving cycle is slowed down(e) There is change in size and shape of spring

COMPUTER MS EXCEL 2013 -INTRODUCTION

UNDERSTANDING EXCEL STRUCTUREA SPREADSHEET IS A FILE THAT EXISTS OF CELLS IN ROWS AND COLUMNS AND CAN HELP ARRANGE CALCULATE AND SORT DATA DATA IN A SPREADSHEET CAN BE NUMERIC VALUES AS WELL AS TEXT

FORMULAS REFERENCES AND FUNCTIONS

WORKSHEETA WORKSHEET IS ALSO KNOWN AS SPREADSHEETIT IS A COLLECTION OF CELLS ON A SINGLE SHEET WHERE YOU KEEP AND CHANGE DATA

WORKBOOKWORKBOOK IS PMS EXCEL FILE IN WHICH THE DATA CAN BE STORED EACH WORKBOOK CAN CONTAIN MANY WORKSHEETS

ROWS AND COLUMNSIN MS EXCEL A ROW IS A GROUP OF CELLS THAT RUN FROM LEFT TO RIGHT OF A PAGEA COLUMN IS A GROUPING OF CELLS THAT RUN FROM THE TOP TO THE BOTTOM OF A PAGE

CELLTHE INTERSECTION POINT BETWEEN A ROW AND THE COLUMN IS CALLED A CELL WHICH IS THE BASIC STORAGE UNIT FOR DATA IN A SPREADSHEET EACH CELL HAS SPECIFIC ADDRESS WHICH IS THE COMBINATION OF THE COLUMN NAME FOLLOWED BY THE ROW NUMBER

CHEMISTRY Chapter ndash Common Laboratory Apparatus and equipments

Objective type questionFill in the blanks (a) Experiment and observation are the two important basics of chemistry(b) A porcelain dish is used for evaporation(c) A test tube holder is used to hold the test tube while-it is heated(d) Mortar and pestle is used for grinding and crushing solid substances into a powder(e) Glass apparatus is made of Pyrex or borosil glass

Class VIISubject Topic Summary Execution

Hindi 2ndlang

ए था राम( डॉ शरी परसाद)

सगकित ा परभाव मानव 0ीवन पर अवशय पडता ह

हमशा मनषय ो अचछो ी सगकित म रहना चाकिहए

शरषठ परो सग स मनषय चरिरतर ा शीघर ही उदय और किवास हो 0ाता

ह इसलिलए वयलि` ो सदा शरषठ परो ा ही सग रना चाकिहए

इसान अगर चाह वह सवय ो बदल भी सता ह

यह हानी राम ए बचच ी हवह गणिणत ी परीकषा म नल रत हए पडा 0ाता ह और उस अधयाप पडत ह और पछत ह यह कया र रह

हो तभी राम न उनी बइजजती ी

शबदाथब ndashहावा भलावाायवाहीndash ाम किनयम व ानन

ो दिदखानापरिरलिचतndash 0ाना पहचानाघटनाndashघबराहट

उलटा चोर ोतवाल ो डाटndashकिववndash भल बर ा जञानतवयndash म 0ो रना चाकिहएसगकितndash बरी सगत

किबलख नाndashरोना किनशचय रनाndash तय रना

फलndashपरिरणामकिनषालिसतndash बाहर किया हआपशचातापndashदख सपननndashधनी

ldquo हा आपी किहममत स हई नल रत पडन ीrdquo ऐसी बात ही किफर

किपता0ी न भी उस डाटा वह ाफी पशचाताप रन लगा बोला गलत दोसतो

ी सगकित म आ0 कितना अनथ र दिदया किफर उसन अधयाप स माफी मागन ी सची और किफर भी ऐसा

नही रगा यह परण भी लिलया

सोचndashकिहच एात-अला

বইndashবাংলা সাহিতয পহিরচয়

পাঠndash১৬লপndashস-য ও অস-যযলখকndashঈশবরচনদর হিবযাসারঅনশীলনীর পরকে4াততর

৬ অGকেলকেখা -ময়া = পশ হিশকার সহিtহিত = হিনকIবতu সbভরষট = লI হিনরীকষণ =

-াকেলা-াকেব যখাকতাঞজহিলপকেI = যজাাকেত৭ হিবপরীতশবদ -ঈষৎ times পরচর উৎকষট times হিনকষট তাশ times উৎফd তবহিদধ times

বহিদধীNপাহিপষঠ times পণযবান৮ পপহিরবতG ন ককেরা -পশ = পাশহিবক যকাপ = যকাহিপতহিসথর = হিসথরতাএকানত = ঐকাহিনতক পর-াত times পর-াতী

CHEMISTRY

Chapter ndashPhysical and Chemical Changes

Chemical ChangeA chemical change involves a change in chemical composition

Characteristics of Chemical changes 1 They are permanent changes2 They are irreversible changes 3 New substance formed4 A Chemical change involves a

change in its chemical properties

Pg-25Question 8What do you observe when1 water is boiled2 a piece of paper is burnt3 some ice cubes are kept in a glass tumbler4 solid ammonium chloride is heated5 an iron nail is kept in tap water for few days6 a spoon of sugar is heated in a pan7 lighted match stick is brought near the mouth of the test tube containing hydrogen gas8 quick lime is dissolved in water9 little amount of curd is added to a bowl containing warm milk and kept for five hours

10 Water is boiledOn boiling water changes into steam (gas) physical change

11 A piece of paper is burnton burning piece of paper produces carbon dioxide and ash is left behind Is a chemical change

12 some ice cubes are kept in a glass tumblerIce cubes (solid) turn into water

(liquid) only state changes (physical change)

13 Solid ammonium chloride is heatedSolid ammonium chloride on heating changes into vapors (change of state) is physical change

14 An iron nail is kept in tap water for few dayswe observe reddish brown coating on the nail called rust (entirely new substance) is chemical change

15 A spoon of sugar is heated in a panWhen a spoon of sugar is heated in a pan black (charred sugar) (carbon) is seen Is a chemical change

16 Lighted match stick is brought near the mouth of the test tube containing hydrogen gasWe observe that hydrogen bums at the mouth of test tube with blue flame and pop sound is heard It is chemical change

17 Quick lime is dissolved in waterThe following two observations will be observed (i) A hissing sound is observed(ii) The mixture starts boiling and lime water is obtained

18 Little amount of curd is added to a bowl containing warm milk and kept for five hoursWhen a little amount curd is added to a bowl containing warm milk and kept for five hours a permanent change occurredThe milk will change to curd On boiling water changes into steam (gas) physical change

GEOGRAPHY

ATMOSPHERE IMPACT OF GLOBAL WARMING The destructive impart of global warming is observed in various spheres of life and the environment Some of the points are outlined below1 High temperatures lead to high

evaporation rate and drying up of the soil and surface water This affects crop production The occurrence of droughts is aggravating the problem even further

2 The heat waves in summer months

Q1 Write some impact of global warmingA1 The impacts of global warming are as follows1 High temperatures lead to high

evaporate ion rate and drying up of the soil and surface water This affects crop production The occurrence of droughts is aggravating the problem even further

2 The heat waves in summer months lead to a greater number

lead to a greater number of deaths due to heat strokes

3 Forest fires become more frequent4 Tropical cyclones and hurricanes

become common5 Melting of glaciers takes place6 Polar ice caps are becoming thinner

and melting at an alarming rate due to global warming The loss of sea ice

7 Due to increase in sea surface temperature sea levels rise in coastal areas and cause submergence of several islands

WAYS TO REDUCE GLOBAL WARMINGFollowing steps can be taken We need to decrease emission of

green house gases by reducing the burning of fossil fuel such as coal and petroleum

By planting more trees to increase forest cover

The government should also distributes free saplings and organize afforestation programmes to spread awareness regarding the beneficial effects of trees

We should switch to eco-friendly cars and gadgets

Incandescent light bulbs should be replaced by CFL bulbs

We can save electricity and reduce global warming by turning off electrical gadgets such as lights fans air-conditioners television and computer when we do not to use them

Efforts should be made to hasten the development of green cities oreco cities These cities are urban areas around the world striving to lessen the environment a impacts of urbanization

By following the 3Rs-Reduce Recycle and Reuse strategy we can use natural resources for our growth as well as save them for the need of the future generations This is called sustainable development

of deaths due to heat strokes3 Forest fires become more

frequent4 Tropical cyclones and hurricanes

become common5 Melting of glaciers takes place

etc

Q2 How to reduce global warmingA2 Following steps can be taken to reduce global warmingaWe need to decrease emission of

green house gases by reducing the burning of fossil fuel such as coal and petroleum

bBy planting more trees to increase forest cover

c The government should also distributes free saplings and organize afforestation programmes to spread awareness regarding the beneficial effects of trees

dWe should witch to eco-friendly cars and gadgets

eIncandescent light bulbs should be replaced by CFL bulbs

f We can save electricity and reduce global warming by turning off electrical gadgets such as lights fans air-conditioners television and computer when we do not to use them

Q3 What do you mean by 3Rrsquos of resource planningA3 The 3Rs are

1 Reduce 2 Recycle and3 Reuse

Q4 What is Sustainable developmentA4 By following the 3Rs-Reluce Recycle and Reuse strategy we can use natural resources for our growth as well as save them for the need of the future generations This is called sustainable development

English Language

Prepositions A preposition is a word placed before a noun or a pronoun It helps to show how the person or thing denoted by the noun is related to something else in the sentence

Kinds of Prepositions

Simple Prepositions- simple preposition are one word Prepositions such as at by for in of off for from on out through till to up with before amidst towards beyond between over etc

Compound Prepositions ndash There are some words that are always used with fixed Prepositions to convey specific meaning

Example I was unable to meet you dueto a previous engagement ( On account of)Always maintain the queue instead of crowding at the counter ( In place of)

Participial PrepositionsmdashParticiple Prepositions are present or past participles of various verbs which together with a noun phrase or a clause function as prepositions Examples- barring concerning considering notwithstanding pending regarding respecting etc

Exercise A

1 Gauravs fever has come down since Friday He has been absent for a week now

2 The child sat between his father and mother among the parents of all his classmates

3 There are mosquitoes in the room They flew into the room when the door was open

4 My father was inside the drawing room when I was playing outside my house

5 You may sit beside me I will give you a drawing book and pencils besides a storybook

6 We went to the market in the morning and walked towards the riverfront in the evening

7 The child walked along the pavement and across the street safely

8 This table top is made of glass My breakfast fell off it in the morning

9 The pan is on the gas stove There are vegetables in it

10 We will wait for you at the bus top There are a lot of people in the hall

Subject ndash Biology Topic ndash Chapter - 3 Photosynthesis and respiration in plants Summary Execution

All living organism (Plants and animals) need food for energy and growth Green plants (autotrophy) prepare food for all living organisms Today we will discuss about the process photosynthesis And adaptations in a leaf to carry out photosynthesis

Q1What do you mean by photosynthesis and write its word equation The process by which green plants make food (glucose) from carbon dioxide and water

in the presence of sunlight and chlorophyll is called photosynthesis

Carbon dioxide + Water ( Sun light from Sun ) Glucose + Oxygen ( chlorophyll in green leaves )

Q2 What are the adaptations in a leaf to carry out photosynthesisi) Leaves are broad wide and flat for absorbing more light energyii) Presence of chlorophyll in chloroplasts to trap sunlightiii) Presence of stomata which allow carbon dioxide to enter the cell and oxygen to go

out iv) Network of veins ensures continuous supply of water and minerals to the leafv) Thin waxy cuticle protects the leaf without blocking the lightQ3 Draw and label structure of chloroplast

Class VIIISubject Topic Summary Execution

PHYSICS ENERGY Production of Hydro electricity

A hydroelectric dam converts the potential energy stored in a water reservoir behind a dam to mechanical energymdashmechanical energy is also known as kinetic energy As the water flows down through the dam its kinetic energy is used to turn a turbine

The generator converts the turbinersquos mechanical energy into electricity

This electric energy then goes through various transmission processes before it reaches you

Question 2

Fill in the blanks

(a) Work is said to be done by a forte only when the body moves

(b) Work done = Force x distance moved in direction of force

(c) The energy of a body is its capacity to do work

(d) The SI unit of energy is joule

(e) The potential energy is due to its state rest of position and kinetic energy of the body is due to its state of motion

(f) Gravitational potential energy U = mass times force of gravity on unit mass times height

(g) Kinetic energy = frac12 times mass times (speed)2

(h) Power P = work donetime taken

(i) The S I unit of power is watt

(j) IHP = 746 W

BIOLOGY Chapter -5 The endocrine system and adolescence

Today we will discuss about thelocation and functions of secreted hormones of adrenal and Pancreas

Q5 Write location hormone secreted main functions and deficiency diseases of pancreas and adrenal glands

Endocrine Glands

Location Hormones secreted

Functions and Deficiency Diseases

1Adrenal gland

2 Pancreas Gland

On the top of each kidney

In between stomach and small intestine

i)Adrenaline from adrenal medulla

ii)Cortisone from adrenal cortex

i) Insulin

ii) Glucagon

It helps a person deal with any kind of emergency situation or emotional stressIt increases the heart beat rate of respiration and blood pressure

a) It regulates carbohydrates protein and fat metabolism

b) It regulates the salt and water balance in the body

a) It changes excess glucose into glycogen

b) It stimulates the cells to burn extra glucose to provide heat amp energy

Less secretion causes diabetes mellitus

Excessive secretions causeinsulin shock

a) It stimulates the breakdown of glycogen into glucose

b) It increases the level of glucose in blood

History Traders to rulers The Battle of Buxar was fought on 22 October 1764 between the forces under the command of the British East India Company led by Hector Munro and the combined armies of Mir Qasim the Nawab of Bengal till 1763 Mir Jafar was made the Nawab of Bengal for a second time in 1763 by the Company just after the battle After being defeated in 4 battles in katwa and Udaynala the Nawab of Awadh Siraj id Daula and the Mughal emperor Shah Alam II accompanied by Raja Balwant Singh of Kashi made an alliance with Mir Qasim The battle was fought at Buxar a small fortified

Answer the following questions- Short note-Battle of BuxarHomework-learn

town within the territory of Bihar located on the banks of the Ganga river about 130 kilometres (81 mi) west of Patna it was a decisive victory for the British East India Company The war was brought to an end by the Treaty of Allahabad in 1765

EnglishLiterature

The west wind-John Mansfield

In the poem The West Wind by John Masefield the poet starts by describingwith very poetic imagery of birds how the west wind is different from other winds its a warm wind full of birds cries There is a touch of melancholy perhaps home-sickness as he describes how it brings tears too and memories from an old land He goes on to describe the restful pastoral beauty of the land where even the dead can lie in the green He then brings in voicesperhaps of family and friends calling him home as he is missing Aprils beautyThe voices then tempt him some more with idyllic images from home (white blossom young green cornrunning rabbitswarm sun) The voices seem to presume that the poets heart is sorrowful bruised and soreThe end of the poem sees the poet appear to make a decision he will go home as he has decided that is where he truly belongs

Write the synopsis of the following words

1 Daffodils- a tall yellow flower that grows in the spring

2 Orchards- a piece of land on which fruit trees are grown

3 Blossom- a flower or a mass of flowers especially on a fruit tree in spring

4 Thrushes- a bird5 Larks- a small brown bird that

makes a pleasant sound6 Bruised- an injury7 Aching- pain 8 Tread- to put your foot down

while you are walking9 Balm-10 May-11 Fluting-

(Write from the book in your copy)

MAT

HEM

ATIC

S

Ch 1

1Al

gebr

ic E

xpre

ssio

n

1 Constant A symbol which has fixed value is called a constant[eg 8 23 -15 radic3 etc]

2 VariableA symbol which does not have any fixed value but may be assigned value (values) according to the requirement is called variable or literal[eg x y p q etc]

3 TermsA term is a number (constant) a variable a combination (product or quotient) of numbers and variables[eg 7 x 5x etc]

4 Algebric expressionA single term or acombination of two or more terms connected by plus (+) or minus (-) sign forms an algebraic expression[eg 5-y 3x2-5x xy-6z+4 etc]

5 PolynomialAn algebraic expression which contains more than one term is called a polynomial (multinomial)[eg x2-5x 5y+xy+x2y etc]

6 Degree of polynomial(a) When the polynomial contains only one variable the highest power of the variable is the degree of the polynomialeg the degree of the polynomial of 4x-7x5+8 is 5(b) When the polynomial contains two or more variablesStep (i) Find the powers of the variables in each term (ii) The highest sum of the powers is taken to be the degree of the polynomialeg the degree of the polynomial 5x2y-4x3y5+6 is = 3+5 = 8Remember An algebraic expression is a polynomial if degree of each term used in it is a non-negative integer

Exercise ndash 11(A)

1 Separate the constants and variables from the following

-7 7+x 7x+yz radic5 radic xy 3 yz

8 45y -3x

Solution Constant Variables-7 radic5 7+x 7x+yz radic xy

3 yz8

45y -3x

2 Write the number of terms in each of the following polynomials(i) 5x2+3timesax (ii) axdivide4-7 (iii) ax-by+ytimesz (iv) 23+atimesbdivide2

Solution Polynomials Number of terms(i) 5x2+3timesax 2(ii) axdivide4-7 2(iii) ax-by+ytimesz 3(iv) 23+atimesbdivide2 2

4 Write the degree of the each polynomials(i) xy+7z (ii) x2-6x3+8 (iii) y-6y2+5y8 (iv) xyz-3 (vi) x5y7-8x3y8+10x4y4z4

Solution Polynomials Degree(i) xy+7z 2(ii) x2-6x3+8 3(iii) y-6y2+5y8 8(iv) xyz-3 3(vi)x5y7-8x3y8+10x4y4z4 12

5Write the coefficient of(i) ab in 7abx (iv) 8 in a2-8ax+a (v) 4xy in x2-4xy+y2

SolutionCoefficient

(i) ab in 7abx 7x(iv) 8 in a2-8ax+a -ax(v) 4xy in x2-4xy+y2 -1

7 CoefficientAny factor of an algebraic quantity is called the coefficient of the remaining quantityeg in the algebraic term 7xyz 7 is coefficient of xyz 7x is coefficient of yz and so on

8 Like term The terms having the same literal coefficient are called like terms and those having different literal coefficients are called unlike terms

eg (i) 5xyz 8xyz -6xyz and 23xyz are like

terms(ii) 7xy2 8x2yz and -15xyz2 are unlike terms

6 in 57xy2z3 write the coefficient of

(i) 5 (vii) 5xy2 (viii) 17yz (xi) 5xyz

Solution Coefficient

(i) 5 17

xy2z3

(vii) 5xy2 17z3

(viii) 17yz

5xyzsup2

(xi) 5xyz 17yz2

7 In polynomial given below separate the like terms(ii) y2z3 xy2z3 -58x2yz -4y2z3 -8xz3y2 3x2yz and 2z3y2

Solution y2z3 -4y2z3 2z3y2 are like terms

xy2z3 -8xz3y2 are like terms

-58x2yz 3x2yz are like terms

Class IXSubject Topic Summary Execution

Bengali (2nd language)

বাগzwnjধারাzwnj বা ধারা-বা ধারা ল হিবকেশষ পরকার বাক -হিb -াকেবর এক হিবকেশষ পরকাশরীহিত াকেক কতগকেলা কার সমষটির মকেধয এগহিলকেক বা ধারা বকেল আবার কতগকেলা শকেবদর বাধাধরা যকান রীহিত যনই য-াকেব চকেল আসকে যসই -াকেবই চকেল আসকে তখন যসই শবদগহিল খন একক -াকেব অG পরকাশ ককের তখন একের বা ধারা বকেল বা ধারার পরকেয়া -াষাকেক আরও সFর ককের যতাকেল

অকাল পকক(অপহিরনত বয়কেস পাকাহিম)-মাতর শ বর বয়কেস যমকেয়টির া মকেখর কা তাকেত অকালপককতা ধরা পকে

অককা পাওয়া( মারা াওয়া) ndash পকেকIমারটি পকেকIমারকেত হিকেয় বাসাতরীকের াকেত মার যখকেত যখকেত অককা যপল

অহি| পরীকষা ( কঠিন ও পরকত পরীকষা)- যকেলটির আজ ডাকতাহির যরজালট যবকেরাকেব এIাই তার জীবকেনর ব অহি| পরীকষা

অষটরমভা (ফাহিক) ndash রীতা মকেখই বকো বকো কা বকেল আর কাকেজর যবলায় অষটরমভা

অকমGার ধাী (অপাG) ndash সমনকেক হিনকেয় যকান ান কেব না ও একেকবাকেরই অকমGার ধাী

অকেনধর ষটি (অসাকেয়র সায়)- আহিশ বকেরর বকোর নাহিত ল অকেনধর ষটি তাকেক াা বকোর একম চকেল না

আকেককল গড়ম (তবহিদধ)- ার তহিম উপকার করকেল যসই যতামার হিবরকেদধ সাকষয হিকেয়কে শকেনই আমার আকেককল গড়ম

আষাকে লপ( অবাসতব লপ) ndashIাকা এখন যকেব না এIা বলকেলই ত এমন আষাকে লপ ফাার যকান রকার হিল না

Hindi- महायजञ ा इस हानी म लख न या बतान ा परयास किया ह कि किसी भी अचछ

2nd language

परसार(यशपाल ाय या पणय न ा फल अवशय मिमलता ह ोई भी परोपार अथवा पणय लिलए किया गया ाय बार नही 0ाता वह ए परार ा यजञ हए धनी सठ थ धम परायण और किवनमर सठ न आन ी यजञ किए थ और दान म न 0ान कितना धन दिदन दखिखयो म बात दिदया थादिदन पलट और सठ यहा गरीबी आ गई उन दिदनो यजञ बचन ी परथा थी सठ भी अपनी 0गह बचन लिलए डलपर ए सट यहा चलन ो तयार हए सठानी रासत लिलए रोटी पड म बाधर सठ ो द दी रासत म ए भख R ो दखर सठ न चारो रोटी उसो खिखला दी खर वह सठ यहा डलपर पहच तो उनी सठानी न उस महायजञ बचन ो हा यदिद बचन आए सठ न R ो रोटी खिखलान ो महायजञ नही समझा और वापस लौट आया घर आर शाम ो उसी घर म उस ए बडा ख0ाना मिमला 0ो उस दवारा किए गएrsquo महायजञrsquo ा परसार था

English language

Letter formal The heading the name and address of the person you are writing to must be included beneath your own address In formal letters ldquoblock stylerdquo of address is preferred

Subject complain in brief

Salutation If the person you are writing to is known to you you may begin ldquoDear MrrdquoOr ldquoDear Mrsrdquo In all other instances you should begin ldquoDear Sirrdquo or ldquoDear Madamrdquo Or ldquoSirsrdquo

The body A formal or business letter has four partsReference The letter should begin by referring to a letter you have received an advertisement or the reason that has prompted you to writeInformation In the second paragraph it is necessary to supply more detailed information that is related to the referencePurpose Here you must give the reason why you are writing the letter This must be stated clearly and ensure that it is relevant to the question that has been setConclusion round off the letter with some polite remarkThe subscription when a letter has begun with dear sir sirs Madam you should end with Yours faithfully or yours truly When however you address a person by name you must conclude with the words ldquoYours sincerelyrdquo

1 A park in your locality is slowly being used as a rubbish dump Write a letter to the Mayor of your city pointing out the nuisance and danger of this Request that action be taken to stop this immediately

Or2 You being a boarder ordered a set of lab manuals from a famous book shop in the town They sent you a wrong set of books Write a letter to the manager of the book shop

Chemistry Chapter-1 1)CHEMICAL FORMULA- Q What is the Significance of

L-2The Language of Chemistrybull Chemical Formula

Itrsquos a symbolic representation of a chemical substance eg ndash The formula of Sulphuric acid is H2SO4

2) Steps of writing Chemical Formula of a given substance-

1 Write the symbols of the constituent atoms or radicals side by side Keep the basic radical on LHS and acid radical on the RHS ( Na+Cl- )2 In case of a radical having more than one atom( compound radical) enclose the radical in a bracket eg (SO4-)3 Write the valencies of each radical on its right hand top4 If the valencies of the two radicals are divisible by a common factor then divide the valencies by the common factor5 Invert (criss-cross) the valency number ie write the valency of one atom below the second atom and vice versa 6 On interchanging if valency number is lsquoone the figure lsquoonersquo is never writtenFor Example- Compound -Calcium Nitrate1 Writing the symbols- Ca(NO3)2 Writing the valencies on their right hand top- Ca2(NO3)1

3 Valency numeral in simple ratio- Ca2(NO3)1

4 Criss-cross- Ca 2NO3 1

5 Writing the formula of the compound- Ca(NO3)2

Chemical formula

A The formula of a substance conveys the following information regarding a substance 1 The name of the substance (qualitative)2 The elements constituting the substance (qualitative)3 The number of various atoms present in a molecule of the substance (quantitative)4 Molecular weight of the substance and the relative weights of different elements present in it (qualitative)

Q What are the limitations of Chemical Formula

A The chemical formula suffers from the following limitations-I It fails to convey whether the elements in a molecule are present in the form of atoms or ionsFor example the formula KBr fails to tell us whether Potassium and Bromine are present in the form of ions II It does not tell anything about the binding force that holds atom in a molecule togetherIII It does not tell us about the arrangement of various atoms with respect to one another within the molecule

Q Examples of Some Chemicals with their Formula Chemical name and Common Name-

A Given in the class notesCommercial Studies

Joint Stock Company

Let us discuss about the demerits of Joint Stock CompanyDespite so many advantages it has got many disadvantages which are as follows

Difficulty in FormationDelay in Decision makingExcessive Government ControlLack of Secrecy

Company can be classified into several categories based on incorporation

QuestionExplain the demerits of Joint Stock CompanyAnswer) 1 Difficulty in Formation The legal requirements and formalities required to be completed are so many The cost involved is quite heavy It has to approach large number of people for its capital It cannot start its business unless certificate of incorporation has been obtained This is granted after a long time when all the formalities are completed

Chartered CompanyStatutory CompanyRegistered Company

Delay in Decision making In this form of organization decisions are not made by single individual All important decisions are taken by the Board of Directors Decision-making process is time-consuming So many opportunities may be costly because of delay in decision-making Promptness of decisions which is a common feature of sole trader ship and partnership is not found in a company

Excessive Government ControlA company and the management have to function well within the law and the provisions of Companies Act are quite elaborate and complex At every step it is necessary to comply with its provisions lest the company and the management should be penalized The penalties are quite heavy and in several cases officers in default can be punished with imprisonment This hampers the proper functioning of the company

Lack of Secrecy The management of companies remains in the hands of many persons Every important thing is discussed in the meetings of Board of Directors Hence secrets of the business cannot be maintained In case of sole proprietorship and partnership forms of organisation such secrecy is possible because a few persons are involved in the management

2 Define the following

Chartered Company- The crown in exercise of the royal prerogative has power to create a corporation by the grant of a charter to persons assenting to be incorporated Such companies or corporations are known as chartered companies Examples of this type of companies are Bank of England (1694) East India Company (1600) The powers and the nature of business of a chartered company are defined by the charter which incorporates it After the country attained independence these types of companies do not exist

in IndiaStatutory Company- A company may be incorporated by means of a special Act of the Parliament or any state legislature Such companies are called statutory companies Instances of statutory companies in India are Reserve Bank of India the Life Insurance Corporation of India the Food Corporation of India etc The provisions of the Companies Act 1956 apply to statutory companies except where the said provisions are inconsistent with the provisions of the Act creating them Statutory companies are mostly invested with compulsory powersRegistered companiesCompanies registered under the Companies Act 1956 or earlier Companies Acts are called registered companies Such companies come into existence when they are registered under the Companies Act and a certificate of incorporation is granted to them by the Registrar

Economics

Chapter-4Basic problems of Economy

Today let us discuss with the topic Production Possibility curve

QuestionExplain the concept of Production Possibility Curve with the help of diagram

Answer) Production Possibility curve is a locus of all possible combinations of two commodities which can be produced in a country with its given resources and technology

The above diagram shows that with the given resources and technology the economy can produce maximum either 5 thousand meters of cloth or 15 thousand quintals of wheat or any other combination of the two goods like B( 1 thousand meters of cloth and 14 thousand quintals of wheat C ( 2 thousands meters of cloth and 12 thousand quintals of wheat) etcProduction Possibility curve is also called production possibility boundary or frontier as it sets the maximum limit of what it is possible to produce with given resources

Geography

Rotationand Revolution

SUNrsquoS POSITION AND SEASONAL CHANGES EQUINOXES ndash SPRING AND AUTUMN

Q1 What is Spring EquinoxA1 On 21st March sunrays fall directly on the equator On that day

As the Equator divides the Earth into two equal halves the sun rays fall directly on the equator twice in a year Equinoxes means equal Spring EquinoxOn 21st March sunrays fall directly on the equator On that day the duration of day and night both are equal ( 12 hours day and 12 hours night) on every places located on equator This day is called as Spring EquinoxAutumn EquinoxOn 23rd September sunrays fall directly on the equator On that day the duration of day and night both are equal ( 12 hours day and 12 hours night) on every places located on equator This day is called as Autumn Equinox

SOLSTICES ndash SUMMER AND WINTERDue to inclination of the Earth on its axis and the apparent movement of the sun the sun rays fall directly on both tropics once in a year Solstice is a Latin word which mean ldquothe Sun standing stillrdquoSummer SolsticesAfter 21st March there is an apparent movement of the Sun to the north of the equator The apparent northward movement up to 21st June when the Sun appears overhead at the Tropic of Cancer (22frac12degN) The sun appears to stand still at this position and then moves southwards towards the equator This position of the Sun on 21st June is known as Summer Solstices On that day the duration of day and night both are equal ( 12 hours day and 12 hours night) on every places located on Tropic of Cancer (22frac12degN)Winter solstices The apparent southward movement of the Sun continues beyond the equator till 22nd

December On this day the Sun is overhead at the Tropic of Capricorn

the duration of day and night both are equal ( 12 hours day and 12 hours night) on every places located on equator This day is called as Spring Equinox

Q2 What do you mean by EquinoxA2 Equinoxes means equal It is use to explain the equal duration of day and night ( 12 hours day and 12 hours night) on the Earth

Q3 On which date the longest day in Tropic of CancerA3 21st June

Q4 What is the meaning of SolsticeA4 Solstice is a Latin word which mean ldquothe Sun standing stillrdquo

Q5 Which is the longest day in southern hemisphereA5 22nd December

Q6 On what date does the Arctic Circle experience the lsquoMidnight SunrsquoA6 On 21 June the Arctic Circle experiences the lsquoMidnight Sunrsquo

Q7 What is cause of Midnight Sun in NorwayA7 During the summer solstice (21 June) the North Pole is inclined towards the Sun Therefore the duration of sunlight or daytime increases from 12 hours at the Equator to 24 hours at the Arctic Circle and beyond Thatrsquos why The region beyond the Arctic Circle especially Norway is known as the Land of the Midnight Sun because there the Sun does not rise or set on 21 June

Q8 Match the column A with BA B

Summer Solstice 21st March

Autumn Equinox 23rd

September

Winter Solstice 21st June

(22frac12degS) This position of the Sun is referred to as the Winter Solstice because it marks the winter season in the Northern Hemisphere On that day the duration of day and night both are equal ( 12 hours day and 12 hours night) on every places located on Tropic of Capricorn (22frac12degS)SEASONS AND DURATION OF DAY AND NIGHT During the equinoxes all places on the Earth have 12 hours of day and 12 hours of night Due to the revolution of the Earth round the Sun on an inclined axis the duration of day and night varies according to seasons and the latitude of a placeDuring the summer solstice (21 June) the North Pole is inclined towards the Sun Therefore the duration of sunlight or daytime increases from 12 hours at the Equator to 24 hours at the Arctic Circle and beyondThe region beyond the Arctic Circle especially Norway is known as the Land of the Midnight Sun because there the Sun does not rise or set on 21 JuneAt the North Pole there will be six months of daylight The Sun will be seen always above the horizon at a low angle At 66degN 24 hours of sunlight can be seen only on 21 June Hammerfest in northern Norway is a place of tourist attraction for observing the phenomenon of the Midnight Sun This place has continuous daylight from 13 May to 29 July This place is easily accessible to tourists and has hotels and other facilities The view of the midnight Sun from here is enthrallingIn the Southern Hemisphere the duration of daylight decreases from 12 hours at the equator to 0 hours beyond the Antarctic Circle In the South Polar Region there is 24 hours of darkness The Sun is always below the horizon In the Southern Hemisphere which experiences winter the duration of night-time is longer than the duration of daylight

Spring Equinox 22nd

December

A8 A B

Summer Solstice 21st June

Autumn Equinox 23rd

September

Winter Solstice 22nd

December

Spring Equinox 21st March

During winter solstice (22 December) the South Pole is inclined towards the Sun The Southern Hemisphere experiences summer and the Northern Hemisphere has winter Therefore the duration of daylight or sunlight is greater in the Southern Hemisphere than in the Northern HemisphereThe duration of daylight increases from 12 hours at the equator to 24 hours beyond the Antarctic Circle The South Polar Region has 24 hours of sunlight for many days continuously At the South Pole there will be six months of sunlight The Sun will always be seen at a low angle above the horizon In the Northern Hemisphere the duration of daylight will decrease from 12 hours at the equator to 0 hours at the Arctic Circle There are 24 hours of darkness in the North Polar region The duration of night is greater than the duration of daylight as one move northwards from the Equator It is evident from the above table that the duration of daylight is 12 hours throughout the year at the equator only As one moves away from the equator the seasonal variations in the duration of daylight increase The seasonal variations in the duration of daylight are maximum at the Polar Regions

Subject Eng Literature (The Merchant of Venice ndash William Shakespeare)Topic Act II Scene 7 Lines 36 to 80 (End of scene ) [Students should read the original play and also the paraphrase provided]

Summary Questions amp AnswersThe Prince then examines the inscription on the silver casket which says ldquoWho chooseth me shall get as much as he deservesrdquo The Prince says that he deserves Portia more than anybody else because of his high rank his noble birth and his great wealth and power But then he argues that silver is ten times

(1) (Act II Sc 7 L 39-47)

From the four corners of the earth they come

To kiss this shrine this mortal breathing saint

The Hyrcanian deserts and the vasty wildsOf wide Arabia are as through-fares now

inferior to gold and therefore he cannot believe that the portrait of such a beautiful lady as Portia can be contained in the silver casket He decides to see the inscription on the golden casket before making his decision

The Prince goes to examine the inscription on the golden casket which says ldquoWho chooseth me shall get what many men desirerdquo The Prince believes that the whole world desires to possess Portia otherwise so many suitors would not have come from all corners of the world for winning Portia Some of them have come from the distant lands of Persia and Arabia The deserts of Persia (Hyrcanian deserts) and the boundless desolate lands of Arabia have been crossed by the Princes seeking the hand of Portia He contrasts this casket containing Portiarsquos portrait with the old English gold coin bearing the image of the archangel (angel of the highest rank) He goes on to remark that while the figure of the archangel is engraved (Insculped) upon the English coin the picture of Portia who is beautiful as an angel lies hidden inside one of the caskets namely the Golden Casket (Golden Bed)

On the basis of his assessment of the inscription on the golden casket the Prince decides to choose the golden casket He asks for the key and opens the golden casket only to find therein an empty human skull holding a roll of

For princes to come view fair PortiaThe watery kingdom whose ambitious headSpets in the face of heaven is no barTo stop the foreign spirits but they comeAs orsquoer a brook to see fair Portia

(i) Explain the occasion for the above mentioned speech

These are the comments of the Prince of Morocco after he reads the inscription on the golden casket His mental process is revealed to us in these words We find him debating within himself as to which casket he should choose

(ii) What light does the above speech throw on the personality of Prince of Morocco

From the above mentioned speech we come to know that the Prince of Morocco is keen to marry Portia He is the type of person who is easily taken away by outward appearance He is in love with Portia because of her beauty

(iii) What information can you gather about Portia from the above mentioned lines

The given speech shows that Portia is a very beautiful lady She must be possessed of good qualities because many suitors come to her place from all over the world with a desire to get married to her The Prince of Morocco is so impressed by her beauty that he calls her a saint According to him the whole world is desirous of having her

(iv) Elucidate the significance of the first two lines

In these lines the Prince of Morocco pays a compliment to Portia These lines show his admiration for her He says that people come from all parts of the world to see fair Portia

(v) Explain the meaning of the last four lines of the

passage

In these lines the Prince of Morocco says that even the vast oceans which throw a challenge at the sky are unable to prevent men from coming to Portiarsquos place to have a glimpse of her These lines are also a tribute to Portiarsquos beauty and good qualities Many men voyage across the ocean treating it as a mere stream to see the beautiful Portia

paper in which is written that whoever happens to be guided by the glitter of things is invariably deceived

On reading the scroll the Prince says that he is too sad at heart to speak a more formal farewell and leaves with his followers amidst a sound of trumpets

After the Prince of Morocco leaves Portia remarks that the Prince is a gentle fellow but she is rid of him May all persons of his nature make a similar choice

IMPORTANT PASSAGES EXPLAINED

(Act II Sc 7 L 39-43)From the four corners of the earth they come

To kiss this shrine this mortal breathing saintThe Hyrcanian deserts and the vasty wildsOf wide Arabia are as through-fares nowFor princes to come view fair Portia

Context

This passage occurs in Act II Scene 7 in The Merchant of Venice This is part of the speech made by the Prince of Morocco

(2)

(Act II Sc 7 L 48-53)

MOROCCO One of these three contains her heavenly pictureIst like that lead contains her

Twere damnation To think so base a thought it were too grossTo rib her cerecloth in the obscure graveOr shall I think in silver shes immurdBeing ten times undervalued to tried gold

(i) What meaning does the Prince of Morocco find out of the inscription of the golden casket What have Belmont and Portiarsquos house been called and why

The inscription on the golden casket is ldquoWho chooseth me shall gain what many men desirerdquo The Prince finds out that it means that the chooser of the golden casket will get Portia because many men desire her In fact the entire world desires her Because of the coming of many suitors to Belmont from different countries in order to win Portiarsquos hand Belmont has become a centre of pilgrimage and her house is the shrine where saintly Portia is installed

(ii) What does the Prince of Morocco do before making the final choice of the casket Which is the correct casket and who will win Portiarsquos hand

The Prince of Morocco surveys and analyses the inscriptions on the casket of lead silver and gold Before making the final choice like a very systematic and methodical person he once again considers the claims of the caskets The casket containing Portiarsquos picture is the correct casket and the person choosing it will win Portiarsquos hand

Explanation

While praising Portia the Prince of Morocco conceives Portia as a goddess whose image is placed inside one of the caskets Many suitors are coming from far and wide the north and the south the east and the west (Four corners) in order to try their luck Some of them have come from the distant land of Persia and Arabia The deserts of Persia (Hyrcanian deserts) and the boundless desolate lands of Arabia have been crossed by the Princes seeking the hand of Portia All this shows that Portia is indeed the most beautiful lady of the world

(iii) What does the Prince of Morocco say in his estimation while examining the motto on the silver casket What does he find in the golden casket

While examining the motto on the silver casket which says ldquoWho chooseth me shall get as much as he deservesrdquo Morocco says that in his own estimation he surely deserves Portia in all respects ndash rank birth wealth etc

He chooses the golden casket When he opens it he finds an empty human skull holding a scroll in which it is written that those who are attracted by the glittering outside of things are always deceived as Morocco has been deceived

(iv) What kind of nature does the Prince of Morocco have

The Prince of Morocco has a simple nature who does not look deeply into the inner meaning of things but is dazzled by the outward appearance of gold He is inclined to over-estimate his own value and does not realize that it is a duty to ldquogive and hazardrdquo To say that he will not hazard for lead shows that he misreads the true meaning of the inscription which is that he should be prepared to ldquohazard all he hathrdquo for Portia So his feeling is only one of fascination and romantic attraction

(v) Do you think that the lottery of the caskets is not a matter that will be determined by chance

In fact the lottery of the casket is not a matter that will be determined by mere chance but that it is a true test of character and of sincerity which is amply proved not only by Moroccorsquos choice but also by the arguments which he uses to help him in his choice

(Act II Sc 7 L 55-59)

They have in England

A coin that bears the figure of an angelStamped in gold but thats insculpd uponBut here an angel in a golden bedLies all within

Context

(3)

(Act II Sc 7 L 63-77)A carrion Death within whose empty eye

There is a written scroll Ill read the writing

All that glisters is not goldOften have you heard that toldMany a man his life hath soldBut my outside to beholdGilded tombs do worms infoldHad you been as wise as boldYoung in limbs in judgment oldYour answer had not been inscrolld

This passage occurs in Act II Scene 7 in The Merchant of Venice This is part of the speech made by the Prince of Morocco

Explanation

In this passage the Prince of Morocco bestows high praise on Portia whose hand he is seeking He contrasts this casket containing Portiarsquos portrait with the old English gold coin bearing the image of the archangel (angel of the highest rank) He goes on to remark that while the figure of the archangel is engraved (Insculped) upon the English coin the picture of Portia who is beautiful as an angel lies hidden inside one of the caskets namely the Golden Casket (Golden Bed) In the day of Elizabeth silver was ten times inferior in value to gold Therefore the Prince of Morocco believing that Portiarsquos portrait is contained in the Golden Casket decides to choose the Golden Casket

Fare you well your suit is coldCold indeed and labour lostThen farewell heat and welcome frostmdashPortia adieu I have too grievd a heartTo take a tedious leave Thus losers part

(i) What reward does the Prince of Morocco get after making a wrong choice of the Casket How does he feel

After making the wrong choice in selecting the casket of gold the Prince of Morocco as a reward earns a rebuke in the form of a scroll tucked in the empty eye-socket of a skull kept in the casket of gold The Prince is shocked and disappointed He becomes all the more sad and dejected when he reads the scroll which points to his foolishness in being misled by the appearance and outward show as indicative of its worth

(ii) How does the Prince respond after reading the scroll

After reading the scroll the Prince though upset accepts the result with good grace and decorum befitting a royal suitor and true sportsman He says that his love-suit is really cold otherwise he would have chosen correctly but now his efforts have been in vain So he bids farewell to Portia to the warmth and enthusiasm of love and welcomes the cold and bitterness of dejection and misery of life which lies ahead

(iii) What request does he make to Portia and why

After being failure in his mission he requests Portia to give him permission to leave at once because he is too sad to undergo the tediousness of a formal leave-taking He tells that it is the manner in which defeated persons part unceremoniously

(iv) Explain the following lines

ldquoAll that glisters is not goldOften have you heard that toldMany a man his life hath soldBut my outside to beholdGilded tombs do worms infoldrdquo

Mere glitter does not make a metal to be gold Man has often been warned against appearance but it has been of no use Many people have sacrificed their lives only to seek the outer appearance of gold Worms are found inside the gilded

monuments

Class XSubject Topic Summary Execution

Hindi 2ndlang

नया रासता भाग 6 मायाराम 0ी घर म धनी मल 0ी और उनी बटी सरिरता ी ही चचा बनी रहती थी अमिमत ो इसम ोई रलिच ना थी वह धनी घर ी लडी स शादी र सवय ो बचना नही चाहता था उसा भी सवाणिभमान ह ईशवर ी पा

स उस पास पस ी ोई मी नही थी अभी उसन फकटरी ही लगाई थी उसी समझ बाहर था कि उस घर वालो ा झाव पस ी तरफ कयो

ह उसन मा स सवाल किया कि मा तम सरिरता स मरी शादी कयो रना चाहती हो मा न उस समझाया कि वह दखन म बरी नही ह और किफर खानदान अचछा

ह वह ए शल गरहणी रप म घर सभाल सगी अमिमत न मा ो इस बात ा एहसास राया कि मीन सबध लिलए मना रन पर उस दिदल

पर कया बीती होगी मा और अमिमत ी लडी बार म ाफी बात हईमा ा झाव सरिरता ी तरफ था कयोकि वह घर पर अचछा दह0 लर आ रही

थी अमिमत न अपनी मौसी ी बरी हालत बार म बताया कि किस तरह वह बड घर ी खानदानी बटी लाई थी और आ0 उसी हालत कितनी खराब ह लाई थी बहकलब 0ाती ह और बचचो ो भी नही दखती ह बात चल ही रही

थी कि तभी ए ार बाहर आर री धनी मल0ी घर अदर आए और पीछ स डराइवर फल ी ए टोरी लर आया अदर आए और पीछ स

डराइवर ए टोरी फल ी लर आया अमिमत ो फल ी पटी बरी लग रही थी अमिमत न पछ लिलया यह फल कयो ल आए ह प इन सब ी कया

0ररत थी उनो न 0वाब दिदया कि 4 पटी शमीर स मगाए थ अमिमत ो या सनर करोध आ गया तभी उस किपता 0ी आ गए उन आत ही अमिमत उठर बाहर चला गया वहा वहा मा पास आर बठ गया और बोला

अभी रिरशता तय नही हआ और धनी मल 0ी धनी मल 0ी फल ी पटी लर चलआय मा न समझाया कि 0ब सबध 0ड 0ाता ह तो खाली हाथ नही

आत अमिमत न मा स हा कि तम सबन सरिरता ो इस घर म लान ी ठान रखी ह धनीमल 0ी उस दिदन सरिरता ो दखन ी तारीख तय रन आय थ

Commercial Studies

Banking Nowadays Bank provide easy and quick services through internet facilities methods of Banking is called internet bankingIn order to save the time and money involved in visiting Bank branches people increasingly prefer to have internet banking

There are different modes of doing internet banking or transferring money through online They areReal Time Gross Settlement (RTGS)National Electronic Fund Transfers (NEFT)

1

Question

1) Explain the term RTGS Write the features of RTGS

Answer)The acronym RTGS stands for Real Time Gross Settlement which may be defined as the continuous real time settlement of funds transfer individually on and order by order basis without netting lsquoReal timersquo may be defined as the processing of instructions at the time they are received rather than at some letter time lsquoGross settlementrsquo may be defined as the settlement of transfer instructions which occurs

individually

Features of RTGS1It is the continuous settlement of

funds transfer individually on an order by order basis

2RTGS facility is provided only by CBS core banking solution enabled Bank branches

3Amount charged from the customer for RTGS transactions vary from bank to bank

2) Explain the term NEFT Write the features of NEFT

Answer) National electronic funds transfer may be defined as a nationwide system that facilitates individuals Farms and copper operates to electronically transfer funds from any bank branch to any individual farm or corporate having an account with any other bank branch in the country

Features of NEFT2 Transfer can be made 7 times on

weekdays and 6 times on Saturday

3 NEFT cannot be used to receive foreign remittances

4 NEFT transaction takes place in batches

5 A bank branch must be NEFT enabled to become a part of NEFT fund transfer network

6 There is no maximum or minimum amount that can be transferred through NEFT when one bank has a bank account

English Language

CompositionEssay

A composition is an art of creating a piece of writing on any topic or subject It is the writing correctly beautifully and clearly in order to make some interesting reading Structure of the composition

Introduction ( you lay the foundation for your composition)

Body (it constitutes the main part of the essay)

Conclusion (final statement that leaves a lasting impression)

Kinds of essays1 The Narrative essay2 The descriptive essay3 The reflective essay4 The argumentative essay

Write a composition on any one of the following topics (350- 400 words)

1 Friendship Or2 The first day of your school

Subject Eng Literature (The Merchant of Venice ndash William Shakespeare)Topic Act V Scene 1 Lines 127 to 158 (Nerissa helliphellip The clerk will nersquoer wear hair onrsquos face that had it) [Students should read the original play and also the paraphrase given in the school prescribed textbook]

Summary Revision Questions o Soon thereafter Bassanio Gratiano

and Antonio arrive

o Bassanio tells Portia that he is feeling as if it is morning because of the presence of Portia who is shining like the sun When Antonio is introduced by Bassanio to Portia she tells Bassanio that he should be grateful to Antonio who took so much trouble on his account even to the extent of risking his life

o Nerissa starts quarrelling with Gratiano and demands that he show her the ring she had presented to him and which she had warned him not to lose She suspects that Gratiano must have presented the ring to some young woman and not to the lawyerrsquos clerk as he repeatedly says and assures

Answer the following questions to check your preparation of Act IV Scenes 1 and 2

You must attempt only after you have completed your preparation of Act IV The answers must be in complete sentences using textual evidence (with citation) when necessary

[It would be in your own interest to attempt the above questions honestly totally refraining from consulting your textbook or your notes during answering After completion you should correct the paper yourself consulting the textbooknotes etc and award marks as specified Please let me know the marks you scored through WhatsApp in the group or to my personal WhatsApp]

Act IV Scene 1 (each question carries 2 marks)

1 What did the Duke try to do for Antonio

2 Why does Shylock refuse to show mercy How does he justify his stance

3 Why does Antonio say he is ready to die 4 What information is contained in Bellariorsquos letter

5 Why does Portia (as Balthazar) assert that Shylock must show mercy How does he respond

6 What offers are made to Shylock to get him to spare Antonio How are they received

7 What does Antoniorsquos speech as he faces the prospect of Shylockrsquos knife tell you about his character

8 How do Bassanio and Gratiano react to the looming prospect of Antoniorsquos demise

9 How does Portia (as Balthazar) use the law to turn the tables on Shylock

10 What does the Duke decree should happen to Shylock Why What happens to Shylockrsquos estate

11 What does Portia ask Bassanio as payment for her ldquoservicesrdquo What is his initial response What makes him change his mind

Act IV Scene 2 (each question carries 1frac12 marks)

1 What does Gratiano bring to Portia (Balthazar)

2 What does Nerissa plan on getting from Gratiano What does Portiarsquos comment suggest about men

ECO-10 280620 Topic-Supply AnalysisSHIFTING OF SUPPLY

But if there is change in factors other than the price of the commodity then either more is supplied at the same price or less supplied at the same price In such cases the price of the commodity remains constant but there is a change in other factors like change in the price of inputs change in technology of production change in price of other related goods change in taxation policy of the government etc For example there is an improvement in the technology of production of the commodity in question It leads to decrease in per unit of cost production of the commodity The firm is willing to sell more quantity of the commodity at the same price So the supply other commodity increases at the same price This increase in supply is shown by rightward shift of supply curve On the other hand if the firm uses inferior technology of production the cost of production per unit of the commodity increases The firm is willing to sell less quantity at the same price So the supply of the commodity decreases at the same price This decrease in supply is shown by leftward shift of the supply curve The above cases of increase and decrease in supply can be shown with the help of the following figures

Y INCREASE OF SUPPLY Price (Rs) s

P A s1

B

s

X` O s1 X

q q1

Y` Quantity demanded (in units)

Y DECREASE IN SUPPLY s2

s

price (Rs)

C

p A

s2

s

X` o X

q2 q

Y` Quantity demanded ( in units)

Main factors causing increase in supply or rightward shift of supply Curve(i) Fall in the price of other related goods

(ii) Fall in the price of inputsfactors(iii) Use of better technology in production(iv) Decrease in the rate of excise duty by government(v) If the objective of producer changes from profit maximization to salesMaximization

Main factors causing decrease in supply or leftward shift of supply curve(i) Increase in the price of other related goods(ii) Rise in the price of inputsfactors(iii) Use of inferior technology in production(iv) Increase in the rate of excise duty by the government(v) If the objective

Subject - Biology Topic ndash Chapter mdash6 PhotosynthesisSummary Execution

Today we will know about photosynthesis and its stages

Q1 What do you mean by photosynthesis The process by which living plants containing chlorophyll produce food

substances from carbon-di- oxide and water by using light energy Sunlight

6CO2 +12 H2O----------------------- C6 H12O6 + 6H2O + 6O2

Chlorophyll

Q2 What are the importance of photosynthesis I) Food for all Green plants trap solar energy by photosynthesis

process and supply food and energy for all living organisms either directly or indirectly

Ii) Oxygen to breathe in by product of photosynthesis is oxygen which is essential for all living organisms respiration

Q3 Write about two main phases of photosynthesis A Light dependent phase This phase occur in grana of chloroplast I) The chlorophyll on exposure to light energy becomes activated by

absorbing photons Ii) The absorbed energy is used in splitting the water molecules (H2O)

into its two components (H+ and OH- ) and releasing electron s 2H2O------------------------- 4H+ + 4e- +O2

Energy of 4 photons This reaction is known as photolysis

End products are H+ and oxygen water

B Light independent (Dark ) phase The reactions in this phase require no light energy

Here CO2 combine with H+ and produce glucose

Class XI

Subject Topic Summary ExecutionEVS Chapter-4 Legal

regimes for sustainable development

Environmental legislationEnvironmental legislation is the collection of laws and regulations pertaining to air quality water quality the wilderness endangered wildlife and other environmental factors The act ensures that matters important to the environment are thoroughly

Learn -The Forest (Conservation) Act 1980

considered in any decisions made by federal agencies

The Forest (Conservation) Act 1980 The Forest (Conservation) Act 1980 an Act of the Parliament of India to provide for the conservation of forests and for matters connected therewith or ancillary or incidental thereto It was further amended in 1988 This law extends to the whole of IndiaObjects and Reasons of the Forest Conservation Act

Deforestation causes ecological imbalance and leads to environmental deterioration Deforestation had been taking place on a large scale in the country and it had caused widespread concern The act seeks to check upon deforestation and de-reservation of forests

Subject Eng Literature (The Tempest ndash William Shakespeare) Topic Act II Scene 1 Lines 314 to 329 (End of scene)

[Students should read the original play and also the paraphrase given in the school prescribed textbook]Summary Questions amp Answers

Conspiracy of Antonio and Sebastian (Contd)

o As they approach Ariel appears again and wakes up Gonzalo by singing a tune in his ear Alonso also wakes up and they see both Sebastian and Antonio with drawn swords On being caught off guard they make up a story saying that they had heard a bellowing of bulls or lions

o They then moved to another part of the island

o Ariel at once rushes to Prospero to inform him of this development

SUMMING-UP of ACT-2 SCENE-1

(i) Among the survivors Ferdinand is separated from the rest which results in the disconsolate grief of Alonso as he took him for dead

(ii) The villainy of Antonio is confirmed

(iii) The supremacy of Prosperorsquos magic which resulted in the failure of the human conspiracy

(1)

(Act II Sc 1 L 311-325)SEBASTIAN Whiles we stood here securing your repose

Even now we heard a hollow burst of bellowing Like bulls or rather lions Didt not wake youIt struck mine ear most terribly

ALONSO I heard nothingANTONIO O rsquotwas a din to fright a monsters ear

To make an earthquake Sure it was the roarOf a whole herd of lions

ALONSO Heard you this GonzaloGONZALO Upon mine honour sir I heard a humming

And that a strange one too which did awake meI shaked you sir and cried As mine eyes opened I saw their weapons drawn There was a noiseThats verily rsquoTis best we stand upon our guardOr that we quit this place Lets draw our weapons

(i) Why has Prospero sent Ariel to Gonzalo and Alonso What does Ariel do to awaken Gonzalo

Prospero has already come to know by his magic powers the danger which threatens Gonzalo who had been Prosperorsquos friend and so he sent Ariel to preserve the lives of both Gonzalo and Alonso Prospero does not want that his scheme should remain unfulfilled Ariel begins to sing a song in Gonzalorsquos ears to awaken him(ii) Who are ready to carry out their plan Who takes steps to stop them Why does Gonzalo feel surprised after being awakened

Sebastian and Antonio are ready to carry out their plans They are standing with their swords drawn to kill Alonso and

(iv) We see two sets of contrasting characters Gonzalo-Adrian against Antonio-Sebastian

(v) The grief that works in Alonso can be perceived to his repentance for his association in Antoniorsquos crime against Prospero

Gonzalo Ariel takes steps to stop them from carrying out their nefarious scheme When Gonzalo is awakened by the song sung by Ariel into his ears he (Gonzalo) feels surprised because he sees Sebastian and Antonio standing with their swords drawn(iii) What reason do Sebastian and Antonio tell of drawing their swords when they are suspected by Alonso and Gonzalo

When Sebastian and Antonio are seen with their swords drawn they are looked with suspicion by Gonzalo and Alonso At first Sebastian tells them that as they stood here to guard them during their sleep they heard only a little before a sudden loud noise very much like the roaring of bulls or more probably that of lions Then Antonio follows him saying that this was a noise so terrible as to frighten even a monsterrsquos ears and this noise could even have shaken the earth and it was surely like the roaring of a multitude of lions Then seeing the danger they have drawn their swords Perhaps after hearing the terrible noise they (Gonzalo and Alonso) woke up from their sound sleep

(iv) What does Gonzalo tell Alonso about the strange noise What did he see on opening his eyes Gonzalo tells Alonso that he did not hear the sound of roaring but he heard a humming sound which was strange and which woke him up After waking up he gave him (Alonso) a shaking and a loud cry On opening his eyes he saw these two gentlemen standing with their swords drawn(v) What does Gonzalo suggest

Gonzalo suggests that there was a noise indeed and of that he has no doubt at all and suggests that the best course for them would be to remain alert and vigilant against any possible danger to their lives or to leave this place and move to some other part of the island

Class XIISubject Topic Summary Execution

Commerce

Chapter- Management

Today we will discuss about LEVELS OF MANAGEMENT

Levels of management is a series or chain of managerial positions from top to bottom It helps individuals to know their authority responsibilities and superior-subordinate relations among themselves There are mainly three levels of Management TOP LEVEL MANAGEMENTMIDDLE LEVEL MANAGEMENTLOWER LEVEL MANAGEMENT

Top level managementIt consists of members at the highest level in the management hierarchy This level includes Board Of Directors Chief Executive Managing Directors Chairman President Vice President

Rolefunctions of the top levelmanagement1To analyse evaluate and deal

with theexternal environment2 To determine the objectives and

policies of the business3 To strive for welfare and survival

of business

4 To create an organisational Framework consisting of authority responsibility relationship

Middle level management Congress of members or groups who are concerned with implementation of the policies let down by the top managementThis level includes head of the department such as finance manager marketing manager branch and regional managers departmental and divisional heads plant superintendent etc

Role of functions of the middle level management

1 To interpret the policies framed by top management

2 To assign duties and responsibilities to lower level managers

3 To select and appoint employees for middle and supervisory level and evaluate their performance

4 To co-operate with other departments for smooth functioning

Operational or supervisory level managementIt refers to the group are members who are concerned with execution of the work They are also known as fast line managers This level includes supervisor 4 men Section Officer clerk Inspector etc

Role of functions of the lower level management1 To plan and execute day-to-

day operations2 To supervise and control the workers3 To arrange materials and

tools to start the process and make arrangements for training

4 Today present workers grievance and suggestions before the management and

ensure safe and proper working conditions in the factory

Business Studies

Staff Appraisal Chapter- 10 Today let us start with a new chapter

Staff Appraisal

Meaning of Performance Appraisal

Performance Appraisal is the systematic evaluation of the performance of employees and to understand the abilities of a person for further growth and developmentThe supervisors measure the pay of employees and compare it with targets and plansThe supervisor analyses the factors behind work performances of employeesThe employers are in position to guide the employees for a better performance

Objectives of Performance Appraisal

Following are the objectives of Performance Appraisal

To maintain records in order to determine compensation packages wage structure salaries raises etc

To identify the strengths and weaknesses of employees to place right men on right job

To maintain and assess the potential present in a person for further growth and development

To provide a feedback to employees regarding their performance and related status

To provide a feedback to employees regarding their performance and related status

Importance of Performance Appraisal

Performance appraisal provides important and useful information for the assessment of employees skill

knowledge ability and overall job performance The following are the points which indicate the importance of performance appraisal in an organization

1 Performance appraisal helps supervisors to assess the work performance of their subordinates

2 Performance appraisal helps to assess the training and development needs of employees

3 Performance appraisal provides grounds for employees to correct their mistakes and it also provides proper guidance and criticism for employees development4 Performance appraisal provides reward for better performance

5 Performance appraisal helps to improve the communication system of the organization

6 Performance appraisal evaluates whether human resource programs being implemented in the organization have been effective

7 Performance appraisal helps to prepare pay structure for each employee working in the organization

8 Performance appraisal helps to review the potentiality of employees so that their future capability is anticipated

Geography

DRIANAGE The SubarnarekhaThe Subarnarekha and the Brahmaniinterposed between the Ganga and the Mahanadi deltas drain an area of 19300 sq kmand 39033 sq km respectively The drainage basins of these streams are shared byJharkhand Odisha west Bengal and Chhattisgarh The Brahmani is known as southKoel in its upper reaches in Jharkhand

The NarmadaThe Narmada rises in the Amarkantak hills of MadhyaPradesh It flows towards the West in a rift valleyformed due to a geological fault The total length of it is 1300 km All the tributaries of the

Q1 Name the two westward flowing rivers in the peninsular plateauA1 Narmada and Tapi are the only westward flowing rivers of the peninsular plateau

Q2 Differentiate between east-flowing rivers and west-flowing riversA2

East-flowing rivers

West-flowing rivers

Narmada are very short inlength Most of its tributaries join the main streamright anglesThe Narmada basin covers parts of Madhya Pradesh and Gujarat

The Tapi The Tapi rises in the Satpura ranges in the Betul listrictof Madhya Pradesh It flows in a rift valley parallel tothe Narmada but it is much shorter in length It coversparts of Madhya Pradesh Gujarat and MaharashtraThe length is about 724 km

The Sabarmati and the MahiThe Sabarmati rises in the Aravali hills and flows south-south-westwards for a distance of 300 kilometres to the Arabian Sea The Sabarmatibasin extends over an area of 21674 sq km in Rajasthan and Gujarat The Mahi rises inthe east of Udaipur and drains an area of 34842 sq km lying in Madhya PradeshRajasthan and Gujarat It flows south-westwards for a distance of 533 km before it fallsinto the Gulf of Khambhat

The ChambalThe Chambal rises near Mhow in the Vindhya Range and flows towards the northgenerally in a gorge upto Kota Below Kota it turns to the north-east direction and afterreaching Pinahat it turns to the east and runs nearly parallel to the Yamuna beforejoining it in the southern part of the Etawah district in Uttar PradeshMajor Rivers of India with their basin area (Sqkm)

Himalayan System Indus 321290Ganga 861404

Brahmaputra 187110Indus System

Jhelum 34775Beas 20303

Ganga System Yamuna 366223Ghaghra 127950

Peninsular RiversNarmada 98796

Tapi 65145Mahanadi 141600

Subarnarekha 19300Sabarmati 21674

Mahi 34842Godavari 312812

Godavari Krishna Kaveri Mahanadi are the east-flowing rivers

Narmada Tapi west-flowing rivers

They fall into the Bay of Bengal

They fall into Arabian Sea

These rivers form big deltas

These rivers form comparativelysmall deltas

Catchment areas of these rivers are larger

Catchment areas of these rivers are smaller

Krishna 2589488Cauveri 87900

Subject ndashBiology Topic ndashChapter -5 Inheritance amp Variations Summary ExecutionToday we will discussabout linkage and its classification

LINKAGE The tendency of the genes located on the same chromosome to stay together is

hereditary transmission Linked genes the genes responsible for this Genes that exhibit the process of linkage locates in the same chromosome The distance between the linked genes in a chromosome determines the strength

of linkage i e genes that are located close to each other show stronger linkage than that are located far from each other

COMPLETE LINKAGE It is the type of linkage showed by the genes that are closely located or are tightly

linked with each other as they have no chance of separatingby crossing over These genes are always transmitted together to the same gamete and the same

offspring In such condition only parental or non cross over type of gametes are formedINCOMPLETE KINKAGE It is type of linkage showed by the genes that are distantly located orare loosely

linked with each other because they have chance of separating by crossing over

SIGNIFICANCE i) It helps in holding the parental character togetherii) It checks the appearance of new recombination and helps in bringing the

hybrid population which resembles the original parents iii) Linked genes dilute the effects of undesirable traits

Subject Eng Literature (The Tempest ndash William Shakespeare) Topic Essay Questions (EQ-3)Question No 3

Give a character sketch of CalibanAnswer

The character of Caliban has been wonderfully conceived by Shakespeare as the manifestation of all that is gross and earthy ndash a sort of creature of the earth as Ariel is a sort of creature of the air

Calibanrsquos Physical Appearanceo Caliban is lsquofreckledrsquo a lsquomisshapen knaversquo not honoured with human shape

o Prospero calls him lsquothou tortoisersquo (Act I Sc 2 Line 317) Trinculo stumbling upon him describes him as ldquoA strange fish hellip Legged like a man And his fins like armsrdquo He ldquosmells like a fishrdquo (Act II Sc 2 Line 25)

o Prospero also calls him a ldquobeastrdquo (Act IV Sc 1 Line 140) and ldquoThis misshapen knaverdquo (Act V Sc 1 Line 268)

o Further it appears that in addition to his physical deformity his spiritual inferiority is also suggested by Prosperorsquos claim that his birth resulted from the union between his mother the witch Sycorax and the devil

Calibanrsquos ParentageWhen the play opens Caliban is twenty four years of age having been born on the island twelve years before the coming of Prospero His mother was the foul witch Sycorax who was banished from Algiers for ldquomischiefs manifold and sorceries terrible to enter human hearingrdquo (Act I Sc 2 Line 264) and the father was the Devil himself Thus

Caliban is a monster of evil and brute nature ugly deformed and stinking

Calibanrsquos Savage and Malignant Natureo Caliban is entirely a creature of the earth ndash gross brutal and savage He regards himself as the rightful possessor

of the island and Prospero as a usurper

o In his young age he was on good terms with Prospero He had consented to be received by Prospero at his house and to be educated by him He has learnt human language only to curse his master whom he abhors

o His beastly nature soon breaks out and ends in a vicious attack on Miranda This opens the eye of Prospero who becomes severe to him and enforces his service by threats and violence

o Prospero uses him to make dams for fish to fetch firewood scraper trenches wash dishes and keep his cell clean

Calibanrsquos Hatred for ProsperoA profound hatred for Prospero has taken hold of Caliban It springs from a sense of his being dispossessed and ill-treated He would kill Prospero if he could but he knows the power of Prosperorsquos lsquobookrsquo Hence he transfers his allegiance to Stephano who seems like a god to him He also incites the two drunken associates to batter the skull of Prospero when he sleeps in the afternoon

Caliban Shows Considerable Intelligenceo He has learnt Prosperorsquos language

ldquoYou taught me language and my profit onrsquot (Act II Sc 2 Lines 86-89)Is I know how to curserdquo

o He is well aware of the futility of arguing with one who has more power than he has

ldquoI must obey his art is such power (Act I Sc 2 Lines 373-376)It would control my damrsquos god SetebosAnd make a vassal of himrdquo

o He realizes the importance of Prosperorsquos books

ldquoRemember (Act III Sc 2 Lines 89-92)First to possess his books for without themHersquos but a sot as I am nor hath notOne spirit to commandrdquo

o He knows the value of stealth when attacking the enemy

ldquoPray you tread softly that the blind mole may not (Act IV Sc 1 Lines 194-195)Hear a foot fall we now are near his cellrdquo

o Caliban has a better set of values than Stephano and Trinculo They are distracted from their plan by their greed for Prosperorsquos rich garments Only Caliban realizes that such a finery is unimportant

ldquoLeave it alone thou fool it is but trashrdquo (Act IV Sc 1 Lines 224)

Caliban is not a good judge of characterCaliban is not a good judge of character He decides for example that Stephano is a god because he dispenses lsquocelestial liquorrsquo (Act II Sc 2 Line 115) but then it must be remembered that he has only known his mother Sycorax Prospero Miranda and the spirits that torture him However he quickly discovers his error of judgementrdquo

ldquoWhat a thrice-double ass (Act V Sc 1 Lines 295-297)Was I to take this drunkard for a godAnd worship this dull foolrdquo

Calibanrsquos Imaginative NatureIf Caliban is sub-human in what has been said above he is human in the respect of the poetic side of his character He listens to music with rapture He tells of the beautiful dreams in which heaven rains treasures upon him and which upon waking he yearns to renew One of the most poetic passages in whole play is Calibanrsquos description of the island

to Stephano and Trinculo

ldquoBe not afeard The isle is full of noises (Act III Sc 2 Lines 135-143)Sounds and sweet airs that give delight and hurt notSometimes a thousand twangling instrumentsWill hum about mine ears and sometime voicesThat if I then had waked after long sleepWill make me sleep again and then in dreamingThe clouds methought would open and show richesReady to drop upon me that when I wakedI cried to dream againrdquo

Caliban - Less Ignoble Than Some OthersCalibanrsquos motive for murder is less dishonourable than that of Antonio and Sebastian They plan to kill Alonso to gain his power and wealth Caliban merely wants revenge and the return of lsquohisrsquo island

Conclusiono Calibanrsquos character is not portrayed very clearly in the play and hence we cannot decide whether he is a poor

savage being grossly maltreated by Prospero or whether he is evil and must therefore be kept in bondage or enslavement

o Caliban is contrasted with Ariel who is a spirit and thus swift and uninterested in physical activitieso Caliban is also contrasted with Prospero who is the all-powerful master of the island and of the destiny of all

those on the islando Caliban is also contrasted with civilized man showing him to be less evil than Antonio and Stephano and less

materialistic than Stephano and Trinculoo Caliban has suffered at the hands of Prospero and he has learnt to curse by listening to Prosperorsquos abuse He

certainly believes that Prospero has deprived him of his birthrighto Finally the character Caliban is thought to be one of Shakespearersquos masterpieces The complexity of the character

is reflected in the large volume of critical discussion that has grown around it

ECO ndash12 Topic-Forms of market

MonopolyMonopoly is a market structure in which there is a single seller there are no close substitutes for the commodity produced by the firm and there are barriers to entry Example Indian Railways which is operated under government of India Monopoly also implies absence of competitionFeatures of Monopoly Monopoly is characterized by1 Single Seller In monopoly there is only one firm producing the product The whole industry consists of this single firm Thus under monopoly there is no distinction between firm and industry Being the only firm there is significant control of the firm over supply and price Thus under monopoly buyers do not have the option of buying the commodity from any other seller They have to buy the product from the firm or they can go without the commodity This fact gives immense control to the monopolist over the market

2No Close Substitute There are no close substitutes of the product produced by the monopolist firm If there are close substitutes of the product in the market it implies presence of more than one firm and hence no monopoly In order to ensure a total of control over the market by the monopolist firm it is assumed that there are no close substitutes of the product

3 No Entry amp Exit Monopoly can only exist when there is strong barriers before a new firm to enter the market In fact once a monopoly firm starts producing the product no other firm can produce the same One reason for this is the ability of the

monopolist to produce the product at a lower cost than any new firm who thinks to enter the market If a new firm who knows that it cannot produce at a lower cost than the monopolist then that firm will never enter the market for fear of losing out in competition Similarly the monopolist who is operating for a long time may be enjoying reputation among its customers and is in a better position to use the situation in its own benefit A new firm has to take long time to achieve this and so may not be interested to enter the market

4 Price Maker Being the single seller of the product the monopolist has full control over the pricing of the product On the other hand if there is a large number of buyers in the market so no single buyer exercises any significant influence over price determination Thus it is a sellerrsquos market So monopoly firm is a price maker

5 Price Discrimination Having considerable control over the market on account of being single seller with no entry of other firms the monopolist can exercise policy of price discrimination it means that the monopolist can sell different quantities of the same product to a consumer at different price or same quantity to different consumers at different prices by adjudging the standard of living of the consumer

6 Shape of Demand Curve Since a monopolist has full control over the price therefore he can sell more by lowering the price This makes the demand curve downward sloping

Subject Ac-12 290620 Topic- retirement Model sumThe Balance Sheet of Rohit Nisha and Sunil who are partners in a firm sharing profits according to their capitals as on 31st March 2014 was as under

Liabilities Amount Assets Amount (Rs) (` Rs)

Creditors 25000 Machinery 40000Bills Payable 13000 Building 90000General Reserve 22000 Debtors 30000Capital Less Provision for Rohit 60000 Bad debts 1000

29000 Nisha 40000 Stocks 23000 Sunil 40000 140000 Cash at Bank 18000

200000 200000

On the date of Balance Sheet Nisha retired from the firm and following adjustments were made(i) Building is appreciated by 20(ii) Provision for bad debts is increased to 5 on Debtors(iii) Machinery is depreciated by 10(iv) Goodwill of the firm is valued at Rs 56000 and the retiring partnerrsquos share is adjusted

(v) The capital of the new firm is fixed at Rs120000 Prepare Revaluation Account Capital Accounts of the partner and Balance Sheet of the new firm after Nisharsquos retirement Revaluation AccountDr Cr

Particulars Amount Particulars Amount (`Rs) (Rs`)

Provision for Bad debt Ac 500 Building Ac 18000Machinery Ac 4000Profit transferred toCapital Accounts (3 2 2)Rohit 5786Nisha 3857Sunil 3857

13500

18000 18000

Capital Account

Dr Cr

Particulars Rohit Nisha Sunil Particulars Rohit Nisha Sunil (Rs`) (Rs`) (`Rs) (Rs`) (Rs`) (Rs`)

Sunilrsquos Capital ac 9600 mdash 6400 Balance bd 60000 40000 40000Bank - 66143 - General Reserve 9428 6286 6286Balance cd 72000 mdash 48000 Revaluation (Profi 5786 3857 3857 Rohitrsquos Capital Ac mdash 9600 mdash

Sunilrsquos Capital Ac 6400 Bank 6386 - 4257

81600 66143 54400 81600 66143 54400

Balance Sheet as at 31st March 2014

Liabilities Amount Assets Amount (Rs`) (Rs`)

Creditors 25000 Building 108000Bank overdraft 37500 Machinery 36000

Bills Payable 13000 Debtors 30000Capital Less ProvisionRohit 72000 for Bad debts 1500 28500Sunil 48000 120000 Stock 23000

195500 195500

Working Notes (i) (a) Profit sharing ratio is 60000 40000 40000 ie = 3 2 2(b) Gaining Ratio Rohit = 35 ndash 37 = 2135 ndash 1535 = 635Sunil = 25-27 = 1435 ndash 1035 = 435= 635 435= 6 4 = 3 2(c) Nisha Share of Goodwill = Rs 56000 times 27 = Rs16000Share of Goodwill in the gaining ratio by the existing partner ieRohit = Rs16000 times 35 = Rs 9600Sunil = Rs 16000 times 25 = Rs 6400

The journal entry isRohitrsquos Capital Ac Dr 9600Sunilrsquos Capital Ac Dr 6400 To Nisharsquos Capital Ac 16000(Share of Goodwill divided into gaining ratio)

  • 1 Static Friction
  • The frictional force that acts between the surfaces when they are at rest with respect to each other is called Static Friction
    • Static Friction Examples
      • 2 Sliding Friction
        • Examples Of Sliding Friction
          • 3 Rolling Friction
            • Examples Of Rolling Friction
              • Objects and Reasons of the Forest Conservation Act
Page 2:  · Web viewSubject . Topic . Summary . Execution . English 1 . Sounds of animals . Hens –cackle Horses –neigh Lions –roar Owls –hoots Snake –hiss. English 2 . Mother’s

পাঠndash২৩লপndashযমলার মজাঅনশীলনীর পরকে4াততর

খ) রকের হিন সবাই( পাপ যবগহিন ) যখকেত চায়) ( কাকেঠর কাকেজর যতকেল -াজার ) যাকাকেনর সামকেনই যবহিশ হি-ঘ) নারকোলা ( পাচবার শবার ) যঘারার পর যকেম যকেলাঙ) ( -াই যবান ) নার যালা চার বায়না ধরকেলা৪ হিবপরীত শবদ -কের timesকাকে যবাকা timesচালাক উপকেরtimes হিনকেচ রাহিজ timesঅরাহিজ য-তকের timesবাইকের৫ একেলাকেমকেলা শবদ সাহিজকেয় যলকেখা -যল-াকেতজা = যতকেল-াজা যয়ানআজ = আকেয়াজন কাকোন =

যাকান হিলহিজহিপ = হিজহিলহিপ যানারলা = নারকোলা কমারহির = রকমাহির

Class IISubject Topic Summary Execution Eng literature

Ch 2 FriendshipBook Orchid

Refer to pg 15Once there lived a crow a tortoise and a mouse They were very good friends and lived in the forest Once they saw a deer running and coming towards them They understood that the deer was afraid of the hunters so the tortoise told him that the hunters donrsquot come inside this deep forest They wanted the deer to be their friend and they four became good friends Once when they didnrsquot find the deer anywhere they became worried and started looking for him Suddenly the crow saw the deer trapped in the net near river bank and he at once went to inform his other friends

Read pg 15

Eng language

Ch 13 Adverbs Adverbs are words that tells us something more about the verb That is how an action happenedWe add ly to form adverbs For eg A tortoise walks slowly[The word walks is a verb and the word slowly is an adverb which is telling how the tortoise is walking]

Read pg 61

EVS Ch 8 Safety First Refer to pg 139Safety in swimming pool and playgroundChildren the things that we should keep in mind while we are in the swimming pool are that as you are very small never go alone near ituse swimming tube no running near the pool as you might slip and fall into the water wait for your turn and never push each other in the pool If you follow these small safety rules then you can avoid many accidents to happen

Read pg 139 and try to understand

Mathematics

CHAP-6SHAPES

SOLID SHAPES ARE CALLED 3-D SHAPESSOLID SHAPES INCLUDES FACESEDGES AND CORNERS

READ AND UNDERSTAND THE SUMMARY PART

FACE- PART OF THE SHAPE THAT IS FLAT (OR CURVED) IS KNOWN AS FACE OF THE SOLID SHAPE A CUBE HAS 6 FACES

EDGE- THE LINE WHERE TWO FACES MEET IS KNOWN AS EDGEA CUBE HAS 12 EDGES

VERTEX-THE POINT WHERE TWO OR MORE EDGES MEET IS KNOWN AS VERTEXA CUBE HAS 8 VERTICES

General knowledge

Ch 21 part of plants

Refer to pg 25Do in the book and learn [ practise drawing a plant and label the different parts of the plant]

Qs 1 Which part of the plant grow under the groundAns Roots Qs2 Which part of the plant looks colourfulAns FlowerQs 3 Which part of the plant look greenAns Leaves Qs 4 Which part of the plant has leaves flowers and fruitsAns StemQs 5 Which part of a plant do you eatAns Fruit

Bengali বইndashবাংলা সাহিতয পহিরচয়

পাঠndash৭লপndashবকোহিনঅনশীলনীর পরকে4াততর

৩ সঠিক উততর যবকেনাও -ক) ( হিসটানকের বাঙাহিলকের ) সবকেচকেয় বকো উৎসব বহিনখ) এই হিনটি ( জরকেতর হিজশহিকেসটর জনমহিন) এইহিনটি পরম ( আনকেFর ঃকেখর ) হিনঘ) হিজGায় ( বাইকেবল যকারান ) পাঠকরায়ঙ) বহিকেনর হিপরয় খাবার কেলা ( যকক হিখচহি ) ৪ হিবপরীতশবদ -বকো timesযাকেIা জনম timesমতয আনFtimes ঃখ নতন timesপকেরাকেনা হিপরয় timesঅহিপরয় হিশশ times বদধপহিবতর times অপহিবতর আকেলা times অনধকার সমাহিNtimes সচনা৫ একেলাকেমকেলা শবদ সাহিজকেয় যলকেখা -যাহিবন = বকোহিন ইকেববাল = বাইকেবল নমজনহি = জনমহিন পারউ = উপার হিনয়মহিব = হিবহিনময় পাসউনা = উপাসনা

Hindi सवनाम अपन लिलए म हममरा हमारा मझ आदिद शबदो ा परयोग रत ह जि0सस बात र रह हउस लिलएmdashतम तमहारा तमह आप आपा आदिद शबदो ा परयोग रत ह जि0स बार म बात र रह ह या किसी अनय लिलएmdashवहव उस उना उनह उनहोनआदिद शबदो ा परयोग किया 0ाता ह ौन ोईकया आदिद शबद भी सवनाम ह

Read the chapter

Class IIISubject Topic Summary Execution COMPUTER

DRAWING IN MS PAINT

Q3) WHAT TOOL DO YOU USE TO INSERT TEXT IN MS PAINTANS) THE TEXT TOOL IS USED TO ADD TEXT IN MS PAINTQ4) WRITE THE STEPS TO USE THE ERASER TOOLANS) THE STEPS ARE

CLICK ON THE ERASER TOOL IN THE TOOLS GROUP DRAG THE MOUSE POINTER OVER THE AREA YOU WANT TO ERASE

বই ndash বাংলা সাহিতয পহিরচয়

পাঠ ndash ৮লপ ndash IনIহিন ও রাজার কাযলখক ndash উকেপনদরহিককেশার রায়কেচৌধরীঅনশীলনীর পরকে4াততর

৯ বাকয রচনা ককেরা -জবদ ndash সবাই হিমকেল যচারকেক পহিলকেশর াকেত তকেল হিকেয় জবদ করকেলা হিখলহিখল ndash নতন যখলনা যপকেয় হিশশটি হিখলহিখল ককের যকেস উঠকেলা কাতর ndash কাতর ককেS বদধ যলাকটি জল চাইহিল IফI ndash আঘাত লাায় ককরIা IফI করহিল -য়ানক ndash যকারনাকে -য়ানক পরলয় কেয়হিল

Hindi सवनाम अपन लिलए म हममरा हमारा मझ आदिद शबदो ा परयोग रत ह जि0सस बात र रह हउस लिलएmdashतम तमहारा तमह आप आपा आदिद शबदो ा परयोग रत ह mdashजि0स बार म बात र रह ह या किसी अनय लिलए वह व उस उना उनह उनहोन आदिद

शबदो ा परयोग किया 0ाता ह ौन ोई कया आदिद शबद भी सवनाम ह Science

Ch ndash Food we get from Plants

We see many plants around us They are of different shapes and sizes Some plants grow tall some stay short Some plants grow along the ground some plants need support to grow

Trees ndash big and tall plants are called trees Trees have thick hard and woody stem called trunk They have thin stems called branches Most trees live for many years A banyan tree can live for hundreds of years Mango banyan neem are examples of trees

Fill in the blanks (pg no- 63)

The main stem of a tree is called trunk

Cotton and hibiscus are examples of shrubs

Rice and mint are examples of herbs

Social studies

Indian literature Jataka talesThe jataka tales are short educational tales They have stories concerning the previous births of Gautam Buddha in both animal and human form These moral tales were written in fourth century and are based on the life of Gautama Buddha These are a collection of about 550 fables Every fable exhibits some virtue where

Buddha may appear in it as a king God or an elephant There are several translations of these Jataka tales in different languages

Read the portion which has been explainedEng language

Ch 19 Adverbs Adverbs are words that tells us more about an action In other words adding something more to the verb For eg The cat ran quicklyhere the word ran is the verb as it is showing an action done by the cat and quickly is the adverb as it is telling how the cat ran

Refer to pg 68Do in the bookWarm upUnderline the words that tell you how an action happened

The cat ran quicklyThe man ran swiftlyIt hurriedly went into its hole The mouse sat safely in its houseThe cat squeaked loudly

MAT

HEM

ATIC

S

Ch 7

Frac

tions

A fraction is a number that stands for parts of a whole object or a collection of objects

Each fraction has two numbers separated by a rule(a) The number above the rule is called numerator(b) The number below the rule is called denominator

Example 7minusminusminusminusrarr Numerator

13minusminusminusminusrarr Denominator

Notes

(1) 05 is a fraction in which numerator is 0

(2) There is no fraction whose denominator is zero

(3) When the numerator and denominator are equal the fraction represents the whole or 1

Example 33=1and2

2 = 1

Exercise ndash 421 Read the following fractional numbers

(c) 411 (d)

815 (e)

2337

Solution

(c) 411

four by eleven

(d)8

15Eight by fifteen

(e) 2337

twenty three by thirty seven

2 Write the following fractional numbers(d) Three over eleven (e) Eighteen by twenty one (f) Four-seventhsSolution

(d) Three over eleven 311

(e) Eighteen by twenty one 1821

(f) Four-sevenths 47

3 Write the numerator of each of the following

(e) 09 (f)

56 (g)

311 (h)

617

Solution (e) 0 (f) 5 (g) 3 (h) 64 Write the denominator of each of the following

(e) 08 (f)

813 (g)

719 (h)

417

Solution (e) 8 (f) 13 (g) 19 (h) 17

5 Write the fractions whose numerators and denominators are given below

(a) Numerator = 4 and Denominator = 13 (b) Numerator = 6 and Denominator = 17(c) Numerator = 11 and Denominator = 16(d) Numerator = 0 and Denominator = 9

Numerator Denominator

Fraction

(a) 4 13 413

(b) 6 17 617

(c) 11 16 1116

(d) 0 9 09

6 Find the fractions in which the denominator is 3 more than the numerator69

74

811

49

1113

711

1411

1613

2023

Solution 69

811

2023

8 A book seller had 15 books He sold 8 books What fraction of the books did he sell

Solution Out of 15 books the book seller sold 8 books

there4 The fraction of the books he sold =8

15

10 A class has 35 students On Monday 34 students were present What fraction of the students was absent

Solution Total number of student = 35On Monday 34 students were presentthere4 The number of student absent on Monday = (35 ndash 34) = 1

So fraction of the students was = 135

Class IVSubject Topic Summary Execution English language

Verbs the -ing form

Meaning of verb A word used to describe an action is known as verb

The ndash ing form of a verb is a very important form for it is used in many different waysExampleJack is playing(Present continuous)Jack was playing with his friends yesterday ( Past continuous)Jack will be playing in a match tomorrow ( Future continuous)So we see that the ndash ing form of verb used to form the continuous tenses

The ndash ing form of verb can also be used as an adjectiveExampleYou should not disturb a sleeping dogThe kettle is full of boiling water

Say which of the ndash ingforms of verbs in the sentences given below have been used as adjectives and which to form continuous tenses ( solved exercises please follow this)

3Interesting ndash adjective

4Was eating ndash past continuous tense

5terrifying ndashadjective

6are helping is spreading ndash present continuous tense

7will be visiting ndashfuture continuous tense

8twittering ndash adjective

Social studies

Map reading A Map helps us to see the whole World continents countries cities and neighborhood They are of different sizes handy and can be rolled up folded or carried easily However the maps do not show the surface accurately This is because the map is flat whereas the Earth is round in shape Cartography or a map making is a study and practice representation of the

One word answers1 It helps us to see the whole world countries and cities ndash Map

2 A study and practice representation of the Earth on a flat surface ndash Cartography

Earth on a flat surface

Elements of a mapTo be able to read a map we must understand the elements of a map

DirectionsDirections are the basic guidelines that help us to locate places It is very important for us to know the correct directions East West North and South are the four Cardinal directions These directions help us to locate the places Beside these there are four sub directionsndash Northeast Southeast North West and South West Compass is an instrument that helps to locate the directions

3 The basic guidelines to help us to locate places ndash directions

4These four directions help us to locate the places ndash Cardinal directions

COMPUTER

Formatting in ms word

Q3) What do you mean by formattingAns) Formatting is a general arrangement of text in a document We can change the appearance of a document by using the features available in ms word We can use different fonts colours and styles in the textQ4) What is alignmentAns) Alignment of text is the way in which it is placed between the margins of a page Text can be aligned to the left side in the centre or to the right side of a pageQ5) what is fontAns) A font is a style of writing and typing A font provides specific textual appearance to the document You can change the size style or give various effects to a font

Hindi 2ndlang

ाला किहरण सभी धमlt ा मल धम ह दया और रणा मरी हर 0ीव अपन ही दकिनया म रहत ह हम

पराणी कयो उनो परशान रत ह ऐसा हम नही रना चाकिहए

0ानवरो स हम ए सीख मिमलती ह किस तरह हम एता म रहना चाकिहए

हर पराणी पयार ी भाा समझता ह अगर हम पयार ी उममीद रत ह तो कया 0त

0ानवर हमस उममीद नही रत कि हम भी उनह पयार द यह हानी ए ाल किहरण ी ह 0ो अपन

समह ा नततव रता था हमशा कया उलिचत ह यह सोचता था अचान

उन पर लिशारिरयो न हमला बोल दिदया जि0सम ाला किहरण परा गया कयोकि वह

दलभ ह और लिशारिरयो ी न0र उसी पर थी परत 0ानवरो ी एता ो दखर

रा0मार ो दया आ गई और उसन उनो छोड दिदया 0ानवर भी बईमान नही थी वह उनस अकसर मिमलन उन बाद म 0ाता रहता रा0ा ो धनयवाद रता अबोध म

पराणिणयो म एता तजञता अभी भी ह

शबदाथ तराई ndash पहाड आसपास नीच

ी भमिमवश- ल या परिरवारअदवभत- अनोखाचौननाndash सत सावधानओझलndash गायबकिवपणिRndash मसीबतसगठन- एताउममीदndash आशाातर दमिU ndashकिववशता

हिहस ndash हानी पहचान वालातवयndash फ0

दल ndash समहनततवndash सचालन रना

ओट ndash आडआतमसमपणndash अपन ो सौप दनाम- चप

हम उनस सीख लनी चाकिहएবইndashবাংলা সাহিতয পহিরচয়

পাঠndash১০লপndashহিবকেবকানকেFর যকেলকেবলাযলখকndashশশী-ষণ াশগNঅনশীলনীর পরকে4াততর

৯প পহিরবতG ন ককেরা -শরীর = শারীহিরক -ত = য-ৌহিতক সG ার = সG াহির সথায়ী = সথাহিয়তবহিবশবাস = হিবশবাসী া = যকো১০ বাকয রচনা ককেরা -আডডা = পাার বদধরা াতলায় আডডা যয়হিডবাহিজ = রাম া যকেক হিডবাহিজ যখকেয় পকেলাসG ার = বকেলহিল যকেল যর সG ারবকহিন = অঙক -ল করায় রীনা মাকেয়র কাকে বকহিন যখকেলাবহিদধ = বহিদধ াককেল উপায় য়হিনশহিত = হিনশহিত রাকেত রাজবাহিকেত ডাকাত পকেলা-য়ানক = পাাহি রাসতা বষটির পকের -য়ানক হিবপ জনক য়হিবশবাস = মানকেষর পরহিত হিবশবাস ারাকেনা পাপ

Science Ch ndash The Food We Eat

Living things need food to live to grow to stay strong and fit When we need food we feel hungry Food gives us energy to do work It also protects us from diseases and helps us to stay healthy Nutrients in food The food we eat contains many substances that are necessary for our body These substances are called nutrients that help us to grow well and stay healthy

Nutrients give us energy to study work and play

They help our body to grow and repair the damaged parts of our body

They also help our body to fight against diseases and remain healthy

Write T for True or F for False (pg no- 11)

1 Food contains nutrients that help us to grow True

2 Foods rich in carbohydrates are called body-building foods False

MAT

HEM

ATIC

S

Ch 9

Com

mon

Fra

ction

s

A fraction is a number that stands for parts of a whole object or a collection of objects

Each fraction has two numbers One is written above the other separated by a line The one above the line is called numerator and the one below the line is called denominator

Example 5minusminusminusminusrarr Numerator

11minusminusminusminusrarr Denominator

Exercise ndash 362 In following fill in the blanks

(b) 37 ___ is denominator ___ is

numerator

(c) 89 ___ is denominator ___ is

numerator

Solution

(b) 37 7 is denominator 3 is numerator

(c) 89 9 is denominator 8 is numerator

3 Write down the fractions whose numerators and denominators are given below in the bracket The first number stands for numerator and the second number standing for denominator

(25) 25

(311) 311

(416) 416

(712) 712

Class VSubject Topic Summary Execution

Science Ch ndash PlantReproduction

In pollination chapter we have learnt that the flowers change into fruits and the fruits bear seeds Now we are going to learn how this process takes place

FertilizationThe process of fusion of the male reproductive cell (male gamete) and female reproductive cell (female gamete) is known as fertilizationWhen a pollen grain reaches from the anther to a stigma it begins to grow and forms apollentube The Pollen tube then travels down through the style to enter an ovule inside the ovary On reaching the ovule male reproductive cell in the pollen grain unites with the egg cell present in the ovule

Books exercise

A) Tick the correct answer

1Which of the following do ovules change into after fertilization ndashseeds

2Which of the following is not a part of the seed ndash flower

3Which of the following condition is needed for germination of a seed ndash all ofthese

English language

Sentences phrases and

Solved exercisesSay which of the underlined groups of words are phrases and which clauses

clauses8 In her new clothes ndashphrasesAs pretty as a doll ndash phrases

9 looking sad and upset ndash phrasesHe had lost all the tickets for the Test Match ndash clauses

10 During the vacation now only a month away ndashphrases

11 too sweet and too hot ndashphrases

12 At the Olympic Games ndashPhrasesOf Laurel leaves ndash phrases

13 Who played the role of Hamlet ndashclauses

14 However fast ndash phrases

15 When the men fell asleep ndash clausesSocial studies

Indian Government

Lok Sabha (lower house) ndash It has 552 members Of these 530 membersrepresent States 20 members represent the union territories and two members represent the Anglo- Indian community All except the representatives of the Anglo-Indian community are elected by Indian citizens A person above the age of 25 can contest in the elections for Lok Sabha One term of Lok Sabha is for 5 yearsRajya Sabha (upper house) ndash Its members are elected by the MLAs or members of the legislative Assembly There are 250 members in the Rajya Sabha of which 12 are nominated by the President One term of Rajya Sabha is for 6 years Anyone above the age of 30 can be elected as a member of Rajya Sabha

ExecutivePresidentThe President is the head of the country in India He is elected by the MPs and the MLAs for a tenure of 5 years He appoints the Prime Minister and the Council of Ministers Prime MinisterThe party which wins the election forms the government and its leader is elected as the Prime Minister He is the chief advisor to the President The Council of Ministers assists the Prime Minister and is accountable for their roles For example the Education minister is responsible for the education system in our country

1 How many members are there in Lok SabhaAns 552 members

2 What is the term for Lok SabhaAns 5 years

3 How many members are there in the Rajya SabhaAns 250 members

4 Who is the head of our countryAns President

5 Who is the chief advisor to the PresidentAns Prime Minister

Book ndash GK

Ch ndash 1First in space

1 First living being into space in 1957 Ans Laika

Times 2 First person to go into space in 1961 Ans Yuri Gagarin

3 First woman to go into space in 1963 Ans ValentinaTereshkova4 First person ever to walk in space in 1965Ans alexei Leonov5 First person to land on the moon in 1959 Ans Neil Armstrong6 First Indian to go into space in 1984 Ans Rakesh Sharma 7 First Indian woman to go into space in 19978 Ans Kalpana Chawla9 First woman tourist in space in 2006

Ans Anusheh AnsariCOMPUTER

ALGORITHM AND FLOWCHART

Q) DRAW THE SYMBOLS USED IN A FLOWCHART WITH THEIR DESCRIPTIONS(IN EXAM IT CAN COME AS SHORT QUESTIONS ASKING INDIVIDUAL SYMBOLS FUNCTION)ANS)

MAT

HEM

ATIC

S

Ch 6

Com

mon

Fra

ction

s

Multiplication of FractionsA Multiply a fractional number by whole numberTo multiply a fractional number by whole number we multiply the numerator of the fractional number by the whole number and denominator of the fractional number by 1 The first product thus obtained is the numerator and the second product is the denominator of the required product

Exercise ndash 30Multiply

7 2027

times 9

Solution 2027

times 9 = 203 = 6

23

8 611

times11

Solution 611

times11 = 6

15 71

20times16

Solution 71

20times16 =

14120

times16

= 1415

times 4 = 141times 4

5 = 564

5 = 11245

B Multiplication of a fractional number by a fractional number To multiply a fractional number by a fractional number we multiply the numerator of the first fractional number by the numerator of the second fractional number and the denominator of the first fractional number by the denominator of the second fractional number The first product thus obtained is the numerator and the second product is the denominator of the required product

16 2712

times24

Solution 2712

times24 = 3112

times24

= 31times2 = 62

Exercise ndash 31

11 83

times 34

2

Solution 83

times 34 = 2

14 723

times2 25

4

Solution 723

times2 25 =

233

times 125 =

23times 45

= 925 = 18

25

15 1212

times1 13

2

Solution 1212

times1 13 =

252

times 43 =

25times 23

= 503 = 16

23

State the following statements are true or false

17 1912

times 239 = 1

Solution LHS = 1912

times 239

= 392

times 239 = 1 = RHS

[LHS = Left hand side amp RHS = Right hand side]

there4 1912

times 239 = 1 [True]

21 213

times2 13 = 4

19

Solution LHS = 213

times2 13 =

73

times 73

= 7times73times3 =

499 = 5

49

there4 LHS ne RHS

So 213

times2 13 = 4

19 [False]

23 23

times 45 =

2times 5+3 times43times 5

Solution

LHS = 23

times 45 =

2times 43 times5 =

815 again

RHS = 2times 5+3 times4

3times 5 = 10+12

15 = 2215

there4 LHS ne RHS So 23

times 45 =

2times 5+3times43times 5

[False]

25 23 of

13 =

29

Solution

LHS= 23 of

13 =

23 times

13 =

29 = RHS]

there4 23 of

13 =

29 [True]

Practice at HomeExercise ndash 31State the following statements are true or false

24 12 of 4 =

18

Class VISubject Topic Summary Execution

HISTORY AND CIVICS

Chapter 5The Mauryan Empire

DECLINE OF MAURYAN EMPIREDecline of Mauryan empire started after the death of Ashoka at around 232 BCThere are several reasons for break up of the empire1 Weak successor Emperors after Ashoka were

capable of handling vast and mighty Mauryan empire In 185BC the last Mauryan ruler Bri-hadrath was murdered by his Commander-in-Chief Pushyamitra Sunga

2 Provincial Revolts Due to weak central author-ity provincial chiefs of Kalinga and southern provinces revolted against emperor and freed themselves from Mauryan empire

3 Weakness of Economy Prosperity of Mauryan was based on solid economic activities which

ExercisesI Multiple choice questions-1 Chandragupta defeated Seleucus in the year ndashc) 305 BC2 Who killed the last Mauryan ruler Brihadrath b) Pushyamitra3 Which of the following was not a reason for the decline of the Mauryan empirec) Chandraguptarsquos weakness4 Ashoka invaded Kalinga in the year c) 261 BC

II Fill in the blanks1Chandragupta ascended the throne in 324

was taken care by early monarchs Later kings had neither ability nor interest in economic af-fairs That led to failure in tax collection As a result they failed to maintain a large army that were essential to keep empire intact

4 Greek Invasion Greeks freed north-western provinces from weak Mauryan monarchs and reestablished their authority

5 Ashokarsquos Policy some scholar opined that after Kalinga war Ashoka embraced Buddhism re-nounced the policy of war and disbanded the Army But this is partially true as there is no proper evidence of disbanding the army

Based on above points we can conclude that main reason for decline of Mauryan empire is weakness of Ashokarsquos successors Kunal Samprati Dasharath Salisuk all were weak kingsAt last in 185 BCPushyamitra Sunga killed king Brihadrath and established the Sunga dynasty

BC2 Bindusara was the son of Chandragupta and father of Ashoka

3 Pataliputra was administered by City Magistrate committess of 5 members each4 The Greek General Seleucus sent his ambassador Megasthenes to Chandraguptarsquos court5 Ashoka sent his son Prince Mahendra and daughter Sanghamitra to spread his Dhamma6 The Indian Rebublic has adopted the Lion Capital of Saranath Pillar as its national emblem 7 Pushyamitra killed the last Mauryan ruler Brihadrath and founded the Sunga dynasty

III Name the following

1The author of Arthashastra-Kautilya2 The ruler who founded the Mauryan dynasty-Chandragupta3 The author of Indika-Megasthenes 4 The officers who were appointed by Ashoka to spread Dhamma-Dhamma Mahamatras5 The general of Alexander whom Chandragupta defeated-Seleucus

V Match the columns1 Kautilya (c)2 Megasthenes (d)3 Pushyamitra (e)4 Brihadrath (b)5 Bindusara (a)

BENGALI(2ND

LANGUAGE)

পশপাহিখর -াষাসহিবনয় রায়কেচৌধরী

যলখক পহিরহিচহিত- পরখযাত সাহিহিতযক উকেপনদরহিককেশার রায়কেচৌধরীর পতর সহিবনয় রায়কেচৌধরী lsquoসকেFশrsquo পহিতরকার সকেb হিতহিন কত হিকেলন তার উকেdখকোয বই lsquoসহিবনয় রায়কেচৌধরীর রচনা সংগরrsquo

পরম হিকেনর পাঠ- lsquoপশপাহিখর হিক -াষাhelliphellip helliphelliphelliphellipপরসপরকেক জানাবার উপায়ও পশপাহিখরা যবশ জাকেনrsquoপরকেমই আমারা জাহিন -াষা হিক -াষা ল আমাকের মকেনর -াব পরকাশ করার জনয আমরা নানান ধরকেনর -হিb বা হিবকেশষ ধরকেনর আওয়াজ মকেখর মাধযকেম কহির অনযকেক যবাঝাকেনার জনয তাকেল এবার আমরা জাহিন পশপাহিখর -াষা হিক পশ পাহিখরা হিক কা বকেল যা পশপাহিখকেরও -াষা আকে তারা তাকের হিনজসব -াষায় কা বকেল মকেনর -াব পরকাশ ককের পশ পাহিখরা মানকেষর হিক হিক -াষা যবাকেঝ হিকনত তারা বলকেত পাকেরনা পরসপরকেক বহিঝকেয় যবার উপায় তারা জাকেননা তকেব তারা হিবকেশষ ককেয়কটি শকেবদর মাধযকেম তাকের মকেনর -াব বহিঝকেয় যয় হিক বহিদধ মান জীব ndashককর হিবাল বন মানষ যঘাা পর-হিত এরা মানকেষর যওয়া নাম শনকেল কান খাা ককের ndash নাম ধকের ডাককেল কাকে আকেস যমন - মরহিরা lsquoহিত ndashহিতrsquo ডাক শকেন আকেস াল lsquoঅ ndashর -র ডাক শকেন কাকে আকেস াহিত মাহকেতর কা শকেন চকেল ককররা মাহিলকেকর হকম পালন ককের সবসময় তাইকেতা ককরকেক পর- -কত পরানী বলা য় ককর আর হিবাল একের আওয়াজ তহিম লকষয করকেল বঝকেব ককররা যরকে যকেল lsquoযঘউ যঘউrsquo করকেত াকেক আবার কাকেল lsquoযকউ যকউrsquo ককের হিবাল সাধারণ lsquoমযাওrsquo বা lsquoহিমউrsquo ককের রা কেল lsquoওয়াওrsquo আওয়াকেজর মাধযকেম মকেনর -াব পরকাশ ককের একেতা যল পশকের কা পাহিখরাও -য় রা পরকাশ করার জনয হিবকেশষ ধরকেনর শবদ ককের হিবপকের সময় পশ পাহিখরা সবার আকে পরসপরকেক জানাবার উপায় তারা জাকেন বহকাল

১) পশপাহিখর -াষা কেলপর যলখক সমপকেকG হিক জাকেনা

উঃ- পরখযাত সাহিহিতযক উকেপনদরহিককেশার রায়কেচৌধরীর পতর সহিবনয় রায়কেচৌধরী lsquoসকেFশrsquo পহিতরকার সকেb হিতহিন কত হিকেলন তার উকেdখকোয বই lsquoসহিবনয় রায়কেচৌধরীর রচনা সংগরrsquo হিতহিন ারকেমাহিনয়াম এসরাজ পর-হিত বাযনতর বাজাকেত পারকেতন ানও জানকেতন হিতহিন যাকেIাকের জনয মজাার লপ কহিবতা হিলখকেতন

২) পশপাহিখ কেলপর মল-াব হিকউঃ- পশপাহিখকেরও -াষা আকে তারা তাকের হিনজসব -াষায় কা বকেল মকেনর -াব পরকাশ ককের পশ পাহিখরা মানকেষর হিক হিক -াষা যবাকেঝ হিকনত তারা বলকেত পাকেরনা পরসপরকেক বহিঝকেয় যবার উপায় তারা জাকেননা তকেব তারা হিবকেশষ ককেয়কটি শকেবদর মাধযকেম তাকের মকেনর -াব বহিঝকেয় যয় হিরউকেবন কযাসটং সাকেব হিতহিন চহিdশ বর বনযজনত যর সকেb যকেককেন হিতহিন বকেলকেন আমরা হি তাকের -াষা তাকের আব কায়া যমকেন চহিল তাকেল আর -কেয়র যকান কারণ াকেকনা আমরা একI -াকেলাকেবকেস যচষটা করকেল পশপাহিখকের সকেb -াব পাতাকেত পাহির

ধকের মানষ এই পশ পাহিখর -াষা হিনকেয় নানা রককেমর পরীকষা ককের আসকে এইরকম একজন হিরউকেবন কযাসটাং সাকেকেবর কা আমরা জানকেবাhelliphellip

Hindi 2nd

langमतर किनमनलिलखिखतपरशनोउRरदीजि0ए

) बढ वयलि` बचच ो कया हआ था ख) डॉकटर साहबन पाटc किस उददशय स रखी थी ग) ाल साप ो हाथ म लर लाश न कया किया घ) डॉकटर चडढा न बढ पतरो दखन स कयो मना र दिदया था ङ) भगत न लाश ो दखर कया हा

उRर ndash) उस बहत बखार थी और 4 दिदनो स आख भी नही खोला थाख) उन बट ी सालकिगरह थीग) ाल सापो हाथ म लर लाश न उसी गदन 0ोर स दबार पडी थीघ) डॉकटर चडढा न बढ वयलि` पतरो दखन स मना र दिदया कयोकि उनह गोलफ खलन 0ाना थाङ) लाश ो दखर हा कि नारायण चाहग तो आध घट म भया उठ 0ाएग

English literature

In the bazaars of Hyderabad- Sarojini Naidu

Through the poem In The Bazaars of Hyderabad Sarojini wanted to convey the message that India is rich in tradition and they donrsquot need the foreign products So she goes on to give a picture of a bazaar where traditional Indian products are rulingThe poem is in the form of questions and answers The poet asks the questions and the merchants answer them Through this technique she make the picture of the bazaar visible to us

Read the poem

PHYSICS FORCE Types of FrictionThere are three types of friction static sliding rolling Static sliding and rolling friction occur between solid surfaces

1 Static Friction The frictional force that acts between the surfaces when they are at rest with respect to each other is called Static FrictionStatic Friction Examples

Skiing against the snow Creating heat by rubbing both the hands

together Table lamp resting on the table

2 Sliding Friction The resistance that is created between any two objects when they are sliding against each other is called Sliding FrictionExamples Of Sliding Friction

Sliding of the block across the floor Two cards sliding against each other in a

deck

3 Rolling Friction The force which resists the motion of a ball or wheel is called Rolling Friction Is the weakest types of frictionExamples Of Rolling Friction

Rolling of the log on the ground Wheels of the moving vehicles

6What effect can a force produce on a body which is not allowed to move Ans - When a force is applied on a body which is not free to move it gets deformed i e the shape or size of the body changes7Give one example each to indicate that the application of a force

1 produces motion2 stops motion3 slows down motion4 changes the direction of motion5 deforms a body

Ans- 1 A car originally at rest when pushed

begins to move2 A moving bicycle is stopped by

applying the brakes3 The speed of a moving vehicle is

slowed down by applying the brakes4 A player kicks a moving football to

change its direction of motion5 On stretching a rubber string its

length increases

8State the effect produced by a force in the following cases (a) The sling of a rubber catapult is stretched(b) A man pushes a heavy cart(c) A player uses his stick to deflect the ball (d) A cyclist applies brakes(e) A spring is compressedAns- (a) The shape and size of catapult changes ie its length increases(b) The heavy cart begins to move(c) The direction of the ball changes(d) The speed of the moving cycle is slowed down(e) There is change in size and shape of spring

COMPUTER MS EXCEL 2013 -INTRODUCTION

UNDERSTANDING EXCEL STRUCTUREA SPREADSHEET IS A FILE THAT EXISTS OF CELLS IN ROWS AND COLUMNS AND CAN HELP ARRANGE CALCULATE AND SORT DATA DATA IN A SPREADSHEET CAN BE NUMERIC VALUES AS WELL AS TEXT

FORMULAS REFERENCES AND FUNCTIONS

WORKSHEETA WORKSHEET IS ALSO KNOWN AS SPREADSHEETIT IS A COLLECTION OF CELLS ON A SINGLE SHEET WHERE YOU KEEP AND CHANGE DATA

WORKBOOKWORKBOOK IS PMS EXCEL FILE IN WHICH THE DATA CAN BE STORED EACH WORKBOOK CAN CONTAIN MANY WORKSHEETS

ROWS AND COLUMNSIN MS EXCEL A ROW IS A GROUP OF CELLS THAT RUN FROM LEFT TO RIGHT OF A PAGEA COLUMN IS A GROUPING OF CELLS THAT RUN FROM THE TOP TO THE BOTTOM OF A PAGE

CELLTHE INTERSECTION POINT BETWEEN A ROW AND THE COLUMN IS CALLED A CELL WHICH IS THE BASIC STORAGE UNIT FOR DATA IN A SPREADSHEET EACH CELL HAS SPECIFIC ADDRESS WHICH IS THE COMBINATION OF THE COLUMN NAME FOLLOWED BY THE ROW NUMBER

CHEMISTRY Chapter ndash Common Laboratory Apparatus and equipments

Objective type questionFill in the blanks (a) Experiment and observation are the two important basics of chemistry(b) A porcelain dish is used for evaporation(c) A test tube holder is used to hold the test tube while-it is heated(d) Mortar and pestle is used for grinding and crushing solid substances into a powder(e) Glass apparatus is made of Pyrex or borosil glass

Class VIISubject Topic Summary Execution

Hindi 2ndlang

ए था राम( डॉ शरी परसाद)

सगकित ा परभाव मानव 0ीवन पर अवशय पडता ह

हमशा मनषय ो अचछो ी सगकित म रहना चाकिहए

शरषठ परो सग स मनषय चरिरतर ा शीघर ही उदय और किवास हो 0ाता

ह इसलिलए वयलि` ो सदा शरषठ परो ा ही सग रना चाकिहए

इसान अगर चाह वह सवय ो बदल भी सता ह

यह हानी राम ए बचच ी हवह गणिणत ी परीकषा म नल रत हए पडा 0ाता ह और उस अधयाप पडत ह और पछत ह यह कया र रह

हो तभी राम न उनी बइजजती ी

शबदाथब ndashहावा भलावाायवाहीndash ाम किनयम व ानन

ो दिदखानापरिरलिचतndash 0ाना पहचानाघटनाndashघबराहट

उलटा चोर ोतवाल ो डाटndashकिववndash भल बर ा जञानतवयndash म 0ो रना चाकिहएसगकितndash बरी सगत

किबलख नाndashरोना किनशचय रनाndash तय रना

फलndashपरिरणामकिनषालिसतndash बाहर किया हआपशचातापndashदख सपननndashधनी

ldquo हा आपी किहममत स हई नल रत पडन ीrdquo ऐसी बात ही किफर

किपता0ी न भी उस डाटा वह ाफी पशचाताप रन लगा बोला गलत दोसतो

ी सगकित म आ0 कितना अनथ र दिदया किफर उसन अधयाप स माफी मागन ी सची और किफर भी ऐसा

नही रगा यह परण भी लिलया

सोचndashकिहच एात-अला

বইndashবাংলা সাহিতয পহিরচয়

পাঠndash১৬লপndashস-য ও অস-যযলখকndashঈশবরচনদর হিবযাসারঅনশীলনীর পরকে4াততর

৬ অGকেলকেখা -ময়া = পশ হিশকার সহিtহিত = হিনকIবতu সbভরষট = লI হিনরীকষণ =

-াকেলা-াকেব যখাকতাঞজহিলপকেI = যজাাকেত৭ হিবপরীতশবদ -ঈষৎ times পরচর উৎকষট times হিনকষট তাশ times উৎফd তবহিদধ times

বহিদধীNপাহিপষঠ times পণযবান৮ পপহিরবতG ন ককেরা -পশ = পাশহিবক যকাপ = যকাহিপতহিসথর = হিসথরতাএকানত = ঐকাহিনতক পর-াত times পর-াতী

CHEMISTRY

Chapter ndashPhysical and Chemical Changes

Chemical ChangeA chemical change involves a change in chemical composition

Characteristics of Chemical changes 1 They are permanent changes2 They are irreversible changes 3 New substance formed4 A Chemical change involves a

change in its chemical properties

Pg-25Question 8What do you observe when1 water is boiled2 a piece of paper is burnt3 some ice cubes are kept in a glass tumbler4 solid ammonium chloride is heated5 an iron nail is kept in tap water for few days6 a spoon of sugar is heated in a pan7 lighted match stick is brought near the mouth of the test tube containing hydrogen gas8 quick lime is dissolved in water9 little amount of curd is added to a bowl containing warm milk and kept for five hours

10 Water is boiledOn boiling water changes into steam (gas) physical change

11 A piece of paper is burnton burning piece of paper produces carbon dioxide and ash is left behind Is a chemical change

12 some ice cubes are kept in a glass tumblerIce cubes (solid) turn into water

(liquid) only state changes (physical change)

13 Solid ammonium chloride is heatedSolid ammonium chloride on heating changes into vapors (change of state) is physical change

14 An iron nail is kept in tap water for few dayswe observe reddish brown coating on the nail called rust (entirely new substance) is chemical change

15 A spoon of sugar is heated in a panWhen a spoon of sugar is heated in a pan black (charred sugar) (carbon) is seen Is a chemical change

16 Lighted match stick is brought near the mouth of the test tube containing hydrogen gasWe observe that hydrogen bums at the mouth of test tube with blue flame and pop sound is heard It is chemical change

17 Quick lime is dissolved in waterThe following two observations will be observed (i) A hissing sound is observed(ii) The mixture starts boiling and lime water is obtained

18 Little amount of curd is added to a bowl containing warm milk and kept for five hoursWhen a little amount curd is added to a bowl containing warm milk and kept for five hours a permanent change occurredThe milk will change to curd On boiling water changes into steam (gas) physical change

GEOGRAPHY

ATMOSPHERE IMPACT OF GLOBAL WARMING The destructive impart of global warming is observed in various spheres of life and the environment Some of the points are outlined below1 High temperatures lead to high

evaporation rate and drying up of the soil and surface water This affects crop production The occurrence of droughts is aggravating the problem even further

2 The heat waves in summer months

Q1 Write some impact of global warmingA1 The impacts of global warming are as follows1 High temperatures lead to high

evaporate ion rate and drying up of the soil and surface water This affects crop production The occurrence of droughts is aggravating the problem even further

2 The heat waves in summer months lead to a greater number

lead to a greater number of deaths due to heat strokes

3 Forest fires become more frequent4 Tropical cyclones and hurricanes

become common5 Melting of glaciers takes place6 Polar ice caps are becoming thinner

and melting at an alarming rate due to global warming The loss of sea ice

7 Due to increase in sea surface temperature sea levels rise in coastal areas and cause submergence of several islands

WAYS TO REDUCE GLOBAL WARMINGFollowing steps can be taken We need to decrease emission of

green house gases by reducing the burning of fossil fuel such as coal and petroleum

By planting more trees to increase forest cover

The government should also distributes free saplings and organize afforestation programmes to spread awareness regarding the beneficial effects of trees

We should switch to eco-friendly cars and gadgets

Incandescent light bulbs should be replaced by CFL bulbs

We can save electricity and reduce global warming by turning off electrical gadgets such as lights fans air-conditioners television and computer when we do not to use them

Efforts should be made to hasten the development of green cities oreco cities These cities are urban areas around the world striving to lessen the environment a impacts of urbanization

By following the 3Rs-Reduce Recycle and Reuse strategy we can use natural resources for our growth as well as save them for the need of the future generations This is called sustainable development

of deaths due to heat strokes3 Forest fires become more

frequent4 Tropical cyclones and hurricanes

become common5 Melting of glaciers takes place

etc

Q2 How to reduce global warmingA2 Following steps can be taken to reduce global warmingaWe need to decrease emission of

green house gases by reducing the burning of fossil fuel such as coal and petroleum

bBy planting more trees to increase forest cover

c The government should also distributes free saplings and organize afforestation programmes to spread awareness regarding the beneficial effects of trees

dWe should witch to eco-friendly cars and gadgets

eIncandescent light bulbs should be replaced by CFL bulbs

f We can save electricity and reduce global warming by turning off electrical gadgets such as lights fans air-conditioners television and computer when we do not to use them

Q3 What do you mean by 3Rrsquos of resource planningA3 The 3Rs are

1 Reduce 2 Recycle and3 Reuse

Q4 What is Sustainable developmentA4 By following the 3Rs-Reluce Recycle and Reuse strategy we can use natural resources for our growth as well as save them for the need of the future generations This is called sustainable development

English Language

Prepositions A preposition is a word placed before a noun or a pronoun It helps to show how the person or thing denoted by the noun is related to something else in the sentence

Kinds of Prepositions

Simple Prepositions- simple preposition are one word Prepositions such as at by for in of off for from on out through till to up with before amidst towards beyond between over etc

Compound Prepositions ndash There are some words that are always used with fixed Prepositions to convey specific meaning

Example I was unable to meet you dueto a previous engagement ( On account of)Always maintain the queue instead of crowding at the counter ( In place of)

Participial PrepositionsmdashParticiple Prepositions are present or past participles of various verbs which together with a noun phrase or a clause function as prepositions Examples- barring concerning considering notwithstanding pending regarding respecting etc

Exercise A

1 Gauravs fever has come down since Friday He has been absent for a week now

2 The child sat between his father and mother among the parents of all his classmates

3 There are mosquitoes in the room They flew into the room when the door was open

4 My father was inside the drawing room when I was playing outside my house

5 You may sit beside me I will give you a drawing book and pencils besides a storybook

6 We went to the market in the morning and walked towards the riverfront in the evening

7 The child walked along the pavement and across the street safely

8 This table top is made of glass My breakfast fell off it in the morning

9 The pan is on the gas stove There are vegetables in it

10 We will wait for you at the bus top There are a lot of people in the hall

Subject ndash Biology Topic ndash Chapter - 3 Photosynthesis and respiration in plants Summary Execution

All living organism (Plants and animals) need food for energy and growth Green plants (autotrophy) prepare food for all living organisms Today we will discuss about the process photosynthesis And adaptations in a leaf to carry out photosynthesis

Q1What do you mean by photosynthesis and write its word equation The process by which green plants make food (glucose) from carbon dioxide and water

in the presence of sunlight and chlorophyll is called photosynthesis

Carbon dioxide + Water ( Sun light from Sun ) Glucose + Oxygen ( chlorophyll in green leaves )

Q2 What are the adaptations in a leaf to carry out photosynthesisi) Leaves are broad wide and flat for absorbing more light energyii) Presence of chlorophyll in chloroplasts to trap sunlightiii) Presence of stomata which allow carbon dioxide to enter the cell and oxygen to go

out iv) Network of veins ensures continuous supply of water and minerals to the leafv) Thin waxy cuticle protects the leaf without blocking the lightQ3 Draw and label structure of chloroplast

Class VIIISubject Topic Summary Execution

PHYSICS ENERGY Production of Hydro electricity

A hydroelectric dam converts the potential energy stored in a water reservoir behind a dam to mechanical energymdashmechanical energy is also known as kinetic energy As the water flows down through the dam its kinetic energy is used to turn a turbine

The generator converts the turbinersquos mechanical energy into electricity

This electric energy then goes through various transmission processes before it reaches you

Question 2

Fill in the blanks

(a) Work is said to be done by a forte only when the body moves

(b) Work done = Force x distance moved in direction of force

(c) The energy of a body is its capacity to do work

(d) The SI unit of energy is joule

(e) The potential energy is due to its state rest of position and kinetic energy of the body is due to its state of motion

(f) Gravitational potential energy U = mass times force of gravity on unit mass times height

(g) Kinetic energy = frac12 times mass times (speed)2

(h) Power P = work donetime taken

(i) The S I unit of power is watt

(j) IHP = 746 W

BIOLOGY Chapter -5 The endocrine system and adolescence

Today we will discuss about thelocation and functions of secreted hormones of adrenal and Pancreas

Q5 Write location hormone secreted main functions and deficiency diseases of pancreas and adrenal glands

Endocrine Glands

Location Hormones secreted

Functions and Deficiency Diseases

1Adrenal gland

2 Pancreas Gland

On the top of each kidney

In between stomach and small intestine

i)Adrenaline from adrenal medulla

ii)Cortisone from adrenal cortex

i) Insulin

ii) Glucagon

It helps a person deal with any kind of emergency situation or emotional stressIt increases the heart beat rate of respiration and blood pressure

a) It regulates carbohydrates protein and fat metabolism

b) It regulates the salt and water balance in the body

a) It changes excess glucose into glycogen

b) It stimulates the cells to burn extra glucose to provide heat amp energy

Less secretion causes diabetes mellitus

Excessive secretions causeinsulin shock

a) It stimulates the breakdown of glycogen into glucose

b) It increases the level of glucose in blood

History Traders to rulers The Battle of Buxar was fought on 22 October 1764 between the forces under the command of the British East India Company led by Hector Munro and the combined armies of Mir Qasim the Nawab of Bengal till 1763 Mir Jafar was made the Nawab of Bengal for a second time in 1763 by the Company just after the battle After being defeated in 4 battles in katwa and Udaynala the Nawab of Awadh Siraj id Daula and the Mughal emperor Shah Alam II accompanied by Raja Balwant Singh of Kashi made an alliance with Mir Qasim The battle was fought at Buxar a small fortified

Answer the following questions- Short note-Battle of BuxarHomework-learn

town within the territory of Bihar located on the banks of the Ganga river about 130 kilometres (81 mi) west of Patna it was a decisive victory for the British East India Company The war was brought to an end by the Treaty of Allahabad in 1765

EnglishLiterature

The west wind-John Mansfield

In the poem The West Wind by John Masefield the poet starts by describingwith very poetic imagery of birds how the west wind is different from other winds its a warm wind full of birds cries There is a touch of melancholy perhaps home-sickness as he describes how it brings tears too and memories from an old land He goes on to describe the restful pastoral beauty of the land where even the dead can lie in the green He then brings in voicesperhaps of family and friends calling him home as he is missing Aprils beautyThe voices then tempt him some more with idyllic images from home (white blossom young green cornrunning rabbitswarm sun) The voices seem to presume that the poets heart is sorrowful bruised and soreThe end of the poem sees the poet appear to make a decision he will go home as he has decided that is where he truly belongs

Write the synopsis of the following words

1 Daffodils- a tall yellow flower that grows in the spring

2 Orchards- a piece of land on which fruit trees are grown

3 Blossom- a flower or a mass of flowers especially on a fruit tree in spring

4 Thrushes- a bird5 Larks- a small brown bird that

makes a pleasant sound6 Bruised- an injury7 Aching- pain 8 Tread- to put your foot down

while you are walking9 Balm-10 May-11 Fluting-

(Write from the book in your copy)

MAT

HEM

ATIC

S

Ch 1

1Al

gebr

ic E

xpre

ssio

n

1 Constant A symbol which has fixed value is called a constant[eg 8 23 -15 radic3 etc]

2 VariableA symbol which does not have any fixed value but may be assigned value (values) according to the requirement is called variable or literal[eg x y p q etc]

3 TermsA term is a number (constant) a variable a combination (product or quotient) of numbers and variables[eg 7 x 5x etc]

4 Algebric expressionA single term or acombination of two or more terms connected by plus (+) or minus (-) sign forms an algebraic expression[eg 5-y 3x2-5x xy-6z+4 etc]

5 PolynomialAn algebraic expression which contains more than one term is called a polynomial (multinomial)[eg x2-5x 5y+xy+x2y etc]

6 Degree of polynomial(a) When the polynomial contains only one variable the highest power of the variable is the degree of the polynomialeg the degree of the polynomial of 4x-7x5+8 is 5(b) When the polynomial contains two or more variablesStep (i) Find the powers of the variables in each term (ii) The highest sum of the powers is taken to be the degree of the polynomialeg the degree of the polynomial 5x2y-4x3y5+6 is = 3+5 = 8Remember An algebraic expression is a polynomial if degree of each term used in it is a non-negative integer

Exercise ndash 11(A)

1 Separate the constants and variables from the following

-7 7+x 7x+yz radic5 radic xy 3 yz

8 45y -3x

Solution Constant Variables-7 radic5 7+x 7x+yz radic xy

3 yz8

45y -3x

2 Write the number of terms in each of the following polynomials(i) 5x2+3timesax (ii) axdivide4-7 (iii) ax-by+ytimesz (iv) 23+atimesbdivide2

Solution Polynomials Number of terms(i) 5x2+3timesax 2(ii) axdivide4-7 2(iii) ax-by+ytimesz 3(iv) 23+atimesbdivide2 2

4 Write the degree of the each polynomials(i) xy+7z (ii) x2-6x3+8 (iii) y-6y2+5y8 (iv) xyz-3 (vi) x5y7-8x3y8+10x4y4z4

Solution Polynomials Degree(i) xy+7z 2(ii) x2-6x3+8 3(iii) y-6y2+5y8 8(iv) xyz-3 3(vi)x5y7-8x3y8+10x4y4z4 12

5Write the coefficient of(i) ab in 7abx (iv) 8 in a2-8ax+a (v) 4xy in x2-4xy+y2

SolutionCoefficient

(i) ab in 7abx 7x(iv) 8 in a2-8ax+a -ax(v) 4xy in x2-4xy+y2 -1

7 CoefficientAny factor of an algebraic quantity is called the coefficient of the remaining quantityeg in the algebraic term 7xyz 7 is coefficient of xyz 7x is coefficient of yz and so on

8 Like term The terms having the same literal coefficient are called like terms and those having different literal coefficients are called unlike terms

eg (i) 5xyz 8xyz -6xyz and 23xyz are like

terms(ii) 7xy2 8x2yz and -15xyz2 are unlike terms

6 in 57xy2z3 write the coefficient of

(i) 5 (vii) 5xy2 (viii) 17yz (xi) 5xyz

Solution Coefficient

(i) 5 17

xy2z3

(vii) 5xy2 17z3

(viii) 17yz

5xyzsup2

(xi) 5xyz 17yz2

7 In polynomial given below separate the like terms(ii) y2z3 xy2z3 -58x2yz -4y2z3 -8xz3y2 3x2yz and 2z3y2

Solution y2z3 -4y2z3 2z3y2 are like terms

xy2z3 -8xz3y2 are like terms

-58x2yz 3x2yz are like terms

Class IXSubject Topic Summary Execution

Bengali (2nd language)

বাগzwnjধারাzwnj বা ধারা-বা ধারা ল হিবকেশষ পরকার বাক -হিb -াকেবর এক হিবকেশষ পরকাশরীহিত াকেক কতগকেলা কার সমষটির মকেধয এগহিলকেক বা ধারা বকেল আবার কতগকেলা শকেবদর বাধাধরা যকান রীহিত যনই য-াকেব চকেল আসকে যসই -াকেবই চকেল আসকে তখন যসই শবদগহিল খন একক -াকেব অG পরকাশ ককের তখন একের বা ধারা বকেল বা ধারার পরকেয়া -াষাকেক আরও সFর ককের যতাকেল

অকাল পকক(অপহিরনত বয়কেস পাকাহিম)-মাতর শ বর বয়কেস যমকেয়টির া মকেখর কা তাকেত অকালপককতা ধরা পকে

অককা পাওয়া( মারা াওয়া) ndash পকেকIমারটি পকেকIমারকেত হিকেয় বাসাতরীকের াকেত মার যখকেত যখকেত অককা যপল

অহি| পরীকষা ( কঠিন ও পরকত পরীকষা)- যকেলটির আজ ডাকতাহির যরজালট যবকেরাকেব এIাই তার জীবকেনর ব অহি| পরীকষা

অষটরমভা (ফাহিক) ndash রীতা মকেখই বকো বকো কা বকেল আর কাকেজর যবলায় অষটরমভা

অকমGার ধাী (অপাG) ndash সমনকেক হিনকেয় যকান ান কেব না ও একেকবাকেরই অকমGার ধাী

অকেনধর ষটি (অসাকেয়র সায়)- আহিশ বকেরর বকোর নাহিত ল অকেনধর ষটি তাকেক াা বকোর একম চকেল না

আকেককল গড়ম (তবহিদধ)- ার তহিম উপকার করকেল যসই যতামার হিবরকেদধ সাকষয হিকেয়কে শকেনই আমার আকেককল গড়ম

আষাকে লপ( অবাসতব লপ) ndashIাকা এখন যকেব না এIা বলকেলই ত এমন আষাকে লপ ফাার যকান রকার হিল না

Hindi- महायजञ ा इस हानी म लख न या बतान ा परयास किया ह कि किसी भी अचछ

2nd language

परसार(यशपाल ाय या पणय न ा फल अवशय मिमलता ह ोई भी परोपार अथवा पणय लिलए किया गया ाय बार नही 0ाता वह ए परार ा यजञ हए धनी सठ थ धम परायण और किवनमर सठ न आन ी यजञ किए थ और दान म न 0ान कितना धन दिदन दखिखयो म बात दिदया थादिदन पलट और सठ यहा गरीबी आ गई उन दिदनो यजञ बचन ी परथा थी सठ भी अपनी 0गह बचन लिलए डलपर ए सट यहा चलन ो तयार हए सठानी रासत लिलए रोटी पड म बाधर सठ ो द दी रासत म ए भख R ो दखर सठ न चारो रोटी उसो खिखला दी खर वह सठ यहा डलपर पहच तो उनी सठानी न उस महायजञ बचन ो हा यदिद बचन आए सठ न R ो रोटी खिखलान ो महायजञ नही समझा और वापस लौट आया घर आर शाम ो उसी घर म उस ए बडा ख0ाना मिमला 0ो उस दवारा किए गएrsquo महायजञrsquo ा परसार था

English language

Letter formal The heading the name and address of the person you are writing to must be included beneath your own address In formal letters ldquoblock stylerdquo of address is preferred

Subject complain in brief

Salutation If the person you are writing to is known to you you may begin ldquoDear MrrdquoOr ldquoDear Mrsrdquo In all other instances you should begin ldquoDear Sirrdquo or ldquoDear Madamrdquo Or ldquoSirsrdquo

The body A formal or business letter has four partsReference The letter should begin by referring to a letter you have received an advertisement or the reason that has prompted you to writeInformation In the second paragraph it is necessary to supply more detailed information that is related to the referencePurpose Here you must give the reason why you are writing the letter This must be stated clearly and ensure that it is relevant to the question that has been setConclusion round off the letter with some polite remarkThe subscription when a letter has begun with dear sir sirs Madam you should end with Yours faithfully or yours truly When however you address a person by name you must conclude with the words ldquoYours sincerelyrdquo

1 A park in your locality is slowly being used as a rubbish dump Write a letter to the Mayor of your city pointing out the nuisance and danger of this Request that action be taken to stop this immediately

Or2 You being a boarder ordered a set of lab manuals from a famous book shop in the town They sent you a wrong set of books Write a letter to the manager of the book shop

Chemistry Chapter-1 1)CHEMICAL FORMULA- Q What is the Significance of

L-2The Language of Chemistrybull Chemical Formula

Itrsquos a symbolic representation of a chemical substance eg ndash The formula of Sulphuric acid is H2SO4

2) Steps of writing Chemical Formula of a given substance-

1 Write the symbols of the constituent atoms or radicals side by side Keep the basic radical on LHS and acid radical on the RHS ( Na+Cl- )2 In case of a radical having more than one atom( compound radical) enclose the radical in a bracket eg (SO4-)3 Write the valencies of each radical on its right hand top4 If the valencies of the two radicals are divisible by a common factor then divide the valencies by the common factor5 Invert (criss-cross) the valency number ie write the valency of one atom below the second atom and vice versa 6 On interchanging if valency number is lsquoone the figure lsquoonersquo is never writtenFor Example- Compound -Calcium Nitrate1 Writing the symbols- Ca(NO3)2 Writing the valencies on their right hand top- Ca2(NO3)1

3 Valency numeral in simple ratio- Ca2(NO3)1

4 Criss-cross- Ca 2NO3 1

5 Writing the formula of the compound- Ca(NO3)2

Chemical formula

A The formula of a substance conveys the following information regarding a substance 1 The name of the substance (qualitative)2 The elements constituting the substance (qualitative)3 The number of various atoms present in a molecule of the substance (quantitative)4 Molecular weight of the substance and the relative weights of different elements present in it (qualitative)

Q What are the limitations of Chemical Formula

A The chemical formula suffers from the following limitations-I It fails to convey whether the elements in a molecule are present in the form of atoms or ionsFor example the formula KBr fails to tell us whether Potassium and Bromine are present in the form of ions II It does not tell anything about the binding force that holds atom in a molecule togetherIII It does not tell us about the arrangement of various atoms with respect to one another within the molecule

Q Examples of Some Chemicals with their Formula Chemical name and Common Name-

A Given in the class notesCommercial Studies

Joint Stock Company

Let us discuss about the demerits of Joint Stock CompanyDespite so many advantages it has got many disadvantages which are as follows

Difficulty in FormationDelay in Decision makingExcessive Government ControlLack of Secrecy

Company can be classified into several categories based on incorporation

QuestionExplain the demerits of Joint Stock CompanyAnswer) 1 Difficulty in Formation The legal requirements and formalities required to be completed are so many The cost involved is quite heavy It has to approach large number of people for its capital It cannot start its business unless certificate of incorporation has been obtained This is granted after a long time when all the formalities are completed

Chartered CompanyStatutory CompanyRegistered Company

Delay in Decision making In this form of organization decisions are not made by single individual All important decisions are taken by the Board of Directors Decision-making process is time-consuming So many opportunities may be costly because of delay in decision-making Promptness of decisions which is a common feature of sole trader ship and partnership is not found in a company

Excessive Government ControlA company and the management have to function well within the law and the provisions of Companies Act are quite elaborate and complex At every step it is necessary to comply with its provisions lest the company and the management should be penalized The penalties are quite heavy and in several cases officers in default can be punished with imprisonment This hampers the proper functioning of the company

Lack of Secrecy The management of companies remains in the hands of many persons Every important thing is discussed in the meetings of Board of Directors Hence secrets of the business cannot be maintained In case of sole proprietorship and partnership forms of organisation such secrecy is possible because a few persons are involved in the management

2 Define the following

Chartered Company- The crown in exercise of the royal prerogative has power to create a corporation by the grant of a charter to persons assenting to be incorporated Such companies or corporations are known as chartered companies Examples of this type of companies are Bank of England (1694) East India Company (1600) The powers and the nature of business of a chartered company are defined by the charter which incorporates it After the country attained independence these types of companies do not exist

in IndiaStatutory Company- A company may be incorporated by means of a special Act of the Parliament or any state legislature Such companies are called statutory companies Instances of statutory companies in India are Reserve Bank of India the Life Insurance Corporation of India the Food Corporation of India etc The provisions of the Companies Act 1956 apply to statutory companies except where the said provisions are inconsistent with the provisions of the Act creating them Statutory companies are mostly invested with compulsory powersRegistered companiesCompanies registered under the Companies Act 1956 or earlier Companies Acts are called registered companies Such companies come into existence when they are registered under the Companies Act and a certificate of incorporation is granted to them by the Registrar

Economics

Chapter-4Basic problems of Economy

Today let us discuss with the topic Production Possibility curve

QuestionExplain the concept of Production Possibility Curve with the help of diagram

Answer) Production Possibility curve is a locus of all possible combinations of two commodities which can be produced in a country with its given resources and technology

The above diagram shows that with the given resources and technology the economy can produce maximum either 5 thousand meters of cloth or 15 thousand quintals of wheat or any other combination of the two goods like B( 1 thousand meters of cloth and 14 thousand quintals of wheat C ( 2 thousands meters of cloth and 12 thousand quintals of wheat) etcProduction Possibility curve is also called production possibility boundary or frontier as it sets the maximum limit of what it is possible to produce with given resources

Geography

Rotationand Revolution

SUNrsquoS POSITION AND SEASONAL CHANGES EQUINOXES ndash SPRING AND AUTUMN

Q1 What is Spring EquinoxA1 On 21st March sunrays fall directly on the equator On that day

As the Equator divides the Earth into two equal halves the sun rays fall directly on the equator twice in a year Equinoxes means equal Spring EquinoxOn 21st March sunrays fall directly on the equator On that day the duration of day and night both are equal ( 12 hours day and 12 hours night) on every places located on equator This day is called as Spring EquinoxAutumn EquinoxOn 23rd September sunrays fall directly on the equator On that day the duration of day and night both are equal ( 12 hours day and 12 hours night) on every places located on equator This day is called as Autumn Equinox

SOLSTICES ndash SUMMER AND WINTERDue to inclination of the Earth on its axis and the apparent movement of the sun the sun rays fall directly on both tropics once in a year Solstice is a Latin word which mean ldquothe Sun standing stillrdquoSummer SolsticesAfter 21st March there is an apparent movement of the Sun to the north of the equator The apparent northward movement up to 21st June when the Sun appears overhead at the Tropic of Cancer (22frac12degN) The sun appears to stand still at this position and then moves southwards towards the equator This position of the Sun on 21st June is known as Summer Solstices On that day the duration of day and night both are equal ( 12 hours day and 12 hours night) on every places located on Tropic of Cancer (22frac12degN)Winter solstices The apparent southward movement of the Sun continues beyond the equator till 22nd

December On this day the Sun is overhead at the Tropic of Capricorn

the duration of day and night both are equal ( 12 hours day and 12 hours night) on every places located on equator This day is called as Spring Equinox

Q2 What do you mean by EquinoxA2 Equinoxes means equal It is use to explain the equal duration of day and night ( 12 hours day and 12 hours night) on the Earth

Q3 On which date the longest day in Tropic of CancerA3 21st June

Q4 What is the meaning of SolsticeA4 Solstice is a Latin word which mean ldquothe Sun standing stillrdquo

Q5 Which is the longest day in southern hemisphereA5 22nd December

Q6 On what date does the Arctic Circle experience the lsquoMidnight SunrsquoA6 On 21 June the Arctic Circle experiences the lsquoMidnight Sunrsquo

Q7 What is cause of Midnight Sun in NorwayA7 During the summer solstice (21 June) the North Pole is inclined towards the Sun Therefore the duration of sunlight or daytime increases from 12 hours at the Equator to 24 hours at the Arctic Circle and beyond Thatrsquos why The region beyond the Arctic Circle especially Norway is known as the Land of the Midnight Sun because there the Sun does not rise or set on 21 June

Q8 Match the column A with BA B

Summer Solstice 21st March

Autumn Equinox 23rd

September

Winter Solstice 21st June

(22frac12degS) This position of the Sun is referred to as the Winter Solstice because it marks the winter season in the Northern Hemisphere On that day the duration of day and night both are equal ( 12 hours day and 12 hours night) on every places located on Tropic of Capricorn (22frac12degS)SEASONS AND DURATION OF DAY AND NIGHT During the equinoxes all places on the Earth have 12 hours of day and 12 hours of night Due to the revolution of the Earth round the Sun on an inclined axis the duration of day and night varies according to seasons and the latitude of a placeDuring the summer solstice (21 June) the North Pole is inclined towards the Sun Therefore the duration of sunlight or daytime increases from 12 hours at the Equator to 24 hours at the Arctic Circle and beyondThe region beyond the Arctic Circle especially Norway is known as the Land of the Midnight Sun because there the Sun does not rise or set on 21 JuneAt the North Pole there will be six months of daylight The Sun will be seen always above the horizon at a low angle At 66degN 24 hours of sunlight can be seen only on 21 June Hammerfest in northern Norway is a place of tourist attraction for observing the phenomenon of the Midnight Sun This place has continuous daylight from 13 May to 29 July This place is easily accessible to tourists and has hotels and other facilities The view of the midnight Sun from here is enthrallingIn the Southern Hemisphere the duration of daylight decreases from 12 hours at the equator to 0 hours beyond the Antarctic Circle In the South Polar Region there is 24 hours of darkness The Sun is always below the horizon In the Southern Hemisphere which experiences winter the duration of night-time is longer than the duration of daylight

Spring Equinox 22nd

December

A8 A B

Summer Solstice 21st June

Autumn Equinox 23rd

September

Winter Solstice 22nd

December

Spring Equinox 21st March

During winter solstice (22 December) the South Pole is inclined towards the Sun The Southern Hemisphere experiences summer and the Northern Hemisphere has winter Therefore the duration of daylight or sunlight is greater in the Southern Hemisphere than in the Northern HemisphereThe duration of daylight increases from 12 hours at the equator to 24 hours beyond the Antarctic Circle The South Polar Region has 24 hours of sunlight for many days continuously At the South Pole there will be six months of sunlight The Sun will always be seen at a low angle above the horizon In the Northern Hemisphere the duration of daylight will decrease from 12 hours at the equator to 0 hours at the Arctic Circle There are 24 hours of darkness in the North Polar region The duration of night is greater than the duration of daylight as one move northwards from the Equator It is evident from the above table that the duration of daylight is 12 hours throughout the year at the equator only As one moves away from the equator the seasonal variations in the duration of daylight increase The seasonal variations in the duration of daylight are maximum at the Polar Regions

Subject Eng Literature (The Merchant of Venice ndash William Shakespeare)Topic Act II Scene 7 Lines 36 to 80 (End of scene ) [Students should read the original play and also the paraphrase provided]

Summary Questions amp AnswersThe Prince then examines the inscription on the silver casket which says ldquoWho chooseth me shall get as much as he deservesrdquo The Prince says that he deserves Portia more than anybody else because of his high rank his noble birth and his great wealth and power But then he argues that silver is ten times

(1) (Act II Sc 7 L 39-47)

From the four corners of the earth they come

To kiss this shrine this mortal breathing saint

The Hyrcanian deserts and the vasty wildsOf wide Arabia are as through-fares now

inferior to gold and therefore he cannot believe that the portrait of such a beautiful lady as Portia can be contained in the silver casket He decides to see the inscription on the golden casket before making his decision

The Prince goes to examine the inscription on the golden casket which says ldquoWho chooseth me shall get what many men desirerdquo The Prince believes that the whole world desires to possess Portia otherwise so many suitors would not have come from all corners of the world for winning Portia Some of them have come from the distant lands of Persia and Arabia The deserts of Persia (Hyrcanian deserts) and the boundless desolate lands of Arabia have been crossed by the Princes seeking the hand of Portia He contrasts this casket containing Portiarsquos portrait with the old English gold coin bearing the image of the archangel (angel of the highest rank) He goes on to remark that while the figure of the archangel is engraved (Insculped) upon the English coin the picture of Portia who is beautiful as an angel lies hidden inside one of the caskets namely the Golden Casket (Golden Bed)

On the basis of his assessment of the inscription on the golden casket the Prince decides to choose the golden casket He asks for the key and opens the golden casket only to find therein an empty human skull holding a roll of

For princes to come view fair PortiaThe watery kingdom whose ambitious headSpets in the face of heaven is no barTo stop the foreign spirits but they comeAs orsquoer a brook to see fair Portia

(i) Explain the occasion for the above mentioned speech

These are the comments of the Prince of Morocco after he reads the inscription on the golden casket His mental process is revealed to us in these words We find him debating within himself as to which casket he should choose

(ii) What light does the above speech throw on the personality of Prince of Morocco

From the above mentioned speech we come to know that the Prince of Morocco is keen to marry Portia He is the type of person who is easily taken away by outward appearance He is in love with Portia because of her beauty

(iii) What information can you gather about Portia from the above mentioned lines

The given speech shows that Portia is a very beautiful lady She must be possessed of good qualities because many suitors come to her place from all over the world with a desire to get married to her The Prince of Morocco is so impressed by her beauty that he calls her a saint According to him the whole world is desirous of having her

(iv) Elucidate the significance of the first two lines

In these lines the Prince of Morocco pays a compliment to Portia These lines show his admiration for her He says that people come from all parts of the world to see fair Portia

(v) Explain the meaning of the last four lines of the

passage

In these lines the Prince of Morocco says that even the vast oceans which throw a challenge at the sky are unable to prevent men from coming to Portiarsquos place to have a glimpse of her These lines are also a tribute to Portiarsquos beauty and good qualities Many men voyage across the ocean treating it as a mere stream to see the beautiful Portia

paper in which is written that whoever happens to be guided by the glitter of things is invariably deceived

On reading the scroll the Prince says that he is too sad at heart to speak a more formal farewell and leaves with his followers amidst a sound of trumpets

After the Prince of Morocco leaves Portia remarks that the Prince is a gentle fellow but she is rid of him May all persons of his nature make a similar choice

IMPORTANT PASSAGES EXPLAINED

(Act II Sc 7 L 39-43)From the four corners of the earth they come

To kiss this shrine this mortal breathing saintThe Hyrcanian deserts and the vasty wildsOf wide Arabia are as through-fares nowFor princes to come view fair Portia

Context

This passage occurs in Act II Scene 7 in The Merchant of Venice This is part of the speech made by the Prince of Morocco

(2)

(Act II Sc 7 L 48-53)

MOROCCO One of these three contains her heavenly pictureIst like that lead contains her

Twere damnation To think so base a thought it were too grossTo rib her cerecloth in the obscure graveOr shall I think in silver shes immurdBeing ten times undervalued to tried gold

(i) What meaning does the Prince of Morocco find out of the inscription of the golden casket What have Belmont and Portiarsquos house been called and why

The inscription on the golden casket is ldquoWho chooseth me shall gain what many men desirerdquo The Prince finds out that it means that the chooser of the golden casket will get Portia because many men desire her In fact the entire world desires her Because of the coming of many suitors to Belmont from different countries in order to win Portiarsquos hand Belmont has become a centre of pilgrimage and her house is the shrine where saintly Portia is installed

(ii) What does the Prince of Morocco do before making the final choice of the casket Which is the correct casket and who will win Portiarsquos hand

The Prince of Morocco surveys and analyses the inscriptions on the casket of lead silver and gold Before making the final choice like a very systematic and methodical person he once again considers the claims of the caskets The casket containing Portiarsquos picture is the correct casket and the person choosing it will win Portiarsquos hand

Explanation

While praising Portia the Prince of Morocco conceives Portia as a goddess whose image is placed inside one of the caskets Many suitors are coming from far and wide the north and the south the east and the west (Four corners) in order to try their luck Some of them have come from the distant land of Persia and Arabia The deserts of Persia (Hyrcanian deserts) and the boundless desolate lands of Arabia have been crossed by the Princes seeking the hand of Portia All this shows that Portia is indeed the most beautiful lady of the world

(iii) What does the Prince of Morocco say in his estimation while examining the motto on the silver casket What does he find in the golden casket

While examining the motto on the silver casket which says ldquoWho chooseth me shall get as much as he deservesrdquo Morocco says that in his own estimation he surely deserves Portia in all respects ndash rank birth wealth etc

He chooses the golden casket When he opens it he finds an empty human skull holding a scroll in which it is written that those who are attracted by the glittering outside of things are always deceived as Morocco has been deceived

(iv) What kind of nature does the Prince of Morocco have

The Prince of Morocco has a simple nature who does not look deeply into the inner meaning of things but is dazzled by the outward appearance of gold He is inclined to over-estimate his own value and does not realize that it is a duty to ldquogive and hazardrdquo To say that he will not hazard for lead shows that he misreads the true meaning of the inscription which is that he should be prepared to ldquohazard all he hathrdquo for Portia So his feeling is only one of fascination and romantic attraction

(v) Do you think that the lottery of the caskets is not a matter that will be determined by chance

In fact the lottery of the casket is not a matter that will be determined by mere chance but that it is a true test of character and of sincerity which is amply proved not only by Moroccorsquos choice but also by the arguments which he uses to help him in his choice

(Act II Sc 7 L 55-59)

They have in England

A coin that bears the figure of an angelStamped in gold but thats insculpd uponBut here an angel in a golden bedLies all within

Context

(3)

(Act II Sc 7 L 63-77)A carrion Death within whose empty eye

There is a written scroll Ill read the writing

All that glisters is not goldOften have you heard that toldMany a man his life hath soldBut my outside to beholdGilded tombs do worms infoldHad you been as wise as boldYoung in limbs in judgment oldYour answer had not been inscrolld

This passage occurs in Act II Scene 7 in The Merchant of Venice This is part of the speech made by the Prince of Morocco

Explanation

In this passage the Prince of Morocco bestows high praise on Portia whose hand he is seeking He contrasts this casket containing Portiarsquos portrait with the old English gold coin bearing the image of the archangel (angel of the highest rank) He goes on to remark that while the figure of the archangel is engraved (Insculped) upon the English coin the picture of Portia who is beautiful as an angel lies hidden inside one of the caskets namely the Golden Casket (Golden Bed) In the day of Elizabeth silver was ten times inferior in value to gold Therefore the Prince of Morocco believing that Portiarsquos portrait is contained in the Golden Casket decides to choose the Golden Casket

Fare you well your suit is coldCold indeed and labour lostThen farewell heat and welcome frostmdashPortia adieu I have too grievd a heartTo take a tedious leave Thus losers part

(i) What reward does the Prince of Morocco get after making a wrong choice of the Casket How does he feel

After making the wrong choice in selecting the casket of gold the Prince of Morocco as a reward earns a rebuke in the form of a scroll tucked in the empty eye-socket of a skull kept in the casket of gold The Prince is shocked and disappointed He becomes all the more sad and dejected when he reads the scroll which points to his foolishness in being misled by the appearance and outward show as indicative of its worth

(ii) How does the Prince respond after reading the scroll

After reading the scroll the Prince though upset accepts the result with good grace and decorum befitting a royal suitor and true sportsman He says that his love-suit is really cold otherwise he would have chosen correctly but now his efforts have been in vain So he bids farewell to Portia to the warmth and enthusiasm of love and welcomes the cold and bitterness of dejection and misery of life which lies ahead

(iii) What request does he make to Portia and why

After being failure in his mission he requests Portia to give him permission to leave at once because he is too sad to undergo the tediousness of a formal leave-taking He tells that it is the manner in which defeated persons part unceremoniously

(iv) Explain the following lines

ldquoAll that glisters is not goldOften have you heard that toldMany a man his life hath soldBut my outside to beholdGilded tombs do worms infoldrdquo

Mere glitter does not make a metal to be gold Man has often been warned against appearance but it has been of no use Many people have sacrificed their lives only to seek the outer appearance of gold Worms are found inside the gilded

monuments

Class XSubject Topic Summary Execution

Hindi 2ndlang

नया रासता भाग 6 मायाराम 0ी घर म धनी मल 0ी और उनी बटी सरिरता ी ही चचा बनी रहती थी अमिमत ो इसम ोई रलिच ना थी वह धनी घर ी लडी स शादी र सवय ो बचना नही चाहता था उसा भी सवाणिभमान ह ईशवर ी पा

स उस पास पस ी ोई मी नही थी अभी उसन फकटरी ही लगाई थी उसी समझ बाहर था कि उस घर वालो ा झाव पस ी तरफ कयो

ह उसन मा स सवाल किया कि मा तम सरिरता स मरी शादी कयो रना चाहती हो मा न उस समझाया कि वह दखन म बरी नही ह और किफर खानदान अचछा

ह वह ए शल गरहणी रप म घर सभाल सगी अमिमत न मा ो इस बात ा एहसास राया कि मीन सबध लिलए मना रन पर उस दिदल

पर कया बीती होगी मा और अमिमत ी लडी बार म ाफी बात हईमा ा झाव सरिरता ी तरफ था कयोकि वह घर पर अचछा दह0 लर आ रही

थी अमिमत न अपनी मौसी ी बरी हालत बार म बताया कि किस तरह वह बड घर ी खानदानी बटी लाई थी और आ0 उसी हालत कितनी खराब ह लाई थी बहकलब 0ाती ह और बचचो ो भी नही दखती ह बात चल ही रही

थी कि तभी ए ार बाहर आर री धनी मल0ी घर अदर आए और पीछ स डराइवर फल ी ए टोरी लर आया अदर आए और पीछ स

डराइवर ए टोरी फल ी लर आया अमिमत ो फल ी पटी बरी लग रही थी अमिमत न पछ लिलया यह फल कयो ल आए ह प इन सब ी कया

0ररत थी उनो न 0वाब दिदया कि 4 पटी शमीर स मगाए थ अमिमत ो या सनर करोध आ गया तभी उस किपता 0ी आ गए उन आत ही अमिमत उठर बाहर चला गया वहा वहा मा पास आर बठ गया और बोला

अभी रिरशता तय नही हआ और धनी मल 0ी धनी मल 0ी फल ी पटी लर चलआय मा न समझाया कि 0ब सबध 0ड 0ाता ह तो खाली हाथ नही

आत अमिमत न मा स हा कि तम सबन सरिरता ो इस घर म लान ी ठान रखी ह धनीमल 0ी उस दिदन सरिरता ो दखन ी तारीख तय रन आय थ

Commercial Studies

Banking Nowadays Bank provide easy and quick services through internet facilities methods of Banking is called internet bankingIn order to save the time and money involved in visiting Bank branches people increasingly prefer to have internet banking

There are different modes of doing internet banking or transferring money through online They areReal Time Gross Settlement (RTGS)National Electronic Fund Transfers (NEFT)

1

Question

1) Explain the term RTGS Write the features of RTGS

Answer)The acronym RTGS stands for Real Time Gross Settlement which may be defined as the continuous real time settlement of funds transfer individually on and order by order basis without netting lsquoReal timersquo may be defined as the processing of instructions at the time they are received rather than at some letter time lsquoGross settlementrsquo may be defined as the settlement of transfer instructions which occurs

individually

Features of RTGS1It is the continuous settlement of

funds transfer individually on an order by order basis

2RTGS facility is provided only by CBS core banking solution enabled Bank branches

3Amount charged from the customer for RTGS transactions vary from bank to bank

2) Explain the term NEFT Write the features of NEFT

Answer) National electronic funds transfer may be defined as a nationwide system that facilitates individuals Farms and copper operates to electronically transfer funds from any bank branch to any individual farm or corporate having an account with any other bank branch in the country

Features of NEFT2 Transfer can be made 7 times on

weekdays and 6 times on Saturday

3 NEFT cannot be used to receive foreign remittances

4 NEFT transaction takes place in batches

5 A bank branch must be NEFT enabled to become a part of NEFT fund transfer network

6 There is no maximum or minimum amount that can be transferred through NEFT when one bank has a bank account

English Language

CompositionEssay

A composition is an art of creating a piece of writing on any topic or subject It is the writing correctly beautifully and clearly in order to make some interesting reading Structure of the composition

Introduction ( you lay the foundation for your composition)

Body (it constitutes the main part of the essay)

Conclusion (final statement that leaves a lasting impression)

Kinds of essays1 The Narrative essay2 The descriptive essay3 The reflective essay4 The argumentative essay

Write a composition on any one of the following topics (350- 400 words)

1 Friendship Or2 The first day of your school

Subject Eng Literature (The Merchant of Venice ndash William Shakespeare)Topic Act V Scene 1 Lines 127 to 158 (Nerissa helliphellip The clerk will nersquoer wear hair onrsquos face that had it) [Students should read the original play and also the paraphrase given in the school prescribed textbook]

Summary Revision Questions o Soon thereafter Bassanio Gratiano

and Antonio arrive

o Bassanio tells Portia that he is feeling as if it is morning because of the presence of Portia who is shining like the sun When Antonio is introduced by Bassanio to Portia she tells Bassanio that he should be grateful to Antonio who took so much trouble on his account even to the extent of risking his life

o Nerissa starts quarrelling with Gratiano and demands that he show her the ring she had presented to him and which she had warned him not to lose She suspects that Gratiano must have presented the ring to some young woman and not to the lawyerrsquos clerk as he repeatedly says and assures

Answer the following questions to check your preparation of Act IV Scenes 1 and 2

You must attempt only after you have completed your preparation of Act IV The answers must be in complete sentences using textual evidence (with citation) when necessary

[It would be in your own interest to attempt the above questions honestly totally refraining from consulting your textbook or your notes during answering After completion you should correct the paper yourself consulting the textbooknotes etc and award marks as specified Please let me know the marks you scored through WhatsApp in the group or to my personal WhatsApp]

Act IV Scene 1 (each question carries 2 marks)

1 What did the Duke try to do for Antonio

2 Why does Shylock refuse to show mercy How does he justify his stance

3 Why does Antonio say he is ready to die 4 What information is contained in Bellariorsquos letter

5 Why does Portia (as Balthazar) assert that Shylock must show mercy How does he respond

6 What offers are made to Shylock to get him to spare Antonio How are they received

7 What does Antoniorsquos speech as he faces the prospect of Shylockrsquos knife tell you about his character

8 How do Bassanio and Gratiano react to the looming prospect of Antoniorsquos demise

9 How does Portia (as Balthazar) use the law to turn the tables on Shylock

10 What does the Duke decree should happen to Shylock Why What happens to Shylockrsquos estate

11 What does Portia ask Bassanio as payment for her ldquoservicesrdquo What is his initial response What makes him change his mind

Act IV Scene 2 (each question carries 1frac12 marks)

1 What does Gratiano bring to Portia (Balthazar)

2 What does Nerissa plan on getting from Gratiano What does Portiarsquos comment suggest about men

ECO-10 280620 Topic-Supply AnalysisSHIFTING OF SUPPLY

But if there is change in factors other than the price of the commodity then either more is supplied at the same price or less supplied at the same price In such cases the price of the commodity remains constant but there is a change in other factors like change in the price of inputs change in technology of production change in price of other related goods change in taxation policy of the government etc For example there is an improvement in the technology of production of the commodity in question It leads to decrease in per unit of cost production of the commodity The firm is willing to sell more quantity of the commodity at the same price So the supply other commodity increases at the same price This increase in supply is shown by rightward shift of supply curve On the other hand if the firm uses inferior technology of production the cost of production per unit of the commodity increases The firm is willing to sell less quantity at the same price So the supply of the commodity decreases at the same price This decrease in supply is shown by leftward shift of the supply curve The above cases of increase and decrease in supply can be shown with the help of the following figures

Y INCREASE OF SUPPLY Price (Rs) s

P A s1

B

s

X` O s1 X

q q1

Y` Quantity demanded (in units)

Y DECREASE IN SUPPLY s2

s

price (Rs)

C

p A

s2

s

X` o X

q2 q

Y` Quantity demanded ( in units)

Main factors causing increase in supply or rightward shift of supply Curve(i) Fall in the price of other related goods

(ii) Fall in the price of inputsfactors(iii) Use of better technology in production(iv) Decrease in the rate of excise duty by government(v) If the objective of producer changes from profit maximization to salesMaximization

Main factors causing decrease in supply or leftward shift of supply curve(i) Increase in the price of other related goods(ii) Rise in the price of inputsfactors(iii) Use of inferior technology in production(iv) Increase in the rate of excise duty by the government(v) If the objective

Subject - Biology Topic ndash Chapter mdash6 PhotosynthesisSummary Execution

Today we will know about photosynthesis and its stages

Q1 What do you mean by photosynthesis The process by which living plants containing chlorophyll produce food

substances from carbon-di- oxide and water by using light energy Sunlight

6CO2 +12 H2O----------------------- C6 H12O6 + 6H2O + 6O2

Chlorophyll

Q2 What are the importance of photosynthesis I) Food for all Green plants trap solar energy by photosynthesis

process and supply food and energy for all living organisms either directly or indirectly

Ii) Oxygen to breathe in by product of photosynthesis is oxygen which is essential for all living organisms respiration

Q3 Write about two main phases of photosynthesis A Light dependent phase This phase occur in grana of chloroplast I) The chlorophyll on exposure to light energy becomes activated by

absorbing photons Ii) The absorbed energy is used in splitting the water molecules (H2O)

into its two components (H+ and OH- ) and releasing electron s 2H2O------------------------- 4H+ + 4e- +O2

Energy of 4 photons This reaction is known as photolysis

End products are H+ and oxygen water

B Light independent (Dark ) phase The reactions in this phase require no light energy

Here CO2 combine with H+ and produce glucose

Class XI

Subject Topic Summary ExecutionEVS Chapter-4 Legal

regimes for sustainable development

Environmental legislationEnvironmental legislation is the collection of laws and regulations pertaining to air quality water quality the wilderness endangered wildlife and other environmental factors The act ensures that matters important to the environment are thoroughly

Learn -The Forest (Conservation) Act 1980

considered in any decisions made by federal agencies

The Forest (Conservation) Act 1980 The Forest (Conservation) Act 1980 an Act of the Parliament of India to provide for the conservation of forests and for matters connected therewith or ancillary or incidental thereto It was further amended in 1988 This law extends to the whole of IndiaObjects and Reasons of the Forest Conservation Act

Deforestation causes ecological imbalance and leads to environmental deterioration Deforestation had been taking place on a large scale in the country and it had caused widespread concern The act seeks to check upon deforestation and de-reservation of forests

Subject Eng Literature (The Tempest ndash William Shakespeare) Topic Act II Scene 1 Lines 314 to 329 (End of scene)

[Students should read the original play and also the paraphrase given in the school prescribed textbook]Summary Questions amp Answers

Conspiracy of Antonio and Sebastian (Contd)

o As they approach Ariel appears again and wakes up Gonzalo by singing a tune in his ear Alonso also wakes up and they see both Sebastian and Antonio with drawn swords On being caught off guard they make up a story saying that they had heard a bellowing of bulls or lions

o They then moved to another part of the island

o Ariel at once rushes to Prospero to inform him of this development

SUMMING-UP of ACT-2 SCENE-1

(i) Among the survivors Ferdinand is separated from the rest which results in the disconsolate grief of Alonso as he took him for dead

(ii) The villainy of Antonio is confirmed

(iii) The supremacy of Prosperorsquos magic which resulted in the failure of the human conspiracy

(1)

(Act II Sc 1 L 311-325)SEBASTIAN Whiles we stood here securing your repose

Even now we heard a hollow burst of bellowing Like bulls or rather lions Didt not wake youIt struck mine ear most terribly

ALONSO I heard nothingANTONIO O rsquotwas a din to fright a monsters ear

To make an earthquake Sure it was the roarOf a whole herd of lions

ALONSO Heard you this GonzaloGONZALO Upon mine honour sir I heard a humming

And that a strange one too which did awake meI shaked you sir and cried As mine eyes opened I saw their weapons drawn There was a noiseThats verily rsquoTis best we stand upon our guardOr that we quit this place Lets draw our weapons

(i) Why has Prospero sent Ariel to Gonzalo and Alonso What does Ariel do to awaken Gonzalo

Prospero has already come to know by his magic powers the danger which threatens Gonzalo who had been Prosperorsquos friend and so he sent Ariel to preserve the lives of both Gonzalo and Alonso Prospero does not want that his scheme should remain unfulfilled Ariel begins to sing a song in Gonzalorsquos ears to awaken him(ii) Who are ready to carry out their plan Who takes steps to stop them Why does Gonzalo feel surprised after being awakened

Sebastian and Antonio are ready to carry out their plans They are standing with their swords drawn to kill Alonso and

(iv) We see two sets of contrasting characters Gonzalo-Adrian against Antonio-Sebastian

(v) The grief that works in Alonso can be perceived to his repentance for his association in Antoniorsquos crime against Prospero

Gonzalo Ariel takes steps to stop them from carrying out their nefarious scheme When Gonzalo is awakened by the song sung by Ariel into his ears he (Gonzalo) feels surprised because he sees Sebastian and Antonio standing with their swords drawn(iii) What reason do Sebastian and Antonio tell of drawing their swords when they are suspected by Alonso and Gonzalo

When Sebastian and Antonio are seen with their swords drawn they are looked with suspicion by Gonzalo and Alonso At first Sebastian tells them that as they stood here to guard them during their sleep they heard only a little before a sudden loud noise very much like the roaring of bulls or more probably that of lions Then Antonio follows him saying that this was a noise so terrible as to frighten even a monsterrsquos ears and this noise could even have shaken the earth and it was surely like the roaring of a multitude of lions Then seeing the danger they have drawn their swords Perhaps after hearing the terrible noise they (Gonzalo and Alonso) woke up from their sound sleep

(iv) What does Gonzalo tell Alonso about the strange noise What did he see on opening his eyes Gonzalo tells Alonso that he did not hear the sound of roaring but he heard a humming sound which was strange and which woke him up After waking up he gave him (Alonso) a shaking and a loud cry On opening his eyes he saw these two gentlemen standing with their swords drawn(v) What does Gonzalo suggest

Gonzalo suggests that there was a noise indeed and of that he has no doubt at all and suggests that the best course for them would be to remain alert and vigilant against any possible danger to their lives or to leave this place and move to some other part of the island

Class XIISubject Topic Summary Execution

Commerce

Chapter- Management

Today we will discuss about LEVELS OF MANAGEMENT

Levels of management is a series or chain of managerial positions from top to bottom It helps individuals to know their authority responsibilities and superior-subordinate relations among themselves There are mainly three levels of Management TOP LEVEL MANAGEMENTMIDDLE LEVEL MANAGEMENTLOWER LEVEL MANAGEMENT

Top level managementIt consists of members at the highest level in the management hierarchy This level includes Board Of Directors Chief Executive Managing Directors Chairman President Vice President

Rolefunctions of the top levelmanagement1To analyse evaluate and deal

with theexternal environment2 To determine the objectives and

policies of the business3 To strive for welfare and survival

of business

4 To create an organisational Framework consisting of authority responsibility relationship

Middle level management Congress of members or groups who are concerned with implementation of the policies let down by the top managementThis level includes head of the department such as finance manager marketing manager branch and regional managers departmental and divisional heads plant superintendent etc

Role of functions of the middle level management

1 To interpret the policies framed by top management

2 To assign duties and responsibilities to lower level managers

3 To select and appoint employees for middle and supervisory level and evaluate their performance

4 To co-operate with other departments for smooth functioning

Operational or supervisory level managementIt refers to the group are members who are concerned with execution of the work They are also known as fast line managers This level includes supervisor 4 men Section Officer clerk Inspector etc

Role of functions of the lower level management1 To plan and execute day-to-

day operations2 To supervise and control the workers3 To arrange materials and

tools to start the process and make arrangements for training

4 Today present workers grievance and suggestions before the management and

ensure safe and proper working conditions in the factory

Business Studies

Staff Appraisal Chapter- 10 Today let us start with a new chapter

Staff Appraisal

Meaning of Performance Appraisal

Performance Appraisal is the systematic evaluation of the performance of employees and to understand the abilities of a person for further growth and developmentThe supervisors measure the pay of employees and compare it with targets and plansThe supervisor analyses the factors behind work performances of employeesThe employers are in position to guide the employees for a better performance

Objectives of Performance Appraisal

Following are the objectives of Performance Appraisal

To maintain records in order to determine compensation packages wage structure salaries raises etc

To identify the strengths and weaknesses of employees to place right men on right job

To maintain and assess the potential present in a person for further growth and development

To provide a feedback to employees regarding their performance and related status

To provide a feedback to employees regarding their performance and related status

Importance of Performance Appraisal

Performance appraisal provides important and useful information for the assessment of employees skill

knowledge ability and overall job performance The following are the points which indicate the importance of performance appraisal in an organization

1 Performance appraisal helps supervisors to assess the work performance of their subordinates

2 Performance appraisal helps to assess the training and development needs of employees

3 Performance appraisal provides grounds for employees to correct their mistakes and it also provides proper guidance and criticism for employees development4 Performance appraisal provides reward for better performance

5 Performance appraisal helps to improve the communication system of the organization

6 Performance appraisal evaluates whether human resource programs being implemented in the organization have been effective

7 Performance appraisal helps to prepare pay structure for each employee working in the organization

8 Performance appraisal helps to review the potentiality of employees so that their future capability is anticipated

Geography

DRIANAGE The SubarnarekhaThe Subarnarekha and the Brahmaniinterposed between the Ganga and the Mahanadi deltas drain an area of 19300 sq kmand 39033 sq km respectively The drainage basins of these streams are shared byJharkhand Odisha west Bengal and Chhattisgarh The Brahmani is known as southKoel in its upper reaches in Jharkhand

The NarmadaThe Narmada rises in the Amarkantak hills of MadhyaPradesh It flows towards the West in a rift valleyformed due to a geological fault The total length of it is 1300 km All the tributaries of the

Q1 Name the two westward flowing rivers in the peninsular plateauA1 Narmada and Tapi are the only westward flowing rivers of the peninsular plateau

Q2 Differentiate between east-flowing rivers and west-flowing riversA2

East-flowing rivers

West-flowing rivers

Narmada are very short inlength Most of its tributaries join the main streamright anglesThe Narmada basin covers parts of Madhya Pradesh and Gujarat

The Tapi The Tapi rises in the Satpura ranges in the Betul listrictof Madhya Pradesh It flows in a rift valley parallel tothe Narmada but it is much shorter in length It coversparts of Madhya Pradesh Gujarat and MaharashtraThe length is about 724 km

The Sabarmati and the MahiThe Sabarmati rises in the Aravali hills and flows south-south-westwards for a distance of 300 kilometres to the Arabian Sea The Sabarmatibasin extends over an area of 21674 sq km in Rajasthan and Gujarat The Mahi rises inthe east of Udaipur and drains an area of 34842 sq km lying in Madhya PradeshRajasthan and Gujarat It flows south-westwards for a distance of 533 km before it fallsinto the Gulf of Khambhat

The ChambalThe Chambal rises near Mhow in the Vindhya Range and flows towards the northgenerally in a gorge upto Kota Below Kota it turns to the north-east direction and afterreaching Pinahat it turns to the east and runs nearly parallel to the Yamuna beforejoining it in the southern part of the Etawah district in Uttar PradeshMajor Rivers of India with their basin area (Sqkm)

Himalayan System Indus 321290Ganga 861404

Brahmaputra 187110Indus System

Jhelum 34775Beas 20303

Ganga System Yamuna 366223Ghaghra 127950

Peninsular RiversNarmada 98796

Tapi 65145Mahanadi 141600

Subarnarekha 19300Sabarmati 21674

Mahi 34842Godavari 312812

Godavari Krishna Kaveri Mahanadi are the east-flowing rivers

Narmada Tapi west-flowing rivers

They fall into the Bay of Bengal

They fall into Arabian Sea

These rivers form big deltas

These rivers form comparativelysmall deltas

Catchment areas of these rivers are larger

Catchment areas of these rivers are smaller

Krishna 2589488Cauveri 87900

Subject ndashBiology Topic ndashChapter -5 Inheritance amp Variations Summary ExecutionToday we will discussabout linkage and its classification

LINKAGE The tendency of the genes located on the same chromosome to stay together is

hereditary transmission Linked genes the genes responsible for this Genes that exhibit the process of linkage locates in the same chromosome The distance between the linked genes in a chromosome determines the strength

of linkage i e genes that are located close to each other show stronger linkage than that are located far from each other

COMPLETE LINKAGE It is the type of linkage showed by the genes that are closely located or are tightly

linked with each other as they have no chance of separatingby crossing over These genes are always transmitted together to the same gamete and the same

offspring In such condition only parental or non cross over type of gametes are formedINCOMPLETE KINKAGE It is type of linkage showed by the genes that are distantly located orare loosely

linked with each other because they have chance of separating by crossing over

SIGNIFICANCE i) It helps in holding the parental character togetherii) It checks the appearance of new recombination and helps in bringing the

hybrid population which resembles the original parents iii) Linked genes dilute the effects of undesirable traits

Subject Eng Literature (The Tempest ndash William Shakespeare) Topic Essay Questions (EQ-3)Question No 3

Give a character sketch of CalibanAnswer

The character of Caliban has been wonderfully conceived by Shakespeare as the manifestation of all that is gross and earthy ndash a sort of creature of the earth as Ariel is a sort of creature of the air

Calibanrsquos Physical Appearanceo Caliban is lsquofreckledrsquo a lsquomisshapen knaversquo not honoured with human shape

o Prospero calls him lsquothou tortoisersquo (Act I Sc 2 Line 317) Trinculo stumbling upon him describes him as ldquoA strange fish hellip Legged like a man And his fins like armsrdquo He ldquosmells like a fishrdquo (Act II Sc 2 Line 25)

o Prospero also calls him a ldquobeastrdquo (Act IV Sc 1 Line 140) and ldquoThis misshapen knaverdquo (Act V Sc 1 Line 268)

o Further it appears that in addition to his physical deformity his spiritual inferiority is also suggested by Prosperorsquos claim that his birth resulted from the union between his mother the witch Sycorax and the devil

Calibanrsquos ParentageWhen the play opens Caliban is twenty four years of age having been born on the island twelve years before the coming of Prospero His mother was the foul witch Sycorax who was banished from Algiers for ldquomischiefs manifold and sorceries terrible to enter human hearingrdquo (Act I Sc 2 Line 264) and the father was the Devil himself Thus

Caliban is a monster of evil and brute nature ugly deformed and stinking

Calibanrsquos Savage and Malignant Natureo Caliban is entirely a creature of the earth ndash gross brutal and savage He regards himself as the rightful possessor

of the island and Prospero as a usurper

o In his young age he was on good terms with Prospero He had consented to be received by Prospero at his house and to be educated by him He has learnt human language only to curse his master whom he abhors

o His beastly nature soon breaks out and ends in a vicious attack on Miranda This opens the eye of Prospero who becomes severe to him and enforces his service by threats and violence

o Prospero uses him to make dams for fish to fetch firewood scraper trenches wash dishes and keep his cell clean

Calibanrsquos Hatred for ProsperoA profound hatred for Prospero has taken hold of Caliban It springs from a sense of his being dispossessed and ill-treated He would kill Prospero if he could but he knows the power of Prosperorsquos lsquobookrsquo Hence he transfers his allegiance to Stephano who seems like a god to him He also incites the two drunken associates to batter the skull of Prospero when he sleeps in the afternoon

Caliban Shows Considerable Intelligenceo He has learnt Prosperorsquos language

ldquoYou taught me language and my profit onrsquot (Act II Sc 2 Lines 86-89)Is I know how to curserdquo

o He is well aware of the futility of arguing with one who has more power than he has

ldquoI must obey his art is such power (Act I Sc 2 Lines 373-376)It would control my damrsquos god SetebosAnd make a vassal of himrdquo

o He realizes the importance of Prosperorsquos books

ldquoRemember (Act III Sc 2 Lines 89-92)First to possess his books for without themHersquos but a sot as I am nor hath notOne spirit to commandrdquo

o He knows the value of stealth when attacking the enemy

ldquoPray you tread softly that the blind mole may not (Act IV Sc 1 Lines 194-195)Hear a foot fall we now are near his cellrdquo

o Caliban has a better set of values than Stephano and Trinculo They are distracted from their plan by their greed for Prosperorsquos rich garments Only Caliban realizes that such a finery is unimportant

ldquoLeave it alone thou fool it is but trashrdquo (Act IV Sc 1 Lines 224)

Caliban is not a good judge of characterCaliban is not a good judge of character He decides for example that Stephano is a god because he dispenses lsquocelestial liquorrsquo (Act II Sc 2 Line 115) but then it must be remembered that he has only known his mother Sycorax Prospero Miranda and the spirits that torture him However he quickly discovers his error of judgementrdquo

ldquoWhat a thrice-double ass (Act V Sc 1 Lines 295-297)Was I to take this drunkard for a godAnd worship this dull foolrdquo

Calibanrsquos Imaginative NatureIf Caliban is sub-human in what has been said above he is human in the respect of the poetic side of his character He listens to music with rapture He tells of the beautiful dreams in which heaven rains treasures upon him and which upon waking he yearns to renew One of the most poetic passages in whole play is Calibanrsquos description of the island

to Stephano and Trinculo

ldquoBe not afeard The isle is full of noises (Act III Sc 2 Lines 135-143)Sounds and sweet airs that give delight and hurt notSometimes a thousand twangling instrumentsWill hum about mine ears and sometime voicesThat if I then had waked after long sleepWill make me sleep again and then in dreamingThe clouds methought would open and show richesReady to drop upon me that when I wakedI cried to dream againrdquo

Caliban - Less Ignoble Than Some OthersCalibanrsquos motive for murder is less dishonourable than that of Antonio and Sebastian They plan to kill Alonso to gain his power and wealth Caliban merely wants revenge and the return of lsquohisrsquo island

Conclusiono Calibanrsquos character is not portrayed very clearly in the play and hence we cannot decide whether he is a poor

savage being grossly maltreated by Prospero or whether he is evil and must therefore be kept in bondage or enslavement

o Caliban is contrasted with Ariel who is a spirit and thus swift and uninterested in physical activitieso Caliban is also contrasted with Prospero who is the all-powerful master of the island and of the destiny of all

those on the islando Caliban is also contrasted with civilized man showing him to be less evil than Antonio and Stephano and less

materialistic than Stephano and Trinculoo Caliban has suffered at the hands of Prospero and he has learnt to curse by listening to Prosperorsquos abuse He

certainly believes that Prospero has deprived him of his birthrighto Finally the character Caliban is thought to be one of Shakespearersquos masterpieces The complexity of the character

is reflected in the large volume of critical discussion that has grown around it

ECO ndash12 Topic-Forms of market

MonopolyMonopoly is a market structure in which there is a single seller there are no close substitutes for the commodity produced by the firm and there are barriers to entry Example Indian Railways which is operated under government of India Monopoly also implies absence of competitionFeatures of Monopoly Monopoly is characterized by1 Single Seller In monopoly there is only one firm producing the product The whole industry consists of this single firm Thus under monopoly there is no distinction between firm and industry Being the only firm there is significant control of the firm over supply and price Thus under monopoly buyers do not have the option of buying the commodity from any other seller They have to buy the product from the firm or they can go without the commodity This fact gives immense control to the monopolist over the market

2No Close Substitute There are no close substitutes of the product produced by the monopolist firm If there are close substitutes of the product in the market it implies presence of more than one firm and hence no monopoly In order to ensure a total of control over the market by the monopolist firm it is assumed that there are no close substitutes of the product

3 No Entry amp Exit Monopoly can only exist when there is strong barriers before a new firm to enter the market In fact once a monopoly firm starts producing the product no other firm can produce the same One reason for this is the ability of the

monopolist to produce the product at a lower cost than any new firm who thinks to enter the market If a new firm who knows that it cannot produce at a lower cost than the monopolist then that firm will never enter the market for fear of losing out in competition Similarly the monopolist who is operating for a long time may be enjoying reputation among its customers and is in a better position to use the situation in its own benefit A new firm has to take long time to achieve this and so may not be interested to enter the market

4 Price Maker Being the single seller of the product the monopolist has full control over the pricing of the product On the other hand if there is a large number of buyers in the market so no single buyer exercises any significant influence over price determination Thus it is a sellerrsquos market So monopoly firm is a price maker

5 Price Discrimination Having considerable control over the market on account of being single seller with no entry of other firms the monopolist can exercise policy of price discrimination it means that the monopolist can sell different quantities of the same product to a consumer at different price or same quantity to different consumers at different prices by adjudging the standard of living of the consumer

6 Shape of Demand Curve Since a monopolist has full control over the price therefore he can sell more by lowering the price This makes the demand curve downward sloping

Subject Ac-12 290620 Topic- retirement Model sumThe Balance Sheet of Rohit Nisha and Sunil who are partners in a firm sharing profits according to their capitals as on 31st March 2014 was as under

Liabilities Amount Assets Amount (Rs) (` Rs)

Creditors 25000 Machinery 40000Bills Payable 13000 Building 90000General Reserve 22000 Debtors 30000Capital Less Provision for Rohit 60000 Bad debts 1000

29000 Nisha 40000 Stocks 23000 Sunil 40000 140000 Cash at Bank 18000

200000 200000

On the date of Balance Sheet Nisha retired from the firm and following adjustments were made(i) Building is appreciated by 20(ii) Provision for bad debts is increased to 5 on Debtors(iii) Machinery is depreciated by 10(iv) Goodwill of the firm is valued at Rs 56000 and the retiring partnerrsquos share is adjusted

(v) The capital of the new firm is fixed at Rs120000 Prepare Revaluation Account Capital Accounts of the partner and Balance Sheet of the new firm after Nisharsquos retirement Revaluation AccountDr Cr

Particulars Amount Particulars Amount (`Rs) (Rs`)

Provision for Bad debt Ac 500 Building Ac 18000Machinery Ac 4000Profit transferred toCapital Accounts (3 2 2)Rohit 5786Nisha 3857Sunil 3857

13500

18000 18000

Capital Account

Dr Cr

Particulars Rohit Nisha Sunil Particulars Rohit Nisha Sunil (Rs`) (Rs`) (`Rs) (Rs`) (Rs`) (Rs`)

Sunilrsquos Capital ac 9600 mdash 6400 Balance bd 60000 40000 40000Bank - 66143 - General Reserve 9428 6286 6286Balance cd 72000 mdash 48000 Revaluation (Profi 5786 3857 3857 Rohitrsquos Capital Ac mdash 9600 mdash

Sunilrsquos Capital Ac 6400 Bank 6386 - 4257

81600 66143 54400 81600 66143 54400

Balance Sheet as at 31st March 2014

Liabilities Amount Assets Amount (Rs`) (Rs`)

Creditors 25000 Building 108000Bank overdraft 37500 Machinery 36000

Bills Payable 13000 Debtors 30000Capital Less ProvisionRohit 72000 for Bad debts 1500 28500Sunil 48000 120000 Stock 23000

195500 195500

Working Notes (i) (a) Profit sharing ratio is 60000 40000 40000 ie = 3 2 2(b) Gaining Ratio Rohit = 35 ndash 37 = 2135 ndash 1535 = 635Sunil = 25-27 = 1435 ndash 1035 = 435= 635 435= 6 4 = 3 2(c) Nisha Share of Goodwill = Rs 56000 times 27 = Rs16000Share of Goodwill in the gaining ratio by the existing partner ieRohit = Rs16000 times 35 = Rs 9600Sunil = Rs 16000 times 25 = Rs 6400

The journal entry isRohitrsquos Capital Ac Dr 9600Sunilrsquos Capital Ac Dr 6400 To Nisharsquos Capital Ac 16000(Share of Goodwill divided into gaining ratio)

  • 1 Static Friction
  • The frictional force that acts between the surfaces when they are at rest with respect to each other is called Static Friction
    • Static Friction Examples
      • 2 Sliding Friction
        • Examples Of Sliding Friction
          • 3 Rolling Friction
            • Examples Of Rolling Friction
              • Objects and Reasons of the Forest Conservation Act
Page 3:  · Web viewSubject . Topic . Summary . Execution . English 1 . Sounds of animals . Hens –cackle Horses –neigh Lions –roar Owls –hoots Snake –hiss. English 2 . Mother’s

FACE- PART OF THE SHAPE THAT IS FLAT (OR CURVED) IS KNOWN AS FACE OF THE SOLID SHAPE A CUBE HAS 6 FACES

EDGE- THE LINE WHERE TWO FACES MEET IS KNOWN AS EDGEA CUBE HAS 12 EDGES

VERTEX-THE POINT WHERE TWO OR MORE EDGES MEET IS KNOWN AS VERTEXA CUBE HAS 8 VERTICES

General knowledge

Ch 21 part of plants

Refer to pg 25Do in the book and learn [ practise drawing a plant and label the different parts of the plant]

Qs 1 Which part of the plant grow under the groundAns Roots Qs2 Which part of the plant looks colourfulAns FlowerQs 3 Which part of the plant look greenAns Leaves Qs 4 Which part of the plant has leaves flowers and fruitsAns StemQs 5 Which part of a plant do you eatAns Fruit

Bengali বইndashবাংলা সাহিতয পহিরচয়

পাঠndash৭লপndashবকোহিনঅনশীলনীর পরকে4াততর

৩ সঠিক উততর যবকেনাও -ক) ( হিসটানকের বাঙাহিলকের ) সবকেচকেয় বকো উৎসব বহিনখ) এই হিনটি ( জরকেতর হিজশহিকেসটর জনমহিন) এইহিনটি পরম ( আনকেFর ঃকেখর ) হিনঘ) হিজGায় ( বাইকেবল যকারান ) পাঠকরায়ঙ) বহিকেনর হিপরয় খাবার কেলা ( যকক হিখচহি ) ৪ হিবপরীতশবদ -বকো timesযাকেIা জনম timesমতয আনFtimes ঃখ নতন timesপকেরাকেনা হিপরয় timesঅহিপরয় হিশশ times বদধপহিবতর times অপহিবতর আকেলা times অনধকার সমাহিNtimes সচনা৫ একেলাকেমকেলা শবদ সাহিজকেয় যলকেখা -যাহিবন = বকোহিন ইকেববাল = বাইকেবল নমজনহি = জনমহিন পারউ = উপার হিনয়মহিব = হিবহিনময় পাসউনা = উপাসনা

Hindi सवनाम अपन लिलए म हममरा हमारा मझ आदिद शबदो ा परयोग रत ह जि0सस बात र रह हउस लिलएmdashतम तमहारा तमह आप आपा आदिद शबदो ा परयोग रत ह जि0स बार म बात र रह ह या किसी अनय लिलएmdashवहव उस उना उनह उनहोनआदिद शबदो ा परयोग किया 0ाता ह ौन ोईकया आदिद शबद भी सवनाम ह

Read the chapter

Class IIISubject Topic Summary Execution COMPUTER

DRAWING IN MS PAINT

Q3) WHAT TOOL DO YOU USE TO INSERT TEXT IN MS PAINTANS) THE TEXT TOOL IS USED TO ADD TEXT IN MS PAINTQ4) WRITE THE STEPS TO USE THE ERASER TOOLANS) THE STEPS ARE

CLICK ON THE ERASER TOOL IN THE TOOLS GROUP DRAG THE MOUSE POINTER OVER THE AREA YOU WANT TO ERASE

বই ndash বাংলা সাহিতয পহিরচয়

পাঠ ndash ৮লপ ndash IনIহিন ও রাজার কাযলখক ndash উকেপনদরহিককেশার রায়কেচৌধরীঅনশীলনীর পরকে4াততর

৯ বাকয রচনা ককেরা -জবদ ndash সবাই হিমকেল যচারকেক পহিলকেশর াকেত তকেল হিকেয় জবদ করকেলা হিখলহিখল ndash নতন যখলনা যপকেয় হিশশটি হিখলহিখল ককের যকেস উঠকেলা কাতর ndash কাতর ককেS বদধ যলাকটি জল চাইহিল IফI ndash আঘাত লাায় ককরIা IফI করহিল -য়ানক ndash যকারনাকে -য়ানক পরলয় কেয়হিল

Hindi सवनाम अपन लिलए म हममरा हमारा मझ आदिद शबदो ा परयोग रत ह जि0सस बात र रह हउस लिलएmdashतम तमहारा तमह आप आपा आदिद शबदो ा परयोग रत ह mdashजि0स बार म बात र रह ह या किसी अनय लिलए वह व उस उना उनह उनहोन आदिद

शबदो ा परयोग किया 0ाता ह ौन ोई कया आदिद शबद भी सवनाम ह Science

Ch ndash Food we get from Plants

We see many plants around us They are of different shapes and sizes Some plants grow tall some stay short Some plants grow along the ground some plants need support to grow

Trees ndash big and tall plants are called trees Trees have thick hard and woody stem called trunk They have thin stems called branches Most trees live for many years A banyan tree can live for hundreds of years Mango banyan neem are examples of trees

Fill in the blanks (pg no- 63)

The main stem of a tree is called trunk

Cotton and hibiscus are examples of shrubs

Rice and mint are examples of herbs

Social studies

Indian literature Jataka talesThe jataka tales are short educational tales They have stories concerning the previous births of Gautam Buddha in both animal and human form These moral tales were written in fourth century and are based on the life of Gautama Buddha These are a collection of about 550 fables Every fable exhibits some virtue where

Buddha may appear in it as a king God or an elephant There are several translations of these Jataka tales in different languages

Read the portion which has been explainedEng language

Ch 19 Adverbs Adverbs are words that tells us more about an action In other words adding something more to the verb For eg The cat ran quicklyhere the word ran is the verb as it is showing an action done by the cat and quickly is the adverb as it is telling how the cat ran

Refer to pg 68Do in the bookWarm upUnderline the words that tell you how an action happened

The cat ran quicklyThe man ran swiftlyIt hurriedly went into its hole The mouse sat safely in its houseThe cat squeaked loudly

MAT

HEM

ATIC

S

Ch 7

Frac

tions

A fraction is a number that stands for parts of a whole object or a collection of objects

Each fraction has two numbers separated by a rule(a) The number above the rule is called numerator(b) The number below the rule is called denominator

Example 7minusminusminusminusrarr Numerator

13minusminusminusminusrarr Denominator

Notes

(1) 05 is a fraction in which numerator is 0

(2) There is no fraction whose denominator is zero

(3) When the numerator and denominator are equal the fraction represents the whole or 1

Example 33=1and2

2 = 1

Exercise ndash 421 Read the following fractional numbers

(c) 411 (d)

815 (e)

2337

Solution

(c) 411

four by eleven

(d)8

15Eight by fifteen

(e) 2337

twenty three by thirty seven

2 Write the following fractional numbers(d) Three over eleven (e) Eighteen by twenty one (f) Four-seventhsSolution

(d) Three over eleven 311

(e) Eighteen by twenty one 1821

(f) Four-sevenths 47

3 Write the numerator of each of the following

(e) 09 (f)

56 (g)

311 (h)

617

Solution (e) 0 (f) 5 (g) 3 (h) 64 Write the denominator of each of the following

(e) 08 (f)

813 (g)

719 (h)

417

Solution (e) 8 (f) 13 (g) 19 (h) 17

5 Write the fractions whose numerators and denominators are given below

(a) Numerator = 4 and Denominator = 13 (b) Numerator = 6 and Denominator = 17(c) Numerator = 11 and Denominator = 16(d) Numerator = 0 and Denominator = 9

Numerator Denominator

Fraction

(a) 4 13 413

(b) 6 17 617

(c) 11 16 1116

(d) 0 9 09

6 Find the fractions in which the denominator is 3 more than the numerator69

74

811

49

1113

711

1411

1613

2023

Solution 69

811

2023

8 A book seller had 15 books He sold 8 books What fraction of the books did he sell

Solution Out of 15 books the book seller sold 8 books

there4 The fraction of the books he sold =8

15

10 A class has 35 students On Monday 34 students were present What fraction of the students was absent

Solution Total number of student = 35On Monday 34 students were presentthere4 The number of student absent on Monday = (35 ndash 34) = 1

So fraction of the students was = 135

Class IVSubject Topic Summary Execution English language

Verbs the -ing form

Meaning of verb A word used to describe an action is known as verb

The ndash ing form of a verb is a very important form for it is used in many different waysExampleJack is playing(Present continuous)Jack was playing with his friends yesterday ( Past continuous)Jack will be playing in a match tomorrow ( Future continuous)So we see that the ndash ing form of verb used to form the continuous tenses

The ndash ing form of verb can also be used as an adjectiveExampleYou should not disturb a sleeping dogThe kettle is full of boiling water

Say which of the ndash ingforms of verbs in the sentences given below have been used as adjectives and which to form continuous tenses ( solved exercises please follow this)

3Interesting ndash adjective

4Was eating ndash past continuous tense

5terrifying ndashadjective

6are helping is spreading ndash present continuous tense

7will be visiting ndashfuture continuous tense

8twittering ndash adjective

Social studies

Map reading A Map helps us to see the whole World continents countries cities and neighborhood They are of different sizes handy and can be rolled up folded or carried easily However the maps do not show the surface accurately This is because the map is flat whereas the Earth is round in shape Cartography or a map making is a study and practice representation of the

One word answers1 It helps us to see the whole world countries and cities ndash Map

2 A study and practice representation of the Earth on a flat surface ndash Cartography

Earth on a flat surface

Elements of a mapTo be able to read a map we must understand the elements of a map

DirectionsDirections are the basic guidelines that help us to locate places It is very important for us to know the correct directions East West North and South are the four Cardinal directions These directions help us to locate the places Beside these there are four sub directionsndash Northeast Southeast North West and South West Compass is an instrument that helps to locate the directions

3 The basic guidelines to help us to locate places ndash directions

4These four directions help us to locate the places ndash Cardinal directions

COMPUTER

Formatting in ms word

Q3) What do you mean by formattingAns) Formatting is a general arrangement of text in a document We can change the appearance of a document by using the features available in ms word We can use different fonts colours and styles in the textQ4) What is alignmentAns) Alignment of text is the way in which it is placed between the margins of a page Text can be aligned to the left side in the centre or to the right side of a pageQ5) what is fontAns) A font is a style of writing and typing A font provides specific textual appearance to the document You can change the size style or give various effects to a font

Hindi 2ndlang

ाला किहरण सभी धमlt ा मल धम ह दया और रणा मरी हर 0ीव अपन ही दकिनया म रहत ह हम

पराणी कयो उनो परशान रत ह ऐसा हम नही रना चाकिहए

0ानवरो स हम ए सीख मिमलती ह किस तरह हम एता म रहना चाकिहए

हर पराणी पयार ी भाा समझता ह अगर हम पयार ी उममीद रत ह तो कया 0त

0ानवर हमस उममीद नही रत कि हम भी उनह पयार द यह हानी ए ाल किहरण ी ह 0ो अपन

समह ा नततव रता था हमशा कया उलिचत ह यह सोचता था अचान

उन पर लिशारिरयो न हमला बोल दिदया जि0सम ाला किहरण परा गया कयोकि वह

दलभ ह और लिशारिरयो ी न0र उसी पर थी परत 0ानवरो ी एता ो दखर

रा0मार ो दया आ गई और उसन उनो छोड दिदया 0ानवर भी बईमान नही थी वह उनस अकसर मिमलन उन बाद म 0ाता रहता रा0ा ो धनयवाद रता अबोध म

पराणिणयो म एता तजञता अभी भी ह

शबदाथ तराई ndash पहाड आसपास नीच

ी भमिमवश- ल या परिरवारअदवभत- अनोखाचौननाndash सत सावधानओझलndash गायबकिवपणिRndash मसीबतसगठन- एताउममीदndash आशाातर दमिU ndashकिववशता

हिहस ndash हानी पहचान वालातवयndash फ0

दल ndash समहनततवndash सचालन रना

ओट ndash आडआतमसमपणndash अपन ो सौप दनाम- चप

हम उनस सीख लनी चाकिहएবইndashবাংলা সাহিতয পহিরচয়

পাঠndash১০লপndashহিবকেবকানকেFর যকেলকেবলাযলখকndashশশী-ষণ াশগNঅনশীলনীর পরকে4াততর

৯প পহিরবতG ন ককেরা -শরীর = শারীহিরক -ত = য-ৌহিতক সG ার = সG াহির সথায়ী = সথাহিয়তবহিবশবাস = হিবশবাসী া = যকো১০ বাকয রচনা ককেরা -আডডা = পাার বদধরা াতলায় আডডা যয়হিডবাহিজ = রাম া যকেক হিডবাহিজ যখকেয় পকেলাসG ার = বকেলহিল যকেল যর সG ারবকহিন = অঙক -ল করায় রীনা মাকেয়র কাকে বকহিন যখকেলাবহিদধ = বহিদধ াককেল উপায় য়হিনশহিত = হিনশহিত রাকেত রাজবাহিকেত ডাকাত পকেলা-য়ানক = পাাহি রাসতা বষটির পকের -য়ানক হিবপ জনক য়হিবশবাস = মানকেষর পরহিত হিবশবাস ারাকেনা পাপ

Science Ch ndash The Food We Eat

Living things need food to live to grow to stay strong and fit When we need food we feel hungry Food gives us energy to do work It also protects us from diseases and helps us to stay healthy Nutrients in food The food we eat contains many substances that are necessary for our body These substances are called nutrients that help us to grow well and stay healthy

Nutrients give us energy to study work and play

They help our body to grow and repair the damaged parts of our body

They also help our body to fight against diseases and remain healthy

Write T for True or F for False (pg no- 11)

1 Food contains nutrients that help us to grow True

2 Foods rich in carbohydrates are called body-building foods False

MAT

HEM

ATIC

S

Ch 9

Com

mon

Fra

ction

s

A fraction is a number that stands for parts of a whole object or a collection of objects

Each fraction has two numbers One is written above the other separated by a line The one above the line is called numerator and the one below the line is called denominator

Example 5minusminusminusminusrarr Numerator

11minusminusminusminusrarr Denominator

Exercise ndash 362 In following fill in the blanks

(b) 37 ___ is denominator ___ is

numerator

(c) 89 ___ is denominator ___ is

numerator

Solution

(b) 37 7 is denominator 3 is numerator

(c) 89 9 is denominator 8 is numerator

3 Write down the fractions whose numerators and denominators are given below in the bracket The first number stands for numerator and the second number standing for denominator

(25) 25

(311) 311

(416) 416

(712) 712

Class VSubject Topic Summary Execution

Science Ch ndash PlantReproduction

In pollination chapter we have learnt that the flowers change into fruits and the fruits bear seeds Now we are going to learn how this process takes place

FertilizationThe process of fusion of the male reproductive cell (male gamete) and female reproductive cell (female gamete) is known as fertilizationWhen a pollen grain reaches from the anther to a stigma it begins to grow and forms apollentube The Pollen tube then travels down through the style to enter an ovule inside the ovary On reaching the ovule male reproductive cell in the pollen grain unites with the egg cell present in the ovule

Books exercise

A) Tick the correct answer

1Which of the following do ovules change into after fertilization ndashseeds

2Which of the following is not a part of the seed ndash flower

3Which of the following condition is needed for germination of a seed ndash all ofthese

English language

Sentences phrases and

Solved exercisesSay which of the underlined groups of words are phrases and which clauses

clauses8 In her new clothes ndashphrasesAs pretty as a doll ndash phrases

9 looking sad and upset ndash phrasesHe had lost all the tickets for the Test Match ndash clauses

10 During the vacation now only a month away ndashphrases

11 too sweet and too hot ndashphrases

12 At the Olympic Games ndashPhrasesOf Laurel leaves ndash phrases

13 Who played the role of Hamlet ndashclauses

14 However fast ndash phrases

15 When the men fell asleep ndash clausesSocial studies

Indian Government

Lok Sabha (lower house) ndash It has 552 members Of these 530 membersrepresent States 20 members represent the union territories and two members represent the Anglo- Indian community All except the representatives of the Anglo-Indian community are elected by Indian citizens A person above the age of 25 can contest in the elections for Lok Sabha One term of Lok Sabha is for 5 yearsRajya Sabha (upper house) ndash Its members are elected by the MLAs or members of the legislative Assembly There are 250 members in the Rajya Sabha of which 12 are nominated by the President One term of Rajya Sabha is for 6 years Anyone above the age of 30 can be elected as a member of Rajya Sabha

ExecutivePresidentThe President is the head of the country in India He is elected by the MPs and the MLAs for a tenure of 5 years He appoints the Prime Minister and the Council of Ministers Prime MinisterThe party which wins the election forms the government and its leader is elected as the Prime Minister He is the chief advisor to the President The Council of Ministers assists the Prime Minister and is accountable for their roles For example the Education minister is responsible for the education system in our country

1 How many members are there in Lok SabhaAns 552 members

2 What is the term for Lok SabhaAns 5 years

3 How many members are there in the Rajya SabhaAns 250 members

4 Who is the head of our countryAns President

5 Who is the chief advisor to the PresidentAns Prime Minister

Book ndash GK

Ch ndash 1First in space

1 First living being into space in 1957 Ans Laika

Times 2 First person to go into space in 1961 Ans Yuri Gagarin

3 First woman to go into space in 1963 Ans ValentinaTereshkova4 First person ever to walk in space in 1965Ans alexei Leonov5 First person to land on the moon in 1959 Ans Neil Armstrong6 First Indian to go into space in 1984 Ans Rakesh Sharma 7 First Indian woman to go into space in 19978 Ans Kalpana Chawla9 First woman tourist in space in 2006

Ans Anusheh AnsariCOMPUTER

ALGORITHM AND FLOWCHART

Q) DRAW THE SYMBOLS USED IN A FLOWCHART WITH THEIR DESCRIPTIONS(IN EXAM IT CAN COME AS SHORT QUESTIONS ASKING INDIVIDUAL SYMBOLS FUNCTION)ANS)

MAT

HEM

ATIC

S

Ch 6

Com

mon

Fra

ction

s

Multiplication of FractionsA Multiply a fractional number by whole numberTo multiply a fractional number by whole number we multiply the numerator of the fractional number by the whole number and denominator of the fractional number by 1 The first product thus obtained is the numerator and the second product is the denominator of the required product

Exercise ndash 30Multiply

7 2027

times 9

Solution 2027

times 9 = 203 = 6

23

8 611

times11

Solution 611

times11 = 6

15 71

20times16

Solution 71

20times16 =

14120

times16

= 1415

times 4 = 141times 4

5 = 564

5 = 11245

B Multiplication of a fractional number by a fractional number To multiply a fractional number by a fractional number we multiply the numerator of the first fractional number by the numerator of the second fractional number and the denominator of the first fractional number by the denominator of the second fractional number The first product thus obtained is the numerator and the second product is the denominator of the required product

16 2712

times24

Solution 2712

times24 = 3112

times24

= 31times2 = 62

Exercise ndash 31

11 83

times 34

2

Solution 83

times 34 = 2

14 723

times2 25

4

Solution 723

times2 25 =

233

times 125 =

23times 45

= 925 = 18

25

15 1212

times1 13

2

Solution 1212

times1 13 =

252

times 43 =

25times 23

= 503 = 16

23

State the following statements are true or false

17 1912

times 239 = 1

Solution LHS = 1912

times 239

= 392

times 239 = 1 = RHS

[LHS = Left hand side amp RHS = Right hand side]

there4 1912

times 239 = 1 [True]

21 213

times2 13 = 4

19

Solution LHS = 213

times2 13 =

73

times 73

= 7times73times3 =

499 = 5

49

there4 LHS ne RHS

So 213

times2 13 = 4

19 [False]

23 23

times 45 =

2times 5+3 times43times 5

Solution

LHS = 23

times 45 =

2times 43 times5 =

815 again

RHS = 2times 5+3 times4

3times 5 = 10+12

15 = 2215

there4 LHS ne RHS So 23

times 45 =

2times 5+3times43times 5

[False]

25 23 of

13 =

29

Solution

LHS= 23 of

13 =

23 times

13 =

29 = RHS]

there4 23 of

13 =

29 [True]

Practice at HomeExercise ndash 31State the following statements are true or false

24 12 of 4 =

18

Class VISubject Topic Summary Execution

HISTORY AND CIVICS

Chapter 5The Mauryan Empire

DECLINE OF MAURYAN EMPIREDecline of Mauryan empire started after the death of Ashoka at around 232 BCThere are several reasons for break up of the empire1 Weak successor Emperors after Ashoka were

capable of handling vast and mighty Mauryan empire In 185BC the last Mauryan ruler Bri-hadrath was murdered by his Commander-in-Chief Pushyamitra Sunga

2 Provincial Revolts Due to weak central author-ity provincial chiefs of Kalinga and southern provinces revolted against emperor and freed themselves from Mauryan empire

3 Weakness of Economy Prosperity of Mauryan was based on solid economic activities which

ExercisesI Multiple choice questions-1 Chandragupta defeated Seleucus in the year ndashc) 305 BC2 Who killed the last Mauryan ruler Brihadrath b) Pushyamitra3 Which of the following was not a reason for the decline of the Mauryan empirec) Chandraguptarsquos weakness4 Ashoka invaded Kalinga in the year c) 261 BC

II Fill in the blanks1Chandragupta ascended the throne in 324

was taken care by early monarchs Later kings had neither ability nor interest in economic af-fairs That led to failure in tax collection As a result they failed to maintain a large army that were essential to keep empire intact

4 Greek Invasion Greeks freed north-western provinces from weak Mauryan monarchs and reestablished their authority

5 Ashokarsquos Policy some scholar opined that after Kalinga war Ashoka embraced Buddhism re-nounced the policy of war and disbanded the Army But this is partially true as there is no proper evidence of disbanding the army

Based on above points we can conclude that main reason for decline of Mauryan empire is weakness of Ashokarsquos successors Kunal Samprati Dasharath Salisuk all were weak kingsAt last in 185 BCPushyamitra Sunga killed king Brihadrath and established the Sunga dynasty

BC2 Bindusara was the son of Chandragupta and father of Ashoka

3 Pataliputra was administered by City Magistrate committess of 5 members each4 The Greek General Seleucus sent his ambassador Megasthenes to Chandraguptarsquos court5 Ashoka sent his son Prince Mahendra and daughter Sanghamitra to spread his Dhamma6 The Indian Rebublic has adopted the Lion Capital of Saranath Pillar as its national emblem 7 Pushyamitra killed the last Mauryan ruler Brihadrath and founded the Sunga dynasty

III Name the following

1The author of Arthashastra-Kautilya2 The ruler who founded the Mauryan dynasty-Chandragupta3 The author of Indika-Megasthenes 4 The officers who were appointed by Ashoka to spread Dhamma-Dhamma Mahamatras5 The general of Alexander whom Chandragupta defeated-Seleucus

V Match the columns1 Kautilya (c)2 Megasthenes (d)3 Pushyamitra (e)4 Brihadrath (b)5 Bindusara (a)

BENGALI(2ND

LANGUAGE)

পশপাহিখর -াষাসহিবনয় রায়কেচৌধরী

যলখক পহিরহিচহিত- পরখযাত সাহিহিতযক উকেপনদরহিককেশার রায়কেচৌধরীর পতর সহিবনয় রায়কেচৌধরী lsquoসকেFশrsquo পহিতরকার সকেb হিতহিন কত হিকেলন তার উকেdখকোয বই lsquoসহিবনয় রায়কেচৌধরীর রচনা সংগরrsquo

পরম হিকেনর পাঠ- lsquoপশপাহিখর হিক -াষাhelliphellip helliphelliphelliphellipপরসপরকেক জানাবার উপায়ও পশপাহিখরা যবশ জাকেনrsquoপরকেমই আমারা জাহিন -াষা হিক -াষা ল আমাকের মকেনর -াব পরকাশ করার জনয আমরা নানান ধরকেনর -হিb বা হিবকেশষ ধরকেনর আওয়াজ মকেখর মাধযকেম কহির অনযকেক যবাঝাকেনার জনয তাকেল এবার আমরা জাহিন পশপাহিখর -াষা হিক পশ পাহিখরা হিক কা বকেল যা পশপাহিখকেরও -াষা আকে তারা তাকের হিনজসব -াষায় কা বকেল মকেনর -াব পরকাশ ককের পশ পাহিখরা মানকেষর হিক হিক -াষা যবাকেঝ হিকনত তারা বলকেত পাকেরনা পরসপরকেক বহিঝকেয় যবার উপায় তারা জাকেননা তকেব তারা হিবকেশষ ককেয়কটি শকেবদর মাধযকেম তাকের মকেনর -াব বহিঝকেয় যয় হিক বহিদধ মান জীব ndashককর হিবাল বন মানষ যঘাা পর-হিত এরা মানকেষর যওয়া নাম শনকেল কান খাা ককের ndash নাম ধকের ডাককেল কাকে আকেস যমন - মরহিরা lsquoহিত ndashহিতrsquo ডাক শকেন আকেস াল lsquoঅ ndashর -র ডাক শকেন কাকে আকেস াহিত মাহকেতর কা শকেন চকেল ককররা মাহিলকেকর হকম পালন ককের সবসময় তাইকেতা ককরকেক পর- -কত পরানী বলা য় ককর আর হিবাল একের আওয়াজ তহিম লকষয করকেল বঝকেব ককররা যরকে যকেল lsquoযঘউ যঘউrsquo করকেত াকেক আবার কাকেল lsquoযকউ যকউrsquo ককের হিবাল সাধারণ lsquoমযাওrsquo বা lsquoহিমউrsquo ককের রা কেল lsquoওয়াওrsquo আওয়াকেজর মাধযকেম মকেনর -াব পরকাশ ককের একেতা যল পশকের কা পাহিখরাও -য় রা পরকাশ করার জনয হিবকেশষ ধরকেনর শবদ ককের হিবপকের সময় পশ পাহিখরা সবার আকে পরসপরকেক জানাবার উপায় তারা জাকেন বহকাল

১) পশপাহিখর -াষা কেলপর যলখক সমপকেকG হিক জাকেনা

উঃ- পরখযাত সাহিহিতযক উকেপনদরহিককেশার রায়কেচৌধরীর পতর সহিবনয় রায়কেচৌধরী lsquoসকেFশrsquo পহিতরকার সকেb হিতহিন কত হিকেলন তার উকেdখকোয বই lsquoসহিবনয় রায়কেচৌধরীর রচনা সংগরrsquo হিতহিন ারকেমাহিনয়াম এসরাজ পর-হিত বাযনতর বাজাকেত পারকেতন ানও জানকেতন হিতহিন যাকেIাকের জনয মজাার লপ কহিবতা হিলখকেতন

২) পশপাহিখ কেলপর মল-াব হিকউঃ- পশপাহিখকেরও -াষা আকে তারা তাকের হিনজসব -াষায় কা বকেল মকেনর -াব পরকাশ ককের পশ পাহিখরা মানকেষর হিক হিক -াষা যবাকেঝ হিকনত তারা বলকেত পাকেরনা পরসপরকেক বহিঝকেয় যবার উপায় তারা জাকেননা তকেব তারা হিবকেশষ ককেয়কটি শকেবদর মাধযকেম তাকের মকেনর -াব বহিঝকেয় যয় হিরউকেবন কযাসটং সাকেব হিতহিন চহিdশ বর বনযজনত যর সকেb যকেককেন হিতহিন বকেলকেন আমরা হি তাকের -াষা তাকের আব কায়া যমকেন চহিল তাকেল আর -কেয়র যকান কারণ াকেকনা আমরা একI -াকেলাকেবকেস যচষটা করকেল পশপাহিখকের সকেb -াব পাতাকেত পাহির

ধকের মানষ এই পশ পাহিখর -াষা হিনকেয় নানা রককেমর পরীকষা ককের আসকে এইরকম একজন হিরউকেবন কযাসটাং সাকেকেবর কা আমরা জানকেবাhelliphellip

Hindi 2nd

langमतर किनमनलिलखिखतपरशनोउRरदीजि0ए

) बढ वयलि` बचच ो कया हआ था ख) डॉकटर साहबन पाटc किस उददशय स रखी थी ग) ाल साप ो हाथ म लर लाश न कया किया घ) डॉकटर चडढा न बढ पतरो दखन स कयो मना र दिदया था ङ) भगत न लाश ो दखर कया हा

उRर ndash) उस बहत बखार थी और 4 दिदनो स आख भी नही खोला थाख) उन बट ी सालकिगरह थीग) ाल सापो हाथ म लर लाश न उसी गदन 0ोर स दबार पडी थीघ) डॉकटर चडढा न बढ वयलि` पतरो दखन स मना र दिदया कयोकि उनह गोलफ खलन 0ाना थाङ) लाश ो दखर हा कि नारायण चाहग तो आध घट म भया उठ 0ाएग

English literature

In the bazaars of Hyderabad- Sarojini Naidu

Through the poem In The Bazaars of Hyderabad Sarojini wanted to convey the message that India is rich in tradition and they donrsquot need the foreign products So she goes on to give a picture of a bazaar where traditional Indian products are rulingThe poem is in the form of questions and answers The poet asks the questions and the merchants answer them Through this technique she make the picture of the bazaar visible to us

Read the poem

PHYSICS FORCE Types of FrictionThere are three types of friction static sliding rolling Static sliding and rolling friction occur between solid surfaces

1 Static Friction The frictional force that acts between the surfaces when they are at rest with respect to each other is called Static FrictionStatic Friction Examples

Skiing against the snow Creating heat by rubbing both the hands

together Table lamp resting on the table

2 Sliding Friction The resistance that is created between any two objects when they are sliding against each other is called Sliding FrictionExamples Of Sliding Friction

Sliding of the block across the floor Two cards sliding against each other in a

deck

3 Rolling Friction The force which resists the motion of a ball or wheel is called Rolling Friction Is the weakest types of frictionExamples Of Rolling Friction

Rolling of the log on the ground Wheels of the moving vehicles

6What effect can a force produce on a body which is not allowed to move Ans - When a force is applied on a body which is not free to move it gets deformed i e the shape or size of the body changes7Give one example each to indicate that the application of a force

1 produces motion2 stops motion3 slows down motion4 changes the direction of motion5 deforms a body

Ans- 1 A car originally at rest when pushed

begins to move2 A moving bicycle is stopped by

applying the brakes3 The speed of a moving vehicle is

slowed down by applying the brakes4 A player kicks a moving football to

change its direction of motion5 On stretching a rubber string its

length increases

8State the effect produced by a force in the following cases (a) The sling of a rubber catapult is stretched(b) A man pushes a heavy cart(c) A player uses his stick to deflect the ball (d) A cyclist applies brakes(e) A spring is compressedAns- (a) The shape and size of catapult changes ie its length increases(b) The heavy cart begins to move(c) The direction of the ball changes(d) The speed of the moving cycle is slowed down(e) There is change in size and shape of spring

COMPUTER MS EXCEL 2013 -INTRODUCTION

UNDERSTANDING EXCEL STRUCTUREA SPREADSHEET IS A FILE THAT EXISTS OF CELLS IN ROWS AND COLUMNS AND CAN HELP ARRANGE CALCULATE AND SORT DATA DATA IN A SPREADSHEET CAN BE NUMERIC VALUES AS WELL AS TEXT

FORMULAS REFERENCES AND FUNCTIONS

WORKSHEETA WORKSHEET IS ALSO KNOWN AS SPREADSHEETIT IS A COLLECTION OF CELLS ON A SINGLE SHEET WHERE YOU KEEP AND CHANGE DATA

WORKBOOKWORKBOOK IS PMS EXCEL FILE IN WHICH THE DATA CAN BE STORED EACH WORKBOOK CAN CONTAIN MANY WORKSHEETS

ROWS AND COLUMNSIN MS EXCEL A ROW IS A GROUP OF CELLS THAT RUN FROM LEFT TO RIGHT OF A PAGEA COLUMN IS A GROUPING OF CELLS THAT RUN FROM THE TOP TO THE BOTTOM OF A PAGE

CELLTHE INTERSECTION POINT BETWEEN A ROW AND THE COLUMN IS CALLED A CELL WHICH IS THE BASIC STORAGE UNIT FOR DATA IN A SPREADSHEET EACH CELL HAS SPECIFIC ADDRESS WHICH IS THE COMBINATION OF THE COLUMN NAME FOLLOWED BY THE ROW NUMBER

CHEMISTRY Chapter ndash Common Laboratory Apparatus and equipments

Objective type questionFill in the blanks (a) Experiment and observation are the two important basics of chemistry(b) A porcelain dish is used for evaporation(c) A test tube holder is used to hold the test tube while-it is heated(d) Mortar and pestle is used for grinding and crushing solid substances into a powder(e) Glass apparatus is made of Pyrex or borosil glass

Class VIISubject Topic Summary Execution

Hindi 2ndlang

ए था राम( डॉ शरी परसाद)

सगकित ा परभाव मानव 0ीवन पर अवशय पडता ह

हमशा मनषय ो अचछो ी सगकित म रहना चाकिहए

शरषठ परो सग स मनषय चरिरतर ा शीघर ही उदय और किवास हो 0ाता

ह इसलिलए वयलि` ो सदा शरषठ परो ा ही सग रना चाकिहए

इसान अगर चाह वह सवय ो बदल भी सता ह

यह हानी राम ए बचच ी हवह गणिणत ी परीकषा म नल रत हए पडा 0ाता ह और उस अधयाप पडत ह और पछत ह यह कया र रह

हो तभी राम न उनी बइजजती ी

शबदाथब ndashहावा भलावाायवाहीndash ाम किनयम व ानन

ो दिदखानापरिरलिचतndash 0ाना पहचानाघटनाndashघबराहट

उलटा चोर ोतवाल ो डाटndashकिववndash भल बर ा जञानतवयndash म 0ो रना चाकिहएसगकितndash बरी सगत

किबलख नाndashरोना किनशचय रनाndash तय रना

फलndashपरिरणामकिनषालिसतndash बाहर किया हआपशचातापndashदख सपननndashधनी

ldquo हा आपी किहममत स हई नल रत पडन ीrdquo ऐसी बात ही किफर

किपता0ी न भी उस डाटा वह ाफी पशचाताप रन लगा बोला गलत दोसतो

ी सगकित म आ0 कितना अनथ र दिदया किफर उसन अधयाप स माफी मागन ी सची और किफर भी ऐसा

नही रगा यह परण भी लिलया

सोचndashकिहच एात-अला

বইndashবাংলা সাহিতয পহিরচয়

পাঠndash১৬লপndashস-য ও অস-যযলখকndashঈশবরচনদর হিবযাসারঅনশীলনীর পরকে4াততর

৬ অGকেলকেখা -ময়া = পশ হিশকার সহিtহিত = হিনকIবতu সbভরষট = লI হিনরীকষণ =

-াকেলা-াকেব যখাকতাঞজহিলপকেI = যজাাকেত৭ হিবপরীতশবদ -ঈষৎ times পরচর উৎকষট times হিনকষট তাশ times উৎফd তবহিদধ times

বহিদধীNপাহিপষঠ times পণযবান৮ পপহিরবতG ন ককেরা -পশ = পাশহিবক যকাপ = যকাহিপতহিসথর = হিসথরতাএকানত = ঐকাহিনতক পর-াত times পর-াতী

CHEMISTRY

Chapter ndashPhysical and Chemical Changes

Chemical ChangeA chemical change involves a change in chemical composition

Characteristics of Chemical changes 1 They are permanent changes2 They are irreversible changes 3 New substance formed4 A Chemical change involves a

change in its chemical properties

Pg-25Question 8What do you observe when1 water is boiled2 a piece of paper is burnt3 some ice cubes are kept in a glass tumbler4 solid ammonium chloride is heated5 an iron nail is kept in tap water for few days6 a spoon of sugar is heated in a pan7 lighted match stick is brought near the mouth of the test tube containing hydrogen gas8 quick lime is dissolved in water9 little amount of curd is added to a bowl containing warm milk and kept for five hours

10 Water is boiledOn boiling water changes into steam (gas) physical change

11 A piece of paper is burnton burning piece of paper produces carbon dioxide and ash is left behind Is a chemical change

12 some ice cubes are kept in a glass tumblerIce cubes (solid) turn into water

(liquid) only state changes (physical change)

13 Solid ammonium chloride is heatedSolid ammonium chloride on heating changes into vapors (change of state) is physical change

14 An iron nail is kept in tap water for few dayswe observe reddish brown coating on the nail called rust (entirely new substance) is chemical change

15 A spoon of sugar is heated in a panWhen a spoon of sugar is heated in a pan black (charred sugar) (carbon) is seen Is a chemical change

16 Lighted match stick is brought near the mouth of the test tube containing hydrogen gasWe observe that hydrogen bums at the mouth of test tube with blue flame and pop sound is heard It is chemical change

17 Quick lime is dissolved in waterThe following two observations will be observed (i) A hissing sound is observed(ii) The mixture starts boiling and lime water is obtained

18 Little amount of curd is added to a bowl containing warm milk and kept for five hoursWhen a little amount curd is added to a bowl containing warm milk and kept for five hours a permanent change occurredThe milk will change to curd On boiling water changes into steam (gas) physical change

GEOGRAPHY

ATMOSPHERE IMPACT OF GLOBAL WARMING The destructive impart of global warming is observed in various spheres of life and the environment Some of the points are outlined below1 High temperatures lead to high

evaporation rate and drying up of the soil and surface water This affects crop production The occurrence of droughts is aggravating the problem even further

2 The heat waves in summer months

Q1 Write some impact of global warmingA1 The impacts of global warming are as follows1 High temperatures lead to high

evaporate ion rate and drying up of the soil and surface water This affects crop production The occurrence of droughts is aggravating the problem even further

2 The heat waves in summer months lead to a greater number

lead to a greater number of deaths due to heat strokes

3 Forest fires become more frequent4 Tropical cyclones and hurricanes

become common5 Melting of glaciers takes place6 Polar ice caps are becoming thinner

and melting at an alarming rate due to global warming The loss of sea ice

7 Due to increase in sea surface temperature sea levels rise in coastal areas and cause submergence of several islands

WAYS TO REDUCE GLOBAL WARMINGFollowing steps can be taken We need to decrease emission of

green house gases by reducing the burning of fossil fuel such as coal and petroleum

By planting more trees to increase forest cover

The government should also distributes free saplings and organize afforestation programmes to spread awareness regarding the beneficial effects of trees

We should switch to eco-friendly cars and gadgets

Incandescent light bulbs should be replaced by CFL bulbs

We can save electricity and reduce global warming by turning off electrical gadgets such as lights fans air-conditioners television and computer when we do not to use them

Efforts should be made to hasten the development of green cities oreco cities These cities are urban areas around the world striving to lessen the environment a impacts of urbanization

By following the 3Rs-Reduce Recycle and Reuse strategy we can use natural resources for our growth as well as save them for the need of the future generations This is called sustainable development

of deaths due to heat strokes3 Forest fires become more

frequent4 Tropical cyclones and hurricanes

become common5 Melting of glaciers takes place

etc

Q2 How to reduce global warmingA2 Following steps can be taken to reduce global warmingaWe need to decrease emission of

green house gases by reducing the burning of fossil fuel such as coal and petroleum

bBy planting more trees to increase forest cover

c The government should also distributes free saplings and organize afforestation programmes to spread awareness regarding the beneficial effects of trees

dWe should witch to eco-friendly cars and gadgets

eIncandescent light bulbs should be replaced by CFL bulbs

f We can save electricity and reduce global warming by turning off electrical gadgets such as lights fans air-conditioners television and computer when we do not to use them

Q3 What do you mean by 3Rrsquos of resource planningA3 The 3Rs are

1 Reduce 2 Recycle and3 Reuse

Q4 What is Sustainable developmentA4 By following the 3Rs-Reluce Recycle and Reuse strategy we can use natural resources for our growth as well as save them for the need of the future generations This is called sustainable development

English Language

Prepositions A preposition is a word placed before a noun or a pronoun It helps to show how the person or thing denoted by the noun is related to something else in the sentence

Kinds of Prepositions

Simple Prepositions- simple preposition are one word Prepositions such as at by for in of off for from on out through till to up with before amidst towards beyond between over etc

Compound Prepositions ndash There are some words that are always used with fixed Prepositions to convey specific meaning

Example I was unable to meet you dueto a previous engagement ( On account of)Always maintain the queue instead of crowding at the counter ( In place of)

Participial PrepositionsmdashParticiple Prepositions are present or past participles of various verbs which together with a noun phrase or a clause function as prepositions Examples- barring concerning considering notwithstanding pending regarding respecting etc

Exercise A

1 Gauravs fever has come down since Friday He has been absent for a week now

2 The child sat between his father and mother among the parents of all his classmates

3 There are mosquitoes in the room They flew into the room when the door was open

4 My father was inside the drawing room when I was playing outside my house

5 You may sit beside me I will give you a drawing book and pencils besides a storybook

6 We went to the market in the morning and walked towards the riverfront in the evening

7 The child walked along the pavement and across the street safely

8 This table top is made of glass My breakfast fell off it in the morning

9 The pan is on the gas stove There are vegetables in it

10 We will wait for you at the bus top There are a lot of people in the hall

Subject ndash Biology Topic ndash Chapter - 3 Photosynthesis and respiration in plants Summary Execution

All living organism (Plants and animals) need food for energy and growth Green plants (autotrophy) prepare food for all living organisms Today we will discuss about the process photosynthesis And adaptations in a leaf to carry out photosynthesis

Q1What do you mean by photosynthesis and write its word equation The process by which green plants make food (glucose) from carbon dioxide and water

in the presence of sunlight and chlorophyll is called photosynthesis

Carbon dioxide + Water ( Sun light from Sun ) Glucose + Oxygen ( chlorophyll in green leaves )

Q2 What are the adaptations in a leaf to carry out photosynthesisi) Leaves are broad wide and flat for absorbing more light energyii) Presence of chlorophyll in chloroplasts to trap sunlightiii) Presence of stomata which allow carbon dioxide to enter the cell and oxygen to go

out iv) Network of veins ensures continuous supply of water and minerals to the leafv) Thin waxy cuticle protects the leaf without blocking the lightQ3 Draw and label structure of chloroplast

Class VIIISubject Topic Summary Execution

PHYSICS ENERGY Production of Hydro electricity

A hydroelectric dam converts the potential energy stored in a water reservoir behind a dam to mechanical energymdashmechanical energy is also known as kinetic energy As the water flows down through the dam its kinetic energy is used to turn a turbine

The generator converts the turbinersquos mechanical energy into electricity

This electric energy then goes through various transmission processes before it reaches you

Question 2

Fill in the blanks

(a) Work is said to be done by a forte only when the body moves

(b) Work done = Force x distance moved in direction of force

(c) The energy of a body is its capacity to do work

(d) The SI unit of energy is joule

(e) The potential energy is due to its state rest of position and kinetic energy of the body is due to its state of motion

(f) Gravitational potential energy U = mass times force of gravity on unit mass times height

(g) Kinetic energy = frac12 times mass times (speed)2

(h) Power P = work donetime taken

(i) The S I unit of power is watt

(j) IHP = 746 W

BIOLOGY Chapter -5 The endocrine system and adolescence

Today we will discuss about thelocation and functions of secreted hormones of adrenal and Pancreas

Q5 Write location hormone secreted main functions and deficiency diseases of pancreas and adrenal glands

Endocrine Glands

Location Hormones secreted

Functions and Deficiency Diseases

1Adrenal gland

2 Pancreas Gland

On the top of each kidney

In between stomach and small intestine

i)Adrenaline from adrenal medulla

ii)Cortisone from adrenal cortex

i) Insulin

ii) Glucagon

It helps a person deal with any kind of emergency situation or emotional stressIt increases the heart beat rate of respiration and blood pressure

a) It regulates carbohydrates protein and fat metabolism

b) It regulates the salt and water balance in the body

a) It changes excess glucose into glycogen

b) It stimulates the cells to burn extra glucose to provide heat amp energy

Less secretion causes diabetes mellitus

Excessive secretions causeinsulin shock

a) It stimulates the breakdown of glycogen into glucose

b) It increases the level of glucose in blood

History Traders to rulers The Battle of Buxar was fought on 22 October 1764 between the forces under the command of the British East India Company led by Hector Munro and the combined armies of Mir Qasim the Nawab of Bengal till 1763 Mir Jafar was made the Nawab of Bengal for a second time in 1763 by the Company just after the battle After being defeated in 4 battles in katwa and Udaynala the Nawab of Awadh Siraj id Daula and the Mughal emperor Shah Alam II accompanied by Raja Balwant Singh of Kashi made an alliance with Mir Qasim The battle was fought at Buxar a small fortified

Answer the following questions- Short note-Battle of BuxarHomework-learn

town within the territory of Bihar located on the banks of the Ganga river about 130 kilometres (81 mi) west of Patna it was a decisive victory for the British East India Company The war was brought to an end by the Treaty of Allahabad in 1765

EnglishLiterature

The west wind-John Mansfield

In the poem The West Wind by John Masefield the poet starts by describingwith very poetic imagery of birds how the west wind is different from other winds its a warm wind full of birds cries There is a touch of melancholy perhaps home-sickness as he describes how it brings tears too and memories from an old land He goes on to describe the restful pastoral beauty of the land where even the dead can lie in the green He then brings in voicesperhaps of family and friends calling him home as he is missing Aprils beautyThe voices then tempt him some more with idyllic images from home (white blossom young green cornrunning rabbitswarm sun) The voices seem to presume that the poets heart is sorrowful bruised and soreThe end of the poem sees the poet appear to make a decision he will go home as he has decided that is where he truly belongs

Write the synopsis of the following words

1 Daffodils- a tall yellow flower that grows in the spring

2 Orchards- a piece of land on which fruit trees are grown

3 Blossom- a flower or a mass of flowers especially on a fruit tree in spring

4 Thrushes- a bird5 Larks- a small brown bird that

makes a pleasant sound6 Bruised- an injury7 Aching- pain 8 Tread- to put your foot down

while you are walking9 Balm-10 May-11 Fluting-

(Write from the book in your copy)

MAT

HEM

ATIC

S

Ch 1

1Al

gebr

ic E

xpre

ssio

n

1 Constant A symbol which has fixed value is called a constant[eg 8 23 -15 radic3 etc]

2 VariableA symbol which does not have any fixed value but may be assigned value (values) according to the requirement is called variable or literal[eg x y p q etc]

3 TermsA term is a number (constant) a variable a combination (product or quotient) of numbers and variables[eg 7 x 5x etc]

4 Algebric expressionA single term or acombination of two or more terms connected by plus (+) or minus (-) sign forms an algebraic expression[eg 5-y 3x2-5x xy-6z+4 etc]

5 PolynomialAn algebraic expression which contains more than one term is called a polynomial (multinomial)[eg x2-5x 5y+xy+x2y etc]

6 Degree of polynomial(a) When the polynomial contains only one variable the highest power of the variable is the degree of the polynomialeg the degree of the polynomial of 4x-7x5+8 is 5(b) When the polynomial contains two or more variablesStep (i) Find the powers of the variables in each term (ii) The highest sum of the powers is taken to be the degree of the polynomialeg the degree of the polynomial 5x2y-4x3y5+6 is = 3+5 = 8Remember An algebraic expression is a polynomial if degree of each term used in it is a non-negative integer

Exercise ndash 11(A)

1 Separate the constants and variables from the following

-7 7+x 7x+yz radic5 radic xy 3 yz

8 45y -3x

Solution Constant Variables-7 radic5 7+x 7x+yz radic xy

3 yz8

45y -3x

2 Write the number of terms in each of the following polynomials(i) 5x2+3timesax (ii) axdivide4-7 (iii) ax-by+ytimesz (iv) 23+atimesbdivide2

Solution Polynomials Number of terms(i) 5x2+3timesax 2(ii) axdivide4-7 2(iii) ax-by+ytimesz 3(iv) 23+atimesbdivide2 2

4 Write the degree of the each polynomials(i) xy+7z (ii) x2-6x3+8 (iii) y-6y2+5y8 (iv) xyz-3 (vi) x5y7-8x3y8+10x4y4z4

Solution Polynomials Degree(i) xy+7z 2(ii) x2-6x3+8 3(iii) y-6y2+5y8 8(iv) xyz-3 3(vi)x5y7-8x3y8+10x4y4z4 12

5Write the coefficient of(i) ab in 7abx (iv) 8 in a2-8ax+a (v) 4xy in x2-4xy+y2

SolutionCoefficient

(i) ab in 7abx 7x(iv) 8 in a2-8ax+a -ax(v) 4xy in x2-4xy+y2 -1

7 CoefficientAny factor of an algebraic quantity is called the coefficient of the remaining quantityeg in the algebraic term 7xyz 7 is coefficient of xyz 7x is coefficient of yz and so on

8 Like term The terms having the same literal coefficient are called like terms and those having different literal coefficients are called unlike terms

eg (i) 5xyz 8xyz -6xyz and 23xyz are like

terms(ii) 7xy2 8x2yz and -15xyz2 are unlike terms

6 in 57xy2z3 write the coefficient of

(i) 5 (vii) 5xy2 (viii) 17yz (xi) 5xyz

Solution Coefficient

(i) 5 17

xy2z3

(vii) 5xy2 17z3

(viii) 17yz

5xyzsup2

(xi) 5xyz 17yz2

7 In polynomial given below separate the like terms(ii) y2z3 xy2z3 -58x2yz -4y2z3 -8xz3y2 3x2yz and 2z3y2

Solution y2z3 -4y2z3 2z3y2 are like terms

xy2z3 -8xz3y2 are like terms

-58x2yz 3x2yz are like terms

Class IXSubject Topic Summary Execution

Bengali (2nd language)

বাগzwnjধারাzwnj বা ধারা-বা ধারা ল হিবকেশষ পরকার বাক -হিb -াকেবর এক হিবকেশষ পরকাশরীহিত াকেক কতগকেলা কার সমষটির মকেধয এগহিলকেক বা ধারা বকেল আবার কতগকেলা শকেবদর বাধাধরা যকান রীহিত যনই য-াকেব চকেল আসকে যসই -াকেবই চকেল আসকে তখন যসই শবদগহিল খন একক -াকেব অG পরকাশ ককের তখন একের বা ধারা বকেল বা ধারার পরকেয়া -াষাকেক আরও সFর ককের যতাকেল

অকাল পকক(অপহিরনত বয়কেস পাকাহিম)-মাতর শ বর বয়কেস যমকেয়টির া মকেখর কা তাকেত অকালপককতা ধরা পকে

অককা পাওয়া( মারা াওয়া) ndash পকেকIমারটি পকেকIমারকেত হিকেয় বাসাতরীকের াকেত মার যখকেত যখকেত অককা যপল

অহি| পরীকষা ( কঠিন ও পরকত পরীকষা)- যকেলটির আজ ডাকতাহির যরজালট যবকেরাকেব এIাই তার জীবকেনর ব অহি| পরীকষা

অষটরমভা (ফাহিক) ndash রীতা মকেখই বকো বকো কা বকেল আর কাকেজর যবলায় অষটরমভা

অকমGার ধাী (অপাG) ndash সমনকেক হিনকেয় যকান ান কেব না ও একেকবাকেরই অকমGার ধাী

অকেনধর ষটি (অসাকেয়র সায়)- আহিশ বকেরর বকোর নাহিত ল অকেনধর ষটি তাকেক াা বকোর একম চকেল না

আকেককল গড়ম (তবহিদধ)- ার তহিম উপকার করকেল যসই যতামার হিবরকেদধ সাকষয হিকেয়কে শকেনই আমার আকেককল গড়ম

আষাকে লপ( অবাসতব লপ) ndashIাকা এখন যকেব না এIা বলকেলই ত এমন আষাকে লপ ফাার যকান রকার হিল না

Hindi- महायजञ ा इस हानी म लख न या बतान ा परयास किया ह कि किसी भी अचछ

2nd language

परसार(यशपाल ाय या पणय न ा फल अवशय मिमलता ह ोई भी परोपार अथवा पणय लिलए किया गया ाय बार नही 0ाता वह ए परार ा यजञ हए धनी सठ थ धम परायण और किवनमर सठ न आन ी यजञ किए थ और दान म न 0ान कितना धन दिदन दखिखयो म बात दिदया थादिदन पलट और सठ यहा गरीबी आ गई उन दिदनो यजञ बचन ी परथा थी सठ भी अपनी 0गह बचन लिलए डलपर ए सट यहा चलन ो तयार हए सठानी रासत लिलए रोटी पड म बाधर सठ ो द दी रासत म ए भख R ो दखर सठ न चारो रोटी उसो खिखला दी खर वह सठ यहा डलपर पहच तो उनी सठानी न उस महायजञ बचन ो हा यदिद बचन आए सठ न R ो रोटी खिखलान ो महायजञ नही समझा और वापस लौट आया घर आर शाम ो उसी घर म उस ए बडा ख0ाना मिमला 0ो उस दवारा किए गएrsquo महायजञrsquo ा परसार था

English language

Letter formal The heading the name and address of the person you are writing to must be included beneath your own address In formal letters ldquoblock stylerdquo of address is preferred

Subject complain in brief

Salutation If the person you are writing to is known to you you may begin ldquoDear MrrdquoOr ldquoDear Mrsrdquo In all other instances you should begin ldquoDear Sirrdquo or ldquoDear Madamrdquo Or ldquoSirsrdquo

The body A formal or business letter has four partsReference The letter should begin by referring to a letter you have received an advertisement or the reason that has prompted you to writeInformation In the second paragraph it is necessary to supply more detailed information that is related to the referencePurpose Here you must give the reason why you are writing the letter This must be stated clearly and ensure that it is relevant to the question that has been setConclusion round off the letter with some polite remarkThe subscription when a letter has begun with dear sir sirs Madam you should end with Yours faithfully or yours truly When however you address a person by name you must conclude with the words ldquoYours sincerelyrdquo

1 A park in your locality is slowly being used as a rubbish dump Write a letter to the Mayor of your city pointing out the nuisance and danger of this Request that action be taken to stop this immediately

Or2 You being a boarder ordered a set of lab manuals from a famous book shop in the town They sent you a wrong set of books Write a letter to the manager of the book shop

Chemistry Chapter-1 1)CHEMICAL FORMULA- Q What is the Significance of

L-2The Language of Chemistrybull Chemical Formula

Itrsquos a symbolic representation of a chemical substance eg ndash The formula of Sulphuric acid is H2SO4

2) Steps of writing Chemical Formula of a given substance-

1 Write the symbols of the constituent atoms or radicals side by side Keep the basic radical on LHS and acid radical on the RHS ( Na+Cl- )2 In case of a radical having more than one atom( compound radical) enclose the radical in a bracket eg (SO4-)3 Write the valencies of each radical on its right hand top4 If the valencies of the two radicals are divisible by a common factor then divide the valencies by the common factor5 Invert (criss-cross) the valency number ie write the valency of one atom below the second atom and vice versa 6 On interchanging if valency number is lsquoone the figure lsquoonersquo is never writtenFor Example- Compound -Calcium Nitrate1 Writing the symbols- Ca(NO3)2 Writing the valencies on their right hand top- Ca2(NO3)1

3 Valency numeral in simple ratio- Ca2(NO3)1

4 Criss-cross- Ca 2NO3 1

5 Writing the formula of the compound- Ca(NO3)2

Chemical formula

A The formula of a substance conveys the following information regarding a substance 1 The name of the substance (qualitative)2 The elements constituting the substance (qualitative)3 The number of various atoms present in a molecule of the substance (quantitative)4 Molecular weight of the substance and the relative weights of different elements present in it (qualitative)

Q What are the limitations of Chemical Formula

A The chemical formula suffers from the following limitations-I It fails to convey whether the elements in a molecule are present in the form of atoms or ionsFor example the formula KBr fails to tell us whether Potassium and Bromine are present in the form of ions II It does not tell anything about the binding force that holds atom in a molecule togetherIII It does not tell us about the arrangement of various atoms with respect to one another within the molecule

Q Examples of Some Chemicals with their Formula Chemical name and Common Name-

A Given in the class notesCommercial Studies

Joint Stock Company

Let us discuss about the demerits of Joint Stock CompanyDespite so many advantages it has got many disadvantages which are as follows

Difficulty in FormationDelay in Decision makingExcessive Government ControlLack of Secrecy

Company can be classified into several categories based on incorporation

QuestionExplain the demerits of Joint Stock CompanyAnswer) 1 Difficulty in Formation The legal requirements and formalities required to be completed are so many The cost involved is quite heavy It has to approach large number of people for its capital It cannot start its business unless certificate of incorporation has been obtained This is granted after a long time when all the formalities are completed

Chartered CompanyStatutory CompanyRegistered Company

Delay in Decision making In this form of organization decisions are not made by single individual All important decisions are taken by the Board of Directors Decision-making process is time-consuming So many opportunities may be costly because of delay in decision-making Promptness of decisions which is a common feature of sole trader ship and partnership is not found in a company

Excessive Government ControlA company and the management have to function well within the law and the provisions of Companies Act are quite elaborate and complex At every step it is necessary to comply with its provisions lest the company and the management should be penalized The penalties are quite heavy and in several cases officers in default can be punished with imprisonment This hampers the proper functioning of the company

Lack of Secrecy The management of companies remains in the hands of many persons Every important thing is discussed in the meetings of Board of Directors Hence secrets of the business cannot be maintained In case of sole proprietorship and partnership forms of organisation such secrecy is possible because a few persons are involved in the management

2 Define the following

Chartered Company- The crown in exercise of the royal prerogative has power to create a corporation by the grant of a charter to persons assenting to be incorporated Such companies or corporations are known as chartered companies Examples of this type of companies are Bank of England (1694) East India Company (1600) The powers and the nature of business of a chartered company are defined by the charter which incorporates it After the country attained independence these types of companies do not exist

in IndiaStatutory Company- A company may be incorporated by means of a special Act of the Parliament or any state legislature Such companies are called statutory companies Instances of statutory companies in India are Reserve Bank of India the Life Insurance Corporation of India the Food Corporation of India etc The provisions of the Companies Act 1956 apply to statutory companies except where the said provisions are inconsistent with the provisions of the Act creating them Statutory companies are mostly invested with compulsory powersRegistered companiesCompanies registered under the Companies Act 1956 or earlier Companies Acts are called registered companies Such companies come into existence when they are registered under the Companies Act and a certificate of incorporation is granted to them by the Registrar

Economics

Chapter-4Basic problems of Economy

Today let us discuss with the topic Production Possibility curve

QuestionExplain the concept of Production Possibility Curve with the help of diagram

Answer) Production Possibility curve is a locus of all possible combinations of two commodities which can be produced in a country with its given resources and technology

The above diagram shows that with the given resources and technology the economy can produce maximum either 5 thousand meters of cloth or 15 thousand quintals of wheat or any other combination of the two goods like B( 1 thousand meters of cloth and 14 thousand quintals of wheat C ( 2 thousands meters of cloth and 12 thousand quintals of wheat) etcProduction Possibility curve is also called production possibility boundary or frontier as it sets the maximum limit of what it is possible to produce with given resources

Geography

Rotationand Revolution

SUNrsquoS POSITION AND SEASONAL CHANGES EQUINOXES ndash SPRING AND AUTUMN

Q1 What is Spring EquinoxA1 On 21st March sunrays fall directly on the equator On that day

As the Equator divides the Earth into two equal halves the sun rays fall directly on the equator twice in a year Equinoxes means equal Spring EquinoxOn 21st March sunrays fall directly on the equator On that day the duration of day and night both are equal ( 12 hours day and 12 hours night) on every places located on equator This day is called as Spring EquinoxAutumn EquinoxOn 23rd September sunrays fall directly on the equator On that day the duration of day and night both are equal ( 12 hours day and 12 hours night) on every places located on equator This day is called as Autumn Equinox

SOLSTICES ndash SUMMER AND WINTERDue to inclination of the Earth on its axis and the apparent movement of the sun the sun rays fall directly on both tropics once in a year Solstice is a Latin word which mean ldquothe Sun standing stillrdquoSummer SolsticesAfter 21st March there is an apparent movement of the Sun to the north of the equator The apparent northward movement up to 21st June when the Sun appears overhead at the Tropic of Cancer (22frac12degN) The sun appears to stand still at this position and then moves southwards towards the equator This position of the Sun on 21st June is known as Summer Solstices On that day the duration of day and night both are equal ( 12 hours day and 12 hours night) on every places located on Tropic of Cancer (22frac12degN)Winter solstices The apparent southward movement of the Sun continues beyond the equator till 22nd

December On this day the Sun is overhead at the Tropic of Capricorn

the duration of day and night both are equal ( 12 hours day and 12 hours night) on every places located on equator This day is called as Spring Equinox

Q2 What do you mean by EquinoxA2 Equinoxes means equal It is use to explain the equal duration of day and night ( 12 hours day and 12 hours night) on the Earth

Q3 On which date the longest day in Tropic of CancerA3 21st June

Q4 What is the meaning of SolsticeA4 Solstice is a Latin word which mean ldquothe Sun standing stillrdquo

Q5 Which is the longest day in southern hemisphereA5 22nd December

Q6 On what date does the Arctic Circle experience the lsquoMidnight SunrsquoA6 On 21 June the Arctic Circle experiences the lsquoMidnight Sunrsquo

Q7 What is cause of Midnight Sun in NorwayA7 During the summer solstice (21 June) the North Pole is inclined towards the Sun Therefore the duration of sunlight or daytime increases from 12 hours at the Equator to 24 hours at the Arctic Circle and beyond Thatrsquos why The region beyond the Arctic Circle especially Norway is known as the Land of the Midnight Sun because there the Sun does not rise or set on 21 June

Q8 Match the column A with BA B

Summer Solstice 21st March

Autumn Equinox 23rd

September

Winter Solstice 21st June

(22frac12degS) This position of the Sun is referred to as the Winter Solstice because it marks the winter season in the Northern Hemisphere On that day the duration of day and night both are equal ( 12 hours day and 12 hours night) on every places located on Tropic of Capricorn (22frac12degS)SEASONS AND DURATION OF DAY AND NIGHT During the equinoxes all places on the Earth have 12 hours of day and 12 hours of night Due to the revolution of the Earth round the Sun on an inclined axis the duration of day and night varies according to seasons and the latitude of a placeDuring the summer solstice (21 June) the North Pole is inclined towards the Sun Therefore the duration of sunlight or daytime increases from 12 hours at the Equator to 24 hours at the Arctic Circle and beyondThe region beyond the Arctic Circle especially Norway is known as the Land of the Midnight Sun because there the Sun does not rise or set on 21 JuneAt the North Pole there will be six months of daylight The Sun will be seen always above the horizon at a low angle At 66degN 24 hours of sunlight can be seen only on 21 June Hammerfest in northern Norway is a place of tourist attraction for observing the phenomenon of the Midnight Sun This place has continuous daylight from 13 May to 29 July This place is easily accessible to tourists and has hotels and other facilities The view of the midnight Sun from here is enthrallingIn the Southern Hemisphere the duration of daylight decreases from 12 hours at the equator to 0 hours beyond the Antarctic Circle In the South Polar Region there is 24 hours of darkness The Sun is always below the horizon In the Southern Hemisphere which experiences winter the duration of night-time is longer than the duration of daylight

Spring Equinox 22nd

December

A8 A B

Summer Solstice 21st June

Autumn Equinox 23rd

September

Winter Solstice 22nd

December

Spring Equinox 21st March

During winter solstice (22 December) the South Pole is inclined towards the Sun The Southern Hemisphere experiences summer and the Northern Hemisphere has winter Therefore the duration of daylight or sunlight is greater in the Southern Hemisphere than in the Northern HemisphereThe duration of daylight increases from 12 hours at the equator to 24 hours beyond the Antarctic Circle The South Polar Region has 24 hours of sunlight for many days continuously At the South Pole there will be six months of sunlight The Sun will always be seen at a low angle above the horizon In the Northern Hemisphere the duration of daylight will decrease from 12 hours at the equator to 0 hours at the Arctic Circle There are 24 hours of darkness in the North Polar region The duration of night is greater than the duration of daylight as one move northwards from the Equator It is evident from the above table that the duration of daylight is 12 hours throughout the year at the equator only As one moves away from the equator the seasonal variations in the duration of daylight increase The seasonal variations in the duration of daylight are maximum at the Polar Regions

Subject Eng Literature (The Merchant of Venice ndash William Shakespeare)Topic Act II Scene 7 Lines 36 to 80 (End of scene ) [Students should read the original play and also the paraphrase provided]

Summary Questions amp AnswersThe Prince then examines the inscription on the silver casket which says ldquoWho chooseth me shall get as much as he deservesrdquo The Prince says that he deserves Portia more than anybody else because of his high rank his noble birth and his great wealth and power But then he argues that silver is ten times

(1) (Act II Sc 7 L 39-47)

From the four corners of the earth they come

To kiss this shrine this mortal breathing saint

The Hyrcanian deserts and the vasty wildsOf wide Arabia are as through-fares now

inferior to gold and therefore he cannot believe that the portrait of such a beautiful lady as Portia can be contained in the silver casket He decides to see the inscription on the golden casket before making his decision

The Prince goes to examine the inscription on the golden casket which says ldquoWho chooseth me shall get what many men desirerdquo The Prince believes that the whole world desires to possess Portia otherwise so many suitors would not have come from all corners of the world for winning Portia Some of them have come from the distant lands of Persia and Arabia The deserts of Persia (Hyrcanian deserts) and the boundless desolate lands of Arabia have been crossed by the Princes seeking the hand of Portia He contrasts this casket containing Portiarsquos portrait with the old English gold coin bearing the image of the archangel (angel of the highest rank) He goes on to remark that while the figure of the archangel is engraved (Insculped) upon the English coin the picture of Portia who is beautiful as an angel lies hidden inside one of the caskets namely the Golden Casket (Golden Bed)

On the basis of his assessment of the inscription on the golden casket the Prince decides to choose the golden casket He asks for the key and opens the golden casket only to find therein an empty human skull holding a roll of

For princes to come view fair PortiaThe watery kingdom whose ambitious headSpets in the face of heaven is no barTo stop the foreign spirits but they comeAs orsquoer a brook to see fair Portia

(i) Explain the occasion for the above mentioned speech

These are the comments of the Prince of Morocco after he reads the inscription on the golden casket His mental process is revealed to us in these words We find him debating within himself as to which casket he should choose

(ii) What light does the above speech throw on the personality of Prince of Morocco

From the above mentioned speech we come to know that the Prince of Morocco is keen to marry Portia He is the type of person who is easily taken away by outward appearance He is in love with Portia because of her beauty

(iii) What information can you gather about Portia from the above mentioned lines

The given speech shows that Portia is a very beautiful lady She must be possessed of good qualities because many suitors come to her place from all over the world with a desire to get married to her The Prince of Morocco is so impressed by her beauty that he calls her a saint According to him the whole world is desirous of having her

(iv) Elucidate the significance of the first two lines

In these lines the Prince of Morocco pays a compliment to Portia These lines show his admiration for her He says that people come from all parts of the world to see fair Portia

(v) Explain the meaning of the last four lines of the

passage

In these lines the Prince of Morocco says that even the vast oceans which throw a challenge at the sky are unable to prevent men from coming to Portiarsquos place to have a glimpse of her These lines are also a tribute to Portiarsquos beauty and good qualities Many men voyage across the ocean treating it as a mere stream to see the beautiful Portia

paper in which is written that whoever happens to be guided by the glitter of things is invariably deceived

On reading the scroll the Prince says that he is too sad at heart to speak a more formal farewell and leaves with his followers amidst a sound of trumpets

After the Prince of Morocco leaves Portia remarks that the Prince is a gentle fellow but she is rid of him May all persons of his nature make a similar choice

IMPORTANT PASSAGES EXPLAINED

(Act II Sc 7 L 39-43)From the four corners of the earth they come

To kiss this shrine this mortal breathing saintThe Hyrcanian deserts and the vasty wildsOf wide Arabia are as through-fares nowFor princes to come view fair Portia

Context

This passage occurs in Act II Scene 7 in The Merchant of Venice This is part of the speech made by the Prince of Morocco

(2)

(Act II Sc 7 L 48-53)

MOROCCO One of these three contains her heavenly pictureIst like that lead contains her

Twere damnation To think so base a thought it were too grossTo rib her cerecloth in the obscure graveOr shall I think in silver shes immurdBeing ten times undervalued to tried gold

(i) What meaning does the Prince of Morocco find out of the inscription of the golden casket What have Belmont and Portiarsquos house been called and why

The inscription on the golden casket is ldquoWho chooseth me shall gain what many men desirerdquo The Prince finds out that it means that the chooser of the golden casket will get Portia because many men desire her In fact the entire world desires her Because of the coming of many suitors to Belmont from different countries in order to win Portiarsquos hand Belmont has become a centre of pilgrimage and her house is the shrine where saintly Portia is installed

(ii) What does the Prince of Morocco do before making the final choice of the casket Which is the correct casket and who will win Portiarsquos hand

The Prince of Morocco surveys and analyses the inscriptions on the casket of lead silver and gold Before making the final choice like a very systematic and methodical person he once again considers the claims of the caskets The casket containing Portiarsquos picture is the correct casket and the person choosing it will win Portiarsquos hand

Explanation

While praising Portia the Prince of Morocco conceives Portia as a goddess whose image is placed inside one of the caskets Many suitors are coming from far and wide the north and the south the east and the west (Four corners) in order to try their luck Some of them have come from the distant land of Persia and Arabia The deserts of Persia (Hyrcanian deserts) and the boundless desolate lands of Arabia have been crossed by the Princes seeking the hand of Portia All this shows that Portia is indeed the most beautiful lady of the world

(iii) What does the Prince of Morocco say in his estimation while examining the motto on the silver casket What does he find in the golden casket

While examining the motto on the silver casket which says ldquoWho chooseth me shall get as much as he deservesrdquo Morocco says that in his own estimation he surely deserves Portia in all respects ndash rank birth wealth etc

He chooses the golden casket When he opens it he finds an empty human skull holding a scroll in which it is written that those who are attracted by the glittering outside of things are always deceived as Morocco has been deceived

(iv) What kind of nature does the Prince of Morocco have

The Prince of Morocco has a simple nature who does not look deeply into the inner meaning of things but is dazzled by the outward appearance of gold He is inclined to over-estimate his own value and does not realize that it is a duty to ldquogive and hazardrdquo To say that he will not hazard for lead shows that he misreads the true meaning of the inscription which is that he should be prepared to ldquohazard all he hathrdquo for Portia So his feeling is only one of fascination and romantic attraction

(v) Do you think that the lottery of the caskets is not a matter that will be determined by chance

In fact the lottery of the casket is not a matter that will be determined by mere chance but that it is a true test of character and of sincerity which is amply proved not only by Moroccorsquos choice but also by the arguments which he uses to help him in his choice

(Act II Sc 7 L 55-59)

They have in England

A coin that bears the figure of an angelStamped in gold but thats insculpd uponBut here an angel in a golden bedLies all within

Context

(3)

(Act II Sc 7 L 63-77)A carrion Death within whose empty eye

There is a written scroll Ill read the writing

All that glisters is not goldOften have you heard that toldMany a man his life hath soldBut my outside to beholdGilded tombs do worms infoldHad you been as wise as boldYoung in limbs in judgment oldYour answer had not been inscrolld

This passage occurs in Act II Scene 7 in The Merchant of Venice This is part of the speech made by the Prince of Morocco

Explanation

In this passage the Prince of Morocco bestows high praise on Portia whose hand he is seeking He contrasts this casket containing Portiarsquos portrait with the old English gold coin bearing the image of the archangel (angel of the highest rank) He goes on to remark that while the figure of the archangel is engraved (Insculped) upon the English coin the picture of Portia who is beautiful as an angel lies hidden inside one of the caskets namely the Golden Casket (Golden Bed) In the day of Elizabeth silver was ten times inferior in value to gold Therefore the Prince of Morocco believing that Portiarsquos portrait is contained in the Golden Casket decides to choose the Golden Casket

Fare you well your suit is coldCold indeed and labour lostThen farewell heat and welcome frostmdashPortia adieu I have too grievd a heartTo take a tedious leave Thus losers part

(i) What reward does the Prince of Morocco get after making a wrong choice of the Casket How does he feel

After making the wrong choice in selecting the casket of gold the Prince of Morocco as a reward earns a rebuke in the form of a scroll tucked in the empty eye-socket of a skull kept in the casket of gold The Prince is shocked and disappointed He becomes all the more sad and dejected when he reads the scroll which points to his foolishness in being misled by the appearance and outward show as indicative of its worth

(ii) How does the Prince respond after reading the scroll

After reading the scroll the Prince though upset accepts the result with good grace and decorum befitting a royal suitor and true sportsman He says that his love-suit is really cold otherwise he would have chosen correctly but now his efforts have been in vain So he bids farewell to Portia to the warmth and enthusiasm of love and welcomes the cold and bitterness of dejection and misery of life which lies ahead

(iii) What request does he make to Portia and why

After being failure in his mission he requests Portia to give him permission to leave at once because he is too sad to undergo the tediousness of a formal leave-taking He tells that it is the manner in which defeated persons part unceremoniously

(iv) Explain the following lines

ldquoAll that glisters is not goldOften have you heard that toldMany a man his life hath soldBut my outside to beholdGilded tombs do worms infoldrdquo

Mere glitter does not make a metal to be gold Man has often been warned against appearance but it has been of no use Many people have sacrificed their lives only to seek the outer appearance of gold Worms are found inside the gilded

monuments

Class XSubject Topic Summary Execution

Hindi 2ndlang

नया रासता भाग 6 मायाराम 0ी घर म धनी मल 0ी और उनी बटी सरिरता ी ही चचा बनी रहती थी अमिमत ो इसम ोई रलिच ना थी वह धनी घर ी लडी स शादी र सवय ो बचना नही चाहता था उसा भी सवाणिभमान ह ईशवर ी पा

स उस पास पस ी ोई मी नही थी अभी उसन फकटरी ही लगाई थी उसी समझ बाहर था कि उस घर वालो ा झाव पस ी तरफ कयो

ह उसन मा स सवाल किया कि मा तम सरिरता स मरी शादी कयो रना चाहती हो मा न उस समझाया कि वह दखन म बरी नही ह और किफर खानदान अचछा

ह वह ए शल गरहणी रप म घर सभाल सगी अमिमत न मा ो इस बात ा एहसास राया कि मीन सबध लिलए मना रन पर उस दिदल

पर कया बीती होगी मा और अमिमत ी लडी बार म ाफी बात हईमा ा झाव सरिरता ी तरफ था कयोकि वह घर पर अचछा दह0 लर आ रही

थी अमिमत न अपनी मौसी ी बरी हालत बार म बताया कि किस तरह वह बड घर ी खानदानी बटी लाई थी और आ0 उसी हालत कितनी खराब ह लाई थी बहकलब 0ाती ह और बचचो ो भी नही दखती ह बात चल ही रही

थी कि तभी ए ार बाहर आर री धनी मल0ी घर अदर आए और पीछ स डराइवर फल ी ए टोरी लर आया अदर आए और पीछ स

डराइवर ए टोरी फल ी लर आया अमिमत ो फल ी पटी बरी लग रही थी अमिमत न पछ लिलया यह फल कयो ल आए ह प इन सब ी कया

0ररत थी उनो न 0वाब दिदया कि 4 पटी शमीर स मगाए थ अमिमत ो या सनर करोध आ गया तभी उस किपता 0ी आ गए उन आत ही अमिमत उठर बाहर चला गया वहा वहा मा पास आर बठ गया और बोला

अभी रिरशता तय नही हआ और धनी मल 0ी धनी मल 0ी फल ी पटी लर चलआय मा न समझाया कि 0ब सबध 0ड 0ाता ह तो खाली हाथ नही

आत अमिमत न मा स हा कि तम सबन सरिरता ो इस घर म लान ी ठान रखी ह धनीमल 0ी उस दिदन सरिरता ो दखन ी तारीख तय रन आय थ

Commercial Studies

Banking Nowadays Bank provide easy and quick services through internet facilities methods of Banking is called internet bankingIn order to save the time and money involved in visiting Bank branches people increasingly prefer to have internet banking

There are different modes of doing internet banking or transferring money through online They areReal Time Gross Settlement (RTGS)National Electronic Fund Transfers (NEFT)

1

Question

1) Explain the term RTGS Write the features of RTGS

Answer)The acronym RTGS stands for Real Time Gross Settlement which may be defined as the continuous real time settlement of funds transfer individually on and order by order basis without netting lsquoReal timersquo may be defined as the processing of instructions at the time they are received rather than at some letter time lsquoGross settlementrsquo may be defined as the settlement of transfer instructions which occurs

individually

Features of RTGS1It is the continuous settlement of

funds transfer individually on an order by order basis

2RTGS facility is provided only by CBS core banking solution enabled Bank branches

3Amount charged from the customer for RTGS transactions vary from bank to bank

2) Explain the term NEFT Write the features of NEFT

Answer) National electronic funds transfer may be defined as a nationwide system that facilitates individuals Farms and copper operates to electronically transfer funds from any bank branch to any individual farm or corporate having an account with any other bank branch in the country

Features of NEFT2 Transfer can be made 7 times on

weekdays and 6 times on Saturday

3 NEFT cannot be used to receive foreign remittances

4 NEFT transaction takes place in batches

5 A bank branch must be NEFT enabled to become a part of NEFT fund transfer network

6 There is no maximum or minimum amount that can be transferred through NEFT when one bank has a bank account

English Language

CompositionEssay

A composition is an art of creating a piece of writing on any topic or subject It is the writing correctly beautifully and clearly in order to make some interesting reading Structure of the composition

Introduction ( you lay the foundation for your composition)

Body (it constitutes the main part of the essay)

Conclusion (final statement that leaves a lasting impression)

Kinds of essays1 The Narrative essay2 The descriptive essay3 The reflective essay4 The argumentative essay

Write a composition on any one of the following topics (350- 400 words)

1 Friendship Or2 The first day of your school

Subject Eng Literature (The Merchant of Venice ndash William Shakespeare)Topic Act V Scene 1 Lines 127 to 158 (Nerissa helliphellip The clerk will nersquoer wear hair onrsquos face that had it) [Students should read the original play and also the paraphrase given in the school prescribed textbook]

Summary Revision Questions o Soon thereafter Bassanio Gratiano

and Antonio arrive

o Bassanio tells Portia that he is feeling as if it is morning because of the presence of Portia who is shining like the sun When Antonio is introduced by Bassanio to Portia she tells Bassanio that he should be grateful to Antonio who took so much trouble on his account even to the extent of risking his life

o Nerissa starts quarrelling with Gratiano and demands that he show her the ring she had presented to him and which she had warned him not to lose She suspects that Gratiano must have presented the ring to some young woman and not to the lawyerrsquos clerk as he repeatedly says and assures

Answer the following questions to check your preparation of Act IV Scenes 1 and 2

You must attempt only after you have completed your preparation of Act IV The answers must be in complete sentences using textual evidence (with citation) when necessary

[It would be in your own interest to attempt the above questions honestly totally refraining from consulting your textbook or your notes during answering After completion you should correct the paper yourself consulting the textbooknotes etc and award marks as specified Please let me know the marks you scored through WhatsApp in the group or to my personal WhatsApp]

Act IV Scene 1 (each question carries 2 marks)

1 What did the Duke try to do for Antonio

2 Why does Shylock refuse to show mercy How does he justify his stance

3 Why does Antonio say he is ready to die 4 What information is contained in Bellariorsquos letter

5 Why does Portia (as Balthazar) assert that Shylock must show mercy How does he respond

6 What offers are made to Shylock to get him to spare Antonio How are they received

7 What does Antoniorsquos speech as he faces the prospect of Shylockrsquos knife tell you about his character

8 How do Bassanio and Gratiano react to the looming prospect of Antoniorsquos demise

9 How does Portia (as Balthazar) use the law to turn the tables on Shylock

10 What does the Duke decree should happen to Shylock Why What happens to Shylockrsquos estate

11 What does Portia ask Bassanio as payment for her ldquoservicesrdquo What is his initial response What makes him change his mind

Act IV Scene 2 (each question carries 1frac12 marks)

1 What does Gratiano bring to Portia (Balthazar)

2 What does Nerissa plan on getting from Gratiano What does Portiarsquos comment suggest about men

ECO-10 280620 Topic-Supply AnalysisSHIFTING OF SUPPLY

But if there is change in factors other than the price of the commodity then either more is supplied at the same price or less supplied at the same price In such cases the price of the commodity remains constant but there is a change in other factors like change in the price of inputs change in technology of production change in price of other related goods change in taxation policy of the government etc For example there is an improvement in the technology of production of the commodity in question It leads to decrease in per unit of cost production of the commodity The firm is willing to sell more quantity of the commodity at the same price So the supply other commodity increases at the same price This increase in supply is shown by rightward shift of supply curve On the other hand if the firm uses inferior technology of production the cost of production per unit of the commodity increases The firm is willing to sell less quantity at the same price So the supply of the commodity decreases at the same price This decrease in supply is shown by leftward shift of the supply curve The above cases of increase and decrease in supply can be shown with the help of the following figures

Y INCREASE OF SUPPLY Price (Rs) s

P A s1

B

s

X` O s1 X

q q1

Y` Quantity demanded (in units)

Y DECREASE IN SUPPLY s2

s

price (Rs)

C

p A

s2

s

X` o X

q2 q

Y` Quantity demanded ( in units)

Main factors causing increase in supply or rightward shift of supply Curve(i) Fall in the price of other related goods

(ii) Fall in the price of inputsfactors(iii) Use of better technology in production(iv) Decrease in the rate of excise duty by government(v) If the objective of producer changes from profit maximization to salesMaximization

Main factors causing decrease in supply or leftward shift of supply curve(i) Increase in the price of other related goods(ii) Rise in the price of inputsfactors(iii) Use of inferior technology in production(iv) Increase in the rate of excise duty by the government(v) If the objective

Subject - Biology Topic ndash Chapter mdash6 PhotosynthesisSummary Execution

Today we will know about photosynthesis and its stages

Q1 What do you mean by photosynthesis The process by which living plants containing chlorophyll produce food

substances from carbon-di- oxide and water by using light energy Sunlight

6CO2 +12 H2O----------------------- C6 H12O6 + 6H2O + 6O2

Chlorophyll

Q2 What are the importance of photosynthesis I) Food for all Green plants trap solar energy by photosynthesis

process and supply food and energy for all living organisms either directly or indirectly

Ii) Oxygen to breathe in by product of photosynthesis is oxygen which is essential for all living organisms respiration

Q3 Write about two main phases of photosynthesis A Light dependent phase This phase occur in grana of chloroplast I) The chlorophyll on exposure to light energy becomes activated by

absorbing photons Ii) The absorbed energy is used in splitting the water molecules (H2O)

into its two components (H+ and OH- ) and releasing electron s 2H2O------------------------- 4H+ + 4e- +O2

Energy of 4 photons This reaction is known as photolysis

End products are H+ and oxygen water

B Light independent (Dark ) phase The reactions in this phase require no light energy

Here CO2 combine with H+ and produce glucose

Class XI

Subject Topic Summary ExecutionEVS Chapter-4 Legal

regimes for sustainable development

Environmental legislationEnvironmental legislation is the collection of laws and regulations pertaining to air quality water quality the wilderness endangered wildlife and other environmental factors The act ensures that matters important to the environment are thoroughly

Learn -The Forest (Conservation) Act 1980

considered in any decisions made by federal agencies

The Forest (Conservation) Act 1980 The Forest (Conservation) Act 1980 an Act of the Parliament of India to provide for the conservation of forests and for matters connected therewith or ancillary or incidental thereto It was further amended in 1988 This law extends to the whole of IndiaObjects and Reasons of the Forest Conservation Act

Deforestation causes ecological imbalance and leads to environmental deterioration Deforestation had been taking place on a large scale in the country and it had caused widespread concern The act seeks to check upon deforestation and de-reservation of forests

Subject Eng Literature (The Tempest ndash William Shakespeare) Topic Act II Scene 1 Lines 314 to 329 (End of scene)

[Students should read the original play and also the paraphrase given in the school prescribed textbook]Summary Questions amp Answers

Conspiracy of Antonio and Sebastian (Contd)

o As they approach Ariel appears again and wakes up Gonzalo by singing a tune in his ear Alonso also wakes up and they see both Sebastian and Antonio with drawn swords On being caught off guard they make up a story saying that they had heard a bellowing of bulls or lions

o They then moved to another part of the island

o Ariel at once rushes to Prospero to inform him of this development

SUMMING-UP of ACT-2 SCENE-1

(i) Among the survivors Ferdinand is separated from the rest which results in the disconsolate grief of Alonso as he took him for dead

(ii) The villainy of Antonio is confirmed

(iii) The supremacy of Prosperorsquos magic which resulted in the failure of the human conspiracy

(1)

(Act II Sc 1 L 311-325)SEBASTIAN Whiles we stood here securing your repose

Even now we heard a hollow burst of bellowing Like bulls or rather lions Didt not wake youIt struck mine ear most terribly

ALONSO I heard nothingANTONIO O rsquotwas a din to fright a monsters ear

To make an earthquake Sure it was the roarOf a whole herd of lions

ALONSO Heard you this GonzaloGONZALO Upon mine honour sir I heard a humming

And that a strange one too which did awake meI shaked you sir and cried As mine eyes opened I saw their weapons drawn There was a noiseThats verily rsquoTis best we stand upon our guardOr that we quit this place Lets draw our weapons

(i) Why has Prospero sent Ariel to Gonzalo and Alonso What does Ariel do to awaken Gonzalo

Prospero has already come to know by his magic powers the danger which threatens Gonzalo who had been Prosperorsquos friend and so he sent Ariel to preserve the lives of both Gonzalo and Alonso Prospero does not want that his scheme should remain unfulfilled Ariel begins to sing a song in Gonzalorsquos ears to awaken him(ii) Who are ready to carry out their plan Who takes steps to stop them Why does Gonzalo feel surprised after being awakened

Sebastian and Antonio are ready to carry out their plans They are standing with their swords drawn to kill Alonso and

(iv) We see two sets of contrasting characters Gonzalo-Adrian against Antonio-Sebastian

(v) The grief that works in Alonso can be perceived to his repentance for his association in Antoniorsquos crime against Prospero

Gonzalo Ariel takes steps to stop them from carrying out their nefarious scheme When Gonzalo is awakened by the song sung by Ariel into his ears he (Gonzalo) feels surprised because he sees Sebastian and Antonio standing with their swords drawn(iii) What reason do Sebastian and Antonio tell of drawing their swords when they are suspected by Alonso and Gonzalo

When Sebastian and Antonio are seen with their swords drawn they are looked with suspicion by Gonzalo and Alonso At first Sebastian tells them that as they stood here to guard them during their sleep they heard only a little before a sudden loud noise very much like the roaring of bulls or more probably that of lions Then Antonio follows him saying that this was a noise so terrible as to frighten even a monsterrsquos ears and this noise could even have shaken the earth and it was surely like the roaring of a multitude of lions Then seeing the danger they have drawn their swords Perhaps after hearing the terrible noise they (Gonzalo and Alonso) woke up from their sound sleep

(iv) What does Gonzalo tell Alonso about the strange noise What did he see on opening his eyes Gonzalo tells Alonso that he did not hear the sound of roaring but he heard a humming sound which was strange and which woke him up After waking up he gave him (Alonso) a shaking and a loud cry On opening his eyes he saw these two gentlemen standing with their swords drawn(v) What does Gonzalo suggest

Gonzalo suggests that there was a noise indeed and of that he has no doubt at all and suggests that the best course for them would be to remain alert and vigilant against any possible danger to their lives or to leave this place and move to some other part of the island

Class XIISubject Topic Summary Execution

Commerce

Chapter- Management

Today we will discuss about LEVELS OF MANAGEMENT

Levels of management is a series or chain of managerial positions from top to bottom It helps individuals to know their authority responsibilities and superior-subordinate relations among themselves There are mainly three levels of Management TOP LEVEL MANAGEMENTMIDDLE LEVEL MANAGEMENTLOWER LEVEL MANAGEMENT

Top level managementIt consists of members at the highest level in the management hierarchy This level includes Board Of Directors Chief Executive Managing Directors Chairman President Vice President

Rolefunctions of the top levelmanagement1To analyse evaluate and deal

with theexternal environment2 To determine the objectives and

policies of the business3 To strive for welfare and survival

of business

4 To create an organisational Framework consisting of authority responsibility relationship

Middle level management Congress of members or groups who are concerned with implementation of the policies let down by the top managementThis level includes head of the department such as finance manager marketing manager branch and regional managers departmental and divisional heads plant superintendent etc

Role of functions of the middle level management

1 To interpret the policies framed by top management

2 To assign duties and responsibilities to lower level managers

3 To select and appoint employees for middle and supervisory level and evaluate their performance

4 To co-operate with other departments for smooth functioning

Operational or supervisory level managementIt refers to the group are members who are concerned with execution of the work They are also known as fast line managers This level includes supervisor 4 men Section Officer clerk Inspector etc

Role of functions of the lower level management1 To plan and execute day-to-

day operations2 To supervise and control the workers3 To arrange materials and

tools to start the process and make arrangements for training

4 Today present workers grievance and suggestions before the management and

ensure safe and proper working conditions in the factory

Business Studies

Staff Appraisal Chapter- 10 Today let us start with a new chapter

Staff Appraisal

Meaning of Performance Appraisal

Performance Appraisal is the systematic evaluation of the performance of employees and to understand the abilities of a person for further growth and developmentThe supervisors measure the pay of employees and compare it with targets and plansThe supervisor analyses the factors behind work performances of employeesThe employers are in position to guide the employees for a better performance

Objectives of Performance Appraisal

Following are the objectives of Performance Appraisal

To maintain records in order to determine compensation packages wage structure salaries raises etc

To identify the strengths and weaknesses of employees to place right men on right job

To maintain and assess the potential present in a person for further growth and development

To provide a feedback to employees regarding their performance and related status

To provide a feedback to employees regarding their performance and related status

Importance of Performance Appraisal

Performance appraisal provides important and useful information for the assessment of employees skill

knowledge ability and overall job performance The following are the points which indicate the importance of performance appraisal in an organization

1 Performance appraisal helps supervisors to assess the work performance of their subordinates

2 Performance appraisal helps to assess the training and development needs of employees

3 Performance appraisal provides grounds for employees to correct their mistakes and it also provides proper guidance and criticism for employees development4 Performance appraisal provides reward for better performance

5 Performance appraisal helps to improve the communication system of the organization

6 Performance appraisal evaluates whether human resource programs being implemented in the organization have been effective

7 Performance appraisal helps to prepare pay structure for each employee working in the organization

8 Performance appraisal helps to review the potentiality of employees so that their future capability is anticipated

Geography

DRIANAGE The SubarnarekhaThe Subarnarekha and the Brahmaniinterposed between the Ganga and the Mahanadi deltas drain an area of 19300 sq kmand 39033 sq km respectively The drainage basins of these streams are shared byJharkhand Odisha west Bengal and Chhattisgarh The Brahmani is known as southKoel in its upper reaches in Jharkhand

The NarmadaThe Narmada rises in the Amarkantak hills of MadhyaPradesh It flows towards the West in a rift valleyformed due to a geological fault The total length of it is 1300 km All the tributaries of the

Q1 Name the two westward flowing rivers in the peninsular plateauA1 Narmada and Tapi are the only westward flowing rivers of the peninsular plateau

Q2 Differentiate between east-flowing rivers and west-flowing riversA2

East-flowing rivers

West-flowing rivers

Narmada are very short inlength Most of its tributaries join the main streamright anglesThe Narmada basin covers parts of Madhya Pradesh and Gujarat

The Tapi The Tapi rises in the Satpura ranges in the Betul listrictof Madhya Pradesh It flows in a rift valley parallel tothe Narmada but it is much shorter in length It coversparts of Madhya Pradesh Gujarat and MaharashtraThe length is about 724 km

The Sabarmati and the MahiThe Sabarmati rises in the Aravali hills and flows south-south-westwards for a distance of 300 kilometres to the Arabian Sea The Sabarmatibasin extends over an area of 21674 sq km in Rajasthan and Gujarat The Mahi rises inthe east of Udaipur and drains an area of 34842 sq km lying in Madhya PradeshRajasthan and Gujarat It flows south-westwards for a distance of 533 km before it fallsinto the Gulf of Khambhat

The ChambalThe Chambal rises near Mhow in the Vindhya Range and flows towards the northgenerally in a gorge upto Kota Below Kota it turns to the north-east direction and afterreaching Pinahat it turns to the east and runs nearly parallel to the Yamuna beforejoining it in the southern part of the Etawah district in Uttar PradeshMajor Rivers of India with their basin area (Sqkm)

Himalayan System Indus 321290Ganga 861404

Brahmaputra 187110Indus System

Jhelum 34775Beas 20303

Ganga System Yamuna 366223Ghaghra 127950

Peninsular RiversNarmada 98796

Tapi 65145Mahanadi 141600

Subarnarekha 19300Sabarmati 21674

Mahi 34842Godavari 312812

Godavari Krishna Kaveri Mahanadi are the east-flowing rivers

Narmada Tapi west-flowing rivers

They fall into the Bay of Bengal

They fall into Arabian Sea

These rivers form big deltas

These rivers form comparativelysmall deltas

Catchment areas of these rivers are larger

Catchment areas of these rivers are smaller

Krishna 2589488Cauveri 87900

Subject ndashBiology Topic ndashChapter -5 Inheritance amp Variations Summary ExecutionToday we will discussabout linkage and its classification

LINKAGE The tendency of the genes located on the same chromosome to stay together is

hereditary transmission Linked genes the genes responsible for this Genes that exhibit the process of linkage locates in the same chromosome The distance between the linked genes in a chromosome determines the strength

of linkage i e genes that are located close to each other show stronger linkage than that are located far from each other

COMPLETE LINKAGE It is the type of linkage showed by the genes that are closely located or are tightly

linked with each other as they have no chance of separatingby crossing over These genes are always transmitted together to the same gamete and the same

offspring In such condition only parental or non cross over type of gametes are formedINCOMPLETE KINKAGE It is type of linkage showed by the genes that are distantly located orare loosely

linked with each other because they have chance of separating by crossing over

SIGNIFICANCE i) It helps in holding the parental character togetherii) It checks the appearance of new recombination and helps in bringing the

hybrid population which resembles the original parents iii) Linked genes dilute the effects of undesirable traits

Subject Eng Literature (The Tempest ndash William Shakespeare) Topic Essay Questions (EQ-3)Question No 3

Give a character sketch of CalibanAnswer

The character of Caliban has been wonderfully conceived by Shakespeare as the manifestation of all that is gross and earthy ndash a sort of creature of the earth as Ariel is a sort of creature of the air

Calibanrsquos Physical Appearanceo Caliban is lsquofreckledrsquo a lsquomisshapen knaversquo not honoured with human shape

o Prospero calls him lsquothou tortoisersquo (Act I Sc 2 Line 317) Trinculo stumbling upon him describes him as ldquoA strange fish hellip Legged like a man And his fins like armsrdquo He ldquosmells like a fishrdquo (Act II Sc 2 Line 25)

o Prospero also calls him a ldquobeastrdquo (Act IV Sc 1 Line 140) and ldquoThis misshapen knaverdquo (Act V Sc 1 Line 268)

o Further it appears that in addition to his physical deformity his spiritual inferiority is also suggested by Prosperorsquos claim that his birth resulted from the union between his mother the witch Sycorax and the devil

Calibanrsquos ParentageWhen the play opens Caliban is twenty four years of age having been born on the island twelve years before the coming of Prospero His mother was the foul witch Sycorax who was banished from Algiers for ldquomischiefs manifold and sorceries terrible to enter human hearingrdquo (Act I Sc 2 Line 264) and the father was the Devil himself Thus

Caliban is a monster of evil and brute nature ugly deformed and stinking

Calibanrsquos Savage and Malignant Natureo Caliban is entirely a creature of the earth ndash gross brutal and savage He regards himself as the rightful possessor

of the island and Prospero as a usurper

o In his young age he was on good terms with Prospero He had consented to be received by Prospero at his house and to be educated by him He has learnt human language only to curse his master whom he abhors

o His beastly nature soon breaks out and ends in a vicious attack on Miranda This opens the eye of Prospero who becomes severe to him and enforces his service by threats and violence

o Prospero uses him to make dams for fish to fetch firewood scraper trenches wash dishes and keep his cell clean

Calibanrsquos Hatred for ProsperoA profound hatred for Prospero has taken hold of Caliban It springs from a sense of his being dispossessed and ill-treated He would kill Prospero if he could but he knows the power of Prosperorsquos lsquobookrsquo Hence he transfers his allegiance to Stephano who seems like a god to him He also incites the two drunken associates to batter the skull of Prospero when he sleeps in the afternoon

Caliban Shows Considerable Intelligenceo He has learnt Prosperorsquos language

ldquoYou taught me language and my profit onrsquot (Act II Sc 2 Lines 86-89)Is I know how to curserdquo

o He is well aware of the futility of arguing with one who has more power than he has

ldquoI must obey his art is such power (Act I Sc 2 Lines 373-376)It would control my damrsquos god SetebosAnd make a vassal of himrdquo

o He realizes the importance of Prosperorsquos books

ldquoRemember (Act III Sc 2 Lines 89-92)First to possess his books for without themHersquos but a sot as I am nor hath notOne spirit to commandrdquo

o He knows the value of stealth when attacking the enemy

ldquoPray you tread softly that the blind mole may not (Act IV Sc 1 Lines 194-195)Hear a foot fall we now are near his cellrdquo

o Caliban has a better set of values than Stephano and Trinculo They are distracted from their plan by their greed for Prosperorsquos rich garments Only Caliban realizes that such a finery is unimportant

ldquoLeave it alone thou fool it is but trashrdquo (Act IV Sc 1 Lines 224)

Caliban is not a good judge of characterCaliban is not a good judge of character He decides for example that Stephano is a god because he dispenses lsquocelestial liquorrsquo (Act II Sc 2 Line 115) but then it must be remembered that he has only known his mother Sycorax Prospero Miranda and the spirits that torture him However he quickly discovers his error of judgementrdquo

ldquoWhat a thrice-double ass (Act V Sc 1 Lines 295-297)Was I to take this drunkard for a godAnd worship this dull foolrdquo

Calibanrsquos Imaginative NatureIf Caliban is sub-human in what has been said above he is human in the respect of the poetic side of his character He listens to music with rapture He tells of the beautiful dreams in which heaven rains treasures upon him and which upon waking he yearns to renew One of the most poetic passages in whole play is Calibanrsquos description of the island

to Stephano and Trinculo

ldquoBe not afeard The isle is full of noises (Act III Sc 2 Lines 135-143)Sounds and sweet airs that give delight and hurt notSometimes a thousand twangling instrumentsWill hum about mine ears and sometime voicesThat if I then had waked after long sleepWill make me sleep again and then in dreamingThe clouds methought would open and show richesReady to drop upon me that when I wakedI cried to dream againrdquo

Caliban - Less Ignoble Than Some OthersCalibanrsquos motive for murder is less dishonourable than that of Antonio and Sebastian They plan to kill Alonso to gain his power and wealth Caliban merely wants revenge and the return of lsquohisrsquo island

Conclusiono Calibanrsquos character is not portrayed very clearly in the play and hence we cannot decide whether he is a poor

savage being grossly maltreated by Prospero or whether he is evil and must therefore be kept in bondage or enslavement

o Caliban is contrasted with Ariel who is a spirit and thus swift and uninterested in physical activitieso Caliban is also contrasted with Prospero who is the all-powerful master of the island and of the destiny of all

those on the islando Caliban is also contrasted with civilized man showing him to be less evil than Antonio and Stephano and less

materialistic than Stephano and Trinculoo Caliban has suffered at the hands of Prospero and he has learnt to curse by listening to Prosperorsquos abuse He

certainly believes that Prospero has deprived him of his birthrighto Finally the character Caliban is thought to be one of Shakespearersquos masterpieces The complexity of the character

is reflected in the large volume of critical discussion that has grown around it

ECO ndash12 Topic-Forms of market

MonopolyMonopoly is a market structure in which there is a single seller there are no close substitutes for the commodity produced by the firm and there are barriers to entry Example Indian Railways which is operated under government of India Monopoly also implies absence of competitionFeatures of Monopoly Monopoly is characterized by1 Single Seller In monopoly there is only one firm producing the product The whole industry consists of this single firm Thus under monopoly there is no distinction between firm and industry Being the only firm there is significant control of the firm over supply and price Thus under monopoly buyers do not have the option of buying the commodity from any other seller They have to buy the product from the firm or they can go without the commodity This fact gives immense control to the monopolist over the market

2No Close Substitute There are no close substitutes of the product produced by the monopolist firm If there are close substitutes of the product in the market it implies presence of more than one firm and hence no monopoly In order to ensure a total of control over the market by the monopolist firm it is assumed that there are no close substitutes of the product

3 No Entry amp Exit Monopoly can only exist when there is strong barriers before a new firm to enter the market In fact once a monopoly firm starts producing the product no other firm can produce the same One reason for this is the ability of the

monopolist to produce the product at a lower cost than any new firm who thinks to enter the market If a new firm who knows that it cannot produce at a lower cost than the monopolist then that firm will never enter the market for fear of losing out in competition Similarly the monopolist who is operating for a long time may be enjoying reputation among its customers and is in a better position to use the situation in its own benefit A new firm has to take long time to achieve this and so may not be interested to enter the market

4 Price Maker Being the single seller of the product the monopolist has full control over the pricing of the product On the other hand if there is a large number of buyers in the market so no single buyer exercises any significant influence over price determination Thus it is a sellerrsquos market So monopoly firm is a price maker

5 Price Discrimination Having considerable control over the market on account of being single seller with no entry of other firms the monopolist can exercise policy of price discrimination it means that the monopolist can sell different quantities of the same product to a consumer at different price or same quantity to different consumers at different prices by adjudging the standard of living of the consumer

6 Shape of Demand Curve Since a monopolist has full control over the price therefore he can sell more by lowering the price This makes the demand curve downward sloping

Subject Ac-12 290620 Topic- retirement Model sumThe Balance Sheet of Rohit Nisha and Sunil who are partners in a firm sharing profits according to their capitals as on 31st March 2014 was as under

Liabilities Amount Assets Amount (Rs) (` Rs)

Creditors 25000 Machinery 40000Bills Payable 13000 Building 90000General Reserve 22000 Debtors 30000Capital Less Provision for Rohit 60000 Bad debts 1000

29000 Nisha 40000 Stocks 23000 Sunil 40000 140000 Cash at Bank 18000

200000 200000

On the date of Balance Sheet Nisha retired from the firm and following adjustments were made(i) Building is appreciated by 20(ii) Provision for bad debts is increased to 5 on Debtors(iii) Machinery is depreciated by 10(iv) Goodwill of the firm is valued at Rs 56000 and the retiring partnerrsquos share is adjusted

(v) The capital of the new firm is fixed at Rs120000 Prepare Revaluation Account Capital Accounts of the partner and Balance Sheet of the new firm after Nisharsquos retirement Revaluation AccountDr Cr

Particulars Amount Particulars Amount (`Rs) (Rs`)

Provision for Bad debt Ac 500 Building Ac 18000Machinery Ac 4000Profit transferred toCapital Accounts (3 2 2)Rohit 5786Nisha 3857Sunil 3857

13500

18000 18000

Capital Account

Dr Cr

Particulars Rohit Nisha Sunil Particulars Rohit Nisha Sunil (Rs`) (Rs`) (`Rs) (Rs`) (Rs`) (Rs`)

Sunilrsquos Capital ac 9600 mdash 6400 Balance bd 60000 40000 40000Bank - 66143 - General Reserve 9428 6286 6286Balance cd 72000 mdash 48000 Revaluation (Profi 5786 3857 3857 Rohitrsquos Capital Ac mdash 9600 mdash

Sunilrsquos Capital Ac 6400 Bank 6386 - 4257

81600 66143 54400 81600 66143 54400

Balance Sheet as at 31st March 2014

Liabilities Amount Assets Amount (Rs`) (Rs`)

Creditors 25000 Building 108000Bank overdraft 37500 Machinery 36000

Bills Payable 13000 Debtors 30000Capital Less ProvisionRohit 72000 for Bad debts 1500 28500Sunil 48000 120000 Stock 23000

195500 195500

Working Notes (i) (a) Profit sharing ratio is 60000 40000 40000 ie = 3 2 2(b) Gaining Ratio Rohit = 35 ndash 37 = 2135 ndash 1535 = 635Sunil = 25-27 = 1435 ndash 1035 = 435= 635 435= 6 4 = 3 2(c) Nisha Share of Goodwill = Rs 56000 times 27 = Rs16000Share of Goodwill in the gaining ratio by the existing partner ieRohit = Rs16000 times 35 = Rs 9600Sunil = Rs 16000 times 25 = Rs 6400

The journal entry isRohitrsquos Capital Ac Dr 9600Sunilrsquos Capital Ac Dr 6400 To Nisharsquos Capital Ac 16000(Share of Goodwill divided into gaining ratio)

  • 1 Static Friction
  • The frictional force that acts between the surfaces when they are at rest with respect to each other is called Static Friction
    • Static Friction Examples
      • 2 Sliding Friction
        • Examples Of Sliding Friction
          • 3 Rolling Friction
            • Examples Of Rolling Friction
              • Objects and Reasons of the Forest Conservation Act
Page 4:  · Web viewSubject . Topic . Summary . Execution . English 1 . Sounds of animals . Hens –cackle Horses –neigh Lions –roar Owls –hoots Snake –hiss. English 2 . Mother’s

Hindi सवनाम अपन लिलए म हममरा हमारा मझ आदिद शबदो ा परयोग रत ह जि0सस बात र रह हउस लिलएmdashतम तमहारा तमह आप आपा आदिद शबदो ा परयोग रत ह जि0स बार म बात र रह ह या किसी अनय लिलएmdashवहव उस उना उनह उनहोनआदिद शबदो ा परयोग किया 0ाता ह ौन ोईकया आदिद शबद भी सवनाम ह

Read the chapter

Class IIISubject Topic Summary Execution COMPUTER

DRAWING IN MS PAINT

Q3) WHAT TOOL DO YOU USE TO INSERT TEXT IN MS PAINTANS) THE TEXT TOOL IS USED TO ADD TEXT IN MS PAINTQ4) WRITE THE STEPS TO USE THE ERASER TOOLANS) THE STEPS ARE

CLICK ON THE ERASER TOOL IN THE TOOLS GROUP DRAG THE MOUSE POINTER OVER THE AREA YOU WANT TO ERASE

বই ndash বাংলা সাহিতয পহিরচয়

পাঠ ndash ৮লপ ndash IনIহিন ও রাজার কাযলখক ndash উকেপনদরহিককেশার রায়কেচৌধরীঅনশীলনীর পরকে4াততর

৯ বাকয রচনা ককেরা -জবদ ndash সবাই হিমকেল যচারকেক পহিলকেশর াকেত তকেল হিকেয় জবদ করকেলা হিখলহিখল ndash নতন যখলনা যপকেয় হিশশটি হিখলহিখল ককের যকেস উঠকেলা কাতর ndash কাতর ককেS বদধ যলাকটি জল চাইহিল IফI ndash আঘাত লাায় ককরIা IফI করহিল -য়ানক ndash যকারনাকে -য়ানক পরলয় কেয়হিল

Hindi सवनाम अपन लिलए म हममरा हमारा मझ आदिद शबदो ा परयोग रत ह जि0सस बात र रह हउस लिलएmdashतम तमहारा तमह आप आपा आदिद शबदो ा परयोग रत ह mdashजि0स बार म बात र रह ह या किसी अनय लिलए वह व उस उना उनह उनहोन आदिद

शबदो ा परयोग किया 0ाता ह ौन ोई कया आदिद शबद भी सवनाम ह Science

Ch ndash Food we get from Plants

We see many plants around us They are of different shapes and sizes Some plants grow tall some stay short Some plants grow along the ground some plants need support to grow

Trees ndash big and tall plants are called trees Trees have thick hard and woody stem called trunk They have thin stems called branches Most trees live for many years A banyan tree can live for hundreds of years Mango banyan neem are examples of trees

Fill in the blanks (pg no- 63)

The main stem of a tree is called trunk

Cotton and hibiscus are examples of shrubs

Rice and mint are examples of herbs

Social studies

Indian literature Jataka talesThe jataka tales are short educational tales They have stories concerning the previous births of Gautam Buddha in both animal and human form These moral tales were written in fourth century and are based on the life of Gautama Buddha These are a collection of about 550 fables Every fable exhibits some virtue where

Buddha may appear in it as a king God or an elephant There are several translations of these Jataka tales in different languages

Read the portion which has been explainedEng language

Ch 19 Adverbs Adverbs are words that tells us more about an action In other words adding something more to the verb For eg The cat ran quicklyhere the word ran is the verb as it is showing an action done by the cat and quickly is the adverb as it is telling how the cat ran

Refer to pg 68Do in the bookWarm upUnderline the words that tell you how an action happened

The cat ran quicklyThe man ran swiftlyIt hurriedly went into its hole The mouse sat safely in its houseThe cat squeaked loudly

MAT

HEM

ATIC

S

Ch 7

Frac

tions

A fraction is a number that stands for parts of a whole object or a collection of objects

Each fraction has two numbers separated by a rule(a) The number above the rule is called numerator(b) The number below the rule is called denominator

Example 7minusminusminusminusrarr Numerator

13minusminusminusminusrarr Denominator

Notes

(1) 05 is a fraction in which numerator is 0

(2) There is no fraction whose denominator is zero

(3) When the numerator and denominator are equal the fraction represents the whole or 1

Example 33=1and2

2 = 1

Exercise ndash 421 Read the following fractional numbers

(c) 411 (d)

815 (e)

2337

Solution

(c) 411

four by eleven

(d)8

15Eight by fifteen

(e) 2337

twenty three by thirty seven

2 Write the following fractional numbers(d) Three over eleven (e) Eighteen by twenty one (f) Four-seventhsSolution

(d) Three over eleven 311

(e) Eighteen by twenty one 1821

(f) Four-sevenths 47

3 Write the numerator of each of the following

(e) 09 (f)

56 (g)

311 (h)

617

Solution (e) 0 (f) 5 (g) 3 (h) 64 Write the denominator of each of the following

(e) 08 (f)

813 (g)

719 (h)

417

Solution (e) 8 (f) 13 (g) 19 (h) 17

5 Write the fractions whose numerators and denominators are given below

(a) Numerator = 4 and Denominator = 13 (b) Numerator = 6 and Denominator = 17(c) Numerator = 11 and Denominator = 16(d) Numerator = 0 and Denominator = 9

Numerator Denominator

Fraction

(a) 4 13 413

(b) 6 17 617

(c) 11 16 1116

(d) 0 9 09

6 Find the fractions in which the denominator is 3 more than the numerator69

74

811

49

1113

711

1411

1613

2023

Solution 69

811

2023

8 A book seller had 15 books He sold 8 books What fraction of the books did he sell

Solution Out of 15 books the book seller sold 8 books

there4 The fraction of the books he sold =8

15

10 A class has 35 students On Monday 34 students were present What fraction of the students was absent

Solution Total number of student = 35On Monday 34 students were presentthere4 The number of student absent on Monday = (35 ndash 34) = 1

So fraction of the students was = 135

Class IVSubject Topic Summary Execution English language

Verbs the -ing form

Meaning of verb A word used to describe an action is known as verb

The ndash ing form of a verb is a very important form for it is used in many different waysExampleJack is playing(Present continuous)Jack was playing with his friends yesterday ( Past continuous)Jack will be playing in a match tomorrow ( Future continuous)So we see that the ndash ing form of verb used to form the continuous tenses

The ndash ing form of verb can also be used as an adjectiveExampleYou should not disturb a sleeping dogThe kettle is full of boiling water

Say which of the ndash ingforms of verbs in the sentences given below have been used as adjectives and which to form continuous tenses ( solved exercises please follow this)

3Interesting ndash adjective

4Was eating ndash past continuous tense

5terrifying ndashadjective

6are helping is spreading ndash present continuous tense

7will be visiting ndashfuture continuous tense

8twittering ndash adjective

Social studies

Map reading A Map helps us to see the whole World continents countries cities and neighborhood They are of different sizes handy and can be rolled up folded or carried easily However the maps do not show the surface accurately This is because the map is flat whereas the Earth is round in shape Cartography or a map making is a study and practice representation of the

One word answers1 It helps us to see the whole world countries and cities ndash Map

2 A study and practice representation of the Earth on a flat surface ndash Cartography

Earth on a flat surface

Elements of a mapTo be able to read a map we must understand the elements of a map

DirectionsDirections are the basic guidelines that help us to locate places It is very important for us to know the correct directions East West North and South are the four Cardinal directions These directions help us to locate the places Beside these there are four sub directionsndash Northeast Southeast North West and South West Compass is an instrument that helps to locate the directions

3 The basic guidelines to help us to locate places ndash directions

4These four directions help us to locate the places ndash Cardinal directions

COMPUTER

Formatting in ms word

Q3) What do you mean by formattingAns) Formatting is a general arrangement of text in a document We can change the appearance of a document by using the features available in ms word We can use different fonts colours and styles in the textQ4) What is alignmentAns) Alignment of text is the way in which it is placed between the margins of a page Text can be aligned to the left side in the centre or to the right side of a pageQ5) what is fontAns) A font is a style of writing and typing A font provides specific textual appearance to the document You can change the size style or give various effects to a font

Hindi 2ndlang

ाला किहरण सभी धमlt ा मल धम ह दया और रणा मरी हर 0ीव अपन ही दकिनया म रहत ह हम

पराणी कयो उनो परशान रत ह ऐसा हम नही रना चाकिहए

0ानवरो स हम ए सीख मिमलती ह किस तरह हम एता म रहना चाकिहए

हर पराणी पयार ी भाा समझता ह अगर हम पयार ी उममीद रत ह तो कया 0त

0ानवर हमस उममीद नही रत कि हम भी उनह पयार द यह हानी ए ाल किहरण ी ह 0ो अपन

समह ा नततव रता था हमशा कया उलिचत ह यह सोचता था अचान

उन पर लिशारिरयो न हमला बोल दिदया जि0सम ाला किहरण परा गया कयोकि वह

दलभ ह और लिशारिरयो ी न0र उसी पर थी परत 0ानवरो ी एता ो दखर

रा0मार ो दया आ गई और उसन उनो छोड दिदया 0ानवर भी बईमान नही थी वह उनस अकसर मिमलन उन बाद म 0ाता रहता रा0ा ो धनयवाद रता अबोध म

पराणिणयो म एता तजञता अभी भी ह

शबदाथ तराई ndash पहाड आसपास नीच

ी भमिमवश- ल या परिरवारअदवभत- अनोखाचौननाndash सत सावधानओझलndash गायबकिवपणिRndash मसीबतसगठन- एताउममीदndash आशाातर दमिU ndashकिववशता

हिहस ndash हानी पहचान वालातवयndash फ0

दल ndash समहनततवndash सचालन रना

ओट ndash आडआतमसमपणndash अपन ो सौप दनाम- चप

हम उनस सीख लनी चाकिहएবইndashবাংলা সাহিতয পহিরচয়

পাঠndash১০লপndashহিবকেবকানকেFর যকেলকেবলাযলখকndashশশী-ষণ াশগNঅনশীলনীর পরকে4াততর

৯প পহিরবতG ন ককেরা -শরীর = শারীহিরক -ত = য-ৌহিতক সG ার = সG াহির সথায়ী = সথাহিয়তবহিবশবাস = হিবশবাসী া = যকো১০ বাকয রচনা ককেরা -আডডা = পাার বদধরা াতলায় আডডা যয়হিডবাহিজ = রাম া যকেক হিডবাহিজ যখকেয় পকেলাসG ার = বকেলহিল যকেল যর সG ারবকহিন = অঙক -ল করায় রীনা মাকেয়র কাকে বকহিন যখকেলাবহিদধ = বহিদধ াককেল উপায় য়হিনশহিত = হিনশহিত রাকেত রাজবাহিকেত ডাকাত পকেলা-য়ানক = পাাহি রাসতা বষটির পকের -য়ানক হিবপ জনক য়হিবশবাস = মানকেষর পরহিত হিবশবাস ারাকেনা পাপ

Science Ch ndash The Food We Eat

Living things need food to live to grow to stay strong and fit When we need food we feel hungry Food gives us energy to do work It also protects us from diseases and helps us to stay healthy Nutrients in food The food we eat contains many substances that are necessary for our body These substances are called nutrients that help us to grow well and stay healthy

Nutrients give us energy to study work and play

They help our body to grow and repair the damaged parts of our body

They also help our body to fight against diseases and remain healthy

Write T for True or F for False (pg no- 11)

1 Food contains nutrients that help us to grow True

2 Foods rich in carbohydrates are called body-building foods False

MAT

HEM

ATIC

S

Ch 9

Com

mon

Fra

ction

s

A fraction is a number that stands for parts of a whole object or a collection of objects

Each fraction has two numbers One is written above the other separated by a line The one above the line is called numerator and the one below the line is called denominator

Example 5minusminusminusminusrarr Numerator

11minusminusminusminusrarr Denominator

Exercise ndash 362 In following fill in the blanks

(b) 37 ___ is denominator ___ is

numerator

(c) 89 ___ is denominator ___ is

numerator

Solution

(b) 37 7 is denominator 3 is numerator

(c) 89 9 is denominator 8 is numerator

3 Write down the fractions whose numerators and denominators are given below in the bracket The first number stands for numerator and the second number standing for denominator

(25) 25

(311) 311

(416) 416

(712) 712

Class VSubject Topic Summary Execution

Science Ch ndash PlantReproduction

In pollination chapter we have learnt that the flowers change into fruits and the fruits bear seeds Now we are going to learn how this process takes place

FertilizationThe process of fusion of the male reproductive cell (male gamete) and female reproductive cell (female gamete) is known as fertilizationWhen a pollen grain reaches from the anther to a stigma it begins to grow and forms apollentube The Pollen tube then travels down through the style to enter an ovule inside the ovary On reaching the ovule male reproductive cell in the pollen grain unites with the egg cell present in the ovule

Books exercise

A) Tick the correct answer

1Which of the following do ovules change into after fertilization ndashseeds

2Which of the following is not a part of the seed ndash flower

3Which of the following condition is needed for germination of a seed ndash all ofthese

English language

Sentences phrases and

Solved exercisesSay which of the underlined groups of words are phrases and which clauses

clauses8 In her new clothes ndashphrasesAs pretty as a doll ndash phrases

9 looking sad and upset ndash phrasesHe had lost all the tickets for the Test Match ndash clauses

10 During the vacation now only a month away ndashphrases

11 too sweet and too hot ndashphrases

12 At the Olympic Games ndashPhrasesOf Laurel leaves ndash phrases

13 Who played the role of Hamlet ndashclauses

14 However fast ndash phrases

15 When the men fell asleep ndash clausesSocial studies

Indian Government

Lok Sabha (lower house) ndash It has 552 members Of these 530 membersrepresent States 20 members represent the union territories and two members represent the Anglo- Indian community All except the representatives of the Anglo-Indian community are elected by Indian citizens A person above the age of 25 can contest in the elections for Lok Sabha One term of Lok Sabha is for 5 yearsRajya Sabha (upper house) ndash Its members are elected by the MLAs or members of the legislative Assembly There are 250 members in the Rajya Sabha of which 12 are nominated by the President One term of Rajya Sabha is for 6 years Anyone above the age of 30 can be elected as a member of Rajya Sabha

ExecutivePresidentThe President is the head of the country in India He is elected by the MPs and the MLAs for a tenure of 5 years He appoints the Prime Minister and the Council of Ministers Prime MinisterThe party which wins the election forms the government and its leader is elected as the Prime Minister He is the chief advisor to the President The Council of Ministers assists the Prime Minister and is accountable for their roles For example the Education minister is responsible for the education system in our country

1 How many members are there in Lok SabhaAns 552 members

2 What is the term for Lok SabhaAns 5 years

3 How many members are there in the Rajya SabhaAns 250 members

4 Who is the head of our countryAns President

5 Who is the chief advisor to the PresidentAns Prime Minister

Book ndash GK

Ch ndash 1First in space

1 First living being into space in 1957 Ans Laika

Times 2 First person to go into space in 1961 Ans Yuri Gagarin

3 First woman to go into space in 1963 Ans ValentinaTereshkova4 First person ever to walk in space in 1965Ans alexei Leonov5 First person to land on the moon in 1959 Ans Neil Armstrong6 First Indian to go into space in 1984 Ans Rakesh Sharma 7 First Indian woman to go into space in 19978 Ans Kalpana Chawla9 First woman tourist in space in 2006

Ans Anusheh AnsariCOMPUTER

ALGORITHM AND FLOWCHART

Q) DRAW THE SYMBOLS USED IN A FLOWCHART WITH THEIR DESCRIPTIONS(IN EXAM IT CAN COME AS SHORT QUESTIONS ASKING INDIVIDUAL SYMBOLS FUNCTION)ANS)

MAT

HEM

ATIC

S

Ch 6

Com

mon

Fra

ction

s

Multiplication of FractionsA Multiply a fractional number by whole numberTo multiply a fractional number by whole number we multiply the numerator of the fractional number by the whole number and denominator of the fractional number by 1 The first product thus obtained is the numerator and the second product is the denominator of the required product

Exercise ndash 30Multiply

7 2027

times 9

Solution 2027

times 9 = 203 = 6

23

8 611

times11

Solution 611

times11 = 6

15 71

20times16

Solution 71

20times16 =

14120

times16

= 1415

times 4 = 141times 4

5 = 564

5 = 11245

B Multiplication of a fractional number by a fractional number To multiply a fractional number by a fractional number we multiply the numerator of the first fractional number by the numerator of the second fractional number and the denominator of the first fractional number by the denominator of the second fractional number The first product thus obtained is the numerator and the second product is the denominator of the required product

16 2712

times24

Solution 2712

times24 = 3112

times24

= 31times2 = 62

Exercise ndash 31

11 83

times 34

2

Solution 83

times 34 = 2

14 723

times2 25

4

Solution 723

times2 25 =

233

times 125 =

23times 45

= 925 = 18

25

15 1212

times1 13

2

Solution 1212

times1 13 =

252

times 43 =

25times 23

= 503 = 16

23

State the following statements are true or false

17 1912

times 239 = 1

Solution LHS = 1912

times 239

= 392

times 239 = 1 = RHS

[LHS = Left hand side amp RHS = Right hand side]

there4 1912

times 239 = 1 [True]

21 213

times2 13 = 4

19

Solution LHS = 213

times2 13 =

73

times 73

= 7times73times3 =

499 = 5

49

there4 LHS ne RHS

So 213

times2 13 = 4

19 [False]

23 23

times 45 =

2times 5+3 times43times 5

Solution

LHS = 23

times 45 =

2times 43 times5 =

815 again

RHS = 2times 5+3 times4

3times 5 = 10+12

15 = 2215

there4 LHS ne RHS So 23

times 45 =

2times 5+3times43times 5

[False]

25 23 of

13 =

29

Solution

LHS= 23 of

13 =

23 times

13 =

29 = RHS]

there4 23 of

13 =

29 [True]

Practice at HomeExercise ndash 31State the following statements are true or false

24 12 of 4 =

18

Class VISubject Topic Summary Execution

HISTORY AND CIVICS

Chapter 5The Mauryan Empire

DECLINE OF MAURYAN EMPIREDecline of Mauryan empire started after the death of Ashoka at around 232 BCThere are several reasons for break up of the empire1 Weak successor Emperors after Ashoka were

capable of handling vast and mighty Mauryan empire In 185BC the last Mauryan ruler Bri-hadrath was murdered by his Commander-in-Chief Pushyamitra Sunga

2 Provincial Revolts Due to weak central author-ity provincial chiefs of Kalinga and southern provinces revolted against emperor and freed themselves from Mauryan empire

3 Weakness of Economy Prosperity of Mauryan was based on solid economic activities which

ExercisesI Multiple choice questions-1 Chandragupta defeated Seleucus in the year ndashc) 305 BC2 Who killed the last Mauryan ruler Brihadrath b) Pushyamitra3 Which of the following was not a reason for the decline of the Mauryan empirec) Chandraguptarsquos weakness4 Ashoka invaded Kalinga in the year c) 261 BC

II Fill in the blanks1Chandragupta ascended the throne in 324

was taken care by early monarchs Later kings had neither ability nor interest in economic af-fairs That led to failure in tax collection As a result they failed to maintain a large army that were essential to keep empire intact

4 Greek Invasion Greeks freed north-western provinces from weak Mauryan monarchs and reestablished their authority

5 Ashokarsquos Policy some scholar opined that after Kalinga war Ashoka embraced Buddhism re-nounced the policy of war and disbanded the Army But this is partially true as there is no proper evidence of disbanding the army

Based on above points we can conclude that main reason for decline of Mauryan empire is weakness of Ashokarsquos successors Kunal Samprati Dasharath Salisuk all were weak kingsAt last in 185 BCPushyamitra Sunga killed king Brihadrath and established the Sunga dynasty

BC2 Bindusara was the son of Chandragupta and father of Ashoka

3 Pataliputra was administered by City Magistrate committess of 5 members each4 The Greek General Seleucus sent his ambassador Megasthenes to Chandraguptarsquos court5 Ashoka sent his son Prince Mahendra and daughter Sanghamitra to spread his Dhamma6 The Indian Rebublic has adopted the Lion Capital of Saranath Pillar as its national emblem 7 Pushyamitra killed the last Mauryan ruler Brihadrath and founded the Sunga dynasty

III Name the following

1The author of Arthashastra-Kautilya2 The ruler who founded the Mauryan dynasty-Chandragupta3 The author of Indika-Megasthenes 4 The officers who were appointed by Ashoka to spread Dhamma-Dhamma Mahamatras5 The general of Alexander whom Chandragupta defeated-Seleucus

V Match the columns1 Kautilya (c)2 Megasthenes (d)3 Pushyamitra (e)4 Brihadrath (b)5 Bindusara (a)

BENGALI(2ND

LANGUAGE)

পশপাহিখর -াষাসহিবনয় রায়কেচৌধরী

যলখক পহিরহিচহিত- পরখযাত সাহিহিতযক উকেপনদরহিককেশার রায়কেচৌধরীর পতর সহিবনয় রায়কেচৌধরী lsquoসকেFশrsquo পহিতরকার সকেb হিতহিন কত হিকেলন তার উকেdখকোয বই lsquoসহিবনয় রায়কেচৌধরীর রচনা সংগরrsquo

পরম হিকেনর পাঠ- lsquoপশপাহিখর হিক -াষাhelliphellip helliphelliphelliphellipপরসপরকেক জানাবার উপায়ও পশপাহিখরা যবশ জাকেনrsquoপরকেমই আমারা জাহিন -াষা হিক -াষা ল আমাকের মকেনর -াব পরকাশ করার জনয আমরা নানান ধরকেনর -হিb বা হিবকেশষ ধরকেনর আওয়াজ মকেখর মাধযকেম কহির অনযকেক যবাঝাকেনার জনয তাকেল এবার আমরা জাহিন পশপাহিখর -াষা হিক পশ পাহিখরা হিক কা বকেল যা পশপাহিখকেরও -াষা আকে তারা তাকের হিনজসব -াষায় কা বকেল মকেনর -াব পরকাশ ককের পশ পাহিখরা মানকেষর হিক হিক -াষা যবাকেঝ হিকনত তারা বলকেত পাকেরনা পরসপরকেক বহিঝকেয় যবার উপায় তারা জাকেননা তকেব তারা হিবকেশষ ককেয়কটি শকেবদর মাধযকেম তাকের মকেনর -াব বহিঝকেয় যয় হিক বহিদধ মান জীব ndashককর হিবাল বন মানষ যঘাা পর-হিত এরা মানকেষর যওয়া নাম শনকেল কান খাা ককের ndash নাম ধকের ডাককেল কাকে আকেস যমন - মরহিরা lsquoহিত ndashহিতrsquo ডাক শকেন আকেস াল lsquoঅ ndashর -র ডাক শকেন কাকে আকেস াহিত মাহকেতর কা শকেন চকেল ককররা মাহিলকেকর হকম পালন ককের সবসময় তাইকেতা ককরকেক পর- -কত পরানী বলা য় ককর আর হিবাল একের আওয়াজ তহিম লকষয করকেল বঝকেব ককররা যরকে যকেল lsquoযঘউ যঘউrsquo করকেত াকেক আবার কাকেল lsquoযকউ যকউrsquo ককের হিবাল সাধারণ lsquoমযাওrsquo বা lsquoহিমউrsquo ককের রা কেল lsquoওয়াওrsquo আওয়াকেজর মাধযকেম মকেনর -াব পরকাশ ককের একেতা যল পশকের কা পাহিখরাও -য় রা পরকাশ করার জনয হিবকেশষ ধরকেনর শবদ ককের হিবপকের সময় পশ পাহিখরা সবার আকে পরসপরকেক জানাবার উপায় তারা জাকেন বহকাল

১) পশপাহিখর -াষা কেলপর যলখক সমপকেকG হিক জাকেনা

উঃ- পরখযাত সাহিহিতযক উকেপনদরহিককেশার রায়কেচৌধরীর পতর সহিবনয় রায়কেচৌধরী lsquoসকেFশrsquo পহিতরকার সকেb হিতহিন কত হিকেলন তার উকেdখকোয বই lsquoসহিবনয় রায়কেচৌধরীর রচনা সংগরrsquo হিতহিন ারকেমাহিনয়াম এসরাজ পর-হিত বাযনতর বাজাকেত পারকেতন ানও জানকেতন হিতহিন যাকেIাকের জনয মজাার লপ কহিবতা হিলখকেতন

২) পশপাহিখ কেলপর মল-াব হিকউঃ- পশপাহিখকেরও -াষা আকে তারা তাকের হিনজসব -াষায় কা বকেল মকেনর -াব পরকাশ ককের পশ পাহিখরা মানকেষর হিক হিক -াষা যবাকেঝ হিকনত তারা বলকেত পাকেরনা পরসপরকেক বহিঝকেয় যবার উপায় তারা জাকেননা তকেব তারা হিবকেশষ ককেয়কটি শকেবদর মাধযকেম তাকের মকেনর -াব বহিঝকেয় যয় হিরউকেবন কযাসটং সাকেব হিতহিন চহিdশ বর বনযজনত যর সকেb যকেককেন হিতহিন বকেলকেন আমরা হি তাকের -াষা তাকের আব কায়া যমকেন চহিল তাকেল আর -কেয়র যকান কারণ াকেকনা আমরা একI -াকেলাকেবকেস যচষটা করকেল পশপাহিখকের সকেb -াব পাতাকেত পাহির

ধকের মানষ এই পশ পাহিখর -াষা হিনকেয় নানা রককেমর পরীকষা ককের আসকে এইরকম একজন হিরউকেবন কযাসটাং সাকেকেবর কা আমরা জানকেবাhelliphellip

Hindi 2nd

langमतर किनमनलिलखिखतपरशनोउRरदीजि0ए

) बढ वयलि` बचच ो कया हआ था ख) डॉकटर साहबन पाटc किस उददशय स रखी थी ग) ाल साप ो हाथ म लर लाश न कया किया घ) डॉकटर चडढा न बढ पतरो दखन स कयो मना र दिदया था ङ) भगत न लाश ो दखर कया हा

उRर ndash) उस बहत बखार थी और 4 दिदनो स आख भी नही खोला थाख) उन बट ी सालकिगरह थीग) ाल सापो हाथ म लर लाश न उसी गदन 0ोर स दबार पडी थीघ) डॉकटर चडढा न बढ वयलि` पतरो दखन स मना र दिदया कयोकि उनह गोलफ खलन 0ाना थाङ) लाश ो दखर हा कि नारायण चाहग तो आध घट म भया उठ 0ाएग

English literature

In the bazaars of Hyderabad- Sarojini Naidu

Through the poem In The Bazaars of Hyderabad Sarojini wanted to convey the message that India is rich in tradition and they donrsquot need the foreign products So she goes on to give a picture of a bazaar where traditional Indian products are rulingThe poem is in the form of questions and answers The poet asks the questions and the merchants answer them Through this technique she make the picture of the bazaar visible to us

Read the poem

PHYSICS FORCE Types of FrictionThere are three types of friction static sliding rolling Static sliding and rolling friction occur between solid surfaces

1 Static Friction The frictional force that acts between the surfaces when they are at rest with respect to each other is called Static FrictionStatic Friction Examples

Skiing against the snow Creating heat by rubbing both the hands

together Table lamp resting on the table

2 Sliding Friction The resistance that is created between any two objects when they are sliding against each other is called Sliding FrictionExamples Of Sliding Friction

Sliding of the block across the floor Two cards sliding against each other in a

deck

3 Rolling Friction The force which resists the motion of a ball or wheel is called Rolling Friction Is the weakest types of frictionExamples Of Rolling Friction

Rolling of the log on the ground Wheels of the moving vehicles

6What effect can a force produce on a body which is not allowed to move Ans - When a force is applied on a body which is not free to move it gets deformed i e the shape or size of the body changes7Give one example each to indicate that the application of a force

1 produces motion2 stops motion3 slows down motion4 changes the direction of motion5 deforms a body

Ans- 1 A car originally at rest when pushed

begins to move2 A moving bicycle is stopped by

applying the brakes3 The speed of a moving vehicle is

slowed down by applying the brakes4 A player kicks a moving football to

change its direction of motion5 On stretching a rubber string its

length increases

8State the effect produced by a force in the following cases (a) The sling of a rubber catapult is stretched(b) A man pushes a heavy cart(c) A player uses his stick to deflect the ball (d) A cyclist applies brakes(e) A spring is compressedAns- (a) The shape and size of catapult changes ie its length increases(b) The heavy cart begins to move(c) The direction of the ball changes(d) The speed of the moving cycle is slowed down(e) There is change in size and shape of spring

COMPUTER MS EXCEL 2013 -INTRODUCTION

UNDERSTANDING EXCEL STRUCTUREA SPREADSHEET IS A FILE THAT EXISTS OF CELLS IN ROWS AND COLUMNS AND CAN HELP ARRANGE CALCULATE AND SORT DATA DATA IN A SPREADSHEET CAN BE NUMERIC VALUES AS WELL AS TEXT

FORMULAS REFERENCES AND FUNCTIONS

WORKSHEETA WORKSHEET IS ALSO KNOWN AS SPREADSHEETIT IS A COLLECTION OF CELLS ON A SINGLE SHEET WHERE YOU KEEP AND CHANGE DATA

WORKBOOKWORKBOOK IS PMS EXCEL FILE IN WHICH THE DATA CAN BE STORED EACH WORKBOOK CAN CONTAIN MANY WORKSHEETS

ROWS AND COLUMNSIN MS EXCEL A ROW IS A GROUP OF CELLS THAT RUN FROM LEFT TO RIGHT OF A PAGEA COLUMN IS A GROUPING OF CELLS THAT RUN FROM THE TOP TO THE BOTTOM OF A PAGE

CELLTHE INTERSECTION POINT BETWEEN A ROW AND THE COLUMN IS CALLED A CELL WHICH IS THE BASIC STORAGE UNIT FOR DATA IN A SPREADSHEET EACH CELL HAS SPECIFIC ADDRESS WHICH IS THE COMBINATION OF THE COLUMN NAME FOLLOWED BY THE ROW NUMBER

CHEMISTRY Chapter ndash Common Laboratory Apparatus and equipments

Objective type questionFill in the blanks (a) Experiment and observation are the two important basics of chemistry(b) A porcelain dish is used for evaporation(c) A test tube holder is used to hold the test tube while-it is heated(d) Mortar and pestle is used for grinding and crushing solid substances into a powder(e) Glass apparatus is made of Pyrex or borosil glass

Class VIISubject Topic Summary Execution

Hindi 2ndlang

ए था राम( डॉ शरी परसाद)

सगकित ा परभाव मानव 0ीवन पर अवशय पडता ह

हमशा मनषय ो अचछो ी सगकित म रहना चाकिहए

शरषठ परो सग स मनषय चरिरतर ा शीघर ही उदय और किवास हो 0ाता

ह इसलिलए वयलि` ो सदा शरषठ परो ा ही सग रना चाकिहए

इसान अगर चाह वह सवय ो बदल भी सता ह

यह हानी राम ए बचच ी हवह गणिणत ी परीकषा म नल रत हए पडा 0ाता ह और उस अधयाप पडत ह और पछत ह यह कया र रह

हो तभी राम न उनी बइजजती ी

शबदाथब ndashहावा भलावाायवाहीndash ाम किनयम व ानन

ो दिदखानापरिरलिचतndash 0ाना पहचानाघटनाndashघबराहट

उलटा चोर ोतवाल ो डाटndashकिववndash भल बर ा जञानतवयndash म 0ो रना चाकिहएसगकितndash बरी सगत

किबलख नाndashरोना किनशचय रनाndash तय रना

फलndashपरिरणामकिनषालिसतndash बाहर किया हआपशचातापndashदख सपननndashधनी

ldquo हा आपी किहममत स हई नल रत पडन ीrdquo ऐसी बात ही किफर

किपता0ी न भी उस डाटा वह ाफी पशचाताप रन लगा बोला गलत दोसतो

ी सगकित म आ0 कितना अनथ र दिदया किफर उसन अधयाप स माफी मागन ी सची और किफर भी ऐसा

नही रगा यह परण भी लिलया

सोचndashकिहच एात-अला

বইndashবাংলা সাহিতয পহিরচয়

পাঠndash১৬লপndashস-য ও অস-যযলখকndashঈশবরচনদর হিবযাসারঅনশীলনীর পরকে4াততর

৬ অGকেলকেখা -ময়া = পশ হিশকার সহিtহিত = হিনকIবতu সbভরষট = লI হিনরীকষণ =

-াকেলা-াকেব যখাকতাঞজহিলপকেI = যজাাকেত৭ হিবপরীতশবদ -ঈষৎ times পরচর উৎকষট times হিনকষট তাশ times উৎফd তবহিদধ times

বহিদধীNপাহিপষঠ times পণযবান৮ পপহিরবতG ন ককেরা -পশ = পাশহিবক যকাপ = যকাহিপতহিসথর = হিসথরতাএকানত = ঐকাহিনতক পর-াত times পর-াতী

CHEMISTRY

Chapter ndashPhysical and Chemical Changes

Chemical ChangeA chemical change involves a change in chemical composition

Characteristics of Chemical changes 1 They are permanent changes2 They are irreversible changes 3 New substance formed4 A Chemical change involves a

change in its chemical properties

Pg-25Question 8What do you observe when1 water is boiled2 a piece of paper is burnt3 some ice cubes are kept in a glass tumbler4 solid ammonium chloride is heated5 an iron nail is kept in tap water for few days6 a spoon of sugar is heated in a pan7 lighted match stick is brought near the mouth of the test tube containing hydrogen gas8 quick lime is dissolved in water9 little amount of curd is added to a bowl containing warm milk and kept for five hours

10 Water is boiledOn boiling water changes into steam (gas) physical change

11 A piece of paper is burnton burning piece of paper produces carbon dioxide and ash is left behind Is a chemical change

12 some ice cubes are kept in a glass tumblerIce cubes (solid) turn into water

(liquid) only state changes (physical change)

13 Solid ammonium chloride is heatedSolid ammonium chloride on heating changes into vapors (change of state) is physical change

14 An iron nail is kept in tap water for few dayswe observe reddish brown coating on the nail called rust (entirely new substance) is chemical change

15 A spoon of sugar is heated in a panWhen a spoon of sugar is heated in a pan black (charred sugar) (carbon) is seen Is a chemical change

16 Lighted match stick is brought near the mouth of the test tube containing hydrogen gasWe observe that hydrogen bums at the mouth of test tube with blue flame and pop sound is heard It is chemical change

17 Quick lime is dissolved in waterThe following two observations will be observed (i) A hissing sound is observed(ii) The mixture starts boiling and lime water is obtained

18 Little amount of curd is added to a bowl containing warm milk and kept for five hoursWhen a little amount curd is added to a bowl containing warm milk and kept for five hours a permanent change occurredThe milk will change to curd On boiling water changes into steam (gas) physical change

GEOGRAPHY

ATMOSPHERE IMPACT OF GLOBAL WARMING The destructive impart of global warming is observed in various spheres of life and the environment Some of the points are outlined below1 High temperatures lead to high

evaporation rate and drying up of the soil and surface water This affects crop production The occurrence of droughts is aggravating the problem even further

2 The heat waves in summer months

Q1 Write some impact of global warmingA1 The impacts of global warming are as follows1 High temperatures lead to high

evaporate ion rate and drying up of the soil and surface water This affects crop production The occurrence of droughts is aggravating the problem even further

2 The heat waves in summer months lead to a greater number

lead to a greater number of deaths due to heat strokes

3 Forest fires become more frequent4 Tropical cyclones and hurricanes

become common5 Melting of glaciers takes place6 Polar ice caps are becoming thinner

and melting at an alarming rate due to global warming The loss of sea ice

7 Due to increase in sea surface temperature sea levels rise in coastal areas and cause submergence of several islands

WAYS TO REDUCE GLOBAL WARMINGFollowing steps can be taken We need to decrease emission of

green house gases by reducing the burning of fossil fuel such as coal and petroleum

By planting more trees to increase forest cover

The government should also distributes free saplings and organize afforestation programmes to spread awareness regarding the beneficial effects of trees

We should switch to eco-friendly cars and gadgets

Incandescent light bulbs should be replaced by CFL bulbs

We can save electricity and reduce global warming by turning off electrical gadgets such as lights fans air-conditioners television and computer when we do not to use them

Efforts should be made to hasten the development of green cities oreco cities These cities are urban areas around the world striving to lessen the environment a impacts of urbanization

By following the 3Rs-Reduce Recycle and Reuse strategy we can use natural resources for our growth as well as save them for the need of the future generations This is called sustainable development

of deaths due to heat strokes3 Forest fires become more

frequent4 Tropical cyclones and hurricanes

become common5 Melting of glaciers takes place

etc

Q2 How to reduce global warmingA2 Following steps can be taken to reduce global warmingaWe need to decrease emission of

green house gases by reducing the burning of fossil fuel such as coal and petroleum

bBy planting more trees to increase forest cover

c The government should also distributes free saplings and organize afforestation programmes to spread awareness regarding the beneficial effects of trees

dWe should witch to eco-friendly cars and gadgets

eIncandescent light bulbs should be replaced by CFL bulbs

f We can save electricity and reduce global warming by turning off electrical gadgets such as lights fans air-conditioners television and computer when we do not to use them

Q3 What do you mean by 3Rrsquos of resource planningA3 The 3Rs are

1 Reduce 2 Recycle and3 Reuse

Q4 What is Sustainable developmentA4 By following the 3Rs-Reluce Recycle and Reuse strategy we can use natural resources for our growth as well as save them for the need of the future generations This is called sustainable development

English Language

Prepositions A preposition is a word placed before a noun or a pronoun It helps to show how the person or thing denoted by the noun is related to something else in the sentence

Kinds of Prepositions

Simple Prepositions- simple preposition are one word Prepositions such as at by for in of off for from on out through till to up with before amidst towards beyond between over etc

Compound Prepositions ndash There are some words that are always used with fixed Prepositions to convey specific meaning

Example I was unable to meet you dueto a previous engagement ( On account of)Always maintain the queue instead of crowding at the counter ( In place of)

Participial PrepositionsmdashParticiple Prepositions are present or past participles of various verbs which together with a noun phrase or a clause function as prepositions Examples- barring concerning considering notwithstanding pending regarding respecting etc

Exercise A

1 Gauravs fever has come down since Friday He has been absent for a week now

2 The child sat between his father and mother among the parents of all his classmates

3 There are mosquitoes in the room They flew into the room when the door was open

4 My father was inside the drawing room when I was playing outside my house

5 You may sit beside me I will give you a drawing book and pencils besides a storybook

6 We went to the market in the morning and walked towards the riverfront in the evening

7 The child walked along the pavement and across the street safely

8 This table top is made of glass My breakfast fell off it in the morning

9 The pan is on the gas stove There are vegetables in it

10 We will wait for you at the bus top There are a lot of people in the hall

Subject ndash Biology Topic ndash Chapter - 3 Photosynthesis and respiration in plants Summary Execution

All living organism (Plants and animals) need food for energy and growth Green plants (autotrophy) prepare food for all living organisms Today we will discuss about the process photosynthesis And adaptations in a leaf to carry out photosynthesis

Q1What do you mean by photosynthesis and write its word equation The process by which green plants make food (glucose) from carbon dioxide and water

in the presence of sunlight and chlorophyll is called photosynthesis

Carbon dioxide + Water ( Sun light from Sun ) Glucose + Oxygen ( chlorophyll in green leaves )

Q2 What are the adaptations in a leaf to carry out photosynthesisi) Leaves are broad wide and flat for absorbing more light energyii) Presence of chlorophyll in chloroplasts to trap sunlightiii) Presence of stomata which allow carbon dioxide to enter the cell and oxygen to go

out iv) Network of veins ensures continuous supply of water and minerals to the leafv) Thin waxy cuticle protects the leaf without blocking the lightQ3 Draw and label structure of chloroplast

Class VIIISubject Topic Summary Execution

PHYSICS ENERGY Production of Hydro electricity

A hydroelectric dam converts the potential energy stored in a water reservoir behind a dam to mechanical energymdashmechanical energy is also known as kinetic energy As the water flows down through the dam its kinetic energy is used to turn a turbine

The generator converts the turbinersquos mechanical energy into electricity

This electric energy then goes through various transmission processes before it reaches you

Question 2

Fill in the blanks

(a) Work is said to be done by a forte only when the body moves

(b) Work done = Force x distance moved in direction of force

(c) The energy of a body is its capacity to do work

(d) The SI unit of energy is joule

(e) The potential energy is due to its state rest of position and kinetic energy of the body is due to its state of motion

(f) Gravitational potential energy U = mass times force of gravity on unit mass times height

(g) Kinetic energy = frac12 times mass times (speed)2

(h) Power P = work donetime taken

(i) The S I unit of power is watt

(j) IHP = 746 W

BIOLOGY Chapter -5 The endocrine system and adolescence

Today we will discuss about thelocation and functions of secreted hormones of adrenal and Pancreas

Q5 Write location hormone secreted main functions and deficiency diseases of pancreas and adrenal glands

Endocrine Glands

Location Hormones secreted

Functions and Deficiency Diseases

1Adrenal gland

2 Pancreas Gland

On the top of each kidney

In between stomach and small intestine

i)Adrenaline from adrenal medulla

ii)Cortisone from adrenal cortex

i) Insulin

ii) Glucagon

It helps a person deal with any kind of emergency situation or emotional stressIt increases the heart beat rate of respiration and blood pressure

a) It regulates carbohydrates protein and fat metabolism

b) It regulates the salt and water balance in the body

a) It changes excess glucose into glycogen

b) It stimulates the cells to burn extra glucose to provide heat amp energy

Less secretion causes diabetes mellitus

Excessive secretions causeinsulin shock

a) It stimulates the breakdown of glycogen into glucose

b) It increases the level of glucose in blood

History Traders to rulers The Battle of Buxar was fought on 22 October 1764 between the forces under the command of the British East India Company led by Hector Munro and the combined armies of Mir Qasim the Nawab of Bengal till 1763 Mir Jafar was made the Nawab of Bengal for a second time in 1763 by the Company just after the battle After being defeated in 4 battles in katwa and Udaynala the Nawab of Awadh Siraj id Daula and the Mughal emperor Shah Alam II accompanied by Raja Balwant Singh of Kashi made an alliance with Mir Qasim The battle was fought at Buxar a small fortified

Answer the following questions- Short note-Battle of BuxarHomework-learn

town within the territory of Bihar located on the banks of the Ganga river about 130 kilometres (81 mi) west of Patna it was a decisive victory for the British East India Company The war was brought to an end by the Treaty of Allahabad in 1765

EnglishLiterature

The west wind-John Mansfield

In the poem The West Wind by John Masefield the poet starts by describingwith very poetic imagery of birds how the west wind is different from other winds its a warm wind full of birds cries There is a touch of melancholy perhaps home-sickness as he describes how it brings tears too and memories from an old land He goes on to describe the restful pastoral beauty of the land where even the dead can lie in the green He then brings in voicesperhaps of family and friends calling him home as he is missing Aprils beautyThe voices then tempt him some more with idyllic images from home (white blossom young green cornrunning rabbitswarm sun) The voices seem to presume that the poets heart is sorrowful bruised and soreThe end of the poem sees the poet appear to make a decision he will go home as he has decided that is where he truly belongs

Write the synopsis of the following words

1 Daffodils- a tall yellow flower that grows in the spring

2 Orchards- a piece of land on which fruit trees are grown

3 Blossom- a flower or a mass of flowers especially on a fruit tree in spring

4 Thrushes- a bird5 Larks- a small brown bird that

makes a pleasant sound6 Bruised- an injury7 Aching- pain 8 Tread- to put your foot down

while you are walking9 Balm-10 May-11 Fluting-

(Write from the book in your copy)

MAT

HEM

ATIC

S

Ch 1

1Al

gebr

ic E

xpre

ssio

n

1 Constant A symbol which has fixed value is called a constant[eg 8 23 -15 radic3 etc]

2 VariableA symbol which does not have any fixed value but may be assigned value (values) according to the requirement is called variable or literal[eg x y p q etc]

3 TermsA term is a number (constant) a variable a combination (product or quotient) of numbers and variables[eg 7 x 5x etc]

4 Algebric expressionA single term or acombination of two or more terms connected by plus (+) or minus (-) sign forms an algebraic expression[eg 5-y 3x2-5x xy-6z+4 etc]

5 PolynomialAn algebraic expression which contains more than one term is called a polynomial (multinomial)[eg x2-5x 5y+xy+x2y etc]

6 Degree of polynomial(a) When the polynomial contains only one variable the highest power of the variable is the degree of the polynomialeg the degree of the polynomial of 4x-7x5+8 is 5(b) When the polynomial contains two or more variablesStep (i) Find the powers of the variables in each term (ii) The highest sum of the powers is taken to be the degree of the polynomialeg the degree of the polynomial 5x2y-4x3y5+6 is = 3+5 = 8Remember An algebraic expression is a polynomial if degree of each term used in it is a non-negative integer

Exercise ndash 11(A)

1 Separate the constants and variables from the following

-7 7+x 7x+yz radic5 radic xy 3 yz

8 45y -3x

Solution Constant Variables-7 radic5 7+x 7x+yz radic xy

3 yz8

45y -3x

2 Write the number of terms in each of the following polynomials(i) 5x2+3timesax (ii) axdivide4-7 (iii) ax-by+ytimesz (iv) 23+atimesbdivide2

Solution Polynomials Number of terms(i) 5x2+3timesax 2(ii) axdivide4-7 2(iii) ax-by+ytimesz 3(iv) 23+atimesbdivide2 2

4 Write the degree of the each polynomials(i) xy+7z (ii) x2-6x3+8 (iii) y-6y2+5y8 (iv) xyz-3 (vi) x5y7-8x3y8+10x4y4z4

Solution Polynomials Degree(i) xy+7z 2(ii) x2-6x3+8 3(iii) y-6y2+5y8 8(iv) xyz-3 3(vi)x5y7-8x3y8+10x4y4z4 12

5Write the coefficient of(i) ab in 7abx (iv) 8 in a2-8ax+a (v) 4xy in x2-4xy+y2

SolutionCoefficient

(i) ab in 7abx 7x(iv) 8 in a2-8ax+a -ax(v) 4xy in x2-4xy+y2 -1

7 CoefficientAny factor of an algebraic quantity is called the coefficient of the remaining quantityeg in the algebraic term 7xyz 7 is coefficient of xyz 7x is coefficient of yz and so on

8 Like term The terms having the same literal coefficient are called like terms and those having different literal coefficients are called unlike terms

eg (i) 5xyz 8xyz -6xyz and 23xyz are like

terms(ii) 7xy2 8x2yz and -15xyz2 are unlike terms

6 in 57xy2z3 write the coefficient of

(i) 5 (vii) 5xy2 (viii) 17yz (xi) 5xyz

Solution Coefficient

(i) 5 17

xy2z3

(vii) 5xy2 17z3

(viii) 17yz

5xyzsup2

(xi) 5xyz 17yz2

7 In polynomial given below separate the like terms(ii) y2z3 xy2z3 -58x2yz -4y2z3 -8xz3y2 3x2yz and 2z3y2

Solution y2z3 -4y2z3 2z3y2 are like terms

xy2z3 -8xz3y2 are like terms

-58x2yz 3x2yz are like terms

Class IXSubject Topic Summary Execution

Bengali (2nd language)

বাগzwnjধারাzwnj বা ধারা-বা ধারা ল হিবকেশষ পরকার বাক -হিb -াকেবর এক হিবকেশষ পরকাশরীহিত াকেক কতগকেলা কার সমষটির মকেধয এগহিলকেক বা ধারা বকেল আবার কতগকেলা শকেবদর বাধাধরা যকান রীহিত যনই য-াকেব চকেল আসকে যসই -াকেবই চকেল আসকে তখন যসই শবদগহিল খন একক -াকেব অG পরকাশ ককের তখন একের বা ধারা বকেল বা ধারার পরকেয়া -াষাকেক আরও সFর ককের যতাকেল

অকাল পকক(অপহিরনত বয়কেস পাকাহিম)-মাতর শ বর বয়কেস যমকেয়টির া মকেখর কা তাকেত অকালপককতা ধরা পকে

অককা পাওয়া( মারা াওয়া) ndash পকেকIমারটি পকেকIমারকেত হিকেয় বাসাতরীকের াকেত মার যখকেত যখকেত অককা যপল

অহি| পরীকষা ( কঠিন ও পরকত পরীকষা)- যকেলটির আজ ডাকতাহির যরজালট যবকেরাকেব এIাই তার জীবকেনর ব অহি| পরীকষা

অষটরমভা (ফাহিক) ndash রীতা মকেখই বকো বকো কা বকেল আর কাকেজর যবলায় অষটরমভা

অকমGার ধাী (অপাG) ndash সমনকেক হিনকেয় যকান ান কেব না ও একেকবাকেরই অকমGার ধাী

অকেনধর ষটি (অসাকেয়র সায়)- আহিশ বকেরর বকোর নাহিত ল অকেনধর ষটি তাকেক াা বকোর একম চকেল না

আকেককল গড়ম (তবহিদধ)- ার তহিম উপকার করকেল যসই যতামার হিবরকেদধ সাকষয হিকেয়কে শকেনই আমার আকেককল গড়ম

আষাকে লপ( অবাসতব লপ) ndashIাকা এখন যকেব না এIা বলকেলই ত এমন আষাকে লপ ফাার যকান রকার হিল না

Hindi- महायजञ ा इस हानी म लख न या बतान ा परयास किया ह कि किसी भी अचछ

2nd language

परसार(यशपाल ाय या पणय न ा फल अवशय मिमलता ह ोई भी परोपार अथवा पणय लिलए किया गया ाय बार नही 0ाता वह ए परार ा यजञ हए धनी सठ थ धम परायण और किवनमर सठ न आन ी यजञ किए थ और दान म न 0ान कितना धन दिदन दखिखयो म बात दिदया थादिदन पलट और सठ यहा गरीबी आ गई उन दिदनो यजञ बचन ी परथा थी सठ भी अपनी 0गह बचन लिलए डलपर ए सट यहा चलन ो तयार हए सठानी रासत लिलए रोटी पड म बाधर सठ ो द दी रासत म ए भख R ो दखर सठ न चारो रोटी उसो खिखला दी खर वह सठ यहा डलपर पहच तो उनी सठानी न उस महायजञ बचन ो हा यदिद बचन आए सठ न R ो रोटी खिखलान ो महायजञ नही समझा और वापस लौट आया घर आर शाम ो उसी घर म उस ए बडा ख0ाना मिमला 0ो उस दवारा किए गएrsquo महायजञrsquo ा परसार था

English language

Letter formal The heading the name and address of the person you are writing to must be included beneath your own address In formal letters ldquoblock stylerdquo of address is preferred

Subject complain in brief

Salutation If the person you are writing to is known to you you may begin ldquoDear MrrdquoOr ldquoDear Mrsrdquo In all other instances you should begin ldquoDear Sirrdquo or ldquoDear Madamrdquo Or ldquoSirsrdquo

The body A formal or business letter has four partsReference The letter should begin by referring to a letter you have received an advertisement or the reason that has prompted you to writeInformation In the second paragraph it is necessary to supply more detailed information that is related to the referencePurpose Here you must give the reason why you are writing the letter This must be stated clearly and ensure that it is relevant to the question that has been setConclusion round off the letter with some polite remarkThe subscription when a letter has begun with dear sir sirs Madam you should end with Yours faithfully or yours truly When however you address a person by name you must conclude with the words ldquoYours sincerelyrdquo

1 A park in your locality is slowly being used as a rubbish dump Write a letter to the Mayor of your city pointing out the nuisance and danger of this Request that action be taken to stop this immediately

Or2 You being a boarder ordered a set of lab manuals from a famous book shop in the town They sent you a wrong set of books Write a letter to the manager of the book shop

Chemistry Chapter-1 1)CHEMICAL FORMULA- Q What is the Significance of

L-2The Language of Chemistrybull Chemical Formula

Itrsquos a symbolic representation of a chemical substance eg ndash The formula of Sulphuric acid is H2SO4

2) Steps of writing Chemical Formula of a given substance-

1 Write the symbols of the constituent atoms or radicals side by side Keep the basic radical on LHS and acid radical on the RHS ( Na+Cl- )2 In case of a radical having more than one atom( compound radical) enclose the radical in a bracket eg (SO4-)3 Write the valencies of each radical on its right hand top4 If the valencies of the two radicals are divisible by a common factor then divide the valencies by the common factor5 Invert (criss-cross) the valency number ie write the valency of one atom below the second atom and vice versa 6 On interchanging if valency number is lsquoone the figure lsquoonersquo is never writtenFor Example- Compound -Calcium Nitrate1 Writing the symbols- Ca(NO3)2 Writing the valencies on their right hand top- Ca2(NO3)1

3 Valency numeral in simple ratio- Ca2(NO3)1

4 Criss-cross- Ca 2NO3 1

5 Writing the formula of the compound- Ca(NO3)2

Chemical formula

A The formula of a substance conveys the following information regarding a substance 1 The name of the substance (qualitative)2 The elements constituting the substance (qualitative)3 The number of various atoms present in a molecule of the substance (quantitative)4 Molecular weight of the substance and the relative weights of different elements present in it (qualitative)

Q What are the limitations of Chemical Formula

A The chemical formula suffers from the following limitations-I It fails to convey whether the elements in a molecule are present in the form of atoms or ionsFor example the formula KBr fails to tell us whether Potassium and Bromine are present in the form of ions II It does not tell anything about the binding force that holds atom in a molecule togetherIII It does not tell us about the arrangement of various atoms with respect to one another within the molecule

Q Examples of Some Chemicals with their Formula Chemical name and Common Name-

A Given in the class notesCommercial Studies

Joint Stock Company

Let us discuss about the demerits of Joint Stock CompanyDespite so many advantages it has got many disadvantages which are as follows

Difficulty in FormationDelay in Decision makingExcessive Government ControlLack of Secrecy

Company can be classified into several categories based on incorporation

QuestionExplain the demerits of Joint Stock CompanyAnswer) 1 Difficulty in Formation The legal requirements and formalities required to be completed are so many The cost involved is quite heavy It has to approach large number of people for its capital It cannot start its business unless certificate of incorporation has been obtained This is granted after a long time when all the formalities are completed

Chartered CompanyStatutory CompanyRegistered Company

Delay in Decision making In this form of organization decisions are not made by single individual All important decisions are taken by the Board of Directors Decision-making process is time-consuming So many opportunities may be costly because of delay in decision-making Promptness of decisions which is a common feature of sole trader ship and partnership is not found in a company

Excessive Government ControlA company and the management have to function well within the law and the provisions of Companies Act are quite elaborate and complex At every step it is necessary to comply with its provisions lest the company and the management should be penalized The penalties are quite heavy and in several cases officers in default can be punished with imprisonment This hampers the proper functioning of the company

Lack of Secrecy The management of companies remains in the hands of many persons Every important thing is discussed in the meetings of Board of Directors Hence secrets of the business cannot be maintained In case of sole proprietorship and partnership forms of organisation such secrecy is possible because a few persons are involved in the management

2 Define the following

Chartered Company- The crown in exercise of the royal prerogative has power to create a corporation by the grant of a charter to persons assenting to be incorporated Such companies or corporations are known as chartered companies Examples of this type of companies are Bank of England (1694) East India Company (1600) The powers and the nature of business of a chartered company are defined by the charter which incorporates it After the country attained independence these types of companies do not exist

in IndiaStatutory Company- A company may be incorporated by means of a special Act of the Parliament or any state legislature Such companies are called statutory companies Instances of statutory companies in India are Reserve Bank of India the Life Insurance Corporation of India the Food Corporation of India etc The provisions of the Companies Act 1956 apply to statutory companies except where the said provisions are inconsistent with the provisions of the Act creating them Statutory companies are mostly invested with compulsory powersRegistered companiesCompanies registered under the Companies Act 1956 or earlier Companies Acts are called registered companies Such companies come into existence when they are registered under the Companies Act and a certificate of incorporation is granted to them by the Registrar

Economics

Chapter-4Basic problems of Economy

Today let us discuss with the topic Production Possibility curve

QuestionExplain the concept of Production Possibility Curve with the help of diagram

Answer) Production Possibility curve is a locus of all possible combinations of two commodities which can be produced in a country with its given resources and technology

The above diagram shows that with the given resources and technology the economy can produce maximum either 5 thousand meters of cloth or 15 thousand quintals of wheat or any other combination of the two goods like B( 1 thousand meters of cloth and 14 thousand quintals of wheat C ( 2 thousands meters of cloth and 12 thousand quintals of wheat) etcProduction Possibility curve is also called production possibility boundary or frontier as it sets the maximum limit of what it is possible to produce with given resources

Geography

Rotationand Revolution

SUNrsquoS POSITION AND SEASONAL CHANGES EQUINOXES ndash SPRING AND AUTUMN

Q1 What is Spring EquinoxA1 On 21st March sunrays fall directly on the equator On that day

As the Equator divides the Earth into two equal halves the sun rays fall directly on the equator twice in a year Equinoxes means equal Spring EquinoxOn 21st March sunrays fall directly on the equator On that day the duration of day and night both are equal ( 12 hours day and 12 hours night) on every places located on equator This day is called as Spring EquinoxAutumn EquinoxOn 23rd September sunrays fall directly on the equator On that day the duration of day and night both are equal ( 12 hours day and 12 hours night) on every places located on equator This day is called as Autumn Equinox

SOLSTICES ndash SUMMER AND WINTERDue to inclination of the Earth on its axis and the apparent movement of the sun the sun rays fall directly on both tropics once in a year Solstice is a Latin word which mean ldquothe Sun standing stillrdquoSummer SolsticesAfter 21st March there is an apparent movement of the Sun to the north of the equator The apparent northward movement up to 21st June when the Sun appears overhead at the Tropic of Cancer (22frac12degN) The sun appears to stand still at this position and then moves southwards towards the equator This position of the Sun on 21st June is known as Summer Solstices On that day the duration of day and night both are equal ( 12 hours day and 12 hours night) on every places located on Tropic of Cancer (22frac12degN)Winter solstices The apparent southward movement of the Sun continues beyond the equator till 22nd

December On this day the Sun is overhead at the Tropic of Capricorn

the duration of day and night both are equal ( 12 hours day and 12 hours night) on every places located on equator This day is called as Spring Equinox

Q2 What do you mean by EquinoxA2 Equinoxes means equal It is use to explain the equal duration of day and night ( 12 hours day and 12 hours night) on the Earth

Q3 On which date the longest day in Tropic of CancerA3 21st June

Q4 What is the meaning of SolsticeA4 Solstice is a Latin word which mean ldquothe Sun standing stillrdquo

Q5 Which is the longest day in southern hemisphereA5 22nd December

Q6 On what date does the Arctic Circle experience the lsquoMidnight SunrsquoA6 On 21 June the Arctic Circle experiences the lsquoMidnight Sunrsquo

Q7 What is cause of Midnight Sun in NorwayA7 During the summer solstice (21 June) the North Pole is inclined towards the Sun Therefore the duration of sunlight or daytime increases from 12 hours at the Equator to 24 hours at the Arctic Circle and beyond Thatrsquos why The region beyond the Arctic Circle especially Norway is known as the Land of the Midnight Sun because there the Sun does not rise or set on 21 June

Q8 Match the column A with BA B

Summer Solstice 21st March

Autumn Equinox 23rd

September

Winter Solstice 21st June

(22frac12degS) This position of the Sun is referred to as the Winter Solstice because it marks the winter season in the Northern Hemisphere On that day the duration of day and night both are equal ( 12 hours day and 12 hours night) on every places located on Tropic of Capricorn (22frac12degS)SEASONS AND DURATION OF DAY AND NIGHT During the equinoxes all places on the Earth have 12 hours of day and 12 hours of night Due to the revolution of the Earth round the Sun on an inclined axis the duration of day and night varies according to seasons and the latitude of a placeDuring the summer solstice (21 June) the North Pole is inclined towards the Sun Therefore the duration of sunlight or daytime increases from 12 hours at the Equator to 24 hours at the Arctic Circle and beyondThe region beyond the Arctic Circle especially Norway is known as the Land of the Midnight Sun because there the Sun does not rise or set on 21 JuneAt the North Pole there will be six months of daylight The Sun will be seen always above the horizon at a low angle At 66degN 24 hours of sunlight can be seen only on 21 June Hammerfest in northern Norway is a place of tourist attraction for observing the phenomenon of the Midnight Sun This place has continuous daylight from 13 May to 29 July This place is easily accessible to tourists and has hotels and other facilities The view of the midnight Sun from here is enthrallingIn the Southern Hemisphere the duration of daylight decreases from 12 hours at the equator to 0 hours beyond the Antarctic Circle In the South Polar Region there is 24 hours of darkness The Sun is always below the horizon In the Southern Hemisphere which experiences winter the duration of night-time is longer than the duration of daylight

Spring Equinox 22nd

December

A8 A B

Summer Solstice 21st June

Autumn Equinox 23rd

September

Winter Solstice 22nd

December

Spring Equinox 21st March

During winter solstice (22 December) the South Pole is inclined towards the Sun The Southern Hemisphere experiences summer and the Northern Hemisphere has winter Therefore the duration of daylight or sunlight is greater in the Southern Hemisphere than in the Northern HemisphereThe duration of daylight increases from 12 hours at the equator to 24 hours beyond the Antarctic Circle The South Polar Region has 24 hours of sunlight for many days continuously At the South Pole there will be six months of sunlight The Sun will always be seen at a low angle above the horizon In the Northern Hemisphere the duration of daylight will decrease from 12 hours at the equator to 0 hours at the Arctic Circle There are 24 hours of darkness in the North Polar region The duration of night is greater than the duration of daylight as one move northwards from the Equator It is evident from the above table that the duration of daylight is 12 hours throughout the year at the equator only As one moves away from the equator the seasonal variations in the duration of daylight increase The seasonal variations in the duration of daylight are maximum at the Polar Regions

Subject Eng Literature (The Merchant of Venice ndash William Shakespeare)Topic Act II Scene 7 Lines 36 to 80 (End of scene ) [Students should read the original play and also the paraphrase provided]

Summary Questions amp AnswersThe Prince then examines the inscription on the silver casket which says ldquoWho chooseth me shall get as much as he deservesrdquo The Prince says that he deserves Portia more than anybody else because of his high rank his noble birth and his great wealth and power But then he argues that silver is ten times

(1) (Act II Sc 7 L 39-47)

From the four corners of the earth they come

To kiss this shrine this mortal breathing saint

The Hyrcanian deserts and the vasty wildsOf wide Arabia are as through-fares now

inferior to gold and therefore he cannot believe that the portrait of such a beautiful lady as Portia can be contained in the silver casket He decides to see the inscription on the golden casket before making his decision

The Prince goes to examine the inscription on the golden casket which says ldquoWho chooseth me shall get what many men desirerdquo The Prince believes that the whole world desires to possess Portia otherwise so many suitors would not have come from all corners of the world for winning Portia Some of them have come from the distant lands of Persia and Arabia The deserts of Persia (Hyrcanian deserts) and the boundless desolate lands of Arabia have been crossed by the Princes seeking the hand of Portia He contrasts this casket containing Portiarsquos portrait with the old English gold coin bearing the image of the archangel (angel of the highest rank) He goes on to remark that while the figure of the archangel is engraved (Insculped) upon the English coin the picture of Portia who is beautiful as an angel lies hidden inside one of the caskets namely the Golden Casket (Golden Bed)

On the basis of his assessment of the inscription on the golden casket the Prince decides to choose the golden casket He asks for the key and opens the golden casket only to find therein an empty human skull holding a roll of

For princes to come view fair PortiaThe watery kingdom whose ambitious headSpets in the face of heaven is no barTo stop the foreign spirits but they comeAs orsquoer a brook to see fair Portia

(i) Explain the occasion for the above mentioned speech

These are the comments of the Prince of Morocco after he reads the inscription on the golden casket His mental process is revealed to us in these words We find him debating within himself as to which casket he should choose

(ii) What light does the above speech throw on the personality of Prince of Morocco

From the above mentioned speech we come to know that the Prince of Morocco is keen to marry Portia He is the type of person who is easily taken away by outward appearance He is in love with Portia because of her beauty

(iii) What information can you gather about Portia from the above mentioned lines

The given speech shows that Portia is a very beautiful lady She must be possessed of good qualities because many suitors come to her place from all over the world with a desire to get married to her The Prince of Morocco is so impressed by her beauty that he calls her a saint According to him the whole world is desirous of having her

(iv) Elucidate the significance of the first two lines

In these lines the Prince of Morocco pays a compliment to Portia These lines show his admiration for her He says that people come from all parts of the world to see fair Portia

(v) Explain the meaning of the last four lines of the

passage

In these lines the Prince of Morocco says that even the vast oceans which throw a challenge at the sky are unable to prevent men from coming to Portiarsquos place to have a glimpse of her These lines are also a tribute to Portiarsquos beauty and good qualities Many men voyage across the ocean treating it as a mere stream to see the beautiful Portia

paper in which is written that whoever happens to be guided by the glitter of things is invariably deceived

On reading the scroll the Prince says that he is too sad at heart to speak a more formal farewell and leaves with his followers amidst a sound of trumpets

After the Prince of Morocco leaves Portia remarks that the Prince is a gentle fellow but she is rid of him May all persons of his nature make a similar choice

IMPORTANT PASSAGES EXPLAINED

(Act II Sc 7 L 39-43)From the four corners of the earth they come

To kiss this shrine this mortal breathing saintThe Hyrcanian deserts and the vasty wildsOf wide Arabia are as through-fares nowFor princes to come view fair Portia

Context

This passage occurs in Act II Scene 7 in The Merchant of Venice This is part of the speech made by the Prince of Morocco

(2)

(Act II Sc 7 L 48-53)

MOROCCO One of these three contains her heavenly pictureIst like that lead contains her

Twere damnation To think so base a thought it were too grossTo rib her cerecloth in the obscure graveOr shall I think in silver shes immurdBeing ten times undervalued to tried gold

(i) What meaning does the Prince of Morocco find out of the inscription of the golden casket What have Belmont and Portiarsquos house been called and why

The inscription on the golden casket is ldquoWho chooseth me shall gain what many men desirerdquo The Prince finds out that it means that the chooser of the golden casket will get Portia because many men desire her In fact the entire world desires her Because of the coming of many suitors to Belmont from different countries in order to win Portiarsquos hand Belmont has become a centre of pilgrimage and her house is the shrine where saintly Portia is installed

(ii) What does the Prince of Morocco do before making the final choice of the casket Which is the correct casket and who will win Portiarsquos hand

The Prince of Morocco surveys and analyses the inscriptions on the casket of lead silver and gold Before making the final choice like a very systematic and methodical person he once again considers the claims of the caskets The casket containing Portiarsquos picture is the correct casket and the person choosing it will win Portiarsquos hand

Explanation

While praising Portia the Prince of Morocco conceives Portia as a goddess whose image is placed inside one of the caskets Many suitors are coming from far and wide the north and the south the east and the west (Four corners) in order to try their luck Some of them have come from the distant land of Persia and Arabia The deserts of Persia (Hyrcanian deserts) and the boundless desolate lands of Arabia have been crossed by the Princes seeking the hand of Portia All this shows that Portia is indeed the most beautiful lady of the world

(iii) What does the Prince of Morocco say in his estimation while examining the motto on the silver casket What does he find in the golden casket

While examining the motto on the silver casket which says ldquoWho chooseth me shall get as much as he deservesrdquo Morocco says that in his own estimation he surely deserves Portia in all respects ndash rank birth wealth etc

He chooses the golden casket When he opens it he finds an empty human skull holding a scroll in which it is written that those who are attracted by the glittering outside of things are always deceived as Morocco has been deceived

(iv) What kind of nature does the Prince of Morocco have

The Prince of Morocco has a simple nature who does not look deeply into the inner meaning of things but is dazzled by the outward appearance of gold He is inclined to over-estimate his own value and does not realize that it is a duty to ldquogive and hazardrdquo To say that he will not hazard for lead shows that he misreads the true meaning of the inscription which is that he should be prepared to ldquohazard all he hathrdquo for Portia So his feeling is only one of fascination and romantic attraction

(v) Do you think that the lottery of the caskets is not a matter that will be determined by chance

In fact the lottery of the casket is not a matter that will be determined by mere chance but that it is a true test of character and of sincerity which is amply proved not only by Moroccorsquos choice but also by the arguments which he uses to help him in his choice

(Act II Sc 7 L 55-59)

They have in England

A coin that bears the figure of an angelStamped in gold but thats insculpd uponBut here an angel in a golden bedLies all within

Context

(3)

(Act II Sc 7 L 63-77)A carrion Death within whose empty eye

There is a written scroll Ill read the writing

All that glisters is not goldOften have you heard that toldMany a man his life hath soldBut my outside to beholdGilded tombs do worms infoldHad you been as wise as boldYoung in limbs in judgment oldYour answer had not been inscrolld

This passage occurs in Act II Scene 7 in The Merchant of Venice This is part of the speech made by the Prince of Morocco

Explanation

In this passage the Prince of Morocco bestows high praise on Portia whose hand he is seeking He contrasts this casket containing Portiarsquos portrait with the old English gold coin bearing the image of the archangel (angel of the highest rank) He goes on to remark that while the figure of the archangel is engraved (Insculped) upon the English coin the picture of Portia who is beautiful as an angel lies hidden inside one of the caskets namely the Golden Casket (Golden Bed) In the day of Elizabeth silver was ten times inferior in value to gold Therefore the Prince of Morocco believing that Portiarsquos portrait is contained in the Golden Casket decides to choose the Golden Casket

Fare you well your suit is coldCold indeed and labour lostThen farewell heat and welcome frostmdashPortia adieu I have too grievd a heartTo take a tedious leave Thus losers part

(i) What reward does the Prince of Morocco get after making a wrong choice of the Casket How does he feel

After making the wrong choice in selecting the casket of gold the Prince of Morocco as a reward earns a rebuke in the form of a scroll tucked in the empty eye-socket of a skull kept in the casket of gold The Prince is shocked and disappointed He becomes all the more sad and dejected when he reads the scroll which points to his foolishness in being misled by the appearance and outward show as indicative of its worth

(ii) How does the Prince respond after reading the scroll

After reading the scroll the Prince though upset accepts the result with good grace and decorum befitting a royal suitor and true sportsman He says that his love-suit is really cold otherwise he would have chosen correctly but now his efforts have been in vain So he bids farewell to Portia to the warmth and enthusiasm of love and welcomes the cold and bitterness of dejection and misery of life which lies ahead

(iii) What request does he make to Portia and why

After being failure in his mission he requests Portia to give him permission to leave at once because he is too sad to undergo the tediousness of a formal leave-taking He tells that it is the manner in which defeated persons part unceremoniously

(iv) Explain the following lines

ldquoAll that glisters is not goldOften have you heard that toldMany a man his life hath soldBut my outside to beholdGilded tombs do worms infoldrdquo

Mere glitter does not make a metal to be gold Man has often been warned against appearance but it has been of no use Many people have sacrificed their lives only to seek the outer appearance of gold Worms are found inside the gilded

monuments

Class XSubject Topic Summary Execution

Hindi 2ndlang

नया रासता भाग 6 मायाराम 0ी घर म धनी मल 0ी और उनी बटी सरिरता ी ही चचा बनी रहती थी अमिमत ो इसम ोई रलिच ना थी वह धनी घर ी लडी स शादी र सवय ो बचना नही चाहता था उसा भी सवाणिभमान ह ईशवर ी पा

स उस पास पस ी ोई मी नही थी अभी उसन फकटरी ही लगाई थी उसी समझ बाहर था कि उस घर वालो ा झाव पस ी तरफ कयो

ह उसन मा स सवाल किया कि मा तम सरिरता स मरी शादी कयो रना चाहती हो मा न उस समझाया कि वह दखन म बरी नही ह और किफर खानदान अचछा

ह वह ए शल गरहणी रप म घर सभाल सगी अमिमत न मा ो इस बात ा एहसास राया कि मीन सबध लिलए मना रन पर उस दिदल

पर कया बीती होगी मा और अमिमत ी लडी बार म ाफी बात हईमा ा झाव सरिरता ी तरफ था कयोकि वह घर पर अचछा दह0 लर आ रही

थी अमिमत न अपनी मौसी ी बरी हालत बार म बताया कि किस तरह वह बड घर ी खानदानी बटी लाई थी और आ0 उसी हालत कितनी खराब ह लाई थी बहकलब 0ाती ह और बचचो ो भी नही दखती ह बात चल ही रही

थी कि तभी ए ार बाहर आर री धनी मल0ी घर अदर आए और पीछ स डराइवर फल ी ए टोरी लर आया अदर आए और पीछ स

डराइवर ए टोरी फल ी लर आया अमिमत ो फल ी पटी बरी लग रही थी अमिमत न पछ लिलया यह फल कयो ल आए ह प इन सब ी कया

0ररत थी उनो न 0वाब दिदया कि 4 पटी शमीर स मगाए थ अमिमत ो या सनर करोध आ गया तभी उस किपता 0ी आ गए उन आत ही अमिमत उठर बाहर चला गया वहा वहा मा पास आर बठ गया और बोला

अभी रिरशता तय नही हआ और धनी मल 0ी धनी मल 0ी फल ी पटी लर चलआय मा न समझाया कि 0ब सबध 0ड 0ाता ह तो खाली हाथ नही

आत अमिमत न मा स हा कि तम सबन सरिरता ो इस घर म लान ी ठान रखी ह धनीमल 0ी उस दिदन सरिरता ो दखन ी तारीख तय रन आय थ

Commercial Studies

Banking Nowadays Bank provide easy and quick services through internet facilities methods of Banking is called internet bankingIn order to save the time and money involved in visiting Bank branches people increasingly prefer to have internet banking

There are different modes of doing internet banking or transferring money through online They areReal Time Gross Settlement (RTGS)National Electronic Fund Transfers (NEFT)

1

Question

1) Explain the term RTGS Write the features of RTGS

Answer)The acronym RTGS stands for Real Time Gross Settlement which may be defined as the continuous real time settlement of funds transfer individually on and order by order basis without netting lsquoReal timersquo may be defined as the processing of instructions at the time they are received rather than at some letter time lsquoGross settlementrsquo may be defined as the settlement of transfer instructions which occurs

individually

Features of RTGS1It is the continuous settlement of

funds transfer individually on an order by order basis

2RTGS facility is provided only by CBS core banking solution enabled Bank branches

3Amount charged from the customer for RTGS transactions vary from bank to bank

2) Explain the term NEFT Write the features of NEFT

Answer) National electronic funds transfer may be defined as a nationwide system that facilitates individuals Farms and copper operates to electronically transfer funds from any bank branch to any individual farm or corporate having an account with any other bank branch in the country

Features of NEFT2 Transfer can be made 7 times on

weekdays and 6 times on Saturday

3 NEFT cannot be used to receive foreign remittances

4 NEFT transaction takes place in batches

5 A bank branch must be NEFT enabled to become a part of NEFT fund transfer network

6 There is no maximum or minimum amount that can be transferred through NEFT when one bank has a bank account

English Language

CompositionEssay

A composition is an art of creating a piece of writing on any topic or subject It is the writing correctly beautifully and clearly in order to make some interesting reading Structure of the composition

Introduction ( you lay the foundation for your composition)

Body (it constitutes the main part of the essay)

Conclusion (final statement that leaves a lasting impression)

Kinds of essays1 The Narrative essay2 The descriptive essay3 The reflective essay4 The argumentative essay

Write a composition on any one of the following topics (350- 400 words)

1 Friendship Or2 The first day of your school

Subject Eng Literature (The Merchant of Venice ndash William Shakespeare)Topic Act V Scene 1 Lines 127 to 158 (Nerissa helliphellip The clerk will nersquoer wear hair onrsquos face that had it) [Students should read the original play and also the paraphrase given in the school prescribed textbook]

Summary Revision Questions o Soon thereafter Bassanio Gratiano

and Antonio arrive

o Bassanio tells Portia that he is feeling as if it is morning because of the presence of Portia who is shining like the sun When Antonio is introduced by Bassanio to Portia she tells Bassanio that he should be grateful to Antonio who took so much trouble on his account even to the extent of risking his life

o Nerissa starts quarrelling with Gratiano and demands that he show her the ring she had presented to him and which she had warned him not to lose She suspects that Gratiano must have presented the ring to some young woman and not to the lawyerrsquos clerk as he repeatedly says and assures

Answer the following questions to check your preparation of Act IV Scenes 1 and 2

You must attempt only after you have completed your preparation of Act IV The answers must be in complete sentences using textual evidence (with citation) when necessary

[It would be in your own interest to attempt the above questions honestly totally refraining from consulting your textbook or your notes during answering After completion you should correct the paper yourself consulting the textbooknotes etc and award marks as specified Please let me know the marks you scored through WhatsApp in the group or to my personal WhatsApp]

Act IV Scene 1 (each question carries 2 marks)

1 What did the Duke try to do for Antonio

2 Why does Shylock refuse to show mercy How does he justify his stance

3 Why does Antonio say he is ready to die 4 What information is contained in Bellariorsquos letter

5 Why does Portia (as Balthazar) assert that Shylock must show mercy How does he respond

6 What offers are made to Shylock to get him to spare Antonio How are they received

7 What does Antoniorsquos speech as he faces the prospect of Shylockrsquos knife tell you about his character

8 How do Bassanio and Gratiano react to the looming prospect of Antoniorsquos demise

9 How does Portia (as Balthazar) use the law to turn the tables on Shylock

10 What does the Duke decree should happen to Shylock Why What happens to Shylockrsquos estate

11 What does Portia ask Bassanio as payment for her ldquoservicesrdquo What is his initial response What makes him change his mind

Act IV Scene 2 (each question carries 1frac12 marks)

1 What does Gratiano bring to Portia (Balthazar)

2 What does Nerissa plan on getting from Gratiano What does Portiarsquos comment suggest about men

ECO-10 280620 Topic-Supply AnalysisSHIFTING OF SUPPLY

But if there is change in factors other than the price of the commodity then either more is supplied at the same price or less supplied at the same price In such cases the price of the commodity remains constant but there is a change in other factors like change in the price of inputs change in technology of production change in price of other related goods change in taxation policy of the government etc For example there is an improvement in the technology of production of the commodity in question It leads to decrease in per unit of cost production of the commodity The firm is willing to sell more quantity of the commodity at the same price So the supply other commodity increases at the same price This increase in supply is shown by rightward shift of supply curve On the other hand if the firm uses inferior technology of production the cost of production per unit of the commodity increases The firm is willing to sell less quantity at the same price So the supply of the commodity decreases at the same price This decrease in supply is shown by leftward shift of the supply curve The above cases of increase and decrease in supply can be shown with the help of the following figures

Y INCREASE OF SUPPLY Price (Rs) s

P A s1

B

s

X` O s1 X

q q1

Y` Quantity demanded (in units)

Y DECREASE IN SUPPLY s2

s

price (Rs)

C

p A

s2

s

X` o X

q2 q

Y` Quantity demanded ( in units)

Main factors causing increase in supply or rightward shift of supply Curve(i) Fall in the price of other related goods

(ii) Fall in the price of inputsfactors(iii) Use of better technology in production(iv) Decrease in the rate of excise duty by government(v) If the objective of producer changes from profit maximization to salesMaximization

Main factors causing decrease in supply or leftward shift of supply curve(i) Increase in the price of other related goods(ii) Rise in the price of inputsfactors(iii) Use of inferior technology in production(iv) Increase in the rate of excise duty by the government(v) If the objective

Subject - Biology Topic ndash Chapter mdash6 PhotosynthesisSummary Execution

Today we will know about photosynthesis and its stages

Q1 What do you mean by photosynthesis The process by which living plants containing chlorophyll produce food

substances from carbon-di- oxide and water by using light energy Sunlight

6CO2 +12 H2O----------------------- C6 H12O6 + 6H2O + 6O2

Chlorophyll

Q2 What are the importance of photosynthesis I) Food for all Green plants trap solar energy by photosynthesis

process and supply food and energy for all living organisms either directly or indirectly

Ii) Oxygen to breathe in by product of photosynthesis is oxygen which is essential for all living organisms respiration

Q3 Write about two main phases of photosynthesis A Light dependent phase This phase occur in grana of chloroplast I) The chlorophyll on exposure to light energy becomes activated by

absorbing photons Ii) The absorbed energy is used in splitting the water molecules (H2O)

into its two components (H+ and OH- ) and releasing electron s 2H2O------------------------- 4H+ + 4e- +O2

Energy of 4 photons This reaction is known as photolysis

End products are H+ and oxygen water

B Light independent (Dark ) phase The reactions in this phase require no light energy

Here CO2 combine with H+ and produce glucose

Class XI

Subject Topic Summary ExecutionEVS Chapter-4 Legal

regimes for sustainable development

Environmental legislationEnvironmental legislation is the collection of laws and regulations pertaining to air quality water quality the wilderness endangered wildlife and other environmental factors The act ensures that matters important to the environment are thoroughly

Learn -The Forest (Conservation) Act 1980

considered in any decisions made by federal agencies

The Forest (Conservation) Act 1980 The Forest (Conservation) Act 1980 an Act of the Parliament of India to provide for the conservation of forests and for matters connected therewith or ancillary or incidental thereto It was further amended in 1988 This law extends to the whole of IndiaObjects and Reasons of the Forest Conservation Act

Deforestation causes ecological imbalance and leads to environmental deterioration Deforestation had been taking place on a large scale in the country and it had caused widespread concern The act seeks to check upon deforestation and de-reservation of forests

Subject Eng Literature (The Tempest ndash William Shakespeare) Topic Act II Scene 1 Lines 314 to 329 (End of scene)

[Students should read the original play and also the paraphrase given in the school prescribed textbook]Summary Questions amp Answers

Conspiracy of Antonio and Sebastian (Contd)

o As they approach Ariel appears again and wakes up Gonzalo by singing a tune in his ear Alonso also wakes up and they see both Sebastian and Antonio with drawn swords On being caught off guard they make up a story saying that they had heard a bellowing of bulls or lions

o They then moved to another part of the island

o Ariel at once rushes to Prospero to inform him of this development

SUMMING-UP of ACT-2 SCENE-1

(i) Among the survivors Ferdinand is separated from the rest which results in the disconsolate grief of Alonso as he took him for dead

(ii) The villainy of Antonio is confirmed

(iii) The supremacy of Prosperorsquos magic which resulted in the failure of the human conspiracy

(1)

(Act II Sc 1 L 311-325)SEBASTIAN Whiles we stood here securing your repose

Even now we heard a hollow burst of bellowing Like bulls or rather lions Didt not wake youIt struck mine ear most terribly

ALONSO I heard nothingANTONIO O rsquotwas a din to fright a monsters ear

To make an earthquake Sure it was the roarOf a whole herd of lions

ALONSO Heard you this GonzaloGONZALO Upon mine honour sir I heard a humming

And that a strange one too which did awake meI shaked you sir and cried As mine eyes opened I saw their weapons drawn There was a noiseThats verily rsquoTis best we stand upon our guardOr that we quit this place Lets draw our weapons

(i) Why has Prospero sent Ariel to Gonzalo and Alonso What does Ariel do to awaken Gonzalo

Prospero has already come to know by his magic powers the danger which threatens Gonzalo who had been Prosperorsquos friend and so he sent Ariel to preserve the lives of both Gonzalo and Alonso Prospero does not want that his scheme should remain unfulfilled Ariel begins to sing a song in Gonzalorsquos ears to awaken him(ii) Who are ready to carry out their plan Who takes steps to stop them Why does Gonzalo feel surprised after being awakened

Sebastian and Antonio are ready to carry out their plans They are standing with their swords drawn to kill Alonso and

(iv) We see two sets of contrasting characters Gonzalo-Adrian against Antonio-Sebastian

(v) The grief that works in Alonso can be perceived to his repentance for his association in Antoniorsquos crime against Prospero

Gonzalo Ariel takes steps to stop them from carrying out their nefarious scheme When Gonzalo is awakened by the song sung by Ariel into his ears he (Gonzalo) feels surprised because he sees Sebastian and Antonio standing with their swords drawn(iii) What reason do Sebastian and Antonio tell of drawing their swords when they are suspected by Alonso and Gonzalo

When Sebastian and Antonio are seen with their swords drawn they are looked with suspicion by Gonzalo and Alonso At first Sebastian tells them that as they stood here to guard them during their sleep they heard only a little before a sudden loud noise very much like the roaring of bulls or more probably that of lions Then Antonio follows him saying that this was a noise so terrible as to frighten even a monsterrsquos ears and this noise could even have shaken the earth and it was surely like the roaring of a multitude of lions Then seeing the danger they have drawn their swords Perhaps after hearing the terrible noise they (Gonzalo and Alonso) woke up from their sound sleep

(iv) What does Gonzalo tell Alonso about the strange noise What did he see on opening his eyes Gonzalo tells Alonso that he did not hear the sound of roaring but he heard a humming sound which was strange and which woke him up After waking up he gave him (Alonso) a shaking and a loud cry On opening his eyes he saw these two gentlemen standing with their swords drawn(v) What does Gonzalo suggest

Gonzalo suggests that there was a noise indeed and of that he has no doubt at all and suggests that the best course for them would be to remain alert and vigilant against any possible danger to their lives or to leave this place and move to some other part of the island

Class XIISubject Topic Summary Execution

Commerce

Chapter- Management

Today we will discuss about LEVELS OF MANAGEMENT

Levels of management is a series or chain of managerial positions from top to bottom It helps individuals to know their authority responsibilities and superior-subordinate relations among themselves There are mainly three levels of Management TOP LEVEL MANAGEMENTMIDDLE LEVEL MANAGEMENTLOWER LEVEL MANAGEMENT

Top level managementIt consists of members at the highest level in the management hierarchy This level includes Board Of Directors Chief Executive Managing Directors Chairman President Vice President

Rolefunctions of the top levelmanagement1To analyse evaluate and deal

with theexternal environment2 To determine the objectives and

policies of the business3 To strive for welfare and survival

of business

4 To create an organisational Framework consisting of authority responsibility relationship

Middle level management Congress of members or groups who are concerned with implementation of the policies let down by the top managementThis level includes head of the department such as finance manager marketing manager branch and regional managers departmental and divisional heads plant superintendent etc

Role of functions of the middle level management

1 To interpret the policies framed by top management

2 To assign duties and responsibilities to lower level managers

3 To select and appoint employees for middle and supervisory level and evaluate their performance

4 To co-operate with other departments for smooth functioning

Operational or supervisory level managementIt refers to the group are members who are concerned with execution of the work They are also known as fast line managers This level includes supervisor 4 men Section Officer clerk Inspector etc

Role of functions of the lower level management1 To plan and execute day-to-

day operations2 To supervise and control the workers3 To arrange materials and

tools to start the process and make arrangements for training

4 Today present workers grievance and suggestions before the management and

ensure safe and proper working conditions in the factory

Business Studies

Staff Appraisal Chapter- 10 Today let us start with a new chapter

Staff Appraisal

Meaning of Performance Appraisal

Performance Appraisal is the systematic evaluation of the performance of employees and to understand the abilities of a person for further growth and developmentThe supervisors measure the pay of employees and compare it with targets and plansThe supervisor analyses the factors behind work performances of employeesThe employers are in position to guide the employees for a better performance

Objectives of Performance Appraisal

Following are the objectives of Performance Appraisal

To maintain records in order to determine compensation packages wage structure salaries raises etc

To identify the strengths and weaknesses of employees to place right men on right job

To maintain and assess the potential present in a person for further growth and development

To provide a feedback to employees regarding their performance and related status

To provide a feedback to employees regarding their performance and related status

Importance of Performance Appraisal

Performance appraisal provides important and useful information for the assessment of employees skill

knowledge ability and overall job performance The following are the points which indicate the importance of performance appraisal in an organization

1 Performance appraisal helps supervisors to assess the work performance of their subordinates

2 Performance appraisal helps to assess the training and development needs of employees

3 Performance appraisal provides grounds for employees to correct their mistakes and it also provides proper guidance and criticism for employees development4 Performance appraisal provides reward for better performance

5 Performance appraisal helps to improve the communication system of the organization

6 Performance appraisal evaluates whether human resource programs being implemented in the organization have been effective

7 Performance appraisal helps to prepare pay structure for each employee working in the organization

8 Performance appraisal helps to review the potentiality of employees so that their future capability is anticipated

Geography

DRIANAGE The SubarnarekhaThe Subarnarekha and the Brahmaniinterposed between the Ganga and the Mahanadi deltas drain an area of 19300 sq kmand 39033 sq km respectively The drainage basins of these streams are shared byJharkhand Odisha west Bengal and Chhattisgarh The Brahmani is known as southKoel in its upper reaches in Jharkhand

The NarmadaThe Narmada rises in the Amarkantak hills of MadhyaPradesh It flows towards the West in a rift valleyformed due to a geological fault The total length of it is 1300 km All the tributaries of the

Q1 Name the two westward flowing rivers in the peninsular plateauA1 Narmada and Tapi are the only westward flowing rivers of the peninsular plateau

Q2 Differentiate between east-flowing rivers and west-flowing riversA2

East-flowing rivers

West-flowing rivers

Narmada are very short inlength Most of its tributaries join the main streamright anglesThe Narmada basin covers parts of Madhya Pradesh and Gujarat

The Tapi The Tapi rises in the Satpura ranges in the Betul listrictof Madhya Pradesh It flows in a rift valley parallel tothe Narmada but it is much shorter in length It coversparts of Madhya Pradesh Gujarat and MaharashtraThe length is about 724 km

The Sabarmati and the MahiThe Sabarmati rises in the Aravali hills and flows south-south-westwards for a distance of 300 kilometres to the Arabian Sea The Sabarmatibasin extends over an area of 21674 sq km in Rajasthan and Gujarat The Mahi rises inthe east of Udaipur and drains an area of 34842 sq km lying in Madhya PradeshRajasthan and Gujarat It flows south-westwards for a distance of 533 km before it fallsinto the Gulf of Khambhat

The ChambalThe Chambal rises near Mhow in the Vindhya Range and flows towards the northgenerally in a gorge upto Kota Below Kota it turns to the north-east direction and afterreaching Pinahat it turns to the east and runs nearly parallel to the Yamuna beforejoining it in the southern part of the Etawah district in Uttar PradeshMajor Rivers of India with their basin area (Sqkm)

Himalayan System Indus 321290Ganga 861404

Brahmaputra 187110Indus System

Jhelum 34775Beas 20303

Ganga System Yamuna 366223Ghaghra 127950

Peninsular RiversNarmada 98796

Tapi 65145Mahanadi 141600

Subarnarekha 19300Sabarmati 21674

Mahi 34842Godavari 312812

Godavari Krishna Kaveri Mahanadi are the east-flowing rivers

Narmada Tapi west-flowing rivers

They fall into the Bay of Bengal

They fall into Arabian Sea

These rivers form big deltas

These rivers form comparativelysmall deltas

Catchment areas of these rivers are larger

Catchment areas of these rivers are smaller

Krishna 2589488Cauveri 87900

Subject ndashBiology Topic ndashChapter -5 Inheritance amp Variations Summary ExecutionToday we will discussabout linkage and its classification

LINKAGE The tendency of the genes located on the same chromosome to stay together is

hereditary transmission Linked genes the genes responsible for this Genes that exhibit the process of linkage locates in the same chromosome The distance between the linked genes in a chromosome determines the strength

of linkage i e genes that are located close to each other show stronger linkage than that are located far from each other

COMPLETE LINKAGE It is the type of linkage showed by the genes that are closely located or are tightly

linked with each other as they have no chance of separatingby crossing over These genes are always transmitted together to the same gamete and the same

offspring In such condition only parental or non cross over type of gametes are formedINCOMPLETE KINKAGE It is type of linkage showed by the genes that are distantly located orare loosely

linked with each other because they have chance of separating by crossing over

SIGNIFICANCE i) It helps in holding the parental character togetherii) It checks the appearance of new recombination and helps in bringing the

hybrid population which resembles the original parents iii) Linked genes dilute the effects of undesirable traits

Subject Eng Literature (The Tempest ndash William Shakespeare) Topic Essay Questions (EQ-3)Question No 3

Give a character sketch of CalibanAnswer

The character of Caliban has been wonderfully conceived by Shakespeare as the manifestation of all that is gross and earthy ndash a sort of creature of the earth as Ariel is a sort of creature of the air

Calibanrsquos Physical Appearanceo Caliban is lsquofreckledrsquo a lsquomisshapen knaversquo not honoured with human shape

o Prospero calls him lsquothou tortoisersquo (Act I Sc 2 Line 317) Trinculo stumbling upon him describes him as ldquoA strange fish hellip Legged like a man And his fins like armsrdquo He ldquosmells like a fishrdquo (Act II Sc 2 Line 25)

o Prospero also calls him a ldquobeastrdquo (Act IV Sc 1 Line 140) and ldquoThis misshapen knaverdquo (Act V Sc 1 Line 268)

o Further it appears that in addition to his physical deformity his spiritual inferiority is also suggested by Prosperorsquos claim that his birth resulted from the union between his mother the witch Sycorax and the devil

Calibanrsquos ParentageWhen the play opens Caliban is twenty four years of age having been born on the island twelve years before the coming of Prospero His mother was the foul witch Sycorax who was banished from Algiers for ldquomischiefs manifold and sorceries terrible to enter human hearingrdquo (Act I Sc 2 Line 264) and the father was the Devil himself Thus

Caliban is a monster of evil and brute nature ugly deformed and stinking

Calibanrsquos Savage and Malignant Natureo Caliban is entirely a creature of the earth ndash gross brutal and savage He regards himself as the rightful possessor

of the island and Prospero as a usurper

o In his young age he was on good terms with Prospero He had consented to be received by Prospero at his house and to be educated by him He has learnt human language only to curse his master whom he abhors

o His beastly nature soon breaks out and ends in a vicious attack on Miranda This opens the eye of Prospero who becomes severe to him and enforces his service by threats and violence

o Prospero uses him to make dams for fish to fetch firewood scraper trenches wash dishes and keep his cell clean

Calibanrsquos Hatred for ProsperoA profound hatred for Prospero has taken hold of Caliban It springs from a sense of his being dispossessed and ill-treated He would kill Prospero if he could but he knows the power of Prosperorsquos lsquobookrsquo Hence he transfers his allegiance to Stephano who seems like a god to him He also incites the two drunken associates to batter the skull of Prospero when he sleeps in the afternoon

Caliban Shows Considerable Intelligenceo He has learnt Prosperorsquos language

ldquoYou taught me language and my profit onrsquot (Act II Sc 2 Lines 86-89)Is I know how to curserdquo

o He is well aware of the futility of arguing with one who has more power than he has

ldquoI must obey his art is such power (Act I Sc 2 Lines 373-376)It would control my damrsquos god SetebosAnd make a vassal of himrdquo

o He realizes the importance of Prosperorsquos books

ldquoRemember (Act III Sc 2 Lines 89-92)First to possess his books for without themHersquos but a sot as I am nor hath notOne spirit to commandrdquo

o He knows the value of stealth when attacking the enemy

ldquoPray you tread softly that the blind mole may not (Act IV Sc 1 Lines 194-195)Hear a foot fall we now are near his cellrdquo

o Caliban has a better set of values than Stephano and Trinculo They are distracted from their plan by their greed for Prosperorsquos rich garments Only Caliban realizes that such a finery is unimportant

ldquoLeave it alone thou fool it is but trashrdquo (Act IV Sc 1 Lines 224)

Caliban is not a good judge of characterCaliban is not a good judge of character He decides for example that Stephano is a god because he dispenses lsquocelestial liquorrsquo (Act II Sc 2 Line 115) but then it must be remembered that he has only known his mother Sycorax Prospero Miranda and the spirits that torture him However he quickly discovers his error of judgementrdquo

ldquoWhat a thrice-double ass (Act V Sc 1 Lines 295-297)Was I to take this drunkard for a godAnd worship this dull foolrdquo

Calibanrsquos Imaginative NatureIf Caliban is sub-human in what has been said above he is human in the respect of the poetic side of his character He listens to music with rapture He tells of the beautiful dreams in which heaven rains treasures upon him and which upon waking he yearns to renew One of the most poetic passages in whole play is Calibanrsquos description of the island

to Stephano and Trinculo

ldquoBe not afeard The isle is full of noises (Act III Sc 2 Lines 135-143)Sounds and sweet airs that give delight and hurt notSometimes a thousand twangling instrumentsWill hum about mine ears and sometime voicesThat if I then had waked after long sleepWill make me sleep again and then in dreamingThe clouds methought would open and show richesReady to drop upon me that when I wakedI cried to dream againrdquo

Caliban - Less Ignoble Than Some OthersCalibanrsquos motive for murder is less dishonourable than that of Antonio and Sebastian They plan to kill Alonso to gain his power and wealth Caliban merely wants revenge and the return of lsquohisrsquo island

Conclusiono Calibanrsquos character is not portrayed very clearly in the play and hence we cannot decide whether he is a poor

savage being grossly maltreated by Prospero or whether he is evil and must therefore be kept in bondage or enslavement

o Caliban is contrasted with Ariel who is a spirit and thus swift and uninterested in physical activitieso Caliban is also contrasted with Prospero who is the all-powerful master of the island and of the destiny of all

those on the islando Caliban is also contrasted with civilized man showing him to be less evil than Antonio and Stephano and less

materialistic than Stephano and Trinculoo Caliban has suffered at the hands of Prospero and he has learnt to curse by listening to Prosperorsquos abuse He

certainly believes that Prospero has deprived him of his birthrighto Finally the character Caliban is thought to be one of Shakespearersquos masterpieces The complexity of the character

is reflected in the large volume of critical discussion that has grown around it

ECO ndash12 Topic-Forms of market

MonopolyMonopoly is a market structure in which there is a single seller there are no close substitutes for the commodity produced by the firm and there are barriers to entry Example Indian Railways which is operated under government of India Monopoly also implies absence of competitionFeatures of Monopoly Monopoly is characterized by1 Single Seller In monopoly there is only one firm producing the product The whole industry consists of this single firm Thus under monopoly there is no distinction between firm and industry Being the only firm there is significant control of the firm over supply and price Thus under monopoly buyers do not have the option of buying the commodity from any other seller They have to buy the product from the firm or they can go without the commodity This fact gives immense control to the monopolist over the market

2No Close Substitute There are no close substitutes of the product produced by the monopolist firm If there are close substitutes of the product in the market it implies presence of more than one firm and hence no monopoly In order to ensure a total of control over the market by the monopolist firm it is assumed that there are no close substitutes of the product

3 No Entry amp Exit Monopoly can only exist when there is strong barriers before a new firm to enter the market In fact once a monopoly firm starts producing the product no other firm can produce the same One reason for this is the ability of the

monopolist to produce the product at a lower cost than any new firm who thinks to enter the market If a new firm who knows that it cannot produce at a lower cost than the monopolist then that firm will never enter the market for fear of losing out in competition Similarly the monopolist who is operating for a long time may be enjoying reputation among its customers and is in a better position to use the situation in its own benefit A new firm has to take long time to achieve this and so may not be interested to enter the market

4 Price Maker Being the single seller of the product the monopolist has full control over the pricing of the product On the other hand if there is a large number of buyers in the market so no single buyer exercises any significant influence over price determination Thus it is a sellerrsquos market So monopoly firm is a price maker

5 Price Discrimination Having considerable control over the market on account of being single seller with no entry of other firms the monopolist can exercise policy of price discrimination it means that the monopolist can sell different quantities of the same product to a consumer at different price or same quantity to different consumers at different prices by adjudging the standard of living of the consumer

6 Shape of Demand Curve Since a monopolist has full control over the price therefore he can sell more by lowering the price This makes the demand curve downward sloping

Subject Ac-12 290620 Topic- retirement Model sumThe Balance Sheet of Rohit Nisha and Sunil who are partners in a firm sharing profits according to their capitals as on 31st March 2014 was as under

Liabilities Amount Assets Amount (Rs) (` Rs)

Creditors 25000 Machinery 40000Bills Payable 13000 Building 90000General Reserve 22000 Debtors 30000Capital Less Provision for Rohit 60000 Bad debts 1000

29000 Nisha 40000 Stocks 23000 Sunil 40000 140000 Cash at Bank 18000

200000 200000

On the date of Balance Sheet Nisha retired from the firm and following adjustments were made(i) Building is appreciated by 20(ii) Provision for bad debts is increased to 5 on Debtors(iii) Machinery is depreciated by 10(iv) Goodwill of the firm is valued at Rs 56000 and the retiring partnerrsquos share is adjusted

(v) The capital of the new firm is fixed at Rs120000 Prepare Revaluation Account Capital Accounts of the partner and Balance Sheet of the new firm after Nisharsquos retirement Revaluation AccountDr Cr

Particulars Amount Particulars Amount (`Rs) (Rs`)

Provision for Bad debt Ac 500 Building Ac 18000Machinery Ac 4000Profit transferred toCapital Accounts (3 2 2)Rohit 5786Nisha 3857Sunil 3857

13500

18000 18000

Capital Account

Dr Cr

Particulars Rohit Nisha Sunil Particulars Rohit Nisha Sunil (Rs`) (Rs`) (`Rs) (Rs`) (Rs`) (Rs`)

Sunilrsquos Capital ac 9600 mdash 6400 Balance bd 60000 40000 40000Bank - 66143 - General Reserve 9428 6286 6286Balance cd 72000 mdash 48000 Revaluation (Profi 5786 3857 3857 Rohitrsquos Capital Ac mdash 9600 mdash

Sunilrsquos Capital Ac 6400 Bank 6386 - 4257

81600 66143 54400 81600 66143 54400

Balance Sheet as at 31st March 2014

Liabilities Amount Assets Amount (Rs`) (Rs`)

Creditors 25000 Building 108000Bank overdraft 37500 Machinery 36000

Bills Payable 13000 Debtors 30000Capital Less ProvisionRohit 72000 for Bad debts 1500 28500Sunil 48000 120000 Stock 23000

195500 195500

Working Notes (i) (a) Profit sharing ratio is 60000 40000 40000 ie = 3 2 2(b) Gaining Ratio Rohit = 35 ndash 37 = 2135 ndash 1535 = 635Sunil = 25-27 = 1435 ndash 1035 = 435= 635 435= 6 4 = 3 2(c) Nisha Share of Goodwill = Rs 56000 times 27 = Rs16000Share of Goodwill in the gaining ratio by the existing partner ieRohit = Rs16000 times 35 = Rs 9600Sunil = Rs 16000 times 25 = Rs 6400

The journal entry isRohitrsquos Capital Ac Dr 9600Sunilrsquos Capital Ac Dr 6400 To Nisharsquos Capital Ac 16000(Share of Goodwill divided into gaining ratio)

  • 1 Static Friction
  • The frictional force that acts between the surfaces when they are at rest with respect to each other is called Static Friction
    • Static Friction Examples
      • 2 Sliding Friction
        • Examples Of Sliding Friction
          • 3 Rolling Friction
            • Examples Of Rolling Friction
              • Objects and Reasons of the Forest Conservation Act
Page 5:  · Web viewSubject . Topic . Summary . Execution . English 1 . Sounds of animals . Hens –cackle Horses –neigh Lions –roar Owls –hoots Snake –hiss. English 2 . Mother’s

Buddha may appear in it as a king God or an elephant There are several translations of these Jataka tales in different languages

Read the portion which has been explainedEng language

Ch 19 Adverbs Adverbs are words that tells us more about an action In other words adding something more to the verb For eg The cat ran quicklyhere the word ran is the verb as it is showing an action done by the cat and quickly is the adverb as it is telling how the cat ran

Refer to pg 68Do in the bookWarm upUnderline the words that tell you how an action happened

The cat ran quicklyThe man ran swiftlyIt hurriedly went into its hole The mouse sat safely in its houseThe cat squeaked loudly

MAT

HEM

ATIC

S

Ch 7

Frac

tions

A fraction is a number that stands for parts of a whole object or a collection of objects

Each fraction has two numbers separated by a rule(a) The number above the rule is called numerator(b) The number below the rule is called denominator

Example 7minusminusminusminusrarr Numerator

13minusminusminusminusrarr Denominator

Notes

(1) 05 is a fraction in which numerator is 0

(2) There is no fraction whose denominator is zero

(3) When the numerator and denominator are equal the fraction represents the whole or 1

Example 33=1and2

2 = 1

Exercise ndash 421 Read the following fractional numbers

(c) 411 (d)

815 (e)

2337

Solution

(c) 411

four by eleven

(d)8

15Eight by fifteen

(e) 2337

twenty three by thirty seven

2 Write the following fractional numbers(d) Three over eleven (e) Eighteen by twenty one (f) Four-seventhsSolution

(d) Three over eleven 311

(e) Eighteen by twenty one 1821

(f) Four-sevenths 47

3 Write the numerator of each of the following

(e) 09 (f)

56 (g)

311 (h)

617

Solution (e) 0 (f) 5 (g) 3 (h) 64 Write the denominator of each of the following

(e) 08 (f)

813 (g)

719 (h)

417

Solution (e) 8 (f) 13 (g) 19 (h) 17

5 Write the fractions whose numerators and denominators are given below

(a) Numerator = 4 and Denominator = 13 (b) Numerator = 6 and Denominator = 17(c) Numerator = 11 and Denominator = 16(d) Numerator = 0 and Denominator = 9

Numerator Denominator

Fraction

(a) 4 13 413

(b) 6 17 617

(c) 11 16 1116

(d) 0 9 09

6 Find the fractions in which the denominator is 3 more than the numerator69

74

811

49

1113

711

1411

1613

2023

Solution 69

811

2023

8 A book seller had 15 books He sold 8 books What fraction of the books did he sell

Solution Out of 15 books the book seller sold 8 books

there4 The fraction of the books he sold =8

15

10 A class has 35 students On Monday 34 students were present What fraction of the students was absent

Solution Total number of student = 35On Monday 34 students were presentthere4 The number of student absent on Monday = (35 ndash 34) = 1

So fraction of the students was = 135

Class IVSubject Topic Summary Execution English language

Verbs the -ing form

Meaning of verb A word used to describe an action is known as verb

The ndash ing form of a verb is a very important form for it is used in many different waysExampleJack is playing(Present continuous)Jack was playing with his friends yesterday ( Past continuous)Jack will be playing in a match tomorrow ( Future continuous)So we see that the ndash ing form of verb used to form the continuous tenses

The ndash ing form of verb can also be used as an adjectiveExampleYou should not disturb a sleeping dogThe kettle is full of boiling water

Say which of the ndash ingforms of verbs in the sentences given below have been used as adjectives and which to form continuous tenses ( solved exercises please follow this)

3Interesting ndash adjective

4Was eating ndash past continuous tense

5terrifying ndashadjective

6are helping is spreading ndash present continuous tense

7will be visiting ndashfuture continuous tense

8twittering ndash adjective

Social studies

Map reading A Map helps us to see the whole World continents countries cities and neighborhood They are of different sizes handy and can be rolled up folded or carried easily However the maps do not show the surface accurately This is because the map is flat whereas the Earth is round in shape Cartography or a map making is a study and practice representation of the

One word answers1 It helps us to see the whole world countries and cities ndash Map

2 A study and practice representation of the Earth on a flat surface ndash Cartography

Earth on a flat surface

Elements of a mapTo be able to read a map we must understand the elements of a map

DirectionsDirections are the basic guidelines that help us to locate places It is very important for us to know the correct directions East West North and South are the four Cardinal directions These directions help us to locate the places Beside these there are four sub directionsndash Northeast Southeast North West and South West Compass is an instrument that helps to locate the directions

3 The basic guidelines to help us to locate places ndash directions

4These four directions help us to locate the places ndash Cardinal directions

COMPUTER

Formatting in ms word

Q3) What do you mean by formattingAns) Formatting is a general arrangement of text in a document We can change the appearance of a document by using the features available in ms word We can use different fonts colours and styles in the textQ4) What is alignmentAns) Alignment of text is the way in which it is placed between the margins of a page Text can be aligned to the left side in the centre or to the right side of a pageQ5) what is fontAns) A font is a style of writing and typing A font provides specific textual appearance to the document You can change the size style or give various effects to a font

Hindi 2ndlang

ाला किहरण सभी धमlt ा मल धम ह दया और रणा मरी हर 0ीव अपन ही दकिनया म रहत ह हम

पराणी कयो उनो परशान रत ह ऐसा हम नही रना चाकिहए

0ानवरो स हम ए सीख मिमलती ह किस तरह हम एता म रहना चाकिहए

हर पराणी पयार ी भाा समझता ह अगर हम पयार ी उममीद रत ह तो कया 0त

0ानवर हमस उममीद नही रत कि हम भी उनह पयार द यह हानी ए ाल किहरण ी ह 0ो अपन

समह ा नततव रता था हमशा कया उलिचत ह यह सोचता था अचान

उन पर लिशारिरयो न हमला बोल दिदया जि0सम ाला किहरण परा गया कयोकि वह

दलभ ह और लिशारिरयो ी न0र उसी पर थी परत 0ानवरो ी एता ो दखर

रा0मार ो दया आ गई और उसन उनो छोड दिदया 0ानवर भी बईमान नही थी वह उनस अकसर मिमलन उन बाद म 0ाता रहता रा0ा ो धनयवाद रता अबोध म

पराणिणयो म एता तजञता अभी भी ह

शबदाथ तराई ndash पहाड आसपास नीच

ी भमिमवश- ल या परिरवारअदवभत- अनोखाचौननाndash सत सावधानओझलndash गायबकिवपणिRndash मसीबतसगठन- एताउममीदndash आशाातर दमिU ndashकिववशता

हिहस ndash हानी पहचान वालातवयndash फ0

दल ndash समहनततवndash सचालन रना

ओट ndash आडआतमसमपणndash अपन ो सौप दनाम- चप

हम उनस सीख लनी चाकिहएবইndashবাংলা সাহিতয পহিরচয়

পাঠndash১০লপndashহিবকেবকানকেFর যকেলকেবলাযলখকndashশশী-ষণ াশগNঅনশীলনীর পরকে4াততর

৯প পহিরবতG ন ককেরা -শরীর = শারীহিরক -ত = য-ৌহিতক সG ার = সG াহির সথায়ী = সথাহিয়তবহিবশবাস = হিবশবাসী া = যকো১০ বাকয রচনা ককেরা -আডডা = পাার বদধরা াতলায় আডডা যয়হিডবাহিজ = রাম া যকেক হিডবাহিজ যখকেয় পকেলাসG ার = বকেলহিল যকেল যর সG ারবকহিন = অঙক -ল করায় রীনা মাকেয়র কাকে বকহিন যখকেলাবহিদধ = বহিদধ াককেল উপায় য়হিনশহিত = হিনশহিত রাকেত রাজবাহিকেত ডাকাত পকেলা-য়ানক = পাাহি রাসতা বষটির পকের -য়ানক হিবপ জনক য়হিবশবাস = মানকেষর পরহিত হিবশবাস ারাকেনা পাপ

Science Ch ndash The Food We Eat

Living things need food to live to grow to stay strong and fit When we need food we feel hungry Food gives us energy to do work It also protects us from diseases and helps us to stay healthy Nutrients in food The food we eat contains many substances that are necessary for our body These substances are called nutrients that help us to grow well and stay healthy

Nutrients give us energy to study work and play

They help our body to grow and repair the damaged parts of our body

They also help our body to fight against diseases and remain healthy

Write T for True or F for False (pg no- 11)

1 Food contains nutrients that help us to grow True

2 Foods rich in carbohydrates are called body-building foods False

MAT

HEM

ATIC

S

Ch 9

Com

mon

Fra

ction

s

A fraction is a number that stands for parts of a whole object or a collection of objects

Each fraction has two numbers One is written above the other separated by a line The one above the line is called numerator and the one below the line is called denominator

Example 5minusminusminusminusrarr Numerator

11minusminusminusminusrarr Denominator

Exercise ndash 362 In following fill in the blanks

(b) 37 ___ is denominator ___ is

numerator

(c) 89 ___ is denominator ___ is

numerator

Solution

(b) 37 7 is denominator 3 is numerator

(c) 89 9 is denominator 8 is numerator

3 Write down the fractions whose numerators and denominators are given below in the bracket The first number stands for numerator and the second number standing for denominator

(25) 25

(311) 311

(416) 416

(712) 712

Class VSubject Topic Summary Execution

Science Ch ndash PlantReproduction

In pollination chapter we have learnt that the flowers change into fruits and the fruits bear seeds Now we are going to learn how this process takes place

FertilizationThe process of fusion of the male reproductive cell (male gamete) and female reproductive cell (female gamete) is known as fertilizationWhen a pollen grain reaches from the anther to a stigma it begins to grow and forms apollentube The Pollen tube then travels down through the style to enter an ovule inside the ovary On reaching the ovule male reproductive cell in the pollen grain unites with the egg cell present in the ovule

Books exercise

A) Tick the correct answer

1Which of the following do ovules change into after fertilization ndashseeds

2Which of the following is not a part of the seed ndash flower

3Which of the following condition is needed for germination of a seed ndash all ofthese

English language

Sentences phrases and

Solved exercisesSay which of the underlined groups of words are phrases and which clauses

clauses8 In her new clothes ndashphrasesAs pretty as a doll ndash phrases

9 looking sad and upset ndash phrasesHe had lost all the tickets for the Test Match ndash clauses

10 During the vacation now only a month away ndashphrases

11 too sweet and too hot ndashphrases

12 At the Olympic Games ndashPhrasesOf Laurel leaves ndash phrases

13 Who played the role of Hamlet ndashclauses

14 However fast ndash phrases

15 When the men fell asleep ndash clausesSocial studies

Indian Government

Lok Sabha (lower house) ndash It has 552 members Of these 530 membersrepresent States 20 members represent the union territories and two members represent the Anglo- Indian community All except the representatives of the Anglo-Indian community are elected by Indian citizens A person above the age of 25 can contest in the elections for Lok Sabha One term of Lok Sabha is for 5 yearsRajya Sabha (upper house) ndash Its members are elected by the MLAs or members of the legislative Assembly There are 250 members in the Rajya Sabha of which 12 are nominated by the President One term of Rajya Sabha is for 6 years Anyone above the age of 30 can be elected as a member of Rajya Sabha

ExecutivePresidentThe President is the head of the country in India He is elected by the MPs and the MLAs for a tenure of 5 years He appoints the Prime Minister and the Council of Ministers Prime MinisterThe party which wins the election forms the government and its leader is elected as the Prime Minister He is the chief advisor to the President The Council of Ministers assists the Prime Minister and is accountable for their roles For example the Education minister is responsible for the education system in our country

1 How many members are there in Lok SabhaAns 552 members

2 What is the term for Lok SabhaAns 5 years

3 How many members are there in the Rajya SabhaAns 250 members

4 Who is the head of our countryAns President

5 Who is the chief advisor to the PresidentAns Prime Minister

Book ndash GK

Ch ndash 1First in space

1 First living being into space in 1957 Ans Laika

Times 2 First person to go into space in 1961 Ans Yuri Gagarin

3 First woman to go into space in 1963 Ans ValentinaTereshkova4 First person ever to walk in space in 1965Ans alexei Leonov5 First person to land on the moon in 1959 Ans Neil Armstrong6 First Indian to go into space in 1984 Ans Rakesh Sharma 7 First Indian woman to go into space in 19978 Ans Kalpana Chawla9 First woman tourist in space in 2006

Ans Anusheh AnsariCOMPUTER

ALGORITHM AND FLOWCHART

Q) DRAW THE SYMBOLS USED IN A FLOWCHART WITH THEIR DESCRIPTIONS(IN EXAM IT CAN COME AS SHORT QUESTIONS ASKING INDIVIDUAL SYMBOLS FUNCTION)ANS)

MAT

HEM

ATIC

S

Ch 6

Com

mon

Fra

ction

s

Multiplication of FractionsA Multiply a fractional number by whole numberTo multiply a fractional number by whole number we multiply the numerator of the fractional number by the whole number and denominator of the fractional number by 1 The first product thus obtained is the numerator and the second product is the denominator of the required product

Exercise ndash 30Multiply

7 2027

times 9

Solution 2027

times 9 = 203 = 6

23

8 611

times11

Solution 611

times11 = 6

15 71

20times16

Solution 71

20times16 =

14120

times16

= 1415

times 4 = 141times 4

5 = 564

5 = 11245

B Multiplication of a fractional number by a fractional number To multiply a fractional number by a fractional number we multiply the numerator of the first fractional number by the numerator of the second fractional number and the denominator of the first fractional number by the denominator of the second fractional number The first product thus obtained is the numerator and the second product is the denominator of the required product

16 2712

times24

Solution 2712

times24 = 3112

times24

= 31times2 = 62

Exercise ndash 31

11 83

times 34

2

Solution 83

times 34 = 2

14 723

times2 25

4

Solution 723

times2 25 =

233

times 125 =

23times 45

= 925 = 18

25

15 1212

times1 13

2

Solution 1212

times1 13 =

252

times 43 =

25times 23

= 503 = 16

23

State the following statements are true or false

17 1912

times 239 = 1

Solution LHS = 1912

times 239

= 392

times 239 = 1 = RHS

[LHS = Left hand side amp RHS = Right hand side]

there4 1912

times 239 = 1 [True]

21 213

times2 13 = 4

19

Solution LHS = 213

times2 13 =

73

times 73

= 7times73times3 =

499 = 5

49

there4 LHS ne RHS

So 213

times2 13 = 4

19 [False]

23 23

times 45 =

2times 5+3 times43times 5

Solution

LHS = 23

times 45 =

2times 43 times5 =

815 again

RHS = 2times 5+3 times4

3times 5 = 10+12

15 = 2215

there4 LHS ne RHS So 23

times 45 =

2times 5+3times43times 5

[False]

25 23 of

13 =

29

Solution

LHS= 23 of

13 =

23 times

13 =

29 = RHS]

there4 23 of

13 =

29 [True]

Practice at HomeExercise ndash 31State the following statements are true or false

24 12 of 4 =

18

Class VISubject Topic Summary Execution

HISTORY AND CIVICS

Chapter 5The Mauryan Empire

DECLINE OF MAURYAN EMPIREDecline of Mauryan empire started after the death of Ashoka at around 232 BCThere are several reasons for break up of the empire1 Weak successor Emperors after Ashoka were

capable of handling vast and mighty Mauryan empire In 185BC the last Mauryan ruler Bri-hadrath was murdered by his Commander-in-Chief Pushyamitra Sunga

2 Provincial Revolts Due to weak central author-ity provincial chiefs of Kalinga and southern provinces revolted against emperor and freed themselves from Mauryan empire

3 Weakness of Economy Prosperity of Mauryan was based on solid economic activities which

ExercisesI Multiple choice questions-1 Chandragupta defeated Seleucus in the year ndashc) 305 BC2 Who killed the last Mauryan ruler Brihadrath b) Pushyamitra3 Which of the following was not a reason for the decline of the Mauryan empirec) Chandraguptarsquos weakness4 Ashoka invaded Kalinga in the year c) 261 BC

II Fill in the blanks1Chandragupta ascended the throne in 324

was taken care by early monarchs Later kings had neither ability nor interest in economic af-fairs That led to failure in tax collection As a result they failed to maintain a large army that were essential to keep empire intact

4 Greek Invasion Greeks freed north-western provinces from weak Mauryan monarchs and reestablished their authority

5 Ashokarsquos Policy some scholar opined that after Kalinga war Ashoka embraced Buddhism re-nounced the policy of war and disbanded the Army But this is partially true as there is no proper evidence of disbanding the army

Based on above points we can conclude that main reason for decline of Mauryan empire is weakness of Ashokarsquos successors Kunal Samprati Dasharath Salisuk all were weak kingsAt last in 185 BCPushyamitra Sunga killed king Brihadrath and established the Sunga dynasty

BC2 Bindusara was the son of Chandragupta and father of Ashoka

3 Pataliputra was administered by City Magistrate committess of 5 members each4 The Greek General Seleucus sent his ambassador Megasthenes to Chandraguptarsquos court5 Ashoka sent his son Prince Mahendra and daughter Sanghamitra to spread his Dhamma6 The Indian Rebublic has adopted the Lion Capital of Saranath Pillar as its national emblem 7 Pushyamitra killed the last Mauryan ruler Brihadrath and founded the Sunga dynasty

III Name the following

1The author of Arthashastra-Kautilya2 The ruler who founded the Mauryan dynasty-Chandragupta3 The author of Indika-Megasthenes 4 The officers who were appointed by Ashoka to spread Dhamma-Dhamma Mahamatras5 The general of Alexander whom Chandragupta defeated-Seleucus

V Match the columns1 Kautilya (c)2 Megasthenes (d)3 Pushyamitra (e)4 Brihadrath (b)5 Bindusara (a)

BENGALI(2ND

LANGUAGE)

পশপাহিখর -াষাসহিবনয় রায়কেচৌধরী

যলখক পহিরহিচহিত- পরখযাত সাহিহিতযক উকেপনদরহিককেশার রায়কেচৌধরীর পতর সহিবনয় রায়কেচৌধরী lsquoসকেFশrsquo পহিতরকার সকেb হিতহিন কত হিকেলন তার উকেdখকোয বই lsquoসহিবনয় রায়কেচৌধরীর রচনা সংগরrsquo

পরম হিকেনর পাঠ- lsquoপশপাহিখর হিক -াষাhelliphellip helliphelliphelliphellipপরসপরকেক জানাবার উপায়ও পশপাহিখরা যবশ জাকেনrsquoপরকেমই আমারা জাহিন -াষা হিক -াষা ল আমাকের মকেনর -াব পরকাশ করার জনয আমরা নানান ধরকেনর -হিb বা হিবকেশষ ধরকেনর আওয়াজ মকেখর মাধযকেম কহির অনযকেক যবাঝাকেনার জনয তাকেল এবার আমরা জাহিন পশপাহিখর -াষা হিক পশ পাহিখরা হিক কা বকেল যা পশপাহিখকেরও -াষা আকে তারা তাকের হিনজসব -াষায় কা বকেল মকেনর -াব পরকাশ ককের পশ পাহিখরা মানকেষর হিক হিক -াষা যবাকেঝ হিকনত তারা বলকেত পাকেরনা পরসপরকেক বহিঝকেয় যবার উপায় তারা জাকেননা তকেব তারা হিবকেশষ ককেয়কটি শকেবদর মাধযকেম তাকের মকেনর -াব বহিঝকেয় যয় হিক বহিদধ মান জীব ndashককর হিবাল বন মানষ যঘাা পর-হিত এরা মানকেষর যওয়া নাম শনকেল কান খাা ককের ndash নাম ধকের ডাককেল কাকে আকেস যমন - মরহিরা lsquoহিত ndashহিতrsquo ডাক শকেন আকেস াল lsquoঅ ndashর -র ডাক শকেন কাকে আকেস াহিত মাহকেতর কা শকেন চকেল ককররা মাহিলকেকর হকম পালন ককের সবসময় তাইকেতা ককরকেক পর- -কত পরানী বলা য় ককর আর হিবাল একের আওয়াজ তহিম লকষয করকেল বঝকেব ককররা যরকে যকেল lsquoযঘউ যঘউrsquo করকেত াকেক আবার কাকেল lsquoযকউ যকউrsquo ককের হিবাল সাধারণ lsquoমযাওrsquo বা lsquoহিমউrsquo ককের রা কেল lsquoওয়াওrsquo আওয়াকেজর মাধযকেম মকেনর -াব পরকাশ ককের একেতা যল পশকের কা পাহিখরাও -য় রা পরকাশ করার জনয হিবকেশষ ধরকেনর শবদ ককের হিবপকের সময় পশ পাহিখরা সবার আকে পরসপরকেক জানাবার উপায় তারা জাকেন বহকাল

১) পশপাহিখর -াষা কেলপর যলখক সমপকেকG হিক জাকেনা

উঃ- পরখযাত সাহিহিতযক উকেপনদরহিককেশার রায়কেচৌধরীর পতর সহিবনয় রায়কেচৌধরী lsquoসকেFশrsquo পহিতরকার সকেb হিতহিন কত হিকেলন তার উকেdখকোয বই lsquoসহিবনয় রায়কেচৌধরীর রচনা সংগরrsquo হিতহিন ারকেমাহিনয়াম এসরাজ পর-হিত বাযনতর বাজাকেত পারকেতন ানও জানকেতন হিতহিন যাকেIাকের জনয মজাার লপ কহিবতা হিলখকেতন

২) পশপাহিখ কেলপর মল-াব হিকউঃ- পশপাহিখকেরও -াষা আকে তারা তাকের হিনজসব -াষায় কা বকেল মকেনর -াব পরকাশ ককের পশ পাহিখরা মানকেষর হিক হিক -াষা যবাকেঝ হিকনত তারা বলকেত পাকেরনা পরসপরকেক বহিঝকেয় যবার উপায় তারা জাকেননা তকেব তারা হিবকেশষ ককেয়কটি শকেবদর মাধযকেম তাকের মকেনর -াব বহিঝকেয় যয় হিরউকেবন কযাসটং সাকেব হিতহিন চহিdশ বর বনযজনত যর সকেb যকেককেন হিতহিন বকেলকেন আমরা হি তাকের -াষা তাকের আব কায়া যমকেন চহিল তাকেল আর -কেয়র যকান কারণ াকেকনা আমরা একI -াকেলাকেবকেস যচষটা করকেল পশপাহিখকের সকেb -াব পাতাকেত পাহির

ধকের মানষ এই পশ পাহিখর -াষা হিনকেয় নানা রককেমর পরীকষা ককের আসকে এইরকম একজন হিরউকেবন কযাসটাং সাকেকেবর কা আমরা জানকেবাhelliphellip

Hindi 2nd

langमतर किनमनलिलखिखतपरशनोउRरदीजि0ए

) बढ वयलि` बचच ो कया हआ था ख) डॉकटर साहबन पाटc किस उददशय स रखी थी ग) ाल साप ो हाथ म लर लाश न कया किया घ) डॉकटर चडढा न बढ पतरो दखन स कयो मना र दिदया था ङ) भगत न लाश ो दखर कया हा

उRर ndash) उस बहत बखार थी और 4 दिदनो स आख भी नही खोला थाख) उन बट ी सालकिगरह थीग) ाल सापो हाथ म लर लाश न उसी गदन 0ोर स दबार पडी थीघ) डॉकटर चडढा न बढ वयलि` पतरो दखन स मना र दिदया कयोकि उनह गोलफ खलन 0ाना थाङ) लाश ो दखर हा कि नारायण चाहग तो आध घट म भया उठ 0ाएग

English literature

In the bazaars of Hyderabad- Sarojini Naidu

Through the poem In The Bazaars of Hyderabad Sarojini wanted to convey the message that India is rich in tradition and they donrsquot need the foreign products So she goes on to give a picture of a bazaar where traditional Indian products are rulingThe poem is in the form of questions and answers The poet asks the questions and the merchants answer them Through this technique she make the picture of the bazaar visible to us

Read the poem

PHYSICS FORCE Types of FrictionThere are three types of friction static sliding rolling Static sliding and rolling friction occur between solid surfaces

1 Static Friction The frictional force that acts between the surfaces when they are at rest with respect to each other is called Static FrictionStatic Friction Examples

Skiing against the snow Creating heat by rubbing both the hands

together Table lamp resting on the table

2 Sliding Friction The resistance that is created between any two objects when they are sliding against each other is called Sliding FrictionExamples Of Sliding Friction

Sliding of the block across the floor Two cards sliding against each other in a

deck

3 Rolling Friction The force which resists the motion of a ball or wheel is called Rolling Friction Is the weakest types of frictionExamples Of Rolling Friction

Rolling of the log on the ground Wheels of the moving vehicles

6What effect can a force produce on a body which is not allowed to move Ans - When a force is applied on a body which is not free to move it gets deformed i e the shape or size of the body changes7Give one example each to indicate that the application of a force

1 produces motion2 stops motion3 slows down motion4 changes the direction of motion5 deforms a body

Ans- 1 A car originally at rest when pushed

begins to move2 A moving bicycle is stopped by

applying the brakes3 The speed of a moving vehicle is

slowed down by applying the brakes4 A player kicks a moving football to

change its direction of motion5 On stretching a rubber string its

length increases

8State the effect produced by a force in the following cases (a) The sling of a rubber catapult is stretched(b) A man pushes a heavy cart(c) A player uses his stick to deflect the ball (d) A cyclist applies brakes(e) A spring is compressedAns- (a) The shape and size of catapult changes ie its length increases(b) The heavy cart begins to move(c) The direction of the ball changes(d) The speed of the moving cycle is slowed down(e) There is change in size and shape of spring

COMPUTER MS EXCEL 2013 -INTRODUCTION

UNDERSTANDING EXCEL STRUCTUREA SPREADSHEET IS A FILE THAT EXISTS OF CELLS IN ROWS AND COLUMNS AND CAN HELP ARRANGE CALCULATE AND SORT DATA DATA IN A SPREADSHEET CAN BE NUMERIC VALUES AS WELL AS TEXT

FORMULAS REFERENCES AND FUNCTIONS

WORKSHEETA WORKSHEET IS ALSO KNOWN AS SPREADSHEETIT IS A COLLECTION OF CELLS ON A SINGLE SHEET WHERE YOU KEEP AND CHANGE DATA

WORKBOOKWORKBOOK IS PMS EXCEL FILE IN WHICH THE DATA CAN BE STORED EACH WORKBOOK CAN CONTAIN MANY WORKSHEETS

ROWS AND COLUMNSIN MS EXCEL A ROW IS A GROUP OF CELLS THAT RUN FROM LEFT TO RIGHT OF A PAGEA COLUMN IS A GROUPING OF CELLS THAT RUN FROM THE TOP TO THE BOTTOM OF A PAGE

CELLTHE INTERSECTION POINT BETWEEN A ROW AND THE COLUMN IS CALLED A CELL WHICH IS THE BASIC STORAGE UNIT FOR DATA IN A SPREADSHEET EACH CELL HAS SPECIFIC ADDRESS WHICH IS THE COMBINATION OF THE COLUMN NAME FOLLOWED BY THE ROW NUMBER

CHEMISTRY Chapter ndash Common Laboratory Apparatus and equipments

Objective type questionFill in the blanks (a) Experiment and observation are the two important basics of chemistry(b) A porcelain dish is used for evaporation(c) A test tube holder is used to hold the test tube while-it is heated(d) Mortar and pestle is used for grinding and crushing solid substances into a powder(e) Glass apparatus is made of Pyrex or borosil glass

Class VIISubject Topic Summary Execution

Hindi 2ndlang

ए था राम( डॉ शरी परसाद)

सगकित ा परभाव मानव 0ीवन पर अवशय पडता ह

हमशा मनषय ो अचछो ी सगकित म रहना चाकिहए

शरषठ परो सग स मनषय चरिरतर ा शीघर ही उदय और किवास हो 0ाता

ह इसलिलए वयलि` ो सदा शरषठ परो ा ही सग रना चाकिहए

इसान अगर चाह वह सवय ो बदल भी सता ह

यह हानी राम ए बचच ी हवह गणिणत ी परीकषा म नल रत हए पडा 0ाता ह और उस अधयाप पडत ह और पछत ह यह कया र रह

हो तभी राम न उनी बइजजती ी

शबदाथब ndashहावा भलावाायवाहीndash ाम किनयम व ानन

ो दिदखानापरिरलिचतndash 0ाना पहचानाघटनाndashघबराहट

उलटा चोर ोतवाल ो डाटndashकिववndash भल बर ा जञानतवयndash म 0ो रना चाकिहएसगकितndash बरी सगत

किबलख नाndashरोना किनशचय रनाndash तय रना

फलndashपरिरणामकिनषालिसतndash बाहर किया हआपशचातापndashदख सपननndashधनी

ldquo हा आपी किहममत स हई नल रत पडन ीrdquo ऐसी बात ही किफर

किपता0ी न भी उस डाटा वह ाफी पशचाताप रन लगा बोला गलत दोसतो

ी सगकित म आ0 कितना अनथ र दिदया किफर उसन अधयाप स माफी मागन ी सची और किफर भी ऐसा

नही रगा यह परण भी लिलया

सोचndashकिहच एात-अला

বইndashবাংলা সাহিতয পহিরচয়

পাঠndash১৬লপndashস-য ও অস-যযলখকndashঈশবরচনদর হিবযাসারঅনশীলনীর পরকে4াততর

৬ অGকেলকেখা -ময়া = পশ হিশকার সহিtহিত = হিনকIবতu সbভরষট = লI হিনরীকষণ =

-াকেলা-াকেব যখাকতাঞজহিলপকেI = যজাাকেত৭ হিবপরীতশবদ -ঈষৎ times পরচর উৎকষট times হিনকষট তাশ times উৎফd তবহিদধ times

বহিদধীNপাহিপষঠ times পণযবান৮ পপহিরবতG ন ককেরা -পশ = পাশহিবক যকাপ = যকাহিপতহিসথর = হিসথরতাএকানত = ঐকাহিনতক পর-াত times পর-াতী

CHEMISTRY

Chapter ndashPhysical and Chemical Changes

Chemical ChangeA chemical change involves a change in chemical composition

Characteristics of Chemical changes 1 They are permanent changes2 They are irreversible changes 3 New substance formed4 A Chemical change involves a

change in its chemical properties

Pg-25Question 8What do you observe when1 water is boiled2 a piece of paper is burnt3 some ice cubes are kept in a glass tumbler4 solid ammonium chloride is heated5 an iron nail is kept in tap water for few days6 a spoon of sugar is heated in a pan7 lighted match stick is brought near the mouth of the test tube containing hydrogen gas8 quick lime is dissolved in water9 little amount of curd is added to a bowl containing warm milk and kept for five hours

10 Water is boiledOn boiling water changes into steam (gas) physical change

11 A piece of paper is burnton burning piece of paper produces carbon dioxide and ash is left behind Is a chemical change

12 some ice cubes are kept in a glass tumblerIce cubes (solid) turn into water

(liquid) only state changes (physical change)

13 Solid ammonium chloride is heatedSolid ammonium chloride on heating changes into vapors (change of state) is physical change

14 An iron nail is kept in tap water for few dayswe observe reddish brown coating on the nail called rust (entirely new substance) is chemical change

15 A spoon of sugar is heated in a panWhen a spoon of sugar is heated in a pan black (charred sugar) (carbon) is seen Is a chemical change

16 Lighted match stick is brought near the mouth of the test tube containing hydrogen gasWe observe that hydrogen bums at the mouth of test tube with blue flame and pop sound is heard It is chemical change

17 Quick lime is dissolved in waterThe following two observations will be observed (i) A hissing sound is observed(ii) The mixture starts boiling and lime water is obtained

18 Little amount of curd is added to a bowl containing warm milk and kept for five hoursWhen a little amount curd is added to a bowl containing warm milk and kept for five hours a permanent change occurredThe milk will change to curd On boiling water changes into steam (gas) physical change

GEOGRAPHY

ATMOSPHERE IMPACT OF GLOBAL WARMING The destructive impart of global warming is observed in various spheres of life and the environment Some of the points are outlined below1 High temperatures lead to high

evaporation rate and drying up of the soil and surface water This affects crop production The occurrence of droughts is aggravating the problem even further

2 The heat waves in summer months

Q1 Write some impact of global warmingA1 The impacts of global warming are as follows1 High temperatures lead to high

evaporate ion rate and drying up of the soil and surface water This affects crop production The occurrence of droughts is aggravating the problem even further

2 The heat waves in summer months lead to a greater number

lead to a greater number of deaths due to heat strokes

3 Forest fires become more frequent4 Tropical cyclones and hurricanes

become common5 Melting of glaciers takes place6 Polar ice caps are becoming thinner

and melting at an alarming rate due to global warming The loss of sea ice

7 Due to increase in sea surface temperature sea levels rise in coastal areas and cause submergence of several islands

WAYS TO REDUCE GLOBAL WARMINGFollowing steps can be taken We need to decrease emission of

green house gases by reducing the burning of fossil fuel such as coal and petroleum

By planting more trees to increase forest cover

The government should also distributes free saplings and organize afforestation programmes to spread awareness regarding the beneficial effects of trees

We should switch to eco-friendly cars and gadgets

Incandescent light bulbs should be replaced by CFL bulbs

We can save electricity and reduce global warming by turning off electrical gadgets such as lights fans air-conditioners television and computer when we do not to use them

Efforts should be made to hasten the development of green cities oreco cities These cities are urban areas around the world striving to lessen the environment a impacts of urbanization

By following the 3Rs-Reduce Recycle and Reuse strategy we can use natural resources for our growth as well as save them for the need of the future generations This is called sustainable development

of deaths due to heat strokes3 Forest fires become more

frequent4 Tropical cyclones and hurricanes

become common5 Melting of glaciers takes place

etc

Q2 How to reduce global warmingA2 Following steps can be taken to reduce global warmingaWe need to decrease emission of

green house gases by reducing the burning of fossil fuel such as coal and petroleum

bBy planting more trees to increase forest cover

c The government should also distributes free saplings and organize afforestation programmes to spread awareness regarding the beneficial effects of trees

dWe should witch to eco-friendly cars and gadgets

eIncandescent light bulbs should be replaced by CFL bulbs

f We can save electricity and reduce global warming by turning off electrical gadgets such as lights fans air-conditioners television and computer when we do not to use them

Q3 What do you mean by 3Rrsquos of resource planningA3 The 3Rs are

1 Reduce 2 Recycle and3 Reuse

Q4 What is Sustainable developmentA4 By following the 3Rs-Reluce Recycle and Reuse strategy we can use natural resources for our growth as well as save them for the need of the future generations This is called sustainable development

English Language

Prepositions A preposition is a word placed before a noun or a pronoun It helps to show how the person or thing denoted by the noun is related to something else in the sentence

Kinds of Prepositions

Simple Prepositions- simple preposition are one word Prepositions such as at by for in of off for from on out through till to up with before amidst towards beyond between over etc

Compound Prepositions ndash There are some words that are always used with fixed Prepositions to convey specific meaning

Example I was unable to meet you dueto a previous engagement ( On account of)Always maintain the queue instead of crowding at the counter ( In place of)

Participial PrepositionsmdashParticiple Prepositions are present or past participles of various verbs which together with a noun phrase or a clause function as prepositions Examples- barring concerning considering notwithstanding pending regarding respecting etc

Exercise A

1 Gauravs fever has come down since Friday He has been absent for a week now

2 The child sat between his father and mother among the parents of all his classmates

3 There are mosquitoes in the room They flew into the room when the door was open

4 My father was inside the drawing room when I was playing outside my house

5 You may sit beside me I will give you a drawing book and pencils besides a storybook

6 We went to the market in the morning and walked towards the riverfront in the evening

7 The child walked along the pavement and across the street safely

8 This table top is made of glass My breakfast fell off it in the morning

9 The pan is on the gas stove There are vegetables in it

10 We will wait for you at the bus top There are a lot of people in the hall

Subject ndash Biology Topic ndash Chapter - 3 Photosynthesis and respiration in plants Summary Execution

All living organism (Plants and animals) need food for energy and growth Green plants (autotrophy) prepare food for all living organisms Today we will discuss about the process photosynthesis And adaptations in a leaf to carry out photosynthesis

Q1What do you mean by photosynthesis and write its word equation The process by which green plants make food (glucose) from carbon dioxide and water

in the presence of sunlight and chlorophyll is called photosynthesis

Carbon dioxide + Water ( Sun light from Sun ) Glucose + Oxygen ( chlorophyll in green leaves )

Q2 What are the adaptations in a leaf to carry out photosynthesisi) Leaves are broad wide and flat for absorbing more light energyii) Presence of chlorophyll in chloroplasts to trap sunlightiii) Presence of stomata which allow carbon dioxide to enter the cell and oxygen to go

out iv) Network of veins ensures continuous supply of water and minerals to the leafv) Thin waxy cuticle protects the leaf without blocking the lightQ3 Draw and label structure of chloroplast

Class VIIISubject Topic Summary Execution

PHYSICS ENERGY Production of Hydro electricity

A hydroelectric dam converts the potential energy stored in a water reservoir behind a dam to mechanical energymdashmechanical energy is also known as kinetic energy As the water flows down through the dam its kinetic energy is used to turn a turbine

The generator converts the turbinersquos mechanical energy into electricity

This electric energy then goes through various transmission processes before it reaches you

Question 2

Fill in the blanks

(a) Work is said to be done by a forte only when the body moves

(b) Work done = Force x distance moved in direction of force

(c) The energy of a body is its capacity to do work

(d) The SI unit of energy is joule

(e) The potential energy is due to its state rest of position and kinetic energy of the body is due to its state of motion

(f) Gravitational potential energy U = mass times force of gravity on unit mass times height

(g) Kinetic energy = frac12 times mass times (speed)2

(h) Power P = work donetime taken

(i) The S I unit of power is watt

(j) IHP = 746 W

BIOLOGY Chapter -5 The endocrine system and adolescence

Today we will discuss about thelocation and functions of secreted hormones of adrenal and Pancreas

Q5 Write location hormone secreted main functions and deficiency diseases of pancreas and adrenal glands

Endocrine Glands

Location Hormones secreted

Functions and Deficiency Diseases

1Adrenal gland

2 Pancreas Gland

On the top of each kidney

In between stomach and small intestine

i)Adrenaline from adrenal medulla

ii)Cortisone from adrenal cortex

i) Insulin

ii) Glucagon

It helps a person deal with any kind of emergency situation or emotional stressIt increases the heart beat rate of respiration and blood pressure

a) It regulates carbohydrates protein and fat metabolism

b) It regulates the salt and water balance in the body

a) It changes excess glucose into glycogen

b) It stimulates the cells to burn extra glucose to provide heat amp energy

Less secretion causes diabetes mellitus

Excessive secretions causeinsulin shock

a) It stimulates the breakdown of glycogen into glucose

b) It increases the level of glucose in blood

History Traders to rulers The Battle of Buxar was fought on 22 October 1764 between the forces under the command of the British East India Company led by Hector Munro and the combined armies of Mir Qasim the Nawab of Bengal till 1763 Mir Jafar was made the Nawab of Bengal for a second time in 1763 by the Company just after the battle After being defeated in 4 battles in katwa and Udaynala the Nawab of Awadh Siraj id Daula and the Mughal emperor Shah Alam II accompanied by Raja Balwant Singh of Kashi made an alliance with Mir Qasim The battle was fought at Buxar a small fortified

Answer the following questions- Short note-Battle of BuxarHomework-learn

town within the territory of Bihar located on the banks of the Ganga river about 130 kilometres (81 mi) west of Patna it was a decisive victory for the British East India Company The war was brought to an end by the Treaty of Allahabad in 1765

EnglishLiterature

The west wind-John Mansfield

In the poem The West Wind by John Masefield the poet starts by describingwith very poetic imagery of birds how the west wind is different from other winds its a warm wind full of birds cries There is a touch of melancholy perhaps home-sickness as he describes how it brings tears too and memories from an old land He goes on to describe the restful pastoral beauty of the land where even the dead can lie in the green He then brings in voicesperhaps of family and friends calling him home as he is missing Aprils beautyThe voices then tempt him some more with idyllic images from home (white blossom young green cornrunning rabbitswarm sun) The voices seem to presume that the poets heart is sorrowful bruised and soreThe end of the poem sees the poet appear to make a decision he will go home as he has decided that is where he truly belongs

Write the synopsis of the following words

1 Daffodils- a tall yellow flower that grows in the spring

2 Orchards- a piece of land on which fruit trees are grown

3 Blossom- a flower or a mass of flowers especially on a fruit tree in spring

4 Thrushes- a bird5 Larks- a small brown bird that

makes a pleasant sound6 Bruised- an injury7 Aching- pain 8 Tread- to put your foot down

while you are walking9 Balm-10 May-11 Fluting-

(Write from the book in your copy)

MAT

HEM

ATIC

S

Ch 1

1Al

gebr

ic E

xpre

ssio

n

1 Constant A symbol which has fixed value is called a constant[eg 8 23 -15 radic3 etc]

2 VariableA symbol which does not have any fixed value but may be assigned value (values) according to the requirement is called variable or literal[eg x y p q etc]

3 TermsA term is a number (constant) a variable a combination (product or quotient) of numbers and variables[eg 7 x 5x etc]

4 Algebric expressionA single term or acombination of two or more terms connected by plus (+) or minus (-) sign forms an algebraic expression[eg 5-y 3x2-5x xy-6z+4 etc]

5 PolynomialAn algebraic expression which contains more than one term is called a polynomial (multinomial)[eg x2-5x 5y+xy+x2y etc]

6 Degree of polynomial(a) When the polynomial contains only one variable the highest power of the variable is the degree of the polynomialeg the degree of the polynomial of 4x-7x5+8 is 5(b) When the polynomial contains two or more variablesStep (i) Find the powers of the variables in each term (ii) The highest sum of the powers is taken to be the degree of the polynomialeg the degree of the polynomial 5x2y-4x3y5+6 is = 3+5 = 8Remember An algebraic expression is a polynomial if degree of each term used in it is a non-negative integer

Exercise ndash 11(A)

1 Separate the constants and variables from the following

-7 7+x 7x+yz radic5 radic xy 3 yz

8 45y -3x

Solution Constant Variables-7 radic5 7+x 7x+yz radic xy

3 yz8

45y -3x

2 Write the number of terms in each of the following polynomials(i) 5x2+3timesax (ii) axdivide4-7 (iii) ax-by+ytimesz (iv) 23+atimesbdivide2

Solution Polynomials Number of terms(i) 5x2+3timesax 2(ii) axdivide4-7 2(iii) ax-by+ytimesz 3(iv) 23+atimesbdivide2 2

4 Write the degree of the each polynomials(i) xy+7z (ii) x2-6x3+8 (iii) y-6y2+5y8 (iv) xyz-3 (vi) x5y7-8x3y8+10x4y4z4

Solution Polynomials Degree(i) xy+7z 2(ii) x2-6x3+8 3(iii) y-6y2+5y8 8(iv) xyz-3 3(vi)x5y7-8x3y8+10x4y4z4 12

5Write the coefficient of(i) ab in 7abx (iv) 8 in a2-8ax+a (v) 4xy in x2-4xy+y2

SolutionCoefficient

(i) ab in 7abx 7x(iv) 8 in a2-8ax+a -ax(v) 4xy in x2-4xy+y2 -1

7 CoefficientAny factor of an algebraic quantity is called the coefficient of the remaining quantityeg in the algebraic term 7xyz 7 is coefficient of xyz 7x is coefficient of yz and so on

8 Like term The terms having the same literal coefficient are called like terms and those having different literal coefficients are called unlike terms

eg (i) 5xyz 8xyz -6xyz and 23xyz are like

terms(ii) 7xy2 8x2yz and -15xyz2 are unlike terms

6 in 57xy2z3 write the coefficient of

(i) 5 (vii) 5xy2 (viii) 17yz (xi) 5xyz

Solution Coefficient

(i) 5 17

xy2z3

(vii) 5xy2 17z3

(viii) 17yz

5xyzsup2

(xi) 5xyz 17yz2

7 In polynomial given below separate the like terms(ii) y2z3 xy2z3 -58x2yz -4y2z3 -8xz3y2 3x2yz and 2z3y2

Solution y2z3 -4y2z3 2z3y2 are like terms

xy2z3 -8xz3y2 are like terms

-58x2yz 3x2yz are like terms

Class IXSubject Topic Summary Execution

Bengali (2nd language)

বাগzwnjধারাzwnj বা ধারা-বা ধারা ল হিবকেশষ পরকার বাক -হিb -াকেবর এক হিবকেশষ পরকাশরীহিত াকেক কতগকেলা কার সমষটির মকেধয এগহিলকেক বা ধারা বকেল আবার কতগকেলা শকেবদর বাধাধরা যকান রীহিত যনই য-াকেব চকেল আসকে যসই -াকেবই চকেল আসকে তখন যসই শবদগহিল খন একক -াকেব অG পরকাশ ককের তখন একের বা ধারা বকেল বা ধারার পরকেয়া -াষাকেক আরও সFর ককের যতাকেল

অকাল পকক(অপহিরনত বয়কেস পাকাহিম)-মাতর শ বর বয়কেস যমকেয়টির া মকেখর কা তাকেত অকালপককতা ধরা পকে

অককা পাওয়া( মারা াওয়া) ndash পকেকIমারটি পকেকIমারকেত হিকেয় বাসাতরীকের াকেত মার যখকেত যখকেত অককা যপল

অহি| পরীকষা ( কঠিন ও পরকত পরীকষা)- যকেলটির আজ ডাকতাহির যরজালট যবকেরাকেব এIাই তার জীবকেনর ব অহি| পরীকষা

অষটরমভা (ফাহিক) ndash রীতা মকেখই বকো বকো কা বকেল আর কাকেজর যবলায় অষটরমভা

অকমGার ধাী (অপাG) ndash সমনকেক হিনকেয় যকান ান কেব না ও একেকবাকেরই অকমGার ধাী

অকেনধর ষটি (অসাকেয়র সায়)- আহিশ বকেরর বকোর নাহিত ল অকেনধর ষটি তাকেক াা বকোর একম চকেল না

আকেককল গড়ম (তবহিদধ)- ার তহিম উপকার করকেল যসই যতামার হিবরকেদধ সাকষয হিকেয়কে শকেনই আমার আকেককল গড়ম

আষাকে লপ( অবাসতব লপ) ndashIাকা এখন যকেব না এIা বলকেলই ত এমন আষাকে লপ ফাার যকান রকার হিল না

Hindi- महायजञ ा इस हानी म लख न या बतान ा परयास किया ह कि किसी भी अचछ

2nd language

परसार(यशपाल ाय या पणय न ा फल अवशय मिमलता ह ोई भी परोपार अथवा पणय लिलए किया गया ाय बार नही 0ाता वह ए परार ा यजञ हए धनी सठ थ धम परायण और किवनमर सठ न आन ी यजञ किए थ और दान म न 0ान कितना धन दिदन दखिखयो म बात दिदया थादिदन पलट और सठ यहा गरीबी आ गई उन दिदनो यजञ बचन ी परथा थी सठ भी अपनी 0गह बचन लिलए डलपर ए सट यहा चलन ो तयार हए सठानी रासत लिलए रोटी पड म बाधर सठ ो द दी रासत म ए भख R ो दखर सठ न चारो रोटी उसो खिखला दी खर वह सठ यहा डलपर पहच तो उनी सठानी न उस महायजञ बचन ो हा यदिद बचन आए सठ न R ो रोटी खिखलान ो महायजञ नही समझा और वापस लौट आया घर आर शाम ो उसी घर म उस ए बडा ख0ाना मिमला 0ो उस दवारा किए गएrsquo महायजञrsquo ा परसार था

English language

Letter formal The heading the name and address of the person you are writing to must be included beneath your own address In formal letters ldquoblock stylerdquo of address is preferred

Subject complain in brief

Salutation If the person you are writing to is known to you you may begin ldquoDear MrrdquoOr ldquoDear Mrsrdquo In all other instances you should begin ldquoDear Sirrdquo or ldquoDear Madamrdquo Or ldquoSirsrdquo

The body A formal or business letter has four partsReference The letter should begin by referring to a letter you have received an advertisement or the reason that has prompted you to writeInformation In the second paragraph it is necessary to supply more detailed information that is related to the referencePurpose Here you must give the reason why you are writing the letter This must be stated clearly and ensure that it is relevant to the question that has been setConclusion round off the letter with some polite remarkThe subscription when a letter has begun with dear sir sirs Madam you should end with Yours faithfully or yours truly When however you address a person by name you must conclude with the words ldquoYours sincerelyrdquo

1 A park in your locality is slowly being used as a rubbish dump Write a letter to the Mayor of your city pointing out the nuisance and danger of this Request that action be taken to stop this immediately

Or2 You being a boarder ordered a set of lab manuals from a famous book shop in the town They sent you a wrong set of books Write a letter to the manager of the book shop

Chemistry Chapter-1 1)CHEMICAL FORMULA- Q What is the Significance of

L-2The Language of Chemistrybull Chemical Formula

Itrsquos a symbolic representation of a chemical substance eg ndash The formula of Sulphuric acid is H2SO4

2) Steps of writing Chemical Formula of a given substance-

1 Write the symbols of the constituent atoms or radicals side by side Keep the basic radical on LHS and acid radical on the RHS ( Na+Cl- )2 In case of a radical having more than one atom( compound radical) enclose the radical in a bracket eg (SO4-)3 Write the valencies of each radical on its right hand top4 If the valencies of the two radicals are divisible by a common factor then divide the valencies by the common factor5 Invert (criss-cross) the valency number ie write the valency of one atom below the second atom and vice versa 6 On interchanging if valency number is lsquoone the figure lsquoonersquo is never writtenFor Example- Compound -Calcium Nitrate1 Writing the symbols- Ca(NO3)2 Writing the valencies on their right hand top- Ca2(NO3)1

3 Valency numeral in simple ratio- Ca2(NO3)1

4 Criss-cross- Ca 2NO3 1

5 Writing the formula of the compound- Ca(NO3)2

Chemical formula

A The formula of a substance conveys the following information regarding a substance 1 The name of the substance (qualitative)2 The elements constituting the substance (qualitative)3 The number of various atoms present in a molecule of the substance (quantitative)4 Molecular weight of the substance and the relative weights of different elements present in it (qualitative)

Q What are the limitations of Chemical Formula

A The chemical formula suffers from the following limitations-I It fails to convey whether the elements in a molecule are present in the form of atoms or ionsFor example the formula KBr fails to tell us whether Potassium and Bromine are present in the form of ions II It does not tell anything about the binding force that holds atom in a molecule togetherIII It does not tell us about the arrangement of various atoms with respect to one another within the molecule

Q Examples of Some Chemicals with their Formula Chemical name and Common Name-

A Given in the class notesCommercial Studies

Joint Stock Company

Let us discuss about the demerits of Joint Stock CompanyDespite so many advantages it has got many disadvantages which are as follows

Difficulty in FormationDelay in Decision makingExcessive Government ControlLack of Secrecy

Company can be classified into several categories based on incorporation

QuestionExplain the demerits of Joint Stock CompanyAnswer) 1 Difficulty in Formation The legal requirements and formalities required to be completed are so many The cost involved is quite heavy It has to approach large number of people for its capital It cannot start its business unless certificate of incorporation has been obtained This is granted after a long time when all the formalities are completed

Chartered CompanyStatutory CompanyRegistered Company

Delay in Decision making In this form of organization decisions are not made by single individual All important decisions are taken by the Board of Directors Decision-making process is time-consuming So many opportunities may be costly because of delay in decision-making Promptness of decisions which is a common feature of sole trader ship and partnership is not found in a company

Excessive Government ControlA company and the management have to function well within the law and the provisions of Companies Act are quite elaborate and complex At every step it is necessary to comply with its provisions lest the company and the management should be penalized The penalties are quite heavy and in several cases officers in default can be punished with imprisonment This hampers the proper functioning of the company

Lack of Secrecy The management of companies remains in the hands of many persons Every important thing is discussed in the meetings of Board of Directors Hence secrets of the business cannot be maintained In case of sole proprietorship and partnership forms of organisation such secrecy is possible because a few persons are involved in the management

2 Define the following

Chartered Company- The crown in exercise of the royal prerogative has power to create a corporation by the grant of a charter to persons assenting to be incorporated Such companies or corporations are known as chartered companies Examples of this type of companies are Bank of England (1694) East India Company (1600) The powers and the nature of business of a chartered company are defined by the charter which incorporates it After the country attained independence these types of companies do not exist

in IndiaStatutory Company- A company may be incorporated by means of a special Act of the Parliament or any state legislature Such companies are called statutory companies Instances of statutory companies in India are Reserve Bank of India the Life Insurance Corporation of India the Food Corporation of India etc The provisions of the Companies Act 1956 apply to statutory companies except where the said provisions are inconsistent with the provisions of the Act creating them Statutory companies are mostly invested with compulsory powersRegistered companiesCompanies registered under the Companies Act 1956 or earlier Companies Acts are called registered companies Such companies come into existence when they are registered under the Companies Act and a certificate of incorporation is granted to them by the Registrar

Economics

Chapter-4Basic problems of Economy

Today let us discuss with the topic Production Possibility curve

QuestionExplain the concept of Production Possibility Curve with the help of diagram

Answer) Production Possibility curve is a locus of all possible combinations of two commodities which can be produced in a country with its given resources and technology

The above diagram shows that with the given resources and technology the economy can produce maximum either 5 thousand meters of cloth or 15 thousand quintals of wheat or any other combination of the two goods like B( 1 thousand meters of cloth and 14 thousand quintals of wheat C ( 2 thousands meters of cloth and 12 thousand quintals of wheat) etcProduction Possibility curve is also called production possibility boundary or frontier as it sets the maximum limit of what it is possible to produce with given resources

Geography

Rotationand Revolution

SUNrsquoS POSITION AND SEASONAL CHANGES EQUINOXES ndash SPRING AND AUTUMN

Q1 What is Spring EquinoxA1 On 21st March sunrays fall directly on the equator On that day

As the Equator divides the Earth into two equal halves the sun rays fall directly on the equator twice in a year Equinoxes means equal Spring EquinoxOn 21st March sunrays fall directly on the equator On that day the duration of day and night both are equal ( 12 hours day and 12 hours night) on every places located on equator This day is called as Spring EquinoxAutumn EquinoxOn 23rd September sunrays fall directly on the equator On that day the duration of day and night both are equal ( 12 hours day and 12 hours night) on every places located on equator This day is called as Autumn Equinox

SOLSTICES ndash SUMMER AND WINTERDue to inclination of the Earth on its axis and the apparent movement of the sun the sun rays fall directly on both tropics once in a year Solstice is a Latin word which mean ldquothe Sun standing stillrdquoSummer SolsticesAfter 21st March there is an apparent movement of the Sun to the north of the equator The apparent northward movement up to 21st June when the Sun appears overhead at the Tropic of Cancer (22frac12degN) The sun appears to stand still at this position and then moves southwards towards the equator This position of the Sun on 21st June is known as Summer Solstices On that day the duration of day and night both are equal ( 12 hours day and 12 hours night) on every places located on Tropic of Cancer (22frac12degN)Winter solstices The apparent southward movement of the Sun continues beyond the equator till 22nd

December On this day the Sun is overhead at the Tropic of Capricorn

the duration of day and night both are equal ( 12 hours day and 12 hours night) on every places located on equator This day is called as Spring Equinox

Q2 What do you mean by EquinoxA2 Equinoxes means equal It is use to explain the equal duration of day and night ( 12 hours day and 12 hours night) on the Earth

Q3 On which date the longest day in Tropic of CancerA3 21st June

Q4 What is the meaning of SolsticeA4 Solstice is a Latin word which mean ldquothe Sun standing stillrdquo

Q5 Which is the longest day in southern hemisphereA5 22nd December

Q6 On what date does the Arctic Circle experience the lsquoMidnight SunrsquoA6 On 21 June the Arctic Circle experiences the lsquoMidnight Sunrsquo

Q7 What is cause of Midnight Sun in NorwayA7 During the summer solstice (21 June) the North Pole is inclined towards the Sun Therefore the duration of sunlight or daytime increases from 12 hours at the Equator to 24 hours at the Arctic Circle and beyond Thatrsquos why The region beyond the Arctic Circle especially Norway is known as the Land of the Midnight Sun because there the Sun does not rise or set on 21 June

Q8 Match the column A with BA B

Summer Solstice 21st March

Autumn Equinox 23rd

September

Winter Solstice 21st June

(22frac12degS) This position of the Sun is referred to as the Winter Solstice because it marks the winter season in the Northern Hemisphere On that day the duration of day and night both are equal ( 12 hours day and 12 hours night) on every places located on Tropic of Capricorn (22frac12degS)SEASONS AND DURATION OF DAY AND NIGHT During the equinoxes all places on the Earth have 12 hours of day and 12 hours of night Due to the revolution of the Earth round the Sun on an inclined axis the duration of day and night varies according to seasons and the latitude of a placeDuring the summer solstice (21 June) the North Pole is inclined towards the Sun Therefore the duration of sunlight or daytime increases from 12 hours at the Equator to 24 hours at the Arctic Circle and beyondThe region beyond the Arctic Circle especially Norway is known as the Land of the Midnight Sun because there the Sun does not rise or set on 21 JuneAt the North Pole there will be six months of daylight The Sun will be seen always above the horizon at a low angle At 66degN 24 hours of sunlight can be seen only on 21 June Hammerfest in northern Norway is a place of tourist attraction for observing the phenomenon of the Midnight Sun This place has continuous daylight from 13 May to 29 July This place is easily accessible to tourists and has hotels and other facilities The view of the midnight Sun from here is enthrallingIn the Southern Hemisphere the duration of daylight decreases from 12 hours at the equator to 0 hours beyond the Antarctic Circle In the South Polar Region there is 24 hours of darkness The Sun is always below the horizon In the Southern Hemisphere which experiences winter the duration of night-time is longer than the duration of daylight

Spring Equinox 22nd

December

A8 A B

Summer Solstice 21st June

Autumn Equinox 23rd

September

Winter Solstice 22nd

December

Spring Equinox 21st March

During winter solstice (22 December) the South Pole is inclined towards the Sun The Southern Hemisphere experiences summer and the Northern Hemisphere has winter Therefore the duration of daylight or sunlight is greater in the Southern Hemisphere than in the Northern HemisphereThe duration of daylight increases from 12 hours at the equator to 24 hours beyond the Antarctic Circle The South Polar Region has 24 hours of sunlight for many days continuously At the South Pole there will be six months of sunlight The Sun will always be seen at a low angle above the horizon In the Northern Hemisphere the duration of daylight will decrease from 12 hours at the equator to 0 hours at the Arctic Circle There are 24 hours of darkness in the North Polar region The duration of night is greater than the duration of daylight as one move northwards from the Equator It is evident from the above table that the duration of daylight is 12 hours throughout the year at the equator only As one moves away from the equator the seasonal variations in the duration of daylight increase The seasonal variations in the duration of daylight are maximum at the Polar Regions

Subject Eng Literature (The Merchant of Venice ndash William Shakespeare)Topic Act II Scene 7 Lines 36 to 80 (End of scene ) [Students should read the original play and also the paraphrase provided]

Summary Questions amp AnswersThe Prince then examines the inscription on the silver casket which says ldquoWho chooseth me shall get as much as he deservesrdquo The Prince says that he deserves Portia more than anybody else because of his high rank his noble birth and his great wealth and power But then he argues that silver is ten times

(1) (Act II Sc 7 L 39-47)

From the four corners of the earth they come

To kiss this shrine this mortal breathing saint

The Hyrcanian deserts and the vasty wildsOf wide Arabia are as through-fares now

inferior to gold and therefore he cannot believe that the portrait of such a beautiful lady as Portia can be contained in the silver casket He decides to see the inscription on the golden casket before making his decision

The Prince goes to examine the inscription on the golden casket which says ldquoWho chooseth me shall get what many men desirerdquo The Prince believes that the whole world desires to possess Portia otherwise so many suitors would not have come from all corners of the world for winning Portia Some of them have come from the distant lands of Persia and Arabia The deserts of Persia (Hyrcanian deserts) and the boundless desolate lands of Arabia have been crossed by the Princes seeking the hand of Portia He contrasts this casket containing Portiarsquos portrait with the old English gold coin bearing the image of the archangel (angel of the highest rank) He goes on to remark that while the figure of the archangel is engraved (Insculped) upon the English coin the picture of Portia who is beautiful as an angel lies hidden inside one of the caskets namely the Golden Casket (Golden Bed)

On the basis of his assessment of the inscription on the golden casket the Prince decides to choose the golden casket He asks for the key and opens the golden casket only to find therein an empty human skull holding a roll of

For princes to come view fair PortiaThe watery kingdom whose ambitious headSpets in the face of heaven is no barTo stop the foreign spirits but they comeAs orsquoer a brook to see fair Portia

(i) Explain the occasion for the above mentioned speech

These are the comments of the Prince of Morocco after he reads the inscription on the golden casket His mental process is revealed to us in these words We find him debating within himself as to which casket he should choose

(ii) What light does the above speech throw on the personality of Prince of Morocco

From the above mentioned speech we come to know that the Prince of Morocco is keen to marry Portia He is the type of person who is easily taken away by outward appearance He is in love with Portia because of her beauty

(iii) What information can you gather about Portia from the above mentioned lines

The given speech shows that Portia is a very beautiful lady She must be possessed of good qualities because many suitors come to her place from all over the world with a desire to get married to her The Prince of Morocco is so impressed by her beauty that he calls her a saint According to him the whole world is desirous of having her

(iv) Elucidate the significance of the first two lines

In these lines the Prince of Morocco pays a compliment to Portia These lines show his admiration for her He says that people come from all parts of the world to see fair Portia

(v) Explain the meaning of the last four lines of the

passage

In these lines the Prince of Morocco says that even the vast oceans which throw a challenge at the sky are unable to prevent men from coming to Portiarsquos place to have a glimpse of her These lines are also a tribute to Portiarsquos beauty and good qualities Many men voyage across the ocean treating it as a mere stream to see the beautiful Portia

paper in which is written that whoever happens to be guided by the glitter of things is invariably deceived

On reading the scroll the Prince says that he is too sad at heart to speak a more formal farewell and leaves with his followers amidst a sound of trumpets

After the Prince of Morocco leaves Portia remarks that the Prince is a gentle fellow but she is rid of him May all persons of his nature make a similar choice

IMPORTANT PASSAGES EXPLAINED

(Act II Sc 7 L 39-43)From the four corners of the earth they come

To kiss this shrine this mortal breathing saintThe Hyrcanian deserts and the vasty wildsOf wide Arabia are as through-fares nowFor princes to come view fair Portia

Context

This passage occurs in Act II Scene 7 in The Merchant of Venice This is part of the speech made by the Prince of Morocco

(2)

(Act II Sc 7 L 48-53)

MOROCCO One of these three contains her heavenly pictureIst like that lead contains her

Twere damnation To think so base a thought it were too grossTo rib her cerecloth in the obscure graveOr shall I think in silver shes immurdBeing ten times undervalued to tried gold

(i) What meaning does the Prince of Morocco find out of the inscription of the golden casket What have Belmont and Portiarsquos house been called and why

The inscription on the golden casket is ldquoWho chooseth me shall gain what many men desirerdquo The Prince finds out that it means that the chooser of the golden casket will get Portia because many men desire her In fact the entire world desires her Because of the coming of many suitors to Belmont from different countries in order to win Portiarsquos hand Belmont has become a centre of pilgrimage and her house is the shrine where saintly Portia is installed

(ii) What does the Prince of Morocco do before making the final choice of the casket Which is the correct casket and who will win Portiarsquos hand

The Prince of Morocco surveys and analyses the inscriptions on the casket of lead silver and gold Before making the final choice like a very systematic and methodical person he once again considers the claims of the caskets The casket containing Portiarsquos picture is the correct casket and the person choosing it will win Portiarsquos hand

Explanation

While praising Portia the Prince of Morocco conceives Portia as a goddess whose image is placed inside one of the caskets Many suitors are coming from far and wide the north and the south the east and the west (Four corners) in order to try their luck Some of them have come from the distant land of Persia and Arabia The deserts of Persia (Hyrcanian deserts) and the boundless desolate lands of Arabia have been crossed by the Princes seeking the hand of Portia All this shows that Portia is indeed the most beautiful lady of the world

(iii) What does the Prince of Morocco say in his estimation while examining the motto on the silver casket What does he find in the golden casket

While examining the motto on the silver casket which says ldquoWho chooseth me shall get as much as he deservesrdquo Morocco says that in his own estimation he surely deserves Portia in all respects ndash rank birth wealth etc

He chooses the golden casket When he opens it he finds an empty human skull holding a scroll in which it is written that those who are attracted by the glittering outside of things are always deceived as Morocco has been deceived

(iv) What kind of nature does the Prince of Morocco have

The Prince of Morocco has a simple nature who does not look deeply into the inner meaning of things but is dazzled by the outward appearance of gold He is inclined to over-estimate his own value and does not realize that it is a duty to ldquogive and hazardrdquo To say that he will not hazard for lead shows that he misreads the true meaning of the inscription which is that he should be prepared to ldquohazard all he hathrdquo for Portia So his feeling is only one of fascination and romantic attraction

(v) Do you think that the lottery of the caskets is not a matter that will be determined by chance

In fact the lottery of the casket is not a matter that will be determined by mere chance but that it is a true test of character and of sincerity which is amply proved not only by Moroccorsquos choice but also by the arguments which he uses to help him in his choice

(Act II Sc 7 L 55-59)

They have in England

A coin that bears the figure of an angelStamped in gold but thats insculpd uponBut here an angel in a golden bedLies all within

Context

(3)

(Act II Sc 7 L 63-77)A carrion Death within whose empty eye

There is a written scroll Ill read the writing

All that glisters is not goldOften have you heard that toldMany a man his life hath soldBut my outside to beholdGilded tombs do worms infoldHad you been as wise as boldYoung in limbs in judgment oldYour answer had not been inscrolld

This passage occurs in Act II Scene 7 in The Merchant of Venice This is part of the speech made by the Prince of Morocco

Explanation

In this passage the Prince of Morocco bestows high praise on Portia whose hand he is seeking He contrasts this casket containing Portiarsquos portrait with the old English gold coin bearing the image of the archangel (angel of the highest rank) He goes on to remark that while the figure of the archangel is engraved (Insculped) upon the English coin the picture of Portia who is beautiful as an angel lies hidden inside one of the caskets namely the Golden Casket (Golden Bed) In the day of Elizabeth silver was ten times inferior in value to gold Therefore the Prince of Morocco believing that Portiarsquos portrait is contained in the Golden Casket decides to choose the Golden Casket

Fare you well your suit is coldCold indeed and labour lostThen farewell heat and welcome frostmdashPortia adieu I have too grievd a heartTo take a tedious leave Thus losers part

(i) What reward does the Prince of Morocco get after making a wrong choice of the Casket How does he feel

After making the wrong choice in selecting the casket of gold the Prince of Morocco as a reward earns a rebuke in the form of a scroll tucked in the empty eye-socket of a skull kept in the casket of gold The Prince is shocked and disappointed He becomes all the more sad and dejected when he reads the scroll which points to his foolishness in being misled by the appearance and outward show as indicative of its worth

(ii) How does the Prince respond after reading the scroll

After reading the scroll the Prince though upset accepts the result with good grace and decorum befitting a royal suitor and true sportsman He says that his love-suit is really cold otherwise he would have chosen correctly but now his efforts have been in vain So he bids farewell to Portia to the warmth and enthusiasm of love and welcomes the cold and bitterness of dejection and misery of life which lies ahead

(iii) What request does he make to Portia and why

After being failure in his mission he requests Portia to give him permission to leave at once because he is too sad to undergo the tediousness of a formal leave-taking He tells that it is the manner in which defeated persons part unceremoniously

(iv) Explain the following lines

ldquoAll that glisters is not goldOften have you heard that toldMany a man his life hath soldBut my outside to beholdGilded tombs do worms infoldrdquo

Mere glitter does not make a metal to be gold Man has often been warned against appearance but it has been of no use Many people have sacrificed their lives only to seek the outer appearance of gold Worms are found inside the gilded

monuments

Class XSubject Topic Summary Execution

Hindi 2ndlang

नया रासता भाग 6 मायाराम 0ी घर म धनी मल 0ी और उनी बटी सरिरता ी ही चचा बनी रहती थी अमिमत ो इसम ोई रलिच ना थी वह धनी घर ी लडी स शादी र सवय ो बचना नही चाहता था उसा भी सवाणिभमान ह ईशवर ी पा

स उस पास पस ी ोई मी नही थी अभी उसन फकटरी ही लगाई थी उसी समझ बाहर था कि उस घर वालो ा झाव पस ी तरफ कयो

ह उसन मा स सवाल किया कि मा तम सरिरता स मरी शादी कयो रना चाहती हो मा न उस समझाया कि वह दखन म बरी नही ह और किफर खानदान अचछा

ह वह ए शल गरहणी रप म घर सभाल सगी अमिमत न मा ो इस बात ा एहसास राया कि मीन सबध लिलए मना रन पर उस दिदल

पर कया बीती होगी मा और अमिमत ी लडी बार म ाफी बात हईमा ा झाव सरिरता ी तरफ था कयोकि वह घर पर अचछा दह0 लर आ रही

थी अमिमत न अपनी मौसी ी बरी हालत बार म बताया कि किस तरह वह बड घर ी खानदानी बटी लाई थी और आ0 उसी हालत कितनी खराब ह लाई थी बहकलब 0ाती ह और बचचो ो भी नही दखती ह बात चल ही रही

थी कि तभी ए ार बाहर आर री धनी मल0ी घर अदर आए और पीछ स डराइवर फल ी ए टोरी लर आया अदर आए और पीछ स

डराइवर ए टोरी फल ी लर आया अमिमत ो फल ी पटी बरी लग रही थी अमिमत न पछ लिलया यह फल कयो ल आए ह प इन सब ी कया

0ररत थी उनो न 0वाब दिदया कि 4 पटी शमीर स मगाए थ अमिमत ो या सनर करोध आ गया तभी उस किपता 0ी आ गए उन आत ही अमिमत उठर बाहर चला गया वहा वहा मा पास आर बठ गया और बोला

अभी रिरशता तय नही हआ और धनी मल 0ी धनी मल 0ी फल ी पटी लर चलआय मा न समझाया कि 0ब सबध 0ड 0ाता ह तो खाली हाथ नही

आत अमिमत न मा स हा कि तम सबन सरिरता ो इस घर म लान ी ठान रखी ह धनीमल 0ी उस दिदन सरिरता ो दखन ी तारीख तय रन आय थ

Commercial Studies

Banking Nowadays Bank provide easy and quick services through internet facilities methods of Banking is called internet bankingIn order to save the time and money involved in visiting Bank branches people increasingly prefer to have internet banking

There are different modes of doing internet banking or transferring money through online They areReal Time Gross Settlement (RTGS)National Electronic Fund Transfers (NEFT)

1

Question

1) Explain the term RTGS Write the features of RTGS

Answer)The acronym RTGS stands for Real Time Gross Settlement which may be defined as the continuous real time settlement of funds transfer individually on and order by order basis without netting lsquoReal timersquo may be defined as the processing of instructions at the time they are received rather than at some letter time lsquoGross settlementrsquo may be defined as the settlement of transfer instructions which occurs

individually

Features of RTGS1It is the continuous settlement of

funds transfer individually on an order by order basis

2RTGS facility is provided only by CBS core banking solution enabled Bank branches

3Amount charged from the customer for RTGS transactions vary from bank to bank

2) Explain the term NEFT Write the features of NEFT

Answer) National electronic funds transfer may be defined as a nationwide system that facilitates individuals Farms and copper operates to electronically transfer funds from any bank branch to any individual farm or corporate having an account with any other bank branch in the country

Features of NEFT2 Transfer can be made 7 times on

weekdays and 6 times on Saturday

3 NEFT cannot be used to receive foreign remittances

4 NEFT transaction takes place in batches

5 A bank branch must be NEFT enabled to become a part of NEFT fund transfer network

6 There is no maximum or minimum amount that can be transferred through NEFT when one bank has a bank account

English Language

CompositionEssay

A composition is an art of creating a piece of writing on any topic or subject It is the writing correctly beautifully and clearly in order to make some interesting reading Structure of the composition

Introduction ( you lay the foundation for your composition)

Body (it constitutes the main part of the essay)

Conclusion (final statement that leaves a lasting impression)

Kinds of essays1 The Narrative essay2 The descriptive essay3 The reflective essay4 The argumentative essay

Write a composition on any one of the following topics (350- 400 words)

1 Friendship Or2 The first day of your school

Subject Eng Literature (The Merchant of Venice ndash William Shakespeare)Topic Act V Scene 1 Lines 127 to 158 (Nerissa helliphellip The clerk will nersquoer wear hair onrsquos face that had it) [Students should read the original play and also the paraphrase given in the school prescribed textbook]

Summary Revision Questions o Soon thereafter Bassanio Gratiano

and Antonio arrive

o Bassanio tells Portia that he is feeling as if it is morning because of the presence of Portia who is shining like the sun When Antonio is introduced by Bassanio to Portia she tells Bassanio that he should be grateful to Antonio who took so much trouble on his account even to the extent of risking his life

o Nerissa starts quarrelling with Gratiano and demands that he show her the ring she had presented to him and which she had warned him not to lose She suspects that Gratiano must have presented the ring to some young woman and not to the lawyerrsquos clerk as he repeatedly says and assures

Answer the following questions to check your preparation of Act IV Scenes 1 and 2

You must attempt only after you have completed your preparation of Act IV The answers must be in complete sentences using textual evidence (with citation) when necessary

[It would be in your own interest to attempt the above questions honestly totally refraining from consulting your textbook or your notes during answering After completion you should correct the paper yourself consulting the textbooknotes etc and award marks as specified Please let me know the marks you scored through WhatsApp in the group or to my personal WhatsApp]

Act IV Scene 1 (each question carries 2 marks)

1 What did the Duke try to do for Antonio

2 Why does Shylock refuse to show mercy How does he justify his stance

3 Why does Antonio say he is ready to die 4 What information is contained in Bellariorsquos letter

5 Why does Portia (as Balthazar) assert that Shylock must show mercy How does he respond

6 What offers are made to Shylock to get him to spare Antonio How are they received

7 What does Antoniorsquos speech as he faces the prospect of Shylockrsquos knife tell you about his character

8 How do Bassanio and Gratiano react to the looming prospect of Antoniorsquos demise

9 How does Portia (as Balthazar) use the law to turn the tables on Shylock

10 What does the Duke decree should happen to Shylock Why What happens to Shylockrsquos estate

11 What does Portia ask Bassanio as payment for her ldquoservicesrdquo What is his initial response What makes him change his mind

Act IV Scene 2 (each question carries 1frac12 marks)

1 What does Gratiano bring to Portia (Balthazar)

2 What does Nerissa plan on getting from Gratiano What does Portiarsquos comment suggest about men

ECO-10 280620 Topic-Supply AnalysisSHIFTING OF SUPPLY

But if there is change in factors other than the price of the commodity then either more is supplied at the same price or less supplied at the same price In such cases the price of the commodity remains constant but there is a change in other factors like change in the price of inputs change in technology of production change in price of other related goods change in taxation policy of the government etc For example there is an improvement in the technology of production of the commodity in question It leads to decrease in per unit of cost production of the commodity The firm is willing to sell more quantity of the commodity at the same price So the supply other commodity increases at the same price This increase in supply is shown by rightward shift of supply curve On the other hand if the firm uses inferior technology of production the cost of production per unit of the commodity increases The firm is willing to sell less quantity at the same price So the supply of the commodity decreases at the same price This decrease in supply is shown by leftward shift of the supply curve The above cases of increase and decrease in supply can be shown with the help of the following figures

Y INCREASE OF SUPPLY Price (Rs) s

P A s1

B

s

X` O s1 X

q q1

Y` Quantity demanded (in units)

Y DECREASE IN SUPPLY s2

s

price (Rs)

C

p A

s2

s

X` o X

q2 q

Y` Quantity demanded ( in units)

Main factors causing increase in supply or rightward shift of supply Curve(i) Fall in the price of other related goods

(ii) Fall in the price of inputsfactors(iii) Use of better technology in production(iv) Decrease in the rate of excise duty by government(v) If the objective of producer changes from profit maximization to salesMaximization

Main factors causing decrease in supply or leftward shift of supply curve(i) Increase in the price of other related goods(ii) Rise in the price of inputsfactors(iii) Use of inferior technology in production(iv) Increase in the rate of excise duty by the government(v) If the objective

Subject - Biology Topic ndash Chapter mdash6 PhotosynthesisSummary Execution

Today we will know about photosynthesis and its stages

Q1 What do you mean by photosynthesis The process by which living plants containing chlorophyll produce food

substances from carbon-di- oxide and water by using light energy Sunlight

6CO2 +12 H2O----------------------- C6 H12O6 + 6H2O + 6O2

Chlorophyll

Q2 What are the importance of photosynthesis I) Food for all Green plants trap solar energy by photosynthesis

process and supply food and energy for all living organisms either directly or indirectly

Ii) Oxygen to breathe in by product of photosynthesis is oxygen which is essential for all living organisms respiration

Q3 Write about two main phases of photosynthesis A Light dependent phase This phase occur in grana of chloroplast I) The chlorophyll on exposure to light energy becomes activated by

absorbing photons Ii) The absorbed energy is used in splitting the water molecules (H2O)

into its two components (H+ and OH- ) and releasing electron s 2H2O------------------------- 4H+ + 4e- +O2

Energy of 4 photons This reaction is known as photolysis

End products are H+ and oxygen water

B Light independent (Dark ) phase The reactions in this phase require no light energy

Here CO2 combine with H+ and produce glucose

Class XI

Subject Topic Summary ExecutionEVS Chapter-4 Legal

regimes for sustainable development

Environmental legislationEnvironmental legislation is the collection of laws and regulations pertaining to air quality water quality the wilderness endangered wildlife and other environmental factors The act ensures that matters important to the environment are thoroughly

Learn -The Forest (Conservation) Act 1980

considered in any decisions made by federal agencies

The Forest (Conservation) Act 1980 The Forest (Conservation) Act 1980 an Act of the Parliament of India to provide for the conservation of forests and for matters connected therewith or ancillary or incidental thereto It was further amended in 1988 This law extends to the whole of IndiaObjects and Reasons of the Forest Conservation Act

Deforestation causes ecological imbalance and leads to environmental deterioration Deforestation had been taking place on a large scale in the country and it had caused widespread concern The act seeks to check upon deforestation and de-reservation of forests

Subject Eng Literature (The Tempest ndash William Shakespeare) Topic Act II Scene 1 Lines 314 to 329 (End of scene)

[Students should read the original play and also the paraphrase given in the school prescribed textbook]Summary Questions amp Answers

Conspiracy of Antonio and Sebastian (Contd)

o As they approach Ariel appears again and wakes up Gonzalo by singing a tune in his ear Alonso also wakes up and they see both Sebastian and Antonio with drawn swords On being caught off guard they make up a story saying that they had heard a bellowing of bulls or lions

o They then moved to another part of the island

o Ariel at once rushes to Prospero to inform him of this development

SUMMING-UP of ACT-2 SCENE-1

(i) Among the survivors Ferdinand is separated from the rest which results in the disconsolate grief of Alonso as he took him for dead

(ii) The villainy of Antonio is confirmed

(iii) The supremacy of Prosperorsquos magic which resulted in the failure of the human conspiracy

(1)

(Act II Sc 1 L 311-325)SEBASTIAN Whiles we stood here securing your repose

Even now we heard a hollow burst of bellowing Like bulls or rather lions Didt not wake youIt struck mine ear most terribly

ALONSO I heard nothingANTONIO O rsquotwas a din to fright a monsters ear

To make an earthquake Sure it was the roarOf a whole herd of lions

ALONSO Heard you this GonzaloGONZALO Upon mine honour sir I heard a humming

And that a strange one too which did awake meI shaked you sir and cried As mine eyes opened I saw their weapons drawn There was a noiseThats verily rsquoTis best we stand upon our guardOr that we quit this place Lets draw our weapons

(i) Why has Prospero sent Ariel to Gonzalo and Alonso What does Ariel do to awaken Gonzalo

Prospero has already come to know by his magic powers the danger which threatens Gonzalo who had been Prosperorsquos friend and so he sent Ariel to preserve the lives of both Gonzalo and Alonso Prospero does not want that his scheme should remain unfulfilled Ariel begins to sing a song in Gonzalorsquos ears to awaken him(ii) Who are ready to carry out their plan Who takes steps to stop them Why does Gonzalo feel surprised after being awakened

Sebastian and Antonio are ready to carry out their plans They are standing with their swords drawn to kill Alonso and

(iv) We see two sets of contrasting characters Gonzalo-Adrian against Antonio-Sebastian

(v) The grief that works in Alonso can be perceived to his repentance for his association in Antoniorsquos crime against Prospero

Gonzalo Ariel takes steps to stop them from carrying out their nefarious scheme When Gonzalo is awakened by the song sung by Ariel into his ears he (Gonzalo) feels surprised because he sees Sebastian and Antonio standing with their swords drawn(iii) What reason do Sebastian and Antonio tell of drawing their swords when they are suspected by Alonso and Gonzalo

When Sebastian and Antonio are seen with their swords drawn they are looked with suspicion by Gonzalo and Alonso At first Sebastian tells them that as they stood here to guard them during their sleep they heard only a little before a sudden loud noise very much like the roaring of bulls or more probably that of lions Then Antonio follows him saying that this was a noise so terrible as to frighten even a monsterrsquos ears and this noise could even have shaken the earth and it was surely like the roaring of a multitude of lions Then seeing the danger they have drawn their swords Perhaps after hearing the terrible noise they (Gonzalo and Alonso) woke up from their sound sleep

(iv) What does Gonzalo tell Alonso about the strange noise What did he see on opening his eyes Gonzalo tells Alonso that he did not hear the sound of roaring but he heard a humming sound which was strange and which woke him up After waking up he gave him (Alonso) a shaking and a loud cry On opening his eyes he saw these two gentlemen standing with their swords drawn(v) What does Gonzalo suggest

Gonzalo suggests that there was a noise indeed and of that he has no doubt at all and suggests that the best course for them would be to remain alert and vigilant against any possible danger to their lives or to leave this place and move to some other part of the island

Class XIISubject Topic Summary Execution

Commerce

Chapter- Management

Today we will discuss about LEVELS OF MANAGEMENT

Levels of management is a series or chain of managerial positions from top to bottom It helps individuals to know their authority responsibilities and superior-subordinate relations among themselves There are mainly three levels of Management TOP LEVEL MANAGEMENTMIDDLE LEVEL MANAGEMENTLOWER LEVEL MANAGEMENT

Top level managementIt consists of members at the highest level in the management hierarchy This level includes Board Of Directors Chief Executive Managing Directors Chairman President Vice President

Rolefunctions of the top levelmanagement1To analyse evaluate and deal

with theexternal environment2 To determine the objectives and

policies of the business3 To strive for welfare and survival

of business

4 To create an organisational Framework consisting of authority responsibility relationship

Middle level management Congress of members or groups who are concerned with implementation of the policies let down by the top managementThis level includes head of the department such as finance manager marketing manager branch and regional managers departmental and divisional heads plant superintendent etc

Role of functions of the middle level management

1 To interpret the policies framed by top management

2 To assign duties and responsibilities to lower level managers

3 To select and appoint employees for middle and supervisory level and evaluate their performance

4 To co-operate with other departments for smooth functioning

Operational or supervisory level managementIt refers to the group are members who are concerned with execution of the work They are also known as fast line managers This level includes supervisor 4 men Section Officer clerk Inspector etc

Role of functions of the lower level management1 To plan and execute day-to-

day operations2 To supervise and control the workers3 To arrange materials and

tools to start the process and make arrangements for training

4 Today present workers grievance and suggestions before the management and

ensure safe and proper working conditions in the factory

Business Studies

Staff Appraisal Chapter- 10 Today let us start with a new chapter

Staff Appraisal

Meaning of Performance Appraisal

Performance Appraisal is the systematic evaluation of the performance of employees and to understand the abilities of a person for further growth and developmentThe supervisors measure the pay of employees and compare it with targets and plansThe supervisor analyses the factors behind work performances of employeesThe employers are in position to guide the employees for a better performance

Objectives of Performance Appraisal

Following are the objectives of Performance Appraisal

To maintain records in order to determine compensation packages wage structure salaries raises etc

To identify the strengths and weaknesses of employees to place right men on right job

To maintain and assess the potential present in a person for further growth and development

To provide a feedback to employees regarding their performance and related status

To provide a feedback to employees regarding their performance and related status

Importance of Performance Appraisal

Performance appraisal provides important and useful information for the assessment of employees skill

knowledge ability and overall job performance The following are the points which indicate the importance of performance appraisal in an organization

1 Performance appraisal helps supervisors to assess the work performance of their subordinates

2 Performance appraisal helps to assess the training and development needs of employees

3 Performance appraisal provides grounds for employees to correct their mistakes and it also provides proper guidance and criticism for employees development4 Performance appraisal provides reward for better performance

5 Performance appraisal helps to improve the communication system of the organization

6 Performance appraisal evaluates whether human resource programs being implemented in the organization have been effective

7 Performance appraisal helps to prepare pay structure for each employee working in the organization

8 Performance appraisal helps to review the potentiality of employees so that their future capability is anticipated

Geography

DRIANAGE The SubarnarekhaThe Subarnarekha and the Brahmaniinterposed between the Ganga and the Mahanadi deltas drain an area of 19300 sq kmand 39033 sq km respectively The drainage basins of these streams are shared byJharkhand Odisha west Bengal and Chhattisgarh The Brahmani is known as southKoel in its upper reaches in Jharkhand

The NarmadaThe Narmada rises in the Amarkantak hills of MadhyaPradesh It flows towards the West in a rift valleyformed due to a geological fault The total length of it is 1300 km All the tributaries of the

Q1 Name the two westward flowing rivers in the peninsular plateauA1 Narmada and Tapi are the only westward flowing rivers of the peninsular plateau

Q2 Differentiate between east-flowing rivers and west-flowing riversA2

East-flowing rivers

West-flowing rivers

Narmada are very short inlength Most of its tributaries join the main streamright anglesThe Narmada basin covers parts of Madhya Pradesh and Gujarat

The Tapi The Tapi rises in the Satpura ranges in the Betul listrictof Madhya Pradesh It flows in a rift valley parallel tothe Narmada but it is much shorter in length It coversparts of Madhya Pradesh Gujarat and MaharashtraThe length is about 724 km

The Sabarmati and the MahiThe Sabarmati rises in the Aravali hills and flows south-south-westwards for a distance of 300 kilometres to the Arabian Sea The Sabarmatibasin extends over an area of 21674 sq km in Rajasthan and Gujarat The Mahi rises inthe east of Udaipur and drains an area of 34842 sq km lying in Madhya PradeshRajasthan and Gujarat It flows south-westwards for a distance of 533 km before it fallsinto the Gulf of Khambhat

The ChambalThe Chambal rises near Mhow in the Vindhya Range and flows towards the northgenerally in a gorge upto Kota Below Kota it turns to the north-east direction and afterreaching Pinahat it turns to the east and runs nearly parallel to the Yamuna beforejoining it in the southern part of the Etawah district in Uttar PradeshMajor Rivers of India with their basin area (Sqkm)

Himalayan System Indus 321290Ganga 861404

Brahmaputra 187110Indus System

Jhelum 34775Beas 20303

Ganga System Yamuna 366223Ghaghra 127950

Peninsular RiversNarmada 98796

Tapi 65145Mahanadi 141600

Subarnarekha 19300Sabarmati 21674

Mahi 34842Godavari 312812

Godavari Krishna Kaveri Mahanadi are the east-flowing rivers

Narmada Tapi west-flowing rivers

They fall into the Bay of Bengal

They fall into Arabian Sea

These rivers form big deltas

These rivers form comparativelysmall deltas

Catchment areas of these rivers are larger

Catchment areas of these rivers are smaller

Krishna 2589488Cauveri 87900

Subject ndashBiology Topic ndashChapter -5 Inheritance amp Variations Summary ExecutionToday we will discussabout linkage and its classification

LINKAGE The tendency of the genes located on the same chromosome to stay together is

hereditary transmission Linked genes the genes responsible for this Genes that exhibit the process of linkage locates in the same chromosome The distance between the linked genes in a chromosome determines the strength

of linkage i e genes that are located close to each other show stronger linkage than that are located far from each other

COMPLETE LINKAGE It is the type of linkage showed by the genes that are closely located or are tightly

linked with each other as they have no chance of separatingby crossing over These genes are always transmitted together to the same gamete and the same

offspring In such condition only parental or non cross over type of gametes are formedINCOMPLETE KINKAGE It is type of linkage showed by the genes that are distantly located orare loosely

linked with each other because they have chance of separating by crossing over

SIGNIFICANCE i) It helps in holding the parental character togetherii) It checks the appearance of new recombination and helps in bringing the

hybrid population which resembles the original parents iii) Linked genes dilute the effects of undesirable traits

Subject Eng Literature (The Tempest ndash William Shakespeare) Topic Essay Questions (EQ-3)Question No 3

Give a character sketch of CalibanAnswer

The character of Caliban has been wonderfully conceived by Shakespeare as the manifestation of all that is gross and earthy ndash a sort of creature of the earth as Ariel is a sort of creature of the air

Calibanrsquos Physical Appearanceo Caliban is lsquofreckledrsquo a lsquomisshapen knaversquo not honoured with human shape

o Prospero calls him lsquothou tortoisersquo (Act I Sc 2 Line 317) Trinculo stumbling upon him describes him as ldquoA strange fish hellip Legged like a man And his fins like armsrdquo He ldquosmells like a fishrdquo (Act II Sc 2 Line 25)

o Prospero also calls him a ldquobeastrdquo (Act IV Sc 1 Line 140) and ldquoThis misshapen knaverdquo (Act V Sc 1 Line 268)

o Further it appears that in addition to his physical deformity his spiritual inferiority is also suggested by Prosperorsquos claim that his birth resulted from the union between his mother the witch Sycorax and the devil

Calibanrsquos ParentageWhen the play opens Caliban is twenty four years of age having been born on the island twelve years before the coming of Prospero His mother was the foul witch Sycorax who was banished from Algiers for ldquomischiefs manifold and sorceries terrible to enter human hearingrdquo (Act I Sc 2 Line 264) and the father was the Devil himself Thus

Caliban is a monster of evil and brute nature ugly deformed and stinking

Calibanrsquos Savage and Malignant Natureo Caliban is entirely a creature of the earth ndash gross brutal and savage He regards himself as the rightful possessor

of the island and Prospero as a usurper

o In his young age he was on good terms with Prospero He had consented to be received by Prospero at his house and to be educated by him He has learnt human language only to curse his master whom he abhors

o His beastly nature soon breaks out and ends in a vicious attack on Miranda This opens the eye of Prospero who becomes severe to him and enforces his service by threats and violence

o Prospero uses him to make dams for fish to fetch firewood scraper trenches wash dishes and keep his cell clean

Calibanrsquos Hatred for ProsperoA profound hatred for Prospero has taken hold of Caliban It springs from a sense of his being dispossessed and ill-treated He would kill Prospero if he could but he knows the power of Prosperorsquos lsquobookrsquo Hence he transfers his allegiance to Stephano who seems like a god to him He also incites the two drunken associates to batter the skull of Prospero when he sleeps in the afternoon

Caliban Shows Considerable Intelligenceo He has learnt Prosperorsquos language

ldquoYou taught me language and my profit onrsquot (Act II Sc 2 Lines 86-89)Is I know how to curserdquo

o He is well aware of the futility of arguing with one who has more power than he has

ldquoI must obey his art is such power (Act I Sc 2 Lines 373-376)It would control my damrsquos god SetebosAnd make a vassal of himrdquo

o He realizes the importance of Prosperorsquos books

ldquoRemember (Act III Sc 2 Lines 89-92)First to possess his books for without themHersquos but a sot as I am nor hath notOne spirit to commandrdquo

o He knows the value of stealth when attacking the enemy

ldquoPray you tread softly that the blind mole may not (Act IV Sc 1 Lines 194-195)Hear a foot fall we now are near his cellrdquo

o Caliban has a better set of values than Stephano and Trinculo They are distracted from their plan by their greed for Prosperorsquos rich garments Only Caliban realizes that such a finery is unimportant

ldquoLeave it alone thou fool it is but trashrdquo (Act IV Sc 1 Lines 224)

Caliban is not a good judge of characterCaliban is not a good judge of character He decides for example that Stephano is a god because he dispenses lsquocelestial liquorrsquo (Act II Sc 2 Line 115) but then it must be remembered that he has only known his mother Sycorax Prospero Miranda and the spirits that torture him However he quickly discovers his error of judgementrdquo

ldquoWhat a thrice-double ass (Act V Sc 1 Lines 295-297)Was I to take this drunkard for a godAnd worship this dull foolrdquo

Calibanrsquos Imaginative NatureIf Caliban is sub-human in what has been said above he is human in the respect of the poetic side of his character He listens to music with rapture He tells of the beautiful dreams in which heaven rains treasures upon him and which upon waking he yearns to renew One of the most poetic passages in whole play is Calibanrsquos description of the island

to Stephano and Trinculo

ldquoBe not afeard The isle is full of noises (Act III Sc 2 Lines 135-143)Sounds and sweet airs that give delight and hurt notSometimes a thousand twangling instrumentsWill hum about mine ears and sometime voicesThat if I then had waked after long sleepWill make me sleep again and then in dreamingThe clouds methought would open and show richesReady to drop upon me that when I wakedI cried to dream againrdquo

Caliban - Less Ignoble Than Some OthersCalibanrsquos motive for murder is less dishonourable than that of Antonio and Sebastian They plan to kill Alonso to gain his power and wealth Caliban merely wants revenge and the return of lsquohisrsquo island

Conclusiono Calibanrsquos character is not portrayed very clearly in the play and hence we cannot decide whether he is a poor

savage being grossly maltreated by Prospero or whether he is evil and must therefore be kept in bondage or enslavement

o Caliban is contrasted with Ariel who is a spirit and thus swift and uninterested in physical activitieso Caliban is also contrasted with Prospero who is the all-powerful master of the island and of the destiny of all

those on the islando Caliban is also contrasted with civilized man showing him to be less evil than Antonio and Stephano and less

materialistic than Stephano and Trinculoo Caliban has suffered at the hands of Prospero and he has learnt to curse by listening to Prosperorsquos abuse He

certainly believes that Prospero has deprived him of his birthrighto Finally the character Caliban is thought to be one of Shakespearersquos masterpieces The complexity of the character

is reflected in the large volume of critical discussion that has grown around it

ECO ndash12 Topic-Forms of market

MonopolyMonopoly is a market structure in which there is a single seller there are no close substitutes for the commodity produced by the firm and there are barriers to entry Example Indian Railways which is operated under government of India Monopoly also implies absence of competitionFeatures of Monopoly Monopoly is characterized by1 Single Seller In monopoly there is only one firm producing the product The whole industry consists of this single firm Thus under monopoly there is no distinction between firm and industry Being the only firm there is significant control of the firm over supply and price Thus under monopoly buyers do not have the option of buying the commodity from any other seller They have to buy the product from the firm or they can go without the commodity This fact gives immense control to the monopolist over the market

2No Close Substitute There are no close substitutes of the product produced by the monopolist firm If there are close substitutes of the product in the market it implies presence of more than one firm and hence no monopoly In order to ensure a total of control over the market by the monopolist firm it is assumed that there are no close substitutes of the product

3 No Entry amp Exit Monopoly can only exist when there is strong barriers before a new firm to enter the market In fact once a monopoly firm starts producing the product no other firm can produce the same One reason for this is the ability of the

monopolist to produce the product at a lower cost than any new firm who thinks to enter the market If a new firm who knows that it cannot produce at a lower cost than the monopolist then that firm will never enter the market for fear of losing out in competition Similarly the monopolist who is operating for a long time may be enjoying reputation among its customers and is in a better position to use the situation in its own benefit A new firm has to take long time to achieve this and so may not be interested to enter the market

4 Price Maker Being the single seller of the product the monopolist has full control over the pricing of the product On the other hand if there is a large number of buyers in the market so no single buyer exercises any significant influence over price determination Thus it is a sellerrsquos market So monopoly firm is a price maker

5 Price Discrimination Having considerable control over the market on account of being single seller with no entry of other firms the monopolist can exercise policy of price discrimination it means that the monopolist can sell different quantities of the same product to a consumer at different price or same quantity to different consumers at different prices by adjudging the standard of living of the consumer

6 Shape of Demand Curve Since a monopolist has full control over the price therefore he can sell more by lowering the price This makes the demand curve downward sloping

Subject Ac-12 290620 Topic- retirement Model sumThe Balance Sheet of Rohit Nisha and Sunil who are partners in a firm sharing profits according to their capitals as on 31st March 2014 was as under

Liabilities Amount Assets Amount (Rs) (` Rs)

Creditors 25000 Machinery 40000Bills Payable 13000 Building 90000General Reserve 22000 Debtors 30000Capital Less Provision for Rohit 60000 Bad debts 1000

29000 Nisha 40000 Stocks 23000 Sunil 40000 140000 Cash at Bank 18000

200000 200000

On the date of Balance Sheet Nisha retired from the firm and following adjustments were made(i) Building is appreciated by 20(ii) Provision for bad debts is increased to 5 on Debtors(iii) Machinery is depreciated by 10(iv) Goodwill of the firm is valued at Rs 56000 and the retiring partnerrsquos share is adjusted

(v) The capital of the new firm is fixed at Rs120000 Prepare Revaluation Account Capital Accounts of the partner and Balance Sheet of the new firm after Nisharsquos retirement Revaluation AccountDr Cr

Particulars Amount Particulars Amount (`Rs) (Rs`)

Provision for Bad debt Ac 500 Building Ac 18000Machinery Ac 4000Profit transferred toCapital Accounts (3 2 2)Rohit 5786Nisha 3857Sunil 3857

13500

18000 18000

Capital Account

Dr Cr

Particulars Rohit Nisha Sunil Particulars Rohit Nisha Sunil (Rs`) (Rs`) (`Rs) (Rs`) (Rs`) (Rs`)

Sunilrsquos Capital ac 9600 mdash 6400 Balance bd 60000 40000 40000Bank - 66143 - General Reserve 9428 6286 6286Balance cd 72000 mdash 48000 Revaluation (Profi 5786 3857 3857 Rohitrsquos Capital Ac mdash 9600 mdash

Sunilrsquos Capital Ac 6400 Bank 6386 - 4257

81600 66143 54400 81600 66143 54400

Balance Sheet as at 31st March 2014

Liabilities Amount Assets Amount (Rs`) (Rs`)

Creditors 25000 Building 108000Bank overdraft 37500 Machinery 36000

Bills Payable 13000 Debtors 30000Capital Less ProvisionRohit 72000 for Bad debts 1500 28500Sunil 48000 120000 Stock 23000

195500 195500

Working Notes (i) (a) Profit sharing ratio is 60000 40000 40000 ie = 3 2 2(b) Gaining Ratio Rohit = 35 ndash 37 = 2135 ndash 1535 = 635Sunil = 25-27 = 1435 ndash 1035 = 435= 635 435= 6 4 = 3 2(c) Nisha Share of Goodwill = Rs 56000 times 27 = Rs16000Share of Goodwill in the gaining ratio by the existing partner ieRohit = Rs16000 times 35 = Rs 9600Sunil = Rs 16000 times 25 = Rs 6400

The journal entry isRohitrsquos Capital Ac Dr 9600Sunilrsquos Capital Ac Dr 6400 To Nisharsquos Capital Ac 16000(Share of Goodwill divided into gaining ratio)

  • 1 Static Friction
  • The frictional force that acts between the surfaces when they are at rest with respect to each other is called Static Friction
    • Static Friction Examples
      • 2 Sliding Friction
        • Examples Of Sliding Friction
          • 3 Rolling Friction
            • Examples Of Rolling Friction
              • Objects and Reasons of the Forest Conservation Act
Page 6:  · Web viewSubject . Topic . Summary . Execution . English 1 . Sounds of animals . Hens –cackle Horses –neigh Lions –roar Owls –hoots Snake –hiss. English 2 . Mother’s

MAT

HEM

ATIC

S

Ch 7

Frac

tions

A fraction is a number that stands for parts of a whole object or a collection of objects

Each fraction has two numbers separated by a rule(a) The number above the rule is called numerator(b) The number below the rule is called denominator

Example 7minusminusminusminusrarr Numerator

13minusminusminusminusrarr Denominator

Notes

(1) 05 is a fraction in which numerator is 0

(2) There is no fraction whose denominator is zero

(3) When the numerator and denominator are equal the fraction represents the whole or 1

Example 33=1and2

2 = 1

Exercise ndash 421 Read the following fractional numbers

(c) 411 (d)

815 (e)

2337

Solution

(c) 411

four by eleven

(d)8

15Eight by fifteen

(e) 2337

twenty three by thirty seven

2 Write the following fractional numbers(d) Three over eleven (e) Eighteen by twenty one (f) Four-seventhsSolution

(d) Three over eleven 311

(e) Eighteen by twenty one 1821

(f) Four-sevenths 47

3 Write the numerator of each of the following

(e) 09 (f)

56 (g)

311 (h)

617

Solution (e) 0 (f) 5 (g) 3 (h) 64 Write the denominator of each of the following

(e) 08 (f)

813 (g)

719 (h)

417

Solution (e) 8 (f) 13 (g) 19 (h) 17

5 Write the fractions whose numerators and denominators are given below

(a) Numerator = 4 and Denominator = 13 (b) Numerator = 6 and Denominator = 17(c) Numerator = 11 and Denominator = 16(d) Numerator = 0 and Denominator = 9

Numerator Denominator

Fraction

(a) 4 13 413

(b) 6 17 617

(c) 11 16 1116

(d) 0 9 09

6 Find the fractions in which the denominator is 3 more than the numerator69

74

811

49

1113

711

1411

1613

2023

Solution 69

811

2023

8 A book seller had 15 books He sold 8 books What fraction of the books did he sell

Solution Out of 15 books the book seller sold 8 books

there4 The fraction of the books he sold =8

15

10 A class has 35 students On Monday 34 students were present What fraction of the students was absent

Solution Total number of student = 35On Monday 34 students were presentthere4 The number of student absent on Monday = (35 ndash 34) = 1

So fraction of the students was = 135

Class IVSubject Topic Summary Execution English language

Verbs the -ing form

Meaning of verb A word used to describe an action is known as verb

The ndash ing form of a verb is a very important form for it is used in many different waysExampleJack is playing(Present continuous)Jack was playing with his friends yesterday ( Past continuous)Jack will be playing in a match tomorrow ( Future continuous)So we see that the ndash ing form of verb used to form the continuous tenses

The ndash ing form of verb can also be used as an adjectiveExampleYou should not disturb a sleeping dogThe kettle is full of boiling water

Say which of the ndash ingforms of verbs in the sentences given below have been used as adjectives and which to form continuous tenses ( solved exercises please follow this)

3Interesting ndash adjective

4Was eating ndash past continuous tense

5terrifying ndashadjective

6are helping is spreading ndash present continuous tense

7will be visiting ndashfuture continuous tense

8twittering ndash adjective

Social studies

Map reading A Map helps us to see the whole World continents countries cities and neighborhood They are of different sizes handy and can be rolled up folded or carried easily However the maps do not show the surface accurately This is because the map is flat whereas the Earth is round in shape Cartography or a map making is a study and practice representation of the

One word answers1 It helps us to see the whole world countries and cities ndash Map

2 A study and practice representation of the Earth on a flat surface ndash Cartography

Earth on a flat surface

Elements of a mapTo be able to read a map we must understand the elements of a map

DirectionsDirections are the basic guidelines that help us to locate places It is very important for us to know the correct directions East West North and South are the four Cardinal directions These directions help us to locate the places Beside these there are four sub directionsndash Northeast Southeast North West and South West Compass is an instrument that helps to locate the directions

3 The basic guidelines to help us to locate places ndash directions

4These four directions help us to locate the places ndash Cardinal directions

COMPUTER

Formatting in ms word

Q3) What do you mean by formattingAns) Formatting is a general arrangement of text in a document We can change the appearance of a document by using the features available in ms word We can use different fonts colours and styles in the textQ4) What is alignmentAns) Alignment of text is the way in which it is placed between the margins of a page Text can be aligned to the left side in the centre or to the right side of a pageQ5) what is fontAns) A font is a style of writing and typing A font provides specific textual appearance to the document You can change the size style or give various effects to a font

Hindi 2ndlang

ाला किहरण सभी धमlt ा मल धम ह दया और रणा मरी हर 0ीव अपन ही दकिनया म रहत ह हम

पराणी कयो उनो परशान रत ह ऐसा हम नही रना चाकिहए

0ानवरो स हम ए सीख मिमलती ह किस तरह हम एता म रहना चाकिहए

हर पराणी पयार ी भाा समझता ह अगर हम पयार ी उममीद रत ह तो कया 0त

0ानवर हमस उममीद नही रत कि हम भी उनह पयार द यह हानी ए ाल किहरण ी ह 0ो अपन

समह ा नततव रता था हमशा कया उलिचत ह यह सोचता था अचान

उन पर लिशारिरयो न हमला बोल दिदया जि0सम ाला किहरण परा गया कयोकि वह

दलभ ह और लिशारिरयो ी न0र उसी पर थी परत 0ानवरो ी एता ो दखर

रा0मार ो दया आ गई और उसन उनो छोड दिदया 0ानवर भी बईमान नही थी वह उनस अकसर मिमलन उन बाद म 0ाता रहता रा0ा ो धनयवाद रता अबोध म

पराणिणयो म एता तजञता अभी भी ह

शबदाथ तराई ndash पहाड आसपास नीच

ी भमिमवश- ल या परिरवारअदवभत- अनोखाचौननाndash सत सावधानओझलndash गायबकिवपणिRndash मसीबतसगठन- एताउममीदndash आशाातर दमिU ndashकिववशता

हिहस ndash हानी पहचान वालातवयndash फ0

दल ndash समहनततवndash सचालन रना

ओट ndash आडआतमसमपणndash अपन ो सौप दनाम- चप

हम उनस सीख लनी चाकिहएবইndashবাংলা সাহিতয পহিরচয়

পাঠndash১০লপndashহিবকেবকানকেFর যকেলকেবলাযলখকndashশশী-ষণ াশগNঅনশীলনীর পরকে4াততর

৯প পহিরবতG ন ককেরা -শরীর = শারীহিরক -ত = য-ৌহিতক সG ার = সG াহির সথায়ী = সথাহিয়তবহিবশবাস = হিবশবাসী া = যকো১০ বাকয রচনা ককেরা -আডডা = পাার বদধরা াতলায় আডডা যয়হিডবাহিজ = রাম া যকেক হিডবাহিজ যখকেয় পকেলাসG ার = বকেলহিল যকেল যর সG ারবকহিন = অঙক -ল করায় রীনা মাকেয়র কাকে বকহিন যখকেলাবহিদধ = বহিদধ াককেল উপায় য়হিনশহিত = হিনশহিত রাকেত রাজবাহিকেত ডাকাত পকেলা-য়ানক = পাাহি রাসতা বষটির পকের -য়ানক হিবপ জনক য়হিবশবাস = মানকেষর পরহিত হিবশবাস ারাকেনা পাপ

Science Ch ndash The Food We Eat

Living things need food to live to grow to stay strong and fit When we need food we feel hungry Food gives us energy to do work It also protects us from diseases and helps us to stay healthy Nutrients in food The food we eat contains many substances that are necessary for our body These substances are called nutrients that help us to grow well and stay healthy

Nutrients give us energy to study work and play

They help our body to grow and repair the damaged parts of our body

They also help our body to fight against diseases and remain healthy

Write T for True or F for False (pg no- 11)

1 Food contains nutrients that help us to grow True

2 Foods rich in carbohydrates are called body-building foods False

MAT

HEM

ATIC

S

Ch 9

Com

mon

Fra

ction

s

A fraction is a number that stands for parts of a whole object or a collection of objects

Each fraction has two numbers One is written above the other separated by a line The one above the line is called numerator and the one below the line is called denominator

Example 5minusminusminusminusrarr Numerator

11minusminusminusminusrarr Denominator

Exercise ndash 362 In following fill in the blanks

(b) 37 ___ is denominator ___ is

numerator

(c) 89 ___ is denominator ___ is

numerator

Solution

(b) 37 7 is denominator 3 is numerator

(c) 89 9 is denominator 8 is numerator

3 Write down the fractions whose numerators and denominators are given below in the bracket The first number stands for numerator and the second number standing for denominator

(25) 25

(311) 311

(416) 416

(712) 712

Class VSubject Topic Summary Execution

Science Ch ndash PlantReproduction

In pollination chapter we have learnt that the flowers change into fruits and the fruits bear seeds Now we are going to learn how this process takes place

FertilizationThe process of fusion of the male reproductive cell (male gamete) and female reproductive cell (female gamete) is known as fertilizationWhen a pollen grain reaches from the anther to a stigma it begins to grow and forms apollentube The Pollen tube then travels down through the style to enter an ovule inside the ovary On reaching the ovule male reproductive cell in the pollen grain unites with the egg cell present in the ovule

Books exercise

A) Tick the correct answer

1Which of the following do ovules change into after fertilization ndashseeds

2Which of the following is not a part of the seed ndash flower

3Which of the following condition is needed for germination of a seed ndash all ofthese

English language

Sentences phrases and

Solved exercisesSay which of the underlined groups of words are phrases and which clauses

clauses8 In her new clothes ndashphrasesAs pretty as a doll ndash phrases

9 looking sad and upset ndash phrasesHe had lost all the tickets for the Test Match ndash clauses

10 During the vacation now only a month away ndashphrases

11 too sweet and too hot ndashphrases

12 At the Olympic Games ndashPhrasesOf Laurel leaves ndash phrases

13 Who played the role of Hamlet ndashclauses

14 However fast ndash phrases

15 When the men fell asleep ndash clausesSocial studies

Indian Government

Lok Sabha (lower house) ndash It has 552 members Of these 530 membersrepresent States 20 members represent the union territories and two members represent the Anglo- Indian community All except the representatives of the Anglo-Indian community are elected by Indian citizens A person above the age of 25 can contest in the elections for Lok Sabha One term of Lok Sabha is for 5 yearsRajya Sabha (upper house) ndash Its members are elected by the MLAs or members of the legislative Assembly There are 250 members in the Rajya Sabha of which 12 are nominated by the President One term of Rajya Sabha is for 6 years Anyone above the age of 30 can be elected as a member of Rajya Sabha

ExecutivePresidentThe President is the head of the country in India He is elected by the MPs and the MLAs for a tenure of 5 years He appoints the Prime Minister and the Council of Ministers Prime MinisterThe party which wins the election forms the government and its leader is elected as the Prime Minister He is the chief advisor to the President The Council of Ministers assists the Prime Minister and is accountable for their roles For example the Education minister is responsible for the education system in our country

1 How many members are there in Lok SabhaAns 552 members

2 What is the term for Lok SabhaAns 5 years

3 How many members are there in the Rajya SabhaAns 250 members

4 Who is the head of our countryAns President

5 Who is the chief advisor to the PresidentAns Prime Minister

Book ndash GK

Ch ndash 1First in space

1 First living being into space in 1957 Ans Laika

Times 2 First person to go into space in 1961 Ans Yuri Gagarin

3 First woman to go into space in 1963 Ans ValentinaTereshkova4 First person ever to walk in space in 1965Ans alexei Leonov5 First person to land on the moon in 1959 Ans Neil Armstrong6 First Indian to go into space in 1984 Ans Rakesh Sharma 7 First Indian woman to go into space in 19978 Ans Kalpana Chawla9 First woman tourist in space in 2006

Ans Anusheh AnsariCOMPUTER

ALGORITHM AND FLOWCHART

Q) DRAW THE SYMBOLS USED IN A FLOWCHART WITH THEIR DESCRIPTIONS(IN EXAM IT CAN COME AS SHORT QUESTIONS ASKING INDIVIDUAL SYMBOLS FUNCTION)ANS)

MAT

HEM

ATIC

S

Ch 6

Com

mon

Fra

ction

s

Multiplication of FractionsA Multiply a fractional number by whole numberTo multiply a fractional number by whole number we multiply the numerator of the fractional number by the whole number and denominator of the fractional number by 1 The first product thus obtained is the numerator and the second product is the denominator of the required product

Exercise ndash 30Multiply

7 2027

times 9

Solution 2027

times 9 = 203 = 6

23

8 611

times11

Solution 611

times11 = 6

15 71

20times16

Solution 71

20times16 =

14120

times16

= 1415

times 4 = 141times 4

5 = 564

5 = 11245

B Multiplication of a fractional number by a fractional number To multiply a fractional number by a fractional number we multiply the numerator of the first fractional number by the numerator of the second fractional number and the denominator of the first fractional number by the denominator of the second fractional number The first product thus obtained is the numerator and the second product is the denominator of the required product

16 2712

times24

Solution 2712

times24 = 3112

times24

= 31times2 = 62

Exercise ndash 31

11 83

times 34

2

Solution 83

times 34 = 2

14 723

times2 25

4

Solution 723

times2 25 =

233

times 125 =

23times 45

= 925 = 18

25

15 1212

times1 13

2

Solution 1212

times1 13 =

252

times 43 =

25times 23

= 503 = 16

23

State the following statements are true or false

17 1912

times 239 = 1

Solution LHS = 1912

times 239

= 392

times 239 = 1 = RHS

[LHS = Left hand side amp RHS = Right hand side]

there4 1912

times 239 = 1 [True]

21 213

times2 13 = 4

19

Solution LHS = 213

times2 13 =

73

times 73

= 7times73times3 =

499 = 5

49

there4 LHS ne RHS

So 213

times2 13 = 4

19 [False]

23 23

times 45 =

2times 5+3 times43times 5

Solution

LHS = 23

times 45 =

2times 43 times5 =

815 again

RHS = 2times 5+3 times4

3times 5 = 10+12

15 = 2215

there4 LHS ne RHS So 23

times 45 =

2times 5+3times43times 5

[False]

25 23 of

13 =

29

Solution

LHS= 23 of

13 =

23 times

13 =

29 = RHS]

there4 23 of

13 =

29 [True]

Practice at HomeExercise ndash 31State the following statements are true or false

24 12 of 4 =

18

Class VISubject Topic Summary Execution

HISTORY AND CIVICS

Chapter 5The Mauryan Empire

DECLINE OF MAURYAN EMPIREDecline of Mauryan empire started after the death of Ashoka at around 232 BCThere are several reasons for break up of the empire1 Weak successor Emperors after Ashoka were

capable of handling vast and mighty Mauryan empire In 185BC the last Mauryan ruler Bri-hadrath was murdered by his Commander-in-Chief Pushyamitra Sunga

2 Provincial Revolts Due to weak central author-ity provincial chiefs of Kalinga and southern provinces revolted against emperor and freed themselves from Mauryan empire

3 Weakness of Economy Prosperity of Mauryan was based on solid economic activities which

ExercisesI Multiple choice questions-1 Chandragupta defeated Seleucus in the year ndashc) 305 BC2 Who killed the last Mauryan ruler Brihadrath b) Pushyamitra3 Which of the following was not a reason for the decline of the Mauryan empirec) Chandraguptarsquos weakness4 Ashoka invaded Kalinga in the year c) 261 BC

II Fill in the blanks1Chandragupta ascended the throne in 324

was taken care by early monarchs Later kings had neither ability nor interest in economic af-fairs That led to failure in tax collection As a result they failed to maintain a large army that were essential to keep empire intact

4 Greek Invasion Greeks freed north-western provinces from weak Mauryan monarchs and reestablished their authority

5 Ashokarsquos Policy some scholar opined that after Kalinga war Ashoka embraced Buddhism re-nounced the policy of war and disbanded the Army But this is partially true as there is no proper evidence of disbanding the army

Based on above points we can conclude that main reason for decline of Mauryan empire is weakness of Ashokarsquos successors Kunal Samprati Dasharath Salisuk all were weak kingsAt last in 185 BCPushyamitra Sunga killed king Brihadrath and established the Sunga dynasty

BC2 Bindusara was the son of Chandragupta and father of Ashoka

3 Pataliputra was administered by City Magistrate committess of 5 members each4 The Greek General Seleucus sent his ambassador Megasthenes to Chandraguptarsquos court5 Ashoka sent his son Prince Mahendra and daughter Sanghamitra to spread his Dhamma6 The Indian Rebublic has adopted the Lion Capital of Saranath Pillar as its national emblem 7 Pushyamitra killed the last Mauryan ruler Brihadrath and founded the Sunga dynasty

III Name the following

1The author of Arthashastra-Kautilya2 The ruler who founded the Mauryan dynasty-Chandragupta3 The author of Indika-Megasthenes 4 The officers who were appointed by Ashoka to spread Dhamma-Dhamma Mahamatras5 The general of Alexander whom Chandragupta defeated-Seleucus

V Match the columns1 Kautilya (c)2 Megasthenes (d)3 Pushyamitra (e)4 Brihadrath (b)5 Bindusara (a)

BENGALI(2ND

LANGUAGE)

পশপাহিখর -াষাসহিবনয় রায়কেচৌধরী

যলখক পহিরহিচহিত- পরখযাত সাহিহিতযক উকেপনদরহিককেশার রায়কেচৌধরীর পতর সহিবনয় রায়কেচৌধরী lsquoসকেFশrsquo পহিতরকার সকেb হিতহিন কত হিকেলন তার উকেdখকোয বই lsquoসহিবনয় রায়কেচৌধরীর রচনা সংগরrsquo

পরম হিকেনর পাঠ- lsquoপশপাহিখর হিক -াষাhelliphellip helliphelliphelliphellipপরসপরকেক জানাবার উপায়ও পশপাহিখরা যবশ জাকেনrsquoপরকেমই আমারা জাহিন -াষা হিক -াষা ল আমাকের মকেনর -াব পরকাশ করার জনয আমরা নানান ধরকেনর -হিb বা হিবকেশষ ধরকেনর আওয়াজ মকেখর মাধযকেম কহির অনযকেক যবাঝাকেনার জনয তাকেল এবার আমরা জাহিন পশপাহিখর -াষা হিক পশ পাহিখরা হিক কা বকেল যা পশপাহিখকেরও -াষা আকে তারা তাকের হিনজসব -াষায় কা বকেল মকেনর -াব পরকাশ ককের পশ পাহিখরা মানকেষর হিক হিক -াষা যবাকেঝ হিকনত তারা বলকেত পাকেরনা পরসপরকেক বহিঝকেয় যবার উপায় তারা জাকেননা তকেব তারা হিবকেশষ ককেয়কটি শকেবদর মাধযকেম তাকের মকেনর -াব বহিঝকেয় যয় হিক বহিদধ মান জীব ndashককর হিবাল বন মানষ যঘাা পর-হিত এরা মানকেষর যওয়া নাম শনকেল কান খাা ককের ndash নাম ধকের ডাককেল কাকে আকেস যমন - মরহিরা lsquoহিত ndashহিতrsquo ডাক শকেন আকেস াল lsquoঅ ndashর -র ডাক শকেন কাকে আকেস াহিত মাহকেতর কা শকেন চকেল ককররা মাহিলকেকর হকম পালন ককের সবসময় তাইকেতা ককরকেক পর- -কত পরানী বলা য় ককর আর হিবাল একের আওয়াজ তহিম লকষয করকেল বঝকেব ককররা যরকে যকেল lsquoযঘউ যঘউrsquo করকেত াকেক আবার কাকেল lsquoযকউ যকউrsquo ককের হিবাল সাধারণ lsquoমযাওrsquo বা lsquoহিমউrsquo ককের রা কেল lsquoওয়াওrsquo আওয়াকেজর মাধযকেম মকেনর -াব পরকাশ ককের একেতা যল পশকের কা পাহিখরাও -য় রা পরকাশ করার জনয হিবকেশষ ধরকেনর শবদ ককের হিবপকের সময় পশ পাহিখরা সবার আকে পরসপরকেক জানাবার উপায় তারা জাকেন বহকাল

১) পশপাহিখর -াষা কেলপর যলখক সমপকেকG হিক জাকেনা

উঃ- পরখযাত সাহিহিতযক উকেপনদরহিককেশার রায়কেচৌধরীর পতর সহিবনয় রায়কেচৌধরী lsquoসকেFশrsquo পহিতরকার সকেb হিতহিন কত হিকেলন তার উকেdখকোয বই lsquoসহিবনয় রায়কেচৌধরীর রচনা সংগরrsquo হিতহিন ারকেমাহিনয়াম এসরাজ পর-হিত বাযনতর বাজাকেত পারকেতন ানও জানকেতন হিতহিন যাকেIাকের জনয মজাার লপ কহিবতা হিলখকেতন

২) পশপাহিখ কেলপর মল-াব হিকউঃ- পশপাহিখকেরও -াষা আকে তারা তাকের হিনজসব -াষায় কা বকেল মকেনর -াব পরকাশ ককের পশ পাহিখরা মানকেষর হিক হিক -াষা যবাকেঝ হিকনত তারা বলকেত পাকেরনা পরসপরকেক বহিঝকেয় যবার উপায় তারা জাকেননা তকেব তারা হিবকেশষ ককেয়কটি শকেবদর মাধযকেম তাকের মকেনর -াব বহিঝকেয় যয় হিরউকেবন কযাসটং সাকেব হিতহিন চহিdশ বর বনযজনত যর সকেb যকেককেন হিতহিন বকেলকেন আমরা হি তাকের -াষা তাকের আব কায়া যমকেন চহিল তাকেল আর -কেয়র যকান কারণ াকেকনা আমরা একI -াকেলাকেবকেস যচষটা করকেল পশপাহিখকের সকেb -াব পাতাকেত পাহির

ধকের মানষ এই পশ পাহিখর -াষা হিনকেয় নানা রককেমর পরীকষা ককের আসকে এইরকম একজন হিরউকেবন কযাসটাং সাকেকেবর কা আমরা জানকেবাhelliphellip

Hindi 2nd

langमतर किनमनलिलखिखतपरशनोउRरदीजि0ए

) बढ वयलि` बचच ो कया हआ था ख) डॉकटर साहबन पाटc किस उददशय स रखी थी ग) ाल साप ो हाथ म लर लाश न कया किया घ) डॉकटर चडढा न बढ पतरो दखन स कयो मना र दिदया था ङ) भगत न लाश ो दखर कया हा

उRर ndash) उस बहत बखार थी और 4 दिदनो स आख भी नही खोला थाख) उन बट ी सालकिगरह थीग) ाल सापो हाथ म लर लाश न उसी गदन 0ोर स दबार पडी थीघ) डॉकटर चडढा न बढ वयलि` पतरो दखन स मना र दिदया कयोकि उनह गोलफ खलन 0ाना थाङ) लाश ो दखर हा कि नारायण चाहग तो आध घट म भया उठ 0ाएग

English literature

In the bazaars of Hyderabad- Sarojini Naidu

Through the poem In The Bazaars of Hyderabad Sarojini wanted to convey the message that India is rich in tradition and they donrsquot need the foreign products So she goes on to give a picture of a bazaar where traditional Indian products are rulingThe poem is in the form of questions and answers The poet asks the questions and the merchants answer them Through this technique she make the picture of the bazaar visible to us

Read the poem

PHYSICS FORCE Types of FrictionThere are three types of friction static sliding rolling Static sliding and rolling friction occur between solid surfaces

1 Static Friction The frictional force that acts between the surfaces when they are at rest with respect to each other is called Static FrictionStatic Friction Examples

Skiing against the snow Creating heat by rubbing both the hands

together Table lamp resting on the table

2 Sliding Friction The resistance that is created between any two objects when they are sliding against each other is called Sliding FrictionExamples Of Sliding Friction

Sliding of the block across the floor Two cards sliding against each other in a

deck

3 Rolling Friction The force which resists the motion of a ball or wheel is called Rolling Friction Is the weakest types of frictionExamples Of Rolling Friction

Rolling of the log on the ground Wheels of the moving vehicles

6What effect can a force produce on a body which is not allowed to move Ans - When a force is applied on a body which is not free to move it gets deformed i e the shape or size of the body changes7Give one example each to indicate that the application of a force

1 produces motion2 stops motion3 slows down motion4 changes the direction of motion5 deforms a body

Ans- 1 A car originally at rest when pushed

begins to move2 A moving bicycle is stopped by

applying the brakes3 The speed of a moving vehicle is

slowed down by applying the brakes4 A player kicks a moving football to

change its direction of motion5 On stretching a rubber string its

length increases

8State the effect produced by a force in the following cases (a) The sling of a rubber catapult is stretched(b) A man pushes a heavy cart(c) A player uses his stick to deflect the ball (d) A cyclist applies brakes(e) A spring is compressedAns- (a) The shape and size of catapult changes ie its length increases(b) The heavy cart begins to move(c) The direction of the ball changes(d) The speed of the moving cycle is slowed down(e) There is change in size and shape of spring

COMPUTER MS EXCEL 2013 -INTRODUCTION

UNDERSTANDING EXCEL STRUCTUREA SPREADSHEET IS A FILE THAT EXISTS OF CELLS IN ROWS AND COLUMNS AND CAN HELP ARRANGE CALCULATE AND SORT DATA DATA IN A SPREADSHEET CAN BE NUMERIC VALUES AS WELL AS TEXT

FORMULAS REFERENCES AND FUNCTIONS

WORKSHEETA WORKSHEET IS ALSO KNOWN AS SPREADSHEETIT IS A COLLECTION OF CELLS ON A SINGLE SHEET WHERE YOU KEEP AND CHANGE DATA

WORKBOOKWORKBOOK IS PMS EXCEL FILE IN WHICH THE DATA CAN BE STORED EACH WORKBOOK CAN CONTAIN MANY WORKSHEETS

ROWS AND COLUMNSIN MS EXCEL A ROW IS A GROUP OF CELLS THAT RUN FROM LEFT TO RIGHT OF A PAGEA COLUMN IS A GROUPING OF CELLS THAT RUN FROM THE TOP TO THE BOTTOM OF A PAGE

CELLTHE INTERSECTION POINT BETWEEN A ROW AND THE COLUMN IS CALLED A CELL WHICH IS THE BASIC STORAGE UNIT FOR DATA IN A SPREADSHEET EACH CELL HAS SPECIFIC ADDRESS WHICH IS THE COMBINATION OF THE COLUMN NAME FOLLOWED BY THE ROW NUMBER

CHEMISTRY Chapter ndash Common Laboratory Apparatus and equipments

Objective type questionFill in the blanks (a) Experiment and observation are the two important basics of chemistry(b) A porcelain dish is used for evaporation(c) A test tube holder is used to hold the test tube while-it is heated(d) Mortar and pestle is used for grinding and crushing solid substances into a powder(e) Glass apparatus is made of Pyrex or borosil glass

Class VIISubject Topic Summary Execution

Hindi 2ndlang

ए था राम( डॉ शरी परसाद)

सगकित ा परभाव मानव 0ीवन पर अवशय पडता ह

हमशा मनषय ो अचछो ी सगकित म रहना चाकिहए

शरषठ परो सग स मनषय चरिरतर ा शीघर ही उदय और किवास हो 0ाता

ह इसलिलए वयलि` ो सदा शरषठ परो ा ही सग रना चाकिहए

इसान अगर चाह वह सवय ो बदल भी सता ह

यह हानी राम ए बचच ी हवह गणिणत ी परीकषा म नल रत हए पडा 0ाता ह और उस अधयाप पडत ह और पछत ह यह कया र रह

हो तभी राम न उनी बइजजती ी

शबदाथब ndashहावा भलावाायवाहीndash ाम किनयम व ानन

ो दिदखानापरिरलिचतndash 0ाना पहचानाघटनाndashघबराहट

उलटा चोर ोतवाल ो डाटndashकिववndash भल बर ा जञानतवयndash म 0ो रना चाकिहएसगकितndash बरी सगत

किबलख नाndashरोना किनशचय रनाndash तय रना

फलndashपरिरणामकिनषालिसतndash बाहर किया हआपशचातापndashदख सपननndashधनी

ldquo हा आपी किहममत स हई नल रत पडन ीrdquo ऐसी बात ही किफर

किपता0ी न भी उस डाटा वह ाफी पशचाताप रन लगा बोला गलत दोसतो

ी सगकित म आ0 कितना अनथ र दिदया किफर उसन अधयाप स माफी मागन ी सची और किफर भी ऐसा

नही रगा यह परण भी लिलया

सोचndashकिहच एात-अला

বইndashবাংলা সাহিতয পহিরচয়

পাঠndash১৬লপndashস-য ও অস-যযলখকndashঈশবরচনদর হিবযাসারঅনশীলনীর পরকে4াততর

৬ অGকেলকেখা -ময়া = পশ হিশকার সহিtহিত = হিনকIবতu সbভরষট = লI হিনরীকষণ =

-াকেলা-াকেব যখাকতাঞজহিলপকেI = যজাাকেত৭ হিবপরীতশবদ -ঈষৎ times পরচর উৎকষট times হিনকষট তাশ times উৎফd তবহিদধ times

বহিদধীNপাহিপষঠ times পণযবান৮ পপহিরবতG ন ককেরা -পশ = পাশহিবক যকাপ = যকাহিপতহিসথর = হিসথরতাএকানত = ঐকাহিনতক পর-াত times পর-াতী

CHEMISTRY

Chapter ndashPhysical and Chemical Changes

Chemical ChangeA chemical change involves a change in chemical composition

Characteristics of Chemical changes 1 They are permanent changes2 They are irreversible changes 3 New substance formed4 A Chemical change involves a

change in its chemical properties

Pg-25Question 8What do you observe when1 water is boiled2 a piece of paper is burnt3 some ice cubes are kept in a glass tumbler4 solid ammonium chloride is heated5 an iron nail is kept in tap water for few days6 a spoon of sugar is heated in a pan7 lighted match stick is brought near the mouth of the test tube containing hydrogen gas8 quick lime is dissolved in water9 little amount of curd is added to a bowl containing warm milk and kept for five hours

10 Water is boiledOn boiling water changes into steam (gas) physical change

11 A piece of paper is burnton burning piece of paper produces carbon dioxide and ash is left behind Is a chemical change

12 some ice cubes are kept in a glass tumblerIce cubes (solid) turn into water

(liquid) only state changes (physical change)

13 Solid ammonium chloride is heatedSolid ammonium chloride on heating changes into vapors (change of state) is physical change

14 An iron nail is kept in tap water for few dayswe observe reddish brown coating on the nail called rust (entirely new substance) is chemical change

15 A spoon of sugar is heated in a panWhen a spoon of sugar is heated in a pan black (charred sugar) (carbon) is seen Is a chemical change

16 Lighted match stick is brought near the mouth of the test tube containing hydrogen gasWe observe that hydrogen bums at the mouth of test tube with blue flame and pop sound is heard It is chemical change

17 Quick lime is dissolved in waterThe following two observations will be observed (i) A hissing sound is observed(ii) The mixture starts boiling and lime water is obtained

18 Little amount of curd is added to a bowl containing warm milk and kept for five hoursWhen a little amount curd is added to a bowl containing warm milk and kept for five hours a permanent change occurredThe milk will change to curd On boiling water changes into steam (gas) physical change

GEOGRAPHY

ATMOSPHERE IMPACT OF GLOBAL WARMING The destructive impart of global warming is observed in various spheres of life and the environment Some of the points are outlined below1 High temperatures lead to high

evaporation rate and drying up of the soil and surface water This affects crop production The occurrence of droughts is aggravating the problem even further

2 The heat waves in summer months

Q1 Write some impact of global warmingA1 The impacts of global warming are as follows1 High temperatures lead to high

evaporate ion rate and drying up of the soil and surface water This affects crop production The occurrence of droughts is aggravating the problem even further

2 The heat waves in summer months lead to a greater number

lead to a greater number of deaths due to heat strokes

3 Forest fires become more frequent4 Tropical cyclones and hurricanes

become common5 Melting of glaciers takes place6 Polar ice caps are becoming thinner

and melting at an alarming rate due to global warming The loss of sea ice

7 Due to increase in sea surface temperature sea levels rise in coastal areas and cause submergence of several islands

WAYS TO REDUCE GLOBAL WARMINGFollowing steps can be taken We need to decrease emission of

green house gases by reducing the burning of fossil fuel such as coal and petroleum

By planting more trees to increase forest cover

The government should also distributes free saplings and organize afforestation programmes to spread awareness regarding the beneficial effects of trees

We should switch to eco-friendly cars and gadgets

Incandescent light bulbs should be replaced by CFL bulbs

We can save electricity and reduce global warming by turning off electrical gadgets such as lights fans air-conditioners television and computer when we do not to use them

Efforts should be made to hasten the development of green cities oreco cities These cities are urban areas around the world striving to lessen the environment a impacts of urbanization

By following the 3Rs-Reduce Recycle and Reuse strategy we can use natural resources for our growth as well as save them for the need of the future generations This is called sustainable development

of deaths due to heat strokes3 Forest fires become more

frequent4 Tropical cyclones and hurricanes

become common5 Melting of glaciers takes place

etc

Q2 How to reduce global warmingA2 Following steps can be taken to reduce global warmingaWe need to decrease emission of

green house gases by reducing the burning of fossil fuel such as coal and petroleum

bBy planting more trees to increase forest cover

c The government should also distributes free saplings and organize afforestation programmes to spread awareness regarding the beneficial effects of trees

dWe should witch to eco-friendly cars and gadgets

eIncandescent light bulbs should be replaced by CFL bulbs

f We can save electricity and reduce global warming by turning off electrical gadgets such as lights fans air-conditioners television and computer when we do not to use them

Q3 What do you mean by 3Rrsquos of resource planningA3 The 3Rs are

1 Reduce 2 Recycle and3 Reuse

Q4 What is Sustainable developmentA4 By following the 3Rs-Reluce Recycle and Reuse strategy we can use natural resources for our growth as well as save them for the need of the future generations This is called sustainable development

English Language

Prepositions A preposition is a word placed before a noun or a pronoun It helps to show how the person or thing denoted by the noun is related to something else in the sentence

Kinds of Prepositions

Simple Prepositions- simple preposition are one word Prepositions such as at by for in of off for from on out through till to up with before amidst towards beyond between over etc

Compound Prepositions ndash There are some words that are always used with fixed Prepositions to convey specific meaning

Example I was unable to meet you dueto a previous engagement ( On account of)Always maintain the queue instead of crowding at the counter ( In place of)

Participial PrepositionsmdashParticiple Prepositions are present or past participles of various verbs which together with a noun phrase or a clause function as prepositions Examples- barring concerning considering notwithstanding pending regarding respecting etc

Exercise A

1 Gauravs fever has come down since Friday He has been absent for a week now

2 The child sat between his father and mother among the parents of all his classmates

3 There are mosquitoes in the room They flew into the room when the door was open

4 My father was inside the drawing room when I was playing outside my house

5 You may sit beside me I will give you a drawing book and pencils besides a storybook

6 We went to the market in the morning and walked towards the riverfront in the evening

7 The child walked along the pavement and across the street safely

8 This table top is made of glass My breakfast fell off it in the morning

9 The pan is on the gas stove There are vegetables in it

10 We will wait for you at the bus top There are a lot of people in the hall

Subject ndash Biology Topic ndash Chapter - 3 Photosynthesis and respiration in plants Summary Execution

All living organism (Plants and animals) need food for energy and growth Green plants (autotrophy) prepare food for all living organisms Today we will discuss about the process photosynthesis And adaptations in a leaf to carry out photosynthesis

Q1What do you mean by photosynthesis and write its word equation The process by which green plants make food (glucose) from carbon dioxide and water

in the presence of sunlight and chlorophyll is called photosynthesis

Carbon dioxide + Water ( Sun light from Sun ) Glucose + Oxygen ( chlorophyll in green leaves )

Q2 What are the adaptations in a leaf to carry out photosynthesisi) Leaves are broad wide and flat for absorbing more light energyii) Presence of chlorophyll in chloroplasts to trap sunlightiii) Presence of stomata which allow carbon dioxide to enter the cell and oxygen to go

out iv) Network of veins ensures continuous supply of water and minerals to the leafv) Thin waxy cuticle protects the leaf without blocking the lightQ3 Draw and label structure of chloroplast

Class VIIISubject Topic Summary Execution

PHYSICS ENERGY Production of Hydro electricity

A hydroelectric dam converts the potential energy stored in a water reservoir behind a dam to mechanical energymdashmechanical energy is also known as kinetic energy As the water flows down through the dam its kinetic energy is used to turn a turbine

The generator converts the turbinersquos mechanical energy into electricity

This electric energy then goes through various transmission processes before it reaches you

Question 2

Fill in the blanks

(a) Work is said to be done by a forte only when the body moves

(b) Work done = Force x distance moved in direction of force

(c) The energy of a body is its capacity to do work

(d) The SI unit of energy is joule

(e) The potential energy is due to its state rest of position and kinetic energy of the body is due to its state of motion

(f) Gravitational potential energy U = mass times force of gravity on unit mass times height

(g) Kinetic energy = frac12 times mass times (speed)2

(h) Power P = work donetime taken

(i) The S I unit of power is watt

(j) IHP = 746 W

BIOLOGY Chapter -5 The endocrine system and adolescence

Today we will discuss about thelocation and functions of secreted hormones of adrenal and Pancreas

Q5 Write location hormone secreted main functions and deficiency diseases of pancreas and adrenal glands

Endocrine Glands

Location Hormones secreted

Functions and Deficiency Diseases

1Adrenal gland

2 Pancreas Gland

On the top of each kidney

In between stomach and small intestine

i)Adrenaline from adrenal medulla

ii)Cortisone from adrenal cortex

i) Insulin

ii) Glucagon

It helps a person deal with any kind of emergency situation or emotional stressIt increases the heart beat rate of respiration and blood pressure

a) It regulates carbohydrates protein and fat metabolism

b) It regulates the salt and water balance in the body

a) It changes excess glucose into glycogen

b) It stimulates the cells to burn extra glucose to provide heat amp energy

Less secretion causes diabetes mellitus

Excessive secretions causeinsulin shock

a) It stimulates the breakdown of glycogen into glucose

b) It increases the level of glucose in blood

History Traders to rulers The Battle of Buxar was fought on 22 October 1764 between the forces under the command of the British East India Company led by Hector Munro and the combined armies of Mir Qasim the Nawab of Bengal till 1763 Mir Jafar was made the Nawab of Bengal for a second time in 1763 by the Company just after the battle After being defeated in 4 battles in katwa and Udaynala the Nawab of Awadh Siraj id Daula and the Mughal emperor Shah Alam II accompanied by Raja Balwant Singh of Kashi made an alliance with Mir Qasim The battle was fought at Buxar a small fortified

Answer the following questions- Short note-Battle of BuxarHomework-learn

town within the territory of Bihar located on the banks of the Ganga river about 130 kilometres (81 mi) west of Patna it was a decisive victory for the British East India Company The war was brought to an end by the Treaty of Allahabad in 1765

EnglishLiterature

The west wind-John Mansfield

In the poem The West Wind by John Masefield the poet starts by describingwith very poetic imagery of birds how the west wind is different from other winds its a warm wind full of birds cries There is a touch of melancholy perhaps home-sickness as he describes how it brings tears too and memories from an old land He goes on to describe the restful pastoral beauty of the land where even the dead can lie in the green He then brings in voicesperhaps of family and friends calling him home as he is missing Aprils beautyThe voices then tempt him some more with idyllic images from home (white blossom young green cornrunning rabbitswarm sun) The voices seem to presume that the poets heart is sorrowful bruised and soreThe end of the poem sees the poet appear to make a decision he will go home as he has decided that is where he truly belongs

Write the synopsis of the following words

1 Daffodils- a tall yellow flower that grows in the spring

2 Orchards- a piece of land on which fruit trees are grown

3 Blossom- a flower or a mass of flowers especially on a fruit tree in spring

4 Thrushes- a bird5 Larks- a small brown bird that

makes a pleasant sound6 Bruised- an injury7 Aching- pain 8 Tread- to put your foot down

while you are walking9 Balm-10 May-11 Fluting-

(Write from the book in your copy)

MAT

HEM

ATIC

S

Ch 1

1Al

gebr

ic E

xpre

ssio

n

1 Constant A symbol which has fixed value is called a constant[eg 8 23 -15 radic3 etc]

2 VariableA symbol which does not have any fixed value but may be assigned value (values) according to the requirement is called variable or literal[eg x y p q etc]

3 TermsA term is a number (constant) a variable a combination (product or quotient) of numbers and variables[eg 7 x 5x etc]

4 Algebric expressionA single term or acombination of two or more terms connected by plus (+) or minus (-) sign forms an algebraic expression[eg 5-y 3x2-5x xy-6z+4 etc]

5 PolynomialAn algebraic expression which contains more than one term is called a polynomial (multinomial)[eg x2-5x 5y+xy+x2y etc]

6 Degree of polynomial(a) When the polynomial contains only one variable the highest power of the variable is the degree of the polynomialeg the degree of the polynomial of 4x-7x5+8 is 5(b) When the polynomial contains two or more variablesStep (i) Find the powers of the variables in each term (ii) The highest sum of the powers is taken to be the degree of the polynomialeg the degree of the polynomial 5x2y-4x3y5+6 is = 3+5 = 8Remember An algebraic expression is a polynomial if degree of each term used in it is a non-negative integer

Exercise ndash 11(A)

1 Separate the constants and variables from the following

-7 7+x 7x+yz radic5 radic xy 3 yz

8 45y -3x

Solution Constant Variables-7 radic5 7+x 7x+yz radic xy

3 yz8

45y -3x

2 Write the number of terms in each of the following polynomials(i) 5x2+3timesax (ii) axdivide4-7 (iii) ax-by+ytimesz (iv) 23+atimesbdivide2

Solution Polynomials Number of terms(i) 5x2+3timesax 2(ii) axdivide4-7 2(iii) ax-by+ytimesz 3(iv) 23+atimesbdivide2 2

4 Write the degree of the each polynomials(i) xy+7z (ii) x2-6x3+8 (iii) y-6y2+5y8 (iv) xyz-3 (vi) x5y7-8x3y8+10x4y4z4

Solution Polynomials Degree(i) xy+7z 2(ii) x2-6x3+8 3(iii) y-6y2+5y8 8(iv) xyz-3 3(vi)x5y7-8x3y8+10x4y4z4 12

5Write the coefficient of(i) ab in 7abx (iv) 8 in a2-8ax+a (v) 4xy in x2-4xy+y2

SolutionCoefficient

(i) ab in 7abx 7x(iv) 8 in a2-8ax+a -ax(v) 4xy in x2-4xy+y2 -1

7 CoefficientAny factor of an algebraic quantity is called the coefficient of the remaining quantityeg in the algebraic term 7xyz 7 is coefficient of xyz 7x is coefficient of yz and so on

8 Like term The terms having the same literal coefficient are called like terms and those having different literal coefficients are called unlike terms

eg (i) 5xyz 8xyz -6xyz and 23xyz are like

terms(ii) 7xy2 8x2yz and -15xyz2 are unlike terms

6 in 57xy2z3 write the coefficient of

(i) 5 (vii) 5xy2 (viii) 17yz (xi) 5xyz

Solution Coefficient

(i) 5 17

xy2z3

(vii) 5xy2 17z3

(viii) 17yz

5xyzsup2

(xi) 5xyz 17yz2

7 In polynomial given below separate the like terms(ii) y2z3 xy2z3 -58x2yz -4y2z3 -8xz3y2 3x2yz and 2z3y2

Solution y2z3 -4y2z3 2z3y2 are like terms

xy2z3 -8xz3y2 are like terms

-58x2yz 3x2yz are like terms

Class IXSubject Topic Summary Execution

Bengali (2nd language)

বাগzwnjধারাzwnj বা ধারা-বা ধারা ল হিবকেশষ পরকার বাক -হিb -াকেবর এক হিবকেশষ পরকাশরীহিত াকেক কতগকেলা কার সমষটির মকেধয এগহিলকেক বা ধারা বকেল আবার কতগকেলা শকেবদর বাধাধরা যকান রীহিত যনই য-াকেব চকেল আসকে যসই -াকেবই চকেল আসকে তখন যসই শবদগহিল খন একক -াকেব অG পরকাশ ককের তখন একের বা ধারা বকেল বা ধারার পরকেয়া -াষাকেক আরও সFর ককের যতাকেল

অকাল পকক(অপহিরনত বয়কেস পাকাহিম)-মাতর শ বর বয়কেস যমকেয়টির া মকেখর কা তাকেত অকালপককতা ধরা পকে

অককা পাওয়া( মারা াওয়া) ndash পকেকIমারটি পকেকIমারকেত হিকেয় বাসাতরীকের াকেত মার যখকেত যখকেত অককা যপল

অহি| পরীকষা ( কঠিন ও পরকত পরীকষা)- যকেলটির আজ ডাকতাহির যরজালট যবকেরাকেব এIাই তার জীবকেনর ব অহি| পরীকষা

অষটরমভা (ফাহিক) ndash রীতা মকেখই বকো বকো কা বকেল আর কাকেজর যবলায় অষটরমভা

অকমGার ধাী (অপাG) ndash সমনকেক হিনকেয় যকান ান কেব না ও একেকবাকেরই অকমGার ধাী

অকেনধর ষটি (অসাকেয়র সায়)- আহিশ বকেরর বকোর নাহিত ল অকেনধর ষটি তাকেক াা বকোর একম চকেল না

আকেককল গড়ম (তবহিদধ)- ার তহিম উপকার করকেল যসই যতামার হিবরকেদধ সাকষয হিকেয়কে শকেনই আমার আকেককল গড়ম

আষাকে লপ( অবাসতব লপ) ndashIাকা এখন যকেব না এIা বলকেলই ত এমন আষাকে লপ ফাার যকান রকার হিল না

Hindi- महायजञ ा इस हानी म लख न या बतान ा परयास किया ह कि किसी भी अचछ

2nd language

परसार(यशपाल ाय या पणय न ा फल अवशय मिमलता ह ोई भी परोपार अथवा पणय लिलए किया गया ाय बार नही 0ाता वह ए परार ा यजञ हए धनी सठ थ धम परायण और किवनमर सठ न आन ी यजञ किए थ और दान म न 0ान कितना धन दिदन दखिखयो म बात दिदया थादिदन पलट और सठ यहा गरीबी आ गई उन दिदनो यजञ बचन ी परथा थी सठ भी अपनी 0गह बचन लिलए डलपर ए सट यहा चलन ो तयार हए सठानी रासत लिलए रोटी पड म बाधर सठ ो द दी रासत म ए भख R ो दखर सठ न चारो रोटी उसो खिखला दी खर वह सठ यहा डलपर पहच तो उनी सठानी न उस महायजञ बचन ो हा यदिद बचन आए सठ न R ो रोटी खिखलान ो महायजञ नही समझा और वापस लौट आया घर आर शाम ो उसी घर म उस ए बडा ख0ाना मिमला 0ो उस दवारा किए गएrsquo महायजञrsquo ा परसार था

English language

Letter formal The heading the name and address of the person you are writing to must be included beneath your own address In formal letters ldquoblock stylerdquo of address is preferred

Subject complain in brief

Salutation If the person you are writing to is known to you you may begin ldquoDear MrrdquoOr ldquoDear Mrsrdquo In all other instances you should begin ldquoDear Sirrdquo or ldquoDear Madamrdquo Or ldquoSirsrdquo

The body A formal or business letter has four partsReference The letter should begin by referring to a letter you have received an advertisement or the reason that has prompted you to writeInformation In the second paragraph it is necessary to supply more detailed information that is related to the referencePurpose Here you must give the reason why you are writing the letter This must be stated clearly and ensure that it is relevant to the question that has been setConclusion round off the letter with some polite remarkThe subscription when a letter has begun with dear sir sirs Madam you should end with Yours faithfully or yours truly When however you address a person by name you must conclude with the words ldquoYours sincerelyrdquo

1 A park in your locality is slowly being used as a rubbish dump Write a letter to the Mayor of your city pointing out the nuisance and danger of this Request that action be taken to stop this immediately

Or2 You being a boarder ordered a set of lab manuals from a famous book shop in the town They sent you a wrong set of books Write a letter to the manager of the book shop

Chemistry Chapter-1 1)CHEMICAL FORMULA- Q What is the Significance of

L-2The Language of Chemistrybull Chemical Formula

Itrsquos a symbolic representation of a chemical substance eg ndash The formula of Sulphuric acid is H2SO4

2) Steps of writing Chemical Formula of a given substance-

1 Write the symbols of the constituent atoms or radicals side by side Keep the basic radical on LHS and acid radical on the RHS ( Na+Cl- )2 In case of a radical having more than one atom( compound radical) enclose the radical in a bracket eg (SO4-)3 Write the valencies of each radical on its right hand top4 If the valencies of the two radicals are divisible by a common factor then divide the valencies by the common factor5 Invert (criss-cross) the valency number ie write the valency of one atom below the second atom and vice versa 6 On interchanging if valency number is lsquoone the figure lsquoonersquo is never writtenFor Example- Compound -Calcium Nitrate1 Writing the symbols- Ca(NO3)2 Writing the valencies on their right hand top- Ca2(NO3)1

3 Valency numeral in simple ratio- Ca2(NO3)1

4 Criss-cross- Ca 2NO3 1

5 Writing the formula of the compound- Ca(NO3)2

Chemical formula

A The formula of a substance conveys the following information regarding a substance 1 The name of the substance (qualitative)2 The elements constituting the substance (qualitative)3 The number of various atoms present in a molecule of the substance (quantitative)4 Molecular weight of the substance and the relative weights of different elements present in it (qualitative)

Q What are the limitations of Chemical Formula

A The chemical formula suffers from the following limitations-I It fails to convey whether the elements in a molecule are present in the form of atoms or ionsFor example the formula KBr fails to tell us whether Potassium and Bromine are present in the form of ions II It does not tell anything about the binding force that holds atom in a molecule togetherIII It does not tell us about the arrangement of various atoms with respect to one another within the molecule

Q Examples of Some Chemicals with their Formula Chemical name and Common Name-

A Given in the class notesCommercial Studies

Joint Stock Company

Let us discuss about the demerits of Joint Stock CompanyDespite so many advantages it has got many disadvantages which are as follows

Difficulty in FormationDelay in Decision makingExcessive Government ControlLack of Secrecy

Company can be classified into several categories based on incorporation

QuestionExplain the demerits of Joint Stock CompanyAnswer) 1 Difficulty in Formation The legal requirements and formalities required to be completed are so many The cost involved is quite heavy It has to approach large number of people for its capital It cannot start its business unless certificate of incorporation has been obtained This is granted after a long time when all the formalities are completed

Chartered CompanyStatutory CompanyRegistered Company

Delay in Decision making In this form of organization decisions are not made by single individual All important decisions are taken by the Board of Directors Decision-making process is time-consuming So many opportunities may be costly because of delay in decision-making Promptness of decisions which is a common feature of sole trader ship and partnership is not found in a company

Excessive Government ControlA company and the management have to function well within the law and the provisions of Companies Act are quite elaborate and complex At every step it is necessary to comply with its provisions lest the company and the management should be penalized The penalties are quite heavy and in several cases officers in default can be punished with imprisonment This hampers the proper functioning of the company

Lack of Secrecy The management of companies remains in the hands of many persons Every important thing is discussed in the meetings of Board of Directors Hence secrets of the business cannot be maintained In case of sole proprietorship and partnership forms of organisation such secrecy is possible because a few persons are involved in the management

2 Define the following

Chartered Company- The crown in exercise of the royal prerogative has power to create a corporation by the grant of a charter to persons assenting to be incorporated Such companies or corporations are known as chartered companies Examples of this type of companies are Bank of England (1694) East India Company (1600) The powers and the nature of business of a chartered company are defined by the charter which incorporates it After the country attained independence these types of companies do not exist

in IndiaStatutory Company- A company may be incorporated by means of a special Act of the Parliament or any state legislature Such companies are called statutory companies Instances of statutory companies in India are Reserve Bank of India the Life Insurance Corporation of India the Food Corporation of India etc The provisions of the Companies Act 1956 apply to statutory companies except where the said provisions are inconsistent with the provisions of the Act creating them Statutory companies are mostly invested with compulsory powersRegistered companiesCompanies registered under the Companies Act 1956 or earlier Companies Acts are called registered companies Such companies come into existence when they are registered under the Companies Act and a certificate of incorporation is granted to them by the Registrar

Economics

Chapter-4Basic problems of Economy

Today let us discuss with the topic Production Possibility curve

QuestionExplain the concept of Production Possibility Curve with the help of diagram

Answer) Production Possibility curve is a locus of all possible combinations of two commodities which can be produced in a country with its given resources and technology

The above diagram shows that with the given resources and technology the economy can produce maximum either 5 thousand meters of cloth or 15 thousand quintals of wheat or any other combination of the two goods like B( 1 thousand meters of cloth and 14 thousand quintals of wheat C ( 2 thousands meters of cloth and 12 thousand quintals of wheat) etcProduction Possibility curve is also called production possibility boundary or frontier as it sets the maximum limit of what it is possible to produce with given resources

Geography

Rotationand Revolution

SUNrsquoS POSITION AND SEASONAL CHANGES EQUINOXES ndash SPRING AND AUTUMN

Q1 What is Spring EquinoxA1 On 21st March sunrays fall directly on the equator On that day

As the Equator divides the Earth into two equal halves the sun rays fall directly on the equator twice in a year Equinoxes means equal Spring EquinoxOn 21st March sunrays fall directly on the equator On that day the duration of day and night both are equal ( 12 hours day and 12 hours night) on every places located on equator This day is called as Spring EquinoxAutumn EquinoxOn 23rd September sunrays fall directly on the equator On that day the duration of day and night both are equal ( 12 hours day and 12 hours night) on every places located on equator This day is called as Autumn Equinox

SOLSTICES ndash SUMMER AND WINTERDue to inclination of the Earth on its axis and the apparent movement of the sun the sun rays fall directly on both tropics once in a year Solstice is a Latin word which mean ldquothe Sun standing stillrdquoSummer SolsticesAfter 21st March there is an apparent movement of the Sun to the north of the equator The apparent northward movement up to 21st June when the Sun appears overhead at the Tropic of Cancer (22frac12degN) The sun appears to stand still at this position and then moves southwards towards the equator This position of the Sun on 21st June is known as Summer Solstices On that day the duration of day and night both are equal ( 12 hours day and 12 hours night) on every places located on Tropic of Cancer (22frac12degN)Winter solstices The apparent southward movement of the Sun continues beyond the equator till 22nd

December On this day the Sun is overhead at the Tropic of Capricorn

the duration of day and night both are equal ( 12 hours day and 12 hours night) on every places located on equator This day is called as Spring Equinox

Q2 What do you mean by EquinoxA2 Equinoxes means equal It is use to explain the equal duration of day and night ( 12 hours day and 12 hours night) on the Earth

Q3 On which date the longest day in Tropic of CancerA3 21st June

Q4 What is the meaning of SolsticeA4 Solstice is a Latin word which mean ldquothe Sun standing stillrdquo

Q5 Which is the longest day in southern hemisphereA5 22nd December

Q6 On what date does the Arctic Circle experience the lsquoMidnight SunrsquoA6 On 21 June the Arctic Circle experiences the lsquoMidnight Sunrsquo

Q7 What is cause of Midnight Sun in NorwayA7 During the summer solstice (21 June) the North Pole is inclined towards the Sun Therefore the duration of sunlight or daytime increases from 12 hours at the Equator to 24 hours at the Arctic Circle and beyond Thatrsquos why The region beyond the Arctic Circle especially Norway is known as the Land of the Midnight Sun because there the Sun does not rise or set on 21 June

Q8 Match the column A with BA B

Summer Solstice 21st March

Autumn Equinox 23rd

September

Winter Solstice 21st June

(22frac12degS) This position of the Sun is referred to as the Winter Solstice because it marks the winter season in the Northern Hemisphere On that day the duration of day and night both are equal ( 12 hours day and 12 hours night) on every places located on Tropic of Capricorn (22frac12degS)SEASONS AND DURATION OF DAY AND NIGHT During the equinoxes all places on the Earth have 12 hours of day and 12 hours of night Due to the revolution of the Earth round the Sun on an inclined axis the duration of day and night varies according to seasons and the latitude of a placeDuring the summer solstice (21 June) the North Pole is inclined towards the Sun Therefore the duration of sunlight or daytime increases from 12 hours at the Equator to 24 hours at the Arctic Circle and beyondThe region beyond the Arctic Circle especially Norway is known as the Land of the Midnight Sun because there the Sun does not rise or set on 21 JuneAt the North Pole there will be six months of daylight The Sun will be seen always above the horizon at a low angle At 66degN 24 hours of sunlight can be seen only on 21 June Hammerfest in northern Norway is a place of tourist attraction for observing the phenomenon of the Midnight Sun This place has continuous daylight from 13 May to 29 July This place is easily accessible to tourists and has hotels and other facilities The view of the midnight Sun from here is enthrallingIn the Southern Hemisphere the duration of daylight decreases from 12 hours at the equator to 0 hours beyond the Antarctic Circle In the South Polar Region there is 24 hours of darkness The Sun is always below the horizon In the Southern Hemisphere which experiences winter the duration of night-time is longer than the duration of daylight

Spring Equinox 22nd

December

A8 A B

Summer Solstice 21st June

Autumn Equinox 23rd

September

Winter Solstice 22nd

December

Spring Equinox 21st March

During winter solstice (22 December) the South Pole is inclined towards the Sun The Southern Hemisphere experiences summer and the Northern Hemisphere has winter Therefore the duration of daylight or sunlight is greater in the Southern Hemisphere than in the Northern HemisphereThe duration of daylight increases from 12 hours at the equator to 24 hours beyond the Antarctic Circle The South Polar Region has 24 hours of sunlight for many days continuously At the South Pole there will be six months of sunlight The Sun will always be seen at a low angle above the horizon In the Northern Hemisphere the duration of daylight will decrease from 12 hours at the equator to 0 hours at the Arctic Circle There are 24 hours of darkness in the North Polar region The duration of night is greater than the duration of daylight as one move northwards from the Equator It is evident from the above table that the duration of daylight is 12 hours throughout the year at the equator only As one moves away from the equator the seasonal variations in the duration of daylight increase The seasonal variations in the duration of daylight are maximum at the Polar Regions

Subject Eng Literature (The Merchant of Venice ndash William Shakespeare)Topic Act II Scene 7 Lines 36 to 80 (End of scene ) [Students should read the original play and also the paraphrase provided]

Summary Questions amp AnswersThe Prince then examines the inscription on the silver casket which says ldquoWho chooseth me shall get as much as he deservesrdquo The Prince says that he deserves Portia more than anybody else because of his high rank his noble birth and his great wealth and power But then he argues that silver is ten times

(1) (Act II Sc 7 L 39-47)

From the four corners of the earth they come

To kiss this shrine this mortal breathing saint

The Hyrcanian deserts and the vasty wildsOf wide Arabia are as through-fares now

inferior to gold and therefore he cannot believe that the portrait of such a beautiful lady as Portia can be contained in the silver casket He decides to see the inscription on the golden casket before making his decision

The Prince goes to examine the inscription on the golden casket which says ldquoWho chooseth me shall get what many men desirerdquo The Prince believes that the whole world desires to possess Portia otherwise so many suitors would not have come from all corners of the world for winning Portia Some of them have come from the distant lands of Persia and Arabia The deserts of Persia (Hyrcanian deserts) and the boundless desolate lands of Arabia have been crossed by the Princes seeking the hand of Portia He contrasts this casket containing Portiarsquos portrait with the old English gold coin bearing the image of the archangel (angel of the highest rank) He goes on to remark that while the figure of the archangel is engraved (Insculped) upon the English coin the picture of Portia who is beautiful as an angel lies hidden inside one of the caskets namely the Golden Casket (Golden Bed)

On the basis of his assessment of the inscription on the golden casket the Prince decides to choose the golden casket He asks for the key and opens the golden casket only to find therein an empty human skull holding a roll of

For princes to come view fair PortiaThe watery kingdom whose ambitious headSpets in the face of heaven is no barTo stop the foreign spirits but they comeAs orsquoer a brook to see fair Portia

(i) Explain the occasion for the above mentioned speech

These are the comments of the Prince of Morocco after he reads the inscription on the golden casket His mental process is revealed to us in these words We find him debating within himself as to which casket he should choose

(ii) What light does the above speech throw on the personality of Prince of Morocco

From the above mentioned speech we come to know that the Prince of Morocco is keen to marry Portia He is the type of person who is easily taken away by outward appearance He is in love with Portia because of her beauty

(iii) What information can you gather about Portia from the above mentioned lines

The given speech shows that Portia is a very beautiful lady She must be possessed of good qualities because many suitors come to her place from all over the world with a desire to get married to her The Prince of Morocco is so impressed by her beauty that he calls her a saint According to him the whole world is desirous of having her

(iv) Elucidate the significance of the first two lines

In these lines the Prince of Morocco pays a compliment to Portia These lines show his admiration for her He says that people come from all parts of the world to see fair Portia

(v) Explain the meaning of the last four lines of the

passage

In these lines the Prince of Morocco says that even the vast oceans which throw a challenge at the sky are unable to prevent men from coming to Portiarsquos place to have a glimpse of her These lines are also a tribute to Portiarsquos beauty and good qualities Many men voyage across the ocean treating it as a mere stream to see the beautiful Portia

paper in which is written that whoever happens to be guided by the glitter of things is invariably deceived

On reading the scroll the Prince says that he is too sad at heart to speak a more formal farewell and leaves with his followers amidst a sound of trumpets

After the Prince of Morocco leaves Portia remarks that the Prince is a gentle fellow but she is rid of him May all persons of his nature make a similar choice

IMPORTANT PASSAGES EXPLAINED

(Act II Sc 7 L 39-43)From the four corners of the earth they come

To kiss this shrine this mortal breathing saintThe Hyrcanian deserts and the vasty wildsOf wide Arabia are as through-fares nowFor princes to come view fair Portia

Context

This passage occurs in Act II Scene 7 in The Merchant of Venice This is part of the speech made by the Prince of Morocco

(2)

(Act II Sc 7 L 48-53)

MOROCCO One of these three contains her heavenly pictureIst like that lead contains her

Twere damnation To think so base a thought it were too grossTo rib her cerecloth in the obscure graveOr shall I think in silver shes immurdBeing ten times undervalued to tried gold

(i) What meaning does the Prince of Morocco find out of the inscription of the golden casket What have Belmont and Portiarsquos house been called and why

The inscription on the golden casket is ldquoWho chooseth me shall gain what many men desirerdquo The Prince finds out that it means that the chooser of the golden casket will get Portia because many men desire her In fact the entire world desires her Because of the coming of many suitors to Belmont from different countries in order to win Portiarsquos hand Belmont has become a centre of pilgrimage and her house is the shrine where saintly Portia is installed

(ii) What does the Prince of Morocco do before making the final choice of the casket Which is the correct casket and who will win Portiarsquos hand

The Prince of Morocco surveys and analyses the inscriptions on the casket of lead silver and gold Before making the final choice like a very systematic and methodical person he once again considers the claims of the caskets The casket containing Portiarsquos picture is the correct casket and the person choosing it will win Portiarsquos hand

Explanation

While praising Portia the Prince of Morocco conceives Portia as a goddess whose image is placed inside one of the caskets Many suitors are coming from far and wide the north and the south the east and the west (Four corners) in order to try their luck Some of them have come from the distant land of Persia and Arabia The deserts of Persia (Hyrcanian deserts) and the boundless desolate lands of Arabia have been crossed by the Princes seeking the hand of Portia All this shows that Portia is indeed the most beautiful lady of the world

(iii) What does the Prince of Morocco say in his estimation while examining the motto on the silver casket What does he find in the golden casket

While examining the motto on the silver casket which says ldquoWho chooseth me shall get as much as he deservesrdquo Morocco says that in his own estimation he surely deserves Portia in all respects ndash rank birth wealth etc

He chooses the golden casket When he opens it he finds an empty human skull holding a scroll in which it is written that those who are attracted by the glittering outside of things are always deceived as Morocco has been deceived

(iv) What kind of nature does the Prince of Morocco have

The Prince of Morocco has a simple nature who does not look deeply into the inner meaning of things but is dazzled by the outward appearance of gold He is inclined to over-estimate his own value and does not realize that it is a duty to ldquogive and hazardrdquo To say that he will not hazard for lead shows that he misreads the true meaning of the inscription which is that he should be prepared to ldquohazard all he hathrdquo for Portia So his feeling is only one of fascination and romantic attraction

(v) Do you think that the lottery of the caskets is not a matter that will be determined by chance

In fact the lottery of the casket is not a matter that will be determined by mere chance but that it is a true test of character and of sincerity which is amply proved not only by Moroccorsquos choice but also by the arguments which he uses to help him in his choice

(Act II Sc 7 L 55-59)

They have in England

A coin that bears the figure of an angelStamped in gold but thats insculpd uponBut here an angel in a golden bedLies all within

Context

(3)

(Act II Sc 7 L 63-77)A carrion Death within whose empty eye

There is a written scroll Ill read the writing

All that glisters is not goldOften have you heard that toldMany a man his life hath soldBut my outside to beholdGilded tombs do worms infoldHad you been as wise as boldYoung in limbs in judgment oldYour answer had not been inscrolld

This passage occurs in Act II Scene 7 in The Merchant of Venice This is part of the speech made by the Prince of Morocco

Explanation

In this passage the Prince of Morocco bestows high praise on Portia whose hand he is seeking He contrasts this casket containing Portiarsquos portrait with the old English gold coin bearing the image of the archangel (angel of the highest rank) He goes on to remark that while the figure of the archangel is engraved (Insculped) upon the English coin the picture of Portia who is beautiful as an angel lies hidden inside one of the caskets namely the Golden Casket (Golden Bed) In the day of Elizabeth silver was ten times inferior in value to gold Therefore the Prince of Morocco believing that Portiarsquos portrait is contained in the Golden Casket decides to choose the Golden Casket

Fare you well your suit is coldCold indeed and labour lostThen farewell heat and welcome frostmdashPortia adieu I have too grievd a heartTo take a tedious leave Thus losers part

(i) What reward does the Prince of Morocco get after making a wrong choice of the Casket How does he feel

After making the wrong choice in selecting the casket of gold the Prince of Morocco as a reward earns a rebuke in the form of a scroll tucked in the empty eye-socket of a skull kept in the casket of gold The Prince is shocked and disappointed He becomes all the more sad and dejected when he reads the scroll which points to his foolishness in being misled by the appearance and outward show as indicative of its worth

(ii) How does the Prince respond after reading the scroll

After reading the scroll the Prince though upset accepts the result with good grace and decorum befitting a royal suitor and true sportsman He says that his love-suit is really cold otherwise he would have chosen correctly but now his efforts have been in vain So he bids farewell to Portia to the warmth and enthusiasm of love and welcomes the cold and bitterness of dejection and misery of life which lies ahead

(iii) What request does he make to Portia and why

After being failure in his mission he requests Portia to give him permission to leave at once because he is too sad to undergo the tediousness of a formal leave-taking He tells that it is the manner in which defeated persons part unceremoniously

(iv) Explain the following lines

ldquoAll that glisters is not goldOften have you heard that toldMany a man his life hath soldBut my outside to beholdGilded tombs do worms infoldrdquo

Mere glitter does not make a metal to be gold Man has often been warned against appearance but it has been of no use Many people have sacrificed their lives only to seek the outer appearance of gold Worms are found inside the gilded

monuments

Class XSubject Topic Summary Execution

Hindi 2ndlang

नया रासता भाग 6 मायाराम 0ी घर म धनी मल 0ी और उनी बटी सरिरता ी ही चचा बनी रहती थी अमिमत ो इसम ोई रलिच ना थी वह धनी घर ी लडी स शादी र सवय ो बचना नही चाहता था उसा भी सवाणिभमान ह ईशवर ी पा

स उस पास पस ी ोई मी नही थी अभी उसन फकटरी ही लगाई थी उसी समझ बाहर था कि उस घर वालो ा झाव पस ी तरफ कयो

ह उसन मा स सवाल किया कि मा तम सरिरता स मरी शादी कयो रना चाहती हो मा न उस समझाया कि वह दखन म बरी नही ह और किफर खानदान अचछा

ह वह ए शल गरहणी रप म घर सभाल सगी अमिमत न मा ो इस बात ा एहसास राया कि मीन सबध लिलए मना रन पर उस दिदल

पर कया बीती होगी मा और अमिमत ी लडी बार म ाफी बात हईमा ा झाव सरिरता ी तरफ था कयोकि वह घर पर अचछा दह0 लर आ रही

थी अमिमत न अपनी मौसी ी बरी हालत बार म बताया कि किस तरह वह बड घर ी खानदानी बटी लाई थी और आ0 उसी हालत कितनी खराब ह लाई थी बहकलब 0ाती ह और बचचो ो भी नही दखती ह बात चल ही रही

थी कि तभी ए ार बाहर आर री धनी मल0ी घर अदर आए और पीछ स डराइवर फल ी ए टोरी लर आया अदर आए और पीछ स

डराइवर ए टोरी फल ी लर आया अमिमत ो फल ी पटी बरी लग रही थी अमिमत न पछ लिलया यह फल कयो ल आए ह प इन सब ी कया

0ररत थी उनो न 0वाब दिदया कि 4 पटी शमीर स मगाए थ अमिमत ो या सनर करोध आ गया तभी उस किपता 0ी आ गए उन आत ही अमिमत उठर बाहर चला गया वहा वहा मा पास आर बठ गया और बोला

अभी रिरशता तय नही हआ और धनी मल 0ी धनी मल 0ी फल ी पटी लर चलआय मा न समझाया कि 0ब सबध 0ड 0ाता ह तो खाली हाथ नही

आत अमिमत न मा स हा कि तम सबन सरिरता ो इस घर म लान ी ठान रखी ह धनीमल 0ी उस दिदन सरिरता ो दखन ी तारीख तय रन आय थ

Commercial Studies

Banking Nowadays Bank provide easy and quick services through internet facilities methods of Banking is called internet bankingIn order to save the time and money involved in visiting Bank branches people increasingly prefer to have internet banking

There are different modes of doing internet banking or transferring money through online They areReal Time Gross Settlement (RTGS)National Electronic Fund Transfers (NEFT)

1

Question

1) Explain the term RTGS Write the features of RTGS

Answer)The acronym RTGS stands for Real Time Gross Settlement which may be defined as the continuous real time settlement of funds transfer individually on and order by order basis without netting lsquoReal timersquo may be defined as the processing of instructions at the time they are received rather than at some letter time lsquoGross settlementrsquo may be defined as the settlement of transfer instructions which occurs

individually

Features of RTGS1It is the continuous settlement of

funds transfer individually on an order by order basis

2RTGS facility is provided only by CBS core banking solution enabled Bank branches

3Amount charged from the customer for RTGS transactions vary from bank to bank

2) Explain the term NEFT Write the features of NEFT

Answer) National electronic funds transfer may be defined as a nationwide system that facilitates individuals Farms and copper operates to electronically transfer funds from any bank branch to any individual farm or corporate having an account with any other bank branch in the country

Features of NEFT2 Transfer can be made 7 times on

weekdays and 6 times on Saturday

3 NEFT cannot be used to receive foreign remittances

4 NEFT transaction takes place in batches

5 A bank branch must be NEFT enabled to become a part of NEFT fund transfer network

6 There is no maximum or minimum amount that can be transferred through NEFT when one bank has a bank account

English Language

CompositionEssay

A composition is an art of creating a piece of writing on any topic or subject It is the writing correctly beautifully and clearly in order to make some interesting reading Structure of the composition

Introduction ( you lay the foundation for your composition)

Body (it constitutes the main part of the essay)

Conclusion (final statement that leaves a lasting impression)

Kinds of essays1 The Narrative essay2 The descriptive essay3 The reflective essay4 The argumentative essay

Write a composition on any one of the following topics (350- 400 words)

1 Friendship Or2 The first day of your school

Subject Eng Literature (The Merchant of Venice ndash William Shakespeare)Topic Act V Scene 1 Lines 127 to 158 (Nerissa helliphellip The clerk will nersquoer wear hair onrsquos face that had it) [Students should read the original play and also the paraphrase given in the school prescribed textbook]

Summary Revision Questions o Soon thereafter Bassanio Gratiano

and Antonio arrive

o Bassanio tells Portia that he is feeling as if it is morning because of the presence of Portia who is shining like the sun When Antonio is introduced by Bassanio to Portia she tells Bassanio that he should be grateful to Antonio who took so much trouble on his account even to the extent of risking his life

o Nerissa starts quarrelling with Gratiano and demands that he show her the ring she had presented to him and which she had warned him not to lose She suspects that Gratiano must have presented the ring to some young woman and not to the lawyerrsquos clerk as he repeatedly says and assures

Answer the following questions to check your preparation of Act IV Scenes 1 and 2

You must attempt only after you have completed your preparation of Act IV The answers must be in complete sentences using textual evidence (with citation) when necessary

[It would be in your own interest to attempt the above questions honestly totally refraining from consulting your textbook or your notes during answering After completion you should correct the paper yourself consulting the textbooknotes etc and award marks as specified Please let me know the marks you scored through WhatsApp in the group or to my personal WhatsApp]

Act IV Scene 1 (each question carries 2 marks)

1 What did the Duke try to do for Antonio

2 Why does Shylock refuse to show mercy How does he justify his stance

3 Why does Antonio say he is ready to die 4 What information is contained in Bellariorsquos letter

5 Why does Portia (as Balthazar) assert that Shylock must show mercy How does he respond

6 What offers are made to Shylock to get him to spare Antonio How are they received

7 What does Antoniorsquos speech as he faces the prospect of Shylockrsquos knife tell you about his character

8 How do Bassanio and Gratiano react to the looming prospect of Antoniorsquos demise

9 How does Portia (as Balthazar) use the law to turn the tables on Shylock

10 What does the Duke decree should happen to Shylock Why What happens to Shylockrsquos estate

11 What does Portia ask Bassanio as payment for her ldquoservicesrdquo What is his initial response What makes him change his mind

Act IV Scene 2 (each question carries 1frac12 marks)

1 What does Gratiano bring to Portia (Balthazar)

2 What does Nerissa plan on getting from Gratiano What does Portiarsquos comment suggest about men

ECO-10 280620 Topic-Supply AnalysisSHIFTING OF SUPPLY

But if there is change in factors other than the price of the commodity then either more is supplied at the same price or less supplied at the same price In such cases the price of the commodity remains constant but there is a change in other factors like change in the price of inputs change in technology of production change in price of other related goods change in taxation policy of the government etc For example there is an improvement in the technology of production of the commodity in question It leads to decrease in per unit of cost production of the commodity The firm is willing to sell more quantity of the commodity at the same price So the supply other commodity increases at the same price This increase in supply is shown by rightward shift of supply curve On the other hand if the firm uses inferior technology of production the cost of production per unit of the commodity increases The firm is willing to sell less quantity at the same price So the supply of the commodity decreases at the same price This decrease in supply is shown by leftward shift of the supply curve The above cases of increase and decrease in supply can be shown with the help of the following figures

Y INCREASE OF SUPPLY Price (Rs) s

P A s1

B

s

X` O s1 X

q q1

Y` Quantity demanded (in units)

Y DECREASE IN SUPPLY s2

s

price (Rs)

C

p A

s2

s

X` o X

q2 q

Y` Quantity demanded ( in units)

Main factors causing increase in supply or rightward shift of supply Curve(i) Fall in the price of other related goods

(ii) Fall in the price of inputsfactors(iii) Use of better technology in production(iv) Decrease in the rate of excise duty by government(v) If the objective of producer changes from profit maximization to salesMaximization

Main factors causing decrease in supply or leftward shift of supply curve(i) Increase in the price of other related goods(ii) Rise in the price of inputsfactors(iii) Use of inferior technology in production(iv) Increase in the rate of excise duty by the government(v) If the objective

Subject - Biology Topic ndash Chapter mdash6 PhotosynthesisSummary Execution

Today we will know about photosynthesis and its stages

Q1 What do you mean by photosynthesis The process by which living plants containing chlorophyll produce food

substances from carbon-di- oxide and water by using light energy Sunlight

6CO2 +12 H2O----------------------- C6 H12O6 + 6H2O + 6O2

Chlorophyll

Q2 What are the importance of photosynthesis I) Food for all Green plants trap solar energy by photosynthesis

process and supply food and energy for all living organisms either directly or indirectly

Ii) Oxygen to breathe in by product of photosynthesis is oxygen which is essential for all living organisms respiration

Q3 Write about two main phases of photosynthesis A Light dependent phase This phase occur in grana of chloroplast I) The chlorophyll on exposure to light energy becomes activated by

absorbing photons Ii) The absorbed energy is used in splitting the water molecules (H2O)

into its two components (H+ and OH- ) and releasing electron s 2H2O------------------------- 4H+ + 4e- +O2

Energy of 4 photons This reaction is known as photolysis

End products are H+ and oxygen water

B Light independent (Dark ) phase The reactions in this phase require no light energy

Here CO2 combine with H+ and produce glucose

Class XI

Subject Topic Summary ExecutionEVS Chapter-4 Legal

regimes for sustainable development

Environmental legislationEnvironmental legislation is the collection of laws and regulations pertaining to air quality water quality the wilderness endangered wildlife and other environmental factors The act ensures that matters important to the environment are thoroughly

Learn -The Forest (Conservation) Act 1980

considered in any decisions made by federal agencies

The Forest (Conservation) Act 1980 The Forest (Conservation) Act 1980 an Act of the Parliament of India to provide for the conservation of forests and for matters connected therewith or ancillary or incidental thereto It was further amended in 1988 This law extends to the whole of IndiaObjects and Reasons of the Forest Conservation Act

Deforestation causes ecological imbalance and leads to environmental deterioration Deforestation had been taking place on a large scale in the country and it had caused widespread concern The act seeks to check upon deforestation and de-reservation of forests

Subject Eng Literature (The Tempest ndash William Shakespeare) Topic Act II Scene 1 Lines 314 to 329 (End of scene)

[Students should read the original play and also the paraphrase given in the school prescribed textbook]Summary Questions amp Answers

Conspiracy of Antonio and Sebastian (Contd)

o As they approach Ariel appears again and wakes up Gonzalo by singing a tune in his ear Alonso also wakes up and they see both Sebastian and Antonio with drawn swords On being caught off guard they make up a story saying that they had heard a bellowing of bulls or lions

o They then moved to another part of the island

o Ariel at once rushes to Prospero to inform him of this development

SUMMING-UP of ACT-2 SCENE-1

(i) Among the survivors Ferdinand is separated from the rest which results in the disconsolate grief of Alonso as he took him for dead

(ii) The villainy of Antonio is confirmed

(iii) The supremacy of Prosperorsquos magic which resulted in the failure of the human conspiracy

(1)

(Act II Sc 1 L 311-325)SEBASTIAN Whiles we stood here securing your repose

Even now we heard a hollow burst of bellowing Like bulls or rather lions Didt not wake youIt struck mine ear most terribly

ALONSO I heard nothingANTONIO O rsquotwas a din to fright a monsters ear

To make an earthquake Sure it was the roarOf a whole herd of lions

ALONSO Heard you this GonzaloGONZALO Upon mine honour sir I heard a humming

And that a strange one too which did awake meI shaked you sir and cried As mine eyes opened I saw their weapons drawn There was a noiseThats verily rsquoTis best we stand upon our guardOr that we quit this place Lets draw our weapons

(i) Why has Prospero sent Ariel to Gonzalo and Alonso What does Ariel do to awaken Gonzalo

Prospero has already come to know by his magic powers the danger which threatens Gonzalo who had been Prosperorsquos friend and so he sent Ariel to preserve the lives of both Gonzalo and Alonso Prospero does not want that his scheme should remain unfulfilled Ariel begins to sing a song in Gonzalorsquos ears to awaken him(ii) Who are ready to carry out their plan Who takes steps to stop them Why does Gonzalo feel surprised after being awakened

Sebastian and Antonio are ready to carry out their plans They are standing with their swords drawn to kill Alonso and

(iv) We see two sets of contrasting characters Gonzalo-Adrian against Antonio-Sebastian

(v) The grief that works in Alonso can be perceived to his repentance for his association in Antoniorsquos crime against Prospero

Gonzalo Ariel takes steps to stop them from carrying out their nefarious scheme When Gonzalo is awakened by the song sung by Ariel into his ears he (Gonzalo) feels surprised because he sees Sebastian and Antonio standing with their swords drawn(iii) What reason do Sebastian and Antonio tell of drawing their swords when they are suspected by Alonso and Gonzalo

When Sebastian and Antonio are seen with their swords drawn they are looked with suspicion by Gonzalo and Alonso At first Sebastian tells them that as they stood here to guard them during their sleep they heard only a little before a sudden loud noise very much like the roaring of bulls or more probably that of lions Then Antonio follows him saying that this was a noise so terrible as to frighten even a monsterrsquos ears and this noise could even have shaken the earth and it was surely like the roaring of a multitude of lions Then seeing the danger they have drawn their swords Perhaps after hearing the terrible noise they (Gonzalo and Alonso) woke up from their sound sleep

(iv) What does Gonzalo tell Alonso about the strange noise What did he see on opening his eyes Gonzalo tells Alonso that he did not hear the sound of roaring but he heard a humming sound which was strange and which woke him up After waking up he gave him (Alonso) a shaking and a loud cry On opening his eyes he saw these two gentlemen standing with their swords drawn(v) What does Gonzalo suggest

Gonzalo suggests that there was a noise indeed and of that he has no doubt at all and suggests that the best course for them would be to remain alert and vigilant against any possible danger to their lives or to leave this place and move to some other part of the island

Class XIISubject Topic Summary Execution

Commerce

Chapter- Management

Today we will discuss about LEVELS OF MANAGEMENT

Levels of management is a series or chain of managerial positions from top to bottom It helps individuals to know their authority responsibilities and superior-subordinate relations among themselves There are mainly three levels of Management TOP LEVEL MANAGEMENTMIDDLE LEVEL MANAGEMENTLOWER LEVEL MANAGEMENT

Top level managementIt consists of members at the highest level in the management hierarchy This level includes Board Of Directors Chief Executive Managing Directors Chairman President Vice President

Rolefunctions of the top levelmanagement1To analyse evaluate and deal

with theexternal environment2 To determine the objectives and

policies of the business3 To strive for welfare and survival

of business

4 To create an organisational Framework consisting of authority responsibility relationship

Middle level management Congress of members or groups who are concerned with implementation of the policies let down by the top managementThis level includes head of the department such as finance manager marketing manager branch and regional managers departmental and divisional heads plant superintendent etc

Role of functions of the middle level management

1 To interpret the policies framed by top management

2 To assign duties and responsibilities to lower level managers

3 To select and appoint employees for middle and supervisory level and evaluate their performance

4 To co-operate with other departments for smooth functioning

Operational or supervisory level managementIt refers to the group are members who are concerned with execution of the work They are also known as fast line managers This level includes supervisor 4 men Section Officer clerk Inspector etc

Role of functions of the lower level management1 To plan and execute day-to-

day operations2 To supervise and control the workers3 To arrange materials and

tools to start the process and make arrangements for training

4 Today present workers grievance and suggestions before the management and

ensure safe and proper working conditions in the factory

Business Studies

Staff Appraisal Chapter- 10 Today let us start with a new chapter

Staff Appraisal

Meaning of Performance Appraisal

Performance Appraisal is the systematic evaluation of the performance of employees and to understand the abilities of a person for further growth and developmentThe supervisors measure the pay of employees and compare it with targets and plansThe supervisor analyses the factors behind work performances of employeesThe employers are in position to guide the employees for a better performance

Objectives of Performance Appraisal

Following are the objectives of Performance Appraisal

To maintain records in order to determine compensation packages wage structure salaries raises etc

To identify the strengths and weaknesses of employees to place right men on right job

To maintain and assess the potential present in a person for further growth and development

To provide a feedback to employees regarding their performance and related status

To provide a feedback to employees regarding their performance and related status

Importance of Performance Appraisal

Performance appraisal provides important and useful information for the assessment of employees skill

knowledge ability and overall job performance The following are the points which indicate the importance of performance appraisal in an organization

1 Performance appraisal helps supervisors to assess the work performance of their subordinates

2 Performance appraisal helps to assess the training and development needs of employees

3 Performance appraisal provides grounds for employees to correct their mistakes and it also provides proper guidance and criticism for employees development4 Performance appraisal provides reward for better performance

5 Performance appraisal helps to improve the communication system of the organization

6 Performance appraisal evaluates whether human resource programs being implemented in the organization have been effective

7 Performance appraisal helps to prepare pay structure for each employee working in the organization

8 Performance appraisal helps to review the potentiality of employees so that their future capability is anticipated

Geography

DRIANAGE The SubarnarekhaThe Subarnarekha and the Brahmaniinterposed between the Ganga and the Mahanadi deltas drain an area of 19300 sq kmand 39033 sq km respectively The drainage basins of these streams are shared byJharkhand Odisha west Bengal and Chhattisgarh The Brahmani is known as southKoel in its upper reaches in Jharkhand

The NarmadaThe Narmada rises in the Amarkantak hills of MadhyaPradesh It flows towards the West in a rift valleyformed due to a geological fault The total length of it is 1300 km All the tributaries of the

Q1 Name the two westward flowing rivers in the peninsular plateauA1 Narmada and Tapi are the only westward flowing rivers of the peninsular plateau

Q2 Differentiate between east-flowing rivers and west-flowing riversA2

East-flowing rivers

West-flowing rivers

Narmada are very short inlength Most of its tributaries join the main streamright anglesThe Narmada basin covers parts of Madhya Pradesh and Gujarat

The Tapi The Tapi rises in the Satpura ranges in the Betul listrictof Madhya Pradesh It flows in a rift valley parallel tothe Narmada but it is much shorter in length It coversparts of Madhya Pradesh Gujarat and MaharashtraThe length is about 724 km

The Sabarmati and the MahiThe Sabarmati rises in the Aravali hills and flows south-south-westwards for a distance of 300 kilometres to the Arabian Sea The Sabarmatibasin extends over an area of 21674 sq km in Rajasthan and Gujarat The Mahi rises inthe east of Udaipur and drains an area of 34842 sq km lying in Madhya PradeshRajasthan and Gujarat It flows south-westwards for a distance of 533 km before it fallsinto the Gulf of Khambhat

The ChambalThe Chambal rises near Mhow in the Vindhya Range and flows towards the northgenerally in a gorge upto Kota Below Kota it turns to the north-east direction and afterreaching Pinahat it turns to the east and runs nearly parallel to the Yamuna beforejoining it in the southern part of the Etawah district in Uttar PradeshMajor Rivers of India with their basin area (Sqkm)

Himalayan System Indus 321290Ganga 861404

Brahmaputra 187110Indus System

Jhelum 34775Beas 20303

Ganga System Yamuna 366223Ghaghra 127950

Peninsular RiversNarmada 98796

Tapi 65145Mahanadi 141600

Subarnarekha 19300Sabarmati 21674

Mahi 34842Godavari 312812

Godavari Krishna Kaveri Mahanadi are the east-flowing rivers

Narmada Tapi west-flowing rivers

They fall into the Bay of Bengal

They fall into Arabian Sea

These rivers form big deltas

These rivers form comparativelysmall deltas

Catchment areas of these rivers are larger

Catchment areas of these rivers are smaller

Krishna 2589488Cauveri 87900

Subject ndashBiology Topic ndashChapter -5 Inheritance amp Variations Summary ExecutionToday we will discussabout linkage and its classification

LINKAGE The tendency of the genes located on the same chromosome to stay together is

hereditary transmission Linked genes the genes responsible for this Genes that exhibit the process of linkage locates in the same chromosome The distance between the linked genes in a chromosome determines the strength

of linkage i e genes that are located close to each other show stronger linkage than that are located far from each other

COMPLETE LINKAGE It is the type of linkage showed by the genes that are closely located or are tightly

linked with each other as they have no chance of separatingby crossing over These genes are always transmitted together to the same gamete and the same

offspring In such condition only parental or non cross over type of gametes are formedINCOMPLETE KINKAGE It is type of linkage showed by the genes that are distantly located orare loosely

linked with each other because they have chance of separating by crossing over

SIGNIFICANCE i) It helps in holding the parental character togetherii) It checks the appearance of new recombination and helps in bringing the

hybrid population which resembles the original parents iii) Linked genes dilute the effects of undesirable traits

Subject Eng Literature (The Tempest ndash William Shakespeare) Topic Essay Questions (EQ-3)Question No 3

Give a character sketch of CalibanAnswer

The character of Caliban has been wonderfully conceived by Shakespeare as the manifestation of all that is gross and earthy ndash a sort of creature of the earth as Ariel is a sort of creature of the air

Calibanrsquos Physical Appearanceo Caliban is lsquofreckledrsquo a lsquomisshapen knaversquo not honoured with human shape

o Prospero calls him lsquothou tortoisersquo (Act I Sc 2 Line 317) Trinculo stumbling upon him describes him as ldquoA strange fish hellip Legged like a man And his fins like armsrdquo He ldquosmells like a fishrdquo (Act II Sc 2 Line 25)

o Prospero also calls him a ldquobeastrdquo (Act IV Sc 1 Line 140) and ldquoThis misshapen knaverdquo (Act V Sc 1 Line 268)

o Further it appears that in addition to his physical deformity his spiritual inferiority is also suggested by Prosperorsquos claim that his birth resulted from the union between his mother the witch Sycorax and the devil

Calibanrsquos ParentageWhen the play opens Caliban is twenty four years of age having been born on the island twelve years before the coming of Prospero His mother was the foul witch Sycorax who was banished from Algiers for ldquomischiefs manifold and sorceries terrible to enter human hearingrdquo (Act I Sc 2 Line 264) and the father was the Devil himself Thus

Caliban is a monster of evil and brute nature ugly deformed and stinking

Calibanrsquos Savage and Malignant Natureo Caliban is entirely a creature of the earth ndash gross brutal and savage He regards himself as the rightful possessor

of the island and Prospero as a usurper

o In his young age he was on good terms with Prospero He had consented to be received by Prospero at his house and to be educated by him He has learnt human language only to curse his master whom he abhors

o His beastly nature soon breaks out and ends in a vicious attack on Miranda This opens the eye of Prospero who becomes severe to him and enforces his service by threats and violence

o Prospero uses him to make dams for fish to fetch firewood scraper trenches wash dishes and keep his cell clean

Calibanrsquos Hatred for ProsperoA profound hatred for Prospero has taken hold of Caliban It springs from a sense of his being dispossessed and ill-treated He would kill Prospero if he could but he knows the power of Prosperorsquos lsquobookrsquo Hence he transfers his allegiance to Stephano who seems like a god to him He also incites the two drunken associates to batter the skull of Prospero when he sleeps in the afternoon

Caliban Shows Considerable Intelligenceo He has learnt Prosperorsquos language

ldquoYou taught me language and my profit onrsquot (Act II Sc 2 Lines 86-89)Is I know how to curserdquo

o He is well aware of the futility of arguing with one who has more power than he has

ldquoI must obey his art is such power (Act I Sc 2 Lines 373-376)It would control my damrsquos god SetebosAnd make a vassal of himrdquo

o He realizes the importance of Prosperorsquos books

ldquoRemember (Act III Sc 2 Lines 89-92)First to possess his books for without themHersquos but a sot as I am nor hath notOne spirit to commandrdquo

o He knows the value of stealth when attacking the enemy

ldquoPray you tread softly that the blind mole may not (Act IV Sc 1 Lines 194-195)Hear a foot fall we now are near his cellrdquo

o Caliban has a better set of values than Stephano and Trinculo They are distracted from their plan by their greed for Prosperorsquos rich garments Only Caliban realizes that such a finery is unimportant

ldquoLeave it alone thou fool it is but trashrdquo (Act IV Sc 1 Lines 224)

Caliban is not a good judge of characterCaliban is not a good judge of character He decides for example that Stephano is a god because he dispenses lsquocelestial liquorrsquo (Act II Sc 2 Line 115) but then it must be remembered that he has only known his mother Sycorax Prospero Miranda and the spirits that torture him However he quickly discovers his error of judgementrdquo

ldquoWhat a thrice-double ass (Act V Sc 1 Lines 295-297)Was I to take this drunkard for a godAnd worship this dull foolrdquo

Calibanrsquos Imaginative NatureIf Caliban is sub-human in what has been said above he is human in the respect of the poetic side of his character He listens to music with rapture He tells of the beautiful dreams in which heaven rains treasures upon him and which upon waking he yearns to renew One of the most poetic passages in whole play is Calibanrsquos description of the island

to Stephano and Trinculo

ldquoBe not afeard The isle is full of noises (Act III Sc 2 Lines 135-143)Sounds and sweet airs that give delight and hurt notSometimes a thousand twangling instrumentsWill hum about mine ears and sometime voicesThat if I then had waked after long sleepWill make me sleep again and then in dreamingThe clouds methought would open and show richesReady to drop upon me that when I wakedI cried to dream againrdquo

Caliban - Less Ignoble Than Some OthersCalibanrsquos motive for murder is less dishonourable than that of Antonio and Sebastian They plan to kill Alonso to gain his power and wealth Caliban merely wants revenge and the return of lsquohisrsquo island

Conclusiono Calibanrsquos character is not portrayed very clearly in the play and hence we cannot decide whether he is a poor

savage being grossly maltreated by Prospero or whether he is evil and must therefore be kept in bondage or enslavement

o Caliban is contrasted with Ariel who is a spirit and thus swift and uninterested in physical activitieso Caliban is also contrasted with Prospero who is the all-powerful master of the island and of the destiny of all

those on the islando Caliban is also contrasted with civilized man showing him to be less evil than Antonio and Stephano and less

materialistic than Stephano and Trinculoo Caliban has suffered at the hands of Prospero and he has learnt to curse by listening to Prosperorsquos abuse He

certainly believes that Prospero has deprived him of his birthrighto Finally the character Caliban is thought to be one of Shakespearersquos masterpieces The complexity of the character

is reflected in the large volume of critical discussion that has grown around it

ECO ndash12 Topic-Forms of market

MonopolyMonopoly is a market structure in which there is a single seller there are no close substitutes for the commodity produced by the firm and there are barriers to entry Example Indian Railways which is operated under government of India Monopoly also implies absence of competitionFeatures of Monopoly Monopoly is characterized by1 Single Seller In monopoly there is only one firm producing the product The whole industry consists of this single firm Thus under monopoly there is no distinction between firm and industry Being the only firm there is significant control of the firm over supply and price Thus under monopoly buyers do not have the option of buying the commodity from any other seller They have to buy the product from the firm or they can go without the commodity This fact gives immense control to the monopolist over the market

2No Close Substitute There are no close substitutes of the product produced by the monopolist firm If there are close substitutes of the product in the market it implies presence of more than one firm and hence no monopoly In order to ensure a total of control over the market by the monopolist firm it is assumed that there are no close substitutes of the product

3 No Entry amp Exit Monopoly can only exist when there is strong barriers before a new firm to enter the market In fact once a monopoly firm starts producing the product no other firm can produce the same One reason for this is the ability of the

monopolist to produce the product at a lower cost than any new firm who thinks to enter the market If a new firm who knows that it cannot produce at a lower cost than the monopolist then that firm will never enter the market for fear of losing out in competition Similarly the monopolist who is operating for a long time may be enjoying reputation among its customers and is in a better position to use the situation in its own benefit A new firm has to take long time to achieve this and so may not be interested to enter the market

4 Price Maker Being the single seller of the product the monopolist has full control over the pricing of the product On the other hand if there is a large number of buyers in the market so no single buyer exercises any significant influence over price determination Thus it is a sellerrsquos market So monopoly firm is a price maker

5 Price Discrimination Having considerable control over the market on account of being single seller with no entry of other firms the monopolist can exercise policy of price discrimination it means that the monopolist can sell different quantities of the same product to a consumer at different price or same quantity to different consumers at different prices by adjudging the standard of living of the consumer

6 Shape of Demand Curve Since a monopolist has full control over the price therefore he can sell more by lowering the price This makes the demand curve downward sloping

Subject Ac-12 290620 Topic- retirement Model sumThe Balance Sheet of Rohit Nisha and Sunil who are partners in a firm sharing profits according to their capitals as on 31st March 2014 was as under

Liabilities Amount Assets Amount (Rs) (` Rs)

Creditors 25000 Machinery 40000Bills Payable 13000 Building 90000General Reserve 22000 Debtors 30000Capital Less Provision for Rohit 60000 Bad debts 1000

29000 Nisha 40000 Stocks 23000 Sunil 40000 140000 Cash at Bank 18000

200000 200000

On the date of Balance Sheet Nisha retired from the firm and following adjustments were made(i) Building is appreciated by 20(ii) Provision for bad debts is increased to 5 on Debtors(iii) Machinery is depreciated by 10(iv) Goodwill of the firm is valued at Rs 56000 and the retiring partnerrsquos share is adjusted

(v) The capital of the new firm is fixed at Rs120000 Prepare Revaluation Account Capital Accounts of the partner and Balance Sheet of the new firm after Nisharsquos retirement Revaluation AccountDr Cr

Particulars Amount Particulars Amount (`Rs) (Rs`)

Provision for Bad debt Ac 500 Building Ac 18000Machinery Ac 4000Profit transferred toCapital Accounts (3 2 2)Rohit 5786Nisha 3857Sunil 3857

13500

18000 18000

Capital Account

Dr Cr

Particulars Rohit Nisha Sunil Particulars Rohit Nisha Sunil (Rs`) (Rs`) (`Rs) (Rs`) (Rs`) (Rs`)

Sunilrsquos Capital ac 9600 mdash 6400 Balance bd 60000 40000 40000Bank - 66143 - General Reserve 9428 6286 6286Balance cd 72000 mdash 48000 Revaluation (Profi 5786 3857 3857 Rohitrsquos Capital Ac mdash 9600 mdash

Sunilrsquos Capital Ac 6400 Bank 6386 - 4257

81600 66143 54400 81600 66143 54400

Balance Sheet as at 31st March 2014

Liabilities Amount Assets Amount (Rs`) (Rs`)

Creditors 25000 Building 108000Bank overdraft 37500 Machinery 36000

Bills Payable 13000 Debtors 30000Capital Less ProvisionRohit 72000 for Bad debts 1500 28500Sunil 48000 120000 Stock 23000

195500 195500

Working Notes (i) (a) Profit sharing ratio is 60000 40000 40000 ie = 3 2 2(b) Gaining Ratio Rohit = 35 ndash 37 = 2135 ndash 1535 = 635Sunil = 25-27 = 1435 ndash 1035 = 435= 635 435= 6 4 = 3 2(c) Nisha Share of Goodwill = Rs 56000 times 27 = Rs16000Share of Goodwill in the gaining ratio by the existing partner ieRohit = Rs16000 times 35 = Rs 9600Sunil = Rs 16000 times 25 = Rs 6400

The journal entry isRohitrsquos Capital Ac Dr 9600Sunilrsquos Capital Ac Dr 6400 To Nisharsquos Capital Ac 16000(Share of Goodwill divided into gaining ratio)

  • 1 Static Friction
  • The frictional force that acts between the surfaces when they are at rest with respect to each other is called Static Friction
    • Static Friction Examples
      • 2 Sliding Friction
        • Examples Of Sliding Friction
          • 3 Rolling Friction
            • Examples Of Rolling Friction
              • Objects and Reasons of the Forest Conservation Act
Page 7:  · Web viewSubject . Topic . Summary . Execution . English 1 . Sounds of animals . Hens –cackle Horses –neigh Lions –roar Owls –hoots Snake –hiss. English 2 . Mother’s

6 Find the fractions in which the denominator is 3 more than the numerator69

74

811

49

1113

711

1411

1613

2023

Solution 69

811

2023

8 A book seller had 15 books He sold 8 books What fraction of the books did he sell

Solution Out of 15 books the book seller sold 8 books

there4 The fraction of the books he sold =8

15

10 A class has 35 students On Monday 34 students were present What fraction of the students was absent

Solution Total number of student = 35On Monday 34 students were presentthere4 The number of student absent on Monday = (35 ndash 34) = 1

So fraction of the students was = 135

Class IVSubject Topic Summary Execution English language

Verbs the -ing form

Meaning of verb A word used to describe an action is known as verb

The ndash ing form of a verb is a very important form for it is used in many different waysExampleJack is playing(Present continuous)Jack was playing with his friends yesterday ( Past continuous)Jack will be playing in a match tomorrow ( Future continuous)So we see that the ndash ing form of verb used to form the continuous tenses

The ndash ing form of verb can also be used as an adjectiveExampleYou should not disturb a sleeping dogThe kettle is full of boiling water

Say which of the ndash ingforms of verbs in the sentences given below have been used as adjectives and which to form continuous tenses ( solved exercises please follow this)

3Interesting ndash adjective

4Was eating ndash past continuous tense

5terrifying ndashadjective

6are helping is spreading ndash present continuous tense

7will be visiting ndashfuture continuous tense

8twittering ndash adjective

Social studies

Map reading A Map helps us to see the whole World continents countries cities and neighborhood They are of different sizes handy and can be rolled up folded or carried easily However the maps do not show the surface accurately This is because the map is flat whereas the Earth is round in shape Cartography or a map making is a study and practice representation of the

One word answers1 It helps us to see the whole world countries and cities ndash Map

2 A study and practice representation of the Earth on a flat surface ndash Cartography

Earth on a flat surface

Elements of a mapTo be able to read a map we must understand the elements of a map

DirectionsDirections are the basic guidelines that help us to locate places It is very important for us to know the correct directions East West North and South are the four Cardinal directions These directions help us to locate the places Beside these there are four sub directionsndash Northeast Southeast North West and South West Compass is an instrument that helps to locate the directions

3 The basic guidelines to help us to locate places ndash directions

4These four directions help us to locate the places ndash Cardinal directions

COMPUTER

Formatting in ms word

Q3) What do you mean by formattingAns) Formatting is a general arrangement of text in a document We can change the appearance of a document by using the features available in ms word We can use different fonts colours and styles in the textQ4) What is alignmentAns) Alignment of text is the way in which it is placed between the margins of a page Text can be aligned to the left side in the centre or to the right side of a pageQ5) what is fontAns) A font is a style of writing and typing A font provides specific textual appearance to the document You can change the size style or give various effects to a font

Hindi 2ndlang

ाला किहरण सभी धमlt ा मल धम ह दया और रणा मरी हर 0ीव अपन ही दकिनया म रहत ह हम

पराणी कयो उनो परशान रत ह ऐसा हम नही रना चाकिहए

0ानवरो स हम ए सीख मिमलती ह किस तरह हम एता म रहना चाकिहए

हर पराणी पयार ी भाा समझता ह अगर हम पयार ी उममीद रत ह तो कया 0त

0ानवर हमस उममीद नही रत कि हम भी उनह पयार द यह हानी ए ाल किहरण ी ह 0ो अपन

समह ा नततव रता था हमशा कया उलिचत ह यह सोचता था अचान

उन पर लिशारिरयो न हमला बोल दिदया जि0सम ाला किहरण परा गया कयोकि वह

दलभ ह और लिशारिरयो ी न0र उसी पर थी परत 0ानवरो ी एता ो दखर

रा0मार ो दया आ गई और उसन उनो छोड दिदया 0ानवर भी बईमान नही थी वह उनस अकसर मिमलन उन बाद म 0ाता रहता रा0ा ो धनयवाद रता अबोध म

पराणिणयो म एता तजञता अभी भी ह

शबदाथ तराई ndash पहाड आसपास नीच

ी भमिमवश- ल या परिरवारअदवभत- अनोखाचौननाndash सत सावधानओझलndash गायबकिवपणिRndash मसीबतसगठन- एताउममीदndash आशाातर दमिU ndashकिववशता

हिहस ndash हानी पहचान वालातवयndash फ0

दल ndash समहनततवndash सचालन रना

ओट ndash आडआतमसमपणndash अपन ो सौप दनाम- चप

हम उनस सीख लनी चाकिहएবইndashবাংলা সাহিতয পহিরচয়

পাঠndash১০লপndashহিবকেবকানকেFর যকেলকেবলাযলখকndashশশী-ষণ াশগNঅনশীলনীর পরকে4াততর

৯প পহিরবতG ন ককেরা -শরীর = শারীহিরক -ত = য-ৌহিতক সG ার = সG াহির সথায়ী = সথাহিয়তবহিবশবাস = হিবশবাসী া = যকো১০ বাকয রচনা ককেরা -আডডা = পাার বদধরা াতলায় আডডা যয়হিডবাহিজ = রাম া যকেক হিডবাহিজ যখকেয় পকেলাসG ার = বকেলহিল যকেল যর সG ারবকহিন = অঙক -ল করায় রীনা মাকেয়র কাকে বকহিন যখকেলাবহিদধ = বহিদধ াককেল উপায় য়হিনশহিত = হিনশহিত রাকেত রাজবাহিকেত ডাকাত পকেলা-য়ানক = পাাহি রাসতা বষটির পকের -য়ানক হিবপ জনক য়হিবশবাস = মানকেষর পরহিত হিবশবাস ারাকেনা পাপ

Science Ch ndash The Food We Eat

Living things need food to live to grow to stay strong and fit When we need food we feel hungry Food gives us energy to do work It also protects us from diseases and helps us to stay healthy Nutrients in food The food we eat contains many substances that are necessary for our body These substances are called nutrients that help us to grow well and stay healthy

Nutrients give us energy to study work and play

They help our body to grow and repair the damaged parts of our body

They also help our body to fight against diseases and remain healthy

Write T for True or F for False (pg no- 11)

1 Food contains nutrients that help us to grow True

2 Foods rich in carbohydrates are called body-building foods False

MAT

HEM

ATIC

S

Ch 9

Com

mon

Fra

ction

s

A fraction is a number that stands for parts of a whole object or a collection of objects

Each fraction has two numbers One is written above the other separated by a line The one above the line is called numerator and the one below the line is called denominator

Example 5minusminusminusminusrarr Numerator

11minusminusminusminusrarr Denominator

Exercise ndash 362 In following fill in the blanks

(b) 37 ___ is denominator ___ is

numerator

(c) 89 ___ is denominator ___ is

numerator

Solution

(b) 37 7 is denominator 3 is numerator

(c) 89 9 is denominator 8 is numerator

3 Write down the fractions whose numerators and denominators are given below in the bracket The first number stands for numerator and the second number standing for denominator

(25) 25

(311) 311

(416) 416

(712) 712

Class VSubject Topic Summary Execution

Science Ch ndash PlantReproduction

In pollination chapter we have learnt that the flowers change into fruits and the fruits bear seeds Now we are going to learn how this process takes place

FertilizationThe process of fusion of the male reproductive cell (male gamete) and female reproductive cell (female gamete) is known as fertilizationWhen a pollen grain reaches from the anther to a stigma it begins to grow and forms apollentube The Pollen tube then travels down through the style to enter an ovule inside the ovary On reaching the ovule male reproductive cell in the pollen grain unites with the egg cell present in the ovule

Books exercise

A) Tick the correct answer

1Which of the following do ovules change into after fertilization ndashseeds

2Which of the following is not a part of the seed ndash flower

3Which of the following condition is needed for germination of a seed ndash all ofthese

English language

Sentences phrases and

Solved exercisesSay which of the underlined groups of words are phrases and which clauses

clauses8 In her new clothes ndashphrasesAs pretty as a doll ndash phrases

9 looking sad and upset ndash phrasesHe had lost all the tickets for the Test Match ndash clauses

10 During the vacation now only a month away ndashphrases

11 too sweet and too hot ndashphrases

12 At the Olympic Games ndashPhrasesOf Laurel leaves ndash phrases

13 Who played the role of Hamlet ndashclauses

14 However fast ndash phrases

15 When the men fell asleep ndash clausesSocial studies

Indian Government

Lok Sabha (lower house) ndash It has 552 members Of these 530 membersrepresent States 20 members represent the union territories and two members represent the Anglo- Indian community All except the representatives of the Anglo-Indian community are elected by Indian citizens A person above the age of 25 can contest in the elections for Lok Sabha One term of Lok Sabha is for 5 yearsRajya Sabha (upper house) ndash Its members are elected by the MLAs or members of the legislative Assembly There are 250 members in the Rajya Sabha of which 12 are nominated by the President One term of Rajya Sabha is for 6 years Anyone above the age of 30 can be elected as a member of Rajya Sabha

ExecutivePresidentThe President is the head of the country in India He is elected by the MPs and the MLAs for a tenure of 5 years He appoints the Prime Minister and the Council of Ministers Prime MinisterThe party which wins the election forms the government and its leader is elected as the Prime Minister He is the chief advisor to the President The Council of Ministers assists the Prime Minister and is accountable for their roles For example the Education minister is responsible for the education system in our country

1 How many members are there in Lok SabhaAns 552 members

2 What is the term for Lok SabhaAns 5 years

3 How many members are there in the Rajya SabhaAns 250 members

4 Who is the head of our countryAns President

5 Who is the chief advisor to the PresidentAns Prime Minister

Book ndash GK

Ch ndash 1First in space

1 First living being into space in 1957 Ans Laika

Times 2 First person to go into space in 1961 Ans Yuri Gagarin

3 First woman to go into space in 1963 Ans ValentinaTereshkova4 First person ever to walk in space in 1965Ans alexei Leonov5 First person to land on the moon in 1959 Ans Neil Armstrong6 First Indian to go into space in 1984 Ans Rakesh Sharma 7 First Indian woman to go into space in 19978 Ans Kalpana Chawla9 First woman tourist in space in 2006

Ans Anusheh AnsariCOMPUTER

ALGORITHM AND FLOWCHART

Q) DRAW THE SYMBOLS USED IN A FLOWCHART WITH THEIR DESCRIPTIONS(IN EXAM IT CAN COME AS SHORT QUESTIONS ASKING INDIVIDUAL SYMBOLS FUNCTION)ANS)

MAT

HEM

ATIC

S

Ch 6

Com

mon

Fra

ction

s

Multiplication of FractionsA Multiply a fractional number by whole numberTo multiply a fractional number by whole number we multiply the numerator of the fractional number by the whole number and denominator of the fractional number by 1 The first product thus obtained is the numerator and the second product is the denominator of the required product

Exercise ndash 30Multiply

7 2027

times 9

Solution 2027

times 9 = 203 = 6

23

8 611

times11

Solution 611

times11 = 6

15 71

20times16

Solution 71

20times16 =

14120

times16

= 1415

times 4 = 141times 4

5 = 564

5 = 11245

B Multiplication of a fractional number by a fractional number To multiply a fractional number by a fractional number we multiply the numerator of the first fractional number by the numerator of the second fractional number and the denominator of the first fractional number by the denominator of the second fractional number The first product thus obtained is the numerator and the second product is the denominator of the required product

16 2712

times24

Solution 2712

times24 = 3112

times24

= 31times2 = 62

Exercise ndash 31

11 83

times 34

2

Solution 83

times 34 = 2

14 723

times2 25

4

Solution 723

times2 25 =

233

times 125 =

23times 45

= 925 = 18

25

15 1212

times1 13

2

Solution 1212

times1 13 =

252

times 43 =

25times 23

= 503 = 16

23

State the following statements are true or false

17 1912

times 239 = 1

Solution LHS = 1912

times 239

= 392

times 239 = 1 = RHS

[LHS = Left hand side amp RHS = Right hand side]

there4 1912

times 239 = 1 [True]

21 213

times2 13 = 4

19

Solution LHS = 213

times2 13 =

73

times 73

= 7times73times3 =

499 = 5

49

there4 LHS ne RHS

So 213

times2 13 = 4

19 [False]

23 23

times 45 =

2times 5+3 times43times 5

Solution

LHS = 23

times 45 =

2times 43 times5 =

815 again

RHS = 2times 5+3 times4

3times 5 = 10+12

15 = 2215

there4 LHS ne RHS So 23

times 45 =

2times 5+3times43times 5

[False]

25 23 of

13 =

29

Solution

LHS= 23 of

13 =

23 times

13 =

29 = RHS]

there4 23 of

13 =

29 [True]

Practice at HomeExercise ndash 31State the following statements are true or false

24 12 of 4 =

18

Class VISubject Topic Summary Execution

HISTORY AND CIVICS

Chapter 5The Mauryan Empire

DECLINE OF MAURYAN EMPIREDecline of Mauryan empire started after the death of Ashoka at around 232 BCThere are several reasons for break up of the empire1 Weak successor Emperors after Ashoka were

capable of handling vast and mighty Mauryan empire In 185BC the last Mauryan ruler Bri-hadrath was murdered by his Commander-in-Chief Pushyamitra Sunga

2 Provincial Revolts Due to weak central author-ity provincial chiefs of Kalinga and southern provinces revolted against emperor and freed themselves from Mauryan empire

3 Weakness of Economy Prosperity of Mauryan was based on solid economic activities which

ExercisesI Multiple choice questions-1 Chandragupta defeated Seleucus in the year ndashc) 305 BC2 Who killed the last Mauryan ruler Brihadrath b) Pushyamitra3 Which of the following was not a reason for the decline of the Mauryan empirec) Chandraguptarsquos weakness4 Ashoka invaded Kalinga in the year c) 261 BC

II Fill in the blanks1Chandragupta ascended the throne in 324

was taken care by early monarchs Later kings had neither ability nor interest in economic af-fairs That led to failure in tax collection As a result they failed to maintain a large army that were essential to keep empire intact

4 Greek Invasion Greeks freed north-western provinces from weak Mauryan monarchs and reestablished their authority

5 Ashokarsquos Policy some scholar opined that after Kalinga war Ashoka embraced Buddhism re-nounced the policy of war and disbanded the Army But this is partially true as there is no proper evidence of disbanding the army

Based on above points we can conclude that main reason for decline of Mauryan empire is weakness of Ashokarsquos successors Kunal Samprati Dasharath Salisuk all were weak kingsAt last in 185 BCPushyamitra Sunga killed king Brihadrath and established the Sunga dynasty

BC2 Bindusara was the son of Chandragupta and father of Ashoka

3 Pataliputra was administered by City Magistrate committess of 5 members each4 The Greek General Seleucus sent his ambassador Megasthenes to Chandraguptarsquos court5 Ashoka sent his son Prince Mahendra and daughter Sanghamitra to spread his Dhamma6 The Indian Rebublic has adopted the Lion Capital of Saranath Pillar as its national emblem 7 Pushyamitra killed the last Mauryan ruler Brihadrath and founded the Sunga dynasty

III Name the following

1The author of Arthashastra-Kautilya2 The ruler who founded the Mauryan dynasty-Chandragupta3 The author of Indika-Megasthenes 4 The officers who were appointed by Ashoka to spread Dhamma-Dhamma Mahamatras5 The general of Alexander whom Chandragupta defeated-Seleucus

V Match the columns1 Kautilya (c)2 Megasthenes (d)3 Pushyamitra (e)4 Brihadrath (b)5 Bindusara (a)

BENGALI(2ND

LANGUAGE)

পশপাহিখর -াষাসহিবনয় রায়কেচৌধরী

যলখক পহিরহিচহিত- পরখযাত সাহিহিতযক উকেপনদরহিককেশার রায়কেচৌধরীর পতর সহিবনয় রায়কেচৌধরী lsquoসকেFশrsquo পহিতরকার সকেb হিতহিন কত হিকেলন তার উকেdখকোয বই lsquoসহিবনয় রায়কেচৌধরীর রচনা সংগরrsquo

পরম হিকেনর পাঠ- lsquoপশপাহিখর হিক -াষাhelliphellip helliphelliphelliphellipপরসপরকেক জানাবার উপায়ও পশপাহিখরা যবশ জাকেনrsquoপরকেমই আমারা জাহিন -াষা হিক -াষা ল আমাকের মকেনর -াব পরকাশ করার জনয আমরা নানান ধরকেনর -হিb বা হিবকেশষ ধরকেনর আওয়াজ মকেখর মাধযকেম কহির অনযকেক যবাঝাকেনার জনয তাকেল এবার আমরা জাহিন পশপাহিখর -াষা হিক পশ পাহিখরা হিক কা বকেল যা পশপাহিখকেরও -াষা আকে তারা তাকের হিনজসব -াষায় কা বকেল মকেনর -াব পরকাশ ককের পশ পাহিখরা মানকেষর হিক হিক -াষা যবাকেঝ হিকনত তারা বলকেত পাকেরনা পরসপরকেক বহিঝকেয় যবার উপায় তারা জাকেননা তকেব তারা হিবকেশষ ককেয়কটি শকেবদর মাধযকেম তাকের মকেনর -াব বহিঝকেয় যয় হিক বহিদধ মান জীব ndashককর হিবাল বন মানষ যঘাা পর-হিত এরা মানকেষর যওয়া নাম শনকেল কান খাা ককের ndash নাম ধকের ডাককেল কাকে আকেস যমন - মরহিরা lsquoহিত ndashহিতrsquo ডাক শকেন আকেস াল lsquoঅ ndashর -র ডাক শকেন কাকে আকেস াহিত মাহকেতর কা শকেন চকেল ককররা মাহিলকেকর হকম পালন ককের সবসময় তাইকেতা ককরকেক পর- -কত পরানী বলা য় ককর আর হিবাল একের আওয়াজ তহিম লকষয করকেল বঝকেব ককররা যরকে যকেল lsquoযঘউ যঘউrsquo করকেত াকেক আবার কাকেল lsquoযকউ যকউrsquo ককের হিবাল সাধারণ lsquoমযাওrsquo বা lsquoহিমউrsquo ককের রা কেল lsquoওয়াওrsquo আওয়াকেজর মাধযকেম মকেনর -াব পরকাশ ককের একেতা যল পশকের কা পাহিখরাও -য় রা পরকাশ করার জনয হিবকেশষ ধরকেনর শবদ ককের হিবপকের সময় পশ পাহিখরা সবার আকে পরসপরকেক জানাবার উপায় তারা জাকেন বহকাল

১) পশপাহিখর -াষা কেলপর যলখক সমপকেকG হিক জাকেনা

উঃ- পরখযাত সাহিহিতযক উকেপনদরহিককেশার রায়কেচৌধরীর পতর সহিবনয় রায়কেচৌধরী lsquoসকেFশrsquo পহিতরকার সকেb হিতহিন কত হিকেলন তার উকেdখকোয বই lsquoসহিবনয় রায়কেচৌধরীর রচনা সংগরrsquo হিতহিন ারকেমাহিনয়াম এসরাজ পর-হিত বাযনতর বাজাকেত পারকেতন ানও জানকেতন হিতহিন যাকেIাকের জনয মজাার লপ কহিবতা হিলখকেতন

২) পশপাহিখ কেলপর মল-াব হিকউঃ- পশপাহিখকেরও -াষা আকে তারা তাকের হিনজসব -াষায় কা বকেল মকেনর -াব পরকাশ ককের পশ পাহিখরা মানকেষর হিক হিক -াষা যবাকেঝ হিকনত তারা বলকেত পাকেরনা পরসপরকেক বহিঝকেয় যবার উপায় তারা জাকেননা তকেব তারা হিবকেশষ ককেয়কটি শকেবদর মাধযকেম তাকের মকেনর -াব বহিঝকেয় যয় হিরউকেবন কযাসটং সাকেব হিতহিন চহিdশ বর বনযজনত যর সকেb যকেককেন হিতহিন বকেলকেন আমরা হি তাকের -াষা তাকের আব কায়া যমকেন চহিল তাকেল আর -কেয়র যকান কারণ াকেকনা আমরা একI -াকেলাকেবকেস যচষটা করকেল পশপাহিখকের সকেb -াব পাতাকেত পাহির

ধকের মানষ এই পশ পাহিখর -াষা হিনকেয় নানা রককেমর পরীকষা ককের আসকে এইরকম একজন হিরউকেবন কযাসটাং সাকেকেবর কা আমরা জানকেবাhelliphellip

Hindi 2nd

langमतर किनमनलिलखिखतपरशनोउRरदीजि0ए

) बढ वयलि` बचच ो कया हआ था ख) डॉकटर साहबन पाटc किस उददशय स रखी थी ग) ाल साप ो हाथ म लर लाश न कया किया घ) डॉकटर चडढा न बढ पतरो दखन स कयो मना र दिदया था ङ) भगत न लाश ो दखर कया हा

उRर ndash) उस बहत बखार थी और 4 दिदनो स आख भी नही खोला थाख) उन बट ी सालकिगरह थीग) ाल सापो हाथ म लर लाश न उसी गदन 0ोर स दबार पडी थीघ) डॉकटर चडढा न बढ वयलि` पतरो दखन स मना र दिदया कयोकि उनह गोलफ खलन 0ाना थाङ) लाश ो दखर हा कि नारायण चाहग तो आध घट म भया उठ 0ाएग

English literature

In the bazaars of Hyderabad- Sarojini Naidu

Through the poem In The Bazaars of Hyderabad Sarojini wanted to convey the message that India is rich in tradition and they donrsquot need the foreign products So she goes on to give a picture of a bazaar where traditional Indian products are rulingThe poem is in the form of questions and answers The poet asks the questions and the merchants answer them Through this technique she make the picture of the bazaar visible to us

Read the poem

PHYSICS FORCE Types of FrictionThere are three types of friction static sliding rolling Static sliding and rolling friction occur between solid surfaces

1 Static Friction The frictional force that acts between the surfaces when they are at rest with respect to each other is called Static FrictionStatic Friction Examples

Skiing against the snow Creating heat by rubbing both the hands

together Table lamp resting on the table

2 Sliding Friction The resistance that is created between any two objects when they are sliding against each other is called Sliding FrictionExamples Of Sliding Friction

Sliding of the block across the floor Two cards sliding against each other in a

deck

3 Rolling Friction The force which resists the motion of a ball or wheel is called Rolling Friction Is the weakest types of frictionExamples Of Rolling Friction

Rolling of the log on the ground Wheels of the moving vehicles

6What effect can a force produce on a body which is not allowed to move Ans - When a force is applied on a body which is not free to move it gets deformed i e the shape or size of the body changes7Give one example each to indicate that the application of a force

1 produces motion2 stops motion3 slows down motion4 changes the direction of motion5 deforms a body

Ans- 1 A car originally at rest when pushed

begins to move2 A moving bicycle is stopped by

applying the brakes3 The speed of a moving vehicle is

slowed down by applying the brakes4 A player kicks a moving football to

change its direction of motion5 On stretching a rubber string its

length increases

8State the effect produced by a force in the following cases (a) The sling of a rubber catapult is stretched(b) A man pushes a heavy cart(c) A player uses his stick to deflect the ball (d) A cyclist applies brakes(e) A spring is compressedAns- (a) The shape and size of catapult changes ie its length increases(b) The heavy cart begins to move(c) The direction of the ball changes(d) The speed of the moving cycle is slowed down(e) There is change in size and shape of spring

COMPUTER MS EXCEL 2013 -INTRODUCTION

UNDERSTANDING EXCEL STRUCTUREA SPREADSHEET IS A FILE THAT EXISTS OF CELLS IN ROWS AND COLUMNS AND CAN HELP ARRANGE CALCULATE AND SORT DATA DATA IN A SPREADSHEET CAN BE NUMERIC VALUES AS WELL AS TEXT

FORMULAS REFERENCES AND FUNCTIONS

WORKSHEETA WORKSHEET IS ALSO KNOWN AS SPREADSHEETIT IS A COLLECTION OF CELLS ON A SINGLE SHEET WHERE YOU KEEP AND CHANGE DATA

WORKBOOKWORKBOOK IS PMS EXCEL FILE IN WHICH THE DATA CAN BE STORED EACH WORKBOOK CAN CONTAIN MANY WORKSHEETS

ROWS AND COLUMNSIN MS EXCEL A ROW IS A GROUP OF CELLS THAT RUN FROM LEFT TO RIGHT OF A PAGEA COLUMN IS A GROUPING OF CELLS THAT RUN FROM THE TOP TO THE BOTTOM OF A PAGE

CELLTHE INTERSECTION POINT BETWEEN A ROW AND THE COLUMN IS CALLED A CELL WHICH IS THE BASIC STORAGE UNIT FOR DATA IN A SPREADSHEET EACH CELL HAS SPECIFIC ADDRESS WHICH IS THE COMBINATION OF THE COLUMN NAME FOLLOWED BY THE ROW NUMBER

CHEMISTRY Chapter ndash Common Laboratory Apparatus and equipments

Objective type questionFill in the blanks (a) Experiment and observation are the two important basics of chemistry(b) A porcelain dish is used for evaporation(c) A test tube holder is used to hold the test tube while-it is heated(d) Mortar and pestle is used for grinding and crushing solid substances into a powder(e) Glass apparatus is made of Pyrex or borosil glass

Class VIISubject Topic Summary Execution

Hindi 2ndlang

ए था राम( डॉ शरी परसाद)

सगकित ा परभाव मानव 0ीवन पर अवशय पडता ह

हमशा मनषय ो अचछो ी सगकित म रहना चाकिहए

शरषठ परो सग स मनषय चरिरतर ा शीघर ही उदय और किवास हो 0ाता

ह इसलिलए वयलि` ो सदा शरषठ परो ा ही सग रना चाकिहए

इसान अगर चाह वह सवय ो बदल भी सता ह

यह हानी राम ए बचच ी हवह गणिणत ी परीकषा म नल रत हए पडा 0ाता ह और उस अधयाप पडत ह और पछत ह यह कया र रह

हो तभी राम न उनी बइजजती ी

शबदाथब ndashहावा भलावाायवाहीndash ाम किनयम व ानन

ो दिदखानापरिरलिचतndash 0ाना पहचानाघटनाndashघबराहट

उलटा चोर ोतवाल ो डाटndashकिववndash भल बर ा जञानतवयndash म 0ो रना चाकिहएसगकितndash बरी सगत

किबलख नाndashरोना किनशचय रनाndash तय रना

फलndashपरिरणामकिनषालिसतndash बाहर किया हआपशचातापndashदख सपननndashधनी

ldquo हा आपी किहममत स हई नल रत पडन ीrdquo ऐसी बात ही किफर

किपता0ी न भी उस डाटा वह ाफी पशचाताप रन लगा बोला गलत दोसतो

ी सगकित म आ0 कितना अनथ र दिदया किफर उसन अधयाप स माफी मागन ी सची और किफर भी ऐसा

नही रगा यह परण भी लिलया

सोचndashकिहच एात-अला

বইndashবাংলা সাহিতয পহিরচয়

পাঠndash১৬লপndashস-য ও অস-যযলখকndashঈশবরচনদর হিবযাসারঅনশীলনীর পরকে4াততর

৬ অGকেলকেখা -ময়া = পশ হিশকার সহিtহিত = হিনকIবতu সbভরষট = লI হিনরীকষণ =

-াকেলা-াকেব যখাকতাঞজহিলপকেI = যজাাকেত৭ হিবপরীতশবদ -ঈষৎ times পরচর উৎকষট times হিনকষট তাশ times উৎফd তবহিদধ times

বহিদধীNপাহিপষঠ times পণযবান৮ পপহিরবতG ন ককেরা -পশ = পাশহিবক যকাপ = যকাহিপতহিসথর = হিসথরতাএকানত = ঐকাহিনতক পর-াত times পর-াতী

CHEMISTRY

Chapter ndashPhysical and Chemical Changes

Chemical ChangeA chemical change involves a change in chemical composition

Characteristics of Chemical changes 1 They are permanent changes2 They are irreversible changes 3 New substance formed4 A Chemical change involves a

change in its chemical properties

Pg-25Question 8What do you observe when1 water is boiled2 a piece of paper is burnt3 some ice cubes are kept in a glass tumbler4 solid ammonium chloride is heated5 an iron nail is kept in tap water for few days6 a spoon of sugar is heated in a pan7 lighted match stick is brought near the mouth of the test tube containing hydrogen gas8 quick lime is dissolved in water9 little amount of curd is added to a bowl containing warm milk and kept for five hours

10 Water is boiledOn boiling water changes into steam (gas) physical change

11 A piece of paper is burnton burning piece of paper produces carbon dioxide and ash is left behind Is a chemical change

12 some ice cubes are kept in a glass tumblerIce cubes (solid) turn into water

(liquid) only state changes (physical change)

13 Solid ammonium chloride is heatedSolid ammonium chloride on heating changes into vapors (change of state) is physical change

14 An iron nail is kept in tap water for few dayswe observe reddish brown coating on the nail called rust (entirely new substance) is chemical change

15 A spoon of sugar is heated in a panWhen a spoon of sugar is heated in a pan black (charred sugar) (carbon) is seen Is a chemical change

16 Lighted match stick is brought near the mouth of the test tube containing hydrogen gasWe observe that hydrogen bums at the mouth of test tube with blue flame and pop sound is heard It is chemical change

17 Quick lime is dissolved in waterThe following two observations will be observed (i) A hissing sound is observed(ii) The mixture starts boiling and lime water is obtained

18 Little amount of curd is added to a bowl containing warm milk and kept for five hoursWhen a little amount curd is added to a bowl containing warm milk and kept for five hours a permanent change occurredThe milk will change to curd On boiling water changes into steam (gas) physical change

GEOGRAPHY

ATMOSPHERE IMPACT OF GLOBAL WARMING The destructive impart of global warming is observed in various spheres of life and the environment Some of the points are outlined below1 High temperatures lead to high

evaporation rate and drying up of the soil and surface water This affects crop production The occurrence of droughts is aggravating the problem even further

2 The heat waves in summer months

Q1 Write some impact of global warmingA1 The impacts of global warming are as follows1 High temperatures lead to high

evaporate ion rate and drying up of the soil and surface water This affects crop production The occurrence of droughts is aggravating the problem even further

2 The heat waves in summer months lead to a greater number

lead to a greater number of deaths due to heat strokes

3 Forest fires become more frequent4 Tropical cyclones and hurricanes

become common5 Melting of glaciers takes place6 Polar ice caps are becoming thinner

and melting at an alarming rate due to global warming The loss of sea ice

7 Due to increase in sea surface temperature sea levels rise in coastal areas and cause submergence of several islands

WAYS TO REDUCE GLOBAL WARMINGFollowing steps can be taken We need to decrease emission of

green house gases by reducing the burning of fossil fuel such as coal and petroleum

By planting more trees to increase forest cover

The government should also distributes free saplings and organize afforestation programmes to spread awareness regarding the beneficial effects of trees

We should switch to eco-friendly cars and gadgets

Incandescent light bulbs should be replaced by CFL bulbs

We can save electricity and reduce global warming by turning off electrical gadgets such as lights fans air-conditioners television and computer when we do not to use them

Efforts should be made to hasten the development of green cities oreco cities These cities are urban areas around the world striving to lessen the environment a impacts of urbanization

By following the 3Rs-Reduce Recycle and Reuse strategy we can use natural resources for our growth as well as save them for the need of the future generations This is called sustainable development

of deaths due to heat strokes3 Forest fires become more

frequent4 Tropical cyclones and hurricanes

become common5 Melting of glaciers takes place

etc

Q2 How to reduce global warmingA2 Following steps can be taken to reduce global warmingaWe need to decrease emission of

green house gases by reducing the burning of fossil fuel such as coal and petroleum

bBy planting more trees to increase forest cover

c The government should also distributes free saplings and organize afforestation programmes to spread awareness regarding the beneficial effects of trees

dWe should witch to eco-friendly cars and gadgets

eIncandescent light bulbs should be replaced by CFL bulbs

f We can save electricity and reduce global warming by turning off electrical gadgets such as lights fans air-conditioners television and computer when we do not to use them

Q3 What do you mean by 3Rrsquos of resource planningA3 The 3Rs are

1 Reduce 2 Recycle and3 Reuse

Q4 What is Sustainable developmentA4 By following the 3Rs-Reluce Recycle and Reuse strategy we can use natural resources for our growth as well as save them for the need of the future generations This is called sustainable development

English Language

Prepositions A preposition is a word placed before a noun or a pronoun It helps to show how the person or thing denoted by the noun is related to something else in the sentence

Kinds of Prepositions

Simple Prepositions- simple preposition are one word Prepositions such as at by for in of off for from on out through till to up with before amidst towards beyond between over etc

Compound Prepositions ndash There are some words that are always used with fixed Prepositions to convey specific meaning

Example I was unable to meet you dueto a previous engagement ( On account of)Always maintain the queue instead of crowding at the counter ( In place of)

Participial PrepositionsmdashParticiple Prepositions are present or past participles of various verbs which together with a noun phrase or a clause function as prepositions Examples- barring concerning considering notwithstanding pending regarding respecting etc

Exercise A

1 Gauravs fever has come down since Friday He has been absent for a week now

2 The child sat between his father and mother among the parents of all his classmates

3 There are mosquitoes in the room They flew into the room when the door was open

4 My father was inside the drawing room when I was playing outside my house

5 You may sit beside me I will give you a drawing book and pencils besides a storybook

6 We went to the market in the morning and walked towards the riverfront in the evening

7 The child walked along the pavement and across the street safely

8 This table top is made of glass My breakfast fell off it in the morning

9 The pan is on the gas stove There are vegetables in it

10 We will wait for you at the bus top There are a lot of people in the hall

Subject ndash Biology Topic ndash Chapter - 3 Photosynthesis and respiration in plants Summary Execution

All living organism (Plants and animals) need food for energy and growth Green plants (autotrophy) prepare food for all living organisms Today we will discuss about the process photosynthesis And adaptations in a leaf to carry out photosynthesis

Q1What do you mean by photosynthesis and write its word equation The process by which green plants make food (glucose) from carbon dioxide and water

in the presence of sunlight and chlorophyll is called photosynthesis

Carbon dioxide + Water ( Sun light from Sun ) Glucose + Oxygen ( chlorophyll in green leaves )

Q2 What are the adaptations in a leaf to carry out photosynthesisi) Leaves are broad wide and flat for absorbing more light energyii) Presence of chlorophyll in chloroplasts to trap sunlightiii) Presence of stomata which allow carbon dioxide to enter the cell and oxygen to go

out iv) Network of veins ensures continuous supply of water and minerals to the leafv) Thin waxy cuticle protects the leaf without blocking the lightQ3 Draw and label structure of chloroplast

Class VIIISubject Topic Summary Execution

PHYSICS ENERGY Production of Hydro electricity

A hydroelectric dam converts the potential energy stored in a water reservoir behind a dam to mechanical energymdashmechanical energy is also known as kinetic energy As the water flows down through the dam its kinetic energy is used to turn a turbine

The generator converts the turbinersquos mechanical energy into electricity

This electric energy then goes through various transmission processes before it reaches you

Question 2

Fill in the blanks

(a) Work is said to be done by a forte only when the body moves

(b) Work done = Force x distance moved in direction of force

(c) The energy of a body is its capacity to do work

(d) The SI unit of energy is joule

(e) The potential energy is due to its state rest of position and kinetic energy of the body is due to its state of motion

(f) Gravitational potential energy U = mass times force of gravity on unit mass times height

(g) Kinetic energy = frac12 times mass times (speed)2

(h) Power P = work donetime taken

(i) The S I unit of power is watt

(j) IHP = 746 W

BIOLOGY Chapter -5 The endocrine system and adolescence

Today we will discuss about thelocation and functions of secreted hormones of adrenal and Pancreas

Q5 Write location hormone secreted main functions and deficiency diseases of pancreas and adrenal glands

Endocrine Glands

Location Hormones secreted

Functions and Deficiency Diseases

1Adrenal gland

2 Pancreas Gland

On the top of each kidney

In between stomach and small intestine

i)Adrenaline from adrenal medulla

ii)Cortisone from adrenal cortex

i) Insulin

ii) Glucagon

It helps a person deal with any kind of emergency situation or emotional stressIt increases the heart beat rate of respiration and blood pressure

a) It regulates carbohydrates protein and fat metabolism

b) It regulates the salt and water balance in the body

a) It changes excess glucose into glycogen

b) It stimulates the cells to burn extra glucose to provide heat amp energy

Less secretion causes diabetes mellitus

Excessive secretions causeinsulin shock

a) It stimulates the breakdown of glycogen into glucose

b) It increases the level of glucose in blood

History Traders to rulers The Battle of Buxar was fought on 22 October 1764 between the forces under the command of the British East India Company led by Hector Munro and the combined armies of Mir Qasim the Nawab of Bengal till 1763 Mir Jafar was made the Nawab of Bengal for a second time in 1763 by the Company just after the battle After being defeated in 4 battles in katwa and Udaynala the Nawab of Awadh Siraj id Daula and the Mughal emperor Shah Alam II accompanied by Raja Balwant Singh of Kashi made an alliance with Mir Qasim The battle was fought at Buxar a small fortified

Answer the following questions- Short note-Battle of BuxarHomework-learn

town within the territory of Bihar located on the banks of the Ganga river about 130 kilometres (81 mi) west of Patna it was a decisive victory for the British East India Company The war was brought to an end by the Treaty of Allahabad in 1765

EnglishLiterature

The west wind-John Mansfield

In the poem The West Wind by John Masefield the poet starts by describingwith very poetic imagery of birds how the west wind is different from other winds its a warm wind full of birds cries There is a touch of melancholy perhaps home-sickness as he describes how it brings tears too and memories from an old land He goes on to describe the restful pastoral beauty of the land where even the dead can lie in the green He then brings in voicesperhaps of family and friends calling him home as he is missing Aprils beautyThe voices then tempt him some more with idyllic images from home (white blossom young green cornrunning rabbitswarm sun) The voices seem to presume that the poets heart is sorrowful bruised and soreThe end of the poem sees the poet appear to make a decision he will go home as he has decided that is where he truly belongs

Write the synopsis of the following words

1 Daffodils- a tall yellow flower that grows in the spring

2 Orchards- a piece of land on which fruit trees are grown

3 Blossom- a flower or a mass of flowers especially on a fruit tree in spring

4 Thrushes- a bird5 Larks- a small brown bird that

makes a pleasant sound6 Bruised- an injury7 Aching- pain 8 Tread- to put your foot down

while you are walking9 Balm-10 May-11 Fluting-

(Write from the book in your copy)

MAT

HEM

ATIC

S

Ch 1

1Al

gebr

ic E

xpre

ssio

n

1 Constant A symbol which has fixed value is called a constant[eg 8 23 -15 radic3 etc]

2 VariableA symbol which does not have any fixed value but may be assigned value (values) according to the requirement is called variable or literal[eg x y p q etc]

3 TermsA term is a number (constant) a variable a combination (product or quotient) of numbers and variables[eg 7 x 5x etc]

4 Algebric expressionA single term or acombination of two or more terms connected by plus (+) or minus (-) sign forms an algebraic expression[eg 5-y 3x2-5x xy-6z+4 etc]

5 PolynomialAn algebraic expression which contains more than one term is called a polynomial (multinomial)[eg x2-5x 5y+xy+x2y etc]

6 Degree of polynomial(a) When the polynomial contains only one variable the highest power of the variable is the degree of the polynomialeg the degree of the polynomial of 4x-7x5+8 is 5(b) When the polynomial contains two or more variablesStep (i) Find the powers of the variables in each term (ii) The highest sum of the powers is taken to be the degree of the polynomialeg the degree of the polynomial 5x2y-4x3y5+6 is = 3+5 = 8Remember An algebraic expression is a polynomial if degree of each term used in it is a non-negative integer

Exercise ndash 11(A)

1 Separate the constants and variables from the following

-7 7+x 7x+yz radic5 radic xy 3 yz

8 45y -3x

Solution Constant Variables-7 radic5 7+x 7x+yz radic xy

3 yz8

45y -3x

2 Write the number of terms in each of the following polynomials(i) 5x2+3timesax (ii) axdivide4-7 (iii) ax-by+ytimesz (iv) 23+atimesbdivide2

Solution Polynomials Number of terms(i) 5x2+3timesax 2(ii) axdivide4-7 2(iii) ax-by+ytimesz 3(iv) 23+atimesbdivide2 2

4 Write the degree of the each polynomials(i) xy+7z (ii) x2-6x3+8 (iii) y-6y2+5y8 (iv) xyz-3 (vi) x5y7-8x3y8+10x4y4z4

Solution Polynomials Degree(i) xy+7z 2(ii) x2-6x3+8 3(iii) y-6y2+5y8 8(iv) xyz-3 3(vi)x5y7-8x3y8+10x4y4z4 12

5Write the coefficient of(i) ab in 7abx (iv) 8 in a2-8ax+a (v) 4xy in x2-4xy+y2

SolutionCoefficient

(i) ab in 7abx 7x(iv) 8 in a2-8ax+a -ax(v) 4xy in x2-4xy+y2 -1

7 CoefficientAny factor of an algebraic quantity is called the coefficient of the remaining quantityeg in the algebraic term 7xyz 7 is coefficient of xyz 7x is coefficient of yz and so on

8 Like term The terms having the same literal coefficient are called like terms and those having different literal coefficients are called unlike terms

eg (i) 5xyz 8xyz -6xyz and 23xyz are like

terms(ii) 7xy2 8x2yz and -15xyz2 are unlike terms

6 in 57xy2z3 write the coefficient of

(i) 5 (vii) 5xy2 (viii) 17yz (xi) 5xyz

Solution Coefficient

(i) 5 17

xy2z3

(vii) 5xy2 17z3

(viii) 17yz

5xyzsup2

(xi) 5xyz 17yz2

7 In polynomial given below separate the like terms(ii) y2z3 xy2z3 -58x2yz -4y2z3 -8xz3y2 3x2yz and 2z3y2

Solution y2z3 -4y2z3 2z3y2 are like terms

xy2z3 -8xz3y2 are like terms

-58x2yz 3x2yz are like terms

Class IXSubject Topic Summary Execution

Bengali (2nd language)

বাগzwnjধারাzwnj বা ধারা-বা ধারা ল হিবকেশষ পরকার বাক -হিb -াকেবর এক হিবকেশষ পরকাশরীহিত াকেক কতগকেলা কার সমষটির মকেধয এগহিলকেক বা ধারা বকেল আবার কতগকেলা শকেবদর বাধাধরা যকান রীহিত যনই য-াকেব চকেল আসকে যসই -াকেবই চকেল আসকে তখন যসই শবদগহিল খন একক -াকেব অG পরকাশ ককের তখন একের বা ধারা বকেল বা ধারার পরকেয়া -াষাকেক আরও সFর ককের যতাকেল

অকাল পকক(অপহিরনত বয়কেস পাকাহিম)-মাতর শ বর বয়কেস যমকেয়টির া মকেখর কা তাকেত অকালপককতা ধরা পকে

অককা পাওয়া( মারা াওয়া) ndash পকেকIমারটি পকেকIমারকেত হিকেয় বাসাতরীকের াকেত মার যখকেত যখকেত অককা যপল

অহি| পরীকষা ( কঠিন ও পরকত পরীকষা)- যকেলটির আজ ডাকতাহির যরজালট যবকেরাকেব এIাই তার জীবকেনর ব অহি| পরীকষা

অষটরমভা (ফাহিক) ndash রীতা মকেখই বকো বকো কা বকেল আর কাকেজর যবলায় অষটরমভা

অকমGার ধাী (অপাG) ndash সমনকেক হিনকেয় যকান ান কেব না ও একেকবাকেরই অকমGার ধাী

অকেনধর ষটি (অসাকেয়র সায়)- আহিশ বকেরর বকোর নাহিত ল অকেনধর ষটি তাকেক াা বকোর একম চকেল না

আকেককল গড়ম (তবহিদধ)- ার তহিম উপকার করকেল যসই যতামার হিবরকেদধ সাকষয হিকেয়কে শকেনই আমার আকেককল গড়ম

আষাকে লপ( অবাসতব লপ) ndashIাকা এখন যকেব না এIা বলকেলই ত এমন আষাকে লপ ফাার যকান রকার হিল না

Hindi- महायजञ ा इस हानी म लख न या बतान ा परयास किया ह कि किसी भी अचछ

2nd language

परसार(यशपाल ाय या पणय न ा फल अवशय मिमलता ह ोई भी परोपार अथवा पणय लिलए किया गया ाय बार नही 0ाता वह ए परार ा यजञ हए धनी सठ थ धम परायण और किवनमर सठ न आन ी यजञ किए थ और दान म न 0ान कितना धन दिदन दखिखयो म बात दिदया थादिदन पलट और सठ यहा गरीबी आ गई उन दिदनो यजञ बचन ी परथा थी सठ भी अपनी 0गह बचन लिलए डलपर ए सट यहा चलन ो तयार हए सठानी रासत लिलए रोटी पड म बाधर सठ ो द दी रासत म ए भख R ो दखर सठ न चारो रोटी उसो खिखला दी खर वह सठ यहा डलपर पहच तो उनी सठानी न उस महायजञ बचन ो हा यदिद बचन आए सठ न R ो रोटी खिखलान ो महायजञ नही समझा और वापस लौट आया घर आर शाम ो उसी घर म उस ए बडा ख0ाना मिमला 0ो उस दवारा किए गएrsquo महायजञrsquo ा परसार था

English language

Letter formal The heading the name and address of the person you are writing to must be included beneath your own address In formal letters ldquoblock stylerdquo of address is preferred

Subject complain in brief

Salutation If the person you are writing to is known to you you may begin ldquoDear MrrdquoOr ldquoDear Mrsrdquo In all other instances you should begin ldquoDear Sirrdquo or ldquoDear Madamrdquo Or ldquoSirsrdquo

The body A formal or business letter has four partsReference The letter should begin by referring to a letter you have received an advertisement or the reason that has prompted you to writeInformation In the second paragraph it is necessary to supply more detailed information that is related to the referencePurpose Here you must give the reason why you are writing the letter This must be stated clearly and ensure that it is relevant to the question that has been setConclusion round off the letter with some polite remarkThe subscription when a letter has begun with dear sir sirs Madam you should end with Yours faithfully or yours truly When however you address a person by name you must conclude with the words ldquoYours sincerelyrdquo

1 A park in your locality is slowly being used as a rubbish dump Write a letter to the Mayor of your city pointing out the nuisance and danger of this Request that action be taken to stop this immediately

Or2 You being a boarder ordered a set of lab manuals from a famous book shop in the town They sent you a wrong set of books Write a letter to the manager of the book shop

Chemistry Chapter-1 1)CHEMICAL FORMULA- Q What is the Significance of

L-2The Language of Chemistrybull Chemical Formula

Itrsquos a symbolic representation of a chemical substance eg ndash The formula of Sulphuric acid is H2SO4

2) Steps of writing Chemical Formula of a given substance-

1 Write the symbols of the constituent atoms or radicals side by side Keep the basic radical on LHS and acid radical on the RHS ( Na+Cl- )2 In case of a radical having more than one atom( compound radical) enclose the radical in a bracket eg (SO4-)3 Write the valencies of each radical on its right hand top4 If the valencies of the two radicals are divisible by a common factor then divide the valencies by the common factor5 Invert (criss-cross) the valency number ie write the valency of one atom below the second atom and vice versa 6 On interchanging if valency number is lsquoone the figure lsquoonersquo is never writtenFor Example- Compound -Calcium Nitrate1 Writing the symbols- Ca(NO3)2 Writing the valencies on their right hand top- Ca2(NO3)1

3 Valency numeral in simple ratio- Ca2(NO3)1

4 Criss-cross- Ca 2NO3 1

5 Writing the formula of the compound- Ca(NO3)2

Chemical formula

A The formula of a substance conveys the following information regarding a substance 1 The name of the substance (qualitative)2 The elements constituting the substance (qualitative)3 The number of various atoms present in a molecule of the substance (quantitative)4 Molecular weight of the substance and the relative weights of different elements present in it (qualitative)

Q What are the limitations of Chemical Formula

A The chemical formula suffers from the following limitations-I It fails to convey whether the elements in a molecule are present in the form of atoms or ionsFor example the formula KBr fails to tell us whether Potassium and Bromine are present in the form of ions II It does not tell anything about the binding force that holds atom in a molecule togetherIII It does not tell us about the arrangement of various atoms with respect to one another within the molecule

Q Examples of Some Chemicals with their Formula Chemical name and Common Name-

A Given in the class notesCommercial Studies

Joint Stock Company

Let us discuss about the demerits of Joint Stock CompanyDespite so many advantages it has got many disadvantages which are as follows

Difficulty in FormationDelay in Decision makingExcessive Government ControlLack of Secrecy

Company can be classified into several categories based on incorporation

QuestionExplain the demerits of Joint Stock CompanyAnswer) 1 Difficulty in Formation The legal requirements and formalities required to be completed are so many The cost involved is quite heavy It has to approach large number of people for its capital It cannot start its business unless certificate of incorporation has been obtained This is granted after a long time when all the formalities are completed

Chartered CompanyStatutory CompanyRegistered Company

Delay in Decision making In this form of organization decisions are not made by single individual All important decisions are taken by the Board of Directors Decision-making process is time-consuming So many opportunities may be costly because of delay in decision-making Promptness of decisions which is a common feature of sole trader ship and partnership is not found in a company

Excessive Government ControlA company and the management have to function well within the law and the provisions of Companies Act are quite elaborate and complex At every step it is necessary to comply with its provisions lest the company and the management should be penalized The penalties are quite heavy and in several cases officers in default can be punished with imprisonment This hampers the proper functioning of the company

Lack of Secrecy The management of companies remains in the hands of many persons Every important thing is discussed in the meetings of Board of Directors Hence secrets of the business cannot be maintained In case of sole proprietorship and partnership forms of organisation such secrecy is possible because a few persons are involved in the management

2 Define the following

Chartered Company- The crown in exercise of the royal prerogative has power to create a corporation by the grant of a charter to persons assenting to be incorporated Such companies or corporations are known as chartered companies Examples of this type of companies are Bank of England (1694) East India Company (1600) The powers and the nature of business of a chartered company are defined by the charter which incorporates it After the country attained independence these types of companies do not exist

in IndiaStatutory Company- A company may be incorporated by means of a special Act of the Parliament or any state legislature Such companies are called statutory companies Instances of statutory companies in India are Reserve Bank of India the Life Insurance Corporation of India the Food Corporation of India etc The provisions of the Companies Act 1956 apply to statutory companies except where the said provisions are inconsistent with the provisions of the Act creating them Statutory companies are mostly invested with compulsory powersRegistered companiesCompanies registered under the Companies Act 1956 or earlier Companies Acts are called registered companies Such companies come into existence when they are registered under the Companies Act and a certificate of incorporation is granted to them by the Registrar

Economics

Chapter-4Basic problems of Economy

Today let us discuss with the topic Production Possibility curve

QuestionExplain the concept of Production Possibility Curve with the help of diagram

Answer) Production Possibility curve is a locus of all possible combinations of two commodities which can be produced in a country with its given resources and technology

The above diagram shows that with the given resources and technology the economy can produce maximum either 5 thousand meters of cloth or 15 thousand quintals of wheat or any other combination of the two goods like B( 1 thousand meters of cloth and 14 thousand quintals of wheat C ( 2 thousands meters of cloth and 12 thousand quintals of wheat) etcProduction Possibility curve is also called production possibility boundary or frontier as it sets the maximum limit of what it is possible to produce with given resources

Geography

Rotationand Revolution

SUNrsquoS POSITION AND SEASONAL CHANGES EQUINOXES ndash SPRING AND AUTUMN

Q1 What is Spring EquinoxA1 On 21st March sunrays fall directly on the equator On that day

As the Equator divides the Earth into two equal halves the sun rays fall directly on the equator twice in a year Equinoxes means equal Spring EquinoxOn 21st March sunrays fall directly on the equator On that day the duration of day and night both are equal ( 12 hours day and 12 hours night) on every places located on equator This day is called as Spring EquinoxAutumn EquinoxOn 23rd September sunrays fall directly on the equator On that day the duration of day and night both are equal ( 12 hours day and 12 hours night) on every places located on equator This day is called as Autumn Equinox

SOLSTICES ndash SUMMER AND WINTERDue to inclination of the Earth on its axis and the apparent movement of the sun the sun rays fall directly on both tropics once in a year Solstice is a Latin word which mean ldquothe Sun standing stillrdquoSummer SolsticesAfter 21st March there is an apparent movement of the Sun to the north of the equator The apparent northward movement up to 21st June when the Sun appears overhead at the Tropic of Cancer (22frac12degN) The sun appears to stand still at this position and then moves southwards towards the equator This position of the Sun on 21st June is known as Summer Solstices On that day the duration of day and night both are equal ( 12 hours day and 12 hours night) on every places located on Tropic of Cancer (22frac12degN)Winter solstices The apparent southward movement of the Sun continues beyond the equator till 22nd

December On this day the Sun is overhead at the Tropic of Capricorn

the duration of day and night both are equal ( 12 hours day and 12 hours night) on every places located on equator This day is called as Spring Equinox

Q2 What do you mean by EquinoxA2 Equinoxes means equal It is use to explain the equal duration of day and night ( 12 hours day and 12 hours night) on the Earth

Q3 On which date the longest day in Tropic of CancerA3 21st June

Q4 What is the meaning of SolsticeA4 Solstice is a Latin word which mean ldquothe Sun standing stillrdquo

Q5 Which is the longest day in southern hemisphereA5 22nd December

Q6 On what date does the Arctic Circle experience the lsquoMidnight SunrsquoA6 On 21 June the Arctic Circle experiences the lsquoMidnight Sunrsquo

Q7 What is cause of Midnight Sun in NorwayA7 During the summer solstice (21 June) the North Pole is inclined towards the Sun Therefore the duration of sunlight or daytime increases from 12 hours at the Equator to 24 hours at the Arctic Circle and beyond Thatrsquos why The region beyond the Arctic Circle especially Norway is known as the Land of the Midnight Sun because there the Sun does not rise or set on 21 June

Q8 Match the column A with BA B

Summer Solstice 21st March

Autumn Equinox 23rd

September

Winter Solstice 21st June

(22frac12degS) This position of the Sun is referred to as the Winter Solstice because it marks the winter season in the Northern Hemisphere On that day the duration of day and night both are equal ( 12 hours day and 12 hours night) on every places located on Tropic of Capricorn (22frac12degS)SEASONS AND DURATION OF DAY AND NIGHT During the equinoxes all places on the Earth have 12 hours of day and 12 hours of night Due to the revolution of the Earth round the Sun on an inclined axis the duration of day and night varies according to seasons and the latitude of a placeDuring the summer solstice (21 June) the North Pole is inclined towards the Sun Therefore the duration of sunlight or daytime increases from 12 hours at the Equator to 24 hours at the Arctic Circle and beyondThe region beyond the Arctic Circle especially Norway is known as the Land of the Midnight Sun because there the Sun does not rise or set on 21 JuneAt the North Pole there will be six months of daylight The Sun will be seen always above the horizon at a low angle At 66degN 24 hours of sunlight can be seen only on 21 June Hammerfest in northern Norway is a place of tourist attraction for observing the phenomenon of the Midnight Sun This place has continuous daylight from 13 May to 29 July This place is easily accessible to tourists and has hotels and other facilities The view of the midnight Sun from here is enthrallingIn the Southern Hemisphere the duration of daylight decreases from 12 hours at the equator to 0 hours beyond the Antarctic Circle In the South Polar Region there is 24 hours of darkness The Sun is always below the horizon In the Southern Hemisphere which experiences winter the duration of night-time is longer than the duration of daylight

Spring Equinox 22nd

December

A8 A B

Summer Solstice 21st June

Autumn Equinox 23rd

September

Winter Solstice 22nd

December

Spring Equinox 21st March

During winter solstice (22 December) the South Pole is inclined towards the Sun The Southern Hemisphere experiences summer and the Northern Hemisphere has winter Therefore the duration of daylight or sunlight is greater in the Southern Hemisphere than in the Northern HemisphereThe duration of daylight increases from 12 hours at the equator to 24 hours beyond the Antarctic Circle The South Polar Region has 24 hours of sunlight for many days continuously At the South Pole there will be six months of sunlight The Sun will always be seen at a low angle above the horizon In the Northern Hemisphere the duration of daylight will decrease from 12 hours at the equator to 0 hours at the Arctic Circle There are 24 hours of darkness in the North Polar region The duration of night is greater than the duration of daylight as one move northwards from the Equator It is evident from the above table that the duration of daylight is 12 hours throughout the year at the equator only As one moves away from the equator the seasonal variations in the duration of daylight increase The seasonal variations in the duration of daylight are maximum at the Polar Regions

Subject Eng Literature (The Merchant of Venice ndash William Shakespeare)Topic Act II Scene 7 Lines 36 to 80 (End of scene ) [Students should read the original play and also the paraphrase provided]

Summary Questions amp AnswersThe Prince then examines the inscription on the silver casket which says ldquoWho chooseth me shall get as much as he deservesrdquo The Prince says that he deserves Portia more than anybody else because of his high rank his noble birth and his great wealth and power But then he argues that silver is ten times

(1) (Act II Sc 7 L 39-47)

From the four corners of the earth they come

To kiss this shrine this mortal breathing saint

The Hyrcanian deserts and the vasty wildsOf wide Arabia are as through-fares now

inferior to gold and therefore he cannot believe that the portrait of such a beautiful lady as Portia can be contained in the silver casket He decides to see the inscription on the golden casket before making his decision

The Prince goes to examine the inscription on the golden casket which says ldquoWho chooseth me shall get what many men desirerdquo The Prince believes that the whole world desires to possess Portia otherwise so many suitors would not have come from all corners of the world for winning Portia Some of them have come from the distant lands of Persia and Arabia The deserts of Persia (Hyrcanian deserts) and the boundless desolate lands of Arabia have been crossed by the Princes seeking the hand of Portia He contrasts this casket containing Portiarsquos portrait with the old English gold coin bearing the image of the archangel (angel of the highest rank) He goes on to remark that while the figure of the archangel is engraved (Insculped) upon the English coin the picture of Portia who is beautiful as an angel lies hidden inside one of the caskets namely the Golden Casket (Golden Bed)

On the basis of his assessment of the inscription on the golden casket the Prince decides to choose the golden casket He asks for the key and opens the golden casket only to find therein an empty human skull holding a roll of

For princes to come view fair PortiaThe watery kingdom whose ambitious headSpets in the face of heaven is no barTo stop the foreign spirits but they comeAs orsquoer a brook to see fair Portia

(i) Explain the occasion for the above mentioned speech

These are the comments of the Prince of Morocco after he reads the inscription on the golden casket His mental process is revealed to us in these words We find him debating within himself as to which casket he should choose

(ii) What light does the above speech throw on the personality of Prince of Morocco

From the above mentioned speech we come to know that the Prince of Morocco is keen to marry Portia He is the type of person who is easily taken away by outward appearance He is in love with Portia because of her beauty

(iii) What information can you gather about Portia from the above mentioned lines

The given speech shows that Portia is a very beautiful lady She must be possessed of good qualities because many suitors come to her place from all over the world with a desire to get married to her The Prince of Morocco is so impressed by her beauty that he calls her a saint According to him the whole world is desirous of having her

(iv) Elucidate the significance of the first two lines

In these lines the Prince of Morocco pays a compliment to Portia These lines show his admiration for her He says that people come from all parts of the world to see fair Portia

(v) Explain the meaning of the last four lines of the

passage

In these lines the Prince of Morocco says that even the vast oceans which throw a challenge at the sky are unable to prevent men from coming to Portiarsquos place to have a glimpse of her These lines are also a tribute to Portiarsquos beauty and good qualities Many men voyage across the ocean treating it as a mere stream to see the beautiful Portia

paper in which is written that whoever happens to be guided by the glitter of things is invariably deceived

On reading the scroll the Prince says that he is too sad at heart to speak a more formal farewell and leaves with his followers amidst a sound of trumpets

After the Prince of Morocco leaves Portia remarks that the Prince is a gentle fellow but she is rid of him May all persons of his nature make a similar choice

IMPORTANT PASSAGES EXPLAINED

(Act II Sc 7 L 39-43)From the four corners of the earth they come

To kiss this shrine this mortal breathing saintThe Hyrcanian deserts and the vasty wildsOf wide Arabia are as through-fares nowFor princes to come view fair Portia

Context

This passage occurs in Act II Scene 7 in The Merchant of Venice This is part of the speech made by the Prince of Morocco

(2)

(Act II Sc 7 L 48-53)

MOROCCO One of these three contains her heavenly pictureIst like that lead contains her

Twere damnation To think so base a thought it were too grossTo rib her cerecloth in the obscure graveOr shall I think in silver shes immurdBeing ten times undervalued to tried gold

(i) What meaning does the Prince of Morocco find out of the inscription of the golden casket What have Belmont and Portiarsquos house been called and why

The inscription on the golden casket is ldquoWho chooseth me shall gain what many men desirerdquo The Prince finds out that it means that the chooser of the golden casket will get Portia because many men desire her In fact the entire world desires her Because of the coming of many suitors to Belmont from different countries in order to win Portiarsquos hand Belmont has become a centre of pilgrimage and her house is the shrine where saintly Portia is installed

(ii) What does the Prince of Morocco do before making the final choice of the casket Which is the correct casket and who will win Portiarsquos hand

The Prince of Morocco surveys and analyses the inscriptions on the casket of lead silver and gold Before making the final choice like a very systematic and methodical person he once again considers the claims of the caskets The casket containing Portiarsquos picture is the correct casket and the person choosing it will win Portiarsquos hand

Explanation

While praising Portia the Prince of Morocco conceives Portia as a goddess whose image is placed inside one of the caskets Many suitors are coming from far and wide the north and the south the east and the west (Four corners) in order to try their luck Some of them have come from the distant land of Persia and Arabia The deserts of Persia (Hyrcanian deserts) and the boundless desolate lands of Arabia have been crossed by the Princes seeking the hand of Portia All this shows that Portia is indeed the most beautiful lady of the world

(iii) What does the Prince of Morocco say in his estimation while examining the motto on the silver casket What does he find in the golden casket

While examining the motto on the silver casket which says ldquoWho chooseth me shall get as much as he deservesrdquo Morocco says that in his own estimation he surely deserves Portia in all respects ndash rank birth wealth etc

He chooses the golden casket When he opens it he finds an empty human skull holding a scroll in which it is written that those who are attracted by the glittering outside of things are always deceived as Morocco has been deceived

(iv) What kind of nature does the Prince of Morocco have

The Prince of Morocco has a simple nature who does not look deeply into the inner meaning of things but is dazzled by the outward appearance of gold He is inclined to over-estimate his own value and does not realize that it is a duty to ldquogive and hazardrdquo To say that he will not hazard for lead shows that he misreads the true meaning of the inscription which is that he should be prepared to ldquohazard all he hathrdquo for Portia So his feeling is only one of fascination and romantic attraction

(v) Do you think that the lottery of the caskets is not a matter that will be determined by chance

In fact the lottery of the casket is not a matter that will be determined by mere chance but that it is a true test of character and of sincerity which is amply proved not only by Moroccorsquos choice but also by the arguments which he uses to help him in his choice

(Act II Sc 7 L 55-59)

They have in England

A coin that bears the figure of an angelStamped in gold but thats insculpd uponBut here an angel in a golden bedLies all within

Context

(3)

(Act II Sc 7 L 63-77)A carrion Death within whose empty eye

There is a written scroll Ill read the writing

All that glisters is not goldOften have you heard that toldMany a man his life hath soldBut my outside to beholdGilded tombs do worms infoldHad you been as wise as boldYoung in limbs in judgment oldYour answer had not been inscrolld

This passage occurs in Act II Scene 7 in The Merchant of Venice This is part of the speech made by the Prince of Morocco

Explanation

In this passage the Prince of Morocco bestows high praise on Portia whose hand he is seeking He contrasts this casket containing Portiarsquos portrait with the old English gold coin bearing the image of the archangel (angel of the highest rank) He goes on to remark that while the figure of the archangel is engraved (Insculped) upon the English coin the picture of Portia who is beautiful as an angel lies hidden inside one of the caskets namely the Golden Casket (Golden Bed) In the day of Elizabeth silver was ten times inferior in value to gold Therefore the Prince of Morocco believing that Portiarsquos portrait is contained in the Golden Casket decides to choose the Golden Casket

Fare you well your suit is coldCold indeed and labour lostThen farewell heat and welcome frostmdashPortia adieu I have too grievd a heartTo take a tedious leave Thus losers part

(i) What reward does the Prince of Morocco get after making a wrong choice of the Casket How does he feel

After making the wrong choice in selecting the casket of gold the Prince of Morocco as a reward earns a rebuke in the form of a scroll tucked in the empty eye-socket of a skull kept in the casket of gold The Prince is shocked and disappointed He becomes all the more sad and dejected when he reads the scroll which points to his foolishness in being misled by the appearance and outward show as indicative of its worth

(ii) How does the Prince respond after reading the scroll

After reading the scroll the Prince though upset accepts the result with good grace and decorum befitting a royal suitor and true sportsman He says that his love-suit is really cold otherwise he would have chosen correctly but now his efforts have been in vain So he bids farewell to Portia to the warmth and enthusiasm of love and welcomes the cold and bitterness of dejection and misery of life which lies ahead

(iii) What request does he make to Portia and why

After being failure in his mission he requests Portia to give him permission to leave at once because he is too sad to undergo the tediousness of a formal leave-taking He tells that it is the manner in which defeated persons part unceremoniously

(iv) Explain the following lines

ldquoAll that glisters is not goldOften have you heard that toldMany a man his life hath soldBut my outside to beholdGilded tombs do worms infoldrdquo

Mere glitter does not make a metal to be gold Man has often been warned against appearance but it has been of no use Many people have sacrificed their lives only to seek the outer appearance of gold Worms are found inside the gilded

monuments

Class XSubject Topic Summary Execution

Hindi 2ndlang

नया रासता भाग 6 मायाराम 0ी घर म धनी मल 0ी और उनी बटी सरिरता ी ही चचा बनी रहती थी अमिमत ो इसम ोई रलिच ना थी वह धनी घर ी लडी स शादी र सवय ो बचना नही चाहता था उसा भी सवाणिभमान ह ईशवर ी पा

स उस पास पस ी ोई मी नही थी अभी उसन फकटरी ही लगाई थी उसी समझ बाहर था कि उस घर वालो ा झाव पस ी तरफ कयो

ह उसन मा स सवाल किया कि मा तम सरिरता स मरी शादी कयो रना चाहती हो मा न उस समझाया कि वह दखन म बरी नही ह और किफर खानदान अचछा

ह वह ए शल गरहणी रप म घर सभाल सगी अमिमत न मा ो इस बात ा एहसास राया कि मीन सबध लिलए मना रन पर उस दिदल

पर कया बीती होगी मा और अमिमत ी लडी बार म ाफी बात हईमा ा झाव सरिरता ी तरफ था कयोकि वह घर पर अचछा दह0 लर आ रही

थी अमिमत न अपनी मौसी ी बरी हालत बार म बताया कि किस तरह वह बड घर ी खानदानी बटी लाई थी और आ0 उसी हालत कितनी खराब ह लाई थी बहकलब 0ाती ह और बचचो ो भी नही दखती ह बात चल ही रही

थी कि तभी ए ार बाहर आर री धनी मल0ी घर अदर आए और पीछ स डराइवर फल ी ए टोरी लर आया अदर आए और पीछ स

डराइवर ए टोरी फल ी लर आया अमिमत ो फल ी पटी बरी लग रही थी अमिमत न पछ लिलया यह फल कयो ल आए ह प इन सब ी कया

0ररत थी उनो न 0वाब दिदया कि 4 पटी शमीर स मगाए थ अमिमत ो या सनर करोध आ गया तभी उस किपता 0ी आ गए उन आत ही अमिमत उठर बाहर चला गया वहा वहा मा पास आर बठ गया और बोला

अभी रिरशता तय नही हआ और धनी मल 0ी धनी मल 0ी फल ी पटी लर चलआय मा न समझाया कि 0ब सबध 0ड 0ाता ह तो खाली हाथ नही

आत अमिमत न मा स हा कि तम सबन सरिरता ो इस घर म लान ी ठान रखी ह धनीमल 0ी उस दिदन सरिरता ो दखन ी तारीख तय रन आय थ

Commercial Studies

Banking Nowadays Bank provide easy and quick services through internet facilities methods of Banking is called internet bankingIn order to save the time and money involved in visiting Bank branches people increasingly prefer to have internet banking

There are different modes of doing internet banking or transferring money through online They areReal Time Gross Settlement (RTGS)National Electronic Fund Transfers (NEFT)

1

Question

1) Explain the term RTGS Write the features of RTGS

Answer)The acronym RTGS stands for Real Time Gross Settlement which may be defined as the continuous real time settlement of funds transfer individually on and order by order basis without netting lsquoReal timersquo may be defined as the processing of instructions at the time they are received rather than at some letter time lsquoGross settlementrsquo may be defined as the settlement of transfer instructions which occurs

individually

Features of RTGS1It is the continuous settlement of

funds transfer individually on an order by order basis

2RTGS facility is provided only by CBS core banking solution enabled Bank branches

3Amount charged from the customer for RTGS transactions vary from bank to bank

2) Explain the term NEFT Write the features of NEFT

Answer) National electronic funds transfer may be defined as a nationwide system that facilitates individuals Farms and copper operates to electronically transfer funds from any bank branch to any individual farm or corporate having an account with any other bank branch in the country

Features of NEFT2 Transfer can be made 7 times on

weekdays and 6 times on Saturday

3 NEFT cannot be used to receive foreign remittances

4 NEFT transaction takes place in batches

5 A bank branch must be NEFT enabled to become a part of NEFT fund transfer network

6 There is no maximum or minimum amount that can be transferred through NEFT when one bank has a bank account

English Language

CompositionEssay

A composition is an art of creating a piece of writing on any topic or subject It is the writing correctly beautifully and clearly in order to make some interesting reading Structure of the composition

Introduction ( you lay the foundation for your composition)

Body (it constitutes the main part of the essay)

Conclusion (final statement that leaves a lasting impression)

Kinds of essays1 The Narrative essay2 The descriptive essay3 The reflective essay4 The argumentative essay

Write a composition on any one of the following topics (350- 400 words)

1 Friendship Or2 The first day of your school

Subject Eng Literature (The Merchant of Venice ndash William Shakespeare)Topic Act V Scene 1 Lines 127 to 158 (Nerissa helliphellip The clerk will nersquoer wear hair onrsquos face that had it) [Students should read the original play and also the paraphrase given in the school prescribed textbook]

Summary Revision Questions o Soon thereafter Bassanio Gratiano

and Antonio arrive

o Bassanio tells Portia that he is feeling as if it is morning because of the presence of Portia who is shining like the sun When Antonio is introduced by Bassanio to Portia she tells Bassanio that he should be grateful to Antonio who took so much trouble on his account even to the extent of risking his life

o Nerissa starts quarrelling with Gratiano and demands that he show her the ring she had presented to him and which she had warned him not to lose She suspects that Gratiano must have presented the ring to some young woman and not to the lawyerrsquos clerk as he repeatedly says and assures

Answer the following questions to check your preparation of Act IV Scenes 1 and 2

You must attempt only after you have completed your preparation of Act IV The answers must be in complete sentences using textual evidence (with citation) when necessary

[It would be in your own interest to attempt the above questions honestly totally refraining from consulting your textbook or your notes during answering After completion you should correct the paper yourself consulting the textbooknotes etc and award marks as specified Please let me know the marks you scored through WhatsApp in the group or to my personal WhatsApp]

Act IV Scene 1 (each question carries 2 marks)

1 What did the Duke try to do for Antonio

2 Why does Shylock refuse to show mercy How does he justify his stance

3 Why does Antonio say he is ready to die 4 What information is contained in Bellariorsquos letter

5 Why does Portia (as Balthazar) assert that Shylock must show mercy How does he respond

6 What offers are made to Shylock to get him to spare Antonio How are they received

7 What does Antoniorsquos speech as he faces the prospect of Shylockrsquos knife tell you about his character

8 How do Bassanio and Gratiano react to the looming prospect of Antoniorsquos demise

9 How does Portia (as Balthazar) use the law to turn the tables on Shylock

10 What does the Duke decree should happen to Shylock Why What happens to Shylockrsquos estate

11 What does Portia ask Bassanio as payment for her ldquoservicesrdquo What is his initial response What makes him change his mind

Act IV Scene 2 (each question carries 1frac12 marks)

1 What does Gratiano bring to Portia (Balthazar)

2 What does Nerissa plan on getting from Gratiano What does Portiarsquos comment suggest about men

ECO-10 280620 Topic-Supply AnalysisSHIFTING OF SUPPLY

But if there is change in factors other than the price of the commodity then either more is supplied at the same price or less supplied at the same price In such cases the price of the commodity remains constant but there is a change in other factors like change in the price of inputs change in technology of production change in price of other related goods change in taxation policy of the government etc For example there is an improvement in the technology of production of the commodity in question It leads to decrease in per unit of cost production of the commodity The firm is willing to sell more quantity of the commodity at the same price So the supply other commodity increases at the same price This increase in supply is shown by rightward shift of supply curve On the other hand if the firm uses inferior technology of production the cost of production per unit of the commodity increases The firm is willing to sell less quantity at the same price So the supply of the commodity decreases at the same price This decrease in supply is shown by leftward shift of the supply curve The above cases of increase and decrease in supply can be shown with the help of the following figures

Y INCREASE OF SUPPLY Price (Rs) s

P A s1

B

s

X` O s1 X

q q1

Y` Quantity demanded (in units)

Y DECREASE IN SUPPLY s2

s

price (Rs)

C

p A

s2

s

X` o X

q2 q

Y` Quantity demanded ( in units)

Main factors causing increase in supply or rightward shift of supply Curve(i) Fall in the price of other related goods

(ii) Fall in the price of inputsfactors(iii) Use of better technology in production(iv) Decrease in the rate of excise duty by government(v) If the objective of producer changes from profit maximization to salesMaximization

Main factors causing decrease in supply or leftward shift of supply curve(i) Increase in the price of other related goods(ii) Rise in the price of inputsfactors(iii) Use of inferior technology in production(iv) Increase in the rate of excise duty by the government(v) If the objective

Subject - Biology Topic ndash Chapter mdash6 PhotosynthesisSummary Execution

Today we will know about photosynthesis and its stages

Q1 What do you mean by photosynthesis The process by which living plants containing chlorophyll produce food

substances from carbon-di- oxide and water by using light energy Sunlight

6CO2 +12 H2O----------------------- C6 H12O6 + 6H2O + 6O2

Chlorophyll

Q2 What are the importance of photosynthesis I) Food for all Green plants trap solar energy by photosynthesis

process and supply food and energy for all living organisms either directly or indirectly

Ii) Oxygen to breathe in by product of photosynthesis is oxygen which is essential for all living organisms respiration

Q3 Write about two main phases of photosynthesis A Light dependent phase This phase occur in grana of chloroplast I) The chlorophyll on exposure to light energy becomes activated by

absorbing photons Ii) The absorbed energy is used in splitting the water molecules (H2O)

into its two components (H+ and OH- ) and releasing electron s 2H2O------------------------- 4H+ + 4e- +O2

Energy of 4 photons This reaction is known as photolysis

End products are H+ and oxygen water

B Light independent (Dark ) phase The reactions in this phase require no light energy

Here CO2 combine with H+ and produce glucose

Class XI

Subject Topic Summary ExecutionEVS Chapter-4 Legal

regimes for sustainable development

Environmental legislationEnvironmental legislation is the collection of laws and regulations pertaining to air quality water quality the wilderness endangered wildlife and other environmental factors The act ensures that matters important to the environment are thoroughly

Learn -The Forest (Conservation) Act 1980

considered in any decisions made by federal agencies

The Forest (Conservation) Act 1980 The Forest (Conservation) Act 1980 an Act of the Parliament of India to provide for the conservation of forests and for matters connected therewith or ancillary or incidental thereto It was further amended in 1988 This law extends to the whole of IndiaObjects and Reasons of the Forest Conservation Act

Deforestation causes ecological imbalance and leads to environmental deterioration Deforestation had been taking place on a large scale in the country and it had caused widespread concern The act seeks to check upon deforestation and de-reservation of forests

Subject Eng Literature (The Tempest ndash William Shakespeare) Topic Act II Scene 1 Lines 314 to 329 (End of scene)

[Students should read the original play and also the paraphrase given in the school prescribed textbook]Summary Questions amp Answers

Conspiracy of Antonio and Sebastian (Contd)

o As they approach Ariel appears again and wakes up Gonzalo by singing a tune in his ear Alonso also wakes up and they see both Sebastian and Antonio with drawn swords On being caught off guard they make up a story saying that they had heard a bellowing of bulls or lions

o They then moved to another part of the island

o Ariel at once rushes to Prospero to inform him of this development

SUMMING-UP of ACT-2 SCENE-1

(i) Among the survivors Ferdinand is separated from the rest which results in the disconsolate grief of Alonso as he took him for dead

(ii) The villainy of Antonio is confirmed

(iii) The supremacy of Prosperorsquos magic which resulted in the failure of the human conspiracy

(1)

(Act II Sc 1 L 311-325)SEBASTIAN Whiles we stood here securing your repose

Even now we heard a hollow burst of bellowing Like bulls or rather lions Didt not wake youIt struck mine ear most terribly

ALONSO I heard nothingANTONIO O rsquotwas a din to fright a monsters ear

To make an earthquake Sure it was the roarOf a whole herd of lions

ALONSO Heard you this GonzaloGONZALO Upon mine honour sir I heard a humming

And that a strange one too which did awake meI shaked you sir and cried As mine eyes opened I saw their weapons drawn There was a noiseThats verily rsquoTis best we stand upon our guardOr that we quit this place Lets draw our weapons

(i) Why has Prospero sent Ariel to Gonzalo and Alonso What does Ariel do to awaken Gonzalo

Prospero has already come to know by his magic powers the danger which threatens Gonzalo who had been Prosperorsquos friend and so he sent Ariel to preserve the lives of both Gonzalo and Alonso Prospero does not want that his scheme should remain unfulfilled Ariel begins to sing a song in Gonzalorsquos ears to awaken him(ii) Who are ready to carry out their plan Who takes steps to stop them Why does Gonzalo feel surprised after being awakened

Sebastian and Antonio are ready to carry out their plans They are standing with their swords drawn to kill Alonso and

(iv) We see two sets of contrasting characters Gonzalo-Adrian against Antonio-Sebastian

(v) The grief that works in Alonso can be perceived to his repentance for his association in Antoniorsquos crime against Prospero

Gonzalo Ariel takes steps to stop them from carrying out their nefarious scheme When Gonzalo is awakened by the song sung by Ariel into his ears he (Gonzalo) feels surprised because he sees Sebastian and Antonio standing with their swords drawn(iii) What reason do Sebastian and Antonio tell of drawing their swords when they are suspected by Alonso and Gonzalo

When Sebastian and Antonio are seen with their swords drawn they are looked with suspicion by Gonzalo and Alonso At first Sebastian tells them that as they stood here to guard them during their sleep they heard only a little before a sudden loud noise very much like the roaring of bulls or more probably that of lions Then Antonio follows him saying that this was a noise so terrible as to frighten even a monsterrsquos ears and this noise could even have shaken the earth and it was surely like the roaring of a multitude of lions Then seeing the danger they have drawn their swords Perhaps after hearing the terrible noise they (Gonzalo and Alonso) woke up from their sound sleep

(iv) What does Gonzalo tell Alonso about the strange noise What did he see on opening his eyes Gonzalo tells Alonso that he did not hear the sound of roaring but he heard a humming sound which was strange and which woke him up After waking up he gave him (Alonso) a shaking and a loud cry On opening his eyes he saw these two gentlemen standing with their swords drawn(v) What does Gonzalo suggest

Gonzalo suggests that there was a noise indeed and of that he has no doubt at all and suggests that the best course for them would be to remain alert and vigilant against any possible danger to their lives or to leave this place and move to some other part of the island

Class XIISubject Topic Summary Execution

Commerce

Chapter- Management

Today we will discuss about LEVELS OF MANAGEMENT

Levels of management is a series or chain of managerial positions from top to bottom It helps individuals to know their authority responsibilities and superior-subordinate relations among themselves There are mainly three levels of Management TOP LEVEL MANAGEMENTMIDDLE LEVEL MANAGEMENTLOWER LEVEL MANAGEMENT

Top level managementIt consists of members at the highest level in the management hierarchy This level includes Board Of Directors Chief Executive Managing Directors Chairman President Vice President

Rolefunctions of the top levelmanagement1To analyse evaluate and deal

with theexternal environment2 To determine the objectives and

policies of the business3 To strive for welfare and survival

of business

4 To create an organisational Framework consisting of authority responsibility relationship

Middle level management Congress of members or groups who are concerned with implementation of the policies let down by the top managementThis level includes head of the department such as finance manager marketing manager branch and regional managers departmental and divisional heads plant superintendent etc

Role of functions of the middle level management

1 To interpret the policies framed by top management

2 To assign duties and responsibilities to lower level managers

3 To select and appoint employees for middle and supervisory level and evaluate their performance

4 To co-operate with other departments for smooth functioning

Operational or supervisory level managementIt refers to the group are members who are concerned with execution of the work They are also known as fast line managers This level includes supervisor 4 men Section Officer clerk Inspector etc

Role of functions of the lower level management1 To plan and execute day-to-

day operations2 To supervise and control the workers3 To arrange materials and

tools to start the process and make arrangements for training

4 Today present workers grievance and suggestions before the management and

ensure safe and proper working conditions in the factory

Business Studies

Staff Appraisal Chapter- 10 Today let us start with a new chapter

Staff Appraisal

Meaning of Performance Appraisal

Performance Appraisal is the systematic evaluation of the performance of employees and to understand the abilities of a person for further growth and developmentThe supervisors measure the pay of employees and compare it with targets and plansThe supervisor analyses the factors behind work performances of employeesThe employers are in position to guide the employees for a better performance

Objectives of Performance Appraisal

Following are the objectives of Performance Appraisal

To maintain records in order to determine compensation packages wage structure salaries raises etc

To identify the strengths and weaknesses of employees to place right men on right job

To maintain and assess the potential present in a person for further growth and development

To provide a feedback to employees regarding their performance and related status

To provide a feedback to employees regarding their performance and related status

Importance of Performance Appraisal

Performance appraisal provides important and useful information for the assessment of employees skill

knowledge ability and overall job performance The following are the points which indicate the importance of performance appraisal in an organization

1 Performance appraisal helps supervisors to assess the work performance of their subordinates

2 Performance appraisal helps to assess the training and development needs of employees

3 Performance appraisal provides grounds for employees to correct their mistakes and it also provides proper guidance and criticism for employees development4 Performance appraisal provides reward for better performance

5 Performance appraisal helps to improve the communication system of the organization

6 Performance appraisal evaluates whether human resource programs being implemented in the organization have been effective

7 Performance appraisal helps to prepare pay structure for each employee working in the organization

8 Performance appraisal helps to review the potentiality of employees so that their future capability is anticipated

Geography

DRIANAGE The SubarnarekhaThe Subarnarekha and the Brahmaniinterposed between the Ganga and the Mahanadi deltas drain an area of 19300 sq kmand 39033 sq km respectively The drainage basins of these streams are shared byJharkhand Odisha west Bengal and Chhattisgarh The Brahmani is known as southKoel in its upper reaches in Jharkhand

The NarmadaThe Narmada rises in the Amarkantak hills of MadhyaPradesh It flows towards the West in a rift valleyformed due to a geological fault The total length of it is 1300 km All the tributaries of the

Q1 Name the two westward flowing rivers in the peninsular plateauA1 Narmada and Tapi are the only westward flowing rivers of the peninsular plateau

Q2 Differentiate between east-flowing rivers and west-flowing riversA2

East-flowing rivers

West-flowing rivers

Narmada are very short inlength Most of its tributaries join the main streamright anglesThe Narmada basin covers parts of Madhya Pradesh and Gujarat

The Tapi The Tapi rises in the Satpura ranges in the Betul listrictof Madhya Pradesh It flows in a rift valley parallel tothe Narmada but it is much shorter in length It coversparts of Madhya Pradesh Gujarat and MaharashtraThe length is about 724 km

The Sabarmati and the MahiThe Sabarmati rises in the Aravali hills and flows south-south-westwards for a distance of 300 kilometres to the Arabian Sea The Sabarmatibasin extends over an area of 21674 sq km in Rajasthan and Gujarat The Mahi rises inthe east of Udaipur and drains an area of 34842 sq km lying in Madhya PradeshRajasthan and Gujarat It flows south-westwards for a distance of 533 km before it fallsinto the Gulf of Khambhat

The ChambalThe Chambal rises near Mhow in the Vindhya Range and flows towards the northgenerally in a gorge upto Kota Below Kota it turns to the north-east direction and afterreaching Pinahat it turns to the east and runs nearly parallel to the Yamuna beforejoining it in the southern part of the Etawah district in Uttar PradeshMajor Rivers of India with their basin area (Sqkm)

Himalayan System Indus 321290Ganga 861404

Brahmaputra 187110Indus System

Jhelum 34775Beas 20303

Ganga System Yamuna 366223Ghaghra 127950

Peninsular RiversNarmada 98796

Tapi 65145Mahanadi 141600

Subarnarekha 19300Sabarmati 21674

Mahi 34842Godavari 312812

Godavari Krishna Kaveri Mahanadi are the east-flowing rivers

Narmada Tapi west-flowing rivers

They fall into the Bay of Bengal

They fall into Arabian Sea

These rivers form big deltas

These rivers form comparativelysmall deltas

Catchment areas of these rivers are larger

Catchment areas of these rivers are smaller

Krishna 2589488Cauveri 87900

Subject ndashBiology Topic ndashChapter -5 Inheritance amp Variations Summary ExecutionToday we will discussabout linkage and its classification

LINKAGE The tendency of the genes located on the same chromosome to stay together is

hereditary transmission Linked genes the genes responsible for this Genes that exhibit the process of linkage locates in the same chromosome The distance between the linked genes in a chromosome determines the strength

of linkage i e genes that are located close to each other show stronger linkage than that are located far from each other

COMPLETE LINKAGE It is the type of linkage showed by the genes that are closely located or are tightly

linked with each other as they have no chance of separatingby crossing over These genes are always transmitted together to the same gamete and the same

offspring In such condition only parental or non cross over type of gametes are formedINCOMPLETE KINKAGE It is type of linkage showed by the genes that are distantly located orare loosely

linked with each other because they have chance of separating by crossing over

SIGNIFICANCE i) It helps in holding the parental character togetherii) It checks the appearance of new recombination and helps in bringing the

hybrid population which resembles the original parents iii) Linked genes dilute the effects of undesirable traits

Subject Eng Literature (The Tempest ndash William Shakespeare) Topic Essay Questions (EQ-3)Question No 3

Give a character sketch of CalibanAnswer

The character of Caliban has been wonderfully conceived by Shakespeare as the manifestation of all that is gross and earthy ndash a sort of creature of the earth as Ariel is a sort of creature of the air

Calibanrsquos Physical Appearanceo Caliban is lsquofreckledrsquo a lsquomisshapen knaversquo not honoured with human shape

o Prospero calls him lsquothou tortoisersquo (Act I Sc 2 Line 317) Trinculo stumbling upon him describes him as ldquoA strange fish hellip Legged like a man And his fins like armsrdquo He ldquosmells like a fishrdquo (Act II Sc 2 Line 25)

o Prospero also calls him a ldquobeastrdquo (Act IV Sc 1 Line 140) and ldquoThis misshapen knaverdquo (Act V Sc 1 Line 268)

o Further it appears that in addition to his physical deformity his spiritual inferiority is also suggested by Prosperorsquos claim that his birth resulted from the union between his mother the witch Sycorax and the devil

Calibanrsquos ParentageWhen the play opens Caliban is twenty four years of age having been born on the island twelve years before the coming of Prospero His mother was the foul witch Sycorax who was banished from Algiers for ldquomischiefs manifold and sorceries terrible to enter human hearingrdquo (Act I Sc 2 Line 264) and the father was the Devil himself Thus

Caliban is a monster of evil and brute nature ugly deformed and stinking

Calibanrsquos Savage and Malignant Natureo Caliban is entirely a creature of the earth ndash gross brutal and savage He regards himself as the rightful possessor

of the island and Prospero as a usurper

o In his young age he was on good terms with Prospero He had consented to be received by Prospero at his house and to be educated by him He has learnt human language only to curse his master whom he abhors

o His beastly nature soon breaks out and ends in a vicious attack on Miranda This opens the eye of Prospero who becomes severe to him and enforces his service by threats and violence

o Prospero uses him to make dams for fish to fetch firewood scraper trenches wash dishes and keep his cell clean

Calibanrsquos Hatred for ProsperoA profound hatred for Prospero has taken hold of Caliban It springs from a sense of his being dispossessed and ill-treated He would kill Prospero if he could but he knows the power of Prosperorsquos lsquobookrsquo Hence he transfers his allegiance to Stephano who seems like a god to him He also incites the two drunken associates to batter the skull of Prospero when he sleeps in the afternoon

Caliban Shows Considerable Intelligenceo He has learnt Prosperorsquos language

ldquoYou taught me language and my profit onrsquot (Act II Sc 2 Lines 86-89)Is I know how to curserdquo

o He is well aware of the futility of arguing with one who has more power than he has

ldquoI must obey his art is such power (Act I Sc 2 Lines 373-376)It would control my damrsquos god SetebosAnd make a vassal of himrdquo

o He realizes the importance of Prosperorsquos books

ldquoRemember (Act III Sc 2 Lines 89-92)First to possess his books for without themHersquos but a sot as I am nor hath notOne spirit to commandrdquo

o He knows the value of stealth when attacking the enemy

ldquoPray you tread softly that the blind mole may not (Act IV Sc 1 Lines 194-195)Hear a foot fall we now are near his cellrdquo

o Caliban has a better set of values than Stephano and Trinculo They are distracted from their plan by their greed for Prosperorsquos rich garments Only Caliban realizes that such a finery is unimportant

ldquoLeave it alone thou fool it is but trashrdquo (Act IV Sc 1 Lines 224)

Caliban is not a good judge of characterCaliban is not a good judge of character He decides for example that Stephano is a god because he dispenses lsquocelestial liquorrsquo (Act II Sc 2 Line 115) but then it must be remembered that he has only known his mother Sycorax Prospero Miranda and the spirits that torture him However he quickly discovers his error of judgementrdquo

ldquoWhat a thrice-double ass (Act V Sc 1 Lines 295-297)Was I to take this drunkard for a godAnd worship this dull foolrdquo

Calibanrsquos Imaginative NatureIf Caliban is sub-human in what has been said above he is human in the respect of the poetic side of his character He listens to music with rapture He tells of the beautiful dreams in which heaven rains treasures upon him and which upon waking he yearns to renew One of the most poetic passages in whole play is Calibanrsquos description of the island

to Stephano and Trinculo

ldquoBe not afeard The isle is full of noises (Act III Sc 2 Lines 135-143)Sounds and sweet airs that give delight and hurt notSometimes a thousand twangling instrumentsWill hum about mine ears and sometime voicesThat if I then had waked after long sleepWill make me sleep again and then in dreamingThe clouds methought would open and show richesReady to drop upon me that when I wakedI cried to dream againrdquo

Caliban - Less Ignoble Than Some OthersCalibanrsquos motive for murder is less dishonourable than that of Antonio and Sebastian They plan to kill Alonso to gain his power and wealth Caliban merely wants revenge and the return of lsquohisrsquo island

Conclusiono Calibanrsquos character is not portrayed very clearly in the play and hence we cannot decide whether he is a poor

savage being grossly maltreated by Prospero or whether he is evil and must therefore be kept in bondage or enslavement

o Caliban is contrasted with Ariel who is a spirit and thus swift and uninterested in physical activitieso Caliban is also contrasted with Prospero who is the all-powerful master of the island and of the destiny of all

those on the islando Caliban is also contrasted with civilized man showing him to be less evil than Antonio and Stephano and less

materialistic than Stephano and Trinculoo Caliban has suffered at the hands of Prospero and he has learnt to curse by listening to Prosperorsquos abuse He

certainly believes that Prospero has deprived him of his birthrighto Finally the character Caliban is thought to be one of Shakespearersquos masterpieces The complexity of the character

is reflected in the large volume of critical discussion that has grown around it

ECO ndash12 Topic-Forms of market

MonopolyMonopoly is a market structure in which there is a single seller there are no close substitutes for the commodity produced by the firm and there are barriers to entry Example Indian Railways which is operated under government of India Monopoly also implies absence of competitionFeatures of Monopoly Monopoly is characterized by1 Single Seller In monopoly there is only one firm producing the product The whole industry consists of this single firm Thus under monopoly there is no distinction between firm and industry Being the only firm there is significant control of the firm over supply and price Thus under monopoly buyers do not have the option of buying the commodity from any other seller They have to buy the product from the firm or they can go without the commodity This fact gives immense control to the monopolist over the market

2No Close Substitute There are no close substitutes of the product produced by the monopolist firm If there are close substitutes of the product in the market it implies presence of more than one firm and hence no monopoly In order to ensure a total of control over the market by the monopolist firm it is assumed that there are no close substitutes of the product

3 No Entry amp Exit Monopoly can only exist when there is strong barriers before a new firm to enter the market In fact once a monopoly firm starts producing the product no other firm can produce the same One reason for this is the ability of the

monopolist to produce the product at a lower cost than any new firm who thinks to enter the market If a new firm who knows that it cannot produce at a lower cost than the monopolist then that firm will never enter the market for fear of losing out in competition Similarly the monopolist who is operating for a long time may be enjoying reputation among its customers and is in a better position to use the situation in its own benefit A new firm has to take long time to achieve this and so may not be interested to enter the market

4 Price Maker Being the single seller of the product the monopolist has full control over the pricing of the product On the other hand if there is a large number of buyers in the market so no single buyer exercises any significant influence over price determination Thus it is a sellerrsquos market So monopoly firm is a price maker

5 Price Discrimination Having considerable control over the market on account of being single seller with no entry of other firms the monopolist can exercise policy of price discrimination it means that the monopolist can sell different quantities of the same product to a consumer at different price or same quantity to different consumers at different prices by adjudging the standard of living of the consumer

6 Shape of Demand Curve Since a monopolist has full control over the price therefore he can sell more by lowering the price This makes the demand curve downward sloping

Subject Ac-12 290620 Topic- retirement Model sumThe Balance Sheet of Rohit Nisha and Sunil who are partners in a firm sharing profits according to their capitals as on 31st March 2014 was as under

Liabilities Amount Assets Amount (Rs) (` Rs)

Creditors 25000 Machinery 40000Bills Payable 13000 Building 90000General Reserve 22000 Debtors 30000Capital Less Provision for Rohit 60000 Bad debts 1000

29000 Nisha 40000 Stocks 23000 Sunil 40000 140000 Cash at Bank 18000

200000 200000

On the date of Balance Sheet Nisha retired from the firm and following adjustments were made(i) Building is appreciated by 20(ii) Provision for bad debts is increased to 5 on Debtors(iii) Machinery is depreciated by 10(iv) Goodwill of the firm is valued at Rs 56000 and the retiring partnerrsquos share is adjusted

(v) The capital of the new firm is fixed at Rs120000 Prepare Revaluation Account Capital Accounts of the partner and Balance Sheet of the new firm after Nisharsquos retirement Revaluation AccountDr Cr

Particulars Amount Particulars Amount (`Rs) (Rs`)

Provision for Bad debt Ac 500 Building Ac 18000Machinery Ac 4000Profit transferred toCapital Accounts (3 2 2)Rohit 5786Nisha 3857Sunil 3857

13500

18000 18000

Capital Account

Dr Cr

Particulars Rohit Nisha Sunil Particulars Rohit Nisha Sunil (Rs`) (Rs`) (`Rs) (Rs`) (Rs`) (Rs`)

Sunilrsquos Capital ac 9600 mdash 6400 Balance bd 60000 40000 40000Bank - 66143 - General Reserve 9428 6286 6286Balance cd 72000 mdash 48000 Revaluation (Profi 5786 3857 3857 Rohitrsquos Capital Ac mdash 9600 mdash

Sunilrsquos Capital Ac 6400 Bank 6386 - 4257

81600 66143 54400 81600 66143 54400

Balance Sheet as at 31st March 2014

Liabilities Amount Assets Amount (Rs`) (Rs`)

Creditors 25000 Building 108000Bank overdraft 37500 Machinery 36000

Bills Payable 13000 Debtors 30000Capital Less ProvisionRohit 72000 for Bad debts 1500 28500Sunil 48000 120000 Stock 23000

195500 195500

Working Notes (i) (a) Profit sharing ratio is 60000 40000 40000 ie = 3 2 2(b) Gaining Ratio Rohit = 35 ndash 37 = 2135 ndash 1535 = 635Sunil = 25-27 = 1435 ndash 1035 = 435= 635 435= 6 4 = 3 2(c) Nisha Share of Goodwill = Rs 56000 times 27 = Rs16000Share of Goodwill in the gaining ratio by the existing partner ieRohit = Rs16000 times 35 = Rs 9600Sunil = Rs 16000 times 25 = Rs 6400

The journal entry isRohitrsquos Capital Ac Dr 9600Sunilrsquos Capital Ac Dr 6400 To Nisharsquos Capital Ac 16000(Share of Goodwill divided into gaining ratio)

  • 1 Static Friction
  • The frictional force that acts between the surfaces when they are at rest with respect to each other is called Static Friction
    • Static Friction Examples
      • 2 Sliding Friction
        • Examples Of Sliding Friction
          • 3 Rolling Friction
            • Examples Of Rolling Friction
              • Objects and Reasons of the Forest Conservation Act
Page 8:  · Web viewSubject . Topic . Summary . Execution . English 1 . Sounds of animals . Hens –cackle Horses –neigh Lions –roar Owls –hoots Snake –hiss. English 2 . Mother’s

Earth on a flat surface

Elements of a mapTo be able to read a map we must understand the elements of a map

DirectionsDirections are the basic guidelines that help us to locate places It is very important for us to know the correct directions East West North and South are the four Cardinal directions These directions help us to locate the places Beside these there are four sub directionsndash Northeast Southeast North West and South West Compass is an instrument that helps to locate the directions

3 The basic guidelines to help us to locate places ndash directions

4These four directions help us to locate the places ndash Cardinal directions

COMPUTER

Formatting in ms word

Q3) What do you mean by formattingAns) Formatting is a general arrangement of text in a document We can change the appearance of a document by using the features available in ms word We can use different fonts colours and styles in the textQ4) What is alignmentAns) Alignment of text is the way in which it is placed between the margins of a page Text can be aligned to the left side in the centre or to the right side of a pageQ5) what is fontAns) A font is a style of writing and typing A font provides specific textual appearance to the document You can change the size style or give various effects to a font

Hindi 2ndlang

ाला किहरण सभी धमlt ा मल धम ह दया और रणा मरी हर 0ीव अपन ही दकिनया म रहत ह हम

पराणी कयो उनो परशान रत ह ऐसा हम नही रना चाकिहए

0ानवरो स हम ए सीख मिमलती ह किस तरह हम एता म रहना चाकिहए

हर पराणी पयार ी भाा समझता ह अगर हम पयार ी उममीद रत ह तो कया 0त

0ानवर हमस उममीद नही रत कि हम भी उनह पयार द यह हानी ए ाल किहरण ी ह 0ो अपन

समह ा नततव रता था हमशा कया उलिचत ह यह सोचता था अचान

उन पर लिशारिरयो न हमला बोल दिदया जि0सम ाला किहरण परा गया कयोकि वह

दलभ ह और लिशारिरयो ी न0र उसी पर थी परत 0ानवरो ी एता ो दखर

रा0मार ो दया आ गई और उसन उनो छोड दिदया 0ानवर भी बईमान नही थी वह उनस अकसर मिमलन उन बाद म 0ाता रहता रा0ा ो धनयवाद रता अबोध म

पराणिणयो म एता तजञता अभी भी ह

शबदाथ तराई ndash पहाड आसपास नीच

ी भमिमवश- ल या परिरवारअदवभत- अनोखाचौननाndash सत सावधानओझलndash गायबकिवपणिRndash मसीबतसगठन- एताउममीदndash आशाातर दमिU ndashकिववशता

हिहस ndash हानी पहचान वालातवयndash फ0

दल ndash समहनततवndash सचालन रना

ओट ndash आडआतमसमपणndash अपन ो सौप दनाम- चप

हम उनस सीख लनी चाकिहएবইndashবাংলা সাহিতয পহিরচয়

পাঠndash১০লপndashহিবকেবকানকেFর যকেলকেবলাযলখকndashশশী-ষণ াশগNঅনশীলনীর পরকে4াততর

৯প পহিরবতG ন ককেরা -শরীর = শারীহিরক -ত = য-ৌহিতক সG ার = সG াহির সথায়ী = সথাহিয়তবহিবশবাস = হিবশবাসী া = যকো১০ বাকয রচনা ককেরা -আডডা = পাার বদধরা াতলায় আডডা যয়হিডবাহিজ = রাম া যকেক হিডবাহিজ যখকেয় পকেলাসG ার = বকেলহিল যকেল যর সG ারবকহিন = অঙক -ল করায় রীনা মাকেয়র কাকে বকহিন যখকেলাবহিদধ = বহিদধ াককেল উপায় য়হিনশহিত = হিনশহিত রাকেত রাজবাহিকেত ডাকাত পকেলা-য়ানক = পাাহি রাসতা বষটির পকের -য়ানক হিবপ জনক য়হিবশবাস = মানকেষর পরহিত হিবশবাস ারাকেনা পাপ

Science Ch ndash The Food We Eat

Living things need food to live to grow to stay strong and fit When we need food we feel hungry Food gives us energy to do work It also protects us from diseases and helps us to stay healthy Nutrients in food The food we eat contains many substances that are necessary for our body These substances are called nutrients that help us to grow well and stay healthy

Nutrients give us energy to study work and play

They help our body to grow and repair the damaged parts of our body

They also help our body to fight against diseases and remain healthy

Write T for True or F for False (pg no- 11)

1 Food contains nutrients that help us to grow True

2 Foods rich in carbohydrates are called body-building foods False

MAT

HEM

ATIC

S

Ch 9

Com

mon

Fra

ction

s

A fraction is a number that stands for parts of a whole object or a collection of objects

Each fraction has two numbers One is written above the other separated by a line The one above the line is called numerator and the one below the line is called denominator

Example 5minusminusminusminusrarr Numerator

11minusminusminusminusrarr Denominator

Exercise ndash 362 In following fill in the blanks

(b) 37 ___ is denominator ___ is

numerator

(c) 89 ___ is denominator ___ is

numerator

Solution

(b) 37 7 is denominator 3 is numerator

(c) 89 9 is denominator 8 is numerator

3 Write down the fractions whose numerators and denominators are given below in the bracket The first number stands for numerator and the second number standing for denominator

(25) 25

(311) 311

(416) 416

(712) 712

Class VSubject Topic Summary Execution

Science Ch ndash PlantReproduction

In pollination chapter we have learnt that the flowers change into fruits and the fruits bear seeds Now we are going to learn how this process takes place

FertilizationThe process of fusion of the male reproductive cell (male gamete) and female reproductive cell (female gamete) is known as fertilizationWhen a pollen grain reaches from the anther to a stigma it begins to grow and forms apollentube The Pollen tube then travels down through the style to enter an ovule inside the ovary On reaching the ovule male reproductive cell in the pollen grain unites with the egg cell present in the ovule

Books exercise

A) Tick the correct answer

1Which of the following do ovules change into after fertilization ndashseeds

2Which of the following is not a part of the seed ndash flower

3Which of the following condition is needed for germination of a seed ndash all ofthese

English language

Sentences phrases and

Solved exercisesSay which of the underlined groups of words are phrases and which clauses

clauses8 In her new clothes ndashphrasesAs pretty as a doll ndash phrases

9 looking sad and upset ndash phrasesHe had lost all the tickets for the Test Match ndash clauses

10 During the vacation now only a month away ndashphrases

11 too sweet and too hot ndashphrases

12 At the Olympic Games ndashPhrasesOf Laurel leaves ndash phrases

13 Who played the role of Hamlet ndashclauses

14 However fast ndash phrases

15 When the men fell asleep ndash clausesSocial studies

Indian Government

Lok Sabha (lower house) ndash It has 552 members Of these 530 membersrepresent States 20 members represent the union territories and two members represent the Anglo- Indian community All except the representatives of the Anglo-Indian community are elected by Indian citizens A person above the age of 25 can contest in the elections for Lok Sabha One term of Lok Sabha is for 5 yearsRajya Sabha (upper house) ndash Its members are elected by the MLAs or members of the legislative Assembly There are 250 members in the Rajya Sabha of which 12 are nominated by the President One term of Rajya Sabha is for 6 years Anyone above the age of 30 can be elected as a member of Rajya Sabha

ExecutivePresidentThe President is the head of the country in India He is elected by the MPs and the MLAs for a tenure of 5 years He appoints the Prime Minister and the Council of Ministers Prime MinisterThe party which wins the election forms the government and its leader is elected as the Prime Minister He is the chief advisor to the President The Council of Ministers assists the Prime Minister and is accountable for their roles For example the Education minister is responsible for the education system in our country

1 How many members are there in Lok SabhaAns 552 members

2 What is the term for Lok SabhaAns 5 years

3 How many members are there in the Rajya SabhaAns 250 members

4 Who is the head of our countryAns President

5 Who is the chief advisor to the PresidentAns Prime Minister

Book ndash GK

Ch ndash 1First in space

1 First living being into space in 1957 Ans Laika

Times 2 First person to go into space in 1961 Ans Yuri Gagarin

3 First woman to go into space in 1963 Ans ValentinaTereshkova4 First person ever to walk in space in 1965Ans alexei Leonov5 First person to land on the moon in 1959 Ans Neil Armstrong6 First Indian to go into space in 1984 Ans Rakesh Sharma 7 First Indian woman to go into space in 19978 Ans Kalpana Chawla9 First woman tourist in space in 2006

Ans Anusheh AnsariCOMPUTER

ALGORITHM AND FLOWCHART

Q) DRAW THE SYMBOLS USED IN A FLOWCHART WITH THEIR DESCRIPTIONS(IN EXAM IT CAN COME AS SHORT QUESTIONS ASKING INDIVIDUAL SYMBOLS FUNCTION)ANS)

MAT

HEM

ATIC

S

Ch 6

Com

mon

Fra

ction

s

Multiplication of FractionsA Multiply a fractional number by whole numberTo multiply a fractional number by whole number we multiply the numerator of the fractional number by the whole number and denominator of the fractional number by 1 The first product thus obtained is the numerator and the second product is the denominator of the required product

Exercise ndash 30Multiply

7 2027

times 9

Solution 2027

times 9 = 203 = 6

23

8 611

times11

Solution 611

times11 = 6

15 71

20times16

Solution 71

20times16 =

14120

times16

= 1415

times 4 = 141times 4

5 = 564

5 = 11245

B Multiplication of a fractional number by a fractional number To multiply a fractional number by a fractional number we multiply the numerator of the first fractional number by the numerator of the second fractional number and the denominator of the first fractional number by the denominator of the second fractional number The first product thus obtained is the numerator and the second product is the denominator of the required product

16 2712

times24

Solution 2712

times24 = 3112

times24

= 31times2 = 62

Exercise ndash 31

11 83

times 34

2

Solution 83

times 34 = 2

14 723

times2 25

4

Solution 723

times2 25 =

233

times 125 =

23times 45

= 925 = 18

25

15 1212

times1 13

2

Solution 1212

times1 13 =

252

times 43 =

25times 23

= 503 = 16

23

State the following statements are true or false

17 1912

times 239 = 1

Solution LHS = 1912

times 239

= 392

times 239 = 1 = RHS

[LHS = Left hand side amp RHS = Right hand side]

there4 1912

times 239 = 1 [True]

21 213

times2 13 = 4

19

Solution LHS = 213

times2 13 =

73

times 73

= 7times73times3 =

499 = 5

49

there4 LHS ne RHS

So 213

times2 13 = 4

19 [False]

23 23

times 45 =

2times 5+3 times43times 5

Solution

LHS = 23

times 45 =

2times 43 times5 =

815 again

RHS = 2times 5+3 times4

3times 5 = 10+12

15 = 2215

there4 LHS ne RHS So 23

times 45 =

2times 5+3times43times 5

[False]

25 23 of

13 =

29

Solution

LHS= 23 of

13 =

23 times

13 =

29 = RHS]

there4 23 of

13 =

29 [True]

Practice at HomeExercise ndash 31State the following statements are true or false

24 12 of 4 =

18

Class VISubject Topic Summary Execution

HISTORY AND CIVICS

Chapter 5The Mauryan Empire

DECLINE OF MAURYAN EMPIREDecline of Mauryan empire started after the death of Ashoka at around 232 BCThere are several reasons for break up of the empire1 Weak successor Emperors after Ashoka were

capable of handling vast and mighty Mauryan empire In 185BC the last Mauryan ruler Bri-hadrath was murdered by his Commander-in-Chief Pushyamitra Sunga

2 Provincial Revolts Due to weak central author-ity provincial chiefs of Kalinga and southern provinces revolted against emperor and freed themselves from Mauryan empire

3 Weakness of Economy Prosperity of Mauryan was based on solid economic activities which

ExercisesI Multiple choice questions-1 Chandragupta defeated Seleucus in the year ndashc) 305 BC2 Who killed the last Mauryan ruler Brihadrath b) Pushyamitra3 Which of the following was not a reason for the decline of the Mauryan empirec) Chandraguptarsquos weakness4 Ashoka invaded Kalinga in the year c) 261 BC

II Fill in the blanks1Chandragupta ascended the throne in 324

was taken care by early monarchs Later kings had neither ability nor interest in economic af-fairs That led to failure in tax collection As a result they failed to maintain a large army that were essential to keep empire intact

4 Greek Invasion Greeks freed north-western provinces from weak Mauryan monarchs and reestablished their authority

5 Ashokarsquos Policy some scholar opined that after Kalinga war Ashoka embraced Buddhism re-nounced the policy of war and disbanded the Army But this is partially true as there is no proper evidence of disbanding the army

Based on above points we can conclude that main reason for decline of Mauryan empire is weakness of Ashokarsquos successors Kunal Samprati Dasharath Salisuk all were weak kingsAt last in 185 BCPushyamitra Sunga killed king Brihadrath and established the Sunga dynasty

BC2 Bindusara was the son of Chandragupta and father of Ashoka

3 Pataliputra was administered by City Magistrate committess of 5 members each4 The Greek General Seleucus sent his ambassador Megasthenes to Chandraguptarsquos court5 Ashoka sent his son Prince Mahendra and daughter Sanghamitra to spread his Dhamma6 The Indian Rebublic has adopted the Lion Capital of Saranath Pillar as its national emblem 7 Pushyamitra killed the last Mauryan ruler Brihadrath and founded the Sunga dynasty

III Name the following

1The author of Arthashastra-Kautilya2 The ruler who founded the Mauryan dynasty-Chandragupta3 The author of Indika-Megasthenes 4 The officers who were appointed by Ashoka to spread Dhamma-Dhamma Mahamatras5 The general of Alexander whom Chandragupta defeated-Seleucus

V Match the columns1 Kautilya (c)2 Megasthenes (d)3 Pushyamitra (e)4 Brihadrath (b)5 Bindusara (a)

BENGALI(2ND

LANGUAGE)

পশপাহিখর -াষাসহিবনয় রায়কেচৌধরী

যলখক পহিরহিচহিত- পরখযাত সাহিহিতযক উকেপনদরহিককেশার রায়কেচৌধরীর পতর সহিবনয় রায়কেচৌধরী lsquoসকেFশrsquo পহিতরকার সকেb হিতহিন কত হিকেলন তার উকেdখকোয বই lsquoসহিবনয় রায়কেচৌধরীর রচনা সংগরrsquo

পরম হিকেনর পাঠ- lsquoপশপাহিখর হিক -াষাhelliphellip helliphelliphelliphellipপরসপরকেক জানাবার উপায়ও পশপাহিখরা যবশ জাকেনrsquoপরকেমই আমারা জাহিন -াষা হিক -াষা ল আমাকের মকেনর -াব পরকাশ করার জনয আমরা নানান ধরকেনর -হিb বা হিবকেশষ ধরকেনর আওয়াজ মকেখর মাধযকেম কহির অনযকেক যবাঝাকেনার জনয তাকেল এবার আমরা জাহিন পশপাহিখর -াষা হিক পশ পাহিখরা হিক কা বকেল যা পশপাহিখকেরও -াষা আকে তারা তাকের হিনজসব -াষায় কা বকেল মকেনর -াব পরকাশ ককের পশ পাহিখরা মানকেষর হিক হিক -াষা যবাকেঝ হিকনত তারা বলকেত পাকেরনা পরসপরকেক বহিঝকেয় যবার উপায় তারা জাকেননা তকেব তারা হিবকেশষ ককেয়কটি শকেবদর মাধযকেম তাকের মকেনর -াব বহিঝকেয় যয় হিক বহিদধ মান জীব ndashককর হিবাল বন মানষ যঘাা পর-হিত এরা মানকেষর যওয়া নাম শনকেল কান খাা ককের ndash নাম ধকের ডাককেল কাকে আকেস যমন - মরহিরা lsquoহিত ndashহিতrsquo ডাক শকেন আকেস াল lsquoঅ ndashর -র ডাক শকেন কাকে আকেস াহিত মাহকেতর কা শকেন চকেল ককররা মাহিলকেকর হকম পালন ককের সবসময় তাইকেতা ককরকেক পর- -কত পরানী বলা য় ককর আর হিবাল একের আওয়াজ তহিম লকষয করকেল বঝকেব ককররা যরকে যকেল lsquoযঘউ যঘউrsquo করকেত াকেক আবার কাকেল lsquoযকউ যকউrsquo ককের হিবাল সাধারণ lsquoমযাওrsquo বা lsquoহিমউrsquo ককের রা কেল lsquoওয়াওrsquo আওয়াকেজর মাধযকেম মকেনর -াব পরকাশ ককের একেতা যল পশকের কা পাহিখরাও -য় রা পরকাশ করার জনয হিবকেশষ ধরকেনর শবদ ককের হিবপকের সময় পশ পাহিখরা সবার আকে পরসপরকেক জানাবার উপায় তারা জাকেন বহকাল

১) পশপাহিখর -াষা কেলপর যলখক সমপকেকG হিক জাকেনা

উঃ- পরখযাত সাহিহিতযক উকেপনদরহিককেশার রায়কেচৌধরীর পতর সহিবনয় রায়কেচৌধরী lsquoসকেFশrsquo পহিতরকার সকেb হিতহিন কত হিকেলন তার উকেdখকোয বই lsquoসহিবনয় রায়কেচৌধরীর রচনা সংগরrsquo হিতহিন ারকেমাহিনয়াম এসরাজ পর-হিত বাযনতর বাজাকেত পারকেতন ানও জানকেতন হিতহিন যাকেIাকের জনয মজাার লপ কহিবতা হিলখকেতন

২) পশপাহিখ কেলপর মল-াব হিকউঃ- পশপাহিখকেরও -াষা আকে তারা তাকের হিনজসব -াষায় কা বকেল মকেনর -াব পরকাশ ককের পশ পাহিখরা মানকেষর হিক হিক -াষা যবাকেঝ হিকনত তারা বলকেত পাকেরনা পরসপরকেক বহিঝকেয় যবার উপায় তারা জাকেননা তকেব তারা হিবকেশষ ককেয়কটি শকেবদর মাধযকেম তাকের মকেনর -াব বহিঝকেয় যয় হিরউকেবন কযাসটং সাকেব হিতহিন চহিdশ বর বনযজনত যর সকেb যকেককেন হিতহিন বকেলকেন আমরা হি তাকের -াষা তাকের আব কায়া যমকেন চহিল তাকেল আর -কেয়র যকান কারণ াকেকনা আমরা একI -াকেলাকেবকেস যচষটা করকেল পশপাহিখকের সকেb -াব পাতাকেত পাহির

ধকের মানষ এই পশ পাহিখর -াষা হিনকেয় নানা রককেমর পরীকষা ককের আসকে এইরকম একজন হিরউকেবন কযাসটাং সাকেকেবর কা আমরা জানকেবাhelliphellip

Hindi 2nd

langमतर किनमनलिलखिखतपरशनोउRरदीजि0ए

) बढ वयलि` बचच ो कया हआ था ख) डॉकटर साहबन पाटc किस उददशय स रखी थी ग) ाल साप ो हाथ म लर लाश न कया किया घ) डॉकटर चडढा न बढ पतरो दखन स कयो मना र दिदया था ङ) भगत न लाश ो दखर कया हा

उRर ndash) उस बहत बखार थी और 4 दिदनो स आख भी नही खोला थाख) उन बट ी सालकिगरह थीग) ाल सापो हाथ म लर लाश न उसी गदन 0ोर स दबार पडी थीघ) डॉकटर चडढा न बढ वयलि` पतरो दखन स मना र दिदया कयोकि उनह गोलफ खलन 0ाना थाङ) लाश ो दखर हा कि नारायण चाहग तो आध घट म भया उठ 0ाएग

English literature

In the bazaars of Hyderabad- Sarojini Naidu

Through the poem In The Bazaars of Hyderabad Sarojini wanted to convey the message that India is rich in tradition and they donrsquot need the foreign products So she goes on to give a picture of a bazaar where traditional Indian products are rulingThe poem is in the form of questions and answers The poet asks the questions and the merchants answer them Through this technique she make the picture of the bazaar visible to us

Read the poem

PHYSICS FORCE Types of FrictionThere are three types of friction static sliding rolling Static sliding and rolling friction occur between solid surfaces

1 Static Friction The frictional force that acts between the surfaces when they are at rest with respect to each other is called Static FrictionStatic Friction Examples

Skiing against the snow Creating heat by rubbing both the hands

together Table lamp resting on the table

2 Sliding Friction The resistance that is created between any two objects when they are sliding against each other is called Sliding FrictionExamples Of Sliding Friction

Sliding of the block across the floor Two cards sliding against each other in a

deck

3 Rolling Friction The force which resists the motion of a ball or wheel is called Rolling Friction Is the weakest types of frictionExamples Of Rolling Friction

Rolling of the log on the ground Wheels of the moving vehicles

6What effect can a force produce on a body which is not allowed to move Ans - When a force is applied on a body which is not free to move it gets deformed i e the shape or size of the body changes7Give one example each to indicate that the application of a force

1 produces motion2 stops motion3 slows down motion4 changes the direction of motion5 deforms a body

Ans- 1 A car originally at rest when pushed

begins to move2 A moving bicycle is stopped by

applying the brakes3 The speed of a moving vehicle is

slowed down by applying the brakes4 A player kicks a moving football to

change its direction of motion5 On stretching a rubber string its

length increases

8State the effect produced by a force in the following cases (a) The sling of a rubber catapult is stretched(b) A man pushes a heavy cart(c) A player uses his stick to deflect the ball (d) A cyclist applies brakes(e) A spring is compressedAns- (a) The shape and size of catapult changes ie its length increases(b) The heavy cart begins to move(c) The direction of the ball changes(d) The speed of the moving cycle is slowed down(e) There is change in size and shape of spring

COMPUTER MS EXCEL 2013 -INTRODUCTION

UNDERSTANDING EXCEL STRUCTUREA SPREADSHEET IS A FILE THAT EXISTS OF CELLS IN ROWS AND COLUMNS AND CAN HELP ARRANGE CALCULATE AND SORT DATA DATA IN A SPREADSHEET CAN BE NUMERIC VALUES AS WELL AS TEXT

FORMULAS REFERENCES AND FUNCTIONS

WORKSHEETA WORKSHEET IS ALSO KNOWN AS SPREADSHEETIT IS A COLLECTION OF CELLS ON A SINGLE SHEET WHERE YOU KEEP AND CHANGE DATA

WORKBOOKWORKBOOK IS PMS EXCEL FILE IN WHICH THE DATA CAN BE STORED EACH WORKBOOK CAN CONTAIN MANY WORKSHEETS

ROWS AND COLUMNSIN MS EXCEL A ROW IS A GROUP OF CELLS THAT RUN FROM LEFT TO RIGHT OF A PAGEA COLUMN IS A GROUPING OF CELLS THAT RUN FROM THE TOP TO THE BOTTOM OF A PAGE

CELLTHE INTERSECTION POINT BETWEEN A ROW AND THE COLUMN IS CALLED A CELL WHICH IS THE BASIC STORAGE UNIT FOR DATA IN A SPREADSHEET EACH CELL HAS SPECIFIC ADDRESS WHICH IS THE COMBINATION OF THE COLUMN NAME FOLLOWED BY THE ROW NUMBER

CHEMISTRY Chapter ndash Common Laboratory Apparatus and equipments

Objective type questionFill in the blanks (a) Experiment and observation are the two important basics of chemistry(b) A porcelain dish is used for evaporation(c) A test tube holder is used to hold the test tube while-it is heated(d) Mortar and pestle is used for grinding and crushing solid substances into a powder(e) Glass apparatus is made of Pyrex or borosil glass

Class VIISubject Topic Summary Execution

Hindi 2ndlang

ए था राम( डॉ शरी परसाद)

सगकित ा परभाव मानव 0ीवन पर अवशय पडता ह

हमशा मनषय ो अचछो ी सगकित म रहना चाकिहए

शरषठ परो सग स मनषय चरिरतर ा शीघर ही उदय और किवास हो 0ाता

ह इसलिलए वयलि` ो सदा शरषठ परो ा ही सग रना चाकिहए

इसान अगर चाह वह सवय ो बदल भी सता ह

यह हानी राम ए बचच ी हवह गणिणत ी परीकषा म नल रत हए पडा 0ाता ह और उस अधयाप पडत ह और पछत ह यह कया र रह

हो तभी राम न उनी बइजजती ी

शबदाथब ndashहावा भलावाायवाहीndash ाम किनयम व ानन

ो दिदखानापरिरलिचतndash 0ाना पहचानाघटनाndashघबराहट

उलटा चोर ोतवाल ो डाटndashकिववndash भल बर ा जञानतवयndash म 0ो रना चाकिहएसगकितndash बरी सगत

किबलख नाndashरोना किनशचय रनाndash तय रना

फलndashपरिरणामकिनषालिसतndash बाहर किया हआपशचातापndashदख सपननndashधनी

ldquo हा आपी किहममत स हई नल रत पडन ीrdquo ऐसी बात ही किफर

किपता0ी न भी उस डाटा वह ाफी पशचाताप रन लगा बोला गलत दोसतो

ी सगकित म आ0 कितना अनथ र दिदया किफर उसन अधयाप स माफी मागन ी सची और किफर भी ऐसा

नही रगा यह परण भी लिलया

सोचndashकिहच एात-अला

বইndashবাংলা সাহিতয পহিরচয়

পাঠndash১৬লপndashস-য ও অস-যযলখকndashঈশবরচনদর হিবযাসারঅনশীলনীর পরকে4াততর

৬ অGকেলকেখা -ময়া = পশ হিশকার সহিtহিত = হিনকIবতu সbভরষট = লI হিনরীকষণ =

-াকেলা-াকেব যখাকতাঞজহিলপকেI = যজাাকেত৭ হিবপরীতশবদ -ঈষৎ times পরচর উৎকষট times হিনকষট তাশ times উৎফd তবহিদধ times

বহিদধীNপাহিপষঠ times পণযবান৮ পপহিরবতG ন ককেরা -পশ = পাশহিবক যকাপ = যকাহিপতহিসথর = হিসথরতাএকানত = ঐকাহিনতক পর-াত times পর-াতী

CHEMISTRY

Chapter ndashPhysical and Chemical Changes

Chemical ChangeA chemical change involves a change in chemical composition

Characteristics of Chemical changes 1 They are permanent changes2 They are irreversible changes 3 New substance formed4 A Chemical change involves a

change in its chemical properties

Pg-25Question 8What do you observe when1 water is boiled2 a piece of paper is burnt3 some ice cubes are kept in a glass tumbler4 solid ammonium chloride is heated5 an iron nail is kept in tap water for few days6 a spoon of sugar is heated in a pan7 lighted match stick is brought near the mouth of the test tube containing hydrogen gas8 quick lime is dissolved in water9 little amount of curd is added to a bowl containing warm milk and kept for five hours

10 Water is boiledOn boiling water changes into steam (gas) physical change

11 A piece of paper is burnton burning piece of paper produces carbon dioxide and ash is left behind Is a chemical change

12 some ice cubes are kept in a glass tumblerIce cubes (solid) turn into water

(liquid) only state changes (physical change)

13 Solid ammonium chloride is heatedSolid ammonium chloride on heating changes into vapors (change of state) is physical change

14 An iron nail is kept in tap water for few dayswe observe reddish brown coating on the nail called rust (entirely new substance) is chemical change

15 A spoon of sugar is heated in a panWhen a spoon of sugar is heated in a pan black (charred sugar) (carbon) is seen Is a chemical change

16 Lighted match stick is brought near the mouth of the test tube containing hydrogen gasWe observe that hydrogen bums at the mouth of test tube with blue flame and pop sound is heard It is chemical change

17 Quick lime is dissolved in waterThe following two observations will be observed (i) A hissing sound is observed(ii) The mixture starts boiling and lime water is obtained

18 Little amount of curd is added to a bowl containing warm milk and kept for five hoursWhen a little amount curd is added to a bowl containing warm milk and kept for five hours a permanent change occurredThe milk will change to curd On boiling water changes into steam (gas) physical change

GEOGRAPHY

ATMOSPHERE IMPACT OF GLOBAL WARMING The destructive impart of global warming is observed in various spheres of life and the environment Some of the points are outlined below1 High temperatures lead to high

evaporation rate and drying up of the soil and surface water This affects crop production The occurrence of droughts is aggravating the problem even further

2 The heat waves in summer months

Q1 Write some impact of global warmingA1 The impacts of global warming are as follows1 High temperatures lead to high

evaporate ion rate and drying up of the soil and surface water This affects crop production The occurrence of droughts is aggravating the problem even further

2 The heat waves in summer months lead to a greater number

lead to a greater number of deaths due to heat strokes

3 Forest fires become more frequent4 Tropical cyclones and hurricanes

become common5 Melting of glaciers takes place6 Polar ice caps are becoming thinner

and melting at an alarming rate due to global warming The loss of sea ice

7 Due to increase in sea surface temperature sea levels rise in coastal areas and cause submergence of several islands

WAYS TO REDUCE GLOBAL WARMINGFollowing steps can be taken We need to decrease emission of

green house gases by reducing the burning of fossil fuel such as coal and petroleum

By planting more trees to increase forest cover

The government should also distributes free saplings and organize afforestation programmes to spread awareness regarding the beneficial effects of trees

We should switch to eco-friendly cars and gadgets

Incandescent light bulbs should be replaced by CFL bulbs

We can save electricity and reduce global warming by turning off electrical gadgets such as lights fans air-conditioners television and computer when we do not to use them

Efforts should be made to hasten the development of green cities oreco cities These cities are urban areas around the world striving to lessen the environment a impacts of urbanization

By following the 3Rs-Reduce Recycle and Reuse strategy we can use natural resources for our growth as well as save them for the need of the future generations This is called sustainable development

of deaths due to heat strokes3 Forest fires become more

frequent4 Tropical cyclones and hurricanes

become common5 Melting of glaciers takes place

etc

Q2 How to reduce global warmingA2 Following steps can be taken to reduce global warmingaWe need to decrease emission of

green house gases by reducing the burning of fossil fuel such as coal and petroleum

bBy planting more trees to increase forest cover

c The government should also distributes free saplings and organize afforestation programmes to spread awareness regarding the beneficial effects of trees

dWe should witch to eco-friendly cars and gadgets

eIncandescent light bulbs should be replaced by CFL bulbs

f We can save electricity and reduce global warming by turning off electrical gadgets such as lights fans air-conditioners television and computer when we do not to use them

Q3 What do you mean by 3Rrsquos of resource planningA3 The 3Rs are

1 Reduce 2 Recycle and3 Reuse

Q4 What is Sustainable developmentA4 By following the 3Rs-Reluce Recycle and Reuse strategy we can use natural resources for our growth as well as save them for the need of the future generations This is called sustainable development

English Language

Prepositions A preposition is a word placed before a noun or a pronoun It helps to show how the person or thing denoted by the noun is related to something else in the sentence

Kinds of Prepositions

Simple Prepositions- simple preposition are one word Prepositions such as at by for in of off for from on out through till to up with before amidst towards beyond between over etc

Compound Prepositions ndash There are some words that are always used with fixed Prepositions to convey specific meaning

Example I was unable to meet you dueto a previous engagement ( On account of)Always maintain the queue instead of crowding at the counter ( In place of)

Participial PrepositionsmdashParticiple Prepositions are present or past participles of various verbs which together with a noun phrase or a clause function as prepositions Examples- barring concerning considering notwithstanding pending regarding respecting etc

Exercise A

1 Gauravs fever has come down since Friday He has been absent for a week now

2 The child sat between his father and mother among the parents of all his classmates

3 There are mosquitoes in the room They flew into the room when the door was open

4 My father was inside the drawing room when I was playing outside my house

5 You may sit beside me I will give you a drawing book and pencils besides a storybook

6 We went to the market in the morning and walked towards the riverfront in the evening

7 The child walked along the pavement and across the street safely

8 This table top is made of glass My breakfast fell off it in the morning

9 The pan is on the gas stove There are vegetables in it

10 We will wait for you at the bus top There are a lot of people in the hall

Subject ndash Biology Topic ndash Chapter - 3 Photosynthesis and respiration in plants Summary Execution

All living organism (Plants and animals) need food for energy and growth Green plants (autotrophy) prepare food for all living organisms Today we will discuss about the process photosynthesis And adaptations in a leaf to carry out photosynthesis

Q1What do you mean by photosynthesis and write its word equation The process by which green plants make food (glucose) from carbon dioxide and water

in the presence of sunlight and chlorophyll is called photosynthesis

Carbon dioxide + Water ( Sun light from Sun ) Glucose + Oxygen ( chlorophyll in green leaves )

Q2 What are the adaptations in a leaf to carry out photosynthesisi) Leaves are broad wide and flat for absorbing more light energyii) Presence of chlorophyll in chloroplasts to trap sunlightiii) Presence of stomata which allow carbon dioxide to enter the cell and oxygen to go

out iv) Network of veins ensures continuous supply of water and minerals to the leafv) Thin waxy cuticle protects the leaf without blocking the lightQ3 Draw and label structure of chloroplast

Class VIIISubject Topic Summary Execution

PHYSICS ENERGY Production of Hydro electricity

A hydroelectric dam converts the potential energy stored in a water reservoir behind a dam to mechanical energymdashmechanical energy is also known as kinetic energy As the water flows down through the dam its kinetic energy is used to turn a turbine

The generator converts the turbinersquos mechanical energy into electricity

This electric energy then goes through various transmission processes before it reaches you

Question 2

Fill in the blanks

(a) Work is said to be done by a forte only when the body moves

(b) Work done = Force x distance moved in direction of force

(c) The energy of a body is its capacity to do work

(d) The SI unit of energy is joule

(e) The potential energy is due to its state rest of position and kinetic energy of the body is due to its state of motion

(f) Gravitational potential energy U = mass times force of gravity on unit mass times height

(g) Kinetic energy = frac12 times mass times (speed)2

(h) Power P = work donetime taken

(i) The S I unit of power is watt

(j) IHP = 746 W

BIOLOGY Chapter -5 The endocrine system and adolescence

Today we will discuss about thelocation and functions of secreted hormones of adrenal and Pancreas

Q5 Write location hormone secreted main functions and deficiency diseases of pancreas and adrenal glands

Endocrine Glands

Location Hormones secreted

Functions and Deficiency Diseases

1Adrenal gland

2 Pancreas Gland

On the top of each kidney

In between stomach and small intestine

i)Adrenaline from adrenal medulla

ii)Cortisone from adrenal cortex

i) Insulin

ii) Glucagon

It helps a person deal with any kind of emergency situation or emotional stressIt increases the heart beat rate of respiration and blood pressure

a) It regulates carbohydrates protein and fat metabolism

b) It regulates the salt and water balance in the body

a) It changes excess glucose into glycogen

b) It stimulates the cells to burn extra glucose to provide heat amp energy

Less secretion causes diabetes mellitus

Excessive secretions causeinsulin shock

a) It stimulates the breakdown of glycogen into glucose

b) It increases the level of glucose in blood

History Traders to rulers The Battle of Buxar was fought on 22 October 1764 between the forces under the command of the British East India Company led by Hector Munro and the combined armies of Mir Qasim the Nawab of Bengal till 1763 Mir Jafar was made the Nawab of Bengal for a second time in 1763 by the Company just after the battle After being defeated in 4 battles in katwa and Udaynala the Nawab of Awadh Siraj id Daula and the Mughal emperor Shah Alam II accompanied by Raja Balwant Singh of Kashi made an alliance with Mir Qasim The battle was fought at Buxar a small fortified

Answer the following questions- Short note-Battle of BuxarHomework-learn

town within the territory of Bihar located on the banks of the Ganga river about 130 kilometres (81 mi) west of Patna it was a decisive victory for the British East India Company The war was brought to an end by the Treaty of Allahabad in 1765

EnglishLiterature

The west wind-John Mansfield

In the poem The West Wind by John Masefield the poet starts by describingwith very poetic imagery of birds how the west wind is different from other winds its a warm wind full of birds cries There is a touch of melancholy perhaps home-sickness as he describes how it brings tears too and memories from an old land He goes on to describe the restful pastoral beauty of the land where even the dead can lie in the green He then brings in voicesperhaps of family and friends calling him home as he is missing Aprils beautyThe voices then tempt him some more with idyllic images from home (white blossom young green cornrunning rabbitswarm sun) The voices seem to presume that the poets heart is sorrowful bruised and soreThe end of the poem sees the poet appear to make a decision he will go home as he has decided that is where he truly belongs

Write the synopsis of the following words

1 Daffodils- a tall yellow flower that grows in the spring

2 Orchards- a piece of land on which fruit trees are grown

3 Blossom- a flower or a mass of flowers especially on a fruit tree in spring

4 Thrushes- a bird5 Larks- a small brown bird that

makes a pleasant sound6 Bruised- an injury7 Aching- pain 8 Tread- to put your foot down

while you are walking9 Balm-10 May-11 Fluting-

(Write from the book in your copy)

MAT

HEM

ATIC

S

Ch 1

1Al

gebr

ic E

xpre

ssio

n

1 Constant A symbol which has fixed value is called a constant[eg 8 23 -15 radic3 etc]

2 VariableA symbol which does not have any fixed value but may be assigned value (values) according to the requirement is called variable or literal[eg x y p q etc]

3 TermsA term is a number (constant) a variable a combination (product or quotient) of numbers and variables[eg 7 x 5x etc]

4 Algebric expressionA single term or acombination of two or more terms connected by plus (+) or minus (-) sign forms an algebraic expression[eg 5-y 3x2-5x xy-6z+4 etc]

5 PolynomialAn algebraic expression which contains more than one term is called a polynomial (multinomial)[eg x2-5x 5y+xy+x2y etc]

6 Degree of polynomial(a) When the polynomial contains only one variable the highest power of the variable is the degree of the polynomialeg the degree of the polynomial of 4x-7x5+8 is 5(b) When the polynomial contains two or more variablesStep (i) Find the powers of the variables in each term (ii) The highest sum of the powers is taken to be the degree of the polynomialeg the degree of the polynomial 5x2y-4x3y5+6 is = 3+5 = 8Remember An algebraic expression is a polynomial if degree of each term used in it is a non-negative integer

Exercise ndash 11(A)

1 Separate the constants and variables from the following

-7 7+x 7x+yz radic5 radic xy 3 yz

8 45y -3x

Solution Constant Variables-7 radic5 7+x 7x+yz radic xy

3 yz8

45y -3x

2 Write the number of terms in each of the following polynomials(i) 5x2+3timesax (ii) axdivide4-7 (iii) ax-by+ytimesz (iv) 23+atimesbdivide2

Solution Polynomials Number of terms(i) 5x2+3timesax 2(ii) axdivide4-7 2(iii) ax-by+ytimesz 3(iv) 23+atimesbdivide2 2

4 Write the degree of the each polynomials(i) xy+7z (ii) x2-6x3+8 (iii) y-6y2+5y8 (iv) xyz-3 (vi) x5y7-8x3y8+10x4y4z4

Solution Polynomials Degree(i) xy+7z 2(ii) x2-6x3+8 3(iii) y-6y2+5y8 8(iv) xyz-3 3(vi)x5y7-8x3y8+10x4y4z4 12

5Write the coefficient of(i) ab in 7abx (iv) 8 in a2-8ax+a (v) 4xy in x2-4xy+y2

SolutionCoefficient

(i) ab in 7abx 7x(iv) 8 in a2-8ax+a -ax(v) 4xy in x2-4xy+y2 -1

7 CoefficientAny factor of an algebraic quantity is called the coefficient of the remaining quantityeg in the algebraic term 7xyz 7 is coefficient of xyz 7x is coefficient of yz and so on

8 Like term The terms having the same literal coefficient are called like terms and those having different literal coefficients are called unlike terms

eg (i) 5xyz 8xyz -6xyz and 23xyz are like

terms(ii) 7xy2 8x2yz and -15xyz2 are unlike terms

6 in 57xy2z3 write the coefficient of

(i) 5 (vii) 5xy2 (viii) 17yz (xi) 5xyz

Solution Coefficient

(i) 5 17

xy2z3

(vii) 5xy2 17z3

(viii) 17yz

5xyzsup2

(xi) 5xyz 17yz2

7 In polynomial given below separate the like terms(ii) y2z3 xy2z3 -58x2yz -4y2z3 -8xz3y2 3x2yz and 2z3y2

Solution y2z3 -4y2z3 2z3y2 are like terms

xy2z3 -8xz3y2 are like terms

-58x2yz 3x2yz are like terms

Class IXSubject Topic Summary Execution

Bengali (2nd language)

বাগzwnjধারাzwnj বা ধারা-বা ধারা ল হিবকেশষ পরকার বাক -হিb -াকেবর এক হিবকেশষ পরকাশরীহিত াকেক কতগকেলা কার সমষটির মকেধয এগহিলকেক বা ধারা বকেল আবার কতগকেলা শকেবদর বাধাধরা যকান রীহিত যনই য-াকেব চকেল আসকে যসই -াকেবই চকেল আসকে তখন যসই শবদগহিল খন একক -াকেব অG পরকাশ ককের তখন একের বা ধারা বকেল বা ধারার পরকেয়া -াষাকেক আরও সFর ককের যতাকেল

অকাল পকক(অপহিরনত বয়কেস পাকাহিম)-মাতর শ বর বয়কেস যমকেয়টির া মকেখর কা তাকেত অকালপককতা ধরা পকে

অককা পাওয়া( মারা াওয়া) ndash পকেকIমারটি পকেকIমারকেত হিকেয় বাসাতরীকের াকেত মার যখকেত যখকেত অককা যপল

অহি| পরীকষা ( কঠিন ও পরকত পরীকষা)- যকেলটির আজ ডাকতাহির যরজালট যবকেরাকেব এIাই তার জীবকেনর ব অহি| পরীকষা

অষটরমভা (ফাহিক) ndash রীতা মকেখই বকো বকো কা বকেল আর কাকেজর যবলায় অষটরমভা

অকমGার ধাী (অপাG) ndash সমনকেক হিনকেয় যকান ান কেব না ও একেকবাকেরই অকমGার ধাী

অকেনধর ষটি (অসাকেয়র সায়)- আহিশ বকেরর বকোর নাহিত ল অকেনধর ষটি তাকেক াা বকোর একম চকেল না

আকেককল গড়ম (তবহিদধ)- ার তহিম উপকার করকেল যসই যতামার হিবরকেদধ সাকষয হিকেয়কে শকেনই আমার আকেককল গড়ম

আষাকে লপ( অবাসতব লপ) ndashIাকা এখন যকেব না এIা বলকেলই ত এমন আষাকে লপ ফাার যকান রকার হিল না

Hindi- महायजञ ा इस हानी म लख न या बतान ा परयास किया ह कि किसी भी अचछ

2nd language

परसार(यशपाल ाय या पणय न ा फल अवशय मिमलता ह ोई भी परोपार अथवा पणय लिलए किया गया ाय बार नही 0ाता वह ए परार ा यजञ हए धनी सठ थ धम परायण और किवनमर सठ न आन ी यजञ किए थ और दान म न 0ान कितना धन दिदन दखिखयो म बात दिदया थादिदन पलट और सठ यहा गरीबी आ गई उन दिदनो यजञ बचन ी परथा थी सठ भी अपनी 0गह बचन लिलए डलपर ए सट यहा चलन ो तयार हए सठानी रासत लिलए रोटी पड म बाधर सठ ो द दी रासत म ए भख R ो दखर सठ न चारो रोटी उसो खिखला दी खर वह सठ यहा डलपर पहच तो उनी सठानी न उस महायजञ बचन ो हा यदिद बचन आए सठ न R ो रोटी खिखलान ो महायजञ नही समझा और वापस लौट आया घर आर शाम ो उसी घर म उस ए बडा ख0ाना मिमला 0ो उस दवारा किए गएrsquo महायजञrsquo ा परसार था

English language

Letter formal The heading the name and address of the person you are writing to must be included beneath your own address In formal letters ldquoblock stylerdquo of address is preferred

Subject complain in brief

Salutation If the person you are writing to is known to you you may begin ldquoDear MrrdquoOr ldquoDear Mrsrdquo In all other instances you should begin ldquoDear Sirrdquo or ldquoDear Madamrdquo Or ldquoSirsrdquo

The body A formal or business letter has four partsReference The letter should begin by referring to a letter you have received an advertisement or the reason that has prompted you to writeInformation In the second paragraph it is necessary to supply more detailed information that is related to the referencePurpose Here you must give the reason why you are writing the letter This must be stated clearly and ensure that it is relevant to the question that has been setConclusion round off the letter with some polite remarkThe subscription when a letter has begun with dear sir sirs Madam you should end with Yours faithfully or yours truly When however you address a person by name you must conclude with the words ldquoYours sincerelyrdquo

1 A park in your locality is slowly being used as a rubbish dump Write a letter to the Mayor of your city pointing out the nuisance and danger of this Request that action be taken to stop this immediately

Or2 You being a boarder ordered a set of lab manuals from a famous book shop in the town They sent you a wrong set of books Write a letter to the manager of the book shop

Chemistry Chapter-1 1)CHEMICAL FORMULA- Q What is the Significance of

L-2The Language of Chemistrybull Chemical Formula

Itrsquos a symbolic representation of a chemical substance eg ndash The formula of Sulphuric acid is H2SO4

2) Steps of writing Chemical Formula of a given substance-

1 Write the symbols of the constituent atoms or radicals side by side Keep the basic radical on LHS and acid radical on the RHS ( Na+Cl- )2 In case of a radical having more than one atom( compound radical) enclose the radical in a bracket eg (SO4-)3 Write the valencies of each radical on its right hand top4 If the valencies of the two radicals are divisible by a common factor then divide the valencies by the common factor5 Invert (criss-cross) the valency number ie write the valency of one atom below the second atom and vice versa 6 On interchanging if valency number is lsquoone the figure lsquoonersquo is never writtenFor Example- Compound -Calcium Nitrate1 Writing the symbols- Ca(NO3)2 Writing the valencies on their right hand top- Ca2(NO3)1

3 Valency numeral in simple ratio- Ca2(NO3)1

4 Criss-cross- Ca 2NO3 1

5 Writing the formula of the compound- Ca(NO3)2

Chemical formula

A The formula of a substance conveys the following information regarding a substance 1 The name of the substance (qualitative)2 The elements constituting the substance (qualitative)3 The number of various atoms present in a molecule of the substance (quantitative)4 Molecular weight of the substance and the relative weights of different elements present in it (qualitative)

Q What are the limitations of Chemical Formula

A The chemical formula suffers from the following limitations-I It fails to convey whether the elements in a molecule are present in the form of atoms or ionsFor example the formula KBr fails to tell us whether Potassium and Bromine are present in the form of ions II It does not tell anything about the binding force that holds atom in a molecule togetherIII It does not tell us about the arrangement of various atoms with respect to one another within the molecule

Q Examples of Some Chemicals with their Formula Chemical name and Common Name-

A Given in the class notesCommercial Studies

Joint Stock Company

Let us discuss about the demerits of Joint Stock CompanyDespite so many advantages it has got many disadvantages which are as follows

Difficulty in FormationDelay in Decision makingExcessive Government ControlLack of Secrecy

Company can be classified into several categories based on incorporation

QuestionExplain the demerits of Joint Stock CompanyAnswer) 1 Difficulty in Formation The legal requirements and formalities required to be completed are so many The cost involved is quite heavy It has to approach large number of people for its capital It cannot start its business unless certificate of incorporation has been obtained This is granted after a long time when all the formalities are completed

Chartered CompanyStatutory CompanyRegistered Company

Delay in Decision making In this form of organization decisions are not made by single individual All important decisions are taken by the Board of Directors Decision-making process is time-consuming So many opportunities may be costly because of delay in decision-making Promptness of decisions which is a common feature of sole trader ship and partnership is not found in a company

Excessive Government ControlA company and the management have to function well within the law and the provisions of Companies Act are quite elaborate and complex At every step it is necessary to comply with its provisions lest the company and the management should be penalized The penalties are quite heavy and in several cases officers in default can be punished with imprisonment This hampers the proper functioning of the company

Lack of Secrecy The management of companies remains in the hands of many persons Every important thing is discussed in the meetings of Board of Directors Hence secrets of the business cannot be maintained In case of sole proprietorship and partnership forms of organisation such secrecy is possible because a few persons are involved in the management

2 Define the following

Chartered Company- The crown in exercise of the royal prerogative has power to create a corporation by the grant of a charter to persons assenting to be incorporated Such companies or corporations are known as chartered companies Examples of this type of companies are Bank of England (1694) East India Company (1600) The powers and the nature of business of a chartered company are defined by the charter which incorporates it After the country attained independence these types of companies do not exist

in IndiaStatutory Company- A company may be incorporated by means of a special Act of the Parliament or any state legislature Such companies are called statutory companies Instances of statutory companies in India are Reserve Bank of India the Life Insurance Corporation of India the Food Corporation of India etc The provisions of the Companies Act 1956 apply to statutory companies except where the said provisions are inconsistent with the provisions of the Act creating them Statutory companies are mostly invested with compulsory powersRegistered companiesCompanies registered under the Companies Act 1956 or earlier Companies Acts are called registered companies Such companies come into existence when they are registered under the Companies Act and a certificate of incorporation is granted to them by the Registrar

Economics

Chapter-4Basic problems of Economy

Today let us discuss with the topic Production Possibility curve

QuestionExplain the concept of Production Possibility Curve with the help of diagram

Answer) Production Possibility curve is a locus of all possible combinations of two commodities which can be produced in a country with its given resources and technology

The above diagram shows that with the given resources and technology the economy can produce maximum either 5 thousand meters of cloth or 15 thousand quintals of wheat or any other combination of the two goods like B( 1 thousand meters of cloth and 14 thousand quintals of wheat C ( 2 thousands meters of cloth and 12 thousand quintals of wheat) etcProduction Possibility curve is also called production possibility boundary or frontier as it sets the maximum limit of what it is possible to produce with given resources

Geography

Rotationand Revolution

SUNrsquoS POSITION AND SEASONAL CHANGES EQUINOXES ndash SPRING AND AUTUMN

Q1 What is Spring EquinoxA1 On 21st March sunrays fall directly on the equator On that day

As the Equator divides the Earth into two equal halves the sun rays fall directly on the equator twice in a year Equinoxes means equal Spring EquinoxOn 21st March sunrays fall directly on the equator On that day the duration of day and night both are equal ( 12 hours day and 12 hours night) on every places located on equator This day is called as Spring EquinoxAutumn EquinoxOn 23rd September sunrays fall directly on the equator On that day the duration of day and night both are equal ( 12 hours day and 12 hours night) on every places located on equator This day is called as Autumn Equinox

SOLSTICES ndash SUMMER AND WINTERDue to inclination of the Earth on its axis and the apparent movement of the sun the sun rays fall directly on both tropics once in a year Solstice is a Latin word which mean ldquothe Sun standing stillrdquoSummer SolsticesAfter 21st March there is an apparent movement of the Sun to the north of the equator The apparent northward movement up to 21st June when the Sun appears overhead at the Tropic of Cancer (22frac12degN) The sun appears to stand still at this position and then moves southwards towards the equator This position of the Sun on 21st June is known as Summer Solstices On that day the duration of day and night both are equal ( 12 hours day and 12 hours night) on every places located on Tropic of Cancer (22frac12degN)Winter solstices The apparent southward movement of the Sun continues beyond the equator till 22nd

December On this day the Sun is overhead at the Tropic of Capricorn

the duration of day and night both are equal ( 12 hours day and 12 hours night) on every places located on equator This day is called as Spring Equinox

Q2 What do you mean by EquinoxA2 Equinoxes means equal It is use to explain the equal duration of day and night ( 12 hours day and 12 hours night) on the Earth

Q3 On which date the longest day in Tropic of CancerA3 21st June

Q4 What is the meaning of SolsticeA4 Solstice is a Latin word which mean ldquothe Sun standing stillrdquo

Q5 Which is the longest day in southern hemisphereA5 22nd December

Q6 On what date does the Arctic Circle experience the lsquoMidnight SunrsquoA6 On 21 June the Arctic Circle experiences the lsquoMidnight Sunrsquo

Q7 What is cause of Midnight Sun in NorwayA7 During the summer solstice (21 June) the North Pole is inclined towards the Sun Therefore the duration of sunlight or daytime increases from 12 hours at the Equator to 24 hours at the Arctic Circle and beyond Thatrsquos why The region beyond the Arctic Circle especially Norway is known as the Land of the Midnight Sun because there the Sun does not rise or set on 21 June

Q8 Match the column A with BA B

Summer Solstice 21st March

Autumn Equinox 23rd

September

Winter Solstice 21st June

(22frac12degS) This position of the Sun is referred to as the Winter Solstice because it marks the winter season in the Northern Hemisphere On that day the duration of day and night both are equal ( 12 hours day and 12 hours night) on every places located on Tropic of Capricorn (22frac12degS)SEASONS AND DURATION OF DAY AND NIGHT During the equinoxes all places on the Earth have 12 hours of day and 12 hours of night Due to the revolution of the Earth round the Sun on an inclined axis the duration of day and night varies according to seasons and the latitude of a placeDuring the summer solstice (21 June) the North Pole is inclined towards the Sun Therefore the duration of sunlight or daytime increases from 12 hours at the Equator to 24 hours at the Arctic Circle and beyondThe region beyond the Arctic Circle especially Norway is known as the Land of the Midnight Sun because there the Sun does not rise or set on 21 JuneAt the North Pole there will be six months of daylight The Sun will be seen always above the horizon at a low angle At 66degN 24 hours of sunlight can be seen only on 21 June Hammerfest in northern Norway is a place of tourist attraction for observing the phenomenon of the Midnight Sun This place has continuous daylight from 13 May to 29 July This place is easily accessible to tourists and has hotels and other facilities The view of the midnight Sun from here is enthrallingIn the Southern Hemisphere the duration of daylight decreases from 12 hours at the equator to 0 hours beyond the Antarctic Circle In the South Polar Region there is 24 hours of darkness The Sun is always below the horizon In the Southern Hemisphere which experiences winter the duration of night-time is longer than the duration of daylight

Spring Equinox 22nd

December

A8 A B

Summer Solstice 21st June

Autumn Equinox 23rd

September

Winter Solstice 22nd

December

Spring Equinox 21st March

During winter solstice (22 December) the South Pole is inclined towards the Sun The Southern Hemisphere experiences summer and the Northern Hemisphere has winter Therefore the duration of daylight or sunlight is greater in the Southern Hemisphere than in the Northern HemisphereThe duration of daylight increases from 12 hours at the equator to 24 hours beyond the Antarctic Circle The South Polar Region has 24 hours of sunlight for many days continuously At the South Pole there will be six months of sunlight The Sun will always be seen at a low angle above the horizon In the Northern Hemisphere the duration of daylight will decrease from 12 hours at the equator to 0 hours at the Arctic Circle There are 24 hours of darkness in the North Polar region The duration of night is greater than the duration of daylight as one move northwards from the Equator It is evident from the above table that the duration of daylight is 12 hours throughout the year at the equator only As one moves away from the equator the seasonal variations in the duration of daylight increase The seasonal variations in the duration of daylight are maximum at the Polar Regions

Subject Eng Literature (The Merchant of Venice ndash William Shakespeare)Topic Act II Scene 7 Lines 36 to 80 (End of scene ) [Students should read the original play and also the paraphrase provided]

Summary Questions amp AnswersThe Prince then examines the inscription on the silver casket which says ldquoWho chooseth me shall get as much as he deservesrdquo The Prince says that he deserves Portia more than anybody else because of his high rank his noble birth and his great wealth and power But then he argues that silver is ten times

(1) (Act II Sc 7 L 39-47)

From the four corners of the earth they come

To kiss this shrine this mortal breathing saint

The Hyrcanian deserts and the vasty wildsOf wide Arabia are as through-fares now

inferior to gold and therefore he cannot believe that the portrait of such a beautiful lady as Portia can be contained in the silver casket He decides to see the inscription on the golden casket before making his decision

The Prince goes to examine the inscription on the golden casket which says ldquoWho chooseth me shall get what many men desirerdquo The Prince believes that the whole world desires to possess Portia otherwise so many suitors would not have come from all corners of the world for winning Portia Some of them have come from the distant lands of Persia and Arabia The deserts of Persia (Hyrcanian deserts) and the boundless desolate lands of Arabia have been crossed by the Princes seeking the hand of Portia He contrasts this casket containing Portiarsquos portrait with the old English gold coin bearing the image of the archangel (angel of the highest rank) He goes on to remark that while the figure of the archangel is engraved (Insculped) upon the English coin the picture of Portia who is beautiful as an angel lies hidden inside one of the caskets namely the Golden Casket (Golden Bed)

On the basis of his assessment of the inscription on the golden casket the Prince decides to choose the golden casket He asks for the key and opens the golden casket only to find therein an empty human skull holding a roll of

For princes to come view fair PortiaThe watery kingdom whose ambitious headSpets in the face of heaven is no barTo stop the foreign spirits but they comeAs orsquoer a brook to see fair Portia

(i) Explain the occasion for the above mentioned speech

These are the comments of the Prince of Morocco after he reads the inscription on the golden casket His mental process is revealed to us in these words We find him debating within himself as to which casket he should choose

(ii) What light does the above speech throw on the personality of Prince of Morocco

From the above mentioned speech we come to know that the Prince of Morocco is keen to marry Portia He is the type of person who is easily taken away by outward appearance He is in love with Portia because of her beauty

(iii) What information can you gather about Portia from the above mentioned lines

The given speech shows that Portia is a very beautiful lady She must be possessed of good qualities because many suitors come to her place from all over the world with a desire to get married to her The Prince of Morocco is so impressed by her beauty that he calls her a saint According to him the whole world is desirous of having her

(iv) Elucidate the significance of the first two lines

In these lines the Prince of Morocco pays a compliment to Portia These lines show his admiration for her He says that people come from all parts of the world to see fair Portia

(v) Explain the meaning of the last four lines of the

passage

In these lines the Prince of Morocco says that even the vast oceans which throw a challenge at the sky are unable to prevent men from coming to Portiarsquos place to have a glimpse of her These lines are also a tribute to Portiarsquos beauty and good qualities Many men voyage across the ocean treating it as a mere stream to see the beautiful Portia

paper in which is written that whoever happens to be guided by the glitter of things is invariably deceived

On reading the scroll the Prince says that he is too sad at heart to speak a more formal farewell and leaves with his followers amidst a sound of trumpets

After the Prince of Morocco leaves Portia remarks that the Prince is a gentle fellow but she is rid of him May all persons of his nature make a similar choice

IMPORTANT PASSAGES EXPLAINED

(Act II Sc 7 L 39-43)From the four corners of the earth they come

To kiss this shrine this mortal breathing saintThe Hyrcanian deserts and the vasty wildsOf wide Arabia are as through-fares nowFor princes to come view fair Portia

Context

This passage occurs in Act II Scene 7 in The Merchant of Venice This is part of the speech made by the Prince of Morocco

(2)

(Act II Sc 7 L 48-53)

MOROCCO One of these three contains her heavenly pictureIst like that lead contains her

Twere damnation To think so base a thought it were too grossTo rib her cerecloth in the obscure graveOr shall I think in silver shes immurdBeing ten times undervalued to tried gold

(i) What meaning does the Prince of Morocco find out of the inscription of the golden casket What have Belmont and Portiarsquos house been called and why

The inscription on the golden casket is ldquoWho chooseth me shall gain what many men desirerdquo The Prince finds out that it means that the chooser of the golden casket will get Portia because many men desire her In fact the entire world desires her Because of the coming of many suitors to Belmont from different countries in order to win Portiarsquos hand Belmont has become a centre of pilgrimage and her house is the shrine where saintly Portia is installed

(ii) What does the Prince of Morocco do before making the final choice of the casket Which is the correct casket and who will win Portiarsquos hand

The Prince of Morocco surveys and analyses the inscriptions on the casket of lead silver and gold Before making the final choice like a very systematic and methodical person he once again considers the claims of the caskets The casket containing Portiarsquos picture is the correct casket and the person choosing it will win Portiarsquos hand

Explanation

While praising Portia the Prince of Morocco conceives Portia as a goddess whose image is placed inside one of the caskets Many suitors are coming from far and wide the north and the south the east and the west (Four corners) in order to try their luck Some of them have come from the distant land of Persia and Arabia The deserts of Persia (Hyrcanian deserts) and the boundless desolate lands of Arabia have been crossed by the Princes seeking the hand of Portia All this shows that Portia is indeed the most beautiful lady of the world

(iii) What does the Prince of Morocco say in his estimation while examining the motto on the silver casket What does he find in the golden casket

While examining the motto on the silver casket which says ldquoWho chooseth me shall get as much as he deservesrdquo Morocco says that in his own estimation he surely deserves Portia in all respects ndash rank birth wealth etc

He chooses the golden casket When he opens it he finds an empty human skull holding a scroll in which it is written that those who are attracted by the glittering outside of things are always deceived as Morocco has been deceived

(iv) What kind of nature does the Prince of Morocco have

The Prince of Morocco has a simple nature who does not look deeply into the inner meaning of things but is dazzled by the outward appearance of gold He is inclined to over-estimate his own value and does not realize that it is a duty to ldquogive and hazardrdquo To say that he will not hazard for lead shows that he misreads the true meaning of the inscription which is that he should be prepared to ldquohazard all he hathrdquo for Portia So his feeling is only one of fascination and romantic attraction

(v) Do you think that the lottery of the caskets is not a matter that will be determined by chance

In fact the lottery of the casket is not a matter that will be determined by mere chance but that it is a true test of character and of sincerity which is amply proved not only by Moroccorsquos choice but also by the arguments which he uses to help him in his choice

(Act II Sc 7 L 55-59)

They have in England

A coin that bears the figure of an angelStamped in gold but thats insculpd uponBut here an angel in a golden bedLies all within

Context

(3)

(Act II Sc 7 L 63-77)A carrion Death within whose empty eye

There is a written scroll Ill read the writing

All that glisters is not goldOften have you heard that toldMany a man his life hath soldBut my outside to beholdGilded tombs do worms infoldHad you been as wise as boldYoung in limbs in judgment oldYour answer had not been inscrolld

This passage occurs in Act II Scene 7 in The Merchant of Venice This is part of the speech made by the Prince of Morocco

Explanation

In this passage the Prince of Morocco bestows high praise on Portia whose hand he is seeking He contrasts this casket containing Portiarsquos portrait with the old English gold coin bearing the image of the archangel (angel of the highest rank) He goes on to remark that while the figure of the archangel is engraved (Insculped) upon the English coin the picture of Portia who is beautiful as an angel lies hidden inside one of the caskets namely the Golden Casket (Golden Bed) In the day of Elizabeth silver was ten times inferior in value to gold Therefore the Prince of Morocco believing that Portiarsquos portrait is contained in the Golden Casket decides to choose the Golden Casket

Fare you well your suit is coldCold indeed and labour lostThen farewell heat and welcome frostmdashPortia adieu I have too grievd a heartTo take a tedious leave Thus losers part

(i) What reward does the Prince of Morocco get after making a wrong choice of the Casket How does he feel

After making the wrong choice in selecting the casket of gold the Prince of Morocco as a reward earns a rebuke in the form of a scroll tucked in the empty eye-socket of a skull kept in the casket of gold The Prince is shocked and disappointed He becomes all the more sad and dejected when he reads the scroll which points to his foolishness in being misled by the appearance and outward show as indicative of its worth

(ii) How does the Prince respond after reading the scroll

After reading the scroll the Prince though upset accepts the result with good grace and decorum befitting a royal suitor and true sportsman He says that his love-suit is really cold otherwise he would have chosen correctly but now his efforts have been in vain So he bids farewell to Portia to the warmth and enthusiasm of love and welcomes the cold and bitterness of dejection and misery of life which lies ahead

(iii) What request does he make to Portia and why

After being failure in his mission he requests Portia to give him permission to leave at once because he is too sad to undergo the tediousness of a formal leave-taking He tells that it is the manner in which defeated persons part unceremoniously

(iv) Explain the following lines

ldquoAll that glisters is not goldOften have you heard that toldMany a man his life hath soldBut my outside to beholdGilded tombs do worms infoldrdquo

Mere glitter does not make a metal to be gold Man has often been warned against appearance but it has been of no use Many people have sacrificed their lives only to seek the outer appearance of gold Worms are found inside the gilded

monuments

Class XSubject Topic Summary Execution

Hindi 2ndlang

नया रासता भाग 6 मायाराम 0ी घर म धनी मल 0ी और उनी बटी सरिरता ी ही चचा बनी रहती थी अमिमत ो इसम ोई रलिच ना थी वह धनी घर ी लडी स शादी र सवय ो बचना नही चाहता था उसा भी सवाणिभमान ह ईशवर ी पा

स उस पास पस ी ोई मी नही थी अभी उसन फकटरी ही लगाई थी उसी समझ बाहर था कि उस घर वालो ा झाव पस ी तरफ कयो

ह उसन मा स सवाल किया कि मा तम सरिरता स मरी शादी कयो रना चाहती हो मा न उस समझाया कि वह दखन म बरी नही ह और किफर खानदान अचछा

ह वह ए शल गरहणी रप म घर सभाल सगी अमिमत न मा ो इस बात ा एहसास राया कि मीन सबध लिलए मना रन पर उस दिदल

पर कया बीती होगी मा और अमिमत ी लडी बार म ाफी बात हईमा ा झाव सरिरता ी तरफ था कयोकि वह घर पर अचछा दह0 लर आ रही

थी अमिमत न अपनी मौसी ी बरी हालत बार म बताया कि किस तरह वह बड घर ी खानदानी बटी लाई थी और आ0 उसी हालत कितनी खराब ह लाई थी बहकलब 0ाती ह और बचचो ो भी नही दखती ह बात चल ही रही

थी कि तभी ए ार बाहर आर री धनी मल0ी घर अदर आए और पीछ स डराइवर फल ी ए टोरी लर आया अदर आए और पीछ स

डराइवर ए टोरी फल ी लर आया अमिमत ो फल ी पटी बरी लग रही थी अमिमत न पछ लिलया यह फल कयो ल आए ह प इन सब ी कया

0ररत थी उनो न 0वाब दिदया कि 4 पटी शमीर स मगाए थ अमिमत ो या सनर करोध आ गया तभी उस किपता 0ी आ गए उन आत ही अमिमत उठर बाहर चला गया वहा वहा मा पास आर बठ गया और बोला

अभी रिरशता तय नही हआ और धनी मल 0ी धनी मल 0ी फल ी पटी लर चलआय मा न समझाया कि 0ब सबध 0ड 0ाता ह तो खाली हाथ नही

आत अमिमत न मा स हा कि तम सबन सरिरता ो इस घर म लान ी ठान रखी ह धनीमल 0ी उस दिदन सरिरता ो दखन ी तारीख तय रन आय थ

Commercial Studies

Banking Nowadays Bank provide easy and quick services through internet facilities methods of Banking is called internet bankingIn order to save the time and money involved in visiting Bank branches people increasingly prefer to have internet banking

There are different modes of doing internet banking or transferring money through online They areReal Time Gross Settlement (RTGS)National Electronic Fund Transfers (NEFT)

1

Question

1) Explain the term RTGS Write the features of RTGS

Answer)The acronym RTGS stands for Real Time Gross Settlement which may be defined as the continuous real time settlement of funds transfer individually on and order by order basis without netting lsquoReal timersquo may be defined as the processing of instructions at the time they are received rather than at some letter time lsquoGross settlementrsquo may be defined as the settlement of transfer instructions which occurs

individually

Features of RTGS1It is the continuous settlement of

funds transfer individually on an order by order basis

2RTGS facility is provided only by CBS core banking solution enabled Bank branches

3Amount charged from the customer for RTGS transactions vary from bank to bank

2) Explain the term NEFT Write the features of NEFT

Answer) National electronic funds transfer may be defined as a nationwide system that facilitates individuals Farms and copper operates to electronically transfer funds from any bank branch to any individual farm or corporate having an account with any other bank branch in the country

Features of NEFT2 Transfer can be made 7 times on

weekdays and 6 times on Saturday

3 NEFT cannot be used to receive foreign remittances

4 NEFT transaction takes place in batches

5 A bank branch must be NEFT enabled to become a part of NEFT fund transfer network

6 There is no maximum or minimum amount that can be transferred through NEFT when one bank has a bank account

English Language

CompositionEssay

A composition is an art of creating a piece of writing on any topic or subject It is the writing correctly beautifully and clearly in order to make some interesting reading Structure of the composition

Introduction ( you lay the foundation for your composition)

Body (it constitutes the main part of the essay)

Conclusion (final statement that leaves a lasting impression)

Kinds of essays1 The Narrative essay2 The descriptive essay3 The reflective essay4 The argumentative essay

Write a composition on any one of the following topics (350- 400 words)

1 Friendship Or2 The first day of your school

Subject Eng Literature (The Merchant of Venice ndash William Shakespeare)Topic Act V Scene 1 Lines 127 to 158 (Nerissa helliphellip The clerk will nersquoer wear hair onrsquos face that had it) [Students should read the original play and also the paraphrase given in the school prescribed textbook]

Summary Revision Questions o Soon thereafter Bassanio Gratiano

and Antonio arrive

o Bassanio tells Portia that he is feeling as if it is morning because of the presence of Portia who is shining like the sun When Antonio is introduced by Bassanio to Portia she tells Bassanio that he should be grateful to Antonio who took so much trouble on his account even to the extent of risking his life

o Nerissa starts quarrelling with Gratiano and demands that he show her the ring she had presented to him and which she had warned him not to lose She suspects that Gratiano must have presented the ring to some young woman and not to the lawyerrsquos clerk as he repeatedly says and assures

Answer the following questions to check your preparation of Act IV Scenes 1 and 2

You must attempt only after you have completed your preparation of Act IV The answers must be in complete sentences using textual evidence (with citation) when necessary

[It would be in your own interest to attempt the above questions honestly totally refraining from consulting your textbook or your notes during answering After completion you should correct the paper yourself consulting the textbooknotes etc and award marks as specified Please let me know the marks you scored through WhatsApp in the group or to my personal WhatsApp]

Act IV Scene 1 (each question carries 2 marks)

1 What did the Duke try to do for Antonio

2 Why does Shylock refuse to show mercy How does he justify his stance

3 Why does Antonio say he is ready to die 4 What information is contained in Bellariorsquos letter

5 Why does Portia (as Balthazar) assert that Shylock must show mercy How does he respond

6 What offers are made to Shylock to get him to spare Antonio How are they received

7 What does Antoniorsquos speech as he faces the prospect of Shylockrsquos knife tell you about his character

8 How do Bassanio and Gratiano react to the looming prospect of Antoniorsquos demise

9 How does Portia (as Balthazar) use the law to turn the tables on Shylock

10 What does the Duke decree should happen to Shylock Why What happens to Shylockrsquos estate

11 What does Portia ask Bassanio as payment for her ldquoservicesrdquo What is his initial response What makes him change his mind

Act IV Scene 2 (each question carries 1frac12 marks)

1 What does Gratiano bring to Portia (Balthazar)

2 What does Nerissa plan on getting from Gratiano What does Portiarsquos comment suggest about men

ECO-10 280620 Topic-Supply AnalysisSHIFTING OF SUPPLY

But if there is change in factors other than the price of the commodity then either more is supplied at the same price or less supplied at the same price In such cases the price of the commodity remains constant but there is a change in other factors like change in the price of inputs change in technology of production change in price of other related goods change in taxation policy of the government etc For example there is an improvement in the technology of production of the commodity in question It leads to decrease in per unit of cost production of the commodity The firm is willing to sell more quantity of the commodity at the same price So the supply other commodity increases at the same price This increase in supply is shown by rightward shift of supply curve On the other hand if the firm uses inferior technology of production the cost of production per unit of the commodity increases The firm is willing to sell less quantity at the same price So the supply of the commodity decreases at the same price This decrease in supply is shown by leftward shift of the supply curve The above cases of increase and decrease in supply can be shown with the help of the following figures

Y INCREASE OF SUPPLY Price (Rs) s

P A s1

B

s

X` O s1 X

q q1

Y` Quantity demanded (in units)

Y DECREASE IN SUPPLY s2

s

price (Rs)

C

p A

s2

s

X` o X

q2 q

Y` Quantity demanded ( in units)

Main factors causing increase in supply or rightward shift of supply Curve(i) Fall in the price of other related goods

(ii) Fall in the price of inputsfactors(iii) Use of better technology in production(iv) Decrease in the rate of excise duty by government(v) If the objective of producer changes from profit maximization to salesMaximization

Main factors causing decrease in supply or leftward shift of supply curve(i) Increase in the price of other related goods(ii) Rise in the price of inputsfactors(iii) Use of inferior technology in production(iv) Increase in the rate of excise duty by the government(v) If the objective

Subject - Biology Topic ndash Chapter mdash6 PhotosynthesisSummary Execution

Today we will know about photosynthesis and its stages

Q1 What do you mean by photosynthesis The process by which living plants containing chlorophyll produce food

substances from carbon-di- oxide and water by using light energy Sunlight

6CO2 +12 H2O----------------------- C6 H12O6 + 6H2O + 6O2

Chlorophyll

Q2 What are the importance of photosynthesis I) Food for all Green plants trap solar energy by photosynthesis

process and supply food and energy for all living organisms either directly or indirectly

Ii) Oxygen to breathe in by product of photosynthesis is oxygen which is essential for all living organisms respiration

Q3 Write about two main phases of photosynthesis A Light dependent phase This phase occur in grana of chloroplast I) The chlorophyll on exposure to light energy becomes activated by

absorbing photons Ii) The absorbed energy is used in splitting the water molecules (H2O)

into its two components (H+ and OH- ) and releasing electron s 2H2O------------------------- 4H+ + 4e- +O2

Energy of 4 photons This reaction is known as photolysis

End products are H+ and oxygen water

B Light independent (Dark ) phase The reactions in this phase require no light energy

Here CO2 combine with H+ and produce glucose

Class XI

Subject Topic Summary ExecutionEVS Chapter-4 Legal

regimes for sustainable development

Environmental legislationEnvironmental legislation is the collection of laws and regulations pertaining to air quality water quality the wilderness endangered wildlife and other environmental factors The act ensures that matters important to the environment are thoroughly

Learn -The Forest (Conservation) Act 1980

considered in any decisions made by federal agencies

The Forest (Conservation) Act 1980 The Forest (Conservation) Act 1980 an Act of the Parliament of India to provide for the conservation of forests and for matters connected therewith or ancillary or incidental thereto It was further amended in 1988 This law extends to the whole of IndiaObjects and Reasons of the Forest Conservation Act

Deforestation causes ecological imbalance and leads to environmental deterioration Deforestation had been taking place on a large scale in the country and it had caused widespread concern The act seeks to check upon deforestation and de-reservation of forests

Subject Eng Literature (The Tempest ndash William Shakespeare) Topic Act II Scene 1 Lines 314 to 329 (End of scene)

[Students should read the original play and also the paraphrase given in the school prescribed textbook]Summary Questions amp Answers

Conspiracy of Antonio and Sebastian (Contd)

o As they approach Ariel appears again and wakes up Gonzalo by singing a tune in his ear Alonso also wakes up and they see both Sebastian and Antonio with drawn swords On being caught off guard they make up a story saying that they had heard a bellowing of bulls or lions

o They then moved to another part of the island

o Ariel at once rushes to Prospero to inform him of this development

SUMMING-UP of ACT-2 SCENE-1

(i) Among the survivors Ferdinand is separated from the rest which results in the disconsolate grief of Alonso as he took him for dead

(ii) The villainy of Antonio is confirmed

(iii) The supremacy of Prosperorsquos magic which resulted in the failure of the human conspiracy

(1)

(Act II Sc 1 L 311-325)SEBASTIAN Whiles we stood here securing your repose

Even now we heard a hollow burst of bellowing Like bulls or rather lions Didt not wake youIt struck mine ear most terribly

ALONSO I heard nothingANTONIO O rsquotwas a din to fright a monsters ear

To make an earthquake Sure it was the roarOf a whole herd of lions

ALONSO Heard you this GonzaloGONZALO Upon mine honour sir I heard a humming

And that a strange one too which did awake meI shaked you sir and cried As mine eyes opened I saw their weapons drawn There was a noiseThats verily rsquoTis best we stand upon our guardOr that we quit this place Lets draw our weapons

(i) Why has Prospero sent Ariel to Gonzalo and Alonso What does Ariel do to awaken Gonzalo

Prospero has already come to know by his magic powers the danger which threatens Gonzalo who had been Prosperorsquos friend and so he sent Ariel to preserve the lives of both Gonzalo and Alonso Prospero does not want that his scheme should remain unfulfilled Ariel begins to sing a song in Gonzalorsquos ears to awaken him(ii) Who are ready to carry out their plan Who takes steps to stop them Why does Gonzalo feel surprised after being awakened

Sebastian and Antonio are ready to carry out their plans They are standing with their swords drawn to kill Alonso and

(iv) We see two sets of contrasting characters Gonzalo-Adrian against Antonio-Sebastian

(v) The grief that works in Alonso can be perceived to his repentance for his association in Antoniorsquos crime against Prospero

Gonzalo Ariel takes steps to stop them from carrying out their nefarious scheme When Gonzalo is awakened by the song sung by Ariel into his ears he (Gonzalo) feels surprised because he sees Sebastian and Antonio standing with their swords drawn(iii) What reason do Sebastian and Antonio tell of drawing their swords when they are suspected by Alonso and Gonzalo

When Sebastian and Antonio are seen with their swords drawn they are looked with suspicion by Gonzalo and Alonso At first Sebastian tells them that as they stood here to guard them during their sleep they heard only a little before a sudden loud noise very much like the roaring of bulls or more probably that of lions Then Antonio follows him saying that this was a noise so terrible as to frighten even a monsterrsquos ears and this noise could even have shaken the earth and it was surely like the roaring of a multitude of lions Then seeing the danger they have drawn their swords Perhaps after hearing the terrible noise they (Gonzalo and Alonso) woke up from their sound sleep

(iv) What does Gonzalo tell Alonso about the strange noise What did he see on opening his eyes Gonzalo tells Alonso that he did not hear the sound of roaring but he heard a humming sound which was strange and which woke him up After waking up he gave him (Alonso) a shaking and a loud cry On opening his eyes he saw these two gentlemen standing with their swords drawn(v) What does Gonzalo suggest

Gonzalo suggests that there was a noise indeed and of that he has no doubt at all and suggests that the best course for them would be to remain alert and vigilant against any possible danger to their lives or to leave this place and move to some other part of the island

Class XIISubject Topic Summary Execution

Commerce

Chapter- Management

Today we will discuss about LEVELS OF MANAGEMENT

Levels of management is a series or chain of managerial positions from top to bottom It helps individuals to know their authority responsibilities and superior-subordinate relations among themselves There are mainly three levels of Management TOP LEVEL MANAGEMENTMIDDLE LEVEL MANAGEMENTLOWER LEVEL MANAGEMENT

Top level managementIt consists of members at the highest level in the management hierarchy This level includes Board Of Directors Chief Executive Managing Directors Chairman President Vice President

Rolefunctions of the top levelmanagement1To analyse evaluate and deal

with theexternal environment2 To determine the objectives and

policies of the business3 To strive for welfare and survival

of business

4 To create an organisational Framework consisting of authority responsibility relationship

Middle level management Congress of members or groups who are concerned with implementation of the policies let down by the top managementThis level includes head of the department such as finance manager marketing manager branch and regional managers departmental and divisional heads plant superintendent etc

Role of functions of the middle level management

1 To interpret the policies framed by top management

2 To assign duties and responsibilities to lower level managers

3 To select and appoint employees for middle and supervisory level and evaluate their performance

4 To co-operate with other departments for smooth functioning

Operational or supervisory level managementIt refers to the group are members who are concerned with execution of the work They are also known as fast line managers This level includes supervisor 4 men Section Officer clerk Inspector etc

Role of functions of the lower level management1 To plan and execute day-to-

day operations2 To supervise and control the workers3 To arrange materials and

tools to start the process and make arrangements for training

4 Today present workers grievance and suggestions before the management and

ensure safe and proper working conditions in the factory

Business Studies

Staff Appraisal Chapter- 10 Today let us start with a new chapter

Staff Appraisal

Meaning of Performance Appraisal

Performance Appraisal is the systematic evaluation of the performance of employees and to understand the abilities of a person for further growth and developmentThe supervisors measure the pay of employees and compare it with targets and plansThe supervisor analyses the factors behind work performances of employeesThe employers are in position to guide the employees for a better performance

Objectives of Performance Appraisal

Following are the objectives of Performance Appraisal

To maintain records in order to determine compensation packages wage structure salaries raises etc

To identify the strengths and weaknesses of employees to place right men on right job

To maintain and assess the potential present in a person for further growth and development

To provide a feedback to employees regarding their performance and related status

To provide a feedback to employees regarding their performance and related status

Importance of Performance Appraisal

Performance appraisal provides important and useful information for the assessment of employees skill

knowledge ability and overall job performance The following are the points which indicate the importance of performance appraisal in an organization

1 Performance appraisal helps supervisors to assess the work performance of their subordinates

2 Performance appraisal helps to assess the training and development needs of employees

3 Performance appraisal provides grounds for employees to correct their mistakes and it also provides proper guidance and criticism for employees development4 Performance appraisal provides reward for better performance

5 Performance appraisal helps to improve the communication system of the organization

6 Performance appraisal evaluates whether human resource programs being implemented in the organization have been effective

7 Performance appraisal helps to prepare pay structure for each employee working in the organization

8 Performance appraisal helps to review the potentiality of employees so that their future capability is anticipated

Geography

DRIANAGE The SubarnarekhaThe Subarnarekha and the Brahmaniinterposed between the Ganga and the Mahanadi deltas drain an area of 19300 sq kmand 39033 sq km respectively The drainage basins of these streams are shared byJharkhand Odisha west Bengal and Chhattisgarh The Brahmani is known as southKoel in its upper reaches in Jharkhand

The NarmadaThe Narmada rises in the Amarkantak hills of MadhyaPradesh It flows towards the West in a rift valleyformed due to a geological fault The total length of it is 1300 km All the tributaries of the

Q1 Name the two westward flowing rivers in the peninsular plateauA1 Narmada and Tapi are the only westward flowing rivers of the peninsular plateau

Q2 Differentiate between east-flowing rivers and west-flowing riversA2

East-flowing rivers

West-flowing rivers

Narmada are very short inlength Most of its tributaries join the main streamright anglesThe Narmada basin covers parts of Madhya Pradesh and Gujarat

The Tapi The Tapi rises in the Satpura ranges in the Betul listrictof Madhya Pradesh It flows in a rift valley parallel tothe Narmada but it is much shorter in length It coversparts of Madhya Pradesh Gujarat and MaharashtraThe length is about 724 km

The Sabarmati and the MahiThe Sabarmati rises in the Aravali hills and flows south-south-westwards for a distance of 300 kilometres to the Arabian Sea The Sabarmatibasin extends over an area of 21674 sq km in Rajasthan and Gujarat The Mahi rises inthe east of Udaipur and drains an area of 34842 sq km lying in Madhya PradeshRajasthan and Gujarat It flows south-westwards for a distance of 533 km before it fallsinto the Gulf of Khambhat

The ChambalThe Chambal rises near Mhow in the Vindhya Range and flows towards the northgenerally in a gorge upto Kota Below Kota it turns to the north-east direction and afterreaching Pinahat it turns to the east and runs nearly parallel to the Yamuna beforejoining it in the southern part of the Etawah district in Uttar PradeshMajor Rivers of India with their basin area (Sqkm)

Himalayan System Indus 321290Ganga 861404

Brahmaputra 187110Indus System

Jhelum 34775Beas 20303

Ganga System Yamuna 366223Ghaghra 127950

Peninsular RiversNarmada 98796

Tapi 65145Mahanadi 141600

Subarnarekha 19300Sabarmati 21674

Mahi 34842Godavari 312812

Godavari Krishna Kaveri Mahanadi are the east-flowing rivers

Narmada Tapi west-flowing rivers

They fall into the Bay of Bengal

They fall into Arabian Sea

These rivers form big deltas

These rivers form comparativelysmall deltas

Catchment areas of these rivers are larger

Catchment areas of these rivers are smaller

Krishna 2589488Cauveri 87900

Subject ndashBiology Topic ndashChapter -5 Inheritance amp Variations Summary ExecutionToday we will discussabout linkage and its classification

LINKAGE The tendency of the genes located on the same chromosome to stay together is

hereditary transmission Linked genes the genes responsible for this Genes that exhibit the process of linkage locates in the same chromosome The distance between the linked genes in a chromosome determines the strength

of linkage i e genes that are located close to each other show stronger linkage than that are located far from each other

COMPLETE LINKAGE It is the type of linkage showed by the genes that are closely located or are tightly

linked with each other as they have no chance of separatingby crossing over These genes are always transmitted together to the same gamete and the same

offspring In such condition only parental or non cross over type of gametes are formedINCOMPLETE KINKAGE It is type of linkage showed by the genes that are distantly located orare loosely

linked with each other because they have chance of separating by crossing over

SIGNIFICANCE i) It helps in holding the parental character togetherii) It checks the appearance of new recombination and helps in bringing the

hybrid population which resembles the original parents iii) Linked genes dilute the effects of undesirable traits

Subject Eng Literature (The Tempest ndash William Shakespeare) Topic Essay Questions (EQ-3)Question No 3

Give a character sketch of CalibanAnswer

The character of Caliban has been wonderfully conceived by Shakespeare as the manifestation of all that is gross and earthy ndash a sort of creature of the earth as Ariel is a sort of creature of the air

Calibanrsquos Physical Appearanceo Caliban is lsquofreckledrsquo a lsquomisshapen knaversquo not honoured with human shape

o Prospero calls him lsquothou tortoisersquo (Act I Sc 2 Line 317) Trinculo stumbling upon him describes him as ldquoA strange fish hellip Legged like a man And his fins like armsrdquo He ldquosmells like a fishrdquo (Act II Sc 2 Line 25)

o Prospero also calls him a ldquobeastrdquo (Act IV Sc 1 Line 140) and ldquoThis misshapen knaverdquo (Act V Sc 1 Line 268)

o Further it appears that in addition to his physical deformity his spiritual inferiority is also suggested by Prosperorsquos claim that his birth resulted from the union between his mother the witch Sycorax and the devil

Calibanrsquos ParentageWhen the play opens Caliban is twenty four years of age having been born on the island twelve years before the coming of Prospero His mother was the foul witch Sycorax who was banished from Algiers for ldquomischiefs manifold and sorceries terrible to enter human hearingrdquo (Act I Sc 2 Line 264) and the father was the Devil himself Thus

Caliban is a monster of evil and brute nature ugly deformed and stinking

Calibanrsquos Savage and Malignant Natureo Caliban is entirely a creature of the earth ndash gross brutal and savage He regards himself as the rightful possessor

of the island and Prospero as a usurper

o In his young age he was on good terms with Prospero He had consented to be received by Prospero at his house and to be educated by him He has learnt human language only to curse his master whom he abhors

o His beastly nature soon breaks out and ends in a vicious attack on Miranda This opens the eye of Prospero who becomes severe to him and enforces his service by threats and violence

o Prospero uses him to make dams for fish to fetch firewood scraper trenches wash dishes and keep his cell clean

Calibanrsquos Hatred for ProsperoA profound hatred for Prospero has taken hold of Caliban It springs from a sense of his being dispossessed and ill-treated He would kill Prospero if he could but he knows the power of Prosperorsquos lsquobookrsquo Hence he transfers his allegiance to Stephano who seems like a god to him He also incites the two drunken associates to batter the skull of Prospero when he sleeps in the afternoon

Caliban Shows Considerable Intelligenceo He has learnt Prosperorsquos language

ldquoYou taught me language and my profit onrsquot (Act II Sc 2 Lines 86-89)Is I know how to curserdquo

o He is well aware of the futility of arguing with one who has more power than he has

ldquoI must obey his art is such power (Act I Sc 2 Lines 373-376)It would control my damrsquos god SetebosAnd make a vassal of himrdquo

o He realizes the importance of Prosperorsquos books

ldquoRemember (Act III Sc 2 Lines 89-92)First to possess his books for without themHersquos but a sot as I am nor hath notOne spirit to commandrdquo

o He knows the value of stealth when attacking the enemy

ldquoPray you tread softly that the blind mole may not (Act IV Sc 1 Lines 194-195)Hear a foot fall we now are near his cellrdquo

o Caliban has a better set of values than Stephano and Trinculo They are distracted from their plan by their greed for Prosperorsquos rich garments Only Caliban realizes that such a finery is unimportant

ldquoLeave it alone thou fool it is but trashrdquo (Act IV Sc 1 Lines 224)

Caliban is not a good judge of characterCaliban is not a good judge of character He decides for example that Stephano is a god because he dispenses lsquocelestial liquorrsquo (Act II Sc 2 Line 115) but then it must be remembered that he has only known his mother Sycorax Prospero Miranda and the spirits that torture him However he quickly discovers his error of judgementrdquo

ldquoWhat a thrice-double ass (Act V Sc 1 Lines 295-297)Was I to take this drunkard for a godAnd worship this dull foolrdquo

Calibanrsquos Imaginative NatureIf Caliban is sub-human in what has been said above he is human in the respect of the poetic side of his character He listens to music with rapture He tells of the beautiful dreams in which heaven rains treasures upon him and which upon waking he yearns to renew One of the most poetic passages in whole play is Calibanrsquos description of the island

to Stephano and Trinculo

ldquoBe not afeard The isle is full of noises (Act III Sc 2 Lines 135-143)Sounds and sweet airs that give delight and hurt notSometimes a thousand twangling instrumentsWill hum about mine ears and sometime voicesThat if I then had waked after long sleepWill make me sleep again and then in dreamingThe clouds methought would open and show richesReady to drop upon me that when I wakedI cried to dream againrdquo

Caliban - Less Ignoble Than Some OthersCalibanrsquos motive for murder is less dishonourable than that of Antonio and Sebastian They plan to kill Alonso to gain his power and wealth Caliban merely wants revenge and the return of lsquohisrsquo island

Conclusiono Calibanrsquos character is not portrayed very clearly in the play and hence we cannot decide whether he is a poor

savage being grossly maltreated by Prospero or whether he is evil and must therefore be kept in bondage or enslavement

o Caliban is contrasted with Ariel who is a spirit and thus swift and uninterested in physical activitieso Caliban is also contrasted with Prospero who is the all-powerful master of the island and of the destiny of all

those on the islando Caliban is also contrasted with civilized man showing him to be less evil than Antonio and Stephano and less

materialistic than Stephano and Trinculoo Caliban has suffered at the hands of Prospero and he has learnt to curse by listening to Prosperorsquos abuse He

certainly believes that Prospero has deprived him of his birthrighto Finally the character Caliban is thought to be one of Shakespearersquos masterpieces The complexity of the character

is reflected in the large volume of critical discussion that has grown around it

ECO ndash12 Topic-Forms of market

MonopolyMonopoly is a market structure in which there is a single seller there are no close substitutes for the commodity produced by the firm and there are barriers to entry Example Indian Railways which is operated under government of India Monopoly also implies absence of competitionFeatures of Monopoly Monopoly is characterized by1 Single Seller In monopoly there is only one firm producing the product The whole industry consists of this single firm Thus under monopoly there is no distinction between firm and industry Being the only firm there is significant control of the firm over supply and price Thus under monopoly buyers do not have the option of buying the commodity from any other seller They have to buy the product from the firm or they can go without the commodity This fact gives immense control to the monopolist over the market

2No Close Substitute There are no close substitutes of the product produced by the monopolist firm If there are close substitutes of the product in the market it implies presence of more than one firm and hence no monopoly In order to ensure a total of control over the market by the monopolist firm it is assumed that there are no close substitutes of the product

3 No Entry amp Exit Monopoly can only exist when there is strong barriers before a new firm to enter the market In fact once a monopoly firm starts producing the product no other firm can produce the same One reason for this is the ability of the

monopolist to produce the product at a lower cost than any new firm who thinks to enter the market If a new firm who knows that it cannot produce at a lower cost than the monopolist then that firm will never enter the market for fear of losing out in competition Similarly the monopolist who is operating for a long time may be enjoying reputation among its customers and is in a better position to use the situation in its own benefit A new firm has to take long time to achieve this and so may not be interested to enter the market

4 Price Maker Being the single seller of the product the monopolist has full control over the pricing of the product On the other hand if there is a large number of buyers in the market so no single buyer exercises any significant influence over price determination Thus it is a sellerrsquos market So monopoly firm is a price maker

5 Price Discrimination Having considerable control over the market on account of being single seller with no entry of other firms the monopolist can exercise policy of price discrimination it means that the monopolist can sell different quantities of the same product to a consumer at different price or same quantity to different consumers at different prices by adjudging the standard of living of the consumer

6 Shape of Demand Curve Since a monopolist has full control over the price therefore he can sell more by lowering the price This makes the demand curve downward sloping

Subject Ac-12 290620 Topic- retirement Model sumThe Balance Sheet of Rohit Nisha and Sunil who are partners in a firm sharing profits according to their capitals as on 31st March 2014 was as under

Liabilities Amount Assets Amount (Rs) (` Rs)

Creditors 25000 Machinery 40000Bills Payable 13000 Building 90000General Reserve 22000 Debtors 30000Capital Less Provision for Rohit 60000 Bad debts 1000

29000 Nisha 40000 Stocks 23000 Sunil 40000 140000 Cash at Bank 18000

200000 200000

On the date of Balance Sheet Nisha retired from the firm and following adjustments were made(i) Building is appreciated by 20(ii) Provision for bad debts is increased to 5 on Debtors(iii) Machinery is depreciated by 10(iv) Goodwill of the firm is valued at Rs 56000 and the retiring partnerrsquos share is adjusted

(v) The capital of the new firm is fixed at Rs120000 Prepare Revaluation Account Capital Accounts of the partner and Balance Sheet of the new firm after Nisharsquos retirement Revaluation AccountDr Cr

Particulars Amount Particulars Amount (`Rs) (Rs`)

Provision for Bad debt Ac 500 Building Ac 18000Machinery Ac 4000Profit transferred toCapital Accounts (3 2 2)Rohit 5786Nisha 3857Sunil 3857

13500

18000 18000

Capital Account

Dr Cr

Particulars Rohit Nisha Sunil Particulars Rohit Nisha Sunil (Rs`) (Rs`) (`Rs) (Rs`) (Rs`) (Rs`)

Sunilrsquos Capital ac 9600 mdash 6400 Balance bd 60000 40000 40000Bank - 66143 - General Reserve 9428 6286 6286Balance cd 72000 mdash 48000 Revaluation (Profi 5786 3857 3857 Rohitrsquos Capital Ac mdash 9600 mdash

Sunilrsquos Capital Ac 6400 Bank 6386 - 4257

81600 66143 54400 81600 66143 54400

Balance Sheet as at 31st March 2014

Liabilities Amount Assets Amount (Rs`) (Rs`)

Creditors 25000 Building 108000Bank overdraft 37500 Machinery 36000

Bills Payable 13000 Debtors 30000Capital Less ProvisionRohit 72000 for Bad debts 1500 28500Sunil 48000 120000 Stock 23000

195500 195500

Working Notes (i) (a) Profit sharing ratio is 60000 40000 40000 ie = 3 2 2(b) Gaining Ratio Rohit = 35 ndash 37 = 2135 ndash 1535 = 635Sunil = 25-27 = 1435 ndash 1035 = 435= 635 435= 6 4 = 3 2(c) Nisha Share of Goodwill = Rs 56000 times 27 = Rs16000Share of Goodwill in the gaining ratio by the existing partner ieRohit = Rs16000 times 35 = Rs 9600Sunil = Rs 16000 times 25 = Rs 6400

The journal entry isRohitrsquos Capital Ac Dr 9600Sunilrsquos Capital Ac Dr 6400 To Nisharsquos Capital Ac 16000(Share of Goodwill divided into gaining ratio)

  • 1 Static Friction
  • The frictional force that acts between the surfaces when they are at rest with respect to each other is called Static Friction
    • Static Friction Examples
      • 2 Sliding Friction
        • Examples Of Sliding Friction
          • 3 Rolling Friction
            • Examples Of Rolling Friction
              • Objects and Reasons of the Forest Conservation Act
Page 9:  · Web viewSubject . Topic . Summary . Execution . English 1 . Sounds of animals . Hens –cackle Horses –neigh Lions –roar Owls –hoots Snake –hiss. English 2 . Mother’s

हम उनस सीख लनी चाकिहएবইndashবাংলা সাহিতয পহিরচয়

পাঠndash১০লপndashহিবকেবকানকেFর যকেলকেবলাযলখকndashশশী-ষণ াশগNঅনশীলনীর পরকে4াততর

৯প পহিরবতG ন ককেরা -শরীর = শারীহিরক -ত = য-ৌহিতক সG ার = সG াহির সথায়ী = সথাহিয়তবহিবশবাস = হিবশবাসী া = যকো১০ বাকয রচনা ককেরা -আডডা = পাার বদধরা াতলায় আডডা যয়হিডবাহিজ = রাম া যকেক হিডবাহিজ যখকেয় পকেলাসG ার = বকেলহিল যকেল যর সG ারবকহিন = অঙক -ল করায় রীনা মাকেয়র কাকে বকহিন যখকেলাবহিদধ = বহিদধ াককেল উপায় য়হিনশহিত = হিনশহিত রাকেত রাজবাহিকেত ডাকাত পকেলা-য়ানক = পাাহি রাসতা বষটির পকের -য়ানক হিবপ জনক য়হিবশবাস = মানকেষর পরহিত হিবশবাস ারাকেনা পাপ

Science Ch ndash The Food We Eat

Living things need food to live to grow to stay strong and fit When we need food we feel hungry Food gives us energy to do work It also protects us from diseases and helps us to stay healthy Nutrients in food The food we eat contains many substances that are necessary for our body These substances are called nutrients that help us to grow well and stay healthy

Nutrients give us energy to study work and play

They help our body to grow and repair the damaged parts of our body

They also help our body to fight against diseases and remain healthy

Write T for True or F for False (pg no- 11)

1 Food contains nutrients that help us to grow True

2 Foods rich in carbohydrates are called body-building foods False

MAT

HEM

ATIC

S

Ch 9

Com

mon

Fra

ction

s

A fraction is a number that stands for parts of a whole object or a collection of objects

Each fraction has two numbers One is written above the other separated by a line The one above the line is called numerator and the one below the line is called denominator

Example 5minusminusminusminusrarr Numerator

11minusminusminusminusrarr Denominator

Exercise ndash 362 In following fill in the blanks

(b) 37 ___ is denominator ___ is

numerator

(c) 89 ___ is denominator ___ is

numerator

Solution

(b) 37 7 is denominator 3 is numerator

(c) 89 9 is denominator 8 is numerator

3 Write down the fractions whose numerators and denominators are given below in the bracket The first number stands for numerator and the second number standing for denominator

(25) 25

(311) 311

(416) 416

(712) 712

Class VSubject Topic Summary Execution

Science Ch ndash PlantReproduction

In pollination chapter we have learnt that the flowers change into fruits and the fruits bear seeds Now we are going to learn how this process takes place

FertilizationThe process of fusion of the male reproductive cell (male gamete) and female reproductive cell (female gamete) is known as fertilizationWhen a pollen grain reaches from the anther to a stigma it begins to grow and forms apollentube The Pollen tube then travels down through the style to enter an ovule inside the ovary On reaching the ovule male reproductive cell in the pollen grain unites with the egg cell present in the ovule

Books exercise

A) Tick the correct answer

1Which of the following do ovules change into after fertilization ndashseeds

2Which of the following is not a part of the seed ndash flower

3Which of the following condition is needed for germination of a seed ndash all ofthese

English language

Sentences phrases and

Solved exercisesSay which of the underlined groups of words are phrases and which clauses

clauses8 In her new clothes ndashphrasesAs pretty as a doll ndash phrases

9 looking sad and upset ndash phrasesHe had lost all the tickets for the Test Match ndash clauses

10 During the vacation now only a month away ndashphrases

11 too sweet and too hot ndashphrases

12 At the Olympic Games ndashPhrasesOf Laurel leaves ndash phrases

13 Who played the role of Hamlet ndashclauses

14 However fast ndash phrases

15 When the men fell asleep ndash clausesSocial studies

Indian Government

Lok Sabha (lower house) ndash It has 552 members Of these 530 membersrepresent States 20 members represent the union territories and two members represent the Anglo- Indian community All except the representatives of the Anglo-Indian community are elected by Indian citizens A person above the age of 25 can contest in the elections for Lok Sabha One term of Lok Sabha is for 5 yearsRajya Sabha (upper house) ndash Its members are elected by the MLAs or members of the legislative Assembly There are 250 members in the Rajya Sabha of which 12 are nominated by the President One term of Rajya Sabha is for 6 years Anyone above the age of 30 can be elected as a member of Rajya Sabha

ExecutivePresidentThe President is the head of the country in India He is elected by the MPs and the MLAs for a tenure of 5 years He appoints the Prime Minister and the Council of Ministers Prime MinisterThe party which wins the election forms the government and its leader is elected as the Prime Minister He is the chief advisor to the President The Council of Ministers assists the Prime Minister and is accountable for their roles For example the Education minister is responsible for the education system in our country

1 How many members are there in Lok SabhaAns 552 members

2 What is the term for Lok SabhaAns 5 years

3 How many members are there in the Rajya SabhaAns 250 members

4 Who is the head of our countryAns President

5 Who is the chief advisor to the PresidentAns Prime Minister

Book ndash GK

Ch ndash 1First in space

1 First living being into space in 1957 Ans Laika

Times 2 First person to go into space in 1961 Ans Yuri Gagarin

3 First woman to go into space in 1963 Ans ValentinaTereshkova4 First person ever to walk in space in 1965Ans alexei Leonov5 First person to land on the moon in 1959 Ans Neil Armstrong6 First Indian to go into space in 1984 Ans Rakesh Sharma 7 First Indian woman to go into space in 19978 Ans Kalpana Chawla9 First woman tourist in space in 2006

Ans Anusheh AnsariCOMPUTER

ALGORITHM AND FLOWCHART

Q) DRAW THE SYMBOLS USED IN A FLOWCHART WITH THEIR DESCRIPTIONS(IN EXAM IT CAN COME AS SHORT QUESTIONS ASKING INDIVIDUAL SYMBOLS FUNCTION)ANS)

MAT

HEM

ATIC

S

Ch 6

Com

mon

Fra

ction

s

Multiplication of FractionsA Multiply a fractional number by whole numberTo multiply a fractional number by whole number we multiply the numerator of the fractional number by the whole number and denominator of the fractional number by 1 The first product thus obtained is the numerator and the second product is the denominator of the required product

Exercise ndash 30Multiply

7 2027

times 9

Solution 2027

times 9 = 203 = 6

23

8 611

times11

Solution 611

times11 = 6

15 71

20times16

Solution 71

20times16 =

14120

times16

= 1415

times 4 = 141times 4

5 = 564

5 = 11245

B Multiplication of a fractional number by a fractional number To multiply a fractional number by a fractional number we multiply the numerator of the first fractional number by the numerator of the second fractional number and the denominator of the first fractional number by the denominator of the second fractional number The first product thus obtained is the numerator and the second product is the denominator of the required product

16 2712

times24

Solution 2712

times24 = 3112

times24

= 31times2 = 62

Exercise ndash 31

11 83

times 34

2

Solution 83

times 34 = 2

14 723

times2 25

4

Solution 723

times2 25 =

233

times 125 =

23times 45

= 925 = 18

25

15 1212

times1 13

2

Solution 1212

times1 13 =

252

times 43 =

25times 23

= 503 = 16

23

State the following statements are true or false

17 1912

times 239 = 1

Solution LHS = 1912

times 239

= 392

times 239 = 1 = RHS

[LHS = Left hand side amp RHS = Right hand side]

there4 1912

times 239 = 1 [True]

21 213

times2 13 = 4

19

Solution LHS = 213

times2 13 =

73

times 73

= 7times73times3 =

499 = 5

49

there4 LHS ne RHS

So 213

times2 13 = 4

19 [False]

23 23

times 45 =

2times 5+3 times43times 5

Solution

LHS = 23

times 45 =

2times 43 times5 =

815 again

RHS = 2times 5+3 times4

3times 5 = 10+12

15 = 2215

there4 LHS ne RHS So 23

times 45 =

2times 5+3times43times 5

[False]

25 23 of

13 =

29

Solution

LHS= 23 of

13 =

23 times

13 =

29 = RHS]

there4 23 of

13 =

29 [True]

Practice at HomeExercise ndash 31State the following statements are true or false

24 12 of 4 =

18

Class VISubject Topic Summary Execution

HISTORY AND CIVICS

Chapter 5The Mauryan Empire

DECLINE OF MAURYAN EMPIREDecline of Mauryan empire started after the death of Ashoka at around 232 BCThere are several reasons for break up of the empire1 Weak successor Emperors after Ashoka were

capable of handling vast and mighty Mauryan empire In 185BC the last Mauryan ruler Bri-hadrath was murdered by his Commander-in-Chief Pushyamitra Sunga

2 Provincial Revolts Due to weak central author-ity provincial chiefs of Kalinga and southern provinces revolted against emperor and freed themselves from Mauryan empire

3 Weakness of Economy Prosperity of Mauryan was based on solid economic activities which

ExercisesI Multiple choice questions-1 Chandragupta defeated Seleucus in the year ndashc) 305 BC2 Who killed the last Mauryan ruler Brihadrath b) Pushyamitra3 Which of the following was not a reason for the decline of the Mauryan empirec) Chandraguptarsquos weakness4 Ashoka invaded Kalinga in the year c) 261 BC

II Fill in the blanks1Chandragupta ascended the throne in 324

was taken care by early monarchs Later kings had neither ability nor interest in economic af-fairs That led to failure in tax collection As a result they failed to maintain a large army that were essential to keep empire intact

4 Greek Invasion Greeks freed north-western provinces from weak Mauryan monarchs and reestablished their authority

5 Ashokarsquos Policy some scholar opined that after Kalinga war Ashoka embraced Buddhism re-nounced the policy of war and disbanded the Army But this is partially true as there is no proper evidence of disbanding the army

Based on above points we can conclude that main reason for decline of Mauryan empire is weakness of Ashokarsquos successors Kunal Samprati Dasharath Salisuk all were weak kingsAt last in 185 BCPushyamitra Sunga killed king Brihadrath and established the Sunga dynasty

BC2 Bindusara was the son of Chandragupta and father of Ashoka

3 Pataliputra was administered by City Magistrate committess of 5 members each4 The Greek General Seleucus sent his ambassador Megasthenes to Chandraguptarsquos court5 Ashoka sent his son Prince Mahendra and daughter Sanghamitra to spread his Dhamma6 The Indian Rebublic has adopted the Lion Capital of Saranath Pillar as its national emblem 7 Pushyamitra killed the last Mauryan ruler Brihadrath and founded the Sunga dynasty

III Name the following

1The author of Arthashastra-Kautilya2 The ruler who founded the Mauryan dynasty-Chandragupta3 The author of Indika-Megasthenes 4 The officers who were appointed by Ashoka to spread Dhamma-Dhamma Mahamatras5 The general of Alexander whom Chandragupta defeated-Seleucus

V Match the columns1 Kautilya (c)2 Megasthenes (d)3 Pushyamitra (e)4 Brihadrath (b)5 Bindusara (a)

BENGALI(2ND

LANGUAGE)

পশপাহিখর -াষাসহিবনয় রায়কেচৌধরী

যলখক পহিরহিচহিত- পরখযাত সাহিহিতযক উকেপনদরহিককেশার রায়কেচৌধরীর পতর সহিবনয় রায়কেচৌধরী lsquoসকেFশrsquo পহিতরকার সকেb হিতহিন কত হিকেলন তার উকেdখকোয বই lsquoসহিবনয় রায়কেচৌধরীর রচনা সংগরrsquo

পরম হিকেনর পাঠ- lsquoপশপাহিখর হিক -াষাhelliphellip helliphelliphelliphellipপরসপরকেক জানাবার উপায়ও পশপাহিখরা যবশ জাকেনrsquoপরকেমই আমারা জাহিন -াষা হিক -াষা ল আমাকের মকেনর -াব পরকাশ করার জনয আমরা নানান ধরকেনর -হিb বা হিবকেশষ ধরকেনর আওয়াজ মকেখর মাধযকেম কহির অনযকেক যবাঝাকেনার জনয তাকেল এবার আমরা জাহিন পশপাহিখর -াষা হিক পশ পাহিখরা হিক কা বকেল যা পশপাহিখকেরও -াষা আকে তারা তাকের হিনজসব -াষায় কা বকেল মকেনর -াব পরকাশ ককের পশ পাহিখরা মানকেষর হিক হিক -াষা যবাকেঝ হিকনত তারা বলকেত পাকেরনা পরসপরকেক বহিঝকেয় যবার উপায় তারা জাকেননা তকেব তারা হিবকেশষ ককেয়কটি শকেবদর মাধযকেম তাকের মকেনর -াব বহিঝকেয় যয় হিক বহিদধ মান জীব ndashককর হিবাল বন মানষ যঘাা পর-হিত এরা মানকেষর যওয়া নাম শনকেল কান খাা ককের ndash নাম ধকের ডাককেল কাকে আকেস যমন - মরহিরা lsquoহিত ndashহিতrsquo ডাক শকেন আকেস াল lsquoঅ ndashর -র ডাক শকেন কাকে আকেস াহিত মাহকেতর কা শকেন চকেল ককররা মাহিলকেকর হকম পালন ককের সবসময় তাইকেতা ককরকেক পর- -কত পরানী বলা য় ককর আর হিবাল একের আওয়াজ তহিম লকষয করকেল বঝকেব ককররা যরকে যকেল lsquoযঘউ যঘউrsquo করকেত াকেক আবার কাকেল lsquoযকউ যকউrsquo ককের হিবাল সাধারণ lsquoমযাওrsquo বা lsquoহিমউrsquo ককের রা কেল lsquoওয়াওrsquo আওয়াকেজর মাধযকেম মকেনর -াব পরকাশ ককের একেতা যল পশকের কা পাহিখরাও -য় রা পরকাশ করার জনয হিবকেশষ ধরকেনর শবদ ককের হিবপকের সময় পশ পাহিখরা সবার আকে পরসপরকেক জানাবার উপায় তারা জাকেন বহকাল

১) পশপাহিখর -াষা কেলপর যলখক সমপকেকG হিক জাকেনা

উঃ- পরখযাত সাহিহিতযক উকেপনদরহিককেশার রায়কেচৌধরীর পতর সহিবনয় রায়কেচৌধরী lsquoসকেFশrsquo পহিতরকার সকেb হিতহিন কত হিকেলন তার উকেdখকোয বই lsquoসহিবনয় রায়কেচৌধরীর রচনা সংগরrsquo হিতহিন ারকেমাহিনয়াম এসরাজ পর-হিত বাযনতর বাজাকেত পারকেতন ানও জানকেতন হিতহিন যাকেIাকের জনয মজাার লপ কহিবতা হিলখকেতন

২) পশপাহিখ কেলপর মল-াব হিকউঃ- পশপাহিখকেরও -াষা আকে তারা তাকের হিনজসব -াষায় কা বকেল মকেনর -াব পরকাশ ককের পশ পাহিখরা মানকেষর হিক হিক -াষা যবাকেঝ হিকনত তারা বলকেত পাকেরনা পরসপরকেক বহিঝকেয় যবার উপায় তারা জাকেননা তকেব তারা হিবকেশষ ককেয়কটি শকেবদর মাধযকেম তাকের মকেনর -াব বহিঝকেয় যয় হিরউকেবন কযাসটং সাকেব হিতহিন চহিdশ বর বনযজনত যর সকেb যকেককেন হিতহিন বকেলকেন আমরা হি তাকের -াষা তাকের আব কায়া যমকেন চহিল তাকেল আর -কেয়র যকান কারণ াকেকনা আমরা একI -াকেলাকেবকেস যচষটা করকেল পশপাহিখকের সকেb -াব পাতাকেত পাহির

ধকের মানষ এই পশ পাহিখর -াষা হিনকেয় নানা রককেমর পরীকষা ককের আসকে এইরকম একজন হিরউকেবন কযাসটাং সাকেকেবর কা আমরা জানকেবাhelliphellip

Hindi 2nd

langमतर किनमनलिलखिखतपरशनोउRरदीजि0ए

) बढ वयलि` बचच ो कया हआ था ख) डॉकटर साहबन पाटc किस उददशय स रखी थी ग) ाल साप ो हाथ म लर लाश न कया किया घ) डॉकटर चडढा न बढ पतरो दखन स कयो मना र दिदया था ङ) भगत न लाश ो दखर कया हा

उRर ndash) उस बहत बखार थी और 4 दिदनो स आख भी नही खोला थाख) उन बट ी सालकिगरह थीग) ाल सापो हाथ म लर लाश न उसी गदन 0ोर स दबार पडी थीघ) डॉकटर चडढा न बढ वयलि` पतरो दखन स मना र दिदया कयोकि उनह गोलफ खलन 0ाना थाङ) लाश ो दखर हा कि नारायण चाहग तो आध घट म भया उठ 0ाएग

English literature

In the bazaars of Hyderabad- Sarojini Naidu

Through the poem In The Bazaars of Hyderabad Sarojini wanted to convey the message that India is rich in tradition and they donrsquot need the foreign products So she goes on to give a picture of a bazaar where traditional Indian products are rulingThe poem is in the form of questions and answers The poet asks the questions and the merchants answer them Through this technique she make the picture of the bazaar visible to us

Read the poem

PHYSICS FORCE Types of FrictionThere are three types of friction static sliding rolling Static sliding and rolling friction occur between solid surfaces

1 Static Friction The frictional force that acts between the surfaces when they are at rest with respect to each other is called Static FrictionStatic Friction Examples

Skiing against the snow Creating heat by rubbing both the hands

together Table lamp resting on the table

2 Sliding Friction The resistance that is created between any two objects when they are sliding against each other is called Sliding FrictionExamples Of Sliding Friction

Sliding of the block across the floor Two cards sliding against each other in a

deck

3 Rolling Friction The force which resists the motion of a ball or wheel is called Rolling Friction Is the weakest types of frictionExamples Of Rolling Friction

Rolling of the log on the ground Wheels of the moving vehicles

6What effect can a force produce on a body which is not allowed to move Ans - When a force is applied on a body which is not free to move it gets deformed i e the shape or size of the body changes7Give one example each to indicate that the application of a force

1 produces motion2 stops motion3 slows down motion4 changes the direction of motion5 deforms a body

Ans- 1 A car originally at rest when pushed

begins to move2 A moving bicycle is stopped by

applying the brakes3 The speed of a moving vehicle is

slowed down by applying the brakes4 A player kicks a moving football to

change its direction of motion5 On stretching a rubber string its

length increases

8State the effect produced by a force in the following cases (a) The sling of a rubber catapult is stretched(b) A man pushes a heavy cart(c) A player uses his stick to deflect the ball (d) A cyclist applies brakes(e) A spring is compressedAns- (a) The shape and size of catapult changes ie its length increases(b) The heavy cart begins to move(c) The direction of the ball changes(d) The speed of the moving cycle is slowed down(e) There is change in size and shape of spring

COMPUTER MS EXCEL 2013 -INTRODUCTION

UNDERSTANDING EXCEL STRUCTUREA SPREADSHEET IS A FILE THAT EXISTS OF CELLS IN ROWS AND COLUMNS AND CAN HELP ARRANGE CALCULATE AND SORT DATA DATA IN A SPREADSHEET CAN BE NUMERIC VALUES AS WELL AS TEXT

FORMULAS REFERENCES AND FUNCTIONS

WORKSHEETA WORKSHEET IS ALSO KNOWN AS SPREADSHEETIT IS A COLLECTION OF CELLS ON A SINGLE SHEET WHERE YOU KEEP AND CHANGE DATA

WORKBOOKWORKBOOK IS PMS EXCEL FILE IN WHICH THE DATA CAN BE STORED EACH WORKBOOK CAN CONTAIN MANY WORKSHEETS

ROWS AND COLUMNSIN MS EXCEL A ROW IS A GROUP OF CELLS THAT RUN FROM LEFT TO RIGHT OF A PAGEA COLUMN IS A GROUPING OF CELLS THAT RUN FROM THE TOP TO THE BOTTOM OF A PAGE

CELLTHE INTERSECTION POINT BETWEEN A ROW AND THE COLUMN IS CALLED A CELL WHICH IS THE BASIC STORAGE UNIT FOR DATA IN A SPREADSHEET EACH CELL HAS SPECIFIC ADDRESS WHICH IS THE COMBINATION OF THE COLUMN NAME FOLLOWED BY THE ROW NUMBER

CHEMISTRY Chapter ndash Common Laboratory Apparatus and equipments

Objective type questionFill in the blanks (a) Experiment and observation are the two important basics of chemistry(b) A porcelain dish is used for evaporation(c) A test tube holder is used to hold the test tube while-it is heated(d) Mortar and pestle is used for grinding and crushing solid substances into a powder(e) Glass apparatus is made of Pyrex or borosil glass

Class VIISubject Topic Summary Execution

Hindi 2ndlang

ए था राम( डॉ शरी परसाद)

सगकित ा परभाव मानव 0ीवन पर अवशय पडता ह

हमशा मनषय ो अचछो ी सगकित म रहना चाकिहए

शरषठ परो सग स मनषय चरिरतर ा शीघर ही उदय और किवास हो 0ाता

ह इसलिलए वयलि` ो सदा शरषठ परो ा ही सग रना चाकिहए

इसान अगर चाह वह सवय ो बदल भी सता ह

यह हानी राम ए बचच ी हवह गणिणत ी परीकषा म नल रत हए पडा 0ाता ह और उस अधयाप पडत ह और पछत ह यह कया र रह

हो तभी राम न उनी बइजजती ी

शबदाथब ndashहावा भलावाायवाहीndash ाम किनयम व ानन

ो दिदखानापरिरलिचतndash 0ाना पहचानाघटनाndashघबराहट

उलटा चोर ोतवाल ो डाटndashकिववndash भल बर ा जञानतवयndash म 0ो रना चाकिहएसगकितndash बरी सगत

किबलख नाndashरोना किनशचय रनाndash तय रना

फलndashपरिरणामकिनषालिसतndash बाहर किया हआपशचातापndashदख सपननndashधनी

ldquo हा आपी किहममत स हई नल रत पडन ीrdquo ऐसी बात ही किफर

किपता0ी न भी उस डाटा वह ाफी पशचाताप रन लगा बोला गलत दोसतो

ी सगकित म आ0 कितना अनथ र दिदया किफर उसन अधयाप स माफी मागन ी सची और किफर भी ऐसा

नही रगा यह परण भी लिलया

सोचndashकिहच एात-अला

বইndashবাংলা সাহিতয পহিরচয়

পাঠndash১৬লপndashস-য ও অস-যযলখকndashঈশবরচনদর হিবযাসারঅনশীলনীর পরকে4াততর

৬ অGকেলকেখা -ময়া = পশ হিশকার সহিtহিত = হিনকIবতu সbভরষট = লI হিনরীকষণ =

-াকেলা-াকেব যখাকতাঞজহিলপকেI = যজাাকেত৭ হিবপরীতশবদ -ঈষৎ times পরচর উৎকষট times হিনকষট তাশ times উৎফd তবহিদধ times

বহিদধীNপাহিপষঠ times পণযবান৮ পপহিরবতG ন ককেরা -পশ = পাশহিবক যকাপ = যকাহিপতহিসথর = হিসথরতাএকানত = ঐকাহিনতক পর-াত times পর-াতী

CHEMISTRY

Chapter ndashPhysical and Chemical Changes

Chemical ChangeA chemical change involves a change in chemical composition

Characteristics of Chemical changes 1 They are permanent changes2 They are irreversible changes 3 New substance formed4 A Chemical change involves a

change in its chemical properties

Pg-25Question 8What do you observe when1 water is boiled2 a piece of paper is burnt3 some ice cubes are kept in a glass tumbler4 solid ammonium chloride is heated5 an iron nail is kept in tap water for few days6 a spoon of sugar is heated in a pan7 lighted match stick is brought near the mouth of the test tube containing hydrogen gas8 quick lime is dissolved in water9 little amount of curd is added to a bowl containing warm milk and kept for five hours

10 Water is boiledOn boiling water changes into steam (gas) physical change

11 A piece of paper is burnton burning piece of paper produces carbon dioxide and ash is left behind Is a chemical change

12 some ice cubes are kept in a glass tumblerIce cubes (solid) turn into water

(liquid) only state changes (physical change)

13 Solid ammonium chloride is heatedSolid ammonium chloride on heating changes into vapors (change of state) is physical change

14 An iron nail is kept in tap water for few dayswe observe reddish brown coating on the nail called rust (entirely new substance) is chemical change

15 A spoon of sugar is heated in a panWhen a spoon of sugar is heated in a pan black (charred sugar) (carbon) is seen Is a chemical change

16 Lighted match stick is brought near the mouth of the test tube containing hydrogen gasWe observe that hydrogen bums at the mouth of test tube with blue flame and pop sound is heard It is chemical change

17 Quick lime is dissolved in waterThe following two observations will be observed (i) A hissing sound is observed(ii) The mixture starts boiling and lime water is obtained

18 Little amount of curd is added to a bowl containing warm milk and kept for five hoursWhen a little amount curd is added to a bowl containing warm milk and kept for five hours a permanent change occurredThe milk will change to curd On boiling water changes into steam (gas) physical change

GEOGRAPHY

ATMOSPHERE IMPACT OF GLOBAL WARMING The destructive impart of global warming is observed in various spheres of life and the environment Some of the points are outlined below1 High temperatures lead to high

evaporation rate and drying up of the soil and surface water This affects crop production The occurrence of droughts is aggravating the problem even further

2 The heat waves in summer months

Q1 Write some impact of global warmingA1 The impacts of global warming are as follows1 High temperatures lead to high

evaporate ion rate and drying up of the soil and surface water This affects crop production The occurrence of droughts is aggravating the problem even further

2 The heat waves in summer months lead to a greater number

lead to a greater number of deaths due to heat strokes

3 Forest fires become more frequent4 Tropical cyclones and hurricanes

become common5 Melting of glaciers takes place6 Polar ice caps are becoming thinner

and melting at an alarming rate due to global warming The loss of sea ice

7 Due to increase in sea surface temperature sea levels rise in coastal areas and cause submergence of several islands

WAYS TO REDUCE GLOBAL WARMINGFollowing steps can be taken We need to decrease emission of

green house gases by reducing the burning of fossil fuel such as coal and petroleum

By planting more trees to increase forest cover

The government should also distributes free saplings and organize afforestation programmes to spread awareness regarding the beneficial effects of trees

We should switch to eco-friendly cars and gadgets

Incandescent light bulbs should be replaced by CFL bulbs

We can save electricity and reduce global warming by turning off electrical gadgets such as lights fans air-conditioners television and computer when we do not to use them

Efforts should be made to hasten the development of green cities oreco cities These cities are urban areas around the world striving to lessen the environment a impacts of urbanization

By following the 3Rs-Reduce Recycle and Reuse strategy we can use natural resources for our growth as well as save them for the need of the future generations This is called sustainable development

of deaths due to heat strokes3 Forest fires become more

frequent4 Tropical cyclones and hurricanes

become common5 Melting of glaciers takes place

etc

Q2 How to reduce global warmingA2 Following steps can be taken to reduce global warmingaWe need to decrease emission of

green house gases by reducing the burning of fossil fuel such as coal and petroleum

bBy planting more trees to increase forest cover

c The government should also distributes free saplings and organize afforestation programmes to spread awareness regarding the beneficial effects of trees

dWe should witch to eco-friendly cars and gadgets

eIncandescent light bulbs should be replaced by CFL bulbs

f We can save electricity and reduce global warming by turning off electrical gadgets such as lights fans air-conditioners television and computer when we do not to use them

Q3 What do you mean by 3Rrsquos of resource planningA3 The 3Rs are

1 Reduce 2 Recycle and3 Reuse

Q4 What is Sustainable developmentA4 By following the 3Rs-Reluce Recycle and Reuse strategy we can use natural resources for our growth as well as save them for the need of the future generations This is called sustainable development

English Language

Prepositions A preposition is a word placed before a noun or a pronoun It helps to show how the person or thing denoted by the noun is related to something else in the sentence

Kinds of Prepositions

Simple Prepositions- simple preposition are one word Prepositions such as at by for in of off for from on out through till to up with before amidst towards beyond between over etc

Compound Prepositions ndash There are some words that are always used with fixed Prepositions to convey specific meaning

Example I was unable to meet you dueto a previous engagement ( On account of)Always maintain the queue instead of crowding at the counter ( In place of)

Participial PrepositionsmdashParticiple Prepositions are present or past participles of various verbs which together with a noun phrase or a clause function as prepositions Examples- barring concerning considering notwithstanding pending regarding respecting etc

Exercise A

1 Gauravs fever has come down since Friday He has been absent for a week now

2 The child sat between his father and mother among the parents of all his classmates

3 There are mosquitoes in the room They flew into the room when the door was open

4 My father was inside the drawing room when I was playing outside my house

5 You may sit beside me I will give you a drawing book and pencils besides a storybook

6 We went to the market in the morning and walked towards the riverfront in the evening

7 The child walked along the pavement and across the street safely

8 This table top is made of glass My breakfast fell off it in the morning

9 The pan is on the gas stove There are vegetables in it

10 We will wait for you at the bus top There are a lot of people in the hall

Subject ndash Biology Topic ndash Chapter - 3 Photosynthesis and respiration in plants Summary Execution

All living organism (Plants and animals) need food for energy and growth Green plants (autotrophy) prepare food for all living organisms Today we will discuss about the process photosynthesis And adaptations in a leaf to carry out photosynthesis

Q1What do you mean by photosynthesis and write its word equation The process by which green plants make food (glucose) from carbon dioxide and water

in the presence of sunlight and chlorophyll is called photosynthesis

Carbon dioxide + Water ( Sun light from Sun ) Glucose + Oxygen ( chlorophyll in green leaves )

Q2 What are the adaptations in a leaf to carry out photosynthesisi) Leaves are broad wide and flat for absorbing more light energyii) Presence of chlorophyll in chloroplasts to trap sunlightiii) Presence of stomata which allow carbon dioxide to enter the cell and oxygen to go

out iv) Network of veins ensures continuous supply of water and minerals to the leafv) Thin waxy cuticle protects the leaf without blocking the lightQ3 Draw and label structure of chloroplast

Class VIIISubject Topic Summary Execution

PHYSICS ENERGY Production of Hydro electricity

A hydroelectric dam converts the potential energy stored in a water reservoir behind a dam to mechanical energymdashmechanical energy is also known as kinetic energy As the water flows down through the dam its kinetic energy is used to turn a turbine

The generator converts the turbinersquos mechanical energy into electricity

This electric energy then goes through various transmission processes before it reaches you

Question 2

Fill in the blanks

(a) Work is said to be done by a forte only when the body moves

(b) Work done = Force x distance moved in direction of force

(c) The energy of a body is its capacity to do work

(d) The SI unit of energy is joule

(e) The potential energy is due to its state rest of position and kinetic energy of the body is due to its state of motion

(f) Gravitational potential energy U = mass times force of gravity on unit mass times height

(g) Kinetic energy = frac12 times mass times (speed)2

(h) Power P = work donetime taken

(i) The S I unit of power is watt

(j) IHP = 746 W

BIOLOGY Chapter -5 The endocrine system and adolescence

Today we will discuss about thelocation and functions of secreted hormones of adrenal and Pancreas

Q5 Write location hormone secreted main functions and deficiency diseases of pancreas and adrenal glands

Endocrine Glands

Location Hormones secreted

Functions and Deficiency Diseases

1Adrenal gland

2 Pancreas Gland

On the top of each kidney

In between stomach and small intestine

i)Adrenaline from adrenal medulla

ii)Cortisone from adrenal cortex

i) Insulin

ii) Glucagon

It helps a person deal with any kind of emergency situation or emotional stressIt increases the heart beat rate of respiration and blood pressure

a) It regulates carbohydrates protein and fat metabolism

b) It regulates the salt and water balance in the body

a) It changes excess glucose into glycogen

b) It stimulates the cells to burn extra glucose to provide heat amp energy

Less secretion causes diabetes mellitus

Excessive secretions causeinsulin shock

a) It stimulates the breakdown of glycogen into glucose

b) It increases the level of glucose in blood

History Traders to rulers The Battle of Buxar was fought on 22 October 1764 between the forces under the command of the British East India Company led by Hector Munro and the combined armies of Mir Qasim the Nawab of Bengal till 1763 Mir Jafar was made the Nawab of Bengal for a second time in 1763 by the Company just after the battle After being defeated in 4 battles in katwa and Udaynala the Nawab of Awadh Siraj id Daula and the Mughal emperor Shah Alam II accompanied by Raja Balwant Singh of Kashi made an alliance with Mir Qasim The battle was fought at Buxar a small fortified

Answer the following questions- Short note-Battle of BuxarHomework-learn

town within the territory of Bihar located on the banks of the Ganga river about 130 kilometres (81 mi) west of Patna it was a decisive victory for the British East India Company The war was brought to an end by the Treaty of Allahabad in 1765

EnglishLiterature

The west wind-John Mansfield

In the poem The West Wind by John Masefield the poet starts by describingwith very poetic imagery of birds how the west wind is different from other winds its a warm wind full of birds cries There is a touch of melancholy perhaps home-sickness as he describes how it brings tears too and memories from an old land He goes on to describe the restful pastoral beauty of the land where even the dead can lie in the green He then brings in voicesperhaps of family and friends calling him home as he is missing Aprils beautyThe voices then tempt him some more with idyllic images from home (white blossom young green cornrunning rabbitswarm sun) The voices seem to presume that the poets heart is sorrowful bruised and soreThe end of the poem sees the poet appear to make a decision he will go home as he has decided that is where he truly belongs

Write the synopsis of the following words

1 Daffodils- a tall yellow flower that grows in the spring

2 Orchards- a piece of land on which fruit trees are grown

3 Blossom- a flower or a mass of flowers especially on a fruit tree in spring

4 Thrushes- a bird5 Larks- a small brown bird that

makes a pleasant sound6 Bruised- an injury7 Aching- pain 8 Tread- to put your foot down

while you are walking9 Balm-10 May-11 Fluting-

(Write from the book in your copy)

MAT

HEM

ATIC

S

Ch 1

1Al

gebr

ic E

xpre

ssio

n

1 Constant A symbol which has fixed value is called a constant[eg 8 23 -15 radic3 etc]

2 VariableA symbol which does not have any fixed value but may be assigned value (values) according to the requirement is called variable or literal[eg x y p q etc]

3 TermsA term is a number (constant) a variable a combination (product or quotient) of numbers and variables[eg 7 x 5x etc]

4 Algebric expressionA single term or acombination of two or more terms connected by plus (+) or minus (-) sign forms an algebraic expression[eg 5-y 3x2-5x xy-6z+4 etc]

5 PolynomialAn algebraic expression which contains more than one term is called a polynomial (multinomial)[eg x2-5x 5y+xy+x2y etc]

6 Degree of polynomial(a) When the polynomial contains only one variable the highest power of the variable is the degree of the polynomialeg the degree of the polynomial of 4x-7x5+8 is 5(b) When the polynomial contains two or more variablesStep (i) Find the powers of the variables in each term (ii) The highest sum of the powers is taken to be the degree of the polynomialeg the degree of the polynomial 5x2y-4x3y5+6 is = 3+5 = 8Remember An algebraic expression is a polynomial if degree of each term used in it is a non-negative integer

Exercise ndash 11(A)

1 Separate the constants and variables from the following

-7 7+x 7x+yz radic5 radic xy 3 yz

8 45y -3x

Solution Constant Variables-7 radic5 7+x 7x+yz radic xy

3 yz8

45y -3x

2 Write the number of terms in each of the following polynomials(i) 5x2+3timesax (ii) axdivide4-7 (iii) ax-by+ytimesz (iv) 23+atimesbdivide2

Solution Polynomials Number of terms(i) 5x2+3timesax 2(ii) axdivide4-7 2(iii) ax-by+ytimesz 3(iv) 23+atimesbdivide2 2

4 Write the degree of the each polynomials(i) xy+7z (ii) x2-6x3+8 (iii) y-6y2+5y8 (iv) xyz-3 (vi) x5y7-8x3y8+10x4y4z4

Solution Polynomials Degree(i) xy+7z 2(ii) x2-6x3+8 3(iii) y-6y2+5y8 8(iv) xyz-3 3(vi)x5y7-8x3y8+10x4y4z4 12

5Write the coefficient of(i) ab in 7abx (iv) 8 in a2-8ax+a (v) 4xy in x2-4xy+y2

SolutionCoefficient

(i) ab in 7abx 7x(iv) 8 in a2-8ax+a -ax(v) 4xy in x2-4xy+y2 -1

7 CoefficientAny factor of an algebraic quantity is called the coefficient of the remaining quantityeg in the algebraic term 7xyz 7 is coefficient of xyz 7x is coefficient of yz and so on

8 Like term The terms having the same literal coefficient are called like terms and those having different literal coefficients are called unlike terms

eg (i) 5xyz 8xyz -6xyz and 23xyz are like

terms(ii) 7xy2 8x2yz and -15xyz2 are unlike terms

6 in 57xy2z3 write the coefficient of

(i) 5 (vii) 5xy2 (viii) 17yz (xi) 5xyz

Solution Coefficient

(i) 5 17

xy2z3

(vii) 5xy2 17z3

(viii) 17yz

5xyzsup2

(xi) 5xyz 17yz2

7 In polynomial given below separate the like terms(ii) y2z3 xy2z3 -58x2yz -4y2z3 -8xz3y2 3x2yz and 2z3y2

Solution y2z3 -4y2z3 2z3y2 are like terms

xy2z3 -8xz3y2 are like terms

-58x2yz 3x2yz are like terms

Class IXSubject Topic Summary Execution

Bengali (2nd language)

বাগzwnjধারাzwnj বা ধারা-বা ধারা ল হিবকেশষ পরকার বাক -হিb -াকেবর এক হিবকেশষ পরকাশরীহিত াকেক কতগকেলা কার সমষটির মকেধয এগহিলকেক বা ধারা বকেল আবার কতগকেলা শকেবদর বাধাধরা যকান রীহিত যনই য-াকেব চকেল আসকে যসই -াকেবই চকেল আসকে তখন যসই শবদগহিল খন একক -াকেব অG পরকাশ ককের তখন একের বা ধারা বকেল বা ধারার পরকেয়া -াষাকেক আরও সFর ককের যতাকেল

অকাল পকক(অপহিরনত বয়কেস পাকাহিম)-মাতর শ বর বয়কেস যমকেয়টির া মকেখর কা তাকেত অকালপককতা ধরা পকে

অককা পাওয়া( মারা াওয়া) ndash পকেকIমারটি পকেকIমারকেত হিকেয় বাসাতরীকের াকেত মার যখকেত যখকেত অককা যপল

অহি| পরীকষা ( কঠিন ও পরকত পরীকষা)- যকেলটির আজ ডাকতাহির যরজালট যবকেরাকেব এIাই তার জীবকেনর ব অহি| পরীকষা

অষটরমভা (ফাহিক) ndash রীতা মকেখই বকো বকো কা বকেল আর কাকেজর যবলায় অষটরমভা

অকমGার ধাী (অপাG) ndash সমনকেক হিনকেয় যকান ান কেব না ও একেকবাকেরই অকমGার ধাী

অকেনধর ষটি (অসাকেয়র সায়)- আহিশ বকেরর বকোর নাহিত ল অকেনধর ষটি তাকেক াা বকোর একম চকেল না

আকেককল গড়ম (তবহিদধ)- ার তহিম উপকার করকেল যসই যতামার হিবরকেদধ সাকষয হিকেয়কে শকেনই আমার আকেককল গড়ম

আষাকে লপ( অবাসতব লপ) ndashIাকা এখন যকেব না এIা বলকেলই ত এমন আষাকে লপ ফাার যকান রকার হিল না

Hindi- महायजञ ा इस हानी म लख न या बतान ा परयास किया ह कि किसी भी अचछ

2nd language

परसार(यशपाल ाय या पणय न ा फल अवशय मिमलता ह ोई भी परोपार अथवा पणय लिलए किया गया ाय बार नही 0ाता वह ए परार ा यजञ हए धनी सठ थ धम परायण और किवनमर सठ न आन ी यजञ किए थ और दान म न 0ान कितना धन दिदन दखिखयो म बात दिदया थादिदन पलट और सठ यहा गरीबी आ गई उन दिदनो यजञ बचन ी परथा थी सठ भी अपनी 0गह बचन लिलए डलपर ए सट यहा चलन ो तयार हए सठानी रासत लिलए रोटी पड म बाधर सठ ो द दी रासत म ए भख R ो दखर सठ न चारो रोटी उसो खिखला दी खर वह सठ यहा डलपर पहच तो उनी सठानी न उस महायजञ बचन ो हा यदिद बचन आए सठ न R ो रोटी खिखलान ो महायजञ नही समझा और वापस लौट आया घर आर शाम ो उसी घर म उस ए बडा ख0ाना मिमला 0ो उस दवारा किए गएrsquo महायजञrsquo ा परसार था

English language

Letter formal The heading the name and address of the person you are writing to must be included beneath your own address In formal letters ldquoblock stylerdquo of address is preferred

Subject complain in brief

Salutation If the person you are writing to is known to you you may begin ldquoDear MrrdquoOr ldquoDear Mrsrdquo In all other instances you should begin ldquoDear Sirrdquo or ldquoDear Madamrdquo Or ldquoSirsrdquo

The body A formal or business letter has four partsReference The letter should begin by referring to a letter you have received an advertisement or the reason that has prompted you to writeInformation In the second paragraph it is necessary to supply more detailed information that is related to the referencePurpose Here you must give the reason why you are writing the letter This must be stated clearly and ensure that it is relevant to the question that has been setConclusion round off the letter with some polite remarkThe subscription when a letter has begun with dear sir sirs Madam you should end with Yours faithfully or yours truly When however you address a person by name you must conclude with the words ldquoYours sincerelyrdquo

1 A park in your locality is slowly being used as a rubbish dump Write a letter to the Mayor of your city pointing out the nuisance and danger of this Request that action be taken to stop this immediately

Or2 You being a boarder ordered a set of lab manuals from a famous book shop in the town They sent you a wrong set of books Write a letter to the manager of the book shop

Chemistry Chapter-1 1)CHEMICAL FORMULA- Q What is the Significance of

L-2The Language of Chemistrybull Chemical Formula

Itrsquos a symbolic representation of a chemical substance eg ndash The formula of Sulphuric acid is H2SO4

2) Steps of writing Chemical Formula of a given substance-

1 Write the symbols of the constituent atoms or radicals side by side Keep the basic radical on LHS and acid radical on the RHS ( Na+Cl- )2 In case of a radical having more than one atom( compound radical) enclose the radical in a bracket eg (SO4-)3 Write the valencies of each radical on its right hand top4 If the valencies of the two radicals are divisible by a common factor then divide the valencies by the common factor5 Invert (criss-cross) the valency number ie write the valency of one atom below the second atom and vice versa 6 On interchanging if valency number is lsquoone the figure lsquoonersquo is never writtenFor Example- Compound -Calcium Nitrate1 Writing the symbols- Ca(NO3)2 Writing the valencies on their right hand top- Ca2(NO3)1

3 Valency numeral in simple ratio- Ca2(NO3)1

4 Criss-cross- Ca 2NO3 1

5 Writing the formula of the compound- Ca(NO3)2

Chemical formula

A The formula of a substance conveys the following information regarding a substance 1 The name of the substance (qualitative)2 The elements constituting the substance (qualitative)3 The number of various atoms present in a molecule of the substance (quantitative)4 Molecular weight of the substance and the relative weights of different elements present in it (qualitative)

Q What are the limitations of Chemical Formula

A The chemical formula suffers from the following limitations-I It fails to convey whether the elements in a molecule are present in the form of atoms or ionsFor example the formula KBr fails to tell us whether Potassium and Bromine are present in the form of ions II It does not tell anything about the binding force that holds atom in a molecule togetherIII It does not tell us about the arrangement of various atoms with respect to one another within the molecule

Q Examples of Some Chemicals with their Formula Chemical name and Common Name-

A Given in the class notesCommercial Studies

Joint Stock Company

Let us discuss about the demerits of Joint Stock CompanyDespite so many advantages it has got many disadvantages which are as follows

Difficulty in FormationDelay in Decision makingExcessive Government ControlLack of Secrecy

Company can be classified into several categories based on incorporation

QuestionExplain the demerits of Joint Stock CompanyAnswer) 1 Difficulty in Formation The legal requirements and formalities required to be completed are so many The cost involved is quite heavy It has to approach large number of people for its capital It cannot start its business unless certificate of incorporation has been obtained This is granted after a long time when all the formalities are completed

Chartered CompanyStatutory CompanyRegistered Company

Delay in Decision making In this form of organization decisions are not made by single individual All important decisions are taken by the Board of Directors Decision-making process is time-consuming So many opportunities may be costly because of delay in decision-making Promptness of decisions which is a common feature of sole trader ship and partnership is not found in a company

Excessive Government ControlA company and the management have to function well within the law and the provisions of Companies Act are quite elaborate and complex At every step it is necessary to comply with its provisions lest the company and the management should be penalized The penalties are quite heavy and in several cases officers in default can be punished with imprisonment This hampers the proper functioning of the company

Lack of Secrecy The management of companies remains in the hands of many persons Every important thing is discussed in the meetings of Board of Directors Hence secrets of the business cannot be maintained In case of sole proprietorship and partnership forms of organisation such secrecy is possible because a few persons are involved in the management

2 Define the following

Chartered Company- The crown in exercise of the royal prerogative has power to create a corporation by the grant of a charter to persons assenting to be incorporated Such companies or corporations are known as chartered companies Examples of this type of companies are Bank of England (1694) East India Company (1600) The powers and the nature of business of a chartered company are defined by the charter which incorporates it After the country attained independence these types of companies do not exist

in IndiaStatutory Company- A company may be incorporated by means of a special Act of the Parliament or any state legislature Such companies are called statutory companies Instances of statutory companies in India are Reserve Bank of India the Life Insurance Corporation of India the Food Corporation of India etc The provisions of the Companies Act 1956 apply to statutory companies except where the said provisions are inconsistent with the provisions of the Act creating them Statutory companies are mostly invested with compulsory powersRegistered companiesCompanies registered under the Companies Act 1956 or earlier Companies Acts are called registered companies Such companies come into existence when they are registered under the Companies Act and a certificate of incorporation is granted to them by the Registrar

Economics

Chapter-4Basic problems of Economy

Today let us discuss with the topic Production Possibility curve

QuestionExplain the concept of Production Possibility Curve with the help of diagram

Answer) Production Possibility curve is a locus of all possible combinations of two commodities which can be produced in a country with its given resources and technology

The above diagram shows that with the given resources and technology the economy can produce maximum either 5 thousand meters of cloth or 15 thousand quintals of wheat or any other combination of the two goods like B( 1 thousand meters of cloth and 14 thousand quintals of wheat C ( 2 thousands meters of cloth and 12 thousand quintals of wheat) etcProduction Possibility curve is also called production possibility boundary or frontier as it sets the maximum limit of what it is possible to produce with given resources

Geography

Rotationand Revolution

SUNrsquoS POSITION AND SEASONAL CHANGES EQUINOXES ndash SPRING AND AUTUMN

Q1 What is Spring EquinoxA1 On 21st March sunrays fall directly on the equator On that day

As the Equator divides the Earth into two equal halves the sun rays fall directly on the equator twice in a year Equinoxes means equal Spring EquinoxOn 21st March sunrays fall directly on the equator On that day the duration of day and night both are equal ( 12 hours day and 12 hours night) on every places located on equator This day is called as Spring EquinoxAutumn EquinoxOn 23rd September sunrays fall directly on the equator On that day the duration of day and night both are equal ( 12 hours day and 12 hours night) on every places located on equator This day is called as Autumn Equinox

SOLSTICES ndash SUMMER AND WINTERDue to inclination of the Earth on its axis and the apparent movement of the sun the sun rays fall directly on both tropics once in a year Solstice is a Latin word which mean ldquothe Sun standing stillrdquoSummer SolsticesAfter 21st March there is an apparent movement of the Sun to the north of the equator The apparent northward movement up to 21st June when the Sun appears overhead at the Tropic of Cancer (22frac12degN) The sun appears to stand still at this position and then moves southwards towards the equator This position of the Sun on 21st June is known as Summer Solstices On that day the duration of day and night both are equal ( 12 hours day and 12 hours night) on every places located on Tropic of Cancer (22frac12degN)Winter solstices The apparent southward movement of the Sun continues beyond the equator till 22nd

December On this day the Sun is overhead at the Tropic of Capricorn

the duration of day and night both are equal ( 12 hours day and 12 hours night) on every places located on equator This day is called as Spring Equinox

Q2 What do you mean by EquinoxA2 Equinoxes means equal It is use to explain the equal duration of day and night ( 12 hours day and 12 hours night) on the Earth

Q3 On which date the longest day in Tropic of CancerA3 21st June

Q4 What is the meaning of SolsticeA4 Solstice is a Latin word which mean ldquothe Sun standing stillrdquo

Q5 Which is the longest day in southern hemisphereA5 22nd December

Q6 On what date does the Arctic Circle experience the lsquoMidnight SunrsquoA6 On 21 June the Arctic Circle experiences the lsquoMidnight Sunrsquo

Q7 What is cause of Midnight Sun in NorwayA7 During the summer solstice (21 June) the North Pole is inclined towards the Sun Therefore the duration of sunlight or daytime increases from 12 hours at the Equator to 24 hours at the Arctic Circle and beyond Thatrsquos why The region beyond the Arctic Circle especially Norway is known as the Land of the Midnight Sun because there the Sun does not rise or set on 21 June

Q8 Match the column A with BA B

Summer Solstice 21st March

Autumn Equinox 23rd

September

Winter Solstice 21st June

(22frac12degS) This position of the Sun is referred to as the Winter Solstice because it marks the winter season in the Northern Hemisphere On that day the duration of day and night both are equal ( 12 hours day and 12 hours night) on every places located on Tropic of Capricorn (22frac12degS)SEASONS AND DURATION OF DAY AND NIGHT During the equinoxes all places on the Earth have 12 hours of day and 12 hours of night Due to the revolution of the Earth round the Sun on an inclined axis the duration of day and night varies according to seasons and the latitude of a placeDuring the summer solstice (21 June) the North Pole is inclined towards the Sun Therefore the duration of sunlight or daytime increases from 12 hours at the Equator to 24 hours at the Arctic Circle and beyondThe region beyond the Arctic Circle especially Norway is known as the Land of the Midnight Sun because there the Sun does not rise or set on 21 JuneAt the North Pole there will be six months of daylight The Sun will be seen always above the horizon at a low angle At 66degN 24 hours of sunlight can be seen only on 21 June Hammerfest in northern Norway is a place of tourist attraction for observing the phenomenon of the Midnight Sun This place has continuous daylight from 13 May to 29 July This place is easily accessible to tourists and has hotels and other facilities The view of the midnight Sun from here is enthrallingIn the Southern Hemisphere the duration of daylight decreases from 12 hours at the equator to 0 hours beyond the Antarctic Circle In the South Polar Region there is 24 hours of darkness The Sun is always below the horizon In the Southern Hemisphere which experiences winter the duration of night-time is longer than the duration of daylight

Spring Equinox 22nd

December

A8 A B

Summer Solstice 21st June

Autumn Equinox 23rd

September

Winter Solstice 22nd

December

Spring Equinox 21st March

During winter solstice (22 December) the South Pole is inclined towards the Sun The Southern Hemisphere experiences summer and the Northern Hemisphere has winter Therefore the duration of daylight or sunlight is greater in the Southern Hemisphere than in the Northern HemisphereThe duration of daylight increases from 12 hours at the equator to 24 hours beyond the Antarctic Circle The South Polar Region has 24 hours of sunlight for many days continuously At the South Pole there will be six months of sunlight The Sun will always be seen at a low angle above the horizon In the Northern Hemisphere the duration of daylight will decrease from 12 hours at the equator to 0 hours at the Arctic Circle There are 24 hours of darkness in the North Polar region The duration of night is greater than the duration of daylight as one move northwards from the Equator It is evident from the above table that the duration of daylight is 12 hours throughout the year at the equator only As one moves away from the equator the seasonal variations in the duration of daylight increase The seasonal variations in the duration of daylight are maximum at the Polar Regions

Subject Eng Literature (The Merchant of Venice ndash William Shakespeare)Topic Act II Scene 7 Lines 36 to 80 (End of scene ) [Students should read the original play and also the paraphrase provided]

Summary Questions amp AnswersThe Prince then examines the inscription on the silver casket which says ldquoWho chooseth me shall get as much as he deservesrdquo The Prince says that he deserves Portia more than anybody else because of his high rank his noble birth and his great wealth and power But then he argues that silver is ten times

(1) (Act II Sc 7 L 39-47)

From the four corners of the earth they come

To kiss this shrine this mortal breathing saint

The Hyrcanian deserts and the vasty wildsOf wide Arabia are as through-fares now

inferior to gold and therefore he cannot believe that the portrait of such a beautiful lady as Portia can be contained in the silver casket He decides to see the inscription on the golden casket before making his decision

The Prince goes to examine the inscription on the golden casket which says ldquoWho chooseth me shall get what many men desirerdquo The Prince believes that the whole world desires to possess Portia otherwise so many suitors would not have come from all corners of the world for winning Portia Some of them have come from the distant lands of Persia and Arabia The deserts of Persia (Hyrcanian deserts) and the boundless desolate lands of Arabia have been crossed by the Princes seeking the hand of Portia He contrasts this casket containing Portiarsquos portrait with the old English gold coin bearing the image of the archangel (angel of the highest rank) He goes on to remark that while the figure of the archangel is engraved (Insculped) upon the English coin the picture of Portia who is beautiful as an angel lies hidden inside one of the caskets namely the Golden Casket (Golden Bed)

On the basis of his assessment of the inscription on the golden casket the Prince decides to choose the golden casket He asks for the key and opens the golden casket only to find therein an empty human skull holding a roll of

For princes to come view fair PortiaThe watery kingdom whose ambitious headSpets in the face of heaven is no barTo stop the foreign spirits but they comeAs orsquoer a brook to see fair Portia

(i) Explain the occasion for the above mentioned speech

These are the comments of the Prince of Morocco after he reads the inscription on the golden casket His mental process is revealed to us in these words We find him debating within himself as to which casket he should choose

(ii) What light does the above speech throw on the personality of Prince of Morocco

From the above mentioned speech we come to know that the Prince of Morocco is keen to marry Portia He is the type of person who is easily taken away by outward appearance He is in love with Portia because of her beauty

(iii) What information can you gather about Portia from the above mentioned lines

The given speech shows that Portia is a very beautiful lady She must be possessed of good qualities because many suitors come to her place from all over the world with a desire to get married to her The Prince of Morocco is so impressed by her beauty that he calls her a saint According to him the whole world is desirous of having her

(iv) Elucidate the significance of the first two lines

In these lines the Prince of Morocco pays a compliment to Portia These lines show his admiration for her He says that people come from all parts of the world to see fair Portia

(v) Explain the meaning of the last four lines of the

passage

In these lines the Prince of Morocco says that even the vast oceans which throw a challenge at the sky are unable to prevent men from coming to Portiarsquos place to have a glimpse of her These lines are also a tribute to Portiarsquos beauty and good qualities Many men voyage across the ocean treating it as a mere stream to see the beautiful Portia

paper in which is written that whoever happens to be guided by the glitter of things is invariably deceived

On reading the scroll the Prince says that he is too sad at heart to speak a more formal farewell and leaves with his followers amidst a sound of trumpets

After the Prince of Morocco leaves Portia remarks that the Prince is a gentle fellow but she is rid of him May all persons of his nature make a similar choice

IMPORTANT PASSAGES EXPLAINED

(Act II Sc 7 L 39-43)From the four corners of the earth they come

To kiss this shrine this mortal breathing saintThe Hyrcanian deserts and the vasty wildsOf wide Arabia are as through-fares nowFor princes to come view fair Portia

Context

This passage occurs in Act II Scene 7 in The Merchant of Venice This is part of the speech made by the Prince of Morocco

(2)

(Act II Sc 7 L 48-53)

MOROCCO One of these three contains her heavenly pictureIst like that lead contains her

Twere damnation To think so base a thought it were too grossTo rib her cerecloth in the obscure graveOr shall I think in silver shes immurdBeing ten times undervalued to tried gold

(i) What meaning does the Prince of Morocco find out of the inscription of the golden casket What have Belmont and Portiarsquos house been called and why

The inscription on the golden casket is ldquoWho chooseth me shall gain what many men desirerdquo The Prince finds out that it means that the chooser of the golden casket will get Portia because many men desire her In fact the entire world desires her Because of the coming of many suitors to Belmont from different countries in order to win Portiarsquos hand Belmont has become a centre of pilgrimage and her house is the shrine where saintly Portia is installed

(ii) What does the Prince of Morocco do before making the final choice of the casket Which is the correct casket and who will win Portiarsquos hand

The Prince of Morocco surveys and analyses the inscriptions on the casket of lead silver and gold Before making the final choice like a very systematic and methodical person he once again considers the claims of the caskets The casket containing Portiarsquos picture is the correct casket and the person choosing it will win Portiarsquos hand

Explanation

While praising Portia the Prince of Morocco conceives Portia as a goddess whose image is placed inside one of the caskets Many suitors are coming from far and wide the north and the south the east and the west (Four corners) in order to try their luck Some of them have come from the distant land of Persia and Arabia The deserts of Persia (Hyrcanian deserts) and the boundless desolate lands of Arabia have been crossed by the Princes seeking the hand of Portia All this shows that Portia is indeed the most beautiful lady of the world

(iii) What does the Prince of Morocco say in his estimation while examining the motto on the silver casket What does he find in the golden casket

While examining the motto on the silver casket which says ldquoWho chooseth me shall get as much as he deservesrdquo Morocco says that in his own estimation he surely deserves Portia in all respects ndash rank birth wealth etc

He chooses the golden casket When he opens it he finds an empty human skull holding a scroll in which it is written that those who are attracted by the glittering outside of things are always deceived as Morocco has been deceived

(iv) What kind of nature does the Prince of Morocco have

The Prince of Morocco has a simple nature who does not look deeply into the inner meaning of things but is dazzled by the outward appearance of gold He is inclined to over-estimate his own value and does not realize that it is a duty to ldquogive and hazardrdquo To say that he will not hazard for lead shows that he misreads the true meaning of the inscription which is that he should be prepared to ldquohazard all he hathrdquo for Portia So his feeling is only one of fascination and romantic attraction

(v) Do you think that the lottery of the caskets is not a matter that will be determined by chance

In fact the lottery of the casket is not a matter that will be determined by mere chance but that it is a true test of character and of sincerity which is amply proved not only by Moroccorsquos choice but also by the arguments which he uses to help him in his choice

(Act II Sc 7 L 55-59)

They have in England

A coin that bears the figure of an angelStamped in gold but thats insculpd uponBut here an angel in a golden bedLies all within

Context

(3)

(Act II Sc 7 L 63-77)A carrion Death within whose empty eye

There is a written scroll Ill read the writing

All that glisters is not goldOften have you heard that toldMany a man his life hath soldBut my outside to beholdGilded tombs do worms infoldHad you been as wise as boldYoung in limbs in judgment oldYour answer had not been inscrolld

This passage occurs in Act II Scene 7 in The Merchant of Venice This is part of the speech made by the Prince of Morocco

Explanation

In this passage the Prince of Morocco bestows high praise on Portia whose hand he is seeking He contrasts this casket containing Portiarsquos portrait with the old English gold coin bearing the image of the archangel (angel of the highest rank) He goes on to remark that while the figure of the archangel is engraved (Insculped) upon the English coin the picture of Portia who is beautiful as an angel lies hidden inside one of the caskets namely the Golden Casket (Golden Bed) In the day of Elizabeth silver was ten times inferior in value to gold Therefore the Prince of Morocco believing that Portiarsquos portrait is contained in the Golden Casket decides to choose the Golden Casket

Fare you well your suit is coldCold indeed and labour lostThen farewell heat and welcome frostmdashPortia adieu I have too grievd a heartTo take a tedious leave Thus losers part

(i) What reward does the Prince of Morocco get after making a wrong choice of the Casket How does he feel

After making the wrong choice in selecting the casket of gold the Prince of Morocco as a reward earns a rebuke in the form of a scroll tucked in the empty eye-socket of a skull kept in the casket of gold The Prince is shocked and disappointed He becomes all the more sad and dejected when he reads the scroll which points to his foolishness in being misled by the appearance and outward show as indicative of its worth

(ii) How does the Prince respond after reading the scroll

After reading the scroll the Prince though upset accepts the result with good grace and decorum befitting a royal suitor and true sportsman He says that his love-suit is really cold otherwise he would have chosen correctly but now his efforts have been in vain So he bids farewell to Portia to the warmth and enthusiasm of love and welcomes the cold and bitterness of dejection and misery of life which lies ahead

(iii) What request does he make to Portia and why

After being failure in his mission he requests Portia to give him permission to leave at once because he is too sad to undergo the tediousness of a formal leave-taking He tells that it is the manner in which defeated persons part unceremoniously

(iv) Explain the following lines

ldquoAll that glisters is not goldOften have you heard that toldMany a man his life hath soldBut my outside to beholdGilded tombs do worms infoldrdquo

Mere glitter does not make a metal to be gold Man has often been warned against appearance but it has been of no use Many people have sacrificed their lives only to seek the outer appearance of gold Worms are found inside the gilded

monuments

Class XSubject Topic Summary Execution

Hindi 2ndlang

नया रासता भाग 6 मायाराम 0ी घर म धनी मल 0ी और उनी बटी सरिरता ी ही चचा बनी रहती थी अमिमत ो इसम ोई रलिच ना थी वह धनी घर ी लडी स शादी र सवय ो बचना नही चाहता था उसा भी सवाणिभमान ह ईशवर ी पा

स उस पास पस ी ोई मी नही थी अभी उसन फकटरी ही लगाई थी उसी समझ बाहर था कि उस घर वालो ा झाव पस ी तरफ कयो

ह उसन मा स सवाल किया कि मा तम सरिरता स मरी शादी कयो रना चाहती हो मा न उस समझाया कि वह दखन म बरी नही ह और किफर खानदान अचछा

ह वह ए शल गरहणी रप म घर सभाल सगी अमिमत न मा ो इस बात ा एहसास राया कि मीन सबध लिलए मना रन पर उस दिदल

पर कया बीती होगी मा और अमिमत ी लडी बार म ाफी बात हईमा ा झाव सरिरता ी तरफ था कयोकि वह घर पर अचछा दह0 लर आ रही

थी अमिमत न अपनी मौसी ी बरी हालत बार म बताया कि किस तरह वह बड घर ी खानदानी बटी लाई थी और आ0 उसी हालत कितनी खराब ह लाई थी बहकलब 0ाती ह और बचचो ो भी नही दखती ह बात चल ही रही

थी कि तभी ए ार बाहर आर री धनी मल0ी घर अदर आए और पीछ स डराइवर फल ी ए टोरी लर आया अदर आए और पीछ स

डराइवर ए टोरी फल ी लर आया अमिमत ो फल ी पटी बरी लग रही थी अमिमत न पछ लिलया यह फल कयो ल आए ह प इन सब ी कया

0ररत थी उनो न 0वाब दिदया कि 4 पटी शमीर स मगाए थ अमिमत ो या सनर करोध आ गया तभी उस किपता 0ी आ गए उन आत ही अमिमत उठर बाहर चला गया वहा वहा मा पास आर बठ गया और बोला

अभी रिरशता तय नही हआ और धनी मल 0ी धनी मल 0ी फल ी पटी लर चलआय मा न समझाया कि 0ब सबध 0ड 0ाता ह तो खाली हाथ नही

आत अमिमत न मा स हा कि तम सबन सरिरता ो इस घर म लान ी ठान रखी ह धनीमल 0ी उस दिदन सरिरता ो दखन ी तारीख तय रन आय थ

Commercial Studies

Banking Nowadays Bank provide easy and quick services through internet facilities methods of Banking is called internet bankingIn order to save the time and money involved in visiting Bank branches people increasingly prefer to have internet banking

There are different modes of doing internet banking or transferring money through online They areReal Time Gross Settlement (RTGS)National Electronic Fund Transfers (NEFT)

1

Question

1) Explain the term RTGS Write the features of RTGS

Answer)The acronym RTGS stands for Real Time Gross Settlement which may be defined as the continuous real time settlement of funds transfer individually on and order by order basis without netting lsquoReal timersquo may be defined as the processing of instructions at the time they are received rather than at some letter time lsquoGross settlementrsquo may be defined as the settlement of transfer instructions which occurs

individually

Features of RTGS1It is the continuous settlement of

funds transfer individually on an order by order basis

2RTGS facility is provided only by CBS core banking solution enabled Bank branches

3Amount charged from the customer for RTGS transactions vary from bank to bank

2) Explain the term NEFT Write the features of NEFT

Answer) National electronic funds transfer may be defined as a nationwide system that facilitates individuals Farms and copper operates to electronically transfer funds from any bank branch to any individual farm or corporate having an account with any other bank branch in the country

Features of NEFT2 Transfer can be made 7 times on

weekdays and 6 times on Saturday

3 NEFT cannot be used to receive foreign remittances

4 NEFT transaction takes place in batches

5 A bank branch must be NEFT enabled to become a part of NEFT fund transfer network

6 There is no maximum or minimum amount that can be transferred through NEFT when one bank has a bank account

English Language

CompositionEssay

A composition is an art of creating a piece of writing on any topic or subject It is the writing correctly beautifully and clearly in order to make some interesting reading Structure of the composition

Introduction ( you lay the foundation for your composition)

Body (it constitutes the main part of the essay)

Conclusion (final statement that leaves a lasting impression)

Kinds of essays1 The Narrative essay2 The descriptive essay3 The reflective essay4 The argumentative essay

Write a composition on any one of the following topics (350- 400 words)

1 Friendship Or2 The first day of your school

Subject Eng Literature (The Merchant of Venice ndash William Shakespeare)Topic Act V Scene 1 Lines 127 to 158 (Nerissa helliphellip The clerk will nersquoer wear hair onrsquos face that had it) [Students should read the original play and also the paraphrase given in the school prescribed textbook]

Summary Revision Questions o Soon thereafter Bassanio Gratiano

and Antonio arrive

o Bassanio tells Portia that he is feeling as if it is morning because of the presence of Portia who is shining like the sun When Antonio is introduced by Bassanio to Portia she tells Bassanio that he should be grateful to Antonio who took so much trouble on his account even to the extent of risking his life

o Nerissa starts quarrelling with Gratiano and demands that he show her the ring she had presented to him and which she had warned him not to lose She suspects that Gratiano must have presented the ring to some young woman and not to the lawyerrsquos clerk as he repeatedly says and assures

Answer the following questions to check your preparation of Act IV Scenes 1 and 2

You must attempt only after you have completed your preparation of Act IV The answers must be in complete sentences using textual evidence (with citation) when necessary

[It would be in your own interest to attempt the above questions honestly totally refraining from consulting your textbook or your notes during answering After completion you should correct the paper yourself consulting the textbooknotes etc and award marks as specified Please let me know the marks you scored through WhatsApp in the group or to my personal WhatsApp]

Act IV Scene 1 (each question carries 2 marks)

1 What did the Duke try to do for Antonio

2 Why does Shylock refuse to show mercy How does he justify his stance

3 Why does Antonio say he is ready to die 4 What information is contained in Bellariorsquos letter

5 Why does Portia (as Balthazar) assert that Shylock must show mercy How does he respond

6 What offers are made to Shylock to get him to spare Antonio How are they received

7 What does Antoniorsquos speech as he faces the prospect of Shylockrsquos knife tell you about his character

8 How do Bassanio and Gratiano react to the looming prospect of Antoniorsquos demise

9 How does Portia (as Balthazar) use the law to turn the tables on Shylock

10 What does the Duke decree should happen to Shylock Why What happens to Shylockrsquos estate

11 What does Portia ask Bassanio as payment for her ldquoservicesrdquo What is his initial response What makes him change his mind

Act IV Scene 2 (each question carries 1frac12 marks)

1 What does Gratiano bring to Portia (Balthazar)

2 What does Nerissa plan on getting from Gratiano What does Portiarsquos comment suggest about men

ECO-10 280620 Topic-Supply AnalysisSHIFTING OF SUPPLY

But if there is change in factors other than the price of the commodity then either more is supplied at the same price or less supplied at the same price In such cases the price of the commodity remains constant but there is a change in other factors like change in the price of inputs change in technology of production change in price of other related goods change in taxation policy of the government etc For example there is an improvement in the technology of production of the commodity in question It leads to decrease in per unit of cost production of the commodity The firm is willing to sell more quantity of the commodity at the same price So the supply other commodity increases at the same price This increase in supply is shown by rightward shift of supply curve On the other hand if the firm uses inferior technology of production the cost of production per unit of the commodity increases The firm is willing to sell less quantity at the same price So the supply of the commodity decreases at the same price This decrease in supply is shown by leftward shift of the supply curve The above cases of increase and decrease in supply can be shown with the help of the following figures

Y INCREASE OF SUPPLY Price (Rs) s

P A s1

B

s

X` O s1 X

q q1

Y` Quantity demanded (in units)

Y DECREASE IN SUPPLY s2

s

price (Rs)

C

p A

s2

s

X` o X

q2 q

Y` Quantity demanded ( in units)

Main factors causing increase in supply or rightward shift of supply Curve(i) Fall in the price of other related goods

(ii) Fall in the price of inputsfactors(iii) Use of better technology in production(iv) Decrease in the rate of excise duty by government(v) If the objective of producer changes from profit maximization to salesMaximization

Main factors causing decrease in supply or leftward shift of supply curve(i) Increase in the price of other related goods(ii) Rise in the price of inputsfactors(iii) Use of inferior technology in production(iv) Increase in the rate of excise duty by the government(v) If the objective

Subject - Biology Topic ndash Chapter mdash6 PhotosynthesisSummary Execution

Today we will know about photosynthesis and its stages

Q1 What do you mean by photosynthesis The process by which living plants containing chlorophyll produce food

substances from carbon-di- oxide and water by using light energy Sunlight

6CO2 +12 H2O----------------------- C6 H12O6 + 6H2O + 6O2

Chlorophyll

Q2 What are the importance of photosynthesis I) Food for all Green plants trap solar energy by photosynthesis

process and supply food and energy for all living organisms either directly or indirectly

Ii) Oxygen to breathe in by product of photosynthesis is oxygen which is essential for all living organisms respiration

Q3 Write about two main phases of photosynthesis A Light dependent phase This phase occur in grana of chloroplast I) The chlorophyll on exposure to light energy becomes activated by

absorbing photons Ii) The absorbed energy is used in splitting the water molecules (H2O)

into its two components (H+ and OH- ) and releasing electron s 2H2O------------------------- 4H+ + 4e- +O2

Energy of 4 photons This reaction is known as photolysis

End products are H+ and oxygen water

B Light independent (Dark ) phase The reactions in this phase require no light energy

Here CO2 combine with H+ and produce glucose

Class XI

Subject Topic Summary ExecutionEVS Chapter-4 Legal

regimes for sustainable development

Environmental legislationEnvironmental legislation is the collection of laws and regulations pertaining to air quality water quality the wilderness endangered wildlife and other environmental factors The act ensures that matters important to the environment are thoroughly

Learn -The Forest (Conservation) Act 1980

considered in any decisions made by federal agencies

The Forest (Conservation) Act 1980 The Forest (Conservation) Act 1980 an Act of the Parliament of India to provide for the conservation of forests and for matters connected therewith or ancillary or incidental thereto It was further amended in 1988 This law extends to the whole of IndiaObjects and Reasons of the Forest Conservation Act

Deforestation causes ecological imbalance and leads to environmental deterioration Deforestation had been taking place on a large scale in the country and it had caused widespread concern The act seeks to check upon deforestation and de-reservation of forests

Subject Eng Literature (The Tempest ndash William Shakespeare) Topic Act II Scene 1 Lines 314 to 329 (End of scene)

[Students should read the original play and also the paraphrase given in the school prescribed textbook]Summary Questions amp Answers

Conspiracy of Antonio and Sebastian (Contd)

o As they approach Ariel appears again and wakes up Gonzalo by singing a tune in his ear Alonso also wakes up and they see both Sebastian and Antonio with drawn swords On being caught off guard they make up a story saying that they had heard a bellowing of bulls or lions

o They then moved to another part of the island

o Ariel at once rushes to Prospero to inform him of this development

SUMMING-UP of ACT-2 SCENE-1

(i) Among the survivors Ferdinand is separated from the rest which results in the disconsolate grief of Alonso as he took him for dead

(ii) The villainy of Antonio is confirmed

(iii) The supremacy of Prosperorsquos magic which resulted in the failure of the human conspiracy

(1)

(Act II Sc 1 L 311-325)SEBASTIAN Whiles we stood here securing your repose

Even now we heard a hollow burst of bellowing Like bulls or rather lions Didt not wake youIt struck mine ear most terribly

ALONSO I heard nothingANTONIO O rsquotwas a din to fright a monsters ear

To make an earthquake Sure it was the roarOf a whole herd of lions

ALONSO Heard you this GonzaloGONZALO Upon mine honour sir I heard a humming

And that a strange one too which did awake meI shaked you sir and cried As mine eyes opened I saw their weapons drawn There was a noiseThats verily rsquoTis best we stand upon our guardOr that we quit this place Lets draw our weapons

(i) Why has Prospero sent Ariel to Gonzalo and Alonso What does Ariel do to awaken Gonzalo

Prospero has already come to know by his magic powers the danger which threatens Gonzalo who had been Prosperorsquos friend and so he sent Ariel to preserve the lives of both Gonzalo and Alonso Prospero does not want that his scheme should remain unfulfilled Ariel begins to sing a song in Gonzalorsquos ears to awaken him(ii) Who are ready to carry out their plan Who takes steps to stop them Why does Gonzalo feel surprised after being awakened

Sebastian and Antonio are ready to carry out their plans They are standing with their swords drawn to kill Alonso and

(iv) We see two sets of contrasting characters Gonzalo-Adrian against Antonio-Sebastian

(v) The grief that works in Alonso can be perceived to his repentance for his association in Antoniorsquos crime against Prospero

Gonzalo Ariel takes steps to stop them from carrying out their nefarious scheme When Gonzalo is awakened by the song sung by Ariel into his ears he (Gonzalo) feels surprised because he sees Sebastian and Antonio standing with their swords drawn(iii) What reason do Sebastian and Antonio tell of drawing their swords when they are suspected by Alonso and Gonzalo

When Sebastian and Antonio are seen with their swords drawn they are looked with suspicion by Gonzalo and Alonso At first Sebastian tells them that as they stood here to guard them during their sleep they heard only a little before a sudden loud noise very much like the roaring of bulls or more probably that of lions Then Antonio follows him saying that this was a noise so terrible as to frighten even a monsterrsquos ears and this noise could even have shaken the earth and it was surely like the roaring of a multitude of lions Then seeing the danger they have drawn their swords Perhaps after hearing the terrible noise they (Gonzalo and Alonso) woke up from their sound sleep

(iv) What does Gonzalo tell Alonso about the strange noise What did he see on opening his eyes Gonzalo tells Alonso that he did not hear the sound of roaring but he heard a humming sound which was strange and which woke him up After waking up he gave him (Alonso) a shaking and a loud cry On opening his eyes he saw these two gentlemen standing with their swords drawn(v) What does Gonzalo suggest

Gonzalo suggests that there was a noise indeed and of that he has no doubt at all and suggests that the best course for them would be to remain alert and vigilant against any possible danger to their lives or to leave this place and move to some other part of the island

Class XIISubject Topic Summary Execution

Commerce

Chapter- Management

Today we will discuss about LEVELS OF MANAGEMENT

Levels of management is a series or chain of managerial positions from top to bottom It helps individuals to know their authority responsibilities and superior-subordinate relations among themselves There are mainly three levels of Management TOP LEVEL MANAGEMENTMIDDLE LEVEL MANAGEMENTLOWER LEVEL MANAGEMENT

Top level managementIt consists of members at the highest level in the management hierarchy This level includes Board Of Directors Chief Executive Managing Directors Chairman President Vice President

Rolefunctions of the top levelmanagement1To analyse evaluate and deal

with theexternal environment2 To determine the objectives and

policies of the business3 To strive for welfare and survival

of business

4 To create an organisational Framework consisting of authority responsibility relationship

Middle level management Congress of members or groups who are concerned with implementation of the policies let down by the top managementThis level includes head of the department such as finance manager marketing manager branch and regional managers departmental and divisional heads plant superintendent etc

Role of functions of the middle level management

1 To interpret the policies framed by top management

2 To assign duties and responsibilities to lower level managers

3 To select and appoint employees for middle and supervisory level and evaluate their performance

4 To co-operate with other departments for smooth functioning

Operational or supervisory level managementIt refers to the group are members who are concerned with execution of the work They are also known as fast line managers This level includes supervisor 4 men Section Officer clerk Inspector etc

Role of functions of the lower level management1 To plan and execute day-to-

day operations2 To supervise and control the workers3 To arrange materials and

tools to start the process and make arrangements for training

4 Today present workers grievance and suggestions before the management and

ensure safe and proper working conditions in the factory

Business Studies

Staff Appraisal Chapter- 10 Today let us start with a new chapter

Staff Appraisal

Meaning of Performance Appraisal

Performance Appraisal is the systematic evaluation of the performance of employees and to understand the abilities of a person for further growth and developmentThe supervisors measure the pay of employees and compare it with targets and plansThe supervisor analyses the factors behind work performances of employeesThe employers are in position to guide the employees for a better performance

Objectives of Performance Appraisal

Following are the objectives of Performance Appraisal

To maintain records in order to determine compensation packages wage structure salaries raises etc

To identify the strengths and weaknesses of employees to place right men on right job

To maintain and assess the potential present in a person for further growth and development

To provide a feedback to employees regarding their performance and related status

To provide a feedback to employees regarding their performance and related status

Importance of Performance Appraisal

Performance appraisal provides important and useful information for the assessment of employees skill

knowledge ability and overall job performance The following are the points which indicate the importance of performance appraisal in an organization

1 Performance appraisal helps supervisors to assess the work performance of their subordinates

2 Performance appraisal helps to assess the training and development needs of employees

3 Performance appraisal provides grounds for employees to correct their mistakes and it also provides proper guidance and criticism for employees development4 Performance appraisal provides reward for better performance

5 Performance appraisal helps to improve the communication system of the organization

6 Performance appraisal evaluates whether human resource programs being implemented in the organization have been effective

7 Performance appraisal helps to prepare pay structure for each employee working in the organization

8 Performance appraisal helps to review the potentiality of employees so that their future capability is anticipated

Geography

DRIANAGE The SubarnarekhaThe Subarnarekha and the Brahmaniinterposed between the Ganga and the Mahanadi deltas drain an area of 19300 sq kmand 39033 sq km respectively The drainage basins of these streams are shared byJharkhand Odisha west Bengal and Chhattisgarh The Brahmani is known as southKoel in its upper reaches in Jharkhand

The NarmadaThe Narmada rises in the Amarkantak hills of MadhyaPradesh It flows towards the West in a rift valleyformed due to a geological fault The total length of it is 1300 km All the tributaries of the

Q1 Name the two westward flowing rivers in the peninsular plateauA1 Narmada and Tapi are the only westward flowing rivers of the peninsular plateau

Q2 Differentiate between east-flowing rivers and west-flowing riversA2

East-flowing rivers

West-flowing rivers

Narmada are very short inlength Most of its tributaries join the main streamright anglesThe Narmada basin covers parts of Madhya Pradesh and Gujarat

The Tapi The Tapi rises in the Satpura ranges in the Betul listrictof Madhya Pradesh It flows in a rift valley parallel tothe Narmada but it is much shorter in length It coversparts of Madhya Pradesh Gujarat and MaharashtraThe length is about 724 km

The Sabarmati and the MahiThe Sabarmati rises in the Aravali hills and flows south-south-westwards for a distance of 300 kilometres to the Arabian Sea The Sabarmatibasin extends over an area of 21674 sq km in Rajasthan and Gujarat The Mahi rises inthe east of Udaipur and drains an area of 34842 sq km lying in Madhya PradeshRajasthan and Gujarat It flows south-westwards for a distance of 533 km before it fallsinto the Gulf of Khambhat

The ChambalThe Chambal rises near Mhow in the Vindhya Range and flows towards the northgenerally in a gorge upto Kota Below Kota it turns to the north-east direction and afterreaching Pinahat it turns to the east and runs nearly parallel to the Yamuna beforejoining it in the southern part of the Etawah district in Uttar PradeshMajor Rivers of India with their basin area (Sqkm)

Himalayan System Indus 321290Ganga 861404

Brahmaputra 187110Indus System

Jhelum 34775Beas 20303

Ganga System Yamuna 366223Ghaghra 127950

Peninsular RiversNarmada 98796

Tapi 65145Mahanadi 141600

Subarnarekha 19300Sabarmati 21674

Mahi 34842Godavari 312812

Godavari Krishna Kaveri Mahanadi are the east-flowing rivers

Narmada Tapi west-flowing rivers

They fall into the Bay of Bengal

They fall into Arabian Sea

These rivers form big deltas

These rivers form comparativelysmall deltas

Catchment areas of these rivers are larger

Catchment areas of these rivers are smaller

Krishna 2589488Cauveri 87900

Subject ndashBiology Topic ndashChapter -5 Inheritance amp Variations Summary ExecutionToday we will discussabout linkage and its classification

LINKAGE The tendency of the genes located on the same chromosome to stay together is

hereditary transmission Linked genes the genes responsible for this Genes that exhibit the process of linkage locates in the same chromosome The distance between the linked genes in a chromosome determines the strength

of linkage i e genes that are located close to each other show stronger linkage than that are located far from each other

COMPLETE LINKAGE It is the type of linkage showed by the genes that are closely located or are tightly

linked with each other as they have no chance of separatingby crossing over These genes are always transmitted together to the same gamete and the same

offspring In such condition only parental or non cross over type of gametes are formedINCOMPLETE KINKAGE It is type of linkage showed by the genes that are distantly located orare loosely

linked with each other because they have chance of separating by crossing over

SIGNIFICANCE i) It helps in holding the parental character togetherii) It checks the appearance of new recombination and helps in bringing the

hybrid population which resembles the original parents iii) Linked genes dilute the effects of undesirable traits

Subject Eng Literature (The Tempest ndash William Shakespeare) Topic Essay Questions (EQ-3)Question No 3

Give a character sketch of CalibanAnswer

The character of Caliban has been wonderfully conceived by Shakespeare as the manifestation of all that is gross and earthy ndash a sort of creature of the earth as Ariel is a sort of creature of the air

Calibanrsquos Physical Appearanceo Caliban is lsquofreckledrsquo a lsquomisshapen knaversquo not honoured with human shape

o Prospero calls him lsquothou tortoisersquo (Act I Sc 2 Line 317) Trinculo stumbling upon him describes him as ldquoA strange fish hellip Legged like a man And his fins like armsrdquo He ldquosmells like a fishrdquo (Act II Sc 2 Line 25)

o Prospero also calls him a ldquobeastrdquo (Act IV Sc 1 Line 140) and ldquoThis misshapen knaverdquo (Act V Sc 1 Line 268)

o Further it appears that in addition to his physical deformity his spiritual inferiority is also suggested by Prosperorsquos claim that his birth resulted from the union between his mother the witch Sycorax and the devil

Calibanrsquos ParentageWhen the play opens Caliban is twenty four years of age having been born on the island twelve years before the coming of Prospero His mother was the foul witch Sycorax who was banished from Algiers for ldquomischiefs manifold and sorceries terrible to enter human hearingrdquo (Act I Sc 2 Line 264) and the father was the Devil himself Thus

Caliban is a monster of evil and brute nature ugly deformed and stinking

Calibanrsquos Savage and Malignant Natureo Caliban is entirely a creature of the earth ndash gross brutal and savage He regards himself as the rightful possessor

of the island and Prospero as a usurper

o In his young age he was on good terms with Prospero He had consented to be received by Prospero at his house and to be educated by him He has learnt human language only to curse his master whom he abhors

o His beastly nature soon breaks out and ends in a vicious attack on Miranda This opens the eye of Prospero who becomes severe to him and enforces his service by threats and violence

o Prospero uses him to make dams for fish to fetch firewood scraper trenches wash dishes and keep his cell clean

Calibanrsquos Hatred for ProsperoA profound hatred for Prospero has taken hold of Caliban It springs from a sense of his being dispossessed and ill-treated He would kill Prospero if he could but he knows the power of Prosperorsquos lsquobookrsquo Hence he transfers his allegiance to Stephano who seems like a god to him He also incites the two drunken associates to batter the skull of Prospero when he sleeps in the afternoon

Caliban Shows Considerable Intelligenceo He has learnt Prosperorsquos language

ldquoYou taught me language and my profit onrsquot (Act II Sc 2 Lines 86-89)Is I know how to curserdquo

o He is well aware of the futility of arguing with one who has more power than he has

ldquoI must obey his art is such power (Act I Sc 2 Lines 373-376)It would control my damrsquos god SetebosAnd make a vassal of himrdquo

o He realizes the importance of Prosperorsquos books

ldquoRemember (Act III Sc 2 Lines 89-92)First to possess his books for without themHersquos but a sot as I am nor hath notOne spirit to commandrdquo

o He knows the value of stealth when attacking the enemy

ldquoPray you tread softly that the blind mole may not (Act IV Sc 1 Lines 194-195)Hear a foot fall we now are near his cellrdquo

o Caliban has a better set of values than Stephano and Trinculo They are distracted from their plan by their greed for Prosperorsquos rich garments Only Caliban realizes that such a finery is unimportant

ldquoLeave it alone thou fool it is but trashrdquo (Act IV Sc 1 Lines 224)

Caliban is not a good judge of characterCaliban is not a good judge of character He decides for example that Stephano is a god because he dispenses lsquocelestial liquorrsquo (Act II Sc 2 Line 115) but then it must be remembered that he has only known his mother Sycorax Prospero Miranda and the spirits that torture him However he quickly discovers his error of judgementrdquo

ldquoWhat a thrice-double ass (Act V Sc 1 Lines 295-297)Was I to take this drunkard for a godAnd worship this dull foolrdquo

Calibanrsquos Imaginative NatureIf Caliban is sub-human in what has been said above he is human in the respect of the poetic side of his character He listens to music with rapture He tells of the beautiful dreams in which heaven rains treasures upon him and which upon waking he yearns to renew One of the most poetic passages in whole play is Calibanrsquos description of the island

to Stephano and Trinculo

ldquoBe not afeard The isle is full of noises (Act III Sc 2 Lines 135-143)Sounds and sweet airs that give delight and hurt notSometimes a thousand twangling instrumentsWill hum about mine ears and sometime voicesThat if I then had waked after long sleepWill make me sleep again and then in dreamingThe clouds methought would open and show richesReady to drop upon me that when I wakedI cried to dream againrdquo

Caliban - Less Ignoble Than Some OthersCalibanrsquos motive for murder is less dishonourable than that of Antonio and Sebastian They plan to kill Alonso to gain his power and wealth Caliban merely wants revenge and the return of lsquohisrsquo island

Conclusiono Calibanrsquos character is not portrayed very clearly in the play and hence we cannot decide whether he is a poor

savage being grossly maltreated by Prospero or whether he is evil and must therefore be kept in bondage or enslavement

o Caliban is contrasted with Ariel who is a spirit and thus swift and uninterested in physical activitieso Caliban is also contrasted with Prospero who is the all-powerful master of the island and of the destiny of all

those on the islando Caliban is also contrasted with civilized man showing him to be less evil than Antonio and Stephano and less

materialistic than Stephano and Trinculoo Caliban has suffered at the hands of Prospero and he has learnt to curse by listening to Prosperorsquos abuse He

certainly believes that Prospero has deprived him of his birthrighto Finally the character Caliban is thought to be one of Shakespearersquos masterpieces The complexity of the character

is reflected in the large volume of critical discussion that has grown around it

ECO ndash12 Topic-Forms of market

MonopolyMonopoly is a market structure in which there is a single seller there are no close substitutes for the commodity produced by the firm and there are barriers to entry Example Indian Railways which is operated under government of India Monopoly also implies absence of competitionFeatures of Monopoly Monopoly is characterized by1 Single Seller In monopoly there is only one firm producing the product The whole industry consists of this single firm Thus under monopoly there is no distinction between firm and industry Being the only firm there is significant control of the firm over supply and price Thus under monopoly buyers do not have the option of buying the commodity from any other seller They have to buy the product from the firm or they can go without the commodity This fact gives immense control to the monopolist over the market

2No Close Substitute There are no close substitutes of the product produced by the monopolist firm If there are close substitutes of the product in the market it implies presence of more than one firm and hence no monopoly In order to ensure a total of control over the market by the monopolist firm it is assumed that there are no close substitutes of the product

3 No Entry amp Exit Monopoly can only exist when there is strong barriers before a new firm to enter the market In fact once a monopoly firm starts producing the product no other firm can produce the same One reason for this is the ability of the

monopolist to produce the product at a lower cost than any new firm who thinks to enter the market If a new firm who knows that it cannot produce at a lower cost than the monopolist then that firm will never enter the market for fear of losing out in competition Similarly the monopolist who is operating for a long time may be enjoying reputation among its customers and is in a better position to use the situation in its own benefit A new firm has to take long time to achieve this and so may not be interested to enter the market

4 Price Maker Being the single seller of the product the monopolist has full control over the pricing of the product On the other hand if there is a large number of buyers in the market so no single buyer exercises any significant influence over price determination Thus it is a sellerrsquos market So monopoly firm is a price maker

5 Price Discrimination Having considerable control over the market on account of being single seller with no entry of other firms the monopolist can exercise policy of price discrimination it means that the monopolist can sell different quantities of the same product to a consumer at different price or same quantity to different consumers at different prices by adjudging the standard of living of the consumer

6 Shape of Demand Curve Since a monopolist has full control over the price therefore he can sell more by lowering the price This makes the demand curve downward sloping

Subject Ac-12 290620 Topic- retirement Model sumThe Balance Sheet of Rohit Nisha and Sunil who are partners in a firm sharing profits according to their capitals as on 31st March 2014 was as under

Liabilities Amount Assets Amount (Rs) (` Rs)

Creditors 25000 Machinery 40000Bills Payable 13000 Building 90000General Reserve 22000 Debtors 30000Capital Less Provision for Rohit 60000 Bad debts 1000

29000 Nisha 40000 Stocks 23000 Sunil 40000 140000 Cash at Bank 18000

200000 200000

On the date of Balance Sheet Nisha retired from the firm and following adjustments were made(i) Building is appreciated by 20(ii) Provision for bad debts is increased to 5 on Debtors(iii) Machinery is depreciated by 10(iv) Goodwill of the firm is valued at Rs 56000 and the retiring partnerrsquos share is adjusted

(v) The capital of the new firm is fixed at Rs120000 Prepare Revaluation Account Capital Accounts of the partner and Balance Sheet of the new firm after Nisharsquos retirement Revaluation AccountDr Cr

Particulars Amount Particulars Amount (`Rs) (Rs`)

Provision for Bad debt Ac 500 Building Ac 18000Machinery Ac 4000Profit transferred toCapital Accounts (3 2 2)Rohit 5786Nisha 3857Sunil 3857

13500

18000 18000

Capital Account

Dr Cr

Particulars Rohit Nisha Sunil Particulars Rohit Nisha Sunil (Rs`) (Rs`) (`Rs) (Rs`) (Rs`) (Rs`)

Sunilrsquos Capital ac 9600 mdash 6400 Balance bd 60000 40000 40000Bank - 66143 - General Reserve 9428 6286 6286Balance cd 72000 mdash 48000 Revaluation (Profi 5786 3857 3857 Rohitrsquos Capital Ac mdash 9600 mdash

Sunilrsquos Capital Ac 6400 Bank 6386 - 4257

81600 66143 54400 81600 66143 54400

Balance Sheet as at 31st March 2014

Liabilities Amount Assets Amount (Rs`) (Rs`)

Creditors 25000 Building 108000Bank overdraft 37500 Machinery 36000

Bills Payable 13000 Debtors 30000Capital Less ProvisionRohit 72000 for Bad debts 1500 28500Sunil 48000 120000 Stock 23000

195500 195500

Working Notes (i) (a) Profit sharing ratio is 60000 40000 40000 ie = 3 2 2(b) Gaining Ratio Rohit = 35 ndash 37 = 2135 ndash 1535 = 635Sunil = 25-27 = 1435 ndash 1035 = 435= 635 435= 6 4 = 3 2(c) Nisha Share of Goodwill = Rs 56000 times 27 = Rs16000Share of Goodwill in the gaining ratio by the existing partner ieRohit = Rs16000 times 35 = Rs 9600Sunil = Rs 16000 times 25 = Rs 6400

The journal entry isRohitrsquos Capital Ac Dr 9600Sunilrsquos Capital Ac Dr 6400 To Nisharsquos Capital Ac 16000(Share of Goodwill divided into gaining ratio)

  • 1 Static Friction
  • The frictional force that acts between the surfaces when they are at rest with respect to each other is called Static Friction
    • Static Friction Examples
      • 2 Sliding Friction
        • Examples Of Sliding Friction
          • 3 Rolling Friction
            • Examples Of Rolling Friction
              • Objects and Reasons of the Forest Conservation Act
Page 10:  · Web viewSubject . Topic . Summary . Execution . English 1 . Sounds of animals . Hens –cackle Horses –neigh Lions –roar Owls –hoots Snake –hiss. English 2 . Mother’s

MAT

HEM

ATIC

S

Ch 9

Com

mon

Fra

ction

s

A fraction is a number that stands for parts of a whole object or a collection of objects

Each fraction has two numbers One is written above the other separated by a line The one above the line is called numerator and the one below the line is called denominator

Example 5minusminusminusminusrarr Numerator

11minusminusminusminusrarr Denominator

Exercise ndash 362 In following fill in the blanks

(b) 37 ___ is denominator ___ is

numerator

(c) 89 ___ is denominator ___ is

numerator

Solution

(b) 37 7 is denominator 3 is numerator

(c) 89 9 is denominator 8 is numerator

3 Write down the fractions whose numerators and denominators are given below in the bracket The first number stands for numerator and the second number standing for denominator

(25) 25

(311) 311

(416) 416

(712) 712

Class VSubject Topic Summary Execution

Science Ch ndash PlantReproduction

In pollination chapter we have learnt that the flowers change into fruits and the fruits bear seeds Now we are going to learn how this process takes place

FertilizationThe process of fusion of the male reproductive cell (male gamete) and female reproductive cell (female gamete) is known as fertilizationWhen a pollen grain reaches from the anther to a stigma it begins to grow and forms apollentube The Pollen tube then travels down through the style to enter an ovule inside the ovary On reaching the ovule male reproductive cell in the pollen grain unites with the egg cell present in the ovule

Books exercise

A) Tick the correct answer

1Which of the following do ovules change into after fertilization ndashseeds

2Which of the following is not a part of the seed ndash flower

3Which of the following condition is needed for germination of a seed ndash all ofthese

English language

Sentences phrases and

Solved exercisesSay which of the underlined groups of words are phrases and which clauses

clauses8 In her new clothes ndashphrasesAs pretty as a doll ndash phrases

9 looking sad and upset ndash phrasesHe had lost all the tickets for the Test Match ndash clauses

10 During the vacation now only a month away ndashphrases

11 too sweet and too hot ndashphrases

12 At the Olympic Games ndashPhrasesOf Laurel leaves ndash phrases

13 Who played the role of Hamlet ndashclauses

14 However fast ndash phrases

15 When the men fell asleep ndash clausesSocial studies

Indian Government

Lok Sabha (lower house) ndash It has 552 members Of these 530 membersrepresent States 20 members represent the union territories and two members represent the Anglo- Indian community All except the representatives of the Anglo-Indian community are elected by Indian citizens A person above the age of 25 can contest in the elections for Lok Sabha One term of Lok Sabha is for 5 yearsRajya Sabha (upper house) ndash Its members are elected by the MLAs or members of the legislative Assembly There are 250 members in the Rajya Sabha of which 12 are nominated by the President One term of Rajya Sabha is for 6 years Anyone above the age of 30 can be elected as a member of Rajya Sabha

ExecutivePresidentThe President is the head of the country in India He is elected by the MPs and the MLAs for a tenure of 5 years He appoints the Prime Minister and the Council of Ministers Prime MinisterThe party which wins the election forms the government and its leader is elected as the Prime Minister He is the chief advisor to the President The Council of Ministers assists the Prime Minister and is accountable for their roles For example the Education minister is responsible for the education system in our country

1 How many members are there in Lok SabhaAns 552 members

2 What is the term for Lok SabhaAns 5 years

3 How many members are there in the Rajya SabhaAns 250 members

4 Who is the head of our countryAns President

5 Who is the chief advisor to the PresidentAns Prime Minister

Book ndash GK

Ch ndash 1First in space

1 First living being into space in 1957 Ans Laika

Times 2 First person to go into space in 1961 Ans Yuri Gagarin

3 First woman to go into space in 1963 Ans ValentinaTereshkova4 First person ever to walk in space in 1965Ans alexei Leonov5 First person to land on the moon in 1959 Ans Neil Armstrong6 First Indian to go into space in 1984 Ans Rakesh Sharma 7 First Indian woman to go into space in 19978 Ans Kalpana Chawla9 First woman tourist in space in 2006

Ans Anusheh AnsariCOMPUTER

ALGORITHM AND FLOWCHART

Q) DRAW THE SYMBOLS USED IN A FLOWCHART WITH THEIR DESCRIPTIONS(IN EXAM IT CAN COME AS SHORT QUESTIONS ASKING INDIVIDUAL SYMBOLS FUNCTION)ANS)

MAT

HEM

ATIC

S

Ch 6

Com

mon

Fra

ction

s

Multiplication of FractionsA Multiply a fractional number by whole numberTo multiply a fractional number by whole number we multiply the numerator of the fractional number by the whole number and denominator of the fractional number by 1 The first product thus obtained is the numerator and the second product is the denominator of the required product

Exercise ndash 30Multiply

7 2027

times 9

Solution 2027

times 9 = 203 = 6

23

8 611

times11

Solution 611

times11 = 6

15 71

20times16

Solution 71

20times16 =

14120

times16

= 1415

times 4 = 141times 4

5 = 564

5 = 11245

B Multiplication of a fractional number by a fractional number To multiply a fractional number by a fractional number we multiply the numerator of the first fractional number by the numerator of the second fractional number and the denominator of the first fractional number by the denominator of the second fractional number The first product thus obtained is the numerator and the second product is the denominator of the required product

16 2712

times24

Solution 2712

times24 = 3112

times24

= 31times2 = 62

Exercise ndash 31

11 83

times 34

2

Solution 83

times 34 = 2

14 723

times2 25

4

Solution 723

times2 25 =

233

times 125 =

23times 45

= 925 = 18

25

15 1212

times1 13

2

Solution 1212

times1 13 =

252

times 43 =

25times 23

= 503 = 16

23

State the following statements are true or false

17 1912

times 239 = 1

Solution LHS = 1912

times 239

= 392

times 239 = 1 = RHS

[LHS = Left hand side amp RHS = Right hand side]

there4 1912

times 239 = 1 [True]

21 213

times2 13 = 4

19

Solution LHS = 213

times2 13 =

73

times 73

= 7times73times3 =

499 = 5

49

there4 LHS ne RHS

So 213

times2 13 = 4

19 [False]

23 23

times 45 =

2times 5+3 times43times 5

Solution

LHS = 23

times 45 =

2times 43 times5 =

815 again

RHS = 2times 5+3 times4

3times 5 = 10+12

15 = 2215

there4 LHS ne RHS So 23

times 45 =

2times 5+3times43times 5

[False]

25 23 of

13 =

29

Solution

LHS= 23 of

13 =

23 times

13 =

29 = RHS]

there4 23 of

13 =

29 [True]

Practice at HomeExercise ndash 31State the following statements are true or false

24 12 of 4 =

18

Class VISubject Topic Summary Execution

HISTORY AND CIVICS

Chapter 5The Mauryan Empire

DECLINE OF MAURYAN EMPIREDecline of Mauryan empire started after the death of Ashoka at around 232 BCThere are several reasons for break up of the empire1 Weak successor Emperors after Ashoka were

capable of handling vast and mighty Mauryan empire In 185BC the last Mauryan ruler Bri-hadrath was murdered by his Commander-in-Chief Pushyamitra Sunga

2 Provincial Revolts Due to weak central author-ity provincial chiefs of Kalinga and southern provinces revolted against emperor and freed themselves from Mauryan empire

3 Weakness of Economy Prosperity of Mauryan was based on solid economic activities which

ExercisesI Multiple choice questions-1 Chandragupta defeated Seleucus in the year ndashc) 305 BC2 Who killed the last Mauryan ruler Brihadrath b) Pushyamitra3 Which of the following was not a reason for the decline of the Mauryan empirec) Chandraguptarsquos weakness4 Ashoka invaded Kalinga in the year c) 261 BC

II Fill in the blanks1Chandragupta ascended the throne in 324

was taken care by early monarchs Later kings had neither ability nor interest in economic af-fairs That led to failure in tax collection As a result they failed to maintain a large army that were essential to keep empire intact

4 Greek Invasion Greeks freed north-western provinces from weak Mauryan monarchs and reestablished their authority

5 Ashokarsquos Policy some scholar opined that after Kalinga war Ashoka embraced Buddhism re-nounced the policy of war and disbanded the Army But this is partially true as there is no proper evidence of disbanding the army

Based on above points we can conclude that main reason for decline of Mauryan empire is weakness of Ashokarsquos successors Kunal Samprati Dasharath Salisuk all were weak kingsAt last in 185 BCPushyamitra Sunga killed king Brihadrath and established the Sunga dynasty

BC2 Bindusara was the son of Chandragupta and father of Ashoka

3 Pataliputra was administered by City Magistrate committess of 5 members each4 The Greek General Seleucus sent his ambassador Megasthenes to Chandraguptarsquos court5 Ashoka sent his son Prince Mahendra and daughter Sanghamitra to spread his Dhamma6 The Indian Rebublic has adopted the Lion Capital of Saranath Pillar as its national emblem 7 Pushyamitra killed the last Mauryan ruler Brihadrath and founded the Sunga dynasty

III Name the following

1The author of Arthashastra-Kautilya2 The ruler who founded the Mauryan dynasty-Chandragupta3 The author of Indika-Megasthenes 4 The officers who were appointed by Ashoka to spread Dhamma-Dhamma Mahamatras5 The general of Alexander whom Chandragupta defeated-Seleucus

V Match the columns1 Kautilya (c)2 Megasthenes (d)3 Pushyamitra (e)4 Brihadrath (b)5 Bindusara (a)

BENGALI(2ND

LANGUAGE)

পশপাহিখর -াষাসহিবনয় রায়কেচৌধরী

যলখক পহিরহিচহিত- পরখযাত সাহিহিতযক উকেপনদরহিককেশার রায়কেচৌধরীর পতর সহিবনয় রায়কেচৌধরী lsquoসকেFশrsquo পহিতরকার সকেb হিতহিন কত হিকেলন তার উকেdখকোয বই lsquoসহিবনয় রায়কেচৌধরীর রচনা সংগরrsquo

পরম হিকেনর পাঠ- lsquoপশপাহিখর হিক -াষাhelliphellip helliphelliphelliphellipপরসপরকেক জানাবার উপায়ও পশপাহিখরা যবশ জাকেনrsquoপরকেমই আমারা জাহিন -াষা হিক -াষা ল আমাকের মকেনর -াব পরকাশ করার জনয আমরা নানান ধরকেনর -হিb বা হিবকেশষ ধরকেনর আওয়াজ মকেখর মাধযকেম কহির অনযকেক যবাঝাকেনার জনয তাকেল এবার আমরা জাহিন পশপাহিখর -াষা হিক পশ পাহিখরা হিক কা বকেল যা পশপাহিখকেরও -াষা আকে তারা তাকের হিনজসব -াষায় কা বকেল মকেনর -াব পরকাশ ককের পশ পাহিখরা মানকেষর হিক হিক -াষা যবাকেঝ হিকনত তারা বলকেত পাকেরনা পরসপরকেক বহিঝকেয় যবার উপায় তারা জাকেননা তকেব তারা হিবকেশষ ককেয়কটি শকেবদর মাধযকেম তাকের মকেনর -াব বহিঝকেয় যয় হিক বহিদধ মান জীব ndashককর হিবাল বন মানষ যঘাা পর-হিত এরা মানকেষর যওয়া নাম শনকেল কান খাা ককের ndash নাম ধকের ডাককেল কাকে আকেস যমন - মরহিরা lsquoহিত ndashহিতrsquo ডাক শকেন আকেস াল lsquoঅ ndashর -র ডাক শকেন কাকে আকেস াহিত মাহকেতর কা শকেন চকেল ককররা মাহিলকেকর হকম পালন ককের সবসময় তাইকেতা ককরকেক পর- -কত পরানী বলা য় ককর আর হিবাল একের আওয়াজ তহিম লকষয করকেল বঝকেব ককররা যরকে যকেল lsquoযঘউ যঘউrsquo করকেত াকেক আবার কাকেল lsquoযকউ যকউrsquo ককের হিবাল সাধারণ lsquoমযাওrsquo বা lsquoহিমউrsquo ককের রা কেল lsquoওয়াওrsquo আওয়াকেজর মাধযকেম মকেনর -াব পরকাশ ককের একেতা যল পশকের কা পাহিখরাও -য় রা পরকাশ করার জনয হিবকেশষ ধরকেনর শবদ ককের হিবপকের সময় পশ পাহিখরা সবার আকে পরসপরকেক জানাবার উপায় তারা জাকেন বহকাল

১) পশপাহিখর -াষা কেলপর যলখক সমপকেকG হিক জাকেনা

উঃ- পরখযাত সাহিহিতযক উকেপনদরহিককেশার রায়কেচৌধরীর পতর সহিবনয় রায়কেচৌধরী lsquoসকেFশrsquo পহিতরকার সকেb হিতহিন কত হিকেলন তার উকেdখকোয বই lsquoসহিবনয় রায়কেচৌধরীর রচনা সংগরrsquo হিতহিন ারকেমাহিনয়াম এসরাজ পর-হিত বাযনতর বাজাকেত পারকেতন ানও জানকেতন হিতহিন যাকেIাকের জনয মজাার লপ কহিবতা হিলখকেতন

২) পশপাহিখ কেলপর মল-াব হিকউঃ- পশপাহিখকেরও -াষা আকে তারা তাকের হিনজসব -াষায় কা বকেল মকেনর -াব পরকাশ ককের পশ পাহিখরা মানকেষর হিক হিক -াষা যবাকেঝ হিকনত তারা বলকেত পাকেরনা পরসপরকেক বহিঝকেয় যবার উপায় তারা জাকেননা তকেব তারা হিবকেশষ ককেয়কটি শকেবদর মাধযকেম তাকের মকেনর -াব বহিঝকেয় যয় হিরউকেবন কযাসটং সাকেব হিতহিন চহিdশ বর বনযজনত যর সকেb যকেককেন হিতহিন বকেলকেন আমরা হি তাকের -াষা তাকের আব কায়া যমকেন চহিল তাকেল আর -কেয়র যকান কারণ াকেকনা আমরা একI -াকেলাকেবকেস যচষটা করকেল পশপাহিখকের সকেb -াব পাতাকেত পাহির

ধকের মানষ এই পশ পাহিখর -াষা হিনকেয় নানা রককেমর পরীকষা ককের আসকে এইরকম একজন হিরউকেবন কযাসটাং সাকেকেবর কা আমরা জানকেবাhelliphellip

Hindi 2nd

langमतर किनमनलिलखिखतपरशनोउRरदीजि0ए

) बढ वयलि` बचच ो कया हआ था ख) डॉकटर साहबन पाटc किस उददशय स रखी थी ग) ाल साप ो हाथ म लर लाश न कया किया घ) डॉकटर चडढा न बढ पतरो दखन स कयो मना र दिदया था ङ) भगत न लाश ो दखर कया हा

उRर ndash) उस बहत बखार थी और 4 दिदनो स आख भी नही खोला थाख) उन बट ी सालकिगरह थीग) ाल सापो हाथ म लर लाश न उसी गदन 0ोर स दबार पडी थीघ) डॉकटर चडढा न बढ वयलि` पतरो दखन स मना र दिदया कयोकि उनह गोलफ खलन 0ाना थाङ) लाश ो दखर हा कि नारायण चाहग तो आध घट म भया उठ 0ाएग

English literature

In the bazaars of Hyderabad- Sarojini Naidu

Through the poem In The Bazaars of Hyderabad Sarojini wanted to convey the message that India is rich in tradition and they donrsquot need the foreign products So she goes on to give a picture of a bazaar where traditional Indian products are rulingThe poem is in the form of questions and answers The poet asks the questions and the merchants answer them Through this technique she make the picture of the bazaar visible to us

Read the poem

PHYSICS FORCE Types of FrictionThere are three types of friction static sliding rolling Static sliding and rolling friction occur between solid surfaces

1 Static Friction The frictional force that acts between the surfaces when they are at rest with respect to each other is called Static FrictionStatic Friction Examples

Skiing against the snow Creating heat by rubbing both the hands

together Table lamp resting on the table

2 Sliding Friction The resistance that is created between any two objects when they are sliding against each other is called Sliding FrictionExamples Of Sliding Friction

Sliding of the block across the floor Two cards sliding against each other in a

deck

3 Rolling Friction The force which resists the motion of a ball or wheel is called Rolling Friction Is the weakest types of frictionExamples Of Rolling Friction

Rolling of the log on the ground Wheels of the moving vehicles

6What effect can a force produce on a body which is not allowed to move Ans - When a force is applied on a body which is not free to move it gets deformed i e the shape or size of the body changes7Give one example each to indicate that the application of a force

1 produces motion2 stops motion3 slows down motion4 changes the direction of motion5 deforms a body

Ans- 1 A car originally at rest when pushed

begins to move2 A moving bicycle is stopped by

applying the brakes3 The speed of a moving vehicle is

slowed down by applying the brakes4 A player kicks a moving football to

change its direction of motion5 On stretching a rubber string its

length increases

8State the effect produced by a force in the following cases (a) The sling of a rubber catapult is stretched(b) A man pushes a heavy cart(c) A player uses his stick to deflect the ball (d) A cyclist applies brakes(e) A spring is compressedAns- (a) The shape and size of catapult changes ie its length increases(b) The heavy cart begins to move(c) The direction of the ball changes(d) The speed of the moving cycle is slowed down(e) There is change in size and shape of spring

COMPUTER MS EXCEL 2013 -INTRODUCTION

UNDERSTANDING EXCEL STRUCTUREA SPREADSHEET IS A FILE THAT EXISTS OF CELLS IN ROWS AND COLUMNS AND CAN HELP ARRANGE CALCULATE AND SORT DATA DATA IN A SPREADSHEET CAN BE NUMERIC VALUES AS WELL AS TEXT

FORMULAS REFERENCES AND FUNCTIONS

WORKSHEETA WORKSHEET IS ALSO KNOWN AS SPREADSHEETIT IS A COLLECTION OF CELLS ON A SINGLE SHEET WHERE YOU KEEP AND CHANGE DATA

WORKBOOKWORKBOOK IS PMS EXCEL FILE IN WHICH THE DATA CAN BE STORED EACH WORKBOOK CAN CONTAIN MANY WORKSHEETS

ROWS AND COLUMNSIN MS EXCEL A ROW IS A GROUP OF CELLS THAT RUN FROM LEFT TO RIGHT OF A PAGEA COLUMN IS A GROUPING OF CELLS THAT RUN FROM THE TOP TO THE BOTTOM OF A PAGE

CELLTHE INTERSECTION POINT BETWEEN A ROW AND THE COLUMN IS CALLED A CELL WHICH IS THE BASIC STORAGE UNIT FOR DATA IN A SPREADSHEET EACH CELL HAS SPECIFIC ADDRESS WHICH IS THE COMBINATION OF THE COLUMN NAME FOLLOWED BY THE ROW NUMBER

CHEMISTRY Chapter ndash Common Laboratory Apparatus and equipments

Objective type questionFill in the blanks (a) Experiment and observation are the two important basics of chemistry(b) A porcelain dish is used for evaporation(c) A test tube holder is used to hold the test tube while-it is heated(d) Mortar and pestle is used for grinding and crushing solid substances into a powder(e) Glass apparatus is made of Pyrex or borosil glass

Class VIISubject Topic Summary Execution

Hindi 2ndlang

ए था राम( डॉ शरी परसाद)

सगकित ा परभाव मानव 0ीवन पर अवशय पडता ह

हमशा मनषय ो अचछो ी सगकित म रहना चाकिहए

शरषठ परो सग स मनषय चरिरतर ा शीघर ही उदय और किवास हो 0ाता

ह इसलिलए वयलि` ो सदा शरषठ परो ा ही सग रना चाकिहए

इसान अगर चाह वह सवय ो बदल भी सता ह

यह हानी राम ए बचच ी हवह गणिणत ी परीकषा म नल रत हए पडा 0ाता ह और उस अधयाप पडत ह और पछत ह यह कया र रह

हो तभी राम न उनी बइजजती ी

शबदाथब ndashहावा भलावाायवाहीndash ाम किनयम व ानन

ो दिदखानापरिरलिचतndash 0ाना पहचानाघटनाndashघबराहट

उलटा चोर ोतवाल ो डाटndashकिववndash भल बर ा जञानतवयndash म 0ो रना चाकिहएसगकितndash बरी सगत

किबलख नाndashरोना किनशचय रनाndash तय रना

फलndashपरिरणामकिनषालिसतndash बाहर किया हआपशचातापndashदख सपननndashधनी

ldquo हा आपी किहममत स हई नल रत पडन ीrdquo ऐसी बात ही किफर

किपता0ी न भी उस डाटा वह ाफी पशचाताप रन लगा बोला गलत दोसतो

ी सगकित म आ0 कितना अनथ र दिदया किफर उसन अधयाप स माफी मागन ी सची और किफर भी ऐसा

नही रगा यह परण भी लिलया

सोचndashकिहच एात-अला

বইndashবাংলা সাহিতয পহিরচয়

পাঠndash১৬লপndashস-য ও অস-যযলখকndashঈশবরচনদর হিবযাসারঅনশীলনীর পরকে4াততর

৬ অGকেলকেখা -ময়া = পশ হিশকার সহিtহিত = হিনকIবতu সbভরষট = লI হিনরীকষণ =

-াকেলা-াকেব যখাকতাঞজহিলপকেI = যজাাকেত৭ হিবপরীতশবদ -ঈষৎ times পরচর উৎকষট times হিনকষট তাশ times উৎফd তবহিদধ times

বহিদধীNপাহিপষঠ times পণযবান৮ পপহিরবতG ন ককেরা -পশ = পাশহিবক যকাপ = যকাহিপতহিসথর = হিসথরতাএকানত = ঐকাহিনতক পর-াত times পর-াতী

CHEMISTRY

Chapter ndashPhysical and Chemical Changes

Chemical ChangeA chemical change involves a change in chemical composition

Characteristics of Chemical changes 1 They are permanent changes2 They are irreversible changes 3 New substance formed4 A Chemical change involves a

change in its chemical properties

Pg-25Question 8What do you observe when1 water is boiled2 a piece of paper is burnt3 some ice cubes are kept in a glass tumbler4 solid ammonium chloride is heated5 an iron nail is kept in tap water for few days6 a spoon of sugar is heated in a pan7 lighted match stick is brought near the mouth of the test tube containing hydrogen gas8 quick lime is dissolved in water9 little amount of curd is added to a bowl containing warm milk and kept for five hours

10 Water is boiledOn boiling water changes into steam (gas) physical change

11 A piece of paper is burnton burning piece of paper produces carbon dioxide and ash is left behind Is a chemical change

12 some ice cubes are kept in a glass tumblerIce cubes (solid) turn into water

(liquid) only state changes (physical change)

13 Solid ammonium chloride is heatedSolid ammonium chloride on heating changes into vapors (change of state) is physical change

14 An iron nail is kept in tap water for few dayswe observe reddish brown coating on the nail called rust (entirely new substance) is chemical change

15 A spoon of sugar is heated in a panWhen a spoon of sugar is heated in a pan black (charred sugar) (carbon) is seen Is a chemical change

16 Lighted match stick is brought near the mouth of the test tube containing hydrogen gasWe observe that hydrogen bums at the mouth of test tube with blue flame and pop sound is heard It is chemical change

17 Quick lime is dissolved in waterThe following two observations will be observed (i) A hissing sound is observed(ii) The mixture starts boiling and lime water is obtained

18 Little amount of curd is added to a bowl containing warm milk and kept for five hoursWhen a little amount curd is added to a bowl containing warm milk and kept for five hours a permanent change occurredThe milk will change to curd On boiling water changes into steam (gas) physical change

GEOGRAPHY

ATMOSPHERE IMPACT OF GLOBAL WARMING The destructive impart of global warming is observed in various spheres of life and the environment Some of the points are outlined below1 High temperatures lead to high

evaporation rate and drying up of the soil and surface water This affects crop production The occurrence of droughts is aggravating the problem even further

2 The heat waves in summer months

Q1 Write some impact of global warmingA1 The impacts of global warming are as follows1 High temperatures lead to high

evaporate ion rate and drying up of the soil and surface water This affects crop production The occurrence of droughts is aggravating the problem even further

2 The heat waves in summer months lead to a greater number

lead to a greater number of deaths due to heat strokes

3 Forest fires become more frequent4 Tropical cyclones and hurricanes

become common5 Melting of glaciers takes place6 Polar ice caps are becoming thinner

and melting at an alarming rate due to global warming The loss of sea ice

7 Due to increase in sea surface temperature sea levels rise in coastal areas and cause submergence of several islands

WAYS TO REDUCE GLOBAL WARMINGFollowing steps can be taken We need to decrease emission of

green house gases by reducing the burning of fossil fuel such as coal and petroleum

By planting more trees to increase forest cover

The government should also distributes free saplings and organize afforestation programmes to spread awareness regarding the beneficial effects of trees

We should switch to eco-friendly cars and gadgets

Incandescent light bulbs should be replaced by CFL bulbs

We can save electricity and reduce global warming by turning off electrical gadgets such as lights fans air-conditioners television and computer when we do not to use them

Efforts should be made to hasten the development of green cities oreco cities These cities are urban areas around the world striving to lessen the environment a impacts of urbanization

By following the 3Rs-Reduce Recycle and Reuse strategy we can use natural resources for our growth as well as save them for the need of the future generations This is called sustainable development

of deaths due to heat strokes3 Forest fires become more

frequent4 Tropical cyclones and hurricanes

become common5 Melting of glaciers takes place

etc

Q2 How to reduce global warmingA2 Following steps can be taken to reduce global warmingaWe need to decrease emission of

green house gases by reducing the burning of fossil fuel such as coal and petroleum

bBy planting more trees to increase forest cover

c The government should also distributes free saplings and organize afforestation programmes to spread awareness regarding the beneficial effects of trees

dWe should witch to eco-friendly cars and gadgets

eIncandescent light bulbs should be replaced by CFL bulbs

f We can save electricity and reduce global warming by turning off electrical gadgets such as lights fans air-conditioners television and computer when we do not to use them

Q3 What do you mean by 3Rrsquos of resource planningA3 The 3Rs are

1 Reduce 2 Recycle and3 Reuse

Q4 What is Sustainable developmentA4 By following the 3Rs-Reluce Recycle and Reuse strategy we can use natural resources for our growth as well as save them for the need of the future generations This is called sustainable development

English Language

Prepositions A preposition is a word placed before a noun or a pronoun It helps to show how the person or thing denoted by the noun is related to something else in the sentence

Kinds of Prepositions

Simple Prepositions- simple preposition are one word Prepositions such as at by for in of off for from on out through till to up with before amidst towards beyond between over etc

Compound Prepositions ndash There are some words that are always used with fixed Prepositions to convey specific meaning

Example I was unable to meet you dueto a previous engagement ( On account of)Always maintain the queue instead of crowding at the counter ( In place of)

Participial PrepositionsmdashParticiple Prepositions are present or past participles of various verbs which together with a noun phrase or a clause function as prepositions Examples- barring concerning considering notwithstanding pending regarding respecting etc

Exercise A

1 Gauravs fever has come down since Friday He has been absent for a week now

2 The child sat between his father and mother among the parents of all his classmates

3 There are mosquitoes in the room They flew into the room when the door was open

4 My father was inside the drawing room when I was playing outside my house

5 You may sit beside me I will give you a drawing book and pencils besides a storybook

6 We went to the market in the morning and walked towards the riverfront in the evening

7 The child walked along the pavement and across the street safely

8 This table top is made of glass My breakfast fell off it in the morning

9 The pan is on the gas stove There are vegetables in it

10 We will wait for you at the bus top There are a lot of people in the hall

Subject ndash Biology Topic ndash Chapter - 3 Photosynthesis and respiration in plants Summary Execution

All living organism (Plants and animals) need food for energy and growth Green plants (autotrophy) prepare food for all living organisms Today we will discuss about the process photosynthesis And adaptations in a leaf to carry out photosynthesis

Q1What do you mean by photosynthesis and write its word equation The process by which green plants make food (glucose) from carbon dioxide and water

in the presence of sunlight and chlorophyll is called photosynthesis

Carbon dioxide + Water ( Sun light from Sun ) Glucose + Oxygen ( chlorophyll in green leaves )

Q2 What are the adaptations in a leaf to carry out photosynthesisi) Leaves are broad wide and flat for absorbing more light energyii) Presence of chlorophyll in chloroplasts to trap sunlightiii) Presence of stomata which allow carbon dioxide to enter the cell and oxygen to go

out iv) Network of veins ensures continuous supply of water and minerals to the leafv) Thin waxy cuticle protects the leaf without blocking the lightQ3 Draw and label structure of chloroplast

Class VIIISubject Topic Summary Execution

PHYSICS ENERGY Production of Hydro electricity

A hydroelectric dam converts the potential energy stored in a water reservoir behind a dam to mechanical energymdashmechanical energy is also known as kinetic energy As the water flows down through the dam its kinetic energy is used to turn a turbine

The generator converts the turbinersquos mechanical energy into electricity

This electric energy then goes through various transmission processes before it reaches you

Question 2

Fill in the blanks

(a) Work is said to be done by a forte only when the body moves

(b) Work done = Force x distance moved in direction of force

(c) The energy of a body is its capacity to do work

(d) The SI unit of energy is joule

(e) The potential energy is due to its state rest of position and kinetic energy of the body is due to its state of motion

(f) Gravitational potential energy U = mass times force of gravity on unit mass times height

(g) Kinetic energy = frac12 times mass times (speed)2

(h) Power P = work donetime taken

(i) The S I unit of power is watt

(j) IHP = 746 W

BIOLOGY Chapter -5 The endocrine system and adolescence

Today we will discuss about thelocation and functions of secreted hormones of adrenal and Pancreas

Q5 Write location hormone secreted main functions and deficiency diseases of pancreas and adrenal glands

Endocrine Glands

Location Hormones secreted

Functions and Deficiency Diseases

1Adrenal gland

2 Pancreas Gland

On the top of each kidney

In between stomach and small intestine

i)Adrenaline from adrenal medulla

ii)Cortisone from adrenal cortex

i) Insulin

ii) Glucagon

It helps a person deal with any kind of emergency situation or emotional stressIt increases the heart beat rate of respiration and blood pressure

a) It regulates carbohydrates protein and fat metabolism

b) It regulates the salt and water balance in the body

a) It changes excess glucose into glycogen

b) It stimulates the cells to burn extra glucose to provide heat amp energy

Less secretion causes diabetes mellitus

Excessive secretions causeinsulin shock

a) It stimulates the breakdown of glycogen into glucose

b) It increases the level of glucose in blood

History Traders to rulers The Battle of Buxar was fought on 22 October 1764 between the forces under the command of the British East India Company led by Hector Munro and the combined armies of Mir Qasim the Nawab of Bengal till 1763 Mir Jafar was made the Nawab of Bengal for a second time in 1763 by the Company just after the battle After being defeated in 4 battles in katwa and Udaynala the Nawab of Awadh Siraj id Daula and the Mughal emperor Shah Alam II accompanied by Raja Balwant Singh of Kashi made an alliance with Mir Qasim The battle was fought at Buxar a small fortified

Answer the following questions- Short note-Battle of BuxarHomework-learn

town within the territory of Bihar located on the banks of the Ganga river about 130 kilometres (81 mi) west of Patna it was a decisive victory for the British East India Company The war was brought to an end by the Treaty of Allahabad in 1765

EnglishLiterature

The west wind-John Mansfield

In the poem The West Wind by John Masefield the poet starts by describingwith very poetic imagery of birds how the west wind is different from other winds its a warm wind full of birds cries There is a touch of melancholy perhaps home-sickness as he describes how it brings tears too and memories from an old land He goes on to describe the restful pastoral beauty of the land where even the dead can lie in the green He then brings in voicesperhaps of family and friends calling him home as he is missing Aprils beautyThe voices then tempt him some more with idyllic images from home (white blossom young green cornrunning rabbitswarm sun) The voices seem to presume that the poets heart is sorrowful bruised and soreThe end of the poem sees the poet appear to make a decision he will go home as he has decided that is where he truly belongs

Write the synopsis of the following words

1 Daffodils- a tall yellow flower that grows in the spring

2 Orchards- a piece of land on which fruit trees are grown

3 Blossom- a flower or a mass of flowers especially on a fruit tree in spring

4 Thrushes- a bird5 Larks- a small brown bird that

makes a pleasant sound6 Bruised- an injury7 Aching- pain 8 Tread- to put your foot down

while you are walking9 Balm-10 May-11 Fluting-

(Write from the book in your copy)

MAT

HEM

ATIC

S

Ch 1

1Al

gebr

ic E

xpre

ssio

n

1 Constant A symbol which has fixed value is called a constant[eg 8 23 -15 radic3 etc]

2 VariableA symbol which does not have any fixed value but may be assigned value (values) according to the requirement is called variable or literal[eg x y p q etc]

3 TermsA term is a number (constant) a variable a combination (product or quotient) of numbers and variables[eg 7 x 5x etc]

4 Algebric expressionA single term or acombination of two or more terms connected by plus (+) or minus (-) sign forms an algebraic expression[eg 5-y 3x2-5x xy-6z+4 etc]

5 PolynomialAn algebraic expression which contains more than one term is called a polynomial (multinomial)[eg x2-5x 5y+xy+x2y etc]

6 Degree of polynomial(a) When the polynomial contains only one variable the highest power of the variable is the degree of the polynomialeg the degree of the polynomial of 4x-7x5+8 is 5(b) When the polynomial contains two or more variablesStep (i) Find the powers of the variables in each term (ii) The highest sum of the powers is taken to be the degree of the polynomialeg the degree of the polynomial 5x2y-4x3y5+6 is = 3+5 = 8Remember An algebraic expression is a polynomial if degree of each term used in it is a non-negative integer

Exercise ndash 11(A)

1 Separate the constants and variables from the following

-7 7+x 7x+yz radic5 radic xy 3 yz

8 45y -3x

Solution Constant Variables-7 radic5 7+x 7x+yz radic xy

3 yz8

45y -3x

2 Write the number of terms in each of the following polynomials(i) 5x2+3timesax (ii) axdivide4-7 (iii) ax-by+ytimesz (iv) 23+atimesbdivide2

Solution Polynomials Number of terms(i) 5x2+3timesax 2(ii) axdivide4-7 2(iii) ax-by+ytimesz 3(iv) 23+atimesbdivide2 2

4 Write the degree of the each polynomials(i) xy+7z (ii) x2-6x3+8 (iii) y-6y2+5y8 (iv) xyz-3 (vi) x5y7-8x3y8+10x4y4z4

Solution Polynomials Degree(i) xy+7z 2(ii) x2-6x3+8 3(iii) y-6y2+5y8 8(iv) xyz-3 3(vi)x5y7-8x3y8+10x4y4z4 12

5Write the coefficient of(i) ab in 7abx (iv) 8 in a2-8ax+a (v) 4xy in x2-4xy+y2

SolutionCoefficient

(i) ab in 7abx 7x(iv) 8 in a2-8ax+a -ax(v) 4xy in x2-4xy+y2 -1

7 CoefficientAny factor of an algebraic quantity is called the coefficient of the remaining quantityeg in the algebraic term 7xyz 7 is coefficient of xyz 7x is coefficient of yz and so on

8 Like term The terms having the same literal coefficient are called like terms and those having different literal coefficients are called unlike terms

eg (i) 5xyz 8xyz -6xyz and 23xyz are like

terms(ii) 7xy2 8x2yz and -15xyz2 are unlike terms

6 in 57xy2z3 write the coefficient of

(i) 5 (vii) 5xy2 (viii) 17yz (xi) 5xyz

Solution Coefficient

(i) 5 17

xy2z3

(vii) 5xy2 17z3

(viii) 17yz

5xyzsup2

(xi) 5xyz 17yz2

7 In polynomial given below separate the like terms(ii) y2z3 xy2z3 -58x2yz -4y2z3 -8xz3y2 3x2yz and 2z3y2

Solution y2z3 -4y2z3 2z3y2 are like terms

xy2z3 -8xz3y2 are like terms

-58x2yz 3x2yz are like terms

Class IXSubject Topic Summary Execution

Bengali (2nd language)

বাগzwnjধারাzwnj বা ধারা-বা ধারা ল হিবকেশষ পরকার বাক -হিb -াকেবর এক হিবকেশষ পরকাশরীহিত াকেক কতগকেলা কার সমষটির মকেধয এগহিলকেক বা ধারা বকেল আবার কতগকেলা শকেবদর বাধাধরা যকান রীহিত যনই য-াকেব চকেল আসকে যসই -াকেবই চকেল আসকে তখন যসই শবদগহিল খন একক -াকেব অG পরকাশ ককের তখন একের বা ধারা বকেল বা ধারার পরকেয়া -াষাকেক আরও সFর ককের যতাকেল

অকাল পকক(অপহিরনত বয়কেস পাকাহিম)-মাতর শ বর বয়কেস যমকেয়টির া মকেখর কা তাকেত অকালপককতা ধরা পকে

অককা পাওয়া( মারা াওয়া) ndash পকেকIমারটি পকেকIমারকেত হিকেয় বাসাতরীকের াকেত মার যখকেত যখকেত অককা যপল

অহি| পরীকষা ( কঠিন ও পরকত পরীকষা)- যকেলটির আজ ডাকতাহির যরজালট যবকেরাকেব এIাই তার জীবকেনর ব অহি| পরীকষা

অষটরমভা (ফাহিক) ndash রীতা মকেখই বকো বকো কা বকেল আর কাকেজর যবলায় অষটরমভা

অকমGার ধাী (অপাG) ndash সমনকেক হিনকেয় যকান ান কেব না ও একেকবাকেরই অকমGার ধাী

অকেনধর ষটি (অসাকেয়র সায়)- আহিশ বকেরর বকোর নাহিত ল অকেনধর ষটি তাকেক াা বকোর একম চকেল না

আকেককল গড়ম (তবহিদধ)- ার তহিম উপকার করকেল যসই যতামার হিবরকেদধ সাকষয হিকেয়কে শকেনই আমার আকেককল গড়ম

আষাকে লপ( অবাসতব লপ) ndashIাকা এখন যকেব না এIা বলকেলই ত এমন আষাকে লপ ফাার যকান রকার হিল না

Hindi- महायजञ ा इस हानी म लख न या बतान ा परयास किया ह कि किसी भी अचछ

2nd language

परसार(यशपाल ाय या पणय न ा फल अवशय मिमलता ह ोई भी परोपार अथवा पणय लिलए किया गया ाय बार नही 0ाता वह ए परार ा यजञ हए धनी सठ थ धम परायण और किवनमर सठ न आन ी यजञ किए थ और दान म न 0ान कितना धन दिदन दखिखयो म बात दिदया थादिदन पलट और सठ यहा गरीबी आ गई उन दिदनो यजञ बचन ी परथा थी सठ भी अपनी 0गह बचन लिलए डलपर ए सट यहा चलन ो तयार हए सठानी रासत लिलए रोटी पड म बाधर सठ ो द दी रासत म ए भख R ो दखर सठ न चारो रोटी उसो खिखला दी खर वह सठ यहा डलपर पहच तो उनी सठानी न उस महायजञ बचन ो हा यदिद बचन आए सठ न R ो रोटी खिखलान ो महायजञ नही समझा और वापस लौट आया घर आर शाम ो उसी घर म उस ए बडा ख0ाना मिमला 0ो उस दवारा किए गएrsquo महायजञrsquo ा परसार था

English language

Letter formal The heading the name and address of the person you are writing to must be included beneath your own address In formal letters ldquoblock stylerdquo of address is preferred

Subject complain in brief

Salutation If the person you are writing to is known to you you may begin ldquoDear MrrdquoOr ldquoDear Mrsrdquo In all other instances you should begin ldquoDear Sirrdquo or ldquoDear Madamrdquo Or ldquoSirsrdquo

The body A formal or business letter has four partsReference The letter should begin by referring to a letter you have received an advertisement or the reason that has prompted you to writeInformation In the second paragraph it is necessary to supply more detailed information that is related to the referencePurpose Here you must give the reason why you are writing the letter This must be stated clearly and ensure that it is relevant to the question that has been setConclusion round off the letter with some polite remarkThe subscription when a letter has begun with dear sir sirs Madam you should end with Yours faithfully or yours truly When however you address a person by name you must conclude with the words ldquoYours sincerelyrdquo

1 A park in your locality is slowly being used as a rubbish dump Write a letter to the Mayor of your city pointing out the nuisance and danger of this Request that action be taken to stop this immediately

Or2 You being a boarder ordered a set of lab manuals from a famous book shop in the town They sent you a wrong set of books Write a letter to the manager of the book shop

Chemistry Chapter-1 1)CHEMICAL FORMULA- Q What is the Significance of

L-2The Language of Chemistrybull Chemical Formula

Itrsquos a symbolic representation of a chemical substance eg ndash The formula of Sulphuric acid is H2SO4

2) Steps of writing Chemical Formula of a given substance-

1 Write the symbols of the constituent atoms or radicals side by side Keep the basic radical on LHS and acid radical on the RHS ( Na+Cl- )2 In case of a radical having more than one atom( compound radical) enclose the radical in a bracket eg (SO4-)3 Write the valencies of each radical on its right hand top4 If the valencies of the two radicals are divisible by a common factor then divide the valencies by the common factor5 Invert (criss-cross) the valency number ie write the valency of one atom below the second atom and vice versa 6 On interchanging if valency number is lsquoone the figure lsquoonersquo is never writtenFor Example- Compound -Calcium Nitrate1 Writing the symbols- Ca(NO3)2 Writing the valencies on their right hand top- Ca2(NO3)1

3 Valency numeral in simple ratio- Ca2(NO3)1

4 Criss-cross- Ca 2NO3 1

5 Writing the formula of the compound- Ca(NO3)2

Chemical formula

A The formula of a substance conveys the following information regarding a substance 1 The name of the substance (qualitative)2 The elements constituting the substance (qualitative)3 The number of various atoms present in a molecule of the substance (quantitative)4 Molecular weight of the substance and the relative weights of different elements present in it (qualitative)

Q What are the limitations of Chemical Formula

A The chemical formula suffers from the following limitations-I It fails to convey whether the elements in a molecule are present in the form of atoms or ionsFor example the formula KBr fails to tell us whether Potassium and Bromine are present in the form of ions II It does not tell anything about the binding force that holds atom in a molecule togetherIII It does not tell us about the arrangement of various atoms with respect to one another within the molecule

Q Examples of Some Chemicals with their Formula Chemical name and Common Name-

A Given in the class notesCommercial Studies

Joint Stock Company

Let us discuss about the demerits of Joint Stock CompanyDespite so many advantages it has got many disadvantages which are as follows

Difficulty in FormationDelay in Decision makingExcessive Government ControlLack of Secrecy

Company can be classified into several categories based on incorporation

QuestionExplain the demerits of Joint Stock CompanyAnswer) 1 Difficulty in Formation The legal requirements and formalities required to be completed are so many The cost involved is quite heavy It has to approach large number of people for its capital It cannot start its business unless certificate of incorporation has been obtained This is granted after a long time when all the formalities are completed

Chartered CompanyStatutory CompanyRegistered Company

Delay in Decision making In this form of organization decisions are not made by single individual All important decisions are taken by the Board of Directors Decision-making process is time-consuming So many opportunities may be costly because of delay in decision-making Promptness of decisions which is a common feature of sole trader ship and partnership is not found in a company

Excessive Government ControlA company and the management have to function well within the law and the provisions of Companies Act are quite elaborate and complex At every step it is necessary to comply with its provisions lest the company and the management should be penalized The penalties are quite heavy and in several cases officers in default can be punished with imprisonment This hampers the proper functioning of the company

Lack of Secrecy The management of companies remains in the hands of many persons Every important thing is discussed in the meetings of Board of Directors Hence secrets of the business cannot be maintained In case of sole proprietorship and partnership forms of organisation such secrecy is possible because a few persons are involved in the management

2 Define the following

Chartered Company- The crown in exercise of the royal prerogative has power to create a corporation by the grant of a charter to persons assenting to be incorporated Such companies or corporations are known as chartered companies Examples of this type of companies are Bank of England (1694) East India Company (1600) The powers and the nature of business of a chartered company are defined by the charter which incorporates it After the country attained independence these types of companies do not exist

in IndiaStatutory Company- A company may be incorporated by means of a special Act of the Parliament or any state legislature Such companies are called statutory companies Instances of statutory companies in India are Reserve Bank of India the Life Insurance Corporation of India the Food Corporation of India etc The provisions of the Companies Act 1956 apply to statutory companies except where the said provisions are inconsistent with the provisions of the Act creating them Statutory companies are mostly invested with compulsory powersRegistered companiesCompanies registered under the Companies Act 1956 or earlier Companies Acts are called registered companies Such companies come into existence when they are registered under the Companies Act and a certificate of incorporation is granted to them by the Registrar

Economics

Chapter-4Basic problems of Economy

Today let us discuss with the topic Production Possibility curve

QuestionExplain the concept of Production Possibility Curve with the help of diagram

Answer) Production Possibility curve is a locus of all possible combinations of two commodities which can be produced in a country with its given resources and technology

The above diagram shows that with the given resources and technology the economy can produce maximum either 5 thousand meters of cloth or 15 thousand quintals of wheat or any other combination of the two goods like B( 1 thousand meters of cloth and 14 thousand quintals of wheat C ( 2 thousands meters of cloth and 12 thousand quintals of wheat) etcProduction Possibility curve is also called production possibility boundary or frontier as it sets the maximum limit of what it is possible to produce with given resources

Geography

Rotationand Revolution

SUNrsquoS POSITION AND SEASONAL CHANGES EQUINOXES ndash SPRING AND AUTUMN

Q1 What is Spring EquinoxA1 On 21st March sunrays fall directly on the equator On that day

As the Equator divides the Earth into two equal halves the sun rays fall directly on the equator twice in a year Equinoxes means equal Spring EquinoxOn 21st March sunrays fall directly on the equator On that day the duration of day and night both are equal ( 12 hours day and 12 hours night) on every places located on equator This day is called as Spring EquinoxAutumn EquinoxOn 23rd September sunrays fall directly on the equator On that day the duration of day and night both are equal ( 12 hours day and 12 hours night) on every places located on equator This day is called as Autumn Equinox

SOLSTICES ndash SUMMER AND WINTERDue to inclination of the Earth on its axis and the apparent movement of the sun the sun rays fall directly on both tropics once in a year Solstice is a Latin word which mean ldquothe Sun standing stillrdquoSummer SolsticesAfter 21st March there is an apparent movement of the Sun to the north of the equator The apparent northward movement up to 21st June when the Sun appears overhead at the Tropic of Cancer (22frac12degN) The sun appears to stand still at this position and then moves southwards towards the equator This position of the Sun on 21st June is known as Summer Solstices On that day the duration of day and night both are equal ( 12 hours day and 12 hours night) on every places located on Tropic of Cancer (22frac12degN)Winter solstices The apparent southward movement of the Sun continues beyond the equator till 22nd

December On this day the Sun is overhead at the Tropic of Capricorn

the duration of day and night both are equal ( 12 hours day and 12 hours night) on every places located on equator This day is called as Spring Equinox

Q2 What do you mean by EquinoxA2 Equinoxes means equal It is use to explain the equal duration of day and night ( 12 hours day and 12 hours night) on the Earth

Q3 On which date the longest day in Tropic of CancerA3 21st June

Q4 What is the meaning of SolsticeA4 Solstice is a Latin word which mean ldquothe Sun standing stillrdquo

Q5 Which is the longest day in southern hemisphereA5 22nd December

Q6 On what date does the Arctic Circle experience the lsquoMidnight SunrsquoA6 On 21 June the Arctic Circle experiences the lsquoMidnight Sunrsquo

Q7 What is cause of Midnight Sun in NorwayA7 During the summer solstice (21 June) the North Pole is inclined towards the Sun Therefore the duration of sunlight or daytime increases from 12 hours at the Equator to 24 hours at the Arctic Circle and beyond Thatrsquos why The region beyond the Arctic Circle especially Norway is known as the Land of the Midnight Sun because there the Sun does not rise or set on 21 June

Q8 Match the column A with BA B

Summer Solstice 21st March

Autumn Equinox 23rd

September

Winter Solstice 21st June

(22frac12degS) This position of the Sun is referred to as the Winter Solstice because it marks the winter season in the Northern Hemisphere On that day the duration of day and night both are equal ( 12 hours day and 12 hours night) on every places located on Tropic of Capricorn (22frac12degS)SEASONS AND DURATION OF DAY AND NIGHT During the equinoxes all places on the Earth have 12 hours of day and 12 hours of night Due to the revolution of the Earth round the Sun on an inclined axis the duration of day and night varies according to seasons and the latitude of a placeDuring the summer solstice (21 June) the North Pole is inclined towards the Sun Therefore the duration of sunlight or daytime increases from 12 hours at the Equator to 24 hours at the Arctic Circle and beyondThe region beyond the Arctic Circle especially Norway is known as the Land of the Midnight Sun because there the Sun does not rise or set on 21 JuneAt the North Pole there will be six months of daylight The Sun will be seen always above the horizon at a low angle At 66degN 24 hours of sunlight can be seen only on 21 June Hammerfest in northern Norway is a place of tourist attraction for observing the phenomenon of the Midnight Sun This place has continuous daylight from 13 May to 29 July This place is easily accessible to tourists and has hotels and other facilities The view of the midnight Sun from here is enthrallingIn the Southern Hemisphere the duration of daylight decreases from 12 hours at the equator to 0 hours beyond the Antarctic Circle In the South Polar Region there is 24 hours of darkness The Sun is always below the horizon In the Southern Hemisphere which experiences winter the duration of night-time is longer than the duration of daylight

Spring Equinox 22nd

December

A8 A B

Summer Solstice 21st June

Autumn Equinox 23rd

September

Winter Solstice 22nd

December

Spring Equinox 21st March

During winter solstice (22 December) the South Pole is inclined towards the Sun The Southern Hemisphere experiences summer and the Northern Hemisphere has winter Therefore the duration of daylight or sunlight is greater in the Southern Hemisphere than in the Northern HemisphereThe duration of daylight increases from 12 hours at the equator to 24 hours beyond the Antarctic Circle The South Polar Region has 24 hours of sunlight for many days continuously At the South Pole there will be six months of sunlight The Sun will always be seen at a low angle above the horizon In the Northern Hemisphere the duration of daylight will decrease from 12 hours at the equator to 0 hours at the Arctic Circle There are 24 hours of darkness in the North Polar region The duration of night is greater than the duration of daylight as one move northwards from the Equator It is evident from the above table that the duration of daylight is 12 hours throughout the year at the equator only As one moves away from the equator the seasonal variations in the duration of daylight increase The seasonal variations in the duration of daylight are maximum at the Polar Regions

Subject Eng Literature (The Merchant of Venice ndash William Shakespeare)Topic Act II Scene 7 Lines 36 to 80 (End of scene ) [Students should read the original play and also the paraphrase provided]

Summary Questions amp AnswersThe Prince then examines the inscription on the silver casket which says ldquoWho chooseth me shall get as much as he deservesrdquo The Prince says that he deserves Portia more than anybody else because of his high rank his noble birth and his great wealth and power But then he argues that silver is ten times

(1) (Act II Sc 7 L 39-47)

From the four corners of the earth they come

To kiss this shrine this mortal breathing saint

The Hyrcanian deserts and the vasty wildsOf wide Arabia are as through-fares now

inferior to gold and therefore he cannot believe that the portrait of such a beautiful lady as Portia can be contained in the silver casket He decides to see the inscription on the golden casket before making his decision

The Prince goes to examine the inscription on the golden casket which says ldquoWho chooseth me shall get what many men desirerdquo The Prince believes that the whole world desires to possess Portia otherwise so many suitors would not have come from all corners of the world for winning Portia Some of them have come from the distant lands of Persia and Arabia The deserts of Persia (Hyrcanian deserts) and the boundless desolate lands of Arabia have been crossed by the Princes seeking the hand of Portia He contrasts this casket containing Portiarsquos portrait with the old English gold coin bearing the image of the archangel (angel of the highest rank) He goes on to remark that while the figure of the archangel is engraved (Insculped) upon the English coin the picture of Portia who is beautiful as an angel lies hidden inside one of the caskets namely the Golden Casket (Golden Bed)

On the basis of his assessment of the inscription on the golden casket the Prince decides to choose the golden casket He asks for the key and opens the golden casket only to find therein an empty human skull holding a roll of

For princes to come view fair PortiaThe watery kingdom whose ambitious headSpets in the face of heaven is no barTo stop the foreign spirits but they comeAs orsquoer a brook to see fair Portia

(i) Explain the occasion for the above mentioned speech

These are the comments of the Prince of Morocco after he reads the inscription on the golden casket His mental process is revealed to us in these words We find him debating within himself as to which casket he should choose

(ii) What light does the above speech throw on the personality of Prince of Morocco

From the above mentioned speech we come to know that the Prince of Morocco is keen to marry Portia He is the type of person who is easily taken away by outward appearance He is in love with Portia because of her beauty

(iii) What information can you gather about Portia from the above mentioned lines

The given speech shows that Portia is a very beautiful lady She must be possessed of good qualities because many suitors come to her place from all over the world with a desire to get married to her The Prince of Morocco is so impressed by her beauty that he calls her a saint According to him the whole world is desirous of having her

(iv) Elucidate the significance of the first two lines

In these lines the Prince of Morocco pays a compliment to Portia These lines show his admiration for her He says that people come from all parts of the world to see fair Portia

(v) Explain the meaning of the last four lines of the

passage

In these lines the Prince of Morocco says that even the vast oceans which throw a challenge at the sky are unable to prevent men from coming to Portiarsquos place to have a glimpse of her These lines are also a tribute to Portiarsquos beauty and good qualities Many men voyage across the ocean treating it as a mere stream to see the beautiful Portia

paper in which is written that whoever happens to be guided by the glitter of things is invariably deceived

On reading the scroll the Prince says that he is too sad at heart to speak a more formal farewell and leaves with his followers amidst a sound of trumpets

After the Prince of Morocco leaves Portia remarks that the Prince is a gentle fellow but she is rid of him May all persons of his nature make a similar choice

IMPORTANT PASSAGES EXPLAINED

(Act II Sc 7 L 39-43)From the four corners of the earth they come

To kiss this shrine this mortal breathing saintThe Hyrcanian deserts and the vasty wildsOf wide Arabia are as through-fares nowFor princes to come view fair Portia

Context

This passage occurs in Act II Scene 7 in The Merchant of Venice This is part of the speech made by the Prince of Morocco

(2)

(Act II Sc 7 L 48-53)

MOROCCO One of these three contains her heavenly pictureIst like that lead contains her

Twere damnation To think so base a thought it were too grossTo rib her cerecloth in the obscure graveOr shall I think in silver shes immurdBeing ten times undervalued to tried gold

(i) What meaning does the Prince of Morocco find out of the inscription of the golden casket What have Belmont and Portiarsquos house been called and why

The inscription on the golden casket is ldquoWho chooseth me shall gain what many men desirerdquo The Prince finds out that it means that the chooser of the golden casket will get Portia because many men desire her In fact the entire world desires her Because of the coming of many suitors to Belmont from different countries in order to win Portiarsquos hand Belmont has become a centre of pilgrimage and her house is the shrine where saintly Portia is installed

(ii) What does the Prince of Morocco do before making the final choice of the casket Which is the correct casket and who will win Portiarsquos hand

The Prince of Morocco surveys and analyses the inscriptions on the casket of lead silver and gold Before making the final choice like a very systematic and methodical person he once again considers the claims of the caskets The casket containing Portiarsquos picture is the correct casket and the person choosing it will win Portiarsquos hand

Explanation

While praising Portia the Prince of Morocco conceives Portia as a goddess whose image is placed inside one of the caskets Many suitors are coming from far and wide the north and the south the east and the west (Four corners) in order to try their luck Some of them have come from the distant land of Persia and Arabia The deserts of Persia (Hyrcanian deserts) and the boundless desolate lands of Arabia have been crossed by the Princes seeking the hand of Portia All this shows that Portia is indeed the most beautiful lady of the world

(iii) What does the Prince of Morocco say in his estimation while examining the motto on the silver casket What does he find in the golden casket

While examining the motto on the silver casket which says ldquoWho chooseth me shall get as much as he deservesrdquo Morocco says that in his own estimation he surely deserves Portia in all respects ndash rank birth wealth etc

He chooses the golden casket When he opens it he finds an empty human skull holding a scroll in which it is written that those who are attracted by the glittering outside of things are always deceived as Morocco has been deceived

(iv) What kind of nature does the Prince of Morocco have

The Prince of Morocco has a simple nature who does not look deeply into the inner meaning of things but is dazzled by the outward appearance of gold He is inclined to over-estimate his own value and does not realize that it is a duty to ldquogive and hazardrdquo To say that he will not hazard for lead shows that he misreads the true meaning of the inscription which is that he should be prepared to ldquohazard all he hathrdquo for Portia So his feeling is only one of fascination and romantic attraction

(v) Do you think that the lottery of the caskets is not a matter that will be determined by chance

In fact the lottery of the casket is not a matter that will be determined by mere chance but that it is a true test of character and of sincerity which is amply proved not only by Moroccorsquos choice but also by the arguments which he uses to help him in his choice

(Act II Sc 7 L 55-59)

They have in England

A coin that bears the figure of an angelStamped in gold but thats insculpd uponBut here an angel in a golden bedLies all within

Context

(3)

(Act II Sc 7 L 63-77)A carrion Death within whose empty eye

There is a written scroll Ill read the writing

All that glisters is not goldOften have you heard that toldMany a man his life hath soldBut my outside to beholdGilded tombs do worms infoldHad you been as wise as boldYoung in limbs in judgment oldYour answer had not been inscrolld

This passage occurs in Act II Scene 7 in The Merchant of Venice This is part of the speech made by the Prince of Morocco

Explanation

In this passage the Prince of Morocco bestows high praise on Portia whose hand he is seeking He contrasts this casket containing Portiarsquos portrait with the old English gold coin bearing the image of the archangel (angel of the highest rank) He goes on to remark that while the figure of the archangel is engraved (Insculped) upon the English coin the picture of Portia who is beautiful as an angel lies hidden inside one of the caskets namely the Golden Casket (Golden Bed) In the day of Elizabeth silver was ten times inferior in value to gold Therefore the Prince of Morocco believing that Portiarsquos portrait is contained in the Golden Casket decides to choose the Golden Casket

Fare you well your suit is coldCold indeed and labour lostThen farewell heat and welcome frostmdashPortia adieu I have too grievd a heartTo take a tedious leave Thus losers part

(i) What reward does the Prince of Morocco get after making a wrong choice of the Casket How does he feel

After making the wrong choice in selecting the casket of gold the Prince of Morocco as a reward earns a rebuke in the form of a scroll tucked in the empty eye-socket of a skull kept in the casket of gold The Prince is shocked and disappointed He becomes all the more sad and dejected when he reads the scroll which points to his foolishness in being misled by the appearance and outward show as indicative of its worth

(ii) How does the Prince respond after reading the scroll

After reading the scroll the Prince though upset accepts the result with good grace and decorum befitting a royal suitor and true sportsman He says that his love-suit is really cold otherwise he would have chosen correctly but now his efforts have been in vain So he bids farewell to Portia to the warmth and enthusiasm of love and welcomes the cold and bitterness of dejection and misery of life which lies ahead

(iii) What request does he make to Portia and why

After being failure in his mission he requests Portia to give him permission to leave at once because he is too sad to undergo the tediousness of a formal leave-taking He tells that it is the manner in which defeated persons part unceremoniously

(iv) Explain the following lines

ldquoAll that glisters is not goldOften have you heard that toldMany a man his life hath soldBut my outside to beholdGilded tombs do worms infoldrdquo

Mere glitter does not make a metal to be gold Man has often been warned against appearance but it has been of no use Many people have sacrificed their lives only to seek the outer appearance of gold Worms are found inside the gilded

monuments

Class XSubject Topic Summary Execution

Hindi 2ndlang

नया रासता भाग 6 मायाराम 0ी घर म धनी मल 0ी और उनी बटी सरिरता ी ही चचा बनी रहती थी अमिमत ो इसम ोई रलिच ना थी वह धनी घर ी लडी स शादी र सवय ो बचना नही चाहता था उसा भी सवाणिभमान ह ईशवर ी पा

स उस पास पस ी ोई मी नही थी अभी उसन फकटरी ही लगाई थी उसी समझ बाहर था कि उस घर वालो ा झाव पस ी तरफ कयो

ह उसन मा स सवाल किया कि मा तम सरिरता स मरी शादी कयो रना चाहती हो मा न उस समझाया कि वह दखन म बरी नही ह और किफर खानदान अचछा

ह वह ए शल गरहणी रप म घर सभाल सगी अमिमत न मा ो इस बात ा एहसास राया कि मीन सबध लिलए मना रन पर उस दिदल

पर कया बीती होगी मा और अमिमत ी लडी बार म ाफी बात हईमा ा झाव सरिरता ी तरफ था कयोकि वह घर पर अचछा दह0 लर आ रही

थी अमिमत न अपनी मौसी ी बरी हालत बार म बताया कि किस तरह वह बड घर ी खानदानी बटी लाई थी और आ0 उसी हालत कितनी खराब ह लाई थी बहकलब 0ाती ह और बचचो ो भी नही दखती ह बात चल ही रही

थी कि तभी ए ार बाहर आर री धनी मल0ी घर अदर आए और पीछ स डराइवर फल ी ए टोरी लर आया अदर आए और पीछ स

डराइवर ए टोरी फल ी लर आया अमिमत ो फल ी पटी बरी लग रही थी अमिमत न पछ लिलया यह फल कयो ल आए ह प इन सब ी कया

0ररत थी उनो न 0वाब दिदया कि 4 पटी शमीर स मगाए थ अमिमत ो या सनर करोध आ गया तभी उस किपता 0ी आ गए उन आत ही अमिमत उठर बाहर चला गया वहा वहा मा पास आर बठ गया और बोला

अभी रिरशता तय नही हआ और धनी मल 0ी धनी मल 0ी फल ी पटी लर चलआय मा न समझाया कि 0ब सबध 0ड 0ाता ह तो खाली हाथ नही

आत अमिमत न मा स हा कि तम सबन सरिरता ो इस घर म लान ी ठान रखी ह धनीमल 0ी उस दिदन सरिरता ो दखन ी तारीख तय रन आय थ

Commercial Studies

Banking Nowadays Bank provide easy and quick services through internet facilities methods of Banking is called internet bankingIn order to save the time and money involved in visiting Bank branches people increasingly prefer to have internet banking

There are different modes of doing internet banking or transferring money through online They areReal Time Gross Settlement (RTGS)National Electronic Fund Transfers (NEFT)

1

Question

1) Explain the term RTGS Write the features of RTGS

Answer)The acronym RTGS stands for Real Time Gross Settlement which may be defined as the continuous real time settlement of funds transfer individually on and order by order basis without netting lsquoReal timersquo may be defined as the processing of instructions at the time they are received rather than at some letter time lsquoGross settlementrsquo may be defined as the settlement of transfer instructions which occurs

individually

Features of RTGS1It is the continuous settlement of

funds transfer individually on an order by order basis

2RTGS facility is provided only by CBS core banking solution enabled Bank branches

3Amount charged from the customer for RTGS transactions vary from bank to bank

2) Explain the term NEFT Write the features of NEFT

Answer) National electronic funds transfer may be defined as a nationwide system that facilitates individuals Farms and copper operates to electronically transfer funds from any bank branch to any individual farm or corporate having an account with any other bank branch in the country

Features of NEFT2 Transfer can be made 7 times on

weekdays and 6 times on Saturday

3 NEFT cannot be used to receive foreign remittances

4 NEFT transaction takes place in batches

5 A bank branch must be NEFT enabled to become a part of NEFT fund transfer network

6 There is no maximum or minimum amount that can be transferred through NEFT when one bank has a bank account

English Language

CompositionEssay

A composition is an art of creating a piece of writing on any topic or subject It is the writing correctly beautifully and clearly in order to make some interesting reading Structure of the composition

Introduction ( you lay the foundation for your composition)

Body (it constitutes the main part of the essay)

Conclusion (final statement that leaves a lasting impression)

Kinds of essays1 The Narrative essay2 The descriptive essay3 The reflective essay4 The argumentative essay

Write a composition on any one of the following topics (350- 400 words)

1 Friendship Or2 The first day of your school

Subject Eng Literature (The Merchant of Venice ndash William Shakespeare)Topic Act V Scene 1 Lines 127 to 158 (Nerissa helliphellip The clerk will nersquoer wear hair onrsquos face that had it) [Students should read the original play and also the paraphrase given in the school prescribed textbook]

Summary Revision Questions o Soon thereafter Bassanio Gratiano

and Antonio arrive

o Bassanio tells Portia that he is feeling as if it is morning because of the presence of Portia who is shining like the sun When Antonio is introduced by Bassanio to Portia she tells Bassanio that he should be grateful to Antonio who took so much trouble on his account even to the extent of risking his life

o Nerissa starts quarrelling with Gratiano and demands that he show her the ring she had presented to him and which she had warned him not to lose She suspects that Gratiano must have presented the ring to some young woman and not to the lawyerrsquos clerk as he repeatedly says and assures

Answer the following questions to check your preparation of Act IV Scenes 1 and 2

You must attempt only after you have completed your preparation of Act IV The answers must be in complete sentences using textual evidence (with citation) when necessary

[It would be in your own interest to attempt the above questions honestly totally refraining from consulting your textbook or your notes during answering After completion you should correct the paper yourself consulting the textbooknotes etc and award marks as specified Please let me know the marks you scored through WhatsApp in the group or to my personal WhatsApp]

Act IV Scene 1 (each question carries 2 marks)

1 What did the Duke try to do for Antonio

2 Why does Shylock refuse to show mercy How does he justify his stance

3 Why does Antonio say he is ready to die 4 What information is contained in Bellariorsquos letter

5 Why does Portia (as Balthazar) assert that Shylock must show mercy How does he respond

6 What offers are made to Shylock to get him to spare Antonio How are they received

7 What does Antoniorsquos speech as he faces the prospect of Shylockrsquos knife tell you about his character

8 How do Bassanio and Gratiano react to the looming prospect of Antoniorsquos demise

9 How does Portia (as Balthazar) use the law to turn the tables on Shylock

10 What does the Duke decree should happen to Shylock Why What happens to Shylockrsquos estate

11 What does Portia ask Bassanio as payment for her ldquoservicesrdquo What is his initial response What makes him change his mind

Act IV Scene 2 (each question carries 1frac12 marks)

1 What does Gratiano bring to Portia (Balthazar)

2 What does Nerissa plan on getting from Gratiano What does Portiarsquos comment suggest about men

ECO-10 280620 Topic-Supply AnalysisSHIFTING OF SUPPLY

But if there is change in factors other than the price of the commodity then either more is supplied at the same price or less supplied at the same price In such cases the price of the commodity remains constant but there is a change in other factors like change in the price of inputs change in technology of production change in price of other related goods change in taxation policy of the government etc For example there is an improvement in the technology of production of the commodity in question It leads to decrease in per unit of cost production of the commodity The firm is willing to sell more quantity of the commodity at the same price So the supply other commodity increases at the same price This increase in supply is shown by rightward shift of supply curve On the other hand if the firm uses inferior technology of production the cost of production per unit of the commodity increases The firm is willing to sell less quantity at the same price So the supply of the commodity decreases at the same price This decrease in supply is shown by leftward shift of the supply curve The above cases of increase and decrease in supply can be shown with the help of the following figures

Y INCREASE OF SUPPLY Price (Rs) s

P A s1

B

s

X` O s1 X

q q1

Y` Quantity demanded (in units)

Y DECREASE IN SUPPLY s2

s

price (Rs)

C

p A

s2

s

X` o X

q2 q

Y` Quantity demanded ( in units)

Main factors causing increase in supply or rightward shift of supply Curve(i) Fall in the price of other related goods

(ii) Fall in the price of inputsfactors(iii) Use of better technology in production(iv) Decrease in the rate of excise duty by government(v) If the objective of producer changes from profit maximization to salesMaximization

Main factors causing decrease in supply or leftward shift of supply curve(i) Increase in the price of other related goods(ii) Rise in the price of inputsfactors(iii) Use of inferior technology in production(iv) Increase in the rate of excise duty by the government(v) If the objective

Subject - Biology Topic ndash Chapter mdash6 PhotosynthesisSummary Execution

Today we will know about photosynthesis and its stages

Q1 What do you mean by photosynthesis The process by which living plants containing chlorophyll produce food

substances from carbon-di- oxide and water by using light energy Sunlight

6CO2 +12 H2O----------------------- C6 H12O6 + 6H2O + 6O2

Chlorophyll

Q2 What are the importance of photosynthesis I) Food for all Green plants trap solar energy by photosynthesis

process and supply food and energy for all living organisms either directly or indirectly

Ii) Oxygen to breathe in by product of photosynthesis is oxygen which is essential for all living organisms respiration

Q3 Write about two main phases of photosynthesis A Light dependent phase This phase occur in grana of chloroplast I) The chlorophyll on exposure to light energy becomes activated by

absorbing photons Ii) The absorbed energy is used in splitting the water molecules (H2O)

into its two components (H+ and OH- ) and releasing electron s 2H2O------------------------- 4H+ + 4e- +O2

Energy of 4 photons This reaction is known as photolysis

End products are H+ and oxygen water

B Light independent (Dark ) phase The reactions in this phase require no light energy

Here CO2 combine with H+ and produce glucose

Class XI

Subject Topic Summary ExecutionEVS Chapter-4 Legal

regimes for sustainable development

Environmental legislationEnvironmental legislation is the collection of laws and regulations pertaining to air quality water quality the wilderness endangered wildlife and other environmental factors The act ensures that matters important to the environment are thoroughly

Learn -The Forest (Conservation) Act 1980

considered in any decisions made by federal agencies

The Forest (Conservation) Act 1980 The Forest (Conservation) Act 1980 an Act of the Parliament of India to provide for the conservation of forests and for matters connected therewith or ancillary or incidental thereto It was further amended in 1988 This law extends to the whole of IndiaObjects and Reasons of the Forest Conservation Act

Deforestation causes ecological imbalance and leads to environmental deterioration Deforestation had been taking place on a large scale in the country and it had caused widespread concern The act seeks to check upon deforestation and de-reservation of forests

Subject Eng Literature (The Tempest ndash William Shakespeare) Topic Act II Scene 1 Lines 314 to 329 (End of scene)

[Students should read the original play and also the paraphrase given in the school prescribed textbook]Summary Questions amp Answers

Conspiracy of Antonio and Sebastian (Contd)

o As they approach Ariel appears again and wakes up Gonzalo by singing a tune in his ear Alonso also wakes up and they see both Sebastian and Antonio with drawn swords On being caught off guard they make up a story saying that they had heard a bellowing of bulls or lions

o They then moved to another part of the island

o Ariel at once rushes to Prospero to inform him of this development

SUMMING-UP of ACT-2 SCENE-1

(i) Among the survivors Ferdinand is separated from the rest which results in the disconsolate grief of Alonso as he took him for dead

(ii) The villainy of Antonio is confirmed

(iii) The supremacy of Prosperorsquos magic which resulted in the failure of the human conspiracy

(1)

(Act II Sc 1 L 311-325)SEBASTIAN Whiles we stood here securing your repose

Even now we heard a hollow burst of bellowing Like bulls or rather lions Didt not wake youIt struck mine ear most terribly

ALONSO I heard nothingANTONIO O rsquotwas a din to fright a monsters ear

To make an earthquake Sure it was the roarOf a whole herd of lions

ALONSO Heard you this GonzaloGONZALO Upon mine honour sir I heard a humming

And that a strange one too which did awake meI shaked you sir and cried As mine eyes opened I saw their weapons drawn There was a noiseThats verily rsquoTis best we stand upon our guardOr that we quit this place Lets draw our weapons

(i) Why has Prospero sent Ariel to Gonzalo and Alonso What does Ariel do to awaken Gonzalo

Prospero has already come to know by his magic powers the danger which threatens Gonzalo who had been Prosperorsquos friend and so he sent Ariel to preserve the lives of both Gonzalo and Alonso Prospero does not want that his scheme should remain unfulfilled Ariel begins to sing a song in Gonzalorsquos ears to awaken him(ii) Who are ready to carry out their plan Who takes steps to stop them Why does Gonzalo feel surprised after being awakened

Sebastian and Antonio are ready to carry out their plans They are standing with their swords drawn to kill Alonso and

(iv) We see two sets of contrasting characters Gonzalo-Adrian against Antonio-Sebastian

(v) The grief that works in Alonso can be perceived to his repentance for his association in Antoniorsquos crime against Prospero

Gonzalo Ariel takes steps to stop them from carrying out their nefarious scheme When Gonzalo is awakened by the song sung by Ariel into his ears he (Gonzalo) feels surprised because he sees Sebastian and Antonio standing with their swords drawn(iii) What reason do Sebastian and Antonio tell of drawing their swords when they are suspected by Alonso and Gonzalo

When Sebastian and Antonio are seen with their swords drawn they are looked with suspicion by Gonzalo and Alonso At first Sebastian tells them that as they stood here to guard them during their sleep they heard only a little before a sudden loud noise very much like the roaring of bulls or more probably that of lions Then Antonio follows him saying that this was a noise so terrible as to frighten even a monsterrsquos ears and this noise could even have shaken the earth and it was surely like the roaring of a multitude of lions Then seeing the danger they have drawn their swords Perhaps after hearing the terrible noise they (Gonzalo and Alonso) woke up from their sound sleep

(iv) What does Gonzalo tell Alonso about the strange noise What did he see on opening his eyes Gonzalo tells Alonso that he did not hear the sound of roaring but he heard a humming sound which was strange and which woke him up After waking up he gave him (Alonso) a shaking and a loud cry On opening his eyes he saw these two gentlemen standing with their swords drawn(v) What does Gonzalo suggest

Gonzalo suggests that there was a noise indeed and of that he has no doubt at all and suggests that the best course for them would be to remain alert and vigilant against any possible danger to their lives or to leave this place and move to some other part of the island

Class XIISubject Topic Summary Execution

Commerce

Chapter- Management

Today we will discuss about LEVELS OF MANAGEMENT

Levels of management is a series or chain of managerial positions from top to bottom It helps individuals to know their authority responsibilities and superior-subordinate relations among themselves There are mainly three levels of Management TOP LEVEL MANAGEMENTMIDDLE LEVEL MANAGEMENTLOWER LEVEL MANAGEMENT

Top level managementIt consists of members at the highest level in the management hierarchy This level includes Board Of Directors Chief Executive Managing Directors Chairman President Vice President

Rolefunctions of the top levelmanagement1To analyse evaluate and deal

with theexternal environment2 To determine the objectives and

policies of the business3 To strive for welfare and survival

of business

4 To create an organisational Framework consisting of authority responsibility relationship

Middle level management Congress of members or groups who are concerned with implementation of the policies let down by the top managementThis level includes head of the department such as finance manager marketing manager branch and regional managers departmental and divisional heads plant superintendent etc

Role of functions of the middle level management

1 To interpret the policies framed by top management

2 To assign duties and responsibilities to lower level managers

3 To select and appoint employees for middle and supervisory level and evaluate their performance

4 To co-operate with other departments for smooth functioning

Operational or supervisory level managementIt refers to the group are members who are concerned with execution of the work They are also known as fast line managers This level includes supervisor 4 men Section Officer clerk Inspector etc

Role of functions of the lower level management1 To plan and execute day-to-

day operations2 To supervise and control the workers3 To arrange materials and

tools to start the process and make arrangements for training

4 Today present workers grievance and suggestions before the management and

ensure safe and proper working conditions in the factory

Business Studies

Staff Appraisal Chapter- 10 Today let us start with a new chapter

Staff Appraisal

Meaning of Performance Appraisal

Performance Appraisal is the systematic evaluation of the performance of employees and to understand the abilities of a person for further growth and developmentThe supervisors measure the pay of employees and compare it with targets and plansThe supervisor analyses the factors behind work performances of employeesThe employers are in position to guide the employees for a better performance

Objectives of Performance Appraisal

Following are the objectives of Performance Appraisal

To maintain records in order to determine compensation packages wage structure salaries raises etc

To identify the strengths and weaknesses of employees to place right men on right job

To maintain and assess the potential present in a person for further growth and development

To provide a feedback to employees regarding their performance and related status

To provide a feedback to employees regarding their performance and related status

Importance of Performance Appraisal

Performance appraisal provides important and useful information for the assessment of employees skill

knowledge ability and overall job performance The following are the points which indicate the importance of performance appraisal in an organization

1 Performance appraisal helps supervisors to assess the work performance of their subordinates

2 Performance appraisal helps to assess the training and development needs of employees

3 Performance appraisal provides grounds for employees to correct their mistakes and it also provides proper guidance and criticism for employees development4 Performance appraisal provides reward for better performance

5 Performance appraisal helps to improve the communication system of the organization

6 Performance appraisal evaluates whether human resource programs being implemented in the organization have been effective

7 Performance appraisal helps to prepare pay structure for each employee working in the organization

8 Performance appraisal helps to review the potentiality of employees so that their future capability is anticipated

Geography

DRIANAGE The SubarnarekhaThe Subarnarekha and the Brahmaniinterposed between the Ganga and the Mahanadi deltas drain an area of 19300 sq kmand 39033 sq km respectively The drainage basins of these streams are shared byJharkhand Odisha west Bengal and Chhattisgarh The Brahmani is known as southKoel in its upper reaches in Jharkhand

The NarmadaThe Narmada rises in the Amarkantak hills of MadhyaPradesh It flows towards the West in a rift valleyformed due to a geological fault The total length of it is 1300 km All the tributaries of the

Q1 Name the two westward flowing rivers in the peninsular plateauA1 Narmada and Tapi are the only westward flowing rivers of the peninsular plateau

Q2 Differentiate between east-flowing rivers and west-flowing riversA2

East-flowing rivers

West-flowing rivers

Narmada are very short inlength Most of its tributaries join the main streamright anglesThe Narmada basin covers parts of Madhya Pradesh and Gujarat

The Tapi The Tapi rises in the Satpura ranges in the Betul listrictof Madhya Pradesh It flows in a rift valley parallel tothe Narmada but it is much shorter in length It coversparts of Madhya Pradesh Gujarat and MaharashtraThe length is about 724 km

The Sabarmati and the MahiThe Sabarmati rises in the Aravali hills and flows south-south-westwards for a distance of 300 kilometres to the Arabian Sea The Sabarmatibasin extends over an area of 21674 sq km in Rajasthan and Gujarat The Mahi rises inthe east of Udaipur and drains an area of 34842 sq km lying in Madhya PradeshRajasthan and Gujarat It flows south-westwards for a distance of 533 km before it fallsinto the Gulf of Khambhat

The ChambalThe Chambal rises near Mhow in the Vindhya Range and flows towards the northgenerally in a gorge upto Kota Below Kota it turns to the north-east direction and afterreaching Pinahat it turns to the east and runs nearly parallel to the Yamuna beforejoining it in the southern part of the Etawah district in Uttar PradeshMajor Rivers of India with their basin area (Sqkm)

Himalayan System Indus 321290Ganga 861404

Brahmaputra 187110Indus System

Jhelum 34775Beas 20303

Ganga System Yamuna 366223Ghaghra 127950

Peninsular RiversNarmada 98796

Tapi 65145Mahanadi 141600

Subarnarekha 19300Sabarmati 21674

Mahi 34842Godavari 312812

Godavari Krishna Kaveri Mahanadi are the east-flowing rivers

Narmada Tapi west-flowing rivers

They fall into the Bay of Bengal

They fall into Arabian Sea

These rivers form big deltas

These rivers form comparativelysmall deltas

Catchment areas of these rivers are larger

Catchment areas of these rivers are smaller

Krishna 2589488Cauveri 87900

Subject ndashBiology Topic ndashChapter -5 Inheritance amp Variations Summary ExecutionToday we will discussabout linkage and its classification

LINKAGE The tendency of the genes located on the same chromosome to stay together is

hereditary transmission Linked genes the genes responsible for this Genes that exhibit the process of linkage locates in the same chromosome The distance between the linked genes in a chromosome determines the strength

of linkage i e genes that are located close to each other show stronger linkage than that are located far from each other

COMPLETE LINKAGE It is the type of linkage showed by the genes that are closely located or are tightly

linked with each other as they have no chance of separatingby crossing over These genes are always transmitted together to the same gamete and the same

offspring In such condition only parental or non cross over type of gametes are formedINCOMPLETE KINKAGE It is type of linkage showed by the genes that are distantly located orare loosely

linked with each other because they have chance of separating by crossing over

SIGNIFICANCE i) It helps in holding the parental character togetherii) It checks the appearance of new recombination and helps in bringing the

hybrid population which resembles the original parents iii) Linked genes dilute the effects of undesirable traits

Subject Eng Literature (The Tempest ndash William Shakespeare) Topic Essay Questions (EQ-3)Question No 3

Give a character sketch of CalibanAnswer

The character of Caliban has been wonderfully conceived by Shakespeare as the manifestation of all that is gross and earthy ndash a sort of creature of the earth as Ariel is a sort of creature of the air

Calibanrsquos Physical Appearanceo Caliban is lsquofreckledrsquo a lsquomisshapen knaversquo not honoured with human shape

o Prospero calls him lsquothou tortoisersquo (Act I Sc 2 Line 317) Trinculo stumbling upon him describes him as ldquoA strange fish hellip Legged like a man And his fins like armsrdquo He ldquosmells like a fishrdquo (Act II Sc 2 Line 25)

o Prospero also calls him a ldquobeastrdquo (Act IV Sc 1 Line 140) and ldquoThis misshapen knaverdquo (Act V Sc 1 Line 268)

o Further it appears that in addition to his physical deformity his spiritual inferiority is also suggested by Prosperorsquos claim that his birth resulted from the union between his mother the witch Sycorax and the devil

Calibanrsquos ParentageWhen the play opens Caliban is twenty four years of age having been born on the island twelve years before the coming of Prospero His mother was the foul witch Sycorax who was banished from Algiers for ldquomischiefs manifold and sorceries terrible to enter human hearingrdquo (Act I Sc 2 Line 264) and the father was the Devil himself Thus

Caliban is a monster of evil and brute nature ugly deformed and stinking

Calibanrsquos Savage and Malignant Natureo Caliban is entirely a creature of the earth ndash gross brutal and savage He regards himself as the rightful possessor

of the island and Prospero as a usurper

o In his young age he was on good terms with Prospero He had consented to be received by Prospero at his house and to be educated by him He has learnt human language only to curse his master whom he abhors

o His beastly nature soon breaks out and ends in a vicious attack on Miranda This opens the eye of Prospero who becomes severe to him and enforces his service by threats and violence

o Prospero uses him to make dams for fish to fetch firewood scraper trenches wash dishes and keep his cell clean

Calibanrsquos Hatred for ProsperoA profound hatred for Prospero has taken hold of Caliban It springs from a sense of his being dispossessed and ill-treated He would kill Prospero if he could but he knows the power of Prosperorsquos lsquobookrsquo Hence he transfers his allegiance to Stephano who seems like a god to him He also incites the two drunken associates to batter the skull of Prospero when he sleeps in the afternoon

Caliban Shows Considerable Intelligenceo He has learnt Prosperorsquos language

ldquoYou taught me language and my profit onrsquot (Act II Sc 2 Lines 86-89)Is I know how to curserdquo

o He is well aware of the futility of arguing with one who has more power than he has

ldquoI must obey his art is such power (Act I Sc 2 Lines 373-376)It would control my damrsquos god SetebosAnd make a vassal of himrdquo

o He realizes the importance of Prosperorsquos books

ldquoRemember (Act III Sc 2 Lines 89-92)First to possess his books for without themHersquos but a sot as I am nor hath notOne spirit to commandrdquo

o He knows the value of stealth when attacking the enemy

ldquoPray you tread softly that the blind mole may not (Act IV Sc 1 Lines 194-195)Hear a foot fall we now are near his cellrdquo

o Caliban has a better set of values than Stephano and Trinculo They are distracted from their plan by their greed for Prosperorsquos rich garments Only Caliban realizes that such a finery is unimportant

ldquoLeave it alone thou fool it is but trashrdquo (Act IV Sc 1 Lines 224)

Caliban is not a good judge of characterCaliban is not a good judge of character He decides for example that Stephano is a god because he dispenses lsquocelestial liquorrsquo (Act II Sc 2 Line 115) but then it must be remembered that he has only known his mother Sycorax Prospero Miranda and the spirits that torture him However he quickly discovers his error of judgementrdquo

ldquoWhat a thrice-double ass (Act V Sc 1 Lines 295-297)Was I to take this drunkard for a godAnd worship this dull foolrdquo

Calibanrsquos Imaginative NatureIf Caliban is sub-human in what has been said above he is human in the respect of the poetic side of his character He listens to music with rapture He tells of the beautiful dreams in which heaven rains treasures upon him and which upon waking he yearns to renew One of the most poetic passages in whole play is Calibanrsquos description of the island

to Stephano and Trinculo

ldquoBe not afeard The isle is full of noises (Act III Sc 2 Lines 135-143)Sounds and sweet airs that give delight and hurt notSometimes a thousand twangling instrumentsWill hum about mine ears and sometime voicesThat if I then had waked after long sleepWill make me sleep again and then in dreamingThe clouds methought would open and show richesReady to drop upon me that when I wakedI cried to dream againrdquo

Caliban - Less Ignoble Than Some OthersCalibanrsquos motive for murder is less dishonourable than that of Antonio and Sebastian They plan to kill Alonso to gain his power and wealth Caliban merely wants revenge and the return of lsquohisrsquo island

Conclusiono Calibanrsquos character is not portrayed very clearly in the play and hence we cannot decide whether he is a poor

savage being grossly maltreated by Prospero or whether he is evil and must therefore be kept in bondage or enslavement

o Caliban is contrasted with Ariel who is a spirit and thus swift and uninterested in physical activitieso Caliban is also contrasted with Prospero who is the all-powerful master of the island and of the destiny of all

those on the islando Caliban is also contrasted with civilized man showing him to be less evil than Antonio and Stephano and less

materialistic than Stephano and Trinculoo Caliban has suffered at the hands of Prospero and he has learnt to curse by listening to Prosperorsquos abuse He

certainly believes that Prospero has deprived him of his birthrighto Finally the character Caliban is thought to be one of Shakespearersquos masterpieces The complexity of the character

is reflected in the large volume of critical discussion that has grown around it

ECO ndash12 Topic-Forms of market

MonopolyMonopoly is a market structure in which there is a single seller there are no close substitutes for the commodity produced by the firm and there are barriers to entry Example Indian Railways which is operated under government of India Monopoly also implies absence of competitionFeatures of Monopoly Monopoly is characterized by1 Single Seller In monopoly there is only one firm producing the product The whole industry consists of this single firm Thus under monopoly there is no distinction between firm and industry Being the only firm there is significant control of the firm over supply and price Thus under monopoly buyers do not have the option of buying the commodity from any other seller They have to buy the product from the firm or they can go without the commodity This fact gives immense control to the monopolist over the market

2No Close Substitute There are no close substitutes of the product produced by the monopolist firm If there are close substitutes of the product in the market it implies presence of more than one firm and hence no monopoly In order to ensure a total of control over the market by the monopolist firm it is assumed that there are no close substitutes of the product

3 No Entry amp Exit Monopoly can only exist when there is strong barriers before a new firm to enter the market In fact once a monopoly firm starts producing the product no other firm can produce the same One reason for this is the ability of the

monopolist to produce the product at a lower cost than any new firm who thinks to enter the market If a new firm who knows that it cannot produce at a lower cost than the monopolist then that firm will never enter the market for fear of losing out in competition Similarly the monopolist who is operating for a long time may be enjoying reputation among its customers and is in a better position to use the situation in its own benefit A new firm has to take long time to achieve this and so may not be interested to enter the market

4 Price Maker Being the single seller of the product the monopolist has full control over the pricing of the product On the other hand if there is a large number of buyers in the market so no single buyer exercises any significant influence over price determination Thus it is a sellerrsquos market So monopoly firm is a price maker

5 Price Discrimination Having considerable control over the market on account of being single seller with no entry of other firms the monopolist can exercise policy of price discrimination it means that the monopolist can sell different quantities of the same product to a consumer at different price or same quantity to different consumers at different prices by adjudging the standard of living of the consumer

6 Shape of Demand Curve Since a monopolist has full control over the price therefore he can sell more by lowering the price This makes the demand curve downward sloping

Subject Ac-12 290620 Topic- retirement Model sumThe Balance Sheet of Rohit Nisha and Sunil who are partners in a firm sharing profits according to their capitals as on 31st March 2014 was as under

Liabilities Amount Assets Amount (Rs) (` Rs)

Creditors 25000 Machinery 40000Bills Payable 13000 Building 90000General Reserve 22000 Debtors 30000Capital Less Provision for Rohit 60000 Bad debts 1000

29000 Nisha 40000 Stocks 23000 Sunil 40000 140000 Cash at Bank 18000

200000 200000

On the date of Balance Sheet Nisha retired from the firm and following adjustments were made(i) Building is appreciated by 20(ii) Provision for bad debts is increased to 5 on Debtors(iii) Machinery is depreciated by 10(iv) Goodwill of the firm is valued at Rs 56000 and the retiring partnerrsquos share is adjusted

(v) The capital of the new firm is fixed at Rs120000 Prepare Revaluation Account Capital Accounts of the partner and Balance Sheet of the new firm after Nisharsquos retirement Revaluation AccountDr Cr

Particulars Amount Particulars Amount (`Rs) (Rs`)

Provision for Bad debt Ac 500 Building Ac 18000Machinery Ac 4000Profit transferred toCapital Accounts (3 2 2)Rohit 5786Nisha 3857Sunil 3857

13500

18000 18000

Capital Account

Dr Cr

Particulars Rohit Nisha Sunil Particulars Rohit Nisha Sunil (Rs`) (Rs`) (`Rs) (Rs`) (Rs`) (Rs`)

Sunilrsquos Capital ac 9600 mdash 6400 Balance bd 60000 40000 40000Bank - 66143 - General Reserve 9428 6286 6286Balance cd 72000 mdash 48000 Revaluation (Profi 5786 3857 3857 Rohitrsquos Capital Ac mdash 9600 mdash

Sunilrsquos Capital Ac 6400 Bank 6386 - 4257

81600 66143 54400 81600 66143 54400

Balance Sheet as at 31st March 2014

Liabilities Amount Assets Amount (Rs`) (Rs`)

Creditors 25000 Building 108000Bank overdraft 37500 Machinery 36000

Bills Payable 13000 Debtors 30000Capital Less ProvisionRohit 72000 for Bad debts 1500 28500Sunil 48000 120000 Stock 23000

195500 195500

Working Notes (i) (a) Profit sharing ratio is 60000 40000 40000 ie = 3 2 2(b) Gaining Ratio Rohit = 35 ndash 37 = 2135 ndash 1535 = 635Sunil = 25-27 = 1435 ndash 1035 = 435= 635 435= 6 4 = 3 2(c) Nisha Share of Goodwill = Rs 56000 times 27 = Rs16000Share of Goodwill in the gaining ratio by the existing partner ieRohit = Rs16000 times 35 = Rs 9600Sunil = Rs 16000 times 25 = Rs 6400

The journal entry isRohitrsquos Capital Ac Dr 9600Sunilrsquos Capital Ac Dr 6400 To Nisharsquos Capital Ac 16000(Share of Goodwill divided into gaining ratio)

  • 1 Static Friction
  • The frictional force that acts between the surfaces when they are at rest with respect to each other is called Static Friction
    • Static Friction Examples
      • 2 Sliding Friction
        • Examples Of Sliding Friction
          • 3 Rolling Friction
            • Examples Of Rolling Friction
              • Objects and Reasons of the Forest Conservation Act
Page 11:  · Web viewSubject . Topic . Summary . Execution . English 1 . Sounds of animals . Hens –cackle Horses –neigh Lions –roar Owls –hoots Snake –hiss. English 2 . Mother’s

clauses8 In her new clothes ndashphrasesAs pretty as a doll ndash phrases

9 looking sad and upset ndash phrasesHe had lost all the tickets for the Test Match ndash clauses

10 During the vacation now only a month away ndashphrases

11 too sweet and too hot ndashphrases

12 At the Olympic Games ndashPhrasesOf Laurel leaves ndash phrases

13 Who played the role of Hamlet ndashclauses

14 However fast ndash phrases

15 When the men fell asleep ndash clausesSocial studies

Indian Government

Lok Sabha (lower house) ndash It has 552 members Of these 530 membersrepresent States 20 members represent the union territories and two members represent the Anglo- Indian community All except the representatives of the Anglo-Indian community are elected by Indian citizens A person above the age of 25 can contest in the elections for Lok Sabha One term of Lok Sabha is for 5 yearsRajya Sabha (upper house) ndash Its members are elected by the MLAs or members of the legislative Assembly There are 250 members in the Rajya Sabha of which 12 are nominated by the President One term of Rajya Sabha is for 6 years Anyone above the age of 30 can be elected as a member of Rajya Sabha

ExecutivePresidentThe President is the head of the country in India He is elected by the MPs and the MLAs for a tenure of 5 years He appoints the Prime Minister and the Council of Ministers Prime MinisterThe party which wins the election forms the government and its leader is elected as the Prime Minister He is the chief advisor to the President The Council of Ministers assists the Prime Minister and is accountable for their roles For example the Education minister is responsible for the education system in our country

1 How many members are there in Lok SabhaAns 552 members

2 What is the term for Lok SabhaAns 5 years

3 How many members are there in the Rajya SabhaAns 250 members

4 Who is the head of our countryAns President

5 Who is the chief advisor to the PresidentAns Prime Minister

Book ndash GK

Ch ndash 1First in space

1 First living being into space in 1957 Ans Laika

Times 2 First person to go into space in 1961 Ans Yuri Gagarin

3 First woman to go into space in 1963 Ans ValentinaTereshkova4 First person ever to walk in space in 1965Ans alexei Leonov5 First person to land on the moon in 1959 Ans Neil Armstrong6 First Indian to go into space in 1984 Ans Rakesh Sharma 7 First Indian woman to go into space in 19978 Ans Kalpana Chawla9 First woman tourist in space in 2006

Ans Anusheh AnsariCOMPUTER

ALGORITHM AND FLOWCHART

Q) DRAW THE SYMBOLS USED IN A FLOWCHART WITH THEIR DESCRIPTIONS(IN EXAM IT CAN COME AS SHORT QUESTIONS ASKING INDIVIDUAL SYMBOLS FUNCTION)ANS)

MAT

HEM

ATIC

S

Ch 6

Com

mon

Fra

ction

s

Multiplication of FractionsA Multiply a fractional number by whole numberTo multiply a fractional number by whole number we multiply the numerator of the fractional number by the whole number and denominator of the fractional number by 1 The first product thus obtained is the numerator and the second product is the denominator of the required product

Exercise ndash 30Multiply

7 2027

times 9

Solution 2027

times 9 = 203 = 6

23

8 611

times11

Solution 611

times11 = 6

15 71

20times16

Solution 71

20times16 =

14120

times16

= 1415

times 4 = 141times 4

5 = 564

5 = 11245

B Multiplication of a fractional number by a fractional number To multiply a fractional number by a fractional number we multiply the numerator of the first fractional number by the numerator of the second fractional number and the denominator of the first fractional number by the denominator of the second fractional number The first product thus obtained is the numerator and the second product is the denominator of the required product

16 2712

times24

Solution 2712

times24 = 3112

times24

= 31times2 = 62

Exercise ndash 31

11 83

times 34

2

Solution 83

times 34 = 2

14 723

times2 25

4

Solution 723

times2 25 =

233

times 125 =

23times 45

= 925 = 18

25

15 1212

times1 13

2

Solution 1212

times1 13 =

252

times 43 =

25times 23

= 503 = 16

23

State the following statements are true or false

17 1912

times 239 = 1

Solution LHS = 1912

times 239

= 392

times 239 = 1 = RHS

[LHS = Left hand side amp RHS = Right hand side]

there4 1912

times 239 = 1 [True]

21 213

times2 13 = 4

19

Solution LHS = 213

times2 13 =

73

times 73

= 7times73times3 =

499 = 5

49

there4 LHS ne RHS

So 213

times2 13 = 4

19 [False]

23 23

times 45 =

2times 5+3 times43times 5

Solution

LHS = 23

times 45 =

2times 43 times5 =

815 again

RHS = 2times 5+3 times4

3times 5 = 10+12

15 = 2215

there4 LHS ne RHS So 23

times 45 =

2times 5+3times43times 5

[False]

25 23 of

13 =

29

Solution

LHS= 23 of

13 =

23 times

13 =

29 = RHS]

there4 23 of

13 =

29 [True]

Practice at HomeExercise ndash 31State the following statements are true or false

24 12 of 4 =

18

Class VISubject Topic Summary Execution

HISTORY AND CIVICS

Chapter 5The Mauryan Empire

DECLINE OF MAURYAN EMPIREDecline of Mauryan empire started after the death of Ashoka at around 232 BCThere are several reasons for break up of the empire1 Weak successor Emperors after Ashoka were

capable of handling vast and mighty Mauryan empire In 185BC the last Mauryan ruler Bri-hadrath was murdered by his Commander-in-Chief Pushyamitra Sunga

2 Provincial Revolts Due to weak central author-ity provincial chiefs of Kalinga and southern provinces revolted against emperor and freed themselves from Mauryan empire

3 Weakness of Economy Prosperity of Mauryan was based on solid economic activities which

ExercisesI Multiple choice questions-1 Chandragupta defeated Seleucus in the year ndashc) 305 BC2 Who killed the last Mauryan ruler Brihadrath b) Pushyamitra3 Which of the following was not a reason for the decline of the Mauryan empirec) Chandraguptarsquos weakness4 Ashoka invaded Kalinga in the year c) 261 BC

II Fill in the blanks1Chandragupta ascended the throne in 324

was taken care by early monarchs Later kings had neither ability nor interest in economic af-fairs That led to failure in tax collection As a result they failed to maintain a large army that were essential to keep empire intact

4 Greek Invasion Greeks freed north-western provinces from weak Mauryan monarchs and reestablished their authority

5 Ashokarsquos Policy some scholar opined that after Kalinga war Ashoka embraced Buddhism re-nounced the policy of war and disbanded the Army But this is partially true as there is no proper evidence of disbanding the army

Based on above points we can conclude that main reason for decline of Mauryan empire is weakness of Ashokarsquos successors Kunal Samprati Dasharath Salisuk all were weak kingsAt last in 185 BCPushyamitra Sunga killed king Brihadrath and established the Sunga dynasty

BC2 Bindusara was the son of Chandragupta and father of Ashoka

3 Pataliputra was administered by City Magistrate committess of 5 members each4 The Greek General Seleucus sent his ambassador Megasthenes to Chandraguptarsquos court5 Ashoka sent his son Prince Mahendra and daughter Sanghamitra to spread his Dhamma6 The Indian Rebublic has adopted the Lion Capital of Saranath Pillar as its national emblem 7 Pushyamitra killed the last Mauryan ruler Brihadrath and founded the Sunga dynasty

III Name the following

1The author of Arthashastra-Kautilya2 The ruler who founded the Mauryan dynasty-Chandragupta3 The author of Indika-Megasthenes 4 The officers who were appointed by Ashoka to spread Dhamma-Dhamma Mahamatras5 The general of Alexander whom Chandragupta defeated-Seleucus

V Match the columns1 Kautilya (c)2 Megasthenes (d)3 Pushyamitra (e)4 Brihadrath (b)5 Bindusara (a)

BENGALI(2ND

LANGUAGE)

পশপাহিখর -াষাসহিবনয় রায়কেচৌধরী

যলখক পহিরহিচহিত- পরখযাত সাহিহিতযক উকেপনদরহিককেশার রায়কেচৌধরীর পতর সহিবনয় রায়কেচৌধরী lsquoসকেFশrsquo পহিতরকার সকেb হিতহিন কত হিকেলন তার উকেdখকোয বই lsquoসহিবনয় রায়কেচৌধরীর রচনা সংগরrsquo

পরম হিকেনর পাঠ- lsquoপশপাহিখর হিক -াষাhelliphellip helliphelliphelliphellipপরসপরকেক জানাবার উপায়ও পশপাহিখরা যবশ জাকেনrsquoপরকেমই আমারা জাহিন -াষা হিক -াষা ল আমাকের মকেনর -াব পরকাশ করার জনয আমরা নানান ধরকেনর -হিb বা হিবকেশষ ধরকেনর আওয়াজ মকেখর মাধযকেম কহির অনযকেক যবাঝাকেনার জনয তাকেল এবার আমরা জাহিন পশপাহিখর -াষা হিক পশ পাহিখরা হিক কা বকেল যা পশপাহিখকেরও -াষা আকে তারা তাকের হিনজসব -াষায় কা বকেল মকেনর -াব পরকাশ ককের পশ পাহিখরা মানকেষর হিক হিক -াষা যবাকেঝ হিকনত তারা বলকেত পাকেরনা পরসপরকেক বহিঝকেয় যবার উপায় তারা জাকেননা তকেব তারা হিবকেশষ ককেয়কটি শকেবদর মাধযকেম তাকের মকেনর -াব বহিঝকেয় যয় হিক বহিদধ মান জীব ndashককর হিবাল বন মানষ যঘাা পর-হিত এরা মানকেষর যওয়া নাম শনকেল কান খাা ককের ndash নাম ধকের ডাককেল কাকে আকেস যমন - মরহিরা lsquoহিত ndashহিতrsquo ডাক শকেন আকেস াল lsquoঅ ndashর -র ডাক শকেন কাকে আকেস াহিত মাহকেতর কা শকেন চকেল ককররা মাহিলকেকর হকম পালন ককের সবসময় তাইকেতা ককরকেক পর- -কত পরানী বলা য় ককর আর হিবাল একের আওয়াজ তহিম লকষয করকেল বঝকেব ককররা যরকে যকেল lsquoযঘউ যঘউrsquo করকেত াকেক আবার কাকেল lsquoযকউ যকউrsquo ককের হিবাল সাধারণ lsquoমযাওrsquo বা lsquoহিমউrsquo ককের রা কেল lsquoওয়াওrsquo আওয়াকেজর মাধযকেম মকেনর -াব পরকাশ ককের একেতা যল পশকের কা পাহিখরাও -য় রা পরকাশ করার জনয হিবকেশষ ধরকেনর শবদ ককের হিবপকের সময় পশ পাহিখরা সবার আকে পরসপরকেক জানাবার উপায় তারা জাকেন বহকাল

১) পশপাহিখর -াষা কেলপর যলখক সমপকেকG হিক জাকেনা

উঃ- পরখযাত সাহিহিতযক উকেপনদরহিককেশার রায়কেচৌধরীর পতর সহিবনয় রায়কেচৌধরী lsquoসকেFশrsquo পহিতরকার সকেb হিতহিন কত হিকেলন তার উকেdখকোয বই lsquoসহিবনয় রায়কেচৌধরীর রচনা সংগরrsquo হিতহিন ারকেমাহিনয়াম এসরাজ পর-হিত বাযনতর বাজাকেত পারকেতন ানও জানকেতন হিতহিন যাকেIাকের জনয মজাার লপ কহিবতা হিলখকেতন

২) পশপাহিখ কেলপর মল-াব হিকউঃ- পশপাহিখকেরও -াষা আকে তারা তাকের হিনজসব -াষায় কা বকেল মকেনর -াব পরকাশ ককের পশ পাহিখরা মানকেষর হিক হিক -াষা যবাকেঝ হিকনত তারা বলকেত পাকেরনা পরসপরকেক বহিঝকেয় যবার উপায় তারা জাকেননা তকেব তারা হিবকেশষ ককেয়কটি শকেবদর মাধযকেম তাকের মকেনর -াব বহিঝকেয় যয় হিরউকেবন কযাসটং সাকেব হিতহিন চহিdশ বর বনযজনত যর সকেb যকেককেন হিতহিন বকেলকেন আমরা হি তাকের -াষা তাকের আব কায়া যমকেন চহিল তাকেল আর -কেয়র যকান কারণ াকেকনা আমরা একI -াকেলাকেবকেস যচষটা করকেল পশপাহিখকের সকেb -াব পাতাকেত পাহির

ধকের মানষ এই পশ পাহিখর -াষা হিনকেয় নানা রককেমর পরীকষা ককের আসকে এইরকম একজন হিরউকেবন কযাসটাং সাকেকেবর কা আমরা জানকেবাhelliphellip

Hindi 2nd

langमतर किनमनलिलखिखतपरशनोउRरदीजि0ए

) बढ वयलि` बचच ो कया हआ था ख) डॉकटर साहबन पाटc किस उददशय स रखी थी ग) ाल साप ो हाथ म लर लाश न कया किया घ) डॉकटर चडढा न बढ पतरो दखन स कयो मना र दिदया था ङ) भगत न लाश ो दखर कया हा

उRर ndash) उस बहत बखार थी और 4 दिदनो स आख भी नही खोला थाख) उन बट ी सालकिगरह थीग) ाल सापो हाथ म लर लाश न उसी गदन 0ोर स दबार पडी थीघ) डॉकटर चडढा न बढ वयलि` पतरो दखन स मना र दिदया कयोकि उनह गोलफ खलन 0ाना थाङ) लाश ो दखर हा कि नारायण चाहग तो आध घट म भया उठ 0ाएग

English literature

In the bazaars of Hyderabad- Sarojini Naidu

Through the poem In The Bazaars of Hyderabad Sarojini wanted to convey the message that India is rich in tradition and they donrsquot need the foreign products So she goes on to give a picture of a bazaar where traditional Indian products are rulingThe poem is in the form of questions and answers The poet asks the questions and the merchants answer them Through this technique she make the picture of the bazaar visible to us

Read the poem

PHYSICS FORCE Types of FrictionThere are three types of friction static sliding rolling Static sliding and rolling friction occur between solid surfaces

1 Static Friction The frictional force that acts between the surfaces when they are at rest with respect to each other is called Static FrictionStatic Friction Examples

Skiing against the snow Creating heat by rubbing both the hands

together Table lamp resting on the table

2 Sliding Friction The resistance that is created between any two objects when they are sliding against each other is called Sliding FrictionExamples Of Sliding Friction

Sliding of the block across the floor Two cards sliding against each other in a

deck

3 Rolling Friction The force which resists the motion of a ball or wheel is called Rolling Friction Is the weakest types of frictionExamples Of Rolling Friction

Rolling of the log on the ground Wheels of the moving vehicles

6What effect can a force produce on a body which is not allowed to move Ans - When a force is applied on a body which is not free to move it gets deformed i e the shape or size of the body changes7Give one example each to indicate that the application of a force

1 produces motion2 stops motion3 slows down motion4 changes the direction of motion5 deforms a body

Ans- 1 A car originally at rest when pushed

begins to move2 A moving bicycle is stopped by

applying the brakes3 The speed of a moving vehicle is

slowed down by applying the brakes4 A player kicks a moving football to

change its direction of motion5 On stretching a rubber string its

length increases

8State the effect produced by a force in the following cases (a) The sling of a rubber catapult is stretched(b) A man pushes a heavy cart(c) A player uses his stick to deflect the ball (d) A cyclist applies brakes(e) A spring is compressedAns- (a) The shape and size of catapult changes ie its length increases(b) The heavy cart begins to move(c) The direction of the ball changes(d) The speed of the moving cycle is slowed down(e) There is change in size and shape of spring

COMPUTER MS EXCEL 2013 -INTRODUCTION

UNDERSTANDING EXCEL STRUCTUREA SPREADSHEET IS A FILE THAT EXISTS OF CELLS IN ROWS AND COLUMNS AND CAN HELP ARRANGE CALCULATE AND SORT DATA DATA IN A SPREADSHEET CAN BE NUMERIC VALUES AS WELL AS TEXT

FORMULAS REFERENCES AND FUNCTIONS

WORKSHEETA WORKSHEET IS ALSO KNOWN AS SPREADSHEETIT IS A COLLECTION OF CELLS ON A SINGLE SHEET WHERE YOU KEEP AND CHANGE DATA

WORKBOOKWORKBOOK IS PMS EXCEL FILE IN WHICH THE DATA CAN BE STORED EACH WORKBOOK CAN CONTAIN MANY WORKSHEETS

ROWS AND COLUMNSIN MS EXCEL A ROW IS A GROUP OF CELLS THAT RUN FROM LEFT TO RIGHT OF A PAGEA COLUMN IS A GROUPING OF CELLS THAT RUN FROM THE TOP TO THE BOTTOM OF A PAGE

CELLTHE INTERSECTION POINT BETWEEN A ROW AND THE COLUMN IS CALLED A CELL WHICH IS THE BASIC STORAGE UNIT FOR DATA IN A SPREADSHEET EACH CELL HAS SPECIFIC ADDRESS WHICH IS THE COMBINATION OF THE COLUMN NAME FOLLOWED BY THE ROW NUMBER

CHEMISTRY Chapter ndash Common Laboratory Apparatus and equipments

Objective type questionFill in the blanks (a) Experiment and observation are the two important basics of chemistry(b) A porcelain dish is used for evaporation(c) A test tube holder is used to hold the test tube while-it is heated(d) Mortar and pestle is used for grinding and crushing solid substances into a powder(e) Glass apparatus is made of Pyrex or borosil glass

Class VIISubject Topic Summary Execution

Hindi 2ndlang

ए था राम( डॉ शरी परसाद)

सगकित ा परभाव मानव 0ीवन पर अवशय पडता ह

हमशा मनषय ो अचछो ी सगकित म रहना चाकिहए

शरषठ परो सग स मनषय चरिरतर ा शीघर ही उदय और किवास हो 0ाता

ह इसलिलए वयलि` ो सदा शरषठ परो ा ही सग रना चाकिहए

इसान अगर चाह वह सवय ो बदल भी सता ह

यह हानी राम ए बचच ी हवह गणिणत ी परीकषा म नल रत हए पडा 0ाता ह और उस अधयाप पडत ह और पछत ह यह कया र रह

हो तभी राम न उनी बइजजती ी

शबदाथब ndashहावा भलावाायवाहीndash ाम किनयम व ानन

ो दिदखानापरिरलिचतndash 0ाना पहचानाघटनाndashघबराहट

उलटा चोर ोतवाल ो डाटndashकिववndash भल बर ा जञानतवयndash म 0ो रना चाकिहएसगकितndash बरी सगत

किबलख नाndashरोना किनशचय रनाndash तय रना

फलndashपरिरणामकिनषालिसतndash बाहर किया हआपशचातापndashदख सपननndashधनी

ldquo हा आपी किहममत स हई नल रत पडन ीrdquo ऐसी बात ही किफर

किपता0ी न भी उस डाटा वह ाफी पशचाताप रन लगा बोला गलत दोसतो

ी सगकित म आ0 कितना अनथ र दिदया किफर उसन अधयाप स माफी मागन ी सची और किफर भी ऐसा

नही रगा यह परण भी लिलया

सोचndashकिहच एात-अला

বইndashবাংলা সাহিতয পহিরচয়

পাঠndash১৬লপndashস-য ও অস-যযলখকndashঈশবরচনদর হিবযাসারঅনশীলনীর পরকে4াততর

৬ অGকেলকেখা -ময়া = পশ হিশকার সহিtহিত = হিনকIবতu সbভরষট = লI হিনরীকষণ =

-াকেলা-াকেব যখাকতাঞজহিলপকেI = যজাাকেত৭ হিবপরীতশবদ -ঈষৎ times পরচর উৎকষট times হিনকষট তাশ times উৎফd তবহিদধ times

বহিদধীNপাহিপষঠ times পণযবান৮ পপহিরবতG ন ককেরা -পশ = পাশহিবক যকাপ = যকাহিপতহিসথর = হিসথরতাএকানত = ঐকাহিনতক পর-াত times পর-াতী

CHEMISTRY

Chapter ndashPhysical and Chemical Changes

Chemical ChangeA chemical change involves a change in chemical composition

Characteristics of Chemical changes 1 They are permanent changes2 They are irreversible changes 3 New substance formed4 A Chemical change involves a

change in its chemical properties

Pg-25Question 8What do you observe when1 water is boiled2 a piece of paper is burnt3 some ice cubes are kept in a glass tumbler4 solid ammonium chloride is heated5 an iron nail is kept in tap water for few days6 a spoon of sugar is heated in a pan7 lighted match stick is brought near the mouth of the test tube containing hydrogen gas8 quick lime is dissolved in water9 little amount of curd is added to a bowl containing warm milk and kept for five hours

10 Water is boiledOn boiling water changes into steam (gas) physical change

11 A piece of paper is burnton burning piece of paper produces carbon dioxide and ash is left behind Is a chemical change

12 some ice cubes are kept in a glass tumblerIce cubes (solid) turn into water

(liquid) only state changes (physical change)

13 Solid ammonium chloride is heatedSolid ammonium chloride on heating changes into vapors (change of state) is physical change

14 An iron nail is kept in tap water for few dayswe observe reddish brown coating on the nail called rust (entirely new substance) is chemical change

15 A spoon of sugar is heated in a panWhen a spoon of sugar is heated in a pan black (charred sugar) (carbon) is seen Is a chemical change

16 Lighted match stick is brought near the mouth of the test tube containing hydrogen gasWe observe that hydrogen bums at the mouth of test tube with blue flame and pop sound is heard It is chemical change

17 Quick lime is dissolved in waterThe following two observations will be observed (i) A hissing sound is observed(ii) The mixture starts boiling and lime water is obtained

18 Little amount of curd is added to a bowl containing warm milk and kept for five hoursWhen a little amount curd is added to a bowl containing warm milk and kept for five hours a permanent change occurredThe milk will change to curd On boiling water changes into steam (gas) physical change

GEOGRAPHY

ATMOSPHERE IMPACT OF GLOBAL WARMING The destructive impart of global warming is observed in various spheres of life and the environment Some of the points are outlined below1 High temperatures lead to high

evaporation rate and drying up of the soil and surface water This affects crop production The occurrence of droughts is aggravating the problem even further

2 The heat waves in summer months

Q1 Write some impact of global warmingA1 The impacts of global warming are as follows1 High temperatures lead to high

evaporate ion rate and drying up of the soil and surface water This affects crop production The occurrence of droughts is aggravating the problem even further

2 The heat waves in summer months lead to a greater number

lead to a greater number of deaths due to heat strokes

3 Forest fires become more frequent4 Tropical cyclones and hurricanes

become common5 Melting of glaciers takes place6 Polar ice caps are becoming thinner

and melting at an alarming rate due to global warming The loss of sea ice

7 Due to increase in sea surface temperature sea levels rise in coastal areas and cause submergence of several islands

WAYS TO REDUCE GLOBAL WARMINGFollowing steps can be taken We need to decrease emission of

green house gases by reducing the burning of fossil fuel such as coal and petroleum

By planting more trees to increase forest cover

The government should also distributes free saplings and organize afforestation programmes to spread awareness regarding the beneficial effects of trees

We should switch to eco-friendly cars and gadgets

Incandescent light bulbs should be replaced by CFL bulbs

We can save electricity and reduce global warming by turning off electrical gadgets such as lights fans air-conditioners television and computer when we do not to use them

Efforts should be made to hasten the development of green cities oreco cities These cities are urban areas around the world striving to lessen the environment a impacts of urbanization

By following the 3Rs-Reduce Recycle and Reuse strategy we can use natural resources for our growth as well as save them for the need of the future generations This is called sustainable development

of deaths due to heat strokes3 Forest fires become more

frequent4 Tropical cyclones and hurricanes

become common5 Melting of glaciers takes place

etc

Q2 How to reduce global warmingA2 Following steps can be taken to reduce global warmingaWe need to decrease emission of

green house gases by reducing the burning of fossil fuel such as coal and petroleum

bBy planting more trees to increase forest cover

c The government should also distributes free saplings and organize afforestation programmes to spread awareness regarding the beneficial effects of trees

dWe should witch to eco-friendly cars and gadgets

eIncandescent light bulbs should be replaced by CFL bulbs

f We can save electricity and reduce global warming by turning off electrical gadgets such as lights fans air-conditioners television and computer when we do not to use them

Q3 What do you mean by 3Rrsquos of resource planningA3 The 3Rs are

1 Reduce 2 Recycle and3 Reuse

Q4 What is Sustainable developmentA4 By following the 3Rs-Reluce Recycle and Reuse strategy we can use natural resources for our growth as well as save them for the need of the future generations This is called sustainable development

English Language

Prepositions A preposition is a word placed before a noun or a pronoun It helps to show how the person or thing denoted by the noun is related to something else in the sentence

Kinds of Prepositions

Simple Prepositions- simple preposition are one word Prepositions such as at by for in of off for from on out through till to up with before amidst towards beyond between over etc

Compound Prepositions ndash There are some words that are always used with fixed Prepositions to convey specific meaning

Example I was unable to meet you dueto a previous engagement ( On account of)Always maintain the queue instead of crowding at the counter ( In place of)

Participial PrepositionsmdashParticiple Prepositions are present or past participles of various verbs which together with a noun phrase or a clause function as prepositions Examples- barring concerning considering notwithstanding pending regarding respecting etc

Exercise A

1 Gauravs fever has come down since Friday He has been absent for a week now

2 The child sat between his father and mother among the parents of all his classmates

3 There are mosquitoes in the room They flew into the room when the door was open

4 My father was inside the drawing room when I was playing outside my house

5 You may sit beside me I will give you a drawing book and pencils besides a storybook

6 We went to the market in the morning and walked towards the riverfront in the evening

7 The child walked along the pavement and across the street safely

8 This table top is made of glass My breakfast fell off it in the morning

9 The pan is on the gas stove There are vegetables in it

10 We will wait for you at the bus top There are a lot of people in the hall

Subject ndash Biology Topic ndash Chapter - 3 Photosynthesis and respiration in plants Summary Execution

All living organism (Plants and animals) need food for energy and growth Green plants (autotrophy) prepare food for all living organisms Today we will discuss about the process photosynthesis And adaptations in a leaf to carry out photosynthesis

Q1What do you mean by photosynthesis and write its word equation The process by which green plants make food (glucose) from carbon dioxide and water

in the presence of sunlight and chlorophyll is called photosynthesis

Carbon dioxide + Water ( Sun light from Sun ) Glucose + Oxygen ( chlorophyll in green leaves )

Q2 What are the adaptations in a leaf to carry out photosynthesisi) Leaves are broad wide and flat for absorbing more light energyii) Presence of chlorophyll in chloroplasts to trap sunlightiii) Presence of stomata which allow carbon dioxide to enter the cell and oxygen to go

out iv) Network of veins ensures continuous supply of water and minerals to the leafv) Thin waxy cuticle protects the leaf without blocking the lightQ3 Draw and label structure of chloroplast

Class VIIISubject Topic Summary Execution

PHYSICS ENERGY Production of Hydro electricity

A hydroelectric dam converts the potential energy stored in a water reservoir behind a dam to mechanical energymdashmechanical energy is also known as kinetic energy As the water flows down through the dam its kinetic energy is used to turn a turbine

The generator converts the turbinersquos mechanical energy into electricity

This electric energy then goes through various transmission processes before it reaches you

Question 2

Fill in the blanks

(a) Work is said to be done by a forte only when the body moves

(b) Work done = Force x distance moved in direction of force

(c) The energy of a body is its capacity to do work

(d) The SI unit of energy is joule

(e) The potential energy is due to its state rest of position and kinetic energy of the body is due to its state of motion

(f) Gravitational potential energy U = mass times force of gravity on unit mass times height

(g) Kinetic energy = frac12 times mass times (speed)2

(h) Power P = work donetime taken

(i) The S I unit of power is watt

(j) IHP = 746 W

BIOLOGY Chapter -5 The endocrine system and adolescence

Today we will discuss about thelocation and functions of secreted hormones of adrenal and Pancreas

Q5 Write location hormone secreted main functions and deficiency diseases of pancreas and adrenal glands

Endocrine Glands

Location Hormones secreted

Functions and Deficiency Diseases

1Adrenal gland

2 Pancreas Gland

On the top of each kidney

In between stomach and small intestine

i)Adrenaline from adrenal medulla

ii)Cortisone from adrenal cortex

i) Insulin

ii) Glucagon

It helps a person deal with any kind of emergency situation or emotional stressIt increases the heart beat rate of respiration and blood pressure

a) It regulates carbohydrates protein and fat metabolism

b) It regulates the salt and water balance in the body

a) It changes excess glucose into glycogen

b) It stimulates the cells to burn extra glucose to provide heat amp energy

Less secretion causes diabetes mellitus

Excessive secretions causeinsulin shock

a) It stimulates the breakdown of glycogen into glucose

b) It increases the level of glucose in blood

History Traders to rulers The Battle of Buxar was fought on 22 October 1764 between the forces under the command of the British East India Company led by Hector Munro and the combined armies of Mir Qasim the Nawab of Bengal till 1763 Mir Jafar was made the Nawab of Bengal for a second time in 1763 by the Company just after the battle After being defeated in 4 battles in katwa and Udaynala the Nawab of Awadh Siraj id Daula and the Mughal emperor Shah Alam II accompanied by Raja Balwant Singh of Kashi made an alliance with Mir Qasim The battle was fought at Buxar a small fortified

Answer the following questions- Short note-Battle of BuxarHomework-learn

town within the territory of Bihar located on the banks of the Ganga river about 130 kilometres (81 mi) west of Patna it was a decisive victory for the British East India Company The war was brought to an end by the Treaty of Allahabad in 1765

EnglishLiterature

The west wind-John Mansfield

In the poem The West Wind by John Masefield the poet starts by describingwith very poetic imagery of birds how the west wind is different from other winds its a warm wind full of birds cries There is a touch of melancholy perhaps home-sickness as he describes how it brings tears too and memories from an old land He goes on to describe the restful pastoral beauty of the land where even the dead can lie in the green He then brings in voicesperhaps of family and friends calling him home as he is missing Aprils beautyThe voices then tempt him some more with idyllic images from home (white blossom young green cornrunning rabbitswarm sun) The voices seem to presume that the poets heart is sorrowful bruised and soreThe end of the poem sees the poet appear to make a decision he will go home as he has decided that is where he truly belongs

Write the synopsis of the following words

1 Daffodils- a tall yellow flower that grows in the spring

2 Orchards- a piece of land on which fruit trees are grown

3 Blossom- a flower or a mass of flowers especially on a fruit tree in spring

4 Thrushes- a bird5 Larks- a small brown bird that

makes a pleasant sound6 Bruised- an injury7 Aching- pain 8 Tread- to put your foot down

while you are walking9 Balm-10 May-11 Fluting-

(Write from the book in your copy)

MAT

HEM

ATIC

S

Ch 1

1Al

gebr

ic E

xpre

ssio

n

1 Constant A symbol which has fixed value is called a constant[eg 8 23 -15 radic3 etc]

2 VariableA symbol which does not have any fixed value but may be assigned value (values) according to the requirement is called variable or literal[eg x y p q etc]

3 TermsA term is a number (constant) a variable a combination (product or quotient) of numbers and variables[eg 7 x 5x etc]

4 Algebric expressionA single term or acombination of two or more terms connected by plus (+) or minus (-) sign forms an algebraic expression[eg 5-y 3x2-5x xy-6z+4 etc]

5 PolynomialAn algebraic expression which contains more than one term is called a polynomial (multinomial)[eg x2-5x 5y+xy+x2y etc]

6 Degree of polynomial(a) When the polynomial contains only one variable the highest power of the variable is the degree of the polynomialeg the degree of the polynomial of 4x-7x5+8 is 5(b) When the polynomial contains two or more variablesStep (i) Find the powers of the variables in each term (ii) The highest sum of the powers is taken to be the degree of the polynomialeg the degree of the polynomial 5x2y-4x3y5+6 is = 3+5 = 8Remember An algebraic expression is a polynomial if degree of each term used in it is a non-negative integer

Exercise ndash 11(A)

1 Separate the constants and variables from the following

-7 7+x 7x+yz radic5 radic xy 3 yz

8 45y -3x

Solution Constant Variables-7 radic5 7+x 7x+yz radic xy

3 yz8

45y -3x

2 Write the number of terms in each of the following polynomials(i) 5x2+3timesax (ii) axdivide4-7 (iii) ax-by+ytimesz (iv) 23+atimesbdivide2

Solution Polynomials Number of terms(i) 5x2+3timesax 2(ii) axdivide4-7 2(iii) ax-by+ytimesz 3(iv) 23+atimesbdivide2 2

4 Write the degree of the each polynomials(i) xy+7z (ii) x2-6x3+8 (iii) y-6y2+5y8 (iv) xyz-3 (vi) x5y7-8x3y8+10x4y4z4

Solution Polynomials Degree(i) xy+7z 2(ii) x2-6x3+8 3(iii) y-6y2+5y8 8(iv) xyz-3 3(vi)x5y7-8x3y8+10x4y4z4 12

5Write the coefficient of(i) ab in 7abx (iv) 8 in a2-8ax+a (v) 4xy in x2-4xy+y2

SolutionCoefficient

(i) ab in 7abx 7x(iv) 8 in a2-8ax+a -ax(v) 4xy in x2-4xy+y2 -1

7 CoefficientAny factor of an algebraic quantity is called the coefficient of the remaining quantityeg in the algebraic term 7xyz 7 is coefficient of xyz 7x is coefficient of yz and so on

8 Like term The terms having the same literal coefficient are called like terms and those having different literal coefficients are called unlike terms

eg (i) 5xyz 8xyz -6xyz and 23xyz are like

terms(ii) 7xy2 8x2yz and -15xyz2 are unlike terms

6 in 57xy2z3 write the coefficient of

(i) 5 (vii) 5xy2 (viii) 17yz (xi) 5xyz

Solution Coefficient

(i) 5 17

xy2z3

(vii) 5xy2 17z3

(viii) 17yz

5xyzsup2

(xi) 5xyz 17yz2

7 In polynomial given below separate the like terms(ii) y2z3 xy2z3 -58x2yz -4y2z3 -8xz3y2 3x2yz and 2z3y2

Solution y2z3 -4y2z3 2z3y2 are like terms

xy2z3 -8xz3y2 are like terms

-58x2yz 3x2yz are like terms

Class IXSubject Topic Summary Execution

Bengali (2nd language)

বাগzwnjধারাzwnj বা ধারা-বা ধারা ল হিবকেশষ পরকার বাক -হিb -াকেবর এক হিবকেশষ পরকাশরীহিত াকেক কতগকেলা কার সমষটির মকেধয এগহিলকেক বা ধারা বকেল আবার কতগকেলা শকেবদর বাধাধরা যকান রীহিত যনই য-াকেব চকেল আসকে যসই -াকেবই চকেল আসকে তখন যসই শবদগহিল খন একক -াকেব অG পরকাশ ককের তখন একের বা ধারা বকেল বা ধারার পরকেয়া -াষাকেক আরও সFর ককের যতাকেল

অকাল পকক(অপহিরনত বয়কেস পাকাহিম)-মাতর শ বর বয়কেস যমকেয়টির া মকেখর কা তাকেত অকালপককতা ধরা পকে

অককা পাওয়া( মারা াওয়া) ndash পকেকIমারটি পকেকIমারকেত হিকেয় বাসাতরীকের াকেত মার যখকেত যখকেত অককা যপল

অহি| পরীকষা ( কঠিন ও পরকত পরীকষা)- যকেলটির আজ ডাকতাহির যরজালট যবকেরাকেব এIাই তার জীবকেনর ব অহি| পরীকষা

অষটরমভা (ফাহিক) ndash রীতা মকেখই বকো বকো কা বকেল আর কাকেজর যবলায় অষটরমভা

অকমGার ধাী (অপাG) ndash সমনকেক হিনকেয় যকান ান কেব না ও একেকবাকেরই অকমGার ধাী

অকেনধর ষটি (অসাকেয়র সায়)- আহিশ বকেরর বকোর নাহিত ল অকেনধর ষটি তাকেক াা বকোর একম চকেল না

আকেককল গড়ম (তবহিদধ)- ার তহিম উপকার করকেল যসই যতামার হিবরকেদধ সাকষয হিকেয়কে শকেনই আমার আকেককল গড়ম

আষাকে লপ( অবাসতব লপ) ndashIাকা এখন যকেব না এIা বলকেলই ত এমন আষাকে লপ ফাার যকান রকার হিল না

Hindi- महायजञ ा इस हानी म लख न या बतान ा परयास किया ह कि किसी भी अचछ

2nd language

परसार(यशपाल ाय या पणय न ा फल अवशय मिमलता ह ोई भी परोपार अथवा पणय लिलए किया गया ाय बार नही 0ाता वह ए परार ा यजञ हए धनी सठ थ धम परायण और किवनमर सठ न आन ी यजञ किए थ और दान म न 0ान कितना धन दिदन दखिखयो म बात दिदया थादिदन पलट और सठ यहा गरीबी आ गई उन दिदनो यजञ बचन ी परथा थी सठ भी अपनी 0गह बचन लिलए डलपर ए सट यहा चलन ो तयार हए सठानी रासत लिलए रोटी पड म बाधर सठ ो द दी रासत म ए भख R ो दखर सठ न चारो रोटी उसो खिखला दी खर वह सठ यहा डलपर पहच तो उनी सठानी न उस महायजञ बचन ो हा यदिद बचन आए सठ न R ो रोटी खिखलान ो महायजञ नही समझा और वापस लौट आया घर आर शाम ो उसी घर म उस ए बडा ख0ाना मिमला 0ो उस दवारा किए गएrsquo महायजञrsquo ा परसार था

English language

Letter formal The heading the name and address of the person you are writing to must be included beneath your own address In formal letters ldquoblock stylerdquo of address is preferred

Subject complain in brief

Salutation If the person you are writing to is known to you you may begin ldquoDear MrrdquoOr ldquoDear Mrsrdquo In all other instances you should begin ldquoDear Sirrdquo or ldquoDear Madamrdquo Or ldquoSirsrdquo

The body A formal or business letter has four partsReference The letter should begin by referring to a letter you have received an advertisement or the reason that has prompted you to writeInformation In the second paragraph it is necessary to supply more detailed information that is related to the referencePurpose Here you must give the reason why you are writing the letter This must be stated clearly and ensure that it is relevant to the question that has been setConclusion round off the letter with some polite remarkThe subscription when a letter has begun with dear sir sirs Madam you should end with Yours faithfully or yours truly When however you address a person by name you must conclude with the words ldquoYours sincerelyrdquo

1 A park in your locality is slowly being used as a rubbish dump Write a letter to the Mayor of your city pointing out the nuisance and danger of this Request that action be taken to stop this immediately

Or2 You being a boarder ordered a set of lab manuals from a famous book shop in the town They sent you a wrong set of books Write a letter to the manager of the book shop

Chemistry Chapter-1 1)CHEMICAL FORMULA- Q What is the Significance of

L-2The Language of Chemistrybull Chemical Formula

Itrsquos a symbolic representation of a chemical substance eg ndash The formula of Sulphuric acid is H2SO4

2) Steps of writing Chemical Formula of a given substance-

1 Write the symbols of the constituent atoms or radicals side by side Keep the basic radical on LHS and acid radical on the RHS ( Na+Cl- )2 In case of a radical having more than one atom( compound radical) enclose the radical in a bracket eg (SO4-)3 Write the valencies of each radical on its right hand top4 If the valencies of the two radicals are divisible by a common factor then divide the valencies by the common factor5 Invert (criss-cross) the valency number ie write the valency of one atom below the second atom and vice versa 6 On interchanging if valency number is lsquoone the figure lsquoonersquo is never writtenFor Example- Compound -Calcium Nitrate1 Writing the symbols- Ca(NO3)2 Writing the valencies on their right hand top- Ca2(NO3)1

3 Valency numeral in simple ratio- Ca2(NO3)1

4 Criss-cross- Ca 2NO3 1

5 Writing the formula of the compound- Ca(NO3)2

Chemical formula

A The formula of a substance conveys the following information regarding a substance 1 The name of the substance (qualitative)2 The elements constituting the substance (qualitative)3 The number of various atoms present in a molecule of the substance (quantitative)4 Molecular weight of the substance and the relative weights of different elements present in it (qualitative)

Q What are the limitations of Chemical Formula

A The chemical formula suffers from the following limitations-I It fails to convey whether the elements in a molecule are present in the form of atoms or ionsFor example the formula KBr fails to tell us whether Potassium and Bromine are present in the form of ions II It does not tell anything about the binding force that holds atom in a molecule togetherIII It does not tell us about the arrangement of various atoms with respect to one another within the molecule

Q Examples of Some Chemicals with their Formula Chemical name and Common Name-

A Given in the class notesCommercial Studies

Joint Stock Company

Let us discuss about the demerits of Joint Stock CompanyDespite so many advantages it has got many disadvantages which are as follows

Difficulty in FormationDelay in Decision makingExcessive Government ControlLack of Secrecy

Company can be classified into several categories based on incorporation

QuestionExplain the demerits of Joint Stock CompanyAnswer) 1 Difficulty in Formation The legal requirements and formalities required to be completed are so many The cost involved is quite heavy It has to approach large number of people for its capital It cannot start its business unless certificate of incorporation has been obtained This is granted after a long time when all the formalities are completed

Chartered CompanyStatutory CompanyRegistered Company

Delay in Decision making In this form of organization decisions are not made by single individual All important decisions are taken by the Board of Directors Decision-making process is time-consuming So many opportunities may be costly because of delay in decision-making Promptness of decisions which is a common feature of sole trader ship and partnership is not found in a company

Excessive Government ControlA company and the management have to function well within the law and the provisions of Companies Act are quite elaborate and complex At every step it is necessary to comply with its provisions lest the company and the management should be penalized The penalties are quite heavy and in several cases officers in default can be punished with imprisonment This hampers the proper functioning of the company

Lack of Secrecy The management of companies remains in the hands of many persons Every important thing is discussed in the meetings of Board of Directors Hence secrets of the business cannot be maintained In case of sole proprietorship and partnership forms of organisation such secrecy is possible because a few persons are involved in the management

2 Define the following

Chartered Company- The crown in exercise of the royal prerogative has power to create a corporation by the grant of a charter to persons assenting to be incorporated Such companies or corporations are known as chartered companies Examples of this type of companies are Bank of England (1694) East India Company (1600) The powers and the nature of business of a chartered company are defined by the charter which incorporates it After the country attained independence these types of companies do not exist

in IndiaStatutory Company- A company may be incorporated by means of a special Act of the Parliament or any state legislature Such companies are called statutory companies Instances of statutory companies in India are Reserve Bank of India the Life Insurance Corporation of India the Food Corporation of India etc The provisions of the Companies Act 1956 apply to statutory companies except where the said provisions are inconsistent with the provisions of the Act creating them Statutory companies are mostly invested with compulsory powersRegistered companiesCompanies registered under the Companies Act 1956 or earlier Companies Acts are called registered companies Such companies come into existence when they are registered under the Companies Act and a certificate of incorporation is granted to them by the Registrar

Economics

Chapter-4Basic problems of Economy

Today let us discuss with the topic Production Possibility curve

QuestionExplain the concept of Production Possibility Curve with the help of diagram

Answer) Production Possibility curve is a locus of all possible combinations of two commodities which can be produced in a country with its given resources and technology

The above diagram shows that with the given resources and technology the economy can produce maximum either 5 thousand meters of cloth or 15 thousand quintals of wheat or any other combination of the two goods like B( 1 thousand meters of cloth and 14 thousand quintals of wheat C ( 2 thousands meters of cloth and 12 thousand quintals of wheat) etcProduction Possibility curve is also called production possibility boundary or frontier as it sets the maximum limit of what it is possible to produce with given resources

Geography

Rotationand Revolution

SUNrsquoS POSITION AND SEASONAL CHANGES EQUINOXES ndash SPRING AND AUTUMN

Q1 What is Spring EquinoxA1 On 21st March sunrays fall directly on the equator On that day

As the Equator divides the Earth into two equal halves the sun rays fall directly on the equator twice in a year Equinoxes means equal Spring EquinoxOn 21st March sunrays fall directly on the equator On that day the duration of day and night both are equal ( 12 hours day and 12 hours night) on every places located on equator This day is called as Spring EquinoxAutumn EquinoxOn 23rd September sunrays fall directly on the equator On that day the duration of day and night both are equal ( 12 hours day and 12 hours night) on every places located on equator This day is called as Autumn Equinox

SOLSTICES ndash SUMMER AND WINTERDue to inclination of the Earth on its axis and the apparent movement of the sun the sun rays fall directly on both tropics once in a year Solstice is a Latin word which mean ldquothe Sun standing stillrdquoSummer SolsticesAfter 21st March there is an apparent movement of the Sun to the north of the equator The apparent northward movement up to 21st June when the Sun appears overhead at the Tropic of Cancer (22frac12degN) The sun appears to stand still at this position and then moves southwards towards the equator This position of the Sun on 21st June is known as Summer Solstices On that day the duration of day and night both are equal ( 12 hours day and 12 hours night) on every places located on Tropic of Cancer (22frac12degN)Winter solstices The apparent southward movement of the Sun continues beyond the equator till 22nd

December On this day the Sun is overhead at the Tropic of Capricorn

the duration of day and night both are equal ( 12 hours day and 12 hours night) on every places located on equator This day is called as Spring Equinox

Q2 What do you mean by EquinoxA2 Equinoxes means equal It is use to explain the equal duration of day and night ( 12 hours day and 12 hours night) on the Earth

Q3 On which date the longest day in Tropic of CancerA3 21st June

Q4 What is the meaning of SolsticeA4 Solstice is a Latin word which mean ldquothe Sun standing stillrdquo

Q5 Which is the longest day in southern hemisphereA5 22nd December

Q6 On what date does the Arctic Circle experience the lsquoMidnight SunrsquoA6 On 21 June the Arctic Circle experiences the lsquoMidnight Sunrsquo

Q7 What is cause of Midnight Sun in NorwayA7 During the summer solstice (21 June) the North Pole is inclined towards the Sun Therefore the duration of sunlight or daytime increases from 12 hours at the Equator to 24 hours at the Arctic Circle and beyond Thatrsquos why The region beyond the Arctic Circle especially Norway is known as the Land of the Midnight Sun because there the Sun does not rise or set on 21 June

Q8 Match the column A with BA B

Summer Solstice 21st March

Autumn Equinox 23rd

September

Winter Solstice 21st June

(22frac12degS) This position of the Sun is referred to as the Winter Solstice because it marks the winter season in the Northern Hemisphere On that day the duration of day and night both are equal ( 12 hours day and 12 hours night) on every places located on Tropic of Capricorn (22frac12degS)SEASONS AND DURATION OF DAY AND NIGHT During the equinoxes all places on the Earth have 12 hours of day and 12 hours of night Due to the revolution of the Earth round the Sun on an inclined axis the duration of day and night varies according to seasons and the latitude of a placeDuring the summer solstice (21 June) the North Pole is inclined towards the Sun Therefore the duration of sunlight or daytime increases from 12 hours at the Equator to 24 hours at the Arctic Circle and beyondThe region beyond the Arctic Circle especially Norway is known as the Land of the Midnight Sun because there the Sun does not rise or set on 21 JuneAt the North Pole there will be six months of daylight The Sun will be seen always above the horizon at a low angle At 66degN 24 hours of sunlight can be seen only on 21 June Hammerfest in northern Norway is a place of tourist attraction for observing the phenomenon of the Midnight Sun This place has continuous daylight from 13 May to 29 July This place is easily accessible to tourists and has hotels and other facilities The view of the midnight Sun from here is enthrallingIn the Southern Hemisphere the duration of daylight decreases from 12 hours at the equator to 0 hours beyond the Antarctic Circle In the South Polar Region there is 24 hours of darkness The Sun is always below the horizon In the Southern Hemisphere which experiences winter the duration of night-time is longer than the duration of daylight

Spring Equinox 22nd

December

A8 A B

Summer Solstice 21st June

Autumn Equinox 23rd

September

Winter Solstice 22nd

December

Spring Equinox 21st March

During winter solstice (22 December) the South Pole is inclined towards the Sun The Southern Hemisphere experiences summer and the Northern Hemisphere has winter Therefore the duration of daylight or sunlight is greater in the Southern Hemisphere than in the Northern HemisphereThe duration of daylight increases from 12 hours at the equator to 24 hours beyond the Antarctic Circle The South Polar Region has 24 hours of sunlight for many days continuously At the South Pole there will be six months of sunlight The Sun will always be seen at a low angle above the horizon In the Northern Hemisphere the duration of daylight will decrease from 12 hours at the equator to 0 hours at the Arctic Circle There are 24 hours of darkness in the North Polar region The duration of night is greater than the duration of daylight as one move northwards from the Equator It is evident from the above table that the duration of daylight is 12 hours throughout the year at the equator only As one moves away from the equator the seasonal variations in the duration of daylight increase The seasonal variations in the duration of daylight are maximum at the Polar Regions

Subject Eng Literature (The Merchant of Venice ndash William Shakespeare)Topic Act II Scene 7 Lines 36 to 80 (End of scene ) [Students should read the original play and also the paraphrase provided]

Summary Questions amp AnswersThe Prince then examines the inscription on the silver casket which says ldquoWho chooseth me shall get as much as he deservesrdquo The Prince says that he deserves Portia more than anybody else because of his high rank his noble birth and his great wealth and power But then he argues that silver is ten times

(1) (Act II Sc 7 L 39-47)

From the four corners of the earth they come

To kiss this shrine this mortal breathing saint

The Hyrcanian deserts and the vasty wildsOf wide Arabia are as through-fares now

inferior to gold and therefore he cannot believe that the portrait of such a beautiful lady as Portia can be contained in the silver casket He decides to see the inscription on the golden casket before making his decision

The Prince goes to examine the inscription on the golden casket which says ldquoWho chooseth me shall get what many men desirerdquo The Prince believes that the whole world desires to possess Portia otherwise so many suitors would not have come from all corners of the world for winning Portia Some of them have come from the distant lands of Persia and Arabia The deserts of Persia (Hyrcanian deserts) and the boundless desolate lands of Arabia have been crossed by the Princes seeking the hand of Portia He contrasts this casket containing Portiarsquos portrait with the old English gold coin bearing the image of the archangel (angel of the highest rank) He goes on to remark that while the figure of the archangel is engraved (Insculped) upon the English coin the picture of Portia who is beautiful as an angel lies hidden inside one of the caskets namely the Golden Casket (Golden Bed)

On the basis of his assessment of the inscription on the golden casket the Prince decides to choose the golden casket He asks for the key and opens the golden casket only to find therein an empty human skull holding a roll of

For princes to come view fair PortiaThe watery kingdom whose ambitious headSpets in the face of heaven is no barTo stop the foreign spirits but they comeAs orsquoer a brook to see fair Portia

(i) Explain the occasion for the above mentioned speech

These are the comments of the Prince of Morocco after he reads the inscription on the golden casket His mental process is revealed to us in these words We find him debating within himself as to which casket he should choose

(ii) What light does the above speech throw on the personality of Prince of Morocco

From the above mentioned speech we come to know that the Prince of Morocco is keen to marry Portia He is the type of person who is easily taken away by outward appearance He is in love with Portia because of her beauty

(iii) What information can you gather about Portia from the above mentioned lines

The given speech shows that Portia is a very beautiful lady She must be possessed of good qualities because many suitors come to her place from all over the world with a desire to get married to her The Prince of Morocco is so impressed by her beauty that he calls her a saint According to him the whole world is desirous of having her

(iv) Elucidate the significance of the first two lines

In these lines the Prince of Morocco pays a compliment to Portia These lines show his admiration for her He says that people come from all parts of the world to see fair Portia

(v) Explain the meaning of the last four lines of the

passage

In these lines the Prince of Morocco says that even the vast oceans which throw a challenge at the sky are unable to prevent men from coming to Portiarsquos place to have a glimpse of her These lines are also a tribute to Portiarsquos beauty and good qualities Many men voyage across the ocean treating it as a mere stream to see the beautiful Portia

paper in which is written that whoever happens to be guided by the glitter of things is invariably deceived

On reading the scroll the Prince says that he is too sad at heart to speak a more formal farewell and leaves with his followers amidst a sound of trumpets

After the Prince of Morocco leaves Portia remarks that the Prince is a gentle fellow but she is rid of him May all persons of his nature make a similar choice

IMPORTANT PASSAGES EXPLAINED

(Act II Sc 7 L 39-43)From the four corners of the earth they come

To kiss this shrine this mortal breathing saintThe Hyrcanian deserts and the vasty wildsOf wide Arabia are as through-fares nowFor princes to come view fair Portia

Context

This passage occurs in Act II Scene 7 in The Merchant of Venice This is part of the speech made by the Prince of Morocco

(2)

(Act II Sc 7 L 48-53)

MOROCCO One of these three contains her heavenly pictureIst like that lead contains her

Twere damnation To think so base a thought it were too grossTo rib her cerecloth in the obscure graveOr shall I think in silver shes immurdBeing ten times undervalued to tried gold

(i) What meaning does the Prince of Morocco find out of the inscription of the golden casket What have Belmont and Portiarsquos house been called and why

The inscription on the golden casket is ldquoWho chooseth me shall gain what many men desirerdquo The Prince finds out that it means that the chooser of the golden casket will get Portia because many men desire her In fact the entire world desires her Because of the coming of many suitors to Belmont from different countries in order to win Portiarsquos hand Belmont has become a centre of pilgrimage and her house is the shrine where saintly Portia is installed

(ii) What does the Prince of Morocco do before making the final choice of the casket Which is the correct casket and who will win Portiarsquos hand

The Prince of Morocco surveys and analyses the inscriptions on the casket of lead silver and gold Before making the final choice like a very systematic and methodical person he once again considers the claims of the caskets The casket containing Portiarsquos picture is the correct casket and the person choosing it will win Portiarsquos hand

Explanation

While praising Portia the Prince of Morocco conceives Portia as a goddess whose image is placed inside one of the caskets Many suitors are coming from far and wide the north and the south the east and the west (Four corners) in order to try their luck Some of them have come from the distant land of Persia and Arabia The deserts of Persia (Hyrcanian deserts) and the boundless desolate lands of Arabia have been crossed by the Princes seeking the hand of Portia All this shows that Portia is indeed the most beautiful lady of the world

(iii) What does the Prince of Morocco say in his estimation while examining the motto on the silver casket What does he find in the golden casket

While examining the motto on the silver casket which says ldquoWho chooseth me shall get as much as he deservesrdquo Morocco says that in his own estimation he surely deserves Portia in all respects ndash rank birth wealth etc

He chooses the golden casket When he opens it he finds an empty human skull holding a scroll in which it is written that those who are attracted by the glittering outside of things are always deceived as Morocco has been deceived

(iv) What kind of nature does the Prince of Morocco have

The Prince of Morocco has a simple nature who does not look deeply into the inner meaning of things but is dazzled by the outward appearance of gold He is inclined to over-estimate his own value and does not realize that it is a duty to ldquogive and hazardrdquo To say that he will not hazard for lead shows that he misreads the true meaning of the inscription which is that he should be prepared to ldquohazard all he hathrdquo for Portia So his feeling is only one of fascination and romantic attraction

(v) Do you think that the lottery of the caskets is not a matter that will be determined by chance

In fact the lottery of the casket is not a matter that will be determined by mere chance but that it is a true test of character and of sincerity which is amply proved not only by Moroccorsquos choice but also by the arguments which he uses to help him in his choice

(Act II Sc 7 L 55-59)

They have in England

A coin that bears the figure of an angelStamped in gold but thats insculpd uponBut here an angel in a golden bedLies all within

Context

(3)

(Act II Sc 7 L 63-77)A carrion Death within whose empty eye

There is a written scroll Ill read the writing

All that glisters is not goldOften have you heard that toldMany a man his life hath soldBut my outside to beholdGilded tombs do worms infoldHad you been as wise as boldYoung in limbs in judgment oldYour answer had not been inscrolld

This passage occurs in Act II Scene 7 in The Merchant of Venice This is part of the speech made by the Prince of Morocco

Explanation

In this passage the Prince of Morocco bestows high praise on Portia whose hand he is seeking He contrasts this casket containing Portiarsquos portrait with the old English gold coin bearing the image of the archangel (angel of the highest rank) He goes on to remark that while the figure of the archangel is engraved (Insculped) upon the English coin the picture of Portia who is beautiful as an angel lies hidden inside one of the caskets namely the Golden Casket (Golden Bed) In the day of Elizabeth silver was ten times inferior in value to gold Therefore the Prince of Morocco believing that Portiarsquos portrait is contained in the Golden Casket decides to choose the Golden Casket

Fare you well your suit is coldCold indeed and labour lostThen farewell heat and welcome frostmdashPortia adieu I have too grievd a heartTo take a tedious leave Thus losers part

(i) What reward does the Prince of Morocco get after making a wrong choice of the Casket How does he feel

After making the wrong choice in selecting the casket of gold the Prince of Morocco as a reward earns a rebuke in the form of a scroll tucked in the empty eye-socket of a skull kept in the casket of gold The Prince is shocked and disappointed He becomes all the more sad and dejected when he reads the scroll which points to his foolishness in being misled by the appearance and outward show as indicative of its worth

(ii) How does the Prince respond after reading the scroll

After reading the scroll the Prince though upset accepts the result with good grace and decorum befitting a royal suitor and true sportsman He says that his love-suit is really cold otherwise he would have chosen correctly but now his efforts have been in vain So he bids farewell to Portia to the warmth and enthusiasm of love and welcomes the cold and bitterness of dejection and misery of life which lies ahead

(iii) What request does he make to Portia and why

After being failure in his mission he requests Portia to give him permission to leave at once because he is too sad to undergo the tediousness of a formal leave-taking He tells that it is the manner in which defeated persons part unceremoniously

(iv) Explain the following lines

ldquoAll that glisters is not goldOften have you heard that toldMany a man his life hath soldBut my outside to beholdGilded tombs do worms infoldrdquo

Mere glitter does not make a metal to be gold Man has often been warned against appearance but it has been of no use Many people have sacrificed their lives only to seek the outer appearance of gold Worms are found inside the gilded

monuments

Class XSubject Topic Summary Execution

Hindi 2ndlang

नया रासता भाग 6 मायाराम 0ी घर म धनी मल 0ी और उनी बटी सरिरता ी ही चचा बनी रहती थी अमिमत ो इसम ोई रलिच ना थी वह धनी घर ी लडी स शादी र सवय ो बचना नही चाहता था उसा भी सवाणिभमान ह ईशवर ी पा

स उस पास पस ी ोई मी नही थी अभी उसन फकटरी ही लगाई थी उसी समझ बाहर था कि उस घर वालो ा झाव पस ी तरफ कयो

ह उसन मा स सवाल किया कि मा तम सरिरता स मरी शादी कयो रना चाहती हो मा न उस समझाया कि वह दखन म बरी नही ह और किफर खानदान अचछा

ह वह ए शल गरहणी रप म घर सभाल सगी अमिमत न मा ो इस बात ा एहसास राया कि मीन सबध लिलए मना रन पर उस दिदल

पर कया बीती होगी मा और अमिमत ी लडी बार म ाफी बात हईमा ा झाव सरिरता ी तरफ था कयोकि वह घर पर अचछा दह0 लर आ रही

थी अमिमत न अपनी मौसी ी बरी हालत बार म बताया कि किस तरह वह बड घर ी खानदानी बटी लाई थी और आ0 उसी हालत कितनी खराब ह लाई थी बहकलब 0ाती ह और बचचो ो भी नही दखती ह बात चल ही रही

थी कि तभी ए ार बाहर आर री धनी मल0ी घर अदर आए और पीछ स डराइवर फल ी ए टोरी लर आया अदर आए और पीछ स

डराइवर ए टोरी फल ी लर आया अमिमत ो फल ी पटी बरी लग रही थी अमिमत न पछ लिलया यह फल कयो ल आए ह प इन सब ी कया

0ररत थी उनो न 0वाब दिदया कि 4 पटी शमीर स मगाए थ अमिमत ो या सनर करोध आ गया तभी उस किपता 0ी आ गए उन आत ही अमिमत उठर बाहर चला गया वहा वहा मा पास आर बठ गया और बोला

अभी रिरशता तय नही हआ और धनी मल 0ी धनी मल 0ी फल ी पटी लर चलआय मा न समझाया कि 0ब सबध 0ड 0ाता ह तो खाली हाथ नही

आत अमिमत न मा स हा कि तम सबन सरिरता ो इस घर म लान ी ठान रखी ह धनीमल 0ी उस दिदन सरिरता ो दखन ी तारीख तय रन आय थ

Commercial Studies

Banking Nowadays Bank provide easy and quick services through internet facilities methods of Banking is called internet bankingIn order to save the time and money involved in visiting Bank branches people increasingly prefer to have internet banking

There are different modes of doing internet banking or transferring money through online They areReal Time Gross Settlement (RTGS)National Electronic Fund Transfers (NEFT)

1

Question

1) Explain the term RTGS Write the features of RTGS

Answer)The acronym RTGS stands for Real Time Gross Settlement which may be defined as the continuous real time settlement of funds transfer individually on and order by order basis without netting lsquoReal timersquo may be defined as the processing of instructions at the time they are received rather than at some letter time lsquoGross settlementrsquo may be defined as the settlement of transfer instructions which occurs

individually

Features of RTGS1It is the continuous settlement of

funds transfer individually on an order by order basis

2RTGS facility is provided only by CBS core banking solution enabled Bank branches

3Amount charged from the customer for RTGS transactions vary from bank to bank

2) Explain the term NEFT Write the features of NEFT

Answer) National electronic funds transfer may be defined as a nationwide system that facilitates individuals Farms and copper operates to electronically transfer funds from any bank branch to any individual farm or corporate having an account with any other bank branch in the country

Features of NEFT2 Transfer can be made 7 times on

weekdays and 6 times on Saturday

3 NEFT cannot be used to receive foreign remittances

4 NEFT transaction takes place in batches

5 A bank branch must be NEFT enabled to become a part of NEFT fund transfer network

6 There is no maximum or minimum amount that can be transferred through NEFT when one bank has a bank account

English Language

CompositionEssay

A composition is an art of creating a piece of writing on any topic or subject It is the writing correctly beautifully and clearly in order to make some interesting reading Structure of the composition

Introduction ( you lay the foundation for your composition)

Body (it constitutes the main part of the essay)

Conclusion (final statement that leaves a lasting impression)

Kinds of essays1 The Narrative essay2 The descriptive essay3 The reflective essay4 The argumentative essay

Write a composition on any one of the following topics (350- 400 words)

1 Friendship Or2 The first day of your school

Subject Eng Literature (The Merchant of Venice ndash William Shakespeare)Topic Act V Scene 1 Lines 127 to 158 (Nerissa helliphellip The clerk will nersquoer wear hair onrsquos face that had it) [Students should read the original play and also the paraphrase given in the school prescribed textbook]

Summary Revision Questions o Soon thereafter Bassanio Gratiano

and Antonio arrive

o Bassanio tells Portia that he is feeling as if it is morning because of the presence of Portia who is shining like the sun When Antonio is introduced by Bassanio to Portia she tells Bassanio that he should be grateful to Antonio who took so much trouble on his account even to the extent of risking his life

o Nerissa starts quarrelling with Gratiano and demands that he show her the ring she had presented to him and which she had warned him not to lose She suspects that Gratiano must have presented the ring to some young woman and not to the lawyerrsquos clerk as he repeatedly says and assures

Answer the following questions to check your preparation of Act IV Scenes 1 and 2

You must attempt only after you have completed your preparation of Act IV The answers must be in complete sentences using textual evidence (with citation) when necessary

[It would be in your own interest to attempt the above questions honestly totally refraining from consulting your textbook or your notes during answering After completion you should correct the paper yourself consulting the textbooknotes etc and award marks as specified Please let me know the marks you scored through WhatsApp in the group or to my personal WhatsApp]

Act IV Scene 1 (each question carries 2 marks)

1 What did the Duke try to do for Antonio

2 Why does Shylock refuse to show mercy How does he justify his stance

3 Why does Antonio say he is ready to die 4 What information is contained in Bellariorsquos letter

5 Why does Portia (as Balthazar) assert that Shylock must show mercy How does he respond

6 What offers are made to Shylock to get him to spare Antonio How are they received

7 What does Antoniorsquos speech as he faces the prospect of Shylockrsquos knife tell you about his character

8 How do Bassanio and Gratiano react to the looming prospect of Antoniorsquos demise

9 How does Portia (as Balthazar) use the law to turn the tables on Shylock

10 What does the Duke decree should happen to Shylock Why What happens to Shylockrsquos estate

11 What does Portia ask Bassanio as payment for her ldquoservicesrdquo What is his initial response What makes him change his mind

Act IV Scene 2 (each question carries 1frac12 marks)

1 What does Gratiano bring to Portia (Balthazar)

2 What does Nerissa plan on getting from Gratiano What does Portiarsquos comment suggest about men

ECO-10 280620 Topic-Supply AnalysisSHIFTING OF SUPPLY

But if there is change in factors other than the price of the commodity then either more is supplied at the same price or less supplied at the same price In such cases the price of the commodity remains constant but there is a change in other factors like change in the price of inputs change in technology of production change in price of other related goods change in taxation policy of the government etc For example there is an improvement in the technology of production of the commodity in question It leads to decrease in per unit of cost production of the commodity The firm is willing to sell more quantity of the commodity at the same price So the supply other commodity increases at the same price This increase in supply is shown by rightward shift of supply curve On the other hand if the firm uses inferior technology of production the cost of production per unit of the commodity increases The firm is willing to sell less quantity at the same price So the supply of the commodity decreases at the same price This decrease in supply is shown by leftward shift of the supply curve The above cases of increase and decrease in supply can be shown with the help of the following figures

Y INCREASE OF SUPPLY Price (Rs) s

P A s1

B

s

X` O s1 X

q q1

Y` Quantity demanded (in units)

Y DECREASE IN SUPPLY s2

s

price (Rs)

C

p A

s2

s

X` o X

q2 q

Y` Quantity demanded ( in units)

Main factors causing increase in supply or rightward shift of supply Curve(i) Fall in the price of other related goods

(ii) Fall in the price of inputsfactors(iii) Use of better technology in production(iv) Decrease in the rate of excise duty by government(v) If the objective of producer changes from profit maximization to salesMaximization

Main factors causing decrease in supply or leftward shift of supply curve(i) Increase in the price of other related goods(ii) Rise in the price of inputsfactors(iii) Use of inferior technology in production(iv) Increase in the rate of excise duty by the government(v) If the objective

Subject - Biology Topic ndash Chapter mdash6 PhotosynthesisSummary Execution

Today we will know about photosynthesis and its stages

Q1 What do you mean by photosynthesis The process by which living plants containing chlorophyll produce food

substances from carbon-di- oxide and water by using light energy Sunlight

6CO2 +12 H2O----------------------- C6 H12O6 + 6H2O + 6O2

Chlorophyll

Q2 What are the importance of photosynthesis I) Food for all Green plants trap solar energy by photosynthesis

process and supply food and energy for all living organisms either directly or indirectly

Ii) Oxygen to breathe in by product of photosynthesis is oxygen which is essential for all living organisms respiration

Q3 Write about two main phases of photosynthesis A Light dependent phase This phase occur in grana of chloroplast I) The chlorophyll on exposure to light energy becomes activated by

absorbing photons Ii) The absorbed energy is used in splitting the water molecules (H2O)

into its two components (H+ and OH- ) and releasing electron s 2H2O------------------------- 4H+ + 4e- +O2

Energy of 4 photons This reaction is known as photolysis

End products are H+ and oxygen water

B Light independent (Dark ) phase The reactions in this phase require no light energy

Here CO2 combine with H+ and produce glucose

Class XI

Subject Topic Summary ExecutionEVS Chapter-4 Legal

regimes for sustainable development

Environmental legislationEnvironmental legislation is the collection of laws and regulations pertaining to air quality water quality the wilderness endangered wildlife and other environmental factors The act ensures that matters important to the environment are thoroughly

Learn -The Forest (Conservation) Act 1980

considered in any decisions made by federal agencies

The Forest (Conservation) Act 1980 The Forest (Conservation) Act 1980 an Act of the Parliament of India to provide for the conservation of forests and for matters connected therewith or ancillary or incidental thereto It was further amended in 1988 This law extends to the whole of IndiaObjects and Reasons of the Forest Conservation Act

Deforestation causes ecological imbalance and leads to environmental deterioration Deforestation had been taking place on a large scale in the country and it had caused widespread concern The act seeks to check upon deforestation and de-reservation of forests

Subject Eng Literature (The Tempest ndash William Shakespeare) Topic Act II Scene 1 Lines 314 to 329 (End of scene)

[Students should read the original play and also the paraphrase given in the school prescribed textbook]Summary Questions amp Answers

Conspiracy of Antonio and Sebastian (Contd)

o As they approach Ariel appears again and wakes up Gonzalo by singing a tune in his ear Alonso also wakes up and they see both Sebastian and Antonio with drawn swords On being caught off guard they make up a story saying that they had heard a bellowing of bulls or lions

o They then moved to another part of the island

o Ariel at once rushes to Prospero to inform him of this development

SUMMING-UP of ACT-2 SCENE-1

(i) Among the survivors Ferdinand is separated from the rest which results in the disconsolate grief of Alonso as he took him for dead

(ii) The villainy of Antonio is confirmed

(iii) The supremacy of Prosperorsquos magic which resulted in the failure of the human conspiracy

(1)

(Act II Sc 1 L 311-325)SEBASTIAN Whiles we stood here securing your repose

Even now we heard a hollow burst of bellowing Like bulls or rather lions Didt not wake youIt struck mine ear most terribly

ALONSO I heard nothingANTONIO O rsquotwas a din to fright a monsters ear

To make an earthquake Sure it was the roarOf a whole herd of lions

ALONSO Heard you this GonzaloGONZALO Upon mine honour sir I heard a humming

And that a strange one too which did awake meI shaked you sir and cried As mine eyes opened I saw their weapons drawn There was a noiseThats verily rsquoTis best we stand upon our guardOr that we quit this place Lets draw our weapons

(i) Why has Prospero sent Ariel to Gonzalo and Alonso What does Ariel do to awaken Gonzalo

Prospero has already come to know by his magic powers the danger which threatens Gonzalo who had been Prosperorsquos friend and so he sent Ariel to preserve the lives of both Gonzalo and Alonso Prospero does not want that his scheme should remain unfulfilled Ariel begins to sing a song in Gonzalorsquos ears to awaken him(ii) Who are ready to carry out their plan Who takes steps to stop them Why does Gonzalo feel surprised after being awakened

Sebastian and Antonio are ready to carry out their plans They are standing with their swords drawn to kill Alonso and

(iv) We see two sets of contrasting characters Gonzalo-Adrian against Antonio-Sebastian

(v) The grief that works in Alonso can be perceived to his repentance for his association in Antoniorsquos crime against Prospero

Gonzalo Ariel takes steps to stop them from carrying out their nefarious scheme When Gonzalo is awakened by the song sung by Ariel into his ears he (Gonzalo) feels surprised because he sees Sebastian and Antonio standing with their swords drawn(iii) What reason do Sebastian and Antonio tell of drawing their swords when they are suspected by Alonso and Gonzalo

When Sebastian and Antonio are seen with their swords drawn they are looked with suspicion by Gonzalo and Alonso At first Sebastian tells them that as they stood here to guard them during their sleep they heard only a little before a sudden loud noise very much like the roaring of bulls or more probably that of lions Then Antonio follows him saying that this was a noise so terrible as to frighten even a monsterrsquos ears and this noise could even have shaken the earth and it was surely like the roaring of a multitude of lions Then seeing the danger they have drawn their swords Perhaps after hearing the terrible noise they (Gonzalo and Alonso) woke up from their sound sleep

(iv) What does Gonzalo tell Alonso about the strange noise What did he see on opening his eyes Gonzalo tells Alonso that he did not hear the sound of roaring but he heard a humming sound which was strange and which woke him up After waking up he gave him (Alonso) a shaking and a loud cry On opening his eyes he saw these two gentlemen standing with their swords drawn(v) What does Gonzalo suggest

Gonzalo suggests that there was a noise indeed and of that he has no doubt at all and suggests that the best course for them would be to remain alert and vigilant against any possible danger to their lives or to leave this place and move to some other part of the island

Class XIISubject Topic Summary Execution

Commerce

Chapter- Management

Today we will discuss about LEVELS OF MANAGEMENT

Levels of management is a series or chain of managerial positions from top to bottom It helps individuals to know their authority responsibilities and superior-subordinate relations among themselves There are mainly three levels of Management TOP LEVEL MANAGEMENTMIDDLE LEVEL MANAGEMENTLOWER LEVEL MANAGEMENT

Top level managementIt consists of members at the highest level in the management hierarchy This level includes Board Of Directors Chief Executive Managing Directors Chairman President Vice President

Rolefunctions of the top levelmanagement1To analyse evaluate and deal

with theexternal environment2 To determine the objectives and

policies of the business3 To strive for welfare and survival

of business

4 To create an organisational Framework consisting of authority responsibility relationship

Middle level management Congress of members or groups who are concerned with implementation of the policies let down by the top managementThis level includes head of the department such as finance manager marketing manager branch and regional managers departmental and divisional heads plant superintendent etc

Role of functions of the middle level management

1 To interpret the policies framed by top management

2 To assign duties and responsibilities to lower level managers

3 To select and appoint employees for middle and supervisory level and evaluate their performance

4 To co-operate with other departments for smooth functioning

Operational or supervisory level managementIt refers to the group are members who are concerned with execution of the work They are also known as fast line managers This level includes supervisor 4 men Section Officer clerk Inspector etc

Role of functions of the lower level management1 To plan and execute day-to-

day operations2 To supervise and control the workers3 To arrange materials and

tools to start the process and make arrangements for training

4 Today present workers grievance and suggestions before the management and

ensure safe and proper working conditions in the factory

Business Studies

Staff Appraisal Chapter- 10 Today let us start with a new chapter

Staff Appraisal

Meaning of Performance Appraisal

Performance Appraisal is the systematic evaluation of the performance of employees and to understand the abilities of a person for further growth and developmentThe supervisors measure the pay of employees and compare it with targets and plansThe supervisor analyses the factors behind work performances of employeesThe employers are in position to guide the employees for a better performance

Objectives of Performance Appraisal

Following are the objectives of Performance Appraisal

To maintain records in order to determine compensation packages wage structure salaries raises etc

To identify the strengths and weaknesses of employees to place right men on right job

To maintain and assess the potential present in a person for further growth and development

To provide a feedback to employees regarding their performance and related status

To provide a feedback to employees regarding their performance and related status

Importance of Performance Appraisal

Performance appraisal provides important and useful information for the assessment of employees skill

knowledge ability and overall job performance The following are the points which indicate the importance of performance appraisal in an organization

1 Performance appraisal helps supervisors to assess the work performance of their subordinates

2 Performance appraisal helps to assess the training and development needs of employees

3 Performance appraisal provides grounds for employees to correct their mistakes and it also provides proper guidance and criticism for employees development4 Performance appraisal provides reward for better performance

5 Performance appraisal helps to improve the communication system of the organization

6 Performance appraisal evaluates whether human resource programs being implemented in the organization have been effective

7 Performance appraisal helps to prepare pay structure for each employee working in the organization

8 Performance appraisal helps to review the potentiality of employees so that their future capability is anticipated

Geography

DRIANAGE The SubarnarekhaThe Subarnarekha and the Brahmaniinterposed between the Ganga and the Mahanadi deltas drain an area of 19300 sq kmand 39033 sq km respectively The drainage basins of these streams are shared byJharkhand Odisha west Bengal and Chhattisgarh The Brahmani is known as southKoel in its upper reaches in Jharkhand

The NarmadaThe Narmada rises in the Amarkantak hills of MadhyaPradesh It flows towards the West in a rift valleyformed due to a geological fault The total length of it is 1300 km All the tributaries of the

Q1 Name the two westward flowing rivers in the peninsular plateauA1 Narmada and Tapi are the only westward flowing rivers of the peninsular plateau

Q2 Differentiate between east-flowing rivers and west-flowing riversA2

East-flowing rivers

West-flowing rivers

Narmada are very short inlength Most of its tributaries join the main streamright anglesThe Narmada basin covers parts of Madhya Pradesh and Gujarat

The Tapi The Tapi rises in the Satpura ranges in the Betul listrictof Madhya Pradesh It flows in a rift valley parallel tothe Narmada but it is much shorter in length It coversparts of Madhya Pradesh Gujarat and MaharashtraThe length is about 724 km

The Sabarmati and the MahiThe Sabarmati rises in the Aravali hills and flows south-south-westwards for a distance of 300 kilometres to the Arabian Sea The Sabarmatibasin extends over an area of 21674 sq km in Rajasthan and Gujarat The Mahi rises inthe east of Udaipur and drains an area of 34842 sq km lying in Madhya PradeshRajasthan and Gujarat It flows south-westwards for a distance of 533 km before it fallsinto the Gulf of Khambhat

The ChambalThe Chambal rises near Mhow in the Vindhya Range and flows towards the northgenerally in a gorge upto Kota Below Kota it turns to the north-east direction and afterreaching Pinahat it turns to the east and runs nearly parallel to the Yamuna beforejoining it in the southern part of the Etawah district in Uttar PradeshMajor Rivers of India with their basin area (Sqkm)

Himalayan System Indus 321290Ganga 861404

Brahmaputra 187110Indus System

Jhelum 34775Beas 20303

Ganga System Yamuna 366223Ghaghra 127950

Peninsular RiversNarmada 98796

Tapi 65145Mahanadi 141600

Subarnarekha 19300Sabarmati 21674

Mahi 34842Godavari 312812

Godavari Krishna Kaveri Mahanadi are the east-flowing rivers

Narmada Tapi west-flowing rivers

They fall into the Bay of Bengal

They fall into Arabian Sea

These rivers form big deltas

These rivers form comparativelysmall deltas

Catchment areas of these rivers are larger

Catchment areas of these rivers are smaller

Krishna 2589488Cauveri 87900

Subject ndashBiology Topic ndashChapter -5 Inheritance amp Variations Summary ExecutionToday we will discussabout linkage and its classification

LINKAGE The tendency of the genes located on the same chromosome to stay together is

hereditary transmission Linked genes the genes responsible for this Genes that exhibit the process of linkage locates in the same chromosome The distance between the linked genes in a chromosome determines the strength

of linkage i e genes that are located close to each other show stronger linkage than that are located far from each other

COMPLETE LINKAGE It is the type of linkage showed by the genes that are closely located or are tightly

linked with each other as they have no chance of separatingby crossing over These genes are always transmitted together to the same gamete and the same

offspring In such condition only parental or non cross over type of gametes are formedINCOMPLETE KINKAGE It is type of linkage showed by the genes that are distantly located orare loosely

linked with each other because they have chance of separating by crossing over

SIGNIFICANCE i) It helps in holding the parental character togetherii) It checks the appearance of new recombination and helps in bringing the

hybrid population which resembles the original parents iii) Linked genes dilute the effects of undesirable traits

Subject Eng Literature (The Tempest ndash William Shakespeare) Topic Essay Questions (EQ-3)Question No 3

Give a character sketch of CalibanAnswer

The character of Caliban has been wonderfully conceived by Shakespeare as the manifestation of all that is gross and earthy ndash a sort of creature of the earth as Ariel is a sort of creature of the air

Calibanrsquos Physical Appearanceo Caliban is lsquofreckledrsquo a lsquomisshapen knaversquo not honoured with human shape

o Prospero calls him lsquothou tortoisersquo (Act I Sc 2 Line 317) Trinculo stumbling upon him describes him as ldquoA strange fish hellip Legged like a man And his fins like armsrdquo He ldquosmells like a fishrdquo (Act II Sc 2 Line 25)

o Prospero also calls him a ldquobeastrdquo (Act IV Sc 1 Line 140) and ldquoThis misshapen knaverdquo (Act V Sc 1 Line 268)

o Further it appears that in addition to his physical deformity his spiritual inferiority is also suggested by Prosperorsquos claim that his birth resulted from the union between his mother the witch Sycorax and the devil

Calibanrsquos ParentageWhen the play opens Caliban is twenty four years of age having been born on the island twelve years before the coming of Prospero His mother was the foul witch Sycorax who was banished from Algiers for ldquomischiefs manifold and sorceries terrible to enter human hearingrdquo (Act I Sc 2 Line 264) and the father was the Devil himself Thus

Caliban is a monster of evil and brute nature ugly deformed and stinking

Calibanrsquos Savage and Malignant Natureo Caliban is entirely a creature of the earth ndash gross brutal and savage He regards himself as the rightful possessor

of the island and Prospero as a usurper

o In his young age he was on good terms with Prospero He had consented to be received by Prospero at his house and to be educated by him He has learnt human language only to curse his master whom he abhors

o His beastly nature soon breaks out and ends in a vicious attack on Miranda This opens the eye of Prospero who becomes severe to him and enforces his service by threats and violence

o Prospero uses him to make dams for fish to fetch firewood scraper trenches wash dishes and keep his cell clean

Calibanrsquos Hatred for ProsperoA profound hatred for Prospero has taken hold of Caliban It springs from a sense of his being dispossessed and ill-treated He would kill Prospero if he could but he knows the power of Prosperorsquos lsquobookrsquo Hence he transfers his allegiance to Stephano who seems like a god to him He also incites the two drunken associates to batter the skull of Prospero when he sleeps in the afternoon

Caliban Shows Considerable Intelligenceo He has learnt Prosperorsquos language

ldquoYou taught me language and my profit onrsquot (Act II Sc 2 Lines 86-89)Is I know how to curserdquo

o He is well aware of the futility of arguing with one who has more power than he has

ldquoI must obey his art is such power (Act I Sc 2 Lines 373-376)It would control my damrsquos god SetebosAnd make a vassal of himrdquo

o He realizes the importance of Prosperorsquos books

ldquoRemember (Act III Sc 2 Lines 89-92)First to possess his books for without themHersquos but a sot as I am nor hath notOne spirit to commandrdquo

o He knows the value of stealth when attacking the enemy

ldquoPray you tread softly that the blind mole may not (Act IV Sc 1 Lines 194-195)Hear a foot fall we now are near his cellrdquo

o Caliban has a better set of values than Stephano and Trinculo They are distracted from their plan by their greed for Prosperorsquos rich garments Only Caliban realizes that such a finery is unimportant

ldquoLeave it alone thou fool it is but trashrdquo (Act IV Sc 1 Lines 224)

Caliban is not a good judge of characterCaliban is not a good judge of character He decides for example that Stephano is a god because he dispenses lsquocelestial liquorrsquo (Act II Sc 2 Line 115) but then it must be remembered that he has only known his mother Sycorax Prospero Miranda and the spirits that torture him However he quickly discovers his error of judgementrdquo

ldquoWhat a thrice-double ass (Act V Sc 1 Lines 295-297)Was I to take this drunkard for a godAnd worship this dull foolrdquo

Calibanrsquos Imaginative NatureIf Caliban is sub-human in what has been said above he is human in the respect of the poetic side of his character He listens to music with rapture He tells of the beautiful dreams in which heaven rains treasures upon him and which upon waking he yearns to renew One of the most poetic passages in whole play is Calibanrsquos description of the island

to Stephano and Trinculo

ldquoBe not afeard The isle is full of noises (Act III Sc 2 Lines 135-143)Sounds and sweet airs that give delight and hurt notSometimes a thousand twangling instrumentsWill hum about mine ears and sometime voicesThat if I then had waked after long sleepWill make me sleep again and then in dreamingThe clouds methought would open and show richesReady to drop upon me that when I wakedI cried to dream againrdquo

Caliban - Less Ignoble Than Some OthersCalibanrsquos motive for murder is less dishonourable than that of Antonio and Sebastian They plan to kill Alonso to gain his power and wealth Caliban merely wants revenge and the return of lsquohisrsquo island

Conclusiono Calibanrsquos character is not portrayed very clearly in the play and hence we cannot decide whether he is a poor

savage being grossly maltreated by Prospero or whether he is evil and must therefore be kept in bondage or enslavement

o Caliban is contrasted with Ariel who is a spirit and thus swift and uninterested in physical activitieso Caliban is also contrasted with Prospero who is the all-powerful master of the island and of the destiny of all

those on the islando Caliban is also contrasted with civilized man showing him to be less evil than Antonio and Stephano and less

materialistic than Stephano and Trinculoo Caliban has suffered at the hands of Prospero and he has learnt to curse by listening to Prosperorsquos abuse He

certainly believes that Prospero has deprived him of his birthrighto Finally the character Caliban is thought to be one of Shakespearersquos masterpieces The complexity of the character

is reflected in the large volume of critical discussion that has grown around it

ECO ndash12 Topic-Forms of market

MonopolyMonopoly is a market structure in which there is a single seller there are no close substitutes for the commodity produced by the firm and there are barriers to entry Example Indian Railways which is operated under government of India Monopoly also implies absence of competitionFeatures of Monopoly Monopoly is characterized by1 Single Seller In monopoly there is only one firm producing the product The whole industry consists of this single firm Thus under monopoly there is no distinction between firm and industry Being the only firm there is significant control of the firm over supply and price Thus under monopoly buyers do not have the option of buying the commodity from any other seller They have to buy the product from the firm or they can go without the commodity This fact gives immense control to the monopolist over the market

2No Close Substitute There are no close substitutes of the product produced by the monopolist firm If there are close substitutes of the product in the market it implies presence of more than one firm and hence no monopoly In order to ensure a total of control over the market by the monopolist firm it is assumed that there are no close substitutes of the product

3 No Entry amp Exit Monopoly can only exist when there is strong barriers before a new firm to enter the market In fact once a monopoly firm starts producing the product no other firm can produce the same One reason for this is the ability of the

monopolist to produce the product at a lower cost than any new firm who thinks to enter the market If a new firm who knows that it cannot produce at a lower cost than the monopolist then that firm will never enter the market for fear of losing out in competition Similarly the monopolist who is operating for a long time may be enjoying reputation among its customers and is in a better position to use the situation in its own benefit A new firm has to take long time to achieve this and so may not be interested to enter the market

4 Price Maker Being the single seller of the product the monopolist has full control over the pricing of the product On the other hand if there is a large number of buyers in the market so no single buyer exercises any significant influence over price determination Thus it is a sellerrsquos market So monopoly firm is a price maker

5 Price Discrimination Having considerable control over the market on account of being single seller with no entry of other firms the monopolist can exercise policy of price discrimination it means that the monopolist can sell different quantities of the same product to a consumer at different price or same quantity to different consumers at different prices by adjudging the standard of living of the consumer

6 Shape of Demand Curve Since a monopolist has full control over the price therefore he can sell more by lowering the price This makes the demand curve downward sloping

Subject Ac-12 290620 Topic- retirement Model sumThe Balance Sheet of Rohit Nisha and Sunil who are partners in a firm sharing profits according to their capitals as on 31st March 2014 was as under

Liabilities Amount Assets Amount (Rs) (` Rs)

Creditors 25000 Machinery 40000Bills Payable 13000 Building 90000General Reserve 22000 Debtors 30000Capital Less Provision for Rohit 60000 Bad debts 1000

29000 Nisha 40000 Stocks 23000 Sunil 40000 140000 Cash at Bank 18000

200000 200000

On the date of Balance Sheet Nisha retired from the firm and following adjustments were made(i) Building is appreciated by 20(ii) Provision for bad debts is increased to 5 on Debtors(iii) Machinery is depreciated by 10(iv) Goodwill of the firm is valued at Rs 56000 and the retiring partnerrsquos share is adjusted

(v) The capital of the new firm is fixed at Rs120000 Prepare Revaluation Account Capital Accounts of the partner and Balance Sheet of the new firm after Nisharsquos retirement Revaluation AccountDr Cr

Particulars Amount Particulars Amount (`Rs) (Rs`)

Provision for Bad debt Ac 500 Building Ac 18000Machinery Ac 4000Profit transferred toCapital Accounts (3 2 2)Rohit 5786Nisha 3857Sunil 3857

13500

18000 18000

Capital Account

Dr Cr

Particulars Rohit Nisha Sunil Particulars Rohit Nisha Sunil (Rs`) (Rs`) (`Rs) (Rs`) (Rs`) (Rs`)

Sunilrsquos Capital ac 9600 mdash 6400 Balance bd 60000 40000 40000Bank - 66143 - General Reserve 9428 6286 6286Balance cd 72000 mdash 48000 Revaluation (Profi 5786 3857 3857 Rohitrsquos Capital Ac mdash 9600 mdash

Sunilrsquos Capital Ac 6400 Bank 6386 - 4257

81600 66143 54400 81600 66143 54400

Balance Sheet as at 31st March 2014

Liabilities Amount Assets Amount (Rs`) (Rs`)

Creditors 25000 Building 108000Bank overdraft 37500 Machinery 36000

Bills Payable 13000 Debtors 30000Capital Less ProvisionRohit 72000 for Bad debts 1500 28500Sunil 48000 120000 Stock 23000

195500 195500

Working Notes (i) (a) Profit sharing ratio is 60000 40000 40000 ie = 3 2 2(b) Gaining Ratio Rohit = 35 ndash 37 = 2135 ndash 1535 = 635Sunil = 25-27 = 1435 ndash 1035 = 435= 635 435= 6 4 = 3 2(c) Nisha Share of Goodwill = Rs 56000 times 27 = Rs16000Share of Goodwill in the gaining ratio by the existing partner ieRohit = Rs16000 times 35 = Rs 9600Sunil = Rs 16000 times 25 = Rs 6400

The journal entry isRohitrsquos Capital Ac Dr 9600Sunilrsquos Capital Ac Dr 6400 To Nisharsquos Capital Ac 16000(Share of Goodwill divided into gaining ratio)

  • 1 Static Friction
  • The frictional force that acts between the surfaces when they are at rest with respect to each other is called Static Friction
    • Static Friction Examples
      • 2 Sliding Friction
        • Examples Of Sliding Friction
          • 3 Rolling Friction
            • Examples Of Rolling Friction
              • Objects and Reasons of the Forest Conservation Act
Page 12:  · Web viewSubject . Topic . Summary . Execution . English 1 . Sounds of animals . Hens –cackle Horses –neigh Lions –roar Owls –hoots Snake –hiss. English 2 . Mother’s

Times 2 First person to go into space in 1961 Ans Yuri Gagarin

3 First woman to go into space in 1963 Ans ValentinaTereshkova4 First person ever to walk in space in 1965Ans alexei Leonov5 First person to land on the moon in 1959 Ans Neil Armstrong6 First Indian to go into space in 1984 Ans Rakesh Sharma 7 First Indian woman to go into space in 19978 Ans Kalpana Chawla9 First woman tourist in space in 2006

Ans Anusheh AnsariCOMPUTER

ALGORITHM AND FLOWCHART

Q) DRAW THE SYMBOLS USED IN A FLOWCHART WITH THEIR DESCRIPTIONS(IN EXAM IT CAN COME AS SHORT QUESTIONS ASKING INDIVIDUAL SYMBOLS FUNCTION)ANS)

MAT

HEM

ATIC

S

Ch 6

Com

mon

Fra

ction

s

Multiplication of FractionsA Multiply a fractional number by whole numberTo multiply a fractional number by whole number we multiply the numerator of the fractional number by the whole number and denominator of the fractional number by 1 The first product thus obtained is the numerator and the second product is the denominator of the required product

Exercise ndash 30Multiply

7 2027

times 9

Solution 2027

times 9 = 203 = 6

23

8 611

times11

Solution 611

times11 = 6

15 71

20times16

Solution 71

20times16 =

14120

times16

= 1415

times 4 = 141times 4

5 = 564

5 = 11245

B Multiplication of a fractional number by a fractional number To multiply a fractional number by a fractional number we multiply the numerator of the first fractional number by the numerator of the second fractional number and the denominator of the first fractional number by the denominator of the second fractional number The first product thus obtained is the numerator and the second product is the denominator of the required product

16 2712

times24

Solution 2712

times24 = 3112

times24

= 31times2 = 62

Exercise ndash 31

11 83

times 34

2

Solution 83

times 34 = 2

14 723

times2 25

4

Solution 723

times2 25 =

233

times 125 =

23times 45

= 925 = 18

25

15 1212

times1 13

2

Solution 1212

times1 13 =

252

times 43 =

25times 23

= 503 = 16

23

State the following statements are true or false

17 1912

times 239 = 1

Solution LHS = 1912

times 239

= 392

times 239 = 1 = RHS

[LHS = Left hand side amp RHS = Right hand side]

there4 1912

times 239 = 1 [True]

21 213

times2 13 = 4

19

Solution LHS = 213

times2 13 =

73

times 73

= 7times73times3 =

499 = 5

49

there4 LHS ne RHS

So 213

times2 13 = 4

19 [False]

23 23

times 45 =

2times 5+3 times43times 5

Solution

LHS = 23

times 45 =

2times 43 times5 =

815 again

RHS = 2times 5+3 times4

3times 5 = 10+12

15 = 2215

there4 LHS ne RHS So 23

times 45 =

2times 5+3times43times 5

[False]

25 23 of

13 =

29

Solution

LHS= 23 of

13 =

23 times

13 =

29 = RHS]

there4 23 of

13 =

29 [True]

Practice at HomeExercise ndash 31State the following statements are true or false

24 12 of 4 =

18

Class VISubject Topic Summary Execution

HISTORY AND CIVICS

Chapter 5The Mauryan Empire

DECLINE OF MAURYAN EMPIREDecline of Mauryan empire started after the death of Ashoka at around 232 BCThere are several reasons for break up of the empire1 Weak successor Emperors after Ashoka were

capable of handling vast and mighty Mauryan empire In 185BC the last Mauryan ruler Bri-hadrath was murdered by his Commander-in-Chief Pushyamitra Sunga

2 Provincial Revolts Due to weak central author-ity provincial chiefs of Kalinga and southern provinces revolted against emperor and freed themselves from Mauryan empire

3 Weakness of Economy Prosperity of Mauryan was based on solid economic activities which

ExercisesI Multiple choice questions-1 Chandragupta defeated Seleucus in the year ndashc) 305 BC2 Who killed the last Mauryan ruler Brihadrath b) Pushyamitra3 Which of the following was not a reason for the decline of the Mauryan empirec) Chandraguptarsquos weakness4 Ashoka invaded Kalinga in the year c) 261 BC

II Fill in the blanks1Chandragupta ascended the throne in 324

was taken care by early monarchs Later kings had neither ability nor interest in economic af-fairs That led to failure in tax collection As a result they failed to maintain a large army that were essential to keep empire intact

4 Greek Invasion Greeks freed north-western provinces from weak Mauryan monarchs and reestablished their authority

5 Ashokarsquos Policy some scholar opined that after Kalinga war Ashoka embraced Buddhism re-nounced the policy of war and disbanded the Army But this is partially true as there is no proper evidence of disbanding the army

Based on above points we can conclude that main reason for decline of Mauryan empire is weakness of Ashokarsquos successors Kunal Samprati Dasharath Salisuk all were weak kingsAt last in 185 BCPushyamitra Sunga killed king Brihadrath and established the Sunga dynasty

BC2 Bindusara was the son of Chandragupta and father of Ashoka

3 Pataliputra was administered by City Magistrate committess of 5 members each4 The Greek General Seleucus sent his ambassador Megasthenes to Chandraguptarsquos court5 Ashoka sent his son Prince Mahendra and daughter Sanghamitra to spread his Dhamma6 The Indian Rebublic has adopted the Lion Capital of Saranath Pillar as its national emblem 7 Pushyamitra killed the last Mauryan ruler Brihadrath and founded the Sunga dynasty

III Name the following

1The author of Arthashastra-Kautilya2 The ruler who founded the Mauryan dynasty-Chandragupta3 The author of Indika-Megasthenes 4 The officers who were appointed by Ashoka to spread Dhamma-Dhamma Mahamatras5 The general of Alexander whom Chandragupta defeated-Seleucus

V Match the columns1 Kautilya (c)2 Megasthenes (d)3 Pushyamitra (e)4 Brihadrath (b)5 Bindusara (a)

BENGALI(2ND

LANGUAGE)

পশপাহিখর -াষাসহিবনয় রায়কেচৌধরী

যলখক পহিরহিচহিত- পরখযাত সাহিহিতযক উকেপনদরহিককেশার রায়কেচৌধরীর পতর সহিবনয় রায়কেচৌধরী lsquoসকেFশrsquo পহিতরকার সকেb হিতহিন কত হিকেলন তার উকেdখকোয বই lsquoসহিবনয় রায়কেচৌধরীর রচনা সংগরrsquo

পরম হিকেনর পাঠ- lsquoপশপাহিখর হিক -াষাhelliphellip helliphelliphelliphellipপরসপরকেক জানাবার উপায়ও পশপাহিখরা যবশ জাকেনrsquoপরকেমই আমারা জাহিন -াষা হিক -াষা ল আমাকের মকেনর -াব পরকাশ করার জনয আমরা নানান ধরকেনর -হিb বা হিবকেশষ ধরকেনর আওয়াজ মকেখর মাধযকেম কহির অনযকেক যবাঝাকেনার জনয তাকেল এবার আমরা জাহিন পশপাহিখর -াষা হিক পশ পাহিখরা হিক কা বকেল যা পশপাহিখকেরও -াষা আকে তারা তাকের হিনজসব -াষায় কা বকেল মকেনর -াব পরকাশ ককের পশ পাহিখরা মানকেষর হিক হিক -াষা যবাকেঝ হিকনত তারা বলকেত পাকেরনা পরসপরকেক বহিঝকেয় যবার উপায় তারা জাকেননা তকেব তারা হিবকেশষ ককেয়কটি শকেবদর মাধযকেম তাকের মকেনর -াব বহিঝকেয় যয় হিক বহিদধ মান জীব ndashককর হিবাল বন মানষ যঘাা পর-হিত এরা মানকেষর যওয়া নাম শনকেল কান খাা ককের ndash নাম ধকের ডাককেল কাকে আকেস যমন - মরহিরা lsquoহিত ndashহিতrsquo ডাক শকেন আকেস াল lsquoঅ ndashর -র ডাক শকেন কাকে আকেস াহিত মাহকেতর কা শকেন চকেল ককররা মাহিলকেকর হকম পালন ককের সবসময় তাইকেতা ককরকেক পর- -কত পরানী বলা য় ককর আর হিবাল একের আওয়াজ তহিম লকষয করকেল বঝকেব ককররা যরকে যকেল lsquoযঘউ যঘউrsquo করকেত াকেক আবার কাকেল lsquoযকউ যকউrsquo ককের হিবাল সাধারণ lsquoমযাওrsquo বা lsquoহিমউrsquo ককের রা কেল lsquoওয়াওrsquo আওয়াকেজর মাধযকেম মকেনর -াব পরকাশ ককের একেতা যল পশকের কা পাহিখরাও -য় রা পরকাশ করার জনয হিবকেশষ ধরকেনর শবদ ককের হিবপকের সময় পশ পাহিখরা সবার আকে পরসপরকেক জানাবার উপায় তারা জাকেন বহকাল

১) পশপাহিখর -াষা কেলপর যলখক সমপকেকG হিক জাকেনা

উঃ- পরখযাত সাহিহিতযক উকেপনদরহিককেশার রায়কেচৌধরীর পতর সহিবনয় রায়কেচৌধরী lsquoসকেFশrsquo পহিতরকার সকেb হিতহিন কত হিকেলন তার উকেdখকোয বই lsquoসহিবনয় রায়কেচৌধরীর রচনা সংগরrsquo হিতহিন ারকেমাহিনয়াম এসরাজ পর-হিত বাযনতর বাজাকেত পারকেতন ানও জানকেতন হিতহিন যাকেIাকের জনয মজাার লপ কহিবতা হিলখকেতন

২) পশপাহিখ কেলপর মল-াব হিকউঃ- পশপাহিখকেরও -াষা আকে তারা তাকের হিনজসব -াষায় কা বকেল মকেনর -াব পরকাশ ককের পশ পাহিখরা মানকেষর হিক হিক -াষা যবাকেঝ হিকনত তারা বলকেত পাকেরনা পরসপরকেক বহিঝকেয় যবার উপায় তারা জাকেননা তকেব তারা হিবকেশষ ককেয়কটি শকেবদর মাধযকেম তাকের মকেনর -াব বহিঝকেয় যয় হিরউকেবন কযাসটং সাকেব হিতহিন চহিdশ বর বনযজনত যর সকেb যকেককেন হিতহিন বকেলকেন আমরা হি তাকের -াষা তাকের আব কায়া যমকেন চহিল তাকেল আর -কেয়র যকান কারণ াকেকনা আমরা একI -াকেলাকেবকেস যচষটা করকেল পশপাহিখকের সকেb -াব পাতাকেত পাহির

ধকের মানষ এই পশ পাহিখর -াষা হিনকেয় নানা রককেমর পরীকষা ককের আসকে এইরকম একজন হিরউকেবন কযাসটাং সাকেকেবর কা আমরা জানকেবাhelliphellip

Hindi 2nd

langमतर किनमनलिलखिखतपरशनोउRरदीजि0ए

) बढ वयलि` बचच ो कया हआ था ख) डॉकटर साहबन पाटc किस उददशय स रखी थी ग) ाल साप ो हाथ म लर लाश न कया किया घ) डॉकटर चडढा न बढ पतरो दखन स कयो मना र दिदया था ङ) भगत न लाश ो दखर कया हा

उRर ndash) उस बहत बखार थी और 4 दिदनो स आख भी नही खोला थाख) उन बट ी सालकिगरह थीग) ाल सापो हाथ म लर लाश न उसी गदन 0ोर स दबार पडी थीघ) डॉकटर चडढा न बढ वयलि` पतरो दखन स मना र दिदया कयोकि उनह गोलफ खलन 0ाना थाङ) लाश ो दखर हा कि नारायण चाहग तो आध घट म भया उठ 0ाएग

English literature

In the bazaars of Hyderabad- Sarojini Naidu

Through the poem In The Bazaars of Hyderabad Sarojini wanted to convey the message that India is rich in tradition and they donrsquot need the foreign products So she goes on to give a picture of a bazaar where traditional Indian products are rulingThe poem is in the form of questions and answers The poet asks the questions and the merchants answer them Through this technique she make the picture of the bazaar visible to us

Read the poem

PHYSICS FORCE Types of FrictionThere are three types of friction static sliding rolling Static sliding and rolling friction occur between solid surfaces

1 Static Friction The frictional force that acts between the surfaces when they are at rest with respect to each other is called Static FrictionStatic Friction Examples

Skiing against the snow Creating heat by rubbing both the hands

together Table lamp resting on the table

2 Sliding Friction The resistance that is created between any two objects when they are sliding against each other is called Sliding FrictionExamples Of Sliding Friction

Sliding of the block across the floor Two cards sliding against each other in a

deck

3 Rolling Friction The force which resists the motion of a ball or wheel is called Rolling Friction Is the weakest types of frictionExamples Of Rolling Friction

Rolling of the log on the ground Wheels of the moving vehicles

6What effect can a force produce on a body which is not allowed to move Ans - When a force is applied on a body which is not free to move it gets deformed i e the shape or size of the body changes7Give one example each to indicate that the application of a force

1 produces motion2 stops motion3 slows down motion4 changes the direction of motion5 deforms a body

Ans- 1 A car originally at rest when pushed

begins to move2 A moving bicycle is stopped by

applying the brakes3 The speed of a moving vehicle is

slowed down by applying the brakes4 A player kicks a moving football to

change its direction of motion5 On stretching a rubber string its

length increases

8State the effect produced by a force in the following cases (a) The sling of a rubber catapult is stretched(b) A man pushes a heavy cart(c) A player uses his stick to deflect the ball (d) A cyclist applies brakes(e) A spring is compressedAns- (a) The shape and size of catapult changes ie its length increases(b) The heavy cart begins to move(c) The direction of the ball changes(d) The speed of the moving cycle is slowed down(e) There is change in size and shape of spring

COMPUTER MS EXCEL 2013 -INTRODUCTION

UNDERSTANDING EXCEL STRUCTUREA SPREADSHEET IS A FILE THAT EXISTS OF CELLS IN ROWS AND COLUMNS AND CAN HELP ARRANGE CALCULATE AND SORT DATA DATA IN A SPREADSHEET CAN BE NUMERIC VALUES AS WELL AS TEXT

FORMULAS REFERENCES AND FUNCTIONS

WORKSHEETA WORKSHEET IS ALSO KNOWN AS SPREADSHEETIT IS A COLLECTION OF CELLS ON A SINGLE SHEET WHERE YOU KEEP AND CHANGE DATA

WORKBOOKWORKBOOK IS PMS EXCEL FILE IN WHICH THE DATA CAN BE STORED EACH WORKBOOK CAN CONTAIN MANY WORKSHEETS

ROWS AND COLUMNSIN MS EXCEL A ROW IS A GROUP OF CELLS THAT RUN FROM LEFT TO RIGHT OF A PAGEA COLUMN IS A GROUPING OF CELLS THAT RUN FROM THE TOP TO THE BOTTOM OF A PAGE

CELLTHE INTERSECTION POINT BETWEEN A ROW AND THE COLUMN IS CALLED A CELL WHICH IS THE BASIC STORAGE UNIT FOR DATA IN A SPREADSHEET EACH CELL HAS SPECIFIC ADDRESS WHICH IS THE COMBINATION OF THE COLUMN NAME FOLLOWED BY THE ROW NUMBER

CHEMISTRY Chapter ndash Common Laboratory Apparatus and equipments

Objective type questionFill in the blanks (a) Experiment and observation are the two important basics of chemistry(b) A porcelain dish is used for evaporation(c) A test tube holder is used to hold the test tube while-it is heated(d) Mortar and pestle is used for grinding and crushing solid substances into a powder(e) Glass apparatus is made of Pyrex or borosil glass

Class VIISubject Topic Summary Execution

Hindi 2ndlang

ए था राम( डॉ शरी परसाद)

सगकित ा परभाव मानव 0ीवन पर अवशय पडता ह

हमशा मनषय ो अचछो ी सगकित म रहना चाकिहए

शरषठ परो सग स मनषय चरिरतर ा शीघर ही उदय और किवास हो 0ाता

ह इसलिलए वयलि` ो सदा शरषठ परो ा ही सग रना चाकिहए

इसान अगर चाह वह सवय ो बदल भी सता ह

यह हानी राम ए बचच ी हवह गणिणत ी परीकषा म नल रत हए पडा 0ाता ह और उस अधयाप पडत ह और पछत ह यह कया र रह

हो तभी राम न उनी बइजजती ी

शबदाथब ndashहावा भलावाायवाहीndash ाम किनयम व ानन

ो दिदखानापरिरलिचतndash 0ाना पहचानाघटनाndashघबराहट

उलटा चोर ोतवाल ो डाटndashकिववndash भल बर ा जञानतवयndash म 0ो रना चाकिहएसगकितndash बरी सगत

किबलख नाndashरोना किनशचय रनाndash तय रना

फलndashपरिरणामकिनषालिसतndash बाहर किया हआपशचातापndashदख सपननndashधनी

ldquo हा आपी किहममत स हई नल रत पडन ीrdquo ऐसी बात ही किफर

किपता0ी न भी उस डाटा वह ाफी पशचाताप रन लगा बोला गलत दोसतो

ी सगकित म आ0 कितना अनथ र दिदया किफर उसन अधयाप स माफी मागन ी सची और किफर भी ऐसा

नही रगा यह परण भी लिलया

सोचndashकिहच एात-अला

বইndashবাংলা সাহিতয পহিরচয়

পাঠndash১৬লপndashস-য ও অস-যযলখকndashঈশবরচনদর হিবযাসারঅনশীলনীর পরকে4াততর

৬ অGকেলকেখা -ময়া = পশ হিশকার সহিtহিত = হিনকIবতu সbভরষট = লI হিনরীকষণ =

-াকেলা-াকেব যখাকতাঞজহিলপকেI = যজাাকেত৭ হিবপরীতশবদ -ঈষৎ times পরচর উৎকষট times হিনকষট তাশ times উৎফd তবহিদধ times

বহিদধীNপাহিপষঠ times পণযবান৮ পপহিরবতG ন ককেরা -পশ = পাশহিবক যকাপ = যকাহিপতহিসথর = হিসথরতাএকানত = ঐকাহিনতক পর-াত times পর-াতী

CHEMISTRY

Chapter ndashPhysical and Chemical Changes

Chemical ChangeA chemical change involves a change in chemical composition

Characteristics of Chemical changes 1 They are permanent changes2 They are irreversible changes 3 New substance formed4 A Chemical change involves a

change in its chemical properties

Pg-25Question 8What do you observe when1 water is boiled2 a piece of paper is burnt3 some ice cubes are kept in a glass tumbler4 solid ammonium chloride is heated5 an iron nail is kept in tap water for few days6 a spoon of sugar is heated in a pan7 lighted match stick is brought near the mouth of the test tube containing hydrogen gas8 quick lime is dissolved in water9 little amount of curd is added to a bowl containing warm milk and kept for five hours

10 Water is boiledOn boiling water changes into steam (gas) physical change

11 A piece of paper is burnton burning piece of paper produces carbon dioxide and ash is left behind Is a chemical change

12 some ice cubes are kept in a glass tumblerIce cubes (solid) turn into water

(liquid) only state changes (physical change)

13 Solid ammonium chloride is heatedSolid ammonium chloride on heating changes into vapors (change of state) is physical change

14 An iron nail is kept in tap water for few dayswe observe reddish brown coating on the nail called rust (entirely new substance) is chemical change

15 A spoon of sugar is heated in a panWhen a spoon of sugar is heated in a pan black (charred sugar) (carbon) is seen Is a chemical change

16 Lighted match stick is brought near the mouth of the test tube containing hydrogen gasWe observe that hydrogen bums at the mouth of test tube with blue flame and pop sound is heard It is chemical change

17 Quick lime is dissolved in waterThe following two observations will be observed (i) A hissing sound is observed(ii) The mixture starts boiling and lime water is obtained

18 Little amount of curd is added to a bowl containing warm milk and kept for five hoursWhen a little amount curd is added to a bowl containing warm milk and kept for five hours a permanent change occurredThe milk will change to curd On boiling water changes into steam (gas) physical change

GEOGRAPHY

ATMOSPHERE IMPACT OF GLOBAL WARMING The destructive impart of global warming is observed in various spheres of life and the environment Some of the points are outlined below1 High temperatures lead to high

evaporation rate and drying up of the soil and surface water This affects crop production The occurrence of droughts is aggravating the problem even further

2 The heat waves in summer months

Q1 Write some impact of global warmingA1 The impacts of global warming are as follows1 High temperatures lead to high

evaporate ion rate and drying up of the soil and surface water This affects crop production The occurrence of droughts is aggravating the problem even further

2 The heat waves in summer months lead to a greater number

lead to a greater number of deaths due to heat strokes

3 Forest fires become more frequent4 Tropical cyclones and hurricanes

become common5 Melting of glaciers takes place6 Polar ice caps are becoming thinner

and melting at an alarming rate due to global warming The loss of sea ice

7 Due to increase in sea surface temperature sea levels rise in coastal areas and cause submergence of several islands

WAYS TO REDUCE GLOBAL WARMINGFollowing steps can be taken We need to decrease emission of

green house gases by reducing the burning of fossil fuel such as coal and petroleum

By planting more trees to increase forest cover

The government should also distributes free saplings and organize afforestation programmes to spread awareness regarding the beneficial effects of trees

We should switch to eco-friendly cars and gadgets

Incandescent light bulbs should be replaced by CFL bulbs

We can save electricity and reduce global warming by turning off electrical gadgets such as lights fans air-conditioners television and computer when we do not to use them

Efforts should be made to hasten the development of green cities oreco cities These cities are urban areas around the world striving to lessen the environment a impacts of urbanization

By following the 3Rs-Reduce Recycle and Reuse strategy we can use natural resources for our growth as well as save them for the need of the future generations This is called sustainable development

of deaths due to heat strokes3 Forest fires become more

frequent4 Tropical cyclones and hurricanes

become common5 Melting of glaciers takes place

etc

Q2 How to reduce global warmingA2 Following steps can be taken to reduce global warmingaWe need to decrease emission of

green house gases by reducing the burning of fossil fuel such as coal and petroleum

bBy planting more trees to increase forest cover

c The government should also distributes free saplings and organize afforestation programmes to spread awareness regarding the beneficial effects of trees

dWe should witch to eco-friendly cars and gadgets

eIncandescent light bulbs should be replaced by CFL bulbs

f We can save electricity and reduce global warming by turning off electrical gadgets such as lights fans air-conditioners television and computer when we do not to use them

Q3 What do you mean by 3Rrsquos of resource planningA3 The 3Rs are

1 Reduce 2 Recycle and3 Reuse

Q4 What is Sustainable developmentA4 By following the 3Rs-Reluce Recycle and Reuse strategy we can use natural resources for our growth as well as save them for the need of the future generations This is called sustainable development

English Language

Prepositions A preposition is a word placed before a noun or a pronoun It helps to show how the person or thing denoted by the noun is related to something else in the sentence

Kinds of Prepositions

Simple Prepositions- simple preposition are one word Prepositions such as at by for in of off for from on out through till to up with before amidst towards beyond between over etc

Compound Prepositions ndash There are some words that are always used with fixed Prepositions to convey specific meaning

Example I was unable to meet you dueto a previous engagement ( On account of)Always maintain the queue instead of crowding at the counter ( In place of)

Participial PrepositionsmdashParticiple Prepositions are present or past participles of various verbs which together with a noun phrase or a clause function as prepositions Examples- barring concerning considering notwithstanding pending regarding respecting etc

Exercise A

1 Gauravs fever has come down since Friday He has been absent for a week now

2 The child sat between his father and mother among the parents of all his classmates

3 There are mosquitoes in the room They flew into the room when the door was open

4 My father was inside the drawing room when I was playing outside my house

5 You may sit beside me I will give you a drawing book and pencils besides a storybook

6 We went to the market in the morning and walked towards the riverfront in the evening

7 The child walked along the pavement and across the street safely

8 This table top is made of glass My breakfast fell off it in the morning

9 The pan is on the gas stove There are vegetables in it

10 We will wait for you at the bus top There are a lot of people in the hall

Subject ndash Biology Topic ndash Chapter - 3 Photosynthesis and respiration in plants Summary Execution

All living organism (Plants and animals) need food for energy and growth Green plants (autotrophy) prepare food for all living organisms Today we will discuss about the process photosynthesis And adaptations in a leaf to carry out photosynthesis

Q1What do you mean by photosynthesis and write its word equation The process by which green plants make food (glucose) from carbon dioxide and water

in the presence of sunlight and chlorophyll is called photosynthesis

Carbon dioxide + Water ( Sun light from Sun ) Glucose + Oxygen ( chlorophyll in green leaves )

Q2 What are the adaptations in a leaf to carry out photosynthesisi) Leaves are broad wide and flat for absorbing more light energyii) Presence of chlorophyll in chloroplasts to trap sunlightiii) Presence of stomata which allow carbon dioxide to enter the cell and oxygen to go

out iv) Network of veins ensures continuous supply of water and minerals to the leafv) Thin waxy cuticle protects the leaf without blocking the lightQ3 Draw and label structure of chloroplast

Class VIIISubject Topic Summary Execution

PHYSICS ENERGY Production of Hydro electricity

A hydroelectric dam converts the potential energy stored in a water reservoir behind a dam to mechanical energymdashmechanical energy is also known as kinetic energy As the water flows down through the dam its kinetic energy is used to turn a turbine

The generator converts the turbinersquos mechanical energy into electricity

This electric energy then goes through various transmission processes before it reaches you

Question 2

Fill in the blanks

(a) Work is said to be done by a forte only when the body moves

(b) Work done = Force x distance moved in direction of force

(c) The energy of a body is its capacity to do work

(d) The SI unit of energy is joule

(e) The potential energy is due to its state rest of position and kinetic energy of the body is due to its state of motion

(f) Gravitational potential energy U = mass times force of gravity on unit mass times height

(g) Kinetic energy = frac12 times mass times (speed)2

(h) Power P = work donetime taken

(i) The S I unit of power is watt

(j) IHP = 746 W

BIOLOGY Chapter -5 The endocrine system and adolescence

Today we will discuss about thelocation and functions of secreted hormones of adrenal and Pancreas

Q5 Write location hormone secreted main functions and deficiency diseases of pancreas and adrenal glands

Endocrine Glands

Location Hormones secreted

Functions and Deficiency Diseases

1Adrenal gland

2 Pancreas Gland

On the top of each kidney

In between stomach and small intestine

i)Adrenaline from adrenal medulla

ii)Cortisone from adrenal cortex

i) Insulin

ii) Glucagon

It helps a person deal with any kind of emergency situation or emotional stressIt increases the heart beat rate of respiration and blood pressure

a) It regulates carbohydrates protein and fat metabolism

b) It regulates the salt and water balance in the body

a) It changes excess glucose into glycogen

b) It stimulates the cells to burn extra glucose to provide heat amp energy

Less secretion causes diabetes mellitus

Excessive secretions causeinsulin shock

a) It stimulates the breakdown of glycogen into glucose

b) It increases the level of glucose in blood

History Traders to rulers The Battle of Buxar was fought on 22 October 1764 between the forces under the command of the British East India Company led by Hector Munro and the combined armies of Mir Qasim the Nawab of Bengal till 1763 Mir Jafar was made the Nawab of Bengal for a second time in 1763 by the Company just after the battle After being defeated in 4 battles in katwa and Udaynala the Nawab of Awadh Siraj id Daula and the Mughal emperor Shah Alam II accompanied by Raja Balwant Singh of Kashi made an alliance with Mir Qasim The battle was fought at Buxar a small fortified

Answer the following questions- Short note-Battle of BuxarHomework-learn

town within the territory of Bihar located on the banks of the Ganga river about 130 kilometres (81 mi) west of Patna it was a decisive victory for the British East India Company The war was brought to an end by the Treaty of Allahabad in 1765

EnglishLiterature

The west wind-John Mansfield

In the poem The West Wind by John Masefield the poet starts by describingwith very poetic imagery of birds how the west wind is different from other winds its a warm wind full of birds cries There is a touch of melancholy perhaps home-sickness as he describes how it brings tears too and memories from an old land He goes on to describe the restful pastoral beauty of the land where even the dead can lie in the green He then brings in voicesperhaps of family and friends calling him home as he is missing Aprils beautyThe voices then tempt him some more with idyllic images from home (white blossom young green cornrunning rabbitswarm sun) The voices seem to presume that the poets heart is sorrowful bruised and soreThe end of the poem sees the poet appear to make a decision he will go home as he has decided that is where he truly belongs

Write the synopsis of the following words

1 Daffodils- a tall yellow flower that grows in the spring

2 Orchards- a piece of land on which fruit trees are grown

3 Blossom- a flower or a mass of flowers especially on a fruit tree in spring

4 Thrushes- a bird5 Larks- a small brown bird that

makes a pleasant sound6 Bruised- an injury7 Aching- pain 8 Tread- to put your foot down

while you are walking9 Balm-10 May-11 Fluting-

(Write from the book in your copy)

MAT

HEM

ATIC

S

Ch 1

1Al

gebr

ic E

xpre

ssio

n

1 Constant A symbol which has fixed value is called a constant[eg 8 23 -15 radic3 etc]

2 VariableA symbol which does not have any fixed value but may be assigned value (values) according to the requirement is called variable or literal[eg x y p q etc]

3 TermsA term is a number (constant) a variable a combination (product or quotient) of numbers and variables[eg 7 x 5x etc]

4 Algebric expressionA single term or acombination of two or more terms connected by plus (+) or minus (-) sign forms an algebraic expression[eg 5-y 3x2-5x xy-6z+4 etc]

5 PolynomialAn algebraic expression which contains more than one term is called a polynomial (multinomial)[eg x2-5x 5y+xy+x2y etc]

6 Degree of polynomial(a) When the polynomial contains only one variable the highest power of the variable is the degree of the polynomialeg the degree of the polynomial of 4x-7x5+8 is 5(b) When the polynomial contains two or more variablesStep (i) Find the powers of the variables in each term (ii) The highest sum of the powers is taken to be the degree of the polynomialeg the degree of the polynomial 5x2y-4x3y5+6 is = 3+5 = 8Remember An algebraic expression is a polynomial if degree of each term used in it is a non-negative integer

Exercise ndash 11(A)

1 Separate the constants and variables from the following

-7 7+x 7x+yz radic5 radic xy 3 yz

8 45y -3x

Solution Constant Variables-7 radic5 7+x 7x+yz radic xy

3 yz8

45y -3x

2 Write the number of terms in each of the following polynomials(i) 5x2+3timesax (ii) axdivide4-7 (iii) ax-by+ytimesz (iv) 23+atimesbdivide2

Solution Polynomials Number of terms(i) 5x2+3timesax 2(ii) axdivide4-7 2(iii) ax-by+ytimesz 3(iv) 23+atimesbdivide2 2

4 Write the degree of the each polynomials(i) xy+7z (ii) x2-6x3+8 (iii) y-6y2+5y8 (iv) xyz-3 (vi) x5y7-8x3y8+10x4y4z4

Solution Polynomials Degree(i) xy+7z 2(ii) x2-6x3+8 3(iii) y-6y2+5y8 8(iv) xyz-3 3(vi)x5y7-8x3y8+10x4y4z4 12

5Write the coefficient of(i) ab in 7abx (iv) 8 in a2-8ax+a (v) 4xy in x2-4xy+y2

SolutionCoefficient

(i) ab in 7abx 7x(iv) 8 in a2-8ax+a -ax(v) 4xy in x2-4xy+y2 -1

7 CoefficientAny factor of an algebraic quantity is called the coefficient of the remaining quantityeg in the algebraic term 7xyz 7 is coefficient of xyz 7x is coefficient of yz and so on

8 Like term The terms having the same literal coefficient are called like terms and those having different literal coefficients are called unlike terms

eg (i) 5xyz 8xyz -6xyz and 23xyz are like

terms(ii) 7xy2 8x2yz and -15xyz2 are unlike terms

6 in 57xy2z3 write the coefficient of

(i) 5 (vii) 5xy2 (viii) 17yz (xi) 5xyz

Solution Coefficient

(i) 5 17

xy2z3

(vii) 5xy2 17z3

(viii) 17yz

5xyzsup2

(xi) 5xyz 17yz2

7 In polynomial given below separate the like terms(ii) y2z3 xy2z3 -58x2yz -4y2z3 -8xz3y2 3x2yz and 2z3y2

Solution y2z3 -4y2z3 2z3y2 are like terms

xy2z3 -8xz3y2 are like terms

-58x2yz 3x2yz are like terms

Class IXSubject Topic Summary Execution

Bengali (2nd language)

বাগzwnjধারাzwnj বা ধারা-বা ধারা ল হিবকেশষ পরকার বাক -হিb -াকেবর এক হিবকেশষ পরকাশরীহিত াকেক কতগকেলা কার সমষটির মকেধয এগহিলকেক বা ধারা বকেল আবার কতগকেলা শকেবদর বাধাধরা যকান রীহিত যনই য-াকেব চকেল আসকে যসই -াকেবই চকেল আসকে তখন যসই শবদগহিল খন একক -াকেব অG পরকাশ ককের তখন একের বা ধারা বকেল বা ধারার পরকেয়া -াষাকেক আরও সFর ককের যতাকেল

অকাল পকক(অপহিরনত বয়কেস পাকাহিম)-মাতর শ বর বয়কেস যমকেয়টির া মকেখর কা তাকেত অকালপককতা ধরা পকে

অককা পাওয়া( মারা াওয়া) ndash পকেকIমারটি পকেকIমারকেত হিকেয় বাসাতরীকের াকেত মার যখকেত যখকেত অককা যপল

অহি| পরীকষা ( কঠিন ও পরকত পরীকষা)- যকেলটির আজ ডাকতাহির যরজালট যবকেরাকেব এIাই তার জীবকেনর ব অহি| পরীকষা

অষটরমভা (ফাহিক) ndash রীতা মকেখই বকো বকো কা বকেল আর কাকেজর যবলায় অষটরমভা

অকমGার ধাী (অপাG) ndash সমনকেক হিনকেয় যকান ান কেব না ও একেকবাকেরই অকমGার ধাী

অকেনধর ষটি (অসাকেয়র সায়)- আহিশ বকেরর বকোর নাহিত ল অকেনধর ষটি তাকেক াা বকোর একম চকেল না

আকেককল গড়ম (তবহিদধ)- ার তহিম উপকার করকেল যসই যতামার হিবরকেদধ সাকষয হিকেয়কে শকেনই আমার আকেককল গড়ম

আষাকে লপ( অবাসতব লপ) ndashIাকা এখন যকেব না এIা বলকেলই ত এমন আষাকে লপ ফাার যকান রকার হিল না

Hindi- महायजञ ा इस हानी म लख न या बतान ा परयास किया ह कि किसी भी अचछ

2nd language

परसार(यशपाल ाय या पणय न ा फल अवशय मिमलता ह ोई भी परोपार अथवा पणय लिलए किया गया ाय बार नही 0ाता वह ए परार ा यजञ हए धनी सठ थ धम परायण और किवनमर सठ न आन ी यजञ किए थ और दान म न 0ान कितना धन दिदन दखिखयो म बात दिदया थादिदन पलट और सठ यहा गरीबी आ गई उन दिदनो यजञ बचन ी परथा थी सठ भी अपनी 0गह बचन लिलए डलपर ए सट यहा चलन ो तयार हए सठानी रासत लिलए रोटी पड म बाधर सठ ो द दी रासत म ए भख R ो दखर सठ न चारो रोटी उसो खिखला दी खर वह सठ यहा डलपर पहच तो उनी सठानी न उस महायजञ बचन ो हा यदिद बचन आए सठ न R ो रोटी खिखलान ो महायजञ नही समझा और वापस लौट आया घर आर शाम ो उसी घर म उस ए बडा ख0ाना मिमला 0ो उस दवारा किए गएrsquo महायजञrsquo ा परसार था

English language

Letter formal The heading the name and address of the person you are writing to must be included beneath your own address In formal letters ldquoblock stylerdquo of address is preferred

Subject complain in brief

Salutation If the person you are writing to is known to you you may begin ldquoDear MrrdquoOr ldquoDear Mrsrdquo In all other instances you should begin ldquoDear Sirrdquo or ldquoDear Madamrdquo Or ldquoSirsrdquo

The body A formal or business letter has four partsReference The letter should begin by referring to a letter you have received an advertisement or the reason that has prompted you to writeInformation In the second paragraph it is necessary to supply more detailed information that is related to the referencePurpose Here you must give the reason why you are writing the letter This must be stated clearly and ensure that it is relevant to the question that has been setConclusion round off the letter with some polite remarkThe subscription when a letter has begun with dear sir sirs Madam you should end with Yours faithfully or yours truly When however you address a person by name you must conclude with the words ldquoYours sincerelyrdquo

1 A park in your locality is slowly being used as a rubbish dump Write a letter to the Mayor of your city pointing out the nuisance and danger of this Request that action be taken to stop this immediately

Or2 You being a boarder ordered a set of lab manuals from a famous book shop in the town They sent you a wrong set of books Write a letter to the manager of the book shop

Chemistry Chapter-1 1)CHEMICAL FORMULA- Q What is the Significance of

L-2The Language of Chemistrybull Chemical Formula

Itrsquos a symbolic representation of a chemical substance eg ndash The formula of Sulphuric acid is H2SO4

2) Steps of writing Chemical Formula of a given substance-

1 Write the symbols of the constituent atoms or radicals side by side Keep the basic radical on LHS and acid radical on the RHS ( Na+Cl- )2 In case of a radical having more than one atom( compound radical) enclose the radical in a bracket eg (SO4-)3 Write the valencies of each radical on its right hand top4 If the valencies of the two radicals are divisible by a common factor then divide the valencies by the common factor5 Invert (criss-cross) the valency number ie write the valency of one atom below the second atom and vice versa 6 On interchanging if valency number is lsquoone the figure lsquoonersquo is never writtenFor Example- Compound -Calcium Nitrate1 Writing the symbols- Ca(NO3)2 Writing the valencies on their right hand top- Ca2(NO3)1

3 Valency numeral in simple ratio- Ca2(NO3)1

4 Criss-cross- Ca 2NO3 1

5 Writing the formula of the compound- Ca(NO3)2

Chemical formula

A The formula of a substance conveys the following information regarding a substance 1 The name of the substance (qualitative)2 The elements constituting the substance (qualitative)3 The number of various atoms present in a molecule of the substance (quantitative)4 Molecular weight of the substance and the relative weights of different elements present in it (qualitative)

Q What are the limitations of Chemical Formula

A The chemical formula suffers from the following limitations-I It fails to convey whether the elements in a molecule are present in the form of atoms or ionsFor example the formula KBr fails to tell us whether Potassium and Bromine are present in the form of ions II It does not tell anything about the binding force that holds atom in a molecule togetherIII It does not tell us about the arrangement of various atoms with respect to one another within the molecule

Q Examples of Some Chemicals with their Formula Chemical name and Common Name-

A Given in the class notesCommercial Studies

Joint Stock Company

Let us discuss about the demerits of Joint Stock CompanyDespite so many advantages it has got many disadvantages which are as follows

Difficulty in FormationDelay in Decision makingExcessive Government ControlLack of Secrecy

Company can be classified into several categories based on incorporation

QuestionExplain the demerits of Joint Stock CompanyAnswer) 1 Difficulty in Formation The legal requirements and formalities required to be completed are so many The cost involved is quite heavy It has to approach large number of people for its capital It cannot start its business unless certificate of incorporation has been obtained This is granted after a long time when all the formalities are completed

Chartered CompanyStatutory CompanyRegistered Company

Delay in Decision making In this form of organization decisions are not made by single individual All important decisions are taken by the Board of Directors Decision-making process is time-consuming So many opportunities may be costly because of delay in decision-making Promptness of decisions which is a common feature of sole trader ship and partnership is not found in a company

Excessive Government ControlA company and the management have to function well within the law and the provisions of Companies Act are quite elaborate and complex At every step it is necessary to comply with its provisions lest the company and the management should be penalized The penalties are quite heavy and in several cases officers in default can be punished with imprisonment This hampers the proper functioning of the company

Lack of Secrecy The management of companies remains in the hands of many persons Every important thing is discussed in the meetings of Board of Directors Hence secrets of the business cannot be maintained In case of sole proprietorship and partnership forms of organisation such secrecy is possible because a few persons are involved in the management

2 Define the following

Chartered Company- The crown in exercise of the royal prerogative has power to create a corporation by the grant of a charter to persons assenting to be incorporated Such companies or corporations are known as chartered companies Examples of this type of companies are Bank of England (1694) East India Company (1600) The powers and the nature of business of a chartered company are defined by the charter which incorporates it After the country attained independence these types of companies do not exist

in IndiaStatutory Company- A company may be incorporated by means of a special Act of the Parliament or any state legislature Such companies are called statutory companies Instances of statutory companies in India are Reserve Bank of India the Life Insurance Corporation of India the Food Corporation of India etc The provisions of the Companies Act 1956 apply to statutory companies except where the said provisions are inconsistent with the provisions of the Act creating them Statutory companies are mostly invested with compulsory powersRegistered companiesCompanies registered under the Companies Act 1956 or earlier Companies Acts are called registered companies Such companies come into existence when they are registered under the Companies Act and a certificate of incorporation is granted to them by the Registrar

Economics

Chapter-4Basic problems of Economy

Today let us discuss with the topic Production Possibility curve

QuestionExplain the concept of Production Possibility Curve with the help of diagram

Answer) Production Possibility curve is a locus of all possible combinations of two commodities which can be produced in a country with its given resources and technology

The above diagram shows that with the given resources and technology the economy can produce maximum either 5 thousand meters of cloth or 15 thousand quintals of wheat or any other combination of the two goods like B( 1 thousand meters of cloth and 14 thousand quintals of wheat C ( 2 thousands meters of cloth and 12 thousand quintals of wheat) etcProduction Possibility curve is also called production possibility boundary or frontier as it sets the maximum limit of what it is possible to produce with given resources

Geography

Rotationand Revolution

SUNrsquoS POSITION AND SEASONAL CHANGES EQUINOXES ndash SPRING AND AUTUMN

Q1 What is Spring EquinoxA1 On 21st March sunrays fall directly on the equator On that day

As the Equator divides the Earth into two equal halves the sun rays fall directly on the equator twice in a year Equinoxes means equal Spring EquinoxOn 21st March sunrays fall directly on the equator On that day the duration of day and night both are equal ( 12 hours day and 12 hours night) on every places located on equator This day is called as Spring EquinoxAutumn EquinoxOn 23rd September sunrays fall directly on the equator On that day the duration of day and night both are equal ( 12 hours day and 12 hours night) on every places located on equator This day is called as Autumn Equinox

SOLSTICES ndash SUMMER AND WINTERDue to inclination of the Earth on its axis and the apparent movement of the sun the sun rays fall directly on both tropics once in a year Solstice is a Latin word which mean ldquothe Sun standing stillrdquoSummer SolsticesAfter 21st March there is an apparent movement of the Sun to the north of the equator The apparent northward movement up to 21st June when the Sun appears overhead at the Tropic of Cancer (22frac12degN) The sun appears to stand still at this position and then moves southwards towards the equator This position of the Sun on 21st June is known as Summer Solstices On that day the duration of day and night both are equal ( 12 hours day and 12 hours night) on every places located on Tropic of Cancer (22frac12degN)Winter solstices The apparent southward movement of the Sun continues beyond the equator till 22nd

December On this day the Sun is overhead at the Tropic of Capricorn

the duration of day and night both are equal ( 12 hours day and 12 hours night) on every places located on equator This day is called as Spring Equinox

Q2 What do you mean by EquinoxA2 Equinoxes means equal It is use to explain the equal duration of day and night ( 12 hours day and 12 hours night) on the Earth

Q3 On which date the longest day in Tropic of CancerA3 21st June

Q4 What is the meaning of SolsticeA4 Solstice is a Latin word which mean ldquothe Sun standing stillrdquo

Q5 Which is the longest day in southern hemisphereA5 22nd December

Q6 On what date does the Arctic Circle experience the lsquoMidnight SunrsquoA6 On 21 June the Arctic Circle experiences the lsquoMidnight Sunrsquo

Q7 What is cause of Midnight Sun in NorwayA7 During the summer solstice (21 June) the North Pole is inclined towards the Sun Therefore the duration of sunlight or daytime increases from 12 hours at the Equator to 24 hours at the Arctic Circle and beyond Thatrsquos why The region beyond the Arctic Circle especially Norway is known as the Land of the Midnight Sun because there the Sun does not rise or set on 21 June

Q8 Match the column A with BA B

Summer Solstice 21st March

Autumn Equinox 23rd

September

Winter Solstice 21st June

(22frac12degS) This position of the Sun is referred to as the Winter Solstice because it marks the winter season in the Northern Hemisphere On that day the duration of day and night both are equal ( 12 hours day and 12 hours night) on every places located on Tropic of Capricorn (22frac12degS)SEASONS AND DURATION OF DAY AND NIGHT During the equinoxes all places on the Earth have 12 hours of day and 12 hours of night Due to the revolution of the Earth round the Sun on an inclined axis the duration of day and night varies according to seasons and the latitude of a placeDuring the summer solstice (21 June) the North Pole is inclined towards the Sun Therefore the duration of sunlight or daytime increases from 12 hours at the Equator to 24 hours at the Arctic Circle and beyondThe region beyond the Arctic Circle especially Norway is known as the Land of the Midnight Sun because there the Sun does not rise or set on 21 JuneAt the North Pole there will be six months of daylight The Sun will be seen always above the horizon at a low angle At 66degN 24 hours of sunlight can be seen only on 21 June Hammerfest in northern Norway is a place of tourist attraction for observing the phenomenon of the Midnight Sun This place has continuous daylight from 13 May to 29 July This place is easily accessible to tourists and has hotels and other facilities The view of the midnight Sun from here is enthrallingIn the Southern Hemisphere the duration of daylight decreases from 12 hours at the equator to 0 hours beyond the Antarctic Circle In the South Polar Region there is 24 hours of darkness The Sun is always below the horizon In the Southern Hemisphere which experiences winter the duration of night-time is longer than the duration of daylight

Spring Equinox 22nd

December

A8 A B

Summer Solstice 21st June

Autumn Equinox 23rd

September

Winter Solstice 22nd

December

Spring Equinox 21st March

During winter solstice (22 December) the South Pole is inclined towards the Sun The Southern Hemisphere experiences summer and the Northern Hemisphere has winter Therefore the duration of daylight or sunlight is greater in the Southern Hemisphere than in the Northern HemisphereThe duration of daylight increases from 12 hours at the equator to 24 hours beyond the Antarctic Circle The South Polar Region has 24 hours of sunlight for many days continuously At the South Pole there will be six months of sunlight The Sun will always be seen at a low angle above the horizon In the Northern Hemisphere the duration of daylight will decrease from 12 hours at the equator to 0 hours at the Arctic Circle There are 24 hours of darkness in the North Polar region The duration of night is greater than the duration of daylight as one move northwards from the Equator It is evident from the above table that the duration of daylight is 12 hours throughout the year at the equator only As one moves away from the equator the seasonal variations in the duration of daylight increase The seasonal variations in the duration of daylight are maximum at the Polar Regions

Subject Eng Literature (The Merchant of Venice ndash William Shakespeare)Topic Act II Scene 7 Lines 36 to 80 (End of scene ) [Students should read the original play and also the paraphrase provided]

Summary Questions amp AnswersThe Prince then examines the inscription on the silver casket which says ldquoWho chooseth me shall get as much as he deservesrdquo The Prince says that he deserves Portia more than anybody else because of his high rank his noble birth and his great wealth and power But then he argues that silver is ten times

(1) (Act II Sc 7 L 39-47)

From the four corners of the earth they come

To kiss this shrine this mortal breathing saint

The Hyrcanian deserts and the vasty wildsOf wide Arabia are as through-fares now

inferior to gold and therefore he cannot believe that the portrait of such a beautiful lady as Portia can be contained in the silver casket He decides to see the inscription on the golden casket before making his decision

The Prince goes to examine the inscription on the golden casket which says ldquoWho chooseth me shall get what many men desirerdquo The Prince believes that the whole world desires to possess Portia otherwise so many suitors would not have come from all corners of the world for winning Portia Some of them have come from the distant lands of Persia and Arabia The deserts of Persia (Hyrcanian deserts) and the boundless desolate lands of Arabia have been crossed by the Princes seeking the hand of Portia He contrasts this casket containing Portiarsquos portrait with the old English gold coin bearing the image of the archangel (angel of the highest rank) He goes on to remark that while the figure of the archangel is engraved (Insculped) upon the English coin the picture of Portia who is beautiful as an angel lies hidden inside one of the caskets namely the Golden Casket (Golden Bed)

On the basis of his assessment of the inscription on the golden casket the Prince decides to choose the golden casket He asks for the key and opens the golden casket only to find therein an empty human skull holding a roll of

For princes to come view fair PortiaThe watery kingdom whose ambitious headSpets in the face of heaven is no barTo stop the foreign spirits but they comeAs orsquoer a brook to see fair Portia

(i) Explain the occasion for the above mentioned speech

These are the comments of the Prince of Morocco after he reads the inscription on the golden casket His mental process is revealed to us in these words We find him debating within himself as to which casket he should choose

(ii) What light does the above speech throw on the personality of Prince of Morocco

From the above mentioned speech we come to know that the Prince of Morocco is keen to marry Portia He is the type of person who is easily taken away by outward appearance He is in love with Portia because of her beauty

(iii) What information can you gather about Portia from the above mentioned lines

The given speech shows that Portia is a very beautiful lady She must be possessed of good qualities because many suitors come to her place from all over the world with a desire to get married to her The Prince of Morocco is so impressed by her beauty that he calls her a saint According to him the whole world is desirous of having her

(iv) Elucidate the significance of the first two lines

In these lines the Prince of Morocco pays a compliment to Portia These lines show his admiration for her He says that people come from all parts of the world to see fair Portia

(v) Explain the meaning of the last four lines of the

passage

In these lines the Prince of Morocco says that even the vast oceans which throw a challenge at the sky are unable to prevent men from coming to Portiarsquos place to have a glimpse of her These lines are also a tribute to Portiarsquos beauty and good qualities Many men voyage across the ocean treating it as a mere stream to see the beautiful Portia

paper in which is written that whoever happens to be guided by the glitter of things is invariably deceived

On reading the scroll the Prince says that he is too sad at heart to speak a more formal farewell and leaves with his followers amidst a sound of trumpets

After the Prince of Morocco leaves Portia remarks that the Prince is a gentle fellow but she is rid of him May all persons of his nature make a similar choice

IMPORTANT PASSAGES EXPLAINED

(Act II Sc 7 L 39-43)From the four corners of the earth they come

To kiss this shrine this mortal breathing saintThe Hyrcanian deserts and the vasty wildsOf wide Arabia are as through-fares nowFor princes to come view fair Portia

Context

This passage occurs in Act II Scene 7 in The Merchant of Venice This is part of the speech made by the Prince of Morocco

(2)

(Act II Sc 7 L 48-53)

MOROCCO One of these three contains her heavenly pictureIst like that lead contains her

Twere damnation To think so base a thought it were too grossTo rib her cerecloth in the obscure graveOr shall I think in silver shes immurdBeing ten times undervalued to tried gold

(i) What meaning does the Prince of Morocco find out of the inscription of the golden casket What have Belmont and Portiarsquos house been called and why

The inscription on the golden casket is ldquoWho chooseth me shall gain what many men desirerdquo The Prince finds out that it means that the chooser of the golden casket will get Portia because many men desire her In fact the entire world desires her Because of the coming of many suitors to Belmont from different countries in order to win Portiarsquos hand Belmont has become a centre of pilgrimage and her house is the shrine where saintly Portia is installed

(ii) What does the Prince of Morocco do before making the final choice of the casket Which is the correct casket and who will win Portiarsquos hand

The Prince of Morocco surveys and analyses the inscriptions on the casket of lead silver and gold Before making the final choice like a very systematic and methodical person he once again considers the claims of the caskets The casket containing Portiarsquos picture is the correct casket and the person choosing it will win Portiarsquos hand

Explanation

While praising Portia the Prince of Morocco conceives Portia as a goddess whose image is placed inside one of the caskets Many suitors are coming from far and wide the north and the south the east and the west (Four corners) in order to try their luck Some of them have come from the distant land of Persia and Arabia The deserts of Persia (Hyrcanian deserts) and the boundless desolate lands of Arabia have been crossed by the Princes seeking the hand of Portia All this shows that Portia is indeed the most beautiful lady of the world

(iii) What does the Prince of Morocco say in his estimation while examining the motto on the silver casket What does he find in the golden casket

While examining the motto on the silver casket which says ldquoWho chooseth me shall get as much as he deservesrdquo Morocco says that in his own estimation he surely deserves Portia in all respects ndash rank birth wealth etc

He chooses the golden casket When he opens it he finds an empty human skull holding a scroll in which it is written that those who are attracted by the glittering outside of things are always deceived as Morocco has been deceived

(iv) What kind of nature does the Prince of Morocco have

The Prince of Morocco has a simple nature who does not look deeply into the inner meaning of things but is dazzled by the outward appearance of gold He is inclined to over-estimate his own value and does not realize that it is a duty to ldquogive and hazardrdquo To say that he will not hazard for lead shows that he misreads the true meaning of the inscription which is that he should be prepared to ldquohazard all he hathrdquo for Portia So his feeling is only one of fascination and romantic attraction

(v) Do you think that the lottery of the caskets is not a matter that will be determined by chance

In fact the lottery of the casket is not a matter that will be determined by mere chance but that it is a true test of character and of sincerity which is amply proved not only by Moroccorsquos choice but also by the arguments which he uses to help him in his choice

(Act II Sc 7 L 55-59)

They have in England

A coin that bears the figure of an angelStamped in gold but thats insculpd uponBut here an angel in a golden bedLies all within

Context

(3)

(Act II Sc 7 L 63-77)A carrion Death within whose empty eye

There is a written scroll Ill read the writing

All that glisters is not goldOften have you heard that toldMany a man his life hath soldBut my outside to beholdGilded tombs do worms infoldHad you been as wise as boldYoung in limbs in judgment oldYour answer had not been inscrolld

This passage occurs in Act II Scene 7 in The Merchant of Venice This is part of the speech made by the Prince of Morocco

Explanation

In this passage the Prince of Morocco bestows high praise on Portia whose hand he is seeking He contrasts this casket containing Portiarsquos portrait with the old English gold coin bearing the image of the archangel (angel of the highest rank) He goes on to remark that while the figure of the archangel is engraved (Insculped) upon the English coin the picture of Portia who is beautiful as an angel lies hidden inside one of the caskets namely the Golden Casket (Golden Bed) In the day of Elizabeth silver was ten times inferior in value to gold Therefore the Prince of Morocco believing that Portiarsquos portrait is contained in the Golden Casket decides to choose the Golden Casket

Fare you well your suit is coldCold indeed and labour lostThen farewell heat and welcome frostmdashPortia adieu I have too grievd a heartTo take a tedious leave Thus losers part

(i) What reward does the Prince of Morocco get after making a wrong choice of the Casket How does he feel

After making the wrong choice in selecting the casket of gold the Prince of Morocco as a reward earns a rebuke in the form of a scroll tucked in the empty eye-socket of a skull kept in the casket of gold The Prince is shocked and disappointed He becomes all the more sad and dejected when he reads the scroll which points to his foolishness in being misled by the appearance and outward show as indicative of its worth

(ii) How does the Prince respond after reading the scroll

After reading the scroll the Prince though upset accepts the result with good grace and decorum befitting a royal suitor and true sportsman He says that his love-suit is really cold otherwise he would have chosen correctly but now his efforts have been in vain So he bids farewell to Portia to the warmth and enthusiasm of love and welcomes the cold and bitterness of dejection and misery of life which lies ahead

(iii) What request does he make to Portia and why

After being failure in his mission he requests Portia to give him permission to leave at once because he is too sad to undergo the tediousness of a formal leave-taking He tells that it is the manner in which defeated persons part unceremoniously

(iv) Explain the following lines

ldquoAll that glisters is not goldOften have you heard that toldMany a man his life hath soldBut my outside to beholdGilded tombs do worms infoldrdquo

Mere glitter does not make a metal to be gold Man has often been warned against appearance but it has been of no use Many people have sacrificed their lives only to seek the outer appearance of gold Worms are found inside the gilded

monuments

Class XSubject Topic Summary Execution

Hindi 2ndlang

नया रासता भाग 6 मायाराम 0ी घर म धनी मल 0ी और उनी बटी सरिरता ी ही चचा बनी रहती थी अमिमत ो इसम ोई रलिच ना थी वह धनी घर ी लडी स शादी र सवय ो बचना नही चाहता था उसा भी सवाणिभमान ह ईशवर ी पा

स उस पास पस ी ोई मी नही थी अभी उसन फकटरी ही लगाई थी उसी समझ बाहर था कि उस घर वालो ा झाव पस ी तरफ कयो

ह उसन मा स सवाल किया कि मा तम सरिरता स मरी शादी कयो रना चाहती हो मा न उस समझाया कि वह दखन म बरी नही ह और किफर खानदान अचछा

ह वह ए शल गरहणी रप म घर सभाल सगी अमिमत न मा ो इस बात ा एहसास राया कि मीन सबध लिलए मना रन पर उस दिदल

पर कया बीती होगी मा और अमिमत ी लडी बार म ाफी बात हईमा ा झाव सरिरता ी तरफ था कयोकि वह घर पर अचछा दह0 लर आ रही

थी अमिमत न अपनी मौसी ी बरी हालत बार म बताया कि किस तरह वह बड घर ी खानदानी बटी लाई थी और आ0 उसी हालत कितनी खराब ह लाई थी बहकलब 0ाती ह और बचचो ो भी नही दखती ह बात चल ही रही

थी कि तभी ए ार बाहर आर री धनी मल0ी घर अदर आए और पीछ स डराइवर फल ी ए टोरी लर आया अदर आए और पीछ स

डराइवर ए टोरी फल ी लर आया अमिमत ो फल ी पटी बरी लग रही थी अमिमत न पछ लिलया यह फल कयो ल आए ह प इन सब ी कया

0ररत थी उनो न 0वाब दिदया कि 4 पटी शमीर स मगाए थ अमिमत ो या सनर करोध आ गया तभी उस किपता 0ी आ गए उन आत ही अमिमत उठर बाहर चला गया वहा वहा मा पास आर बठ गया और बोला

अभी रिरशता तय नही हआ और धनी मल 0ी धनी मल 0ी फल ी पटी लर चलआय मा न समझाया कि 0ब सबध 0ड 0ाता ह तो खाली हाथ नही

आत अमिमत न मा स हा कि तम सबन सरिरता ो इस घर म लान ी ठान रखी ह धनीमल 0ी उस दिदन सरिरता ो दखन ी तारीख तय रन आय थ

Commercial Studies

Banking Nowadays Bank provide easy and quick services through internet facilities methods of Banking is called internet bankingIn order to save the time and money involved in visiting Bank branches people increasingly prefer to have internet banking

There are different modes of doing internet banking or transferring money through online They areReal Time Gross Settlement (RTGS)National Electronic Fund Transfers (NEFT)

1

Question

1) Explain the term RTGS Write the features of RTGS

Answer)The acronym RTGS stands for Real Time Gross Settlement which may be defined as the continuous real time settlement of funds transfer individually on and order by order basis without netting lsquoReal timersquo may be defined as the processing of instructions at the time they are received rather than at some letter time lsquoGross settlementrsquo may be defined as the settlement of transfer instructions which occurs

individually

Features of RTGS1It is the continuous settlement of

funds transfer individually on an order by order basis

2RTGS facility is provided only by CBS core banking solution enabled Bank branches

3Amount charged from the customer for RTGS transactions vary from bank to bank

2) Explain the term NEFT Write the features of NEFT

Answer) National electronic funds transfer may be defined as a nationwide system that facilitates individuals Farms and copper operates to electronically transfer funds from any bank branch to any individual farm or corporate having an account with any other bank branch in the country

Features of NEFT2 Transfer can be made 7 times on

weekdays and 6 times on Saturday

3 NEFT cannot be used to receive foreign remittances

4 NEFT transaction takes place in batches

5 A bank branch must be NEFT enabled to become a part of NEFT fund transfer network

6 There is no maximum or minimum amount that can be transferred through NEFT when one bank has a bank account

English Language

CompositionEssay

A composition is an art of creating a piece of writing on any topic or subject It is the writing correctly beautifully and clearly in order to make some interesting reading Structure of the composition

Introduction ( you lay the foundation for your composition)

Body (it constitutes the main part of the essay)

Conclusion (final statement that leaves a lasting impression)

Kinds of essays1 The Narrative essay2 The descriptive essay3 The reflective essay4 The argumentative essay

Write a composition on any one of the following topics (350- 400 words)

1 Friendship Or2 The first day of your school

Subject Eng Literature (The Merchant of Venice ndash William Shakespeare)Topic Act V Scene 1 Lines 127 to 158 (Nerissa helliphellip The clerk will nersquoer wear hair onrsquos face that had it) [Students should read the original play and also the paraphrase given in the school prescribed textbook]

Summary Revision Questions o Soon thereafter Bassanio Gratiano

and Antonio arrive

o Bassanio tells Portia that he is feeling as if it is morning because of the presence of Portia who is shining like the sun When Antonio is introduced by Bassanio to Portia she tells Bassanio that he should be grateful to Antonio who took so much trouble on his account even to the extent of risking his life

o Nerissa starts quarrelling with Gratiano and demands that he show her the ring she had presented to him and which she had warned him not to lose She suspects that Gratiano must have presented the ring to some young woman and not to the lawyerrsquos clerk as he repeatedly says and assures

Answer the following questions to check your preparation of Act IV Scenes 1 and 2

You must attempt only after you have completed your preparation of Act IV The answers must be in complete sentences using textual evidence (with citation) when necessary

[It would be in your own interest to attempt the above questions honestly totally refraining from consulting your textbook or your notes during answering After completion you should correct the paper yourself consulting the textbooknotes etc and award marks as specified Please let me know the marks you scored through WhatsApp in the group or to my personal WhatsApp]

Act IV Scene 1 (each question carries 2 marks)

1 What did the Duke try to do for Antonio

2 Why does Shylock refuse to show mercy How does he justify his stance

3 Why does Antonio say he is ready to die 4 What information is contained in Bellariorsquos letter

5 Why does Portia (as Balthazar) assert that Shylock must show mercy How does he respond

6 What offers are made to Shylock to get him to spare Antonio How are they received

7 What does Antoniorsquos speech as he faces the prospect of Shylockrsquos knife tell you about his character

8 How do Bassanio and Gratiano react to the looming prospect of Antoniorsquos demise

9 How does Portia (as Balthazar) use the law to turn the tables on Shylock

10 What does the Duke decree should happen to Shylock Why What happens to Shylockrsquos estate

11 What does Portia ask Bassanio as payment for her ldquoservicesrdquo What is his initial response What makes him change his mind

Act IV Scene 2 (each question carries 1frac12 marks)

1 What does Gratiano bring to Portia (Balthazar)

2 What does Nerissa plan on getting from Gratiano What does Portiarsquos comment suggest about men

ECO-10 280620 Topic-Supply AnalysisSHIFTING OF SUPPLY

But if there is change in factors other than the price of the commodity then either more is supplied at the same price or less supplied at the same price In such cases the price of the commodity remains constant but there is a change in other factors like change in the price of inputs change in technology of production change in price of other related goods change in taxation policy of the government etc For example there is an improvement in the technology of production of the commodity in question It leads to decrease in per unit of cost production of the commodity The firm is willing to sell more quantity of the commodity at the same price So the supply other commodity increases at the same price This increase in supply is shown by rightward shift of supply curve On the other hand if the firm uses inferior technology of production the cost of production per unit of the commodity increases The firm is willing to sell less quantity at the same price So the supply of the commodity decreases at the same price This decrease in supply is shown by leftward shift of the supply curve The above cases of increase and decrease in supply can be shown with the help of the following figures

Y INCREASE OF SUPPLY Price (Rs) s

P A s1

B

s

X` O s1 X

q q1

Y` Quantity demanded (in units)

Y DECREASE IN SUPPLY s2

s

price (Rs)

C

p A

s2

s

X` o X

q2 q

Y` Quantity demanded ( in units)

Main factors causing increase in supply or rightward shift of supply Curve(i) Fall in the price of other related goods

(ii) Fall in the price of inputsfactors(iii) Use of better technology in production(iv) Decrease in the rate of excise duty by government(v) If the objective of producer changes from profit maximization to salesMaximization

Main factors causing decrease in supply or leftward shift of supply curve(i) Increase in the price of other related goods(ii) Rise in the price of inputsfactors(iii) Use of inferior technology in production(iv) Increase in the rate of excise duty by the government(v) If the objective

Subject - Biology Topic ndash Chapter mdash6 PhotosynthesisSummary Execution

Today we will know about photosynthesis and its stages

Q1 What do you mean by photosynthesis The process by which living plants containing chlorophyll produce food

substances from carbon-di- oxide and water by using light energy Sunlight

6CO2 +12 H2O----------------------- C6 H12O6 + 6H2O + 6O2

Chlorophyll

Q2 What are the importance of photosynthesis I) Food for all Green plants trap solar energy by photosynthesis

process and supply food and energy for all living organisms either directly or indirectly

Ii) Oxygen to breathe in by product of photosynthesis is oxygen which is essential for all living organisms respiration

Q3 Write about two main phases of photosynthesis A Light dependent phase This phase occur in grana of chloroplast I) The chlorophyll on exposure to light energy becomes activated by

absorbing photons Ii) The absorbed energy is used in splitting the water molecules (H2O)

into its two components (H+ and OH- ) and releasing electron s 2H2O------------------------- 4H+ + 4e- +O2

Energy of 4 photons This reaction is known as photolysis

End products are H+ and oxygen water

B Light independent (Dark ) phase The reactions in this phase require no light energy

Here CO2 combine with H+ and produce glucose

Class XI

Subject Topic Summary ExecutionEVS Chapter-4 Legal

regimes for sustainable development

Environmental legislationEnvironmental legislation is the collection of laws and regulations pertaining to air quality water quality the wilderness endangered wildlife and other environmental factors The act ensures that matters important to the environment are thoroughly

Learn -The Forest (Conservation) Act 1980

considered in any decisions made by federal agencies

The Forest (Conservation) Act 1980 The Forest (Conservation) Act 1980 an Act of the Parliament of India to provide for the conservation of forests and for matters connected therewith or ancillary or incidental thereto It was further amended in 1988 This law extends to the whole of IndiaObjects and Reasons of the Forest Conservation Act

Deforestation causes ecological imbalance and leads to environmental deterioration Deforestation had been taking place on a large scale in the country and it had caused widespread concern The act seeks to check upon deforestation and de-reservation of forests

Subject Eng Literature (The Tempest ndash William Shakespeare) Topic Act II Scene 1 Lines 314 to 329 (End of scene)

[Students should read the original play and also the paraphrase given in the school prescribed textbook]Summary Questions amp Answers

Conspiracy of Antonio and Sebastian (Contd)

o As they approach Ariel appears again and wakes up Gonzalo by singing a tune in his ear Alonso also wakes up and they see both Sebastian and Antonio with drawn swords On being caught off guard they make up a story saying that they had heard a bellowing of bulls or lions

o They then moved to another part of the island

o Ariel at once rushes to Prospero to inform him of this development

SUMMING-UP of ACT-2 SCENE-1

(i) Among the survivors Ferdinand is separated from the rest which results in the disconsolate grief of Alonso as he took him for dead

(ii) The villainy of Antonio is confirmed

(iii) The supremacy of Prosperorsquos magic which resulted in the failure of the human conspiracy

(1)

(Act II Sc 1 L 311-325)SEBASTIAN Whiles we stood here securing your repose

Even now we heard a hollow burst of bellowing Like bulls or rather lions Didt not wake youIt struck mine ear most terribly

ALONSO I heard nothingANTONIO O rsquotwas a din to fright a monsters ear

To make an earthquake Sure it was the roarOf a whole herd of lions

ALONSO Heard you this GonzaloGONZALO Upon mine honour sir I heard a humming

And that a strange one too which did awake meI shaked you sir and cried As mine eyes opened I saw their weapons drawn There was a noiseThats verily rsquoTis best we stand upon our guardOr that we quit this place Lets draw our weapons

(i) Why has Prospero sent Ariel to Gonzalo and Alonso What does Ariel do to awaken Gonzalo

Prospero has already come to know by his magic powers the danger which threatens Gonzalo who had been Prosperorsquos friend and so he sent Ariel to preserve the lives of both Gonzalo and Alonso Prospero does not want that his scheme should remain unfulfilled Ariel begins to sing a song in Gonzalorsquos ears to awaken him(ii) Who are ready to carry out their plan Who takes steps to stop them Why does Gonzalo feel surprised after being awakened

Sebastian and Antonio are ready to carry out their plans They are standing with their swords drawn to kill Alonso and

(iv) We see two sets of contrasting characters Gonzalo-Adrian against Antonio-Sebastian

(v) The grief that works in Alonso can be perceived to his repentance for his association in Antoniorsquos crime against Prospero

Gonzalo Ariel takes steps to stop them from carrying out their nefarious scheme When Gonzalo is awakened by the song sung by Ariel into his ears he (Gonzalo) feels surprised because he sees Sebastian and Antonio standing with their swords drawn(iii) What reason do Sebastian and Antonio tell of drawing their swords when they are suspected by Alonso and Gonzalo

When Sebastian and Antonio are seen with their swords drawn they are looked with suspicion by Gonzalo and Alonso At first Sebastian tells them that as they stood here to guard them during their sleep they heard only a little before a sudden loud noise very much like the roaring of bulls or more probably that of lions Then Antonio follows him saying that this was a noise so terrible as to frighten even a monsterrsquos ears and this noise could even have shaken the earth and it was surely like the roaring of a multitude of lions Then seeing the danger they have drawn their swords Perhaps after hearing the terrible noise they (Gonzalo and Alonso) woke up from their sound sleep

(iv) What does Gonzalo tell Alonso about the strange noise What did he see on opening his eyes Gonzalo tells Alonso that he did not hear the sound of roaring but he heard a humming sound which was strange and which woke him up After waking up he gave him (Alonso) a shaking and a loud cry On opening his eyes he saw these two gentlemen standing with their swords drawn(v) What does Gonzalo suggest

Gonzalo suggests that there was a noise indeed and of that he has no doubt at all and suggests that the best course for them would be to remain alert and vigilant against any possible danger to their lives or to leave this place and move to some other part of the island

Class XIISubject Topic Summary Execution

Commerce

Chapter- Management

Today we will discuss about LEVELS OF MANAGEMENT

Levels of management is a series or chain of managerial positions from top to bottom It helps individuals to know their authority responsibilities and superior-subordinate relations among themselves There are mainly three levels of Management TOP LEVEL MANAGEMENTMIDDLE LEVEL MANAGEMENTLOWER LEVEL MANAGEMENT

Top level managementIt consists of members at the highest level in the management hierarchy This level includes Board Of Directors Chief Executive Managing Directors Chairman President Vice President

Rolefunctions of the top levelmanagement1To analyse evaluate and deal

with theexternal environment2 To determine the objectives and

policies of the business3 To strive for welfare and survival

of business

4 To create an organisational Framework consisting of authority responsibility relationship

Middle level management Congress of members or groups who are concerned with implementation of the policies let down by the top managementThis level includes head of the department such as finance manager marketing manager branch and regional managers departmental and divisional heads plant superintendent etc

Role of functions of the middle level management

1 To interpret the policies framed by top management

2 To assign duties and responsibilities to lower level managers

3 To select and appoint employees for middle and supervisory level and evaluate their performance

4 To co-operate with other departments for smooth functioning

Operational or supervisory level managementIt refers to the group are members who are concerned with execution of the work They are also known as fast line managers This level includes supervisor 4 men Section Officer clerk Inspector etc

Role of functions of the lower level management1 To plan and execute day-to-

day operations2 To supervise and control the workers3 To arrange materials and

tools to start the process and make arrangements for training

4 Today present workers grievance and suggestions before the management and

ensure safe and proper working conditions in the factory

Business Studies

Staff Appraisal Chapter- 10 Today let us start with a new chapter

Staff Appraisal

Meaning of Performance Appraisal

Performance Appraisal is the systematic evaluation of the performance of employees and to understand the abilities of a person for further growth and developmentThe supervisors measure the pay of employees and compare it with targets and plansThe supervisor analyses the factors behind work performances of employeesThe employers are in position to guide the employees for a better performance

Objectives of Performance Appraisal

Following are the objectives of Performance Appraisal

To maintain records in order to determine compensation packages wage structure salaries raises etc

To identify the strengths and weaknesses of employees to place right men on right job

To maintain and assess the potential present in a person for further growth and development

To provide a feedback to employees regarding their performance and related status

To provide a feedback to employees regarding their performance and related status

Importance of Performance Appraisal

Performance appraisal provides important and useful information for the assessment of employees skill

knowledge ability and overall job performance The following are the points which indicate the importance of performance appraisal in an organization

1 Performance appraisal helps supervisors to assess the work performance of their subordinates

2 Performance appraisal helps to assess the training and development needs of employees

3 Performance appraisal provides grounds for employees to correct their mistakes and it also provides proper guidance and criticism for employees development4 Performance appraisal provides reward for better performance

5 Performance appraisal helps to improve the communication system of the organization

6 Performance appraisal evaluates whether human resource programs being implemented in the organization have been effective

7 Performance appraisal helps to prepare pay structure for each employee working in the organization

8 Performance appraisal helps to review the potentiality of employees so that their future capability is anticipated

Geography

DRIANAGE The SubarnarekhaThe Subarnarekha and the Brahmaniinterposed between the Ganga and the Mahanadi deltas drain an area of 19300 sq kmand 39033 sq km respectively The drainage basins of these streams are shared byJharkhand Odisha west Bengal and Chhattisgarh The Brahmani is known as southKoel in its upper reaches in Jharkhand

The NarmadaThe Narmada rises in the Amarkantak hills of MadhyaPradesh It flows towards the West in a rift valleyformed due to a geological fault The total length of it is 1300 km All the tributaries of the

Q1 Name the two westward flowing rivers in the peninsular plateauA1 Narmada and Tapi are the only westward flowing rivers of the peninsular plateau

Q2 Differentiate between east-flowing rivers and west-flowing riversA2

East-flowing rivers

West-flowing rivers

Narmada are very short inlength Most of its tributaries join the main streamright anglesThe Narmada basin covers parts of Madhya Pradesh and Gujarat

The Tapi The Tapi rises in the Satpura ranges in the Betul listrictof Madhya Pradesh It flows in a rift valley parallel tothe Narmada but it is much shorter in length It coversparts of Madhya Pradesh Gujarat and MaharashtraThe length is about 724 km

The Sabarmati and the MahiThe Sabarmati rises in the Aravali hills and flows south-south-westwards for a distance of 300 kilometres to the Arabian Sea The Sabarmatibasin extends over an area of 21674 sq km in Rajasthan and Gujarat The Mahi rises inthe east of Udaipur and drains an area of 34842 sq km lying in Madhya PradeshRajasthan and Gujarat It flows south-westwards for a distance of 533 km before it fallsinto the Gulf of Khambhat

The ChambalThe Chambal rises near Mhow in the Vindhya Range and flows towards the northgenerally in a gorge upto Kota Below Kota it turns to the north-east direction and afterreaching Pinahat it turns to the east and runs nearly parallel to the Yamuna beforejoining it in the southern part of the Etawah district in Uttar PradeshMajor Rivers of India with their basin area (Sqkm)

Himalayan System Indus 321290Ganga 861404

Brahmaputra 187110Indus System

Jhelum 34775Beas 20303

Ganga System Yamuna 366223Ghaghra 127950

Peninsular RiversNarmada 98796

Tapi 65145Mahanadi 141600

Subarnarekha 19300Sabarmati 21674

Mahi 34842Godavari 312812

Godavari Krishna Kaveri Mahanadi are the east-flowing rivers

Narmada Tapi west-flowing rivers

They fall into the Bay of Bengal

They fall into Arabian Sea

These rivers form big deltas

These rivers form comparativelysmall deltas

Catchment areas of these rivers are larger

Catchment areas of these rivers are smaller

Krishna 2589488Cauveri 87900

Subject ndashBiology Topic ndashChapter -5 Inheritance amp Variations Summary ExecutionToday we will discussabout linkage and its classification

LINKAGE The tendency of the genes located on the same chromosome to stay together is

hereditary transmission Linked genes the genes responsible for this Genes that exhibit the process of linkage locates in the same chromosome The distance between the linked genes in a chromosome determines the strength

of linkage i e genes that are located close to each other show stronger linkage than that are located far from each other

COMPLETE LINKAGE It is the type of linkage showed by the genes that are closely located or are tightly

linked with each other as they have no chance of separatingby crossing over These genes are always transmitted together to the same gamete and the same

offspring In such condition only parental or non cross over type of gametes are formedINCOMPLETE KINKAGE It is type of linkage showed by the genes that are distantly located orare loosely

linked with each other because they have chance of separating by crossing over

SIGNIFICANCE i) It helps in holding the parental character togetherii) It checks the appearance of new recombination and helps in bringing the

hybrid population which resembles the original parents iii) Linked genes dilute the effects of undesirable traits

Subject Eng Literature (The Tempest ndash William Shakespeare) Topic Essay Questions (EQ-3)Question No 3

Give a character sketch of CalibanAnswer

The character of Caliban has been wonderfully conceived by Shakespeare as the manifestation of all that is gross and earthy ndash a sort of creature of the earth as Ariel is a sort of creature of the air

Calibanrsquos Physical Appearanceo Caliban is lsquofreckledrsquo a lsquomisshapen knaversquo not honoured with human shape

o Prospero calls him lsquothou tortoisersquo (Act I Sc 2 Line 317) Trinculo stumbling upon him describes him as ldquoA strange fish hellip Legged like a man And his fins like armsrdquo He ldquosmells like a fishrdquo (Act II Sc 2 Line 25)

o Prospero also calls him a ldquobeastrdquo (Act IV Sc 1 Line 140) and ldquoThis misshapen knaverdquo (Act V Sc 1 Line 268)

o Further it appears that in addition to his physical deformity his spiritual inferiority is also suggested by Prosperorsquos claim that his birth resulted from the union between his mother the witch Sycorax and the devil

Calibanrsquos ParentageWhen the play opens Caliban is twenty four years of age having been born on the island twelve years before the coming of Prospero His mother was the foul witch Sycorax who was banished from Algiers for ldquomischiefs manifold and sorceries terrible to enter human hearingrdquo (Act I Sc 2 Line 264) and the father was the Devil himself Thus

Caliban is a monster of evil and brute nature ugly deformed and stinking

Calibanrsquos Savage and Malignant Natureo Caliban is entirely a creature of the earth ndash gross brutal and savage He regards himself as the rightful possessor

of the island and Prospero as a usurper

o In his young age he was on good terms with Prospero He had consented to be received by Prospero at his house and to be educated by him He has learnt human language only to curse his master whom he abhors

o His beastly nature soon breaks out and ends in a vicious attack on Miranda This opens the eye of Prospero who becomes severe to him and enforces his service by threats and violence

o Prospero uses him to make dams for fish to fetch firewood scraper trenches wash dishes and keep his cell clean

Calibanrsquos Hatred for ProsperoA profound hatred for Prospero has taken hold of Caliban It springs from a sense of his being dispossessed and ill-treated He would kill Prospero if he could but he knows the power of Prosperorsquos lsquobookrsquo Hence he transfers his allegiance to Stephano who seems like a god to him He also incites the two drunken associates to batter the skull of Prospero when he sleeps in the afternoon

Caliban Shows Considerable Intelligenceo He has learnt Prosperorsquos language

ldquoYou taught me language and my profit onrsquot (Act II Sc 2 Lines 86-89)Is I know how to curserdquo

o He is well aware of the futility of arguing with one who has more power than he has

ldquoI must obey his art is such power (Act I Sc 2 Lines 373-376)It would control my damrsquos god SetebosAnd make a vassal of himrdquo

o He realizes the importance of Prosperorsquos books

ldquoRemember (Act III Sc 2 Lines 89-92)First to possess his books for without themHersquos but a sot as I am nor hath notOne spirit to commandrdquo

o He knows the value of stealth when attacking the enemy

ldquoPray you tread softly that the blind mole may not (Act IV Sc 1 Lines 194-195)Hear a foot fall we now are near his cellrdquo

o Caliban has a better set of values than Stephano and Trinculo They are distracted from their plan by their greed for Prosperorsquos rich garments Only Caliban realizes that such a finery is unimportant

ldquoLeave it alone thou fool it is but trashrdquo (Act IV Sc 1 Lines 224)

Caliban is not a good judge of characterCaliban is not a good judge of character He decides for example that Stephano is a god because he dispenses lsquocelestial liquorrsquo (Act II Sc 2 Line 115) but then it must be remembered that he has only known his mother Sycorax Prospero Miranda and the spirits that torture him However he quickly discovers his error of judgementrdquo

ldquoWhat a thrice-double ass (Act V Sc 1 Lines 295-297)Was I to take this drunkard for a godAnd worship this dull foolrdquo

Calibanrsquos Imaginative NatureIf Caliban is sub-human in what has been said above he is human in the respect of the poetic side of his character He listens to music with rapture He tells of the beautiful dreams in which heaven rains treasures upon him and which upon waking he yearns to renew One of the most poetic passages in whole play is Calibanrsquos description of the island

to Stephano and Trinculo

ldquoBe not afeard The isle is full of noises (Act III Sc 2 Lines 135-143)Sounds and sweet airs that give delight and hurt notSometimes a thousand twangling instrumentsWill hum about mine ears and sometime voicesThat if I then had waked after long sleepWill make me sleep again and then in dreamingThe clouds methought would open and show richesReady to drop upon me that when I wakedI cried to dream againrdquo

Caliban - Less Ignoble Than Some OthersCalibanrsquos motive for murder is less dishonourable than that of Antonio and Sebastian They plan to kill Alonso to gain his power and wealth Caliban merely wants revenge and the return of lsquohisrsquo island

Conclusiono Calibanrsquos character is not portrayed very clearly in the play and hence we cannot decide whether he is a poor

savage being grossly maltreated by Prospero or whether he is evil and must therefore be kept in bondage or enslavement

o Caliban is contrasted with Ariel who is a spirit and thus swift and uninterested in physical activitieso Caliban is also contrasted with Prospero who is the all-powerful master of the island and of the destiny of all

those on the islando Caliban is also contrasted with civilized man showing him to be less evil than Antonio and Stephano and less

materialistic than Stephano and Trinculoo Caliban has suffered at the hands of Prospero and he has learnt to curse by listening to Prosperorsquos abuse He

certainly believes that Prospero has deprived him of his birthrighto Finally the character Caliban is thought to be one of Shakespearersquos masterpieces The complexity of the character

is reflected in the large volume of critical discussion that has grown around it

ECO ndash12 Topic-Forms of market

MonopolyMonopoly is a market structure in which there is a single seller there are no close substitutes for the commodity produced by the firm and there are barriers to entry Example Indian Railways which is operated under government of India Monopoly also implies absence of competitionFeatures of Monopoly Monopoly is characterized by1 Single Seller In monopoly there is only one firm producing the product The whole industry consists of this single firm Thus under monopoly there is no distinction between firm and industry Being the only firm there is significant control of the firm over supply and price Thus under monopoly buyers do not have the option of buying the commodity from any other seller They have to buy the product from the firm or they can go without the commodity This fact gives immense control to the monopolist over the market

2No Close Substitute There are no close substitutes of the product produced by the monopolist firm If there are close substitutes of the product in the market it implies presence of more than one firm and hence no monopoly In order to ensure a total of control over the market by the monopolist firm it is assumed that there are no close substitutes of the product

3 No Entry amp Exit Monopoly can only exist when there is strong barriers before a new firm to enter the market In fact once a monopoly firm starts producing the product no other firm can produce the same One reason for this is the ability of the

monopolist to produce the product at a lower cost than any new firm who thinks to enter the market If a new firm who knows that it cannot produce at a lower cost than the monopolist then that firm will never enter the market for fear of losing out in competition Similarly the monopolist who is operating for a long time may be enjoying reputation among its customers and is in a better position to use the situation in its own benefit A new firm has to take long time to achieve this and so may not be interested to enter the market

4 Price Maker Being the single seller of the product the monopolist has full control over the pricing of the product On the other hand if there is a large number of buyers in the market so no single buyer exercises any significant influence over price determination Thus it is a sellerrsquos market So monopoly firm is a price maker

5 Price Discrimination Having considerable control over the market on account of being single seller with no entry of other firms the monopolist can exercise policy of price discrimination it means that the monopolist can sell different quantities of the same product to a consumer at different price or same quantity to different consumers at different prices by adjudging the standard of living of the consumer

6 Shape of Demand Curve Since a monopolist has full control over the price therefore he can sell more by lowering the price This makes the demand curve downward sloping

Subject Ac-12 290620 Topic- retirement Model sumThe Balance Sheet of Rohit Nisha and Sunil who are partners in a firm sharing profits according to their capitals as on 31st March 2014 was as under

Liabilities Amount Assets Amount (Rs) (` Rs)

Creditors 25000 Machinery 40000Bills Payable 13000 Building 90000General Reserve 22000 Debtors 30000Capital Less Provision for Rohit 60000 Bad debts 1000

29000 Nisha 40000 Stocks 23000 Sunil 40000 140000 Cash at Bank 18000

200000 200000

On the date of Balance Sheet Nisha retired from the firm and following adjustments were made(i) Building is appreciated by 20(ii) Provision for bad debts is increased to 5 on Debtors(iii) Machinery is depreciated by 10(iv) Goodwill of the firm is valued at Rs 56000 and the retiring partnerrsquos share is adjusted

(v) The capital of the new firm is fixed at Rs120000 Prepare Revaluation Account Capital Accounts of the partner and Balance Sheet of the new firm after Nisharsquos retirement Revaluation AccountDr Cr

Particulars Amount Particulars Amount (`Rs) (Rs`)

Provision for Bad debt Ac 500 Building Ac 18000Machinery Ac 4000Profit transferred toCapital Accounts (3 2 2)Rohit 5786Nisha 3857Sunil 3857

13500

18000 18000

Capital Account

Dr Cr

Particulars Rohit Nisha Sunil Particulars Rohit Nisha Sunil (Rs`) (Rs`) (`Rs) (Rs`) (Rs`) (Rs`)

Sunilrsquos Capital ac 9600 mdash 6400 Balance bd 60000 40000 40000Bank - 66143 - General Reserve 9428 6286 6286Balance cd 72000 mdash 48000 Revaluation (Profi 5786 3857 3857 Rohitrsquos Capital Ac mdash 9600 mdash

Sunilrsquos Capital Ac 6400 Bank 6386 - 4257

81600 66143 54400 81600 66143 54400

Balance Sheet as at 31st March 2014

Liabilities Amount Assets Amount (Rs`) (Rs`)

Creditors 25000 Building 108000Bank overdraft 37500 Machinery 36000

Bills Payable 13000 Debtors 30000Capital Less ProvisionRohit 72000 for Bad debts 1500 28500Sunil 48000 120000 Stock 23000

195500 195500

Working Notes (i) (a) Profit sharing ratio is 60000 40000 40000 ie = 3 2 2(b) Gaining Ratio Rohit = 35 ndash 37 = 2135 ndash 1535 = 635Sunil = 25-27 = 1435 ndash 1035 = 435= 635 435= 6 4 = 3 2(c) Nisha Share of Goodwill = Rs 56000 times 27 = Rs16000Share of Goodwill in the gaining ratio by the existing partner ieRohit = Rs16000 times 35 = Rs 9600Sunil = Rs 16000 times 25 = Rs 6400

The journal entry isRohitrsquos Capital Ac Dr 9600Sunilrsquos Capital Ac Dr 6400 To Nisharsquos Capital Ac 16000(Share of Goodwill divided into gaining ratio)

  • 1 Static Friction
  • The frictional force that acts between the surfaces when they are at rest with respect to each other is called Static Friction
    • Static Friction Examples
      • 2 Sliding Friction
        • Examples Of Sliding Friction
          • 3 Rolling Friction
            • Examples Of Rolling Friction
              • Objects and Reasons of the Forest Conservation Act
Page 13:  · Web viewSubject . Topic . Summary . Execution . English 1 . Sounds of animals . Hens –cackle Horses –neigh Lions –roar Owls –hoots Snake –hiss. English 2 . Mother’s

B Multiplication of a fractional number by a fractional number To multiply a fractional number by a fractional number we multiply the numerator of the first fractional number by the numerator of the second fractional number and the denominator of the first fractional number by the denominator of the second fractional number The first product thus obtained is the numerator and the second product is the denominator of the required product

16 2712

times24

Solution 2712

times24 = 3112

times24

= 31times2 = 62

Exercise ndash 31

11 83

times 34

2

Solution 83

times 34 = 2

14 723

times2 25

4

Solution 723

times2 25 =

233

times 125 =

23times 45

= 925 = 18

25

15 1212

times1 13

2

Solution 1212

times1 13 =

252

times 43 =

25times 23

= 503 = 16

23

State the following statements are true or false

17 1912

times 239 = 1

Solution LHS = 1912

times 239

= 392

times 239 = 1 = RHS

[LHS = Left hand side amp RHS = Right hand side]

there4 1912

times 239 = 1 [True]

21 213

times2 13 = 4

19

Solution LHS = 213

times2 13 =

73

times 73

= 7times73times3 =

499 = 5

49

there4 LHS ne RHS

So 213

times2 13 = 4

19 [False]

23 23

times 45 =

2times 5+3 times43times 5

Solution

LHS = 23

times 45 =

2times 43 times5 =

815 again

RHS = 2times 5+3 times4

3times 5 = 10+12

15 = 2215

there4 LHS ne RHS So 23

times 45 =

2times 5+3times43times 5

[False]

25 23 of

13 =

29

Solution

LHS= 23 of

13 =

23 times

13 =

29 = RHS]

there4 23 of

13 =

29 [True]

Practice at HomeExercise ndash 31State the following statements are true or false

24 12 of 4 =

18

Class VISubject Topic Summary Execution

HISTORY AND CIVICS

Chapter 5The Mauryan Empire

DECLINE OF MAURYAN EMPIREDecline of Mauryan empire started after the death of Ashoka at around 232 BCThere are several reasons for break up of the empire1 Weak successor Emperors after Ashoka were

capable of handling vast and mighty Mauryan empire In 185BC the last Mauryan ruler Bri-hadrath was murdered by his Commander-in-Chief Pushyamitra Sunga

2 Provincial Revolts Due to weak central author-ity provincial chiefs of Kalinga and southern provinces revolted against emperor and freed themselves from Mauryan empire

3 Weakness of Economy Prosperity of Mauryan was based on solid economic activities which

ExercisesI Multiple choice questions-1 Chandragupta defeated Seleucus in the year ndashc) 305 BC2 Who killed the last Mauryan ruler Brihadrath b) Pushyamitra3 Which of the following was not a reason for the decline of the Mauryan empirec) Chandraguptarsquos weakness4 Ashoka invaded Kalinga in the year c) 261 BC

II Fill in the blanks1Chandragupta ascended the throne in 324

was taken care by early monarchs Later kings had neither ability nor interest in economic af-fairs That led to failure in tax collection As a result they failed to maintain a large army that were essential to keep empire intact

4 Greek Invasion Greeks freed north-western provinces from weak Mauryan monarchs and reestablished their authority

5 Ashokarsquos Policy some scholar opined that after Kalinga war Ashoka embraced Buddhism re-nounced the policy of war and disbanded the Army But this is partially true as there is no proper evidence of disbanding the army

Based on above points we can conclude that main reason for decline of Mauryan empire is weakness of Ashokarsquos successors Kunal Samprati Dasharath Salisuk all were weak kingsAt last in 185 BCPushyamitra Sunga killed king Brihadrath and established the Sunga dynasty

BC2 Bindusara was the son of Chandragupta and father of Ashoka

3 Pataliputra was administered by City Magistrate committess of 5 members each4 The Greek General Seleucus sent his ambassador Megasthenes to Chandraguptarsquos court5 Ashoka sent his son Prince Mahendra and daughter Sanghamitra to spread his Dhamma6 The Indian Rebublic has adopted the Lion Capital of Saranath Pillar as its national emblem 7 Pushyamitra killed the last Mauryan ruler Brihadrath and founded the Sunga dynasty

III Name the following

1The author of Arthashastra-Kautilya2 The ruler who founded the Mauryan dynasty-Chandragupta3 The author of Indika-Megasthenes 4 The officers who were appointed by Ashoka to spread Dhamma-Dhamma Mahamatras5 The general of Alexander whom Chandragupta defeated-Seleucus

V Match the columns1 Kautilya (c)2 Megasthenes (d)3 Pushyamitra (e)4 Brihadrath (b)5 Bindusara (a)

BENGALI(2ND

LANGUAGE)

পশপাহিখর -াষাসহিবনয় রায়কেচৌধরী

যলখক পহিরহিচহিত- পরখযাত সাহিহিতযক উকেপনদরহিককেশার রায়কেচৌধরীর পতর সহিবনয় রায়কেচৌধরী lsquoসকেFশrsquo পহিতরকার সকেb হিতহিন কত হিকেলন তার উকেdখকোয বই lsquoসহিবনয় রায়কেচৌধরীর রচনা সংগরrsquo

পরম হিকেনর পাঠ- lsquoপশপাহিখর হিক -াষাhelliphellip helliphelliphelliphellipপরসপরকেক জানাবার উপায়ও পশপাহিখরা যবশ জাকেনrsquoপরকেমই আমারা জাহিন -াষা হিক -াষা ল আমাকের মকেনর -াব পরকাশ করার জনয আমরা নানান ধরকেনর -হিb বা হিবকেশষ ধরকেনর আওয়াজ মকেখর মাধযকেম কহির অনযকেক যবাঝাকেনার জনয তাকেল এবার আমরা জাহিন পশপাহিখর -াষা হিক পশ পাহিখরা হিক কা বকেল যা পশপাহিখকেরও -াষা আকে তারা তাকের হিনজসব -াষায় কা বকেল মকেনর -াব পরকাশ ককের পশ পাহিখরা মানকেষর হিক হিক -াষা যবাকেঝ হিকনত তারা বলকেত পাকেরনা পরসপরকেক বহিঝকেয় যবার উপায় তারা জাকেননা তকেব তারা হিবকেশষ ককেয়কটি শকেবদর মাধযকেম তাকের মকেনর -াব বহিঝকেয় যয় হিক বহিদধ মান জীব ndashককর হিবাল বন মানষ যঘাা পর-হিত এরা মানকেষর যওয়া নাম শনকেল কান খাা ককের ndash নাম ধকের ডাককেল কাকে আকেস যমন - মরহিরা lsquoহিত ndashহিতrsquo ডাক শকেন আকেস াল lsquoঅ ndashর -র ডাক শকেন কাকে আকেস াহিত মাহকেতর কা শকেন চকেল ককররা মাহিলকেকর হকম পালন ককের সবসময় তাইকেতা ককরকেক পর- -কত পরানী বলা য় ককর আর হিবাল একের আওয়াজ তহিম লকষয করকেল বঝকেব ককররা যরকে যকেল lsquoযঘউ যঘউrsquo করকেত াকেক আবার কাকেল lsquoযকউ যকউrsquo ককের হিবাল সাধারণ lsquoমযাওrsquo বা lsquoহিমউrsquo ককের রা কেল lsquoওয়াওrsquo আওয়াকেজর মাধযকেম মকেনর -াব পরকাশ ককের একেতা যল পশকের কা পাহিখরাও -য় রা পরকাশ করার জনয হিবকেশষ ধরকেনর শবদ ককের হিবপকের সময় পশ পাহিখরা সবার আকে পরসপরকেক জানাবার উপায় তারা জাকেন বহকাল

১) পশপাহিখর -াষা কেলপর যলখক সমপকেকG হিক জাকেনা

উঃ- পরখযাত সাহিহিতযক উকেপনদরহিককেশার রায়কেচৌধরীর পতর সহিবনয় রায়কেচৌধরী lsquoসকেFশrsquo পহিতরকার সকেb হিতহিন কত হিকেলন তার উকেdখকোয বই lsquoসহিবনয় রায়কেচৌধরীর রচনা সংগরrsquo হিতহিন ারকেমাহিনয়াম এসরাজ পর-হিত বাযনতর বাজাকেত পারকেতন ানও জানকেতন হিতহিন যাকেIাকের জনয মজাার লপ কহিবতা হিলখকেতন

২) পশপাহিখ কেলপর মল-াব হিকউঃ- পশপাহিখকেরও -াষা আকে তারা তাকের হিনজসব -াষায় কা বকেল মকেনর -াব পরকাশ ককের পশ পাহিখরা মানকেষর হিক হিক -াষা যবাকেঝ হিকনত তারা বলকেত পাকেরনা পরসপরকেক বহিঝকেয় যবার উপায় তারা জাকেননা তকেব তারা হিবকেশষ ককেয়কটি শকেবদর মাধযকেম তাকের মকেনর -াব বহিঝকেয় যয় হিরউকেবন কযাসটং সাকেব হিতহিন চহিdশ বর বনযজনত যর সকেb যকেককেন হিতহিন বকেলকেন আমরা হি তাকের -াষা তাকের আব কায়া যমকেন চহিল তাকেল আর -কেয়র যকান কারণ াকেকনা আমরা একI -াকেলাকেবকেস যচষটা করকেল পশপাহিখকের সকেb -াব পাতাকেত পাহির

ধকের মানষ এই পশ পাহিখর -াষা হিনকেয় নানা রককেমর পরীকষা ককের আসকে এইরকম একজন হিরউকেবন কযাসটাং সাকেকেবর কা আমরা জানকেবাhelliphellip

Hindi 2nd

langमतर किनमनलिलखिखतपरशनोउRरदीजि0ए

) बढ वयलि` बचच ो कया हआ था ख) डॉकटर साहबन पाटc किस उददशय स रखी थी ग) ाल साप ो हाथ म लर लाश न कया किया घ) डॉकटर चडढा न बढ पतरो दखन स कयो मना र दिदया था ङ) भगत न लाश ो दखर कया हा

उRर ndash) उस बहत बखार थी और 4 दिदनो स आख भी नही खोला थाख) उन बट ी सालकिगरह थीग) ाल सापो हाथ म लर लाश न उसी गदन 0ोर स दबार पडी थीघ) डॉकटर चडढा न बढ वयलि` पतरो दखन स मना र दिदया कयोकि उनह गोलफ खलन 0ाना थाङ) लाश ो दखर हा कि नारायण चाहग तो आध घट म भया उठ 0ाएग

English literature

In the bazaars of Hyderabad- Sarojini Naidu

Through the poem In The Bazaars of Hyderabad Sarojini wanted to convey the message that India is rich in tradition and they donrsquot need the foreign products So she goes on to give a picture of a bazaar where traditional Indian products are rulingThe poem is in the form of questions and answers The poet asks the questions and the merchants answer them Through this technique she make the picture of the bazaar visible to us

Read the poem

PHYSICS FORCE Types of FrictionThere are three types of friction static sliding rolling Static sliding and rolling friction occur between solid surfaces

1 Static Friction The frictional force that acts between the surfaces when they are at rest with respect to each other is called Static FrictionStatic Friction Examples

Skiing against the snow Creating heat by rubbing both the hands

together Table lamp resting on the table

2 Sliding Friction The resistance that is created between any two objects when they are sliding against each other is called Sliding FrictionExamples Of Sliding Friction

Sliding of the block across the floor Two cards sliding against each other in a

deck

3 Rolling Friction The force which resists the motion of a ball or wheel is called Rolling Friction Is the weakest types of frictionExamples Of Rolling Friction

Rolling of the log on the ground Wheels of the moving vehicles

6What effect can a force produce on a body which is not allowed to move Ans - When a force is applied on a body which is not free to move it gets deformed i e the shape or size of the body changes7Give one example each to indicate that the application of a force

1 produces motion2 stops motion3 slows down motion4 changes the direction of motion5 deforms a body

Ans- 1 A car originally at rest when pushed

begins to move2 A moving bicycle is stopped by

applying the brakes3 The speed of a moving vehicle is

slowed down by applying the brakes4 A player kicks a moving football to

change its direction of motion5 On stretching a rubber string its

length increases

8State the effect produced by a force in the following cases (a) The sling of a rubber catapult is stretched(b) A man pushes a heavy cart(c) A player uses his stick to deflect the ball (d) A cyclist applies brakes(e) A spring is compressedAns- (a) The shape and size of catapult changes ie its length increases(b) The heavy cart begins to move(c) The direction of the ball changes(d) The speed of the moving cycle is slowed down(e) There is change in size and shape of spring

COMPUTER MS EXCEL 2013 -INTRODUCTION

UNDERSTANDING EXCEL STRUCTUREA SPREADSHEET IS A FILE THAT EXISTS OF CELLS IN ROWS AND COLUMNS AND CAN HELP ARRANGE CALCULATE AND SORT DATA DATA IN A SPREADSHEET CAN BE NUMERIC VALUES AS WELL AS TEXT

FORMULAS REFERENCES AND FUNCTIONS

WORKSHEETA WORKSHEET IS ALSO KNOWN AS SPREADSHEETIT IS A COLLECTION OF CELLS ON A SINGLE SHEET WHERE YOU KEEP AND CHANGE DATA

WORKBOOKWORKBOOK IS PMS EXCEL FILE IN WHICH THE DATA CAN BE STORED EACH WORKBOOK CAN CONTAIN MANY WORKSHEETS

ROWS AND COLUMNSIN MS EXCEL A ROW IS A GROUP OF CELLS THAT RUN FROM LEFT TO RIGHT OF A PAGEA COLUMN IS A GROUPING OF CELLS THAT RUN FROM THE TOP TO THE BOTTOM OF A PAGE

CELLTHE INTERSECTION POINT BETWEEN A ROW AND THE COLUMN IS CALLED A CELL WHICH IS THE BASIC STORAGE UNIT FOR DATA IN A SPREADSHEET EACH CELL HAS SPECIFIC ADDRESS WHICH IS THE COMBINATION OF THE COLUMN NAME FOLLOWED BY THE ROW NUMBER

CHEMISTRY Chapter ndash Common Laboratory Apparatus and equipments

Objective type questionFill in the blanks (a) Experiment and observation are the two important basics of chemistry(b) A porcelain dish is used for evaporation(c) A test tube holder is used to hold the test tube while-it is heated(d) Mortar and pestle is used for grinding and crushing solid substances into a powder(e) Glass apparatus is made of Pyrex or borosil glass

Class VIISubject Topic Summary Execution

Hindi 2ndlang

ए था राम( डॉ शरी परसाद)

सगकित ा परभाव मानव 0ीवन पर अवशय पडता ह

हमशा मनषय ो अचछो ी सगकित म रहना चाकिहए

शरषठ परो सग स मनषय चरिरतर ा शीघर ही उदय और किवास हो 0ाता

ह इसलिलए वयलि` ो सदा शरषठ परो ा ही सग रना चाकिहए

इसान अगर चाह वह सवय ो बदल भी सता ह

यह हानी राम ए बचच ी हवह गणिणत ी परीकषा म नल रत हए पडा 0ाता ह और उस अधयाप पडत ह और पछत ह यह कया र रह

हो तभी राम न उनी बइजजती ी

शबदाथब ndashहावा भलावाायवाहीndash ाम किनयम व ानन

ो दिदखानापरिरलिचतndash 0ाना पहचानाघटनाndashघबराहट

उलटा चोर ोतवाल ो डाटndashकिववndash भल बर ा जञानतवयndash म 0ो रना चाकिहएसगकितndash बरी सगत

किबलख नाndashरोना किनशचय रनाndash तय रना

फलndashपरिरणामकिनषालिसतndash बाहर किया हआपशचातापndashदख सपननndashधनी

ldquo हा आपी किहममत स हई नल रत पडन ीrdquo ऐसी बात ही किफर

किपता0ी न भी उस डाटा वह ाफी पशचाताप रन लगा बोला गलत दोसतो

ी सगकित म आ0 कितना अनथ र दिदया किफर उसन अधयाप स माफी मागन ी सची और किफर भी ऐसा

नही रगा यह परण भी लिलया

सोचndashकिहच एात-अला

বইndashবাংলা সাহিতয পহিরচয়

পাঠndash১৬লপndashস-য ও অস-যযলখকndashঈশবরচনদর হিবযাসারঅনশীলনীর পরকে4াততর

৬ অGকেলকেখা -ময়া = পশ হিশকার সহিtহিত = হিনকIবতu সbভরষট = লI হিনরীকষণ =

-াকেলা-াকেব যখাকতাঞজহিলপকেI = যজাাকেত৭ হিবপরীতশবদ -ঈষৎ times পরচর উৎকষট times হিনকষট তাশ times উৎফd তবহিদধ times

বহিদধীNপাহিপষঠ times পণযবান৮ পপহিরবতG ন ককেরা -পশ = পাশহিবক যকাপ = যকাহিপতহিসথর = হিসথরতাএকানত = ঐকাহিনতক পর-াত times পর-াতী

CHEMISTRY

Chapter ndashPhysical and Chemical Changes

Chemical ChangeA chemical change involves a change in chemical composition

Characteristics of Chemical changes 1 They are permanent changes2 They are irreversible changes 3 New substance formed4 A Chemical change involves a

change in its chemical properties

Pg-25Question 8What do you observe when1 water is boiled2 a piece of paper is burnt3 some ice cubes are kept in a glass tumbler4 solid ammonium chloride is heated5 an iron nail is kept in tap water for few days6 a spoon of sugar is heated in a pan7 lighted match stick is brought near the mouth of the test tube containing hydrogen gas8 quick lime is dissolved in water9 little amount of curd is added to a bowl containing warm milk and kept for five hours

10 Water is boiledOn boiling water changes into steam (gas) physical change

11 A piece of paper is burnton burning piece of paper produces carbon dioxide and ash is left behind Is a chemical change

12 some ice cubes are kept in a glass tumblerIce cubes (solid) turn into water

(liquid) only state changes (physical change)

13 Solid ammonium chloride is heatedSolid ammonium chloride on heating changes into vapors (change of state) is physical change

14 An iron nail is kept in tap water for few dayswe observe reddish brown coating on the nail called rust (entirely new substance) is chemical change

15 A spoon of sugar is heated in a panWhen a spoon of sugar is heated in a pan black (charred sugar) (carbon) is seen Is a chemical change

16 Lighted match stick is brought near the mouth of the test tube containing hydrogen gasWe observe that hydrogen bums at the mouth of test tube with blue flame and pop sound is heard It is chemical change

17 Quick lime is dissolved in waterThe following two observations will be observed (i) A hissing sound is observed(ii) The mixture starts boiling and lime water is obtained

18 Little amount of curd is added to a bowl containing warm milk and kept for five hoursWhen a little amount curd is added to a bowl containing warm milk and kept for five hours a permanent change occurredThe milk will change to curd On boiling water changes into steam (gas) physical change

GEOGRAPHY

ATMOSPHERE IMPACT OF GLOBAL WARMING The destructive impart of global warming is observed in various spheres of life and the environment Some of the points are outlined below1 High temperatures lead to high

evaporation rate and drying up of the soil and surface water This affects crop production The occurrence of droughts is aggravating the problem even further

2 The heat waves in summer months

Q1 Write some impact of global warmingA1 The impacts of global warming are as follows1 High temperatures lead to high

evaporate ion rate and drying up of the soil and surface water This affects crop production The occurrence of droughts is aggravating the problem even further

2 The heat waves in summer months lead to a greater number

lead to a greater number of deaths due to heat strokes

3 Forest fires become more frequent4 Tropical cyclones and hurricanes

become common5 Melting of glaciers takes place6 Polar ice caps are becoming thinner

and melting at an alarming rate due to global warming The loss of sea ice

7 Due to increase in sea surface temperature sea levels rise in coastal areas and cause submergence of several islands

WAYS TO REDUCE GLOBAL WARMINGFollowing steps can be taken We need to decrease emission of

green house gases by reducing the burning of fossil fuel such as coal and petroleum

By planting more trees to increase forest cover

The government should also distributes free saplings and organize afforestation programmes to spread awareness regarding the beneficial effects of trees

We should switch to eco-friendly cars and gadgets

Incandescent light bulbs should be replaced by CFL bulbs

We can save electricity and reduce global warming by turning off electrical gadgets such as lights fans air-conditioners television and computer when we do not to use them

Efforts should be made to hasten the development of green cities oreco cities These cities are urban areas around the world striving to lessen the environment a impacts of urbanization

By following the 3Rs-Reduce Recycle and Reuse strategy we can use natural resources for our growth as well as save them for the need of the future generations This is called sustainable development

of deaths due to heat strokes3 Forest fires become more

frequent4 Tropical cyclones and hurricanes

become common5 Melting of glaciers takes place

etc

Q2 How to reduce global warmingA2 Following steps can be taken to reduce global warmingaWe need to decrease emission of

green house gases by reducing the burning of fossil fuel such as coal and petroleum

bBy planting more trees to increase forest cover

c The government should also distributes free saplings and organize afforestation programmes to spread awareness regarding the beneficial effects of trees

dWe should witch to eco-friendly cars and gadgets

eIncandescent light bulbs should be replaced by CFL bulbs

f We can save electricity and reduce global warming by turning off electrical gadgets such as lights fans air-conditioners television and computer when we do not to use them

Q3 What do you mean by 3Rrsquos of resource planningA3 The 3Rs are

1 Reduce 2 Recycle and3 Reuse

Q4 What is Sustainable developmentA4 By following the 3Rs-Reluce Recycle and Reuse strategy we can use natural resources for our growth as well as save them for the need of the future generations This is called sustainable development

English Language

Prepositions A preposition is a word placed before a noun or a pronoun It helps to show how the person or thing denoted by the noun is related to something else in the sentence

Kinds of Prepositions

Simple Prepositions- simple preposition are one word Prepositions such as at by for in of off for from on out through till to up with before amidst towards beyond between over etc

Compound Prepositions ndash There are some words that are always used with fixed Prepositions to convey specific meaning

Example I was unable to meet you dueto a previous engagement ( On account of)Always maintain the queue instead of crowding at the counter ( In place of)

Participial PrepositionsmdashParticiple Prepositions are present or past participles of various verbs which together with a noun phrase or a clause function as prepositions Examples- barring concerning considering notwithstanding pending regarding respecting etc

Exercise A

1 Gauravs fever has come down since Friday He has been absent for a week now

2 The child sat between his father and mother among the parents of all his classmates

3 There are mosquitoes in the room They flew into the room when the door was open

4 My father was inside the drawing room when I was playing outside my house

5 You may sit beside me I will give you a drawing book and pencils besides a storybook

6 We went to the market in the morning and walked towards the riverfront in the evening

7 The child walked along the pavement and across the street safely

8 This table top is made of glass My breakfast fell off it in the morning

9 The pan is on the gas stove There are vegetables in it

10 We will wait for you at the bus top There are a lot of people in the hall

Subject ndash Biology Topic ndash Chapter - 3 Photosynthesis and respiration in plants Summary Execution

All living organism (Plants and animals) need food for energy and growth Green plants (autotrophy) prepare food for all living organisms Today we will discuss about the process photosynthesis And adaptations in a leaf to carry out photosynthesis

Q1What do you mean by photosynthesis and write its word equation The process by which green plants make food (glucose) from carbon dioxide and water

in the presence of sunlight and chlorophyll is called photosynthesis

Carbon dioxide + Water ( Sun light from Sun ) Glucose + Oxygen ( chlorophyll in green leaves )

Q2 What are the adaptations in a leaf to carry out photosynthesisi) Leaves are broad wide and flat for absorbing more light energyii) Presence of chlorophyll in chloroplasts to trap sunlightiii) Presence of stomata which allow carbon dioxide to enter the cell and oxygen to go

out iv) Network of veins ensures continuous supply of water and minerals to the leafv) Thin waxy cuticle protects the leaf without blocking the lightQ3 Draw and label structure of chloroplast

Class VIIISubject Topic Summary Execution

PHYSICS ENERGY Production of Hydro electricity

A hydroelectric dam converts the potential energy stored in a water reservoir behind a dam to mechanical energymdashmechanical energy is also known as kinetic energy As the water flows down through the dam its kinetic energy is used to turn a turbine

The generator converts the turbinersquos mechanical energy into electricity

This electric energy then goes through various transmission processes before it reaches you

Question 2

Fill in the blanks

(a) Work is said to be done by a forte only when the body moves

(b) Work done = Force x distance moved in direction of force

(c) The energy of a body is its capacity to do work

(d) The SI unit of energy is joule

(e) The potential energy is due to its state rest of position and kinetic energy of the body is due to its state of motion

(f) Gravitational potential energy U = mass times force of gravity on unit mass times height

(g) Kinetic energy = frac12 times mass times (speed)2

(h) Power P = work donetime taken

(i) The S I unit of power is watt

(j) IHP = 746 W

BIOLOGY Chapter -5 The endocrine system and adolescence

Today we will discuss about thelocation and functions of secreted hormones of adrenal and Pancreas

Q5 Write location hormone secreted main functions and deficiency diseases of pancreas and adrenal glands

Endocrine Glands

Location Hormones secreted

Functions and Deficiency Diseases

1Adrenal gland

2 Pancreas Gland

On the top of each kidney

In between stomach and small intestine

i)Adrenaline from adrenal medulla

ii)Cortisone from adrenal cortex

i) Insulin

ii) Glucagon

It helps a person deal with any kind of emergency situation or emotional stressIt increases the heart beat rate of respiration and blood pressure

a) It regulates carbohydrates protein and fat metabolism

b) It regulates the salt and water balance in the body

a) It changes excess glucose into glycogen

b) It stimulates the cells to burn extra glucose to provide heat amp energy

Less secretion causes diabetes mellitus

Excessive secretions causeinsulin shock

a) It stimulates the breakdown of glycogen into glucose

b) It increases the level of glucose in blood

History Traders to rulers The Battle of Buxar was fought on 22 October 1764 between the forces under the command of the British East India Company led by Hector Munro and the combined armies of Mir Qasim the Nawab of Bengal till 1763 Mir Jafar was made the Nawab of Bengal for a second time in 1763 by the Company just after the battle After being defeated in 4 battles in katwa and Udaynala the Nawab of Awadh Siraj id Daula and the Mughal emperor Shah Alam II accompanied by Raja Balwant Singh of Kashi made an alliance with Mir Qasim The battle was fought at Buxar a small fortified

Answer the following questions- Short note-Battle of BuxarHomework-learn

town within the territory of Bihar located on the banks of the Ganga river about 130 kilometres (81 mi) west of Patna it was a decisive victory for the British East India Company The war was brought to an end by the Treaty of Allahabad in 1765

EnglishLiterature

The west wind-John Mansfield

In the poem The West Wind by John Masefield the poet starts by describingwith very poetic imagery of birds how the west wind is different from other winds its a warm wind full of birds cries There is a touch of melancholy perhaps home-sickness as he describes how it brings tears too and memories from an old land He goes on to describe the restful pastoral beauty of the land where even the dead can lie in the green He then brings in voicesperhaps of family and friends calling him home as he is missing Aprils beautyThe voices then tempt him some more with idyllic images from home (white blossom young green cornrunning rabbitswarm sun) The voices seem to presume that the poets heart is sorrowful bruised and soreThe end of the poem sees the poet appear to make a decision he will go home as he has decided that is where he truly belongs

Write the synopsis of the following words

1 Daffodils- a tall yellow flower that grows in the spring

2 Orchards- a piece of land on which fruit trees are grown

3 Blossom- a flower or a mass of flowers especially on a fruit tree in spring

4 Thrushes- a bird5 Larks- a small brown bird that

makes a pleasant sound6 Bruised- an injury7 Aching- pain 8 Tread- to put your foot down

while you are walking9 Balm-10 May-11 Fluting-

(Write from the book in your copy)

MAT

HEM

ATIC

S

Ch 1

1Al

gebr

ic E

xpre

ssio

n

1 Constant A symbol which has fixed value is called a constant[eg 8 23 -15 radic3 etc]

2 VariableA symbol which does not have any fixed value but may be assigned value (values) according to the requirement is called variable or literal[eg x y p q etc]

3 TermsA term is a number (constant) a variable a combination (product or quotient) of numbers and variables[eg 7 x 5x etc]

4 Algebric expressionA single term or acombination of two or more terms connected by plus (+) or minus (-) sign forms an algebraic expression[eg 5-y 3x2-5x xy-6z+4 etc]

5 PolynomialAn algebraic expression which contains more than one term is called a polynomial (multinomial)[eg x2-5x 5y+xy+x2y etc]

6 Degree of polynomial(a) When the polynomial contains only one variable the highest power of the variable is the degree of the polynomialeg the degree of the polynomial of 4x-7x5+8 is 5(b) When the polynomial contains two or more variablesStep (i) Find the powers of the variables in each term (ii) The highest sum of the powers is taken to be the degree of the polynomialeg the degree of the polynomial 5x2y-4x3y5+6 is = 3+5 = 8Remember An algebraic expression is a polynomial if degree of each term used in it is a non-negative integer

Exercise ndash 11(A)

1 Separate the constants and variables from the following

-7 7+x 7x+yz radic5 radic xy 3 yz

8 45y -3x

Solution Constant Variables-7 radic5 7+x 7x+yz radic xy

3 yz8

45y -3x

2 Write the number of terms in each of the following polynomials(i) 5x2+3timesax (ii) axdivide4-7 (iii) ax-by+ytimesz (iv) 23+atimesbdivide2

Solution Polynomials Number of terms(i) 5x2+3timesax 2(ii) axdivide4-7 2(iii) ax-by+ytimesz 3(iv) 23+atimesbdivide2 2

4 Write the degree of the each polynomials(i) xy+7z (ii) x2-6x3+8 (iii) y-6y2+5y8 (iv) xyz-3 (vi) x5y7-8x3y8+10x4y4z4

Solution Polynomials Degree(i) xy+7z 2(ii) x2-6x3+8 3(iii) y-6y2+5y8 8(iv) xyz-3 3(vi)x5y7-8x3y8+10x4y4z4 12

5Write the coefficient of(i) ab in 7abx (iv) 8 in a2-8ax+a (v) 4xy in x2-4xy+y2

SolutionCoefficient

(i) ab in 7abx 7x(iv) 8 in a2-8ax+a -ax(v) 4xy in x2-4xy+y2 -1

7 CoefficientAny factor of an algebraic quantity is called the coefficient of the remaining quantityeg in the algebraic term 7xyz 7 is coefficient of xyz 7x is coefficient of yz and so on

8 Like term The terms having the same literal coefficient are called like terms and those having different literal coefficients are called unlike terms

eg (i) 5xyz 8xyz -6xyz and 23xyz are like

terms(ii) 7xy2 8x2yz and -15xyz2 are unlike terms

6 in 57xy2z3 write the coefficient of

(i) 5 (vii) 5xy2 (viii) 17yz (xi) 5xyz

Solution Coefficient

(i) 5 17

xy2z3

(vii) 5xy2 17z3

(viii) 17yz

5xyzsup2

(xi) 5xyz 17yz2

7 In polynomial given below separate the like terms(ii) y2z3 xy2z3 -58x2yz -4y2z3 -8xz3y2 3x2yz and 2z3y2

Solution y2z3 -4y2z3 2z3y2 are like terms

xy2z3 -8xz3y2 are like terms

-58x2yz 3x2yz are like terms

Class IXSubject Topic Summary Execution

Bengali (2nd language)

বাগzwnjধারাzwnj বা ধারা-বা ধারা ল হিবকেশষ পরকার বাক -হিb -াকেবর এক হিবকেশষ পরকাশরীহিত াকেক কতগকেলা কার সমষটির মকেধয এগহিলকেক বা ধারা বকেল আবার কতগকেলা শকেবদর বাধাধরা যকান রীহিত যনই য-াকেব চকেল আসকে যসই -াকেবই চকেল আসকে তখন যসই শবদগহিল খন একক -াকেব অG পরকাশ ককের তখন একের বা ধারা বকেল বা ধারার পরকেয়া -াষাকেক আরও সFর ককের যতাকেল

অকাল পকক(অপহিরনত বয়কেস পাকাহিম)-মাতর শ বর বয়কেস যমকেয়টির া মকেখর কা তাকেত অকালপককতা ধরা পকে

অককা পাওয়া( মারা াওয়া) ndash পকেকIমারটি পকেকIমারকেত হিকেয় বাসাতরীকের াকেত মার যখকেত যখকেত অককা যপল

অহি| পরীকষা ( কঠিন ও পরকত পরীকষা)- যকেলটির আজ ডাকতাহির যরজালট যবকেরাকেব এIাই তার জীবকেনর ব অহি| পরীকষা

অষটরমভা (ফাহিক) ndash রীতা মকেখই বকো বকো কা বকেল আর কাকেজর যবলায় অষটরমভা

অকমGার ধাী (অপাG) ndash সমনকেক হিনকেয় যকান ান কেব না ও একেকবাকেরই অকমGার ধাী

অকেনধর ষটি (অসাকেয়র সায়)- আহিশ বকেরর বকোর নাহিত ল অকেনধর ষটি তাকেক াা বকোর একম চকেল না

আকেককল গড়ম (তবহিদধ)- ার তহিম উপকার করকেল যসই যতামার হিবরকেদধ সাকষয হিকেয়কে শকেনই আমার আকেককল গড়ম

আষাকে লপ( অবাসতব লপ) ndashIাকা এখন যকেব না এIা বলকেলই ত এমন আষাকে লপ ফাার যকান রকার হিল না

Hindi- महायजञ ा इस हानी म लख न या बतान ा परयास किया ह कि किसी भी अचछ

2nd language

परसार(यशपाल ाय या पणय न ा फल अवशय मिमलता ह ोई भी परोपार अथवा पणय लिलए किया गया ाय बार नही 0ाता वह ए परार ा यजञ हए धनी सठ थ धम परायण और किवनमर सठ न आन ी यजञ किए थ और दान म न 0ान कितना धन दिदन दखिखयो म बात दिदया थादिदन पलट और सठ यहा गरीबी आ गई उन दिदनो यजञ बचन ी परथा थी सठ भी अपनी 0गह बचन लिलए डलपर ए सट यहा चलन ो तयार हए सठानी रासत लिलए रोटी पड म बाधर सठ ो द दी रासत म ए भख R ो दखर सठ न चारो रोटी उसो खिखला दी खर वह सठ यहा डलपर पहच तो उनी सठानी न उस महायजञ बचन ो हा यदिद बचन आए सठ न R ो रोटी खिखलान ो महायजञ नही समझा और वापस लौट आया घर आर शाम ो उसी घर म उस ए बडा ख0ाना मिमला 0ो उस दवारा किए गएrsquo महायजञrsquo ा परसार था

English language

Letter formal The heading the name and address of the person you are writing to must be included beneath your own address In formal letters ldquoblock stylerdquo of address is preferred

Subject complain in brief

Salutation If the person you are writing to is known to you you may begin ldquoDear MrrdquoOr ldquoDear Mrsrdquo In all other instances you should begin ldquoDear Sirrdquo or ldquoDear Madamrdquo Or ldquoSirsrdquo

The body A formal or business letter has four partsReference The letter should begin by referring to a letter you have received an advertisement or the reason that has prompted you to writeInformation In the second paragraph it is necessary to supply more detailed information that is related to the referencePurpose Here you must give the reason why you are writing the letter This must be stated clearly and ensure that it is relevant to the question that has been setConclusion round off the letter with some polite remarkThe subscription when a letter has begun with dear sir sirs Madam you should end with Yours faithfully or yours truly When however you address a person by name you must conclude with the words ldquoYours sincerelyrdquo

1 A park in your locality is slowly being used as a rubbish dump Write a letter to the Mayor of your city pointing out the nuisance and danger of this Request that action be taken to stop this immediately

Or2 You being a boarder ordered a set of lab manuals from a famous book shop in the town They sent you a wrong set of books Write a letter to the manager of the book shop

Chemistry Chapter-1 1)CHEMICAL FORMULA- Q What is the Significance of

L-2The Language of Chemistrybull Chemical Formula

Itrsquos a symbolic representation of a chemical substance eg ndash The formula of Sulphuric acid is H2SO4

2) Steps of writing Chemical Formula of a given substance-

1 Write the symbols of the constituent atoms or radicals side by side Keep the basic radical on LHS and acid radical on the RHS ( Na+Cl- )2 In case of a radical having more than one atom( compound radical) enclose the radical in a bracket eg (SO4-)3 Write the valencies of each radical on its right hand top4 If the valencies of the two radicals are divisible by a common factor then divide the valencies by the common factor5 Invert (criss-cross) the valency number ie write the valency of one atom below the second atom and vice versa 6 On interchanging if valency number is lsquoone the figure lsquoonersquo is never writtenFor Example- Compound -Calcium Nitrate1 Writing the symbols- Ca(NO3)2 Writing the valencies on their right hand top- Ca2(NO3)1

3 Valency numeral in simple ratio- Ca2(NO3)1

4 Criss-cross- Ca 2NO3 1

5 Writing the formula of the compound- Ca(NO3)2

Chemical formula

A The formula of a substance conveys the following information regarding a substance 1 The name of the substance (qualitative)2 The elements constituting the substance (qualitative)3 The number of various atoms present in a molecule of the substance (quantitative)4 Molecular weight of the substance and the relative weights of different elements present in it (qualitative)

Q What are the limitations of Chemical Formula

A The chemical formula suffers from the following limitations-I It fails to convey whether the elements in a molecule are present in the form of atoms or ionsFor example the formula KBr fails to tell us whether Potassium and Bromine are present in the form of ions II It does not tell anything about the binding force that holds atom in a molecule togetherIII It does not tell us about the arrangement of various atoms with respect to one another within the molecule

Q Examples of Some Chemicals with their Formula Chemical name and Common Name-

A Given in the class notesCommercial Studies

Joint Stock Company

Let us discuss about the demerits of Joint Stock CompanyDespite so many advantages it has got many disadvantages which are as follows

Difficulty in FormationDelay in Decision makingExcessive Government ControlLack of Secrecy

Company can be classified into several categories based on incorporation

QuestionExplain the demerits of Joint Stock CompanyAnswer) 1 Difficulty in Formation The legal requirements and formalities required to be completed are so many The cost involved is quite heavy It has to approach large number of people for its capital It cannot start its business unless certificate of incorporation has been obtained This is granted after a long time when all the formalities are completed

Chartered CompanyStatutory CompanyRegistered Company

Delay in Decision making In this form of organization decisions are not made by single individual All important decisions are taken by the Board of Directors Decision-making process is time-consuming So many opportunities may be costly because of delay in decision-making Promptness of decisions which is a common feature of sole trader ship and partnership is not found in a company

Excessive Government ControlA company and the management have to function well within the law and the provisions of Companies Act are quite elaborate and complex At every step it is necessary to comply with its provisions lest the company and the management should be penalized The penalties are quite heavy and in several cases officers in default can be punished with imprisonment This hampers the proper functioning of the company

Lack of Secrecy The management of companies remains in the hands of many persons Every important thing is discussed in the meetings of Board of Directors Hence secrets of the business cannot be maintained In case of sole proprietorship and partnership forms of organisation such secrecy is possible because a few persons are involved in the management

2 Define the following

Chartered Company- The crown in exercise of the royal prerogative has power to create a corporation by the grant of a charter to persons assenting to be incorporated Such companies or corporations are known as chartered companies Examples of this type of companies are Bank of England (1694) East India Company (1600) The powers and the nature of business of a chartered company are defined by the charter which incorporates it After the country attained independence these types of companies do not exist

in IndiaStatutory Company- A company may be incorporated by means of a special Act of the Parliament or any state legislature Such companies are called statutory companies Instances of statutory companies in India are Reserve Bank of India the Life Insurance Corporation of India the Food Corporation of India etc The provisions of the Companies Act 1956 apply to statutory companies except where the said provisions are inconsistent with the provisions of the Act creating them Statutory companies are mostly invested with compulsory powersRegistered companiesCompanies registered under the Companies Act 1956 or earlier Companies Acts are called registered companies Such companies come into existence when they are registered under the Companies Act and a certificate of incorporation is granted to them by the Registrar

Economics

Chapter-4Basic problems of Economy

Today let us discuss with the topic Production Possibility curve

QuestionExplain the concept of Production Possibility Curve with the help of diagram

Answer) Production Possibility curve is a locus of all possible combinations of two commodities which can be produced in a country with its given resources and technology

The above diagram shows that with the given resources and technology the economy can produce maximum either 5 thousand meters of cloth or 15 thousand quintals of wheat or any other combination of the two goods like B( 1 thousand meters of cloth and 14 thousand quintals of wheat C ( 2 thousands meters of cloth and 12 thousand quintals of wheat) etcProduction Possibility curve is also called production possibility boundary or frontier as it sets the maximum limit of what it is possible to produce with given resources

Geography

Rotationand Revolution

SUNrsquoS POSITION AND SEASONAL CHANGES EQUINOXES ndash SPRING AND AUTUMN

Q1 What is Spring EquinoxA1 On 21st March sunrays fall directly on the equator On that day

As the Equator divides the Earth into two equal halves the sun rays fall directly on the equator twice in a year Equinoxes means equal Spring EquinoxOn 21st March sunrays fall directly on the equator On that day the duration of day and night both are equal ( 12 hours day and 12 hours night) on every places located on equator This day is called as Spring EquinoxAutumn EquinoxOn 23rd September sunrays fall directly on the equator On that day the duration of day and night both are equal ( 12 hours day and 12 hours night) on every places located on equator This day is called as Autumn Equinox

SOLSTICES ndash SUMMER AND WINTERDue to inclination of the Earth on its axis and the apparent movement of the sun the sun rays fall directly on both tropics once in a year Solstice is a Latin word which mean ldquothe Sun standing stillrdquoSummer SolsticesAfter 21st March there is an apparent movement of the Sun to the north of the equator The apparent northward movement up to 21st June when the Sun appears overhead at the Tropic of Cancer (22frac12degN) The sun appears to stand still at this position and then moves southwards towards the equator This position of the Sun on 21st June is known as Summer Solstices On that day the duration of day and night both are equal ( 12 hours day and 12 hours night) on every places located on Tropic of Cancer (22frac12degN)Winter solstices The apparent southward movement of the Sun continues beyond the equator till 22nd

December On this day the Sun is overhead at the Tropic of Capricorn

the duration of day and night both are equal ( 12 hours day and 12 hours night) on every places located on equator This day is called as Spring Equinox

Q2 What do you mean by EquinoxA2 Equinoxes means equal It is use to explain the equal duration of day and night ( 12 hours day and 12 hours night) on the Earth

Q3 On which date the longest day in Tropic of CancerA3 21st June

Q4 What is the meaning of SolsticeA4 Solstice is a Latin word which mean ldquothe Sun standing stillrdquo

Q5 Which is the longest day in southern hemisphereA5 22nd December

Q6 On what date does the Arctic Circle experience the lsquoMidnight SunrsquoA6 On 21 June the Arctic Circle experiences the lsquoMidnight Sunrsquo

Q7 What is cause of Midnight Sun in NorwayA7 During the summer solstice (21 June) the North Pole is inclined towards the Sun Therefore the duration of sunlight or daytime increases from 12 hours at the Equator to 24 hours at the Arctic Circle and beyond Thatrsquos why The region beyond the Arctic Circle especially Norway is known as the Land of the Midnight Sun because there the Sun does not rise or set on 21 June

Q8 Match the column A with BA B

Summer Solstice 21st March

Autumn Equinox 23rd

September

Winter Solstice 21st June

(22frac12degS) This position of the Sun is referred to as the Winter Solstice because it marks the winter season in the Northern Hemisphere On that day the duration of day and night both are equal ( 12 hours day and 12 hours night) on every places located on Tropic of Capricorn (22frac12degS)SEASONS AND DURATION OF DAY AND NIGHT During the equinoxes all places on the Earth have 12 hours of day and 12 hours of night Due to the revolution of the Earth round the Sun on an inclined axis the duration of day and night varies according to seasons and the latitude of a placeDuring the summer solstice (21 June) the North Pole is inclined towards the Sun Therefore the duration of sunlight or daytime increases from 12 hours at the Equator to 24 hours at the Arctic Circle and beyondThe region beyond the Arctic Circle especially Norway is known as the Land of the Midnight Sun because there the Sun does not rise or set on 21 JuneAt the North Pole there will be six months of daylight The Sun will be seen always above the horizon at a low angle At 66degN 24 hours of sunlight can be seen only on 21 June Hammerfest in northern Norway is a place of tourist attraction for observing the phenomenon of the Midnight Sun This place has continuous daylight from 13 May to 29 July This place is easily accessible to tourists and has hotels and other facilities The view of the midnight Sun from here is enthrallingIn the Southern Hemisphere the duration of daylight decreases from 12 hours at the equator to 0 hours beyond the Antarctic Circle In the South Polar Region there is 24 hours of darkness The Sun is always below the horizon In the Southern Hemisphere which experiences winter the duration of night-time is longer than the duration of daylight

Spring Equinox 22nd

December

A8 A B

Summer Solstice 21st June

Autumn Equinox 23rd

September

Winter Solstice 22nd

December

Spring Equinox 21st March

During winter solstice (22 December) the South Pole is inclined towards the Sun The Southern Hemisphere experiences summer and the Northern Hemisphere has winter Therefore the duration of daylight or sunlight is greater in the Southern Hemisphere than in the Northern HemisphereThe duration of daylight increases from 12 hours at the equator to 24 hours beyond the Antarctic Circle The South Polar Region has 24 hours of sunlight for many days continuously At the South Pole there will be six months of sunlight The Sun will always be seen at a low angle above the horizon In the Northern Hemisphere the duration of daylight will decrease from 12 hours at the equator to 0 hours at the Arctic Circle There are 24 hours of darkness in the North Polar region The duration of night is greater than the duration of daylight as one move northwards from the Equator It is evident from the above table that the duration of daylight is 12 hours throughout the year at the equator only As one moves away from the equator the seasonal variations in the duration of daylight increase The seasonal variations in the duration of daylight are maximum at the Polar Regions

Subject Eng Literature (The Merchant of Venice ndash William Shakespeare)Topic Act II Scene 7 Lines 36 to 80 (End of scene ) [Students should read the original play and also the paraphrase provided]

Summary Questions amp AnswersThe Prince then examines the inscription on the silver casket which says ldquoWho chooseth me shall get as much as he deservesrdquo The Prince says that he deserves Portia more than anybody else because of his high rank his noble birth and his great wealth and power But then he argues that silver is ten times

(1) (Act II Sc 7 L 39-47)

From the four corners of the earth they come

To kiss this shrine this mortal breathing saint

The Hyrcanian deserts and the vasty wildsOf wide Arabia are as through-fares now

inferior to gold and therefore he cannot believe that the portrait of such a beautiful lady as Portia can be contained in the silver casket He decides to see the inscription on the golden casket before making his decision

The Prince goes to examine the inscription on the golden casket which says ldquoWho chooseth me shall get what many men desirerdquo The Prince believes that the whole world desires to possess Portia otherwise so many suitors would not have come from all corners of the world for winning Portia Some of them have come from the distant lands of Persia and Arabia The deserts of Persia (Hyrcanian deserts) and the boundless desolate lands of Arabia have been crossed by the Princes seeking the hand of Portia He contrasts this casket containing Portiarsquos portrait with the old English gold coin bearing the image of the archangel (angel of the highest rank) He goes on to remark that while the figure of the archangel is engraved (Insculped) upon the English coin the picture of Portia who is beautiful as an angel lies hidden inside one of the caskets namely the Golden Casket (Golden Bed)

On the basis of his assessment of the inscription on the golden casket the Prince decides to choose the golden casket He asks for the key and opens the golden casket only to find therein an empty human skull holding a roll of

For princes to come view fair PortiaThe watery kingdom whose ambitious headSpets in the face of heaven is no barTo stop the foreign spirits but they comeAs orsquoer a brook to see fair Portia

(i) Explain the occasion for the above mentioned speech

These are the comments of the Prince of Morocco after he reads the inscription on the golden casket His mental process is revealed to us in these words We find him debating within himself as to which casket he should choose

(ii) What light does the above speech throw on the personality of Prince of Morocco

From the above mentioned speech we come to know that the Prince of Morocco is keen to marry Portia He is the type of person who is easily taken away by outward appearance He is in love with Portia because of her beauty

(iii) What information can you gather about Portia from the above mentioned lines

The given speech shows that Portia is a very beautiful lady She must be possessed of good qualities because many suitors come to her place from all over the world with a desire to get married to her The Prince of Morocco is so impressed by her beauty that he calls her a saint According to him the whole world is desirous of having her

(iv) Elucidate the significance of the first two lines

In these lines the Prince of Morocco pays a compliment to Portia These lines show his admiration for her He says that people come from all parts of the world to see fair Portia

(v) Explain the meaning of the last four lines of the

passage

In these lines the Prince of Morocco says that even the vast oceans which throw a challenge at the sky are unable to prevent men from coming to Portiarsquos place to have a glimpse of her These lines are also a tribute to Portiarsquos beauty and good qualities Many men voyage across the ocean treating it as a mere stream to see the beautiful Portia

paper in which is written that whoever happens to be guided by the glitter of things is invariably deceived

On reading the scroll the Prince says that he is too sad at heart to speak a more formal farewell and leaves with his followers amidst a sound of trumpets

After the Prince of Morocco leaves Portia remarks that the Prince is a gentle fellow but she is rid of him May all persons of his nature make a similar choice

IMPORTANT PASSAGES EXPLAINED

(Act II Sc 7 L 39-43)From the four corners of the earth they come

To kiss this shrine this mortal breathing saintThe Hyrcanian deserts and the vasty wildsOf wide Arabia are as through-fares nowFor princes to come view fair Portia

Context

This passage occurs in Act II Scene 7 in The Merchant of Venice This is part of the speech made by the Prince of Morocco

(2)

(Act II Sc 7 L 48-53)

MOROCCO One of these three contains her heavenly pictureIst like that lead contains her

Twere damnation To think so base a thought it were too grossTo rib her cerecloth in the obscure graveOr shall I think in silver shes immurdBeing ten times undervalued to tried gold

(i) What meaning does the Prince of Morocco find out of the inscription of the golden casket What have Belmont and Portiarsquos house been called and why

The inscription on the golden casket is ldquoWho chooseth me shall gain what many men desirerdquo The Prince finds out that it means that the chooser of the golden casket will get Portia because many men desire her In fact the entire world desires her Because of the coming of many suitors to Belmont from different countries in order to win Portiarsquos hand Belmont has become a centre of pilgrimage and her house is the shrine where saintly Portia is installed

(ii) What does the Prince of Morocco do before making the final choice of the casket Which is the correct casket and who will win Portiarsquos hand

The Prince of Morocco surveys and analyses the inscriptions on the casket of lead silver and gold Before making the final choice like a very systematic and methodical person he once again considers the claims of the caskets The casket containing Portiarsquos picture is the correct casket and the person choosing it will win Portiarsquos hand

Explanation

While praising Portia the Prince of Morocco conceives Portia as a goddess whose image is placed inside one of the caskets Many suitors are coming from far and wide the north and the south the east and the west (Four corners) in order to try their luck Some of them have come from the distant land of Persia and Arabia The deserts of Persia (Hyrcanian deserts) and the boundless desolate lands of Arabia have been crossed by the Princes seeking the hand of Portia All this shows that Portia is indeed the most beautiful lady of the world

(iii) What does the Prince of Morocco say in his estimation while examining the motto on the silver casket What does he find in the golden casket

While examining the motto on the silver casket which says ldquoWho chooseth me shall get as much as he deservesrdquo Morocco says that in his own estimation he surely deserves Portia in all respects ndash rank birth wealth etc

He chooses the golden casket When he opens it he finds an empty human skull holding a scroll in which it is written that those who are attracted by the glittering outside of things are always deceived as Morocco has been deceived

(iv) What kind of nature does the Prince of Morocco have

The Prince of Morocco has a simple nature who does not look deeply into the inner meaning of things but is dazzled by the outward appearance of gold He is inclined to over-estimate his own value and does not realize that it is a duty to ldquogive and hazardrdquo To say that he will not hazard for lead shows that he misreads the true meaning of the inscription which is that he should be prepared to ldquohazard all he hathrdquo for Portia So his feeling is only one of fascination and romantic attraction

(v) Do you think that the lottery of the caskets is not a matter that will be determined by chance

In fact the lottery of the casket is not a matter that will be determined by mere chance but that it is a true test of character and of sincerity which is amply proved not only by Moroccorsquos choice but also by the arguments which he uses to help him in his choice

(Act II Sc 7 L 55-59)

They have in England

A coin that bears the figure of an angelStamped in gold but thats insculpd uponBut here an angel in a golden bedLies all within

Context

(3)

(Act II Sc 7 L 63-77)A carrion Death within whose empty eye

There is a written scroll Ill read the writing

All that glisters is not goldOften have you heard that toldMany a man his life hath soldBut my outside to beholdGilded tombs do worms infoldHad you been as wise as boldYoung in limbs in judgment oldYour answer had not been inscrolld

This passage occurs in Act II Scene 7 in The Merchant of Venice This is part of the speech made by the Prince of Morocco

Explanation

In this passage the Prince of Morocco bestows high praise on Portia whose hand he is seeking He contrasts this casket containing Portiarsquos portrait with the old English gold coin bearing the image of the archangel (angel of the highest rank) He goes on to remark that while the figure of the archangel is engraved (Insculped) upon the English coin the picture of Portia who is beautiful as an angel lies hidden inside one of the caskets namely the Golden Casket (Golden Bed) In the day of Elizabeth silver was ten times inferior in value to gold Therefore the Prince of Morocco believing that Portiarsquos portrait is contained in the Golden Casket decides to choose the Golden Casket

Fare you well your suit is coldCold indeed and labour lostThen farewell heat and welcome frostmdashPortia adieu I have too grievd a heartTo take a tedious leave Thus losers part

(i) What reward does the Prince of Morocco get after making a wrong choice of the Casket How does he feel

After making the wrong choice in selecting the casket of gold the Prince of Morocco as a reward earns a rebuke in the form of a scroll tucked in the empty eye-socket of a skull kept in the casket of gold The Prince is shocked and disappointed He becomes all the more sad and dejected when he reads the scroll which points to his foolishness in being misled by the appearance and outward show as indicative of its worth

(ii) How does the Prince respond after reading the scroll

After reading the scroll the Prince though upset accepts the result with good grace and decorum befitting a royal suitor and true sportsman He says that his love-suit is really cold otherwise he would have chosen correctly but now his efforts have been in vain So he bids farewell to Portia to the warmth and enthusiasm of love and welcomes the cold and bitterness of dejection and misery of life which lies ahead

(iii) What request does he make to Portia and why

After being failure in his mission he requests Portia to give him permission to leave at once because he is too sad to undergo the tediousness of a formal leave-taking He tells that it is the manner in which defeated persons part unceremoniously

(iv) Explain the following lines

ldquoAll that glisters is not goldOften have you heard that toldMany a man his life hath soldBut my outside to beholdGilded tombs do worms infoldrdquo

Mere glitter does not make a metal to be gold Man has often been warned against appearance but it has been of no use Many people have sacrificed their lives only to seek the outer appearance of gold Worms are found inside the gilded

monuments

Class XSubject Topic Summary Execution

Hindi 2ndlang

नया रासता भाग 6 मायाराम 0ी घर म धनी मल 0ी और उनी बटी सरिरता ी ही चचा बनी रहती थी अमिमत ो इसम ोई रलिच ना थी वह धनी घर ी लडी स शादी र सवय ो बचना नही चाहता था उसा भी सवाणिभमान ह ईशवर ी पा

स उस पास पस ी ोई मी नही थी अभी उसन फकटरी ही लगाई थी उसी समझ बाहर था कि उस घर वालो ा झाव पस ी तरफ कयो

ह उसन मा स सवाल किया कि मा तम सरिरता स मरी शादी कयो रना चाहती हो मा न उस समझाया कि वह दखन म बरी नही ह और किफर खानदान अचछा

ह वह ए शल गरहणी रप म घर सभाल सगी अमिमत न मा ो इस बात ा एहसास राया कि मीन सबध लिलए मना रन पर उस दिदल

पर कया बीती होगी मा और अमिमत ी लडी बार म ाफी बात हईमा ा झाव सरिरता ी तरफ था कयोकि वह घर पर अचछा दह0 लर आ रही

थी अमिमत न अपनी मौसी ी बरी हालत बार म बताया कि किस तरह वह बड घर ी खानदानी बटी लाई थी और आ0 उसी हालत कितनी खराब ह लाई थी बहकलब 0ाती ह और बचचो ो भी नही दखती ह बात चल ही रही

थी कि तभी ए ार बाहर आर री धनी मल0ी घर अदर आए और पीछ स डराइवर फल ी ए टोरी लर आया अदर आए और पीछ स

डराइवर ए टोरी फल ी लर आया अमिमत ो फल ी पटी बरी लग रही थी अमिमत न पछ लिलया यह फल कयो ल आए ह प इन सब ी कया

0ररत थी उनो न 0वाब दिदया कि 4 पटी शमीर स मगाए थ अमिमत ो या सनर करोध आ गया तभी उस किपता 0ी आ गए उन आत ही अमिमत उठर बाहर चला गया वहा वहा मा पास आर बठ गया और बोला

अभी रिरशता तय नही हआ और धनी मल 0ी धनी मल 0ी फल ी पटी लर चलआय मा न समझाया कि 0ब सबध 0ड 0ाता ह तो खाली हाथ नही

आत अमिमत न मा स हा कि तम सबन सरिरता ो इस घर म लान ी ठान रखी ह धनीमल 0ी उस दिदन सरिरता ो दखन ी तारीख तय रन आय थ

Commercial Studies

Banking Nowadays Bank provide easy and quick services through internet facilities methods of Banking is called internet bankingIn order to save the time and money involved in visiting Bank branches people increasingly prefer to have internet banking

There are different modes of doing internet banking or transferring money through online They areReal Time Gross Settlement (RTGS)National Electronic Fund Transfers (NEFT)

1

Question

1) Explain the term RTGS Write the features of RTGS

Answer)The acronym RTGS stands for Real Time Gross Settlement which may be defined as the continuous real time settlement of funds transfer individually on and order by order basis without netting lsquoReal timersquo may be defined as the processing of instructions at the time they are received rather than at some letter time lsquoGross settlementrsquo may be defined as the settlement of transfer instructions which occurs

individually

Features of RTGS1It is the continuous settlement of

funds transfer individually on an order by order basis

2RTGS facility is provided only by CBS core banking solution enabled Bank branches

3Amount charged from the customer for RTGS transactions vary from bank to bank

2) Explain the term NEFT Write the features of NEFT

Answer) National electronic funds transfer may be defined as a nationwide system that facilitates individuals Farms and copper operates to electronically transfer funds from any bank branch to any individual farm or corporate having an account with any other bank branch in the country

Features of NEFT2 Transfer can be made 7 times on

weekdays and 6 times on Saturday

3 NEFT cannot be used to receive foreign remittances

4 NEFT transaction takes place in batches

5 A bank branch must be NEFT enabled to become a part of NEFT fund transfer network

6 There is no maximum or minimum amount that can be transferred through NEFT when one bank has a bank account

English Language

CompositionEssay

A composition is an art of creating a piece of writing on any topic or subject It is the writing correctly beautifully and clearly in order to make some interesting reading Structure of the composition

Introduction ( you lay the foundation for your composition)

Body (it constitutes the main part of the essay)

Conclusion (final statement that leaves a lasting impression)

Kinds of essays1 The Narrative essay2 The descriptive essay3 The reflective essay4 The argumentative essay

Write a composition on any one of the following topics (350- 400 words)

1 Friendship Or2 The first day of your school

Subject Eng Literature (The Merchant of Venice ndash William Shakespeare)Topic Act V Scene 1 Lines 127 to 158 (Nerissa helliphellip The clerk will nersquoer wear hair onrsquos face that had it) [Students should read the original play and also the paraphrase given in the school prescribed textbook]

Summary Revision Questions o Soon thereafter Bassanio Gratiano

and Antonio arrive

o Bassanio tells Portia that he is feeling as if it is morning because of the presence of Portia who is shining like the sun When Antonio is introduced by Bassanio to Portia she tells Bassanio that he should be grateful to Antonio who took so much trouble on his account even to the extent of risking his life

o Nerissa starts quarrelling with Gratiano and demands that he show her the ring she had presented to him and which she had warned him not to lose She suspects that Gratiano must have presented the ring to some young woman and not to the lawyerrsquos clerk as he repeatedly says and assures

Answer the following questions to check your preparation of Act IV Scenes 1 and 2

You must attempt only after you have completed your preparation of Act IV The answers must be in complete sentences using textual evidence (with citation) when necessary

[It would be in your own interest to attempt the above questions honestly totally refraining from consulting your textbook or your notes during answering After completion you should correct the paper yourself consulting the textbooknotes etc and award marks as specified Please let me know the marks you scored through WhatsApp in the group or to my personal WhatsApp]

Act IV Scene 1 (each question carries 2 marks)

1 What did the Duke try to do for Antonio

2 Why does Shylock refuse to show mercy How does he justify his stance

3 Why does Antonio say he is ready to die 4 What information is contained in Bellariorsquos letter

5 Why does Portia (as Balthazar) assert that Shylock must show mercy How does he respond

6 What offers are made to Shylock to get him to spare Antonio How are they received

7 What does Antoniorsquos speech as he faces the prospect of Shylockrsquos knife tell you about his character

8 How do Bassanio and Gratiano react to the looming prospect of Antoniorsquos demise

9 How does Portia (as Balthazar) use the law to turn the tables on Shylock

10 What does the Duke decree should happen to Shylock Why What happens to Shylockrsquos estate

11 What does Portia ask Bassanio as payment for her ldquoservicesrdquo What is his initial response What makes him change his mind

Act IV Scene 2 (each question carries 1frac12 marks)

1 What does Gratiano bring to Portia (Balthazar)

2 What does Nerissa plan on getting from Gratiano What does Portiarsquos comment suggest about men

ECO-10 280620 Topic-Supply AnalysisSHIFTING OF SUPPLY

But if there is change in factors other than the price of the commodity then either more is supplied at the same price or less supplied at the same price In such cases the price of the commodity remains constant but there is a change in other factors like change in the price of inputs change in technology of production change in price of other related goods change in taxation policy of the government etc For example there is an improvement in the technology of production of the commodity in question It leads to decrease in per unit of cost production of the commodity The firm is willing to sell more quantity of the commodity at the same price So the supply other commodity increases at the same price This increase in supply is shown by rightward shift of supply curve On the other hand if the firm uses inferior technology of production the cost of production per unit of the commodity increases The firm is willing to sell less quantity at the same price So the supply of the commodity decreases at the same price This decrease in supply is shown by leftward shift of the supply curve The above cases of increase and decrease in supply can be shown with the help of the following figures

Y INCREASE OF SUPPLY Price (Rs) s

P A s1

B

s

X` O s1 X

q q1

Y` Quantity demanded (in units)

Y DECREASE IN SUPPLY s2

s

price (Rs)

C

p A

s2

s

X` o X

q2 q

Y` Quantity demanded ( in units)

Main factors causing increase in supply or rightward shift of supply Curve(i) Fall in the price of other related goods

(ii) Fall in the price of inputsfactors(iii) Use of better technology in production(iv) Decrease in the rate of excise duty by government(v) If the objective of producer changes from profit maximization to salesMaximization

Main factors causing decrease in supply or leftward shift of supply curve(i) Increase in the price of other related goods(ii) Rise in the price of inputsfactors(iii) Use of inferior technology in production(iv) Increase in the rate of excise duty by the government(v) If the objective

Subject - Biology Topic ndash Chapter mdash6 PhotosynthesisSummary Execution

Today we will know about photosynthesis and its stages

Q1 What do you mean by photosynthesis The process by which living plants containing chlorophyll produce food

substances from carbon-di- oxide and water by using light energy Sunlight

6CO2 +12 H2O----------------------- C6 H12O6 + 6H2O + 6O2

Chlorophyll

Q2 What are the importance of photosynthesis I) Food for all Green plants trap solar energy by photosynthesis

process and supply food and energy for all living organisms either directly or indirectly

Ii) Oxygen to breathe in by product of photosynthesis is oxygen which is essential for all living organisms respiration

Q3 Write about two main phases of photosynthesis A Light dependent phase This phase occur in grana of chloroplast I) The chlorophyll on exposure to light energy becomes activated by

absorbing photons Ii) The absorbed energy is used in splitting the water molecules (H2O)

into its two components (H+ and OH- ) and releasing electron s 2H2O------------------------- 4H+ + 4e- +O2

Energy of 4 photons This reaction is known as photolysis

End products are H+ and oxygen water

B Light independent (Dark ) phase The reactions in this phase require no light energy

Here CO2 combine with H+ and produce glucose

Class XI

Subject Topic Summary ExecutionEVS Chapter-4 Legal

regimes for sustainable development

Environmental legislationEnvironmental legislation is the collection of laws and regulations pertaining to air quality water quality the wilderness endangered wildlife and other environmental factors The act ensures that matters important to the environment are thoroughly

Learn -The Forest (Conservation) Act 1980

considered in any decisions made by federal agencies

The Forest (Conservation) Act 1980 The Forest (Conservation) Act 1980 an Act of the Parliament of India to provide for the conservation of forests and for matters connected therewith or ancillary or incidental thereto It was further amended in 1988 This law extends to the whole of IndiaObjects and Reasons of the Forest Conservation Act

Deforestation causes ecological imbalance and leads to environmental deterioration Deforestation had been taking place on a large scale in the country and it had caused widespread concern The act seeks to check upon deforestation and de-reservation of forests

Subject Eng Literature (The Tempest ndash William Shakespeare) Topic Act II Scene 1 Lines 314 to 329 (End of scene)

[Students should read the original play and also the paraphrase given in the school prescribed textbook]Summary Questions amp Answers

Conspiracy of Antonio and Sebastian (Contd)

o As they approach Ariel appears again and wakes up Gonzalo by singing a tune in his ear Alonso also wakes up and they see both Sebastian and Antonio with drawn swords On being caught off guard they make up a story saying that they had heard a bellowing of bulls or lions

o They then moved to another part of the island

o Ariel at once rushes to Prospero to inform him of this development

SUMMING-UP of ACT-2 SCENE-1

(i) Among the survivors Ferdinand is separated from the rest which results in the disconsolate grief of Alonso as he took him for dead

(ii) The villainy of Antonio is confirmed

(iii) The supremacy of Prosperorsquos magic which resulted in the failure of the human conspiracy

(1)

(Act II Sc 1 L 311-325)SEBASTIAN Whiles we stood here securing your repose

Even now we heard a hollow burst of bellowing Like bulls or rather lions Didt not wake youIt struck mine ear most terribly

ALONSO I heard nothingANTONIO O rsquotwas a din to fright a monsters ear

To make an earthquake Sure it was the roarOf a whole herd of lions

ALONSO Heard you this GonzaloGONZALO Upon mine honour sir I heard a humming

And that a strange one too which did awake meI shaked you sir and cried As mine eyes opened I saw their weapons drawn There was a noiseThats verily rsquoTis best we stand upon our guardOr that we quit this place Lets draw our weapons

(i) Why has Prospero sent Ariel to Gonzalo and Alonso What does Ariel do to awaken Gonzalo

Prospero has already come to know by his magic powers the danger which threatens Gonzalo who had been Prosperorsquos friend and so he sent Ariel to preserve the lives of both Gonzalo and Alonso Prospero does not want that his scheme should remain unfulfilled Ariel begins to sing a song in Gonzalorsquos ears to awaken him(ii) Who are ready to carry out their plan Who takes steps to stop them Why does Gonzalo feel surprised after being awakened

Sebastian and Antonio are ready to carry out their plans They are standing with their swords drawn to kill Alonso and

(iv) We see two sets of contrasting characters Gonzalo-Adrian against Antonio-Sebastian

(v) The grief that works in Alonso can be perceived to his repentance for his association in Antoniorsquos crime against Prospero

Gonzalo Ariel takes steps to stop them from carrying out their nefarious scheme When Gonzalo is awakened by the song sung by Ariel into his ears he (Gonzalo) feels surprised because he sees Sebastian and Antonio standing with their swords drawn(iii) What reason do Sebastian and Antonio tell of drawing their swords when they are suspected by Alonso and Gonzalo

When Sebastian and Antonio are seen with their swords drawn they are looked with suspicion by Gonzalo and Alonso At first Sebastian tells them that as they stood here to guard them during their sleep they heard only a little before a sudden loud noise very much like the roaring of bulls or more probably that of lions Then Antonio follows him saying that this was a noise so terrible as to frighten even a monsterrsquos ears and this noise could even have shaken the earth and it was surely like the roaring of a multitude of lions Then seeing the danger they have drawn their swords Perhaps after hearing the terrible noise they (Gonzalo and Alonso) woke up from their sound sleep

(iv) What does Gonzalo tell Alonso about the strange noise What did he see on opening his eyes Gonzalo tells Alonso that he did not hear the sound of roaring but he heard a humming sound which was strange and which woke him up After waking up he gave him (Alonso) a shaking and a loud cry On opening his eyes he saw these two gentlemen standing with their swords drawn(v) What does Gonzalo suggest

Gonzalo suggests that there was a noise indeed and of that he has no doubt at all and suggests that the best course for them would be to remain alert and vigilant against any possible danger to their lives or to leave this place and move to some other part of the island

Class XIISubject Topic Summary Execution

Commerce

Chapter- Management

Today we will discuss about LEVELS OF MANAGEMENT

Levels of management is a series or chain of managerial positions from top to bottom It helps individuals to know their authority responsibilities and superior-subordinate relations among themselves There are mainly three levels of Management TOP LEVEL MANAGEMENTMIDDLE LEVEL MANAGEMENTLOWER LEVEL MANAGEMENT

Top level managementIt consists of members at the highest level in the management hierarchy This level includes Board Of Directors Chief Executive Managing Directors Chairman President Vice President

Rolefunctions of the top levelmanagement1To analyse evaluate and deal

with theexternal environment2 To determine the objectives and

policies of the business3 To strive for welfare and survival

of business

4 To create an organisational Framework consisting of authority responsibility relationship

Middle level management Congress of members or groups who are concerned with implementation of the policies let down by the top managementThis level includes head of the department such as finance manager marketing manager branch and regional managers departmental and divisional heads plant superintendent etc

Role of functions of the middle level management

1 To interpret the policies framed by top management

2 To assign duties and responsibilities to lower level managers

3 To select and appoint employees for middle and supervisory level and evaluate their performance

4 To co-operate with other departments for smooth functioning

Operational or supervisory level managementIt refers to the group are members who are concerned with execution of the work They are also known as fast line managers This level includes supervisor 4 men Section Officer clerk Inspector etc

Role of functions of the lower level management1 To plan and execute day-to-

day operations2 To supervise and control the workers3 To arrange materials and

tools to start the process and make arrangements for training

4 Today present workers grievance and suggestions before the management and

ensure safe and proper working conditions in the factory

Business Studies

Staff Appraisal Chapter- 10 Today let us start with a new chapter

Staff Appraisal

Meaning of Performance Appraisal

Performance Appraisal is the systematic evaluation of the performance of employees and to understand the abilities of a person for further growth and developmentThe supervisors measure the pay of employees and compare it with targets and plansThe supervisor analyses the factors behind work performances of employeesThe employers are in position to guide the employees for a better performance

Objectives of Performance Appraisal

Following are the objectives of Performance Appraisal

To maintain records in order to determine compensation packages wage structure salaries raises etc

To identify the strengths and weaknesses of employees to place right men on right job

To maintain and assess the potential present in a person for further growth and development

To provide a feedback to employees regarding their performance and related status

To provide a feedback to employees regarding their performance and related status

Importance of Performance Appraisal

Performance appraisal provides important and useful information for the assessment of employees skill

knowledge ability and overall job performance The following are the points which indicate the importance of performance appraisal in an organization

1 Performance appraisal helps supervisors to assess the work performance of their subordinates

2 Performance appraisal helps to assess the training and development needs of employees

3 Performance appraisal provides grounds for employees to correct their mistakes and it also provides proper guidance and criticism for employees development4 Performance appraisal provides reward for better performance

5 Performance appraisal helps to improve the communication system of the organization

6 Performance appraisal evaluates whether human resource programs being implemented in the organization have been effective

7 Performance appraisal helps to prepare pay structure for each employee working in the organization

8 Performance appraisal helps to review the potentiality of employees so that their future capability is anticipated

Geography

DRIANAGE The SubarnarekhaThe Subarnarekha and the Brahmaniinterposed between the Ganga and the Mahanadi deltas drain an area of 19300 sq kmand 39033 sq km respectively The drainage basins of these streams are shared byJharkhand Odisha west Bengal and Chhattisgarh The Brahmani is known as southKoel in its upper reaches in Jharkhand

The NarmadaThe Narmada rises in the Amarkantak hills of MadhyaPradesh It flows towards the West in a rift valleyformed due to a geological fault The total length of it is 1300 km All the tributaries of the

Q1 Name the two westward flowing rivers in the peninsular plateauA1 Narmada and Tapi are the only westward flowing rivers of the peninsular plateau

Q2 Differentiate between east-flowing rivers and west-flowing riversA2

East-flowing rivers

West-flowing rivers

Narmada are very short inlength Most of its tributaries join the main streamright anglesThe Narmada basin covers parts of Madhya Pradesh and Gujarat

The Tapi The Tapi rises in the Satpura ranges in the Betul listrictof Madhya Pradesh It flows in a rift valley parallel tothe Narmada but it is much shorter in length It coversparts of Madhya Pradesh Gujarat and MaharashtraThe length is about 724 km

The Sabarmati and the MahiThe Sabarmati rises in the Aravali hills and flows south-south-westwards for a distance of 300 kilometres to the Arabian Sea The Sabarmatibasin extends over an area of 21674 sq km in Rajasthan and Gujarat The Mahi rises inthe east of Udaipur and drains an area of 34842 sq km lying in Madhya PradeshRajasthan and Gujarat It flows south-westwards for a distance of 533 km before it fallsinto the Gulf of Khambhat

The ChambalThe Chambal rises near Mhow in the Vindhya Range and flows towards the northgenerally in a gorge upto Kota Below Kota it turns to the north-east direction and afterreaching Pinahat it turns to the east and runs nearly parallel to the Yamuna beforejoining it in the southern part of the Etawah district in Uttar PradeshMajor Rivers of India with their basin area (Sqkm)

Himalayan System Indus 321290Ganga 861404

Brahmaputra 187110Indus System

Jhelum 34775Beas 20303

Ganga System Yamuna 366223Ghaghra 127950

Peninsular RiversNarmada 98796

Tapi 65145Mahanadi 141600

Subarnarekha 19300Sabarmati 21674

Mahi 34842Godavari 312812

Godavari Krishna Kaveri Mahanadi are the east-flowing rivers

Narmada Tapi west-flowing rivers

They fall into the Bay of Bengal

They fall into Arabian Sea

These rivers form big deltas

These rivers form comparativelysmall deltas

Catchment areas of these rivers are larger

Catchment areas of these rivers are smaller

Krishna 2589488Cauveri 87900

Subject ndashBiology Topic ndashChapter -5 Inheritance amp Variations Summary ExecutionToday we will discussabout linkage and its classification

LINKAGE The tendency of the genes located on the same chromosome to stay together is

hereditary transmission Linked genes the genes responsible for this Genes that exhibit the process of linkage locates in the same chromosome The distance between the linked genes in a chromosome determines the strength

of linkage i e genes that are located close to each other show stronger linkage than that are located far from each other

COMPLETE LINKAGE It is the type of linkage showed by the genes that are closely located or are tightly

linked with each other as they have no chance of separatingby crossing over These genes are always transmitted together to the same gamete and the same

offspring In such condition only parental or non cross over type of gametes are formedINCOMPLETE KINKAGE It is type of linkage showed by the genes that are distantly located orare loosely

linked with each other because they have chance of separating by crossing over

SIGNIFICANCE i) It helps in holding the parental character togetherii) It checks the appearance of new recombination and helps in bringing the

hybrid population which resembles the original parents iii) Linked genes dilute the effects of undesirable traits

Subject Eng Literature (The Tempest ndash William Shakespeare) Topic Essay Questions (EQ-3)Question No 3

Give a character sketch of CalibanAnswer

The character of Caliban has been wonderfully conceived by Shakespeare as the manifestation of all that is gross and earthy ndash a sort of creature of the earth as Ariel is a sort of creature of the air

Calibanrsquos Physical Appearanceo Caliban is lsquofreckledrsquo a lsquomisshapen knaversquo not honoured with human shape

o Prospero calls him lsquothou tortoisersquo (Act I Sc 2 Line 317) Trinculo stumbling upon him describes him as ldquoA strange fish hellip Legged like a man And his fins like armsrdquo He ldquosmells like a fishrdquo (Act II Sc 2 Line 25)

o Prospero also calls him a ldquobeastrdquo (Act IV Sc 1 Line 140) and ldquoThis misshapen knaverdquo (Act V Sc 1 Line 268)

o Further it appears that in addition to his physical deformity his spiritual inferiority is also suggested by Prosperorsquos claim that his birth resulted from the union between his mother the witch Sycorax and the devil

Calibanrsquos ParentageWhen the play opens Caliban is twenty four years of age having been born on the island twelve years before the coming of Prospero His mother was the foul witch Sycorax who was banished from Algiers for ldquomischiefs manifold and sorceries terrible to enter human hearingrdquo (Act I Sc 2 Line 264) and the father was the Devil himself Thus

Caliban is a monster of evil and brute nature ugly deformed and stinking

Calibanrsquos Savage and Malignant Natureo Caliban is entirely a creature of the earth ndash gross brutal and savage He regards himself as the rightful possessor

of the island and Prospero as a usurper

o In his young age he was on good terms with Prospero He had consented to be received by Prospero at his house and to be educated by him He has learnt human language only to curse his master whom he abhors

o His beastly nature soon breaks out and ends in a vicious attack on Miranda This opens the eye of Prospero who becomes severe to him and enforces his service by threats and violence

o Prospero uses him to make dams for fish to fetch firewood scraper trenches wash dishes and keep his cell clean

Calibanrsquos Hatred for ProsperoA profound hatred for Prospero has taken hold of Caliban It springs from a sense of his being dispossessed and ill-treated He would kill Prospero if he could but he knows the power of Prosperorsquos lsquobookrsquo Hence he transfers his allegiance to Stephano who seems like a god to him He also incites the two drunken associates to batter the skull of Prospero when he sleeps in the afternoon

Caliban Shows Considerable Intelligenceo He has learnt Prosperorsquos language

ldquoYou taught me language and my profit onrsquot (Act II Sc 2 Lines 86-89)Is I know how to curserdquo

o He is well aware of the futility of arguing with one who has more power than he has

ldquoI must obey his art is such power (Act I Sc 2 Lines 373-376)It would control my damrsquos god SetebosAnd make a vassal of himrdquo

o He realizes the importance of Prosperorsquos books

ldquoRemember (Act III Sc 2 Lines 89-92)First to possess his books for without themHersquos but a sot as I am nor hath notOne spirit to commandrdquo

o He knows the value of stealth when attacking the enemy

ldquoPray you tread softly that the blind mole may not (Act IV Sc 1 Lines 194-195)Hear a foot fall we now are near his cellrdquo

o Caliban has a better set of values than Stephano and Trinculo They are distracted from their plan by their greed for Prosperorsquos rich garments Only Caliban realizes that such a finery is unimportant

ldquoLeave it alone thou fool it is but trashrdquo (Act IV Sc 1 Lines 224)

Caliban is not a good judge of characterCaliban is not a good judge of character He decides for example that Stephano is a god because he dispenses lsquocelestial liquorrsquo (Act II Sc 2 Line 115) but then it must be remembered that he has only known his mother Sycorax Prospero Miranda and the spirits that torture him However he quickly discovers his error of judgementrdquo

ldquoWhat a thrice-double ass (Act V Sc 1 Lines 295-297)Was I to take this drunkard for a godAnd worship this dull foolrdquo

Calibanrsquos Imaginative NatureIf Caliban is sub-human in what has been said above he is human in the respect of the poetic side of his character He listens to music with rapture He tells of the beautiful dreams in which heaven rains treasures upon him and which upon waking he yearns to renew One of the most poetic passages in whole play is Calibanrsquos description of the island

to Stephano and Trinculo

ldquoBe not afeard The isle is full of noises (Act III Sc 2 Lines 135-143)Sounds and sweet airs that give delight and hurt notSometimes a thousand twangling instrumentsWill hum about mine ears and sometime voicesThat if I then had waked after long sleepWill make me sleep again and then in dreamingThe clouds methought would open and show richesReady to drop upon me that when I wakedI cried to dream againrdquo

Caliban - Less Ignoble Than Some OthersCalibanrsquos motive for murder is less dishonourable than that of Antonio and Sebastian They plan to kill Alonso to gain his power and wealth Caliban merely wants revenge and the return of lsquohisrsquo island

Conclusiono Calibanrsquos character is not portrayed very clearly in the play and hence we cannot decide whether he is a poor

savage being grossly maltreated by Prospero or whether he is evil and must therefore be kept in bondage or enslavement

o Caliban is contrasted with Ariel who is a spirit and thus swift and uninterested in physical activitieso Caliban is also contrasted with Prospero who is the all-powerful master of the island and of the destiny of all

those on the islando Caliban is also contrasted with civilized man showing him to be less evil than Antonio and Stephano and less

materialistic than Stephano and Trinculoo Caliban has suffered at the hands of Prospero and he has learnt to curse by listening to Prosperorsquos abuse He

certainly believes that Prospero has deprived him of his birthrighto Finally the character Caliban is thought to be one of Shakespearersquos masterpieces The complexity of the character

is reflected in the large volume of critical discussion that has grown around it

ECO ndash12 Topic-Forms of market

MonopolyMonopoly is a market structure in which there is a single seller there are no close substitutes for the commodity produced by the firm and there are barriers to entry Example Indian Railways which is operated under government of India Monopoly also implies absence of competitionFeatures of Monopoly Monopoly is characterized by1 Single Seller In monopoly there is only one firm producing the product The whole industry consists of this single firm Thus under monopoly there is no distinction between firm and industry Being the only firm there is significant control of the firm over supply and price Thus under monopoly buyers do not have the option of buying the commodity from any other seller They have to buy the product from the firm or they can go without the commodity This fact gives immense control to the monopolist over the market

2No Close Substitute There are no close substitutes of the product produced by the monopolist firm If there are close substitutes of the product in the market it implies presence of more than one firm and hence no monopoly In order to ensure a total of control over the market by the monopolist firm it is assumed that there are no close substitutes of the product

3 No Entry amp Exit Monopoly can only exist when there is strong barriers before a new firm to enter the market In fact once a monopoly firm starts producing the product no other firm can produce the same One reason for this is the ability of the

monopolist to produce the product at a lower cost than any new firm who thinks to enter the market If a new firm who knows that it cannot produce at a lower cost than the monopolist then that firm will never enter the market for fear of losing out in competition Similarly the monopolist who is operating for a long time may be enjoying reputation among its customers and is in a better position to use the situation in its own benefit A new firm has to take long time to achieve this and so may not be interested to enter the market

4 Price Maker Being the single seller of the product the monopolist has full control over the pricing of the product On the other hand if there is a large number of buyers in the market so no single buyer exercises any significant influence over price determination Thus it is a sellerrsquos market So monopoly firm is a price maker

5 Price Discrimination Having considerable control over the market on account of being single seller with no entry of other firms the monopolist can exercise policy of price discrimination it means that the monopolist can sell different quantities of the same product to a consumer at different price or same quantity to different consumers at different prices by adjudging the standard of living of the consumer

6 Shape of Demand Curve Since a monopolist has full control over the price therefore he can sell more by lowering the price This makes the demand curve downward sloping

Subject Ac-12 290620 Topic- retirement Model sumThe Balance Sheet of Rohit Nisha and Sunil who are partners in a firm sharing profits according to their capitals as on 31st March 2014 was as under

Liabilities Amount Assets Amount (Rs) (` Rs)

Creditors 25000 Machinery 40000Bills Payable 13000 Building 90000General Reserve 22000 Debtors 30000Capital Less Provision for Rohit 60000 Bad debts 1000

29000 Nisha 40000 Stocks 23000 Sunil 40000 140000 Cash at Bank 18000

200000 200000

On the date of Balance Sheet Nisha retired from the firm and following adjustments were made(i) Building is appreciated by 20(ii) Provision for bad debts is increased to 5 on Debtors(iii) Machinery is depreciated by 10(iv) Goodwill of the firm is valued at Rs 56000 and the retiring partnerrsquos share is adjusted

(v) The capital of the new firm is fixed at Rs120000 Prepare Revaluation Account Capital Accounts of the partner and Balance Sheet of the new firm after Nisharsquos retirement Revaluation AccountDr Cr

Particulars Amount Particulars Amount (`Rs) (Rs`)

Provision for Bad debt Ac 500 Building Ac 18000Machinery Ac 4000Profit transferred toCapital Accounts (3 2 2)Rohit 5786Nisha 3857Sunil 3857

13500

18000 18000

Capital Account

Dr Cr

Particulars Rohit Nisha Sunil Particulars Rohit Nisha Sunil (Rs`) (Rs`) (`Rs) (Rs`) (Rs`) (Rs`)

Sunilrsquos Capital ac 9600 mdash 6400 Balance bd 60000 40000 40000Bank - 66143 - General Reserve 9428 6286 6286Balance cd 72000 mdash 48000 Revaluation (Profi 5786 3857 3857 Rohitrsquos Capital Ac mdash 9600 mdash

Sunilrsquos Capital Ac 6400 Bank 6386 - 4257

81600 66143 54400 81600 66143 54400

Balance Sheet as at 31st March 2014

Liabilities Amount Assets Amount (Rs`) (Rs`)

Creditors 25000 Building 108000Bank overdraft 37500 Machinery 36000

Bills Payable 13000 Debtors 30000Capital Less ProvisionRohit 72000 for Bad debts 1500 28500Sunil 48000 120000 Stock 23000

195500 195500

Working Notes (i) (a) Profit sharing ratio is 60000 40000 40000 ie = 3 2 2(b) Gaining Ratio Rohit = 35 ndash 37 = 2135 ndash 1535 = 635Sunil = 25-27 = 1435 ndash 1035 = 435= 635 435= 6 4 = 3 2(c) Nisha Share of Goodwill = Rs 56000 times 27 = Rs16000Share of Goodwill in the gaining ratio by the existing partner ieRohit = Rs16000 times 35 = Rs 9600Sunil = Rs 16000 times 25 = Rs 6400

The journal entry isRohitrsquos Capital Ac Dr 9600Sunilrsquos Capital Ac Dr 6400 To Nisharsquos Capital Ac 16000(Share of Goodwill divided into gaining ratio)

  • 1 Static Friction
  • The frictional force that acts between the surfaces when they are at rest with respect to each other is called Static Friction
    • Static Friction Examples
      • 2 Sliding Friction
        • Examples Of Sliding Friction
          • 3 Rolling Friction
            • Examples Of Rolling Friction
              • Objects and Reasons of the Forest Conservation Act
Page 14:  · Web viewSubject . Topic . Summary . Execution . English 1 . Sounds of animals . Hens –cackle Horses –neigh Lions –roar Owls –hoots Snake –hiss. English 2 . Mother’s

21 213

times2 13 = 4

19

Solution LHS = 213

times2 13 =

73

times 73

= 7times73times3 =

499 = 5

49

there4 LHS ne RHS

So 213

times2 13 = 4

19 [False]

23 23

times 45 =

2times 5+3 times43times 5

Solution

LHS = 23

times 45 =

2times 43 times5 =

815 again

RHS = 2times 5+3 times4

3times 5 = 10+12

15 = 2215

there4 LHS ne RHS So 23

times 45 =

2times 5+3times43times 5

[False]

25 23 of

13 =

29

Solution

LHS= 23 of

13 =

23 times

13 =

29 = RHS]

there4 23 of

13 =

29 [True]

Practice at HomeExercise ndash 31State the following statements are true or false

24 12 of 4 =

18

Class VISubject Topic Summary Execution

HISTORY AND CIVICS

Chapter 5The Mauryan Empire

DECLINE OF MAURYAN EMPIREDecline of Mauryan empire started after the death of Ashoka at around 232 BCThere are several reasons for break up of the empire1 Weak successor Emperors after Ashoka were

capable of handling vast and mighty Mauryan empire In 185BC the last Mauryan ruler Bri-hadrath was murdered by his Commander-in-Chief Pushyamitra Sunga

2 Provincial Revolts Due to weak central author-ity provincial chiefs of Kalinga and southern provinces revolted against emperor and freed themselves from Mauryan empire

3 Weakness of Economy Prosperity of Mauryan was based on solid economic activities which

ExercisesI Multiple choice questions-1 Chandragupta defeated Seleucus in the year ndashc) 305 BC2 Who killed the last Mauryan ruler Brihadrath b) Pushyamitra3 Which of the following was not a reason for the decline of the Mauryan empirec) Chandraguptarsquos weakness4 Ashoka invaded Kalinga in the year c) 261 BC

II Fill in the blanks1Chandragupta ascended the throne in 324

was taken care by early monarchs Later kings had neither ability nor interest in economic af-fairs That led to failure in tax collection As a result they failed to maintain a large army that were essential to keep empire intact

4 Greek Invasion Greeks freed north-western provinces from weak Mauryan monarchs and reestablished their authority

5 Ashokarsquos Policy some scholar opined that after Kalinga war Ashoka embraced Buddhism re-nounced the policy of war and disbanded the Army But this is partially true as there is no proper evidence of disbanding the army

Based on above points we can conclude that main reason for decline of Mauryan empire is weakness of Ashokarsquos successors Kunal Samprati Dasharath Salisuk all were weak kingsAt last in 185 BCPushyamitra Sunga killed king Brihadrath and established the Sunga dynasty

BC2 Bindusara was the son of Chandragupta and father of Ashoka

3 Pataliputra was administered by City Magistrate committess of 5 members each4 The Greek General Seleucus sent his ambassador Megasthenes to Chandraguptarsquos court5 Ashoka sent his son Prince Mahendra and daughter Sanghamitra to spread his Dhamma6 The Indian Rebublic has adopted the Lion Capital of Saranath Pillar as its national emblem 7 Pushyamitra killed the last Mauryan ruler Brihadrath and founded the Sunga dynasty

III Name the following

1The author of Arthashastra-Kautilya2 The ruler who founded the Mauryan dynasty-Chandragupta3 The author of Indika-Megasthenes 4 The officers who were appointed by Ashoka to spread Dhamma-Dhamma Mahamatras5 The general of Alexander whom Chandragupta defeated-Seleucus

V Match the columns1 Kautilya (c)2 Megasthenes (d)3 Pushyamitra (e)4 Brihadrath (b)5 Bindusara (a)

BENGALI(2ND

LANGUAGE)

পশপাহিখর -াষাসহিবনয় রায়কেচৌধরী

যলখক পহিরহিচহিত- পরখযাত সাহিহিতযক উকেপনদরহিককেশার রায়কেচৌধরীর পতর সহিবনয় রায়কেচৌধরী lsquoসকেFশrsquo পহিতরকার সকেb হিতহিন কত হিকেলন তার উকেdখকোয বই lsquoসহিবনয় রায়কেচৌধরীর রচনা সংগরrsquo

পরম হিকেনর পাঠ- lsquoপশপাহিখর হিক -াষাhelliphellip helliphelliphelliphellipপরসপরকেক জানাবার উপায়ও পশপাহিখরা যবশ জাকেনrsquoপরকেমই আমারা জাহিন -াষা হিক -াষা ল আমাকের মকেনর -াব পরকাশ করার জনয আমরা নানান ধরকেনর -হিb বা হিবকেশষ ধরকেনর আওয়াজ মকেখর মাধযকেম কহির অনযকেক যবাঝাকেনার জনয তাকেল এবার আমরা জাহিন পশপাহিখর -াষা হিক পশ পাহিখরা হিক কা বকেল যা পশপাহিখকেরও -াষা আকে তারা তাকের হিনজসব -াষায় কা বকেল মকেনর -াব পরকাশ ককের পশ পাহিখরা মানকেষর হিক হিক -াষা যবাকেঝ হিকনত তারা বলকেত পাকেরনা পরসপরকেক বহিঝকেয় যবার উপায় তারা জাকেননা তকেব তারা হিবকেশষ ককেয়কটি শকেবদর মাধযকেম তাকের মকেনর -াব বহিঝকেয় যয় হিক বহিদধ মান জীব ndashককর হিবাল বন মানষ যঘাা পর-হিত এরা মানকেষর যওয়া নাম শনকেল কান খাা ককের ndash নাম ধকের ডাককেল কাকে আকেস যমন - মরহিরা lsquoহিত ndashহিতrsquo ডাক শকেন আকেস াল lsquoঅ ndashর -র ডাক শকেন কাকে আকেস াহিত মাহকেতর কা শকেন চকেল ককররা মাহিলকেকর হকম পালন ককের সবসময় তাইকেতা ককরকেক পর- -কত পরানী বলা য় ককর আর হিবাল একের আওয়াজ তহিম লকষয করকেল বঝকেব ককররা যরকে যকেল lsquoযঘউ যঘউrsquo করকেত াকেক আবার কাকেল lsquoযকউ যকউrsquo ককের হিবাল সাধারণ lsquoমযাওrsquo বা lsquoহিমউrsquo ককের রা কেল lsquoওয়াওrsquo আওয়াকেজর মাধযকেম মকেনর -াব পরকাশ ককের একেতা যল পশকের কা পাহিখরাও -য় রা পরকাশ করার জনয হিবকেশষ ধরকেনর শবদ ককের হিবপকের সময় পশ পাহিখরা সবার আকে পরসপরকেক জানাবার উপায় তারা জাকেন বহকাল

১) পশপাহিখর -াষা কেলপর যলখক সমপকেকG হিক জাকেনা

উঃ- পরখযাত সাহিহিতযক উকেপনদরহিককেশার রায়কেচৌধরীর পতর সহিবনয় রায়কেচৌধরী lsquoসকেFশrsquo পহিতরকার সকেb হিতহিন কত হিকেলন তার উকেdখকোয বই lsquoসহিবনয় রায়কেচৌধরীর রচনা সংগরrsquo হিতহিন ারকেমাহিনয়াম এসরাজ পর-হিত বাযনতর বাজাকেত পারকেতন ানও জানকেতন হিতহিন যাকেIাকের জনয মজাার লপ কহিবতা হিলখকেতন

২) পশপাহিখ কেলপর মল-াব হিকউঃ- পশপাহিখকেরও -াষা আকে তারা তাকের হিনজসব -াষায় কা বকেল মকেনর -াব পরকাশ ককের পশ পাহিখরা মানকেষর হিক হিক -াষা যবাকেঝ হিকনত তারা বলকেত পাকেরনা পরসপরকেক বহিঝকেয় যবার উপায় তারা জাকেননা তকেব তারা হিবকেশষ ককেয়কটি শকেবদর মাধযকেম তাকের মকেনর -াব বহিঝকেয় যয় হিরউকেবন কযাসটং সাকেব হিতহিন চহিdশ বর বনযজনত যর সকেb যকেককেন হিতহিন বকেলকেন আমরা হি তাকের -াষা তাকের আব কায়া যমকেন চহিল তাকেল আর -কেয়র যকান কারণ াকেকনা আমরা একI -াকেলাকেবকেস যচষটা করকেল পশপাহিখকের সকেb -াব পাতাকেত পাহির

ধকের মানষ এই পশ পাহিখর -াষা হিনকেয় নানা রককেমর পরীকষা ককের আসকে এইরকম একজন হিরউকেবন কযাসটাং সাকেকেবর কা আমরা জানকেবাhelliphellip

Hindi 2nd

langमतर किनमनलिलखिखतपरशनोउRरदीजि0ए

) बढ वयलि` बचच ो कया हआ था ख) डॉकटर साहबन पाटc किस उददशय स रखी थी ग) ाल साप ो हाथ म लर लाश न कया किया घ) डॉकटर चडढा न बढ पतरो दखन स कयो मना र दिदया था ङ) भगत न लाश ो दखर कया हा

उRर ndash) उस बहत बखार थी और 4 दिदनो स आख भी नही खोला थाख) उन बट ी सालकिगरह थीग) ाल सापो हाथ म लर लाश न उसी गदन 0ोर स दबार पडी थीघ) डॉकटर चडढा न बढ वयलि` पतरो दखन स मना र दिदया कयोकि उनह गोलफ खलन 0ाना थाङ) लाश ो दखर हा कि नारायण चाहग तो आध घट म भया उठ 0ाएग

English literature

In the bazaars of Hyderabad- Sarojini Naidu

Through the poem In The Bazaars of Hyderabad Sarojini wanted to convey the message that India is rich in tradition and they donrsquot need the foreign products So she goes on to give a picture of a bazaar where traditional Indian products are rulingThe poem is in the form of questions and answers The poet asks the questions and the merchants answer them Through this technique she make the picture of the bazaar visible to us

Read the poem

PHYSICS FORCE Types of FrictionThere are three types of friction static sliding rolling Static sliding and rolling friction occur between solid surfaces

1 Static Friction The frictional force that acts between the surfaces when they are at rest with respect to each other is called Static FrictionStatic Friction Examples

Skiing against the snow Creating heat by rubbing both the hands

together Table lamp resting on the table

2 Sliding Friction The resistance that is created between any two objects when they are sliding against each other is called Sliding FrictionExamples Of Sliding Friction

Sliding of the block across the floor Two cards sliding against each other in a

deck

3 Rolling Friction The force which resists the motion of a ball or wheel is called Rolling Friction Is the weakest types of frictionExamples Of Rolling Friction

Rolling of the log on the ground Wheels of the moving vehicles

6What effect can a force produce on a body which is not allowed to move Ans - When a force is applied on a body which is not free to move it gets deformed i e the shape or size of the body changes7Give one example each to indicate that the application of a force

1 produces motion2 stops motion3 slows down motion4 changes the direction of motion5 deforms a body

Ans- 1 A car originally at rest when pushed

begins to move2 A moving bicycle is stopped by

applying the brakes3 The speed of a moving vehicle is

slowed down by applying the brakes4 A player kicks a moving football to

change its direction of motion5 On stretching a rubber string its

length increases

8State the effect produced by a force in the following cases (a) The sling of a rubber catapult is stretched(b) A man pushes a heavy cart(c) A player uses his stick to deflect the ball (d) A cyclist applies brakes(e) A spring is compressedAns- (a) The shape and size of catapult changes ie its length increases(b) The heavy cart begins to move(c) The direction of the ball changes(d) The speed of the moving cycle is slowed down(e) There is change in size and shape of spring

COMPUTER MS EXCEL 2013 -INTRODUCTION

UNDERSTANDING EXCEL STRUCTUREA SPREADSHEET IS A FILE THAT EXISTS OF CELLS IN ROWS AND COLUMNS AND CAN HELP ARRANGE CALCULATE AND SORT DATA DATA IN A SPREADSHEET CAN BE NUMERIC VALUES AS WELL AS TEXT

FORMULAS REFERENCES AND FUNCTIONS

WORKSHEETA WORKSHEET IS ALSO KNOWN AS SPREADSHEETIT IS A COLLECTION OF CELLS ON A SINGLE SHEET WHERE YOU KEEP AND CHANGE DATA

WORKBOOKWORKBOOK IS PMS EXCEL FILE IN WHICH THE DATA CAN BE STORED EACH WORKBOOK CAN CONTAIN MANY WORKSHEETS

ROWS AND COLUMNSIN MS EXCEL A ROW IS A GROUP OF CELLS THAT RUN FROM LEFT TO RIGHT OF A PAGEA COLUMN IS A GROUPING OF CELLS THAT RUN FROM THE TOP TO THE BOTTOM OF A PAGE

CELLTHE INTERSECTION POINT BETWEEN A ROW AND THE COLUMN IS CALLED A CELL WHICH IS THE BASIC STORAGE UNIT FOR DATA IN A SPREADSHEET EACH CELL HAS SPECIFIC ADDRESS WHICH IS THE COMBINATION OF THE COLUMN NAME FOLLOWED BY THE ROW NUMBER

CHEMISTRY Chapter ndash Common Laboratory Apparatus and equipments

Objective type questionFill in the blanks (a) Experiment and observation are the two important basics of chemistry(b) A porcelain dish is used for evaporation(c) A test tube holder is used to hold the test tube while-it is heated(d) Mortar and pestle is used for grinding and crushing solid substances into a powder(e) Glass apparatus is made of Pyrex or borosil glass

Class VIISubject Topic Summary Execution

Hindi 2ndlang

ए था राम( डॉ शरी परसाद)

सगकित ा परभाव मानव 0ीवन पर अवशय पडता ह

हमशा मनषय ो अचछो ी सगकित म रहना चाकिहए

शरषठ परो सग स मनषय चरिरतर ा शीघर ही उदय और किवास हो 0ाता

ह इसलिलए वयलि` ो सदा शरषठ परो ा ही सग रना चाकिहए

इसान अगर चाह वह सवय ो बदल भी सता ह

यह हानी राम ए बचच ी हवह गणिणत ी परीकषा म नल रत हए पडा 0ाता ह और उस अधयाप पडत ह और पछत ह यह कया र रह

हो तभी राम न उनी बइजजती ी

शबदाथब ndashहावा भलावाायवाहीndash ाम किनयम व ानन

ो दिदखानापरिरलिचतndash 0ाना पहचानाघटनाndashघबराहट

उलटा चोर ोतवाल ो डाटndashकिववndash भल बर ा जञानतवयndash म 0ो रना चाकिहएसगकितndash बरी सगत

किबलख नाndashरोना किनशचय रनाndash तय रना

फलndashपरिरणामकिनषालिसतndash बाहर किया हआपशचातापndashदख सपननndashधनी

ldquo हा आपी किहममत स हई नल रत पडन ीrdquo ऐसी बात ही किफर

किपता0ी न भी उस डाटा वह ाफी पशचाताप रन लगा बोला गलत दोसतो

ी सगकित म आ0 कितना अनथ र दिदया किफर उसन अधयाप स माफी मागन ी सची और किफर भी ऐसा

नही रगा यह परण भी लिलया

सोचndashकिहच एात-अला

বইndashবাংলা সাহিতয পহিরচয়

পাঠndash১৬লপndashস-য ও অস-যযলখকndashঈশবরচনদর হিবযাসারঅনশীলনীর পরকে4াততর

৬ অGকেলকেখা -ময়া = পশ হিশকার সহিtহিত = হিনকIবতu সbভরষট = লI হিনরীকষণ =

-াকেলা-াকেব যখাকতাঞজহিলপকেI = যজাাকেত৭ হিবপরীতশবদ -ঈষৎ times পরচর উৎকষট times হিনকষট তাশ times উৎফd তবহিদধ times

বহিদধীNপাহিপষঠ times পণযবান৮ পপহিরবতG ন ককেরা -পশ = পাশহিবক যকাপ = যকাহিপতহিসথর = হিসথরতাএকানত = ঐকাহিনতক পর-াত times পর-াতী

CHEMISTRY

Chapter ndashPhysical and Chemical Changes

Chemical ChangeA chemical change involves a change in chemical composition

Characteristics of Chemical changes 1 They are permanent changes2 They are irreversible changes 3 New substance formed4 A Chemical change involves a

change in its chemical properties

Pg-25Question 8What do you observe when1 water is boiled2 a piece of paper is burnt3 some ice cubes are kept in a glass tumbler4 solid ammonium chloride is heated5 an iron nail is kept in tap water for few days6 a spoon of sugar is heated in a pan7 lighted match stick is brought near the mouth of the test tube containing hydrogen gas8 quick lime is dissolved in water9 little amount of curd is added to a bowl containing warm milk and kept for five hours

10 Water is boiledOn boiling water changes into steam (gas) physical change

11 A piece of paper is burnton burning piece of paper produces carbon dioxide and ash is left behind Is a chemical change

12 some ice cubes are kept in a glass tumblerIce cubes (solid) turn into water

(liquid) only state changes (physical change)

13 Solid ammonium chloride is heatedSolid ammonium chloride on heating changes into vapors (change of state) is physical change

14 An iron nail is kept in tap water for few dayswe observe reddish brown coating on the nail called rust (entirely new substance) is chemical change

15 A spoon of sugar is heated in a panWhen a spoon of sugar is heated in a pan black (charred sugar) (carbon) is seen Is a chemical change

16 Lighted match stick is brought near the mouth of the test tube containing hydrogen gasWe observe that hydrogen bums at the mouth of test tube with blue flame and pop sound is heard It is chemical change

17 Quick lime is dissolved in waterThe following two observations will be observed (i) A hissing sound is observed(ii) The mixture starts boiling and lime water is obtained

18 Little amount of curd is added to a bowl containing warm milk and kept for five hoursWhen a little amount curd is added to a bowl containing warm milk and kept for five hours a permanent change occurredThe milk will change to curd On boiling water changes into steam (gas) physical change

GEOGRAPHY

ATMOSPHERE IMPACT OF GLOBAL WARMING The destructive impart of global warming is observed in various spheres of life and the environment Some of the points are outlined below1 High temperatures lead to high

evaporation rate and drying up of the soil and surface water This affects crop production The occurrence of droughts is aggravating the problem even further

2 The heat waves in summer months

Q1 Write some impact of global warmingA1 The impacts of global warming are as follows1 High temperatures lead to high

evaporate ion rate and drying up of the soil and surface water This affects crop production The occurrence of droughts is aggravating the problem even further

2 The heat waves in summer months lead to a greater number

lead to a greater number of deaths due to heat strokes

3 Forest fires become more frequent4 Tropical cyclones and hurricanes

become common5 Melting of glaciers takes place6 Polar ice caps are becoming thinner

and melting at an alarming rate due to global warming The loss of sea ice

7 Due to increase in sea surface temperature sea levels rise in coastal areas and cause submergence of several islands

WAYS TO REDUCE GLOBAL WARMINGFollowing steps can be taken We need to decrease emission of

green house gases by reducing the burning of fossil fuel such as coal and petroleum

By planting more trees to increase forest cover

The government should also distributes free saplings and organize afforestation programmes to spread awareness regarding the beneficial effects of trees

We should switch to eco-friendly cars and gadgets

Incandescent light bulbs should be replaced by CFL bulbs

We can save electricity and reduce global warming by turning off electrical gadgets such as lights fans air-conditioners television and computer when we do not to use them

Efforts should be made to hasten the development of green cities oreco cities These cities are urban areas around the world striving to lessen the environment a impacts of urbanization

By following the 3Rs-Reduce Recycle and Reuse strategy we can use natural resources for our growth as well as save them for the need of the future generations This is called sustainable development

of deaths due to heat strokes3 Forest fires become more

frequent4 Tropical cyclones and hurricanes

become common5 Melting of glaciers takes place

etc

Q2 How to reduce global warmingA2 Following steps can be taken to reduce global warmingaWe need to decrease emission of

green house gases by reducing the burning of fossil fuel such as coal and petroleum

bBy planting more trees to increase forest cover

c The government should also distributes free saplings and organize afforestation programmes to spread awareness regarding the beneficial effects of trees

dWe should witch to eco-friendly cars and gadgets

eIncandescent light bulbs should be replaced by CFL bulbs

f We can save electricity and reduce global warming by turning off electrical gadgets such as lights fans air-conditioners television and computer when we do not to use them

Q3 What do you mean by 3Rrsquos of resource planningA3 The 3Rs are

1 Reduce 2 Recycle and3 Reuse

Q4 What is Sustainable developmentA4 By following the 3Rs-Reluce Recycle and Reuse strategy we can use natural resources for our growth as well as save them for the need of the future generations This is called sustainable development

English Language

Prepositions A preposition is a word placed before a noun or a pronoun It helps to show how the person or thing denoted by the noun is related to something else in the sentence

Kinds of Prepositions

Simple Prepositions- simple preposition are one word Prepositions such as at by for in of off for from on out through till to up with before amidst towards beyond between over etc

Compound Prepositions ndash There are some words that are always used with fixed Prepositions to convey specific meaning

Example I was unable to meet you dueto a previous engagement ( On account of)Always maintain the queue instead of crowding at the counter ( In place of)

Participial PrepositionsmdashParticiple Prepositions are present or past participles of various verbs which together with a noun phrase or a clause function as prepositions Examples- barring concerning considering notwithstanding pending regarding respecting etc

Exercise A

1 Gauravs fever has come down since Friday He has been absent for a week now

2 The child sat between his father and mother among the parents of all his classmates

3 There are mosquitoes in the room They flew into the room when the door was open

4 My father was inside the drawing room when I was playing outside my house

5 You may sit beside me I will give you a drawing book and pencils besides a storybook

6 We went to the market in the morning and walked towards the riverfront in the evening

7 The child walked along the pavement and across the street safely

8 This table top is made of glass My breakfast fell off it in the morning

9 The pan is on the gas stove There are vegetables in it

10 We will wait for you at the bus top There are a lot of people in the hall

Subject ndash Biology Topic ndash Chapter - 3 Photosynthesis and respiration in plants Summary Execution

All living organism (Plants and animals) need food for energy and growth Green plants (autotrophy) prepare food for all living organisms Today we will discuss about the process photosynthesis And adaptations in a leaf to carry out photosynthesis

Q1What do you mean by photosynthesis and write its word equation The process by which green plants make food (glucose) from carbon dioxide and water

in the presence of sunlight and chlorophyll is called photosynthesis

Carbon dioxide + Water ( Sun light from Sun ) Glucose + Oxygen ( chlorophyll in green leaves )

Q2 What are the adaptations in a leaf to carry out photosynthesisi) Leaves are broad wide and flat for absorbing more light energyii) Presence of chlorophyll in chloroplasts to trap sunlightiii) Presence of stomata which allow carbon dioxide to enter the cell and oxygen to go

out iv) Network of veins ensures continuous supply of water and minerals to the leafv) Thin waxy cuticle protects the leaf without blocking the lightQ3 Draw and label structure of chloroplast

Class VIIISubject Topic Summary Execution

PHYSICS ENERGY Production of Hydro electricity

A hydroelectric dam converts the potential energy stored in a water reservoir behind a dam to mechanical energymdashmechanical energy is also known as kinetic energy As the water flows down through the dam its kinetic energy is used to turn a turbine

The generator converts the turbinersquos mechanical energy into electricity

This electric energy then goes through various transmission processes before it reaches you

Question 2

Fill in the blanks

(a) Work is said to be done by a forte only when the body moves

(b) Work done = Force x distance moved in direction of force

(c) The energy of a body is its capacity to do work

(d) The SI unit of energy is joule

(e) The potential energy is due to its state rest of position and kinetic energy of the body is due to its state of motion

(f) Gravitational potential energy U = mass times force of gravity on unit mass times height

(g) Kinetic energy = frac12 times mass times (speed)2

(h) Power P = work donetime taken

(i) The S I unit of power is watt

(j) IHP = 746 W

BIOLOGY Chapter -5 The endocrine system and adolescence

Today we will discuss about thelocation and functions of secreted hormones of adrenal and Pancreas

Q5 Write location hormone secreted main functions and deficiency diseases of pancreas and adrenal glands

Endocrine Glands

Location Hormones secreted

Functions and Deficiency Diseases

1Adrenal gland

2 Pancreas Gland

On the top of each kidney

In between stomach and small intestine

i)Adrenaline from adrenal medulla

ii)Cortisone from adrenal cortex

i) Insulin

ii) Glucagon

It helps a person deal with any kind of emergency situation or emotional stressIt increases the heart beat rate of respiration and blood pressure

a) It regulates carbohydrates protein and fat metabolism

b) It regulates the salt and water balance in the body

a) It changes excess glucose into glycogen

b) It stimulates the cells to burn extra glucose to provide heat amp energy

Less secretion causes diabetes mellitus

Excessive secretions causeinsulin shock

a) It stimulates the breakdown of glycogen into glucose

b) It increases the level of glucose in blood

History Traders to rulers The Battle of Buxar was fought on 22 October 1764 between the forces under the command of the British East India Company led by Hector Munro and the combined armies of Mir Qasim the Nawab of Bengal till 1763 Mir Jafar was made the Nawab of Bengal for a second time in 1763 by the Company just after the battle After being defeated in 4 battles in katwa and Udaynala the Nawab of Awadh Siraj id Daula and the Mughal emperor Shah Alam II accompanied by Raja Balwant Singh of Kashi made an alliance with Mir Qasim The battle was fought at Buxar a small fortified

Answer the following questions- Short note-Battle of BuxarHomework-learn

town within the territory of Bihar located on the banks of the Ganga river about 130 kilometres (81 mi) west of Patna it was a decisive victory for the British East India Company The war was brought to an end by the Treaty of Allahabad in 1765

EnglishLiterature

The west wind-John Mansfield

In the poem The West Wind by John Masefield the poet starts by describingwith very poetic imagery of birds how the west wind is different from other winds its a warm wind full of birds cries There is a touch of melancholy perhaps home-sickness as he describes how it brings tears too and memories from an old land He goes on to describe the restful pastoral beauty of the land where even the dead can lie in the green He then brings in voicesperhaps of family and friends calling him home as he is missing Aprils beautyThe voices then tempt him some more with idyllic images from home (white blossom young green cornrunning rabbitswarm sun) The voices seem to presume that the poets heart is sorrowful bruised and soreThe end of the poem sees the poet appear to make a decision he will go home as he has decided that is where he truly belongs

Write the synopsis of the following words

1 Daffodils- a tall yellow flower that grows in the spring

2 Orchards- a piece of land on which fruit trees are grown

3 Blossom- a flower or a mass of flowers especially on a fruit tree in spring

4 Thrushes- a bird5 Larks- a small brown bird that

makes a pleasant sound6 Bruised- an injury7 Aching- pain 8 Tread- to put your foot down

while you are walking9 Balm-10 May-11 Fluting-

(Write from the book in your copy)

MAT

HEM

ATIC

S

Ch 1

1Al

gebr

ic E

xpre

ssio

n

1 Constant A symbol which has fixed value is called a constant[eg 8 23 -15 radic3 etc]

2 VariableA symbol which does not have any fixed value but may be assigned value (values) according to the requirement is called variable or literal[eg x y p q etc]

3 TermsA term is a number (constant) a variable a combination (product or quotient) of numbers and variables[eg 7 x 5x etc]

4 Algebric expressionA single term or acombination of two or more terms connected by plus (+) or minus (-) sign forms an algebraic expression[eg 5-y 3x2-5x xy-6z+4 etc]

5 PolynomialAn algebraic expression which contains more than one term is called a polynomial (multinomial)[eg x2-5x 5y+xy+x2y etc]

6 Degree of polynomial(a) When the polynomial contains only one variable the highest power of the variable is the degree of the polynomialeg the degree of the polynomial of 4x-7x5+8 is 5(b) When the polynomial contains two or more variablesStep (i) Find the powers of the variables in each term (ii) The highest sum of the powers is taken to be the degree of the polynomialeg the degree of the polynomial 5x2y-4x3y5+6 is = 3+5 = 8Remember An algebraic expression is a polynomial if degree of each term used in it is a non-negative integer

Exercise ndash 11(A)

1 Separate the constants and variables from the following

-7 7+x 7x+yz radic5 radic xy 3 yz

8 45y -3x

Solution Constant Variables-7 radic5 7+x 7x+yz radic xy

3 yz8

45y -3x

2 Write the number of terms in each of the following polynomials(i) 5x2+3timesax (ii) axdivide4-7 (iii) ax-by+ytimesz (iv) 23+atimesbdivide2

Solution Polynomials Number of terms(i) 5x2+3timesax 2(ii) axdivide4-7 2(iii) ax-by+ytimesz 3(iv) 23+atimesbdivide2 2

4 Write the degree of the each polynomials(i) xy+7z (ii) x2-6x3+8 (iii) y-6y2+5y8 (iv) xyz-3 (vi) x5y7-8x3y8+10x4y4z4

Solution Polynomials Degree(i) xy+7z 2(ii) x2-6x3+8 3(iii) y-6y2+5y8 8(iv) xyz-3 3(vi)x5y7-8x3y8+10x4y4z4 12

5Write the coefficient of(i) ab in 7abx (iv) 8 in a2-8ax+a (v) 4xy in x2-4xy+y2

SolutionCoefficient

(i) ab in 7abx 7x(iv) 8 in a2-8ax+a -ax(v) 4xy in x2-4xy+y2 -1

7 CoefficientAny factor of an algebraic quantity is called the coefficient of the remaining quantityeg in the algebraic term 7xyz 7 is coefficient of xyz 7x is coefficient of yz and so on

8 Like term The terms having the same literal coefficient are called like terms and those having different literal coefficients are called unlike terms

eg (i) 5xyz 8xyz -6xyz and 23xyz are like

terms(ii) 7xy2 8x2yz and -15xyz2 are unlike terms

6 in 57xy2z3 write the coefficient of

(i) 5 (vii) 5xy2 (viii) 17yz (xi) 5xyz

Solution Coefficient

(i) 5 17

xy2z3

(vii) 5xy2 17z3

(viii) 17yz

5xyzsup2

(xi) 5xyz 17yz2

7 In polynomial given below separate the like terms(ii) y2z3 xy2z3 -58x2yz -4y2z3 -8xz3y2 3x2yz and 2z3y2

Solution y2z3 -4y2z3 2z3y2 are like terms

xy2z3 -8xz3y2 are like terms

-58x2yz 3x2yz are like terms

Class IXSubject Topic Summary Execution

Bengali (2nd language)

বাগzwnjধারাzwnj বা ধারা-বা ধারা ল হিবকেশষ পরকার বাক -হিb -াকেবর এক হিবকেশষ পরকাশরীহিত াকেক কতগকেলা কার সমষটির মকেধয এগহিলকেক বা ধারা বকেল আবার কতগকেলা শকেবদর বাধাধরা যকান রীহিত যনই য-াকেব চকেল আসকে যসই -াকেবই চকেল আসকে তখন যসই শবদগহিল খন একক -াকেব অG পরকাশ ককের তখন একের বা ধারা বকেল বা ধারার পরকেয়া -াষাকেক আরও সFর ককের যতাকেল

অকাল পকক(অপহিরনত বয়কেস পাকাহিম)-মাতর শ বর বয়কেস যমকেয়টির া মকেখর কা তাকেত অকালপককতা ধরা পকে

অককা পাওয়া( মারা াওয়া) ndash পকেকIমারটি পকেকIমারকেত হিকেয় বাসাতরীকের াকেত মার যখকেত যখকেত অককা যপল

অহি| পরীকষা ( কঠিন ও পরকত পরীকষা)- যকেলটির আজ ডাকতাহির যরজালট যবকেরাকেব এIাই তার জীবকেনর ব অহি| পরীকষা

অষটরমভা (ফাহিক) ndash রীতা মকেখই বকো বকো কা বকেল আর কাকেজর যবলায় অষটরমভা

অকমGার ধাী (অপাG) ndash সমনকেক হিনকেয় যকান ান কেব না ও একেকবাকেরই অকমGার ধাী

অকেনধর ষটি (অসাকেয়র সায়)- আহিশ বকেরর বকোর নাহিত ল অকেনধর ষটি তাকেক াা বকোর একম চকেল না

আকেককল গড়ম (তবহিদধ)- ার তহিম উপকার করকেল যসই যতামার হিবরকেদধ সাকষয হিকেয়কে শকেনই আমার আকেককল গড়ম

আষাকে লপ( অবাসতব লপ) ndashIাকা এখন যকেব না এIা বলকেলই ত এমন আষাকে লপ ফাার যকান রকার হিল না

Hindi- महायजञ ा इस हानी म लख न या बतान ा परयास किया ह कि किसी भी अचछ

2nd language

परसार(यशपाल ाय या पणय न ा फल अवशय मिमलता ह ोई भी परोपार अथवा पणय लिलए किया गया ाय बार नही 0ाता वह ए परार ा यजञ हए धनी सठ थ धम परायण और किवनमर सठ न आन ी यजञ किए थ और दान म न 0ान कितना धन दिदन दखिखयो म बात दिदया थादिदन पलट और सठ यहा गरीबी आ गई उन दिदनो यजञ बचन ी परथा थी सठ भी अपनी 0गह बचन लिलए डलपर ए सट यहा चलन ो तयार हए सठानी रासत लिलए रोटी पड म बाधर सठ ो द दी रासत म ए भख R ो दखर सठ न चारो रोटी उसो खिखला दी खर वह सठ यहा डलपर पहच तो उनी सठानी न उस महायजञ बचन ो हा यदिद बचन आए सठ न R ो रोटी खिखलान ो महायजञ नही समझा और वापस लौट आया घर आर शाम ो उसी घर म उस ए बडा ख0ाना मिमला 0ो उस दवारा किए गएrsquo महायजञrsquo ा परसार था

English language

Letter formal The heading the name and address of the person you are writing to must be included beneath your own address In formal letters ldquoblock stylerdquo of address is preferred

Subject complain in brief

Salutation If the person you are writing to is known to you you may begin ldquoDear MrrdquoOr ldquoDear Mrsrdquo In all other instances you should begin ldquoDear Sirrdquo or ldquoDear Madamrdquo Or ldquoSirsrdquo

The body A formal or business letter has four partsReference The letter should begin by referring to a letter you have received an advertisement or the reason that has prompted you to writeInformation In the second paragraph it is necessary to supply more detailed information that is related to the referencePurpose Here you must give the reason why you are writing the letter This must be stated clearly and ensure that it is relevant to the question that has been setConclusion round off the letter with some polite remarkThe subscription when a letter has begun with dear sir sirs Madam you should end with Yours faithfully or yours truly When however you address a person by name you must conclude with the words ldquoYours sincerelyrdquo

1 A park in your locality is slowly being used as a rubbish dump Write a letter to the Mayor of your city pointing out the nuisance and danger of this Request that action be taken to stop this immediately

Or2 You being a boarder ordered a set of lab manuals from a famous book shop in the town They sent you a wrong set of books Write a letter to the manager of the book shop

Chemistry Chapter-1 1)CHEMICAL FORMULA- Q What is the Significance of

L-2The Language of Chemistrybull Chemical Formula

Itrsquos a symbolic representation of a chemical substance eg ndash The formula of Sulphuric acid is H2SO4

2) Steps of writing Chemical Formula of a given substance-

1 Write the symbols of the constituent atoms or radicals side by side Keep the basic radical on LHS and acid radical on the RHS ( Na+Cl- )2 In case of a radical having more than one atom( compound radical) enclose the radical in a bracket eg (SO4-)3 Write the valencies of each radical on its right hand top4 If the valencies of the two radicals are divisible by a common factor then divide the valencies by the common factor5 Invert (criss-cross) the valency number ie write the valency of one atom below the second atom and vice versa 6 On interchanging if valency number is lsquoone the figure lsquoonersquo is never writtenFor Example- Compound -Calcium Nitrate1 Writing the symbols- Ca(NO3)2 Writing the valencies on their right hand top- Ca2(NO3)1

3 Valency numeral in simple ratio- Ca2(NO3)1

4 Criss-cross- Ca 2NO3 1

5 Writing the formula of the compound- Ca(NO3)2

Chemical formula

A The formula of a substance conveys the following information regarding a substance 1 The name of the substance (qualitative)2 The elements constituting the substance (qualitative)3 The number of various atoms present in a molecule of the substance (quantitative)4 Molecular weight of the substance and the relative weights of different elements present in it (qualitative)

Q What are the limitations of Chemical Formula

A The chemical formula suffers from the following limitations-I It fails to convey whether the elements in a molecule are present in the form of atoms or ionsFor example the formula KBr fails to tell us whether Potassium and Bromine are present in the form of ions II It does not tell anything about the binding force that holds atom in a molecule togetherIII It does not tell us about the arrangement of various atoms with respect to one another within the molecule

Q Examples of Some Chemicals with their Formula Chemical name and Common Name-

A Given in the class notesCommercial Studies

Joint Stock Company

Let us discuss about the demerits of Joint Stock CompanyDespite so many advantages it has got many disadvantages which are as follows

Difficulty in FormationDelay in Decision makingExcessive Government ControlLack of Secrecy

Company can be classified into several categories based on incorporation

QuestionExplain the demerits of Joint Stock CompanyAnswer) 1 Difficulty in Formation The legal requirements and formalities required to be completed are so many The cost involved is quite heavy It has to approach large number of people for its capital It cannot start its business unless certificate of incorporation has been obtained This is granted after a long time when all the formalities are completed

Chartered CompanyStatutory CompanyRegistered Company

Delay in Decision making In this form of organization decisions are not made by single individual All important decisions are taken by the Board of Directors Decision-making process is time-consuming So many opportunities may be costly because of delay in decision-making Promptness of decisions which is a common feature of sole trader ship and partnership is not found in a company

Excessive Government ControlA company and the management have to function well within the law and the provisions of Companies Act are quite elaborate and complex At every step it is necessary to comply with its provisions lest the company and the management should be penalized The penalties are quite heavy and in several cases officers in default can be punished with imprisonment This hampers the proper functioning of the company

Lack of Secrecy The management of companies remains in the hands of many persons Every important thing is discussed in the meetings of Board of Directors Hence secrets of the business cannot be maintained In case of sole proprietorship and partnership forms of organisation such secrecy is possible because a few persons are involved in the management

2 Define the following

Chartered Company- The crown in exercise of the royal prerogative has power to create a corporation by the grant of a charter to persons assenting to be incorporated Such companies or corporations are known as chartered companies Examples of this type of companies are Bank of England (1694) East India Company (1600) The powers and the nature of business of a chartered company are defined by the charter which incorporates it After the country attained independence these types of companies do not exist

in IndiaStatutory Company- A company may be incorporated by means of a special Act of the Parliament or any state legislature Such companies are called statutory companies Instances of statutory companies in India are Reserve Bank of India the Life Insurance Corporation of India the Food Corporation of India etc The provisions of the Companies Act 1956 apply to statutory companies except where the said provisions are inconsistent with the provisions of the Act creating them Statutory companies are mostly invested with compulsory powersRegistered companiesCompanies registered under the Companies Act 1956 or earlier Companies Acts are called registered companies Such companies come into existence when they are registered under the Companies Act and a certificate of incorporation is granted to them by the Registrar

Economics

Chapter-4Basic problems of Economy

Today let us discuss with the topic Production Possibility curve

QuestionExplain the concept of Production Possibility Curve with the help of diagram

Answer) Production Possibility curve is a locus of all possible combinations of two commodities which can be produced in a country with its given resources and technology

The above diagram shows that with the given resources and technology the economy can produce maximum either 5 thousand meters of cloth or 15 thousand quintals of wheat or any other combination of the two goods like B( 1 thousand meters of cloth and 14 thousand quintals of wheat C ( 2 thousands meters of cloth and 12 thousand quintals of wheat) etcProduction Possibility curve is also called production possibility boundary or frontier as it sets the maximum limit of what it is possible to produce with given resources

Geography

Rotationand Revolution

SUNrsquoS POSITION AND SEASONAL CHANGES EQUINOXES ndash SPRING AND AUTUMN

Q1 What is Spring EquinoxA1 On 21st March sunrays fall directly on the equator On that day

As the Equator divides the Earth into two equal halves the sun rays fall directly on the equator twice in a year Equinoxes means equal Spring EquinoxOn 21st March sunrays fall directly on the equator On that day the duration of day and night both are equal ( 12 hours day and 12 hours night) on every places located on equator This day is called as Spring EquinoxAutumn EquinoxOn 23rd September sunrays fall directly on the equator On that day the duration of day and night both are equal ( 12 hours day and 12 hours night) on every places located on equator This day is called as Autumn Equinox

SOLSTICES ndash SUMMER AND WINTERDue to inclination of the Earth on its axis and the apparent movement of the sun the sun rays fall directly on both tropics once in a year Solstice is a Latin word which mean ldquothe Sun standing stillrdquoSummer SolsticesAfter 21st March there is an apparent movement of the Sun to the north of the equator The apparent northward movement up to 21st June when the Sun appears overhead at the Tropic of Cancer (22frac12degN) The sun appears to stand still at this position and then moves southwards towards the equator This position of the Sun on 21st June is known as Summer Solstices On that day the duration of day and night both are equal ( 12 hours day and 12 hours night) on every places located on Tropic of Cancer (22frac12degN)Winter solstices The apparent southward movement of the Sun continues beyond the equator till 22nd

December On this day the Sun is overhead at the Tropic of Capricorn

the duration of day and night both are equal ( 12 hours day and 12 hours night) on every places located on equator This day is called as Spring Equinox

Q2 What do you mean by EquinoxA2 Equinoxes means equal It is use to explain the equal duration of day and night ( 12 hours day and 12 hours night) on the Earth

Q3 On which date the longest day in Tropic of CancerA3 21st June

Q4 What is the meaning of SolsticeA4 Solstice is a Latin word which mean ldquothe Sun standing stillrdquo

Q5 Which is the longest day in southern hemisphereA5 22nd December

Q6 On what date does the Arctic Circle experience the lsquoMidnight SunrsquoA6 On 21 June the Arctic Circle experiences the lsquoMidnight Sunrsquo

Q7 What is cause of Midnight Sun in NorwayA7 During the summer solstice (21 June) the North Pole is inclined towards the Sun Therefore the duration of sunlight or daytime increases from 12 hours at the Equator to 24 hours at the Arctic Circle and beyond Thatrsquos why The region beyond the Arctic Circle especially Norway is known as the Land of the Midnight Sun because there the Sun does not rise or set on 21 June

Q8 Match the column A with BA B

Summer Solstice 21st March

Autumn Equinox 23rd

September

Winter Solstice 21st June

(22frac12degS) This position of the Sun is referred to as the Winter Solstice because it marks the winter season in the Northern Hemisphere On that day the duration of day and night both are equal ( 12 hours day and 12 hours night) on every places located on Tropic of Capricorn (22frac12degS)SEASONS AND DURATION OF DAY AND NIGHT During the equinoxes all places on the Earth have 12 hours of day and 12 hours of night Due to the revolution of the Earth round the Sun on an inclined axis the duration of day and night varies according to seasons and the latitude of a placeDuring the summer solstice (21 June) the North Pole is inclined towards the Sun Therefore the duration of sunlight or daytime increases from 12 hours at the Equator to 24 hours at the Arctic Circle and beyondThe region beyond the Arctic Circle especially Norway is known as the Land of the Midnight Sun because there the Sun does not rise or set on 21 JuneAt the North Pole there will be six months of daylight The Sun will be seen always above the horizon at a low angle At 66degN 24 hours of sunlight can be seen only on 21 June Hammerfest in northern Norway is a place of tourist attraction for observing the phenomenon of the Midnight Sun This place has continuous daylight from 13 May to 29 July This place is easily accessible to tourists and has hotels and other facilities The view of the midnight Sun from here is enthrallingIn the Southern Hemisphere the duration of daylight decreases from 12 hours at the equator to 0 hours beyond the Antarctic Circle In the South Polar Region there is 24 hours of darkness The Sun is always below the horizon In the Southern Hemisphere which experiences winter the duration of night-time is longer than the duration of daylight

Spring Equinox 22nd

December

A8 A B

Summer Solstice 21st June

Autumn Equinox 23rd

September

Winter Solstice 22nd

December

Spring Equinox 21st March

During winter solstice (22 December) the South Pole is inclined towards the Sun The Southern Hemisphere experiences summer and the Northern Hemisphere has winter Therefore the duration of daylight or sunlight is greater in the Southern Hemisphere than in the Northern HemisphereThe duration of daylight increases from 12 hours at the equator to 24 hours beyond the Antarctic Circle The South Polar Region has 24 hours of sunlight for many days continuously At the South Pole there will be six months of sunlight The Sun will always be seen at a low angle above the horizon In the Northern Hemisphere the duration of daylight will decrease from 12 hours at the equator to 0 hours at the Arctic Circle There are 24 hours of darkness in the North Polar region The duration of night is greater than the duration of daylight as one move northwards from the Equator It is evident from the above table that the duration of daylight is 12 hours throughout the year at the equator only As one moves away from the equator the seasonal variations in the duration of daylight increase The seasonal variations in the duration of daylight are maximum at the Polar Regions

Subject Eng Literature (The Merchant of Venice ndash William Shakespeare)Topic Act II Scene 7 Lines 36 to 80 (End of scene ) [Students should read the original play and also the paraphrase provided]

Summary Questions amp AnswersThe Prince then examines the inscription on the silver casket which says ldquoWho chooseth me shall get as much as he deservesrdquo The Prince says that he deserves Portia more than anybody else because of his high rank his noble birth and his great wealth and power But then he argues that silver is ten times

(1) (Act II Sc 7 L 39-47)

From the four corners of the earth they come

To kiss this shrine this mortal breathing saint

The Hyrcanian deserts and the vasty wildsOf wide Arabia are as through-fares now

inferior to gold and therefore he cannot believe that the portrait of such a beautiful lady as Portia can be contained in the silver casket He decides to see the inscription on the golden casket before making his decision

The Prince goes to examine the inscription on the golden casket which says ldquoWho chooseth me shall get what many men desirerdquo The Prince believes that the whole world desires to possess Portia otherwise so many suitors would not have come from all corners of the world for winning Portia Some of them have come from the distant lands of Persia and Arabia The deserts of Persia (Hyrcanian deserts) and the boundless desolate lands of Arabia have been crossed by the Princes seeking the hand of Portia He contrasts this casket containing Portiarsquos portrait with the old English gold coin bearing the image of the archangel (angel of the highest rank) He goes on to remark that while the figure of the archangel is engraved (Insculped) upon the English coin the picture of Portia who is beautiful as an angel lies hidden inside one of the caskets namely the Golden Casket (Golden Bed)

On the basis of his assessment of the inscription on the golden casket the Prince decides to choose the golden casket He asks for the key and opens the golden casket only to find therein an empty human skull holding a roll of

For princes to come view fair PortiaThe watery kingdom whose ambitious headSpets in the face of heaven is no barTo stop the foreign spirits but they comeAs orsquoer a brook to see fair Portia

(i) Explain the occasion for the above mentioned speech

These are the comments of the Prince of Morocco after he reads the inscription on the golden casket His mental process is revealed to us in these words We find him debating within himself as to which casket he should choose

(ii) What light does the above speech throw on the personality of Prince of Morocco

From the above mentioned speech we come to know that the Prince of Morocco is keen to marry Portia He is the type of person who is easily taken away by outward appearance He is in love with Portia because of her beauty

(iii) What information can you gather about Portia from the above mentioned lines

The given speech shows that Portia is a very beautiful lady She must be possessed of good qualities because many suitors come to her place from all over the world with a desire to get married to her The Prince of Morocco is so impressed by her beauty that he calls her a saint According to him the whole world is desirous of having her

(iv) Elucidate the significance of the first two lines

In these lines the Prince of Morocco pays a compliment to Portia These lines show his admiration for her He says that people come from all parts of the world to see fair Portia

(v) Explain the meaning of the last four lines of the

passage

In these lines the Prince of Morocco says that even the vast oceans which throw a challenge at the sky are unable to prevent men from coming to Portiarsquos place to have a glimpse of her These lines are also a tribute to Portiarsquos beauty and good qualities Many men voyage across the ocean treating it as a mere stream to see the beautiful Portia

paper in which is written that whoever happens to be guided by the glitter of things is invariably deceived

On reading the scroll the Prince says that he is too sad at heart to speak a more formal farewell and leaves with his followers amidst a sound of trumpets

After the Prince of Morocco leaves Portia remarks that the Prince is a gentle fellow but she is rid of him May all persons of his nature make a similar choice

IMPORTANT PASSAGES EXPLAINED

(Act II Sc 7 L 39-43)From the four corners of the earth they come

To kiss this shrine this mortal breathing saintThe Hyrcanian deserts and the vasty wildsOf wide Arabia are as through-fares nowFor princes to come view fair Portia

Context

This passage occurs in Act II Scene 7 in The Merchant of Venice This is part of the speech made by the Prince of Morocco

(2)

(Act II Sc 7 L 48-53)

MOROCCO One of these three contains her heavenly pictureIst like that lead contains her

Twere damnation To think so base a thought it were too grossTo rib her cerecloth in the obscure graveOr shall I think in silver shes immurdBeing ten times undervalued to tried gold

(i) What meaning does the Prince of Morocco find out of the inscription of the golden casket What have Belmont and Portiarsquos house been called and why

The inscription on the golden casket is ldquoWho chooseth me shall gain what many men desirerdquo The Prince finds out that it means that the chooser of the golden casket will get Portia because many men desire her In fact the entire world desires her Because of the coming of many suitors to Belmont from different countries in order to win Portiarsquos hand Belmont has become a centre of pilgrimage and her house is the shrine where saintly Portia is installed

(ii) What does the Prince of Morocco do before making the final choice of the casket Which is the correct casket and who will win Portiarsquos hand

The Prince of Morocco surveys and analyses the inscriptions on the casket of lead silver and gold Before making the final choice like a very systematic and methodical person he once again considers the claims of the caskets The casket containing Portiarsquos picture is the correct casket and the person choosing it will win Portiarsquos hand

Explanation

While praising Portia the Prince of Morocco conceives Portia as a goddess whose image is placed inside one of the caskets Many suitors are coming from far and wide the north and the south the east and the west (Four corners) in order to try their luck Some of them have come from the distant land of Persia and Arabia The deserts of Persia (Hyrcanian deserts) and the boundless desolate lands of Arabia have been crossed by the Princes seeking the hand of Portia All this shows that Portia is indeed the most beautiful lady of the world

(iii) What does the Prince of Morocco say in his estimation while examining the motto on the silver casket What does he find in the golden casket

While examining the motto on the silver casket which says ldquoWho chooseth me shall get as much as he deservesrdquo Morocco says that in his own estimation he surely deserves Portia in all respects ndash rank birth wealth etc

He chooses the golden casket When he opens it he finds an empty human skull holding a scroll in which it is written that those who are attracted by the glittering outside of things are always deceived as Morocco has been deceived

(iv) What kind of nature does the Prince of Morocco have

The Prince of Morocco has a simple nature who does not look deeply into the inner meaning of things but is dazzled by the outward appearance of gold He is inclined to over-estimate his own value and does not realize that it is a duty to ldquogive and hazardrdquo To say that he will not hazard for lead shows that he misreads the true meaning of the inscription which is that he should be prepared to ldquohazard all he hathrdquo for Portia So his feeling is only one of fascination and romantic attraction

(v) Do you think that the lottery of the caskets is not a matter that will be determined by chance

In fact the lottery of the casket is not a matter that will be determined by mere chance but that it is a true test of character and of sincerity which is amply proved not only by Moroccorsquos choice but also by the arguments which he uses to help him in his choice

(Act II Sc 7 L 55-59)

They have in England

A coin that bears the figure of an angelStamped in gold but thats insculpd uponBut here an angel in a golden bedLies all within

Context

(3)

(Act II Sc 7 L 63-77)A carrion Death within whose empty eye

There is a written scroll Ill read the writing

All that glisters is not goldOften have you heard that toldMany a man his life hath soldBut my outside to beholdGilded tombs do worms infoldHad you been as wise as boldYoung in limbs in judgment oldYour answer had not been inscrolld

This passage occurs in Act II Scene 7 in The Merchant of Venice This is part of the speech made by the Prince of Morocco

Explanation

In this passage the Prince of Morocco bestows high praise on Portia whose hand he is seeking He contrasts this casket containing Portiarsquos portrait with the old English gold coin bearing the image of the archangel (angel of the highest rank) He goes on to remark that while the figure of the archangel is engraved (Insculped) upon the English coin the picture of Portia who is beautiful as an angel lies hidden inside one of the caskets namely the Golden Casket (Golden Bed) In the day of Elizabeth silver was ten times inferior in value to gold Therefore the Prince of Morocco believing that Portiarsquos portrait is contained in the Golden Casket decides to choose the Golden Casket

Fare you well your suit is coldCold indeed and labour lostThen farewell heat and welcome frostmdashPortia adieu I have too grievd a heartTo take a tedious leave Thus losers part

(i) What reward does the Prince of Morocco get after making a wrong choice of the Casket How does he feel

After making the wrong choice in selecting the casket of gold the Prince of Morocco as a reward earns a rebuke in the form of a scroll tucked in the empty eye-socket of a skull kept in the casket of gold The Prince is shocked and disappointed He becomes all the more sad and dejected when he reads the scroll which points to his foolishness in being misled by the appearance and outward show as indicative of its worth

(ii) How does the Prince respond after reading the scroll

After reading the scroll the Prince though upset accepts the result with good grace and decorum befitting a royal suitor and true sportsman He says that his love-suit is really cold otherwise he would have chosen correctly but now his efforts have been in vain So he bids farewell to Portia to the warmth and enthusiasm of love and welcomes the cold and bitterness of dejection and misery of life which lies ahead

(iii) What request does he make to Portia and why

After being failure in his mission he requests Portia to give him permission to leave at once because he is too sad to undergo the tediousness of a formal leave-taking He tells that it is the manner in which defeated persons part unceremoniously

(iv) Explain the following lines

ldquoAll that glisters is not goldOften have you heard that toldMany a man his life hath soldBut my outside to beholdGilded tombs do worms infoldrdquo

Mere glitter does not make a metal to be gold Man has often been warned against appearance but it has been of no use Many people have sacrificed their lives only to seek the outer appearance of gold Worms are found inside the gilded

monuments

Class XSubject Topic Summary Execution

Hindi 2ndlang

नया रासता भाग 6 मायाराम 0ी घर म धनी मल 0ी और उनी बटी सरिरता ी ही चचा बनी रहती थी अमिमत ो इसम ोई रलिच ना थी वह धनी घर ी लडी स शादी र सवय ो बचना नही चाहता था उसा भी सवाणिभमान ह ईशवर ी पा

स उस पास पस ी ोई मी नही थी अभी उसन फकटरी ही लगाई थी उसी समझ बाहर था कि उस घर वालो ा झाव पस ी तरफ कयो

ह उसन मा स सवाल किया कि मा तम सरिरता स मरी शादी कयो रना चाहती हो मा न उस समझाया कि वह दखन म बरी नही ह और किफर खानदान अचछा

ह वह ए शल गरहणी रप म घर सभाल सगी अमिमत न मा ो इस बात ा एहसास राया कि मीन सबध लिलए मना रन पर उस दिदल

पर कया बीती होगी मा और अमिमत ी लडी बार म ाफी बात हईमा ा झाव सरिरता ी तरफ था कयोकि वह घर पर अचछा दह0 लर आ रही

थी अमिमत न अपनी मौसी ी बरी हालत बार म बताया कि किस तरह वह बड घर ी खानदानी बटी लाई थी और आ0 उसी हालत कितनी खराब ह लाई थी बहकलब 0ाती ह और बचचो ो भी नही दखती ह बात चल ही रही

थी कि तभी ए ार बाहर आर री धनी मल0ी घर अदर आए और पीछ स डराइवर फल ी ए टोरी लर आया अदर आए और पीछ स

डराइवर ए टोरी फल ी लर आया अमिमत ो फल ी पटी बरी लग रही थी अमिमत न पछ लिलया यह फल कयो ल आए ह प इन सब ी कया

0ररत थी उनो न 0वाब दिदया कि 4 पटी शमीर स मगाए थ अमिमत ो या सनर करोध आ गया तभी उस किपता 0ी आ गए उन आत ही अमिमत उठर बाहर चला गया वहा वहा मा पास आर बठ गया और बोला

अभी रिरशता तय नही हआ और धनी मल 0ी धनी मल 0ी फल ी पटी लर चलआय मा न समझाया कि 0ब सबध 0ड 0ाता ह तो खाली हाथ नही

आत अमिमत न मा स हा कि तम सबन सरिरता ो इस घर म लान ी ठान रखी ह धनीमल 0ी उस दिदन सरिरता ो दखन ी तारीख तय रन आय थ

Commercial Studies

Banking Nowadays Bank provide easy and quick services through internet facilities methods of Banking is called internet bankingIn order to save the time and money involved in visiting Bank branches people increasingly prefer to have internet banking

There are different modes of doing internet banking or transferring money through online They areReal Time Gross Settlement (RTGS)National Electronic Fund Transfers (NEFT)

1

Question

1) Explain the term RTGS Write the features of RTGS

Answer)The acronym RTGS stands for Real Time Gross Settlement which may be defined as the continuous real time settlement of funds transfer individually on and order by order basis without netting lsquoReal timersquo may be defined as the processing of instructions at the time they are received rather than at some letter time lsquoGross settlementrsquo may be defined as the settlement of transfer instructions which occurs

individually

Features of RTGS1It is the continuous settlement of

funds transfer individually on an order by order basis

2RTGS facility is provided only by CBS core banking solution enabled Bank branches

3Amount charged from the customer for RTGS transactions vary from bank to bank

2) Explain the term NEFT Write the features of NEFT

Answer) National electronic funds transfer may be defined as a nationwide system that facilitates individuals Farms and copper operates to electronically transfer funds from any bank branch to any individual farm or corporate having an account with any other bank branch in the country

Features of NEFT2 Transfer can be made 7 times on

weekdays and 6 times on Saturday

3 NEFT cannot be used to receive foreign remittances

4 NEFT transaction takes place in batches

5 A bank branch must be NEFT enabled to become a part of NEFT fund transfer network

6 There is no maximum or minimum amount that can be transferred through NEFT when one bank has a bank account

English Language

CompositionEssay

A composition is an art of creating a piece of writing on any topic or subject It is the writing correctly beautifully and clearly in order to make some interesting reading Structure of the composition

Introduction ( you lay the foundation for your composition)

Body (it constitutes the main part of the essay)

Conclusion (final statement that leaves a lasting impression)

Kinds of essays1 The Narrative essay2 The descriptive essay3 The reflective essay4 The argumentative essay

Write a composition on any one of the following topics (350- 400 words)

1 Friendship Or2 The first day of your school

Subject Eng Literature (The Merchant of Venice ndash William Shakespeare)Topic Act V Scene 1 Lines 127 to 158 (Nerissa helliphellip The clerk will nersquoer wear hair onrsquos face that had it) [Students should read the original play and also the paraphrase given in the school prescribed textbook]

Summary Revision Questions o Soon thereafter Bassanio Gratiano

and Antonio arrive

o Bassanio tells Portia that he is feeling as if it is morning because of the presence of Portia who is shining like the sun When Antonio is introduced by Bassanio to Portia she tells Bassanio that he should be grateful to Antonio who took so much trouble on his account even to the extent of risking his life

o Nerissa starts quarrelling with Gratiano and demands that he show her the ring she had presented to him and which she had warned him not to lose She suspects that Gratiano must have presented the ring to some young woman and not to the lawyerrsquos clerk as he repeatedly says and assures

Answer the following questions to check your preparation of Act IV Scenes 1 and 2

You must attempt only after you have completed your preparation of Act IV The answers must be in complete sentences using textual evidence (with citation) when necessary

[It would be in your own interest to attempt the above questions honestly totally refraining from consulting your textbook or your notes during answering After completion you should correct the paper yourself consulting the textbooknotes etc and award marks as specified Please let me know the marks you scored through WhatsApp in the group or to my personal WhatsApp]

Act IV Scene 1 (each question carries 2 marks)

1 What did the Duke try to do for Antonio

2 Why does Shylock refuse to show mercy How does he justify his stance

3 Why does Antonio say he is ready to die 4 What information is contained in Bellariorsquos letter

5 Why does Portia (as Balthazar) assert that Shylock must show mercy How does he respond

6 What offers are made to Shylock to get him to spare Antonio How are they received

7 What does Antoniorsquos speech as he faces the prospect of Shylockrsquos knife tell you about his character

8 How do Bassanio and Gratiano react to the looming prospect of Antoniorsquos demise

9 How does Portia (as Balthazar) use the law to turn the tables on Shylock

10 What does the Duke decree should happen to Shylock Why What happens to Shylockrsquos estate

11 What does Portia ask Bassanio as payment for her ldquoservicesrdquo What is his initial response What makes him change his mind

Act IV Scene 2 (each question carries 1frac12 marks)

1 What does Gratiano bring to Portia (Balthazar)

2 What does Nerissa plan on getting from Gratiano What does Portiarsquos comment suggest about men

ECO-10 280620 Topic-Supply AnalysisSHIFTING OF SUPPLY

But if there is change in factors other than the price of the commodity then either more is supplied at the same price or less supplied at the same price In such cases the price of the commodity remains constant but there is a change in other factors like change in the price of inputs change in technology of production change in price of other related goods change in taxation policy of the government etc For example there is an improvement in the technology of production of the commodity in question It leads to decrease in per unit of cost production of the commodity The firm is willing to sell more quantity of the commodity at the same price So the supply other commodity increases at the same price This increase in supply is shown by rightward shift of supply curve On the other hand if the firm uses inferior technology of production the cost of production per unit of the commodity increases The firm is willing to sell less quantity at the same price So the supply of the commodity decreases at the same price This decrease in supply is shown by leftward shift of the supply curve The above cases of increase and decrease in supply can be shown with the help of the following figures

Y INCREASE OF SUPPLY Price (Rs) s

P A s1

B

s

X` O s1 X

q q1

Y` Quantity demanded (in units)

Y DECREASE IN SUPPLY s2

s

price (Rs)

C

p A

s2

s

X` o X

q2 q

Y` Quantity demanded ( in units)

Main factors causing increase in supply or rightward shift of supply Curve(i) Fall in the price of other related goods

(ii) Fall in the price of inputsfactors(iii) Use of better technology in production(iv) Decrease in the rate of excise duty by government(v) If the objective of producer changes from profit maximization to salesMaximization

Main factors causing decrease in supply or leftward shift of supply curve(i) Increase in the price of other related goods(ii) Rise in the price of inputsfactors(iii) Use of inferior technology in production(iv) Increase in the rate of excise duty by the government(v) If the objective

Subject - Biology Topic ndash Chapter mdash6 PhotosynthesisSummary Execution

Today we will know about photosynthesis and its stages

Q1 What do you mean by photosynthesis The process by which living plants containing chlorophyll produce food

substances from carbon-di- oxide and water by using light energy Sunlight

6CO2 +12 H2O----------------------- C6 H12O6 + 6H2O + 6O2

Chlorophyll

Q2 What are the importance of photosynthesis I) Food for all Green plants trap solar energy by photosynthesis

process and supply food and energy for all living organisms either directly or indirectly

Ii) Oxygen to breathe in by product of photosynthesis is oxygen which is essential for all living organisms respiration

Q3 Write about two main phases of photosynthesis A Light dependent phase This phase occur in grana of chloroplast I) The chlorophyll on exposure to light energy becomes activated by

absorbing photons Ii) The absorbed energy is used in splitting the water molecules (H2O)

into its two components (H+ and OH- ) and releasing electron s 2H2O------------------------- 4H+ + 4e- +O2

Energy of 4 photons This reaction is known as photolysis

End products are H+ and oxygen water

B Light independent (Dark ) phase The reactions in this phase require no light energy

Here CO2 combine with H+ and produce glucose

Class XI

Subject Topic Summary ExecutionEVS Chapter-4 Legal

regimes for sustainable development

Environmental legislationEnvironmental legislation is the collection of laws and regulations pertaining to air quality water quality the wilderness endangered wildlife and other environmental factors The act ensures that matters important to the environment are thoroughly

Learn -The Forest (Conservation) Act 1980

considered in any decisions made by federal agencies

The Forest (Conservation) Act 1980 The Forest (Conservation) Act 1980 an Act of the Parliament of India to provide for the conservation of forests and for matters connected therewith or ancillary or incidental thereto It was further amended in 1988 This law extends to the whole of IndiaObjects and Reasons of the Forest Conservation Act

Deforestation causes ecological imbalance and leads to environmental deterioration Deforestation had been taking place on a large scale in the country and it had caused widespread concern The act seeks to check upon deforestation and de-reservation of forests

Subject Eng Literature (The Tempest ndash William Shakespeare) Topic Act II Scene 1 Lines 314 to 329 (End of scene)

[Students should read the original play and also the paraphrase given in the school prescribed textbook]Summary Questions amp Answers

Conspiracy of Antonio and Sebastian (Contd)

o As they approach Ariel appears again and wakes up Gonzalo by singing a tune in his ear Alonso also wakes up and they see both Sebastian and Antonio with drawn swords On being caught off guard they make up a story saying that they had heard a bellowing of bulls or lions

o They then moved to another part of the island

o Ariel at once rushes to Prospero to inform him of this development

SUMMING-UP of ACT-2 SCENE-1

(i) Among the survivors Ferdinand is separated from the rest which results in the disconsolate grief of Alonso as he took him for dead

(ii) The villainy of Antonio is confirmed

(iii) The supremacy of Prosperorsquos magic which resulted in the failure of the human conspiracy

(1)

(Act II Sc 1 L 311-325)SEBASTIAN Whiles we stood here securing your repose

Even now we heard a hollow burst of bellowing Like bulls or rather lions Didt not wake youIt struck mine ear most terribly

ALONSO I heard nothingANTONIO O rsquotwas a din to fright a monsters ear

To make an earthquake Sure it was the roarOf a whole herd of lions

ALONSO Heard you this GonzaloGONZALO Upon mine honour sir I heard a humming

And that a strange one too which did awake meI shaked you sir and cried As mine eyes opened I saw their weapons drawn There was a noiseThats verily rsquoTis best we stand upon our guardOr that we quit this place Lets draw our weapons

(i) Why has Prospero sent Ariel to Gonzalo and Alonso What does Ariel do to awaken Gonzalo

Prospero has already come to know by his magic powers the danger which threatens Gonzalo who had been Prosperorsquos friend and so he sent Ariel to preserve the lives of both Gonzalo and Alonso Prospero does not want that his scheme should remain unfulfilled Ariel begins to sing a song in Gonzalorsquos ears to awaken him(ii) Who are ready to carry out their plan Who takes steps to stop them Why does Gonzalo feel surprised after being awakened

Sebastian and Antonio are ready to carry out their plans They are standing with their swords drawn to kill Alonso and

(iv) We see two sets of contrasting characters Gonzalo-Adrian against Antonio-Sebastian

(v) The grief that works in Alonso can be perceived to his repentance for his association in Antoniorsquos crime against Prospero

Gonzalo Ariel takes steps to stop them from carrying out their nefarious scheme When Gonzalo is awakened by the song sung by Ariel into his ears he (Gonzalo) feels surprised because he sees Sebastian and Antonio standing with their swords drawn(iii) What reason do Sebastian and Antonio tell of drawing their swords when they are suspected by Alonso and Gonzalo

When Sebastian and Antonio are seen with their swords drawn they are looked with suspicion by Gonzalo and Alonso At first Sebastian tells them that as they stood here to guard them during their sleep they heard only a little before a sudden loud noise very much like the roaring of bulls or more probably that of lions Then Antonio follows him saying that this was a noise so terrible as to frighten even a monsterrsquos ears and this noise could even have shaken the earth and it was surely like the roaring of a multitude of lions Then seeing the danger they have drawn their swords Perhaps after hearing the terrible noise they (Gonzalo and Alonso) woke up from their sound sleep

(iv) What does Gonzalo tell Alonso about the strange noise What did he see on opening his eyes Gonzalo tells Alonso that he did not hear the sound of roaring but he heard a humming sound which was strange and which woke him up After waking up he gave him (Alonso) a shaking and a loud cry On opening his eyes he saw these two gentlemen standing with their swords drawn(v) What does Gonzalo suggest

Gonzalo suggests that there was a noise indeed and of that he has no doubt at all and suggests that the best course for them would be to remain alert and vigilant against any possible danger to their lives or to leave this place and move to some other part of the island

Class XIISubject Topic Summary Execution

Commerce

Chapter- Management

Today we will discuss about LEVELS OF MANAGEMENT

Levels of management is a series or chain of managerial positions from top to bottom It helps individuals to know their authority responsibilities and superior-subordinate relations among themselves There are mainly three levels of Management TOP LEVEL MANAGEMENTMIDDLE LEVEL MANAGEMENTLOWER LEVEL MANAGEMENT

Top level managementIt consists of members at the highest level in the management hierarchy This level includes Board Of Directors Chief Executive Managing Directors Chairman President Vice President

Rolefunctions of the top levelmanagement1To analyse evaluate and deal

with theexternal environment2 To determine the objectives and

policies of the business3 To strive for welfare and survival

of business

4 To create an organisational Framework consisting of authority responsibility relationship

Middle level management Congress of members or groups who are concerned with implementation of the policies let down by the top managementThis level includes head of the department such as finance manager marketing manager branch and regional managers departmental and divisional heads plant superintendent etc

Role of functions of the middle level management

1 To interpret the policies framed by top management

2 To assign duties and responsibilities to lower level managers

3 To select and appoint employees for middle and supervisory level and evaluate their performance

4 To co-operate with other departments for smooth functioning

Operational or supervisory level managementIt refers to the group are members who are concerned with execution of the work They are also known as fast line managers This level includes supervisor 4 men Section Officer clerk Inspector etc

Role of functions of the lower level management1 To plan and execute day-to-

day operations2 To supervise and control the workers3 To arrange materials and

tools to start the process and make arrangements for training

4 Today present workers grievance and suggestions before the management and

ensure safe and proper working conditions in the factory

Business Studies

Staff Appraisal Chapter- 10 Today let us start with a new chapter

Staff Appraisal

Meaning of Performance Appraisal

Performance Appraisal is the systematic evaluation of the performance of employees and to understand the abilities of a person for further growth and developmentThe supervisors measure the pay of employees and compare it with targets and plansThe supervisor analyses the factors behind work performances of employeesThe employers are in position to guide the employees for a better performance

Objectives of Performance Appraisal

Following are the objectives of Performance Appraisal

To maintain records in order to determine compensation packages wage structure salaries raises etc

To identify the strengths and weaknesses of employees to place right men on right job

To maintain and assess the potential present in a person for further growth and development

To provide a feedback to employees regarding their performance and related status

To provide a feedback to employees regarding their performance and related status

Importance of Performance Appraisal

Performance appraisal provides important and useful information for the assessment of employees skill

knowledge ability and overall job performance The following are the points which indicate the importance of performance appraisal in an organization

1 Performance appraisal helps supervisors to assess the work performance of their subordinates

2 Performance appraisal helps to assess the training and development needs of employees

3 Performance appraisal provides grounds for employees to correct their mistakes and it also provides proper guidance and criticism for employees development4 Performance appraisal provides reward for better performance

5 Performance appraisal helps to improve the communication system of the organization

6 Performance appraisal evaluates whether human resource programs being implemented in the organization have been effective

7 Performance appraisal helps to prepare pay structure for each employee working in the organization

8 Performance appraisal helps to review the potentiality of employees so that their future capability is anticipated

Geography

DRIANAGE The SubarnarekhaThe Subarnarekha and the Brahmaniinterposed between the Ganga and the Mahanadi deltas drain an area of 19300 sq kmand 39033 sq km respectively The drainage basins of these streams are shared byJharkhand Odisha west Bengal and Chhattisgarh The Brahmani is known as southKoel in its upper reaches in Jharkhand

The NarmadaThe Narmada rises in the Amarkantak hills of MadhyaPradesh It flows towards the West in a rift valleyformed due to a geological fault The total length of it is 1300 km All the tributaries of the

Q1 Name the two westward flowing rivers in the peninsular plateauA1 Narmada and Tapi are the only westward flowing rivers of the peninsular plateau

Q2 Differentiate between east-flowing rivers and west-flowing riversA2

East-flowing rivers

West-flowing rivers

Narmada are very short inlength Most of its tributaries join the main streamright anglesThe Narmada basin covers parts of Madhya Pradesh and Gujarat

The Tapi The Tapi rises in the Satpura ranges in the Betul listrictof Madhya Pradesh It flows in a rift valley parallel tothe Narmada but it is much shorter in length It coversparts of Madhya Pradesh Gujarat and MaharashtraThe length is about 724 km

The Sabarmati and the MahiThe Sabarmati rises in the Aravali hills and flows south-south-westwards for a distance of 300 kilometres to the Arabian Sea The Sabarmatibasin extends over an area of 21674 sq km in Rajasthan and Gujarat The Mahi rises inthe east of Udaipur and drains an area of 34842 sq km lying in Madhya PradeshRajasthan and Gujarat It flows south-westwards for a distance of 533 km before it fallsinto the Gulf of Khambhat

The ChambalThe Chambal rises near Mhow in the Vindhya Range and flows towards the northgenerally in a gorge upto Kota Below Kota it turns to the north-east direction and afterreaching Pinahat it turns to the east and runs nearly parallel to the Yamuna beforejoining it in the southern part of the Etawah district in Uttar PradeshMajor Rivers of India with their basin area (Sqkm)

Himalayan System Indus 321290Ganga 861404

Brahmaputra 187110Indus System

Jhelum 34775Beas 20303

Ganga System Yamuna 366223Ghaghra 127950

Peninsular RiversNarmada 98796

Tapi 65145Mahanadi 141600

Subarnarekha 19300Sabarmati 21674

Mahi 34842Godavari 312812

Godavari Krishna Kaveri Mahanadi are the east-flowing rivers

Narmada Tapi west-flowing rivers

They fall into the Bay of Bengal

They fall into Arabian Sea

These rivers form big deltas

These rivers form comparativelysmall deltas

Catchment areas of these rivers are larger

Catchment areas of these rivers are smaller

Krishna 2589488Cauveri 87900

Subject ndashBiology Topic ndashChapter -5 Inheritance amp Variations Summary ExecutionToday we will discussabout linkage and its classification

LINKAGE The tendency of the genes located on the same chromosome to stay together is

hereditary transmission Linked genes the genes responsible for this Genes that exhibit the process of linkage locates in the same chromosome The distance between the linked genes in a chromosome determines the strength

of linkage i e genes that are located close to each other show stronger linkage than that are located far from each other

COMPLETE LINKAGE It is the type of linkage showed by the genes that are closely located or are tightly

linked with each other as they have no chance of separatingby crossing over These genes are always transmitted together to the same gamete and the same

offspring In such condition only parental or non cross over type of gametes are formedINCOMPLETE KINKAGE It is type of linkage showed by the genes that are distantly located orare loosely

linked with each other because they have chance of separating by crossing over

SIGNIFICANCE i) It helps in holding the parental character togetherii) It checks the appearance of new recombination and helps in bringing the

hybrid population which resembles the original parents iii) Linked genes dilute the effects of undesirable traits

Subject Eng Literature (The Tempest ndash William Shakespeare) Topic Essay Questions (EQ-3)Question No 3

Give a character sketch of CalibanAnswer

The character of Caliban has been wonderfully conceived by Shakespeare as the manifestation of all that is gross and earthy ndash a sort of creature of the earth as Ariel is a sort of creature of the air

Calibanrsquos Physical Appearanceo Caliban is lsquofreckledrsquo a lsquomisshapen knaversquo not honoured with human shape

o Prospero calls him lsquothou tortoisersquo (Act I Sc 2 Line 317) Trinculo stumbling upon him describes him as ldquoA strange fish hellip Legged like a man And his fins like armsrdquo He ldquosmells like a fishrdquo (Act II Sc 2 Line 25)

o Prospero also calls him a ldquobeastrdquo (Act IV Sc 1 Line 140) and ldquoThis misshapen knaverdquo (Act V Sc 1 Line 268)

o Further it appears that in addition to his physical deformity his spiritual inferiority is also suggested by Prosperorsquos claim that his birth resulted from the union between his mother the witch Sycorax and the devil

Calibanrsquos ParentageWhen the play opens Caliban is twenty four years of age having been born on the island twelve years before the coming of Prospero His mother was the foul witch Sycorax who was banished from Algiers for ldquomischiefs manifold and sorceries terrible to enter human hearingrdquo (Act I Sc 2 Line 264) and the father was the Devil himself Thus

Caliban is a monster of evil and brute nature ugly deformed and stinking

Calibanrsquos Savage and Malignant Natureo Caliban is entirely a creature of the earth ndash gross brutal and savage He regards himself as the rightful possessor

of the island and Prospero as a usurper

o In his young age he was on good terms with Prospero He had consented to be received by Prospero at his house and to be educated by him He has learnt human language only to curse his master whom he abhors

o His beastly nature soon breaks out and ends in a vicious attack on Miranda This opens the eye of Prospero who becomes severe to him and enforces his service by threats and violence

o Prospero uses him to make dams for fish to fetch firewood scraper trenches wash dishes and keep his cell clean

Calibanrsquos Hatred for ProsperoA profound hatred for Prospero has taken hold of Caliban It springs from a sense of his being dispossessed and ill-treated He would kill Prospero if he could but he knows the power of Prosperorsquos lsquobookrsquo Hence he transfers his allegiance to Stephano who seems like a god to him He also incites the two drunken associates to batter the skull of Prospero when he sleeps in the afternoon

Caliban Shows Considerable Intelligenceo He has learnt Prosperorsquos language

ldquoYou taught me language and my profit onrsquot (Act II Sc 2 Lines 86-89)Is I know how to curserdquo

o He is well aware of the futility of arguing with one who has more power than he has

ldquoI must obey his art is such power (Act I Sc 2 Lines 373-376)It would control my damrsquos god SetebosAnd make a vassal of himrdquo

o He realizes the importance of Prosperorsquos books

ldquoRemember (Act III Sc 2 Lines 89-92)First to possess his books for without themHersquos but a sot as I am nor hath notOne spirit to commandrdquo

o He knows the value of stealth when attacking the enemy

ldquoPray you tread softly that the blind mole may not (Act IV Sc 1 Lines 194-195)Hear a foot fall we now are near his cellrdquo

o Caliban has a better set of values than Stephano and Trinculo They are distracted from their plan by their greed for Prosperorsquos rich garments Only Caliban realizes that such a finery is unimportant

ldquoLeave it alone thou fool it is but trashrdquo (Act IV Sc 1 Lines 224)

Caliban is not a good judge of characterCaliban is not a good judge of character He decides for example that Stephano is a god because he dispenses lsquocelestial liquorrsquo (Act II Sc 2 Line 115) but then it must be remembered that he has only known his mother Sycorax Prospero Miranda and the spirits that torture him However he quickly discovers his error of judgementrdquo

ldquoWhat a thrice-double ass (Act V Sc 1 Lines 295-297)Was I to take this drunkard for a godAnd worship this dull foolrdquo

Calibanrsquos Imaginative NatureIf Caliban is sub-human in what has been said above he is human in the respect of the poetic side of his character He listens to music with rapture He tells of the beautiful dreams in which heaven rains treasures upon him and which upon waking he yearns to renew One of the most poetic passages in whole play is Calibanrsquos description of the island

to Stephano and Trinculo

ldquoBe not afeard The isle is full of noises (Act III Sc 2 Lines 135-143)Sounds and sweet airs that give delight and hurt notSometimes a thousand twangling instrumentsWill hum about mine ears and sometime voicesThat if I then had waked after long sleepWill make me sleep again and then in dreamingThe clouds methought would open and show richesReady to drop upon me that when I wakedI cried to dream againrdquo

Caliban - Less Ignoble Than Some OthersCalibanrsquos motive for murder is less dishonourable than that of Antonio and Sebastian They plan to kill Alonso to gain his power and wealth Caliban merely wants revenge and the return of lsquohisrsquo island

Conclusiono Calibanrsquos character is not portrayed very clearly in the play and hence we cannot decide whether he is a poor

savage being grossly maltreated by Prospero or whether he is evil and must therefore be kept in bondage or enslavement

o Caliban is contrasted with Ariel who is a spirit and thus swift and uninterested in physical activitieso Caliban is also contrasted with Prospero who is the all-powerful master of the island and of the destiny of all

those on the islando Caliban is also contrasted with civilized man showing him to be less evil than Antonio and Stephano and less

materialistic than Stephano and Trinculoo Caliban has suffered at the hands of Prospero and he has learnt to curse by listening to Prosperorsquos abuse He

certainly believes that Prospero has deprived him of his birthrighto Finally the character Caliban is thought to be one of Shakespearersquos masterpieces The complexity of the character

is reflected in the large volume of critical discussion that has grown around it

ECO ndash12 Topic-Forms of market

MonopolyMonopoly is a market structure in which there is a single seller there are no close substitutes for the commodity produced by the firm and there are barriers to entry Example Indian Railways which is operated under government of India Monopoly also implies absence of competitionFeatures of Monopoly Monopoly is characterized by1 Single Seller In monopoly there is only one firm producing the product The whole industry consists of this single firm Thus under monopoly there is no distinction between firm and industry Being the only firm there is significant control of the firm over supply and price Thus under monopoly buyers do not have the option of buying the commodity from any other seller They have to buy the product from the firm or they can go without the commodity This fact gives immense control to the monopolist over the market

2No Close Substitute There are no close substitutes of the product produced by the monopolist firm If there are close substitutes of the product in the market it implies presence of more than one firm and hence no monopoly In order to ensure a total of control over the market by the monopolist firm it is assumed that there are no close substitutes of the product

3 No Entry amp Exit Monopoly can only exist when there is strong barriers before a new firm to enter the market In fact once a monopoly firm starts producing the product no other firm can produce the same One reason for this is the ability of the

monopolist to produce the product at a lower cost than any new firm who thinks to enter the market If a new firm who knows that it cannot produce at a lower cost than the monopolist then that firm will never enter the market for fear of losing out in competition Similarly the monopolist who is operating for a long time may be enjoying reputation among its customers and is in a better position to use the situation in its own benefit A new firm has to take long time to achieve this and so may not be interested to enter the market

4 Price Maker Being the single seller of the product the monopolist has full control over the pricing of the product On the other hand if there is a large number of buyers in the market so no single buyer exercises any significant influence over price determination Thus it is a sellerrsquos market So monopoly firm is a price maker

5 Price Discrimination Having considerable control over the market on account of being single seller with no entry of other firms the monopolist can exercise policy of price discrimination it means that the monopolist can sell different quantities of the same product to a consumer at different price or same quantity to different consumers at different prices by adjudging the standard of living of the consumer

6 Shape of Demand Curve Since a monopolist has full control over the price therefore he can sell more by lowering the price This makes the demand curve downward sloping

Subject Ac-12 290620 Topic- retirement Model sumThe Balance Sheet of Rohit Nisha and Sunil who are partners in a firm sharing profits according to their capitals as on 31st March 2014 was as under

Liabilities Amount Assets Amount (Rs) (` Rs)

Creditors 25000 Machinery 40000Bills Payable 13000 Building 90000General Reserve 22000 Debtors 30000Capital Less Provision for Rohit 60000 Bad debts 1000

29000 Nisha 40000 Stocks 23000 Sunil 40000 140000 Cash at Bank 18000

200000 200000

On the date of Balance Sheet Nisha retired from the firm and following adjustments were made(i) Building is appreciated by 20(ii) Provision for bad debts is increased to 5 on Debtors(iii) Machinery is depreciated by 10(iv) Goodwill of the firm is valued at Rs 56000 and the retiring partnerrsquos share is adjusted

(v) The capital of the new firm is fixed at Rs120000 Prepare Revaluation Account Capital Accounts of the partner and Balance Sheet of the new firm after Nisharsquos retirement Revaluation AccountDr Cr

Particulars Amount Particulars Amount (`Rs) (Rs`)

Provision for Bad debt Ac 500 Building Ac 18000Machinery Ac 4000Profit transferred toCapital Accounts (3 2 2)Rohit 5786Nisha 3857Sunil 3857

13500

18000 18000

Capital Account

Dr Cr

Particulars Rohit Nisha Sunil Particulars Rohit Nisha Sunil (Rs`) (Rs`) (`Rs) (Rs`) (Rs`) (Rs`)

Sunilrsquos Capital ac 9600 mdash 6400 Balance bd 60000 40000 40000Bank - 66143 - General Reserve 9428 6286 6286Balance cd 72000 mdash 48000 Revaluation (Profi 5786 3857 3857 Rohitrsquos Capital Ac mdash 9600 mdash

Sunilrsquos Capital Ac 6400 Bank 6386 - 4257

81600 66143 54400 81600 66143 54400

Balance Sheet as at 31st March 2014

Liabilities Amount Assets Amount (Rs`) (Rs`)

Creditors 25000 Building 108000Bank overdraft 37500 Machinery 36000

Bills Payable 13000 Debtors 30000Capital Less ProvisionRohit 72000 for Bad debts 1500 28500Sunil 48000 120000 Stock 23000

195500 195500

Working Notes (i) (a) Profit sharing ratio is 60000 40000 40000 ie = 3 2 2(b) Gaining Ratio Rohit = 35 ndash 37 = 2135 ndash 1535 = 635Sunil = 25-27 = 1435 ndash 1035 = 435= 635 435= 6 4 = 3 2(c) Nisha Share of Goodwill = Rs 56000 times 27 = Rs16000Share of Goodwill in the gaining ratio by the existing partner ieRohit = Rs16000 times 35 = Rs 9600Sunil = Rs 16000 times 25 = Rs 6400

The journal entry isRohitrsquos Capital Ac Dr 9600Sunilrsquos Capital Ac Dr 6400 To Nisharsquos Capital Ac 16000(Share of Goodwill divided into gaining ratio)

  • 1 Static Friction
  • The frictional force that acts between the surfaces when they are at rest with respect to each other is called Static Friction
    • Static Friction Examples
      • 2 Sliding Friction
        • Examples Of Sliding Friction
          • 3 Rolling Friction
            • Examples Of Rolling Friction
              • Objects and Reasons of the Forest Conservation Act
Page 15:  · Web viewSubject . Topic . Summary . Execution . English 1 . Sounds of animals . Hens –cackle Horses –neigh Lions –roar Owls –hoots Snake –hiss. English 2 . Mother’s

was taken care by early monarchs Later kings had neither ability nor interest in economic af-fairs That led to failure in tax collection As a result they failed to maintain a large army that were essential to keep empire intact

4 Greek Invasion Greeks freed north-western provinces from weak Mauryan monarchs and reestablished their authority

5 Ashokarsquos Policy some scholar opined that after Kalinga war Ashoka embraced Buddhism re-nounced the policy of war and disbanded the Army But this is partially true as there is no proper evidence of disbanding the army

Based on above points we can conclude that main reason for decline of Mauryan empire is weakness of Ashokarsquos successors Kunal Samprati Dasharath Salisuk all were weak kingsAt last in 185 BCPushyamitra Sunga killed king Brihadrath and established the Sunga dynasty

BC2 Bindusara was the son of Chandragupta and father of Ashoka

3 Pataliputra was administered by City Magistrate committess of 5 members each4 The Greek General Seleucus sent his ambassador Megasthenes to Chandraguptarsquos court5 Ashoka sent his son Prince Mahendra and daughter Sanghamitra to spread his Dhamma6 The Indian Rebublic has adopted the Lion Capital of Saranath Pillar as its national emblem 7 Pushyamitra killed the last Mauryan ruler Brihadrath and founded the Sunga dynasty

III Name the following

1The author of Arthashastra-Kautilya2 The ruler who founded the Mauryan dynasty-Chandragupta3 The author of Indika-Megasthenes 4 The officers who were appointed by Ashoka to spread Dhamma-Dhamma Mahamatras5 The general of Alexander whom Chandragupta defeated-Seleucus

V Match the columns1 Kautilya (c)2 Megasthenes (d)3 Pushyamitra (e)4 Brihadrath (b)5 Bindusara (a)

BENGALI(2ND

LANGUAGE)

পশপাহিখর -াষাসহিবনয় রায়কেচৌধরী

যলখক পহিরহিচহিত- পরখযাত সাহিহিতযক উকেপনদরহিককেশার রায়কেচৌধরীর পতর সহিবনয় রায়কেচৌধরী lsquoসকেFশrsquo পহিতরকার সকেb হিতহিন কত হিকেলন তার উকেdখকোয বই lsquoসহিবনয় রায়কেচৌধরীর রচনা সংগরrsquo

পরম হিকেনর পাঠ- lsquoপশপাহিখর হিক -াষাhelliphellip helliphelliphelliphellipপরসপরকেক জানাবার উপায়ও পশপাহিখরা যবশ জাকেনrsquoপরকেমই আমারা জাহিন -াষা হিক -াষা ল আমাকের মকেনর -াব পরকাশ করার জনয আমরা নানান ধরকেনর -হিb বা হিবকেশষ ধরকেনর আওয়াজ মকেখর মাধযকেম কহির অনযকেক যবাঝাকেনার জনয তাকেল এবার আমরা জাহিন পশপাহিখর -াষা হিক পশ পাহিখরা হিক কা বকেল যা পশপাহিখকেরও -াষা আকে তারা তাকের হিনজসব -াষায় কা বকেল মকেনর -াব পরকাশ ককের পশ পাহিখরা মানকেষর হিক হিক -াষা যবাকেঝ হিকনত তারা বলকেত পাকেরনা পরসপরকেক বহিঝকেয় যবার উপায় তারা জাকেননা তকেব তারা হিবকেশষ ককেয়কটি শকেবদর মাধযকেম তাকের মকেনর -াব বহিঝকেয় যয় হিক বহিদধ মান জীব ndashককর হিবাল বন মানষ যঘাা পর-হিত এরা মানকেষর যওয়া নাম শনকেল কান খাা ককের ndash নাম ধকের ডাককেল কাকে আকেস যমন - মরহিরা lsquoহিত ndashহিতrsquo ডাক শকেন আকেস াল lsquoঅ ndashর -র ডাক শকেন কাকে আকেস াহিত মাহকেতর কা শকেন চকেল ককররা মাহিলকেকর হকম পালন ককের সবসময় তাইকেতা ককরকেক পর- -কত পরানী বলা য় ককর আর হিবাল একের আওয়াজ তহিম লকষয করকেল বঝকেব ককররা যরকে যকেল lsquoযঘউ যঘউrsquo করকেত াকেক আবার কাকেল lsquoযকউ যকউrsquo ককের হিবাল সাধারণ lsquoমযাওrsquo বা lsquoহিমউrsquo ককের রা কেল lsquoওয়াওrsquo আওয়াকেজর মাধযকেম মকেনর -াব পরকাশ ককের একেতা যল পশকের কা পাহিখরাও -য় রা পরকাশ করার জনয হিবকেশষ ধরকেনর শবদ ককের হিবপকের সময় পশ পাহিখরা সবার আকে পরসপরকেক জানাবার উপায় তারা জাকেন বহকাল

১) পশপাহিখর -াষা কেলপর যলখক সমপকেকG হিক জাকেনা

উঃ- পরখযাত সাহিহিতযক উকেপনদরহিককেশার রায়কেচৌধরীর পতর সহিবনয় রায়কেচৌধরী lsquoসকেFশrsquo পহিতরকার সকেb হিতহিন কত হিকেলন তার উকেdখকোয বই lsquoসহিবনয় রায়কেচৌধরীর রচনা সংগরrsquo হিতহিন ারকেমাহিনয়াম এসরাজ পর-হিত বাযনতর বাজাকেত পারকেতন ানও জানকেতন হিতহিন যাকেIাকের জনয মজাার লপ কহিবতা হিলখকেতন

২) পশপাহিখ কেলপর মল-াব হিকউঃ- পশপাহিখকেরও -াষা আকে তারা তাকের হিনজসব -াষায় কা বকেল মকেনর -াব পরকাশ ককের পশ পাহিখরা মানকেষর হিক হিক -াষা যবাকেঝ হিকনত তারা বলকেত পাকেরনা পরসপরকেক বহিঝকেয় যবার উপায় তারা জাকেননা তকেব তারা হিবকেশষ ককেয়কটি শকেবদর মাধযকেম তাকের মকেনর -াব বহিঝকেয় যয় হিরউকেবন কযাসটং সাকেব হিতহিন চহিdশ বর বনযজনত যর সকেb যকেককেন হিতহিন বকেলকেন আমরা হি তাকের -াষা তাকের আব কায়া যমকেন চহিল তাকেল আর -কেয়র যকান কারণ াকেকনা আমরা একI -াকেলাকেবকেস যচষটা করকেল পশপাহিখকের সকেb -াব পাতাকেত পাহির

ধকের মানষ এই পশ পাহিখর -াষা হিনকেয় নানা রককেমর পরীকষা ককের আসকে এইরকম একজন হিরউকেবন কযাসটাং সাকেকেবর কা আমরা জানকেবাhelliphellip

Hindi 2nd

langमतर किनमनलिलखिखतपरशनोउRरदीजि0ए

) बढ वयलि` बचच ो कया हआ था ख) डॉकटर साहबन पाटc किस उददशय स रखी थी ग) ाल साप ो हाथ म लर लाश न कया किया घ) डॉकटर चडढा न बढ पतरो दखन स कयो मना र दिदया था ङ) भगत न लाश ो दखर कया हा

उRर ndash) उस बहत बखार थी और 4 दिदनो स आख भी नही खोला थाख) उन बट ी सालकिगरह थीग) ाल सापो हाथ म लर लाश न उसी गदन 0ोर स दबार पडी थीघ) डॉकटर चडढा न बढ वयलि` पतरो दखन स मना र दिदया कयोकि उनह गोलफ खलन 0ाना थाङ) लाश ो दखर हा कि नारायण चाहग तो आध घट म भया उठ 0ाएग

English literature

In the bazaars of Hyderabad- Sarojini Naidu

Through the poem In The Bazaars of Hyderabad Sarojini wanted to convey the message that India is rich in tradition and they donrsquot need the foreign products So she goes on to give a picture of a bazaar where traditional Indian products are rulingThe poem is in the form of questions and answers The poet asks the questions and the merchants answer them Through this technique she make the picture of the bazaar visible to us

Read the poem

PHYSICS FORCE Types of FrictionThere are three types of friction static sliding rolling Static sliding and rolling friction occur between solid surfaces

1 Static Friction The frictional force that acts between the surfaces when they are at rest with respect to each other is called Static FrictionStatic Friction Examples

Skiing against the snow Creating heat by rubbing both the hands

together Table lamp resting on the table

2 Sliding Friction The resistance that is created between any two objects when they are sliding against each other is called Sliding FrictionExamples Of Sliding Friction

Sliding of the block across the floor Two cards sliding against each other in a

deck

3 Rolling Friction The force which resists the motion of a ball or wheel is called Rolling Friction Is the weakest types of frictionExamples Of Rolling Friction

Rolling of the log on the ground Wheels of the moving vehicles

6What effect can a force produce on a body which is not allowed to move Ans - When a force is applied on a body which is not free to move it gets deformed i e the shape or size of the body changes7Give one example each to indicate that the application of a force

1 produces motion2 stops motion3 slows down motion4 changes the direction of motion5 deforms a body

Ans- 1 A car originally at rest when pushed

begins to move2 A moving bicycle is stopped by

applying the brakes3 The speed of a moving vehicle is

slowed down by applying the brakes4 A player kicks a moving football to

change its direction of motion5 On stretching a rubber string its

length increases

8State the effect produced by a force in the following cases (a) The sling of a rubber catapult is stretched(b) A man pushes a heavy cart(c) A player uses his stick to deflect the ball (d) A cyclist applies brakes(e) A spring is compressedAns- (a) The shape and size of catapult changes ie its length increases(b) The heavy cart begins to move(c) The direction of the ball changes(d) The speed of the moving cycle is slowed down(e) There is change in size and shape of spring

COMPUTER MS EXCEL 2013 -INTRODUCTION

UNDERSTANDING EXCEL STRUCTUREA SPREADSHEET IS A FILE THAT EXISTS OF CELLS IN ROWS AND COLUMNS AND CAN HELP ARRANGE CALCULATE AND SORT DATA DATA IN A SPREADSHEET CAN BE NUMERIC VALUES AS WELL AS TEXT

FORMULAS REFERENCES AND FUNCTIONS

WORKSHEETA WORKSHEET IS ALSO KNOWN AS SPREADSHEETIT IS A COLLECTION OF CELLS ON A SINGLE SHEET WHERE YOU KEEP AND CHANGE DATA

WORKBOOKWORKBOOK IS PMS EXCEL FILE IN WHICH THE DATA CAN BE STORED EACH WORKBOOK CAN CONTAIN MANY WORKSHEETS

ROWS AND COLUMNSIN MS EXCEL A ROW IS A GROUP OF CELLS THAT RUN FROM LEFT TO RIGHT OF A PAGEA COLUMN IS A GROUPING OF CELLS THAT RUN FROM THE TOP TO THE BOTTOM OF A PAGE

CELLTHE INTERSECTION POINT BETWEEN A ROW AND THE COLUMN IS CALLED A CELL WHICH IS THE BASIC STORAGE UNIT FOR DATA IN A SPREADSHEET EACH CELL HAS SPECIFIC ADDRESS WHICH IS THE COMBINATION OF THE COLUMN NAME FOLLOWED BY THE ROW NUMBER

CHEMISTRY Chapter ndash Common Laboratory Apparatus and equipments

Objective type questionFill in the blanks (a) Experiment and observation are the two important basics of chemistry(b) A porcelain dish is used for evaporation(c) A test tube holder is used to hold the test tube while-it is heated(d) Mortar and pestle is used for grinding and crushing solid substances into a powder(e) Glass apparatus is made of Pyrex or borosil glass

Class VIISubject Topic Summary Execution

Hindi 2ndlang

ए था राम( डॉ शरी परसाद)

सगकित ा परभाव मानव 0ीवन पर अवशय पडता ह

हमशा मनषय ो अचछो ी सगकित म रहना चाकिहए

शरषठ परो सग स मनषय चरिरतर ा शीघर ही उदय और किवास हो 0ाता

ह इसलिलए वयलि` ो सदा शरषठ परो ा ही सग रना चाकिहए

इसान अगर चाह वह सवय ो बदल भी सता ह

यह हानी राम ए बचच ी हवह गणिणत ी परीकषा म नल रत हए पडा 0ाता ह और उस अधयाप पडत ह और पछत ह यह कया र रह

हो तभी राम न उनी बइजजती ी

शबदाथब ndashहावा भलावाायवाहीndash ाम किनयम व ानन

ो दिदखानापरिरलिचतndash 0ाना पहचानाघटनाndashघबराहट

उलटा चोर ोतवाल ो डाटndashकिववndash भल बर ा जञानतवयndash म 0ो रना चाकिहएसगकितndash बरी सगत

किबलख नाndashरोना किनशचय रनाndash तय रना

फलndashपरिरणामकिनषालिसतndash बाहर किया हआपशचातापndashदख सपननndashधनी

ldquo हा आपी किहममत स हई नल रत पडन ीrdquo ऐसी बात ही किफर

किपता0ी न भी उस डाटा वह ाफी पशचाताप रन लगा बोला गलत दोसतो

ी सगकित म आ0 कितना अनथ र दिदया किफर उसन अधयाप स माफी मागन ी सची और किफर भी ऐसा

नही रगा यह परण भी लिलया

सोचndashकिहच एात-अला

বইndashবাংলা সাহিতয পহিরচয়

পাঠndash১৬লপndashস-য ও অস-যযলখকndashঈশবরচনদর হিবযাসারঅনশীলনীর পরকে4াততর

৬ অGকেলকেখা -ময়া = পশ হিশকার সহিtহিত = হিনকIবতu সbভরষট = লI হিনরীকষণ =

-াকেলা-াকেব যখাকতাঞজহিলপকেI = যজাাকেত৭ হিবপরীতশবদ -ঈষৎ times পরচর উৎকষট times হিনকষট তাশ times উৎফd তবহিদধ times

বহিদধীNপাহিপষঠ times পণযবান৮ পপহিরবতG ন ককেরা -পশ = পাশহিবক যকাপ = যকাহিপতহিসথর = হিসথরতাএকানত = ঐকাহিনতক পর-াত times পর-াতী

CHEMISTRY

Chapter ndashPhysical and Chemical Changes

Chemical ChangeA chemical change involves a change in chemical composition

Characteristics of Chemical changes 1 They are permanent changes2 They are irreversible changes 3 New substance formed4 A Chemical change involves a

change in its chemical properties

Pg-25Question 8What do you observe when1 water is boiled2 a piece of paper is burnt3 some ice cubes are kept in a glass tumbler4 solid ammonium chloride is heated5 an iron nail is kept in tap water for few days6 a spoon of sugar is heated in a pan7 lighted match stick is brought near the mouth of the test tube containing hydrogen gas8 quick lime is dissolved in water9 little amount of curd is added to a bowl containing warm milk and kept for five hours

10 Water is boiledOn boiling water changes into steam (gas) physical change

11 A piece of paper is burnton burning piece of paper produces carbon dioxide and ash is left behind Is a chemical change

12 some ice cubes are kept in a glass tumblerIce cubes (solid) turn into water

(liquid) only state changes (physical change)

13 Solid ammonium chloride is heatedSolid ammonium chloride on heating changes into vapors (change of state) is physical change

14 An iron nail is kept in tap water for few dayswe observe reddish brown coating on the nail called rust (entirely new substance) is chemical change

15 A spoon of sugar is heated in a panWhen a spoon of sugar is heated in a pan black (charred sugar) (carbon) is seen Is a chemical change

16 Lighted match stick is brought near the mouth of the test tube containing hydrogen gasWe observe that hydrogen bums at the mouth of test tube with blue flame and pop sound is heard It is chemical change

17 Quick lime is dissolved in waterThe following two observations will be observed (i) A hissing sound is observed(ii) The mixture starts boiling and lime water is obtained

18 Little amount of curd is added to a bowl containing warm milk and kept for five hoursWhen a little amount curd is added to a bowl containing warm milk and kept for five hours a permanent change occurredThe milk will change to curd On boiling water changes into steam (gas) physical change

GEOGRAPHY

ATMOSPHERE IMPACT OF GLOBAL WARMING The destructive impart of global warming is observed in various spheres of life and the environment Some of the points are outlined below1 High temperatures lead to high

evaporation rate and drying up of the soil and surface water This affects crop production The occurrence of droughts is aggravating the problem even further

2 The heat waves in summer months

Q1 Write some impact of global warmingA1 The impacts of global warming are as follows1 High temperatures lead to high

evaporate ion rate and drying up of the soil and surface water This affects crop production The occurrence of droughts is aggravating the problem even further

2 The heat waves in summer months lead to a greater number

lead to a greater number of deaths due to heat strokes

3 Forest fires become more frequent4 Tropical cyclones and hurricanes

become common5 Melting of glaciers takes place6 Polar ice caps are becoming thinner

and melting at an alarming rate due to global warming The loss of sea ice

7 Due to increase in sea surface temperature sea levels rise in coastal areas and cause submergence of several islands

WAYS TO REDUCE GLOBAL WARMINGFollowing steps can be taken We need to decrease emission of

green house gases by reducing the burning of fossil fuel such as coal and petroleum

By planting more trees to increase forest cover

The government should also distributes free saplings and organize afforestation programmes to spread awareness regarding the beneficial effects of trees

We should switch to eco-friendly cars and gadgets

Incandescent light bulbs should be replaced by CFL bulbs

We can save electricity and reduce global warming by turning off electrical gadgets such as lights fans air-conditioners television and computer when we do not to use them

Efforts should be made to hasten the development of green cities oreco cities These cities are urban areas around the world striving to lessen the environment a impacts of urbanization

By following the 3Rs-Reduce Recycle and Reuse strategy we can use natural resources for our growth as well as save them for the need of the future generations This is called sustainable development

of deaths due to heat strokes3 Forest fires become more

frequent4 Tropical cyclones and hurricanes

become common5 Melting of glaciers takes place

etc

Q2 How to reduce global warmingA2 Following steps can be taken to reduce global warmingaWe need to decrease emission of

green house gases by reducing the burning of fossil fuel such as coal and petroleum

bBy planting more trees to increase forest cover

c The government should also distributes free saplings and organize afforestation programmes to spread awareness regarding the beneficial effects of trees

dWe should witch to eco-friendly cars and gadgets

eIncandescent light bulbs should be replaced by CFL bulbs

f We can save electricity and reduce global warming by turning off electrical gadgets such as lights fans air-conditioners television and computer when we do not to use them

Q3 What do you mean by 3Rrsquos of resource planningA3 The 3Rs are

1 Reduce 2 Recycle and3 Reuse

Q4 What is Sustainable developmentA4 By following the 3Rs-Reluce Recycle and Reuse strategy we can use natural resources for our growth as well as save them for the need of the future generations This is called sustainable development

English Language

Prepositions A preposition is a word placed before a noun or a pronoun It helps to show how the person or thing denoted by the noun is related to something else in the sentence

Kinds of Prepositions

Simple Prepositions- simple preposition are one word Prepositions such as at by for in of off for from on out through till to up with before amidst towards beyond between over etc

Compound Prepositions ndash There are some words that are always used with fixed Prepositions to convey specific meaning

Example I was unable to meet you dueto a previous engagement ( On account of)Always maintain the queue instead of crowding at the counter ( In place of)

Participial PrepositionsmdashParticiple Prepositions are present or past participles of various verbs which together with a noun phrase or a clause function as prepositions Examples- barring concerning considering notwithstanding pending regarding respecting etc

Exercise A

1 Gauravs fever has come down since Friday He has been absent for a week now

2 The child sat between his father and mother among the parents of all his classmates

3 There are mosquitoes in the room They flew into the room when the door was open

4 My father was inside the drawing room when I was playing outside my house

5 You may sit beside me I will give you a drawing book and pencils besides a storybook

6 We went to the market in the morning and walked towards the riverfront in the evening

7 The child walked along the pavement and across the street safely

8 This table top is made of glass My breakfast fell off it in the morning

9 The pan is on the gas stove There are vegetables in it

10 We will wait for you at the bus top There are a lot of people in the hall

Subject ndash Biology Topic ndash Chapter - 3 Photosynthesis and respiration in plants Summary Execution

All living organism (Plants and animals) need food for energy and growth Green plants (autotrophy) prepare food for all living organisms Today we will discuss about the process photosynthesis And adaptations in a leaf to carry out photosynthesis

Q1What do you mean by photosynthesis and write its word equation The process by which green plants make food (glucose) from carbon dioxide and water

in the presence of sunlight and chlorophyll is called photosynthesis

Carbon dioxide + Water ( Sun light from Sun ) Glucose + Oxygen ( chlorophyll in green leaves )

Q2 What are the adaptations in a leaf to carry out photosynthesisi) Leaves are broad wide and flat for absorbing more light energyii) Presence of chlorophyll in chloroplasts to trap sunlightiii) Presence of stomata which allow carbon dioxide to enter the cell and oxygen to go

out iv) Network of veins ensures continuous supply of water and minerals to the leafv) Thin waxy cuticle protects the leaf without blocking the lightQ3 Draw and label structure of chloroplast

Class VIIISubject Topic Summary Execution

PHYSICS ENERGY Production of Hydro electricity

A hydroelectric dam converts the potential energy stored in a water reservoir behind a dam to mechanical energymdashmechanical energy is also known as kinetic energy As the water flows down through the dam its kinetic energy is used to turn a turbine

The generator converts the turbinersquos mechanical energy into electricity

This electric energy then goes through various transmission processes before it reaches you

Question 2

Fill in the blanks

(a) Work is said to be done by a forte only when the body moves

(b) Work done = Force x distance moved in direction of force

(c) The energy of a body is its capacity to do work

(d) The SI unit of energy is joule

(e) The potential energy is due to its state rest of position and kinetic energy of the body is due to its state of motion

(f) Gravitational potential energy U = mass times force of gravity on unit mass times height

(g) Kinetic energy = frac12 times mass times (speed)2

(h) Power P = work donetime taken

(i) The S I unit of power is watt

(j) IHP = 746 W

BIOLOGY Chapter -5 The endocrine system and adolescence

Today we will discuss about thelocation and functions of secreted hormones of adrenal and Pancreas

Q5 Write location hormone secreted main functions and deficiency diseases of pancreas and adrenal glands

Endocrine Glands

Location Hormones secreted

Functions and Deficiency Diseases

1Adrenal gland

2 Pancreas Gland

On the top of each kidney

In between stomach and small intestine

i)Adrenaline from adrenal medulla

ii)Cortisone from adrenal cortex

i) Insulin

ii) Glucagon

It helps a person deal with any kind of emergency situation or emotional stressIt increases the heart beat rate of respiration and blood pressure

a) It regulates carbohydrates protein and fat metabolism

b) It regulates the salt and water balance in the body

a) It changes excess glucose into glycogen

b) It stimulates the cells to burn extra glucose to provide heat amp energy

Less secretion causes diabetes mellitus

Excessive secretions causeinsulin shock

a) It stimulates the breakdown of glycogen into glucose

b) It increases the level of glucose in blood

History Traders to rulers The Battle of Buxar was fought on 22 October 1764 between the forces under the command of the British East India Company led by Hector Munro and the combined armies of Mir Qasim the Nawab of Bengal till 1763 Mir Jafar was made the Nawab of Bengal for a second time in 1763 by the Company just after the battle After being defeated in 4 battles in katwa and Udaynala the Nawab of Awadh Siraj id Daula and the Mughal emperor Shah Alam II accompanied by Raja Balwant Singh of Kashi made an alliance with Mir Qasim The battle was fought at Buxar a small fortified

Answer the following questions- Short note-Battle of BuxarHomework-learn

town within the territory of Bihar located on the banks of the Ganga river about 130 kilometres (81 mi) west of Patna it was a decisive victory for the British East India Company The war was brought to an end by the Treaty of Allahabad in 1765

EnglishLiterature

The west wind-John Mansfield

In the poem The West Wind by John Masefield the poet starts by describingwith very poetic imagery of birds how the west wind is different from other winds its a warm wind full of birds cries There is a touch of melancholy perhaps home-sickness as he describes how it brings tears too and memories from an old land He goes on to describe the restful pastoral beauty of the land where even the dead can lie in the green He then brings in voicesperhaps of family and friends calling him home as he is missing Aprils beautyThe voices then tempt him some more with idyllic images from home (white blossom young green cornrunning rabbitswarm sun) The voices seem to presume that the poets heart is sorrowful bruised and soreThe end of the poem sees the poet appear to make a decision he will go home as he has decided that is where he truly belongs

Write the synopsis of the following words

1 Daffodils- a tall yellow flower that grows in the spring

2 Orchards- a piece of land on which fruit trees are grown

3 Blossom- a flower or a mass of flowers especially on a fruit tree in spring

4 Thrushes- a bird5 Larks- a small brown bird that

makes a pleasant sound6 Bruised- an injury7 Aching- pain 8 Tread- to put your foot down

while you are walking9 Balm-10 May-11 Fluting-

(Write from the book in your copy)

MAT

HEM

ATIC

S

Ch 1

1Al

gebr

ic E

xpre

ssio

n

1 Constant A symbol which has fixed value is called a constant[eg 8 23 -15 radic3 etc]

2 VariableA symbol which does not have any fixed value but may be assigned value (values) according to the requirement is called variable or literal[eg x y p q etc]

3 TermsA term is a number (constant) a variable a combination (product or quotient) of numbers and variables[eg 7 x 5x etc]

4 Algebric expressionA single term or acombination of two or more terms connected by plus (+) or minus (-) sign forms an algebraic expression[eg 5-y 3x2-5x xy-6z+4 etc]

5 PolynomialAn algebraic expression which contains more than one term is called a polynomial (multinomial)[eg x2-5x 5y+xy+x2y etc]

6 Degree of polynomial(a) When the polynomial contains only one variable the highest power of the variable is the degree of the polynomialeg the degree of the polynomial of 4x-7x5+8 is 5(b) When the polynomial contains two or more variablesStep (i) Find the powers of the variables in each term (ii) The highest sum of the powers is taken to be the degree of the polynomialeg the degree of the polynomial 5x2y-4x3y5+6 is = 3+5 = 8Remember An algebraic expression is a polynomial if degree of each term used in it is a non-negative integer

Exercise ndash 11(A)

1 Separate the constants and variables from the following

-7 7+x 7x+yz radic5 radic xy 3 yz

8 45y -3x

Solution Constant Variables-7 radic5 7+x 7x+yz radic xy

3 yz8

45y -3x

2 Write the number of terms in each of the following polynomials(i) 5x2+3timesax (ii) axdivide4-7 (iii) ax-by+ytimesz (iv) 23+atimesbdivide2

Solution Polynomials Number of terms(i) 5x2+3timesax 2(ii) axdivide4-7 2(iii) ax-by+ytimesz 3(iv) 23+atimesbdivide2 2

4 Write the degree of the each polynomials(i) xy+7z (ii) x2-6x3+8 (iii) y-6y2+5y8 (iv) xyz-3 (vi) x5y7-8x3y8+10x4y4z4

Solution Polynomials Degree(i) xy+7z 2(ii) x2-6x3+8 3(iii) y-6y2+5y8 8(iv) xyz-3 3(vi)x5y7-8x3y8+10x4y4z4 12

5Write the coefficient of(i) ab in 7abx (iv) 8 in a2-8ax+a (v) 4xy in x2-4xy+y2

SolutionCoefficient

(i) ab in 7abx 7x(iv) 8 in a2-8ax+a -ax(v) 4xy in x2-4xy+y2 -1

7 CoefficientAny factor of an algebraic quantity is called the coefficient of the remaining quantityeg in the algebraic term 7xyz 7 is coefficient of xyz 7x is coefficient of yz and so on

8 Like term The terms having the same literal coefficient are called like terms and those having different literal coefficients are called unlike terms

eg (i) 5xyz 8xyz -6xyz and 23xyz are like

terms(ii) 7xy2 8x2yz and -15xyz2 are unlike terms

6 in 57xy2z3 write the coefficient of

(i) 5 (vii) 5xy2 (viii) 17yz (xi) 5xyz

Solution Coefficient

(i) 5 17

xy2z3

(vii) 5xy2 17z3

(viii) 17yz

5xyzsup2

(xi) 5xyz 17yz2

7 In polynomial given below separate the like terms(ii) y2z3 xy2z3 -58x2yz -4y2z3 -8xz3y2 3x2yz and 2z3y2

Solution y2z3 -4y2z3 2z3y2 are like terms

xy2z3 -8xz3y2 are like terms

-58x2yz 3x2yz are like terms

Class IXSubject Topic Summary Execution

Bengali (2nd language)

বাগzwnjধারাzwnj বা ধারা-বা ধারা ল হিবকেশষ পরকার বাক -হিb -াকেবর এক হিবকেশষ পরকাশরীহিত াকেক কতগকেলা কার সমষটির মকেধয এগহিলকেক বা ধারা বকেল আবার কতগকেলা শকেবদর বাধাধরা যকান রীহিত যনই য-াকেব চকেল আসকে যসই -াকেবই চকেল আসকে তখন যসই শবদগহিল খন একক -াকেব অG পরকাশ ককের তখন একের বা ধারা বকেল বা ধারার পরকেয়া -াষাকেক আরও সFর ককের যতাকেল

অকাল পকক(অপহিরনত বয়কেস পাকাহিম)-মাতর শ বর বয়কেস যমকেয়টির া মকেখর কা তাকেত অকালপককতা ধরা পকে

অককা পাওয়া( মারা াওয়া) ndash পকেকIমারটি পকেকIমারকেত হিকেয় বাসাতরীকের াকেত মার যখকেত যখকেত অককা যপল

অহি| পরীকষা ( কঠিন ও পরকত পরীকষা)- যকেলটির আজ ডাকতাহির যরজালট যবকেরাকেব এIাই তার জীবকেনর ব অহি| পরীকষা

অষটরমভা (ফাহিক) ndash রীতা মকেখই বকো বকো কা বকেল আর কাকেজর যবলায় অষটরমভা

অকমGার ধাী (অপাG) ndash সমনকেক হিনকেয় যকান ান কেব না ও একেকবাকেরই অকমGার ধাী

অকেনধর ষটি (অসাকেয়র সায়)- আহিশ বকেরর বকোর নাহিত ল অকেনধর ষটি তাকেক াা বকোর একম চকেল না

আকেককল গড়ম (তবহিদধ)- ার তহিম উপকার করকেল যসই যতামার হিবরকেদধ সাকষয হিকেয়কে শকেনই আমার আকেককল গড়ম

আষাকে লপ( অবাসতব লপ) ndashIাকা এখন যকেব না এIা বলকেলই ত এমন আষাকে লপ ফাার যকান রকার হিল না

Hindi- महायजञ ा इस हानी म लख न या बतान ा परयास किया ह कि किसी भी अचछ

2nd language

परसार(यशपाल ाय या पणय न ा फल अवशय मिमलता ह ोई भी परोपार अथवा पणय लिलए किया गया ाय बार नही 0ाता वह ए परार ा यजञ हए धनी सठ थ धम परायण और किवनमर सठ न आन ी यजञ किए थ और दान म न 0ान कितना धन दिदन दखिखयो म बात दिदया थादिदन पलट और सठ यहा गरीबी आ गई उन दिदनो यजञ बचन ी परथा थी सठ भी अपनी 0गह बचन लिलए डलपर ए सट यहा चलन ो तयार हए सठानी रासत लिलए रोटी पड म बाधर सठ ो द दी रासत म ए भख R ो दखर सठ न चारो रोटी उसो खिखला दी खर वह सठ यहा डलपर पहच तो उनी सठानी न उस महायजञ बचन ो हा यदिद बचन आए सठ न R ो रोटी खिखलान ो महायजञ नही समझा और वापस लौट आया घर आर शाम ो उसी घर म उस ए बडा ख0ाना मिमला 0ो उस दवारा किए गएrsquo महायजञrsquo ा परसार था

English language

Letter formal The heading the name and address of the person you are writing to must be included beneath your own address In formal letters ldquoblock stylerdquo of address is preferred

Subject complain in brief

Salutation If the person you are writing to is known to you you may begin ldquoDear MrrdquoOr ldquoDear Mrsrdquo In all other instances you should begin ldquoDear Sirrdquo or ldquoDear Madamrdquo Or ldquoSirsrdquo

The body A formal or business letter has four partsReference The letter should begin by referring to a letter you have received an advertisement or the reason that has prompted you to writeInformation In the second paragraph it is necessary to supply more detailed information that is related to the referencePurpose Here you must give the reason why you are writing the letter This must be stated clearly and ensure that it is relevant to the question that has been setConclusion round off the letter with some polite remarkThe subscription when a letter has begun with dear sir sirs Madam you should end with Yours faithfully or yours truly When however you address a person by name you must conclude with the words ldquoYours sincerelyrdquo

1 A park in your locality is slowly being used as a rubbish dump Write a letter to the Mayor of your city pointing out the nuisance and danger of this Request that action be taken to stop this immediately

Or2 You being a boarder ordered a set of lab manuals from a famous book shop in the town They sent you a wrong set of books Write a letter to the manager of the book shop

Chemistry Chapter-1 1)CHEMICAL FORMULA- Q What is the Significance of

L-2The Language of Chemistrybull Chemical Formula

Itrsquos a symbolic representation of a chemical substance eg ndash The formula of Sulphuric acid is H2SO4

2) Steps of writing Chemical Formula of a given substance-

1 Write the symbols of the constituent atoms or radicals side by side Keep the basic radical on LHS and acid radical on the RHS ( Na+Cl- )2 In case of a radical having more than one atom( compound radical) enclose the radical in a bracket eg (SO4-)3 Write the valencies of each radical on its right hand top4 If the valencies of the two radicals are divisible by a common factor then divide the valencies by the common factor5 Invert (criss-cross) the valency number ie write the valency of one atom below the second atom and vice versa 6 On interchanging if valency number is lsquoone the figure lsquoonersquo is never writtenFor Example- Compound -Calcium Nitrate1 Writing the symbols- Ca(NO3)2 Writing the valencies on their right hand top- Ca2(NO3)1

3 Valency numeral in simple ratio- Ca2(NO3)1

4 Criss-cross- Ca 2NO3 1

5 Writing the formula of the compound- Ca(NO3)2

Chemical formula

A The formula of a substance conveys the following information regarding a substance 1 The name of the substance (qualitative)2 The elements constituting the substance (qualitative)3 The number of various atoms present in a molecule of the substance (quantitative)4 Molecular weight of the substance and the relative weights of different elements present in it (qualitative)

Q What are the limitations of Chemical Formula

A The chemical formula suffers from the following limitations-I It fails to convey whether the elements in a molecule are present in the form of atoms or ionsFor example the formula KBr fails to tell us whether Potassium and Bromine are present in the form of ions II It does not tell anything about the binding force that holds atom in a molecule togetherIII It does not tell us about the arrangement of various atoms with respect to one another within the molecule

Q Examples of Some Chemicals with their Formula Chemical name and Common Name-

A Given in the class notesCommercial Studies

Joint Stock Company

Let us discuss about the demerits of Joint Stock CompanyDespite so many advantages it has got many disadvantages which are as follows

Difficulty in FormationDelay in Decision makingExcessive Government ControlLack of Secrecy

Company can be classified into several categories based on incorporation

QuestionExplain the demerits of Joint Stock CompanyAnswer) 1 Difficulty in Formation The legal requirements and formalities required to be completed are so many The cost involved is quite heavy It has to approach large number of people for its capital It cannot start its business unless certificate of incorporation has been obtained This is granted after a long time when all the formalities are completed

Chartered CompanyStatutory CompanyRegistered Company

Delay in Decision making In this form of organization decisions are not made by single individual All important decisions are taken by the Board of Directors Decision-making process is time-consuming So many opportunities may be costly because of delay in decision-making Promptness of decisions which is a common feature of sole trader ship and partnership is not found in a company

Excessive Government ControlA company and the management have to function well within the law and the provisions of Companies Act are quite elaborate and complex At every step it is necessary to comply with its provisions lest the company and the management should be penalized The penalties are quite heavy and in several cases officers in default can be punished with imprisonment This hampers the proper functioning of the company

Lack of Secrecy The management of companies remains in the hands of many persons Every important thing is discussed in the meetings of Board of Directors Hence secrets of the business cannot be maintained In case of sole proprietorship and partnership forms of organisation such secrecy is possible because a few persons are involved in the management

2 Define the following

Chartered Company- The crown in exercise of the royal prerogative has power to create a corporation by the grant of a charter to persons assenting to be incorporated Such companies or corporations are known as chartered companies Examples of this type of companies are Bank of England (1694) East India Company (1600) The powers and the nature of business of a chartered company are defined by the charter which incorporates it After the country attained independence these types of companies do not exist

in IndiaStatutory Company- A company may be incorporated by means of a special Act of the Parliament or any state legislature Such companies are called statutory companies Instances of statutory companies in India are Reserve Bank of India the Life Insurance Corporation of India the Food Corporation of India etc The provisions of the Companies Act 1956 apply to statutory companies except where the said provisions are inconsistent with the provisions of the Act creating them Statutory companies are mostly invested with compulsory powersRegistered companiesCompanies registered under the Companies Act 1956 or earlier Companies Acts are called registered companies Such companies come into existence when they are registered under the Companies Act and a certificate of incorporation is granted to them by the Registrar

Economics

Chapter-4Basic problems of Economy

Today let us discuss with the topic Production Possibility curve

QuestionExplain the concept of Production Possibility Curve with the help of diagram

Answer) Production Possibility curve is a locus of all possible combinations of two commodities which can be produced in a country with its given resources and technology

The above diagram shows that with the given resources and technology the economy can produce maximum either 5 thousand meters of cloth or 15 thousand quintals of wheat or any other combination of the two goods like B( 1 thousand meters of cloth and 14 thousand quintals of wheat C ( 2 thousands meters of cloth and 12 thousand quintals of wheat) etcProduction Possibility curve is also called production possibility boundary or frontier as it sets the maximum limit of what it is possible to produce with given resources

Geography

Rotationand Revolution

SUNrsquoS POSITION AND SEASONAL CHANGES EQUINOXES ndash SPRING AND AUTUMN

Q1 What is Spring EquinoxA1 On 21st March sunrays fall directly on the equator On that day

As the Equator divides the Earth into two equal halves the sun rays fall directly on the equator twice in a year Equinoxes means equal Spring EquinoxOn 21st March sunrays fall directly on the equator On that day the duration of day and night both are equal ( 12 hours day and 12 hours night) on every places located on equator This day is called as Spring EquinoxAutumn EquinoxOn 23rd September sunrays fall directly on the equator On that day the duration of day and night both are equal ( 12 hours day and 12 hours night) on every places located on equator This day is called as Autumn Equinox

SOLSTICES ndash SUMMER AND WINTERDue to inclination of the Earth on its axis and the apparent movement of the sun the sun rays fall directly on both tropics once in a year Solstice is a Latin word which mean ldquothe Sun standing stillrdquoSummer SolsticesAfter 21st March there is an apparent movement of the Sun to the north of the equator The apparent northward movement up to 21st June when the Sun appears overhead at the Tropic of Cancer (22frac12degN) The sun appears to stand still at this position and then moves southwards towards the equator This position of the Sun on 21st June is known as Summer Solstices On that day the duration of day and night both are equal ( 12 hours day and 12 hours night) on every places located on Tropic of Cancer (22frac12degN)Winter solstices The apparent southward movement of the Sun continues beyond the equator till 22nd

December On this day the Sun is overhead at the Tropic of Capricorn

the duration of day and night both are equal ( 12 hours day and 12 hours night) on every places located on equator This day is called as Spring Equinox

Q2 What do you mean by EquinoxA2 Equinoxes means equal It is use to explain the equal duration of day and night ( 12 hours day and 12 hours night) on the Earth

Q3 On which date the longest day in Tropic of CancerA3 21st June

Q4 What is the meaning of SolsticeA4 Solstice is a Latin word which mean ldquothe Sun standing stillrdquo

Q5 Which is the longest day in southern hemisphereA5 22nd December

Q6 On what date does the Arctic Circle experience the lsquoMidnight SunrsquoA6 On 21 June the Arctic Circle experiences the lsquoMidnight Sunrsquo

Q7 What is cause of Midnight Sun in NorwayA7 During the summer solstice (21 June) the North Pole is inclined towards the Sun Therefore the duration of sunlight or daytime increases from 12 hours at the Equator to 24 hours at the Arctic Circle and beyond Thatrsquos why The region beyond the Arctic Circle especially Norway is known as the Land of the Midnight Sun because there the Sun does not rise or set on 21 June

Q8 Match the column A with BA B

Summer Solstice 21st March

Autumn Equinox 23rd

September

Winter Solstice 21st June

(22frac12degS) This position of the Sun is referred to as the Winter Solstice because it marks the winter season in the Northern Hemisphere On that day the duration of day and night both are equal ( 12 hours day and 12 hours night) on every places located on Tropic of Capricorn (22frac12degS)SEASONS AND DURATION OF DAY AND NIGHT During the equinoxes all places on the Earth have 12 hours of day and 12 hours of night Due to the revolution of the Earth round the Sun on an inclined axis the duration of day and night varies according to seasons and the latitude of a placeDuring the summer solstice (21 June) the North Pole is inclined towards the Sun Therefore the duration of sunlight or daytime increases from 12 hours at the Equator to 24 hours at the Arctic Circle and beyondThe region beyond the Arctic Circle especially Norway is known as the Land of the Midnight Sun because there the Sun does not rise or set on 21 JuneAt the North Pole there will be six months of daylight The Sun will be seen always above the horizon at a low angle At 66degN 24 hours of sunlight can be seen only on 21 June Hammerfest in northern Norway is a place of tourist attraction for observing the phenomenon of the Midnight Sun This place has continuous daylight from 13 May to 29 July This place is easily accessible to tourists and has hotels and other facilities The view of the midnight Sun from here is enthrallingIn the Southern Hemisphere the duration of daylight decreases from 12 hours at the equator to 0 hours beyond the Antarctic Circle In the South Polar Region there is 24 hours of darkness The Sun is always below the horizon In the Southern Hemisphere which experiences winter the duration of night-time is longer than the duration of daylight

Spring Equinox 22nd

December

A8 A B

Summer Solstice 21st June

Autumn Equinox 23rd

September

Winter Solstice 22nd

December

Spring Equinox 21st March

During winter solstice (22 December) the South Pole is inclined towards the Sun The Southern Hemisphere experiences summer and the Northern Hemisphere has winter Therefore the duration of daylight or sunlight is greater in the Southern Hemisphere than in the Northern HemisphereThe duration of daylight increases from 12 hours at the equator to 24 hours beyond the Antarctic Circle The South Polar Region has 24 hours of sunlight for many days continuously At the South Pole there will be six months of sunlight The Sun will always be seen at a low angle above the horizon In the Northern Hemisphere the duration of daylight will decrease from 12 hours at the equator to 0 hours at the Arctic Circle There are 24 hours of darkness in the North Polar region The duration of night is greater than the duration of daylight as one move northwards from the Equator It is evident from the above table that the duration of daylight is 12 hours throughout the year at the equator only As one moves away from the equator the seasonal variations in the duration of daylight increase The seasonal variations in the duration of daylight are maximum at the Polar Regions

Subject Eng Literature (The Merchant of Venice ndash William Shakespeare)Topic Act II Scene 7 Lines 36 to 80 (End of scene ) [Students should read the original play and also the paraphrase provided]

Summary Questions amp AnswersThe Prince then examines the inscription on the silver casket which says ldquoWho chooseth me shall get as much as he deservesrdquo The Prince says that he deserves Portia more than anybody else because of his high rank his noble birth and his great wealth and power But then he argues that silver is ten times

(1) (Act II Sc 7 L 39-47)

From the four corners of the earth they come

To kiss this shrine this mortal breathing saint

The Hyrcanian deserts and the vasty wildsOf wide Arabia are as through-fares now

inferior to gold and therefore he cannot believe that the portrait of such a beautiful lady as Portia can be contained in the silver casket He decides to see the inscription on the golden casket before making his decision

The Prince goes to examine the inscription on the golden casket which says ldquoWho chooseth me shall get what many men desirerdquo The Prince believes that the whole world desires to possess Portia otherwise so many suitors would not have come from all corners of the world for winning Portia Some of them have come from the distant lands of Persia and Arabia The deserts of Persia (Hyrcanian deserts) and the boundless desolate lands of Arabia have been crossed by the Princes seeking the hand of Portia He contrasts this casket containing Portiarsquos portrait with the old English gold coin bearing the image of the archangel (angel of the highest rank) He goes on to remark that while the figure of the archangel is engraved (Insculped) upon the English coin the picture of Portia who is beautiful as an angel lies hidden inside one of the caskets namely the Golden Casket (Golden Bed)

On the basis of his assessment of the inscription on the golden casket the Prince decides to choose the golden casket He asks for the key and opens the golden casket only to find therein an empty human skull holding a roll of

For princes to come view fair PortiaThe watery kingdom whose ambitious headSpets in the face of heaven is no barTo stop the foreign spirits but they comeAs orsquoer a brook to see fair Portia

(i) Explain the occasion for the above mentioned speech

These are the comments of the Prince of Morocco after he reads the inscription on the golden casket His mental process is revealed to us in these words We find him debating within himself as to which casket he should choose

(ii) What light does the above speech throw on the personality of Prince of Morocco

From the above mentioned speech we come to know that the Prince of Morocco is keen to marry Portia He is the type of person who is easily taken away by outward appearance He is in love with Portia because of her beauty

(iii) What information can you gather about Portia from the above mentioned lines

The given speech shows that Portia is a very beautiful lady She must be possessed of good qualities because many suitors come to her place from all over the world with a desire to get married to her The Prince of Morocco is so impressed by her beauty that he calls her a saint According to him the whole world is desirous of having her

(iv) Elucidate the significance of the first two lines

In these lines the Prince of Morocco pays a compliment to Portia These lines show his admiration for her He says that people come from all parts of the world to see fair Portia

(v) Explain the meaning of the last four lines of the

passage

In these lines the Prince of Morocco says that even the vast oceans which throw a challenge at the sky are unable to prevent men from coming to Portiarsquos place to have a glimpse of her These lines are also a tribute to Portiarsquos beauty and good qualities Many men voyage across the ocean treating it as a mere stream to see the beautiful Portia

paper in which is written that whoever happens to be guided by the glitter of things is invariably deceived

On reading the scroll the Prince says that he is too sad at heart to speak a more formal farewell and leaves with his followers amidst a sound of trumpets

After the Prince of Morocco leaves Portia remarks that the Prince is a gentle fellow but she is rid of him May all persons of his nature make a similar choice

IMPORTANT PASSAGES EXPLAINED

(Act II Sc 7 L 39-43)From the four corners of the earth they come

To kiss this shrine this mortal breathing saintThe Hyrcanian deserts and the vasty wildsOf wide Arabia are as through-fares nowFor princes to come view fair Portia

Context

This passage occurs in Act II Scene 7 in The Merchant of Venice This is part of the speech made by the Prince of Morocco

(2)

(Act II Sc 7 L 48-53)

MOROCCO One of these three contains her heavenly pictureIst like that lead contains her

Twere damnation To think so base a thought it were too grossTo rib her cerecloth in the obscure graveOr shall I think in silver shes immurdBeing ten times undervalued to tried gold

(i) What meaning does the Prince of Morocco find out of the inscription of the golden casket What have Belmont and Portiarsquos house been called and why

The inscription on the golden casket is ldquoWho chooseth me shall gain what many men desirerdquo The Prince finds out that it means that the chooser of the golden casket will get Portia because many men desire her In fact the entire world desires her Because of the coming of many suitors to Belmont from different countries in order to win Portiarsquos hand Belmont has become a centre of pilgrimage and her house is the shrine where saintly Portia is installed

(ii) What does the Prince of Morocco do before making the final choice of the casket Which is the correct casket and who will win Portiarsquos hand

The Prince of Morocco surveys and analyses the inscriptions on the casket of lead silver and gold Before making the final choice like a very systematic and methodical person he once again considers the claims of the caskets The casket containing Portiarsquos picture is the correct casket and the person choosing it will win Portiarsquos hand

Explanation

While praising Portia the Prince of Morocco conceives Portia as a goddess whose image is placed inside one of the caskets Many suitors are coming from far and wide the north and the south the east and the west (Four corners) in order to try their luck Some of them have come from the distant land of Persia and Arabia The deserts of Persia (Hyrcanian deserts) and the boundless desolate lands of Arabia have been crossed by the Princes seeking the hand of Portia All this shows that Portia is indeed the most beautiful lady of the world

(iii) What does the Prince of Morocco say in his estimation while examining the motto on the silver casket What does he find in the golden casket

While examining the motto on the silver casket which says ldquoWho chooseth me shall get as much as he deservesrdquo Morocco says that in his own estimation he surely deserves Portia in all respects ndash rank birth wealth etc

He chooses the golden casket When he opens it he finds an empty human skull holding a scroll in which it is written that those who are attracted by the glittering outside of things are always deceived as Morocco has been deceived

(iv) What kind of nature does the Prince of Morocco have

The Prince of Morocco has a simple nature who does not look deeply into the inner meaning of things but is dazzled by the outward appearance of gold He is inclined to over-estimate his own value and does not realize that it is a duty to ldquogive and hazardrdquo To say that he will not hazard for lead shows that he misreads the true meaning of the inscription which is that he should be prepared to ldquohazard all he hathrdquo for Portia So his feeling is only one of fascination and romantic attraction

(v) Do you think that the lottery of the caskets is not a matter that will be determined by chance

In fact the lottery of the casket is not a matter that will be determined by mere chance but that it is a true test of character and of sincerity which is amply proved not only by Moroccorsquos choice but also by the arguments which he uses to help him in his choice

(Act II Sc 7 L 55-59)

They have in England

A coin that bears the figure of an angelStamped in gold but thats insculpd uponBut here an angel in a golden bedLies all within

Context

(3)

(Act II Sc 7 L 63-77)A carrion Death within whose empty eye

There is a written scroll Ill read the writing

All that glisters is not goldOften have you heard that toldMany a man his life hath soldBut my outside to beholdGilded tombs do worms infoldHad you been as wise as boldYoung in limbs in judgment oldYour answer had not been inscrolld

This passage occurs in Act II Scene 7 in The Merchant of Venice This is part of the speech made by the Prince of Morocco

Explanation

In this passage the Prince of Morocco bestows high praise on Portia whose hand he is seeking He contrasts this casket containing Portiarsquos portrait with the old English gold coin bearing the image of the archangel (angel of the highest rank) He goes on to remark that while the figure of the archangel is engraved (Insculped) upon the English coin the picture of Portia who is beautiful as an angel lies hidden inside one of the caskets namely the Golden Casket (Golden Bed) In the day of Elizabeth silver was ten times inferior in value to gold Therefore the Prince of Morocco believing that Portiarsquos portrait is contained in the Golden Casket decides to choose the Golden Casket

Fare you well your suit is coldCold indeed and labour lostThen farewell heat and welcome frostmdashPortia adieu I have too grievd a heartTo take a tedious leave Thus losers part

(i) What reward does the Prince of Morocco get after making a wrong choice of the Casket How does he feel

After making the wrong choice in selecting the casket of gold the Prince of Morocco as a reward earns a rebuke in the form of a scroll tucked in the empty eye-socket of a skull kept in the casket of gold The Prince is shocked and disappointed He becomes all the more sad and dejected when he reads the scroll which points to his foolishness in being misled by the appearance and outward show as indicative of its worth

(ii) How does the Prince respond after reading the scroll

After reading the scroll the Prince though upset accepts the result with good grace and decorum befitting a royal suitor and true sportsman He says that his love-suit is really cold otherwise he would have chosen correctly but now his efforts have been in vain So he bids farewell to Portia to the warmth and enthusiasm of love and welcomes the cold and bitterness of dejection and misery of life which lies ahead

(iii) What request does he make to Portia and why

After being failure in his mission he requests Portia to give him permission to leave at once because he is too sad to undergo the tediousness of a formal leave-taking He tells that it is the manner in which defeated persons part unceremoniously

(iv) Explain the following lines

ldquoAll that glisters is not goldOften have you heard that toldMany a man his life hath soldBut my outside to beholdGilded tombs do worms infoldrdquo

Mere glitter does not make a metal to be gold Man has often been warned against appearance but it has been of no use Many people have sacrificed their lives only to seek the outer appearance of gold Worms are found inside the gilded

monuments

Class XSubject Topic Summary Execution

Hindi 2ndlang

नया रासता भाग 6 मायाराम 0ी घर म धनी मल 0ी और उनी बटी सरिरता ी ही चचा बनी रहती थी अमिमत ो इसम ोई रलिच ना थी वह धनी घर ी लडी स शादी र सवय ो बचना नही चाहता था उसा भी सवाणिभमान ह ईशवर ी पा

स उस पास पस ी ोई मी नही थी अभी उसन फकटरी ही लगाई थी उसी समझ बाहर था कि उस घर वालो ा झाव पस ी तरफ कयो

ह उसन मा स सवाल किया कि मा तम सरिरता स मरी शादी कयो रना चाहती हो मा न उस समझाया कि वह दखन म बरी नही ह और किफर खानदान अचछा

ह वह ए शल गरहणी रप म घर सभाल सगी अमिमत न मा ो इस बात ा एहसास राया कि मीन सबध लिलए मना रन पर उस दिदल

पर कया बीती होगी मा और अमिमत ी लडी बार म ाफी बात हईमा ा झाव सरिरता ी तरफ था कयोकि वह घर पर अचछा दह0 लर आ रही

थी अमिमत न अपनी मौसी ी बरी हालत बार म बताया कि किस तरह वह बड घर ी खानदानी बटी लाई थी और आ0 उसी हालत कितनी खराब ह लाई थी बहकलब 0ाती ह और बचचो ो भी नही दखती ह बात चल ही रही

थी कि तभी ए ार बाहर आर री धनी मल0ी घर अदर आए और पीछ स डराइवर फल ी ए टोरी लर आया अदर आए और पीछ स

डराइवर ए टोरी फल ी लर आया अमिमत ो फल ी पटी बरी लग रही थी अमिमत न पछ लिलया यह फल कयो ल आए ह प इन सब ी कया

0ररत थी उनो न 0वाब दिदया कि 4 पटी शमीर स मगाए थ अमिमत ो या सनर करोध आ गया तभी उस किपता 0ी आ गए उन आत ही अमिमत उठर बाहर चला गया वहा वहा मा पास आर बठ गया और बोला

अभी रिरशता तय नही हआ और धनी मल 0ी धनी मल 0ी फल ी पटी लर चलआय मा न समझाया कि 0ब सबध 0ड 0ाता ह तो खाली हाथ नही

आत अमिमत न मा स हा कि तम सबन सरिरता ो इस घर म लान ी ठान रखी ह धनीमल 0ी उस दिदन सरिरता ो दखन ी तारीख तय रन आय थ

Commercial Studies

Banking Nowadays Bank provide easy and quick services through internet facilities methods of Banking is called internet bankingIn order to save the time and money involved in visiting Bank branches people increasingly prefer to have internet banking

There are different modes of doing internet banking or transferring money through online They areReal Time Gross Settlement (RTGS)National Electronic Fund Transfers (NEFT)

1

Question

1) Explain the term RTGS Write the features of RTGS

Answer)The acronym RTGS stands for Real Time Gross Settlement which may be defined as the continuous real time settlement of funds transfer individually on and order by order basis without netting lsquoReal timersquo may be defined as the processing of instructions at the time they are received rather than at some letter time lsquoGross settlementrsquo may be defined as the settlement of transfer instructions which occurs

individually

Features of RTGS1It is the continuous settlement of

funds transfer individually on an order by order basis

2RTGS facility is provided only by CBS core banking solution enabled Bank branches

3Amount charged from the customer for RTGS transactions vary from bank to bank

2) Explain the term NEFT Write the features of NEFT

Answer) National electronic funds transfer may be defined as a nationwide system that facilitates individuals Farms and copper operates to electronically transfer funds from any bank branch to any individual farm or corporate having an account with any other bank branch in the country

Features of NEFT2 Transfer can be made 7 times on

weekdays and 6 times on Saturday

3 NEFT cannot be used to receive foreign remittances

4 NEFT transaction takes place in batches

5 A bank branch must be NEFT enabled to become a part of NEFT fund transfer network

6 There is no maximum or minimum amount that can be transferred through NEFT when one bank has a bank account

English Language

CompositionEssay

A composition is an art of creating a piece of writing on any topic or subject It is the writing correctly beautifully and clearly in order to make some interesting reading Structure of the composition

Introduction ( you lay the foundation for your composition)

Body (it constitutes the main part of the essay)

Conclusion (final statement that leaves a lasting impression)

Kinds of essays1 The Narrative essay2 The descriptive essay3 The reflective essay4 The argumentative essay

Write a composition on any one of the following topics (350- 400 words)

1 Friendship Or2 The first day of your school

Subject Eng Literature (The Merchant of Venice ndash William Shakespeare)Topic Act V Scene 1 Lines 127 to 158 (Nerissa helliphellip The clerk will nersquoer wear hair onrsquos face that had it) [Students should read the original play and also the paraphrase given in the school prescribed textbook]

Summary Revision Questions o Soon thereafter Bassanio Gratiano

and Antonio arrive

o Bassanio tells Portia that he is feeling as if it is morning because of the presence of Portia who is shining like the sun When Antonio is introduced by Bassanio to Portia she tells Bassanio that he should be grateful to Antonio who took so much trouble on his account even to the extent of risking his life

o Nerissa starts quarrelling with Gratiano and demands that he show her the ring she had presented to him and which she had warned him not to lose She suspects that Gratiano must have presented the ring to some young woman and not to the lawyerrsquos clerk as he repeatedly says and assures

Answer the following questions to check your preparation of Act IV Scenes 1 and 2

You must attempt only after you have completed your preparation of Act IV The answers must be in complete sentences using textual evidence (with citation) when necessary

[It would be in your own interest to attempt the above questions honestly totally refraining from consulting your textbook or your notes during answering After completion you should correct the paper yourself consulting the textbooknotes etc and award marks as specified Please let me know the marks you scored through WhatsApp in the group or to my personal WhatsApp]

Act IV Scene 1 (each question carries 2 marks)

1 What did the Duke try to do for Antonio

2 Why does Shylock refuse to show mercy How does he justify his stance

3 Why does Antonio say he is ready to die 4 What information is contained in Bellariorsquos letter

5 Why does Portia (as Balthazar) assert that Shylock must show mercy How does he respond

6 What offers are made to Shylock to get him to spare Antonio How are they received

7 What does Antoniorsquos speech as he faces the prospect of Shylockrsquos knife tell you about his character

8 How do Bassanio and Gratiano react to the looming prospect of Antoniorsquos demise

9 How does Portia (as Balthazar) use the law to turn the tables on Shylock

10 What does the Duke decree should happen to Shylock Why What happens to Shylockrsquos estate

11 What does Portia ask Bassanio as payment for her ldquoservicesrdquo What is his initial response What makes him change his mind

Act IV Scene 2 (each question carries 1frac12 marks)

1 What does Gratiano bring to Portia (Balthazar)

2 What does Nerissa plan on getting from Gratiano What does Portiarsquos comment suggest about men

ECO-10 280620 Topic-Supply AnalysisSHIFTING OF SUPPLY

But if there is change in factors other than the price of the commodity then either more is supplied at the same price or less supplied at the same price In such cases the price of the commodity remains constant but there is a change in other factors like change in the price of inputs change in technology of production change in price of other related goods change in taxation policy of the government etc For example there is an improvement in the technology of production of the commodity in question It leads to decrease in per unit of cost production of the commodity The firm is willing to sell more quantity of the commodity at the same price So the supply other commodity increases at the same price This increase in supply is shown by rightward shift of supply curve On the other hand if the firm uses inferior technology of production the cost of production per unit of the commodity increases The firm is willing to sell less quantity at the same price So the supply of the commodity decreases at the same price This decrease in supply is shown by leftward shift of the supply curve The above cases of increase and decrease in supply can be shown with the help of the following figures

Y INCREASE OF SUPPLY Price (Rs) s

P A s1

B

s

X` O s1 X

q q1

Y` Quantity demanded (in units)

Y DECREASE IN SUPPLY s2

s

price (Rs)

C

p A

s2

s

X` o X

q2 q

Y` Quantity demanded ( in units)

Main factors causing increase in supply or rightward shift of supply Curve(i) Fall in the price of other related goods

(ii) Fall in the price of inputsfactors(iii) Use of better technology in production(iv) Decrease in the rate of excise duty by government(v) If the objective of producer changes from profit maximization to salesMaximization

Main factors causing decrease in supply or leftward shift of supply curve(i) Increase in the price of other related goods(ii) Rise in the price of inputsfactors(iii) Use of inferior technology in production(iv) Increase in the rate of excise duty by the government(v) If the objective

Subject - Biology Topic ndash Chapter mdash6 PhotosynthesisSummary Execution

Today we will know about photosynthesis and its stages

Q1 What do you mean by photosynthesis The process by which living plants containing chlorophyll produce food

substances from carbon-di- oxide and water by using light energy Sunlight

6CO2 +12 H2O----------------------- C6 H12O6 + 6H2O + 6O2

Chlorophyll

Q2 What are the importance of photosynthesis I) Food for all Green plants trap solar energy by photosynthesis

process and supply food and energy for all living organisms either directly or indirectly

Ii) Oxygen to breathe in by product of photosynthesis is oxygen which is essential for all living organisms respiration

Q3 Write about two main phases of photosynthesis A Light dependent phase This phase occur in grana of chloroplast I) The chlorophyll on exposure to light energy becomes activated by

absorbing photons Ii) The absorbed energy is used in splitting the water molecules (H2O)

into its two components (H+ and OH- ) and releasing electron s 2H2O------------------------- 4H+ + 4e- +O2

Energy of 4 photons This reaction is known as photolysis

End products are H+ and oxygen water

B Light independent (Dark ) phase The reactions in this phase require no light energy

Here CO2 combine with H+ and produce glucose

Class XI

Subject Topic Summary ExecutionEVS Chapter-4 Legal

regimes for sustainable development

Environmental legislationEnvironmental legislation is the collection of laws and regulations pertaining to air quality water quality the wilderness endangered wildlife and other environmental factors The act ensures that matters important to the environment are thoroughly

Learn -The Forest (Conservation) Act 1980

considered in any decisions made by federal agencies

The Forest (Conservation) Act 1980 The Forest (Conservation) Act 1980 an Act of the Parliament of India to provide for the conservation of forests and for matters connected therewith or ancillary or incidental thereto It was further amended in 1988 This law extends to the whole of IndiaObjects and Reasons of the Forest Conservation Act

Deforestation causes ecological imbalance and leads to environmental deterioration Deforestation had been taking place on a large scale in the country and it had caused widespread concern The act seeks to check upon deforestation and de-reservation of forests

Subject Eng Literature (The Tempest ndash William Shakespeare) Topic Act II Scene 1 Lines 314 to 329 (End of scene)

[Students should read the original play and also the paraphrase given in the school prescribed textbook]Summary Questions amp Answers

Conspiracy of Antonio and Sebastian (Contd)

o As they approach Ariel appears again and wakes up Gonzalo by singing a tune in his ear Alonso also wakes up and they see both Sebastian and Antonio with drawn swords On being caught off guard they make up a story saying that they had heard a bellowing of bulls or lions

o They then moved to another part of the island

o Ariel at once rushes to Prospero to inform him of this development

SUMMING-UP of ACT-2 SCENE-1

(i) Among the survivors Ferdinand is separated from the rest which results in the disconsolate grief of Alonso as he took him for dead

(ii) The villainy of Antonio is confirmed

(iii) The supremacy of Prosperorsquos magic which resulted in the failure of the human conspiracy

(1)

(Act II Sc 1 L 311-325)SEBASTIAN Whiles we stood here securing your repose

Even now we heard a hollow burst of bellowing Like bulls or rather lions Didt not wake youIt struck mine ear most terribly

ALONSO I heard nothingANTONIO O rsquotwas a din to fright a monsters ear

To make an earthquake Sure it was the roarOf a whole herd of lions

ALONSO Heard you this GonzaloGONZALO Upon mine honour sir I heard a humming

And that a strange one too which did awake meI shaked you sir and cried As mine eyes opened I saw their weapons drawn There was a noiseThats verily rsquoTis best we stand upon our guardOr that we quit this place Lets draw our weapons

(i) Why has Prospero sent Ariel to Gonzalo and Alonso What does Ariel do to awaken Gonzalo

Prospero has already come to know by his magic powers the danger which threatens Gonzalo who had been Prosperorsquos friend and so he sent Ariel to preserve the lives of both Gonzalo and Alonso Prospero does not want that his scheme should remain unfulfilled Ariel begins to sing a song in Gonzalorsquos ears to awaken him(ii) Who are ready to carry out their plan Who takes steps to stop them Why does Gonzalo feel surprised after being awakened

Sebastian and Antonio are ready to carry out their plans They are standing with their swords drawn to kill Alonso and

(iv) We see two sets of contrasting characters Gonzalo-Adrian against Antonio-Sebastian

(v) The grief that works in Alonso can be perceived to his repentance for his association in Antoniorsquos crime against Prospero

Gonzalo Ariel takes steps to stop them from carrying out their nefarious scheme When Gonzalo is awakened by the song sung by Ariel into his ears he (Gonzalo) feels surprised because he sees Sebastian and Antonio standing with their swords drawn(iii) What reason do Sebastian and Antonio tell of drawing their swords when they are suspected by Alonso and Gonzalo

When Sebastian and Antonio are seen with their swords drawn they are looked with suspicion by Gonzalo and Alonso At first Sebastian tells them that as they stood here to guard them during their sleep they heard only a little before a sudden loud noise very much like the roaring of bulls or more probably that of lions Then Antonio follows him saying that this was a noise so terrible as to frighten even a monsterrsquos ears and this noise could even have shaken the earth and it was surely like the roaring of a multitude of lions Then seeing the danger they have drawn their swords Perhaps after hearing the terrible noise they (Gonzalo and Alonso) woke up from their sound sleep

(iv) What does Gonzalo tell Alonso about the strange noise What did he see on opening his eyes Gonzalo tells Alonso that he did not hear the sound of roaring but he heard a humming sound which was strange and which woke him up After waking up he gave him (Alonso) a shaking and a loud cry On opening his eyes he saw these two gentlemen standing with their swords drawn(v) What does Gonzalo suggest

Gonzalo suggests that there was a noise indeed and of that he has no doubt at all and suggests that the best course for them would be to remain alert and vigilant against any possible danger to their lives or to leave this place and move to some other part of the island

Class XIISubject Topic Summary Execution

Commerce

Chapter- Management

Today we will discuss about LEVELS OF MANAGEMENT

Levels of management is a series or chain of managerial positions from top to bottom It helps individuals to know their authority responsibilities and superior-subordinate relations among themselves There are mainly three levels of Management TOP LEVEL MANAGEMENTMIDDLE LEVEL MANAGEMENTLOWER LEVEL MANAGEMENT

Top level managementIt consists of members at the highest level in the management hierarchy This level includes Board Of Directors Chief Executive Managing Directors Chairman President Vice President

Rolefunctions of the top levelmanagement1To analyse evaluate and deal

with theexternal environment2 To determine the objectives and

policies of the business3 To strive for welfare and survival

of business

4 To create an organisational Framework consisting of authority responsibility relationship

Middle level management Congress of members or groups who are concerned with implementation of the policies let down by the top managementThis level includes head of the department such as finance manager marketing manager branch and regional managers departmental and divisional heads plant superintendent etc

Role of functions of the middle level management

1 To interpret the policies framed by top management

2 To assign duties and responsibilities to lower level managers

3 To select and appoint employees for middle and supervisory level and evaluate their performance

4 To co-operate with other departments for smooth functioning

Operational or supervisory level managementIt refers to the group are members who are concerned with execution of the work They are also known as fast line managers This level includes supervisor 4 men Section Officer clerk Inspector etc

Role of functions of the lower level management1 To plan and execute day-to-

day operations2 To supervise and control the workers3 To arrange materials and

tools to start the process and make arrangements for training

4 Today present workers grievance and suggestions before the management and

ensure safe and proper working conditions in the factory

Business Studies

Staff Appraisal Chapter- 10 Today let us start with a new chapter

Staff Appraisal

Meaning of Performance Appraisal

Performance Appraisal is the systematic evaluation of the performance of employees and to understand the abilities of a person for further growth and developmentThe supervisors measure the pay of employees and compare it with targets and plansThe supervisor analyses the factors behind work performances of employeesThe employers are in position to guide the employees for a better performance

Objectives of Performance Appraisal

Following are the objectives of Performance Appraisal

To maintain records in order to determine compensation packages wage structure salaries raises etc

To identify the strengths and weaknesses of employees to place right men on right job

To maintain and assess the potential present in a person for further growth and development

To provide a feedback to employees regarding their performance and related status

To provide a feedback to employees regarding their performance and related status

Importance of Performance Appraisal

Performance appraisal provides important and useful information for the assessment of employees skill

knowledge ability and overall job performance The following are the points which indicate the importance of performance appraisal in an organization

1 Performance appraisal helps supervisors to assess the work performance of their subordinates

2 Performance appraisal helps to assess the training and development needs of employees

3 Performance appraisal provides grounds for employees to correct their mistakes and it also provides proper guidance and criticism for employees development4 Performance appraisal provides reward for better performance

5 Performance appraisal helps to improve the communication system of the organization

6 Performance appraisal evaluates whether human resource programs being implemented in the organization have been effective

7 Performance appraisal helps to prepare pay structure for each employee working in the organization

8 Performance appraisal helps to review the potentiality of employees so that their future capability is anticipated

Geography

DRIANAGE The SubarnarekhaThe Subarnarekha and the Brahmaniinterposed between the Ganga and the Mahanadi deltas drain an area of 19300 sq kmand 39033 sq km respectively The drainage basins of these streams are shared byJharkhand Odisha west Bengal and Chhattisgarh The Brahmani is known as southKoel in its upper reaches in Jharkhand

The NarmadaThe Narmada rises in the Amarkantak hills of MadhyaPradesh It flows towards the West in a rift valleyformed due to a geological fault The total length of it is 1300 km All the tributaries of the

Q1 Name the two westward flowing rivers in the peninsular plateauA1 Narmada and Tapi are the only westward flowing rivers of the peninsular plateau

Q2 Differentiate between east-flowing rivers and west-flowing riversA2

East-flowing rivers

West-flowing rivers

Narmada are very short inlength Most of its tributaries join the main streamright anglesThe Narmada basin covers parts of Madhya Pradesh and Gujarat

The Tapi The Tapi rises in the Satpura ranges in the Betul listrictof Madhya Pradesh It flows in a rift valley parallel tothe Narmada but it is much shorter in length It coversparts of Madhya Pradesh Gujarat and MaharashtraThe length is about 724 km

The Sabarmati and the MahiThe Sabarmati rises in the Aravali hills and flows south-south-westwards for a distance of 300 kilometres to the Arabian Sea The Sabarmatibasin extends over an area of 21674 sq km in Rajasthan and Gujarat The Mahi rises inthe east of Udaipur and drains an area of 34842 sq km lying in Madhya PradeshRajasthan and Gujarat It flows south-westwards for a distance of 533 km before it fallsinto the Gulf of Khambhat

The ChambalThe Chambal rises near Mhow in the Vindhya Range and flows towards the northgenerally in a gorge upto Kota Below Kota it turns to the north-east direction and afterreaching Pinahat it turns to the east and runs nearly parallel to the Yamuna beforejoining it in the southern part of the Etawah district in Uttar PradeshMajor Rivers of India with their basin area (Sqkm)

Himalayan System Indus 321290Ganga 861404

Brahmaputra 187110Indus System

Jhelum 34775Beas 20303

Ganga System Yamuna 366223Ghaghra 127950

Peninsular RiversNarmada 98796

Tapi 65145Mahanadi 141600

Subarnarekha 19300Sabarmati 21674

Mahi 34842Godavari 312812

Godavari Krishna Kaveri Mahanadi are the east-flowing rivers

Narmada Tapi west-flowing rivers

They fall into the Bay of Bengal

They fall into Arabian Sea

These rivers form big deltas

These rivers form comparativelysmall deltas

Catchment areas of these rivers are larger

Catchment areas of these rivers are smaller

Krishna 2589488Cauveri 87900

Subject ndashBiology Topic ndashChapter -5 Inheritance amp Variations Summary ExecutionToday we will discussabout linkage and its classification

LINKAGE The tendency of the genes located on the same chromosome to stay together is

hereditary transmission Linked genes the genes responsible for this Genes that exhibit the process of linkage locates in the same chromosome The distance between the linked genes in a chromosome determines the strength

of linkage i e genes that are located close to each other show stronger linkage than that are located far from each other

COMPLETE LINKAGE It is the type of linkage showed by the genes that are closely located or are tightly

linked with each other as they have no chance of separatingby crossing over These genes are always transmitted together to the same gamete and the same

offspring In such condition only parental or non cross over type of gametes are formedINCOMPLETE KINKAGE It is type of linkage showed by the genes that are distantly located orare loosely

linked with each other because they have chance of separating by crossing over

SIGNIFICANCE i) It helps in holding the parental character togetherii) It checks the appearance of new recombination and helps in bringing the

hybrid population which resembles the original parents iii) Linked genes dilute the effects of undesirable traits

Subject Eng Literature (The Tempest ndash William Shakespeare) Topic Essay Questions (EQ-3)Question No 3

Give a character sketch of CalibanAnswer

The character of Caliban has been wonderfully conceived by Shakespeare as the manifestation of all that is gross and earthy ndash a sort of creature of the earth as Ariel is a sort of creature of the air

Calibanrsquos Physical Appearanceo Caliban is lsquofreckledrsquo a lsquomisshapen knaversquo not honoured with human shape

o Prospero calls him lsquothou tortoisersquo (Act I Sc 2 Line 317) Trinculo stumbling upon him describes him as ldquoA strange fish hellip Legged like a man And his fins like armsrdquo He ldquosmells like a fishrdquo (Act II Sc 2 Line 25)

o Prospero also calls him a ldquobeastrdquo (Act IV Sc 1 Line 140) and ldquoThis misshapen knaverdquo (Act V Sc 1 Line 268)

o Further it appears that in addition to his physical deformity his spiritual inferiority is also suggested by Prosperorsquos claim that his birth resulted from the union between his mother the witch Sycorax and the devil

Calibanrsquos ParentageWhen the play opens Caliban is twenty four years of age having been born on the island twelve years before the coming of Prospero His mother was the foul witch Sycorax who was banished from Algiers for ldquomischiefs manifold and sorceries terrible to enter human hearingrdquo (Act I Sc 2 Line 264) and the father was the Devil himself Thus

Caliban is a monster of evil and brute nature ugly deformed and stinking

Calibanrsquos Savage and Malignant Natureo Caliban is entirely a creature of the earth ndash gross brutal and savage He regards himself as the rightful possessor

of the island and Prospero as a usurper

o In his young age he was on good terms with Prospero He had consented to be received by Prospero at his house and to be educated by him He has learnt human language only to curse his master whom he abhors

o His beastly nature soon breaks out and ends in a vicious attack on Miranda This opens the eye of Prospero who becomes severe to him and enforces his service by threats and violence

o Prospero uses him to make dams for fish to fetch firewood scraper trenches wash dishes and keep his cell clean

Calibanrsquos Hatred for ProsperoA profound hatred for Prospero has taken hold of Caliban It springs from a sense of his being dispossessed and ill-treated He would kill Prospero if he could but he knows the power of Prosperorsquos lsquobookrsquo Hence he transfers his allegiance to Stephano who seems like a god to him He also incites the two drunken associates to batter the skull of Prospero when he sleeps in the afternoon

Caliban Shows Considerable Intelligenceo He has learnt Prosperorsquos language

ldquoYou taught me language and my profit onrsquot (Act II Sc 2 Lines 86-89)Is I know how to curserdquo

o He is well aware of the futility of arguing with one who has more power than he has

ldquoI must obey his art is such power (Act I Sc 2 Lines 373-376)It would control my damrsquos god SetebosAnd make a vassal of himrdquo

o He realizes the importance of Prosperorsquos books

ldquoRemember (Act III Sc 2 Lines 89-92)First to possess his books for without themHersquos but a sot as I am nor hath notOne spirit to commandrdquo

o He knows the value of stealth when attacking the enemy

ldquoPray you tread softly that the blind mole may not (Act IV Sc 1 Lines 194-195)Hear a foot fall we now are near his cellrdquo

o Caliban has a better set of values than Stephano and Trinculo They are distracted from their plan by their greed for Prosperorsquos rich garments Only Caliban realizes that such a finery is unimportant

ldquoLeave it alone thou fool it is but trashrdquo (Act IV Sc 1 Lines 224)

Caliban is not a good judge of characterCaliban is not a good judge of character He decides for example that Stephano is a god because he dispenses lsquocelestial liquorrsquo (Act II Sc 2 Line 115) but then it must be remembered that he has only known his mother Sycorax Prospero Miranda and the spirits that torture him However he quickly discovers his error of judgementrdquo

ldquoWhat a thrice-double ass (Act V Sc 1 Lines 295-297)Was I to take this drunkard for a godAnd worship this dull foolrdquo

Calibanrsquos Imaginative NatureIf Caliban is sub-human in what has been said above he is human in the respect of the poetic side of his character He listens to music with rapture He tells of the beautiful dreams in which heaven rains treasures upon him and which upon waking he yearns to renew One of the most poetic passages in whole play is Calibanrsquos description of the island

to Stephano and Trinculo

ldquoBe not afeard The isle is full of noises (Act III Sc 2 Lines 135-143)Sounds and sweet airs that give delight and hurt notSometimes a thousand twangling instrumentsWill hum about mine ears and sometime voicesThat if I then had waked after long sleepWill make me sleep again and then in dreamingThe clouds methought would open and show richesReady to drop upon me that when I wakedI cried to dream againrdquo

Caliban - Less Ignoble Than Some OthersCalibanrsquos motive for murder is less dishonourable than that of Antonio and Sebastian They plan to kill Alonso to gain his power and wealth Caliban merely wants revenge and the return of lsquohisrsquo island

Conclusiono Calibanrsquos character is not portrayed very clearly in the play and hence we cannot decide whether he is a poor

savage being grossly maltreated by Prospero or whether he is evil and must therefore be kept in bondage or enslavement

o Caliban is contrasted with Ariel who is a spirit and thus swift and uninterested in physical activitieso Caliban is also contrasted with Prospero who is the all-powerful master of the island and of the destiny of all

those on the islando Caliban is also contrasted with civilized man showing him to be less evil than Antonio and Stephano and less

materialistic than Stephano and Trinculoo Caliban has suffered at the hands of Prospero and he has learnt to curse by listening to Prosperorsquos abuse He

certainly believes that Prospero has deprived him of his birthrighto Finally the character Caliban is thought to be one of Shakespearersquos masterpieces The complexity of the character

is reflected in the large volume of critical discussion that has grown around it

ECO ndash12 Topic-Forms of market

MonopolyMonopoly is a market structure in which there is a single seller there are no close substitutes for the commodity produced by the firm and there are barriers to entry Example Indian Railways which is operated under government of India Monopoly also implies absence of competitionFeatures of Monopoly Monopoly is characterized by1 Single Seller In monopoly there is only one firm producing the product The whole industry consists of this single firm Thus under monopoly there is no distinction between firm and industry Being the only firm there is significant control of the firm over supply and price Thus under monopoly buyers do not have the option of buying the commodity from any other seller They have to buy the product from the firm or they can go without the commodity This fact gives immense control to the monopolist over the market

2No Close Substitute There are no close substitutes of the product produced by the monopolist firm If there are close substitutes of the product in the market it implies presence of more than one firm and hence no monopoly In order to ensure a total of control over the market by the monopolist firm it is assumed that there are no close substitutes of the product

3 No Entry amp Exit Monopoly can only exist when there is strong barriers before a new firm to enter the market In fact once a monopoly firm starts producing the product no other firm can produce the same One reason for this is the ability of the

monopolist to produce the product at a lower cost than any new firm who thinks to enter the market If a new firm who knows that it cannot produce at a lower cost than the monopolist then that firm will never enter the market for fear of losing out in competition Similarly the monopolist who is operating for a long time may be enjoying reputation among its customers and is in a better position to use the situation in its own benefit A new firm has to take long time to achieve this and so may not be interested to enter the market

4 Price Maker Being the single seller of the product the monopolist has full control over the pricing of the product On the other hand if there is a large number of buyers in the market so no single buyer exercises any significant influence over price determination Thus it is a sellerrsquos market So monopoly firm is a price maker

5 Price Discrimination Having considerable control over the market on account of being single seller with no entry of other firms the monopolist can exercise policy of price discrimination it means that the monopolist can sell different quantities of the same product to a consumer at different price or same quantity to different consumers at different prices by adjudging the standard of living of the consumer

6 Shape of Demand Curve Since a monopolist has full control over the price therefore he can sell more by lowering the price This makes the demand curve downward sloping

Subject Ac-12 290620 Topic- retirement Model sumThe Balance Sheet of Rohit Nisha and Sunil who are partners in a firm sharing profits according to their capitals as on 31st March 2014 was as under

Liabilities Amount Assets Amount (Rs) (` Rs)

Creditors 25000 Machinery 40000Bills Payable 13000 Building 90000General Reserve 22000 Debtors 30000Capital Less Provision for Rohit 60000 Bad debts 1000

29000 Nisha 40000 Stocks 23000 Sunil 40000 140000 Cash at Bank 18000

200000 200000

On the date of Balance Sheet Nisha retired from the firm and following adjustments were made(i) Building is appreciated by 20(ii) Provision for bad debts is increased to 5 on Debtors(iii) Machinery is depreciated by 10(iv) Goodwill of the firm is valued at Rs 56000 and the retiring partnerrsquos share is adjusted

(v) The capital of the new firm is fixed at Rs120000 Prepare Revaluation Account Capital Accounts of the partner and Balance Sheet of the new firm after Nisharsquos retirement Revaluation AccountDr Cr

Particulars Amount Particulars Amount (`Rs) (Rs`)

Provision for Bad debt Ac 500 Building Ac 18000Machinery Ac 4000Profit transferred toCapital Accounts (3 2 2)Rohit 5786Nisha 3857Sunil 3857

13500

18000 18000

Capital Account

Dr Cr

Particulars Rohit Nisha Sunil Particulars Rohit Nisha Sunil (Rs`) (Rs`) (`Rs) (Rs`) (Rs`) (Rs`)

Sunilrsquos Capital ac 9600 mdash 6400 Balance bd 60000 40000 40000Bank - 66143 - General Reserve 9428 6286 6286Balance cd 72000 mdash 48000 Revaluation (Profi 5786 3857 3857 Rohitrsquos Capital Ac mdash 9600 mdash

Sunilrsquos Capital Ac 6400 Bank 6386 - 4257

81600 66143 54400 81600 66143 54400

Balance Sheet as at 31st March 2014

Liabilities Amount Assets Amount (Rs`) (Rs`)

Creditors 25000 Building 108000Bank overdraft 37500 Machinery 36000

Bills Payable 13000 Debtors 30000Capital Less ProvisionRohit 72000 for Bad debts 1500 28500Sunil 48000 120000 Stock 23000

195500 195500

Working Notes (i) (a) Profit sharing ratio is 60000 40000 40000 ie = 3 2 2(b) Gaining Ratio Rohit = 35 ndash 37 = 2135 ndash 1535 = 635Sunil = 25-27 = 1435 ndash 1035 = 435= 635 435= 6 4 = 3 2(c) Nisha Share of Goodwill = Rs 56000 times 27 = Rs16000Share of Goodwill in the gaining ratio by the existing partner ieRohit = Rs16000 times 35 = Rs 9600Sunil = Rs 16000 times 25 = Rs 6400

The journal entry isRohitrsquos Capital Ac Dr 9600Sunilrsquos Capital Ac Dr 6400 To Nisharsquos Capital Ac 16000(Share of Goodwill divided into gaining ratio)

  • 1 Static Friction
  • The frictional force that acts between the surfaces when they are at rest with respect to each other is called Static Friction
    • Static Friction Examples
      • 2 Sliding Friction
        • Examples Of Sliding Friction
          • 3 Rolling Friction
            • Examples Of Rolling Friction
              • Objects and Reasons of the Forest Conservation Act
Page 16:  · Web viewSubject . Topic . Summary . Execution . English 1 . Sounds of animals . Hens –cackle Horses –neigh Lions –roar Owls –hoots Snake –hiss. English 2 . Mother’s

ধকের মানষ এই পশ পাহিখর -াষা হিনকেয় নানা রককেমর পরীকষা ককের আসকে এইরকম একজন হিরউকেবন কযাসটাং সাকেকেবর কা আমরা জানকেবাhelliphellip

Hindi 2nd

langमतर किनमनलिलखिखतपरशनोउRरदीजि0ए

) बढ वयलि` बचच ो कया हआ था ख) डॉकटर साहबन पाटc किस उददशय स रखी थी ग) ाल साप ो हाथ म लर लाश न कया किया घ) डॉकटर चडढा न बढ पतरो दखन स कयो मना र दिदया था ङ) भगत न लाश ो दखर कया हा

उRर ndash) उस बहत बखार थी और 4 दिदनो स आख भी नही खोला थाख) उन बट ी सालकिगरह थीग) ाल सापो हाथ म लर लाश न उसी गदन 0ोर स दबार पडी थीघ) डॉकटर चडढा न बढ वयलि` पतरो दखन स मना र दिदया कयोकि उनह गोलफ खलन 0ाना थाङ) लाश ो दखर हा कि नारायण चाहग तो आध घट म भया उठ 0ाएग

English literature

In the bazaars of Hyderabad- Sarojini Naidu

Through the poem In The Bazaars of Hyderabad Sarojini wanted to convey the message that India is rich in tradition and they donrsquot need the foreign products So she goes on to give a picture of a bazaar where traditional Indian products are rulingThe poem is in the form of questions and answers The poet asks the questions and the merchants answer them Through this technique she make the picture of the bazaar visible to us

Read the poem

PHYSICS FORCE Types of FrictionThere are three types of friction static sliding rolling Static sliding and rolling friction occur between solid surfaces

1 Static Friction The frictional force that acts between the surfaces when they are at rest with respect to each other is called Static FrictionStatic Friction Examples

Skiing against the snow Creating heat by rubbing both the hands

together Table lamp resting on the table

2 Sliding Friction The resistance that is created between any two objects when they are sliding against each other is called Sliding FrictionExamples Of Sliding Friction

Sliding of the block across the floor Two cards sliding against each other in a

deck

3 Rolling Friction The force which resists the motion of a ball or wheel is called Rolling Friction Is the weakest types of frictionExamples Of Rolling Friction

Rolling of the log on the ground Wheels of the moving vehicles

6What effect can a force produce on a body which is not allowed to move Ans - When a force is applied on a body which is not free to move it gets deformed i e the shape or size of the body changes7Give one example each to indicate that the application of a force

1 produces motion2 stops motion3 slows down motion4 changes the direction of motion5 deforms a body

Ans- 1 A car originally at rest when pushed

begins to move2 A moving bicycle is stopped by

applying the brakes3 The speed of a moving vehicle is

slowed down by applying the brakes4 A player kicks a moving football to

change its direction of motion5 On stretching a rubber string its

length increases

8State the effect produced by a force in the following cases (a) The sling of a rubber catapult is stretched(b) A man pushes a heavy cart(c) A player uses his stick to deflect the ball (d) A cyclist applies brakes(e) A spring is compressedAns- (a) The shape and size of catapult changes ie its length increases(b) The heavy cart begins to move(c) The direction of the ball changes(d) The speed of the moving cycle is slowed down(e) There is change in size and shape of spring

COMPUTER MS EXCEL 2013 -INTRODUCTION

UNDERSTANDING EXCEL STRUCTUREA SPREADSHEET IS A FILE THAT EXISTS OF CELLS IN ROWS AND COLUMNS AND CAN HELP ARRANGE CALCULATE AND SORT DATA DATA IN A SPREADSHEET CAN BE NUMERIC VALUES AS WELL AS TEXT

FORMULAS REFERENCES AND FUNCTIONS

WORKSHEETA WORKSHEET IS ALSO KNOWN AS SPREADSHEETIT IS A COLLECTION OF CELLS ON A SINGLE SHEET WHERE YOU KEEP AND CHANGE DATA

WORKBOOKWORKBOOK IS PMS EXCEL FILE IN WHICH THE DATA CAN BE STORED EACH WORKBOOK CAN CONTAIN MANY WORKSHEETS

ROWS AND COLUMNSIN MS EXCEL A ROW IS A GROUP OF CELLS THAT RUN FROM LEFT TO RIGHT OF A PAGEA COLUMN IS A GROUPING OF CELLS THAT RUN FROM THE TOP TO THE BOTTOM OF A PAGE

CELLTHE INTERSECTION POINT BETWEEN A ROW AND THE COLUMN IS CALLED A CELL WHICH IS THE BASIC STORAGE UNIT FOR DATA IN A SPREADSHEET EACH CELL HAS SPECIFIC ADDRESS WHICH IS THE COMBINATION OF THE COLUMN NAME FOLLOWED BY THE ROW NUMBER

CHEMISTRY Chapter ndash Common Laboratory Apparatus and equipments

Objective type questionFill in the blanks (a) Experiment and observation are the two important basics of chemistry(b) A porcelain dish is used for evaporation(c) A test tube holder is used to hold the test tube while-it is heated(d) Mortar and pestle is used for grinding and crushing solid substances into a powder(e) Glass apparatus is made of Pyrex or borosil glass

Class VIISubject Topic Summary Execution

Hindi 2ndlang

ए था राम( डॉ शरी परसाद)

सगकित ा परभाव मानव 0ीवन पर अवशय पडता ह

हमशा मनषय ो अचछो ी सगकित म रहना चाकिहए

शरषठ परो सग स मनषय चरिरतर ा शीघर ही उदय और किवास हो 0ाता

ह इसलिलए वयलि` ो सदा शरषठ परो ा ही सग रना चाकिहए

इसान अगर चाह वह सवय ो बदल भी सता ह

यह हानी राम ए बचच ी हवह गणिणत ी परीकषा म नल रत हए पडा 0ाता ह और उस अधयाप पडत ह और पछत ह यह कया र रह

हो तभी राम न उनी बइजजती ी

शबदाथब ndashहावा भलावाायवाहीndash ाम किनयम व ानन

ो दिदखानापरिरलिचतndash 0ाना पहचानाघटनाndashघबराहट

उलटा चोर ोतवाल ो डाटndashकिववndash भल बर ा जञानतवयndash म 0ो रना चाकिहएसगकितndash बरी सगत

किबलख नाndashरोना किनशचय रनाndash तय रना

फलndashपरिरणामकिनषालिसतndash बाहर किया हआपशचातापndashदख सपननndashधनी

ldquo हा आपी किहममत स हई नल रत पडन ीrdquo ऐसी बात ही किफर

किपता0ी न भी उस डाटा वह ाफी पशचाताप रन लगा बोला गलत दोसतो

ी सगकित म आ0 कितना अनथ र दिदया किफर उसन अधयाप स माफी मागन ी सची और किफर भी ऐसा

नही रगा यह परण भी लिलया

सोचndashकिहच एात-अला

বইndashবাংলা সাহিতয পহিরচয়

পাঠndash১৬লপndashস-য ও অস-যযলখকndashঈশবরচনদর হিবযাসারঅনশীলনীর পরকে4াততর

৬ অGকেলকেখা -ময়া = পশ হিশকার সহিtহিত = হিনকIবতu সbভরষট = লI হিনরীকষণ =

-াকেলা-াকেব যখাকতাঞজহিলপকেI = যজাাকেত৭ হিবপরীতশবদ -ঈষৎ times পরচর উৎকষট times হিনকষট তাশ times উৎফd তবহিদধ times

বহিদধীNপাহিপষঠ times পণযবান৮ পপহিরবতG ন ককেরা -পশ = পাশহিবক যকাপ = যকাহিপতহিসথর = হিসথরতাএকানত = ঐকাহিনতক পর-াত times পর-াতী

CHEMISTRY

Chapter ndashPhysical and Chemical Changes

Chemical ChangeA chemical change involves a change in chemical composition

Characteristics of Chemical changes 1 They are permanent changes2 They are irreversible changes 3 New substance formed4 A Chemical change involves a

change in its chemical properties

Pg-25Question 8What do you observe when1 water is boiled2 a piece of paper is burnt3 some ice cubes are kept in a glass tumbler4 solid ammonium chloride is heated5 an iron nail is kept in tap water for few days6 a spoon of sugar is heated in a pan7 lighted match stick is brought near the mouth of the test tube containing hydrogen gas8 quick lime is dissolved in water9 little amount of curd is added to a bowl containing warm milk and kept for five hours

10 Water is boiledOn boiling water changes into steam (gas) physical change

11 A piece of paper is burnton burning piece of paper produces carbon dioxide and ash is left behind Is a chemical change

12 some ice cubes are kept in a glass tumblerIce cubes (solid) turn into water

(liquid) only state changes (physical change)

13 Solid ammonium chloride is heatedSolid ammonium chloride on heating changes into vapors (change of state) is physical change

14 An iron nail is kept in tap water for few dayswe observe reddish brown coating on the nail called rust (entirely new substance) is chemical change

15 A spoon of sugar is heated in a panWhen a spoon of sugar is heated in a pan black (charred sugar) (carbon) is seen Is a chemical change

16 Lighted match stick is brought near the mouth of the test tube containing hydrogen gasWe observe that hydrogen bums at the mouth of test tube with blue flame and pop sound is heard It is chemical change

17 Quick lime is dissolved in waterThe following two observations will be observed (i) A hissing sound is observed(ii) The mixture starts boiling and lime water is obtained

18 Little amount of curd is added to a bowl containing warm milk and kept for five hoursWhen a little amount curd is added to a bowl containing warm milk and kept for five hours a permanent change occurredThe milk will change to curd On boiling water changes into steam (gas) physical change

GEOGRAPHY

ATMOSPHERE IMPACT OF GLOBAL WARMING The destructive impart of global warming is observed in various spheres of life and the environment Some of the points are outlined below1 High temperatures lead to high

evaporation rate and drying up of the soil and surface water This affects crop production The occurrence of droughts is aggravating the problem even further

2 The heat waves in summer months

Q1 Write some impact of global warmingA1 The impacts of global warming are as follows1 High temperatures lead to high

evaporate ion rate and drying up of the soil and surface water This affects crop production The occurrence of droughts is aggravating the problem even further

2 The heat waves in summer months lead to a greater number

lead to a greater number of deaths due to heat strokes

3 Forest fires become more frequent4 Tropical cyclones and hurricanes

become common5 Melting of glaciers takes place6 Polar ice caps are becoming thinner

and melting at an alarming rate due to global warming The loss of sea ice

7 Due to increase in sea surface temperature sea levels rise in coastal areas and cause submergence of several islands

WAYS TO REDUCE GLOBAL WARMINGFollowing steps can be taken We need to decrease emission of

green house gases by reducing the burning of fossil fuel such as coal and petroleum

By planting more trees to increase forest cover

The government should also distributes free saplings and organize afforestation programmes to spread awareness regarding the beneficial effects of trees

We should switch to eco-friendly cars and gadgets

Incandescent light bulbs should be replaced by CFL bulbs

We can save electricity and reduce global warming by turning off electrical gadgets such as lights fans air-conditioners television and computer when we do not to use them

Efforts should be made to hasten the development of green cities oreco cities These cities are urban areas around the world striving to lessen the environment a impacts of urbanization

By following the 3Rs-Reduce Recycle and Reuse strategy we can use natural resources for our growth as well as save them for the need of the future generations This is called sustainable development

of deaths due to heat strokes3 Forest fires become more

frequent4 Tropical cyclones and hurricanes

become common5 Melting of glaciers takes place

etc

Q2 How to reduce global warmingA2 Following steps can be taken to reduce global warmingaWe need to decrease emission of

green house gases by reducing the burning of fossil fuel such as coal and petroleum

bBy planting more trees to increase forest cover

c The government should also distributes free saplings and organize afforestation programmes to spread awareness regarding the beneficial effects of trees

dWe should witch to eco-friendly cars and gadgets

eIncandescent light bulbs should be replaced by CFL bulbs

f We can save electricity and reduce global warming by turning off electrical gadgets such as lights fans air-conditioners television and computer when we do not to use them

Q3 What do you mean by 3Rrsquos of resource planningA3 The 3Rs are

1 Reduce 2 Recycle and3 Reuse

Q4 What is Sustainable developmentA4 By following the 3Rs-Reluce Recycle and Reuse strategy we can use natural resources for our growth as well as save them for the need of the future generations This is called sustainable development

English Language

Prepositions A preposition is a word placed before a noun or a pronoun It helps to show how the person or thing denoted by the noun is related to something else in the sentence

Kinds of Prepositions

Simple Prepositions- simple preposition are one word Prepositions such as at by for in of off for from on out through till to up with before amidst towards beyond between over etc

Compound Prepositions ndash There are some words that are always used with fixed Prepositions to convey specific meaning

Example I was unable to meet you dueto a previous engagement ( On account of)Always maintain the queue instead of crowding at the counter ( In place of)

Participial PrepositionsmdashParticiple Prepositions are present or past participles of various verbs which together with a noun phrase or a clause function as prepositions Examples- barring concerning considering notwithstanding pending regarding respecting etc

Exercise A

1 Gauravs fever has come down since Friday He has been absent for a week now

2 The child sat between his father and mother among the parents of all his classmates

3 There are mosquitoes in the room They flew into the room when the door was open

4 My father was inside the drawing room when I was playing outside my house

5 You may sit beside me I will give you a drawing book and pencils besides a storybook

6 We went to the market in the morning and walked towards the riverfront in the evening

7 The child walked along the pavement and across the street safely

8 This table top is made of glass My breakfast fell off it in the morning

9 The pan is on the gas stove There are vegetables in it

10 We will wait for you at the bus top There are a lot of people in the hall

Subject ndash Biology Topic ndash Chapter - 3 Photosynthesis and respiration in plants Summary Execution

All living organism (Plants and animals) need food for energy and growth Green plants (autotrophy) prepare food for all living organisms Today we will discuss about the process photosynthesis And adaptations in a leaf to carry out photosynthesis

Q1What do you mean by photosynthesis and write its word equation The process by which green plants make food (glucose) from carbon dioxide and water

in the presence of sunlight and chlorophyll is called photosynthesis

Carbon dioxide + Water ( Sun light from Sun ) Glucose + Oxygen ( chlorophyll in green leaves )

Q2 What are the adaptations in a leaf to carry out photosynthesisi) Leaves are broad wide and flat for absorbing more light energyii) Presence of chlorophyll in chloroplasts to trap sunlightiii) Presence of stomata which allow carbon dioxide to enter the cell and oxygen to go

out iv) Network of veins ensures continuous supply of water and minerals to the leafv) Thin waxy cuticle protects the leaf without blocking the lightQ3 Draw and label structure of chloroplast

Class VIIISubject Topic Summary Execution

PHYSICS ENERGY Production of Hydro electricity

A hydroelectric dam converts the potential energy stored in a water reservoir behind a dam to mechanical energymdashmechanical energy is also known as kinetic energy As the water flows down through the dam its kinetic energy is used to turn a turbine

The generator converts the turbinersquos mechanical energy into electricity

This electric energy then goes through various transmission processes before it reaches you

Question 2

Fill in the blanks

(a) Work is said to be done by a forte only when the body moves

(b) Work done = Force x distance moved in direction of force

(c) The energy of a body is its capacity to do work

(d) The SI unit of energy is joule

(e) The potential energy is due to its state rest of position and kinetic energy of the body is due to its state of motion

(f) Gravitational potential energy U = mass times force of gravity on unit mass times height

(g) Kinetic energy = frac12 times mass times (speed)2

(h) Power P = work donetime taken

(i) The S I unit of power is watt

(j) IHP = 746 W

BIOLOGY Chapter -5 The endocrine system and adolescence

Today we will discuss about thelocation and functions of secreted hormones of adrenal and Pancreas

Q5 Write location hormone secreted main functions and deficiency diseases of pancreas and adrenal glands

Endocrine Glands

Location Hormones secreted

Functions and Deficiency Diseases

1Adrenal gland

2 Pancreas Gland

On the top of each kidney

In between stomach and small intestine

i)Adrenaline from adrenal medulla

ii)Cortisone from adrenal cortex

i) Insulin

ii) Glucagon

It helps a person deal with any kind of emergency situation or emotional stressIt increases the heart beat rate of respiration and blood pressure

a) It regulates carbohydrates protein and fat metabolism

b) It regulates the salt and water balance in the body

a) It changes excess glucose into glycogen

b) It stimulates the cells to burn extra glucose to provide heat amp energy

Less secretion causes diabetes mellitus

Excessive secretions causeinsulin shock

a) It stimulates the breakdown of glycogen into glucose

b) It increases the level of glucose in blood

History Traders to rulers The Battle of Buxar was fought on 22 October 1764 between the forces under the command of the British East India Company led by Hector Munro and the combined armies of Mir Qasim the Nawab of Bengal till 1763 Mir Jafar was made the Nawab of Bengal for a second time in 1763 by the Company just after the battle After being defeated in 4 battles in katwa and Udaynala the Nawab of Awadh Siraj id Daula and the Mughal emperor Shah Alam II accompanied by Raja Balwant Singh of Kashi made an alliance with Mir Qasim The battle was fought at Buxar a small fortified

Answer the following questions- Short note-Battle of BuxarHomework-learn

town within the territory of Bihar located on the banks of the Ganga river about 130 kilometres (81 mi) west of Patna it was a decisive victory for the British East India Company The war was brought to an end by the Treaty of Allahabad in 1765

EnglishLiterature

The west wind-John Mansfield

In the poem The West Wind by John Masefield the poet starts by describingwith very poetic imagery of birds how the west wind is different from other winds its a warm wind full of birds cries There is a touch of melancholy perhaps home-sickness as he describes how it brings tears too and memories from an old land He goes on to describe the restful pastoral beauty of the land where even the dead can lie in the green He then brings in voicesperhaps of family and friends calling him home as he is missing Aprils beautyThe voices then tempt him some more with idyllic images from home (white blossom young green cornrunning rabbitswarm sun) The voices seem to presume that the poets heart is sorrowful bruised and soreThe end of the poem sees the poet appear to make a decision he will go home as he has decided that is where he truly belongs

Write the synopsis of the following words

1 Daffodils- a tall yellow flower that grows in the spring

2 Orchards- a piece of land on which fruit trees are grown

3 Blossom- a flower or a mass of flowers especially on a fruit tree in spring

4 Thrushes- a bird5 Larks- a small brown bird that

makes a pleasant sound6 Bruised- an injury7 Aching- pain 8 Tread- to put your foot down

while you are walking9 Balm-10 May-11 Fluting-

(Write from the book in your copy)

MAT

HEM

ATIC

S

Ch 1

1Al

gebr

ic E

xpre

ssio

n

1 Constant A symbol which has fixed value is called a constant[eg 8 23 -15 radic3 etc]

2 VariableA symbol which does not have any fixed value but may be assigned value (values) according to the requirement is called variable or literal[eg x y p q etc]

3 TermsA term is a number (constant) a variable a combination (product or quotient) of numbers and variables[eg 7 x 5x etc]

4 Algebric expressionA single term or acombination of two or more terms connected by plus (+) or minus (-) sign forms an algebraic expression[eg 5-y 3x2-5x xy-6z+4 etc]

5 PolynomialAn algebraic expression which contains more than one term is called a polynomial (multinomial)[eg x2-5x 5y+xy+x2y etc]

6 Degree of polynomial(a) When the polynomial contains only one variable the highest power of the variable is the degree of the polynomialeg the degree of the polynomial of 4x-7x5+8 is 5(b) When the polynomial contains two or more variablesStep (i) Find the powers of the variables in each term (ii) The highest sum of the powers is taken to be the degree of the polynomialeg the degree of the polynomial 5x2y-4x3y5+6 is = 3+5 = 8Remember An algebraic expression is a polynomial if degree of each term used in it is a non-negative integer

Exercise ndash 11(A)

1 Separate the constants and variables from the following

-7 7+x 7x+yz radic5 radic xy 3 yz

8 45y -3x

Solution Constant Variables-7 radic5 7+x 7x+yz radic xy

3 yz8

45y -3x

2 Write the number of terms in each of the following polynomials(i) 5x2+3timesax (ii) axdivide4-7 (iii) ax-by+ytimesz (iv) 23+atimesbdivide2

Solution Polynomials Number of terms(i) 5x2+3timesax 2(ii) axdivide4-7 2(iii) ax-by+ytimesz 3(iv) 23+atimesbdivide2 2

4 Write the degree of the each polynomials(i) xy+7z (ii) x2-6x3+8 (iii) y-6y2+5y8 (iv) xyz-3 (vi) x5y7-8x3y8+10x4y4z4

Solution Polynomials Degree(i) xy+7z 2(ii) x2-6x3+8 3(iii) y-6y2+5y8 8(iv) xyz-3 3(vi)x5y7-8x3y8+10x4y4z4 12

5Write the coefficient of(i) ab in 7abx (iv) 8 in a2-8ax+a (v) 4xy in x2-4xy+y2

SolutionCoefficient

(i) ab in 7abx 7x(iv) 8 in a2-8ax+a -ax(v) 4xy in x2-4xy+y2 -1

7 CoefficientAny factor of an algebraic quantity is called the coefficient of the remaining quantityeg in the algebraic term 7xyz 7 is coefficient of xyz 7x is coefficient of yz and so on

8 Like term The terms having the same literal coefficient are called like terms and those having different literal coefficients are called unlike terms

eg (i) 5xyz 8xyz -6xyz and 23xyz are like

terms(ii) 7xy2 8x2yz and -15xyz2 are unlike terms

6 in 57xy2z3 write the coefficient of

(i) 5 (vii) 5xy2 (viii) 17yz (xi) 5xyz

Solution Coefficient

(i) 5 17

xy2z3

(vii) 5xy2 17z3

(viii) 17yz

5xyzsup2

(xi) 5xyz 17yz2

7 In polynomial given below separate the like terms(ii) y2z3 xy2z3 -58x2yz -4y2z3 -8xz3y2 3x2yz and 2z3y2

Solution y2z3 -4y2z3 2z3y2 are like terms

xy2z3 -8xz3y2 are like terms

-58x2yz 3x2yz are like terms

Class IXSubject Topic Summary Execution

Bengali (2nd language)

বাগzwnjধারাzwnj বা ধারা-বা ধারা ল হিবকেশষ পরকার বাক -হিb -াকেবর এক হিবকেশষ পরকাশরীহিত াকেক কতগকেলা কার সমষটির মকেধয এগহিলকেক বা ধারা বকেল আবার কতগকেলা শকেবদর বাধাধরা যকান রীহিত যনই য-াকেব চকেল আসকে যসই -াকেবই চকেল আসকে তখন যসই শবদগহিল খন একক -াকেব অG পরকাশ ককের তখন একের বা ধারা বকেল বা ধারার পরকেয়া -াষাকেক আরও সFর ককের যতাকেল

অকাল পকক(অপহিরনত বয়কেস পাকাহিম)-মাতর শ বর বয়কেস যমকেয়টির া মকেখর কা তাকেত অকালপককতা ধরা পকে

অককা পাওয়া( মারা াওয়া) ndash পকেকIমারটি পকেকIমারকেত হিকেয় বাসাতরীকের াকেত মার যখকেত যখকেত অককা যপল

অহি| পরীকষা ( কঠিন ও পরকত পরীকষা)- যকেলটির আজ ডাকতাহির যরজালট যবকেরাকেব এIাই তার জীবকেনর ব অহি| পরীকষা

অষটরমভা (ফাহিক) ndash রীতা মকেখই বকো বকো কা বকেল আর কাকেজর যবলায় অষটরমভা

অকমGার ধাী (অপাG) ndash সমনকেক হিনকেয় যকান ান কেব না ও একেকবাকেরই অকমGার ধাী

অকেনধর ষটি (অসাকেয়র সায়)- আহিশ বকেরর বকোর নাহিত ল অকেনধর ষটি তাকেক াা বকোর একম চকেল না

আকেককল গড়ম (তবহিদধ)- ার তহিম উপকার করকেল যসই যতামার হিবরকেদধ সাকষয হিকেয়কে শকেনই আমার আকেককল গড়ম

আষাকে লপ( অবাসতব লপ) ndashIাকা এখন যকেব না এIা বলকেলই ত এমন আষাকে লপ ফাার যকান রকার হিল না

Hindi- महायजञ ा इस हानी म लख न या बतान ा परयास किया ह कि किसी भी अचछ

2nd language

परसार(यशपाल ाय या पणय न ा फल अवशय मिमलता ह ोई भी परोपार अथवा पणय लिलए किया गया ाय बार नही 0ाता वह ए परार ा यजञ हए धनी सठ थ धम परायण और किवनमर सठ न आन ी यजञ किए थ और दान म न 0ान कितना धन दिदन दखिखयो म बात दिदया थादिदन पलट और सठ यहा गरीबी आ गई उन दिदनो यजञ बचन ी परथा थी सठ भी अपनी 0गह बचन लिलए डलपर ए सट यहा चलन ो तयार हए सठानी रासत लिलए रोटी पड म बाधर सठ ो द दी रासत म ए भख R ो दखर सठ न चारो रोटी उसो खिखला दी खर वह सठ यहा डलपर पहच तो उनी सठानी न उस महायजञ बचन ो हा यदिद बचन आए सठ न R ो रोटी खिखलान ो महायजञ नही समझा और वापस लौट आया घर आर शाम ो उसी घर म उस ए बडा ख0ाना मिमला 0ो उस दवारा किए गएrsquo महायजञrsquo ा परसार था

English language

Letter formal The heading the name and address of the person you are writing to must be included beneath your own address In formal letters ldquoblock stylerdquo of address is preferred

Subject complain in brief

Salutation If the person you are writing to is known to you you may begin ldquoDear MrrdquoOr ldquoDear Mrsrdquo In all other instances you should begin ldquoDear Sirrdquo or ldquoDear Madamrdquo Or ldquoSirsrdquo

The body A formal or business letter has four partsReference The letter should begin by referring to a letter you have received an advertisement or the reason that has prompted you to writeInformation In the second paragraph it is necessary to supply more detailed information that is related to the referencePurpose Here you must give the reason why you are writing the letter This must be stated clearly and ensure that it is relevant to the question that has been setConclusion round off the letter with some polite remarkThe subscription when a letter has begun with dear sir sirs Madam you should end with Yours faithfully or yours truly When however you address a person by name you must conclude with the words ldquoYours sincerelyrdquo

1 A park in your locality is slowly being used as a rubbish dump Write a letter to the Mayor of your city pointing out the nuisance and danger of this Request that action be taken to stop this immediately

Or2 You being a boarder ordered a set of lab manuals from a famous book shop in the town They sent you a wrong set of books Write a letter to the manager of the book shop

Chemistry Chapter-1 1)CHEMICAL FORMULA- Q What is the Significance of

L-2The Language of Chemistrybull Chemical Formula

Itrsquos a symbolic representation of a chemical substance eg ndash The formula of Sulphuric acid is H2SO4

2) Steps of writing Chemical Formula of a given substance-

1 Write the symbols of the constituent atoms or radicals side by side Keep the basic radical on LHS and acid radical on the RHS ( Na+Cl- )2 In case of a radical having more than one atom( compound radical) enclose the radical in a bracket eg (SO4-)3 Write the valencies of each radical on its right hand top4 If the valencies of the two radicals are divisible by a common factor then divide the valencies by the common factor5 Invert (criss-cross) the valency number ie write the valency of one atom below the second atom and vice versa 6 On interchanging if valency number is lsquoone the figure lsquoonersquo is never writtenFor Example- Compound -Calcium Nitrate1 Writing the symbols- Ca(NO3)2 Writing the valencies on their right hand top- Ca2(NO3)1

3 Valency numeral in simple ratio- Ca2(NO3)1

4 Criss-cross- Ca 2NO3 1

5 Writing the formula of the compound- Ca(NO3)2

Chemical formula

A The formula of a substance conveys the following information regarding a substance 1 The name of the substance (qualitative)2 The elements constituting the substance (qualitative)3 The number of various atoms present in a molecule of the substance (quantitative)4 Molecular weight of the substance and the relative weights of different elements present in it (qualitative)

Q What are the limitations of Chemical Formula

A The chemical formula suffers from the following limitations-I It fails to convey whether the elements in a molecule are present in the form of atoms or ionsFor example the formula KBr fails to tell us whether Potassium and Bromine are present in the form of ions II It does not tell anything about the binding force that holds atom in a molecule togetherIII It does not tell us about the arrangement of various atoms with respect to one another within the molecule

Q Examples of Some Chemicals with their Formula Chemical name and Common Name-

A Given in the class notesCommercial Studies

Joint Stock Company

Let us discuss about the demerits of Joint Stock CompanyDespite so many advantages it has got many disadvantages which are as follows

Difficulty in FormationDelay in Decision makingExcessive Government ControlLack of Secrecy

Company can be classified into several categories based on incorporation

QuestionExplain the demerits of Joint Stock CompanyAnswer) 1 Difficulty in Formation The legal requirements and formalities required to be completed are so many The cost involved is quite heavy It has to approach large number of people for its capital It cannot start its business unless certificate of incorporation has been obtained This is granted after a long time when all the formalities are completed

Chartered CompanyStatutory CompanyRegistered Company

Delay in Decision making In this form of organization decisions are not made by single individual All important decisions are taken by the Board of Directors Decision-making process is time-consuming So many opportunities may be costly because of delay in decision-making Promptness of decisions which is a common feature of sole trader ship and partnership is not found in a company

Excessive Government ControlA company and the management have to function well within the law and the provisions of Companies Act are quite elaborate and complex At every step it is necessary to comply with its provisions lest the company and the management should be penalized The penalties are quite heavy and in several cases officers in default can be punished with imprisonment This hampers the proper functioning of the company

Lack of Secrecy The management of companies remains in the hands of many persons Every important thing is discussed in the meetings of Board of Directors Hence secrets of the business cannot be maintained In case of sole proprietorship and partnership forms of organisation such secrecy is possible because a few persons are involved in the management

2 Define the following

Chartered Company- The crown in exercise of the royal prerogative has power to create a corporation by the grant of a charter to persons assenting to be incorporated Such companies or corporations are known as chartered companies Examples of this type of companies are Bank of England (1694) East India Company (1600) The powers and the nature of business of a chartered company are defined by the charter which incorporates it After the country attained independence these types of companies do not exist

in IndiaStatutory Company- A company may be incorporated by means of a special Act of the Parliament or any state legislature Such companies are called statutory companies Instances of statutory companies in India are Reserve Bank of India the Life Insurance Corporation of India the Food Corporation of India etc The provisions of the Companies Act 1956 apply to statutory companies except where the said provisions are inconsistent with the provisions of the Act creating them Statutory companies are mostly invested with compulsory powersRegistered companiesCompanies registered under the Companies Act 1956 or earlier Companies Acts are called registered companies Such companies come into existence when they are registered under the Companies Act and a certificate of incorporation is granted to them by the Registrar

Economics

Chapter-4Basic problems of Economy

Today let us discuss with the topic Production Possibility curve

QuestionExplain the concept of Production Possibility Curve with the help of diagram

Answer) Production Possibility curve is a locus of all possible combinations of two commodities which can be produced in a country with its given resources and technology

The above diagram shows that with the given resources and technology the economy can produce maximum either 5 thousand meters of cloth or 15 thousand quintals of wheat or any other combination of the two goods like B( 1 thousand meters of cloth and 14 thousand quintals of wheat C ( 2 thousands meters of cloth and 12 thousand quintals of wheat) etcProduction Possibility curve is also called production possibility boundary or frontier as it sets the maximum limit of what it is possible to produce with given resources

Geography

Rotationand Revolution

SUNrsquoS POSITION AND SEASONAL CHANGES EQUINOXES ndash SPRING AND AUTUMN

Q1 What is Spring EquinoxA1 On 21st March sunrays fall directly on the equator On that day

As the Equator divides the Earth into two equal halves the sun rays fall directly on the equator twice in a year Equinoxes means equal Spring EquinoxOn 21st March sunrays fall directly on the equator On that day the duration of day and night both are equal ( 12 hours day and 12 hours night) on every places located on equator This day is called as Spring EquinoxAutumn EquinoxOn 23rd September sunrays fall directly on the equator On that day the duration of day and night both are equal ( 12 hours day and 12 hours night) on every places located on equator This day is called as Autumn Equinox

SOLSTICES ndash SUMMER AND WINTERDue to inclination of the Earth on its axis and the apparent movement of the sun the sun rays fall directly on both tropics once in a year Solstice is a Latin word which mean ldquothe Sun standing stillrdquoSummer SolsticesAfter 21st March there is an apparent movement of the Sun to the north of the equator The apparent northward movement up to 21st June when the Sun appears overhead at the Tropic of Cancer (22frac12degN) The sun appears to stand still at this position and then moves southwards towards the equator This position of the Sun on 21st June is known as Summer Solstices On that day the duration of day and night both are equal ( 12 hours day and 12 hours night) on every places located on Tropic of Cancer (22frac12degN)Winter solstices The apparent southward movement of the Sun continues beyond the equator till 22nd

December On this day the Sun is overhead at the Tropic of Capricorn

the duration of day and night both are equal ( 12 hours day and 12 hours night) on every places located on equator This day is called as Spring Equinox

Q2 What do you mean by EquinoxA2 Equinoxes means equal It is use to explain the equal duration of day and night ( 12 hours day and 12 hours night) on the Earth

Q3 On which date the longest day in Tropic of CancerA3 21st June

Q4 What is the meaning of SolsticeA4 Solstice is a Latin word which mean ldquothe Sun standing stillrdquo

Q5 Which is the longest day in southern hemisphereA5 22nd December

Q6 On what date does the Arctic Circle experience the lsquoMidnight SunrsquoA6 On 21 June the Arctic Circle experiences the lsquoMidnight Sunrsquo

Q7 What is cause of Midnight Sun in NorwayA7 During the summer solstice (21 June) the North Pole is inclined towards the Sun Therefore the duration of sunlight or daytime increases from 12 hours at the Equator to 24 hours at the Arctic Circle and beyond Thatrsquos why The region beyond the Arctic Circle especially Norway is known as the Land of the Midnight Sun because there the Sun does not rise or set on 21 June

Q8 Match the column A with BA B

Summer Solstice 21st March

Autumn Equinox 23rd

September

Winter Solstice 21st June

(22frac12degS) This position of the Sun is referred to as the Winter Solstice because it marks the winter season in the Northern Hemisphere On that day the duration of day and night both are equal ( 12 hours day and 12 hours night) on every places located on Tropic of Capricorn (22frac12degS)SEASONS AND DURATION OF DAY AND NIGHT During the equinoxes all places on the Earth have 12 hours of day and 12 hours of night Due to the revolution of the Earth round the Sun on an inclined axis the duration of day and night varies according to seasons and the latitude of a placeDuring the summer solstice (21 June) the North Pole is inclined towards the Sun Therefore the duration of sunlight or daytime increases from 12 hours at the Equator to 24 hours at the Arctic Circle and beyondThe region beyond the Arctic Circle especially Norway is known as the Land of the Midnight Sun because there the Sun does not rise or set on 21 JuneAt the North Pole there will be six months of daylight The Sun will be seen always above the horizon at a low angle At 66degN 24 hours of sunlight can be seen only on 21 June Hammerfest in northern Norway is a place of tourist attraction for observing the phenomenon of the Midnight Sun This place has continuous daylight from 13 May to 29 July This place is easily accessible to tourists and has hotels and other facilities The view of the midnight Sun from here is enthrallingIn the Southern Hemisphere the duration of daylight decreases from 12 hours at the equator to 0 hours beyond the Antarctic Circle In the South Polar Region there is 24 hours of darkness The Sun is always below the horizon In the Southern Hemisphere which experiences winter the duration of night-time is longer than the duration of daylight

Spring Equinox 22nd

December

A8 A B

Summer Solstice 21st June

Autumn Equinox 23rd

September

Winter Solstice 22nd

December

Spring Equinox 21st March

During winter solstice (22 December) the South Pole is inclined towards the Sun The Southern Hemisphere experiences summer and the Northern Hemisphere has winter Therefore the duration of daylight or sunlight is greater in the Southern Hemisphere than in the Northern HemisphereThe duration of daylight increases from 12 hours at the equator to 24 hours beyond the Antarctic Circle The South Polar Region has 24 hours of sunlight for many days continuously At the South Pole there will be six months of sunlight The Sun will always be seen at a low angle above the horizon In the Northern Hemisphere the duration of daylight will decrease from 12 hours at the equator to 0 hours at the Arctic Circle There are 24 hours of darkness in the North Polar region The duration of night is greater than the duration of daylight as one move northwards from the Equator It is evident from the above table that the duration of daylight is 12 hours throughout the year at the equator only As one moves away from the equator the seasonal variations in the duration of daylight increase The seasonal variations in the duration of daylight are maximum at the Polar Regions

Subject Eng Literature (The Merchant of Venice ndash William Shakespeare)Topic Act II Scene 7 Lines 36 to 80 (End of scene ) [Students should read the original play and also the paraphrase provided]

Summary Questions amp AnswersThe Prince then examines the inscription on the silver casket which says ldquoWho chooseth me shall get as much as he deservesrdquo The Prince says that he deserves Portia more than anybody else because of his high rank his noble birth and his great wealth and power But then he argues that silver is ten times

(1) (Act II Sc 7 L 39-47)

From the four corners of the earth they come

To kiss this shrine this mortal breathing saint

The Hyrcanian deserts and the vasty wildsOf wide Arabia are as through-fares now

inferior to gold and therefore he cannot believe that the portrait of such a beautiful lady as Portia can be contained in the silver casket He decides to see the inscription on the golden casket before making his decision

The Prince goes to examine the inscription on the golden casket which says ldquoWho chooseth me shall get what many men desirerdquo The Prince believes that the whole world desires to possess Portia otherwise so many suitors would not have come from all corners of the world for winning Portia Some of them have come from the distant lands of Persia and Arabia The deserts of Persia (Hyrcanian deserts) and the boundless desolate lands of Arabia have been crossed by the Princes seeking the hand of Portia He contrasts this casket containing Portiarsquos portrait with the old English gold coin bearing the image of the archangel (angel of the highest rank) He goes on to remark that while the figure of the archangel is engraved (Insculped) upon the English coin the picture of Portia who is beautiful as an angel lies hidden inside one of the caskets namely the Golden Casket (Golden Bed)

On the basis of his assessment of the inscription on the golden casket the Prince decides to choose the golden casket He asks for the key and opens the golden casket only to find therein an empty human skull holding a roll of

For princes to come view fair PortiaThe watery kingdom whose ambitious headSpets in the face of heaven is no barTo stop the foreign spirits but they comeAs orsquoer a brook to see fair Portia

(i) Explain the occasion for the above mentioned speech

These are the comments of the Prince of Morocco after he reads the inscription on the golden casket His mental process is revealed to us in these words We find him debating within himself as to which casket he should choose

(ii) What light does the above speech throw on the personality of Prince of Morocco

From the above mentioned speech we come to know that the Prince of Morocco is keen to marry Portia He is the type of person who is easily taken away by outward appearance He is in love with Portia because of her beauty

(iii) What information can you gather about Portia from the above mentioned lines

The given speech shows that Portia is a very beautiful lady She must be possessed of good qualities because many suitors come to her place from all over the world with a desire to get married to her The Prince of Morocco is so impressed by her beauty that he calls her a saint According to him the whole world is desirous of having her

(iv) Elucidate the significance of the first two lines

In these lines the Prince of Morocco pays a compliment to Portia These lines show his admiration for her He says that people come from all parts of the world to see fair Portia

(v) Explain the meaning of the last four lines of the

passage

In these lines the Prince of Morocco says that even the vast oceans which throw a challenge at the sky are unable to prevent men from coming to Portiarsquos place to have a glimpse of her These lines are also a tribute to Portiarsquos beauty and good qualities Many men voyage across the ocean treating it as a mere stream to see the beautiful Portia

paper in which is written that whoever happens to be guided by the glitter of things is invariably deceived

On reading the scroll the Prince says that he is too sad at heart to speak a more formal farewell and leaves with his followers amidst a sound of trumpets

After the Prince of Morocco leaves Portia remarks that the Prince is a gentle fellow but she is rid of him May all persons of his nature make a similar choice

IMPORTANT PASSAGES EXPLAINED

(Act II Sc 7 L 39-43)From the four corners of the earth they come

To kiss this shrine this mortal breathing saintThe Hyrcanian deserts and the vasty wildsOf wide Arabia are as through-fares nowFor princes to come view fair Portia

Context

This passage occurs in Act II Scene 7 in The Merchant of Venice This is part of the speech made by the Prince of Morocco

(2)

(Act II Sc 7 L 48-53)

MOROCCO One of these three contains her heavenly pictureIst like that lead contains her

Twere damnation To think so base a thought it were too grossTo rib her cerecloth in the obscure graveOr shall I think in silver shes immurdBeing ten times undervalued to tried gold

(i) What meaning does the Prince of Morocco find out of the inscription of the golden casket What have Belmont and Portiarsquos house been called and why

The inscription on the golden casket is ldquoWho chooseth me shall gain what many men desirerdquo The Prince finds out that it means that the chooser of the golden casket will get Portia because many men desire her In fact the entire world desires her Because of the coming of many suitors to Belmont from different countries in order to win Portiarsquos hand Belmont has become a centre of pilgrimage and her house is the shrine where saintly Portia is installed

(ii) What does the Prince of Morocco do before making the final choice of the casket Which is the correct casket and who will win Portiarsquos hand

The Prince of Morocco surveys and analyses the inscriptions on the casket of lead silver and gold Before making the final choice like a very systematic and methodical person he once again considers the claims of the caskets The casket containing Portiarsquos picture is the correct casket and the person choosing it will win Portiarsquos hand

Explanation

While praising Portia the Prince of Morocco conceives Portia as a goddess whose image is placed inside one of the caskets Many suitors are coming from far and wide the north and the south the east and the west (Four corners) in order to try their luck Some of them have come from the distant land of Persia and Arabia The deserts of Persia (Hyrcanian deserts) and the boundless desolate lands of Arabia have been crossed by the Princes seeking the hand of Portia All this shows that Portia is indeed the most beautiful lady of the world

(iii) What does the Prince of Morocco say in his estimation while examining the motto on the silver casket What does he find in the golden casket

While examining the motto on the silver casket which says ldquoWho chooseth me shall get as much as he deservesrdquo Morocco says that in his own estimation he surely deserves Portia in all respects ndash rank birth wealth etc

He chooses the golden casket When he opens it he finds an empty human skull holding a scroll in which it is written that those who are attracted by the glittering outside of things are always deceived as Morocco has been deceived

(iv) What kind of nature does the Prince of Morocco have

The Prince of Morocco has a simple nature who does not look deeply into the inner meaning of things but is dazzled by the outward appearance of gold He is inclined to over-estimate his own value and does not realize that it is a duty to ldquogive and hazardrdquo To say that he will not hazard for lead shows that he misreads the true meaning of the inscription which is that he should be prepared to ldquohazard all he hathrdquo for Portia So his feeling is only one of fascination and romantic attraction

(v) Do you think that the lottery of the caskets is not a matter that will be determined by chance

In fact the lottery of the casket is not a matter that will be determined by mere chance but that it is a true test of character and of sincerity which is amply proved not only by Moroccorsquos choice but also by the arguments which he uses to help him in his choice

(Act II Sc 7 L 55-59)

They have in England

A coin that bears the figure of an angelStamped in gold but thats insculpd uponBut here an angel in a golden bedLies all within

Context

(3)

(Act II Sc 7 L 63-77)A carrion Death within whose empty eye

There is a written scroll Ill read the writing

All that glisters is not goldOften have you heard that toldMany a man his life hath soldBut my outside to beholdGilded tombs do worms infoldHad you been as wise as boldYoung in limbs in judgment oldYour answer had not been inscrolld

This passage occurs in Act II Scene 7 in The Merchant of Venice This is part of the speech made by the Prince of Morocco

Explanation

In this passage the Prince of Morocco bestows high praise on Portia whose hand he is seeking He contrasts this casket containing Portiarsquos portrait with the old English gold coin bearing the image of the archangel (angel of the highest rank) He goes on to remark that while the figure of the archangel is engraved (Insculped) upon the English coin the picture of Portia who is beautiful as an angel lies hidden inside one of the caskets namely the Golden Casket (Golden Bed) In the day of Elizabeth silver was ten times inferior in value to gold Therefore the Prince of Morocco believing that Portiarsquos portrait is contained in the Golden Casket decides to choose the Golden Casket

Fare you well your suit is coldCold indeed and labour lostThen farewell heat and welcome frostmdashPortia adieu I have too grievd a heartTo take a tedious leave Thus losers part

(i) What reward does the Prince of Morocco get after making a wrong choice of the Casket How does he feel

After making the wrong choice in selecting the casket of gold the Prince of Morocco as a reward earns a rebuke in the form of a scroll tucked in the empty eye-socket of a skull kept in the casket of gold The Prince is shocked and disappointed He becomes all the more sad and dejected when he reads the scroll which points to his foolishness in being misled by the appearance and outward show as indicative of its worth

(ii) How does the Prince respond after reading the scroll

After reading the scroll the Prince though upset accepts the result with good grace and decorum befitting a royal suitor and true sportsman He says that his love-suit is really cold otherwise he would have chosen correctly but now his efforts have been in vain So he bids farewell to Portia to the warmth and enthusiasm of love and welcomes the cold and bitterness of dejection and misery of life which lies ahead

(iii) What request does he make to Portia and why

After being failure in his mission he requests Portia to give him permission to leave at once because he is too sad to undergo the tediousness of a formal leave-taking He tells that it is the manner in which defeated persons part unceremoniously

(iv) Explain the following lines

ldquoAll that glisters is not goldOften have you heard that toldMany a man his life hath soldBut my outside to beholdGilded tombs do worms infoldrdquo

Mere glitter does not make a metal to be gold Man has often been warned against appearance but it has been of no use Many people have sacrificed their lives only to seek the outer appearance of gold Worms are found inside the gilded

monuments

Class XSubject Topic Summary Execution

Hindi 2ndlang

नया रासता भाग 6 मायाराम 0ी घर म धनी मल 0ी और उनी बटी सरिरता ी ही चचा बनी रहती थी अमिमत ो इसम ोई रलिच ना थी वह धनी घर ी लडी स शादी र सवय ो बचना नही चाहता था उसा भी सवाणिभमान ह ईशवर ी पा

स उस पास पस ी ोई मी नही थी अभी उसन फकटरी ही लगाई थी उसी समझ बाहर था कि उस घर वालो ा झाव पस ी तरफ कयो

ह उसन मा स सवाल किया कि मा तम सरिरता स मरी शादी कयो रना चाहती हो मा न उस समझाया कि वह दखन म बरी नही ह और किफर खानदान अचछा

ह वह ए शल गरहणी रप म घर सभाल सगी अमिमत न मा ो इस बात ा एहसास राया कि मीन सबध लिलए मना रन पर उस दिदल

पर कया बीती होगी मा और अमिमत ी लडी बार म ाफी बात हईमा ा झाव सरिरता ी तरफ था कयोकि वह घर पर अचछा दह0 लर आ रही

थी अमिमत न अपनी मौसी ी बरी हालत बार म बताया कि किस तरह वह बड घर ी खानदानी बटी लाई थी और आ0 उसी हालत कितनी खराब ह लाई थी बहकलब 0ाती ह और बचचो ो भी नही दखती ह बात चल ही रही

थी कि तभी ए ार बाहर आर री धनी मल0ी घर अदर आए और पीछ स डराइवर फल ी ए टोरी लर आया अदर आए और पीछ स

डराइवर ए टोरी फल ी लर आया अमिमत ो फल ी पटी बरी लग रही थी अमिमत न पछ लिलया यह फल कयो ल आए ह प इन सब ी कया

0ररत थी उनो न 0वाब दिदया कि 4 पटी शमीर स मगाए थ अमिमत ो या सनर करोध आ गया तभी उस किपता 0ी आ गए उन आत ही अमिमत उठर बाहर चला गया वहा वहा मा पास आर बठ गया और बोला

अभी रिरशता तय नही हआ और धनी मल 0ी धनी मल 0ी फल ी पटी लर चलआय मा न समझाया कि 0ब सबध 0ड 0ाता ह तो खाली हाथ नही

आत अमिमत न मा स हा कि तम सबन सरिरता ो इस घर म लान ी ठान रखी ह धनीमल 0ी उस दिदन सरिरता ो दखन ी तारीख तय रन आय थ

Commercial Studies

Banking Nowadays Bank provide easy and quick services through internet facilities methods of Banking is called internet bankingIn order to save the time and money involved in visiting Bank branches people increasingly prefer to have internet banking

There are different modes of doing internet banking or transferring money through online They areReal Time Gross Settlement (RTGS)National Electronic Fund Transfers (NEFT)

1

Question

1) Explain the term RTGS Write the features of RTGS

Answer)The acronym RTGS stands for Real Time Gross Settlement which may be defined as the continuous real time settlement of funds transfer individually on and order by order basis without netting lsquoReal timersquo may be defined as the processing of instructions at the time they are received rather than at some letter time lsquoGross settlementrsquo may be defined as the settlement of transfer instructions which occurs

individually

Features of RTGS1It is the continuous settlement of

funds transfer individually on an order by order basis

2RTGS facility is provided only by CBS core banking solution enabled Bank branches

3Amount charged from the customer for RTGS transactions vary from bank to bank

2) Explain the term NEFT Write the features of NEFT

Answer) National electronic funds transfer may be defined as a nationwide system that facilitates individuals Farms and copper operates to electronically transfer funds from any bank branch to any individual farm or corporate having an account with any other bank branch in the country

Features of NEFT2 Transfer can be made 7 times on

weekdays and 6 times on Saturday

3 NEFT cannot be used to receive foreign remittances

4 NEFT transaction takes place in batches

5 A bank branch must be NEFT enabled to become a part of NEFT fund transfer network

6 There is no maximum or minimum amount that can be transferred through NEFT when one bank has a bank account

English Language

CompositionEssay

A composition is an art of creating a piece of writing on any topic or subject It is the writing correctly beautifully and clearly in order to make some interesting reading Structure of the composition

Introduction ( you lay the foundation for your composition)

Body (it constitutes the main part of the essay)

Conclusion (final statement that leaves a lasting impression)

Kinds of essays1 The Narrative essay2 The descriptive essay3 The reflective essay4 The argumentative essay

Write a composition on any one of the following topics (350- 400 words)

1 Friendship Or2 The first day of your school

Subject Eng Literature (The Merchant of Venice ndash William Shakespeare)Topic Act V Scene 1 Lines 127 to 158 (Nerissa helliphellip The clerk will nersquoer wear hair onrsquos face that had it) [Students should read the original play and also the paraphrase given in the school prescribed textbook]

Summary Revision Questions o Soon thereafter Bassanio Gratiano

and Antonio arrive

o Bassanio tells Portia that he is feeling as if it is morning because of the presence of Portia who is shining like the sun When Antonio is introduced by Bassanio to Portia she tells Bassanio that he should be grateful to Antonio who took so much trouble on his account even to the extent of risking his life

o Nerissa starts quarrelling with Gratiano and demands that he show her the ring she had presented to him and which she had warned him not to lose She suspects that Gratiano must have presented the ring to some young woman and not to the lawyerrsquos clerk as he repeatedly says and assures

Answer the following questions to check your preparation of Act IV Scenes 1 and 2

You must attempt only after you have completed your preparation of Act IV The answers must be in complete sentences using textual evidence (with citation) when necessary

[It would be in your own interest to attempt the above questions honestly totally refraining from consulting your textbook or your notes during answering After completion you should correct the paper yourself consulting the textbooknotes etc and award marks as specified Please let me know the marks you scored through WhatsApp in the group or to my personal WhatsApp]

Act IV Scene 1 (each question carries 2 marks)

1 What did the Duke try to do for Antonio

2 Why does Shylock refuse to show mercy How does he justify his stance

3 Why does Antonio say he is ready to die 4 What information is contained in Bellariorsquos letter

5 Why does Portia (as Balthazar) assert that Shylock must show mercy How does he respond

6 What offers are made to Shylock to get him to spare Antonio How are they received

7 What does Antoniorsquos speech as he faces the prospect of Shylockrsquos knife tell you about his character

8 How do Bassanio and Gratiano react to the looming prospect of Antoniorsquos demise

9 How does Portia (as Balthazar) use the law to turn the tables on Shylock

10 What does the Duke decree should happen to Shylock Why What happens to Shylockrsquos estate

11 What does Portia ask Bassanio as payment for her ldquoservicesrdquo What is his initial response What makes him change his mind

Act IV Scene 2 (each question carries 1frac12 marks)

1 What does Gratiano bring to Portia (Balthazar)

2 What does Nerissa plan on getting from Gratiano What does Portiarsquos comment suggest about men

ECO-10 280620 Topic-Supply AnalysisSHIFTING OF SUPPLY

But if there is change in factors other than the price of the commodity then either more is supplied at the same price or less supplied at the same price In such cases the price of the commodity remains constant but there is a change in other factors like change in the price of inputs change in technology of production change in price of other related goods change in taxation policy of the government etc For example there is an improvement in the technology of production of the commodity in question It leads to decrease in per unit of cost production of the commodity The firm is willing to sell more quantity of the commodity at the same price So the supply other commodity increases at the same price This increase in supply is shown by rightward shift of supply curve On the other hand if the firm uses inferior technology of production the cost of production per unit of the commodity increases The firm is willing to sell less quantity at the same price So the supply of the commodity decreases at the same price This decrease in supply is shown by leftward shift of the supply curve The above cases of increase and decrease in supply can be shown with the help of the following figures

Y INCREASE OF SUPPLY Price (Rs) s

P A s1

B

s

X` O s1 X

q q1

Y` Quantity demanded (in units)

Y DECREASE IN SUPPLY s2

s

price (Rs)

C

p A

s2

s

X` o X

q2 q

Y` Quantity demanded ( in units)

Main factors causing increase in supply or rightward shift of supply Curve(i) Fall in the price of other related goods

(ii) Fall in the price of inputsfactors(iii) Use of better technology in production(iv) Decrease in the rate of excise duty by government(v) If the objective of producer changes from profit maximization to salesMaximization

Main factors causing decrease in supply or leftward shift of supply curve(i) Increase in the price of other related goods(ii) Rise in the price of inputsfactors(iii) Use of inferior technology in production(iv) Increase in the rate of excise duty by the government(v) If the objective

Subject - Biology Topic ndash Chapter mdash6 PhotosynthesisSummary Execution

Today we will know about photosynthesis and its stages

Q1 What do you mean by photosynthesis The process by which living plants containing chlorophyll produce food

substances from carbon-di- oxide and water by using light energy Sunlight

6CO2 +12 H2O----------------------- C6 H12O6 + 6H2O + 6O2

Chlorophyll

Q2 What are the importance of photosynthesis I) Food for all Green plants trap solar energy by photosynthesis

process and supply food and energy for all living organisms either directly or indirectly

Ii) Oxygen to breathe in by product of photosynthesis is oxygen which is essential for all living organisms respiration

Q3 Write about two main phases of photosynthesis A Light dependent phase This phase occur in grana of chloroplast I) The chlorophyll on exposure to light energy becomes activated by

absorbing photons Ii) The absorbed energy is used in splitting the water molecules (H2O)

into its two components (H+ and OH- ) and releasing electron s 2H2O------------------------- 4H+ + 4e- +O2

Energy of 4 photons This reaction is known as photolysis

End products are H+ and oxygen water

B Light independent (Dark ) phase The reactions in this phase require no light energy

Here CO2 combine with H+ and produce glucose

Class XI

Subject Topic Summary ExecutionEVS Chapter-4 Legal

regimes for sustainable development

Environmental legislationEnvironmental legislation is the collection of laws and regulations pertaining to air quality water quality the wilderness endangered wildlife and other environmental factors The act ensures that matters important to the environment are thoroughly

Learn -The Forest (Conservation) Act 1980

considered in any decisions made by federal agencies

The Forest (Conservation) Act 1980 The Forest (Conservation) Act 1980 an Act of the Parliament of India to provide for the conservation of forests and for matters connected therewith or ancillary or incidental thereto It was further amended in 1988 This law extends to the whole of IndiaObjects and Reasons of the Forest Conservation Act

Deforestation causes ecological imbalance and leads to environmental deterioration Deforestation had been taking place on a large scale in the country and it had caused widespread concern The act seeks to check upon deforestation and de-reservation of forests

Subject Eng Literature (The Tempest ndash William Shakespeare) Topic Act II Scene 1 Lines 314 to 329 (End of scene)

[Students should read the original play and also the paraphrase given in the school prescribed textbook]Summary Questions amp Answers

Conspiracy of Antonio and Sebastian (Contd)

o As they approach Ariel appears again and wakes up Gonzalo by singing a tune in his ear Alonso also wakes up and they see both Sebastian and Antonio with drawn swords On being caught off guard they make up a story saying that they had heard a bellowing of bulls or lions

o They then moved to another part of the island

o Ariel at once rushes to Prospero to inform him of this development

SUMMING-UP of ACT-2 SCENE-1

(i) Among the survivors Ferdinand is separated from the rest which results in the disconsolate grief of Alonso as he took him for dead

(ii) The villainy of Antonio is confirmed

(iii) The supremacy of Prosperorsquos magic which resulted in the failure of the human conspiracy

(1)

(Act II Sc 1 L 311-325)SEBASTIAN Whiles we stood here securing your repose

Even now we heard a hollow burst of bellowing Like bulls or rather lions Didt not wake youIt struck mine ear most terribly

ALONSO I heard nothingANTONIO O rsquotwas a din to fright a monsters ear

To make an earthquake Sure it was the roarOf a whole herd of lions

ALONSO Heard you this GonzaloGONZALO Upon mine honour sir I heard a humming

And that a strange one too which did awake meI shaked you sir and cried As mine eyes opened I saw their weapons drawn There was a noiseThats verily rsquoTis best we stand upon our guardOr that we quit this place Lets draw our weapons

(i) Why has Prospero sent Ariel to Gonzalo and Alonso What does Ariel do to awaken Gonzalo

Prospero has already come to know by his magic powers the danger which threatens Gonzalo who had been Prosperorsquos friend and so he sent Ariel to preserve the lives of both Gonzalo and Alonso Prospero does not want that his scheme should remain unfulfilled Ariel begins to sing a song in Gonzalorsquos ears to awaken him(ii) Who are ready to carry out their plan Who takes steps to stop them Why does Gonzalo feel surprised after being awakened

Sebastian and Antonio are ready to carry out their plans They are standing with their swords drawn to kill Alonso and

(iv) We see two sets of contrasting characters Gonzalo-Adrian against Antonio-Sebastian

(v) The grief that works in Alonso can be perceived to his repentance for his association in Antoniorsquos crime against Prospero

Gonzalo Ariel takes steps to stop them from carrying out their nefarious scheme When Gonzalo is awakened by the song sung by Ariel into his ears he (Gonzalo) feels surprised because he sees Sebastian and Antonio standing with their swords drawn(iii) What reason do Sebastian and Antonio tell of drawing their swords when they are suspected by Alonso and Gonzalo

When Sebastian and Antonio are seen with their swords drawn they are looked with suspicion by Gonzalo and Alonso At first Sebastian tells them that as they stood here to guard them during their sleep they heard only a little before a sudden loud noise very much like the roaring of bulls or more probably that of lions Then Antonio follows him saying that this was a noise so terrible as to frighten even a monsterrsquos ears and this noise could even have shaken the earth and it was surely like the roaring of a multitude of lions Then seeing the danger they have drawn their swords Perhaps after hearing the terrible noise they (Gonzalo and Alonso) woke up from their sound sleep

(iv) What does Gonzalo tell Alonso about the strange noise What did he see on opening his eyes Gonzalo tells Alonso that he did not hear the sound of roaring but he heard a humming sound which was strange and which woke him up After waking up he gave him (Alonso) a shaking and a loud cry On opening his eyes he saw these two gentlemen standing with their swords drawn(v) What does Gonzalo suggest

Gonzalo suggests that there was a noise indeed and of that he has no doubt at all and suggests that the best course for them would be to remain alert and vigilant against any possible danger to their lives or to leave this place and move to some other part of the island

Class XIISubject Topic Summary Execution

Commerce

Chapter- Management

Today we will discuss about LEVELS OF MANAGEMENT

Levels of management is a series or chain of managerial positions from top to bottom It helps individuals to know their authority responsibilities and superior-subordinate relations among themselves There are mainly three levels of Management TOP LEVEL MANAGEMENTMIDDLE LEVEL MANAGEMENTLOWER LEVEL MANAGEMENT

Top level managementIt consists of members at the highest level in the management hierarchy This level includes Board Of Directors Chief Executive Managing Directors Chairman President Vice President

Rolefunctions of the top levelmanagement1To analyse evaluate and deal

with theexternal environment2 To determine the objectives and

policies of the business3 To strive for welfare and survival

of business

4 To create an organisational Framework consisting of authority responsibility relationship

Middle level management Congress of members or groups who are concerned with implementation of the policies let down by the top managementThis level includes head of the department such as finance manager marketing manager branch and regional managers departmental and divisional heads plant superintendent etc

Role of functions of the middle level management

1 To interpret the policies framed by top management

2 To assign duties and responsibilities to lower level managers

3 To select and appoint employees for middle and supervisory level and evaluate their performance

4 To co-operate with other departments for smooth functioning

Operational or supervisory level managementIt refers to the group are members who are concerned with execution of the work They are also known as fast line managers This level includes supervisor 4 men Section Officer clerk Inspector etc

Role of functions of the lower level management1 To plan and execute day-to-

day operations2 To supervise and control the workers3 To arrange materials and

tools to start the process and make arrangements for training

4 Today present workers grievance and suggestions before the management and

ensure safe and proper working conditions in the factory

Business Studies

Staff Appraisal Chapter- 10 Today let us start with a new chapter

Staff Appraisal

Meaning of Performance Appraisal

Performance Appraisal is the systematic evaluation of the performance of employees and to understand the abilities of a person for further growth and developmentThe supervisors measure the pay of employees and compare it with targets and plansThe supervisor analyses the factors behind work performances of employeesThe employers are in position to guide the employees for a better performance

Objectives of Performance Appraisal

Following are the objectives of Performance Appraisal

To maintain records in order to determine compensation packages wage structure salaries raises etc

To identify the strengths and weaknesses of employees to place right men on right job

To maintain and assess the potential present in a person for further growth and development

To provide a feedback to employees regarding their performance and related status

To provide a feedback to employees regarding their performance and related status

Importance of Performance Appraisal

Performance appraisal provides important and useful information for the assessment of employees skill

knowledge ability and overall job performance The following are the points which indicate the importance of performance appraisal in an organization

1 Performance appraisal helps supervisors to assess the work performance of their subordinates

2 Performance appraisal helps to assess the training and development needs of employees

3 Performance appraisal provides grounds for employees to correct their mistakes and it also provides proper guidance and criticism for employees development4 Performance appraisal provides reward for better performance

5 Performance appraisal helps to improve the communication system of the organization

6 Performance appraisal evaluates whether human resource programs being implemented in the organization have been effective

7 Performance appraisal helps to prepare pay structure for each employee working in the organization

8 Performance appraisal helps to review the potentiality of employees so that their future capability is anticipated

Geography

DRIANAGE The SubarnarekhaThe Subarnarekha and the Brahmaniinterposed between the Ganga and the Mahanadi deltas drain an area of 19300 sq kmand 39033 sq km respectively The drainage basins of these streams are shared byJharkhand Odisha west Bengal and Chhattisgarh The Brahmani is known as southKoel in its upper reaches in Jharkhand

The NarmadaThe Narmada rises in the Amarkantak hills of MadhyaPradesh It flows towards the West in a rift valleyformed due to a geological fault The total length of it is 1300 km All the tributaries of the

Q1 Name the two westward flowing rivers in the peninsular plateauA1 Narmada and Tapi are the only westward flowing rivers of the peninsular plateau

Q2 Differentiate between east-flowing rivers and west-flowing riversA2

East-flowing rivers

West-flowing rivers

Narmada are very short inlength Most of its tributaries join the main streamright anglesThe Narmada basin covers parts of Madhya Pradesh and Gujarat

The Tapi The Tapi rises in the Satpura ranges in the Betul listrictof Madhya Pradesh It flows in a rift valley parallel tothe Narmada but it is much shorter in length It coversparts of Madhya Pradesh Gujarat and MaharashtraThe length is about 724 km

The Sabarmati and the MahiThe Sabarmati rises in the Aravali hills and flows south-south-westwards for a distance of 300 kilometres to the Arabian Sea The Sabarmatibasin extends over an area of 21674 sq km in Rajasthan and Gujarat The Mahi rises inthe east of Udaipur and drains an area of 34842 sq km lying in Madhya PradeshRajasthan and Gujarat It flows south-westwards for a distance of 533 km before it fallsinto the Gulf of Khambhat

The ChambalThe Chambal rises near Mhow in the Vindhya Range and flows towards the northgenerally in a gorge upto Kota Below Kota it turns to the north-east direction and afterreaching Pinahat it turns to the east and runs nearly parallel to the Yamuna beforejoining it in the southern part of the Etawah district in Uttar PradeshMajor Rivers of India with their basin area (Sqkm)

Himalayan System Indus 321290Ganga 861404

Brahmaputra 187110Indus System

Jhelum 34775Beas 20303

Ganga System Yamuna 366223Ghaghra 127950

Peninsular RiversNarmada 98796

Tapi 65145Mahanadi 141600

Subarnarekha 19300Sabarmati 21674

Mahi 34842Godavari 312812

Godavari Krishna Kaveri Mahanadi are the east-flowing rivers

Narmada Tapi west-flowing rivers

They fall into the Bay of Bengal

They fall into Arabian Sea

These rivers form big deltas

These rivers form comparativelysmall deltas

Catchment areas of these rivers are larger

Catchment areas of these rivers are smaller

Krishna 2589488Cauveri 87900

Subject ndashBiology Topic ndashChapter -5 Inheritance amp Variations Summary ExecutionToday we will discussabout linkage and its classification

LINKAGE The tendency of the genes located on the same chromosome to stay together is

hereditary transmission Linked genes the genes responsible for this Genes that exhibit the process of linkage locates in the same chromosome The distance between the linked genes in a chromosome determines the strength

of linkage i e genes that are located close to each other show stronger linkage than that are located far from each other

COMPLETE LINKAGE It is the type of linkage showed by the genes that are closely located or are tightly

linked with each other as they have no chance of separatingby crossing over These genes are always transmitted together to the same gamete and the same

offspring In such condition only parental or non cross over type of gametes are formedINCOMPLETE KINKAGE It is type of linkage showed by the genes that are distantly located orare loosely

linked with each other because they have chance of separating by crossing over

SIGNIFICANCE i) It helps in holding the parental character togetherii) It checks the appearance of new recombination and helps in bringing the

hybrid population which resembles the original parents iii) Linked genes dilute the effects of undesirable traits

Subject Eng Literature (The Tempest ndash William Shakespeare) Topic Essay Questions (EQ-3)Question No 3

Give a character sketch of CalibanAnswer

The character of Caliban has been wonderfully conceived by Shakespeare as the manifestation of all that is gross and earthy ndash a sort of creature of the earth as Ariel is a sort of creature of the air

Calibanrsquos Physical Appearanceo Caliban is lsquofreckledrsquo a lsquomisshapen knaversquo not honoured with human shape

o Prospero calls him lsquothou tortoisersquo (Act I Sc 2 Line 317) Trinculo stumbling upon him describes him as ldquoA strange fish hellip Legged like a man And his fins like armsrdquo He ldquosmells like a fishrdquo (Act II Sc 2 Line 25)

o Prospero also calls him a ldquobeastrdquo (Act IV Sc 1 Line 140) and ldquoThis misshapen knaverdquo (Act V Sc 1 Line 268)

o Further it appears that in addition to his physical deformity his spiritual inferiority is also suggested by Prosperorsquos claim that his birth resulted from the union between his mother the witch Sycorax and the devil

Calibanrsquos ParentageWhen the play opens Caliban is twenty four years of age having been born on the island twelve years before the coming of Prospero His mother was the foul witch Sycorax who was banished from Algiers for ldquomischiefs manifold and sorceries terrible to enter human hearingrdquo (Act I Sc 2 Line 264) and the father was the Devil himself Thus

Caliban is a monster of evil and brute nature ugly deformed and stinking

Calibanrsquos Savage and Malignant Natureo Caliban is entirely a creature of the earth ndash gross brutal and savage He regards himself as the rightful possessor

of the island and Prospero as a usurper

o In his young age he was on good terms with Prospero He had consented to be received by Prospero at his house and to be educated by him He has learnt human language only to curse his master whom he abhors

o His beastly nature soon breaks out and ends in a vicious attack on Miranda This opens the eye of Prospero who becomes severe to him and enforces his service by threats and violence

o Prospero uses him to make dams for fish to fetch firewood scraper trenches wash dishes and keep his cell clean

Calibanrsquos Hatred for ProsperoA profound hatred for Prospero has taken hold of Caliban It springs from a sense of his being dispossessed and ill-treated He would kill Prospero if he could but he knows the power of Prosperorsquos lsquobookrsquo Hence he transfers his allegiance to Stephano who seems like a god to him He also incites the two drunken associates to batter the skull of Prospero when he sleeps in the afternoon

Caliban Shows Considerable Intelligenceo He has learnt Prosperorsquos language

ldquoYou taught me language and my profit onrsquot (Act II Sc 2 Lines 86-89)Is I know how to curserdquo

o He is well aware of the futility of arguing with one who has more power than he has

ldquoI must obey his art is such power (Act I Sc 2 Lines 373-376)It would control my damrsquos god SetebosAnd make a vassal of himrdquo

o He realizes the importance of Prosperorsquos books

ldquoRemember (Act III Sc 2 Lines 89-92)First to possess his books for without themHersquos but a sot as I am nor hath notOne spirit to commandrdquo

o He knows the value of stealth when attacking the enemy

ldquoPray you tread softly that the blind mole may not (Act IV Sc 1 Lines 194-195)Hear a foot fall we now are near his cellrdquo

o Caliban has a better set of values than Stephano and Trinculo They are distracted from their plan by their greed for Prosperorsquos rich garments Only Caliban realizes that such a finery is unimportant

ldquoLeave it alone thou fool it is but trashrdquo (Act IV Sc 1 Lines 224)

Caliban is not a good judge of characterCaliban is not a good judge of character He decides for example that Stephano is a god because he dispenses lsquocelestial liquorrsquo (Act II Sc 2 Line 115) but then it must be remembered that he has only known his mother Sycorax Prospero Miranda and the spirits that torture him However he quickly discovers his error of judgementrdquo

ldquoWhat a thrice-double ass (Act V Sc 1 Lines 295-297)Was I to take this drunkard for a godAnd worship this dull foolrdquo

Calibanrsquos Imaginative NatureIf Caliban is sub-human in what has been said above he is human in the respect of the poetic side of his character He listens to music with rapture He tells of the beautiful dreams in which heaven rains treasures upon him and which upon waking he yearns to renew One of the most poetic passages in whole play is Calibanrsquos description of the island

to Stephano and Trinculo

ldquoBe not afeard The isle is full of noises (Act III Sc 2 Lines 135-143)Sounds and sweet airs that give delight and hurt notSometimes a thousand twangling instrumentsWill hum about mine ears and sometime voicesThat if I then had waked after long sleepWill make me sleep again and then in dreamingThe clouds methought would open and show richesReady to drop upon me that when I wakedI cried to dream againrdquo

Caliban - Less Ignoble Than Some OthersCalibanrsquos motive for murder is less dishonourable than that of Antonio and Sebastian They plan to kill Alonso to gain his power and wealth Caliban merely wants revenge and the return of lsquohisrsquo island

Conclusiono Calibanrsquos character is not portrayed very clearly in the play and hence we cannot decide whether he is a poor

savage being grossly maltreated by Prospero or whether he is evil and must therefore be kept in bondage or enslavement

o Caliban is contrasted with Ariel who is a spirit and thus swift and uninterested in physical activitieso Caliban is also contrasted with Prospero who is the all-powerful master of the island and of the destiny of all

those on the islando Caliban is also contrasted with civilized man showing him to be less evil than Antonio and Stephano and less

materialistic than Stephano and Trinculoo Caliban has suffered at the hands of Prospero and he has learnt to curse by listening to Prosperorsquos abuse He

certainly believes that Prospero has deprived him of his birthrighto Finally the character Caliban is thought to be one of Shakespearersquos masterpieces The complexity of the character

is reflected in the large volume of critical discussion that has grown around it

ECO ndash12 Topic-Forms of market

MonopolyMonopoly is a market structure in which there is a single seller there are no close substitutes for the commodity produced by the firm and there are barriers to entry Example Indian Railways which is operated under government of India Monopoly also implies absence of competitionFeatures of Monopoly Monopoly is characterized by1 Single Seller In monopoly there is only one firm producing the product The whole industry consists of this single firm Thus under monopoly there is no distinction between firm and industry Being the only firm there is significant control of the firm over supply and price Thus under monopoly buyers do not have the option of buying the commodity from any other seller They have to buy the product from the firm or they can go without the commodity This fact gives immense control to the monopolist over the market

2No Close Substitute There are no close substitutes of the product produced by the monopolist firm If there are close substitutes of the product in the market it implies presence of more than one firm and hence no monopoly In order to ensure a total of control over the market by the monopolist firm it is assumed that there are no close substitutes of the product

3 No Entry amp Exit Monopoly can only exist when there is strong barriers before a new firm to enter the market In fact once a monopoly firm starts producing the product no other firm can produce the same One reason for this is the ability of the

monopolist to produce the product at a lower cost than any new firm who thinks to enter the market If a new firm who knows that it cannot produce at a lower cost than the monopolist then that firm will never enter the market for fear of losing out in competition Similarly the monopolist who is operating for a long time may be enjoying reputation among its customers and is in a better position to use the situation in its own benefit A new firm has to take long time to achieve this and so may not be interested to enter the market

4 Price Maker Being the single seller of the product the monopolist has full control over the pricing of the product On the other hand if there is a large number of buyers in the market so no single buyer exercises any significant influence over price determination Thus it is a sellerrsquos market So monopoly firm is a price maker

5 Price Discrimination Having considerable control over the market on account of being single seller with no entry of other firms the monopolist can exercise policy of price discrimination it means that the monopolist can sell different quantities of the same product to a consumer at different price or same quantity to different consumers at different prices by adjudging the standard of living of the consumer

6 Shape of Demand Curve Since a monopolist has full control over the price therefore he can sell more by lowering the price This makes the demand curve downward sloping

Subject Ac-12 290620 Topic- retirement Model sumThe Balance Sheet of Rohit Nisha and Sunil who are partners in a firm sharing profits according to their capitals as on 31st March 2014 was as under

Liabilities Amount Assets Amount (Rs) (` Rs)

Creditors 25000 Machinery 40000Bills Payable 13000 Building 90000General Reserve 22000 Debtors 30000Capital Less Provision for Rohit 60000 Bad debts 1000

29000 Nisha 40000 Stocks 23000 Sunil 40000 140000 Cash at Bank 18000

200000 200000

On the date of Balance Sheet Nisha retired from the firm and following adjustments were made(i) Building is appreciated by 20(ii) Provision for bad debts is increased to 5 on Debtors(iii) Machinery is depreciated by 10(iv) Goodwill of the firm is valued at Rs 56000 and the retiring partnerrsquos share is adjusted

(v) The capital of the new firm is fixed at Rs120000 Prepare Revaluation Account Capital Accounts of the partner and Balance Sheet of the new firm after Nisharsquos retirement Revaluation AccountDr Cr

Particulars Amount Particulars Amount (`Rs) (Rs`)

Provision for Bad debt Ac 500 Building Ac 18000Machinery Ac 4000Profit transferred toCapital Accounts (3 2 2)Rohit 5786Nisha 3857Sunil 3857

13500

18000 18000

Capital Account

Dr Cr

Particulars Rohit Nisha Sunil Particulars Rohit Nisha Sunil (Rs`) (Rs`) (`Rs) (Rs`) (Rs`) (Rs`)

Sunilrsquos Capital ac 9600 mdash 6400 Balance bd 60000 40000 40000Bank - 66143 - General Reserve 9428 6286 6286Balance cd 72000 mdash 48000 Revaluation (Profi 5786 3857 3857 Rohitrsquos Capital Ac mdash 9600 mdash

Sunilrsquos Capital Ac 6400 Bank 6386 - 4257

81600 66143 54400 81600 66143 54400

Balance Sheet as at 31st March 2014

Liabilities Amount Assets Amount (Rs`) (Rs`)

Creditors 25000 Building 108000Bank overdraft 37500 Machinery 36000

Bills Payable 13000 Debtors 30000Capital Less ProvisionRohit 72000 for Bad debts 1500 28500Sunil 48000 120000 Stock 23000

195500 195500

Working Notes (i) (a) Profit sharing ratio is 60000 40000 40000 ie = 3 2 2(b) Gaining Ratio Rohit = 35 ndash 37 = 2135 ndash 1535 = 635Sunil = 25-27 = 1435 ndash 1035 = 435= 635 435= 6 4 = 3 2(c) Nisha Share of Goodwill = Rs 56000 times 27 = Rs16000Share of Goodwill in the gaining ratio by the existing partner ieRohit = Rs16000 times 35 = Rs 9600Sunil = Rs 16000 times 25 = Rs 6400

The journal entry isRohitrsquos Capital Ac Dr 9600Sunilrsquos Capital Ac Dr 6400 To Nisharsquos Capital Ac 16000(Share of Goodwill divided into gaining ratio)

  • 1 Static Friction
  • The frictional force that acts between the surfaces when they are at rest with respect to each other is called Static Friction
    • Static Friction Examples
      • 2 Sliding Friction
        • Examples Of Sliding Friction
          • 3 Rolling Friction
            • Examples Of Rolling Friction
              • Objects and Reasons of the Forest Conservation Act
Page 17:  · Web viewSubject . Topic . Summary . Execution . English 1 . Sounds of animals . Hens –cackle Horses –neigh Lions –roar Owls –hoots Snake –hiss. English 2 . Mother’s

FORMULAS REFERENCES AND FUNCTIONS

WORKSHEETA WORKSHEET IS ALSO KNOWN AS SPREADSHEETIT IS A COLLECTION OF CELLS ON A SINGLE SHEET WHERE YOU KEEP AND CHANGE DATA

WORKBOOKWORKBOOK IS PMS EXCEL FILE IN WHICH THE DATA CAN BE STORED EACH WORKBOOK CAN CONTAIN MANY WORKSHEETS

ROWS AND COLUMNSIN MS EXCEL A ROW IS A GROUP OF CELLS THAT RUN FROM LEFT TO RIGHT OF A PAGEA COLUMN IS A GROUPING OF CELLS THAT RUN FROM THE TOP TO THE BOTTOM OF A PAGE

CELLTHE INTERSECTION POINT BETWEEN A ROW AND THE COLUMN IS CALLED A CELL WHICH IS THE BASIC STORAGE UNIT FOR DATA IN A SPREADSHEET EACH CELL HAS SPECIFIC ADDRESS WHICH IS THE COMBINATION OF THE COLUMN NAME FOLLOWED BY THE ROW NUMBER

CHEMISTRY Chapter ndash Common Laboratory Apparatus and equipments

Objective type questionFill in the blanks (a) Experiment and observation are the two important basics of chemistry(b) A porcelain dish is used for evaporation(c) A test tube holder is used to hold the test tube while-it is heated(d) Mortar and pestle is used for grinding and crushing solid substances into a powder(e) Glass apparatus is made of Pyrex or borosil glass

Class VIISubject Topic Summary Execution

Hindi 2ndlang

ए था राम( डॉ शरी परसाद)

सगकित ा परभाव मानव 0ीवन पर अवशय पडता ह

हमशा मनषय ो अचछो ी सगकित म रहना चाकिहए

शरषठ परो सग स मनषय चरिरतर ा शीघर ही उदय और किवास हो 0ाता

ह इसलिलए वयलि` ो सदा शरषठ परो ा ही सग रना चाकिहए

इसान अगर चाह वह सवय ो बदल भी सता ह

यह हानी राम ए बचच ी हवह गणिणत ी परीकषा म नल रत हए पडा 0ाता ह और उस अधयाप पडत ह और पछत ह यह कया र रह

हो तभी राम न उनी बइजजती ी

शबदाथब ndashहावा भलावाायवाहीndash ाम किनयम व ानन

ो दिदखानापरिरलिचतndash 0ाना पहचानाघटनाndashघबराहट

उलटा चोर ोतवाल ो डाटndashकिववndash भल बर ा जञानतवयndash म 0ो रना चाकिहएसगकितndash बरी सगत

किबलख नाndashरोना किनशचय रनाndash तय रना

फलndashपरिरणामकिनषालिसतndash बाहर किया हआपशचातापndashदख सपननndashधनी

ldquo हा आपी किहममत स हई नल रत पडन ीrdquo ऐसी बात ही किफर

किपता0ी न भी उस डाटा वह ाफी पशचाताप रन लगा बोला गलत दोसतो

ी सगकित म आ0 कितना अनथ र दिदया किफर उसन अधयाप स माफी मागन ी सची और किफर भी ऐसा

नही रगा यह परण भी लिलया

सोचndashकिहच एात-अला

বইndashবাংলা সাহিতয পহিরচয়

পাঠndash১৬লপndashস-য ও অস-যযলখকndashঈশবরচনদর হিবযাসারঅনশীলনীর পরকে4াততর

৬ অGকেলকেখা -ময়া = পশ হিশকার সহিtহিত = হিনকIবতu সbভরষট = লI হিনরীকষণ =

-াকেলা-াকেব যখাকতাঞজহিলপকেI = যজাাকেত৭ হিবপরীতশবদ -ঈষৎ times পরচর উৎকষট times হিনকষট তাশ times উৎফd তবহিদধ times

বহিদধীNপাহিপষঠ times পণযবান৮ পপহিরবতG ন ককেরা -পশ = পাশহিবক যকাপ = যকাহিপতহিসথর = হিসথরতাএকানত = ঐকাহিনতক পর-াত times পর-াতী

CHEMISTRY

Chapter ndashPhysical and Chemical Changes

Chemical ChangeA chemical change involves a change in chemical composition

Characteristics of Chemical changes 1 They are permanent changes2 They are irreversible changes 3 New substance formed4 A Chemical change involves a

change in its chemical properties

Pg-25Question 8What do you observe when1 water is boiled2 a piece of paper is burnt3 some ice cubes are kept in a glass tumbler4 solid ammonium chloride is heated5 an iron nail is kept in tap water for few days6 a spoon of sugar is heated in a pan7 lighted match stick is brought near the mouth of the test tube containing hydrogen gas8 quick lime is dissolved in water9 little amount of curd is added to a bowl containing warm milk and kept for five hours

10 Water is boiledOn boiling water changes into steam (gas) physical change

11 A piece of paper is burnton burning piece of paper produces carbon dioxide and ash is left behind Is a chemical change

12 some ice cubes are kept in a glass tumblerIce cubes (solid) turn into water

(liquid) only state changes (physical change)

13 Solid ammonium chloride is heatedSolid ammonium chloride on heating changes into vapors (change of state) is physical change

14 An iron nail is kept in tap water for few dayswe observe reddish brown coating on the nail called rust (entirely new substance) is chemical change

15 A spoon of sugar is heated in a panWhen a spoon of sugar is heated in a pan black (charred sugar) (carbon) is seen Is a chemical change

16 Lighted match stick is brought near the mouth of the test tube containing hydrogen gasWe observe that hydrogen bums at the mouth of test tube with blue flame and pop sound is heard It is chemical change

17 Quick lime is dissolved in waterThe following two observations will be observed (i) A hissing sound is observed(ii) The mixture starts boiling and lime water is obtained

18 Little amount of curd is added to a bowl containing warm milk and kept for five hoursWhen a little amount curd is added to a bowl containing warm milk and kept for five hours a permanent change occurredThe milk will change to curd On boiling water changes into steam (gas) physical change

GEOGRAPHY

ATMOSPHERE IMPACT OF GLOBAL WARMING The destructive impart of global warming is observed in various spheres of life and the environment Some of the points are outlined below1 High temperatures lead to high

evaporation rate and drying up of the soil and surface water This affects crop production The occurrence of droughts is aggravating the problem even further

2 The heat waves in summer months

Q1 Write some impact of global warmingA1 The impacts of global warming are as follows1 High temperatures lead to high

evaporate ion rate and drying up of the soil and surface water This affects crop production The occurrence of droughts is aggravating the problem even further

2 The heat waves in summer months lead to a greater number

lead to a greater number of deaths due to heat strokes

3 Forest fires become more frequent4 Tropical cyclones and hurricanes

become common5 Melting of glaciers takes place6 Polar ice caps are becoming thinner

and melting at an alarming rate due to global warming The loss of sea ice

7 Due to increase in sea surface temperature sea levels rise in coastal areas and cause submergence of several islands

WAYS TO REDUCE GLOBAL WARMINGFollowing steps can be taken We need to decrease emission of

green house gases by reducing the burning of fossil fuel such as coal and petroleum

By planting more trees to increase forest cover

The government should also distributes free saplings and organize afforestation programmes to spread awareness regarding the beneficial effects of trees

We should switch to eco-friendly cars and gadgets

Incandescent light bulbs should be replaced by CFL bulbs

We can save electricity and reduce global warming by turning off electrical gadgets such as lights fans air-conditioners television and computer when we do not to use them

Efforts should be made to hasten the development of green cities oreco cities These cities are urban areas around the world striving to lessen the environment a impacts of urbanization

By following the 3Rs-Reduce Recycle and Reuse strategy we can use natural resources for our growth as well as save them for the need of the future generations This is called sustainable development

of deaths due to heat strokes3 Forest fires become more

frequent4 Tropical cyclones and hurricanes

become common5 Melting of glaciers takes place

etc

Q2 How to reduce global warmingA2 Following steps can be taken to reduce global warmingaWe need to decrease emission of

green house gases by reducing the burning of fossil fuel such as coal and petroleum

bBy planting more trees to increase forest cover

c The government should also distributes free saplings and organize afforestation programmes to spread awareness regarding the beneficial effects of trees

dWe should witch to eco-friendly cars and gadgets

eIncandescent light bulbs should be replaced by CFL bulbs

f We can save electricity and reduce global warming by turning off electrical gadgets such as lights fans air-conditioners television and computer when we do not to use them

Q3 What do you mean by 3Rrsquos of resource planningA3 The 3Rs are

1 Reduce 2 Recycle and3 Reuse

Q4 What is Sustainable developmentA4 By following the 3Rs-Reluce Recycle and Reuse strategy we can use natural resources for our growth as well as save them for the need of the future generations This is called sustainable development

English Language

Prepositions A preposition is a word placed before a noun or a pronoun It helps to show how the person or thing denoted by the noun is related to something else in the sentence

Kinds of Prepositions

Simple Prepositions- simple preposition are one word Prepositions such as at by for in of off for from on out through till to up with before amidst towards beyond between over etc

Compound Prepositions ndash There are some words that are always used with fixed Prepositions to convey specific meaning

Example I was unable to meet you dueto a previous engagement ( On account of)Always maintain the queue instead of crowding at the counter ( In place of)

Participial PrepositionsmdashParticiple Prepositions are present or past participles of various verbs which together with a noun phrase or a clause function as prepositions Examples- barring concerning considering notwithstanding pending regarding respecting etc

Exercise A

1 Gauravs fever has come down since Friday He has been absent for a week now

2 The child sat between his father and mother among the parents of all his classmates

3 There are mosquitoes in the room They flew into the room when the door was open

4 My father was inside the drawing room when I was playing outside my house

5 You may sit beside me I will give you a drawing book and pencils besides a storybook

6 We went to the market in the morning and walked towards the riverfront in the evening

7 The child walked along the pavement and across the street safely

8 This table top is made of glass My breakfast fell off it in the morning

9 The pan is on the gas stove There are vegetables in it

10 We will wait for you at the bus top There are a lot of people in the hall

Subject ndash Biology Topic ndash Chapter - 3 Photosynthesis and respiration in plants Summary Execution

All living organism (Plants and animals) need food for energy and growth Green plants (autotrophy) prepare food for all living organisms Today we will discuss about the process photosynthesis And adaptations in a leaf to carry out photosynthesis

Q1What do you mean by photosynthesis and write its word equation The process by which green plants make food (glucose) from carbon dioxide and water

in the presence of sunlight and chlorophyll is called photosynthesis

Carbon dioxide + Water ( Sun light from Sun ) Glucose + Oxygen ( chlorophyll in green leaves )

Q2 What are the adaptations in a leaf to carry out photosynthesisi) Leaves are broad wide and flat for absorbing more light energyii) Presence of chlorophyll in chloroplasts to trap sunlightiii) Presence of stomata which allow carbon dioxide to enter the cell and oxygen to go

out iv) Network of veins ensures continuous supply of water and minerals to the leafv) Thin waxy cuticle protects the leaf without blocking the lightQ3 Draw and label structure of chloroplast

Class VIIISubject Topic Summary Execution

PHYSICS ENERGY Production of Hydro electricity

A hydroelectric dam converts the potential energy stored in a water reservoir behind a dam to mechanical energymdashmechanical energy is also known as kinetic energy As the water flows down through the dam its kinetic energy is used to turn a turbine

The generator converts the turbinersquos mechanical energy into electricity

This electric energy then goes through various transmission processes before it reaches you

Question 2

Fill in the blanks

(a) Work is said to be done by a forte only when the body moves

(b) Work done = Force x distance moved in direction of force

(c) The energy of a body is its capacity to do work

(d) The SI unit of energy is joule

(e) The potential energy is due to its state rest of position and kinetic energy of the body is due to its state of motion

(f) Gravitational potential energy U = mass times force of gravity on unit mass times height

(g) Kinetic energy = frac12 times mass times (speed)2

(h) Power P = work donetime taken

(i) The S I unit of power is watt

(j) IHP = 746 W

BIOLOGY Chapter -5 The endocrine system and adolescence

Today we will discuss about thelocation and functions of secreted hormones of adrenal and Pancreas

Q5 Write location hormone secreted main functions and deficiency diseases of pancreas and adrenal glands

Endocrine Glands

Location Hormones secreted

Functions and Deficiency Diseases

1Adrenal gland

2 Pancreas Gland

On the top of each kidney

In between stomach and small intestine

i)Adrenaline from adrenal medulla

ii)Cortisone from adrenal cortex

i) Insulin

ii) Glucagon

It helps a person deal with any kind of emergency situation or emotional stressIt increases the heart beat rate of respiration and blood pressure

a) It regulates carbohydrates protein and fat metabolism

b) It regulates the salt and water balance in the body

a) It changes excess glucose into glycogen

b) It stimulates the cells to burn extra glucose to provide heat amp energy

Less secretion causes diabetes mellitus

Excessive secretions causeinsulin shock

a) It stimulates the breakdown of glycogen into glucose

b) It increases the level of glucose in blood

History Traders to rulers The Battle of Buxar was fought on 22 October 1764 between the forces under the command of the British East India Company led by Hector Munro and the combined armies of Mir Qasim the Nawab of Bengal till 1763 Mir Jafar was made the Nawab of Bengal for a second time in 1763 by the Company just after the battle After being defeated in 4 battles in katwa and Udaynala the Nawab of Awadh Siraj id Daula and the Mughal emperor Shah Alam II accompanied by Raja Balwant Singh of Kashi made an alliance with Mir Qasim The battle was fought at Buxar a small fortified

Answer the following questions- Short note-Battle of BuxarHomework-learn

town within the territory of Bihar located on the banks of the Ganga river about 130 kilometres (81 mi) west of Patna it was a decisive victory for the British East India Company The war was brought to an end by the Treaty of Allahabad in 1765

EnglishLiterature

The west wind-John Mansfield

In the poem The West Wind by John Masefield the poet starts by describingwith very poetic imagery of birds how the west wind is different from other winds its a warm wind full of birds cries There is a touch of melancholy perhaps home-sickness as he describes how it brings tears too and memories from an old land He goes on to describe the restful pastoral beauty of the land where even the dead can lie in the green He then brings in voicesperhaps of family and friends calling him home as he is missing Aprils beautyThe voices then tempt him some more with idyllic images from home (white blossom young green cornrunning rabbitswarm sun) The voices seem to presume that the poets heart is sorrowful bruised and soreThe end of the poem sees the poet appear to make a decision he will go home as he has decided that is where he truly belongs

Write the synopsis of the following words

1 Daffodils- a tall yellow flower that grows in the spring

2 Orchards- a piece of land on which fruit trees are grown

3 Blossom- a flower or a mass of flowers especially on a fruit tree in spring

4 Thrushes- a bird5 Larks- a small brown bird that

makes a pleasant sound6 Bruised- an injury7 Aching- pain 8 Tread- to put your foot down

while you are walking9 Balm-10 May-11 Fluting-

(Write from the book in your copy)

MAT

HEM

ATIC

S

Ch 1

1Al

gebr

ic E

xpre

ssio

n

1 Constant A symbol which has fixed value is called a constant[eg 8 23 -15 radic3 etc]

2 VariableA symbol which does not have any fixed value but may be assigned value (values) according to the requirement is called variable or literal[eg x y p q etc]

3 TermsA term is a number (constant) a variable a combination (product or quotient) of numbers and variables[eg 7 x 5x etc]

4 Algebric expressionA single term or acombination of two or more terms connected by plus (+) or minus (-) sign forms an algebraic expression[eg 5-y 3x2-5x xy-6z+4 etc]

5 PolynomialAn algebraic expression which contains more than one term is called a polynomial (multinomial)[eg x2-5x 5y+xy+x2y etc]

6 Degree of polynomial(a) When the polynomial contains only one variable the highest power of the variable is the degree of the polynomialeg the degree of the polynomial of 4x-7x5+8 is 5(b) When the polynomial contains two or more variablesStep (i) Find the powers of the variables in each term (ii) The highest sum of the powers is taken to be the degree of the polynomialeg the degree of the polynomial 5x2y-4x3y5+6 is = 3+5 = 8Remember An algebraic expression is a polynomial if degree of each term used in it is a non-negative integer

Exercise ndash 11(A)

1 Separate the constants and variables from the following

-7 7+x 7x+yz radic5 radic xy 3 yz

8 45y -3x

Solution Constant Variables-7 radic5 7+x 7x+yz radic xy

3 yz8

45y -3x

2 Write the number of terms in each of the following polynomials(i) 5x2+3timesax (ii) axdivide4-7 (iii) ax-by+ytimesz (iv) 23+atimesbdivide2

Solution Polynomials Number of terms(i) 5x2+3timesax 2(ii) axdivide4-7 2(iii) ax-by+ytimesz 3(iv) 23+atimesbdivide2 2

4 Write the degree of the each polynomials(i) xy+7z (ii) x2-6x3+8 (iii) y-6y2+5y8 (iv) xyz-3 (vi) x5y7-8x3y8+10x4y4z4

Solution Polynomials Degree(i) xy+7z 2(ii) x2-6x3+8 3(iii) y-6y2+5y8 8(iv) xyz-3 3(vi)x5y7-8x3y8+10x4y4z4 12

5Write the coefficient of(i) ab in 7abx (iv) 8 in a2-8ax+a (v) 4xy in x2-4xy+y2

SolutionCoefficient

(i) ab in 7abx 7x(iv) 8 in a2-8ax+a -ax(v) 4xy in x2-4xy+y2 -1

7 CoefficientAny factor of an algebraic quantity is called the coefficient of the remaining quantityeg in the algebraic term 7xyz 7 is coefficient of xyz 7x is coefficient of yz and so on

8 Like term The terms having the same literal coefficient are called like terms and those having different literal coefficients are called unlike terms

eg (i) 5xyz 8xyz -6xyz and 23xyz are like

terms(ii) 7xy2 8x2yz and -15xyz2 are unlike terms

6 in 57xy2z3 write the coefficient of

(i) 5 (vii) 5xy2 (viii) 17yz (xi) 5xyz

Solution Coefficient

(i) 5 17

xy2z3

(vii) 5xy2 17z3

(viii) 17yz

5xyzsup2

(xi) 5xyz 17yz2

7 In polynomial given below separate the like terms(ii) y2z3 xy2z3 -58x2yz -4y2z3 -8xz3y2 3x2yz and 2z3y2

Solution y2z3 -4y2z3 2z3y2 are like terms

xy2z3 -8xz3y2 are like terms

-58x2yz 3x2yz are like terms

Class IXSubject Topic Summary Execution

Bengali (2nd language)

বাগzwnjধারাzwnj বা ধারা-বা ধারা ল হিবকেশষ পরকার বাক -হিb -াকেবর এক হিবকেশষ পরকাশরীহিত াকেক কতগকেলা কার সমষটির মকেধয এগহিলকেক বা ধারা বকেল আবার কতগকেলা শকেবদর বাধাধরা যকান রীহিত যনই য-াকেব চকেল আসকে যসই -াকেবই চকেল আসকে তখন যসই শবদগহিল খন একক -াকেব অG পরকাশ ককের তখন একের বা ধারা বকেল বা ধারার পরকেয়া -াষাকেক আরও সFর ককের যতাকেল

অকাল পকক(অপহিরনত বয়কেস পাকাহিম)-মাতর শ বর বয়কেস যমকেয়টির া মকেখর কা তাকেত অকালপককতা ধরা পকে

অককা পাওয়া( মারা াওয়া) ndash পকেকIমারটি পকেকIমারকেত হিকেয় বাসাতরীকের াকেত মার যখকেত যখকেত অককা যপল

অহি| পরীকষা ( কঠিন ও পরকত পরীকষা)- যকেলটির আজ ডাকতাহির যরজালট যবকেরাকেব এIাই তার জীবকেনর ব অহি| পরীকষা

অষটরমভা (ফাহিক) ndash রীতা মকেখই বকো বকো কা বকেল আর কাকেজর যবলায় অষটরমভা

অকমGার ধাী (অপাG) ndash সমনকেক হিনকেয় যকান ান কেব না ও একেকবাকেরই অকমGার ধাী

অকেনধর ষটি (অসাকেয়র সায়)- আহিশ বকেরর বকোর নাহিত ল অকেনধর ষটি তাকেক াা বকোর একম চকেল না

আকেককল গড়ম (তবহিদধ)- ার তহিম উপকার করকেল যসই যতামার হিবরকেদধ সাকষয হিকেয়কে শকেনই আমার আকেককল গড়ম

আষাকে লপ( অবাসতব লপ) ndashIাকা এখন যকেব না এIা বলকেলই ত এমন আষাকে লপ ফাার যকান রকার হিল না

Hindi- महायजञ ा इस हानी म लख न या बतान ा परयास किया ह कि किसी भी अचछ

2nd language

परसार(यशपाल ाय या पणय न ा फल अवशय मिमलता ह ोई भी परोपार अथवा पणय लिलए किया गया ाय बार नही 0ाता वह ए परार ा यजञ हए धनी सठ थ धम परायण और किवनमर सठ न आन ी यजञ किए थ और दान म न 0ान कितना धन दिदन दखिखयो म बात दिदया थादिदन पलट और सठ यहा गरीबी आ गई उन दिदनो यजञ बचन ी परथा थी सठ भी अपनी 0गह बचन लिलए डलपर ए सट यहा चलन ो तयार हए सठानी रासत लिलए रोटी पड म बाधर सठ ो द दी रासत म ए भख R ो दखर सठ न चारो रोटी उसो खिखला दी खर वह सठ यहा डलपर पहच तो उनी सठानी न उस महायजञ बचन ो हा यदिद बचन आए सठ न R ो रोटी खिखलान ो महायजञ नही समझा और वापस लौट आया घर आर शाम ो उसी घर म उस ए बडा ख0ाना मिमला 0ो उस दवारा किए गएrsquo महायजञrsquo ा परसार था

English language

Letter formal The heading the name and address of the person you are writing to must be included beneath your own address In formal letters ldquoblock stylerdquo of address is preferred

Subject complain in brief

Salutation If the person you are writing to is known to you you may begin ldquoDear MrrdquoOr ldquoDear Mrsrdquo In all other instances you should begin ldquoDear Sirrdquo or ldquoDear Madamrdquo Or ldquoSirsrdquo

The body A formal or business letter has four partsReference The letter should begin by referring to a letter you have received an advertisement or the reason that has prompted you to writeInformation In the second paragraph it is necessary to supply more detailed information that is related to the referencePurpose Here you must give the reason why you are writing the letter This must be stated clearly and ensure that it is relevant to the question that has been setConclusion round off the letter with some polite remarkThe subscription when a letter has begun with dear sir sirs Madam you should end with Yours faithfully or yours truly When however you address a person by name you must conclude with the words ldquoYours sincerelyrdquo

1 A park in your locality is slowly being used as a rubbish dump Write a letter to the Mayor of your city pointing out the nuisance and danger of this Request that action be taken to stop this immediately

Or2 You being a boarder ordered a set of lab manuals from a famous book shop in the town They sent you a wrong set of books Write a letter to the manager of the book shop

Chemistry Chapter-1 1)CHEMICAL FORMULA- Q What is the Significance of

L-2The Language of Chemistrybull Chemical Formula

Itrsquos a symbolic representation of a chemical substance eg ndash The formula of Sulphuric acid is H2SO4

2) Steps of writing Chemical Formula of a given substance-

1 Write the symbols of the constituent atoms or radicals side by side Keep the basic radical on LHS and acid radical on the RHS ( Na+Cl- )2 In case of a radical having more than one atom( compound radical) enclose the radical in a bracket eg (SO4-)3 Write the valencies of each radical on its right hand top4 If the valencies of the two radicals are divisible by a common factor then divide the valencies by the common factor5 Invert (criss-cross) the valency number ie write the valency of one atom below the second atom and vice versa 6 On interchanging if valency number is lsquoone the figure lsquoonersquo is never writtenFor Example- Compound -Calcium Nitrate1 Writing the symbols- Ca(NO3)2 Writing the valencies on their right hand top- Ca2(NO3)1

3 Valency numeral in simple ratio- Ca2(NO3)1

4 Criss-cross- Ca 2NO3 1

5 Writing the formula of the compound- Ca(NO3)2

Chemical formula

A The formula of a substance conveys the following information regarding a substance 1 The name of the substance (qualitative)2 The elements constituting the substance (qualitative)3 The number of various atoms present in a molecule of the substance (quantitative)4 Molecular weight of the substance and the relative weights of different elements present in it (qualitative)

Q What are the limitations of Chemical Formula

A The chemical formula suffers from the following limitations-I It fails to convey whether the elements in a molecule are present in the form of atoms or ionsFor example the formula KBr fails to tell us whether Potassium and Bromine are present in the form of ions II It does not tell anything about the binding force that holds atom in a molecule togetherIII It does not tell us about the arrangement of various atoms with respect to one another within the molecule

Q Examples of Some Chemicals with their Formula Chemical name and Common Name-

A Given in the class notesCommercial Studies

Joint Stock Company

Let us discuss about the demerits of Joint Stock CompanyDespite so many advantages it has got many disadvantages which are as follows

Difficulty in FormationDelay in Decision makingExcessive Government ControlLack of Secrecy

Company can be classified into several categories based on incorporation

QuestionExplain the demerits of Joint Stock CompanyAnswer) 1 Difficulty in Formation The legal requirements and formalities required to be completed are so many The cost involved is quite heavy It has to approach large number of people for its capital It cannot start its business unless certificate of incorporation has been obtained This is granted after a long time when all the formalities are completed

Chartered CompanyStatutory CompanyRegistered Company

Delay in Decision making In this form of organization decisions are not made by single individual All important decisions are taken by the Board of Directors Decision-making process is time-consuming So many opportunities may be costly because of delay in decision-making Promptness of decisions which is a common feature of sole trader ship and partnership is not found in a company

Excessive Government ControlA company and the management have to function well within the law and the provisions of Companies Act are quite elaborate and complex At every step it is necessary to comply with its provisions lest the company and the management should be penalized The penalties are quite heavy and in several cases officers in default can be punished with imprisonment This hampers the proper functioning of the company

Lack of Secrecy The management of companies remains in the hands of many persons Every important thing is discussed in the meetings of Board of Directors Hence secrets of the business cannot be maintained In case of sole proprietorship and partnership forms of organisation such secrecy is possible because a few persons are involved in the management

2 Define the following

Chartered Company- The crown in exercise of the royal prerogative has power to create a corporation by the grant of a charter to persons assenting to be incorporated Such companies or corporations are known as chartered companies Examples of this type of companies are Bank of England (1694) East India Company (1600) The powers and the nature of business of a chartered company are defined by the charter which incorporates it After the country attained independence these types of companies do not exist

in IndiaStatutory Company- A company may be incorporated by means of a special Act of the Parliament or any state legislature Such companies are called statutory companies Instances of statutory companies in India are Reserve Bank of India the Life Insurance Corporation of India the Food Corporation of India etc The provisions of the Companies Act 1956 apply to statutory companies except where the said provisions are inconsistent with the provisions of the Act creating them Statutory companies are mostly invested with compulsory powersRegistered companiesCompanies registered under the Companies Act 1956 or earlier Companies Acts are called registered companies Such companies come into existence when they are registered under the Companies Act and a certificate of incorporation is granted to them by the Registrar

Economics

Chapter-4Basic problems of Economy

Today let us discuss with the topic Production Possibility curve

QuestionExplain the concept of Production Possibility Curve with the help of diagram

Answer) Production Possibility curve is a locus of all possible combinations of two commodities which can be produced in a country with its given resources and technology

The above diagram shows that with the given resources and technology the economy can produce maximum either 5 thousand meters of cloth or 15 thousand quintals of wheat or any other combination of the two goods like B( 1 thousand meters of cloth and 14 thousand quintals of wheat C ( 2 thousands meters of cloth and 12 thousand quintals of wheat) etcProduction Possibility curve is also called production possibility boundary or frontier as it sets the maximum limit of what it is possible to produce with given resources

Geography

Rotationand Revolution

SUNrsquoS POSITION AND SEASONAL CHANGES EQUINOXES ndash SPRING AND AUTUMN

Q1 What is Spring EquinoxA1 On 21st March sunrays fall directly on the equator On that day

As the Equator divides the Earth into two equal halves the sun rays fall directly on the equator twice in a year Equinoxes means equal Spring EquinoxOn 21st March sunrays fall directly on the equator On that day the duration of day and night both are equal ( 12 hours day and 12 hours night) on every places located on equator This day is called as Spring EquinoxAutumn EquinoxOn 23rd September sunrays fall directly on the equator On that day the duration of day and night both are equal ( 12 hours day and 12 hours night) on every places located on equator This day is called as Autumn Equinox

SOLSTICES ndash SUMMER AND WINTERDue to inclination of the Earth on its axis and the apparent movement of the sun the sun rays fall directly on both tropics once in a year Solstice is a Latin word which mean ldquothe Sun standing stillrdquoSummer SolsticesAfter 21st March there is an apparent movement of the Sun to the north of the equator The apparent northward movement up to 21st June when the Sun appears overhead at the Tropic of Cancer (22frac12degN) The sun appears to stand still at this position and then moves southwards towards the equator This position of the Sun on 21st June is known as Summer Solstices On that day the duration of day and night both are equal ( 12 hours day and 12 hours night) on every places located on Tropic of Cancer (22frac12degN)Winter solstices The apparent southward movement of the Sun continues beyond the equator till 22nd

December On this day the Sun is overhead at the Tropic of Capricorn

the duration of day and night both are equal ( 12 hours day and 12 hours night) on every places located on equator This day is called as Spring Equinox

Q2 What do you mean by EquinoxA2 Equinoxes means equal It is use to explain the equal duration of day and night ( 12 hours day and 12 hours night) on the Earth

Q3 On which date the longest day in Tropic of CancerA3 21st June

Q4 What is the meaning of SolsticeA4 Solstice is a Latin word which mean ldquothe Sun standing stillrdquo

Q5 Which is the longest day in southern hemisphereA5 22nd December

Q6 On what date does the Arctic Circle experience the lsquoMidnight SunrsquoA6 On 21 June the Arctic Circle experiences the lsquoMidnight Sunrsquo

Q7 What is cause of Midnight Sun in NorwayA7 During the summer solstice (21 June) the North Pole is inclined towards the Sun Therefore the duration of sunlight or daytime increases from 12 hours at the Equator to 24 hours at the Arctic Circle and beyond Thatrsquos why The region beyond the Arctic Circle especially Norway is known as the Land of the Midnight Sun because there the Sun does not rise or set on 21 June

Q8 Match the column A with BA B

Summer Solstice 21st March

Autumn Equinox 23rd

September

Winter Solstice 21st June

(22frac12degS) This position of the Sun is referred to as the Winter Solstice because it marks the winter season in the Northern Hemisphere On that day the duration of day and night both are equal ( 12 hours day and 12 hours night) on every places located on Tropic of Capricorn (22frac12degS)SEASONS AND DURATION OF DAY AND NIGHT During the equinoxes all places on the Earth have 12 hours of day and 12 hours of night Due to the revolution of the Earth round the Sun on an inclined axis the duration of day and night varies according to seasons and the latitude of a placeDuring the summer solstice (21 June) the North Pole is inclined towards the Sun Therefore the duration of sunlight or daytime increases from 12 hours at the Equator to 24 hours at the Arctic Circle and beyondThe region beyond the Arctic Circle especially Norway is known as the Land of the Midnight Sun because there the Sun does not rise or set on 21 JuneAt the North Pole there will be six months of daylight The Sun will be seen always above the horizon at a low angle At 66degN 24 hours of sunlight can be seen only on 21 June Hammerfest in northern Norway is a place of tourist attraction for observing the phenomenon of the Midnight Sun This place has continuous daylight from 13 May to 29 July This place is easily accessible to tourists and has hotels and other facilities The view of the midnight Sun from here is enthrallingIn the Southern Hemisphere the duration of daylight decreases from 12 hours at the equator to 0 hours beyond the Antarctic Circle In the South Polar Region there is 24 hours of darkness The Sun is always below the horizon In the Southern Hemisphere which experiences winter the duration of night-time is longer than the duration of daylight

Spring Equinox 22nd

December

A8 A B

Summer Solstice 21st June

Autumn Equinox 23rd

September

Winter Solstice 22nd

December

Spring Equinox 21st March

During winter solstice (22 December) the South Pole is inclined towards the Sun The Southern Hemisphere experiences summer and the Northern Hemisphere has winter Therefore the duration of daylight or sunlight is greater in the Southern Hemisphere than in the Northern HemisphereThe duration of daylight increases from 12 hours at the equator to 24 hours beyond the Antarctic Circle The South Polar Region has 24 hours of sunlight for many days continuously At the South Pole there will be six months of sunlight The Sun will always be seen at a low angle above the horizon In the Northern Hemisphere the duration of daylight will decrease from 12 hours at the equator to 0 hours at the Arctic Circle There are 24 hours of darkness in the North Polar region The duration of night is greater than the duration of daylight as one move northwards from the Equator It is evident from the above table that the duration of daylight is 12 hours throughout the year at the equator only As one moves away from the equator the seasonal variations in the duration of daylight increase The seasonal variations in the duration of daylight are maximum at the Polar Regions

Subject Eng Literature (The Merchant of Venice ndash William Shakespeare)Topic Act II Scene 7 Lines 36 to 80 (End of scene ) [Students should read the original play and also the paraphrase provided]

Summary Questions amp AnswersThe Prince then examines the inscription on the silver casket which says ldquoWho chooseth me shall get as much as he deservesrdquo The Prince says that he deserves Portia more than anybody else because of his high rank his noble birth and his great wealth and power But then he argues that silver is ten times

(1) (Act II Sc 7 L 39-47)

From the four corners of the earth they come

To kiss this shrine this mortal breathing saint

The Hyrcanian deserts and the vasty wildsOf wide Arabia are as through-fares now

inferior to gold and therefore he cannot believe that the portrait of such a beautiful lady as Portia can be contained in the silver casket He decides to see the inscription on the golden casket before making his decision

The Prince goes to examine the inscription on the golden casket which says ldquoWho chooseth me shall get what many men desirerdquo The Prince believes that the whole world desires to possess Portia otherwise so many suitors would not have come from all corners of the world for winning Portia Some of them have come from the distant lands of Persia and Arabia The deserts of Persia (Hyrcanian deserts) and the boundless desolate lands of Arabia have been crossed by the Princes seeking the hand of Portia He contrasts this casket containing Portiarsquos portrait with the old English gold coin bearing the image of the archangel (angel of the highest rank) He goes on to remark that while the figure of the archangel is engraved (Insculped) upon the English coin the picture of Portia who is beautiful as an angel lies hidden inside one of the caskets namely the Golden Casket (Golden Bed)

On the basis of his assessment of the inscription on the golden casket the Prince decides to choose the golden casket He asks for the key and opens the golden casket only to find therein an empty human skull holding a roll of

For princes to come view fair PortiaThe watery kingdom whose ambitious headSpets in the face of heaven is no barTo stop the foreign spirits but they comeAs orsquoer a brook to see fair Portia

(i) Explain the occasion for the above mentioned speech

These are the comments of the Prince of Morocco after he reads the inscription on the golden casket His mental process is revealed to us in these words We find him debating within himself as to which casket he should choose

(ii) What light does the above speech throw on the personality of Prince of Morocco

From the above mentioned speech we come to know that the Prince of Morocco is keen to marry Portia He is the type of person who is easily taken away by outward appearance He is in love with Portia because of her beauty

(iii) What information can you gather about Portia from the above mentioned lines

The given speech shows that Portia is a very beautiful lady She must be possessed of good qualities because many suitors come to her place from all over the world with a desire to get married to her The Prince of Morocco is so impressed by her beauty that he calls her a saint According to him the whole world is desirous of having her

(iv) Elucidate the significance of the first two lines

In these lines the Prince of Morocco pays a compliment to Portia These lines show his admiration for her He says that people come from all parts of the world to see fair Portia

(v) Explain the meaning of the last four lines of the

passage

In these lines the Prince of Morocco says that even the vast oceans which throw a challenge at the sky are unable to prevent men from coming to Portiarsquos place to have a glimpse of her These lines are also a tribute to Portiarsquos beauty and good qualities Many men voyage across the ocean treating it as a mere stream to see the beautiful Portia

paper in which is written that whoever happens to be guided by the glitter of things is invariably deceived

On reading the scroll the Prince says that he is too sad at heart to speak a more formal farewell and leaves with his followers amidst a sound of trumpets

After the Prince of Morocco leaves Portia remarks that the Prince is a gentle fellow but she is rid of him May all persons of his nature make a similar choice

IMPORTANT PASSAGES EXPLAINED

(Act II Sc 7 L 39-43)From the four corners of the earth they come

To kiss this shrine this mortal breathing saintThe Hyrcanian deserts and the vasty wildsOf wide Arabia are as through-fares nowFor princes to come view fair Portia

Context

This passage occurs in Act II Scene 7 in The Merchant of Venice This is part of the speech made by the Prince of Morocco

(2)

(Act II Sc 7 L 48-53)

MOROCCO One of these three contains her heavenly pictureIst like that lead contains her

Twere damnation To think so base a thought it were too grossTo rib her cerecloth in the obscure graveOr shall I think in silver shes immurdBeing ten times undervalued to tried gold

(i) What meaning does the Prince of Morocco find out of the inscription of the golden casket What have Belmont and Portiarsquos house been called and why

The inscription on the golden casket is ldquoWho chooseth me shall gain what many men desirerdquo The Prince finds out that it means that the chooser of the golden casket will get Portia because many men desire her In fact the entire world desires her Because of the coming of many suitors to Belmont from different countries in order to win Portiarsquos hand Belmont has become a centre of pilgrimage and her house is the shrine where saintly Portia is installed

(ii) What does the Prince of Morocco do before making the final choice of the casket Which is the correct casket and who will win Portiarsquos hand

The Prince of Morocco surveys and analyses the inscriptions on the casket of lead silver and gold Before making the final choice like a very systematic and methodical person he once again considers the claims of the caskets The casket containing Portiarsquos picture is the correct casket and the person choosing it will win Portiarsquos hand

Explanation

While praising Portia the Prince of Morocco conceives Portia as a goddess whose image is placed inside one of the caskets Many suitors are coming from far and wide the north and the south the east and the west (Four corners) in order to try their luck Some of them have come from the distant land of Persia and Arabia The deserts of Persia (Hyrcanian deserts) and the boundless desolate lands of Arabia have been crossed by the Princes seeking the hand of Portia All this shows that Portia is indeed the most beautiful lady of the world

(iii) What does the Prince of Morocco say in his estimation while examining the motto on the silver casket What does he find in the golden casket

While examining the motto on the silver casket which says ldquoWho chooseth me shall get as much as he deservesrdquo Morocco says that in his own estimation he surely deserves Portia in all respects ndash rank birth wealth etc

He chooses the golden casket When he opens it he finds an empty human skull holding a scroll in which it is written that those who are attracted by the glittering outside of things are always deceived as Morocco has been deceived

(iv) What kind of nature does the Prince of Morocco have

The Prince of Morocco has a simple nature who does not look deeply into the inner meaning of things but is dazzled by the outward appearance of gold He is inclined to over-estimate his own value and does not realize that it is a duty to ldquogive and hazardrdquo To say that he will not hazard for lead shows that he misreads the true meaning of the inscription which is that he should be prepared to ldquohazard all he hathrdquo for Portia So his feeling is only one of fascination and romantic attraction

(v) Do you think that the lottery of the caskets is not a matter that will be determined by chance

In fact the lottery of the casket is not a matter that will be determined by mere chance but that it is a true test of character and of sincerity which is amply proved not only by Moroccorsquos choice but also by the arguments which he uses to help him in his choice

(Act II Sc 7 L 55-59)

They have in England

A coin that bears the figure of an angelStamped in gold but thats insculpd uponBut here an angel in a golden bedLies all within

Context

(3)

(Act II Sc 7 L 63-77)A carrion Death within whose empty eye

There is a written scroll Ill read the writing

All that glisters is not goldOften have you heard that toldMany a man his life hath soldBut my outside to beholdGilded tombs do worms infoldHad you been as wise as boldYoung in limbs in judgment oldYour answer had not been inscrolld

This passage occurs in Act II Scene 7 in The Merchant of Venice This is part of the speech made by the Prince of Morocco

Explanation

In this passage the Prince of Morocco bestows high praise on Portia whose hand he is seeking He contrasts this casket containing Portiarsquos portrait with the old English gold coin bearing the image of the archangel (angel of the highest rank) He goes on to remark that while the figure of the archangel is engraved (Insculped) upon the English coin the picture of Portia who is beautiful as an angel lies hidden inside one of the caskets namely the Golden Casket (Golden Bed) In the day of Elizabeth silver was ten times inferior in value to gold Therefore the Prince of Morocco believing that Portiarsquos portrait is contained in the Golden Casket decides to choose the Golden Casket

Fare you well your suit is coldCold indeed and labour lostThen farewell heat and welcome frostmdashPortia adieu I have too grievd a heartTo take a tedious leave Thus losers part

(i) What reward does the Prince of Morocco get after making a wrong choice of the Casket How does he feel

After making the wrong choice in selecting the casket of gold the Prince of Morocco as a reward earns a rebuke in the form of a scroll tucked in the empty eye-socket of a skull kept in the casket of gold The Prince is shocked and disappointed He becomes all the more sad and dejected when he reads the scroll which points to his foolishness in being misled by the appearance and outward show as indicative of its worth

(ii) How does the Prince respond after reading the scroll

After reading the scroll the Prince though upset accepts the result with good grace and decorum befitting a royal suitor and true sportsman He says that his love-suit is really cold otherwise he would have chosen correctly but now his efforts have been in vain So he bids farewell to Portia to the warmth and enthusiasm of love and welcomes the cold and bitterness of dejection and misery of life which lies ahead

(iii) What request does he make to Portia and why

After being failure in his mission he requests Portia to give him permission to leave at once because he is too sad to undergo the tediousness of a formal leave-taking He tells that it is the manner in which defeated persons part unceremoniously

(iv) Explain the following lines

ldquoAll that glisters is not goldOften have you heard that toldMany a man his life hath soldBut my outside to beholdGilded tombs do worms infoldrdquo

Mere glitter does not make a metal to be gold Man has often been warned against appearance but it has been of no use Many people have sacrificed their lives only to seek the outer appearance of gold Worms are found inside the gilded

monuments

Class XSubject Topic Summary Execution

Hindi 2ndlang

नया रासता भाग 6 मायाराम 0ी घर म धनी मल 0ी और उनी बटी सरिरता ी ही चचा बनी रहती थी अमिमत ो इसम ोई रलिच ना थी वह धनी घर ी लडी स शादी र सवय ो बचना नही चाहता था उसा भी सवाणिभमान ह ईशवर ी पा

स उस पास पस ी ोई मी नही थी अभी उसन फकटरी ही लगाई थी उसी समझ बाहर था कि उस घर वालो ा झाव पस ी तरफ कयो

ह उसन मा स सवाल किया कि मा तम सरिरता स मरी शादी कयो रना चाहती हो मा न उस समझाया कि वह दखन म बरी नही ह और किफर खानदान अचछा

ह वह ए शल गरहणी रप म घर सभाल सगी अमिमत न मा ो इस बात ा एहसास राया कि मीन सबध लिलए मना रन पर उस दिदल

पर कया बीती होगी मा और अमिमत ी लडी बार म ाफी बात हईमा ा झाव सरिरता ी तरफ था कयोकि वह घर पर अचछा दह0 लर आ रही

थी अमिमत न अपनी मौसी ी बरी हालत बार म बताया कि किस तरह वह बड घर ी खानदानी बटी लाई थी और आ0 उसी हालत कितनी खराब ह लाई थी बहकलब 0ाती ह और बचचो ो भी नही दखती ह बात चल ही रही

थी कि तभी ए ार बाहर आर री धनी मल0ी घर अदर आए और पीछ स डराइवर फल ी ए टोरी लर आया अदर आए और पीछ स

डराइवर ए टोरी फल ी लर आया अमिमत ो फल ी पटी बरी लग रही थी अमिमत न पछ लिलया यह फल कयो ल आए ह प इन सब ी कया

0ररत थी उनो न 0वाब दिदया कि 4 पटी शमीर स मगाए थ अमिमत ो या सनर करोध आ गया तभी उस किपता 0ी आ गए उन आत ही अमिमत उठर बाहर चला गया वहा वहा मा पास आर बठ गया और बोला

अभी रिरशता तय नही हआ और धनी मल 0ी धनी मल 0ी फल ी पटी लर चलआय मा न समझाया कि 0ब सबध 0ड 0ाता ह तो खाली हाथ नही

आत अमिमत न मा स हा कि तम सबन सरिरता ो इस घर म लान ी ठान रखी ह धनीमल 0ी उस दिदन सरिरता ो दखन ी तारीख तय रन आय थ

Commercial Studies

Banking Nowadays Bank provide easy and quick services through internet facilities methods of Banking is called internet bankingIn order to save the time and money involved in visiting Bank branches people increasingly prefer to have internet banking

There are different modes of doing internet banking or transferring money through online They areReal Time Gross Settlement (RTGS)National Electronic Fund Transfers (NEFT)

1

Question

1) Explain the term RTGS Write the features of RTGS

Answer)The acronym RTGS stands for Real Time Gross Settlement which may be defined as the continuous real time settlement of funds transfer individually on and order by order basis without netting lsquoReal timersquo may be defined as the processing of instructions at the time they are received rather than at some letter time lsquoGross settlementrsquo may be defined as the settlement of transfer instructions which occurs

individually

Features of RTGS1It is the continuous settlement of

funds transfer individually on an order by order basis

2RTGS facility is provided only by CBS core banking solution enabled Bank branches

3Amount charged from the customer for RTGS transactions vary from bank to bank

2) Explain the term NEFT Write the features of NEFT

Answer) National electronic funds transfer may be defined as a nationwide system that facilitates individuals Farms and copper operates to electronically transfer funds from any bank branch to any individual farm or corporate having an account with any other bank branch in the country

Features of NEFT2 Transfer can be made 7 times on

weekdays and 6 times on Saturday

3 NEFT cannot be used to receive foreign remittances

4 NEFT transaction takes place in batches

5 A bank branch must be NEFT enabled to become a part of NEFT fund transfer network

6 There is no maximum or minimum amount that can be transferred through NEFT when one bank has a bank account

English Language

CompositionEssay

A composition is an art of creating a piece of writing on any topic or subject It is the writing correctly beautifully and clearly in order to make some interesting reading Structure of the composition

Introduction ( you lay the foundation for your composition)

Body (it constitutes the main part of the essay)

Conclusion (final statement that leaves a lasting impression)

Kinds of essays1 The Narrative essay2 The descriptive essay3 The reflective essay4 The argumentative essay

Write a composition on any one of the following topics (350- 400 words)

1 Friendship Or2 The first day of your school

Subject Eng Literature (The Merchant of Venice ndash William Shakespeare)Topic Act V Scene 1 Lines 127 to 158 (Nerissa helliphellip The clerk will nersquoer wear hair onrsquos face that had it) [Students should read the original play and also the paraphrase given in the school prescribed textbook]

Summary Revision Questions o Soon thereafter Bassanio Gratiano

and Antonio arrive

o Bassanio tells Portia that he is feeling as if it is morning because of the presence of Portia who is shining like the sun When Antonio is introduced by Bassanio to Portia she tells Bassanio that he should be grateful to Antonio who took so much trouble on his account even to the extent of risking his life

o Nerissa starts quarrelling with Gratiano and demands that he show her the ring she had presented to him and which she had warned him not to lose She suspects that Gratiano must have presented the ring to some young woman and not to the lawyerrsquos clerk as he repeatedly says and assures

Answer the following questions to check your preparation of Act IV Scenes 1 and 2

You must attempt only after you have completed your preparation of Act IV The answers must be in complete sentences using textual evidence (with citation) when necessary

[It would be in your own interest to attempt the above questions honestly totally refraining from consulting your textbook or your notes during answering After completion you should correct the paper yourself consulting the textbooknotes etc and award marks as specified Please let me know the marks you scored through WhatsApp in the group or to my personal WhatsApp]

Act IV Scene 1 (each question carries 2 marks)

1 What did the Duke try to do for Antonio

2 Why does Shylock refuse to show mercy How does he justify his stance

3 Why does Antonio say he is ready to die 4 What information is contained in Bellariorsquos letter

5 Why does Portia (as Balthazar) assert that Shylock must show mercy How does he respond

6 What offers are made to Shylock to get him to spare Antonio How are they received

7 What does Antoniorsquos speech as he faces the prospect of Shylockrsquos knife tell you about his character

8 How do Bassanio and Gratiano react to the looming prospect of Antoniorsquos demise

9 How does Portia (as Balthazar) use the law to turn the tables on Shylock

10 What does the Duke decree should happen to Shylock Why What happens to Shylockrsquos estate

11 What does Portia ask Bassanio as payment for her ldquoservicesrdquo What is his initial response What makes him change his mind

Act IV Scene 2 (each question carries 1frac12 marks)

1 What does Gratiano bring to Portia (Balthazar)

2 What does Nerissa plan on getting from Gratiano What does Portiarsquos comment suggest about men

ECO-10 280620 Topic-Supply AnalysisSHIFTING OF SUPPLY

But if there is change in factors other than the price of the commodity then either more is supplied at the same price or less supplied at the same price In such cases the price of the commodity remains constant but there is a change in other factors like change in the price of inputs change in technology of production change in price of other related goods change in taxation policy of the government etc For example there is an improvement in the technology of production of the commodity in question It leads to decrease in per unit of cost production of the commodity The firm is willing to sell more quantity of the commodity at the same price So the supply other commodity increases at the same price This increase in supply is shown by rightward shift of supply curve On the other hand if the firm uses inferior technology of production the cost of production per unit of the commodity increases The firm is willing to sell less quantity at the same price So the supply of the commodity decreases at the same price This decrease in supply is shown by leftward shift of the supply curve The above cases of increase and decrease in supply can be shown with the help of the following figures

Y INCREASE OF SUPPLY Price (Rs) s

P A s1

B

s

X` O s1 X

q q1

Y` Quantity demanded (in units)

Y DECREASE IN SUPPLY s2

s

price (Rs)

C

p A

s2

s

X` o X

q2 q

Y` Quantity demanded ( in units)

Main factors causing increase in supply or rightward shift of supply Curve(i) Fall in the price of other related goods

(ii) Fall in the price of inputsfactors(iii) Use of better technology in production(iv) Decrease in the rate of excise duty by government(v) If the objective of producer changes from profit maximization to salesMaximization

Main factors causing decrease in supply or leftward shift of supply curve(i) Increase in the price of other related goods(ii) Rise in the price of inputsfactors(iii) Use of inferior technology in production(iv) Increase in the rate of excise duty by the government(v) If the objective

Subject - Biology Topic ndash Chapter mdash6 PhotosynthesisSummary Execution

Today we will know about photosynthesis and its stages

Q1 What do you mean by photosynthesis The process by which living plants containing chlorophyll produce food

substances from carbon-di- oxide and water by using light energy Sunlight

6CO2 +12 H2O----------------------- C6 H12O6 + 6H2O + 6O2

Chlorophyll

Q2 What are the importance of photosynthesis I) Food for all Green plants trap solar energy by photosynthesis

process and supply food and energy for all living organisms either directly or indirectly

Ii) Oxygen to breathe in by product of photosynthesis is oxygen which is essential for all living organisms respiration

Q3 Write about two main phases of photosynthesis A Light dependent phase This phase occur in grana of chloroplast I) The chlorophyll on exposure to light energy becomes activated by

absorbing photons Ii) The absorbed energy is used in splitting the water molecules (H2O)

into its two components (H+ and OH- ) and releasing electron s 2H2O------------------------- 4H+ + 4e- +O2

Energy of 4 photons This reaction is known as photolysis

End products are H+ and oxygen water

B Light independent (Dark ) phase The reactions in this phase require no light energy

Here CO2 combine with H+ and produce glucose

Class XI

Subject Topic Summary ExecutionEVS Chapter-4 Legal

regimes for sustainable development

Environmental legislationEnvironmental legislation is the collection of laws and regulations pertaining to air quality water quality the wilderness endangered wildlife and other environmental factors The act ensures that matters important to the environment are thoroughly

Learn -The Forest (Conservation) Act 1980

considered in any decisions made by federal agencies

The Forest (Conservation) Act 1980 The Forest (Conservation) Act 1980 an Act of the Parliament of India to provide for the conservation of forests and for matters connected therewith or ancillary or incidental thereto It was further amended in 1988 This law extends to the whole of IndiaObjects and Reasons of the Forest Conservation Act

Deforestation causes ecological imbalance and leads to environmental deterioration Deforestation had been taking place on a large scale in the country and it had caused widespread concern The act seeks to check upon deforestation and de-reservation of forests

Subject Eng Literature (The Tempest ndash William Shakespeare) Topic Act II Scene 1 Lines 314 to 329 (End of scene)

[Students should read the original play and also the paraphrase given in the school prescribed textbook]Summary Questions amp Answers

Conspiracy of Antonio and Sebastian (Contd)

o As they approach Ariel appears again and wakes up Gonzalo by singing a tune in his ear Alonso also wakes up and they see both Sebastian and Antonio with drawn swords On being caught off guard they make up a story saying that they had heard a bellowing of bulls or lions

o They then moved to another part of the island

o Ariel at once rushes to Prospero to inform him of this development

SUMMING-UP of ACT-2 SCENE-1

(i) Among the survivors Ferdinand is separated from the rest which results in the disconsolate grief of Alonso as he took him for dead

(ii) The villainy of Antonio is confirmed

(iii) The supremacy of Prosperorsquos magic which resulted in the failure of the human conspiracy

(1)

(Act II Sc 1 L 311-325)SEBASTIAN Whiles we stood here securing your repose

Even now we heard a hollow burst of bellowing Like bulls or rather lions Didt not wake youIt struck mine ear most terribly

ALONSO I heard nothingANTONIO O rsquotwas a din to fright a monsters ear

To make an earthquake Sure it was the roarOf a whole herd of lions

ALONSO Heard you this GonzaloGONZALO Upon mine honour sir I heard a humming

And that a strange one too which did awake meI shaked you sir and cried As mine eyes opened I saw their weapons drawn There was a noiseThats verily rsquoTis best we stand upon our guardOr that we quit this place Lets draw our weapons

(i) Why has Prospero sent Ariel to Gonzalo and Alonso What does Ariel do to awaken Gonzalo

Prospero has already come to know by his magic powers the danger which threatens Gonzalo who had been Prosperorsquos friend and so he sent Ariel to preserve the lives of both Gonzalo and Alonso Prospero does not want that his scheme should remain unfulfilled Ariel begins to sing a song in Gonzalorsquos ears to awaken him(ii) Who are ready to carry out their plan Who takes steps to stop them Why does Gonzalo feel surprised after being awakened

Sebastian and Antonio are ready to carry out their plans They are standing with their swords drawn to kill Alonso and

(iv) We see two sets of contrasting characters Gonzalo-Adrian against Antonio-Sebastian

(v) The grief that works in Alonso can be perceived to his repentance for his association in Antoniorsquos crime against Prospero

Gonzalo Ariel takes steps to stop them from carrying out their nefarious scheme When Gonzalo is awakened by the song sung by Ariel into his ears he (Gonzalo) feels surprised because he sees Sebastian and Antonio standing with their swords drawn(iii) What reason do Sebastian and Antonio tell of drawing their swords when they are suspected by Alonso and Gonzalo

When Sebastian and Antonio are seen with their swords drawn they are looked with suspicion by Gonzalo and Alonso At first Sebastian tells them that as they stood here to guard them during their sleep they heard only a little before a sudden loud noise very much like the roaring of bulls or more probably that of lions Then Antonio follows him saying that this was a noise so terrible as to frighten even a monsterrsquos ears and this noise could even have shaken the earth and it was surely like the roaring of a multitude of lions Then seeing the danger they have drawn their swords Perhaps after hearing the terrible noise they (Gonzalo and Alonso) woke up from their sound sleep

(iv) What does Gonzalo tell Alonso about the strange noise What did he see on opening his eyes Gonzalo tells Alonso that he did not hear the sound of roaring but he heard a humming sound which was strange and which woke him up After waking up he gave him (Alonso) a shaking and a loud cry On opening his eyes he saw these two gentlemen standing with their swords drawn(v) What does Gonzalo suggest

Gonzalo suggests that there was a noise indeed and of that he has no doubt at all and suggests that the best course for them would be to remain alert and vigilant against any possible danger to their lives or to leave this place and move to some other part of the island

Class XIISubject Topic Summary Execution

Commerce

Chapter- Management

Today we will discuss about LEVELS OF MANAGEMENT

Levels of management is a series or chain of managerial positions from top to bottom It helps individuals to know their authority responsibilities and superior-subordinate relations among themselves There are mainly three levels of Management TOP LEVEL MANAGEMENTMIDDLE LEVEL MANAGEMENTLOWER LEVEL MANAGEMENT

Top level managementIt consists of members at the highest level in the management hierarchy This level includes Board Of Directors Chief Executive Managing Directors Chairman President Vice President

Rolefunctions of the top levelmanagement1To analyse evaluate and deal

with theexternal environment2 To determine the objectives and

policies of the business3 To strive for welfare and survival

of business

4 To create an organisational Framework consisting of authority responsibility relationship

Middle level management Congress of members or groups who are concerned with implementation of the policies let down by the top managementThis level includes head of the department such as finance manager marketing manager branch and regional managers departmental and divisional heads plant superintendent etc

Role of functions of the middle level management

1 To interpret the policies framed by top management

2 To assign duties and responsibilities to lower level managers

3 To select and appoint employees for middle and supervisory level and evaluate their performance

4 To co-operate with other departments for smooth functioning

Operational or supervisory level managementIt refers to the group are members who are concerned with execution of the work They are also known as fast line managers This level includes supervisor 4 men Section Officer clerk Inspector etc

Role of functions of the lower level management1 To plan and execute day-to-

day operations2 To supervise and control the workers3 To arrange materials and

tools to start the process and make arrangements for training

4 Today present workers grievance and suggestions before the management and

ensure safe and proper working conditions in the factory

Business Studies

Staff Appraisal Chapter- 10 Today let us start with a new chapter

Staff Appraisal

Meaning of Performance Appraisal

Performance Appraisal is the systematic evaluation of the performance of employees and to understand the abilities of a person for further growth and developmentThe supervisors measure the pay of employees and compare it with targets and plansThe supervisor analyses the factors behind work performances of employeesThe employers are in position to guide the employees for a better performance

Objectives of Performance Appraisal

Following are the objectives of Performance Appraisal

To maintain records in order to determine compensation packages wage structure salaries raises etc

To identify the strengths and weaknesses of employees to place right men on right job

To maintain and assess the potential present in a person for further growth and development

To provide a feedback to employees regarding their performance and related status

To provide a feedback to employees regarding their performance and related status

Importance of Performance Appraisal

Performance appraisal provides important and useful information for the assessment of employees skill

knowledge ability and overall job performance The following are the points which indicate the importance of performance appraisal in an organization

1 Performance appraisal helps supervisors to assess the work performance of their subordinates

2 Performance appraisal helps to assess the training and development needs of employees

3 Performance appraisal provides grounds for employees to correct their mistakes and it also provides proper guidance and criticism for employees development4 Performance appraisal provides reward for better performance

5 Performance appraisal helps to improve the communication system of the organization

6 Performance appraisal evaluates whether human resource programs being implemented in the organization have been effective

7 Performance appraisal helps to prepare pay structure for each employee working in the organization

8 Performance appraisal helps to review the potentiality of employees so that their future capability is anticipated

Geography

DRIANAGE The SubarnarekhaThe Subarnarekha and the Brahmaniinterposed between the Ganga and the Mahanadi deltas drain an area of 19300 sq kmand 39033 sq km respectively The drainage basins of these streams are shared byJharkhand Odisha west Bengal and Chhattisgarh The Brahmani is known as southKoel in its upper reaches in Jharkhand

The NarmadaThe Narmada rises in the Amarkantak hills of MadhyaPradesh It flows towards the West in a rift valleyformed due to a geological fault The total length of it is 1300 km All the tributaries of the

Q1 Name the two westward flowing rivers in the peninsular plateauA1 Narmada and Tapi are the only westward flowing rivers of the peninsular plateau

Q2 Differentiate between east-flowing rivers and west-flowing riversA2

East-flowing rivers

West-flowing rivers

Narmada are very short inlength Most of its tributaries join the main streamright anglesThe Narmada basin covers parts of Madhya Pradesh and Gujarat

The Tapi The Tapi rises in the Satpura ranges in the Betul listrictof Madhya Pradesh It flows in a rift valley parallel tothe Narmada but it is much shorter in length It coversparts of Madhya Pradesh Gujarat and MaharashtraThe length is about 724 km

The Sabarmati and the MahiThe Sabarmati rises in the Aravali hills and flows south-south-westwards for a distance of 300 kilometres to the Arabian Sea The Sabarmatibasin extends over an area of 21674 sq km in Rajasthan and Gujarat The Mahi rises inthe east of Udaipur and drains an area of 34842 sq km lying in Madhya PradeshRajasthan and Gujarat It flows south-westwards for a distance of 533 km before it fallsinto the Gulf of Khambhat

The ChambalThe Chambal rises near Mhow in the Vindhya Range and flows towards the northgenerally in a gorge upto Kota Below Kota it turns to the north-east direction and afterreaching Pinahat it turns to the east and runs nearly parallel to the Yamuna beforejoining it in the southern part of the Etawah district in Uttar PradeshMajor Rivers of India with their basin area (Sqkm)

Himalayan System Indus 321290Ganga 861404

Brahmaputra 187110Indus System

Jhelum 34775Beas 20303

Ganga System Yamuna 366223Ghaghra 127950

Peninsular RiversNarmada 98796

Tapi 65145Mahanadi 141600

Subarnarekha 19300Sabarmati 21674

Mahi 34842Godavari 312812

Godavari Krishna Kaveri Mahanadi are the east-flowing rivers

Narmada Tapi west-flowing rivers

They fall into the Bay of Bengal

They fall into Arabian Sea

These rivers form big deltas

These rivers form comparativelysmall deltas

Catchment areas of these rivers are larger

Catchment areas of these rivers are smaller

Krishna 2589488Cauveri 87900

Subject ndashBiology Topic ndashChapter -5 Inheritance amp Variations Summary ExecutionToday we will discussabout linkage and its classification

LINKAGE The tendency of the genes located on the same chromosome to stay together is

hereditary transmission Linked genes the genes responsible for this Genes that exhibit the process of linkage locates in the same chromosome The distance between the linked genes in a chromosome determines the strength

of linkage i e genes that are located close to each other show stronger linkage than that are located far from each other

COMPLETE LINKAGE It is the type of linkage showed by the genes that are closely located or are tightly

linked with each other as they have no chance of separatingby crossing over These genes are always transmitted together to the same gamete and the same

offspring In such condition only parental or non cross over type of gametes are formedINCOMPLETE KINKAGE It is type of linkage showed by the genes that are distantly located orare loosely

linked with each other because they have chance of separating by crossing over

SIGNIFICANCE i) It helps in holding the parental character togetherii) It checks the appearance of new recombination and helps in bringing the

hybrid population which resembles the original parents iii) Linked genes dilute the effects of undesirable traits

Subject Eng Literature (The Tempest ndash William Shakespeare) Topic Essay Questions (EQ-3)Question No 3

Give a character sketch of CalibanAnswer

The character of Caliban has been wonderfully conceived by Shakespeare as the manifestation of all that is gross and earthy ndash a sort of creature of the earth as Ariel is a sort of creature of the air

Calibanrsquos Physical Appearanceo Caliban is lsquofreckledrsquo a lsquomisshapen knaversquo not honoured with human shape

o Prospero calls him lsquothou tortoisersquo (Act I Sc 2 Line 317) Trinculo stumbling upon him describes him as ldquoA strange fish hellip Legged like a man And his fins like armsrdquo He ldquosmells like a fishrdquo (Act II Sc 2 Line 25)

o Prospero also calls him a ldquobeastrdquo (Act IV Sc 1 Line 140) and ldquoThis misshapen knaverdquo (Act V Sc 1 Line 268)

o Further it appears that in addition to his physical deformity his spiritual inferiority is also suggested by Prosperorsquos claim that his birth resulted from the union between his mother the witch Sycorax and the devil

Calibanrsquos ParentageWhen the play opens Caliban is twenty four years of age having been born on the island twelve years before the coming of Prospero His mother was the foul witch Sycorax who was banished from Algiers for ldquomischiefs manifold and sorceries terrible to enter human hearingrdquo (Act I Sc 2 Line 264) and the father was the Devil himself Thus

Caliban is a monster of evil and brute nature ugly deformed and stinking

Calibanrsquos Savage and Malignant Natureo Caliban is entirely a creature of the earth ndash gross brutal and savage He regards himself as the rightful possessor

of the island and Prospero as a usurper

o In his young age he was on good terms with Prospero He had consented to be received by Prospero at his house and to be educated by him He has learnt human language only to curse his master whom he abhors

o His beastly nature soon breaks out and ends in a vicious attack on Miranda This opens the eye of Prospero who becomes severe to him and enforces his service by threats and violence

o Prospero uses him to make dams for fish to fetch firewood scraper trenches wash dishes and keep his cell clean

Calibanrsquos Hatred for ProsperoA profound hatred for Prospero has taken hold of Caliban It springs from a sense of his being dispossessed and ill-treated He would kill Prospero if he could but he knows the power of Prosperorsquos lsquobookrsquo Hence he transfers his allegiance to Stephano who seems like a god to him He also incites the two drunken associates to batter the skull of Prospero when he sleeps in the afternoon

Caliban Shows Considerable Intelligenceo He has learnt Prosperorsquos language

ldquoYou taught me language and my profit onrsquot (Act II Sc 2 Lines 86-89)Is I know how to curserdquo

o He is well aware of the futility of arguing with one who has more power than he has

ldquoI must obey his art is such power (Act I Sc 2 Lines 373-376)It would control my damrsquos god SetebosAnd make a vassal of himrdquo

o He realizes the importance of Prosperorsquos books

ldquoRemember (Act III Sc 2 Lines 89-92)First to possess his books for without themHersquos but a sot as I am nor hath notOne spirit to commandrdquo

o He knows the value of stealth when attacking the enemy

ldquoPray you tread softly that the blind mole may not (Act IV Sc 1 Lines 194-195)Hear a foot fall we now are near his cellrdquo

o Caliban has a better set of values than Stephano and Trinculo They are distracted from their plan by their greed for Prosperorsquos rich garments Only Caliban realizes that such a finery is unimportant

ldquoLeave it alone thou fool it is but trashrdquo (Act IV Sc 1 Lines 224)

Caliban is not a good judge of characterCaliban is not a good judge of character He decides for example that Stephano is a god because he dispenses lsquocelestial liquorrsquo (Act II Sc 2 Line 115) but then it must be remembered that he has only known his mother Sycorax Prospero Miranda and the spirits that torture him However he quickly discovers his error of judgementrdquo

ldquoWhat a thrice-double ass (Act V Sc 1 Lines 295-297)Was I to take this drunkard for a godAnd worship this dull foolrdquo

Calibanrsquos Imaginative NatureIf Caliban is sub-human in what has been said above he is human in the respect of the poetic side of his character He listens to music with rapture He tells of the beautiful dreams in which heaven rains treasures upon him and which upon waking he yearns to renew One of the most poetic passages in whole play is Calibanrsquos description of the island

to Stephano and Trinculo

ldquoBe not afeard The isle is full of noises (Act III Sc 2 Lines 135-143)Sounds and sweet airs that give delight and hurt notSometimes a thousand twangling instrumentsWill hum about mine ears and sometime voicesThat if I then had waked after long sleepWill make me sleep again and then in dreamingThe clouds methought would open and show richesReady to drop upon me that when I wakedI cried to dream againrdquo

Caliban - Less Ignoble Than Some OthersCalibanrsquos motive for murder is less dishonourable than that of Antonio and Sebastian They plan to kill Alonso to gain his power and wealth Caliban merely wants revenge and the return of lsquohisrsquo island

Conclusiono Calibanrsquos character is not portrayed very clearly in the play and hence we cannot decide whether he is a poor

savage being grossly maltreated by Prospero or whether he is evil and must therefore be kept in bondage or enslavement

o Caliban is contrasted with Ariel who is a spirit and thus swift and uninterested in physical activitieso Caliban is also contrasted with Prospero who is the all-powerful master of the island and of the destiny of all

those on the islando Caliban is also contrasted with civilized man showing him to be less evil than Antonio and Stephano and less

materialistic than Stephano and Trinculoo Caliban has suffered at the hands of Prospero and he has learnt to curse by listening to Prosperorsquos abuse He

certainly believes that Prospero has deprived him of his birthrighto Finally the character Caliban is thought to be one of Shakespearersquos masterpieces The complexity of the character

is reflected in the large volume of critical discussion that has grown around it

ECO ndash12 Topic-Forms of market

MonopolyMonopoly is a market structure in which there is a single seller there are no close substitutes for the commodity produced by the firm and there are barriers to entry Example Indian Railways which is operated under government of India Monopoly also implies absence of competitionFeatures of Monopoly Monopoly is characterized by1 Single Seller In monopoly there is only one firm producing the product The whole industry consists of this single firm Thus under monopoly there is no distinction between firm and industry Being the only firm there is significant control of the firm over supply and price Thus under monopoly buyers do not have the option of buying the commodity from any other seller They have to buy the product from the firm or they can go without the commodity This fact gives immense control to the monopolist over the market

2No Close Substitute There are no close substitutes of the product produced by the monopolist firm If there are close substitutes of the product in the market it implies presence of more than one firm and hence no monopoly In order to ensure a total of control over the market by the monopolist firm it is assumed that there are no close substitutes of the product

3 No Entry amp Exit Monopoly can only exist when there is strong barriers before a new firm to enter the market In fact once a monopoly firm starts producing the product no other firm can produce the same One reason for this is the ability of the

monopolist to produce the product at a lower cost than any new firm who thinks to enter the market If a new firm who knows that it cannot produce at a lower cost than the monopolist then that firm will never enter the market for fear of losing out in competition Similarly the monopolist who is operating for a long time may be enjoying reputation among its customers and is in a better position to use the situation in its own benefit A new firm has to take long time to achieve this and so may not be interested to enter the market

4 Price Maker Being the single seller of the product the monopolist has full control over the pricing of the product On the other hand if there is a large number of buyers in the market so no single buyer exercises any significant influence over price determination Thus it is a sellerrsquos market So monopoly firm is a price maker

5 Price Discrimination Having considerable control over the market on account of being single seller with no entry of other firms the monopolist can exercise policy of price discrimination it means that the monopolist can sell different quantities of the same product to a consumer at different price or same quantity to different consumers at different prices by adjudging the standard of living of the consumer

6 Shape of Demand Curve Since a monopolist has full control over the price therefore he can sell more by lowering the price This makes the demand curve downward sloping

Subject Ac-12 290620 Topic- retirement Model sumThe Balance Sheet of Rohit Nisha and Sunil who are partners in a firm sharing profits according to their capitals as on 31st March 2014 was as under

Liabilities Amount Assets Amount (Rs) (` Rs)

Creditors 25000 Machinery 40000Bills Payable 13000 Building 90000General Reserve 22000 Debtors 30000Capital Less Provision for Rohit 60000 Bad debts 1000

29000 Nisha 40000 Stocks 23000 Sunil 40000 140000 Cash at Bank 18000

200000 200000

On the date of Balance Sheet Nisha retired from the firm and following adjustments were made(i) Building is appreciated by 20(ii) Provision for bad debts is increased to 5 on Debtors(iii) Machinery is depreciated by 10(iv) Goodwill of the firm is valued at Rs 56000 and the retiring partnerrsquos share is adjusted

(v) The capital of the new firm is fixed at Rs120000 Prepare Revaluation Account Capital Accounts of the partner and Balance Sheet of the new firm after Nisharsquos retirement Revaluation AccountDr Cr

Particulars Amount Particulars Amount (`Rs) (Rs`)

Provision for Bad debt Ac 500 Building Ac 18000Machinery Ac 4000Profit transferred toCapital Accounts (3 2 2)Rohit 5786Nisha 3857Sunil 3857

13500

18000 18000

Capital Account

Dr Cr

Particulars Rohit Nisha Sunil Particulars Rohit Nisha Sunil (Rs`) (Rs`) (`Rs) (Rs`) (Rs`) (Rs`)

Sunilrsquos Capital ac 9600 mdash 6400 Balance bd 60000 40000 40000Bank - 66143 - General Reserve 9428 6286 6286Balance cd 72000 mdash 48000 Revaluation (Profi 5786 3857 3857 Rohitrsquos Capital Ac mdash 9600 mdash

Sunilrsquos Capital Ac 6400 Bank 6386 - 4257

81600 66143 54400 81600 66143 54400

Balance Sheet as at 31st March 2014

Liabilities Amount Assets Amount (Rs`) (Rs`)

Creditors 25000 Building 108000Bank overdraft 37500 Machinery 36000

Bills Payable 13000 Debtors 30000Capital Less ProvisionRohit 72000 for Bad debts 1500 28500Sunil 48000 120000 Stock 23000

195500 195500

Working Notes (i) (a) Profit sharing ratio is 60000 40000 40000 ie = 3 2 2(b) Gaining Ratio Rohit = 35 ndash 37 = 2135 ndash 1535 = 635Sunil = 25-27 = 1435 ndash 1035 = 435= 635 435= 6 4 = 3 2(c) Nisha Share of Goodwill = Rs 56000 times 27 = Rs16000Share of Goodwill in the gaining ratio by the existing partner ieRohit = Rs16000 times 35 = Rs 9600Sunil = Rs 16000 times 25 = Rs 6400

The journal entry isRohitrsquos Capital Ac Dr 9600Sunilrsquos Capital Ac Dr 6400 To Nisharsquos Capital Ac 16000(Share of Goodwill divided into gaining ratio)

  • 1 Static Friction
  • The frictional force that acts between the surfaces when they are at rest with respect to each other is called Static Friction
    • Static Friction Examples
      • 2 Sliding Friction
        • Examples Of Sliding Friction
          • 3 Rolling Friction
            • Examples Of Rolling Friction
              • Objects and Reasons of the Forest Conservation Act
Page 18:  · Web viewSubject . Topic . Summary . Execution . English 1 . Sounds of animals . Hens –cackle Horses –neigh Lions –roar Owls –hoots Snake –hiss. English 2 . Mother’s

ldquo हा आपी किहममत स हई नल रत पडन ीrdquo ऐसी बात ही किफर

किपता0ी न भी उस डाटा वह ाफी पशचाताप रन लगा बोला गलत दोसतो

ी सगकित म आ0 कितना अनथ र दिदया किफर उसन अधयाप स माफी मागन ी सची और किफर भी ऐसा

नही रगा यह परण भी लिलया

सोचndashकिहच एात-अला

বইndashবাংলা সাহিতয পহিরচয়

পাঠndash১৬লপndashস-য ও অস-যযলখকndashঈশবরচনদর হিবযাসারঅনশীলনীর পরকে4াততর

৬ অGকেলকেখা -ময়া = পশ হিশকার সহিtহিত = হিনকIবতu সbভরষট = লI হিনরীকষণ =

-াকেলা-াকেব যখাকতাঞজহিলপকেI = যজাাকেত৭ হিবপরীতশবদ -ঈষৎ times পরচর উৎকষট times হিনকষট তাশ times উৎফd তবহিদধ times

বহিদধীNপাহিপষঠ times পণযবান৮ পপহিরবতG ন ককেরা -পশ = পাশহিবক যকাপ = যকাহিপতহিসথর = হিসথরতাএকানত = ঐকাহিনতক পর-াত times পর-াতী

CHEMISTRY

Chapter ndashPhysical and Chemical Changes

Chemical ChangeA chemical change involves a change in chemical composition

Characteristics of Chemical changes 1 They are permanent changes2 They are irreversible changes 3 New substance formed4 A Chemical change involves a

change in its chemical properties

Pg-25Question 8What do you observe when1 water is boiled2 a piece of paper is burnt3 some ice cubes are kept in a glass tumbler4 solid ammonium chloride is heated5 an iron nail is kept in tap water for few days6 a spoon of sugar is heated in a pan7 lighted match stick is brought near the mouth of the test tube containing hydrogen gas8 quick lime is dissolved in water9 little amount of curd is added to a bowl containing warm milk and kept for five hours

10 Water is boiledOn boiling water changes into steam (gas) physical change

11 A piece of paper is burnton burning piece of paper produces carbon dioxide and ash is left behind Is a chemical change

12 some ice cubes are kept in a glass tumblerIce cubes (solid) turn into water

(liquid) only state changes (physical change)

13 Solid ammonium chloride is heatedSolid ammonium chloride on heating changes into vapors (change of state) is physical change

14 An iron nail is kept in tap water for few dayswe observe reddish brown coating on the nail called rust (entirely new substance) is chemical change

15 A spoon of sugar is heated in a panWhen a spoon of sugar is heated in a pan black (charred sugar) (carbon) is seen Is a chemical change

16 Lighted match stick is brought near the mouth of the test tube containing hydrogen gasWe observe that hydrogen bums at the mouth of test tube with blue flame and pop sound is heard It is chemical change

17 Quick lime is dissolved in waterThe following two observations will be observed (i) A hissing sound is observed(ii) The mixture starts boiling and lime water is obtained

18 Little amount of curd is added to a bowl containing warm milk and kept for five hoursWhen a little amount curd is added to a bowl containing warm milk and kept for five hours a permanent change occurredThe milk will change to curd On boiling water changes into steam (gas) physical change

GEOGRAPHY

ATMOSPHERE IMPACT OF GLOBAL WARMING The destructive impart of global warming is observed in various spheres of life and the environment Some of the points are outlined below1 High temperatures lead to high

evaporation rate and drying up of the soil and surface water This affects crop production The occurrence of droughts is aggravating the problem even further

2 The heat waves in summer months

Q1 Write some impact of global warmingA1 The impacts of global warming are as follows1 High temperatures lead to high

evaporate ion rate and drying up of the soil and surface water This affects crop production The occurrence of droughts is aggravating the problem even further

2 The heat waves in summer months lead to a greater number

lead to a greater number of deaths due to heat strokes

3 Forest fires become more frequent4 Tropical cyclones and hurricanes

become common5 Melting of glaciers takes place6 Polar ice caps are becoming thinner

and melting at an alarming rate due to global warming The loss of sea ice

7 Due to increase in sea surface temperature sea levels rise in coastal areas and cause submergence of several islands

WAYS TO REDUCE GLOBAL WARMINGFollowing steps can be taken We need to decrease emission of

green house gases by reducing the burning of fossil fuel such as coal and petroleum

By planting more trees to increase forest cover

The government should also distributes free saplings and organize afforestation programmes to spread awareness regarding the beneficial effects of trees

We should switch to eco-friendly cars and gadgets

Incandescent light bulbs should be replaced by CFL bulbs

We can save electricity and reduce global warming by turning off electrical gadgets such as lights fans air-conditioners television and computer when we do not to use them

Efforts should be made to hasten the development of green cities oreco cities These cities are urban areas around the world striving to lessen the environment a impacts of urbanization

By following the 3Rs-Reduce Recycle and Reuse strategy we can use natural resources for our growth as well as save them for the need of the future generations This is called sustainable development

of deaths due to heat strokes3 Forest fires become more

frequent4 Tropical cyclones and hurricanes

become common5 Melting of glaciers takes place

etc

Q2 How to reduce global warmingA2 Following steps can be taken to reduce global warmingaWe need to decrease emission of

green house gases by reducing the burning of fossil fuel such as coal and petroleum

bBy planting more trees to increase forest cover

c The government should also distributes free saplings and organize afforestation programmes to spread awareness regarding the beneficial effects of trees

dWe should witch to eco-friendly cars and gadgets

eIncandescent light bulbs should be replaced by CFL bulbs

f We can save electricity and reduce global warming by turning off electrical gadgets such as lights fans air-conditioners television and computer when we do not to use them

Q3 What do you mean by 3Rrsquos of resource planningA3 The 3Rs are

1 Reduce 2 Recycle and3 Reuse

Q4 What is Sustainable developmentA4 By following the 3Rs-Reluce Recycle and Reuse strategy we can use natural resources for our growth as well as save them for the need of the future generations This is called sustainable development

English Language

Prepositions A preposition is a word placed before a noun or a pronoun It helps to show how the person or thing denoted by the noun is related to something else in the sentence

Kinds of Prepositions

Simple Prepositions- simple preposition are one word Prepositions such as at by for in of off for from on out through till to up with before amidst towards beyond between over etc

Compound Prepositions ndash There are some words that are always used with fixed Prepositions to convey specific meaning

Example I was unable to meet you dueto a previous engagement ( On account of)Always maintain the queue instead of crowding at the counter ( In place of)

Participial PrepositionsmdashParticiple Prepositions are present or past participles of various verbs which together with a noun phrase or a clause function as prepositions Examples- barring concerning considering notwithstanding pending regarding respecting etc

Exercise A

1 Gauravs fever has come down since Friday He has been absent for a week now

2 The child sat between his father and mother among the parents of all his classmates

3 There are mosquitoes in the room They flew into the room when the door was open

4 My father was inside the drawing room when I was playing outside my house

5 You may sit beside me I will give you a drawing book and pencils besides a storybook

6 We went to the market in the morning and walked towards the riverfront in the evening

7 The child walked along the pavement and across the street safely

8 This table top is made of glass My breakfast fell off it in the morning

9 The pan is on the gas stove There are vegetables in it

10 We will wait for you at the bus top There are a lot of people in the hall

Subject ndash Biology Topic ndash Chapter - 3 Photosynthesis and respiration in plants Summary Execution

All living organism (Plants and animals) need food for energy and growth Green plants (autotrophy) prepare food for all living organisms Today we will discuss about the process photosynthesis And adaptations in a leaf to carry out photosynthesis

Q1What do you mean by photosynthesis and write its word equation The process by which green plants make food (glucose) from carbon dioxide and water

in the presence of sunlight and chlorophyll is called photosynthesis

Carbon dioxide + Water ( Sun light from Sun ) Glucose + Oxygen ( chlorophyll in green leaves )

Q2 What are the adaptations in a leaf to carry out photosynthesisi) Leaves are broad wide and flat for absorbing more light energyii) Presence of chlorophyll in chloroplasts to trap sunlightiii) Presence of stomata which allow carbon dioxide to enter the cell and oxygen to go

out iv) Network of veins ensures continuous supply of water and minerals to the leafv) Thin waxy cuticle protects the leaf without blocking the lightQ3 Draw and label structure of chloroplast

Class VIIISubject Topic Summary Execution

PHYSICS ENERGY Production of Hydro electricity

A hydroelectric dam converts the potential energy stored in a water reservoir behind a dam to mechanical energymdashmechanical energy is also known as kinetic energy As the water flows down through the dam its kinetic energy is used to turn a turbine

The generator converts the turbinersquos mechanical energy into electricity

This electric energy then goes through various transmission processes before it reaches you

Question 2

Fill in the blanks

(a) Work is said to be done by a forte only when the body moves

(b) Work done = Force x distance moved in direction of force

(c) The energy of a body is its capacity to do work

(d) The SI unit of energy is joule

(e) The potential energy is due to its state rest of position and kinetic energy of the body is due to its state of motion

(f) Gravitational potential energy U = mass times force of gravity on unit mass times height

(g) Kinetic energy = frac12 times mass times (speed)2

(h) Power P = work donetime taken

(i) The S I unit of power is watt

(j) IHP = 746 W

BIOLOGY Chapter -5 The endocrine system and adolescence

Today we will discuss about thelocation and functions of secreted hormones of adrenal and Pancreas

Q5 Write location hormone secreted main functions and deficiency diseases of pancreas and adrenal glands

Endocrine Glands

Location Hormones secreted

Functions and Deficiency Diseases

1Adrenal gland

2 Pancreas Gland

On the top of each kidney

In between stomach and small intestine

i)Adrenaline from adrenal medulla

ii)Cortisone from adrenal cortex

i) Insulin

ii) Glucagon

It helps a person deal with any kind of emergency situation or emotional stressIt increases the heart beat rate of respiration and blood pressure

a) It regulates carbohydrates protein and fat metabolism

b) It regulates the salt and water balance in the body

a) It changes excess glucose into glycogen

b) It stimulates the cells to burn extra glucose to provide heat amp energy

Less secretion causes diabetes mellitus

Excessive secretions causeinsulin shock

a) It stimulates the breakdown of glycogen into glucose

b) It increases the level of glucose in blood

History Traders to rulers The Battle of Buxar was fought on 22 October 1764 between the forces under the command of the British East India Company led by Hector Munro and the combined armies of Mir Qasim the Nawab of Bengal till 1763 Mir Jafar was made the Nawab of Bengal for a second time in 1763 by the Company just after the battle After being defeated in 4 battles in katwa and Udaynala the Nawab of Awadh Siraj id Daula and the Mughal emperor Shah Alam II accompanied by Raja Balwant Singh of Kashi made an alliance with Mir Qasim The battle was fought at Buxar a small fortified

Answer the following questions- Short note-Battle of BuxarHomework-learn

town within the territory of Bihar located on the banks of the Ganga river about 130 kilometres (81 mi) west of Patna it was a decisive victory for the British East India Company The war was brought to an end by the Treaty of Allahabad in 1765

EnglishLiterature

The west wind-John Mansfield

In the poem The West Wind by John Masefield the poet starts by describingwith very poetic imagery of birds how the west wind is different from other winds its a warm wind full of birds cries There is a touch of melancholy perhaps home-sickness as he describes how it brings tears too and memories from an old land He goes on to describe the restful pastoral beauty of the land where even the dead can lie in the green He then brings in voicesperhaps of family and friends calling him home as he is missing Aprils beautyThe voices then tempt him some more with idyllic images from home (white blossom young green cornrunning rabbitswarm sun) The voices seem to presume that the poets heart is sorrowful bruised and soreThe end of the poem sees the poet appear to make a decision he will go home as he has decided that is where he truly belongs

Write the synopsis of the following words

1 Daffodils- a tall yellow flower that grows in the spring

2 Orchards- a piece of land on which fruit trees are grown

3 Blossom- a flower or a mass of flowers especially on a fruit tree in spring

4 Thrushes- a bird5 Larks- a small brown bird that

makes a pleasant sound6 Bruised- an injury7 Aching- pain 8 Tread- to put your foot down

while you are walking9 Balm-10 May-11 Fluting-

(Write from the book in your copy)

MAT

HEM

ATIC

S

Ch 1

1Al

gebr

ic E

xpre

ssio

n

1 Constant A symbol which has fixed value is called a constant[eg 8 23 -15 radic3 etc]

2 VariableA symbol which does not have any fixed value but may be assigned value (values) according to the requirement is called variable or literal[eg x y p q etc]

3 TermsA term is a number (constant) a variable a combination (product or quotient) of numbers and variables[eg 7 x 5x etc]

4 Algebric expressionA single term or acombination of two or more terms connected by plus (+) or minus (-) sign forms an algebraic expression[eg 5-y 3x2-5x xy-6z+4 etc]

5 PolynomialAn algebraic expression which contains more than one term is called a polynomial (multinomial)[eg x2-5x 5y+xy+x2y etc]

6 Degree of polynomial(a) When the polynomial contains only one variable the highest power of the variable is the degree of the polynomialeg the degree of the polynomial of 4x-7x5+8 is 5(b) When the polynomial contains two or more variablesStep (i) Find the powers of the variables in each term (ii) The highest sum of the powers is taken to be the degree of the polynomialeg the degree of the polynomial 5x2y-4x3y5+6 is = 3+5 = 8Remember An algebraic expression is a polynomial if degree of each term used in it is a non-negative integer

Exercise ndash 11(A)

1 Separate the constants and variables from the following

-7 7+x 7x+yz radic5 radic xy 3 yz

8 45y -3x

Solution Constant Variables-7 radic5 7+x 7x+yz radic xy

3 yz8

45y -3x

2 Write the number of terms in each of the following polynomials(i) 5x2+3timesax (ii) axdivide4-7 (iii) ax-by+ytimesz (iv) 23+atimesbdivide2

Solution Polynomials Number of terms(i) 5x2+3timesax 2(ii) axdivide4-7 2(iii) ax-by+ytimesz 3(iv) 23+atimesbdivide2 2

4 Write the degree of the each polynomials(i) xy+7z (ii) x2-6x3+8 (iii) y-6y2+5y8 (iv) xyz-3 (vi) x5y7-8x3y8+10x4y4z4

Solution Polynomials Degree(i) xy+7z 2(ii) x2-6x3+8 3(iii) y-6y2+5y8 8(iv) xyz-3 3(vi)x5y7-8x3y8+10x4y4z4 12

5Write the coefficient of(i) ab in 7abx (iv) 8 in a2-8ax+a (v) 4xy in x2-4xy+y2

SolutionCoefficient

(i) ab in 7abx 7x(iv) 8 in a2-8ax+a -ax(v) 4xy in x2-4xy+y2 -1

7 CoefficientAny factor of an algebraic quantity is called the coefficient of the remaining quantityeg in the algebraic term 7xyz 7 is coefficient of xyz 7x is coefficient of yz and so on

8 Like term The terms having the same literal coefficient are called like terms and those having different literal coefficients are called unlike terms

eg (i) 5xyz 8xyz -6xyz and 23xyz are like

terms(ii) 7xy2 8x2yz and -15xyz2 are unlike terms

6 in 57xy2z3 write the coefficient of

(i) 5 (vii) 5xy2 (viii) 17yz (xi) 5xyz

Solution Coefficient

(i) 5 17

xy2z3

(vii) 5xy2 17z3

(viii) 17yz

5xyzsup2

(xi) 5xyz 17yz2

7 In polynomial given below separate the like terms(ii) y2z3 xy2z3 -58x2yz -4y2z3 -8xz3y2 3x2yz and 2z3y2

Solution y2z3 -4y2z3 2z3y2 are like terms

xy2z3 -8xz3y2 are like terms

-58x2yz 3x2yz are like terms

Class IXSubject Topic Summary Execution

Bengali (2nd language)

বাগzwnjধারাzwnj বা ধারা-বা ধারা ল হিবকেশষ পরকার বাক -হিb -াকেবর এক হিবকেশষ পরকাশরীহিত াকেক কতগকেলা কার সমষটির মকেধয এগহিলকেক বা ধারা বকেল আবার কতগকেলা শকেবদর বাধাধরা যকান রীহিত যনই য-াকেব চকেল আসকে যসই -াকেবই চকেল আসকে তখন যসই শবদগহিল খন একক -াকেব অG পরকাশ ককের তখন একের বা ধারা বকেল বা ধারার পরকেয়া -াষাকেক আরও সFর ককের যতাকেল

অকাল পকক(অপহিরনত বয়কেস পাকাহিম)-মাতর শ বর বয়কেস যমকেয়টির া মকেখর কা তাকেত অকালপককতা ধরা পকে

অককা পাওয়া( মারা াওয়া) ndash পকেকIমারটি পকেকIমারকেত হিকেয় বাসাতরীকের াকেত মার যখকেত যখকেত অককা যপল

অহি| পরীকষা ( কঠিন ও পরকত পরীকষা)- যকেলটির আজ ডাকতাহির যরজালট যবকেরাকেব এIাই তার জীবকেনর ব অহি| পরীকষা

অষটরমভা (ফাহিক) ndash রীতা মকেখই বকো বকো কা বকেল আর কাকেজর যবলায় অষটরমভা

অকমGার ধাী (অপাG) ndash সমনকেক হিনকেয় যকান ান কেব না ও একেকবাকেরই অকমGার ধাী

অকেনধর ষটি (অসাকেয়র সায়)- আহিশ বকেরর বকোর নাহিত ল অকেনধর ষটি তাকেক াা বকোর একম চকেল না

আকেককল গড়ম (তবহিদধ)- ার তহিম উপকার করকেল যসই যতামার হিবরকেদধ সাকষয হিকেয়কে শকেনই আমার আকেককল গড়ম

আষাকে লপ( অবাসতব লপ) ndashIাকা এখন যকেব না এIা বলকেলই ত এমন আষাকে লপ ফাার যকান রকার হিল না

Hindi- महायजञ ा इस हानी म लख न या बतान ा परयास किया ह कि किसी भी अचछ

2nd language

परसार(यशपाल ाय या पणय न ा फल अवशय मिमलता ह ोई भी परोपार अथवा पणय लिलए किया गया ाय बार नही 0ाता वह ए परार ा यजञ हए धनी सठ थ धम परायण और किवनमर सठ न आन ी यजञ किए थ और दान म न 0ान कितना धन दिदन दखिखयो म बात दिदया थादिदन पलट और सठ यहा गरीबी आ गई उन दिदनो यजञ बचन ी परथा थी सठ भी अपनी 0गह बचन लिलए डलपर ए सट यहा चलन ो तयार हए सठानी रासत लिलए रोटी पड म बाधर सठ ो द दी रासत म ए भख R ो दखर सठ न चारो रोटी उसो खिखला दी खर वह सठ यहा डलपर पहच तो उनी सठानी न उस महायजञ बचन ो हा यदिद बचन आए सठ न R ो रोटी खिखलान ो महायजञ नही समझा और वापस लौट आया घर आर शाम ो उसी घर म उस ए बडा ख0ाना मिमला 0ो उस दवारा किए गएrsquo महायजञrsquo ा परसार था

English language

Letter formal The heading the name and address of the person you are writing to must be included beneath your own address In formal letters ldquoblock stylerdquo of address is preferred

Subject complain in brief

Salutation If the person you are writing to is known to you you may begin ldquoDear MrrdquoOr ldquoDear Mrsrdquo In all other instances you should begin ldquoDear Sirrdquo or ldquoDear Madamrdquo Or ldquoSirsrdquo

The body A formal or business letter has four partsReference The letter should begin by referring to a letter you have received an advertisement or the reason that has prompted you to writeInformation In the second paragraph it is necessary to supply more detailed information that is related to the referencePurpose Here you must give the reason why you are writing the letter This must be stated clearly and ensure that it is relevant to the question that has been setConclusion round off the letter with some polite remarkThe subscription when a letter has begun with dear sir sirs Madam you should end with Yours faithfully or yours truly When however you address a person by name you must conclude with the words ldquoYours sincerelyrdquo

1 A park in your locality is slowly being used as a rubbish dump Write a letter to the Mayor of your city pointing out the nuisance and danger of this Request that action be taken to stop this immediately

Or2 You being a boarder ordered a set of lab manuals from a famous book shop in the town They sent you a wrong set of books Write a letter to the manager of the book shop

Chemistry Chapter-1 1)CHEMICAL FORMULA- Q What is the Significance of

L-2The Language of Chemistrybull Chemical Formula

Itrsquos a symbolic representation of a chemical substance eg ndash The formula of Sulphuric acid is H2SO4

2) Steps of writing Chemical Formula of a given substance-

1 Write the symbols of the constituent atoms or radicals side by side Keep the basic radical on LHS and acid radical on the RHS ( Na+Cl- )2 In case of a radical having more than one atom( compound radical) enclose the radical in a bracket eg (SO4-)3 Write the valencies of each radical on its right hand top4 If the valencies of the two radicals are divisible by a common factor then divide the valencies by the common factor5 Invert (criss-cross) the valency number ie write the valency of one atom below the second atom and vice versa 6 On interchanging if valency number is lsquoone the figure lsquoonersquo is never writtenFor Example- Compound -Calcium Nitrate1 Writing the symbols- Ca(NO3)2 Writing the valencies on their right hand top- Ca2(NO3)1

3 Valency numeral in simple ratio- Ca2(NO3)1

4 Criss-cross- Ca 2NO3 1

5 Writing the formula of the compound- Ca(NO3)2

Chemical formula

A The formula of a substance conveys the following information regarding a substance 1 The name of the substance (qualitative)2 The elements constituting the substance (qualitative)3 The number of various atoms present in a molecule of the substance (quantitative)4 Molecular weight of the substance and the relative weights of different elements present in it (qualitative)

Q What are the limitations of Chemical Formula

A The chemical formula suffers from the following limitations-I It fails to convey whether the elements in a molecule are present in the form of atoms or ionsFor example the formula KBr fails to tell us whether Potassium and Bromine are present in the form of ions II It does not tell anything about the binding force that holds atom in a molecule togetherIII It does not tell us about the arrangement of various atoms with respect to one another within the molecule

Q Examples of Some Chemicals with their Formula Chemical name and Common Name-

A Given in the class notesCommercial Studies

Joint Stock Company

Let us discuss about the demerits of Joint Stock CompanyDespite so many advantages it has got many disadvantages which are as follows

Difficulty in FormationDelay in Decision makingExcessive Government ControlLack of Secrecy

Company can be classified into several categories based on incorporation

QuestionExplain the demerits of Joint Stock CompanyAnswer) 1 Difficulty in Formation The legal requirements and formalities required to be completed are so many The cost involved is quite heavy It has to approach large number of people for its capital It cannot start its business unless certificate of incorporation has been obtained This is granted after a long time when all the formalities are completed

Chartered CompanyStatutory CompanyRegistered Company

Delay in Decision making In this form of organization decisions are not made by single individual All important decisions are taken by the Board of Directors Decision-making process is time-consuming So many opportunities may be costly because of delay in decision-making Promptness of decisions which is a common feature of sole trader ship and partnership is not found in a company

Excessive Government ControlA company and the management have to function well within the law and the provisions of Companies Act are quite elaborate and complex At every step it is necessary to comply with its provisions lest the company and the management should be penalized The penalties are quite heavy and in several cases officers in default can be punished with imprisonment This hampers the proper functioning of the company

Lack of Secrecy The management of companies remains in the hands of many persons Every important thing is discussed in the meetings of Board of Directors Hence secrets of the business cannot be maintained In case of sole proprietorship and partnership forms of organisation such secrecy is possible because a few persons are involved in the management

2 Define the following

Chartered Company- The crown in exercise of the royal prerogative has power to create a corporation by the grant of a charter to persons assenting to be incorporated Such companies or corporations are known as chartered companies Examples of this type of companies are Bank of England (1694) East India Company (1600) The powers and the nature of business of a chartered company are defined by the charter which incorporates it After the country attained independence these types of companies do not exist

in IndiaStatutory Company- A company may be incorporated by means of a special Act of the Parliament or any state legislature Such companies are called statutory companies Instances of statutory companies in India are Reserve Bank of India the Life Insurance Corporation of India the Food Corporation of India etc The provisions of the Companies Act 1956 apply to statutory companies except where the said provisions are inconsistent with the provisions of the Act creating them Statutory companies are mostly invested with compulsory powersRegistered companiesCompanies registered under the Companies Act 1956 or earlier Companies Acts are called registered companies Such companies come into existence when they are registered under the Companies Act and a certificate of incorporation is granted to them by the Registrar

Economics

Chapter-4Basic problems of Economy

Today let us discuss with the topic Production Possibility curve

QuestionExplain the concept of Production Possibility Curve with the help of diagram

Answer) Production Possibility curve is a locus of all possible combinations of two commodities which can be produced in a country with its given resources and technology

The above diagram shows that with the given resources and technology the economy can produce maximum either 5 thousand meters of cloth or 15 thousand quintals of wheat or any other combination of the two goods like B( 1 thousand meters of cloth and 14 thousand quintals of wheat C ( 2 thousands meters of cloth and 12 thousand quintals of wheat) etcProduction Possibility curve is also called production possibility boundary or frontier as it sets the maximum limit of what it is possible to produce with given resources

Geography

Rotationand Revolution

SUNrsquoS POSITION AND SEASONAL CHANGES EQUINOXES ndash SPRING AND AUTUMN

Q1 What is Spring EquinoxA1 On 21st March sunrays fall directly on the equator On that day

As the Equator divides the Earth into two equal halves the sun rays fall directly on the equator twice in a year Equinoxes means equal Spring EquinoxOn 21st March sunrays fall directly on the equator On that day the duration of day and night both are equal ( 12 hours day and 12 hours night) on every places located on equator This day is called as Spring EquinoxAutumn EquinoxOn 23rd September sunrays fall directly on the equator On that day the duration of day and night both are equal ( 12 hours day and 12 hours night) on every places located on equator This day is called as Autumn Equinox

SOLSTICES ndash SUMMER AND WINTERDue to inclination of the Earth on its axis and the apparent movement of the sun the sun rays fall directly on both tropics once in a year Solstice is a Latin word which mean ldquothe Sun standing stillrdquoSummer SolsticesAfter 21st March there is an apparent movement of the Sun to the north of the equator The apparent northward movement up to 21st June when the Sun appears overhead at the Tropic of Cancer (22frac12degN) The sun appears to stand still at this position and then moves southwards towards the equator This position of the Sun on 21st June is known as Summer Solstices On that day the duration of day and night both are equal ( 12 hours day and 12 hours night) on every places located on Tropic of Cancer (22frac12degN)Winter solstices The apparent southward movement of the Sun continues beyond the equator till 22nd

December On this day the Sun is overhead at the Tropic of Capricorn

the duration of day and night both are equal ( 12 hours day and 12 hours night) on every places located on equator This day is called as Spring Equinox

Q2 What do you mean by EquinoxA2 Equinoxes means equal It is use to explain the equal duration of day and night ( 12 hours day and 12 hours night) on the Earth

Q3 On which date the longest day in Tropic of CancerA3 21st June

Q4 What is the meaning of SolsticeA4 Solstice is a Latin word which mean ldquothe Sun standing stillrdquo

Q5 Which is the longest day in southern hemisphereA5 22nd December

Q6 On what date does the Arctic Circle experience the lsquoMidnight SunrsquoA6 On 21 June the Arctic Circle experiences the lsquoMidnight Sunrsquo

Q7 What is cause of Midnight Sun in NorwayA7 During the summer solstice (21 June) the North Pole is inclined towards the Sun Therefore the duration of sunlight or daytime increases from 12 hours at the Equator to 24 hours at the Arctic Circle and beyond Thatrsquos why The region beyond the Arctic Circle especially Norway is known as the Land of the Midnight Sun because there the Sun does not rise or set on 21 June

Q8 Match the column A with BA B

Summer Solstice 21st March

Autumn Equinox 23rd

September

Winter Solstice 21st June

(22frac12degS) This position of the Sun is referred to as the Winter Solstice because it marks the winter season in the Northern Hemisphere On that day the duration of day and night both are equal ( 12 hours day and 12 hours night) on every places located on Tropic of Capricorn (22frac12degS)SEASONS AND DURATION OF DAY AND NIGHT During the equinoxes all places on the Earth have 12 hours of day and 12 hours of night Due to the revolution of the Earth round the Sun on an inclined axis the duration of day and night varies according to seasons and the latitude of a placeDuring the summer solstice (21 June) the North Pole is inclined towards the Sun Therefore the duration of sunlight or daytime increases from 12 hours at the Equator to 24 hours at the Arctic Circle and beyondThe region beyond the Arctic Circle especially Norway is known as the Land of the Midnight Sun because there the Sun does not rise or set on 21 JuneAt the North Pole there will be six months of daylight The Sun will be seen always above the horizon at a low angle At 66degN 24 hours of sunlight can be seen only on 21 June Hammerfest in northern Norway is a place of tourist attraction for observing the phenomenon of the Midnight Sun This place has continuous daylight from 13 May to 29 July This place is easily accessible to tourists and has hotels and other facilities The view of the midnight Sun from here is enthrallingIn the Southern Hemisphere the duration of daylight decreases from 12 hours at the equator to 0 hours beyond the Antarctic Circle In the South Polar Region there is 24 hours of darkness The Sun is always below the horizon In the Southern Hemisphere which experiences winter the duration of night-time is longer than the duration of daylight

Spring Equinox 22nd

December

A8 A B

Summer Solstice 21st June

Autumn Equinox 23rd

September

Winter Solstice 22nd

December

Spring Equinox 21st March

During winter solstice (22 December) the South Pole is inclined towards the Sun The Southern Hemisphere experiences summer and the Northern Hemisphere has winter Therefore the duration of daylight or sunlight is greater in the Southern Hemisphere than in the Northern HemisphereThe duration of daylight increases from 12 hours at the equator to 24 hours beyond the Antarctic Circle The South Polar Region has 24 hours of sunlight for many days continuously At the South Pole there will be six months of sunlight The Sun will always be seen at a low angle above the horizon In the Northern Hemisphere the duration of daylight will decrease from 12 hours at the equator to 0 hours at the Arctic Circle There are 24 hours of darkness in the North Polar region The duration of night is greater than the duration of daylight as one move northwards from the Equator It is evident from the above table that the duration of daylight is 12 hours throughout the year at the equator only As one moves away from the equator the seasonal variations in the duration of daylight increase The seasonal variations in the duration of daylight are maximum at the Polar Regions

Subject Eng Literature (The Merchant of Venice ndash William Shakespeare)Topic Act II Scene 7 Lines 36 to 80 (End of scene ) [Students should read the original play and also the paraphrase provided]

Summary Questions amp AnswersThe Prince then examines the inscription on the silver casket which says ldquoWho chooseth me shall get as much as he deservesrdquo The Prince says that he deserves Portia more than anybody else because of his high rank his noble birth and his great wealth and power But then he argues that silver is ten times

(1) (Act II Sc 7 L 39-47)

From the four corners of the earth they come

To kiss this shrine this mortal breathing saint

The Hyrcanian deserts and the vasty wildsOf wide Arabia are as through-fares now

inferior to gold and therefore he cannot believe that the portrait of such a beautiful lady as Portia can be contained in the silver casket He decides to see the inscription on the golden casket before making his decision

The Prince goes to examine the inscription on the golden casket which says ldquoWho chooseth me shall get what many men desirerdquo The Prince believes that the whole world desires to possess Portia otherwise so many suitors would not have come from all corners of the world for winning Portia Some of them have come from the distant lands of Persia and Arabia The deserts of Persia (Hyrcanian deserts) and the boundless desolate lands of Arabia have been crossed by the Princes seeking the hand of Portia He contrasts this casket containing Portiarsquos portrait with the old English gold coin bearing the image of the archangel (angel of the highest rank) He goes on to remark that while the figure of the archangel is engraved (Insculped) upon the English coin the picture of Portia who is beautiful as an angel lies hidden inside one of the caskets namely the Golden Casket (Golden Bed)

On the basis of his assessment of the inscription on the golden casket the Prince decides to choose the golden casket He asks for the key and opens the golden casket only to find therein an empty human skull holding a roll of

For princes to come view fair PortiaThe watery kingdom whose ambitious headSpets in the face of heaven is no barTo stop the foreign spirits but they comeAs orsquoer a brook to see fair Portia

(i) Explain the occasion for the above mentioned speech

These are the comments of the Prince of Morocco after he reads the inscription on the golden casket His mental process is revealed to us in these words We find him debating within himself as to which casket he should choose

(ii) What light does the above speech throw on the personality of Prince of Morocco

From the above mentioned speech we come to know that the Prince of Morocco is keen to marry Portia He is the type of person who is easily taken away by outward appearance He is in love with Portia because of her beauty

(iii) What information can you gather about Portia from the above mentioned lines

The given speech shows that Portia is a very beautiful lady She must be possessed of good qualities because many suitors come to her place from all over the world with a desire to get married to her The Prince of Morocco is so impressed by her beauty that he calls her a saint According to him the whole world is desirous of having her

(iv) Elucidate the significance of the first two lines

In these lines the Prince of Morocco pays a compliment to Portia These lines show his admiration for her He says that people come from all parts of the world to see fair Portia

(v) Explain the meaning of the last four lines of the

passage

In these lines the Prince of Morocco says that even the vast oceans which throw a challenge at the sky are unable to prevent men from coming to Portiarsquos place to have a glimpse of her These lines are also a tribute to Portiarsquos beauty and good qualities Many men voyage across the ocean treating it as a mere stream to see the beautiful Portia

paper in which is written that whoever happens to be guided by the glitter of things is invariably deceived

On reading the scroll the Prince says that he is too sad at heart to speak a more formal farewell and leaves with his followers amidst a sound of trumpets

After the Prince of Morocco leaves Portia remarks that the Prince is a gentle fellow but she is rid of him May all persons of his nature make a similar choice

IMPORTANT PASSAGES EXPLAINED

(Act II Sc 7 L 39-43)From the four corners of the earth they come

To kiss this shrine this mortal breathing saintThe Hyrcanian deserts and the vasty wildsOf wide Arabia are as through-fares nowFor princes to come view fair Portia

Context

This passage occurs in Act II Scene 7 in The Merchant of Venice This is part of the speech made by the Prince of Morocco

(2)

(Act II Sc 7 L 48-53)

MOROCCO One of these three contains her heavenly pictureIst like that lead contains her

Twere damnation To think so base a thought it were too grossTo rib her cerecloth in the obscure graveOr shall I think in silver shes immurdBeing ten times undervalued to tried gold

(i) What meaning does the Prince of Morocco find out of the inscription of the golden casket What have Belmont and Portiarsquos house been called and why

The inscription on the golden casket is ldquoWho chooseth me shall gain what many men desirerdquo The Prince finds out that it means that the chooser of the golden casket will get Portia because many men desire her In fact the entire world desires her Because of the coming of many suitors to Belmont from different countries in order to win Portiarsquos hand Belmont has become a centre of pilgrimage and her house is the shrine where saintly Portia is installed

(ii) What does the Prince of Morocco do before making the final choice of the casket Which is the correct casket and who will win Portiarsquos hand

The Prince of Morocco surveys and analyses the inscriptions on the casket of lead silver and gold Before making the final choice like a very systematic and methodical person he once again considers the claims of the caskets The casket containing Portiarsquos picture is the correct casket and the person choosing it will win Portiarsquos hand

Explanation

While praising Portia the Prince of Morocco conceives Portia as a goddess whose image is placed inside one of the caskets Many suitors are coming from far and wide the north and the south the east and the west (Four corners) in order to try their luck Some of them have come from the distant land of Persia and Arabia The deserts of Persia (Hyrcanian deserts) and the boundless desolate lands of Arabia have been crossed by the Princes seeking the hand of Portia All this shows that Portia is indeed the most beautiful lady of the world

(iii) What does the Prince of Morocco say in his estimation while examining the motto on the silver casket What does he find in the golden casket

While examining the motto on the silver casket which says ldquoWho chooseth me shall get as much as he deservesrdquo Morocco says that in his own estimation he surely deserves Portia in all respects ndash rank birth wealth etc

He chooses the golden casket When he opens it he finds an empty human skull holding a scroll in which it is written that those who are attracted by the glittering outside of things are always deceived as Morocco has been deceived

(iv) What kind of nature does the Prince of Morocco have

The Prince of Morocco has a simple nature who does not look deeply into the inner meaning of things but is dazzled by the outward appearance of gold He is inclined to over-estimate his own value and does not realize that it is a duty to ldquogive and hazardrdquo To say that he will not hazard for lead shows that he misreads the true meaning of the inscription which is that he should be prepared to ldquohazard all he hathrdquo for Portia So his feeling is only one of fascination and romantic attraction

(v) Do you think that the lottery of the caskets is not a matter that will be determined by chance

In fact the lottery of the casket is not a matter that will be determined by mere chance but that it is a true test of character and of sincerity which is amply proved not only by Moroccorsquos choice but also by the arguments which he uses to help him in his choice

(Act II Sc 7 L 55-59)

They have in England

A coin that bears the figure of an angelStamped in gold but thats insculpd uponBut here an angel in a golden bedLies all within

Context

(3)

(Act II Sc 7 L 63-77)A carrion Death within whose empty eye

There is a written scroll Ill read the writing

All that glisters is not goldOften have you heard that toldMany a man his life hath soldBut my outside to beholdGilded tombs do worms infoldHad you been as wise as boldYoung in limbs in judgment oldYour answer had not been inscrolld

This passage occurs in Act II Scene 7 in The Merchant of Venice This is part of the speech made by the Prince of Morocco

Explanation

In this passage the Prince of Morocco bestows high praise on Portia whose hand he is seeking He contrasts this casket containing Portiarsquos portrait with the old English gold coin bearing the image of the archangel (angel of the highest rank) He goes on to remark that while the figure of the archangel is engraved (Insculped) upon the English coin the picture of Portia who is beautiful as an angel lies hidden inside one of the caskets namely the Golden Casket (Golden Bed) In the day of Elizabeth silver was ten times inferior in value to gold Therefore the Prince of Morocco believing that Portiarsquos portrait is contained in the Golden Casket decides to choose the Golden Casket

Fare you well your suit is coldCold indeed and labour lostThen farewell heat and welcome frostmdashPortia adieu I have too grievd a heartTo take a tedious leave Thus losers part

(i) What reward does the Prince of Morocco get after making a wrong choice of the Casket How does he feel

After making the wrong choice in selecting the casket of gold the Prince of Morocco as a reward earns a rebuke in the form of a scroll tucked in the empty eye-socket of a skull kept in the casket of gold The Prince is shocked and disappointed He becomes all the more sad and dejected when he reads the scroll which points to his foolishness in being misled by the appearance and outward show as indicative of its worth

(ii) How does the Prince respond after reading the scroll

After reading the scroll the Prince though upset accepts the result with good grace and decorum befitting a royal suitor and true sportsman He says that his love-suit is really cold otherwise he would have chosen correctly but now his efforts have been in vain So he bids farewell to Portia to the warmth and enthusiasm of love and welcomes the cold and bitterness of dejection and misery of life which lies ahead

(iii) What request does he make to Portia and why

After being failure in his mission he requests Portia to give him permission to leave at once because he is too sad to undergo the tediousness of a formal leave-taking He tells that it is the manner in which defeated persons part unceremoniously

(iv) Explain the following lines

ldquoAll that glisters is not goldOften have you heard that toldMany a man his life hath soldBut my outside to beholdGilded tombs do worms infoldrdquo

Mere glitter does not make a metal to be gold Man has often been warned against appearance but it has been of no use Many people have sacrificed their lives only to seek the outer appearance of gold Worms are found inside the gilded

monuments

Class XSubject Topic Summary Execution

Hindi 2ndlang

नया रासता भाग 6 मायाराम 0ी घर म धनी मल 0ी और उनी बटी सरिरता ी ही चचा बनी रहती थी अमिमत ो इसम ोई रलिच ना थी वह धनी घर ी लडी स शादी र सवय ो बचना नही चाहता था उसा भी सवाणिभमान ह ईशवर ी पा

स उस पास पस ी ोई मी नही थी अभी उसन फकटरी ही लगाई थी उसी समझ बाहर था कि उस घर वालो ा झाव पस ी तरफ कयो

ह उसन मा स सवाल किया कि मा तम सरिरता स मरी शादी कयो रना चाहती हो मा न उस समझाया कि वह दखन म बरी नही ह और किफर खानदान अचछा

ह वह ए शल गरहणी रप म घर सभाल सगी अमिमत न मा ो इस बात ा एहसास राया कि मीन सबध लिलए मना रन पर उस दिदल

पर कया बीती होगी मा और अमिमत ी लडी बार म ाफी बात हईमा ा झाव सरिरता ी तरफ था कयोकि वह घर पर अचछा दह0 लर आ रही

थी अमिमत न अपनी मौसी ी बरी हालत बार म बताया कि किस तरह वह बड घर ी खानदानी बटी लाई थी और आ0 उसी हालत कितनी खराब ह लाई थी बहकलब 0ाती ह और बचचो ो भी नही दखती ह बात चल ही रही

थी कि तभी ए ार बाहर आर री धनी मल0ी घर अदर आए और पीछ स डराइवर फल ी ए टोरी लर आया अदर आए और पीछ स

डराइवर ए टोरी फल ी लर आया अमिमत ो फल ी पटी बरी लग रही थी अमिमत न पछ लिलया यह फल कयो ल आए ह प इन सब ी कया

0ररत थी उनो न 0वाब दिदया कि 4 पटी शमीर स मगाए थ अमिमत ो या सनर करोध आ गया तभी उस किपता 0ी आ गए उन आत ही अमिमत उठर बाहर चला गया वहा वहा मा पास आर बठ गया और बोला

अभी रिरशता तय नही हआ और धनी मल 0ी धनी मल 0ी फल ी पटी लर चलआय मा न समझाया कि 0ब सबध 0ड 0ाता ह तो खाली हाथ नही

आत अमिमत न मा स हा कि तम सबन सरिरता ो इस घर म लान ी ठान रखी ह धनीमल 0ी उस दिदन सरिरता ो दखन ी तारीख तय रन आय थ

Commercial Studies

Banking Nowadays Bank provide easy and quick services through internet facilities methods of Banking is called internet bankingIn order to save the time and money involved in visiting Bank branches people increasingly prefer to have internet banking

There are different modes of doing internet banking or transferring money through online They areReal Time Gross Settlement (RTGS)National Electronic Fund Transfers (NEFT)

1

Question

1) Explain the term RTGS Write the features of RTGS

Answer)The acronym RTGS stands for Real Time Gross Settlement which may be defined as the continuous real time settlement of funds transfer individually on and order by order basis without netting lsquoReal timersquo may be defined as the processing of instructions at the time they are received rather than at some letter time lsquoGross settlementrsquo may be defined as the settlement of transfer instructions which occurs

individually

Features of RTGS1It is the continuous settlement of

funds transfer individually on an order by order basis

2RTGS facility is provided only by CBS core banking solution enabled Bank branches

3Amount charged from the customer for RTGS transactions vary from bank to bank

2) Explain the term NEFT Write the features of NEFT

Answer) National electronic funds transfer may be defined as a nationwide system that facilitates individuals Farms and copper operates to electronically transfer funds from any bank branch to any individual farm or corporate having an account with any other bank branch in the country

Features of NEFT2 Transfer can be made 7 times on

weekdays and 6 times on Saturday

3 NEFT cannot be used to receive foreign remittances

4 NEFT transaction takes place in batches

5 A bank branch must be NEFT enabled to become a part of NEFT fund transfer network

6 There is no maximum or minimum amount that can be transferred through NEFT when one bank has a bank account

English Language

CompositionEssay

A composition is an art of creating a piece of writing on any topic or subject It is the writing correctly beautifully and clearly in order to make some interesting reading Structure of the composition

Introduction ( you lay the foundation for your composition)

Body (it constitutes the main part of the essay)

Conclusion (final statement that leaves a lasting impression)

Kinds of essays1 The Narrative essay2 The descriptive essay3 The reflective essay4 The argumentative essay

Write a composition on any one of the following topics (350- 400 words)

1 Friendship Or2 The first day of your school

Subject Eng Literature (The Merchant of Venice ndash William Shakespeare)Topic Act V Scene 1 Lines 127 to 158 (Nerissa helliphellip The clerk will nersquoer wear hair onrsquos face that had it) [Students should read the original play and also the paraphrase given in the school prescribed textbook]

Summary Revision Questions o Soon thereafter Bassanio Gratiano

and Antonio arrive

o Bassanio tells Portia that he is feeling as if it is morning because of the presence of Portia who is shining like the sun When Antonio is introduced by Bassanio to Portia she tells Bassanio that he should be grateful to Antonio who took so much trouble on his account even to the extent of risking his life

o Nerissa starts quarrelling with Gratiano and demands that he show her the ring she had presented to him and which she had warned him not to lose She suspects that Gratiano must have presented the ring to some young woman and not to the lawyerrsquos clerk as he repeatedly says and assures

Answer the following questions to check your preparation of Act IV Scenes 1 and 2

You must attempt only after you have completed your preparation of Act IV The answers must be in complete sentences using textual evidence (with citation) when necessary

[It would be in your own interest to attempt the above questions honestly totally refraining from consulting your textbook or your notes during answering After completion you should correct the paper yourself consulting the textbooknotes etc and award marks as specified Please let me know the marks you scored through WhatsApp in the group or to my personal WhatsApp]

Act IV Scene 1 (each question carries 2 marks)

1 What did the Duke try to do for Antonio

2 Why does Shylock refuse to show mercy How does he justify his stance

3 Why does Antonio say he is ready to die 4 What information is contained in Bellariorsquos letter

5 Why does Portia (as Balthazar) assert that Shylock must show mercy How does he respond

6 What offers are made to Shylock to get him to spare Antonio How are they received

7 What does Antoniorsquos speech as he faces the prospect of Shylockrsquos knife tell you about his character

8 How do Bassanio and Gratiano react to the looming prospect of Antoniorsquos demise

9 How does Portia (as Balthazar) use the law to turn the tables on Shylock

10 What does the Duke decree should happen to Shylock Why What happens to Shylockrsquos estate

11 What does Portia ask Bassanio as payment for her ldquoservicesrdquo What is his initial response What makes him change his mind

Act IV Scene 2 (each question carries 1frac12 marks)

1 What does Gratiano bring to Portia (Balthazar)

2 What does Nerissa plan on getting from Gratiano What does Portiarsquos comment suggest about men

ECO-10 280620 Topic-Supply AnalysisSHIFTING OF SUPPLY

But if there is change in factors other than the price of the commodity then either more is supplied at the same price or less supplied at the same price In such cases the price of the commodity remains constant but there is a change in other factors like change in the price of inputs change in technology of production change in price of other related goods change in taxation policy of the government etc For example there is an improvement in the technology of production of the commodity in question It leads to decrease in per unit of cost production of the commodity The firm is willing to sell more quantity of the commodity at the same price So the supply other commodity increases at the same price This increase in supply is shown by rightward shift of supply curve On the other hand if the firm uses inferior technology of production the cost of production per unit of the commodity increases The firm is willing to sell less quantity at the same price So the supply of the commodity decreases at the same price This decrease in supply is shown by leftward shift of the supply curve The above cases of increase and decrease in supply can be shown with the help of the following figures

Y INCREASE OF SUPPLY Price (Rs) s

P A s1

B

s

X` O s1 X

q q1

Y` Quantity demanded (in units)

Y DECREASE IN SUPPLY s2

s

price (Rs)

C

p A

s2

s

X` o X

q2 q

Y` Quantity demanded ( in units)

Main factors causing increase in supply or rightward shift of supply Curve(i) Fall in the price of other related goods

(ii) Fall in the price of inputsfactors(iii) Use of better technology in production(iv) Decrease in the rate of excise duty by government(v) If the objective of producer changes from profit maximization to salesMaximization

Main factors causing decrease in supply or leftward shift of supply curve(i) Increase in the price of other related goods(ii) Rise in the price of inputsfactors(iii) Use of inferior technology in production(iv) Increase in the rate of excise duty by the government(v) If the objective

Subject - Biology Topic ndash Chapter mdash6 PhotosynthesisSummary Execution

Today we will know about photosynthesis and its stages

Q1 What do you mean by photosynthesis The process by which living plants containing chlorophyll produce food

substances from carbon-di- oxide and water by using light energy Sunlight

6CO2 +12 H2O----------------------- C6 H12O6 + 6H2O + 6O2

Chlorophyll

Q2 What are the importance of photosynthesis I) Food for all Green plants trap solar energy by photosynthesis

process and supply food and energy for all living organisms either directly or indirectly

Ii) Oxygen to breathe in by product of photosynthesis is oxygen which is essential for all living organisms respiration

Q3 Write about two main phases of photosynthesis A Light dependent phase This phase occur in grana of chloroplast I) The chlorophyll on exposure to light energy becomes activated by

absorbing photons Ii) The absorbed energy is used in splitting the water molecules (H2O)

into its two components (H+ and OH- ) and releasing electron s 2H2O------------------------- 4H+ + 4e- +O2

Energy of 4 photons This reaction is known as photolysis

End products are H+ and oxygen water

B Light independent (Dark ) phase The reactions in this phase require no light energy

Here CO2 combine with H+ and produce glucose

Class XI

Subject Topic Summary ExecutionEVS Chapter-4 Legal

regimes for sustainable development

Environmental legislationEnvironmental legislation is the collection of laws and regulations pertaining to air quality water quality the wilderness endangered wildlife and other environmental factors The act ensures that matters important to the environment are thoroughly

Learn -The Forest (Conservation) Act 1980

considered in any decisions made by federal agencies

The Forest (Conservation) Act 1980 The Forest (Conservation) Act 1980 an Act of the Parliament of India to provide for the conservation of forests and for matters connected therewith or ancillary or incidental thereto It was further amended in 1988 This law extends to the whole of IndiaObjects and Reasons of the Forest Conservation Act

Deforestation causes ecological imbalance and leads to environmental deterioration Deforestation had been taking place on a large scale in the country and it had caused widespread concern The act seeks to check upon deforestation and de-reservation of forests

Subject Eng Literature (The Tempest ndash William Shakespeare) Topic Act II Scene 1 Lines 314 to 329 (End of scene)

[Students should read the original play and also the paraphrase given in the school prescribed textbook]Summary Questions amp Answers

Conspiracy of Antonio and Sebastian (Contd)

o As they approach Ariel appears again and wakes up Gonzalo by singing a tune in his ear Alonso also wakes up and they see both Sebastian and Antonio with drawn swords On being caught off guard they make up a story saying that they had heard a bellowing of bulls or lions

o They then moved to another part of the island

o Ariel at once rushes to Prospero to inform him of this development

SUMMING-UP of ACT-2 SCENE-1

(i) Among the survivors Ferdinand is separated from the rest which results in the disconsolate grief of Alonso as he took him for dead

(ii) The villainy of Antonio is confirmed

(iii) The supremacy of Prosperorsquos magic which resulted in the failure of the human conspiracy

(1)

(Act II Sc 1 L 311-325)SEBASTIAN Whiles we stood here securing your repose

Even now we heard a hollow burst of bellowing Like bulls or rather lions Didt not wake youIt struck mine ear most terribly

ALONSO I heard nothingANTONIO O rsquotwas a din to fright a monsters ear

To make an earthquake Sure it was the roarOf a whole herd of lions

ALONSO Heard you this GonzaloGONZALO Upon mine honour sir I heard a humming

And that a strange one too which did awake meI shaked you sir and cried As mine eyes opened I saw their weapons drawn There was a noiseThats verily rsquoTis best we stand upon our guardOr that we quit this place Lets draw our weapons

(i) Why has Prospero sent Ariel to Gonzalo and Alonso What does Ariel do to awaken Gonzalo

Prospero has already come to know by his magic powers the danger which threatens Gonzalo who had been Prosperorsquos friend and so he sent Ariel to preserve the lives of both Gonzalo and Alonso Prospero does not want that his scheme should remain unfulfilled Ariel begins to sing a song in Gonzalorsquos ears to awaken him(ii) Who are ready to carry out their plan Who takes steps to stop them Why does Gonzalo feel surprised after being awakened

Sebastian and Antonio are ready to carry out their plans They are standing with their swords drawn to kill Alonso and

(iv) We see two sets of contrasting characters Gonzalo-Adrian against Antonio-Sebastian

(v) The grief that works in Alonso can be perceived to his repentance for his association in Antoniorsquos crime against Prospero

Gonzalo Ariel takes steps to stop them from carrying out their nefarious scheme When Gonzalo is awakened by the song sung by Ariel into his ears he (Gonzalo) feels surprised because he sees Sebastian and Antonio standing with their swords drawn(iii) What reason do Sebastian and Antonio tell of drawing their swords when they are suspected by Alonso and Gonzalo

When Sebastian and Antonio are seen with their swords drawn they are looked with suspicion by Gonzalo and Alonso At first Sebastian tells them that as they stood here to guard them during their sleep they heard only a little before a sudden loud noise very much like the roaring of bulls or more probably that of lions Then Antonio follows him saying that this was a noise so terrible as to frighten even a monsterrsquos ears and this noise could even have shaken the earth and it was surely like the roaring of a multitude of lions Then seeing the danger they have drawn their swords Perhaps after hearing the terrible noise they (Gonzalo and Alonso) woke up from their sound sleep

(iv) What does Gonzalo tell Alonso about the strange noise What did he see on opening his eyes Gonzalo tells Alonso that he did not hear the sound of roaring but he heard a humming sound which was strange and which woke him up After waking up he gave him (Alonso) a shaking and a loud cry On opening his eyes he saw these two gentlemen standing with their swords drawn(v) What does Gonzalo suggest

Gonzalo suggests that there was a noise indeed and of that he has no doubt at all and suggests that the best course for them would be to remain alert and vigilant against any possible danger to their lives or to leave this place and move to some other part of the island

Class XIISubject Topic Summary Execution

Commerce

Chapter- Management

Today we will discuss about LEVELS OF MANAGEMENT

Levels of management is a series or chain of managerial positions from top to bottom It helps individuals to know their authority responsibilities and superior-subordinate relations among themselves There are mainly three levels of Management TOP LEVEL MANAGEMENTMIDDLE LEVEL MANAGEMENTLOWER LEVEL MANAGEMENT

Top level managementIt consists of members at the highest level in the management hierarchy This level includes Board Of Directors Chief Executive Managing Directors Chairman President Vice President

Rolefunctions of the top levelmanagement1To analyse evaluate and deal

with theexternal environment2 To determine the objectives and

policies of the business3 To strive for welfare and survival

of business

4 To create an organisational Framework consisting of authority responsibility relationship

Middle level management Congress of members or groups who are concerned with implementation of the policies let down by the top managementThis level includes head of the department such as finance manager marketing manager branch and regional managers departmental and divisional heads plant superintendent etc

Role of functions of the middle level management

1 To interpret the policies framed by top management

2 To assign duties and responsibilities to lower level managers

3 To select and appoint employees for middle and supervisory level and evaluate their performance

4 To co-operate with other departments for smooth functioning

Operational or supervisory level managementIt refers to the group are members who are concerned with execution of the work They are also known as fast line managers This level includes supervisor 4 men Section Officer clerk Inspector etc

Role of functions of the lower level management1 To plan and execute day-to-

day operations2 To supervise and control the workers3 To arrange materials and

tools to start the process and make arrangements for training

4 Today present workers grievance and suggestions before the management and

ensure safe and proper working conditions in the factory

Business Studies

Staff Appraisal Chapter- 10 Today let us start with a new chapter

Staff Appraisal

Meaning of Performance Appraisal

Performance Appraisal is the systematic evaluation of the performance of employees and to understand the abilities of a person for further growth and developmentThe supervisors measure the pay of employees and compare it with targets and plansThe supervisor analyses the factors behind work performances of employeesThe employers are in position to guide the employees for a better performance

Objectives of Performance Appraisal

Following are the objectives of Performance Appraisal

To maintain records in order to determine compensation packages wage structure salaries raises etc

To identify the strengths and weaknesses of employees to place right men on right job

To maintain and assess the potential present in a person for further growth and development

To provide a feedback to employees regarding their performance and related status

To provide a feedback to employees regarding their performance and related status

Importance of Performance Appraisal

Performance appraisal provides important and useful information for the assessment of employees skill

knowledge ability and overall job performance The following are the points which indicate the importance of performance appraisal in an organization

1 Performance appraisal helps supervisors to assess the work performance of their subordinates

2 Performance appraisal helps to assess the training and development needs of employees

3 Performance appraisal provides grounds for employees to correct their mistakes and it also provides proper guidance and criticism for employees development4 Performance appraisal provides reward for better performance

5 Performance appraisal helps to improve the communication system of the organization

6 Performance appraisal evaluates whether human resource programs being implemented in the organization have been effective

7 Performance appraisal helps to prepare pay structure for each employee working in the organization

8 Performance appraisal helps to review the potentiality of employees so that their future capability is anticipated

Geography

DRIANAGE The SubarnarekhaThe Subarnarekha and the Brahmaniinterposed between the Ganga and the Mahanadi deltas drain an area of 19300 sq kmand 39033 sq km respectively The drainage basins of these streams are shared byJharkhand Odisha west Bengal and Chhattisgarh The Brahmani is known as southKoel in its upper reaches in Jharkhand

The NarmadaThe Narmada rises in the Amarkantak hills of MadhyaPradesh It flows towards the West in a rift valleyformed due to a geological fault The total length of it is 1300 km All the tributaries of the

Q1 Name the two westward flowing rivers in the peninsular plateauA1 Narmada and Tapi are the only westward flowing rivers of the peninsular plateau

Q2 Differentiate between east-flowing rivers and west-flowing riversA2

East-flowing rivers

West-flowing rivers

Narmada are very short inlength Most of its tributaries join the main streamright anglesThe Narmada basin covers parts of Madhya Pradesh and Gujarat

The Tapi The Tapi rises in the Satpura ranges in the Betul listrictof Madhya Pradesh It flows in a rift valley parallel tothe Narmada but it is much shorter in length It coversparts of Madhya Pradesh Gujarat and MaharashtraThe length is about 724 km

The Sabarmati and the MahiThe Sabarmati rises in the Aravali hills and flows south-south-westwards for a distance of 300 kilometres to the Arabian Sea The Sabarmatibasin extends over an area of 21674 sq km in Rajasthan and Gujarat The Mahi rises inthe east of Udaipur and drains an area of 34842 sq km lying in Madhya PradeshRajasthan and Gujarat It flows south-westwards for a distance of 533 km before it fallsinto the Gulf of Khambhat

The ChambalThe Chambal rises near Mhow in the Vindhya Range and flows towards the northgenerally in a gorge upto Kota Below Kota it turns to the north-east direction and afterreaching Pinahat it turns to the east and runs nearly parallel to the Yamuna beforejoining it in the southern part of the Etawah district in Uttar PradeshMajor Rivers of India with their basin area (Sqkm)

Himalayan System Indus 321290Ganga 861404

Brahmaputra 187110Indus System

Jhelum 34775Beas 20303

Ganga System Yamuna 366223Ghaghra 127950

Peninsular RiversNarmada 98796

Tapi 65145Mahanadi 141600

Subarnarekha 19300Sabarmati 21674

Mahi 34842Godavari 312812

Godavari Krishna Kaveri Mahanadi are the east-flowing rivers

Narmada Tapi west-flowing rivers

They fall into the Bay of Bengal

They fall into Arabian Sea

These rivers form big deltas

These rivers form comparativelysmall deltas

Catchment areas of these rivers are larger

Catchment areas of these rivers are smaller

Krishna 2589488Cauveri 87900

Subject ndashBiology Topic ndashChapter -5 Inheritance amp Variations Summary ExecutionToday we will discussabout linkage and its classification

LINKAGE The tendency of the genes located on the same chromosome to stay together is

hereditary transmission Linked genes the genes responsible for this Genes that exhibit the process of linkage locates in the same chromosome The distance between the linked genes in a chromosome determines the strength

of linkage i e genes that are located close to each other show stronger linkage than that are located far from each other

COMPLETE LINKAGE It is the type of linkage showed by the genes that are closely located or are tightly

linked with each other as they have no chance of separatingby crossing over These genes are always transmitted together to the same gamete and the same

offspring In such condition only parental or non cross over type of gametes are formedINCOMPLETE KINKAGE It is type of linkage showed by the genes that are distantly located orare loosely

linked with each other because they have chance of separating by crossing over

SIGNIFICANCE i) It helps in holding the parental character togetherii) It checks the appearance of new recombination and helps in bringing the

hybrid population which resembles the original parents iii) Linked genes dilute the effects of undesirable traits

Subject Eng Literature (The Tempest ndash William Shakespeare) Topic Essay Questions (EQ-3)Question No 3

Give a character sketch of CalibanAnswer

The character of Caliban has been wonderfully conceived by Shakespeare as the manifestation of all that is gross and earthy ndash a sort of creature of the earth as Ariel is a sort of creature of the air

Calibanrsquos Physical Appearanceo Caliban is lsquofreckledrsquo a lsquomisshapen knaversquo not honoured with human shape

o Prospero calls him lsquothou tortoisersquo (Act I Sc 2 Line 317) Trinculo stumbling upon him describes him as ldquoA strange fish hellip Legged like a man And his fins like armsrdquo He ldquosmells like a fishrdquo (Act II Sc 2 Line 25)

o Prospero also calls him a ldquobeastrdquo (Act IV Sc 1 Line 140) and ldquoThis misshapen knaverdquo (Act V Sc 1 Line 268)

o Further it appears that in addition to his physical deformity his spiritual inferiority is also suggested by Prosperorsquos claim that his birth resulted from the union between his mother the witch Sycorax and the devil

Calibanrsquos ParentageWhen the play opens Caliban is twenty four years of age having been born on the island twelve years before the coming of Prospero His mother was the foul witch Sycorax who was banished from Algiers for ldquomischiefs manifold and sorceries terrible to enter human hearingrdquo (Act I Sc 2 Line 264) and the father was the Devil himself Thus

Caliban is a monster of evil and brute nature ugly deformed and stinking

Calibanrsquos Savage and Malignant Natureo Caliban is entirely a creature of the earth ndash gross brutal and savage He regards himself as the rightful possessor

of the island and Prospero as a usurper

o In his young age he was on good terms with Prospero He had consented to be received by Prospero at his house and to be educated by him He has learnt human language only to curse his master whom he abhors

o His beastly nature soon breaks out and ends in a vicious attack on Miranda This opens the eye of Prospero who becomes severe to him and enforces his service by threats and violence

o Prospero uses him to make dams for fish to fetch firewood scraper trenches wash dishes and keep his cell clean

Calibanrsquos Hatred for ProsperoA profound hatred for Prospero has taken hold of Caliban It springs from a sense of his being dispossessed and ill-treated He would kill Prospero if he could but he knows the power of Prosperorsquos lsquobookrsquo Hence he transfers his allegiance to Stephano who seems like a god to him He also incites the two drunken associates to batter the skull of Prospero when he sleeps in the afternoon

Caliban Shows Considerable Intelligenceo He has learnt Prosperorsquos language

ldquoYou taught me language and my profit onrsquot (Act II Sc 2 Lines 86-89)Is I know how to curserdquo

o He is well aware of the futility of arguing with one who has more power than he has

ldquoI must obey his art is such power (Act I Sc 2 Lines 373-376)It would control my damrsquos god SetebosAnd make a vassal of himrdquo

o He realizes the importance of Prosperorsquos books

ldquoRemember (Act III Sc 2 Lines 89-92)First to possess his books for without themHersquos but a sot as I am nor hath notOne spirit to commandrdquo

o He knows the value of stealth when attacking the enemy

ldquoPray you tread softly that the blind mole may not (Act IV Sc 1 Lines 194-195)Hear a foot fall we now are near his cellrdquo

o Caliban has a better set of values than Stephano and Trinculo They are distracted from their plan by their greed for Prosperorsquos rich garments Only Caliban realizes that such a finery is unimportant

ldquoLeave it alone thou fool it is but trashrdquo (Act IV Sc 1 Lines 224)

Caliban is not a good judge of characterCaliban is not a good judge of character He decides for example that Stephano is a god because he dispenses lsquocelestial liquorrsquo (Act II Sc 2 Line 115) but then it must be remembered that he has only known his mother Sycorax Prospero Miranda and the spirits that torture him However he quickly discovers his error of judgementrdquo

ldquoWhat a thrice-double ass (Act V Sc 1 Lines 295-297)Was I to take this drunkard for a godAnd worship this dull foolrdquo

Calibanrsquos Imaginative NatureIf Caliban is sub-human in what has been said above he is human in the respect of the poetic side of his character He listens to music with rapture He tells of the beautiful dreams in which heaven rains treasures upon him and which upon waking he yearns to renew One of the most poetic passages in whole play is Calibanrsquos description of the island

to Stephano and Trinculo

ldquoBe not afeard The isle is full of noises (Act III Sc 2 Lines 135-143)Sounds and sweet airs that give delight and hurt notSometimes a thousand twangling instrumentsWill hum about mine ears and sometime voicesThat if I then had waked after long sleepWill make me sleep again and then in dreamingThe clouds methought would open and show richesReady to drop upon me that when I wakedI cried to dream againrdquo

Caliban - Less Ignoble Than Some OthersCalibanrsquos motive for murder is less dishonourable than that of Antonio and Sebastian They plan to kill Alonso to gain his power and wealth Caliban merely wants revenge and the return of lsquohisrsquo island

Conclusiono Calibanrsquos character is not portrayed very clearly in the play and hence we cannot decide whether he is a poor

savage being grossly maltreated by Prospero or whether he is evil and must therefore be kept in bondage or enslavement

o Caliban is contrasted with Ariel who is a spirit and thus swift and uninterested in physical activitieso Caliban is also contrasted with Prospero who is the all-powerful master of the island and of the destiny of all

those on the islando Caliban is also contrasted with civilized man showing him to be less evil than Antonio and Stephano and less

materialistic than Stephano and Trinculoo Caliban has suffered at the hands of Prospero and he has learnt to curse by listening to Prosperorsquos abuse He

certainly believes that Prospero has deprived him of his birthrighto Finally the character Caliban is thought to be one of Shakespearersquos masterpieces The complexity of the character

is reflected in the large volume of critical discussion that has grown around it

ECO ndash12 Topic-Forms of market

MonopolyMonopoly is a market structure in which there is a single seller there are no close substitutes for the commodity produced by the firm and there are barriers to entry Example Indian Railways which is operated under government of India Monopoly also implies absence of competitionFeatures of Monopoly Monopoly is characterized by1 Single Seller In monopoly there is only one firm producing the product The whole industry consists of this single firm Thus under monopoly there is no distinction between firm and industry Being the only firm there is significant control of the firm over supply and price Thus under monopoly buyers do not have the option of buying the commodity from any other seller They have to buy the product from the firm or they can go without the commodity This fact gives immense control to the monopolist over the market

2No Close Substitute There are no close substitutes of the product produced by the monopolist firm If there are close substitutes of the product in the market it implies presence of more than one firm and hence no monopoly In order to ensure a total of control over the market by the monopolist firm it is assumed that there are no close substitutes of the product

3 No Entry amp Exit Monopoly can only exist when there is strong barriers before a new firm to enter the market In fact once a monopoly firm starts producing the product no other firm can produce the same One reason for this is the ability of the

monopolist to produce the product at a lower cost than any new firm who thinks to enter the market If a new firm who knows that it cannot produce at a lower cost than the monopolist then that firm will never enter the market for fear of losing out in competition Similarly the monopolist who is operating for a long time may be enjoying reputation among its customers and is in a better position to use the situation in its own benefit A new firm has to take long time to achieve this and so may not be interested to enter the market

4 Price Maker Being the single seller of the product the monopolist has full control over the pricing of the product On the other hand if there is a large number of buyers in the market so no single buyer exercises any significant influence over price determination Thus it is a sellerrsquos market So monopoly firm is a price maker

5 Price Discrimination Having considerable control over the market on account of being single seller with no entry of other firms the monopolist can exercise policy of price discrimination it means that the monopolist can sell different quantities of the same product to a consumer at different price or same quantity to different consumers at different prices by adjudging the standard of living of the consumer

6 Shape of Demand Curve Since a monopolist has full control over the price therefore he can sell more by lowering the price This makes the demand curve downward sloping

Subject Ac-12 290620 Topic- retirement Model sumThe Balance Sheet of Rohit Nisha and Sunil who are partners in a firm sharing profits according to their capitals as on 31st March 2014 was as under

Liabilities Amount Assets Amount (Rs) (` Rs)

Creditors 25000 Machinery 40000Bills Payable 13000 Building 90000General Reserve 22000 Debtors 30000Capital Less Provision for Rohit 60000 Bad debts 1000

29000 Nisha 40000 Stocks 23000 Sunil 40000 140000 Cash at Bank 18000

200000 200000

On the date of Balance Sheet Nisha retired from the firm and following adjustments were made(i) Building is appreciated by 20(ii) Provision for bad debts is increased to 5 on Debtors(iii) Machinery is depreciated by 10(iv) Goodwill of the firm is valued at Rs 56000 and the retiring partnerrsquos share is adjusted

(v) The capital of the new firm is fixed at Rs120000 Prepare Revaluation Account Capital Accounts of the partner and Balance Sheet of the new firm after Nisharsquos retirement Revaluation AccountDr Cr

Particulars Amount Particulars Amount (`Rs) (Rs`)

Provision for Bad debt Ac 500 Building Ac 18000Machinery Ac 4000Profit transferred toCapital Accounts (3 2 2)Rohit 5786Nisha 3857Sunil 3857

13500

18000 18000

Capital Account

Dr Cr

Particulars Rohit Nisha Sunil Particulars Rohit Nisha Sunil (Rs`) (Rs`) (`Rs) (Rs`) (Rs`) (Rs`)

Sunilrsquos Capital ac 9600 mdash 6400 Balance bd 60000 40000 40000Bank - 66143 - General Reserve 9428 6286 6286Balance cd 72000 mdash 48000 Revaluation (Profi 5786 3857 3857 Rohitrsquos Capital Ac mdash 9600 mdash

Sunilrsquos Capital Ac 6400 Bank 6386 - 4257

81600 66143 54400 81600 66143 54400

Balance Sheet as at 31st March 2014

Liabilities Amount Assets Amount (Rs`) (Rs`)

Creditors 25000 Building 108000Bank overdraft 37500 Machinery 36000

Bills Payable 13000 Debtors 30000Capital Less ProvisionRohit 72000 for Bad debts 1500 28500Sunil 48000 120000 Stock 23000

195500 195500

Working Notes (i) (a) Profit sharing ratio is 60000 40000 40000 ie = 3 2 2(b) Gaining Ratio Rohit = 35 ndash 37 = 2135 ndash 1535 = 635Sunil = 25-27 = 1435 ndash 1035 = 435= 635 435= 6 4 = 3 2(c) Nisha Share of Goodwill = Rs 56000 times 27 = Rs16000Share of Goodwill in the gaining ratio by the existing partner ieRohit = Rs16000 times 35 = Rs 9600Sunil = Rs 16000 times 25 = Rs 6400

The journal entry isRohitrsquos Capital Ac Dr 9600Sunilrsquos Capital Ac Dr 6400 To Nisharsquos Capital Ac 16000(Share of Goodwill divided into gaining ratio)

  • 1 Static Friction
  • The frictional force that acts between the surfaces when they are at rest with respect to each other is called Static Friction
    • Static Friction Examples
      • 2 Sliding Friction
        • Examples Of Sliding Friction
          • 3 Rolling Friction
            • Examples Of Rolling Friction
              • Objects and Reasons of the Forest Conservation Act
Page 19:  · Web viewSubject . Topic . Summary . Execution . English 1 . Sounds of animals . Hens –cackle Horses –neigh Lions –roar Owls –hoots Snake –hiss. English 2 . Mother’s

(liquid) only state changes (physical change)

13 Solid ammonium chloride is heatedSolid ammonium chloride on heating changes into vapors (change of state) is physical change

14 An iron nail is kept in tap water for few dayswe observe reddish brown coating on the nail called rust (entirely new substance) is chemical change

15 A spoon of sugar is heated in a panWhen a spoon of sugar is heated in a pan black (charred sugar) (carbon) is seen Is a chemical change

16 Lighted match stick is brought near the mouth of the test tube containing hydrogen gasWe observe that hydrogen bums at the mouth of test tube with blue flame and pop sound is heard It is chemical change

17 Quick lime is dissolved in waterThe following two observations will be observed (i) A hissing sound is observed(ii) The mixture starts boiling and lime water is obtained

18 Little amount of curd is added to a bowl containing warm milk and kept for five hoursWhen a little amount curd is added to a bowl containing warm milk and kept for five hours a permanent change occurredThe milk will change to curd On boiling water changes into steam (gas) physical change

GEOGRAPHY

ATMOSPHERE IMPACT OF GLOBAL WARMING The destructive impart of global warming is observed in various spheres of life and the environment Some of the points are outlined below1 High temperatures lead to high

evaporation rate and drying up of the soil and surface water This affects crop production The occurrence of droughts is aggravating the problem even further

2 The heat waves in summer months

Q1 Write some impact of global warmingA1 The impacts of global warming are as follows1 High temperatures lead to high

evaporate ion rate and drying up of the soil and surface water This affects crop production The occurrence of droughts is aggravating the problem even further

2 The heat waves in summer months lead to a greater number

lead to a greater number of deaths due to heat strokes

3 Forest fires become more frequent4 Tropical cyclones and hurricanes

become common5 Melting of glaciers takes place6 Polar ice caps are becoming thinner

and melting at an alarming rate due to global warming The loss of sea ice

7 Due to increase in sea surface temperature sea levels rise in coastal areas and cause submergence of several islands

WAYS TO REDUCE GLOBAL WARMINGFollowing steps can be taken We need to decrease emission of

green house gases by reducing the burning of fossil fuel such as coal and petroleum

By planting more trees to increase forest cover

The government should also distributes free saplings and organize afforestation programmes to spread awareness regarding the beneficial effects of trees

We should switch to eco-friendly cars and gadgets

Incandescent light bulbs should be replaced by CFL bulbs

We can save electricity and reduce global warming by turning off electrical gadgets such as lights fans air-conditioners television and computer when we do not to use them

Efforts should be made to hasten the development of green cities oreco cities These cities are urban areas around the world striving to lessen the environment a impacts of urbanization

By following the 3Rs-Reduce Recycle and Reuse strategy we can use natural resources for our growth as well as save them for the need of the future generations This is called sustainable development

of deaths due to heat strokes3 Forest fires become more

frequent4 Tropical cyclones and hurricanes

become common5 Melting of glaciers takes place

etc

Q2 How to reduce global warmingA2 Following steps can be taken to reduce global warmingaWe need to decrease emission of

green house gases by reducing the burning of fossil fuel such as coal and petroleum

bBy planting more trees to increase forest cover

c The government should also distributes free saplings and organize afforestation programmes to spread awareness regarding the beneficial effects of trees

dWe should witch to eco-friendly cars and gadgets

eIncandescent light bulbs should be replaced by CFL bulbs

f We can save electricity and reduce global warming by turning off electrical gadgets such as lights fans air-conditioners television and computer when we do not to use them

Q3 What do you mean by 3Rrsquos of resource planningA3 The 3Rs are

1 Reduce 2 Recycle and3 Reuse

Q4 What is Sustainable developmentA4 By following the 3Rs-Reluce Recycle and Reuse strategy we can use natural resources for our growth as well as save them for the need of the future generations This is called sustainable development

English Language

Prepositions A preposition is a word placed before a noun or a pronoun It helps to show how the person or thing denoted by the noun is related to something else in the sentence

Kinds of Prepositions

Simple Prepositions- simple preposition are one word Prepositions such as at by for in of off for from on out through till to up with before amidst towards beyond between over etc

Compound Prepositions ndash There are some words that are always used with fixed Prepositions to convey specific meaning

Example I was unable to meet you dueto a previous engagement ( On account of)Always maintain the queue instead of crowding at the counter ( In place of)

Participial PrepositionsmdashParticiple Prepositions are present or past participles of various verbs which together with a noun phrase or a clause function as prepositions Examples- barring concerning considering notwithstanding pending regarding respecting etc

Exercise A

1 Gauravs fever has come down since Friday He has been absent for a week now

2 The child sat between his father and mother among the parents of all his classmates

3 There are mosquitoes in the room They flew into the room when the door was open

4 My father was inside the drawing room when I was playing outside my house

5 You may sit beside me I will give you a drawing book and pencils besides a storybook

6 We went to the market in the morning and walked towards the riverfront in the evening

7 The child walked along the pavement and across the street safely

8 This table top is made of glass My breakfast fell off it in the morning

9 The pan is on the gas stove There are vegetables in it

10 We will wait for you at the bus top There are a lot of people in the hall

Subject ndash Biology Topic ndash Chapter - 3 Photosynthesis and respiration in plants Summary Execution

All living organism (Plants and animals) need food for energy and growth Green plants (autotrophy) prepare food for all living organisms Today we will discuss about the process photosynthesis And adaptations in a leaf to carry out photosynthesis

Q1What do you mean by photosynthesis and write its word equation The process by which green plants make food (glucose) from carbon dioxide and water

in the presence of sunlight and chlorophyll is called photosynthesis

Carbon dioxide + Water ( Sun light from Sun ) Glucose + Oxygen ( chlorophyll in green leaves )

Q2 What are the adaptations in a leaf to carry out photosynthesisi) Leaves are broad wide and flat for absorbing more light energyii) Presence of chlorophyll in chloroplasts to trap sunlightiii) Presence of stomata which allow carbon dioxide to enter the cell and oxygen to go

out iv) Network of veins ensures continuous supply of water and minerals to the leafv) Thin waxy cuticle protects the leaf without blocking the lightQ3 Draw and label structure of chloroplast

Class VIIISubject Topic Summary Execution

PHYSICS ENERGY Production of Hydro electricity

A hydroelectric dam converts the potential energy stored in a water reservoir behind a dam to mechanical energymdashmechanical energy is also known as kinetic energy As the water flows down through the dam its kinetic energy is used to turn a turbine

The generator converts the turbinersquos mechanical energy into electricity

This electric energy then goes through various transmission processes before it reaches you

Question 2

Fill in the blanks

(a) Work is said to be done by a forte only when the body moves

(b) Work done = Force x distance moved in direction of force

(c) The energy of a body is its capacity to do work

(d) The SI unit of energy is joule

(e) The potential energy is due to its state rest of position and kinetic energy of the body is due to its state of motion

(f) Gravitational potential energy U = mass times force of gravity on unit mass times height

(g) Kinetic energy = frac12 times mass times (speed)2

(h) Power P = work donetime taken

(i) The S I unit of power is watt

(j) IHP = 746 W

BIOLOGY Chapter -5 The endocrine system and adolescence

Today we will discuss about thelocation and functions of secreted hormones of adrenal and Pancreas

Q5 Write location hormone secreted main functions and deficiency diseases of pancreas and adrenal glands

Endocrine Glands

Location Hormones secreted

Functions and Deficiency Diseases

1Adrenal gland

2 Pancreas Gland

On the top of each kidney

In between stomach and small intestine

i)Adrenaline from adrenal medulla

ii)Cortisone from adrenal cortex

i) Insulin

ii) Glucagon

It helps a person deal with any kind of emergency situation or emotional stressIt increases the heart beat rate of respiration and blood pressure

a) It regulates carbohydrates protein and fat metabolism

b) It regulates the salt and water balance in the body

a) It changes excess glucose into glycogen

b) It stimulates the cells to burn extra glucose to provide heat amp energy

Less secretion causes diabetes mellitus

Excessive secretions causeinsulin shock

a) It stimulates the breakdown of glycogen into glucose

b) It increases the level of glucose in blood

History Traders to rulers The Battle of Buxar was fought on 22 October 1764 between the forces under the command of the British East India Company led by Hector Munro and the combined armies of Mir Qasim the Nawab of Bengal till 1763 Mir Jafar was made the Nawab of Bengal for a second time in 1763 by the Company just after the battle After being defeated in 4 battles in katwa and Udaynala the Nawab of Awadh Siraj id Daula and the Mughal emperor Shah Alam II accompanied by Raja Balwant Singh of Kashi made an alliance with Mir Qasim The battle was fought at Buxar a small fortified

Answer the following questions- Short note-Battle of BuxarHomework-learn

town within the territory of Bihar located on the banks of the Ganga river about 130 kilometres (81 mi) west of Patna it was a decisive victory for the British East India Company The war was brought to an end by the Treaty of Allahabad in 1765

EnglishLiterature

The west wind-John Mansfield

In the poem The West Wind by John Masefield the poet starts by describingwith very poetic imagery of birds how the west wind is different from other winds its a warm wind full of birds cries There is a touch of melancholy perhaps home-sickness as he describes how it brings tears too and memories from an old land He goes on to describe the restful pastoral beauty of the land where even the dead can lie in the green He then brings in voicesperhaps of family and friends calling him home as he is missing Aprils beautyThe voices then tempt him some more with idyllic images from home (white blossom young green cornrunning rabbitswarm sun) The voices seem to presume that the poets heart is sorrowful bruised and soreThe end of the poem sees the poet appear to make a decision he will go home as he has decided that is where he truly belongs

Write the synopsis of the following words

1 Daffodils- a tall yellow flower that grows in the spring

2 Orchards- a piece of land on which fruit trees are grown

3 Blossom- a flower or a mass of flowers especially on a fruit tree in spring

4 Thrushes- a bird5 Larks- a small brown bird that

makes a pleasant sound6 Bruised- an injury7 Aching- pain 8 Tread- to put your foot down

while you are walking9 Balm-10 May-11 Fluting-

(Write from the book in your copy)

MAT

HEM

ATIC

S

Ch 1

1Al

gebr

ic E

xpre

ssio

n

1 Constant A symbol which has fixed value is called a constant[eg 8 23 -15 radic3 etc]

2 VariableA symbol which does not have any fixed value but may be assigned value (values) according to the requirement is called variable or literal[eg x y p q etc]

3 TermsA term is a number (constant) a variable a combination (product or quotient) of numbers and variables[eg 7 x 5x etc]

4 Algebric expressionA single term or acombination of two or more terms connected by plus (+) or minus (-) sign forms an algebraic expression[eg 5-y 3x2-5x xy-6z+4 etc]

5 PolynomialAn algebraic expression which contains more than one term is called a polynomial (multinomial)[eg x2-5x 5y+xy+x2y etc]

6 Degree of polynomial(a) When the polynomial contains only one variable the highest power of the variable is the degree of the polynomialeg the degree of the polynomial of 4x-7x5+8 is 5(b) When the polynomial contains two or more variablesStep (i) Find the powers of the variables in each term (ii) The highest sum of the powers is taken to be the degree of the polynomialeg the degree of the polynomial 5x2y-4x3y5+6 is = 3+5 = 8Remember An algebraic expression is a polynomial if degree of each term used in it is a non-negative integer

Exercise ndash 11(A)

1 Separate the constants and variables from the following

-7 7+x 7x+yz radic5 radic xy 3 yz

8 45y -3x

Solution Constant Variables-7 radic5 7+x 7x+yz radic xy

3 yz8

45y -3x

2 Write the number of terms in each of the following polynomials(i) 5x2+3timesax (ii) axdivide4-7 (iii) ax-by+ytimesz (iv) 23+atimesbdivide2

Solution Polynomials Number of terms(i) 5x2+3timesax 2(ii) axdivide4-7 2(iii) ax-by+ytimesz 3(iv) 23+atimesbdivide2 2

4 Write the degree of the each polynomials(i) xy+7z (ii) x2-6x3+8 (iii) y-6y2+5y8 (iv) xyz-3 (vi) x5y7-8x3y8+10x4y4z4

Solution Polynomials Degree(i) xy+7z 2(ii) x2-6x3+8 3(iii) y-6y2+5y8 8(iv) xyz-3 3(vi)x5y7-8x3y8+10x4y4z4 12

5Write the coefficient of(i) ab in 7abx (iv) 8 in a2-8ax+a (v) 4xy in x2-4xy+y2

SolutionCoefficient

(i) ab in 7abx 7x(iv) 8 in a2-8ax+a -ax(v) 4xy in x2-4xy+y2 -1

7 CoefficientAny factor of an algebraic quantity is called the coefficient of the remaining quantityeg in the algebraic term 7xyz 7 is coefficient of xyz 7x is coefficient of yz and so on

8 Like term The terms having the same literal coefficient are called like terms and those having different literal coefficients are called unlike terms

eg (i) 5xyz 8xyz -6xyz and 23xyz are like

terms(ii) 7xy2 8x2yz and -15xyz2 are unlike terms

6 in 57xy2z3 write the coefficient of

(i) 5 (vii) 5xy2 (viii) 17yz (xi) 5xyz

Solution Coefficient

(i) 5 17

xy2z3

(vii) 5xy2 17z3

(viii) 17yz

5xyzsup2

(xi) 5xyz 17yz2

7 In polynomial given below separate the like terms(ii) y2z3 xy2z3 -58x2yz -4y2z3 -8xz3y2 3x2yz and 2z3y2

Solution y2z3 -4y2z3 2z3y2 are like terms

xy2z3 -8xz3y2 are like terms

-58x2yz 3x2yz are like terms

Class IXSubject Topic Summary Execution

Bengali (2nd language)

বাগzwnjধারাzwnj বা ধারা-বা ধারা ল হিবকেশষ পরকার বাক -হিb -াকেবর এক হিবকেশষ পরকাশরীহিত াকেক কতগকেলা কার সমষটির মকেধয এগহিলকেক বা ধারা বকেল আবার কতগকেলা শকেবদর বাধাধরা যকান রীহিত যনই য-াকেব চকেল আসকে যসই -াকেবই চকেল আসকে তখন যসই শবদগহিল খন একক -াকেব অG পরকাশ ককের তখন একের বা ধারা বকেল বা ধারার পরকেয়া -াষাকেক আরও সFর ককের যতাকেল

অকাল পকক(অপহিরনত বয়কেস পাকাহিম)-মাতর শ বর বয়কেস যমকেয়টির া মকেখর কা তাকেত অকালপককতা ধরা পকে

অককা পাওয়া( মারা াওয়া) ndash পকেকIমারটি পকেকIমারকেত হিকেয় বাসাতরীকের াকেত মার যখকেত যখকেত অককা যপল

অহি| পরীকষা ( কঠিন ও পরকত পরীকষা)- যকেলটির আজ ডাকতাহির যরজালট যবকেরাকেব এIাই তার জীবকেনর ব অহি| পরীকষা

অষটরমভা (ফাহিক) ndash রীতা মকেখই বকো বকো কা বকেল আর কাকেজর যবলায় অষটরমভা

অকমGার ধাী (অপাG) ndash সমনকেক হিনকেয় যকান ান কেব না ও একেকবাকেরই অকমGার ধাী

অকেনধর ষটি (অসাকেয়র সায়)- আহিশ বকেরর বকোর নাহিত ল অকেনধর ষটি তাকেক াা বকোর একম চকেল না

আকেককল গড়ম (তবহিদধ)- ার তহিম উপকার করকেল যসই যতামার হিবরকেদধ সাকষয হিকেয়কে শকেনই আমার আকেককল গড়ম

আষাকে লপ( অবাসতব লপ) ndashIাকা এখন যকেব না এIা বলকেলই ত এমন আষাকে লপ ফাার যকান রকার হিল না

Hindi- महायजञ ा इस हानी म लख न या बतान ा परयास किया ह कि किसी भी अचछ

2nd language

परसार(यशपाल ाय या पणय न ा फल अवशय मिमलता ह ोई भी परोपार अथवा पणय लिलए किया गया ाय बार नही 0ाता वह ए परार ा यजञ हए धनी सठ थ धम परायण और किवनमर सठ न आन ी यजञ किए थ और दान म न 0ान कितना धन दिदन दखिखयो म बात दिदया थादिदन पलट और सठ यहा गरीबी आ गई उन दिदनो यजञ बचन ी परथा थी सठ भी अपनी 0गह बचन लिलए डलपर ए सट यहा चलन ो तयार हए सठानी रासत लिलए रोटी पड म बाधर सठ ो द दी रासत म ए भख R ो दखर सठ न चारो रोटी उसो खिखला दी खर वह सठ यहा डलपर पहच तो उनी सठानी न उस महायजञ बचन ो हा यदिद बचन आए सठ न R ो रोटी खिखलान ो महायजञ नही समझा और वापस लौट आया घर आर शाम ो उसी घर म उस ए बडा ख0ाना मिमला 0ो उस दवारा किए गएrsquo महायजञrsquo ा परसार था

English language

Letter formal The heading the name and address of the person you are writing to must be included beneath your own address In formal letters ldquoblock stylerdquo of address is preferred

Subject complain in brief

Salutation If the person you are writing to is known to you you may begin ldquoDear MrrdquoOr ldquoDear Mrsrdquo In all other instances you should begin ldquoDear Sirrdquo or ldquoDear Madamrdquo Or ldquoSirsrdquo

The body A formal or business letter has four partsReference The letter should begin by referring to a letter you have received an advertisement or the reason that has prompted you to writeInformation In the second paragraph it is necessary to supply more detailed information that is related to the referencePurpose Here you must give the reason why you are writing the letter This must be stated clearly and ensure that it is relevant to the question that has been setConclusion round off the letter with some polite remarkThe subscription when a letter has begun with dear sir sirs Madam you should end with Yours faithfully or yours truly When however you address a person by name you must conclude with the words ldquoYours sincerelyrdquo

1 A park in your locality is slowly being used as a rubbish dump Write a letter to the Mayor of your city pointing out the nuisance and danger of this Request that action be taken to stop this immediately

Or2 You being a boarder ordered a set of lab manuals from a famous book shop in the town They sent you a wrong set of books Write a letter to the manager of the book shop

Chemistry Chapter-1 1)CHEMICAL FORMULA- Q What is the Significance of

L-2The Language of Chemistrybull Chemical Formula

Itrsquos a symbolic representation of a chemical substance eg ndash The formula of Sulphuric acid is H2SO4

2) Steps of writing Chemical Formula of a given substance-

1 Write the symbols of the constituent atoms or radicals side by side Keep the basic radical on LHS and acid radical on the RHS ( Na+Cl- )2 In case of a radical having more than one atom( compound radical) enclose the radical in a bracket eg (SO4-)3 Write the valencies of each radical on its right hand top4 If the valencies of the two radicals are divisible by a common factor then divide the valencies by the common factor5 Invert (criss-cross) the valency number ie write the valency of one atom below the second atom and vice versa 6 On interchanging if valency number is lsquoone the figure lsquoonersquo is never writtenFor Example- Compound -Calcium Nitrate1 Writing the symbols- Ca(NO3)2 Writing the valencies on their right hand top- Ca2(NO3)1

3 Valency numeral in simple ratio- Ca2(NO3)1

4 Criss-cross- Ca 2NO3 1

5 Writing the formula of the compound- Ca(NO3)2

Chemical formula

A The formula of a substance conveys the following information regarding a substance 1 The name of the substance (qualitative)2 The elements constituting the substance (qualitative)3 The number of various atoms present in a molecule of the substance (quantitative)4 Molecular weight of the substance and the relative weights of different elements present in it (qualitative)

Q What are the limitations of Chemical Formula

A The chemical formula suffers from the following limitations-I It fails to convey whether the elements in a molecule are present in the form of atoms or ionsFor example the formula KBr fails to tell us whether Potassium and Bromine are present in the form of ions II It does not tell anything about the binding force that holds atom in a molecule togetherIII It does not tell us about the arrangement of various atoms with respect to one another within the molecule

Q Examples of Some Chemicals with their Formula Chemical name and Common Name-

A Given in the class notesCommercial Studies

Joint Stock Company

Let us discuss about the demerits of Joint Stock CompanyDespite so many advantages it has got many disadvantages which are as follows

Difficulty in FormationDelay in Decision makingExcessive Government ControlLack of Secrecy

Company can be classified into several categories based on incorporation

QuestionExplain the demerits of Joint Stock CompanyAnswer) 1 Difficulty in Formation The legal requirements and formalities required to be completed are so many The cost involved is quite heavy It has to approach large number of people for its capital It cannot start its business unless certificate of incorporation has been obtained This is granted after a long time when all the formalities are completed

Chartered CompanyStatutory CompanyRegistered Company

Delay in Decision making In this form of organization decisions are not made by single individual All important decisions are taken by the Board of Directors Decision-making process is time-consuming So many opportunities may be costly because of delay in decision-making Promptness of decisions which is a common feature of sole trader ship and partnership is not found in a company

Excessive Government ControlA company and the management have to function well within the law and the provisions of Companies Act are quite elaborate and complex At every step it is necessary to comply with its provisions lest the company and the management should be penalized The penalties are quite heavy and in several cases officers in default can be punished with imprisonment This hampers the proper functioning of the company

Lack of Secrecy The management of companies remains in the hands of many persons Every important thing is discussed in the meetings of Board of Directors Hence secrets of the business cannot be maintained In case of sole proprietorship and partnership forms of organisation such secrecy is possible because a few persons are involved in the management

2 Define the following

Chartered Company- The crown in exercise of the royal prerogative has power to create a corporation by the grant of a charter to persons assenting to be incorporated Such companies or corporations are known as chartered companies Examples of this type of companies are Bank of England (1694) East India Company (1600) The powers and the nature of business of a chartered company are defined by the charter which incorporates it After the country attained independence these types of companies do not exist

in IndiaStatutory Company- A company may be incorporated by means of a special Act of the Parliament or any state legislature Such companies are called statutory companies Instances of statutory companies in India are Reserve Bank of India the Life Insurance Corporation of India the Food Corporation of India etc The provisions of the Companies Act 1956 apply to statutory companies except where the said provisions are inconsistent with the provisions of the Act creating them Statutory companies are mostly invested with compulsory powersRegistered companiesCompanies registered under the Companies Act 1956 or earlier Companies Acts are called registered companies Such companies come into existence when they are registered under the Companies Act and a certificate of incorporation is granted to them by the Registrar

Economics

Chapter-4Basic problems of Economy

Today let us discuss with the topic Production Possibility curve

QuestionExplain the concept of Production Possibility Curve with the help of diagram

Answer) Production Possibility curve is a locus of all possible combinations of two commodities which can be produced in a country with its given resources and technology

The above diagram shows that with the given resources and technology the economy can produce maximum either 5 thousand meters of cloth or 15 thousand quintals of wheat or any other combination of the two goods like B( 1 thousand meters of cloth and 14 thousand quintals of wheat C ( 2 thousands meters of cloth and 12 thousand quintals of wheat) etcProduction Possibility curve is also called production possibility boundary or frontier as it sets the maximum limit of what it is possible to produce with given resources

Geography

Rotationand Revolution

SUNrsquoS POSITION AND SEASONAL CHANGES EQUINOXES ndash SPRING AND AUTUMN

Q1 What is Spring EquinoxA1 On 21st March sunrays fall directly on the equator On that day

As the Equator divides the Earth into two equal halves the sun rays fall directly on the equator twice in a year Equinoxes means equal Spring EquinoxOn 21st March sunrays fall directly on the equator On that day the duration of day and night both are equal ( 12 hours day and 12 hours night) on every places located on equator This day is called as Spring EquinoxAutumn EquinoxOn 23rd September sunrays fall directly on the equator On that day the duration of day and night both are equal ( 12 hours day and 12 hours night) on every places located on equator This day is called as Autumn Equinox

SOLSTICES ndash SUMMER AND WINTERDue to inclination of the Earth on its axis and the apparent movement of the sun the sun rays fall directly on both tropics once in a year Solstice is a Latin word which mean ldquothe Sun standing stillrdquoSummer SolsticesAfter 21st March there is an apparent movement of the Sun to the north of the equator The apparent northward movement up to 21st June when the Sun appears overhead at the Tropic of Cancer (22frac12degN) The sun appears to stand still at this position and then moves southwards towards the equator This position of the Sun on 21st June is known as Summer Solstices On that day the duration of day and night both are equal ( 12 hours day and 12 hours night) on every places located on Tropic of Cancer (22frac12degN)Winter solstices The apparent southward movement of the Sun continues beyond the equator till 22nd

December On this day the Sun is overhead at the Tropic of Capricorn

the duration of day and night both are equal ( 12 hours day and 12 hours night) on every places located on equator This day is called as Spring Equinox

Q2 What do you mean by EquinoxA2 Equinoxes means equal It is use to explain the equal duration of day and night ( 12 hours day and 12 hours night) on the Earth

Q3 On which date the longest day in Tropic of CancerA3 21st June

Q4 What is the meaning of SolsticeA4 Solstice is a Latin word which mean ldquothe Sun standing stillrdquo

Q5 Which is the longest day in southern hemisphereA5 22nd December

Q6 On what date does the Arctic Circle experience the lsquoMidnight SunrsquoA6 On 21 June the Arctic Circle experiences the lsquoMidnight Sunrsquo

Q7 What is cause of Midnight Sun in NorwayA7 During the summer solstice (21 June) the North Pole is inclined towards the Sun Therefore the duration of sunlight or daytime increases from 12 hours at the Equator to 24 hours at the Arctic Circle and beyond Thatrsquos why The region beyond the Arctic Circle especially Norway is known as the Land of the Midnight Sun because there the Sun does not rise or set on 21 June

Q8 Match the column A with BA B

Summer Solstice 21st March

Autumn Equinox 23rd

September

Winter Solstice 21st June

(22frac12degS) This position of the Sun is referred to as the Winter Solstice because it marks the winter season in the Northern Hemisphere On that day the duration of day and night both are equal ( 12 hours day and 12 hours night) on every places located on Tropic of Capricorn (22frac12degS)SEASONS AND DURATION OF DAY AND NIGHT During the equinoxes all places on the Earth have 12 hours of day and 12 hours of night Due to the revolution of the Earth round the Sun on an inclined axis the duration of day and night varies according to seasons and the latitude of a placeDuring the summer solstice (21 June) the North Pole is inclined towards the Sun Therefore the duration of sunlight or daytime increases from 12 hours at the Equator to 24 hours at the Arctic Circle and beyondThe region beyond the Arctic Circle especially Norway is known as the Land of the Midnight Sun because there the Sun does not rise or set on 21 JuneAt the North Pole there will be six months of daylight The Sun will be seen always above the horizon at a low angle At 66degN 24 hours of sunlight can be seen only on 21 June Hammerfest in northern Norway is a place of tourist attraction for observing the phenomenon of the Midnight Sun This place has continuous daylight from 13 May to 29 July This place is easily accessible to tourists and has hotels and other facilities The view of the midnight Sun from here is enthrallingIn the Southern Hemisphere the duration of daylight decreases from 12 hours at the equator to 0 hours beyond the Antarctic Circle In the South Polar Region there is 24 hours of darkness The Sun is always below the horizon In the Southern Hemisphere which experiences winter the duration of night-time is longer than the duration of daylight

Spring Equinox 22nd

December

A8 A B

Summer Solstice 21st June

Autumn Equinox 23rd

September

Winter Solstice 22nd

December

Spring Equinox 21st March

During winter solstice (22 December) the South Pole is inclined towards the Sun The Southern Hemisphere experiences summer and the Northern Hemisphere has winter Therefore the duration of daylight or sunlight is greater in the Southern Hemisphere than in the Northern HemisphereThe duration of daylight increases from 12 hours at the equator to 24 hours beyond the Antarctic Circle The South Polar Region has 24 hours of sunlight for many days continuously At the South Pole there will be six months of sunlight The Sun will always be seen at a low angle above the horizon In the Northern Hemisphere the duration of daylight will decrease from 12 hours at the equator to 0 hours at the Arctic Circle There are 24 hours of darkness in the North Polar region The duration of night is greater than the duration of daylight as one move northwards from the Equator It is evident from the above table that the duration of daylight is 12 hours throughout the year at the equator only As one moves away from the equator the seasonal variations in the duration of daylight increase The seasonal variations in the duration of daylight are maximum at the Polar Regions

Subject Eng Literature (The Merchant of Venice ndash William Shakespeare)Topic Act II Scene 7 Lines 36 to 80 (End of scene ) [Students should read the original play and also the paraphrase provided]

Summary Questions amp AnswersThe Prince then examines the inscription on the silver casket which says ldquoWho chooseth me shall get as much as he deservesrdquo The Prince says that he deserves Portia more than anybody else because of his high rank his noble birth and his great wealth and power But then he argues that silver is ten times

(1) (Act II Sc 7 L 39-47)

From the four corners of the earth they come

To kiss this shrine this mortal breathing saint

The Hyrcanian deserts and the vasty wildsOf wide Arabia are as through-fares now

inferior to gold and therefore he cannot believe that the portrait of such a beautiful lady as Portia can be contained in the silver casket He decides to see the inscription on the golden casket before making his decision

The Prince goes to examine the inscription on the golden casket which says ldquoWho chooseth me shall get what many men desirerdquo The Prince believes that the whole world desires to possess Portia otherwise so many suitors would not have come from all corners of the world for winning Portia Some of them have come from the distant lands of Persia and Arabia The deserts of Persia (Hyrcanian deserts) and the boundless desolate lands of Arabia have been crossed by the Princes seeking the hand of Portia He contrasts this casket containing Portiarsquos portrait with the old English gold coin bearing the image of the archangel (angel of the highest rank) He goes on to remark that while the figure of the archangel is engraved (Insculped) upon the English coin the picture of Portia who is beautiful as an angel lies hidden inside one of the caskets namely the Golden Casket (Golden Bed)

On the basis of his assessment of the inscription on the golden casket the Prince decides to choose the golden casket He asks for the key and opens the golden casket only to find therein an empty human skull holding a roll of

For princes to come view fair PortiaThe watery kingdom whose ambitious headSpets in the face of heaven is no barTo stop the foreign spirits but they comeAs orsquoer a brook to see fair Portia

(i) Explain the occasion for the above mentioned speech

These are the comments of the Prince of Morocco after he reads the inscription on the golden casket His mental process is revealed to us in these words We find him debating within himself as to which casket he should choose

(ii) What light does the above speech throw on the personality of Prince of Morocco

From the above mentioned speech we come to know that the Prince of Morocco is keen to marry Portia He is the type of person who is easily taken away by outward appearance He is in love with Portia because of her beauty

(iii) What information can you gather about Portia from the above mentioned lines

The given speech shows that Portia is a very beautiful lady She must be possessed of good qualities because many suitors come to her place from all over the world with a desire to get married to her The Prince of Morocco is so impressed by her beauty that he calls her a saint According to him the whole world is desirous of having her

(iv) Elucidate the significance of the first two lines

In these lines the Prince of Morocco pays a compliment to Portia These lines show his admiration for her He says that people come from all parts of the world to see fair Portia

(v) Explain the meaning of the last four lines of the

passage

In these lines the Prince of Morocco says that even the vast oceans which throw a challenge at the sky are unable to prevent men from coming to Portiarsquos place to have a glimpse of her These lines are also a tribute to Portiarsquos beauty and good qualities Many men voyage across the ocean treating it as a mere stream to see the beautiful Portia

paper in which is written that whoever happens to be guided by the glitter of things is invariably deceived

On reading the scroll the Prince says that he is too sad at heart to speak a more formal farewell and leaves with his followers amidst a sound of trumpets

After the Prince of Morocco leaves Portia remarks that the Prince is a gentle fellow but she is rid of him May all persons of his nature make a similar choice

IMPORTANT PASSAGES EXPLAINED

(Act II Sc 7 L 39-43)From the four corners of the earth they come

To kiss this shrine this mortal breathing saintThe Hyrcanian deserts and the vasty wildsOf wide Arabia are as through-fares nowFor princes to come view fair Portia

Context

This passage occurs in Act II Scene 7 in The Merchant of Venice This is part of the speech made by the Prince of Morocco

(2)

(Act II Sc 7 L 48-53)

MOROCCO One of these three contains her heavenly pictureIst like that lead contains her

Twere damnation To think so base a thought it were too grossTo rib her cerecloth in the obscure graveOr shall I think in silver shes immurdBeing ten times undervalued to tried gold

(i) What meaning does the Prince of Morocco find out of the inscription of the golden casket What have Belmont and Portiarsquos house been called and why

The inscription on the golden casket is ldquoWho chooseth me shall gain what many men desirerdquo The Prince finds out that it means that the chooser of the golden casket will get Portia because many men desire her In fact the entire world desires her Because of the coming of many suitors to Belmont from different countries in order to win Portiarsquos hand Belmont has become a centre of pilgrimage and her house is the shrine where saintly Portia is installed

(ii) What does the Prince of Morocco do before making the final choice of the casket Which is the correct casket and who will win Portiarsquos hand

The Prince of Morocco surveys and analyses the inscriptions on the casket of lead silver and gold Before making the final choice like a very systematic and methodical person he once again considers the claims of the caskets The casket containing Portiarsquos picture is the correct casket and the person choosing it will win Portiarsquos hand

Explanation

While praising Portia the Prince of Morocco conceives Portia as a goddess whose image is placed inside one of the caskets Many suitors are coming from far and wide the north and the south the east and the west (Four corners) in order to try their luck Some of them have come from the distant land of Persia and Arabia The deserts of Persia (Hyrcanian deserts) and the boundless desolate lands of Arabia have been crossed by the Princes seeking the hand of Portia All this shows that Portia is indeed the most beautiful lady of the world

(iii) What does the Prince of Morocco say in his estimation while examining the motto on the silver casket What does he find in the golden casket

While examining the motto on the silver casket which says ldquoWho chooseth me shall get as much as he deservesrdquo Morocco says that in his own estimation he surely deserves Portia in all respects ndash rank birth wealth etc

He chooses the golden casket When he opens it he finds an empty human skull holding a scroll in which it is written that those who are attracted by the glittering outside of things are always deceived as Morocco has been deceived

(iv) What kind of nature does the Prince of Morocco have

The Prince of Morocco has a simple nature who does not look deeply into the inner meaning of things but is dazzled by the outward appearance of gold He is inclined to over-estimate his own value and does not realize that it is a duty to ldquogive and hazardrdquo To say that he will not hazard for lead shows that he misreads the true meaning of the inscription which is that he should be prepared to ldquohazard all he hathrdquo for Portia So his feeling is only one of fascination and romantic attraction

(v) Do you think that the lottery of the caskets is not a matter that will be determined by chance

In fact the lottery of the casket is not a matter that will be determined by mere chance but that it is a true test of character and of sincerity which is amply proved not only by Moroccorsquos choice but also by the arguments which he uses to help him in his choice

(Act II Sc 7 L 55-59)

They have in England

A coin that bears the figure of an angelStamped in gold but thats insculpd uponBut here an angel in a golden bedLies all within

Context

(3)

(Act II Sc 7 L 63-77)A carrion Death within whose empty eye

There is a written scroll Ill read the writing

All that glisters is not goldOften have you heard that toldMany a man his life hath soldBut my outside to beholdGilded tombs do worms infoldHad you been as wise as boldYoung in limbs in judgment oldYour answer had not been inscrolld

This passage occurs in Act II Scene 7 in The Merchant of Venice This is part of the speech made by the Prince of Morocco

Explanation

In this passage the Prince of Morocco bestows high praise on Portia whose hand he is seeking He contrasts this casket containing Portiarsquos portrait with the old English gold coin bearing the image of the archangel (angel of the highest rank) He goes on to remark that while the figure of the archangel is engraved (Insculped) upon the English coin the picture of Portia who is beautiful as an angel lies hidden inside one of the caskets namely the Golden Casket (Golden Bed) In the day of Elizabeth silver was ten times inferior in value to gold Therefore the Prince of Morocco believing that Portiarsquos portrait is contained in the Golden Casket decides to choose the Golden Casket

Fare you well your suit is coldCold indeed and labour lostThen farewell heat and welcome frostmdashPortia adieu I have too grievd a heartTo take a tedious leave Thus losers part

(i) What reward does the Prince of Morocco get after making a wrong choice of the Casket How does he feel

After making the wrong choice in selecting the casket of gold the Prince of Morocco as a reward earns a rebuke in the form of a scroll tucked in the empty eye-socket of a skull kept in the casket of gold The Prince is shocked and disappointed He becomes all the more sad and dejected when he reads the scroll which points to his foolishness in being misled by the appearance and outward show as indicative of its worth

(ii) How does the Prince respond after reading the scroll

After reading the scroll the Prince though upset accepts the result with good grace and decorum befitting a royal suitor and true sportsman He says that his love-suit is really cold otherwise he would have chosen correctly but now his efforts have been in vain So he bids farewell to Portia to the warmth and enthusiasm of love and welcomes the cold and bitterness of dejection and misery of life which lies ahead

(iii) What request does he make to Portia and why

After being failure in his mission he requests Portia to give him permission to leave at once because he is too sad to undergo the tediousness of a formal leave-taking He tells that it is the manner in which defeated persons part unceremoniously

(iv) Explain the following lines

ldquoAll that glisters is not goldOften have you heard that toldMany a man his life hath soldBut my outside to beholdGilded tombs do worms infoldrdquo

Mere glitter does not make a metal to be gold Man has often been warned against appearance but it has been of no use Many people have sacrificed their lives only to seek the outer appearance of gold Worms are found inside the gilded

monuments

Class XSubject Topic Summary Execution

Hindi 2ndlang

नया रासता भाग 6 मायाराम 0ी घर म धनी मल 0ी और उनी बटी सरिरता ी ही चचा बनी रहती थी अमिमत ो इसम ोई रलिच ना थी वह धनी घर ी लडी स शादी र सवय ो बचना नही चाहता था उसा भी सवाणिभमान ह ईशवर ी पा

स उस पास पस ी ोई मी नही थी अभी उसन फकटरी ही लगाई थी उसी समझ बाहर था कि उस घर वालो ा झाव पस ी तरफ कयो

ह उसन मा स सवाल किया कि मा तम सरिरता स मरी शादी कयो रना चाहती हो मा न उस समझाया कि वह दखन म बरी नही ह और किफर खानदान अचछा

ह वह ए शल गरहणी रप म घर सभाल सगी अमिमत न मा ो इस बात ा एहसास राया कि मीन सबध लिलए मना रन पर उस दिदल

पर कया बीती होगी मा और अमिमत ी लडी बार म ाफी बात हईमा ा झाव सरिरता ी तरफ था कयोकि वह घर पर अचछा दह0 लर आ रही

थी अमिमत न अपनी मौसी ी बरी हालत बार म बताया कि किस तरह वह बड घर ी खानदानी बटी लाई थी और आ0 उसी हालत कितनी खराब ह लाई थी बहकलब 0ाती ह और बचचो ो भी नही दखती ह बात चल ही रही

थी कि तभी ए ार बाहर आर री धनी मल0ी घर अदर आए और पीछ स डराइवर फल ी ए टोरी लर आया अदर आए और पीछ स

डराइवर ए टोरी फल ी लर आया अमिमत ो फल ी पटी बरी लग रही थी अमिमत न पछ लिलया यह फल कयो ल आए ह प इन सब ी कया

0ररत थी उनो न 0वाब दिदया कि 4 पटी शमीर स मगाए थ अमिमत ो या सनर करोध आ गया तभी उस किपता 0ी आ गए उन आत ही अमिमत उठर बाहर चला गया वहा वहा मा पास आर बठ गया और बोला

अभी रिरशता तय नही हआ और धनी मल 0ी धनी मल 0ी फल ी पटी लर चलआय मा न समझाया कि 0ब सबध 0ड 0ाता ह तो खाली हाथ नही

आत अमिमत न मा स हा कि तम सबन सरिरता ो इस घर म लान ी ठान रखी ह धनीमल 0ी उस दिदन सरिरता ो दखन ी तारीख तय रन आय थ

Commercial Studies

Banking Nowadays Bank provide easy and quick services through internet facilities methods of Banking is called internet bankingIn order to save the time and money involved in visiting Bank branches people increasingly prefer to have internet banking

There are different modes of doing internet banking or transferring money through online They areReal Time Gross Settlement (RTGS)National Electronic Fund Transfers (NEFT)

1

Question

1) Explain the term RTGS Write the features of RTGS

Answer)The acronym RTGS stands for Real Time Gross Settlement which may be defined as the continuous real time settlement of funds transfer individually on and order by order basis without netting lsquoReal timersquo may be defined as the processing of instructions at the time they are received rather than at some letter time lsquoGross settlementrsquo may be defined as the settlement of transfer instructions which occurs

individually

Features of RTGS1It is the continuous settlement of

funds transfer individually on an order by order basis

2RTGS facility is provided only by CBS core banking solution enabled Bank branches

3Amount charged from the customer for RTGS transactions vary from bank to bank

2) Explain the term NEFT Write the features of NEFT

Answer) National electronic funds transfer may be defined as a nationwide system that facilitates individuals Farms and copper operates to electronically transfer funds from any bank branch to any individual farm or corporate having an account with any other bank branch in the country

Features of NEFT2 Transfer can be made 7 times on

weekdays and 6 times on Saturday

3 NEFT cannot be used to receive foreign remittances

4 NEFT transaction takes place in batches

5 A bank branch must be NEFT enabled to become a part of NEFT fund transfer network

6 There is no maximum or minimum amount that can be transferred through NEFT when one bank has a bank account

English Language

CompositionEssay

A composition is an art of creating a piece of writing on any topic or subject It is the writing correctly beautifully and clearly in order to make some interesting reading Structure of the composition

Introduction ( you lay the foundation for your composition)

Body (it constitutes the main part of the essay)

Conclusion (final statement that leaves a lasting impression)

Kinds of essays1 The Narrative essay2 The descriptive essay3 The reflective essay4 The argumentative essay

Write a composition on any one of the following topics (350- 400 words)

1 Friendship Or2 The first day of your school

Subject Eng Literature (The Merchant of Venice ndash William Shakespeare)Topic Act V Scene 1 Lines 127 to 158 (Nerissa helliphellip The clerk will nersquoer wear hair onrsquos face that had it) [Students should read the original play and also the paraphrase given in the school prescribed textbook]

Summary Revision Questions o Soon thereafter Bassanio Gratiano

and Antonio arrive

o Bassanio tells Portia that he is feeling as if it is morning because of the presence of Portia who is shining like the sun When Antonio is introduced by Bassanio to Portia she tells Bassanio that he should be grateful to Antonio who took so much trouble on his account even to the extent of risking his life

o Nerissa starts quarrelling with Gratiano and demands that he show her the ring she had presented to him and which she had warned him not to lose She suspects that Gratiano must have presented the ring to some young woman and not to the lawyerrsquos clerk as he repeatedly says and assures

Answer the following questions to check your preparation of Act IV Scenes 1 and 2

You must attempt only after you have completed your preparation of Act IV The answers must be in complete sentences using textual evidence (with citation) when necessary

[It would be in your own interest to attempt the above questions honestly totally refraining from consulting your textbook or your notes during answering After completion you should correct the paper yourself consulting the textbooknotes etc and award marks as specified Please let me know the marks you scored through WhatsApp in the group or to my personal WhatsApp]

Act IV Scene 1 (each question carries 2 marks)

1 What did the Duke try to do for Antonio

2 Why does Shylock refuse to show mercy How does he justify his stance

3 Why does Antonio say he is ready to die 4 What information is contained in Bellariorsquos letter

5 Why does Portia (as Balthazar) assert that Shylock must show mercy How does he respond

6 What offers are made to Shylock to get him to spare Antonio How are they received

7 What does Antoniorsquos speech as he faces the prospect of Shylockrsquos knife tell you about his character

8 How do Bassanio and Gratiano react to the looming prospect of Antoniorsquos demise

9 How does Portia (as Balthazar) use the law to turn the tables on Shylock

10 What does the Duke decree should happen to Shylock Why What happens to Shylockrsquos estate

11 What does Portia ask Bassanio as payment for her ldquoservicesrdquo What is his initial response What makes him change his mind

Act IV Scene 2 (each question carries 1frac12 marks)

1 What does Gratiano bring to Portia (Balthazar)

2 What does Nerissa plan on getting from Gratiano What does Portiarsquos comment suggest about men

ECO-10 280620 Topic-Supply AnalysisSHIFTING OF SUPPLY

But if there is change in factors other than the price of the commodity then either more is supplied at the same price or less supplied at the same price In such cases the price of the commodity remains constant but there is a change in other factors like change in the price of inputs change in technology of production change in price of other related goods change in taxation policy of the government etc For example there is an improvement in the technology of production of the commodity in question It leads to decrease in per unit of cost production of the commodity The firm is willing to sell more quantity of the commodity at the same price So the supply other commodity increases at the same price This increase in supply is shown by rightward shift of supply curve On the other hand if the firm uses inferior technology of production the cost of production per unit of the commodity increases The firm is willing to sell less quantity at the same price So the supply of the commodity decreases at the same price This decrease in supply is shown by leftward shift of the supply curve The above cases of increase and decrease in supply can be shown with the help of the following figures

Y INCREASE OF SUPPLY Price (Rs) s

P A s1

B

s

X` O s1 X

q q1

Y` Quantity demanded (in units)

Y DECREASE IN SUPPLY s2

s

price (Rs)

C

p A

s2

s

X` o X

q2 q

Y` Quantity demanded ( in units)

Main factors causing increase in supply or rightward shift of supply Curve(i) Fall in the price of other related goods

(ii) Fall in the price of inputsfactors(iii) Use of better technology in production(iv) Decrease in the rate of excise duty by government(v) If the objective of producer changes from profit maximization to salesMaximization

Main factors causing decrease in supply or leftward shift of supply curve(i) Increase in the price of other related goods(ii) Rise in the price of inputsfactors(iii) Use of inferior technology in production(iv) Increase in the rate of excise duty by the government(v) If the objective

Subject - Biology Topic ndash Chapter mdash6 PhotosynthesisSummary Execution

Today we will know about photosynthesis and its stages

Q1 What do you mean by photosynthesis The process by which living plants containing chlorophyll produce food

substances from carbon-di- oxide and water by using light energy Sunlight

6CO2 +12 H2O----------------------- C6 H12O6 + 6H2O + 6O2

Chlorophyll

Q2 What are the importance of photosynthesis I) Food for all Green plants trap solar energy by photosynthesis

process and supply food and energy for all living organisms either directly or indirectly

Ii) Oxygen to breathe in by product of photosynthesis is oxygen which is essential for all living organisms respiration

Q3 Write about two main phases of photosynthesis A Light dependent phase This phase occur in grana of chloroplast I) The chlorophyll on exposure to light energy becomes activated by

absorbing photons Ii) The absorbed energy is used in splitting the water molecules (H2O)

into its two components (H+ and OH- ) and releasing electron s 2H2O------------------------- 4H+ + 4e- +O2

Energy of 4 photons This reaction is known as photolysis

End products are H+ and oxygen water

B Light independent (Dark ) phase The reactions in this phase require no light energy

Here CO2 combine with H+ and produce glucose

Class XI

Subject Topic Summary ExecutionEVS Chapter-4 Legal

regimes for sustainable development

Environmental legislationEnvironmental legislation is the collection of laws and regulations pertaining to air quality water quality the wilderness endangered wildlife and other environmental factors The act ensures that matters important to the environment are thoroughly

Learn -The Forest (Conservation) Act 1980

considered in any decisions made by federal agencies

The Forest (Conservation) Act 1980 The Forest (Conservation) Act 1980 an Act of the Parliament of India to provide for the conservation of forests and for matters connected therewith or ancillary or incidental thereto It was further amended in 1988 This law extends to the whole of IndiaObjects and Reasons of the Forest Conservation Act

Deforestation causes ecological imbalance and leads to environmental deterioration Deforestation had been taking place on a large scale in the country and it had caused widespread concern The act seeks to check upon deforestation and de-reservation of forests

Subject Eng Literature (The Tempest ndash William Shakespeare) Topic Act II Scene 1 Lines 314 to 329 (End of scene)

[Students should read the original play and also the paraphrase given in the school prescribed textbook]Summary Questions amp Answers

Conspiracy of Antonio and Sebastian (Contd)

o As they approach Ariel appears again and wakes up Gonzalo by singing a tune in his ear Alonso also wakes up and they see both Sebastian and Antonio with drawn swords On being caught off guard they make up a story saying that they had heard a bellowing of bulls or lions

o They then moved to another part of the island

o Ariel at once rushes to Prospero to inform him of this development

SUMMING-UP of ACT-2 SCENE-1

(i) Among the survivors Ferdinand is separated from the rest which results in the disconsolate grief of Alonso as he took him for dead

(ii) The villainy of Antonio is confirmed

(iii) The supremacy of Prosperorsquos magic which resulted in the failure of the human conspiracy

(1)

(Act II Sc 1 L 311-325)SEBASTIAN Whiles we stood here securing your repose

Even now we heard a hollow burst of bellowing Like bulls or rather lions Didt not wake youIt struck mine ear most terribly

ALONSO I heard nothingANTONIO O rsquotwas a din to fright a monsters ear

To make an earthquake Sure it was the roarOf a whole herd of lions

ALONSO Heard you this GonzaloGONZALO Upon mine honour sir I heard a humming

And that a strange one too which did awake meI shaked you sir and cried As mine eyes opened I saw their weapons drawn There was a noiseThats verily rsquoTis best we stand upon our guardOr that we quit this place Lets draw our weapons

(i) Why has Prospero sent Ariel to Gonzalo and Alonso What does Ariel do to awaken Gonzalo

Prospero has already come to know by his magic powers the danger which threatens Gonzalo who had been Prosperorsquos friend and so he sent Ariel to preserve the lives of both Gonzalo and Alonso Prospero does not want that his scheme should remain unfulfilled Ariel begins to sing a song in Gonzalorsquos ears to awaken him(ii) Who are ready to carry out their plan Who takes steps to stop them Why does Gonzalo feel surprised after being awakened

Sebastian and Antonio are ready to carry out their plans They are standing with their swords drawn to kill Alonso and

(iv) We see two sets of contrasting characters Gonzalo-Adrian against Antonio-Sebastian

(v) The grief that works in Alonso can be perceived to his repentance for his association in Antoniorsquos crime against Prospero

Gonzalo Ariel takes steps to stop them from carrying out their nefarious scheme When Gonzalo is awakened by the song sung by Ariel into his ears he (Gonzalo) feels surprised because he sees Sebastian and Antonio standing with their swords drawn(iii) What reason do Sebastian and Antonio tell of drawing their swords when they are suspected by Alonso and Gonzalo

When Sebastian and Antonio are seen with their swords drawn they are looked with suspicion by Gonzalo and Alonso At first Sebastian tells them that as they stood here to guard them during their sleep they heard only a little before a sudden loud noise very much like the roaring of bulls or more probably that of lions Then Antonio follows him saying that this was a noise so terrible as to frighten even a monsterrsquos ears and this noise could even have shaken the earth and it was surely like the roaring of a multitude of lions Then seeing the danger they have drawn their swords Perhaps after hearing the terrible noise they (Gonzalo and Alonso) woke up from their sound sleep

(iv) What does Gonzalo tell Alonso about the strange noise What did he see on opening his eyes Gonzalo tells Alonso that he did not hear the sound of roaring but he heard a humming sound which was strange and which woke him up After waking up he gave him (Alonso) a shaking and a loud cry On opening his eyes he saw these two gentlemen standing with their swords drawn(v) What does Gonzalo suggest

Gonzalo suggests that there was a noise indeed and of that he has no doubt at all and suggests that the best course for them would be to remain alert and vigilant against any possible danger to their lives or to leave this place and move to some other part of the island

Class XIISubject Topic Summary Execution

Commerce

Chapter- Management

Today we will discuss about LEVELS OF MANAGEMENT

Levels of management is a series or chain of managerial positions from top to bottom It helps individuals to know their authority responsibilities and superior-subordinate relations among themselves There are mainly three levels of Management TOP LEVEL MANAGEMENTMIDDLE LEVEL MANAGEMENTLOWER LEVEL MANAGEMENT

Top level managementIt consists of members at the highest level in the management hierarchy This level includes Board Of Directors Chief Executive Managing Directors Chairman President Vice President

Rolefunctions of the top levelmanagement1To analyse evaluate and deal

with theexternal environment2 To determine the objectives and

policies of the business3 To strive for welfare and survival

of business

4 To create an organisational Framework consisting of authority responsibility relationship

Middle level management Congress of members or groups who are concerned with implementation of the policies let down by the top managementThis level includes head of the department such as finance manager marketing manager branch and regional managers departmental and divisional heads plant superintendent etc

Role of functions of the middle level management

1 To interpret the policies framed by top management

2 To assign duties and responsibilities to lower level managers

3 To select and appoint employees for middle and supervisory level and evaluate their performance

4 To co-operate with other departments for smooth functioning

Operational or supervisory level managementIt refers to the group are members who are concerned with execution of the work They are also known as fast line managers This level includes supervisor 4 men Section Officer clerk Inspector etc

Role of functions of the lower level management1 To plan and execute day-to-

day operations2 To supervise and control the workers3 To arrange materials and

tools to start the process and make arrangements for training

4 Today present workers grievance and suggestions before the management and

ensure safe and proper working conditions in the factory

Business Studies

Staff Appraisal Chapter- 10 Today let us start with a new chapter

Staff Appraisal

Meaning of Performance Appraisal

Performance Appraisal is the systematic evaluation of the performance of employees and to understand the abilities of a person for further growth and developmentThe supervisors measure the pay of employees and compare it with targets and plansThe supervisor analyses the factors behind work performances of employeesThe employers are in position to guide the employees for a better performance

Objectives of Performance Appraisal

Following are the objectives of Performance Appraisal

To maintain records in order to determine compensation packages wage structure salaries raises etc

To identify the strengths and weaknesses of employees to place right men on right job

To maintain and assess the potential present in a person for further growth and development

To provide a feedback to employees regarding their performance and related status

To provide a feedback to employees regarding their performance and related status

Importance of Performance Appraisal

Performance appraisal provides important and useful information for the assessment of employees skill

knowledge ability and overall job performance The following are the points which indicate the importance of performance appraisal in an organization

1 Performance appraisal helps supervisors to assess the work performance of their subordinates

2 Performance appraisal helps to assess the training and development needs of employees

3 Performance appraisal provides grounds for employees to correct their mistakes and it also provides proper guidance and criticism for employees development4 Performance appraisal provides reward for better performance

5 Performance appraisal helps to improve the communication system of the organization

6 Performance appraisal evaluates whether human resource programs being implemented in the organization have been effective

7 Performance appraisal helps to prepare pay structure for each employee working in the organization

8 Performance appraisal helps to review the potentiality of employees so that their future capability is anticipated

Geography

DRIANAGE The SubarnarekhaThe Subarnarekha and the Brahmaniinterposed between the Ganga and the Mahanadi deltas drain an area of 19300 sq kmand 39033 sq km respectively The drainage basins of these streams are shared byJharkhand Odisha west Bengal and Chhattisgarh The Brahmani is known as southKoel in its upper reaches in Jharkhand

The NarmadaThe Narmada rises in the Amarkantak hills of MadhyaPradesh It flows towards the West in a rift valleyformed due to a geological fault The total length of it is 1300 km All the tributaries of the

Q1 Name the two westward flowing rivers in the peninsular plateauA1 Narmada and Tapi are the only westward flowing rivers of the peninsular plateau

Q2 Differentiate between east-flowing rivers and west-flowing riversA2

East-flowing rivers

West-flowing rivers

Narmada are very short inlength Most of its tributaries join the main streamright anglesThe Narmada basin covers parts of Madhya Pradesh and Gujarat

The Tapi The Tapi rises in the Satpura ranges in the Betul listrictof Madhya Pradesh It flows in a rift valley parallel tothe Narmada but it is much shorter in length It coversparts of Madhya Pradesh Gujarat and MaharashtraThe length is about 724 km

The Sabarmati and the MahiThe Sabarmati rises in the Aravali hills and flows south-south-westwards for a distance of 300 kilometres to the Arabian Sea The Sabarmatibasin extends over an area of 21674 sq km in Rajasthan and Gujarat The Mahi rises inthe east of Udaipur and drains an area of 34842 sq km lying in Madhya PradeshRajasthan and Gujarat It flows south-westwards for a distance of 533 km before it fallsinto the Gulf of Khambhat

The ChambalThe Chambal rises near Mhow in the Vindhya Range and flows towards the northgenerally in a gorge upto Kota Below Kota it turns to the north-east direction and afterreaching Pinahat it turns to the east and runs nearly parallel to the Yamuna beforejoining it in the southern part of the Etawah district in Uttar PradeshMajor Rivers of India with their basin area (Sqkm)

Himalayan System Indus 321290Ganga 861404

Brahmaputra 187110Indus System

Jhelum 34775Beas 20303

Ganga System Yamuna 366223Ghaghra 127950

Peninsular RiversNarmada 98796

Tapi 65145Mahanadi 141600

Subarnarekha 19300Sabarmati 21674

Mahi 34842Godavari 312812

Godavari Krishna Kaveri Mahanadi are the east-flowing rivers

Narmada Tapi west-flowing rivers

They fall into the Bay of Bengal

They fall into Arabian Sea

These rivers form big deltas

These rivers form comparativelysmall deltas

Catchment areas of these rivers are larger

Catchment areas of these rivers are smaller

Krishna 2589488Cauveri 87900

Subject ndashBiology Topic ndashChapter -5 Inheritance amp Variations Summary ExecutionToday we will discussabout linkage and its classification

LINKAGE The tendency of the genes located on the same chromosome to stay together is

hereditary transmission Linked genes the genes responsible for this Genes that exhibit the process of linkage locates in the same chromosome The distance between the linked genes in a chromosome determines the strength

of linkage i e genes that are located close to each other show stronger linkage than that are located far from each other

COMPLETE LINKAGE It is the type of linkage showed by the genes that are closely located or are tightly

linked with each other as they have no chance of separatingby crossing over These genes are always transmitted together to the same gamete and the same

offspring In such condition only parental or non cross over type of gametes are formedINCOMPLETE KINKAGE It is type of linkage showed by the genes that are distantly located orare loosely

linked with each other because they have chance of separating by crossing over

SIGNIFICANCE i) It helps in holding the parental character togetherii) It checks the appearance of new recombination and helps in bringing the

hybrid population which resembles the original parents iii) Linked genes dilute the effects of undesirable traits

Subject Eng Literature (The Tempest ndash William Shakespeare) Topic Essay Questions (EQ-3)Question No 3

Give a character sketch of CalibanAnswer

The character of Caliban has been wonderfully conceived by Shakespeare as the manifestation of all that is gross and earthy ndash a sort of creature of the earth as Ariel is a sort of creature of the air

Calibanrsquos Physical Appearanceo Caliban is lsquofreckledrsquo a lsquomisshapen knaversquo not honoured with human shape

o Prospero calls him lsquothou tortoisersquo (Act I Sc 2 Line 317) Trinculo stumbling upon him describes him as ldquoA strange fish hellip Legged like a man And his fins like armsrdquo He ldquosmells like a fishrdquo (Act II Sc 2 Line 25)

o Prospero also calls him a ldquobeastrdquo (Act IV Sc 1 Line 140) and ldquoThis misshapen knaverdquo (Act V Sc 1 Line 268)

o Further it appears that in addition to his physical deformity his spiritual inferiority is also suggested by Prosperorsquos claim that his birth resulted from the union between his mother the witch Sycorax and the devil

Calibanrsquos ParentageWhen the play opens Caliban is twenty four years of age having been born on the island twelve years before the coming of Prospero His mother was the foul witch Sycorax who was banished from Algiers for ldquomischiefs manifold and sorceries terrible to enter human hearingrdquo (Act I Sc 2 Line 264) and the father was the Devil himself Thus

Caliban is a monster of evil and brute nature ugly deformed and stinking

Calibanrsquos Savage and Malignant Natureo Caliban is entirely a creature of the earth ndash gross brutal and savage He regards himself as the rightful possessor

of the island and Prospero as a usurper

o In his young age he was on good terms with Prospero He had consented to be received by Prospero at his house and to be educated by him He has learnt human language only to curse his master whom he abhors

o His beastly nature soon breaks out and ends in a vicious attack on Miranda This opens the eye of Prospero who becomes severe to him and enforces his service by threats and violence

o Prospero uses him to make dams for fish to fetch firewood scraper trenches wash dishes and keep his cell clean

Calibanrsquos Hatred for ProsperoA profound hatred for Prospero has taken hold of Caliban It springs from a sense of his being dispossessed and ill-treated He would kill Prospero if he could but he knows the power of Prosperorsquos lsquobookrsquo Hence he transfers his allegiance to Stephano who seems like a god to him He also incites the two drunken associates to batter the skull of Prospero when he sleeps in the afternoon

Caliban Shows Considerable Intelligenceo He has learnt Prosperorsquos language

ldquoYou taught me language and my profit onrsquot (Act II Sc 2 Lines 86-89)Is I know how to curserdquo

o He is well aware of the futility of arguing with one who has more power than he has

ldquoI must obey his art is such power (Act I Sc 2 Lines 373-376)It would control my damrsquos god SetebosAnd make a vassal of himrdquo

o He realizes the importance of Prosperorsquos books

ldquoRemember (Act III Sc 2 Lines 89-92)First to possess his books for without themHersquos but a sot as I am nor hath notOne spirit to commandrdquo

o He knows the value of stealth when attacking the enemy

ldquoPray you tread softly that the blind mole may not (Act IV Sc 1 Lines 194-195)Hear a foot fall we now are near his cellrdquo

o Caliban has a better set of values than Stephano and Trinculo They are distracted from their plan by their greed for Prosperorsquos rich garments Only Caliban realizes that such a finery is unimportant

ldquoLeave it alone thou fool it is but trashrdquo (Act IV Sc 1 Lines 224)

Caliban is not a good judge of characterCaliban is not a good judge of character He decides for example that Stephano is a god because he dispenses lsquocelestial liquorrsquo (Act II Sc 2 Line 115) but then it must be remembered that he has only known his mother Sycorax Prospero Miranda and the spirits that torture him However he quickly discovers his error of judgementrdquo

ldquoWhat a thrice-double ass (Act V Sc 1 Lines 295-297)Was I to take this drunkard for a godAnd worship this dull foolrdquo

Calibanrsquos Imaginative NatureIf Caliban is sub-human in what has been said above he is human in the respect of the poetic side of his character He listens to music with rapture He tells of the beautiful dreams in which heaven rains treasures upon him and which upon waking he yearns to renew One of the most poetic passages in whole play is Calibanrsquos description of the island

to Stephano and Trinculo

ldquoBe not afeard The isle is full of noises (Act III Sc 2 Lines 135-143)Sounds and sweet airs that give delight and hurt notSometimes a thousand twangling instrumentsWill hum about mine ears and sometime voicesThat if I then had waked after long sleepWill make me sleep again and then in dreamingThe clouds methought would open and show richesReady to drop upon me that when I wakedI cried to dream againrdquo

Caliban - Less Ignoble Than Some OthersCalibanrsquos motive for murder is less dishonourable than that of Antonio and Sebastian They plan to kill Alonso to gain his power and wealth Caliban merely wants revenge and the return of lsquohisrsquo island

Conclusiono Calibanrsquos character is not portrayed very clearly in the play and hence we cannot decide whether he is a poor

savage being grossly maltreated by Prospero or whether he is evil and must therefore be kept in bondage or enslavement

o Caliban is contrasted with Ariel who is a spirit and thus swift and uninterested in physical activitieso Caliban is also contrasted with Prospero who is the all-powerful master of the island and of the destiny of all

those on the islando Caliban is also contrasted with civilized man showing him to be less evil than Antonio and Stephano and less

materialistic than Stephano and Trinculoo Caliban has suffered at the hands of Prospero and he has learnt to curse by listening to Prosperorsquos abuse He

certainly believes that Prospero has deprived him of his birthrighto Finally the character Caliban is thought to be one of Shakespearersquos masterpieces The complexity of the character

is reflected in the large volume of critical discussion that has grown around it

ECO ndash12 Topic-Forms of market

MonopolyMonopoly is a market structure in which there is a single seller there are no close substitutes for the commodity produced by the firm and there are barriers to entry Example Indian Railways which is operated under government of India Monopoly also implies absence of competitionFeatures of Monopoly Monopoly is characterized by1 Single Seller In monopoly there is only one firm producing the product The whole industry consists of this single firm Thus under monopoly there is no distinction between firm and industry Being the only firm there is significant control of the firm over supply and price Thus under monopoly buyers do not have the option of buying the commodity from any other seller They have to buy the product from the firm or they can go without the commodity This fact gives immense control to the monopolist over the market

2No Close Substitute There are no close substitutes of the product produced by the monopolist firm If there are close substitutes of the product in the market it implies presence of more than one firm and hence no monopoly In order to ensure a total of control over the market by the monopolist firm it is assumed that there are no close substitutes of the product

3 No Entry amp Exit Monopoly can only exist when there is strong barriers before a new firm to enter the market In fact once a monopoly firm starts producing the product no other firm can produce the same One reason for this is the ability of the

monopolist to produce the product at a lower cost than any new firm who thinks to enter the market If a new firm who knows that it cannot produce at a lower cost than the monopolist then that firm will never enter the market for fear of losing out in competition Similarly the monopolist who is operating for a long time may be enjoying reputation among its customers and is in a better position to use the situation in its own benefit A new firm has to take long time to achieve this and so may not be interested to enter the market

4 Price Maker Being the single seller of the product the monopolist has full control over the pricing of the product On the other hand if there is a large number of buyers in the market so no single buyer exercises any significant influence over price determination Thus it is a sellerrsquos market So monopoly firm is a price maker

5 Price Discrimination Having considerable control over the market on account of being single seller with no entry of other firms the monopolist can exercise policy of price discrimination it means that the monopolist can sell different quantities of the same product to a consumer at different price or same quantity to different consumers at different prices by adjudging the standard of living of the consumer

6 Shape of Demand Curve Since a monopolist has full control over the price therefore he can sell more by lowering the price This makes the demand curve downward sloping

Subject Ac-12 290620 Topic- retirement Model sumThe Balance Sheet of Rohit Nisha and Sunil who are partners in a firm sharing profits according to their capitals as on 31st March 2014 was as under

Liabilities Amount Assets Amount (Rs) (` Rs)

Creditors 25000 Machinery 40000Bills Payable 13000 Building 90000General Reserve 22000 Debtors 30000Capital Less Provision for Rohit 60000 Bad debts 1000

29000 Nisha 40000 Stocks 23000 Sunil 40000 140000 Cash at Bank 18000

200000 200000

On the date of Balance Sheet Nisha retired from the firm and following adjustments were made(i) Building is appreciated by 20(ii) Provision for bad debts is increased to 5 on Debtors(iii) Machinery is depreciated by 10(iv) Goodwill of the firm is valued at Rs 56000 and the retiring partnerrsquos share is adjusted

(v) The capital of the new firm is fixed at Rs120000 Prepare Revaluation Account Capital Accounts of the partner and Balance Sheet of the new firm after Nisharsquos retirement Revaluation AccountDr Cr

Particulars Amount Particulars Amount (`Rs) (Rs`)

Provision for Bad debt Ac 500 Building Ac 18000Machinery Ac 4000Profit transferred toCapital Accounts (3 2 2)Rohit 5786Nisha 3857Sunil 3857

13500

18000 18000

Capital Account

Dr Cr

Particulars Rohit Nisha Sunil Particulars Rohit Nisha Sunil (Rs`) (Rs`) (`Rs) (Rs`) (Rs`) (Rs`)

Sunilrsquos Capital ac 9600 mdash 6400 Balance bd 60000 40000 40000Bank - 66143 - General Reserve 9428 6286 6286Balance cd 72000 mdash 48000 Revaluation (Profi 5786 3857 3857 Rohitrsquos Capital Ac mdash 9600 mdash

Sunilrsquos Capital Ac 6400 Bank 6386 - 4257

81600 66143 54400 81600 66143 54400

Balance Sheet as at 31st March 2014

Liabilities Amount Assets Amount (Rs`) (Rs`)

Creditors 25000 Building 108000Bank overdraft 37500 Machinery 36000

Bills Payable 13000 Debtors 30000Capital Less ProvisionRohit 72000 for Bad debts 1500 28500Sunil 48000 120000 Stock 23000

195500 195500

Working Notes (i) (a) Profit sharing ratio is 60000 40000 40000 ie = 3 2 2(b) Gaining Ratio Rohit = 35 ndash 37 = 2135 ndash 1535 = 635Sunil = 25-27 = 1435 ndash 1035 = 435= 635 435= 6 4 = 3 2(c) Nisha Share of Goodwill = Rs 56000 times 27 = Rs16000Share of Goodwill in the gaining ratio by the existing partner ieRohit = Rs16000 times 35 = Rs 9600Sunil = Rs 16000 times 25 = Rs 6400

The journal entry isRohitrsquos Capital Ac Dr 9600Sunilrsquos Capital Ac Dr 6400 To Nisharsquos Capital Ac 16000(Share of Goodwill divided into gaining ratio)

  • 1 Static Friction
  • The frictional force that acts between the surfaces when they are at rest with respect to each other is called Static Friction
    • Static Friction Examples
      • 2 Sliding Friction
        • Examples Of Sliding Friction
          • 3 Rolling Friction
            • Examples Of Rolling Friction
              • Objects and Reasons of the Forest Conservation Act
Page 20:  · Web viewSubject . Topic . Summary . Execution . English 1 . Sounds of animals . Hens –cackle Horses –neigh Lions –roar Owls –hoots Snake –hiss. English 2 . Mother’s

lead to a greater number of deaths due to heat strokes

3 Forest fires become more frequent4 Tropical cyclones and hurricanes

become common5 Melting of glaciers takes place6 Polar ice caps are becoming thinner

and melting at an alarming rate due to global warming The loss of sea ice

7 Due to increase in sea surface temperature sea levels rise in coastal areas and cause submergence of several islands

WAYS TO REDUCE GLOBAL WARMINGFollowing steps can be taken We need to decrease emission of

green house gases by reducing the burning of fossil fuel such as coal and petroleum

By planting more trees to increase forest cover

The government should also distributes free saplings and organize afforestation programmes to spread awareness regarding the beneficial effects of trees

We should switch to eco-friendly cars and gadgets

Incandescent light bulbs should be replaced by CFL bulbs

We can save electricity and reduce global warming by turning off electrical gadgets such as lights fans air-conditioners television and computer when we do not to use them

Efforts should be made to hasten the development of green cities oreco cities These cities are urban areas around the world striving to lessen the environment a impacts of urbanization

By following the 3Rs-Reduce Recycle and Reuse strategy we can use natural resources for our growth as well as save them for the need of the future generations This is called sustainable development

of deaths due to heat strokes3 Forest fires become more

frequent4 Tropical cyclones and hurricanes

become common5 Melting of glaciers takes place

etc

Q2 How to reduce global warmingA2 Following steps can be taken to reduce global warmingaWe need to decrease emission of

green house gases by reducing the burning of fossil fuel such as coal and petroleum

bBy planting more trees to increase forest cover

c The government should also distributes free saplings and organize afforestation programmes to spread awareness regarding the beneficial effects of trees

dWe should witch to eco-friendly cars and gadgets

eIncandescent light bulbs should be replaced by CFL bulbs

f We can save electricity and reduce global warming by turning off electrical gadgets such as lights fans air-conditioners television and computer when we do not to use them

Q3 What do you mean by 3Rrsquos of resource planningA3 The 3Rs are

1 Reduce 2 Recycle and3 Reuse

Q4 What is Sustainable developmentA4 By following the 3Rs-Reluce Recycle and Reuse strategy we can use natural resources for our growth as well as save them for the need of the future generations This is called sustainable development

English Language

Prepositions A preposition is a word placed before a noun or a pronoun It helps to show how the person or thing denoted by the noun is related to something else in the sentence

Kinds of Prepositions

Simple Prepositions- simple preposition are one word Prepositions such as at by for in of off for from on out through till to up with before amidst towards beyond between over etc

Compound Prepositions ndash There are some words that are always used with fixed Prepositions to convey specific meaning

Example I was unable to meet you dueto a previous engagement ( On account of)Always maintain the queue instead of crowding at the counter ( In place of)

Participial PrepositionsmdashParticiple Prepositions are present or past participles of various verbs which together with a noun phrase or a clause function as prepositions Examples- barring concerning considering notwithstanding pending regarding respecting etc

Exercise A

1 Gauravs fever has come down since Friday He has been absent for a week now

2 The child sat between his father and mother among the parents of all his classmates

3 There are mosquitoes in the room They flew into the room when the door was open

4 My father was inside the drawing room when I was playing outside my house

5 You may sit beside me I will give you a drawing book and pencils besides a storybook

6 We went to the market in the morning and walked towards the riverfront in the evening

7 The child walked along the pavement and across the street safely

8 This table top is made of glass My breakfast fell off it in the morning

9 The pan is on the gas stove There are vegetables in it

10 We will wait for you at the bus top There are a lot of people in the hall

Subject ndash Biology Topic ndash Chapter - 3 Photosynthesis and respiration in plants Summary Execution

All living organism (Plants and animals) need food for energy and growth Green plants (autotrophy) prepare food for all living organisms Today we will discuss about the process photosynthesis And adaptations in a leaf to carry out photosynthesis

Q1What do you mean by photosynthesis and write its word equation The process by which green plants make food (glucose) from carbon dioxide and water

in the presence of sunlight and chlorophyll is called photosynthesis

Carbon dioxide + Water ( Sun light from Sun ) Glucose + Oxygen ( chlorophyll in green leaves )

Q2 What are the adaptations in a leaf to carry out photosynthesisi) Leaves are broad wide and flat for absorbing more light energyii) Presence of chlorophyll in chloroplasts to trap sunlightiii) Presence of stomata which allow carbon dioxide to enter the cell and oxygen to go

out iv) Network of veins ensures continuous supply of water and minerals to the leafv) Thin waxy cuticle protects the leaf without blocking the lightQ3 Draw and label structure of chloroplast

Class VIIISubject Topic Summary Execution

PHYSICS ENERGY Production of Hydro electricity

A hydroelectric dam converts the potential energy stored in a water reservoir behind a dam to mechanical energymdashmechanical energy is also known as kinetic energy As the water flows down through the dam its kinetic energy is used to turn a turbine

The generator converts the turbinersquos mechanical energy into electricity

This electric energy then goes through various transmission processes before it reaches you

Question 2

Fill in the blanks

(a) Work is said to be done by a forte only when the body moves

(b) Work done = Force x distance moved in direction of force

(c) The energy of a body is its capacity to do work

(d) The SI unit of energy is joule

(e) The potential energy is due to its state rest of position and kinetic energy of the body is due to its state of motion

(f) Gravitational potential energy U = mass times force of gravity on unit mass times height

(g) Kinetic energy = frac12 times mass times (speed)2

(h) Power P = work donetime taken

(i) The S I unit of power is watt

(j) IHP = 746 W

BIOLOGY Chapter -5 The endocrine system and adolescence

Today we will discuss about thelocation and functions of secreted hormones of adrenal and Pancreas

Q5 Write location hormone secreted main functions and deficiency diseases of pancreas and adrenal glands

Endocrine Glands

Location Hormones secreted

Functions and Deficiency Diseases

1Adrenal gland

2 Pancreas Gland

On the top of each kidney

In between stomach and small intestine

i)Adrenaline from adrenal medulla

ii)Cortisone from adrenal cortex

i) Insulin

ii) Glucagon

It helps a person deal with any kind of emergency situation or emotional stressIt increases the heart beat rate of respiration and blood pressure

a) It regulates carbohydrates protein and fat metabolism

b) It regulates the salt and water balance in the body

a) It changes excess glucose into glycogen

b) It stimulates the cells to burn extra glucose to provide heat amp energy

Less secretion causes diabetes mellitus

Excessive secretions causeinsulin shock

a) It stimulates the breakdown of glycogen into glucose

b) It increases the level of glucose in blood

History Traders to rulers The Battle of Buxar was fought on 22 October 1764 between the forces under the command of the British East India Company led by Hector Munro and the combined armies of Mir Qasim the Nawab of Bengal till 1763 Mir Jafar was made the Nawab of Bengal for a second time in 1763 by the Company just after the battle After being defeated in 4 battles in katwa and Udaynala the Nawab of Awadh Siraj id Daula and the Mughal emperor Shah Alam II accompanied by Raja Balwant Singh of Kashi made an alliance with Mir Qasim The battle was fought at Buxar a small fortified

Answer the following questions- Short note-Battle of BuxarHomework-learn

town within the territory of Bihar located on the banks of the Ganga river about 130 kilometres (81 mi) west of Patna it was a decisive victory for the British East India Company The war was brought to an end by the Treaty of Allahabad in 1765

EnglishLiterature

The west wind-John Mansfield

In the poem The West Wind by John Masefield the poet starts by describingwith very poetic imagery of birds how the west wind is different from other winds its a warm wind full of birds cries There is a touch of melancholy perhaps home-sickness as he describes how it brings tears too and memories from an old land He goes on to describe the restful pastoral beauty of the land where even the dead can lie in the green He then brings in voicesperhaps of family and friends calling him home as he is missing Aprils beautyThe voices then tempt him some more with idyllic images from home (white blossom young green cornrunning rabbitswarm sun) The voices seem to presume that the poets heart is sorrowful bruised and soreThe end of the poem sees the poet appear to make a decision he will go home as he has decided that is where he truly belongs

Write the synopsis of the following words

1 Daffodils- a tall yellow flower that grows in the spring

2 Orchards- a piece of land on which fruit trees are grown

3 Blossom- a flower or a mass of flowers especially on a fruit tree in spring

4 Thrushes- a bird5 Larks- a small brown bird that

makes a pleasant sound6 Bruised- an injury7 Aching- pain 8 Tread- to put your foot down

while you are walking9 Balm-10 May-11 Fluting-

(Write from the book in your copy)

MAT

HEM

ATIC

S

Ch 1

1Al

gebr

ic E

xpre

ssio

n

1 Constant A symbol which has fixed value is called a constant[eg 8 23 -15 radic3 etc]

2 VariableA symbol which does not have any fixed value but may be assigned value (values) according to the requirement is called variable or literal[eg x y p q etc]

3 TermsA term is a number (constant) a variable a combination (product or quotient) of numbers and variables[eg 7 x 5x etc]

4 Algebric expressionA single term or acombination of two or more terms connected by plus (+) or minus (-) sign forms an algebraic expression[eg 5-y 3x2-5x xy-6z+4 etc]

5 PolynomialAn algebraic expression which contains more than one term is called a polynomial (multinomial)[eg x2-5x 5y+xy+x2y etc]

6 Degree of polynomial(a) When the polynomial contains only one variable the highest power of the variable is the degree of the polynomialeg the degree of the polynomial of 4x-7x5+8 is 5(b) When the polynomial contains two or more variablesStep (i) Find the powers of the variables in each term (ii) The highest sum of the powers is taken to be the degree of the polynomialeg the degree of the polynomial 5x2y-4x3y5+6 is = 3+5 = 8Remember An algebraic expression is a polynomial if degree of each term used in it is a non-negative integer

Exercise ndash 11(A)

1 Separate the constants and variables from the following

-7 7+x 7x+yz radic5 radic xy 3 yz

8 45y -3x

Solution Constant Variables-7 radic5 7+x 7x+yz radic xy

3 yz8

45y -3x

2 Write the number of terms in each of the following polynomials(i) 5x2+3timesax (ii) axdivide4-7 (iii) ax-by+ytimesz (iv) 23+atimesbdivide2

Solution Polynomials Number of terms(i) 5x2+3timesax 2(ii) axdivide4-7 2(iii) ax-by+ytimesz 3(iv) 23+atimesbdivide2 2

4 Write the degree of the each polynomials(i) xy+7z (ii) x2-6x3+8 (iii) y-6y2+5y8 (iv) xyz-3 (vi) x5y7-8x3y8+10x4y4z4

Solution Polynomials Degree(i) xy+7z 2(ii) x2-6x3+8 3(iii) y-6y2+5y8 8(iv) xyz-3 3(vi)x5y7-8x3y8+10x4y4z4 12

5Write the coefficient of(i) ab in 7abx (iv) 8 in a2-8ax+a (v) 4xy in x2-4xy+y2

SolutionCoefficient

(i) ab in 7abx 7x(iv) 8 in a2-8ax+a -ax(v) 4xy in x2-4xy+y2 -1

7 CoefficientAny factor of an algebraic quantity is called the coefficient of the remaining quantityeg in the algebraic term 7xyz 7 is coefficient of xyz 7x is coefficient of yz and so on

8 Like term The terms having the same literal coefficient are called like terms and those having different literal coefficients are called unlike terms

eg (i) 5xyz 8xyz -6xyz and 23xyz are like

terms(ii) 7xy2 8x2yz and -15xyz2 are unlike terms

6 in 57xy2z3 write the coefficient of

(i) 5 (vii) 5xy2 (viii) 17yz (xi) 5xyz

Solution Coefficient

(i) 5 17

xy2z3

(vii) 5xy2 17z3

(viii) 17yz

5xyzsup2

(xi) 5xyz 17yz2

7 In polynomial given below separate the like terms(ii) y2z3 xy2z3 -58x2yz -4y2z3 -8xz3y2 3x2yz and 2z3y2

Solution y2z3 -4y2z3 2z3y2 are like terms

xy2z3 -8xz3y2 are like terms

-58x2yz 3x2yz are like terms

Class IXSubject Topic Summary Execution

Bengali (2nd language)

বাগzwnjধারাzwnj বা ধারা-বা ধারা ল হিবকেশষ পরকার বাক -হিb -াকেবর এক হিবকেশষ পরকাশরীহিত াকেক কতগকেলা কার সমষটির মকেধয এগহিলকেক বা ধারা বকেল আবার কতগকেলা শকেবদর বাধাধরা যকান রীহিত যনই য-াকেব চকেল আসকে যসই -াকেবই চকেল আসকে তখন যসই শবদগহিল খন একক -াকেব অG পরকাশ ককের তখন একের বা ধারা বকেল বা ধারার পরকেয়া -াষাকেক আরও সFর ককের যতাকেল

অকাল পকক(অপহিরনত বয়কেস পাকাহিম)-মাতর শ বর বয়কেস যমকেয়টির া মকেখর কা তাকেত অকালপককতা ধরা পকে

অককা পাওয়া( মারা াওয়া) ndash পকেকIমারটি পকেকIমারকেত হিকেয় বাসাতরীকের াকেত মার যখকেত যখকেত অককা যপল

অহি| পরীকষা ( কঠিন ও পরকত পরীকষা)- যকেলটির আজ ডাকতাহির যরজালট যবকেরাকেব এIাই তার জীবকেনর ব অহি| পরীকষা

অষটরমভা (ফাহিক) ndash রীতা মকেখই বকো বকো কা বকেল আর কাকেজর যবলায় অষটরমভা

অকমGার ধাী (অপাG) ndash সমনকেক হিনকেয় যকান ান কেব না ও একেকবাকেরই অকমGার ধাী

অকেনধর ষটি (অসাকেয়র সায়)- আহিশ বকেরর বকোর নাহিত ল অকেনধর ষটি তাকেক াা বকোর একম চকেল না

আকেককল গড়ম (তবহিদধ)- ার তহিম উপকার করকেল যসই যতামার হিবরকেদধ সাকষয হিকেয়কে শকেনই আমার আকেককল গড়ম

আষাকে লপ( অবাসতব লপ) ndashIাকা এখন যকেব না এIা বলকেলই ত এমন আষাকে লপ ফাার যকান রকার হিল না

Hindi- महायजञ ा इस हानी म लख न या बतान ा परयास किया ह कि किसी भी अचछ

2nd language

परसार(यशपाल ाय या पणय न ा फल अवशय मिमलता ह ोई भी परोपार अथवा पणय लिलए किया गया ाय बार नही 0ाता वह ए परार ा यजञ हए धनी सठ थ धम परायण और किवनमर सठ न आन ी यजञ किए थ और दान म न 0ान कितना धन दिदन दखिखयो म बात दिदया थादिदन पलट और सठ यहा गरीबी आ गई उन दिदनो यजञ बचन ी परथा थी सठ भी अपनी 0गह बचन लिलए डलपर ए सट यहा चलन ो तयार हए सठानी रासत लिलए रोटी पड म बाधर सठ ो द दी रासत म ए भख R ो दखर सठ न चारो रोटी उसो खिखला दी खर वह सठ यहा डलपर पहच तो उनी सठानी न उस महायजञ बचन ो हा यदिद बचन आए सठ न R ो रोटी खिखलान ो महायजञ नही समझा और वापस लौट आया घर आर शाम ो उसी घर म उस ए बडा ख0ाना मिमला 0ो उस दवारा किए गएrsquo महायजञrsquo ा परसार था

English language

Letter formal The heading the name and address of the person you are writing to must be included beneath your own address In formal letters ldquoblock stylerdquo of address is preferred

Subject complain in brief

Salutation If the person you are writing to is known to you you may begin ldquoDear MrrdquoOr ldquoDear Mrsrdquo In all other instances you should begin ldquoDear Sirrdquo or ldquoDear Madamrdquo Or ldquoSirsrdquo

The body A formal or business letter has four partsReference The letter should begin by referring to a letter you have received an advertisement or the reason that has prompted you to writeInformation In the second paragraph it is necessary to supply more detailed information that is related to the referencePurpose Here you must give the reason why you are writing the letter This must be stated clearly and ensure that it is relevant to the question that has been setConclusion round off the letter with some polite remarkThe subscription when a letter has begun with dear sir sirs Madam you should end with Yours faithfully or yours truly When however you address a person by name you must conclude with the words ldquoYours sincerelyrdquo

1 A park in your locality is slowly being used as a rubbish dump Write a letter to the Mayor of your city pointing out the nuisance and danger of this Request that action be taken to stop this immediately

Or2 You being a boarder ordered a set of lab manuals from a famous book shop in the town They sent you a wrong set of books Write a letter to the manager of the book shop

Chemistry Chapter-1 1)CHEMICAL FORMULA- Q What is the Significance of

L-2The Language of Chemistrybull Chemical Formula

Itrsquos a symbolic representation of a chemical substance eg ndash The formula of Sulphuric acid is H2SO4

2) Steps of writing Chemical Formula of a given substance-

1 Write the symbols of the constituent atoms or radicals side by side Keep the basic radical on LHS and acid radical on the RHS ( Na+Cl- )2 In case of a radical having more than one atom( compound radical) enclose the radical in a bracket eg (SO4-)3 Write the valencies of each radical on its right hand top4 If the valencies of the two radicals are divisible by a common factor then divide the valencies by the common factor5 Invert (criss-cross) the valency number ie write the valency of one atom below the second atom and vice versa 6 On interchanging if valency number is lsquoone the figure lsquoonersquo is never writtenFor Example- Compound -Calcium Nitrate1 Writing the symbols- Ca(NO3)2 Writing the valencies on their right hand top- Ca2(NO3)1

3 Valency numeral in simple ratio- Ca2(NO3)1

4 Criss-cross- Ca 2NO3 1

5 Writing the formula of the compound- Ca(NO3)2

Chemical formula

A The formula of a substance conveys the following information regarding a substance 1 The name of the substance (qualitative)2 The elements constituting the substance (qualitative)3 The number of various atoms present in a molecule of the substance (quantitative)4 Molecular weight of the substance and the relative weights of different elements present in it (qualitative)

Q What are the limitations of Chemical Formula

A The chemical formula suffers from the following limitations-I It fails to convey whether the elements in a molecule are present in the form of atoms or ionsFor example the formula KBr fails to tell us whether Potassium and Bromine are present in the form of ions II It does not tell anything about the binding force that holds atom in a molecule togetherIII It does not tell us about the arrangement of various atoms with respect to one another within the molecule

Q Examples of Some Chemicals with their Formula Chemical name and Common Name-

A Given in the class notesCommercial Studies

Joint Stock Company

Let us discuss about the demerits of Joint Stock CompanyDespite so many advantages it has got many disadvantages which are as follows

Difficulty in FormationDelay in Decision makingExcessive Government ControlLack of Secrecy

Company can be classified into several categories based on incorporation

QuestionExplain the demerits of Joint Stock CompanyAnswer) 1 Difficulty in Formation The legal requirements and formalities required to be completed are so many The cost involved is quite heavy It has to approach large number of people for its capital It cannot start its business unless certificate of incorporation has been obtained This is granted after a long time when all the formalities are completed

Chartered CompanyStatutory CompanyRegistered Company

Delay in Decision making In this form of organization decisions are not made by single individual All important decisions are taken by the Board of Directors Decision-making process is time-consuming So many opportunities may be costly because of delay in decision-making Promptness of decisions which is a common feature of sole trader ship and partnership is not found in a company

Excessive Government ControlA company and the management have to function well within the law and the provisions of Companies Act are quite elaborate and complex At every step it is necessary to comply with its provisions lest the company and the management should be penalized The penalties are quite heavy and in several cases officers in default can be punished with imprisonment This hampers the proper functioning of the company

Lack of Secrecy The management of companies remains in the hands of many persons Every important thing is discussed in the meetings of Board of Directors Hence secrets of the business cannot be maintained In case of sole proprietorship and partnership forms of organisation such secrecy is possible because a few persons are involved in the management

2 Define the following

Chartered Company- The crown in exercise of the royal prerogative has power to create a corporation by the grant of a charter to persons assenting to be incorporated Such companies or corporations are known as chartered companies Examples of this type of companies are Bank of England (1694) East India Company (1600) The powers and the nature of business of a chartered company are defined by the charter which incorporates it After the country attained independence these types of companies do not exist

in IndiaStatutory Company- A company may be incorporated by means of a special Act of the Parliament or any state legislature Such companies are called statutory companies Instances of statutory companies in India are Reserve Bank of India the Life Insurance Corporation of India the Food Corporation of India etc The provisions of the Companies Act 1956 apply to statutory companies except where the said provisions are inconsistent with the provisions of the Act creating them Statutory companies are mostly invested with compulsory powersRegistered companiesCompanies registered under the Companies Act 1956 or earlier Companies Acts are called registered companies Such companies come into existence when they are registered under the Companies Act and a certificate of incorporation is granted to them by the Registrar

Economics

Chapter-4Basic problems of Economy

Today let us discuss with the topic Production Possibility curve

QuestionExplain the concept of Production Possibility Curve with the help of diagram

Answer) Production Possibility curve is a locus of all possible combinations of two commodities which can be produced in a country with its given resources and technology

The above diagram shows that with the given resources and technology the economy can produce maximum either 5 thousand meters of cloth or 15 thousand quintals of wheat or any other combination of the two goods like B( 1 thousand meters of cloth and 14 thousand quintals of wheat C ( 2 thousands meters of cloth and 12 thousand quintals of wheat) etcProduction Possibility curve is also called production possibility boundary or frontier as it sets the maximum limit of what it is possible to produce with given resources

Geography

Rotationand Revolution

SUNrsquoS POSITION AND SEASONAL CHANGES EQUINOXES ndash SPRING AND AUTUMN

Q1 What is Spring EquinoxA1 On 21st March sunrays fall directly on the equator On that day

As the Equator divides the Earth into two equal halves the sun rays fall directly on the equator twice in a year Equinoxes means equal Spring EquinoxOn 21st March sunrays fall directly on the equator On that day the duration of day and night both are equal ( 12 hours day and 12 hours night) on every places located on equator This day is called as Spring EquinoxAutumn EquinoxOn 23rd September sunrays fall directly on the equator On that day the duration of day and night both are equal ( 12 hours day and 12 hours night) on every places located on equator This day is called as Autumn Equinox

SOLSTICES ndash SUMMER AND WINTERDue to inclination of the Earth on its axis and the apparent movement of the sun the sun rays fall directly on both tropics once in a year Solstice is a Latin word which mean ldquothe Sun standing stillrdquoSummer SolsticesAfter 21st March there is an apparent movement of the Sun to the north of the equator The apparent northward movement up to 21st June when the Sun appears overhead at the Tropic of Cancer (22frac12degN) The sun appears to stand still at this position and then moves southwards towards the equator This position of the Sun on 21st June is known as Summer Solstices On that day the duration of day and night both are equal ( 12 hours day and 12 hours night) on every places located on Tropic of Cancer (22frac12degN)Winter solstices The apparent southward movement of the Sun continues beyond the equator till 22nd

December On this day the Sun is overhead at the Tropic of Capricorn

the duration of day and night both are equal ( 12 hours day and 12 hours night) on every places located on equator This day is called as Spring Equinox

Q2 What do you mean by EquinoxA2 Equinoxes means equal It is use to explain the equal duration of day and night ( 12 hours day and 12 hours night) on the Earth

Q3 On which date the longest day in Tropic of CancerA3 21st June

Q4 What is the meaning of SolsticeA4 Solstice is a Latin word which mean ldquothe Sun standing stillrdquo

Q5 Which is the longest day in southern hemisphereA5 22nd December

Q6 On what date does the Arctic Circle experience the lsquoMidnight SunrsquoA6 On 21 June the Arctic Circle experiences the lsquoMidnight Sunrsquo

Q7 What is cause of Midnight Sun in NorwayA7 During the summer solstice (21 June) the North Pole is inclined towards the Sun Therefore the duration of sunlight or daytime increases from 12 hours at the Equator to 24 hours at the Arctic Circle and beyond Thatrsquos why The region beyond the Arctic Circle especially Norway is known as the Land of the Midnight Sun because there the Sun does not rise or set on 21 June

Q8 Match the column A with BA B

Summer Solstice 21st March

Autumn Equinox 23rd

September

Winter Solstice 21st June

(22frac12degS) This position of the Sun is referred to as the Winter Solstice because it marks the winter season in the Northern Hemisphere On that day the duration of day and night both are equal ( 12 hours day and 12 hours night) on every places located on Tropic of Capricorn (22frac12degS)SEASONS AND DURATION OF DAY AND NIGHT During the equinoxes all places on the Earth have 12 hours of day and 12 hours of night Due to the revolution of the Earth round the Sun on an inclined axis the duration of day and night varies according to seasons and the latitude of a placeDuring the summer solstice (21 June) the North Pole is inclined towards the Sun Therefore the duration of sunlight or daytime increases from 12 hours at the Equator to 24 hours at the Arctic Circle and beyondThe region beyond the Arctic Circle especially Norway is known as the Land of the Midnight Sun because there the Sun does not rise or set on 21 JuneAt the North Pole there will be six months of daylight The Sun will be seen always above the horizon at a low angle At 66degN 24 hours of sunlight can be seen only on 21 June Hammerfest in northern Norway is a place of tourist attraction for observing the phenomenon of the Midnight Sun This place has continuous daylight from 13 May to 29 July This place is easily accessible to tourists and has hotels and other facilities The view of the midnight Sun from here is enthrallingIn the Southern Hemisphere the duration of daylight decreases from 12 hours at the equator to 0 hours beyond the Antarctic Circle In the South Polar Region there is 24 hours of darkness The Sun is always below the horizon In the Southern Hemisphere which experiences winter the duration of night-time is longer than the duration of daylight

Spring Equinox 22nd

December

A8 A B

Summer Solstice 21st June

Autumn Equinox 23rd

September

Winter Solstice 22nd

December

Spring Equinox 21st March

During winter solstice (22 December) the South Pole is inclined towards the Sun The Southern Hemisphere experiences summer and the Northern Hemisphere has winter Therefore the duration of daylight or sunlight is greater in the Southern Hemisphere than in the Northern HemisphereThe duration of daylight increases from 12 hours at the equator to 24 hours beyond the Antarctic Circle The South Polar Region has 24 hours of sunlight for many days continuously At the South Pole there will be six months of sunlight The Sun will always be seen at a low angle above the horizon In the Northern Hemisphere the duration of daylight will decrease from 12 hours at the equator to 0 hours at the Arctic Circle There are 24 hours of darkness in the North Polar region The duration of night is greater than the duration of daylight as one move northwards from the Equator It is evident from the above table that the duration of daylight is 12 hours throughout the year at the equator only As one moves away from the equator the seasonal variations in the duration of daylight increase The seasonal variations in the duration of daylight are maximum at the Polar Regions

Subject Eng Literature (The Merchant of Venice ndash William Shakespeare)Topic Act II Scene 7 Lines 36 to 80 (End of scene ) [Students should read the original play and also the paraphrase provided]

Summary Questions amp AnswersThe Prince then examines the inscription on the silver casket which says ldquoWho chooseth me shall get as much as he deservesrdquo The Prince says that he deserves Portia more than anybody else because of his high rank his noble birth and his great wealth and power But then he argues that silver is ten times

(1) (Act II Sc 7 L 39-47)

From the four corners of the earth they come

To kiss this shrine this mortal breathing saint

The Hyrcanian deserts and the vasty wildsOf wide Arabia are as through-fares now

inferior to gold and therefore he cannot believe that the portrait of such a beautiful lady as Portia can be contained in the silver casket He decides to see the inscription on the golden casket before making his decision

The Prince goes to examine the inscription on the golden casket which says ldquoWho chooseth me shall get what many men desirerdquo The Prince believes that the whole world desires to possess Portia otherwise so many suitors would not have come from all corners of the world for winning Portia Some of them have come from the distant lands of Persia and Arabia The deserts of Persia (Hyrcanian deserts) and the boundless desolate lands of Arabia have been crossed by the Princes seeking the hand of Portia He contrasts this casket containing Portiarsquos portrait with the old English gold coin bearing the image of the archangel (angel of the highest rank) He goes on to remark that while the figure of the archangel is engraved (Insculped) upon the English coin the picture of Portia who is beautiful as an angel lies hidden inside one of the caskets namely the Golden Casket (Golden Bed)

On the basis of his assessment of the inscription on the golden casket the Prince decides to choose the golden casket He asks for the key and opens the golden casket only to find therein an empty human skull holding a roll of

For princes to come view fair PortiaThe watery kingdom whose ambitious headSpets in the face of heaven is no barTo stop the foreign spirits but they comeAs orsquoer a brook to see fair Portia

(i) Explain the occasion for the above mentioned speech

These are the comments of the Prince of Morocco after he reads the inscription on the golden casket His mental process is revealed to us in these words We find him debating within himself as to which casket he should choose

(ii) What light does the above speech throw on the personality of Prince of Morocco

From the above mentioned speech we come to know that the Prince of Morocco is keen to marry Portia He is the type of person who is easily taken away by outward appearance He is in love with Portia because of her beauty

(iii) What information can you gather about Portia from the above mentioned lines

The given speech shows that Portia is a very beautiful lady She must be possessed of good qualities because many suitors come to her place from all over the world with a desire to get married to her The Prince of Morocco is so impressed by her beauty that he calls her a saint According to him the whole world is desirous of having her

(iv) Elucidate the significance of the first two lines

In these lines the Prince of Morocco pays a compliment to Portia These lines show his admiration for her He says that people come from all parts of the world to see fair Portia

(v) Explain the meaning of the last four lines of the

passage

In these lines the Prince of Morocco says that even the vast oceans which throw a challenge at the sky are unable to prevent men from coming to Portiarsquos place to have a glimpse of her These lines are also a tribute to Portiarsquos beauty and good qualities Many men voyage across the ocean treating it as a mere stream to see the beautiful Portia

paper in which is written that whoever happens to be guided by the glitter of things is invariably deceived

On reading the scroll the Prince says that he is too sad at heart to speak a more formal farewell and leaves with his followers amidst a sound of trumpets

After the Prince of Morocco leaves Portia remarks that the Prince is a gentle fellow but she is rid of him May all persons of his nature make a similar choice

IMPORTANT PASSAGES EXPLAINED

(Act II Sc 7 L 39-43)From the four corners of the earth they come

To kiss this shrine this mortal breathing saintThe Hyrcanian deserts and the vasty wildsOf wide Arabia are as through-fares nowFor princes to come view fair Portia

Context

This passage occurs in Act II Scene 7 in The Merchant of Venice This is part of the speech made by the Prince of Morocco

(2)

(Act II Sc 7 L 48-53)

MOROCCO One of these three contains her heavenly pictureIst like that lead contains her

Twere damnation To think so base a thought it were too grossTo rib her cerecloth in the obscure graveOr shall I think in silver shes immurdBeing ten times undervalued to tried gold

(i) What meaning does the Prince of Morocco find out of the inscription of the golden casket What have Belmont and Portiarsquos house been called and why

The inscription on the golden casket is ldquoWho chooseth me shall gain what many men desirerdquo The Prince finds out that it means that the chooser of the golden casket will get Portia because many men desire her In fact the entire world desires her Because of the coming of many suitors to Belmont from different countries in order to win Portiarsquos hand Belmont has become a centre of pilgrimage and her house is the shrine where saintly Portia is installed

(ii) What does the Prince of Morocco do before making the final choice of the casket Which is the correct casket and who will win Portiarsquos hand

The Prince of Morocco surveys and analyses the inscriptions on the casket of lead silver and gold Before making the final choice like a very systematic and methodical person he once again considers the claims of the caskets The casket containing Portiarsquos picture is the correct casket and the person choosing it will win Portiarsquos hand

Explanation

While praising Portia the Prince of Morocco conceives Portia as a goddess whose image is placed inside one of the caskets Many suitors are coming from far and wide the north and the south the east and the west (Four corners) in order to try their luck Some of them have come from the distant land of Persia and Arabia The deserts of Persia (Hyrcanian deserts) and the boundless desolate lands of Arabia have been crossed by the Princes seeking the hand of Portia All this shows that Portia is indeed the most beautiful lady of the world

(iii) What does the Prince of Morocco say in his estimation while examining the motto on the silver casket What does he find in the golden casket

While examining the motto on the silver casket which says ldquoWho chooseth me shall get as much as he deservesrdquo Morocco says that in his own estimation he surely deserves Portia in all respects ndash rank birth wealth etc

He chooses the golden casket When he opens it he finds an empty human skull holding a scroll in which it is written that those who are attracted by the glittering outside of things are always deceived as Morocco has been deceived

(iv) What kind of nature does the Prince of Morocco have

The Prince of Morocco has a simple nature who does not look deeply into the inner meaning of things but is dazzled by the outward appearance of gold He is inclined to over-estimate his own value and does not realize that it is a duty to ldquogive and hazardrdquo To say that he will not hazard for lead shows that he misreads the true meaning of the inscription which is that he should be prepared to ldquohazard all he hathrdquo for Portia So his feeling is only one of fascination and romantic attraction

(v) Do you think that the lottery of the caskets is not a matter that will be determined by chance

In fact the lottery of the casket is not a matter that will be determined by mere chance but that it is a true test of character and of sincerity which is amply proved not only by Moroccorsquos choice but also by the arguments which he uses to help him in his choice

(Act II Sc 7 L 55-59)

They have in England

A coin that bears the figure of an angelStamped in gold but thats insculpd uponBut here an angel in a golden bedLies all within

Context

(3)

(Act II Sc 7 L 63-77)A carrion Death within whose empty eye

There is a written scroll Ill read the writing

All that glisters is not goldOften have you heard that toldMany a man his life hath soldBut my outside to beholdGilded tombs do worms infoldHad you been as wise as boldYoung in limbs in judgment oldYour answer had not been inscrolld

This passage occurs in Act II Scene 7 in The Merchant of Venice This is part of the speech made by the Prince of Morocco

Explanation

In this passage the Prince of Morocco bestows high praise on Portia whose hand he is seeking He contrasts this casket containing Portiarsquos portrait with the old English gold coin bearing the image of the archangel (angel of the highest rank) He goes on to remark that while the figure of the archangel is engraved (Insculped) upon the English coin the picture of Portia who is beautiful as an angel lies hidden inside one of the caskets namely the Golden Casket (Golden Bed) In the day of Elizabeth silver was ten times inferior in value to gold Therefore the Prince of Morocco believing that Portiarsquos portrait is contained in the Golden Casket decides to choose the Golden Casket

Fare you well your suit is coldCold indeed and labour lostThen farewell heat and welcome frostmdashPortia adieu I have too grievd a heartTo take a tedious leave Thus losers part

(i) What reward does the Prince of Morocco get after making a wrong choice of the Casket How does he feel

After making the wrong choice in selecting the casket of gold the Prince of Morocco as a reward earns a rebuke in the form of a scroll tucked in the empty eye-socket of a skull kept in the casket of gold The Prince is shocked and disappointed He becomes all the more sad and dejected when he reads the scroll which points to his foolishness in being misled by the appearance and outward show as indicative of its worth

(ii) How does the Prince respond after reading the scroll

After reading the scroll the Prince though upset accepts the result with good grace and decorum befitting a royal suitor and true sportsman He says that his love-suit is really cold otherwise he would have chosen correctly but now his efforts have been in vain So he bids farewell to Portia to the warmth and enthusiasm of love and welcomes the cold and bitterness of dejection and misery of life which lies ahead

(iii) What request does he make to Portia and why

After being failure in his mission he requests Portia to give him permission to leave at once because he is too sad to undergo the tediousness of a formal leave-taking He tells that it is the manner in which defeated persons part unceremoniously

(iv) Explain the following lines

ldquoAll that glisters is not goldOften have you heard that toldMany a man his life hath soldBut my outside to beholdGilded tombs do worms infoldrdquo

Mere glitter does not make a metal to be gold Man has often been warned against appearance but it has been of no use Many people have sacrificed their lives only to seek the outer appearance of gold Worms are found inside the gilded

monuments

Class XSubject Topic Summary Execution

Hindi 2ndlang

नया रासता भाग 6 मायाराम 0ी घर म धनी मल 0ी और उनी बटी सरिरता ी ही चचा बनी रहती थी अमिमत ो इसम ोई रलिच ना थी वह धनी घर ी लडी स शादी र सवय ो बचना नही चाहता था उसा भी सवाणिभमान ह ईशवर ी पा

स उस पास पस ी ोई मी नही थी अभी उसन फकटरी ही लगाई थी उसी समझ बाहर था कि उस घर वालो ा झाव पस ी तरफ कयो

ह उसन मा स सवाल किया कि मा तम सरिरता स मरी शादी कयो रना चाहती हो मा न उस समझाया कि वह दखन म बरी नही ह और किफर खानदान अचछा

ह वह ए शल गरहणी रप म घर सभाल सगी अमिमत न मा ो इस बात ा एहसास राया कि मीन सबध लिलए मना रन पर उस दिदल

पर कया बीती होगी मा और अमिमत ी लडी बार म ाफी बात हईमा ा झाव सरिरता ी तरफ था कयोकि वह घर पर अचछा दह0 लर आ रही

थी अमिमत न अपनी मौसी ी बरी हालत बार म बताया कि किस तरह वह बड घर ी खानदानी बटी लाई थी और आ0 उसी हालत कितनी खराब ह लाई थी बहकलब 0ाती ह और बचचो ो भी नही दखती ह बात चल ही रही

थी कि तभी ए ार बाहर आर री धनी मल0ी घर अदर आए और पीछ स डराइवर फल ी ए टोरी लर आया अदर आए और पीछ स

डराइवर ए टोरी फल ी लर आया अमिमत ो फल ी पटी बरी लग रही थी अमिमत न पछ लिलया यह फल कयो ल आए ह प इन सब ी कया

0ररत थी उनो न 0वाब दिदया कि 4 पटी शमीर स मगाए थ अमिमत ो या सनर करोध आ गया तभी उस किपता 0ी आ गए उन आत ही अमिमत उठर बाहर चला गया वहा वहा मा पास आर बठ गया और बोला

अभी रिरशता तय नही हआ और धनी मल 0ी धनी मल 0ी फल ी पटी लर चलआय मा न समझाया कि 0ब सबध 0ड 0ाता ह तो खाली हाथ नही

आत अमिमत न मा स हा कि तम सबन सरिरता ो इस घर म लान ी ठान रखी ह धनीमल 0ी उस दिदन सरिरता ो दखन ी तारीख तय रन आय थ

Commercial Studies

Banking Nowadays Bank provide easy and quick services through internet facilities methods of Banking is called internet bankingIn order to save the time and money involved in visiting Bank branches people increasingly prefer to have internet banking

There are different modes of doing internet banking or transferring money through online They areReal Time Gross Settlement (RTGS)National Electronic Fund Transfers (NEFT)

1

Question

1) Explain the term RTGS Write the features of RTGS

Answer)The acronym RTGS stands for Real Time Gross Settlement which may be defined as the continuous real time settlement of funds transfer individually on and order by order basis without netting lsquoReal timersquo may be defined as the processing of instructions at the time they are received rather than at some letter time lsquoGross settlementrsquo may be defined as the settlement of transfer instructions which occurs

individually

Features of RTGS1It is the continuous settlement of

funds transfer individually on an order by order basis

2RTGS facility is provided only by CBS core banking solution enabled Bank branches

3Amount charged from the customer for RTGS transactions vary from bank to bank

2) Explain the term NEFT Write the features of NEFT

Answer) National electronic funds transfer may be defined as a nationwide system that facilitates individuals Farms and copper operates to electronically transfer funds from any bank branch to any individual farm or corporate having an account with any other bank branch in the country

Features of NEFT2 Transfer can be made 7 times on

weekdays and 6 times on Saturday

3 NEFT cannot be used to receive foreign remittances

4 NEFT transaction takes place in batches

5 A bank branch must be NEFT enabled to become a part of NEFT fund transfer network

6 There is no maximum or minimum amount that can be transferred through NEFT when one bank has a bank account

English Language

CompositionEssay

A composition is an art of creating a piece of writing on any topic or subject It is the writing correctly beautifully and clearly in order to make some interesting reading Structure of the composition

Introduction ( you lay the foundation for your composition)

Body (it constitutes the main part of the essay)

Conclusion (final statement that leaves a lasting impression)

Kinds of essays1 The Narrative essay2 The descriptive essay3 The reflective essay4 The argumentative essay

Write a composition on any one of the following topics (350- 400 words)

1 Friendship Or2 The first day of your school

Subject Eng Literature (The Merchant of Venice ndash William Shakespeare)Topic Act V Scene 1 Lines 127 to 158 (Nerissa helliphellip The clerk will nersquoer wear hair onrsquos face that had it) [Students should read the original play and also the paraphrase given in the school prescribed textbook]

Summary Revision Questions o Soon thereafter Bassanio Gratiano

and Antonio arrive

o Bassanio tells Portia that he is feeling as if it is morning because of the presence of Portia who is shining like the sun When Antonio is introduced by Bassanio to Portia she tells Bassanio that he should be grateful to Antonio who took so much trouble on his account even to the extent of risking his life

o Nerissa starts quarrelling with Gratiano and demands that he show her the ring she had presented to him and which she had warned him not to lose She suspects that Gratiano must have presented the ring to some young woman and not to the lawyerrsquos clerk as he repeatedly says and assures

Answer the following questions to check your preparation of Act IV Scenes 1 and 2

You must attempt only after you have completed your preparation of Act IV The answers must be in complete sentences using textual evidence (with citation) when necessary

[It would be in your own interest to attempt the above questions honestly totally refraining from consulting your textbook or your notes during answering After completion you should correct the paper yourself consulting the textbooknotes etc and award marks as specified Please let me know the marks you scored through WhatsApp in the group or to my personal WhatsApp]

Act IV Scene 1 (each question carries 2 marks)

1 What did the Duke try to do for Antonio

2 Why does Shylock refuse to show mercy How does he justify his stance

3 Why does Antonio say he is ready to die 4 What information is contained in Bellariorsquos letter

5 Why does Portia (as Balthazar) assert that Shylock must show mercy How does he respond

6 What offers are made to Shylock to get him to spare Antonio How are they received

7 What does Antoniorsquos speech as he faces the prospect of Shylockrsquos knife tell you about his character

8 How do Bassanio and Gratiano react to the looming prospect of Antoniorsquos demise

9 How does Portia (as Balthazar) use the law to turn the tables on Shylock

10 What does the Duke decree should happen to Shylock Why What happens to Shylockrsquos estate

11 What does Portia ask Bassanio as payment for her ldquoservicesrdquo What is his initial response What makes him change his mind

Act IV Scene 2 (each question carries 1frac12 marks)

1 What does Gratiano bring to Portia (Balthazar)

2 What does Nerissa plan on getting from Gratiano What does Portiarsquos comment suggest about men

ECO-10 280620 Topic-Supply AnalysisSHIFTING OF SUPPLY

But if there is change in factors other than the price of the commodity then either more is supplied at the same price or less supplied at the same price In such cases the price of the commodity remains constant but there is a change in other factors like change in the price of inputs change in technology of production change in price of other related goods change in taxation policy of the government etc For example there is an improvement in the technology of production of the commodity in question It leads to decrease in per unit of cost production of the commodity The firm is willing to sell more quantity of the commodity at the same price So the supply other commodity increases at the same price This increase in supply is shown by rightward shift of supply curve On the other hand if the firm uses inferior technology of production the cost of production per unit of the commodity increases The firm is willing to sell less quantity at the same price So the supply of the commodity decreases at the same price This decrease in supply is shown by leftward shift of the supply curve The above cases of increase and decrease in supply can be shown with the help of the following figures

Y INCREASE OF SUPPLY Price (Rs) s

P A s1

B

s

X` O s1 X

q q1

Y` Quantity demanded (in units)

Y DECREASE IN SUPPLY s2

s

price (Rs)

C

p A

s2

s

X` o X

q2 q

Y` Quantity demanded ( in units)

Main factors causing increase in supply or rightward shift of supply Curve(i) Fall in the price of other related goods

(ii) Fall in the price of inputsfactors(iii) Use of better technology in production(iv) Decrease in the rate of excise duty by government(v) If the objective of producer changes from profit maximization to salesMaximization

Main factors causing decrease in supply or leftward shift of supply curve(i) Increase in the price of other related goods(ii) Rise in the price of inputsfactors(iii) Use of inferior technology in production(iv) Increase in the rate of excise duty by the government(v) If the objective

Subject - Biology Topic ndash Chapter mdash6 PhotosynthesisSummary Execution

Today we will know about photosynthesis and its stages

Q1 What do you mean by photosynthesis The process by which living plants containing chlorophyll produce food

substances from carbon-di- oxide and water by using light energy Sunlight

6CO2 +12 H2O----------------------- C6 H12O6 + 6H2O + 6O2

Chlorophyll

Q2 What are the importance of photosynthesis I) Food for all Green plants trap solar energy by photosynthesis

process and supply food and energy for all living organisms either directly or indirectly

Ii) Oxygen to breathe in by product of photosynthesis is oxygen which is essential for all living organisms respiration

Q3 Write about two main phases of photosynthesis A Light dependent phase This phase occur in grana of chloroplast I) The chlorophyll on exposure to light energy becomes activated by

absorbing photons Ii) The absorbed energy is used in splitting the water molecules (H2O)

into its two components (H+ and OH- ) and releasing electron s 2H2O------------------------- 4H+ + 4e- +O2

Energy of 4 photons This reaction is known as photolysis

End products are H+ and oxygen water

B Light independent (Dark ) phase The reactions in this phase require no light energy

Here CO2 combine with H+ and produce glucose

Class XI

Subject Topic Summary ExecutionEVS Chapter-4 Legal

regimes for sustainable development

Environmental legislationEnvironmental legislation is the collection of laws and regulations pertaining to air quality water quality the wilderness endangered wildlife and other environmental factors The act ensures that matters important to the environment are thoroughly

Learn -The Forest (Conservation) Act 1980

considered in any decisions made by federal agencies

The Forest (Conservation) Act 1980 The Forest (Conservation) Act 1980 an Act of the Parliament of India to provide for the conservation of forests and for matters connected therewith or ancillary or incidental thereto It was further amended in 1988 This law extends to the whole of IndiaObjects and Reasons of the Forest Conservation Act

Deforestation causes ecological imbalance and leads to environmental deterioration Deforestation had been taking place on a large scale in the country and it had caused widespread concern The act seeks to check upon deforestation and de-reservation of forests

Subject Eng Literature (The Tempest ndash William Shakespeare) Topic Act II Scene 1 Lines 314 to 329 (End of scene)

[Students should read the original play and also the paraphrase given in the school prescribed textbook]Summary Questions amp Answers

Conspiracy of Antonio and Sebastian (Contd)

o As they approach Ariel appears again and wakes up Gonzalo by singing a tune in his ear Alonso also wakes up and they see both Sebastian and Antonio with drawn swords On being caught off guard they make up a story saying that they had heard a bellowing of bulls or lions

o They then moved to another part of the island

o Ariel at once rushes to Prospero to inform him of this development

SUMMING-UP of ACT-2 SCENE-1

(i) Among the survivors Ferdinand is separated from the rest which results in the disconsolate grief of Alonso as he took him for dead

(ii) The villainy of Antonio is confirmed

(iii) The supremacy of Prosperorsquos magic which resulted in the failure of the human conspiracy

(1)

(Act II Sc 1 L 311-325)SEBASTIAN Whiles we stood here securing your repose

Even now we heard a hollow burst of bellowing Like bulls or rather lions Didt not wake youIt struck mine ear most terribly

ALONSO I heard nothingANTONIO O rsquotwas a din to fright a monsters ear

To make an earthquake Sure it was the roarOf a whole herd of lions

ALONSO Heard you this GonzaloGONZALO Upon mine honour sir I heard a humming

And that a strange one too which did awake meI shaked you sir and cried As mine eyes opened I saw their weapons drawn There was a noiseThats verily rsquoTis best we stand upon our guardOr that we quit this place Lets draw our weapons

(i) Why has Prospero sent Ariel to Gonzalo and Alonso What does Ariel do to awaken Gonzalo

Prospero has already come to know by his magic powers the danger which threatens Gonzalo who had been Prosperorsquos friend and so he sent Ariel to preserve the lives of both Gonzalo and Alonso Prospero does not want that his scheme should remain unfulfilled Ariel begins to sing a song in Gonzalorsquos ears to awaken him(ii) Who are ready to carry out their plan Who takes steps to stop them Why does Gonzalo feel surprised after being awakened

Sebastian and Antonio are ready to carry out their plans They are standing with their swords drawn to kill Alonso and

(iv) We see two sets of contrasting characters Gonzalo-Adrian against Antonio-Sebastian

(v) The grief that works in Alonso can be perceived to his repentance for his association in Antoniorsquos crime against Prospero

Gonzalo Ariel takes steps to stop them from carrying out their nefarious scheme When Gonzalo is awakened by the song sung by Ariel into his ears he (Gonzalo) feels surprised because he sees Sebastian and Antonio standing with their swords drawn(iii) What reason do Sebastian and Antonio tell of drawing their swords when they are suspected by Alonso and Gonzalo

When Sebastian and Antonio are seen with their swords drawn they are looked with suspicion by Gonzalo and Alonso At first Sebastian tells them that as they stood here to guard them during their sleep they heard only a little before a sudden loud noise very much like the roaring of bulls or more probably that of lions Then Antonio follows him saying that this was a noise so terrible as to frighten even a monsterrsquos ears and this noise could even have shaken the earth and it was surely like the roaring of a multitude of lions Then seeing the danger they have drawn their swords Perhaps after hearing the terrible noise they (Gonzalo and Alonso) woke up from their sound sleep

(iv) What does Gonzalo tell Alonso about the strange noise What did he see on opening his eyes Gonzalo tells Alonso that he did not hear the sound of roaring but he heard a humming sound which was strange and which woke him up After waking up he gave him (Alonso) a shaking and a loud cry On opening his eyes he saw these two gentlemen standing with their swords drawn(v) What does Gonzalo suggest

Gonzalo suggests that there was a noise indeed and of that he has no doubt at all and suggests that the best course for them would be to remain alert and vigilant against any possible danger to their lives or to leave this place and move to some other part of the island

Class XIISubject Topic Summary Execution

Commerce

Chapter- Management

Today we will discuss about LEVELS OF MANAGEMENT

Levels of management is a series or chain of managerial positions from top to bottom It helps individuals to know their authority responsibilities and superior-subordinate relations among themselves There are mainly three levels of Management TOP LEVEL MANAGEMENTMIDDLE LEVEL MANAGEMENTLOWER LEVEL MANAGEMENT

Top level managementIt consists of members at the highest level in the management hierarchy This level includes Board Of Directors Chief Executive Managing Directors Chairman President Vice President

Rolefunctions of the top levelmanagement1To analyse evaluate and deal

with theexternal environment2 To determine the objectives and

policies of the business3 To strive for welfare and survival

of business

4 To create an organisational Framework consisting of authority responsibility relationship

Middle level management Congress of members or groups who are concerned with implementation of the policies let down by the top managementThis level includes head of the department such as finance manager marketing manager branch and regional managers departmental and divisional heads plant superintendent etc

Role of functions of the middle level management

1 To interpret the policies framed by top management

2 To assign duties and responsibilities to lower level managers

3 To select and appoint employees for middle and supervisory level and evaluate their performance

4 To co-operate with other departments for smooth functioning

Operational or supervisory level managementIt refers to the group are members who are concerned with execution of the work They are also known as fast line managers This level includes supervisor 4 men Section Officer clerk Inspector etc

Role of functions of the lower level management1 To plan and execute day-to-

day operations2 To supervise and control the workers3 To arrange materials and

tools to start the process and make arrangements for training

4 Today present workers grievance and suggestions before the management and

ensure safe and proper working conditions in the factory

Business Studies

Staff Appraisal Chapter- 10 Today let us start with a new chapter

Staff Appraisal

Meaning of Performance Appraisal

Performance Appraisal is the systematic evaluation of the performance of employees and to understand the abilities of a person for further growth and developmentThe supervisors measure the pay of employees and compare it with targets and plansThe supervisor analyses the factors behind work performances of employeesThe employers are in position to guide the employees for a better performance

Objectives of Performance Appraisal

Following are the objectives of Performance Appraisal

To maintain records in order to determine compensation packages wage structure salaries raises etc

To identify the strengths and weaknesses of employees to place right men on right job

To maintain and assess the potential present in a person for further growth and development

To provide a feedback to employees regarding their performance and related status

To provide a feedback to employees regarding their performance and related status

Importance of Performance Appraisal

Performance appraisal provides important and useful information for the assessment of employees skill

knowledge ability and overall job performance The following are the points which indicate the importance of performance appraisal in an organization

1 Performance appraisal helps supervisors to assess the work performance of their subordinates

2 Performance appraisal helps to assess the training and development needs of employees

3 Performance appraisal provides grounds for employees to correct their mistakes and it also provides proper guidance and criticism for employees development4 Performance appraisal provides reward for better performance

5 Performance appraisal helps to improve the communication system of the organization

6 Performance appraisal evaluates whether human resource programs being implemented in the organization have been effective

7 Performance appraisal helps to prepare pay structure for each employee working in the organization

8 Performance appraisal helps to review the potentiality of employees so that their future capability is anticipated

Geography

DRIANAGE The SubarnarekhaThe Subarnarekha and the Brahmaniinterposed between the Ganga and the Mahanadi deltas drain an area of 19300 sq kmand 39033 sq km respectively The drainage basins of these streams are shared byJharkhand Odisha west Bengal and Chhattisgarh The Brahmani is known as southKoel in its upper reaches in Jharkhand

The NarmadaThe Narmada rises in the Amarkantak hills of MadhyaPradesh It flows towards the West in a rift valleyformed due to a geological fault The total length of it is 1300 km All the tributaries of the

Q1 Name the two westward flowing rivers in the peninsular plateauA1 Narmada and Tapi are the only westward flowing rivers of the peninsular plateau

Q2 Differentiate between east-flowing rivers and west-flowing riversA2

East-flowing rivers

West-flowing rivers

Narmada are very short inlength Most of its tributaries join the main streamright anglesThe Narmada basin covers parts of Madhya Pradesh and Gujarat

The Tapi The Tapi rises in the Satpura ranges in the Betul listrictof Madhya Pradesh It flows in a rift valley parallel tothe Narmada but it is much shorter in length It coversparts of Madhya Pradesh Gujarat and MaharashtraThe length is about 724 km

The Sabarmati and the MahiThe Sabarmati rises in the Aravali hills and flows south-south-westwards for a distance of 300 kilometres to the Arabian Sea The Sabarmatibasin extends over an area of 21674 sq km in Rajasthan and Gujarat The Mahi rises inthe east of Udaipur and drains an area of 34842 sq km lying in Madhya PradeshRajasthan and Gujarat It flows south-westwards for a distance of 533 km before it fallsinto the Gulf of Khambhat

The ChambalThe Chambal rises near Mhow in the Vindhya Range and flows towards the northgenerally in a gorge upto Kota Below Kota it turns to the north-east direction and afterreaching Pinahat it turns to the east and runs nearly parallel to the Yamuna beforejoining it in the southern part of the Etawah district in Uttar PradeshMajor Rivers of India with their basin area (Sqkm)

Himalayan System Indus 321290Ganga 861404

Brahmaputra 187110Indus System

Jhelum 34775Beas 20303

Ganga System Yamuna 366223Ghaghra 127950

Peninsular RiversNarmada 98796

Tapi 65145Mahanadi 141600

Subarnarekha 19300Sabarmati 21674

Mahi 34842Godavari 312812

Godavari Krishna Kaveri Mahanadi are the east-flowing rivers

Narmada Tapi west-flowing rivers

They fall into the Bay of Bengal

They fall into Arabian Sea

These rivers form big deltas

These rivers form comparativelysmall deltas

Catchment areas of these rivers are larger

Catchment areas of these rivers are smaller

Krishna 2589488Cauveri 87900

Subject ndashBiology Topic ndashChapter -5 Inheritance amp Variations Summary ExecutionToday we will discussabout linkage and its classification

LINKAGE The tendency of the genes located on the same chromosome to stay together is

hereditary transmission Linked genes the genes responsible for this Genes that exhibit the process of linkage locates in the same chromosome The distance between the linked genes in a chromosome determines the strength

of linkage i e genes that are located close to each other show stronger linkage than that are located far from each other

COMPLETE LINKAGE It is the type of linkage showed by the genes that are closely located or are tightly

linked with each other as they have no chance of separatingby crossing over These genes are always transmitted together to the same gamete and the same

offspring In such condition only parental or non cross over type of gametes are formedINCOMPLETE KINKAGE It is type of linkage showed by the genes that are distantly located orare loosely

linked with each other because they have chance of separating by crossing over

SIGNIFICANCE i) It helps in holding the parental character togetherii) It checks the appearance of new recombination and helps in bringing the

hybrid population which resembles the original parents iii) Linked genes dilute the effects of undesirable traits

Subject Eng Literature (The Tempest ndash William Shakespeare) Topic Essay Questions (EQ-3)Question No 3

Give a character sketch of CalibanAnswer

The character of Caliban has been wonderfully conceived by Shakespeare as the manifestation of all that is gross and earthy ndash a sort of creature of the earth as Ariel is a sort of creature of the air

Calibanrsquos Physical Appearanceo Caliban is lsquofreckledrsquo a lsquomisshapen knaversquo not honoured with human shape

o Prospero calls him lsquothou tortoisersquo (Act I Sc 2 Line 317) Trinculo stumbling upon him describes him as ldquoA strange fish hellip Legged like a man And his fins like armsrdquo He ldquosmells like a fishrdquo (Act II Sc 2 Line 25)

o Prospero also calls him a ldquobeastrdquo (Act IV Sc 1 Line 140) and ldquoThis misshapen knaverdquo (Act V Sc 1 Line 268)

o Further it appears that in addition to his physical deformity his spiritual inferiority is also suggested by Prosperorsquos claim that his birth resulted from the union between his mother the witch Sycorax and the devil

Calibanrsquos ParentageWhen the play opens Caliban is twenty four years of age having been born on the island twelve years before the coming of Prospero His mother was the foul witch Sycorax who was banished from Algiers for ldquomischiefs manifold and sorceries terrible to enter human hearingrdquo (Act I Sc 2 Line 264) and the father was the Devil himself Thus

Caliban is a monster of evil and brute nature ugly deformed and stinking

Calibanrsquos Savage and Malignant Natureo Caliban is entirely a creature of the earth ndash gross brutal and savage He regards himself as the rightful possessor

of the island and Prospero as a usurper

o In his young age he was on good terms with Prospero He had consented to be received by Prospero at his house and to be educated by him He has learnt human language only to curse his master whom he abhors

o His beastly nature soon breaks out and ends in a vicious attack on Miranda This opens the eye of Prospero who becomes severe to him and enforces his service by threats and violence

o Prospero uses him to make dams for fish to fetch firewood scraper trenches wash dishes and keep his cell clean

Calibanrsquos Hatred for ProsperoA profound hatred for Prospero has taken hold of Caliban It springs from a sense of his being dispossessed and ill-treated He would kill Prospero if he could but he knows the power of Prosperorsquos lsquobookrsquo Hence he transfers his allegiance to Stephano who seems like a god to him He also incites the two drunken associates to batter the skull of Prospero when he sleeps in the afternoon

Caliban Shows Considerable Intelligenceo He has learnt Prosperorsquos language

ldquoYou taught me language and my profit onrsquot (Act II Sc 2 Lines 86-89)Is I know how to curserdquo

o He is well aware of the futility of arguing with one who has more power than he has

ldquoI must obey his art is such power (Act I Sc 2 Lines 373-376)It would control my damrsquos god SetebosAnd make a vassal of himrdquo

o He realizes the importance of Prosperorsquos books

ldquoRemember (Act III Sc 2 Lines 89-92)First to possess his books for without themHersquos but a sot as I am nor hath notOne spirit to commandrdquo

o He knows the value of stealth when attacking the enemy

ldquoPray you tread softly that the blind mole may not (Act IV Sc 1 Lines 194-195)Hear a foot fall we now are near his cellrdquo

o Caliban has a better set of values than Stephano and Trinculo They are distracted from their plan by their greed for Prosperorsquos rich garments Only Caliban realizes that such a finery is unimportant

ldquoLeave it alone thou fool it is but trashrdquo (Act IV Sc 1 Lines 224)

Caliban is not a good judge of characterCaliban is not a good judge of character He decides for example that Stephano is a god because he dispenses lsquocelestial liquorrsquo (Act II Sc 2 Line 115) but then it must be remembered that he has only known his mother Sycorax Prospero Miranda and the spirits that torture him However he quickly discovers his error of judgementrdquo

ldquoWhat a thrice-double ass (Act V Sc 1 Lines 295-297)Was I to take this drunkard for a godAnd worship this dull foolrdquo

Calibanrsquos Imaginative NatureIf Caliban is sub-human in what has been said above he is human in the respect of the poetic side of his character He listens to music with rapture He tells of the beautiful dreams in which heaven rains treasures upon him and which upon waking he yearns to renew One of the most poetic passages in whole play is Calibanrsquos description of the island

to Stephano and Trinculo

ldquoBe not afeard The isle is full of noises (Act III Sc 2 Lines 135-143)Sounds and sweet airs that give delight and hurt notSometimes a thousand twangling instrumentsWill hum about mine ears and sometime voicesThat if I then had waked after long sleepWill make me sleep again and then in dreamingThe clouds methought would open and show richesReady to drop upon me that when I wakedI cried to dream againrdquo

Caliban - Less Ignoble Than Some OthersCalibanrsquos motive for murder is less dishonourable than that of Antonio and Sebastian They plan to kill Alonso to gain his power and wealth Caliban merely wants revenge and the return of lsquohisrsquo island

Conclusiono Calibanrsquos character is not portrayed very clearly in the play and hence we cannot decide whether he is a poor

savage being grossly maltreated by Prospero or whether he is evil and must therefore be kept in bondage or enslavement

o Caliban is contrasted with Ariel who is a spirit and thus swift and uninterested in physical activitieso Caliban is also contrasted with Prospero who is the all-powerful master of the island and of the destiny of all

those on the islando Caliban is also contrasted with civilized man showing him to be less evil than Antonio and Stephano and less

materialistic than Stephano and Trinculoo Caliban has suffered at the hands of Prospero and he has learnt to curse by listening to Prosperorsquos abuse He

certainly believes that Prospero has deprived him of his birthrighto Finally the character Caliban is thought to be one of Shakespearersquos masterpieces The complexity of the character

is reflected in the large volume of critical discussion that has grown around it

ECO ndash12 Topic-Forms of market

MonopolyMonopoly is a market structure in which there is a single seller there are no close substitutes for the commodity produced by the firm and there are barriers to entry Example Indian Railways which is operated under government of India Monopoly also implies absence of competitionFeatures of Monopoly Monopoly is characterized by1 Single Seller In monopoly there is only one firm producing the product The whole industry consists of this single firm Thus under monopoly there is no distinction between firm and industry Being the only firm there is significant control of the firm over supply and price Thus under monopoly buyers do not have the option of buying the commodity from any other seller They have to buy the product from the firm or they can go without the commodity This fact gives immense control to the monopolist over the market

2No Close Substitute There are no close substitutes of the product produced by the monopolist firm If there are close substitutes of the product in the market it implies presence of more than one firm and hence no monopoly In order to ensure a total of control over the market by the monopolist firm it is assumed that there are no close substitutes of the product

3 No Entry amp Exit Monopoly can only exist when there is strong barriers before a new firm to enter the market In fact once a monopoly firm starts producing the product no other firm can produce the same One reason for this is the ability of the

monopolist to produce the product at a lower cost than any new firm who thinks to enter the market If a new firm who knows that it cannot produce at a lower cost than the monopolist then that firm will never enter the market for fear of losing out in competition Similarly the monopolist who is operating for a long time may be enjoying reputation among its customers and is in a better position to use the situation in its own benefit A new firm has to take long time to achieve this and so may not be interested to enter the market

4 Price Maker Being the single seller of the product the monopolist has full control over the pricing of the product On the other hand if there is a large number of buyers in the market so no single buyer exercises any significant influence over price determination Thus it is a sellerrsquos market So monopoly firm is a price maker

5 Price Discrimination Having considerable control over the market on account of being single seller with no entry of other firms the monopolist can exercise policy of price discrimination it means that the monopolist can sell different quantities of the same product to a consumer at different price or same quantity to different consumers at different prices by adjudging the standard of living of the consumer

6 Shape of Demand Curve Since a monopolist has full control over the price therefore he can sell more by lowering the price This makes the demand curve downward sloping

Subject Ac-12 290620 Topic- retirement Model sumThe Balance Sheet of Rohit Nisha and Sunil who are partners in a firm sharing profits according to their capitals as on 31st March 2014 was as under

Liabilities Amount Assets Amount (Rs) (` Rs)

Creditors 25000 Machinery 40000Bills Payable 13000 Building 90000General Reserve 22000 Debtors 30000Capital Less Provision for Rohit 60000 Bad debts 1000

29000 Nisha 40000 Stocks 23000 Sunil 40000 140000 Cash at Bank 18000

200000 200000

On the date of Balance Sheet Nisha retired from the firm and following adjustments were made(i) Building is appreciated by 20(ii) Provision for bad debts is increased to 5 on Debtors(iii) Machinery is depreciated by 10(iv) Goodwill of the firm is valued at Rs 56000 and the retiring partnerrsquos share is adjusted

(v) The capital of the new firm is fixed at Rs120000 Prepare Revaluation Account Capital Accounts of the partner and Balance Sheet of the new firm after Nisharsquos retirement Revaluation AccountDr Cr

Particulars Amount Particulars Amount (`Rs) (Rs`)

Provision for Bad debt Ac 500 Building Ac 18000Machinery Ac 4000Profit transferred toCapital Accounts (3 2 2)Rohit 5786Nisha 3857Sunil 3857

13500

18000 18000

Capital Account

Dr Cr

Particulars Rohit Nisha Sunil Particulars Rohit Nisha Sunil (Rs`) (Rs`) (`Rs) (Rs`) (Rs`) (Rs`)

Sunilrsquos Capital ac 9600 mdash 6400 Balance bd 60000 40000 40000Bank - 66143 - General Reserve 9428 6286 6286Balance cd 72000 mdash 48000 Revaluation (Profi 5786 3857 3857 Rohitrsquos Capital Ac mdash 9600 mdash

Sunilrsquos Capital Ac 6400 Bank 6386 - 4257

81600 66143 54400 81600 66143 54400

Balance Sheet as at 31st March 2014

Liabilities Amount Assets Amount (Rs`) (Rs`)

Creditors 25000 Building 108000Bank overdraft 37500 Machinery 36000

Bills Payable 13000 Debtors 30000Capital Less ProvisionRohit 72000 for Bad debts 1500 28500Sunil 48000 120000 Stock 23000

195500 195500

Working Notes (i) (a) Profit sharing ratio is 60000 40000 40000 ie = 3 2 2(b) Gaining Ratio Rohit = 35 ndash 37 = 2135 ndash 1535 = 635Sunil = 25-27 = 1435 ndash 1035 = 435= 635 435= 6 4 = 3 2(c) Nisha Share of Goodwill = Rs 56000 times 27 = Rs16000Share of Goodwill in the gaining ratio by the existing partner ieRohit = Rs16000 times 35 = Rs 9600Sunil = Rs 16000 times 25 = Rs 6400

The journal entry isRohitrsquos Capital Ac Dr 9600Sunilrsquos Capital Ac Dr 6400 To Nisharsquos Capital Ac 16000(Share of Goodwill divided into gaining ratio)

  • 1 Static Friction
  • The frictional force that acts between the surfaces when they are at rest with respect to each other is called Static Friction
    • Static Friction Examples
      • 2 Sliding Friction
        • Examples Of Sliding Friction
          • 3 Rolling Friction
            • Examples Of Rolling Friction
              • Objects and Reasons of the Forest Conservation Act
Page 21:  · Web viewSubject . Topic . Summary . Execution . English 1 . Sounds of animals . Hens –cackle Horses –neigh Lions –roar Owls –hoots Snake –hiss. English 2 . Mother’s

English Language

Prepositions A preposition is a word placed before a noun or a pronoun It helps to show how the person or thing denoted by the noun is related to something else in the sentence

Kinds of Prepositions

Simple Prepositions- simple preposition are one word Prepositions such as at by for in of off for from on out through till to up with before amidst towards beyond between over etc

Compound Prepositions ndash There are some words that are always used with fixed Prepositions to convey specific meaning

Example I was unable to meet you dueto a previous engagement ( On account of)Always maintain the queue instead of crowding at the counter ( In place of)

Participial PrepositionsmdashParticiple Prepositions are present or past participles of various verbs which together with a noun phrase or a clause function as prepositions Examples- barring concerning considering notwithstanding pending regarding respecting etc

Exercise A

1 Gauravs fever has come down since Friday He has been absent for a week now

2 The child sat between his father and mother among the parents of all his classmates

3 There are mosquitoes in the room They flew into the room when the door was open

4 My father was inside the drawing room when I was playing outside my house

5 You may sit beside me I will give you a drawing book and pencils besides a storybook

6 We went to the market in the morning and walked towards the riverfront in the evening

7 The child walked along the pavement and across the street safely

8 This table top is made of glass My breakfast fell off it in the morning

9 The pan is on the gas stove There are vegetables in it

10 We will wait for you at the bus top There are a lot of people in the hall

Subject ndash Biology Topic ndash Chapter - 3 Photosynthesis and respiration in plants Summary Execution

All living organism (Plants and animals) need food for energy and growth Green plants (autotrophy) prepare food for all living organisms Today we will discuss about the process photosynthesis And adaptations in a leaf to carry out photosynthesis

Q1What do you mean by photosynthesis and write its word equation The process by which green plants make food (glucose) from carbon dioxide and water

in the presence of sunlight and chlorophyll is called photosynthesis

Carbon dioxide + Water ( Sun light from Sun ) Glucose + Oxygen ( chlorophyll in green leaves )

Q2 What are the adaptations in a leaf to carry out photosynthesisi) Leaves are broad wide and flat for absorbing more light energyii) Presence of chlorophyll in chloroplasts to trap sunlightiii) Presence of stomata which allow carbon dioxide to enter the cell and oxygen to go

out iv) Network of veins ensures continuous supply of water and minerals to the leafv) Thin waxy cuticle protects the leaf without blocking the lightQ3 Draw and label structure of chloroplast

Class VIIISubject Topic Summary Execution

PHYSICS ENERGY Production of Hydro electricity

A hydroelectric dam converts the potential energy stored in a water reservoir behind a dam to mechanical energymdashmechanical energy is also known as kinetic energy As the water flows down through the dam its kinetic energy is used to turn a turbine

The generator converts the turbinersquos mechanical energy into electricity

This electric energy then goes through various transmission processes before it reaches you

Question 2

Fill in the blanks

(a) Work is said to be done by a forte only when the body moves

(b) Work done = Force x distance moved in direction of force

(c) The energy of a body is its capacity to do work

(d) The SI unit of energy is joule

(e) The potential energy is due to its state rest of position and kinetic energy of the body is due to its state of motion

(f) Gravitational potential energy U = mass times force of gravity on unit mass times height

(g) Kinetic energy = frac12 times mass times (speed)2

(h) Power P = work donetime taken

(i) The S I unit of power is watt

(j) IHP = 746 W

BIOLOGY Chapter -5 The endocrine system and adolescence

Today we will discuss about thelocation and functions of secreted hormones of adrenal and Pancreas

Q5 Write location hormone secreted main functions and deficiency diseases of pancreas and adrenal glands

Endocrine Glands

Location Hormones secreted

Functions and Deficiency Diseases

1Adrenal gland

2 Pancreas Gland

On the top of each kidney

In between stomach and small intestine

i)Adrenaline from adrenal medulla

ii)Cortisone from adrenal cortex

i) Insulin

ii) Glucagon

It helps a person deal with any kind of emergency situation or emotional stressIt increases the heart beat rate of respiration and blood pressure

a) It regulates carbohydrates protein and fat metabolism

b) It regulates the salt and water balance in the body

a) It changes excess glucose into glycogen

b) It stimulates the cells to burn extra glucose to provide heat amp energy

Less secretion causes diabetes mellitus

Excessive secretions causeinsulin shock

a) It stimulates the breakdown of glycogen into glucose

b) It increases the level of glucose in blood

History Traders to rulers The Battle of Buxar was fought on 22 October 1764 between the forces under the command of the British East India Company led by Hector Munro and the combined armies of Mir Qasim the Nawab of Bengal till 1763 Mir Jafar was made the Nawab of Bengal for a second time in 1763 by the Company just after the battle After being defeated in 4 battles in katwa and Udaynala the Nawab of Awadh Siraj id Daula and the Mughal emperor Shah Alam II accompanied by Raja Balwant Singh of Kashi made an alliance with Mir Qasim The battle was fought at Buxar a small fortified

Answer the following questions- Short note-Battle of BuxarHomework-learn

town within the territory of Bihar located on the banks of the Ganga river about 130 kilometres (81 mi) west of Patna it was a decisive victory for the British East India Company The war was brought to an end by the Treaty of Allahabad in 1765

EnglishLiterature

The west wind-John Mansfield

In the poem The West Wind by John Masefield the poet starts by describingwith very poetic imagery of birds how the west wind is different from other winds its a warm wind full of birds cries There is a touch of melancholy perhaps home-sickness as he describes how it brings tears too and memories from an old land He goes on to describe the restful pastoral beauty of the land where even the dead can lie in the green He then brings in voicesperhaps of family and friends calling him home as he is missing Aprils beautyThe voices then tempt him some more with idyllic images from home (white blossom young green cornrunning rabbitswarm sun) The voices seem to presume that the poets heart is sorrowful bruised and soreThe end of the poem sees the poet appear to make a decision he will go home as he has decided that is where he truly belongs

Write the synopsis of the following words

1 Daffodils- a tall yellow flower that grows in the spring

2 Orchards- a piece of land on which fruit trees are grown

3 Blossom- a flower or a mass of flowers especially on a fruit tree in spring

4 Thrushes- a bird5 Larks- a small brown bird that

makes a pleasant sound6 Bruised- an injury7 Aching- pain 8 Tread- to put your foot down

while you are walking9 Balm-10 May-11 Fluting-

(Write from the book in your copy)

MAT

HEM

ATIC

S

Ch 1

1Al

gebr

ic E

xpre

ssio

n

1 Constant A symbol which has fixed value is called a constant[eg 8 23 -15 radic3 etc]

2 VariableA symbol which does not have any fixed value but may be assigned value (values) according to the requirement is called variable or literal[eg x y p q etc]

3 TermsA term is a number (constant) a variable a combination (product or quotient) of numbers and variables[eg 7 x 5x etc]

4 Algebric expressionA single term or acombination of two or more terms connected by plus (+) or minus (-) sign forms an algebraic expression[eg 5-y 3x2-5x xy-6z+4 etc]

5 PolynomialAn algebraic expression which contains more than one term is called a polynomial (multinomial)[eg x2-5x 5y+xy+x2y etc]

6 Degree of polynomial(a) When the polynomial contains only one variable the highest power of the variable is the degree of the polynomialeg the degree of the polynomial of 4x-7x5+8 is 5(b) When the polynomial contains two or more variablesStep (i) Find the powers of the variables in each term (ii) The highest sum of the powers is taken to be the degree of the polynomialeg the degree of the polynomial 5x2y-4x3y5+6 is = 3+5 = 8Remember An algebraic expression is a polynomial if degree of each term used in it is a non-negative integer

Exercise ndash 11(A)

1 Separate the constants and variables from the following

-7 7+x 7x+yz radic5 radic xy 3 yz

8 45y -3x

Solution Constant Variables-7 radic5 7+x 7x+yz radic xy

3 yz8

45y -3x

2 Write the number of terms in each of the following polynomials(i) 5x2+3timesax (ii) axdivide4-7 (iii) ax-by+ytimesz (iv) 23+atimesbdivide2

Solution Polynomials Number of terms(i) 5x2+3timesax 2(ii) axdivide4-7 2(iii) ax-by+ytimesz 3(iv) 23+atimesbdivide2 2

4 Write the degree of the each polynomials(i) xy+7z (ii) x2-6x3+8 (iii) y-6y2+5y8 (iv) xyz-3 (vi) x5y7-8x3y8+10x4y4z4

Solution Polynomials Degree(i) xy+7z 2(ii) x2-6x3+8 3(iii) y-6y2+5y8 8(iv) xyz-3 3(vi)x5y7-8x3y8+10x4y4z4 12

5Write the coefficient of(i) ab in 7abx (iv) 8 in a2-8ax+a (v) 4xy in x2-4xy+y2

SolutionCoefficient

(i) ab in 7abx 7x(iv) 8 in a2-8ax+a -ax(v) 4xy in x2-4xy+y2 -1

7 CoefficientAny factor of an algebraic quantity is called the coefficient of the remaining quantityeg in the algebraic term 7xyz 7 is coefficient of xyz 7x is coefficient of yz and so on

8 Like term The terms having the same literal coefficient are called like terms and those having different literal coefficients are called unlike terms

eg (i) 5xyz 8xyz -6xyz and 23xyz are like

terms(ii) 7xy2 8x2yz and -15xyz2 are unlike terms

6 in 57xy2z3 write the coefficient of

(i) 5 (vii) 5xy2 (viii) 17yz (xi) 5xyz

Solution Coefficient

(i) 5 17

xy2z3

(vii) 5xy2 17z3

(viii) 17yz

5xyzsup2

(xi) 5xyz 17yz2

7 In polynomial given below separate the like terms(ii) y2z3 xy2z3 -58x2yz -4y2z3 -8xz3y2 3x2yz and 2z3y2

Solution y2z3 -4y2z3 2z3y2 are like terms

xy2z3 -8xz3y2 are like terms

-58x2yz 3x2yz are like terms

Class IXSubject Topic Summary Execution

Bengali (2nd language)

বাগzwnjধারাzwnj বা ধারা-বা ধারা ল হিবকেশষ পরকার বাক -হিb -াকেবর এক হিবকেশষ পরকাশরীহিত াকেক কতগকেলা কার সমষটির মকেধয এগহিলকেক বা ধারা বকেল আবার কতগকেলা শকেবদর বাধাধরা যকান রীহিত যনই য-াকেব চকেল আসকে যসই -াকেবই চকেল আসকে তখন যসই শবদগহিল খন একক -াকেব অG পরকাশ ককের তখন একের বা ধারা বকেল বা ধারার পরকেয়া -াষাকেক আরও সFর ককের যতাকেল

অকাল পকক(অপহিরনত বয়কেস পাকাহিম)-মাতর শ বর বয়কেস যমকেয়টির া মকেখর কা তাকেত অকালপককতা ধরা পকে

অককা পাওয়া( মারা াওয়া) ndash পকেকIমারটি পকেকIমারকেত হিকেয় বাসাতরীকের াকেত মার যখকেত যখকেত অককা যপল

অহি| পরীকষা ( কঠিন ও পরকত পরীকষা)- যকেলটির আজ ডাকতাহির যরজালট যবকেরাকেব এIাই তার জীবকেনর ব অহি| পরীকষা

অষটরমভা (ফাহিক) ndash রীতা মকেখই বকো বকো কা বকেল আর কাকেজর যবলায় অষটরমভা

অকমGার ধাী (অপাG) ndash সমনকেক হিনকেয় যকান ান কেব না ও একেকবাকেরই অকমGার ধাী

অকেনধর ষটি (অসাকেয়র সায়)- আহিশ বকেরর বকোর নাহিত ল অকেনধর ষটি তাকেক াা বকোর একম চকেল না

আকেককল গড়ম (তবহিদধ)- ার তহিম উপকার করকেল যসই যতামার হিবরকেদধ সাকষয হিকেয়কে শকেনই আমার আকেককল গড়ম

আষাকে লপ( অবাসতব লপ) ndashIাকা এখন যকেব না এIা বলকেলই ত এমন আষাকে লপ ফাার যকান রকার হিল না

Hindi- महायजञ ा इस हानी म लख न या बतान ा परयास किया ह कि किसी भी अचछ

2nd language

परसार(यशपाल ाय या पणय न ा फल अवशय मिमलता ह ोई भी परोपार अथवा पणय लिलए किया गया ाय बार नही 0ाता वह ए परार ा यजञ हए धनी सठ थ धम परायण और किवनमर सठ न आन ी यजञ किए थ और दान म न 0ान कितना धन दिदन दखिखयो म बात दिदया थादिदन पलट और सठ यहा गरीबी आ गई उन दिदनो यजञ बचन ी परथा थी सठ भी अपनी 0गह बचन लिलए डलपर ए सट यहा चलन ो तयार हए सठानी रासत लिलए रोटी पड म बाधर सठ ो द दी रासत म ए भख R ो दखर सठ न चारो रोटी उसो खिखला दी खर वह सठ यहा डलपर पहच तो उनी सठानी न उस महायजञ बचन ो हा यदिद बचन आए सठ न R ो रोटी खिखलान ो महायजञ नही समझा और वापस लौट आया घर आर शाम ो उसी घर म उस ए बडा ख0ाना मिमला 0ो उस दवारा किए गएrsquo महायजञrsquo ा परसार था

English language

Letter formal The heading the name and address of the person you are writing to must be included beneath your own address In formal letters ldquoblock stylerdquo of address is preferred

Subject complain in brief

Salutation If the person you are writing to is known to you you may begin ldquoDear MrrdquoOr ldquoDear Mrsrdquo In all other instances you should begin ldquoDear Sirrdquo or ldquoDear Madamrdquo Or ldquoSirsrdquo

The body A formal or business letter has four partsReference The letter should begin by referring to a letter you have received an advertisement or the reason that has prompted you to writeInformation In the second paragraph it is necessary to supply more detailed information that is related to the referencePurpose Here you must give the reason why you are writing the letter This must be stated clearly and ensure that it is relevant to the question that has been setConclusion round off the letter with some polite remarkThe subscription when a letter has begun with dear sir sirs Madam you should end with Yours faithfully or yours truly When however you address a person by name you must conclude with the words ldquoYours sincerelyrdquo

1 A park in your locality is slowly being used as a rubbish dump Write a letter to the Mayor of your city pointing out the nuisance and danger of this Request that action be taken to stop this immediately

Or2 You being a boarder ordered a set of lab manuals from a famous book shop in the town They sent you a wrong set of books Write a letter to the manager of the book shop

Chemistry Chapter-1 1)CHEMICAL FORMULA- Q What is the Significance of

L-2The Language of Chemistrybull Chemical Formula

Itrsquos a symbolic representation of a chemical substance eg ndash The formula of Sulphuric acid is H2SO4

2) Steps of writing Chemical Formula of a given substance-

1 Write the symbols of the constituent atoms or radicals side by side Keep the basic radical on LHS and acid radical on the RHS ( Na+Cl- )2 In case of a radical having more than one atom( compound radical) enclose the radical in a bracket eg (SO4-)3 Write the valencies of each radical on its right hand top4 If the valencies of the two radicals are divisible by a common factor then divide the valencies by the common factor5 Invert (criss-cross) the valency number ie write the valency of one atom below the second atom and vice versa 6 On interchanging if valency number is lsquoone the figure lsquoonersquo is never writtenFor Example- Compound -Calcium Nitrate1 Writing the symbols- Ca(NO3)2 Writing the valencies on their right hand top- Ca2(NO3)1

3 Valency numeral in simple ratio- Ca2(NO3)1

4 Criss-cross- Ca 2NO3 1

5 Writing the formula of the compound- Ca(NO3)2

Chemical formula

A The formula of a substance conveys the following information regarding a substance 1 The name of the substance (qualitative)2 The elements constituting the substance (qualitative)3 The number of various atoms present in a molecule of the substance (quantitative)4 Molecular weight of the substance and the relative weights of different elements present in it (qualitative)

Q What are the limitations of Chemical Formula

A The chemical formula suffers from the following limitations-I It fails to convey whether the elements in a molecule are present in the form of atoms or ionsFor example the formula KBr fails to tell us whether Potassium and Bromine are present in the form of ions II It does not tell anything about the binding force that holds atom in a molecule togetherIII It does not tell us about the arrangement of various atoms with respect to one another within the molecule

Q Examples of Some Chemicals with their Formula Chemical name and Common Name-

A Given in the class notesCommercial Studies

Joint Stock Company

Let us discuss about the demerits of Joint Stock CompanyDespite so many advantages it has got many disadvantages which are as follows

Difficulty in FormationDelay in Decision makingExcessive Government ControlLack of Secrecy

Company can be classified into several categories based on incorporation

QuestionExplain the demerits of Joint Stock CompanyAnswer) 1 Difficulty in Formation The legal requirements and formalities required to be completed are so many The cost involved is quite heavy It has to approach large number of people for its capital It cannot start its business unless certificate of incorporation has been obtained This is granted after a long time when all the formalities are completed

Chartered CompanyStatutory CompanyRegistered Company

Delay in Decision making In this form of organization decisions are not made by single individual All important decisions are taken by the Board of Directors Decision-making process is time-consuming So many opportunities may be costly because of delay in decision-making Promptness of decisions which is a common feature of sole trader ship and partnership is not found in a company

Excessive Government ControlA company and the management have to function well within the law and the provisions of Companies Act are quite elaborate and complex At every step it is necessary to comply with its provisions lest the company and the management should be penalized The penalties are quite heavy and in several cases officers in default can be punished with imprisonment This hampers the proper functioning of the company

Lack of Secrecy The management of companies remains in the hands of many persons Every important thing is discussed in the meetings of Board of Directors Hence secrets of the business cannot be maintained In case of sole proprietorship and partnership forms of organisation such secrecy is possible because a few persons are involved in the management

2 Define the following

Chartered Company- The crown in exercise of the royal prerogative has power to create a corporation by the grant of a charter to persons assenting to be incorporated Such companies or corporations are known as chartered companies Examples of this type of companies are Bank of England (1694) East India Company (1600) The powers and the nature of business of a chartered company are defined by the charter which incorporates it After the country attained independence these types of companies do not exist

in IndiaStatutory Company- A company may be incorporated by means of a special Act of the Parliament or any state legislature Such companies are called statutory companies Instances of statutory companies in India are Reserve Bank of India the Life Insurance Corporation of India the Food Corporation of India etc The provisions of the Companies Act 1956 apply to statutory companies except where the said provisions are inconsistent with the provisions of the Act creating them Statutory companies are mostly invested with compulsory powersRegistered companiesCompanies registered under the Companies Act 1956 or earlier Companies Acts are called registered companies Such companies come into existence when they are registered under the Companies Act and a certificate of incorporation is granted to them by the Registrar

Economics

Chapter-4Basic problems of Economy

Today let us discuss with the topic Production Possibility curve

QuestionExplain the concept of Production Possibility Curve with the help of diagram

Answer) Production Possibility curve is a locus of all possible combinations of two commodities which can be produced in a country with its given resources and technology

The above diagram shows that with the given resources and technology the economy can produce maximum either 5 thousand meters of cloth or 15 thousand quintals of wheat or any other combination of the two goods like B( 1 thousand meters of cloth and 14 thousand quintals of wheat C ( 2 thousands meters of cloth and 12 thousand quintals of wheat) etcProduction Possibility curve is also called production possibility boundary or frontier as it sets the maximum limit of what it is possible to produce with given resources

Geography

Rotationand Revolution

SUNrsquoS POSITION AND SEASONAL CHANGES EQUINOXES ndash SPRING AND AUTUMN

Q1 What is Spring EquinoxA1 On 21st March sunrays fall directly on the equator On that day

As the Equator divides the Earth into two equal halves the sun rays fall directly on the equator twice in a year Equinoxes means equal Spring EquinoxOn 21st March sunrays fall directly on the equator On that day the duration of day and night both are equal ( 12 hours day and 12 hours night) on every places located on equator This day is called as Spring EquinoxAutumn EquinoxOn 23rd September sunrays fall directly on the equator On that day the duration of day and night both are equal ( 12 hours day and 12 hours night) on every places located on equator This day is called as Autumn Equinox

SOLSTICES ndash SUMMER AND WINTERDue to inclination of the Earth on its axis and the apparent movement of the sun the sun rays fall directly on both tropics once in a year Solstice is a Latin word which mean ldquothe Sun standing stillrdquoSummer SolsticesAfter 21st March there is an apparent movement of the Sun to the north of the equator The apparent northward movement up to 21st June when the Sun appears overhead at the Tropic of Cancer (22frac12degN) The sun appears to stand still at this position and then moves southwards towards the equator This position of the Sun on 21st June is known as Summer Solstices On that day the duration of day and night both are equal ( 12 hours day and 12 hours night) on every places located on Tropic of Cancer (22frac12degN)Winter solstices The apparent southward movement of the Sun continues beyond the equator till 22nd

December On this day the Sun is overhead at the Tropic of Capricorn

the duration of day and night both are equal ( 12 hours day and 12 hours night) on every places located on equator This day is called as Spring Equinox

Q2 What do you mean by EquinoxA2 Equinoxes means equal It is use to explain the equal duration of day and night ( 12 hours day and 12 hours night) on the Earth

Q3 On which date the longest day in Tropic of CancerA3 21st June

Q4 What is the meaning of SolsticeA4 Solstice is a Latin word which mean ldquothe Sun standing stillrdquo

Q5 Which is the longest day in southern hemisphereA5 22nd December

Q6 On what date does the Arctic Circle experience the lsquoMidnight SunrsquoA6 On 21 June the Arctic Circle experiences the lsquoMidnight Sunrsquo

Q7 What is cause of Midnight Sun in NorwayA7 During the summer solstice (21 June) the North Pole is inclined towards the Sun Therefore the duration of sunlight or daytime increases from 12 hours at the Equator to 24 hours at the Arctic Circle and beyond Thatrsquos why The region beyond the Arctic Circle especially Norway is known as the Land of the Midnight Sun because there the Sun does not rise or set on 21 June

Q8 Match the column A with BA B

Summer Solstice 21st March

Autumn Equinox 23rd

September

Winter Solstice 21st June

(22frac12degS) This position of the Sun is referred to as the Winter Solstice because it marks the winter season in the Northern Hemisphere On that day the duration of day and night both are equal ( 12 hours day and 12 hours night) on every places located on Tropic of Capricorn (22frac12degS)SEASONS AND DURATION OF DAY AND NIGHT During the equinoxes all places on the Earth have 12 hours of day and 12 hours of night Due to the revolution of the Earth round the Sun on an inclined axis the duration of day and night varies according to seasons and the latitude of a placeDuring the summer solstice (21 June) the North Pole is inclined towards the Sun Therefore the duration of sunlight or daytime increases from 12 hours at the Equator to 24 hours at the Arctic Circle and beyondThe region beyond the Arctic Circle especially Norway is known as the Land of the Midnight Sun because there the Sun does not rise or set on 21 JuneAt the North Pole there will be six months of daylight The Sun will be seen always above the horizon at a low angle At 66degN 24 hours of sunlight can be seen only on 21 June Hammerfest in northern Norway is a place of tourist attraction for observing the phenomenon of the Midnight Sun This place has continuous daylight from 13 May to 29 July This place is easily accessible to tourists and has hotels and other facilities The view of the midnight Sun from here is enthrallingIn the Southern Hemisphere the duration of daylight decreases from 12 hours at the equator to 0 hours beyond the Antarctic Circle In the South Polar Region there is 24 hours of darkness The Sun is always below the horizon In the Southern Hemisphere which experiences winter the duration of night-time is longer than the duration of daylight

Spring Equinox 22nd

December

A8 A B

Summer Solstice 21st June

Autumn Equinox 23rd

September

Winter Solstice 22nd

December

Spring Equinox 21st March

During winter solstice (22 December) the South Pole is inclined towards the Sun The Southern Hemisphere experiences summer and the Northern Hemisphere has winter Therefore the duration of daylight or sunlight is greater in the Southern Hemisphere than in the Northern HemisphereThe duration of daylight increases from 12 hours at the equator to 24 hours beyond the Antarctic Circle The South Polar Region has 24 hours of sunlight for many days continuously At the South Pole there will be six months of sunlight The Sun will always be seen at a low angle above the horizon In the Northern Hemisphere the duration of daylight will decrease from 12 hours at the equator to 0 hours at the Arctic Circle There are 24 hours of darkness in the North Polar region The duration of night is greater than the duration of daylight as one move northwards from the Equator It is evident from the above table that the duration of daylight is 12 hours throughout the year at the equator only As one moves away from the equator the seasonal variations in the duration of daylight increase The seasonal variations in the duration of daylight are maximum at the Polar Regions

Subject Eng Literature (The Merchant of Venice ndash William Shakespeare)Topic Act II Scene 7 Lines 36 to 80 (End of scene ) [Students should read the original play and also the paraphrase provided]

Summary Questions amp AnswersThe Prince then examines the inscription on the silver casket which says ldquoWho chooseth me shall get as much as he deservesrdquo The Prince says that he deserves Portia more than anybody else because of his high rank his noble birth and his great wealth and power But then he argues that silver is ten times

(1) (Act II Sc 7 L 39-47)

From the four corners of the earth they come

To kiss this shrine this mortal breathing saint

The Hyrcanian deserts and the vasty wildsOf wide Arabia are as through-fares now

inferior to gold and therefore he cannot believe that the portrait of such a beautiful lady as Portia can be contained in the silver casket He decides to see the inscription on the golden casket before making his decision

The Prince goes to examine the inscription on the golden casket which says ldquoWho chooseth me shall get what many men desirerdquo The Prince believes that the whole world desires to possess Portia otherwise so many suitors would not have come from all corners of the world for winning Portia Some of them have come from the distant lands of Persia and Arabia The deserts of Persia (Hyrcanian deserts) and the boundless desolate lands of Arabia have been crossed by the Princes seeking the hand of Portia He contrasts this casket containing Portiarsquos portrait with the old English gold coin bearing the image of the archangel (angel of the highest rank) He goes on to remark that while the figure of the archangel is engraved (Insculped) upon the English coin the picture of Portia who is beautiful as an angel lies hidden inside one of the caskets namely the Golden Casket (Golden Bed)

On the basis of his assessment of the inscription on the golden casket the Prince decides to choose the golden casket He asks for the key and opens the golden casket only to find therein an empty human skull holding a roll of

For princes to come view fair PortiaThe watery kingdom whose ambitious headSpets in the face of heaven is no barTo stop the foreign spirits but they comeAs orsquoer a brook to see fair Portia

(i) Explain the occasion for the above mentioned speech

These are the comments of the Prince of Morocco after he reads the inscription on the golden casket His mental process is revealed to us in these words We find him debating within himself as to which casket he should choose

(ii) What light does the above speech throw on the personality of Prince of Morocco

From the above mentioned speech we come to know that the Prince of Morocco is keen to marry Portia He is the type of person who is easily taken away by outward appearance He is in love with Portia because of her beauty

(iii) What information can you gather about Portia from the above mentioned lines

The given speech shows that Portia is a very beautiful lady She must be possessed of good qualities because many suitors come to her place from all over the world with a desire to get married to her The Prince of Morocco is so impressed by her beauty that he calls her a saint According to him the whole world is desirous of having her

(iv) Elucidate the significance of the first two lines

In these lines the Prince of Morocco pays a compliment to Portia These lines show his admiration for her He says that people come from all parts of the world to see fair Portia

(v) Explain the meaning of the last four lines of the

passage

In these lines the Prince of Morocco says that even the vast oceans which throw a challenge at the sky are unable to prevent men from coming to Portiarsquos place to have a glimpse of her These lines are also a tribute to Portiarsquos beauty and good qualities Many men voyage across the ocean treating it as a mere stream to see the beautiful Portia

paper in which is written that whoever happens to be guided by the glitter of things is invariably deceived

On reading the scroll the Prince says that he is too sad at heart to speak a more formal farewell and leaves with his followers amidst a sound of trumpets

After the Prince of Morocco leaves Portia remarks that the Prince is a gentle fellow but she is rid of him May all persons of his nature make a similar choice

IMPORTANT PASSAGES EXPLAINED

(Act II Sc 7 L 39-43)From the four corners of the earth they come

To kiss this shrine this mortal breathing saintThe Hyrcanian deserts and the vasty wildsOf wide Arabia are as through-fares nowFor princes to come view fair Portia

Context

This passage occurs in Act II Scene 7 in The Merchant of Venice This is part of the speech made by the Prince of Morocco

(2)

(Act II Sc 7 L 48-53)

MOROCCO One of these three contains her heavenly pictureIst like that lead contains her

Twere damnation To think so base a thought it were too grossTo rib her cerecloth in the obscure graveOr shall I think in silver shes immurdBeing ten times undervalued to tried gold

(i) What meaning does the Prince of Morocco find out of the inscription of the golden casket What have Belmont and Portiarsquos house been called and why

The inscription on the golden casket is ldquoWho chooseth me shall gain what many men desirerdquo The Prince finds out that it means that the chooser of the golden casket will get Portia because many men desire her In fact the entire world desires her Because of the coming of many suitors to Belmont from different countries in order to win Portiarsquos hand Belmont has become a centre of pilgrimage and her house is the shrine where saintly Portia is installed

(ii) What does the Prince of Morocco do before making the final choice of the casket Which is the correct casket and who will win Portiarsquos hand

The Prince of Morocco surveys and analyses the inscriptions on the casket of lead silver and gold Before making the final choice like a very systematic and methodical person he once again considers the claims of the caskets The casket containing Portiarsquos picture is the correct casket and the person choosing it will win Portiarsquos hand

Explanation

While praising Portia the Prince of Morocco conceives Portia as a goddess whose image is placed inside one of the caskets Many suitors are coming from far and wide the north and the south the east and the west (Four corners) in order to try their luck Some of them have come from the distant land of Persia and Arabia The deserts of Persia (Hyrcanian deserts) and the boundless desolate lands of Arabia have been crossed by the Princes seeking the hand of Portia All this shows that Portia is indeed the most beautiful lady of the world

(iii) What does the Prince of Morocco say in his estimation while examining the motto on the silver casket What does he find in the golden casket

While examining the motto on the silver casket which says ldquoWho chooseth me shall get as much as he deservesrdquo Morocco says that in his own estimation he surely deserves Portia in all respects ndash rank birth wealth etc

He chooses the golden casket When he opens it he finds an empty human skull holding a scroll in which it is written that those who are attracted by the glittering outside of things are always deceived as Morocco has been deceived

(iv) What kind of nature does the Prince of Morocco have

The Prince of Morocco has a simple nature who does not look deeply into the inner meaning of things but is dazzled by the outward appearance of gold He is inclined to over-estimate his own value and does not realize that it is a duty to ldquogive and hazardrdquo To say that he will not hazard for lead shows that he misreads the true meaning of the inscription which is that he should be prepared to ldquohazard all he hathrdquo for Portia So his feeling is only one of fascination and romantic attraction

(v) Do you think that the lottery of the caskets is not a matter that will be determined by chance

In fact the lottery of the casket is not a matter that will be determined by mere chance but that it is a true test of character and of sincerity which is amply proved not only by Moroccorsquos choice but also by the arguments which he uses to help him in his choice

(Act II Sc 7 L 55-59)

They have in England

A coin that bears the figure of an angelStamped in gold but thats insculpd uponBut here an angel in a golden bedLies all within

Context

(3)

(Act II Sc 7 L 63-77)A carrion Death within whose empty eye

There is a written scroll Ill read the writing

All that glisters is not goldOften have you heard that toldMany a man his life hath soldBut my outside to beholdGilded tombs do worms infoldHad you been as wise as boldYoung in limbs in judgment oldYour answer had not been inscrolld

This passage occurs in Act II Scene 7 in The Merchant of Venice This is part of the speech made by the Prince of Morocco

Explanation

In this passage the Prince of Morocco bestows high praise on Portia whose hand he is seeking He contrasts this casket containing Portiarsquos portrait with the old English gold coin bearing the image of the archangel (angel of the highest rank) He goes on to remark that while the figure of the archangel is engraved (Insculped) upon the English coin the picture of Portia who is beautiful as an angel lies hidden inside one of the caskets namely the Golden Casket (Golden Bed) In the day of Elizabeth silver was ten times inferior in value to gold Therefore the Prince of Morocco believing that Portiarsquos portrait is contained in the Golden Casket decides to choose the Golden Casket

Fare you well your suit is coldCold indeed and labour lostThen farewell heat and welcome frostmdashPortia adieu I have too grievd a heartTo take a tedious leave Thus losers part

(i) What reward does the Prince of Morocco get after making a wrong choice of the Casket How does he feel

After making the wrong choice in selecting the casket of gold the Prince of Morocco as a reward earns a rebuke in the form of a scroll tucked in the empty eye-socket of a skull kept in the casket of gold The Prince is shocked and disappointed He becomes all the more sad and dejected when he reads the scroll which points to his foolishness in being misled by the appearance and outward show as indicative of its worth

(ii) How does the Prince respond after reading the scroll

After reading the scroll the Prince though upset accepts the result with good grace and decorum befitting a royal suitor and true sportsman He says that his love-suit is really cold otherwise he would have chosen correctly but now his efforts have been in vain So he bids farewell to Portia to the warmth and enthusiasm of love and welcomes the cold and bitterness of dejection and misery of life which lies ahead

(iii) What request does he make to Portia and why

After being failure in his mission he requests Portia to give him permission to leave at once because he is too sad to undergo the tediousness of a formal leave-taking He tells that it is the manner in which defeated persons part unceremoniously

(iv) Explain the following lines

ldquoAll that glisters is not goldOften have you heard that toldMany a man his life hath soldBut my outside to beholdGilded tombs do worms infoldrdquo

Mere glitter does not make a metal to be gold Man has often been warned against appearance but it has been of no use Many people have sacrificed their lives only to seek the outer appearance of gold Worms are found inside the gilded

monuments

Class XSubject Topic Summary Execution

Hindi 2ndlang

नया रासता भाग 6 मायाराम 0ी घर म धनी मल 0ी और उनी बटी सरिरता ी ही चचा बनी रहती थी अमिमत ो इसम ोई रलिच ना थी वह धनी घर ी लडी स शादी र सवय ो बचना नही चाहता था उसा भी सवाणिभमान ह ईशवर ी पा

स उस पास पस ी ोई मी नही थी अभी उसन फकटरी ही लगाई थी उसी समझ बाहर था कि उस घर वालो ा झाव पस ी तरफ कयो

ह उसन मा स सवाल किया कि मा तम सरिरता स मरी शादी कयो रना चाहती हो मा न उस समझाया कि वह दखन म बरी नही ह और किफर खानदान अचछा

ह वह ए शल गरहणी रप म घर सभाल सगी अमिमत न मा ो इस बात ा एहसास राया कि मीन सबध लिलए मना रन पर उस दिदल

पर कया बीती होगी मा और अमिमत ी लडी बार म ाफी बात हईमा ा झाव सरिरता ी तरफ था कयोकि वह घर पर अचछा दह0 लर आ रही

थी अमिमत न अपनी मौसी ी बरी हालत बार म बताया कि किस तरह वह बड घर ी खानदानी बटी लाई थी और आ0 उसी हालत कितनी खराब ह लाई थी बहकलब 0ाती ह और बचचो ो भी नही दखती ह बात चल ही रही

थी कि तभी ए ार बाहर आर री धनी मल0ी घर अदर आए और पीछ स डराइवर फल ी ए टोरी लर आया अदर आए और पीछ स

डराइवर ए टोरी फल ी लर आया अमिमत ो फल ी पटी बरी लग रही थी अमिमत न पछ लिलया यह फल कयो ल आए ह प इन सब ी कया

0ररत थी उनो न 0वाब दिदया कि 4 पटी शमीर स मगाए थ अमिमत ो या सनर करोध आ गया तभी उस किपता 0ी आ गए उन आत ही अमिमत उठर बाहर चला गया वहा वहा मा पास आर बठ गया और बोला

अभी रिरशता तय नही हआ और धनी मल 0ी धनी मल 0ी फल ी पटी लर चलआय मा न समझाया कि 0ब सबध 0ड 0ाता ह तो खाली हाथ नही

आत अमिमत न मा स हा कि तम सबन सरिरता ो इस घर म लान ी ठान रखी ह धनीमल 0ी उस दिदन सरिरता ो दखन ी तारीख तय रन आय थ

Commercial Studies

Banking Nowadays Bank provide easy and quick services through internet facilities methods of Banking is called internet bankingIn order to save the time and money involved in visiting Bank branches people increasingly prefer to have internet banking

There are different modes of doing internet banking or transferring money through online They areReal Time Gross Settlement (RTGS)National Electronic Fund Transfers (NEFT)

1

Question

1) Explain the term RTGS Write the features of RTGS

Answer)The acronym RTGS stands for Real Time Gross Settlement which may be defined as the continuous real time settlement of funds transfer individually on and order by order basis without netting lsquoReal timersquo may be defined as the processing of instructions at the time they are received rather than at some letter time lsquoGross settlementrsquo may be defined as the settlement of transfer instructions which occurs

individually

Features of RTGS1It is the continuous settlement of

funds transfer individually on an order by order basis

2RTGS facility is provided only by CBS core banking solution enabled Bank branches

3Amount charged from the customer for RTGS transactions vary from bank to bank

2) Explain the term NEFT Write the features of NEFT

Answer) National electronic funds transfer may be defined as a nationwide system that facilitates individuals Farms and copper operates to electronically transfer funds from any bank branch to any individual farm or corporate having an account with any other bank branch in the country

Features of NEFT2 Transfer can be made 7 times on

weekdays and 6 times on Saturday

3 NEFT cannot be used to receive foreign remittances

4 NEFT transaction takes place in batches

5 A bank branch must be NEFT enabled to become a part of NEFT fund transfer network

6 There is no maximum or minimum amount that can be transferred through NEFT when one bank has a bank account

English Language

CompositionEssay

A composition is an art of creating a piece of writing on any topic or subject It is the writing correctly beautifully and clearly in order to make some interesting reading Structure of the composition

Introduction ( you lay the foundation for your composition)

Body (it constitutes the main part of the essay)

Conclusion (final statement that leaves a lasting impression)

Kinds of essays1 The Narrative essay2 The descriptive essay3 The reflective essay4 The argumentative essay

Write a composition on any one of the following topics (350- 400 words)

1 Friendship Or2 The first day of your school

Subject Eng Literature (The Merchant of Venice ndash William Shakespeare)Topic Act V Scene 1 Lines 127 to 158 (Nerissa helliphellip The clerk will nersquoer wear hair onrsquos face that had it) [Students should read the original play and also the paraphrase given in the school prescribed textbook]

Summary Revision Questions o Soon thereafter Bassanio Gratiano

and Antonio arrive

o Bassanio tells Portia that he is feeling as if it is morning because of the presence of Portia who is shining like the sun When Antonio is introduced by Bassanio to Portia she tells Bassanio that he should be grateful to Antonio who took so much trouble on his account even to the extent of risking his life

o Nerissa starts quarrelling with Gratiano and demands that he show her the ring she had presented to him and which she had warned him not to lose She suspects that Gratiano must have presented the ring to some young woman and not to the lawyerrsquos clerk as he repeatedly says and assures

Answer the following questions to check your preparation of Act IV Scenes 1 and 2

You must attempt only after you have completed your preparation of Act IV The answers must be in complete sentences using textual evidence (with citation) when necessary

[It would be in your own interest to attempt the above questions honestly totally refraining from consulting your textbook or your notes during answering After completion you should correct the paper yourself consulting the textbooknotes etc and award marks as specified Please let me know the marks you scored through WhatsApp in the group or to my personal WhatsApp]

Act IV Scene 1 (each question carries 2 marks)

1 What did the Duke try to do for Antonio

2 Why does Shylock refuse to show mercy How does he justify his stance

3 Why does Antonio say he is ready to die 4 What information is contained in Bellariorsquos letter

5 Why does Portia (as Balthazar) assert that Shylock must show mercy How does he respond

6 What offers are made to Shylock to get him to spare Antonio How are they received

7 What does Antoniorsquos speech as he faces the prospect of Shylockrsquos knife tell you about his character

8 How do Bassanio and Gratiano react to the looming prospect of Antoniorsquos demise

9 How does Portia (as Balthazar) use the law to turn the tables on Shylock

10 What does the Duke decree should happen to Shylock Why What happens to Shylockrsquos estate

11 What does Portia ask Bassanio as payment for her ldquoservicesrdquo What is his initial response What makes him change his mind

Act IV Scene 2 (each question carries 1frac12 marks)

1 What does Gratiano bring to Portia (Balthazar)

2 What does Nerissa plan on getting from Gratiano What does Portiarsquos comment suggest about men

ECO-10 280620 Topic-Supply AnalysisSHIFTING OF SUPPLY

But if there is change in factors other than the price of the commodity then either more is supplied at the same price or less supplied at the same price In such cases the price of the commodity remains constant but there is a change in other factors like change in the price of inputs change in technology of production change in price of other related goods change in taxation policy of the government etc For example there is an improvement in the technology of production of the commodity in question It leads to decrease in per unit of cost production of the commodity The firm is willing to sell more quantity of the commodity at the same price So the supply other commodity increases at the same price This increase in supply is shown by rightward shift of supply curve On the other hand if the firm uses inferior technology of production the cost of production per unit of the commodity increases The firm is willing to sell less quantity at the same price So the supply of the commodity decreases at the same price This decrease in supply is shown by leftward shift of the supply curve The above cases of increase and decrease in supply can be shown with the help of the following figures

Y INCREASE OF SUPPLY Price (Rs) s

P A s1

B

s

X` O s1 X

q q1

Y` Quantity demanded (in units)

Y DECREASE IN SUPPLY s2

s

price (Rs)

C

p A

s2

s

X` o X

q2 q

Y` Quantity demanded ( in units)

Main factors causing increase in supply or rightward shift of supply Curve(i) Fall in the price of other related goods

(ii) Fall in the price of inputsfactors(iii) Use of better technology in production(iv) Decrease in the rate of excise duty by government(v) If the objective of producer changes from profit maximization to salesMaximization

Main factors causing decrease in supply or leftward shift of supply curve(i) Increase in the price of other related goods(ii) Rise in the price of inputsfactors(iii) Use of inferior technology in production(iv) Increase in the rate of excise duty by the government(v) If the objective

Subject - Biology Topic ndash Chapter mdash6 PhotosynthesisSummary Execution

Today we will know about photosynthesis and its stages

Q1 What do you mean by photosynthesis The process by which living plants containing chlorophyll produce food

substances from carbon-di- oxide and water by using light energy Sunlight

6CO2 +12 H2O----------------------- C6 H12O6 + 6H2O + 6O2

Chlorophyll

Q2 What are the importance of photosynthesis I) Food for all Green plants trap solar energy by photosynthesis

process and supply food and energy for all living organisms either directly or indirectly

Ii) Oxygen to breathe in by product of photosynthesis is oxygen which is essential for all living organisms respiration

Q3 Write about two main phases of photosynthesis A Light dependent phase This phase occur in grana of chloroplast I) The chlorophyll on exposure to light energy becomes activated by

absorbing photons Ii) The absorbed energy is used in splitting the water molecules (H2O)

into its two components (H+ and OH- ) and releasing electron s 2H2O------------------------- 4H+ + 4e- +O2

Energy of 4 photons This reaction is known as photolysis

End products are H+ and oxygen water

B Light independent (Dark ) phase The reactions in this phase require no light energy

Here CO2 combine with H+ and produce glucose

Class XI

Subject Topic Summary ExecutionEVS Chapter-4 Legal

regimes for sustainable development

Environmental legislationEnvironmental legislation is the collection of laws and regulations pertaining to air quality water quality the wilderness endangered wildlife and other environmental factors The act ensures that matters important to the environment are thoroughly

Learn -The Forest (Conservation) Act 1980

considered in any decisions made by federal agencies

The Forest (Conservation) Act 1980 The Forest (Conservation) Act 1980 an Act of the Parliament of India to provide for the conservation of forests and for matters connected therewith or ancillary or incidental thereto It was further amended in 1988 This law extends to the whole of IndiaObjects and Reasons of the Forest Conservation Act

Deforestation causes ecological imbalance and leads to environmental deterioration Deforestation had been taking place on a large scale in the country and it had caused widespread concern The act seeks to check upon deforestation and de-reservation of forests

Subject Eng Literature (The Tempest ndash William Shakespeare) Topic Act II Scene 1 Lines 314 to 329 (End of scene)

[Students should read the original play and also the paraphrase given in the school prescribed textbook]Summary Questions amp Answers

Conspiracy of Antonio and Sebastian (Contd)

o As they approach Ariel appears again and wakes up Gonzalo by singing a tune in his ear Alonso also wakes up and they see both Sebastian and Antonio with drawn swords On being caught off guard they make up a story saying that they had heard a bellowing of bulls or lions

o They then moved to another part of the island

o Ariel at once rushes to Prospero to inform him of this development

SUMMING-UP of ACT-2 SCENE-1

(i) Among the survivors Ferdinand is separated from the rest which results in the disconsolate grief of Alonso as he took him for dead

(ii) The villainy of Antonio is confirmed

(iii) The supremacy of Prosperorsquos magic which resulted in the failure of the human conspiracy

(1)

(Act II Sc 1 L 311-325)SEBASTIAN Whiles we stood here securing your repose

Even now we heard a hollow burst of bellowing Like bulls or rather lions Didt not wake youIt struck mine ear most terribly

ALONSO I heard nothingANTONIO O rsquotwas a din to fright a monsters ear

To make an earthquake Sure it was the roarOf a whole herd of lions

ALONSO Heard you this GonzaloGONZALO Upon mine honour sir I heard a humming

And that a strange one too which did awake meI shaked you sir and cried As mine eyes opened I saw their weapons drawn There was a noiseThats verily rsquoTis best we stand upon our guardOr that we quit this place Lets draw our weapons

(i) Why has Prospero sent Ariel to Gonzalo and Alonso What does Ariel do to awaken Gonzalo

Prospero has already come to know by his magic powers the danger which threatens Gonzalo who had been Prosperorsquos friend and so he sent Ariel to preserve the lives of both Gonzalo and Alonso Prospero does not want that his scheme should remain unfulfilled Ariel begins to sing a song in Gonzalorsquos ears to awaken him(ii) Who are ready to carry out their plan Who takes steps to stop them Why does Gonzalo feel surprised after being awakened

Sebastian and Antonio are ready to carry out their plans They are standing with their swords drawn to kill Alonso and

(iv) We see two sets of contrasting characters Gonzalo-Adrian against Antonio-Sebastian

(v) The grief that works in Alonso can be perceived to his repentance for his association in Antoniorsquos crime against Prospero

Gonzalo Ariel takes steps to stop them from carrying out their nefarious scheme When Gonzalo is awakened by the song sung by Ariel into his ears he (Gonzalo) feels surprised because he sees Sebastian and Antonio standing with their swords drawn(iii) What reason do Sebastian and Antonio tell of drawing their swords when they are suspected by Alonso and Gonzalo

When Sebastian and Antonio are seen with their swords drawn they are looked with suspicion by Gonzalo and Alonso At first Sebastian tells them that as they stood here to guard them during their sleep they heard only a little before a sudden loud noise very much like the roaring of bulls or more probably that of lions Then Antonio follows him saying that this was a noise so terrible as to frighten even a monsterrsquos ears and this noise could even have shaken the earth and it was surely like the roaring of a multitude of lions Then seeing the danger they have drawn their swords Perhaps after hearing the terrible noise they (Gonzalo and Alonso) woke up from their sound sleep

(iv) What does Gonzalo tell Alonso about the strange noise What did he see on opening his eyes Gonzalo tells Alonso that he did not hear the sound of roaring but he heard a humming sound which was strange and which woke him up After waking up he gave him (Alonso) a shaking and a loud cry On opening his eyes he saw these two gentlemen standing with their swords drawn(v) What does Gonzalo suggest

Gonzalo suggests that there was a noise indeed and of that he has no doubt at all and suggests that the best course for them would be to remain alert and vigilant against any possible danger to their lives or to leave this place and move to some other part of the island

Class XIISubject Topic Summary Execution

Commerce

Chapter- Management

Today we will discuss about LEVELS OF MANAGEMENT

Levels of management is a series or chain of managerial positions from top to bottom It helps individuals to know their authority responsibilities and superior-subordinate relations among themselves There are mainly three levels of Management TOP LEVEL MANAGEMENTMIDDLE LEVEL MANAGEMENTLOWER LEVEL MANAGEMENT

Top level managementIt consists of members at the highest level in the management hierarchy This level includes Board Of Directors Chief Executive Managing Directors Chairman President Vice President

Rolefunctions of the top levelmanagement1To analyse evaluate and deal

with theexternal environment2 To determine the objectives and

policies of the business3 To strive for welfare and survival

of business

4 To create an organisational Framework consisting of authority responsibility relationship

Middle level management Congress of members or groups who are concerned with implementation of the policies let down by the top managementThis level includes head of the department such as finance manager marketing manager branch and regional managers departmental and divisional heads plant superintendent etc

Role of functions of the middle level management

1 To interpret the policies framed by top management

2 To assign duties and responsibilities to lower level managers

3 To select and appoint employees for middle and supervisory level and evaluate their performance

4 To co-operate with other departments for smooth functioning

Operational or supervisory level managementIt refers to the group are members who are concerned with execution of the work They are also known as fast line managers This level includes supervisor 4 men Section Officer clerk Inspector etc

Role of functions of the lower level management1 To plan and execute day-to-

day operations2 To supervise and control the workers3 To arrange materials and

tools to start the process and make arrangements for training

4 Today present workers grievance and suggestions before the management and

ensure safe and proper working conditions in the factory

Business Studies

Staff Appraisal Chapter- 10 Today let us start with a new chapter

Staff Appraisal

Meaning of Performance Appraisal

Performance Appraisal is the systematic evaluation of the performance of employees and to understand the abilities of a person for further growth and developmentThe supervisors measure the pay of employees and compare it with targets and plansThe supervisor analyses the factors behind work performances of employeesThe employers are in position to guide the employees for a better performance

Objectives of Performance Appraisal

Following are the objectives of Performance Appraisal

To maintain records in order to determine compensation packages wage structure salaries raises etc

To identify the strengths and weaknesses of employees to place right men on right job

To maintain and assess the potential present in a person for further growth and development

To provide a feedback to employees regarding their performance and related status

To provide a feedback to employees regarding their performance and related status

Importance of Performance Appraisal

Performance appraisal provides important and useful information for the assessment of employees skill

knowledge ability and overall job performance The following are the points which indicate the importance of performance appraisal in an organization

1 Performance appraisal helps supervisors to assess the work performance of their subordinates

2 Performance appraisal helps to assess the training and development needs of employees

3 Performance appraisal provides grounds for employees to correct their mistakes and it also provides proper guidance and criticism for employees development4 Performance appraisal provides reward for better performance

5 Performance appraisal helps to improve the communication system of the organization

6 Performance appraisal evaluates whether human resource programs being implemented in the organization have been effective

7 Performance appraisal helps to prepare pay structure for each employee working in the organization

8 Performance appraisal helps to review the potentiality of employees so that their future capability is anticipated

Geography

DRIANAGE The SubarnarekhaThe Subarnarekha and the Brahmaniinterposed between the Ganga and the Mahanadi deltas drain an area of 19300 sq kmand 39033 sq km respectively The drainage basins of these streams are shared byJharkhand Odisha west Bengal and Chhattisgarh The Brahmani is known as southKoel in its upper reaches in Jharkhand

The NarmadaThe Narmada rises in the Amarkantak hills of MadhyaPradesh It flows towards the West in a rift valleyformed due to a geological fault The total length of it is 1300 km All the tributaries of the

Q1 Name the two westward flowing rivers in the peninsular plateauA1 Narmada and Tapi are the only westward flowing rivers of the peninsular plateau

Q2 Differentiate between east-flowing rivers and west-flowing riversA2

East-flowing rivers

West-flowing rivers

Narmada are very short inlength Most of its tributaries join the main streamright anglesThe Narmada basin covers parts of Madhya Pradesh and Gujarat

The Tapi The Tapi rises in the Satpura ranges in the Betul listrictof Madhya Pradesh It flows in a rift valley parallel tothe Narmada but it is much shorter in length It coversparts of Madhya Pradesh Gujarat and MaharashtraThe length is about 724 km

The Sabarmati and the MahiThe Sabarmati rises in the Aravali hills and flows south-south-westwards for a distance of 300 kilometres to the Arabian Sea The Sabarmatibasin extends over an area of 21674 sq km in Rajasthan and Gujarat The Mahi rises inthe east of Udaipur and drains an area of 34842 sq km lying in Madhya PradeshRajasthan and Gujarat It flows south-westwards for a distance of 533 km before it fallsinto the Gulf of Khambhat

The ChambalThe Chambal rises near Mhow in the Vindhya Range and flows towards the northgenerally in a gorge upto Kota Below Kota it turns to the north-east direction and afterreaching Pinahat it turns to the east and runs nearly parallel to the Yamuna beforejoining it in the southern part of the Etawah district in Uttar PradeshMajor Rivers of India with their basin area (Sqkm)

Himalayan System Indus 321290Ganga 861404

Brahmaputra 187110Indus System

Jhelum 34775Beas 20303

Ganga System Yamuna 366223Ghaghra 127950

Peninsular RiversNarmada 98796

Tapi 65145Mahanadi 141600

Subarnarekha 19300Sabarmati 21674

Mahi 34842Godavari 312812

Godavari Krishna Kaveri Mahanadi are the east-flowing rivers

Narmada Tapi west-flowing rivers

They fall into the Bay of Bengal

They fall into Arabian Sea

These rivers form big deltas

These rivers form comparativelysmall deltas

Catchment areas of these rivers are larger

Catchment areas of these rivers are smaller

Krishna 2589488Cauveri 87900

Subject ndashBiology Topic ndashChapter -5 Inheritance amp Variations Summary ExecutionToday we will discussabout linkage and its classification

LINKAGE The tendency of the genes located on the same chromosome to stay together is

hereditary transmission Linked genes the genes responsible for this Genes that exhibit the process of linkage locates in the same chromosome The distance between the linked genes in a chromosome determines the strength

of linkage i e genes that are located close to each other show stronger linkage than that are located far from each other

COMPLETE LINKAGE It is the type of linkage showed by the genes that are closely located or are tightly

linked with each other as they have no chance of separatingby crossing over These genes are always transmitted together to the same gamete and the same

offspring In such condition only parental or non cross over type of gametes are formedINCOMPLETE KINKAGE It is type of linkage showed by the genes that are distantly located orare loosely

linked with each other because they have chance of separating by crossing over

SIGNIFICANCE i) It helps in holding the parental character togetherii) It checks the appearance of new recombination and helps in bringing the

hybrid population which resembles the original parents iii) Linked genes dilute the effects of undesirable traits

Subject Eng Literature (The Tempest ndash William Shakespeare) Topic Essay Questions (EQ-3)Question No 3

Give a character sketch of CalibanAnswer

The character of Caliban has been wonderfully conceived by Shakespeare as the manifestation of all that is gross and earthy ndash a sort of creature of the earth as Ariel is a sort of creature of the air

Calibanrsquos Physical Appearanceo Caliban is lsquofreckledrsquo a lsquomisshapen knaversquo not honoured with human shape

o Prospero calls him lsquothou tortoisersquo (Act I Sc 2 Line 317) Trinculo stumbling upon him describes him as ldquoA strange fish hellip Legged like a man And his fins like armsrdquo He ldquosmells like a fishrdquo (Act II Sc 2 Line 25)

o Prospero also calls him a ldquobeastrdquo (Act IV Sc 1 Line 140) and ldquoThis misshapen knaverdquo (Act V Sc 1 Line 268)

o Further it appears that in addition to his physical deformity his spiritual inferiority is also suggested by Prosperorsquos claim that his birth resulted from the union between his mother the witch Sycorax and the devil

Calibanrsquos ParentageWhen the play opens Caliban is twenty four years of age having been born on the island twelve years before the coming of Prospero His mother was the foul witch Sycorax who was banished from Algiers for ldquomischiefs manifold and sorceries terrible to enter human hearingrdquo (Act I Sc 2 Line 264) and the father was the Devil himself Thus

Caliban is a monster of evil and brute nature ugly deformed and stinking

Calibanrsquos Savage and Malignant Natureo Caliban is entirely a creature of the earth ndash gross brutal and savage He regards himself as the rightful possessor

of the island and Prospero as a usurper

o In his young age he was on good terms with Prospero He had consented to be received by Prospero at his house and to be educated by him He has learnt human language only to curse his master whom he abhors

o His beastly nature soon breaks out and ends in a vicious attack on Miranda This opens the eye of Prospero who becomes severe to him and enforces his service by threats and violence

o Prospero uses him to make dams for fish to fetch firewood scraper trenches wash dishes and keep his cell clean

Calibanrsquos Hatred for ProsperoA profound hatred for Prospero has taken hold of Caliban It springs from a sense of his being dispossessed and ill-treated He would kill Prospero if he could but he knows the power of Prosperorsquos lsquobookrsquo Hence he transfers his allegiance to Stephano who seems like a god to him He also incites the two drunken associates to batter the skull of Prospero when he sleeps in the afternoon

Caliban Shows Considerable Intelligenceo He has learnt Prosperorsquos language

ldquoYou taught me language and my profit onrsquot (Act II Sc 2 Lines 86-89)Is I know how to curserdquo

o He is well aware of the futility of arguing with one who has more power than he has

ldquoI must obey his art is such power (Act I Sc 2 Lines 373-376)It would control my damrsquos god SetebosAnd make a vassal of himrdquo

o He realizes the importance of Prosperorsquos books

ldquoRemember (Act III Sc 2 Lines 89-92)First to possess his books for without themHersquos but a sot as I am nor hath notOne spirit to commandrdquo

o He knows the value of stealth when attacking the enemy

ldquoPray you tread softly that the blind mole may not (Act IV Sc 1 Lines 194-195)Hear a foot fall we now are near his cellrdquo

o Caliban has a better set of values than Stephano and Trinculo They are distracted from their plan by their greed for Prosperorsquos rich garments Only Caliban realizes that such a finery is unimportant

ldquoLeave it alone thou fool it is but trashrdquo (Act IV Sc 1 Lines 224)

Caliban is not a good judge of characterCaliban is not a good judge of character He decides for example that Stephano is a god because he dispenses lsquocelestial liquorrsquo (Act II Sc 2 Line 115) but then it must be remembered that he has only known his mother Sycorax Prospero Miranda and the spirits that torture him However he quickly discovers his error of judgementrdquo

ldquoWhat a thrice-double ass (Act V Sc 1 Lines 295-297)Was I to take this drunkard for a godAnd worship this dull foolrdquo

Calibanrsquos Imaginative NatureIf Caliban is sub-human in what has been said above he is human in the respect of the poetic side of his character He listens to music with rapture He tells of the beautiful dreams in which heaven rains treasures upon him and which upon waking he yearns to renew One of the most poetic passages in whole play is Calibanrsquos description of the island

to Stephano and Trinculo

ldquoBe not afeard The isle is full of noises (Act III Sc 2 Lines 135-143)Sounds and sweet airs that give delight and hurt notSometimes a thousand twangling instrumentsWill hum about mine ears and sometime voicesThat if I then had waked after long sleepWill make me sleep again and then in dreamingThe clouds methought would open and show richesReady to drop upon me that when I wakedI cried to dream againrdquo

Caliban - Less Ignoble Than Some OthersCalibanrsquos motive for murder is less dishonourable than that of Antonio and Sebastian They plan to kill Alonso to gain his power and wealth Caliban merely wants revenge and the return of lsquohisrsquo island

Conclusiono Calibanrsquos character is not portrayed very clearly in the play and hence we cannot decide whether he is a poor

savage being grossly maltreated by Prospero or whether he is evil and must therefore be kept in bondage or enslavement

o Caliban is contrasted with Ariel who is a spirit and thus swift and uninterested in physical activitieso Caliban is also contrasted with Prospero who is the all-powerful master of the island and of the destiny of all

those on the islando Caliban is also contrasted with civilized man showing him to be less evil than Antonio and Stephano and less

materialistic than Stephano and Trinculoo Caliban has suffered at the hands of Prospero and he has learnt to curse by listening to Prosperorsquos abuse He

certainly believes that Prospero has deprived him of his birthrighto Finally the character Caliban is thought to be one of Shakespearersquos masterpieces The complexity of the character

is reflected in the large volume of critical discussion that has grown around it

ECO ndash12 Topic-Forms of market

MonopolyMonopoly is a market structure in which there is a single seller there are no close substitutes for the commodity produced by the firm and there are barriers to entry Example Indian Railways which is operated under government of India Monopoly also implies absence of competitionFeatures of Monopoly Monopoly is characterized by1 Single Seller In monopoly there is only one firm producing the product The whole industry consists of this single firm Thus under monopoly there is no distinction between firm and industry Being the only firm there is significant control of the firm over supply and price Thus under monopoly buyers do not have the option of buying the commodity from any other seller They have to buy the product from the firm or they can go without the commodity This fact gives immense control to the monopolist over the market

2No Close Substitute There are no close substitutes of the product produced by the monopolist firm If there are close substitutes of the product in the market it implies presence of more than one firm and hence no monopoly In order to ensure a total of control over the market by the monopolist firm it is assumed that there are no close substitutes of the product

3 No Entry amp Exit Monopoly can only exist when there is strong barriers before a new firm to enter the market In fact once a monopoly firm starts producing the product no other firm can produce the same One reason for this is the ability of the

monopolist to produce the product at a lower cost than any new firm who thinks to enter the market If a new firm who knows that it cannot produce at a lower cost than the monopolist then that firm will never enter the market for fear of losing out in competition Similarly the monopolist who is operating for a long time may be enjoying reputation among its customers and is in a better position to use the situation in its own benefit A new firm has to take long time to achieve this and so may not be interested to enter the market

4 Price Maker Being the single seller of the product the monopolist has full control over the pricing of the product On the other hand if there is a large number of buyers in the market so no single buyer exercises any significant influence over price determination Thus it is a sellerrsquos market So monopoly firm is a price maker

5 Price Discrimination Having considerable control over the market on account of being single seller with no entry of other firms the monopolist can exercise policy of price discrimination it means that the monopolist can sell different quantities of the same product to a consumer at different price or same quantity to different consumers at different prices by adjudging the standard of living of the consumer

6 Shape of Demand Curve Since a monopolist has full control over the price therefore he can sell more by lowering the price This makes the demand curve downward sloping

Subject Ac-12 290620 Topic- retirement Model sumThe Balance Sheet of Rohit Nisha and Sunil who are partners in a firm sharing profits according to their capitals as on 31st March 2014 was as under

Liabilities Amount Assets Amount (Rs) (` Rs)

Creditors 25000 Machinery 40000Bills Payable 13000 Building 90000General Reserve 22000 Debtors 30000Capital Less Provision for Rohit 60000 Bad debts 1000

29000 Nisha 40000 Stocks 23000 Sunil 40000 140000 Cash at Bank 18000

200000 200000

On the date of Balance Sheet Nisha retired from the firm and following adjustments were made(i) Building is appreciated by 20(ii) Provision for bad debts is increased to 5 on Debtors(iii) Machinery is depreciated by 10(iv) Goodwill of the firm is valued at Rs 56000 and the retiring partnerrsquos share is adjusted

(v) The capital of the new firm is fixed at Rs120000 Prepare Revaluation Account Capital Accounts of the partner and Balance Sheet of the new firm after Nisharsquos retirement Revaluation AccountDr Cr

Particulars Amount Particulars Amount (`Rs) (Rs`)

Provision for Bad debt Ac 500 Building Ac 18000Machinery Ac 4000Profit transferred toCapital Accounts (3 2 2)Rohit 5786Nisha 3857Sunil 3857

13500

18000 18000

Capital Account

Dr Cr

Particulars Rohit Nisha Sunil Particulars Rohit Nisha Sunil (Rs`) (Rs`) (`Rs) (Rs`) (Rs`) (Rs`)

Sunilrsquos Capital ac 9600 mdash 6400 Balance bd 60000 40000 40000Bank - 66143 - General Reserve 9428 6286 6286Balance cd 72000 mdash 48000 Revaluation (Profi 5786 3857 3857 Rohitrsquos Capital Ac mdash 9600 mdash

Sunilrsquos Capital Ac 6400 Bank 6386 - 4257

81600 66143 54400 81600 66143 54400

Balance Sheet as at 31st March 2014

Liabilities Amount Assets Amount (Rs`) (Rs`)

Creditors 25000 Building 108000Bank overdraft 37500 Machinery 36000

Bills Payable 13000 Debtors 30000Capital Less ProvisionRohit 72000 for Bad debts 1500 28500Sunil 48000 120000 Stock 23000

195500 195500

Working Notes (i) (a) Profit sharing ratio is 60000 40000 40000 ie = 3 2 2(b) Gaining Ratio Rohit = 35 ndash 37 = 2135 ndash 1535 = 635Sunil = 25-27 = 1435 ndash 1035 = 435= 635 435= 6 4 = 3 2(c) Nisha Share of Goodwill = Rs 56000 times 27 = Rs16000Share of Goodwill in the gaining ratio by the existing partner ieRohit = Rs16000 times 35 = Rs 9600Sunil = Rs 16000 times 25 = Rs 6400

The journal entry isRohitrsquos Capital Ac Dr 9600Sunilrsquos Capital Ac Dr 6400 To Nisharsquos Capital Ac 16000(Share of Goodwill divided into gaining ratio)

  • 1 Static Friction
  • The frictional force that acts between the surfaces when they are at rest with respect to each other is called Static Friction
    • Static Friction Examples
      • 2 Sliding Friction
        • Examples Of Sliding Friction
          • 3 Rolling Friction
            • Examples Of Rolling Friction
              • Objects and Reasons of the Forest Conservation Act
Page 22:  · Web viewSubject . Topic . Summary . Execution . English 1 . Sounds of animals . Hens –cackle Horses –neigh Lions –roar Owls –hoots Snake –hiss. English 2 . Mother’s

All living organism (Plants and animals) need food for energy and growth Green plants (autotrophy) prepare food for all living organisms Today we will discuss about the process photosynthesis And adaptations in a leaf to carry out photosynthesis

Q1What do you mean by photosynthesis and write its word equation The process by which green plants make food (glucose) from carbon dioxide and water

in the presence of sunlight and chlorophyll is called photosynthesis

Carbon dioxide + Water ( Sun light from Sun ) Glucose + Oxygen ( chlorophyll in green leaves )

Q2 What are the adaptations in a leaf to carry out photosynthesisi) Leaves are broad wide and flat for absorbing more light energyii) Presence of chlorophyll in chloroplasts to trap sunlightiii) Presence of stomata which allow carbon dioxide to enter the cell and oxygen to go

out iv) Network of veins ensures continuous supply of water and minerals to the leafv) Thin waxy cuticle protects the leaf without blocking the lightQ3 Draw and label structure of chloroplast

Class VIIISubject Topic Summary Execution

PHYSICS ENERGY Production of Hydro electricity

A hydroelectric dam converts the potential energy stored in a water reservoir behind a dam to mechanical energymdashmechanical energy is also known as kinetic energy As the water flows down through the dam its kinetic energy is used to turn a turbine

The generator converts the turbinersquos mechanical energy into electricity

This electric energy then goes through various transmission processes before it reaches you

Question 2

Fill in the blanks

(a) Work is said to be done by a forte only when the body moves

(b) Work done = Force x distance moved in direction of force

(c) The energy of a body is its capacity to do work

(d) The SI unit of energy is joule

(e) The potential energy is due to its state rest of position and kinetic energy of the body is due to its state of motion

(f) Gravitational potential energy U = mass times force of gravity on unit mass times height

(g) Kinetic energy = frac12 times mass times (speed)2

(h) Power P = work donetime taken

(i) The S I unit of power is watt

(j) IHP = 746 W

BIOLOGY Chapter -5 The endocrine system and adolescence

Today we will discuss about thelocation and functions of secreted hormones of adrenal and Pancreas

Q5 Write location hormone secreted main functions and deficiency diseases of pancreas and adrenal glands

Endocrine Glands

Location Hormones secreted

Functions and Deficiency Diseases

1Adrenal gland

2 Pancreas Gland

On the top of each kidney

In between stomach and small intestine

i)Adrenaline from adrenal medulla

ii)Cortisone from adrenal cortex

i) Insulin

ii) Glucagon

It helps a person deal with any kind of emergency situation or emotional stressIt increases the heart beat rate of respiration and blood pressure

a) It regulates carbohydrates protein and fat metabolism

b) It regulates the salt and water balance in the body

a) It changes excess glucose into glycogen

b) It stimulates the cells to burn extra glucose to provide heat amp energy

Less secretion causes diabetes mellitus

Excessive secretions causeinsulin shock

a) It stimulates the breakdown of glycogen into glucose

b) It increases the level of glucose in blood

History Traders to rulers The Battle of Buxar was fought on 22 October 1764 between the forces under the command of the British East India Company led by Hector Munro and the combined armies of Mir Qasim the Nawab of Bengal till 1763 Mir Jafar was made the Nawab of Bengal for a second time in 1763 by the Company just after the battle After being defeated in 4 battles in katwa and Udaynala the Nawab of Awadh Siraj id Daula and the Mughal emperor Shah Alam II accompanied by Raja Balwant Singh of Kashi made an alliance with Mir Qasim The battle was fought at Buxar a small fortified

Answer the following questions- Short note-Battle of BuxarHomework-learn

town within the territory of Bihar located on the banks of the Ganga river about 130 kilometres (81 mi) west of Patna it was a decisive victory for the British East India Company The war was brought to an end by the Treaty of Allahabad in 1765

EnglishLiterature

The west wind-John Mansfield

In the poem The West Wind by John Masefield the poet starts by describingwith very poetic imagery of birds how the west wind is different from other winds its a warm wind full of birds cries There is a touch of melancholy perhaps home-sickness as he describes how it brings tears too and memories from an old land He goes on to describe the restful pastoral beauty of the land where even the dead can lie in the green He then brings in voicesperhaps of family and friends calling him home as he is missing Aprils beautyThe voices then tempt him some more with idyllic images from home (white blossom young green cornrunning rabbitswarm sun) The voices seem to presume that the poets heart is sorrowful bruised and soreThe end of the poem sees the poet appear to make a decision he will go home as he has decided that is where he truly belongs

Write the synopsis of the following words

1 Daffodils- a tall yellow flower that grows in the spring

2 Orchards- a piece of land on which fruit trees are grown

3 Blossom- a flower or a mass of flowers especially on a fruit tree in spring

4 Thrushes- a bird5 Larks- a small brown bird that

makes a pleasant sound6 Bruised- an injury7 Aching- pain 8 Tread- to put your foot down

while you are walking9 Balm-10 May-11 Fluting-

(Write from the book in your copy)

MAT

HEM

ATIC

S

Ch 1

1Al

gebr

ic E

xpre

ssio

n

1 Constant A symbol which has fixed value is called a constant[eg 8 23 -15 radic3 etc]

2 VariableA symbol which does not have any fixed value but may be assigned value (values) according to the requirement is called variable or literal[eg x y p q etc]

3 TermsA term is a number (constant) a variable a combination (product or quotient) of numbers and variables[eg 7 x 5x etc]

4 Algebric expressionA single term or acombination of two or more terms connected by plus (+) or minus (-) sign forms an algebraic expression[eg 5-y 3x2-5x xy-6z+4 etc]

5 PolynomialAn algebraic expression which contains more than one term is called a polynomial (multinomial)[eg x2-5x 5y+xy+x2y etc]

6 Degree of polynomial(a) When the polynomial contains only one variable the highest power of the variable is the degree of the polynomialeg the degree of the polynomial of 4x-7x5+8 is 5(b) When the polynomial contains two or more variablesStep (i) Find the powers of the variables in each term (ii) The highest sum of the powers is taken to be the degree of the polynomialeg the degree of the polynomial 5x2y-4x3y5+6 is = 3+5 = 8Remember An algebraic expression is a polynomial if degree of each term used in it is a non-negative integer

Exercise ndash 11(A)

1 Separate the constants and variables from the following

-7 7+x 7x+yz radic5 radic xy 3 yz

8 45y -3x

Solution Constant Variables-7 radic5 7+x 7x+yz radic xy

3 yz8

45y -3x

2 Write the number of terms in each of the following polynomials(i) 5x2+3timesax (ii) axdivide4-7 (iii) ax-by+ytimesz (iv) 23+atimesbdivide2

Solution Polynomials Number of terms(i) 5x2+3timesax 2(ii) axdivide4-7 2(iii) ax-by+ytimesz 3(iv) 23+atimesbdivide2 2

4 Write the degree of the each polynomials(i) xy+7z (ii) x2-6x3+8 (iii) y-6y2+5y8 (iv) xyz-3 (vi) x5y7-8x3y8+10x4y4z4

Solution Polynomials Degree(i) xy+7z 2(ii) x2-6x3+8 3(iii) y-6y2+5y8 8(iv) xyz-3 3(vi)x5y7-8x3y8+10x4y4z4 12

5Write the coefficient of(i) ab in 7abx (iv) 8 in a2-8ax+a (v) 4xy in x2-4xy+y2

SolutionCoefficient

(i) ab in 7abx 7x(iv) 8 in a2-8ax+a -ax(v) 4xy in x2-4xy+y2 -1

7 CoefficientAny factor of an algebraic quantity is called the coefficient of the remaining quantityeg in the algebraic term 7xyz 7 is coefficient of xyz 7x is coefficient of yz and so on

8 Like term The terms having the same literal coefficient are called like terms and those having different literal coefficients are called unlike terms

eg (i) 5xyz 8xyz -6xyz and 23xyz are like

terms(ii) 7xy2 8x2yz and -15xyz2 are unlike terms

6 in 57xy2z3 write the coefficient of

(i) 5 (vii) 5xy2 (viii) 17yz (xi) 5xyz

Solution Coefficient

(i) 5 17

xy2z3

(vii) 5xy2 17z3

(viii) 17yz

5xyzsup2

(xi) 5xyz 17yz2

7 In polynomial given below separate the like terms(ii) y2z3 xy2z3 -58x2yz -4y2z3 -8xz3y2 3x2yz and 2z3y2

Solution y2z3 -4y2z3 2z3y2 are like terms

xy2z3 -8xz3y2 are like terms

-58x2yz 3x2yz are like terms

Class IXSubject Topic Summary Execution

Bengali (2nd language)

বাগzwnjধারাzwnj বা ধারা-বা ধারা ল হিবকেশষ পরকার বাক -হিb -াকেবর এক হিবকেশষ পরকাশরীহিত াকেক কতগকেলা কার সমষটির মকেধয এগহিলকেক বা ধারা বকেল আবার কতগকেলা শকেবদর বাধাধরা যকান রীহিত যনই য-াকেব চকেল আসকে যসই -াকেবই চকেল আসকে তখন যসই শবদগহিল খন একক -াকেব অG পরকাশ ককের তখন একের বা ধারা বকেল বা ধারার পরকেয়া -াষাকেক আরও সFর ককের যতাকেল

অকাল পকক(অপহিরনত বয়কেস পাকাহিম)-মাতর শ বর বয়কেস যমকেয়টির া মকেখর কা তাকেত অকালপককতা ধরা পকে

অককা পাওয়া( মারা াওয়া) ndash পকেকIমারটি পকেকIমারকেত হিকেয় বাসাতরীকের াকেত মার যখকেত যখকেত অককা যপল

অহি| পরীকষা ( কঠিন ও পরকত পরীকষা)- যকেলটির আজ ডাকতাহির যরজালট যবকেরাকেব এIাই তার জীবকেনর ব অহি| পরীকষা

অষটরমভা (ফাহিক) ndash রীতা মকেখই বকো বকো কা বকেল আর কাকেজর যবলায় অষটরমভা

অকমGার ধাী (অপাG) ndash সমনকেক হিনকেয় যকান ান কেব না ও একেকবাকেরই অকমGার ধাী

অকেনধর ষটি (অসাকেয়র সায়)- আহিশ বকেরর বকোর নাহিত ল অকেনধর ষটি তাকেক াা বকোর একম চকেল না

আকেককল গড়ম (তবহিদধ)- ার তহিম উপকার করকেল যসই যতামার হিবরকেদধ সাকষয হিকেয়কে শকেনই আমার আকেককল গড়ম

আষাকে লপ( অবাসতব লপ) ndashIাকা এখন যকেব না এIা বলকেলই ত এমন আষাকে লপ ফাার যকান রকার হিল না

Hindi- महायजञ ा इस हानी म लख न या बतान ा परयास किया ह कि किसी भी अचछ

2nd language

परसार(यशपाल ाय या पणय न ा फल अवशय मिमलता ह ोई भी परोपार अथवा पणय लिलए किया गया ाय बार नही 0ाता वह ए परार ा यजञ हए धनी सठ थ धम परायण और किवनमर सठ न आन ी यजञ किए थ और दान म न 0ान कितना धन दिदन दखिखयो म बात दिदया थादिदन पलट और सठ यहा गरीबी आ गई उन दिदनो यजञ बचन ी परथा थी सठ भी अपनी 0गह बचन लिलए डलपर ए सट यहा चलन ो तयार हए सठानी रासत लिलए रोटी पड म बाधर सठ ो द दी रासत म ए भख R ो दखर सठ न चारो रोटी उसो खिखला दी खर वह सठ यहा डलपर पहच तो उनी सठानी न उस महायजञ बचन ो हा यदिद बचन आए सठ न R ो रोटी खिखलान ो महायजञ नही समझा और वापस लौट आया घर आर शाम ो उसी घर म उस ए बडा ख0ाना मिमला 0ो उस दवारा किए गएrsquo महायजञrsquo ा परसार था

English language

Letter formal The heading the name and address of the person you are writing to must be included beneath your own address In formal letters ldquoblock stylerdquo of address is preferred

Subject complain in brief

Salutation If the person you are writing to is known to you you may begin ldquoDear MrrdquoOr ldquoDear Mrsrdquo In all other instances you should begin ldquoDear Sirrdquo or ldquoDear Madamrdquo Or ldquoSirsrdquo

The body A formal or business letter has four partsReference The letter should begin by referring to a letter you have received an advertisement or the reason that has prompted you to writeInformation In the second paragraph it is necessary to supply more detailed information that is related to the referencePurpose Here you must give the reason why you are writing the letter This must be stated clearly and ensure that it is relevant to the question that has been setConclusion round off the letter with some polite remarkThe subscription when a letter has begun with dear sir sirs Madam you should end with Yours faithfully or yours truly When however you address a person by name you must conclude with the words ldquoYours sincerelyrdquo

1 A park in your locality is slowly being used as a rubbish dump Write a letter to the Mayor of your city pointing out the nuisance and danger of this Request that action be taken to stop this immediately

Or2 You being a boarder ordered a set of lab manuals from a famous book shop in the town They sent you a wrong set of books Write a letter to the manager of the book shop

Chemistry Chapter-1 1)CHEMICAL FORMULA- Q What is the Significance of

L-2The Language of Chemistrybull Chemical Formula

Itrsquos a symbolic representation of a chemical substance eg ndash The formula of Sulphuric acid is H2SO4

2) Steps of writing Chemical Formula of a given substance-

1 Write the symbols of the constituent atoms or radicals side by side Keep the basic radical on LHS and acid radical on the RHS ( Na+Cl- )2 In case of a radical having more than one atom( compound radical) enclose the radical in a bracket eg (SO4-)3 Write the valencies of each radical on its right hand top4 If the valencies of the two radicals are divisible by a common factor then divide the valencies by the common factor5 Invert (criss-cross) the valency number ie write the valency of one atom below the second atom and vice versa 6 On interchanging if valency number is lsquoone the figure lsquoonersquo is never writtenFor Example- Compound -Calcium Nitrate1 Writing the symbols- Ca(NO3)2 Writing the valencies on their right hand top- Ca2(NO3)1

3 Valency numeral in simple ratio- Ca2(NO3)1

4 Criss-cross- Ca 2NO3 1

5 Writing the formula of the compound- Ca(NO3)2

Chemical formula

A The formula of a substance conveys the following information regarding a substance 1 The name of the substance (qualitative)2 The elements constituting the substance (qualitative)3 The number of various atoms present in a molecule of the substance (quantitative)4 Molecular weight of the substance and the relative weights of different elements present in it (qualitative)

Q What are the limitations of Chemical Formula

A The chemical formula suffers from the following limitations-I It fails to convey whether the elements in a molecule are present in the form of atoms or ionsFor example the formula KBr fails to tell us whether Potassium and Bromine are present in the form of ions II It does not tell anything about the binding force that holds atom in a molecule togetherIII It does not tell us about the arrangement of various atoms with respect to one another within the molecule

Q Examples of Some Chemicals with their Formula Chemical name and Common Name-

A Given in the class notesCommercial Studies

Joint Stock Company

Let us discuss about the demerits of Joint Stock CompanyDespite so many advantages it has got many disadvantages which are as follows

Difficulty in FormationDelay in Decision makingExcessive Government ControlLack of Secrecy

Company can be classified into several categories based on incorporation

QuestionExplain the demerits of Joint Stock CompanyAnswer) 1 Difficulty in Formation The legal requirements and formalities required to be completed are so many The cost involved is quite heavy It has to approach large number of people for its capital It cannot start its business unless certificate of incorporation has been obtained This is granted after a long time when all the formalities are completed

Chartered CompanyStatutory CompanyRegistered Company

Delay in Decision making In this form of organization decisions are not made by single individual All important decisions are taken by the Board of Directors Decision-making process is time-consuming So many opportunities may be costly because of delay in decision-making Promptness of decisions which is a common feature of sole trader ship and partnership is not found in a company

Excessive Government ControlA company and the management have to function well within the law and the provisions of Companies Act are quite elaborate and complex At every step it is necessary to comply with its provisions lest the company and the management should be penalized The penalties are quite heavy and in several cases officers in default can be punished with imprisonment This hampers the proper functioning of the company

Lack of Secrecy The management of companies remains in the hands of many persons Every important thing is discussed in the meetings of Board of Directors Hence secrets of the business cannot be maintained In case of sole proprietorship and partnership forms of organisation such secrecy is possible because a few persons are involved in the management

2 Define the following

Chartered Company- The crown in exercise of the royal prerogative has power to create a corporation by the grant of a charter to persons assenting to be incorporated Such companies or corporations are known as chartered companies Examples of this type of companies are Bank of England (1694) East India Company (1600) The powers and the nature of business of a chartered company are defined by the charter which incorporates it After the country attained independence these types of companies do not exist

in IndiaStatutory Company- A company may be incorporated by means of a special Act of the Parliament or any state legislature Such companies are called statutory companies Instances of statutory companies in India are Reserve Bank of India the Life Insurance Corporation of India the Food Corporation of India etc The provisions of the Companies Act 1956 apply to statutory companies except where the said provisions are inconsistent with the provisions of the Act creating them Statutory companies are mostly invested with compulsory powersRegistered companiesCompanies registered under the Companies Act 1956 or earlier Companies Acts are called registered companies Such companies come into existence when they are registered under the Companies Act and a certificate of incorporation is granted to them by the Registrar

Economics

Chapter-4Basic problems of Economy

Today let us discuss with the topic Production Possibility curve

QuestionExplain the concept of Production Possibility Curve with the help of diagram

Answer) Production Possibility curve is a locus of all possible combinations of two commodities which can be produced in a country with its given resources and technology

The above diagram shows that with the given resources and technology the economy can produce maximum either 5 thousand meters of cloth or 15 thousand quintals of wheat or any other combination of the two goods like B( 1 thousand meters of cloth and 14 thousand quintals of wheat C ( 2 thousands meters of cloth and 12 thousand quintals of wheat) etcProduction Possibility curve is also called production possibility boundary or frontier as it sets the maximum limit of what it is possible to produce with given resources

Geography

Rotationand Revolution

SUNrsquoS POSITION AND SEASONAL CHANGES EQUINOXES ndash SPRING AND AUTUMN

Q1 What is Spring EquinoxA1 On 21st March sunrays fall directly on the equator On that day

As the Equator divides the Earth into two equal halves the sun rays fall directly on the equator twice in a year Equinoxes means equal Spring EquinoxOn 21st March sunrays fall directly on the equator On that day the duration of day and night both are equal ( 12 hours day and 12 hours night) on every places located on equator This day is called as Spring EquinoxAutumn EquinoxOn 23rd September sunrays fall directly on the equator On that day the duration of day and night both are equal ( 12 hours day and 12 hours night) on every places located on equator This day is called as Autumn Equinox

SOLSTICES ndash SUMMER AND WINTERDue to inclination of the Earth on its axis and the apparent movement of the sun the sun rays fall directly on both tropics once in a year Solstice is a Latin word which mean ldquothe Sun standing stillrdquoSummer SolsticesAfter 21st March there is an apparent movement of the Sun to the north of the equator The apparent northward movement up to 21st June when the Sun appears overhead at the Tropic of Cancer (22frac12degN) The sun appears to stand still at this position and then moves southwards towards the equator This position of the Sun on 21st June is known as Summer Solstices On that day the duration of day and night both are equal ( 12 hours day and 12 hours night) on every places located on Tropic of Cancer (22frac12degN)Winter solstices The apparent southward movement of the Sun continues beyond the equator till 22nd

December On this day the Sun is overhead at the Tropic of Capricorn

the duration of day and night both are equal ( 12 hours day and 12 hours night) on every places located on equator This day is called as Spring Equinox

Q2 What do you mean by EquinoxA2 Equinoxes means equal It is use to explain the equal duration of day and night ( 12 hours day and 12 hours night) on the Earth

Q3 On which date the longest day in Tropic of CancerA3 21st June

Q4 What is the meaning of SolsticeA4 Solstice is a Latin word which mean ldquothe Sun standing stillrdquo

Q5 Which is the longest day in southern hemisphereA5 22nd December

Q6 On what date does the Arctic Circle experience the lsquoMidnight SunrsquoA6 On 21 June the Arctic Circle experiences the lsquoMidnight Sunrsquo

Q7 What is cause of Midnight Sun in NorwayA7 During the summer solstice (21 June) the North Pole is inclined towards the Sun Therefore the duration of sunlight or daytime increases from 12 hours at the Equator to 24 hours at the Arctic Circle and beyond Thatrsquos why The region beyond the Arctic Circle especially Norway is known as the Land of the Midnight Sun because there the Sun does not rise or set on 21 June

Q8 Match the column A with BA B

Summer Solstice 21st March

Autumn Equinox 23rd

September

Winter Solstice 21st June

(22frac12degS) This position of the Sun is referred to as the Winter Solstice because it marks the winter season in the Northern Hemisphere On that day the duration of day and night both are equal ( 12 hours day and 12 hours night) on every places located on Tropic of Capricorn (22frac12degS)SEASONS AND DURATION OF DAY AND NIGHT During the equinoxes all places on the Earth have 12 hours of day and 12 hours of night Due to the revolution of the Earth round the Sun on an inclined axis the duration of day and night varies according to seasons and the latitude of a placeDuring the summer solstice (21 June) the North Pole is inclined towards the Sun Therefore the duration of sunlight or daytime increases from 12 hours at the Equator to 24 hours at the Arctic Circle and beyondThe region beyond the Arctic Circle especially Norway is known as the Land of the Midnight Sun because there the Sun does not rise or set on 21 JuneAt the North Pole there will be six months of daylight The Sun will be seen always above the horizon at a low angle At 66degN 24 hours of sunlight can be seen only on 21 June Hammerfest in northern Norway is a place of tourist attraction for observing the phenomenon of the Midnight Sun This place has continuous daylight from 13 May to 29 July This place is easily accessible to tourists and has hotels and other facilities The view of the midnight Sun from here is enthrallingIn the Southern Hemisphere the duration of daylight decreases from 12 hours at the equator to 0 hours beyond the Antarctic Circle In the South Polar Region there is 24 hours of darkness The Sun is always below the horizon In the Southern Hemisphere which experiences winter the duration of night-time is longer than the duration of daylight

Spring Equinox 22nd

December

A8 A B

Summer Solstice 21st June

Autumn Equinox 23rd

September

Winter Solstice 22nd

December

Spring Equinox 21st March

During winter solstice (22 December) the South Pole is inclined towards the Sun The Southern Hemisphere experiences summer and the Northern Hemisphere has winter Therefore the duration of daylight or sunlight is greater in the Southern Hemisphere than in the Northern HemisphereThe duration of daylight increases from 12 hours at the equator to 24 hours beyond the Antarctic Circle The South Polar Region has 24 hours of sunlight for many days continuously At the South Pole there will be six months of sunlight The Sun will always be seen at a low angle above the horizon In the Northern Hemisphere the duration of daylight will decrease from 12 hours at the equator to 0 hours at the Arctic Circle There are 24 hours of darkness in the North Polar region The duration of night is greater than the duration of daylight as one move northwards from the Equator It is evident from the above table that the duration of daylight is 12 hours throughout the year at the equator only As one moves away from the equator the seasonal variations in the duration of daylight increase The seasonal variations in the duration of daylight are maximum at the Polar Regions

Subject Eng Literature (The Merchant of Venice ndash William Shakespeare)Topic Act II Scene 7 Lines 36 to 80 (End of scene ) [Students should read the original play and also the paraphrase provided]

Summary Questions amp AnswersThe Prince then examines the inscription on the silver casket which says ldquoWho chooseth me shall get as much as he deservesrdquo The Prince says that he deserves Portia more than anybody else because of his high rank his noble birth and his great wealth and power But then he argues that silver is ten times

(1) (Act II Sc 7 L 39-47)

From the four corners of the earth they come

To kiss this shrine this mortal breathing saint

The Hyrcanian deserts and the vasty wildsOf wide Arabia are as through-fares now

inferior to gold and therefore he cannot believe that the portrait of such a beautiful lady as Portia can be contained in the silver casket He decides to see the inscription on the golden casket before making his decision

The Prince goes to examine the inscription on the golden casket which says ldquoWho chooseth me shall get what many men desirerdquo The Prince believes that the whole world desires to possess Portia otherwise so many suitors would not have come from all corners of the world for winning Portia Some of them have come from the distant lands of Persia and Arabia The deserts of Persia (Hyrcanian deserts) and the boundless desolate lands of Arabia have been crossed by the Princes seeking the hand of Portia He contrasts this casket containing Portiarsquos portrait with the old English gold coin bearing the image of the archangel (angel of the highest rank) He goes on to remark that while the figure of the archangel is engraved (Insculped) upon the English coin the picture of Portia who is beautiful as an angel lies hidden inside one of the caskets namely the Golden Casket (Golden Bed)

On the basis of his assessment of the inscription on the golden casket the Prince decides to choose the golden casket He asks for the key and opens the golden casket only to find therein an empty human skull holding a roll of

For princes to come view fair PortiaThe watery kingdom whose ambitious headSpets in the face of heaven is no barTo stop the foreign spirits but they comeAs orsquoer a brook to see fair Portia

(i) Explain the occasion for the above mentioned speech

These are the comments of the Prince of Morocco after he reads the inscription on the golden casket His mental process is revealed to us in these words We find him debating within himself as to which casket he should choose

(ii) What light does the above speech throw on the personality of Prince of Morocco

From the above mentioned speech we come to know that the Prince of Morocco is keen to marry Portia He is the type of person who is easily taken away by outward appearance He is in love with Portia because of her beauty

(iii) What information can you gather about Portia from the above mentioned lines

The given speech shows that Portia is a very beautiful lady She must be possessed of good qualities because many suitors come to her place from all over the world with a desire to get married to her The Prince of Morocco is so impressed by her beauty that he calls her a saint According to him the whole world is desirous of having her

(iv) Elucidate the significance of the first two lines

In these lines the Prince of Morocco pays a compliment to Portia These lines show his admiration for her He says that people come from all parts of the world to see fair Portia

(v) Explain the meaning of the last four lines of the

passage

In these lines the Prince of Morocco says that even the vast oceans which throw a challenge at the sky are unable to prevent men from coming to Portiarsquos place to have a glimpse of her These lines are also a tribute to Portiarsquos beauty and good qualities Many men voyage across the ocean treating it as a mere stream to see the beautiful Portia

paper in which is written that whoever happens to be guided by the glitter of things is invariably deceived

On reading the scroll the Prince says that he is too sad at heart to speak a more formal farewell and leaves with his followers amidst a sound of trumpets

After the Prince of Morocco leaves Portia remarks that the Prince is a gentle fellow but she is rid of him May all persons of his nature make a similar choice

IMPORTANT PASSAGES EXPLAINED

(Act II Sc 7 L 39-43)From the four corners of the earth they come

To kiss this shrine this mortal breathing saintThe Hyrcanian deserts and the vasty wildsOf wide Arabia are as through-fares nowFor princes to come view fair Portia

Context

This passage occurs in Act II Scene 7 in The Merchant of Venice This is part of the speech made by the Prince of Morocco

(2)

(Act II Sc 7 L 48-53)

MOROCCO One of these three contains her heavenly pictureIst like that lead contains her

Twere damnation To think so base a thought it were too grossTo rib her cerecloth in the obscure graveOr shall I think in silver shes immurdBeing ten times undervalued to tried gold

(i) What meaning does the Prince of Morocco find out of the inscription of the golden casket What have Belmont and Portiarsquos house been called and why

The inscription on the golden casket is ldquoWho chooseth me shall gain what many men desirerdquo The Prince finds out that it means that the chooser of the golden casket will get Portia because many men desire her In fact the entire world desires her Because of the coming of many suitors to Belmont from different countries in order to win Portiarsquos hand Belmont has become a centre of pilgrimage and her house is the shrine where saintly Portia is installed

(ii) What does the Prince of Morocco do before making the final choice of the casket Which is the correct casket and who will win Portiarsquos hand

The Prince of Morocco surveys and analyses the inscriptions on the casket of lead silver and gold Before making the final choice like a very systematic and methodical person he once again considers the claims of the caskets The casket containing Portiarsquos picture is the correct casket and the person choosing it will win Portiarsquos hand

Explanation

While praising Portia the Prince of Morocco conceives Portia as a goddess whose image is placed inside one of the caskets Many suitors are coming from far and wide the north and the south the east and the west (Four corners) in order to try their luck Some of them have come from the distant land of Persia and Arabia The deserts of Persia (Hyrcanian deserts) and the boundless desolate lands of Arabia have been crossed by the Princes seeking the hand of Portia All this shows that Portia is indeed the most beautiful lady of the world

(iii) What does the Prince of Morocco say in his estimation while examining the motto on the silver casket What does he find in the golden casket

While examining the motto on the silver casket which says ldquoWho chooseth me shall get as much as he deservesrdquo Morocco says that in his own estimation he surely deserves Portia in all respects ndash rank birth wealth etc

He chooses the golden casket When he opens it he finds an empty human skull holding a scroll in which it is written that those who are attracted by the glittering outside of things are always deceived as Morocco has been deceived

(iv) What kind of nature does the Prince of Morocco have

The Prince of Morocco has a simple nature who does not look deeply into the inner meaning of things but is dazzled by the outward appearance of gold He is inclined to over-estimate his own value and does not realize that it is a duty to ldquogive and hazardrdquo To say that he will not hazard for lead shows that he misreads the true meaning of the inscription which is that he should be prepared to ldquohazard all he hathrdquo for Portia So his feeling is only one of fascination and romantic attraction

(v) Do you think that the lottery of the caskets is not a matter that will be determined by chance

In fact the lottery of the casket is not a matter that will be determined by mere chance but that it is a true test of character and of sincerity which is amply proved not only by Moroccorsquos choice but also by the arguments which he uses to help him in his choice

(Act II Sc 7 L 55-59)

They have in England

A coin that bears the figure of an angelStamped in gold but thats insculpd uponBut here an angel in a golden bedLies all within

Context

(3)

(Act II Sc 7 L 63-77)A carrion Death within whose empty eye

There is a written scroll Ill read the writing

All that glisters is not goldOften have you heard that toldMany a man his life hath soldBut my outside to beholdGilded tombs do worms infoldHad you been as wise as boldYoung in limbs in judgment oldYour answer had not been inscrolld

This passage occurs in Act II Scene 7 in The Merchant of Venice This is part of the speech made by the Prince of Morocco

Explanation

In this passage the Prince of Morocco bestows high praise on Portia whose hand he is seeking He contrasts this casket containing Portiarsquos portrait with the old English gold coin bearing the image of the archangel (angel of the highest rank) He goes on to remark that while the figure of the archangel is engraved (Insculped) upon the English coin the picture of Portia who is beautiful as an angel lies hidden inside one of the caskets namely the Golden Casket (Golden Bed) In the day of Elizabeth silver was ten times inferior in value to gold Therefore the Prince of Morocco believing that Portiarsquos portrait is contained in the Golden Casket decides to choose the Golden Casket

Fare you well your suit is coldCold indeed and labour lostThen farewell heat and welcome frostmdashPortia adieu I have too grievd a heartTo take a tedious leave Thus losers part

(i) What reward does the Prince of Morocco get after making a wrong choice of the Casket How does he feel

After making the wrong choice in selecting the casket of gold the Prince of Morocco as a reward earns a rebuke in the form of a scroll tucked in the empty eye-socket of a skull kept in the casket of gold The Prince is shocked and disappointed He becomes all the more sad and dejected when he reads the scroll which points to his foolishness in being misled by the appearance and outward show as indicative of its worth

(ii) How does the Prince respond after reading the scroll

After reading the scroll the Prince though upset accepts the result with good grace and decorum befitting a royal suitor and true sportsman He says that his love-suit is really cold otherwise he would have chosen correctly but now his efforts have been in vain So he bids farewell to Portia to the warmth and enthusiasm of love and welcomes the cold and bitterness of dejection and misery of life which lies ahead

(iii) What request does he make to Portia and why

After being failure in his mission he requests Portia to give him permission to leave at once because he is too sad to undergo the tediousness of a formal leave-taking He tells that it is the manner in which defeated persons part unceremoniously

(iv) Explain the following lines

ldquoAll that glisters is not goldOften have you heard that toldMany a man his life hath soldBut my outside to beholdGilded tombs do worms infoldrdquo

Mere glitter does not make a metal to be gold Man has often been warned against appearance but it has been of no use Many people have sacrificed their lives only to seek the outer appearance of gold Worms are found inside the gilded

monuments

Class XSubject Topic Summary Execution

Hindi 2ndlang

नया रासता भाग 6 मायाराम 0ी घर म धनी मल 0ी और उनी बटी सरिरता ी ही चचा बनी रहती थी अमिमत ो इसम ोई रलिच ना थी वह धनी घर ी लडी स शादी र सवय ो बचना नही चाहता था उसा भी सवाणिभमान ह ईशवर ी पा

स उस पास पस ी ोई मी नही थी अभी उसन फकटरी ही लगाई थी उसी समझ बाहर था कि उस घर वालो ा झाव पस ी तरफ कयो

ह उसन मा स सवाल किया कि मा तम सरिरता स मरी शादी कयो रना चाहती हो मा न उस समझाया कि वह दखन म बरी नही ह और किफर खानदान अचछा

ह वह ए शल गरहणी रप म घर सभाल सगी अमिमत न मा ो इस बात ा एहसास राया कि मीन सबध लिलए मना रन पर उस दिदल

पर कया बीती होगी मा और अमिमत ी लडी बार म ाफी बात हईमा ा झाव सरिरता ी तरफ था कयोकि वह घर पर अचछा दह0 लर आ रही

थी अमिमत न अपनी मौसी ी बरी हालत बार म बताया कि किस तरह वह बड घर ी खानदानी बटी लाई थी और आ0 उसी हालत कितनी खराब ह लाई थी बहकलब 0ाती ह और बचचो ो भी नही दखती ह बात चल ही रही

थी कि तभी ए ार बाहर आर री धनी मल0ी घर अदर आए और पीछ स डराइवर फल ी ए टोरी लर आया अदर आए और पीछ स

डराइवर ए टोरी फल ी लर आया अमिमत ो फल ी पटी बरी लग रही थी अमिमत न पछ लिलया यह फल कयो ल आए ह प इन सब ी कया

0ररत थी उनो न 0वाब दिदया कि 4 पटी शमीर स मगाए थ अमिमत ो या सनर करोध आ गया तभी उस किपता 0ी आ गए उन आत ही अमिमत उठर बाहर चला गया वहा वहा मा पास आर बठ गया और बोला

अभी रिरशता तय नही हआ और धनी मल 0ी धनी मल 0ी फल ी पटी लर चलआय मा न समझाया कि 0ब सबध 0ड 0ाता ह तो खाली हाथ नही

आत अमिमत न मा स हा कि तम सबन सरिरता ो इस घर म लान ी ठान रखी ह धनीमल 0ी उस दिदन सरिरता ो दखन ी तारीख तय रन आय थ

Commercial Studies

Banking Nowadays Bank provide easy and quick services through internet facilities methods of Banking is called internet bankingIn order to save the time and money involved in visiting Bank branches people increasingly prefer to have internet banking

There are different modes of doing internet banking or transferring money through online They areReal Time Gross Settlement (RTGS)National Electronic Fund Transfers (NEFT)

1

Question

1) Explain the term RTGS Write the features of RTGS

Answer)The acronym RTGS stands for Real Time Gross Settlement which may be defined as the continuous real time settlement of funds transfer individually on and order by order basis without netting lsquoReal timersquo may be defined as the processing of instructions at the time they are received rather than at some letter time lsquoGross settlementrsquo may be defined as the settlement of transfer instructions which occurs

individually

Features of RTGS1It is the continuous settlement of

funds transfer individually on an order by order basis

2RTGS facility is provided only by CBS core banking solution enabled Bank branches

3Amount charged from the customer for RTGS transactions vary from bank to bank

2) Explain the term NEFT Write the features of NEFT

Answer) National electronic funds transfer may be defined as a nationwide system that facilitates individuals Farms and copper operates to electronically transfer funds from any bank branch to any individual farm or corporate having an account with any other bank branch in the country

Features of NEFT2 Transfer can be made 7 times on

weekdays and 6 times on Saturday

3 NEFT cannot be used to receive foreign remittances

4 NEFT transaction takes place in batches

5 A bank branch must be NEFT enabled to become a part of NEFT fund transfer network

6 There is no maximum or minimum amount that can be transferred through NEFT when one bank has a bank account

English Language

CompositionEssay

A composition is an art of creating a piece of writing on any topic or subject It is the writing correctly beautifully and clearly in order to make some interesting reading Structure of the composition

Introduction ( you lay the foundation for your composition)

Body (it constitutes the main part of the essay)

Conclusion (final statement that leaves a lasting impression)

Kinds of essays1 The Narrative essay2 The descriptive essay3 The reflective essay4 The argumentative essay

Write a composition on any one of the following topics (350- 400 words)

1 Friendship Or2 The first day of your school

Subject Eng Literature (The Merchant of Venice ndash William Shakespeare)Topic Act V Scene 1 Lines 127 to 158 (Nerissa helliphellip The clerk will nersquoer wear hair onrsquos face that had it) [Students should read the original play and also the paraphrase given in the school prescribed textbook]

Summary Revision Questions o Soon thereafter Bassanio Gratiano

and Antonio arrive

o Bassanio tells Portia that he is feeling as if it is morning because of the presence of Portia who is shining like the sun When Antonio is introduced by Bassanio to Portia she tells Bassanio that he should be grateful to Antonio who took so much trouble on his account even to the extent of risking his life

o Nerissa starts quarrelling with Gratiano and demands that he show her the ring she had presented to him and which she had warned him not to lose She suspects that Gratiano must have presented the ring to some young woman and not to the lawyerrsquos clerk as he repeatedly says and assures

Answer the following questions to check your preparation of Act IV Scenes 1 and 2

You must attempt only after you have completed your preparation of Act IV The answers must be in complete sentences using textual evidence (with citation) when necessary

[It would be in your own interest to attempt the above questions honestly totally refraining from consulting your textbook or your notes during answering After completion you should correct the paper yourself consulting the textbooknotes etc and award marks as specified Please let me know the marks you scored through WhatsApp in the group or to my personal WhatsApp]

Act IV Scene 1 (each question carries 2 marks)

1 What did the Duke try to do for Antonio

2 Why does Shylock refuse to show mercy How does he justify his stance

3 Why does Antonio say he is ready to die 4 What information is contained in Bellariorsquos letter

5 Why does Portia (as Balthazar) assert that Shylock must show mercy How does he respond

6 What offers are made to Shylock to get him to spare Antonio How are they received

7 What does Antoniorsquos speech as he faces the prospect of Shylockrsquos knife tell you about his character

8 How do Bassanio and Gratiano react to the looming prospect of Antoniorsquos demise

9 How does Portia (as Balthazar) use the law to turn the tables on Shylock

10 What does the Duke decree should happen to Shylock Why What happens to Shylockrsquos estate

11 What does Portia ask Bassanio as payment for her ldquoservicesrdquo What is his initial response What makes him change his mind

Act IV Scene 2 (each question carries 1frac12 marks)

1 What does Gratiano bring to Portia (Balthazar)

2 What does Nerissa plan on getting from Gratiano What does Portiarsquos comment suggest about men

ECO-10 280620 Topic-Supply AnalysisSHIFTING OF SUPPLY

But if there is change in factors other than the price of the commodity then either more is supplied at the same price or less supplied at the same price In such cases the price of the commodity remains constant but there is a change in other factors like change in the price of inputs change in technology of production change in price of other related goods change in taxation policy of the government etc For example there is an improvement in the technology of production of the commodity in question It leads to decrease in per unit of cost production of the commodity The firm is willing to sell more quantity of the commodity at the same price So the supply other commodity increases at the same price This increase in supply is shown by rightward shift of supply curve On the other hand if the firm uses inferior technology of production the cost of production per unit of the commodity increases The firm is willing to sell less quantity at the same price So the supply of the commodity decreases at the same price This decrease in supply is shown by leftward shift of the supply curve The above cases of increase and decrease in supply can be shown with the help of the following figures

Y INCREASE OF SUPPLY Price (Rs) s

P A s1

B

s

X` O s1 X

q q1

Y` Quantity demanded (in units)

Y DECREASE IN SUPPLY s2

s

price (Rs)

C

p A

s2

s

X` o X

q2 q

Y` Quantity demanded ( in units)

Main factors causing increase in supply or rightward shift of supply Curve(i) Fall in the price of other related goods

(ii) Fall in the price of inputsfactors(iii) Use of better technology in production(iv) Decrease in the rate of excise duty by government(v) If the objective of producer changes from profit maximization to salesMaximization

Main factors causing decrease in supply or leftward shift of supply curve(i) Increase in the price of other related goods(ii) Rise in the price of inputsfactors(iii) Use of inferior technology in production(iv) Increase in the rate of excise duty by the government(v) If the objective

Subject - Biology Topic ndash Chapter mdash6 PhotosynthesisSummary Execution

Today we will know about photosynthesis and its stages

Q1 What do you mean by photosynthesis The process by which living plants containing chlorophyll produce food

substances from carbon-di- oxide and water by using light energy Sunlight

6CO2 +12 H2O----------------------- C6 H12O6 + 6H2O + 6O2

Chlorophyll

Q2 What are the importance of photosynthesis I) Food for all Green plants trap solar energy by photosynthesis

process and supply food and energy for all living organisms either directly or indirectly

Ii) Oxygen to breathe in by product of photosynthesis is oxygen which is essential for all living organisms respiration

Q3 Write about two main phases of photosynthesis A Light dependent phase This phase occur in grana of chloroplast I) The chlorophyll on exposure to light energy becomes activated by

absorbing photons Ii) The absorbed energy is used in splitting the water molecules (H2O)

into its two components (H+ and OH- ) and releasing electron s 2H2O------------------------- 4H+ + 4e- +O2

Energy of 4 photons This reaction is known as photolysis

End products are H+ and oxygen water

B Light independent (Dark ) phase The reactions in this phase require no light energy

Here CO2 combine with H+ and produce glucose

Class XI

Subject Topic Summary ExecutionEVS Chapter-4 Legal

regimes for sustainable development

Environmental legislationEnvironmental legislation is the collection of laws and regulations pertaining to air quality water quality the wilderness endangered wildlife and other environmental factors The act ensures that matters important to the environment are thoroughly

Learn -The Forest (Conservation) Act 1980

considered in any decisions made by federal agencies

The Forest (Conservation) Act 1980 The Forest (Conservation) Act 1980 an Act of the Parliament of India to provide for the conservation of forests and for matters connected therewith or ancillary or incidental thereto It was further amended in 1988 This law extends to the whole of IndiaObjects and Reasons of the Forest Conservation Act

Deforestation causes ecological imbalance and leads to environmental deterioration Deforestation had been taking place on a large scale in the country and it had caused widespread concern The act seeks to check upon deforestation and de-reservation of forests

Subject Eng Literature (The Tempest ndash William Shakespeare) Topic Act II Scene 1 Lines 314 to 329 (End of scene)

[Students should read the original play and also the paraphrase given in the school prescribed textbook]Summary Questions amp Answers

Conspiracy of Antonio and Sebastian (Contd)

o As they approach Ariel appears again and wakes up Gonzalo by singing a tune in his ear Alonso also wakes up and they see both Sebastian and Antonio with drawn swords On being caught off guard they make up a story saying that they had heard a bellowing of bulls or lions

o They then moved to another part of the island

o Ariel at once rushes to Prospero to inform him of this development

SUMMING-UP of ACT-2 SCENE-1

(i) Among the survivors Ferdinand is separated from the rest which results in the disconsolate grief of Alonso as he took him for dead

(ii) The villainy of Antonio is confirmed

(iii) The supremacy of Prosperorsquos magic which resulted in the failure of the human conspiracy

(1)

(Act II Sc 1 L 311-325)SEBASTIAN Whiles we stood here securing your repose

Even now we heard a hollow burst of bellowing Like bulls or rather lions Didt not wake youIt struck mine ear most terribly

ALONSO I heard nothingANTONIO O rsquotwas a din to fright a monsters ear

To make an earthquake Sure it was the roarOf a whole herd of lions

ALONSO Heard you this GonzaloGONZALO Upon mine honour sir I heard a humming

And that a strange one too which did awake meI shaked you sir and cried As mine eyes opened I saw their weapons drawn There was a noiseThats verily rsquoTis best we stand upon our guardOr that we quit this place Lets draw our weapons

(i) Why has Prospero sent Ariel to Gonzalo and Alonso What does Ariel do to awaken Gonzalo

Prospero has already come to know by his magic powers the danger which threatens Gonzalo who had been Prosperorsquos friend and so he sent Ariel to preserve the lives of both Gonzalo and Alonso Prospero does not want that his scheme should remain unfulfilled Ariel begins to sing a song in Gonzalorsquos ears to awaken him(ii) Who are ready to carry out their plan Who takes steps to stop them Why does Gonzalo feel surprised after being awakened

Sebastian and Antonio are ready to carry out their plans They are standing with their swords drawn to kill Alonso and

(iv) We see two sets of contrasting characters Gonzalo-Adrian against Antonio-Sebastian

(v) The grief that works in Alonso can be perceived to his repentance for his association in Antoniorsquos crime against Prospero

Gonzalo Ariel takes steps to stop them from carrying out their nefarious scheme When Gonzalo is awakened by the song sung by Ariel into his ears he (Gonzalo) feels surprised because he sees Sebastian and Antonio standing with their swords drawn(iii) What reason do Sebastian and Antonio tell of drawing their swords when they are suspected by Alonso and Gonzalo

When Sebastian and Antonio are seen with their swords drawn they are looked with suspicion by Gonzalo and Alonso At first Sebastian tells them that as they stood here to guard them during their sleep they heard only a little before a sudden loud noise very much like the roaring of bulls or more probably that of lions Then Antonio follows him saying that this was a noise so terrible as to frighten even a monsterrsquos ears and this noise could even have shaken the earth and it was surely like the roaring of a multitude of lions Then seeing the danger they have drawn their swords Perhaps after hearing the terrible noise they (Gonzalo and Alonso) woke up from their sound sleep

(iv) What does Gonzalo tell Alonso about the strange noise What did he see on opening his eyes Gonzalo tells Alonso that he did not hear the sound of roaring but he heard a humming sound which was strange and which woke him up After waking up he gave him (Alonso) a shaking and a loud cry On opening his eyes he saw these two gentlemen standing with their swords drawn(v) What does Gonzalo suggest

Gonzalo suggests that there was a noise indeed and of that he has no doubt at all and suggests that the best course for them would be to remain alert and vigilant against any possible danger to their lives or to leave this place and move to some other part of the island

Class XIISubject Topic Summary Execution

Commerce

Chapter- Management

Today we will discuss about LEVELS OF MANAGEMENT

Levels of management is a series or chain of managerial positions from top to bottom It helps individuals to know their authority responsibilities and superior-subordinate relations among themselves There are mainly three levels of Management TOP LEVEL MANAGEMENTMIDDLE LEVEL MANAGEMENTLOWER LEVEL MANAGEMENT

Top level managementIt consists of members at the highest level in the management hierarchy This level includes Board Of Directors Chief Executive Managing Directors Chairman President Vice President

Rolefunctions of the top levelmanagement1To analyse evaluate and deal

with theexternal environment2 To determine the objectives and

policies of the business3 To strive for welfare and survival

of business

4 To create an organisational Framework consisting of authority responsibility relationship

Middle level management Congress of members or groups who are concerned with implementation of the policies let down by the top managementThis level includes head of the department such as finance manager marketing manager branch and regional managers departmental and divisional heads plant superintendent etc

Role of functions of the middle level management

1 To interpret the policies framed by top management

2 To assign duties and responsibilities to lower level managers

3 To select and appoint employees for middle and supervisory level and evaluate their performance

4 To co-operate with other departments for smooth functioning

Operational or supervisory level managementIt refers to the group are members who are concerned with execution of the work They are also known as fast line managers This level includes supervisor 4 men Section Officer clerk Inspector etc

Role of functions of the lower level management1 To plan and execute day-to-

day operations2 To supervise and control the workers3 To arrange materials and

tools to start the process and make arrangements for training

4 Today present workers grievance and suggestions before the management and

ensure safe and proper working conditions in the factory

Business Studies

Staff Appraisal Chapter- 10 Today let us start with a new chapter

Staff Appraisal

Meaning of Performance Appraisal

Performance Appraisal is the systematic evaluation of the performance of employees and to understand the abilities of a person for further growth and developmentThe supervisors measure the pay of employees and compare it with targets and plansThe supervisor analyses the factors behind work performances of employeesThe employers are in position to guide the employees for a better performance

Objectives of Performance Appraisal

Following are the objectives of Performance Appraisal

To maintain records in order to determine compensation packages wage structure salaries raises etc

To identify the strengths and weaknesses of employees to place right men on right job

To maintain and assess the potential present in a person for further growth and development

To provide a feedback to employees regarding their performance and related status

To provide a feedback to employees regarding their performance and related status

Importance of Performance Appraisal

Performance appraisal provides important and useful information for the assessment of employees skill

knowledge ability and overall job performance The following are the points which indicate the importance of performance appraisal in an organization

1 Performance appraisal helps supervisors to assess the work performance of their subordinates

2 Performance appraisal helps to assess the training and development needs of employees

3 Performance appraisal provides grounds for employees to correct their mistakes and it also provides proper guidance and criticism for employees development4 Performance appraisal provides reward for better performance

5 Performance appraisal helps to improve the communication system of the organization

6 Performance appraisal evaluates whether human resource programs being implemented in the organization have been effective

7 Performance appraisal helps to prepare pay structure for each employee working in the organization

8 Performance appraisal helps to review the potentiality of employees so that their future capability is anticipated

Geography

DRIANAGE The SubarnarekhaThe Subarnarekha and the Brahmaniinterposed between the Ganga and the Mahanadi deltas drain an area of 19300 sq kmand 39033 sq km respectively The drainage basins of these streams are shared byJharkhand Odisha west Bengal and Chhattisgarh The Brahmani is known as southKoel in its upper reaches in Jharkhand

The NarmadaThe Narmada rises in the Amarkantak hills of MadhyaPradesh It flows towards the West in a rift valleyformed due to a geological fault The total length of it is 1300 km All the tributaries of the

Q1 Name the two westward flowing rivers in the peninsular plateauA1 Narmada and Tapi are the only westward flowing rivers of the peninsular plateau

Q2 Differentiate between east-flowing rivers and west-flowing riversA2

East-flowing rivers

West-flowing rivers

Narmada are very short inlength Most of its tributaries join the main streamright anglesThe Narmada basin covers parts of Madhya Pradesh and Gujarat

The Tapi The Tapi rises in the Satpura ranges in the Betul listrictof Madhya Pradesh It flows in a rift valley parallel tothe Narmada but it is much shorter in length It coversparts of Madhya Pradesh Gujarat and MaharashtraThe length is about 724 km

The Sabarmati and the MahiThe Sabarmati rises in the Aravali hills and flows south-south-westwards for a distance of 300 kilometres to the Arabian Sea The Sabarmatibasin extends over an area of 21674 sq km in Rajasthan and Gujarat The Mahi rises inthe east of Udaipur and drains an area of 34842 sq km lying in Madhya PradeshRajasthan and Gujarat It flows south-westwards for a distance of 533 km before it fallsinto the Gulf of Khambhat

The ChambalThe Chambal rises near Mhow in the Vindhya Range and flows towards the northgenerally in a gorge upto Kota Below Kota it turns to the north-east direction and afterreaching Pinahat it turns to the east and runs nearly parallel to the Yamuna beforejoining it in the southern part of the Etawah district in Uttar PradeshMajor Rivers of India with their basin area (Sqkm)

Himalayan System Indus 321290Ganga 861404

Brahmaputra 187110Indus System

Jhelum 34775Beas 20303

Ganga System Yamuna 366223Ghaghra 127950

Peninsular RiversNarmada 98796

Tapi 65145Mahanadi 141600

Subarnarekha 19300Sabarmati 21674

Mahi 34842Godavari 312812

Godavari Krishna Kaveri Mahanadi are the east-flowing rivers

Narmada Tapi west-flowing rivers

They fall into the Bay of Bengal

They fall into Arabian Sea

These rivers form big deltas

These rivers form comparativelysmall deltas

Catchment areas of these rivers are larger

Catchment areas of these rivers are smaller

Krishna 2589488Cauveri 87900

Subject ndashBiology Topic ndashChapter -5 Inheritance amp Variations Summary ExecutionToday we will discussabout linkage and its classification

LINKAGE The tendency of the genes located on the same chromosome to stay together is

hereditary transmission Linked genes the genes responsible for this Genes that exhibit the process of linkage locates in the same chromosome The distance between the linked genes in a chromosome determines the strength

of linkage i e genes that are located close to each other show stronger linkage than that are located far from each other

COMPLETE LINKAGE It is the type of linkage showed by the genes that are closely located or are tightly

linked with each other as they have no chance of separatingby crossing over These genes are always transmitted together to the same gamete and the same

offspring In such condition only parental or non cross over type of gametes are formedINCOMPLETE KINKAGE It is type of linkage showed by the genes that are distantly located orare loosely

linked with each other because they have chance of separating by crossing over

SIGNIFICANCE i) It helps in holding the parental character togetherii) It checks the appearance of new recombination and helps in bringing the

hybrid population which resembles the original parents iii) Linked genes dilute the effects of undesirable traits

Subject Eng Literature (The Tempest ndash William Shakespeare) Topic Essay Questions (EQ-3)Question No 3

Give a character sketch of CalibanAnswer

The character of Caliban has been wonderfully conceived by Shakespeare as the manifestation of all that is gross and earthy ndash a sort of creature of the earth as Ariel is a sort of creature of the air

Calibanrsquos Physical Appearanceo Caliban is lsquofreckledrsquo a lsquomisshapen knaversquo not honoured with human shape

o Prospero calls him lsquothou tortoisersquo (Act I Sc 2 Line 317) Trinculo stumbling upon him describes him as ldquoA strange fish hellip Legged like a man And his fins like armsrdquo He ldquosmells like a fishrdquo (Act II Sc 2 Line 25)

o Prospero also calls him a ldquobeastrdquo (Act IV Sc 1 Line 140) and ldquoThis misshapen knaverdquo (Act V Sc 1 Line 268)

o Further it appears that in addition to his physical deformity his spiritual inferiority is also suggested by Prosperorsquos claim that his birth resulted from the union between his mother the witch Sycorax and the devil

Calibanrsquos ParentageWhen the play opens Caliban is twenty four years of age having been born on the island twelve years before the coming of Prospero His mother was the foul witch Sycorax who was banished from Algiers for ldquomischiefs manifold and sorceries terrible to enter human hearingrdquo (Act I Sc 2 Line 264) and the father was the Devil himself Thus

Caliban is a monster of evil and brute nature ugly deformed and stinking

Calibanrsquos Savage and Malignant Natureo Caliban is entirely a creature of the earth ndash gross brutal and savage He regards himself as the rightful possessor

of the island and Prospero as a usurper

o In his young age he was on good terms with Prospero He had consented to be received by Prospero at his house and to be educated by him He has learnt human language only to curse his master whom he abhors

o His beastly nature soon breaks out and ends in a vicious attack on Miranda This opens the eye of Prospero who becomes severe to him and enforces his service by threats and violence

o Prospero uses him to make dams for fish to fetch firewood scraper trenches wash dishes and keep his cell clean

Calibanrsquos Hatred for ProsperoA profound hatred for Prospero has taken hold of Caliban It springs from a sense of his being dispossessed and ill-treated He would kill Prospero if he could but he knows the power of Prosperorsquos lsquobookrsquo Hence he transfers his allegiance to Stephano who seems like a god to him He also incites the two drunken associates to batter the skull of Prospero when he sleeps in the afternoon

Caliban Shows Considerable Intelligenceo He has learnt Prosperorsquos language

ldquoYou taught me language and my profit onrsquot (Act II Sc 2 Lines 86-89)Is I know how to curserdquo

o He is well aware of the futility of arguing with one who has more power than he has

ldquoI must obey his art is such power (Act I Sc 2 Lines 373-376)It would control my damrsquos god SetebosAnd make a vassal of himrdquo

o He realizes the importance of Prosperorsquos books

ldquoRemember (Act III Sc 2 Lines 89-92)First to possess his books for without themHersquos but a sot as I am nor hath notOne spirit to commandrdquo

o He knows the value of stealth when attacking the enemy

ldquoPray you tread softly that the blind mole may not (Act IV Sc 1 Lines 194-195)Hear a foot fall we now are near his cellrdquo

o Caliban has a better set of values than Stephano and Trinculo They are distracted from their plan by their greed for Prosperorsquos rich garments Only Caliban realizes that such a finery is unimportant

ldquoLeave it alone thou fool it is but trashrdquo (Act IV Sc 1 Lines 224)

Caliban is not a good judge of characterCaliban is not a good judge of character He decides for example that Stephano is a god because he dispenses lsquocelestial liquorrsquo (Act II Sc 2 Line 115) but then it must be remembered that he has only known his mother Sycorax Prospero Miranda and the spirits that torture him However he quickly discovers his error of judgementrdquo

ldquoWhat a thrice-double ass (Act V Sc 1 Lines 295-297)Was I to take this drunkard for a godAnd worship this dull foolrdquo

Calibanrsquos Imaginative NatureIf Caliban is sub-human in what has been said above he is human in the respect of the poetic side of his character He listens to music with rapture He tells of the beautiful dreams in which heaven rains treasures upon him and which upon waking he yearns to renew One of the most poetic passages in whole play is Calibanrsquos description of the island

to Stephano and Trinculo

ldquoBe not afeard The isle is full of noises (Act III Sc 2 Lines 135-143)Sounds and sweet airs that give delight and hurt notSometimes a thousand twangling instrumentsWill hum about mine ears and sometime voicesThat if I then had waked after long sleepWill make me sleep again and then in dreamingThe clouds methought would open and show richesReady to drop upon me that when I wakedI cried to dream againrdquo

Caliban - Less Ignoble Than Some OthersCalibanrsquos motive for murder is less dishonourable than that of Antonio and Sebastian They plan to kill Alonso to gain his power and wealth Caliban merely wants revenge and the return of lsquohisrsquo island

Conclusiono Calibanrsquos character is not portrayed very clearly in the play and hence we cannot decide whether he is a poor

savage being grossly maltreated by Prospero or whether he is evil and must therefore be kept in bondage or enslavement

o Caliban is contrasted with Ariel who is a spirit and thus swift and uninterested in physical activitieso Caliban is also contrasted with Prospero who is the all-powerful master of the island and of the destiny of all

those on the islando Caliban is also contrasted with civilized man showing him to be less evil than Antonio and Stephano and less

materialistic than Stephano and Trinculoo Caliban has suffered at the hands of Prospero and he has learnt to curse by listening to Prosperorsquos abuse He

certainly believes that Prospero has deprived him of his birthrighto Finally the character Caliban is thought to be one of Shakespearersquos masterpieces The complexity of the character

is reflected in the large volume of critical discussion that has grown around it

ECO ndash12 Topic-Forms of market

MonopolyMonopoly is a market structure in which there is a single seller there are no close substitutes for the commodity produced by the firm and there are barriers to entry Example Indian Railways which is operated under government of India Monopoly also implies absence of competitionFeatures of Monopoly Monopoly is characterized by1 Single Seller In monopoly there is only one firm producing the product The whole industry consists of this single firm Thus under monopoly there is no distinction between firm and industry Being the only firm there is significant control of the firm over supply and price Thus under monopoly buyers do not have the option of buying the commodity from any other seller They have to buy the product from the firm or they can go without the commodity This fact gives immense control to the monopolist over the market

2No Close Substitute There are no close substitutes of the product produced by the monopolist firm If there are close substitutes of the product in the market it implies presence of more than one firm and hence no monopoly In order to ensure a total of control over the market by the monopolist firm it is assumed that there are no close substitutes of the product

3 No Entry amp Exit Monopoly can only exist when there is strong barriers before a new firm to enter the market In fact once a monopoly firm starts producing the product no other firm can produce the same One reason for this is the ability of the

monopolist to produce the product at a lower cost than any new firm who thinks to enter the market If a new firm who knows that it cannot produce at a lower cost than the monopolist then that firm will never enter the market for fear of losing out in competition Similarly the monopolist who is operating for a long time may be enjoying reputation among its customers and is in a better position to use the situation in its own benefit A new firm has to take long time to achieve this and so may not be interested to enter the market

4 Price Maker Being the single seller of the product the monopolist has full control over the pricing of the product On the other hand if there is a large number of buyers in the market so no single buyer exercises any significant influence over price determination Thus it is a sellerrsquos market So monopoly firm is a price maker

5 Price Discrimination Having considerable control over the market on account of being single seller with no entry of other firms the monopolist can exercise policy of price discrimination it means that the monopolist can sell different quantities of the same product to a consumer at different price or same quantity to different consumers at different prices by adjudging the standard of living of the consumer

6 Shape of Demand Curve Since a monopolist has full control over the price therefore he can sell more by lowering the price This makes the demand curve downward sloping

Subject Ac-12 290620 Topic- retirement Model sumThe Balance Sheet of Rohit Nisha and Sunil who are partners in a firm sharing profits according to their capitals as on 31st March 2014 was as under

Liabilities Amount Assets Amount (Rs) (` Rs)

Creditors 25000 Machinery 40000Bills Payable 13000 Building 90000General Reserve 22000 Debtors 30000Capital Less Provision for Rohit 60000 Bad debts 1000

29000 Nisha 40000 Stocks 23000 Sunil 40000 140000 Cash at Bank 18000

200000 200000

On the date of Balance Sheet Nisha retired from the firm and following adjustments were made(i) Building is appreciated by 20(ii) Provision for bad debts is increased to 5 on Debtors(iii) Machinery is depreciated by 10(iv) Goodwill of the firm is valued at Rs 56000 and the retiring partnerrsquos share is adjusted

(v) The capital of the new firm is fixed at Rs120000 Prepare Revaluation Account Capital Accounts of the partner and Balance Sheet of the new firm after Nisharsquos retirement Revaluation AccountDr Cr

Particulars Amount Particulars Amount (`Rs) (Rs`)

Provision for Bad debt Ac 500 Building Ac 18000Machinery Ac 4000Profit transferred toCapital Accounts (3 2 2)Rohit 5786Nisha 3857Sunil 3857

13500

18000 18000

Capital Account

Dr Cr

Particulars Rohit Nisha Sunil Particulars Rohit Nisha Sunil (Rs`) (Rs`) (`Rs) (Rs`) (Rs`) (Rs`)

Sunilrsquos Capital ac 9600 mdash 6400 Balance bd 60000 40000 40000Bank - 66143 - General Reserve 9428 6286 6286Balance cd 72000 mdash 48000 Revaluation (Profi 5786 3857 3857 Rohitrsquos Capital Ac mdash 9600 mdash

Sunilrsquos Capital Ac 6400 Bank 6386 - 4257

81600 66143 54400 81600 66143 54400

Balance Sheet as at 31st March 2014

Liabilities Amount Assets Amount (Rs`) (Rs`)

Creditors 25000 Building 108000Bank overdraft 37500 Machinery 36000

Bills Payable 13000 Debtors 30000Capital Less ProvisionRohit 72000 for Bad debts 1500 28500Sunil 48000 120000 Stock 23000

195500 195500

Working Notes (i) (a) Profit sharing ratio is 60000 40000 40000 ie = 3 2 2(b) Gaining Ratio Rohit = 35 ndash 37 = 2135 ndash 1535 = 635Sunil = 25-27 = 1435 ndash 1035 = 435= 635 435= 6 4 = 3 2(c) Nisha Share of Goodwill = Rs 56000 times 27 = Rs16000Share of Goodwill in the gaining ratio by the existing partner ieRohit = Rs16000 times 35 = Rs 9600Sunil = Rs 16000 times 25 = Rs 6400

The journal entry isRohitrsquos Capital Ac Dr 9600Sunilrsquos Capital Ac Dr 6400 To Nisharsquos Capital Ac 16000(Share of Goodwill divided into gaining ratio)

  • 1 Static Friction
  • The frictional force that acts between the surfaces when they are at rest with respect to each other is called Static Friction
    • Static Friction Examples
      • 2 Sliding Friction
        • Examples Of Sliding Friction
          • 3 Rolling Friction
            • Examples Of Rolling Friction
              • Objects and Reasons of the Forest Conservation Act
Page 23:  · Web viewSubject . Topic . Summary . Execution . English 1 . Sounds of animals . Hens –cackle Horses –neigh Lions –roar Owls –hoots Snake –hiss. English 2 . Mother’s

BIOLOGY Chapter -5 The endocrine system and adolescence

Today we will discuss about thelocation and functions of secreted hormones of adrenal and Pancreas

Q5 Write location hormone secreted main functions and deficiency diseases of pancreas and adrenal glands

Endocrine Glands

Location Hormones secreted

Functions and Deficiency Diseases

1Adrenal gland

2 Pancreas Gland

On the top of each kidney

In between stomach and small intestine

i)Adrenaline from adrenal medulla

ii)Cortisone from adrenal cortex

i) Insulin

ii) Glucagon

It helps a person deal with any kind of emergency situation or emotional stressIt increases the heart beat rate of respiration and blood pressure

a) It regulates carbohydrates protein and fat metabolism

b) It regulates the salt and water balance in the body

a) It changes excess glucose into glycogen

b) It stimulates the cells to burn extra glucose to provide heat amp energy

Less secretion causes diabetes mellitus

Excessive secretions causeinsulin shock

a) It stimulates the breakdown of glycogen into glucose

b) It increases the level of glucose in blood

History Traders to rulers The Battle of Buxar was fought on 22 October 1764 between the forces under the command of the British East India Company led by Hector Munro and the combined armies of Mir Qasim the Nawab of Bengal till 1763 Mir Jafar was made the Nawab of Bengal for a second time in 1763 by the Company just after the battle After being defeated in 4 battles in katwa and Udaynala the Nawab of Awadh Siraj id Daula and the Mughal emperor Shah Alam II accompanied by Raja Balwant Singh of Kashi made an alliance with Mir Qasim The battle was fought at Buxar a small fortified

Answer the following questions- Short note-Battle of BuxarHomework-learn

town within the territory of Bihar located on the banks of the Ganga river about 130 kilometres (81 mi) west of Patna it was a decisive victory for the British East India Company The war was brought to an end by the Treaty of Allahabad in 1765

EnglishLiterature

The west wind-John Mansfield

In the poem The West Wind by John Masefield the poet starts by describingwith very poetic imagery of birds how the west wind is different from other winds its a warm wind full of birds cries There is a touch of melancholy perhaps home-sickness as he describes how it brings tears too and memories from an old land He goes on to describe the restful pastoral beauty of the land where even the dead can lie in the green He then brings in voicesperhaps of family and friends calling him home as he is missing Aprils beautyThe voices then tempt him some more with idyllic images from home (white blossom young green cornrunning rabbitswarm sun) The voices seem to presume that the poets heart is sorrowful bruised and soreThe end of the poem sees the poet appear to make a decision he will go home as he has decided that is where he truly belongs

Write the synopsis of the following words

1 Daffodils- a tall yellow flower that grows in the spring

2 Orchards- a piece of land on which fruit trees are grown

3 Blossom- a flower or a mass of flowers especially on a fruit tree in spring

4 Thrushes- a bird5 Larks- a small brown bird that

makes a pleasant sound6 Bruised- an injury7 Aching- pain 8 Tread- to put your foot down

while you are walking9 Balm-10 May-11 Fluting-

(Write from the book in your copy)

MAT

HEM

ATIC

S

Ch 1

1Al

gebr

ic E

xpre

ssio

n

1 Constant A symbol which has fixed value is called a constant[eg 8 23 -15 radic3 etc]

2 VariableA symbol which does not have any fixed value but may be assigned value (values) according to the requirement is called variable or literal[eg x y p q etc]

3 TermsA term is a number (constant) a variable a combination (product or quotient) of numbers and variables[eg 7 x 5x etc]

4 Algebric expressionA single term or acombination of two or more terms connected by plus (+) or minus (-) sign forms an algebraic expression[eg 5-y 3x2-5x xy-6z+4 etc]

5 PolynomialAn algebraic expression which contains more than one term is called a polynomial (multinomial)[eg x2-5x 5y+xy+x2y etc]

6 Degree of polynomial(a) When the polynomial contains only one variable the highest power of the variable is the degree of the polynomialeg the degree of the polynomial of 4x-7x5+8 is 5(b) When the polynomial contains two or more variablesStep (i) Find the powers of the variables in each term (ii) The highest sum of the powers is taken to be the degree of the polynomialeg the degree of the polynomial 5x2y-4x3y5+6 is = 3+5 = 8Remember An algebraic expression is a polynomial if degree of each term used in it is a non-negative integer

Exercise ndash 11(A)

1 Separate the constants and variables from the following

-7 7+x 7x+yz radic5 radic xy 3 yz

8 45y -3x

Solution Constant Variables-7 radic5 7+x 7x+yz radic xy

3 yz8

45y -3x

2 Write the number of terms in each of the following polynomials(i) 5x2+3timesax (ii) axdivide4-7 (iii) ax-by+ytimesz (iv) 23+atimesbdivide2

Solution Polynomials Number of terms(i) 5x2+3timesax 2(ii) axdivide4-7 2(iii) ax-by+ytimesz 3(iv) 23+atimesbdivide2 2

4 Write the degree of the each polynomials(i) xy+7z (ii) x2-6x3+8 (iii) y-6y2+5y8 (iv) xyz-3 (vi) x5y7-8x3y8+10x4y4z4

Solution Polynomials Degree(i) xy+7z 2(ii) x2-6x3+8 3(iii) y-6y2+5y8 8(iv) xyz-3 3(vi)x5y7-8x3y8+10x4y4z4 12

5Write the coefficient of(i) ab in 7abx (iv) 8 in a2-8ax+a (v) 4xy in x2-4xy+y2

SolutionCoefficient

(i) ab in 7abx 7x(iv) 8 in a2-8ax+a -ax(v) 4xy in x2-4xy+y2 -1

7 CoefficientAny factor of an algebraic quantity is called the coefficient of the remaining quantityeg in the algebraic term 7xyz 7 is coefficient of xyz 7x is coefficient of yz and so on

8 Like term The terms having the same literal coefficient are called like terms and those having different literal coefficients are called unlike terms

eg (i) 5xyz 8xyz -6xyz and 23xyz are like

terms(ii) 7xy2 8x2yz and -15xyz2 are unlike terms

6 in 57xy2z3 write the coefficient of

(i) 5 (vii) 5xy2 (viii) 17yz (xi) 5xyz

Solution Coefficient

(i) 5 17

xy2z3

(vii) 5xy2 17z3

(viii) 17yz

5xyzsup2

(xi) 5xyz 17yz2

7 In polynomial given below separate the like terms(ii) y2z3 xy2z3 -58x2yz -4y2z3 -8xz3y2 3x2yz and 2z3y2

Solution y2z3 -4y2z3 2z3y2 are like terms

xy2z3 -8xz3y2 are like terms

-58x2yz 3x2yz are like terms

Class IXSubject Topic Summary Execution

Bengali (2nd language)

বাগzwnjধারাzwnj বা ধারা-বা ধারা ল হিবকেশষ পরকার বাক -হিb -াকেবর এক হিবকেশষ পরকাশরীহিত াকেক কতগকেলা কার সমষটির মকেধয এগহিলকেক বা ধারা বকেল আবার কতগকেলা শকেবদর বাধাধরা যকান রীহিত যনই য-াকেব চকেল আসকে যসই -াকেবই চকেল আসকে তখন যসই শবদগহিল খন একক -াকেব অG পরকাশ ককের তখন একের বা ধারা বকেল বা ধারার পরকেয়া -াষাকেক আরও সFর ককের যতাকেল

অকাল পকক(অপহিরনত বয়কেস পাকাহিম)-মাতর শ বর বয়কেস যমকেয়টির া মকেখর কা তাকেত অকালপককতা ধরা পকে

অককা পাওয়া( মারা াওয়া) ndash পকেকIমারটি পকেকIমারকেত হিকেয় বাসাতরীকের াকেত মার যখকেত যখকেত অককা যপল

অহি| পরীকষা ( কঠিন ও পরকত পরীকষা)- যকেলটির আজ ডাকতাহির যরজালট যবকেরাকেব এIাই তার জীবকেনর ব অহি| পরীকষা

অষটরমভা (ফাহিক) ndash রীতা মকেখই বকো বকো কা বকেল আর কাকেজর যবলায় অষটরমভা

অকমGার ধাী (অপাG) ndash সমনকেক হিনকেয় যকান ান কেব না ও একেকবাকেরই অকমGার ধাী

অকেনধর ষটি (অসাকেয়র সায়)- আহিশ বকেরর বকোর নাহিত ল অকেনধর ষটি তাকেক াা বকোর একম চকেল না

আকেককল গড়ম (তবহিদধ)- ার তহিম উপকার করকেল যসই যতামার হিবরকেদধ সাকষয হিকেয়কে শকেনই আমার আকেককল গড়ম

আষাকে লপ( অবাসতব লপ) ndashIাকা এখন যকেব না এIা বলকেলই ত এমন আষাকে লপ ফাার যকান রকার হিল না

Hindi- महायजञ ा इस हानी म लख न या बतान ा परयास किया ह कि किसी भी अचछ

2nd language

परसार(यशपाल ाय या पणय न ा फल अवशय मिमलता ह ोई भी परोपार अथवा पणय लिलए किया गया ाय बार नही 0ाता वह ए परार ा यजञ हए धनी सठ थ धम परायण और किवनमर सठ न आन ी यजञ किए थ और दान म न 0ान कितना धन दिदन दखिखयो म बात दिदया थादिदन पलट और सठ यहा गरीबी आ गई उन दिदनो यजञ बचन ी परथा थी सठ भी अपनी 0गह बचन लिलए डलपर ए सट यहा चलन ो तयार हए सठानी रासत लिलए रोटी पड म बाधर सठ ो द दी रासत म ए भख R ो दखर सठ न चारो रोटी उसो खिखला दी खर वह सठ यहा डलपर पहच तो उनी सठानी न उस महायजञ बचन ो हा यदिद बचन आए सठ न R ो रोटी खिखलान ो महायजञ नही समझा और वापस लौट आया घर आर शाम ो उसी घर म उस ए बडा ख0ाना मिमला 0ो उस दवारा किए गएrsquo महायजञrsquo ा परसार था

English language

Letter formal The heading the name and address of the person you are writing to must be included beneath your own address In formal letters ldquoblock stylerdquo of address is preferred

Subject complain in brief

Salutation If the person you are writing to is known to you you may begin ldquoDear MrrdquoOr ldquoDear Mrsrdquo In all other instances you should begin ldquoDear Sirrdquo or ldquoDear Madamrdquo Or ldquoSirsrdquo

The body A formal or business letter has four partsReference The letter should begin by referring to a letter you have received an advertisement or the reason that has prompted you to writeInformation In the second paragraph it is necessary to supply more detailed information that is related to the referencePurpose Here you must give the reason why you are writing the letter This must be stated clearly and ensure that it is relevant to the question that has been setConclusion round off the letter with some polite remarkThe subscription when a letter has begun with dear sir sirs Madam you should end with Yours faithfully or yours truly When however you address a person by name you must conclude with the words ldquoYours sincerelyrdquo

1 A park in your locality is slowly being used as a rubbish dump Write a letter to the Mayor of your city pointing out the nuisance and danger of this Request that action be taken to stop this immediately

Or2 You being a boarder ordered a set of lab manuals from a famous book shop in the town They sent you a wrong set of books Write a letter to the manager of the book shop

Chemistry Chapter-1 1)CHEMICAL FORMULA- Q What is the Significance of

L-2The Language of Chemistrybull Chemical Formula

Itrsquos a symbolic representation of a chemical substance eg ndash The formula of Sulphuric acid is H2SO4

2) Steps of writing Chemical Formula of a given substance-

1 Write the symbols of the constituent atoms or radicals side by side Keep the basic radical on LHS and acid radical on the RHS ( Na+Cl- )2 In case of a radical having more than one atom( compound radical) enclose the radical in a bracket eg (SO4-)3 Write the valencies of each radical on its right hand top4 If the valencies of the two radicals are divisible by a common factor then divide the valencies by the common factor5 Invert (criss-cross) the valency number ie write the valency of one atom below the second atom and vice versa 6 On interchanging if valency number is lsquoone the figure lsquoonersquo is never writtenFor Example- Compound -Calcium Nitrate1 Writing the symbols- Ca(NO3)2 Writing the valencies on their right hand top- Ca2(NO3)1

3 Valency numeral in simple ratio- Ca2(NO3)1

4 Criss-cross- Ca 2NO3 1

5 Writing the formula of the compound- Ca(NO3)2

Chemical formula

A The formula of a substance conveys the following information regarding a substance 1 The name of the substance (qualitative)2 The elements constituting the substance (qualitative)3 The number of various atoms present in a molecule of the substance (quantitative)4 Molecular weight of the substance and the relative weights of different elements present in it (qualitative)

Q What are the limitations of Chemical Formula

A The chemical formula suffers from the following limitations-I It fails to convey whether the elements in a molecule are present in the form of atoms or ionsFor example the formula KBr fails to tell us whether Potassium and Bromine are present in the form of ions II It does not tell anything about the binding force that holds atom in a molecule togetherIII It does not tell us about the arrangement of various atoms with respect to one another within the molecule

Q Examples of Some Chemicals with their Formula Chemical name and Common Name-

A Given in the class notesCommercial Studies

Joint Stock Company

Let us discuss about the demerits of Joint Stock CompanyDespite so many advantages it has got many disadvantages which are as follows

Difficulty in FormationDelay in Decision makingExcessive Government ControlLack of Secrecy

Company can be classified into several categories based on incorporation

QuestionExplain the demerits of Joint Stock CompanyAnswer) 1 Difficulty in Formation The legal requirements and formalities required to be completed are so many The cost involved is quite heavy It has to approach large number of people for its capital It cannot start its business unless certificate of incorporation has been obtained This is granted after a long time when all the formalities are completed

Chartered CompanyStatutory CompanyRegistered Company

Delay in Decision making In this form of organization decisions are not made by single individual All important decisions are taken by the Board of Directors Decision-making process is time-consuming So many opportunities may be costly because of delay in decision-making Promptness of decisions which is a common feature of sole trader ship and partnership is not found in a company

Excessive Government ControlA company and the management have to function well within the law and the provisions of Companies Act are quite elaborate and complex At every step it is necessary to comply with its provisions lest the company and the management should be penalized The penalties are quite heavy and in several cases officers in default can be punished with imprisonment This hampers the proper functioning of the company

Lack of Secrecy The management of companies remains in the hands of many persons Every important thing is discussed in the meetings of Board of Directors Hence secrets of the business cannot be maintained In case of sole proprietorship and partnership forms of organisation such secrecy is possible because a few persons are involved in the management

2 Define the following

Chartered Company- The crown in exercise of the royal prerogative has power to create a corporation by the grant of a charter to persons assenting to be incorporated Such companies or corporations are known as chartered companies Examples of this type of companies are Bank of England (1694) East India Company (1600) The powers and the nature of business of a chartered company are defined by the charter which incorporates it After the country attained independence these types of companies do not exist

in IndiaStatutory Company- A company may be incorporated by means of a special Act of the Parliament or any state legislature Such companies are called statutory companies Instances of statutory companies in India are Reserve Bank of India the Life Insurance Corporation of India the Food Corporation of India etc The provisions of the Companies Act 1956 apply to statutory companies except where the said provisions are inconsistent with the provisions of the Act creating them Statutory companies are mostly invested with compulsory powersRegistered companiesCompanies registered under the Companies Act 1956 or earlier Companies Acts are called registered companies Such companies come into existence when they are registered under the Companies Act and a certificate of incorporation is granted to them by the Registrar

Economics

Chapter-4Basic problems of Economy

Today let us discuss with the topic Production Possibility curve

QuestionExplain the concept of Production Possibility Curve with the help of diagram

Answer) Production Possibility curve is a locus of all possible combinations of two commodities which can be produced in a country with its given resources and technology

The above diagram shows that with the given resources and technology the economy can produce maximum either 5 thousand meters of cloth or 15 thousand quintals of wheat or any other combination of the two goods like B( 1 thousand meters of cloth and 14 thousand quintals of wheat C ( 2 thousands meters of cloth and 12 thousand quintals of wheat) etcProduction Possibility curve is also called production possibility boundary or frontier as it sets the maximum limit of what it is possible to produce with given resources

Geography

Rotationand Revolution

SUNrsquoS POSITION AND SEASONAL CHANGES EQUINOXES ndash SPRING AND AUTUMN

Q1 What is Spring EquinoxA1 On 21st March sunrays fall directly on the equator On that day

As the Equator divides the Earth into two equal halves the sun rays fall directly on the equator twice in a year Equinoxes means equal Spring EquinoxOn 21st March sunrays fall directly on the equator On that day the duration of day and night both are equal ( 12 hours day and 12 hours night) on every places located on equator This day is called as Spring EquinoxAutumn EquinoxOn 23rd September sunrays fall directly on the equator On that day the duration of day and night both are equal ( 12 hours day and 12 hours night) on every places located on equator This day is called as Autumn Equinox

SOLSTICES ndash SUMMER AND WINTERDue to inclination of the Earth on its axis and the apparent movement of the sun the sun rays fall directly on both tropics once in a year Solstice is a Latin word which mean ldquothe Sun standing stillrdquoSummer SolsticesAfter 21st March there is an apparent movement of the Sun to the north of the equator The apparent northward movement up to 21st June when the Sun appears overhead at the Tropic of Cancer (22frac12degN) The sun appears to stand still at this position and then moves southwards towards the equator This position of the Sun on 21st June is known as Summer Solstices On that day the duration of day and night both are equal ( 12 hours day and 12 hours night) on every places located on Tropic of Cancer (22frac12degN)Winter solstices The apparent southward movement of the Sun continues beyond the equator till 22nd

December On this day the Sun is overhead at the Tropic of Capricorn

the duration of day and night both are equal ( 12 hours day and 12 hours night) on every places located on equator This day is called as Spring Equinox

Q2 What do you mean by EquinoxA2 Equinoxes means equal It is use to explain the equal duration of day and night ( 12 hours day and 12 hours night) on the Earth

Q3 On which date the longest day in Tropic of CancerA3 21st June

Q4 What is the meaning of SolsticeA4 Solstice is a Latin word which mean ldquothe Sun standing stillrdquo

Q5 Which is the longest day in southern hemisphereA5 22nd December

Q6 On what date does the Arctic Circle experience the lsquoMidnight SunrsquoA6 On 21 June the Arctic Circle experiences the lsquoMidnight Sunrsquo

Q7 What is cause of Midnight Sun in NorwayA7 During the summer solstice (21 June) the North Pole is inclined towards the Sun Therefore the duration of sunlight or daytime increases from 12 hours at the Equator to 24 hours at the Arctic Circle and beyond Thatrsquos why The region beyond the Arctic Circle especially Norway is known as the Land of the Midnight Sun because there the Sun does not rise or set on 21 June

Q8 Match the column A with BA B

Summer Solstice 21st March

Autumn Equinox 23rd

September

Winter Solstice 21st June

(22frac12degS) This position of the Sun is referred to as the Winter Solstice because it marks the winter season in the Northern Hemisphere On that day the duration of day and night both are equal ( 12 hours day and 12 hours night) on every places located on Tropic of Capricorn (22frac12degS)SEASONS AND DURATION OF DAY AND NIGHT During the equinoxes all places on the Earth have 12 hours of day and 12 hours of night Due to the revolution of the Earth round the Sun on an inclined axis the duration of day and night varies according to seasons and the latitude of a placeDuring the summer solstice (21 June) the North Pole is inclined towards the Sun Therefore the duration of sunlight or daytime increases from 12 hours at the Equator to 24 hours at the Arctic Circle and beyondThe region beyond the Arctic Circle especially Norway is known as the Land of the Midnight Sun because there the Sun does not rise or set on 21 JuneAt the North Pole there will be six months of daylight The Sun will be seen always above the horizon at a low angle At 66degN 24 hours of sunlight can be seen only on 21 June Hammerfest in northern Norway is a place of tourist attraction for observing the phenomenon of the Midnight Sun This place has continuous daylight from 13 May to 29 July This place is easily accessible to tourists and has hotels and other facilities The view of the midnight Sun from here is enthrallingIn the Southern Hemisphere the duration of daylight decreases from 12 hours at the equator to 0 hours beyond the Antarctic Circle In the South Polar Region there is 24 hours of darkness The Sun is always below the horizon In the Southern Hemisphere which experiences winter the duration of night-time is longer than the duration of daylight

Spring Equinox 22nd

December

A8 A B

Summer Solstice 21st June

Autumn Equinox 23rd

September

Winter Solstice 22nd

December

Spring Equinox 21st March

During winter solstice (22 December) the South Pole is inclined towards the Sun The Southern Hemisphere experiences summer and the Northern Hemisphere has winter Therefore the duration of daylight or sunlight is greater in the Southern Hemisphere than in the Northern HemisphereThe duration of daylight increases from 12 hours at the equator to 24 hours beyond the Antarctic Circle The South Polar Region has 24 hours of sunlight for many days continuously At the South Pole there will be six months of sunlight The Sun will always be seen at a low angle above the horizon In the Northern Hemisphere the duration of daylight will decrease from 12 hours at the equator to 0 hours at the Arctic Circle There are 24 hours of darkness in the North Polar region The duration of night is greater than the duration of daylight as one move northwards from the Equator It is evident from the above table that the duration of daylight is 12 hours throughout the year at the equator only As one moves away from the equator the seasonal variations in the duration of daylight increase The seasonal variations in the duration of daylight are maximum at the Polar Regions

Subject Eng Literature (The Merchant of Venice ndash William Shakespeare)Topic Act II Scene 7 Lines 36 to 80 (End of scene ) [Students should read the original play and also the paraphrase provided]

Summary Questions amp AnswersThe Prince then examines the inscription on the silver casket which says ldquoWho chooseth me shall get as much as he deservesrdquo The Prince says that he deserves Portia more than anybody else because of his high rank his noble birth and his great wealth and power But then he argues that silver is ten times

(1) (Act II Sc 7 L 39-47)

From the four corners of the earth they come

To kiss this shrine this mortal breathing saint

The Hyrcanian deserts and the vasty wildsOf wide Arabia are as through-fares now

inferior to gold and therefore he cannot believe that the portrait of such a beautiful lady as Portia can be contained in the silver casket He decides to see the inscription on the golden casket before making his decision

The Prince goes to examine the inscription on the golden casket which says ldquoWho chooseth me shall get what many men desirerdquo The Prince believes that the whole world desires to possess Portia otherwise so many suitors would not have come from all corners of the world for winning Portia Some of them have come from the distant lands of Persia and Arabia The deserts of Persia (Hyrcanian deserts) and the boundless desolate lands of Arabia have been crossed by the Princes seeking the hand of Portia He contrasts this casket containing Portiarsquos portrait with the old English gold coin bearing the image of the archangel (angel of the highest rank) He goes on to remark that while the figure of the archangel is engraved (Insculped) upon the English coin the picture of Portia who is beautiful as an angel lies hidden inside one of the caskets namely the Golden Casket (Golden Bed)

On the basis of his assessment of the inscription on the golden casket the Prince decides to choose the golden casket He asks for the key and opens the golden casket only to find therein an empty human skull holding a roll of

For princes to come view fair PortiaThe watery kingdom whose ambitious headSpets in the face of heaven is no barTo stop the foreign spirits but they comeAs orsquoer a brook to see fair Portia

(i) Explain the occasion for the above mentioned speech

These are the comments of the Prince of Morocco after he reads the inscription on the golden casket His mental process is revealed to us in these words We find him debating within himself as to which casket he should choose

(ii) What light does the above speech throw on the personality of Prince of Morocco

From the above mentioned speech we come to know that the Prince of Morocco is keen to marry Portia He is the type of person who is easily taken away by outward appearance He is in love with Portia because of her beauty

(iii) What information can you gather about Portia from the above mentioned lines

The given speech shows that Portia is a very beautiful lady She must be possessed of good qualities because many suitors come to her place from all over the world with a desire to get married to her The Prince of Morocco is so impressed by her beauty that he calls her a saint According to him the whole world is desirous of having her

(iv) Elucidate the significance of the first two lines

In these lines the Prince of Morocco pays a compliment to Portia These lines show his admiration for her He says that people come from all parts of the world to see fair Portia

(v) Explain the meaning of the last four lines of the

passage

In these lines the Prince of Morocco says that even the vast oceans which throw a challenge at the sky are unable to prevent men from coming to Portiarsquos place to have a glimpse of her These lines are also a tribute to Portiarsquos beauty and good qualities Many men voyage across the ocean treating it as a mere stream to see the beautiful Portia

paper in which is written that whoever happens to be guided by the glitter of things is invariably deceived

On reading the scroll the Prince says that he is too sad at heart to speak a more formal farewell and leaves with his followers amidst a sound of trumpets

After the Prince of Morocco leaves Portia remarks that the Prince is a gentle fellow but she is rid of him May all persons of his nature make a similar choice

IMPORTANT PASSAGES EXPLAINED

(Act II Sc 7 L 39-43)From the four corners of the earth they come

To kiss this shrine this mortal breathing saintThe Hyrcanian deserts and the vasty wildsOf wide Arabia are as through-fares nowFor princes to come view fair Portia

Context

This passage occurs in Act II Scene 7 in The Merchant of Venice This is part of the speech made by the Prince of Morocco

(2)

(Act II Sc 7 L 48-53)

MOROCCO One of these three contains her heavenly pictureIst like that lead contains her

Twere damnation To think so base a thought it were too grossTo rib her cerecloth in the obscure graveOr shall I think in silver shes immurdBeing ten times undervalued to tried gold

(i) What meaning does the Prince of Morocco find out of the inscription of the golden casket What have Belmont and Portiarsquos house been called and why

The inscription on the golden casket is ldquoWho chooseth me shall gain what many men desirerdquo The Prince finds out that it means that the chooser of the golden casket will get Portia because many men desire her In fact the entire world desires her Because of the coming of many suitors to Belmont from different countries in order to win Portiarsquos hand Belmont has become a centre of pilgrimage and her house is the shrine where saintly Portia is installed

(ii) What does the Prince of Morocco do before making the final choice of the casket Which is the correct casket and who will win Portiarsquos hand

The Prince of Morocco surveys and analyses the inscriptions on the casket of lead silver and gold Before making the final choice like a very systematic and methodical person he once again considers the claims of the caskets The casket containing Portiarsquos picture is the correct casket and the person choosing it will win Portiarsquos hand

Explanation

While praising Portia the Prince of Morocco conceives Portia as a goddess whose image is placed inside one of the caskets Many suitors are coming from far and wide the north and the south the east and the west (Four corners) in order to try their luck Some of them have come from the distant land of Persia and Arabia The deserts of Persia (Hyrcanian deserts) and the boundless desolate lands of Arabia have been crossed by the Princes seeking the hand of Portia All this shows that Portia is indeed the most beautiful lady of the world

(iii) What does the Prince of Morocco say in his estimation while examining the motto on the silver casket What does he find in the golden casket

While examining the motto on the silver casket which says ldquoWho chooseth me shall get as much as he deservesrdquo Morocco says that in his own estimation he surely deserves Portia in all respects ndash rank birth wealth etc

He chooses the golden casket When he opens it he finds an empty human skull holding a scroll in which it is written that those who are attracted by the glittering outside of things are always deceived as Morocco has been deceived

(iv) What kind of nature does the Prince of Morocco have

The Prince of Morocco has a simple nature who does not look deeply into the inner meaning of things but is dazzled by the outward appearance of gold He is inclined to over-estimate his own value and does not realize that it is a duty to ldquogive and hazardrdquo To say that he will not hazard for lead shows that he misreads the true meaning of the inscription which is that he should be prepared to ldquohazard all he hathrdquo for Portia So his feeling is only one of fascination and romantic attraction

(v) Do you think that the lottery of the caskets is not a matter that will be determined by chance

In fact the lottery of the casket is not a matter that will be determined by mere chance but that it is a true test of character and of sincerity which is amply proved not only by Moroccorsquos choice but also by the arguments which he uses to help him in his choice

(Act II Sc 7 L 55-59)

They have in England

A coin that bears the figure of an angelStamped in gold but thats insculpd uponBut here an angel in a golden bedLies all within

Context

(3)

(Act II Sc 7 L 63-77)A carrion Death within whose empty eye

There is a written scroll Ill read the writing

All that glisters is not goldOften have you heard that toldMany a man his life hath soldBut my outside to beholdGilded tombs do worms infoldHad you been as wise as boldYoung in limbs in judgment oldYour answer had not been inscrolld

This passage occurs in Act II Scene 7 in The Merchant of Venice This is part of the speech made by the Prince of Morocco

Explanation

In this passage the Prince of Morocco bestows high praise on Portia whose hand he is seeking He contrasts this casket containing Portiarsquos portrait with the old English gold coin bearing the image of the archangel (angel of the highest rank) He goes on to remark that while the figure of the archangel is engraved (Insculped) upon the English coin the picture of Portia who is beautiful as an angel lies hidden inside one of the caskets namely the Golden Casket (Golden Bed) In the day of Elizabeth silver was ten times inferior in value to gold Therefore the Prince of Morocco believing that Portiarsquos portrait is contained in the Golden Casket decides to choose the Golden Casket

Fare you well your suit is coldCold indeed and labour lostThen farewell heat and welcome frostmdashPortia adieu I have too grievd a heartTo take a tedious leave Thus losers part

(i) What reward does the Prince of Morocco get after making a wrong choice of the Casket How does he feel

After making the wrong choice in selecting the casket of gold the Prince of Morocco as a reward earns a rebuke in the form of a scroll tucked in the empty eye-socket of a skull kept in the casket of gold The Prince is shocked and disappointed He becomes all the more sad and dejected when he reads the scroll which points to his foolishness in being misled by the appearance and outward show as indicative of its worth

(ii) How does the Prince respond after reading the scroll

After reading the scroll the Prince though upset accepts the result with good grace and decorum befitting a royal suitor and true sportsman He says that his love-suit is really cold otherwise he would have chosen correctly but now his efforts have been in vain So he bids farewell to Portia to the warmth and enthusiasm of love and welcomes the cold and bitterness of dejection and misery of life which lies ahead

(iii) What request does he make to Portia and why

After being failure in his mission he requests Portia to give him permission to leave at once because he is too sad to undergo the tediousness of a formal leave-taking He tells that it is the manner in which defeated persons part unceremoniously

(iv) Explain the following lines

ldquoAll that glisters is not goldOften have you heard that toldMany a man his life hath soldBut my outside to beholdGilded tombs do worms infoldrdquo

Mere glitter does not make a metal to be gold Man has often been warned against appearance but it has been of no use Many people have sacrificed their lives only to seek the outer appearance of gold Worms are found inside the gilded

monuments

Class XSubject Topic Summary Execution

Hindi 2ndlang

नया रासता भाग 6 मायाराम 0ी घर म धनी मल 0ी और उनी बटी सरिरता ी ही चचा बनी रहती थी अमिमत ो इसम ोई रलिच ना थी वह धनी घर ी लडी स शादी र सवय ो बचना नही चाहता था उसा भी सवाणिभमान ह ईशवर ी पा

स उस पास पस ी ोई मी नही थी अभी उसन फकटरी ही लगाई थी उसी समझ बाहर था कि उस घर वालो ा झाव पस ी तरफ कयो

ह उसन मा स सवाल किया कि मा तम सरिरता स मरी शादी कयो रना चाहती हो मा न उस समझाया कि वह दखन म बरी नही ह और किफर खानदान अचछा

ह वह ए शल गरहणी रप म घर सभाल सगी अमिमत न मा ो इस बात ा एहसास राया कि मीन सबध लिलए मना रन पर उस दिदल

पर कया बीती होगी मा और अमिमत ी लडी बार म ाफी बात हईमा ा झाव सरिरता ी तरफ था कयोकि वह घर पर अचछा दह0 लर आ रही

थी अमिमत न अपनी मौसी ी बरी हालत बार म बताया कि किस तरह वह बड घर ी खानदानी बटी लाई थी और आ0 उसी हालत कितनी खराब ह लाई थी बहकलब 0ाती ह और बचचो ो भी नही दखती ह बात चल ही रही

थी कि तभी ए ार बाहर आर री धनी मल0ी घर अदर आए और पीछ स डराइवर फल ी ए टोरी लर आया अदर आए और पीछ स

डराइवर ए टोरी फल ी लर आया अमिमत ो फल ी पटी बरी लग रही थी अमिमत न पछ लिलया यह फल कयो ल आए ह प इन सब ी कया

0ररत थी उनो न 0वाब दिदया कि 4 पटी शमीर स मगाए थ अमिमत ो या सनर करोध आ गया तभी उस किपता 0ी आ गए उन आत ही अमिमत उठर बाहर चला गया वहा वहा मा पास आर बठ गया और बोला

अभी रिरशता तय नही हआ और धनी मल 0ी धनी मल 0ी फल ी पटी लर चलआय मा न समझाया कि 0ब सबध 0ड 0ाता ह तो खाली हाथ नही

आत अमिमत न मा स हा कि तम सबन सरिरता ो इस घर म लान ी ठान रखी ह धनीमल 0ी उस दिदन सरिरता ो दखन ी तारीख तय रन आय थ

Commercial Studies

Banking Nowadays Bank provide easy and quick services through internet facilities methods of Banking is called internet bankingIn order to save the time and money involved in visiting Bank branches people increasingly prefer to have internet banking

There are different modes of doing internet banking or transferring money through online They areReal Time Gross Settlement (RTGS)National Electronic Fund Transfers (NEFT)

1

Question

1) Explain the term RTGS Write the features of RTGS

Answer)The acronym RTGS stands for Real Time Gross Settlement which may be defined as the continuous real time settlement of funds transfer individually on and order by order basis without netting lsquoReal timersquo may be defined as the processing of instructions at the time they are received rather than at some letter time lsquoGross settlementrsquo may be defined as the settlement of transfer instructions which occurs

individually

Features of RTGS1It is the continuous settlement of

funds transfer individually on an order by order basis

2RTGS facility is provided only by CBS core banking solution enabled Bank branches

3Amount charged from the customer for RTGS transactions vary from bank to bank

2) Explain the term NEFT Write the features of NEFT

Answer) National electronic funds transfer may be defined as a nationwide system that facilitates individuals Farms and copper operates to electronically transfer funds from any bank branch to any individual farm or corporate having an account with any other bank branch in the country

Features of NEFT2 Transfer can be made 7 times on

weekdays and 6 times on Saturday

3 NEFT cannot be used to receive foreign remittances

4 NEFT transaction takes place in batches

5 A bank branch must be NEFT enabled to become a part of NEFT fund transfer network

6 There is no maximum or minimum amount that can be transferred through NEFT when one bank has a bank account

English Language

CompositionEssay

A composition is an art of creating a piece of writing on any topic or subject It is the writing correctly beautifully and clearly in order to make some interesting reading Structure of the composition

Introduction ( you lay the foundation for your composition)

Body (it constitutes the main part of the essay)

Conclusion (final statement that leaves a lasting impression)

Kinds of essays1 The Narrative essay2 The descriptive essay3 The reflective essay4 The argumentative essay

Write a composition on any one of the following topics (350- 400 words)

1 Friendship Or2 The first day of your school

Subject Eng Literature (The Merchant of Venice ndash William Shakespeare)Topic Act V Scene 1 Lines 127 to 158 (Nerissa helliphellip The clerk will nersquoer wear hair onrsquos face that had it) [Students should read the original play and also the paraphrase given in the school prescribed textbook]

Summary Revision Questions o Soon thereafter Bassanio Gratiano

and Antonio arrive

o Bassanio tells Portia that he is feeling as if it is morning because of the presence of Portia who is shining like the sun When Antonio is introduced by Bassanio to Portia she tells Bassanio that he should be grateful to Antonio who took so much trouble on his account even to the extent of risking his life

o Nerissa starts quarrelling with Gratiano and demands that he show her the ring she had presented to him and which she had warned him not to lose She suspects that Gratiano must have presented the ring to some young woman and not to the lawyerrsquos clerk as he repeatedly says and assures

Answer the following questions to check your preparation of Act IV Scenes 1 and 2

You must attempt only after you have completed your preparation of Act IV The answers must be in complete sentences using textual evidence (with citation) when necessary

[It would be in your own interest to attempt the above questions honestly totally refraining from consulting your textbook or your notes during answering After completion you should correct the paper yourself consulting the textbooknotes etc and award marks as specified Please let me know the marks you scored through WhatsApp in the group or to my personal WhatsApp]

Act IV Scene 1 (each question carries 2 marks)

1 What did the Duke try to do for Antonio

2 Why does Shylock refuse to show mercy How does he justify his stance

3 Why does Antonio say he is ready to die 4 What information is contained in Bellariorsquos letter

5 Why does Portia (as Balthazar) assert that Shylock must show mercy How does he respond

6 What offers are made to Shylock to get him to spare Antonio How are they received

7 What does Antoniorsquos speech as he faces the prospect of Shylockrsquos knife tell you about his character

8 How do Bassanio and Gratiano react to the looming prospect of Antoniorsquos demise

9 How does Portia (as Balthazar) use the law to turn the tables on Shylock

10 What does the Duke decree should happen to Shylock Why What happens to Shylockrsquos estate

11 What does Portia ask Bassanio as payment for her ldquoservicesrdquo What is his initial response What makes him change his mind

Act IV Scene 2 (each question carries 1frac12 marks)

1 What does Gratiano bring to Portia (Balthazar)

2 What does Nerissa plan on getting from Gratiano What does Portiarsquos comment suggest about men

ECO-10 280620 Topic-Supply AnalysisSHIFTING OF SUPPLY

But if there is change in factors other than the price of the commodity then either more is supplied at the same price or less supplied at the same price In such cases the price of the commodity remains constant but there is a change in other factors like change in the price of inputs change in technology of production change in price of other related goods change in taxation policy of the government etc For example there is an improvement in the technology of production of the commodity in question It leads to decrease in per unit of cost production of the commodity The firm is willing to sell more quantity of the commodity at the same price So the supply other commodity increases at the same price This increase in supply is shown by rightward shift of supply curve On the other hand if the firm uses inferior technology of production the cost of production per unit of the commodity increases The firm is willing to sell less quantity at the same price So the supply of the commodity decreases at the same price This decrease in supply is shown by leftward shift of the supply curve The above cases of increase and decrease in supply can be shown with the help of the following figures

Y INCREASE OF SUPPLY Price (Rs) s

P A s1

B

s

X` O s1 X

q q1

Y` Quantity demanded (in units)

Y DECREASE IN SUPPLY s2

s

price (Rs)

C

p A

s2

s

X` o X

q2 q

Y` Quantity demanded ( in units)

Main factors causing increase in supply or rightward shift of supply Curve(i) Fall in the price of other related goods

(ii) Fall in the price of inputsfactors(iii) Use of better technology in production(iv) Decrease in the rate of excise duty by government(v) If the objective of producer changes from profit maximization to salesMaximization

Main factors causing decrease in supply or leftward shift of supply curve(i) Increase in the price of other related goods(ii) Rise in the price of inputsfactors(iii) Use of inferior technology in production(iv) Increase in the rate of excise duty by the government(v) If the objective

Subject - Biology Topic ndash Chapter mdash6 PhotosynthesisSummary Execution

Today we will know about photosynthesis and its stages

Q1 What do you mean by photosynthesis The process by which living plants containing chlorophyll produce food

substances from carbon-di- oxide and water by using light energy Sunlight

6CO2 +12 H2O----------------------- C6 H12O6 + 6H2O + 6O2

Chlorophyll

Q2 What are the importance of photosynthesis I) Food for all Green plants trap solar energy by photosynthesis

process and supply food and energy for all living organisms either directly or indirectly

Ii) Oxygen to breathe in by product of photosynthesis is oxygen which is essential for all living organisms respiration

Q3 Write about two main phases of photosynthesis A Light dependent phase This phase occur in grana of chloroplast I) The chlorophyll on exposure to light energy becomes activated by

absorbing photons Ii) The absorbed energy is used in splitting the water molecules (H2O)

into its two components (H+ and OH- ) and releasing electron s 2H2O------------------------- 4H+ + 4e- +O2

Energy of 4 photons This reaction is known as photolysis

End products are H+ and oxygen water

B Light independent (Dark ) phase The reactions in this phase require no light energy

Here CO2 combine with H+ and produce glucose

Class XI

Subject Topic Summary ExecutionEVS Chapter-4 Legal

regimes for sustainable development

Environmental legislationEnvironmental legislation is the collection of laws and regulations pertaining to air quality water quality the wilderness endangered wildlife and other environmental factors The act ensures that matters important to the environment are thoroughly

Learn -The Forest (Conservation) Act 1980

considered in any decisions made by federal agencies

The Forest (Conservation) Act 1980 The Forest (Conservation) Act 1980 an Act of the Parliament of India to provide for the conservation of forests and for matters connected therewith or ancillary or incidental thereto It was further amended in 1988 This law extends to the whole of IndiaObjects and Reasons of the Forest Conservation Act

Deforestation causes ecological imbalance and leads to environmental deterioration Deforestation had been taking place on a large scale in the country and it had caused widespread concern The act seeks to check upon deforestation and de-reservation of forests

Subject Eng Literature (The Tempest ndash William Shakespeare) Topic Act II Scene 1 Lines 314 to 329 (End of scene)

[Students should read the original play and also the paraphrase given in the school prescribed textbook]Summary Questions amp Answers

Conspiracy of Antonio and Sebastian (Contd)

o As they approach Ariel appears again and wakes up Gonzalo by singing a tune in his ear Alonso also wakes up and they see both Sebastian and Antonio with drawn swords On being caught off guard they make up a story saying that they had heard a bellowing of bulls or lions

o They then moved to another part of the island

o Ariel at once rushes to Prospero to inform him of this development

SUMMING-UP of ACT-2 SCENE-1

(i) Among the survivors Ferdinand is separated from the rest which results in the disconsolate grief of Alonso as he took him for dead

(ii) The villainy of Antonio is confirmed

(iii) The supremacy of Prosperorsquos magic which resulted in the failure of the human conspiracy

(1)

(Act II Sc 1 L 311-325)SEBASTIAN Whiles we stood here securing your repose

Even now we heard a hollow burst of bellowing Like bulls or rather lions Didt not wake youIt struck mine ear most terribly

ALONSO I heard nothingANTONIO O rsquotwas a din to fright a monsters ear

To make an earthquake Sure it was the roarOf a whole herd of lions

ALONSO Heard you this GonzaloGONZALO Upon mine honour sir I heard a humming

And that a strange one too which did awake meI shaked you sir and cried As mine eyes opened I saw their weapons drawn There was a noiseThats verily rsquoTis best we stand upon our guardOr that we quit this place Lets draw our weapons

(i) Why has Prospero sent Ariel to Gonzalo and Alonso What does Ariel do to awaken Gonzalo

Prospero has already come to know by his magic powers the danger which threatens Gonzalo who had been Prosperorsquos friend and so he sent Ariel to preserve the lives of both Gonzalo and Alonso Prospero does not want that his scheme should remain unfulfilled Ariel begins to sing a song in Gonzalorsquos ears to awaken him(ii) Who are ready to carry out their plan Who takes steps to stop them Why does Gonzalo feel surprised after being awakened

Sebastian and Antonio are ready to carry out their plans They are standing with their swords drawn to kill Alonso and

(iv) We see two sets of contrasting characters Gonzalo-Adrian against Antonio-Sebastian

(v) The grief that works in Alonso can be perceived to his repentance for his association in Antoniorsquos crime against Prospero

Gonzalo Ariel takes steps to stop them from carrying out their nefarious scheme When Gonzalo is awakened by the song sung by Ariel into his ears he (Gonzalo) feels surprised because he sees Sebastian and Antonio standing with their swords drawn(iii) What reason do Sebastian and Antonio tell of drawing their swords when they are suspected by Alonso and Gonzalo

When Sebastian and Antonio are seen with their swords drawn they are looked with suspicion by Gonzalo and Alonso At first Sebastian tells them that as they stood here to guard them during their sleep they heard only a little before a sudden loud noise very much like the roaring of bulls or more probably that of lions Then Antonio follows him saying that this was a noise so terrible as to frighten even a monsterrsquos ears and this noise could even have shaken the earth and it was surely like the roaring of a multitude of lions Then seeing the danger they have drawn their swords Perhaps after hearing the terrible noise they (Gonzalo and Alonso) woke up from their sound sleep

(iv) What does Gonzalo tell Alonso about the strange noise What did he see on opening his eyes Gonzalo tells Alonso that he did not hear the sound of roaring but he heard a humming sound which was strange and which woke him up After waking up he gave him (Alonso) a shaking and a loud cry On opening his eyes he saw these two gentlemen standing with their swords drawn(v) What does Gonzalo suggest

Gonzalo suggests that there was a noise indeed and of that he has no doubt at all and suggests that the best course for them would be to remain alert and vigilant against any possible danger to their lives or to leave this place and move to some other part of the island

Class XIISubject Topic Summary Execution

Commerce

Chapter- Management

Today we will discuss about LEVELS OF MANAGEMENT

Levels of management is a series or chain of managerial positions from top to bottom It helps individuals to know their authority responsibilities and superior-subordinate relations among themselves There are mainly three levels of Management TOP LEVEL MANAGEMENTMIDDLE LEVEL MANAGEMENTLOWER LEVEL MANAGEMENT

Top level managementIt consists of members at the highest level in the management hierarchy This level includes Board Of Directors Chief Executive Managing Directors Chairman President Vice President

Rolefunctions of the top levelmanagement1To analyse evaluate and deal

with theexternal environment2 To determine the objectives and

policies of the business3 To strive for welfare and survival

of business

4 To create an organisational Framework consisting of authority responsibility relationship

Middle level management Congress of members or groups who are concerned with implementation of the policies let down by the top managementThis level includes head of the department such as finance manager marketing manager branch and regional managers departmental and divisional heads plant superintendent etc

Role of functions of the middle level management

1 To interpret the policies framed by top management

2 To assign duties and responsibilities to lower level managers

3 To select and appoint employees for middle and supervisory level and evaluate their performance

4 To co-operate with other departments for smooth functioning

Operational or supervisory level managementIt refers to the group are members who are concerned with execution of the work They are also known as fast line managers This level includes supervisor 4 men Section Officer clerk Inspector etc

Role of functions of the lower level management1 To plan and execute day-to-

day operations2 To supervise and control the workers3 To arrange materials and

tools to start the process and make arrangements for training

4 Today present workers grievance and suggestions before the management and

ensure safe and proper working conditions in the factory

Business Studies

Staff Appraisal Chapter- 10 Today let us start with a new chapter

Staff Appraisal

Meaning of Performance Appraisal

Performance Appraisal is the systematic evaluation of the performance of employees and to understand the abilities of a person for further growth and developmentThe supervisors measure the pay of employees and compare it with targets and plansThe supervisor analyses the factors behind work performances of employeesThe employers are in position to guide the employees for a better performance

Objectives of Performance Appraisal

Following are the objectives of Performance Appraisal

To maintain records in order to determine compensation packages wage structure salaries raises etc

To identify the strengths and weaknesses of employees to place right men on right job

To maintain and assess the potential present in a person for further growth and development

To provide a feedback to employees regarding their performance and related status

To provide a feedback to employees regarding their performance and related status

Importance of Performance Appraisal

Performance appraisal provides important and useful information for the assessment of employees skill

knowledge ability and overall job performance The following are the points which indicate the importance of performance appraisal in an organization

1 Performance appraisal helps supervisors to assess the work performance of their subordinates

2 Performance appraisal helps to assess the training and development needs of employees

3 Performance appraisal provides grounds for employees to correct their mistakes and it also provides proper guidance and criticism for employees development4 Performance appraisal provides reward for better performance

5 Performance appraisal helps to improve the communication system of the organization

6 Performance appraisal evaluates whether human resource programs being implemented in the organization have been effective

7 Performance appraisal helps to prepare pay structure for each employee working in the organization

8 Performance appraisal helps to review the potentiality of employees so that their future capability is anticipated

Geography

DRIANAGE The SubarnarekhaThe Subarnarekha and the Brahmaniinterposed between the Ganga and the Mahanadi deltas drain an area of 19300 sq kmand 39033 sq km respectively The drainage basins of these streams are shared byJharkhand Odisha west Bengal and Chhattisgarh The Brahmani is known as southKoel in its upper reaches in Jharkhand

The NarmadaThe Narmada rises in the Amarkantak hills of MadhyaPradesh It flows towards the West in a rift valleyformed due to a geological fault The total length of it is 1300 km All the tributaries of the

Q1 Name the two westward flowing rivers in the peninsular plateauA1 Narmada and Tapi are the only westward flowing rivers of the peninsular plateau

Q2 Differentiate between east-flowing rivers and west-flowing riversA2

East-flowing rivers

West-flowing rivers

Narmada are very short inlength Most of its tributaries join the main streamright anglesThe Narmada basin covers parts of Madhya Pradesh and Gujarat

The Tapi The Tapi rises in the Satpura ranges in the Betul listrictof Madhya Pradesh It flows in a rift valley parallel tothe Narmada but it is much shorter in length It coversparts of Madhya Pradesh Gujarat and MaharashtraThe length is about 724 km

The Sabarmati and the MahiThe Sabarmati rises in the Aravali hills and flows south-south-westwards for a distance of 300 kilometres to the Arabian Sea The Sabarmatibasin extends over an area of 21674 sq km in Rajasthan and Gujarat The Mahi rises inthe east of Udaipur and drains an area of 34842 sq km lying in Madhya PradeshRajasthan and Gujarat It flows south-westwards for a distance of 533 km before it fallsinto the Gulf of Khambhat

The ChambalThe Chambal rises near Mhow in the Vindhya Range and flows towards the northgenerally in a gorge upto Kota Below Kota it turns to the north-east direction and afterreaching Pinahat it turns to the east and runs nearly parallel to the Yamuna beforejoining it in the southern part of the Etawah district in Uttar PradeshMajor Rivers of India with their basin area (Sqkm)

Himalayan System Indus 321290Ganga 861404

Brahmaputra 187110Indus System

Jhelum 34775Beas 20303

Ganga System Yamuna 366223Ghaghra 127950

Peninsular RiversNarmada 98796

Tapi 65145Mahanadi 141600

Subarnarekha 19300Sabarmati 21674

Mahi 34842Godavari 312812

Godavari Krishna Kaveri Mahanadi are the east-flowing rivers

Narmada Tapi west-flowing rivers

They fall into the Bay of Bengal

They fall into Arabian Sea

These rivers form big deltas

These rivers form comparativelysmall deltas

Catchment areas of these rivers are larger

Catchment areas of these rivers are smaller

Krishna 2589488Cauveri 87900

Subject ndashBiology Topic ndashChapter -5 Inheritance amp Variations Summary ExecutionToday we will discussabout linkage and its classification

LINKAGE The tendency of the genes located on the same chromosome to stay together is

hereditary transmission Linked genes the genes responsible for this Genes that exhibit the process of linkage locates in the same chromosome The distance between the linked genes in a chromosome determines the strength

of linkage i e genes that are located close to each other show stronger linkage than that are located far from each other

COMPLETE LINKAGE It is the type of linkage showed by the genes that are closely located or are tightly

linked with each other as they have no chance of separatingby crossing over These genes are always transmitted together to the same gamete and the same

offspring In such condition only parental or non cross over type of gametes are formedINCOMPLETE KINKAGE It is type of linkage showed by the genes that are distantly located orare loosely

linked with each other because they have chance of separating by crossing over

SIGNIFICANCE i) It helps in holding the parental character togetherii) It checks the appearance of new recombination and helps in bringing the

hybrid population which resembles the original parents iii) Linked genes dilute the effects of undesirable traits

Subject Eng Literature (The Tempest ndash William Shakespeare) Topic Essay Questions (EQ-3)Question No 3

Give a character sketch of CalibanAnswer

The character of Caliban has been wonderfully conceived by Shakespeare as the manifestation of all that is gross and earthy ndash a sort of creature of the earth as Ariel is a sort of creature of the air

Calibanrsquos Physical Appearanceo Caliban is lsquofreckledrsquo a lsquomisshapen knaversquo not honoured with human shape

o Prospero calls him lsquothou tortoisersquo (Act I Sc 2 Line 317) Trinculo stumbling upon him describes him as ldquoA strange fish hellip Legged like a man And his fins like armsrdquo He ldquosmells like a fishrdquo (Act II Sc 2 Line 25)

o Prospero also calls him a ldquobeastrdquo (Act IV Sc 1 Line 140) and ldquoThis misshapen knaverdquo (Act V Sc 1 Line 268)

o Further it appears that in addition to his physical deformity his spiritual inferiority is also suggested by Prosperorsquos claim that his birth resulted from the union between his mother the witch Sycorax and the devil

Calibanrsquos ParentageWhen the play opens Caliban is twenty four years of age having been born on the island twelve years before the coming of Prospero His mother was the foul witch Sycorax who was banished from Algiers for ldquomischiefs manifold and sorceries terrible to enter human hearingrdquo (Act I Sc 2 Line 264) and the father was the Devil himself Thus

Caliban is a monster of evil and brute nature ugly deformed and stinking

Calibanrsquos Savage and Malignant Natureo Caliban is entirely a creature of the earth ndash gross brutal and savage He regards himself as the rightful possessor

of the island and Prospero as a usurper

o In his young age he was on good terms with Prospero He had consented to be received by Prospero at his house and to be educated by him He has learnt human language only to curse his master whom he abhors

o His beastly nature soon breaks out and ends in a vicious attack on Miranda This opens the eye of Prospero who becomes severe to him and enforces his service by threats and violence

o Prospero uses him to make dams for fish to fetch firewood scraper trenches wash dishes and keep his cell clean

Calibanrsquos Hatred for ProsperoA profound hatred for Prospero has taken hold of Caliban It springs from a sense of his being dispossessed and ill-treated He would kill Prospero if he could but he knows the power of Prosperorsquos lsquobookrsquo Hence he transfers his allegiance to Stephano who seems like a god to him He also incites the two drunken associates to batter the skull of Prospero when he sleeps in the afternoon

Caliban Shows Considerable Intelligenceo He has learnt Prosperorsquos language

ldquoYou taught me language and my profit onrsquot (Act II Sc 2 Lines 86-89)Is I know how to curserdquo

o He is well aware of the futility of arguing with one who has more power than he has

ldquoI must obey his art is such power (Act I Sc 2 Lines 373-376)It would control my damrsquos god SetebosAnd make a vassal of himrdquo

o He realizes the importance of Prosperorsquos books

ldquoRemember (Act III Sc 2 Lines 89-92)First to possess his books for without themHersquos but a sot as I am nor hath notOne spirit to commandrdquo

o He knows the value of stealth when attacking the enemy

ldquoPray you tread softly that the blind mole may not (Act IV Sc 1 Lines 194-195)Hear a foot fall we now are near his cellrdquo

o Caliban has a better set of values than Stephano and Trinculo They are distracted from their plan by their greed for Prosperorsquos rich garments Only Caliban realizes that such a finery is unimportant

ldquoLeave it alone thou fool it is but trashrdquo (Act IV Sc 1 Lines 224)

Caliban is not a good judge of characterCaliban is not a good judge of character He decides for example that Stephano is a god because he dispenses lsquocelestial liquorrsquo (Act II Sc 2 Line 115) but then it must be remembered that he has only known his mother Sycorax Prospero Miranda and the spirits that torture him However he quickly discovers his error of judgementrdquo

ldquoWhat a thrice-double ass (Act V Sc 1 Lines 295-297)Was I to take this drunkard for a godAnd worship this dull foolrdquo

Calibanrsquos Imaginative NatureIf Caliban is sub-human in what has been said above he is human in the respect of the poetic side of his character He listens to music with rapture He tells of the beautiful dreams in which heaven rains treasures upon him and which upon waking he yearns to renew One of the most poetic passages in whole play is Calibanrsquos description of the island

to Stephano and Trinculo

ldquoBe not afeard The isle is full of noises (Act III Sc 2 Lines 135-143)Sounds and sweet airs that give delight and hurt notSometimes a thousand twangling instrumentsWill hum about mine ears and sometime voicesThat if I then had waked after long sleepWill make me sleep again and then in dreamingThe clouds methought would open and show richesReady to drop upon me that when I wakedI cried to dream againrdquo

Caliban - Less Ignoble Than Some OthersCalibanrsquos motive for murder is less dishonourable than that of Antonio and Sebastian They plan to kill Alonso to gain his power and wealth Caliban merely wants revenge and the return of lsquohisrsquo island

Conclusiono Calibanrsquos character is not portrayed very clearly in the play and hence we cannot decide whether he is a poor

savage being grossly maltreated by Prospero or whether he is evil and must therefore be kept in bondage or enslavement

o Caliban is contrasted with Ariel who is a spirit and thus swift and uninterested in physical activitieso Caliban is also contrasted with Prospero who is the all-powerful master of the island and of the destiny of all

those on the islando Caliban is also contrasted with civilized man showing him to be less evil than Antonio and Stephano and less

materialistic than Stephano and Trinculoo Caliban has suffered at the hands of Prospero and he has learnt to curse by listening to Prosperorsquos abuse He

certainly believes that Prospero has deprived him of his birthrighto Finally the character Caliban is thought to be one of Shakespearersquos masterpieces The complexity of the character

is reflected in the large volume of critical discussion that has grown around it

ECO ndash12 Topic-Forms of market

MonopolyMonopoly is a market structure in which there is a single seller there are no close substitutes for the commodity produced by the firm and there are barriers to entry Example Indian Railways which is operated under government of India Monopoly also implies absence of competitionFeatures of Monopoly Monopoly is characterized by1 Single Seller In monopoly there is only one firm producing the product The whole industry consists of this single firm Thus under monopoly there is no distinction between firm and industry Being the only firm there is significant control of the firm over supply and price Thus under monopoly buyers do not have the option of buying the commodity from any other seller They have to buy the product from the firm or they can go without the commodity This fact gives immense control to the monopolist over the market

2No Close Substitute There are no close substitutes of the product produced by the monopolist firm If there are close substitutes of the product in the market it implies presence of more than one firm and hence no monopoly In order to ensure a total of control over the market by the monopolist firm it is assumed that there are no close substitutes of the product

3 No Entry amp Exit Monopoly can only exist when there is strong barriers before a new firm to enter the market In fact once a monopoly firm starts producing the product no other firm can produce the same One reason for this is the ability of the

monopolist to produce the product at a lower cost than any new firm who thinks to enter the market If a new firm who knows that it cannot produce at a lower cost than the monopolist then that firm will never enter the market for fear of losing out in competition Similarly the monopolist who is operating for a long time may be enjoying reputation among its customers and is in a better position to use the situation in its own benefit A new firm has to take long time to achieve this and so may not be interested to enter the market

4 Price Maker Being the single seller of the product the monopolist has full control over the pricing of the product On the other hand if there is a large number of buyers in the market so no single buyer exercises any significant influence over price determination Thus it is a sellerrsquos market So monopoly firm is a price maker

5 Price Discrimination Having considerable control over the market on account of being single seller with no entry of other firms the monopolist can exercise policy of price discrimination it means that the monopolist can sell different quantities of the same product to a consumer at different price or same quantity to different consumers at different prices by adjudging the standard of living of the consumer

6 Shape of Demand Curve Since a monopolist has full control over the price therefore he can sell more by lowering the price This makes the demand curve downward sloping

Subject Ac-12 290620 Topic- retirement Model sumThe Balance Sheet of Rohit Nisha and Sunil who are partners in a firm sharing profits according to their capitals as on 31st March 2014 was as under

Liabilities Amount Assets Amount (Rs) (` Rs)

Creditors 25000 Machinery 40000Bills Payable 13000 Building 90000General Reserve 22000 Debtors 30000Capital Less Provision for Rohit 60000 Bad debts 1000

29000 Nisha 40000 Stocks 23000 Sunil 40000 140000 Cash at Bank 18000

200000 200000

On the date of Balance Sheet Nisha retired from the firm and following adjustments were made(i) Building is appreciated by 20(ii) Provision for bad debts is increased to 5 on Debtors(iii) Machinery is depreciated by 10(iv) Goodwill of the firm is valued at Rs 56000 and the retiring partnerrsquos share is adjusted

(v) The capital of the new firm is fixed at Rs120000 Prepare Revaluation Account Capital Accounts of the partner and Balance Sheet of the new firm after Nisharsquos retirement Revaluation AccountDr Cr

Particulars Amount Particulars Amount (`Rs) (Rs`)

Provision for Bad debt Ac 500 Building Ac 18000Machinery Ac 4000Profit transferred toCapital Accounts (3 2 2)Rohit 5786Nisha 3857Sunil 3857

13500

18000 18000

Capital Account

Dr Cr

Particulars Rohit Nisha Sunil Particulars Rohit Nisha Sunil (Rs`) (Rs`) (`Rs) (Rs`) (Rs`) (Rs`)

Sunilrsquos Capital ac 9600 mdash 6400 Balance bd 60000 40000 40000Bank - 66143 - General Reserve 9428 6286 6286Balance cd 72000 mdash 48000 Revaluation (Profi 5786 3857 3857 Rohitrsquos Capital Ac mdash 9600 mdash

Sunilrsquos Capital Ac 6400 Bank 6386 - 4257

81600 66143 54400 81600 66143 54400

Balance Sheet as at 31st March 2014

Liabilities Amount Assets Amount (Rs`) (Rs`)

Creditors 25000 Building 108000Bank overdraft 37500 Machinery 36000

Bills Payable 13000 Debtors 30000Capital Less ProvisionRohit 72000 for Bad debts 1500 28500Sunil 48000 120000 Stock 23000

195500 195500

Working Notes (i) (a) Profit sharing ratio is 60000 40000 40000 ie = 3 2 2(b) Gaining Ratio Rohit = 35 ndash 37 = 2135 ndash 1535 = 635Sunil = 25-27 = 1435 ndash 1035 = 435= 635 435= 6 4 = 3 2(c) Nisha Share of Goodwill = Rs 56000 times 27 = Rs16000Share of Goodwill in the gaining ratio by the existing partner ieRohit = Rs16000 times 35 = Rs 9600Sunil = Rs 16000 times 25 = Rs 6400

The journal entry isRohitrsquos Capital Ac Dr 9600Sunilrsquos Capital Ac Dr 6400 To Nisharsquos Capital Ac 16000(Share of Goodwill divided into gaining ratio)

  • 1 Static Friction
  • The frictional force that acts between the surfaces when they are at rest with respect to each other is called Static Friction
    • Static Friction Examples
      • 2 Sliding Friction
        • Examples Of Sliding Friction
          • 3 Rolling Friction
            • Examples Of Rolling Friction
              • Objects and Reasons of the Forest Conservation Act
Page 24:  · Web viewSubject . Topic . Summary . Execution . English 1 . Sounds of animals . Hens –cackle Horses –neigh Lions –roar Owls –hoots Snake –hiss. English 2 . Mother’s

town within the territory of Bihar located on the banks of the Ganga river about 130 kilometres (81 mi) west of Patna it was a decisive victory for the British East India Company The war was brought to an end by the Treaty of Allahabad in 1765

EnglishLiterature

The west wind-John Mansfield

In the poem The West Wind by John Masefield the poet starts by describingwith very poetic imagery of birds how the west wind is different from other winds its a warm wind full of birds cries There is a touch of melancholy perhaps home-sickness as he describes how it brings tears too and memories from an old land He goes on to describe the restful pastoral beauty of the land where even the dead can lie in the green He then brings in voicesperhaps of family and friends calling him home as he is missing Aprils beautyThe voices then tempt him some more with idyllic images from home (white blossom young green cornrunning rabbitswarm sun) The voices seem to presume that the poets heart is sorrowful bruised and soreThe end of the poem sees the poet appear to make a decision he will go home as he has decided that is where he truly belongs

Write the synopsis of the following words

1 Daffodils- a tall yellow flower that grows in the spring

2 Orchards- a piece of land on which fruit trees are grown

3 Blossom- a flower or a mass of flowers especially on a fruit tree in spring

4 Thrushes- a bird5 Larks- a small brown bird that

makes a pleasant sound6 Bruised- an injury7 Aching- pain 8 Tread- to put your foot down

while you are walking9 Balm-10 May-11 Fluting-

(Write from the book in your copy)

MAT

HEM

ATIC

S

Ch 1

1Al

gebr

ic E

xpre

ssio

n

1 Constant A symbol which has fixed value is called a constant[eg 8 23 -15 radic3 etc]

2 VariableA symbol which does not have any fixed value but may be assigned value (values) according to the requirement is called variable or literal[eg x y p q etc]

3 TermsA term is a number (constant) a variable a combination (product or quotient) of numbers and variables[eg 7 x 5x etc]

4 Algebric expressionA single term or acombination of two or more terms connected by plus (+) or minus (-) sign forms an algebraic expression[eg 5-y 3x2-5x xy-6z+4 etc]

5 PolynomialAn algebraic expression which contains more than one term is called a polynomial (multinomial)[eg x2-5x 5y+xy+x2y etc]

6 Degree of polynomial(a) When the polynomial contains only one variable the highest power of the variable is the degree of the polynomialeg the degree of the polynomial of 4x-7x5+8 is 5(b) When the polynomial contains two or more variablesStep (i) Find the powers of the variables in each term (ii) The highest sum of the powers is taken to be the degree of the polynomialeg the degree of the polynomial 5x2y-4x3y5+6 is = 3+5 = 8Remember An algebraic expression is a polynomial if degree of each term used in it is a non-negative integer

Exercise ndash 11(A)

1 Separate the constants and variables from the following

-7 7+x 7x+yz radic5 radic xy 3 yz

8 45y -3x

Solution Constant Variables-7 radic5 7+x 7x+yz radic xy

3 yz8

45y -3x

2 Write the number of terms in each of the following polynomials(i) 5x2+3timesax (ii) axdivide4-7 (iii) ax-by+ytimesz (iv) 23+atimesbdivide2

Solution Polynomials Number of terms(i) 5x2+3timesax 2(ii) axdivide4-7 2(iii) ax-by+ytimesz 3(iv) 23+atimesbdivide2 2

4 Write the degree of the each polynomials(i) xy+7z (ii) x2-6x3+8 (iii) y-6y2+5y8 (iv) xyz-3 (vi) x5y7-8x3y8+10x4y4z4

Solution Polynomials Degree(i) xy+7z 2(ii) x2-6x3+8 3(iii) y-6y2+5y8 8(iv) xyz-3 3(vi)x5y7-8x3y8+10x4y4z4 12

5Write the coefficient of(i) ab in 7abx (iv) 8 in a2-8ax+a (v) 4xy in x2-4xy+y2

SolutionCoefficient

(i) ab in 7abx 7x(iv) 8 in a2-8ax+a -ax(v) 4xy in x2-4xy+y2 -1

7 CoefficientAny factor of an algebraic quantity is called the coefficient of the remaining quantityeg in the algebraic term 7xyz 7 is coefficient of xyz 7x is coefficient of yz and so on

8 Like term The terms having the same literal coefficient are called like terms and those having different literal coefficients are called unlike terms

eg (i) 5xyz 8xyz -6xyz and 23xyz are like

terms(ii) 7xy2 8x2yz and -15xyz2 are unlike terms

6 in 57xy2z3 write the coefficient of

(i) 5 (vii) 5xy2 (viii) 17yz (xi) 5xyz

Solution Coefficient

(i) 5 17

xy2z3

(vii) 5xy2 17z3

(viii) 17yz

5xyzsup2

(xi) 5xyz 17yz2

7 In polynomial given below separate the like terms(ii) y2z3 xy2z3 -58x2yz -4y2z3 -8xz3y2 3x2yz and 2z3y2

Solution y2z3 -4y2z3 2z3y2 are like terms

xy2z3 -8xz3y2 are like terms

-58x2yz 3x2yz are like terms

Class IXSubject Topic Summary Execution

Bengali (2nd language)

বাগzwnjধারাzwnj বা ধারা-বা ধারা ল হিবকেশষ পরকার বাক -হিb -াকেবর এক হিবকেশষ পরকাশরীহিত াকেক কতগকেলা কার সমষটির মকেধয এগহিলকেক বা ধারা বকেল আবার কতগকেলা শকেবদর বাধাধরা যকান রীহিত যনই য-াকেব চকেল আসকে যসই -াকেবই চকেল আসকে তখন যসই শবদগহিল খন একক -াকেব অG পরকাশ ককের তখন একের বা ধারা বকেল বা ধারার পরকেয়া -াষাকেক আরও সFর ককের যতাকেল

অকাল পকক(অপহিরনত বয়কেস পাকাহিম)-মাতর শ বর বয়কেস যমকেয়টির া মকেখর কা তাকেত অকালপককতা ধরা পকে

অককা পাওয়া( মারা াওয়া) ndash পকেকIমারটি পকেকIমারকেত হিকেয় বাসাতরীকের াকেত মার যখকেত যখকেত অককা যপল

অহি| পরীকষা ( কঠিন ও পরকত পরীকষা)- যকেলটির আজ ডাকতাহির যরজালট যবকেরাকেব এIাই তার জীবকেনর ব অহি| পরীকষা

অষটরমভা (ফাহিক) ndash রীতা মকেখই বকো বকো কা বকেল আর কাকেজর যবলায় অষটরমভা

অকমGার ধাী (অপাG) ndash সমনকেক হিনকেয় যকান ান কেব না ও একেকবাকেরই অকমGার ধাী

অকেনধর ষটি (অসাকেয়র সায়)- আহিশ বকেরর বকোর নাহিত ল অকেনধর ষটি তাকেক াা বকোর একম চকেল না

আকেককল গড়ম (তবহিদধ)- ার তহিম উপকার করকেল যসই যতামার হিবরকেদধ সাকষয হিকেয়কে শকেনই আমার আকেককল গড়ম

আষাকে লপ( অবাসতব লপ) ndashIাকা এখন যকেব না এIা বলকেলই ত এমন আষাকে লপ ফাার যকান রকার হিল না

Hindi- महायजञ ा इस हानी म लख न या बतान ा परयास किया ह कि किसी भी अचछ

2nd language

परसार(यशपाल ाय या पणय न ा फल अवशय मिमलता ह ोई भी परोपार अथवा पणय लिलए किया गया ाय बार नही 0ाता वह ए परार ा यजञ हए धनी सठ थ धम परायण और किवनमर सठ न आन ी यजञ किए थ और दान म न 0ान कितना धन दिदन दखिखयो म बात दिदया थादिदन पलट और सठ यहा गरीबी आ गई उन दिदनो यजञ बचन ी परथा थी सठ भी अपनी 0गह बचन लिलए डलपर ए सट यहा चलन ो तयार हए सठानी रासत लिलए रोटी पड म बाधर सठ ो द दी रासत म ए भख R ो दखर सठ न चारो रोटी उसो खिखला दी खर वह सठ यहा डलपर पहच तो उनी सठानी न उस महायजञ बचन ो हा यदिद बचन आए सठ न R ो रोटी खिखलान ो महायजञ नही समझा और वापस लौट आया घर आर शाम ो उसी घर म उस ए बडा ख0ाना मिमला 0ो उस दवारा किए गएrsquo महायजञrsquo ा परसार था

English language

Letter formal The heading the name and address of the person you are writing to must be included beneath your own address In formal letters ldquoblock stylerdquo of address is preferred

Subject complain in brief

Salutation If the person you are writing to is known to you you may begin ldquoDear MrrdquoOr ldquoDear Mrsrdquo In all other instances you should begin ldquoDear Sirrdquo or ldquoDear Madamrdquo Or ldquoSirsrdquo

The body A formal or business letter has four partsReference The letter should begin by referring to a letter you have received an advertisement or the reason that has prompted you to writeInformation In the second paragraph it is necessary to supply more detailed information that is related to the referencePurpose Here you must give the reason why you are writing the letter This must be stated clearly and ensure that it is relevant to the question that has been setConclusion round off the letter with some polite remarkThe subscription when a letter has begun with dear sir sirs Madam you should end with Yours faithfully or yours truly When however you address a person by name you must conclude with the words ldquoYours sincerelyrdquo

1 A park in your locality is slowly being used as a rubbish dump Write a letter to the Mayor of your city pointing out the nuisance and danger of this Request that action be taken to stop this immediately

Or2 You being a boarder ordered a set of lab manuals from a famous book shop in the town They sent you a wrong set of books Write a letter to the manager of the book shop

Chemistry Chapter-1 1)CHEMICAL FORMULA- Q What is the Significance of

L-2The Language of Chemistrybull Chemical Formula

Itrsquos a symbolic representation of a chemical substance eg ndash The formula of Sulphuric acid is H2SO4

2) Steps of writing Chemical Formula of a given substance-

1 Write the symbols of the constituent atoms or radicals side by side Keep the basic radical on LHS and acid radical on the RHS ( Na+Cl- )2 In case of a radical having more than one atom( compound radical) enclose the radical in a bracket eg (SO4-)3 Write the valencies of each radical on its right hand top4 If the valencies of the two radicals are divisible by a common factor then divide the valencies by the common factor5 Invert (criss-cross) the valency number ie write the valency of one atom below the second atom and vice versa 6 On interchanging if valency number is lsquoone the figure lsquoonersquo is never writtenFor Example- Compound -Calcium Nitrate1 Writing the symbols- Ca(NO3)2 Writing the valencies on their right hand top- Ca2(NO3)1

3 Valency numeral in simple ratio- Ca2(NO3)1

4 Criss-cross- Ca 2NO3 1

5 Writing the formula of the compound- Ca(NO3)2

Chemical formula

A The formula of a substance conveys the following information regarding a substance 1 The name of the substance (qualitative)2 The elements constituting the substance (qualitative)3 The number of various atoms present in a molecule of the substance (quantitative)4 Molecular weight of the substance and the relative weights of different elements present in it (qualitative)

Q What are the limitations of Chemical Formula

A The chemical formula suffers from the following limitations-I It fails to convey whether the elements in a molecule are present in the form of atoms or ionsFor example the formula KBr fails to tell us whether Potassium and Bromine are present in the form of ions II It does not tell anything about the binding force that holds atom in a molecule togetherIII It does not tell us about the arrangement of various atoms with respect to one another within the molecule

Q Examples of Some Chemicals with their Formula Chemical name and Common Name-

A Given in the class notesCommercial Studies

Joint Stock Company

Let us discuss about the demerits of Joint Stock CompanyDespite so many advantages it has got many disadvantages which are as follows

Difficulty in FormationDelay in Decision makingExcessive Government ControlLack of Secrecy

Company can be classified into several categories based on incorporation

QuestionExplain the demerits of Joint Stock CompanyAnswer) 1 Difficulty in Formation The legal requirements and formalities required to be completed are so many The cost involved is quite heavy It has to approach large number of people for its capital It cannot start its business unless certificate of incorporation has been obtained This is granted after a long time when all the formalities are completed

Chartered CompanyStatutory CompanyRegistered Company

Delay in Decision making In this form of organization decisions are not made by single individual All important decisions are taken by the Board of Directors Decision-making process is time-consuming So many opportunities may be costly because of delay in decision-making Promptness of decisions which is a common feature of sole trader ship and partnership is not found in a company

Excessive Government ControlA company and the management have to function well within the law and the provisions of Companies Act are quite elaborate and complex At every step it is necessary to comply with its provisions lest the company and the management should be penalized The penalties are quite heavy and in several cases officers in default can be punished with imprisonment This hampers the proper functioning of the company

Lack of Secrecy The management of companies remains in the hands of many persons Every important thing is discussed in the meetings of Board of Directors Hence secrets of the business cannot be maintained In case of sole proprietorship and partnership forms of organisation such secrecy is possible because a few persons are involved in the management

2 Define the following

Chartered Company- The crown in exercise of the royal prerogative has power to create a corporation by the grant of a charter to persons assenting to be incorporated Such companies or corporations are known as chartered companies Examples of this type of companies are Bank of England (1694) East India Company (1600) The powers and the nature of business of a chartered company are defined by the charter which incorporates it After the country attained independence these types of companies do not exist

in IndiaStatutory Company- A company may be incorporated by means of a special Act of the Parliament or any state legislature Such companies are called statutory companies Instances of statutory companies in India are Reserve Bank of India the Life Insurance Corporation of India the Food Corporation of India etc The provisions of the Companies Act 1956 apply to statutory companies except where the said provisions are inconsistent with the provisions of the Act creating them Statutory companies are mostly invested with compulsory powersRegistered companiesCompanies registered under the Companies Act 1956 or earlier Companies Acts are called registered companies Such companies come into existence when they are registered under the Companies Act and a certificate of incorporation is granted to them by the Registrar

Economics

Chapter-4Basic problems of Economy

Today let us discuss with the topic Production Possibility curve

QuestionExplain the concept of Production Possibility Curve with the help of diagram

Answer) Production Possibility curve is a locus of all possible combinations of two commodities which can be produced in a country with its given resources and technology

The above diagram shows that with the given resources and technology the economy can produce maximum either 5 thousand meters of cloth or 15 thousand quintals of wheat or any other combination of the two goods like B( 1 thousand meters of cloth and 14 thousand quintals of wheat C ( 2 thousands meters of cloth and 12 thousand quintals of wheat) etcProduction Possibility curve is also called production possibility boundary or frontier as it sets the maximum limit of what it is possible to produce with given resources

Geography

Rotationand Revolution

SUNrsquoS POSITION AND SEASONAL CHANGES EQUINOXES ndash SPRING AND AUTUMN

Q1 What is Spring EquinoxA1 On 21st March sunrays fall directly on the equator On that day

As the Equator divides the Earth into two equal halves the sun rays fall directly on the equator twice in a year Equinoxes means equal Spring EquinoxOn 21st March sunrays fall directly on the equator On that day the duration of day and night both are equal ( 12 hours day and 12 hours night) on every places located on equator This day is called as Spring EquinoxAutumn EquinoxOn 23rd September sunrays fall directly on the equator On that day the duration of day and night both are equal ( 12 hours day and 12 hours night) on every places located on equator This day is called as Autumn Equinox

SOLSTICES ndash SUMMER AND WINTERDue to inclination of the Earth on its axis and the apparent movement of the sun the sun rays fall directly on both tropics once in a year Solstice is a Latin word which mean ldquothe Sun standing stillrdquoSummer SolsticesAfter 21st March there is an apparent movement of the Sun to the north of the equator The apparent northward movement up to 21st June when the Sun appears overhead at the Tropic of Cancer (22frac12degN) The sun appears to stand still at this position and then moves southwards towards the equator This position of the Sun on 21st June is known as Summer Solstices On that day the duration of day and night both are equal ( 12 hours day and 12 hours night) on every places located on Tropic of Cancer (22frac12degN)Winter solstices The apparent southward movement of the Sun continues beyond the equator till 22nd

December On this day the Sun is overhead at the Tropic of Capricorn

the duration of day and night both are equal ( 12 hours day and 12 hours night) on every places located on equator This day is called as Spring Equinox

Q2 What do you mean by EquinoxA2 Equinoxes means equal It is use to explain the equal duration of day and night ( 12 hours day and 12 hours night) on the Earth

Q3 On which date the longest day in Tropic of CancerA3 21st June

Q4 What is the meaning of SolsticeA4 Solstice is a Latin word which mean ldquothe Sun standing stillrdquo

Q5 Which is the longest day in southern hemisphereA5 22nd December

Q6 On what date does the Arctic Circle experience the lsquoMidnight SunrsquoA6 On 21 June the Arctic Circle experiences the lsquoMidnight Sunrsquo

Q7 What is cause of Midnight Sun in NorwayA7 During the summer solstice (21 June) the North Pole is inclined towards the Sun Therefore the duration of sunlight or daytime increases from 12 hours at the Equator to 24 hours at the Arctic Circle and beyond Thatrsquos why The region beyond the Arctic Circle especially Norway is known as the Land of the Midnight Sun because there the Sun does not rise or set on 21 June

Q8 Match the column A with BA B

Summer Solstice 21st March

Autumn Equinox 23rd

September

Winter Solstice 21st June

(22frac12degS) This position of the Sun is referred to as the Winter Solstice because it marks the winter season in the Northern Hemisphere On that day the duration of day and night both are equal ( 12 hours day and 12 hours night) on every places located on Tropic of Capricorn (22frac12degS)SEASONS AND DURATION OF DAY AND NIGHT During the equinoxes all places on the Earth have 12 hours of day and 12 hours of night Due to the revolution of the Earth round the Sun on an inclined axis the duration of day and night varies according to seasons and the latitude of a placeDuring the summer solstice (21 June) the North Pole is inclined towards the Sun Therefore the duration of sunlight or daytime increases from 12 hours at the Equator to 24 hours at the Arctic Circle and beyondThe region beyond the Arctic Circle especially Norway is known as the Land of the Midnight Sun because there the Sun does not rise or set on 21 JuneAt the North Pole there will be six months of daylight The Sun will be seen always above the horizon at a low angle At 66degN 24 hours of sunlight can be seen only on 21 June Hammerfest in northern Norway is a place of tourist attraction for observing the phenomenon of the Midnight Sun This place has continuous daylight from 13 May to 29 July This place is easily accessible to tourists and has hotels and other facilities The view of the midnight Sun from here is enthrallingIn the Southern Hemisphere the duration of daylight decreases from 12 hours at the equator to 0 hours beyond the Antarctic Circle In the South Polar Region there is 24 hours of darkness The Sun is always below the horizon In the Southern Hemisphere which experiences winter the duration of night-time is longer than the duration of daylight

Spring Equinox 22nd

December

A8 A B

Summer Solstice 21st June

Autumn Equinox 23rd

September

Winter Solstice 22nd

December

Spring Equinox 21st March

During winter solstice (22 December) the South Pole is inclined towards the Sun The Southern Hemisphere experiences summer and the Northern Hemisphere has winter Therefore the duration of daylight or sunlight is greater in the Southern Hemisphere than in the Northern HemisphereThe duration of daylight increases from 12 hours at the equator to 24 hours beyond the Antarctic Circle The South Polar Region has 24 hours of sunlight for many days continuously At the South Pole there will be six months of sunlight The Sun will always be seen at a low angle above the horizon In the Northern Hemisphere the duration of daylight will decrease from 12 hours at the equator to 0 hours at the Arctic Circle There are 24 hours of darkness in the North Polar region The duration of night is greater than the duration of daylight as one move northwards from the Equator It is evident from the above table that the duration of daylight is 12 hours throughout the year at the equator only As one moves away from the equator the seasonal variations in the duration of daylight increase The seasonal variations in the duration of daylight are maximum at the Polar Regions

Subject Eng Literature (The Merchant of Venice ndash William Shakespeare)Topic Act II Scene 7 Lines 36 to 80 (End of scene ) [Students should read the original play and also the paraphrase provided]

Summary Questions amp AnswersThe Prince then examines the inscription on the silver casket which says ldquoWho chooseth me shall get as much as he deservesrdquo The Prince says that he deserves Portia more than anybody else because of his high rank his noble birth and his great wealth and power But then he argues that silver is ten times

(1) (Act II Sc 7 L 39-47)

From the four corners of the earth they come

To kiss this shrine this mortal breathing saint

The Hyrcanian deserts and the vasty wildsOf wide Arabia are as through-fares now

inferior to gold and therefore he cannot believe that the portrait of such a beautiful lady as Portia can be contained in the silver casket He decides to see the inscription on the golden casket before making his decision

The Prince goes to examine the inscription on the golden casket which says ldquoWho chooseth me shall get what many men desirerdquo The Prince believes that the whole world desires to possess Portia otherwise so many suitors would not have come from all corners of the world for winning Portia Some of them have come from the distant lands of Persia and Arabia The deserts of Persia (Hyrcanian deserts) and the boundless desolate lands of Arabia have been crossed by the Princes seeking the hand of Portia He contrasts this casket containing Portiarsquos portrait with the old English gold coin bearing the image of the archangel (angel of the highest rank) He goes on to remark that while the figure of the archangel is engraved (Insculped) upon the English coin the picture of Portia who is beautiful as an angel lies hidden inside one of the caskets namely the Golden Casket (Golden Bed)

On the basis of his assessment of the inscription on the golden casket the Prince decides to choose the golden casket He asks for the key and opens the golden casket only to find therein an empty human skull holding a roll of

For princes to come view fair PortiaThe watery kingdom whose ambitious headSpets in the face of heaven is no barTo stop the foreign spirits but they comeAs orsquoer a brook to see fair Portia

(i) Explain the occasion for the above mentioned speech

These are the comments of the Prince of Morocco after he reads the inscription on the golden casket His mental process is revealed to us in these words We find him debating within himself as to which casket he should choose

(ii) What light does the above speech throw on the personality of Prince of Morocco

From the above mentioned speech we come to know that the Prince of Morocco is keen to marry Portia He is the type of person who is easily taken away by outward appearance He is in love with Portia because of her beauty

(iii) What information can you gather about Portia from the above mentioned lines

The given speech shows that Portia is a very beautiful lady She must be possessed of good qualities because many suitors come to her place from all over the world with a desire to get married to her The Prince of Morocco is so impressed by her beauty that he calls her a saint According to him the whole world is desirous of having her

(iv) Elucidate the significance of the first two lines

In these lines the Prince of Morocco pays a compliment to Portia These lines show his admiration for her He says that people come from all parts of the world to see fair Portia

(v) Explain the meaning of the last four lines of the

passage

In these lines the Prince of Morocco says that even the vast oceans which throw a challenge at the sky are unable to prevent men from coming to Portiarsquos place to have a glimpse of her These lines are also a tribute to Portiarsquos beauty and good qualities Many men voyage across the ocean treating it as a mere stream to see the beautiful Portia

paper in which is written that whoever happens to be guided by the glitter of things is invariably deceived

On reading the scroll the Prince says that he is too sad at heart to speak a more formal farewell and leaves with his followers amidst a sound of trumpets

After the Prince of Morocco leaves Portia remarks that the Prince is a gentle fellow but she is rid of him May all persons of his nature make a similar choice

IMPORTANT PASSAGES EXPLAINED

(Act II Sc 7 L 39-43)From the four corners of the earth they come

To kiss this shrine this mortal breathing saintThe Hyrcanian deserts and the vasty wildsOf wide Arabia are as through-fares nowFor princes to come view fair Portia

Context

This passage occurs in Act II Scene 7 in The Merchant of Venice This is part of the speech made by the Prince of Morocco

(2)

(Act II Sc 7 L 48-53)

MOROCCO One of these three contains her heavenly pictureIst like that lead contains her

Twere damnation To think so base a thought it were too grossTo rib her cerecloth in the obscure graveOr shall I think in silver shes immurdBeing ten times undervalued to tried gold

(i) What meaning does the Prince of Morocco find out of the inscription of the golden casket What have Belmont and Portiarsquos house been called and why

The inscription on the golden casket is ldquoWho chooseth me shall gain what many men desirerdquo The Prince finds out that it means that the chooser of the golden casket will get Portia because many men desire her In fact the entire world desires her Because of the coming of many suitors to Belmont from different countries in order to win Portiarsquos hand Belmont has become a centre of pilgrimage and her house is the shrine where saintly Portia is installed

(ii) What does the Prince of Morocco do before making the final choice of the casket Which is the correct casket and who will win Portiarsquos hand

The Prince of Morocco surveys and analyses the inscriptions on the casket of lead silver and gold Before making the final choice like a very systematic and methodical person he once again considers the claims of the caskets The casket containing Portiarsquos picture is the correct casket and the person choosing it will win Portiarsquos hand

Explanation

While praising Portia the Prince of Morocco conceives Portia as a goddess whose image is placed inside one of the caskets Many suitors are coming from far and wide the north and the south the east and the west (Four corners) in order to try their luck Some of them have come from the distant land of Persia and Arabia The deserts of Persia (Hyrcanian deserts) and the boundless desolate lands of Arabia have been crossed by the Princes seeking the hand of Portia All this shows that Portia is indeed the most beautiful lady of the world

(iii) What does the Prince of Morocco say in his estimation while examining the motto on the silver casket What does he find in the golden casket

While examining the motto on the silver casket which says ldquoWho chooseth me shall get as much as he deservesrdquo Morocco says that in his own estimation he surely deserves Portia in all respects ndash rank birth wealth etc

He chooses the golden casket When he opens it he finds an empty human skull holding a scroll in which it is written that those who are attracted by the glittering outside of things are always deceived as Morocco has been deceived

(iv) What kind of nature does the Prince of Morocco have

The Prince of Morocco has a simple nature who does not look deeply into the inner meaning of things but is dazzled by the outward appearance of gold He is inclined to over-estimate his own value and does not realize that it is a duty to ldquogive and hazardrdquo To say that he will not hazard for lead shows that he misreads the true meaning of the inscription which is that he should be prepared to ldquohazard all he hathrdquo for Portia So his feeling is only one of fascination and romantic attraction

(v) Do you think that the lottery of the caskets is not a matter that will be determined by chance

In fact the lottery of the casket is not a matter that will be determined by mere chance but that it is a true test of character and of sincerity which is amply proved not only by Moroccorsquos choice but also by the arguments which he uses to help him in his choice

(Act II Sc 7 L 55-59)

They have in England

A coin that bears the figure of an angelStamped in gold but thats insculpd uponBut here an angel in a golden bedLies all within

Context

(3)

(Act II Sc 7 L 63-77)A carrion Death within whose empty eye

There is a written scroll Ill read the writing

All that glisters is not goldOften have you heard that toldMany a man his life hath soldBut my outside to beholdGilded tombs do worms infoldHad you been as wise as boldYoung in limbs in judgment oldYour answer had not been inscrolld

This passage occurs in Act II Scene 7 in The Merchant of Venice This is part of the speech made by the Prince of Morocco

Explanation

In this passage the Prince of Morocco bestows high praise on Portia whose hand he is seeking He contrasts this casket containing Portiarsquos portrait with the old English gold coin bearing the image of the archangel (angel of the highest rank) He goes on to remark that while the figure of the archangel is engraved (Insculped) upon the English coin the picture of Portia who is beautiful as an angel lies hidden inside one of the caskets namely the Golden Casket (Golden Bed) In the day of Elizabeth silver was ten times inferior in value to gold Therefore the Prince of Morocco believing that Portiarsquos portrait is contained in the Golden Casket decides to choose the Golden Casket

Fare you well your suit is coldCold indeed and labour lostThen farewell heat and welcome frostmdashPortia adieu I have too grievd a heartTo take a tedious leave Thus losers part

(i) What reward does the Prince of Morocco get after making a wrong choice of the Casket How does he feel

After making the wrong choice in selecting the casket of gold the Prince of Morocco as a reward earns a rebuke in the form of a scroll tucked in the empty eye-socket of a skull kept in the casket of gold The Prince is shocked and disappointed He becomes all the more sad and dejected when he reads the scroll which points to his foolishness in being misled by the appearance and outward show as indicative of its worth

(ii) How does the Prince respond after reading the scroll

After reading the scroll the Prince though upset accepts the result with good grace and decorum befitting a royal suitor and true sportsman He says that his love-suit is really cold otherwise he would have chosen correctly but now his efforts have been in vain So he bids farewell to Portia to the warmth and enthusiasm of love and welcomes the cold and bitterness of dejection and misery of life which lies ahead

(iii) What request does he make to Portia and why

After being failure in his mission he requests Portia to give him permission to leave at once because he is too sad to undergo the tediousness of a formal leave-taking He tells that it is the manner in which defeated persons part unceremoniously

(iv) Explain the following lines

ldquoAll that glisters is not goldOften have you heard that toldMany a man his life hath soldBut my outside to beholdGilded tombs do worms infoldrdquo

Mere glitter does not make a metal to be gold Man has often been warned against appearance but it has been of no use Many people have sacrificed their lives only to seek the outer appearance of gold Worms are found inside the gilded

monuments

Class XSubject Topic Summary Execution

Hindi 2ndlang

नया रासता भाग 6 मायाराम 0ी घर म धनी मल 0ी और उनी बटी सरिरता ी ही चचा बनी रहती थी अमिमत ो इसम ोई रलिच ना थी वह धनी घर ी लडी स शादी र सवय ो बचना नही चाहता था उसा भी सवाणिभमान ह ईशवर ी पा

स उस पास पस ी ोई मी नही थी अभी उसन फकटरी ही लगाई थी उसी समझ बाहर था कि उस घर वालो ा झाव पस ी तरफ कयो

ह उसन मा स सवाल किया कि मा तम सरिरता स मरी शादी कयो रना चाहती हो मा न उस समझाया कि वह दखन म बरी नही ह और किफर खानदान अचछा

ह वह ए शल गरहणी रप म घर सभाल सगी अमिमत न मा ो इस बात ा एहसास राया कि मीन सबध लिलए मना रन पर उस दिदल

पर कया बीती होगी मा और अमिमत ी लडी बार म ाफी बात हईमा ा झाव सरिरता ी तरफ था कयोकि वह घर पर अचछा दह0 लर आ रही

थी अमिमत न अपनी मौसी ी बरी हालत बार म बताया कि किस तरह वह बड घर ी खानदानी बटी लाई थी और आ0 उसी हालत कितनी खराब ह लाई थी बहकलब 0ाती ह और बचचो ो भी नही दखती ह बात चल ही रही

थी कि तभी ए ार बाहर आर री धनी मल0ी घर अदर आए और पीछ स डराइवर फल ी ए टोरी लर आया अदर आए और पीछ स

डराइवर ए टोरी फल ी लर आया अमिमत ो फल ी पटी बरी लग रही थी अमिमत न पछ लिलया यह फल कयो ल आए ह प इन सब ी कया

0ररत थी उनो न 0वाब दिदया कि 4 पटी शमीर स मगाए थ अमिमत ो या सनर करोध आ गया तभी उस किपता 0ी आ गए उन आत ही अमिमत उठर बाहर चला गया वहा वहा मा पास आर बठ गया और बोला

अभी रिरशता तय नही हआ और धनी मल 0ी धनी मल 0ी फल ी पटी लर चलआय मा न समझाया कि 0ब सबध 0ड 0ाता ह तो खाली हाथ नही

आत अमिमत न मा स हा कि तम सबन सरिरता ो इस घर म लान ी ठान रखी ह धनीमल 0ी उस दिदन सरिरता ो दखन ी तारीख तय रन आय थ

Commercial Studies

Banking Nowadays Bank provide easy and quick services through internet facilities methods of Banking is called internet bankingIn order to save the time and money involved in visiting Bank branches people increasingly prefer to have internet banking

There are different modes of doing internet banking or transferring money through online They areReal Time Gross Settlement (RTGS)National Electronic Fund Transfers (NEFT)

1

Question

1) Explain the term RTGS Write the features of RTGS

Answer)The acronym RTGS stands for Real Time Gross Settlement which may be defined as the continuous real time settlement of funds transfer individually on and order by order basis without netting lsquoReal timersquo may be defined as the processing of instructions at the time they are received rather than at some letter time lsquoGross settlementrsquo may be defined as the settlement of transfer instructions which occurs

individually

Features of RTGS1It is the continuous settlement of

funds transfer individually on an order by order basis

2RTGS facility is provided only by CBS core banking solution enabled Bank branches

3Amount charged from the customer for RTGS transactions vary from bank to bank

2) Explain the term NEFT Write the features of NEFT

Answer) National electronic funds transfer may be defined as a nationwide system that facilitates individuals Farms and copper operates to electronically transfer funds from any bank branch to any individual farm or corporate having an account with any other bank branch in the country

Features of NEFT2 Transfer can be made 7 times on

weekdays and 6 times on Saturday

3 NEFT cannot be used to receive foreign remittances

4 NEFT transaction takes place in batches

5 A bank branch must be NEFT enabled to become a part of NEFT fund transfer network

6 There is no maximum or minimum amount that can be transferred through NEFT when one bank has a bank account

English Language

CompositionEssay

A composition is an art of creating a piece of writing on any topic or subject It is the writing correctly beautifully and clearly in order to make some interesting reading Structure of the composition

Introduction ( you lay the foundation for your composition)

Body (it constitutes the main part of the essay)

Conclusion (final statement that leaves a lasting impression)

Kinds of essays1 The Narrative essay2 The descriptive essay3 The reflective essay4 The argumentative essay

Write a composition on any one of the following topics (350- 400 words)

1 Friendship Or2 The first day of your school

Subject Eng Literature (The Merchant of Venice ndash William Shakespeare)Topic Act V Scene 1 Lines 127 to 158 (Nerissa helliphellip The clerk will nersquoer wear hair onrsquos face that had it) [Students should read the original play and also the paraphrase given in the school prescribed textbook]

Summary Revision Questions o Soon thereafter Bassanio Gratiano

and Antonio arrive

o Bassanio tells Portia that he is feeling as if it is morning because of the presence of Portia who is shining like the sun When Antonio is introduced by Bassanio to Portia she tells Bassanio that he should be grateful to Antonio who took so much trouble on his account even to the extent of risking his life

o Nerissa starts quarrelling with Gratiano and demands that he show her the ring she had presented to him and which she had warned him not to lose She suspects that Gratiano must have presented the ring to some young woman and not to the lawyerrsquos clerk as he repeatedly says and assures

Answer the following questions to check your preparation of Act IV Scenes 1 and 2

You must attempt only after you have completed your preparation of Act IV The answers must be in complete sentences using textual evidence (with citation) when necessary

[It would be in your own interest to attempt the above questions honestly totally refraining from consulting your textbook or your notes during answering After completion you should correct the paper yourself consulting the textbooknotes etc and award marks as specified Please let me know the marks you scored through WhatsApp in the group or to my personal WhatsApp]

Act IV Scene 1 (each question carries 2 marks)

1 What did the Duke try to do for Antonio

2 Why does Shylock refuse to show mercy How does he justify his stance

3 Why does Antonio say he is ready to die 4 What information is contained in Bellariorsquos letter

5 Why does Portia (as Balthazar) assert that Shylock must show mercy How does he respond

6 What offers are made to Shylock to get him to spare Antonio How are they received

7 What does Antoniorsquos speech as he faces the prospect of Shylockrsquos knife tell you about his character

8 How do Bassanio and Gratiano react to the looming prospect of Antoniorsquos demise

9 How does Portia (as Balthazar) use the law to turn the tables on Shylock

10 What does the Duke decree should happen to Shylock Why What happens to Shylockrsquos estate

11 What does Portia ask Bassanio as payment for her ldquoservicesrdquo What is his initial response What makes him change his mind

Act IV Scene 2 (each question carries 1frac12 marks)

1 What does Gratiano bring to Portia (Balthazar)

2 What does Nerissa plan on getting from Gratiano What does Portiarsquos comment suggest about men

ECO-10 280620 Topic-Supply AnalysisSHIFTING OF SUPPLY

But if there is change in factors other than the price of the commodity then either more is supplied at the same price or less supplied at the same price In such cases the price of the commodity remains constant but there is a change in other factors like change in the price of inputs change in technology of production change in price of other related goods change in taxation policy of the government etc For example there is an improvement in the technology of production of the commodity in question It leads to decrease in per unit of cost production of the commodity The firm is willing to sell more quantity of the commodity at the same price So the supply other commodity increases at the same price This increase in supply is shown by rightward shift of supply curve On the other hand if the firm uses inferior technology of production the cost of production per unit of the commodity increases The firm is willing to sell less quantity at the same price So the supply of the commodity decreases at the same price This decrease in supply is shown by leftward shift of the supply curve The above cases of increase and decrease in supply can be shown with the help of the following figures

Y INCREASE OF SUPPLY Price (Rs) s

P A s1

B

s

X` O s1 X

q q1

Y` Quantity demanded (in units)

Y DECREASE IN SUPPLY s2

s

price (Rs)

C

p A

s2

s

X` o X

q2 q

Y` Quantity demanded ( in units)

Main factors causing increase in supply or rightward shift of supply Curve(i) Fall in the price of other related goods

(ii) Fall in the price of inputsfactors(iii) Use of better technology in production(iv) Decrease in the rate of excise duty by government(v) If the objective of producer changes from profit maximization to salesMaximization

Main factors causing decrease in supply or leftward shift of supply curve(i) Increase in the price of other related goods(ii) Rise in the price of inputsfactors(iii) Use of inferior technology in production(iv) Increase in the rate of excise duty by the government(v) If the objective

Subject - Biology Topic ndash Chapter mdash6 PhotosynthesisSummary Execution

Today we will know about photosynthesis and its stages

Q1 What do you mean by photosynthesis The process by which living plants containing chlorophyll produce food

substances from carbon-di- oxide and water by using light energy Sunlight

6CO2 +12 H2O----------------------- C6 H12O6 + 6H2O + 6O2

Chlorophyll

Q2 What are the importance of photosynthesis I) Food for all Green plants trap solar energy by photosynthesis

process and supply food and energy for all living organisms either directly or indirectly

Ii) Oxygen to breathe in by product of photosynthesis is oxygen which is essential for all living organisms respiration

Q3 Write about two main phases of photosynthesis A Light dependent phase This phase occur in grana of chloroplast I) The chlorophyll on exposure to light energy becomes activated by

absorbing photons Ii) The absorbed energy is used in splitting the water molecules (H2O)

into its two components (H+ and OH- ) and releasing electron s 2H2O------------------------- 4H+ + 4e- +O2

Energy of 4 photons This reaction is known as photolysis

End products are H+ and oxygen water

B Light independent (Dark ) phase The reactions in this phase require no light energy

Here CO2 combine with H+ and produce glucose

Class XI

Subject Topic Summary ExecutionEVS Chapter-4 Legal

regimes for sustainable development

Environmental legislationEnvironmental legislation is the collection of laws and regulations pertaining to air quality water quality the wilderness endangered wildlife and other environmental factors The act ensures that matters important to the environment are thoroughly

Learn -The Forest (Conservation) Act 1980

considered in any decisions made by federal agencies

The Forest (Conservation) Act 1980 The Forest (Conservation) Act 1980 an Act of the Parliament of India to provide for the conservation of forests and for matters connected therewith or ancillary or incidental thereto It was further amended in 1988 This law extends to the whole of IndiaObjects and Reasons of the Forest Conservation Act

Deforestation causes ecological imbalance and leads to environmental deterioration Deforestation had been taking place on a large scale in the country and it had caused widespread concern The act seeks to check upon deforestation and de-reservation of forests

Subject Eng Literature (The Tempest ndash William Shakespeare) Topic Act II Scene 1 Lines 314 to 329 (End of scene)

[Students should read the original play and also the paraphrase given in the school prescribed textbook]Summary Questions amp Answers

Conspiracy of Antonio and Sebastian (Contd)

o As they approach Ariel appears again and wakes up Gonzalo by singing a tune in his ear Alonso also wakes up and they see both Sebastian and Antonio with drawn swords On being caught off guard they make up a story saying that they had heard a bellowing of bulls or lions

o They then moved to another part of the island

o Ariel at once rushes to Prospero to inform him of this development

SUMMING-UP of ACT-2 SCENE-1

(i) Among the survivors Ferdinand is separated from the rest which results in the disconsolate grief of Alonso as he took him for dead

(ii) The villainy of Antonio is confirmed

(iii) The supremacy of Prosperorsquos magic which resulted in the failure of the human conspiracy

(1)

(Act II Sc 1 L 311-325)SEBASTIAN Whiles we stood here securing your repose

Even now we heard a hollow burst of bellowing Like bulls or rather lions Didt not wake youIt struck mine ear most terribly

ALONSO I heard nothingANTONIO O rsquotwas a din to fright a monsters ear

To make an earthquake Sure it was the roarOf a whole herd of lions

ALONSO Heard you this GonzaloGONZALO Upon mine honour sir I heard a humming

And that a strange one too which did awake meI shaked you sir and cried As mine eyes opened I saw their weapons drawn There was a noiseThats verily rsquoTis best we stand upon our guardOr that we quit this place Lets draw our weapons

(i) Why has Prospero sent Ariel to Gonzalo and Alonso What does Ariel do to awaken Gonzalo

Prospero has already come to know by his magic powers the danger which threatens Gonzalo who had been Prosperorsquos friend and so he sent Ariel to preserve the lives of both Gonzalo and Alonso Prospero does not want that his scheme should remain unfulfilled Ariel begins to sing a song in Gonzalorsquos ears to awaken him(ii) Who are ready to carry out their plan Who takes steps to stop them Why does Gonzalo feel surprised after being awakened

Sebastian and Antonio are ready to carry out their plans They are standing with their swords drawn to kill Alonso and

(iv) We see two sets of contrasting characters Gonzalo-Adrian against Antonio-Sebastian

(v) The grief that works in Alonso can be perceived to his repentance for his association in Antoniorsquos crime against Prospero

Gonzalo Ariel takes steps to stop them from carrying out their nefarious scheme When Gonzalo is awakened by the song sung by Ariel into his ears he (Gonzalo) feels surprised because he sees Sebastian and Antonio standing with their swords drawn(iii) What reason do Sebastian and Antonio tell of drawing their swords when they are suspected by Alonso and Gonzalo

When Sebastian and Antonio are seen with their swords drawn they are looked with suspicion by Gonzalo and Alonso At first Sebastian tells them that as they stood here to guard them during their sleep they heard only a little before a sudden loud noise very much like the roaring of bulls or more probably that of lions Then Antonio follows him saying that this was a noise so terrible as to frighten even a monsterrsquos ears and this noise could even have shaken the earth and it was surely like the roaring of a multitude of lions Then seeing the danger they have drawn their swords Perhaps after hearing the terrible noise they (Gonzalo and Alonso) woke up from their sound sleep

(iv) What does Gonzalo tell Alonso about the strange noise What did he see on opening his eyes Gonzalo tells Alonso that he did not hear the sound of roaring but he heard a humming sound which was strange and which woke him up After waking up he gave him (Alonso) a shaking and a loud cry On opening his eyes he saw these two gentlemen standing with their swords drawn(v) What does Gonzalo suggest

Gonzalo suggests that there was a noise indeed and of that he has no doubt at all and suggests that the best course for them would be to remain alert and vigilant against any possible danger to their lives or to leave this place and move to some other part of the island

Class XIISubject Topic Summary Execution

Commerce

Chapter- Management

Today we will discuss about LEVELS OF MANAGEMENT

Levels of management is a series or chain of managerial positions from top to bottom It helps individuals to know their authority responsibilities and superior-subordinate relations among themselves There are mainly three levels of Management TOP LEVEL MANAGEMENTMIDDLE LEVEL MANAGEMENTLOWER LEVEL MANAGEMENT

Top level managementIt consists of members at the highest level in the management hierarchy This level includes Board Of Directors Chief Executive Managing Directors Chairman President Vice President

Rolefunctions of the top levelmanagement1To analyse evaluate and deal

with theexternal environment2 To determine the objectives and

policies of the business3 To strive for welfare and survival

of business

4 To create an organisational Framework consisting of authority responsibility relationship

Middle level management Congress of members or groups who are concerned with implementation of the policies let down by the top managementThis level includes head of the department such as finance manager marketing manager branch and regional managers departmental and divisional heads plant superintendent etc

Role of functions of the middle level management

1 To interpret the policies framed by top management

2 To assign duties and responsibilities to lower level managers

3 To select and appoint employees for middle and supervisory level and evaluate their performance

4 To co-operate with other departments for smooth functioning

Operational or supervisory level managementIt refers to the group are members who are concerned with execution of the work They are also known as fast line managers This level includes supervisor 4 men Section Officer clerk Inspector etc

Role of functions of the lower level management1 To plan and execute day-to-

day operations2 To supervise and control the workers3 To arrange materials and

tools to start the process and make arrangements for training

4 Today present workers grievance and suggestions before the management and

ensure safe and proper working conditions in the factory

Business Studies

Staff Appraisal Chapter- 10 Today let us start with a new chapter

Staff Appraisal

Meaning of Performance Appraisal

Performance Appraisal is the systematic evaluation of the performance of employees and to understand the abilities of a person for further growth and developmentThe supervisors measure the pay of employees and compare it with targets and plansThe supervisor analyses the factors behind work performances of employeesThe employers are in position to guide the employees for a better performance

Objectives of Performance Appraisal

Following are the objectives of Performance Appraisal

To maintain records in order to determine compensation packages wage structure salaries raises etc

To identify the strengths and weaknesses of employees to place right men on right job

To maintain and assess the potential present in a person for further growth and development

To provide a feedback to employees regarding their performance and related status

To provide a feedback to employees regarding their performance and related status

Importance of Performance Appraisal

Performance appraisal provides important and useful information for the assessment of employees skill

knowledge ability and overall job performance The following are the points which indicate the importance of performance appraisal in an organization

1 Performance appraisal helps supervisors to assess the work performance of their subordinates

2 Performance appraisal helps to assess the training and development needs of employees

3 Performance appraisal provides grounds for employees to correct their mistakes and it also provides proper guidance and criticism for employees development4 Performance appraisal provides reward for better performance

5 Performance appraisal helps to improve the communication system of the organization

6 Performance appraisal evaluates whether human resource programs being implemented in the organization have been effective

7 Performance appraisal helps to prepare pay structure for each employee working in the organization

8 Performance appraisal helps to review the potentiality of employees so that their future capability is anticipated

Geography

DRIANAGE The SubarnarekhaThe Subarnarekha and the Brahmaniinterposed between the Ganga and the Mahanadi deltas drain an area of 19300 sq kmand 39033 sq km respectively The drainage basins of these streams are shared byJharkhand Odisha west Bengal and Chhattisgarh The Brahmani is known as southKoel in its upper reaches in Jharkhand

The NarmadaThe Narmada rises in the Amarkantak hills of MadhyaPradesh It flows towards the West in a rift valleyformed due to a geological fault The total length of it is 1300 km All the tributaries of the

Q1 Name the two westward flowing rivers in the peninsular plateauA1 Narmada and Tapi are the only westward flowing rivers of the peninsular plateau

Q2 Differentiate between east-flowing rivers and west-flowing riversA2

East-flowing rivers

West-flowing rivers

Narmada are very short inlength Most of its tributaries join the main streamright anglesThe Narmada basin covers parts of Madhya Pradesh and Gujarat

The Tapi The Tapi rises in the Satpura ranges in the Betul listrictof Madhya Pradesh It flows in a rift valley parallel tothe Narmada but it is much shorter in length It coversparts of Madhya Pradesh Gujarat and MaharashtraThe length is about 724 km

The Sabarmati and the MahiThe Sabarmati rises in the Aravali hills and flows south-south-westwards for a distance of 300 kilometres to the Arabian Sea The Sabarmatibasin extends over an area of 21674 sq km in Rajasthan and Gujarat The Mahi rises inthe east of Udaipur and drains an area of 34842 sq km lying in Madhya PradeshRajasthan and Gujarat It flows south-westwards for a distance of 533 km before it fallsinto the Gulf of Khambhat

The ChambalThe Chambal rises near Mhow in the Vindhya Range and flows towards the northgenerally in a gorge upto Kota Below Kota it turns to the north-east direction and afterreaching Pinahat it turns to the east and runs nearly parallel to the Yamuna beforejoining it in the southern part of the Etawah district in Uttar PradeshMajor Rivers of India with their basin area (Sqkm)

Himalayan System Indus 321290Ganga 861404

Brahmaputra 187110Indus System

Jhelum 34775Beas 20303

Ganga System Yamuna 366223Ghaghra 127950

Peninsular RiversNarmada 98796

Tapi 65145Mahanadi 141600

Subarnarekha 19300Sabarmati 21674

Mahi 34842Godavari 312812

Godavari Krishna Kaveri Mahanadi are the east-flowing rivers

Narmada Tapi west-flowing rivers

They fall into the Bay of Bengal

They fall into Arabian Sea

These rivers form big deltas

These rivers form comparativelysmall deltas

Catchment areas of these rivers are larger

Catchment areas of these rivers are smaller

Krishna 2589488Cauveri 87900

Subject ndashBiology Topic ndashChapter -5 Inheritance amp Variations Summary ExecutionToday we will discussabout linkage and its classification

LINKAGE The tendency of the genes located on the same chromosome to stay together is

hereditary transmission Linked genes the genes responsible for this Genes that exhibit the process of linkage locates in the same chromosome The distance between the linked genes in a chromosome determines the strength

of linkage i e genes that are located close to each other show stronger linkage than that are located far from each other

COMPLETE LINKAGE It is the type of linkage showed by the genes that are closely located or are tightly

linked with each other as they have no chance of separatingby crossing over These genes are always transmitted together to the same gamete and the same

offspring In such condition only parental or non cross over type of gametes are formedINCOMPLETE KINKAGE It is type of linkage showed by the genes that are distantly located orare loosely

linked with each other because they have chance of separating by crossing over

SIGNIFICANCE i) It helps in holding the parental character togetherii) It checks the appearance of new recombination and helps in bringing the

hybrid population which resembles the original parents iii) Linked genes dilute the effects of undesirable traits

Subject Eng Literature (The Tempest ndash William Shakespeare) Topic Essay Questions (EQ-3)Question No 3

Give a character sketch of CalibanAnswer

The character of Caliban has been wonderfully conceived by Shakespeare as the manifestation of all that is gross and earthy ndash a sort of creature of the earth as Ariel is a sort of creature of the air

Calibanrsquos Physical Appearanceo Caliban is lsquofreckledrsquo a lsquomisshapen knaversquo not honoured with human shape

o Prospero calls him lsquothou tortoisersquo (Act I Sc 2 Line 317) Trinculo stumbling upon him describes him as ldquoA strange fish hellip Legged like a man And his fins like armsrdquo He ldquosmells like a fishrdquo (Act II Sc 2 Line 25)

o Prospero also calls him a ldquobeastrdquo (Act IV Sc 1 Line 140) and ldquoThis misshapen knaverdquo (Act V Sc 1 Line 268)

o Further it appears that in addition to his physical deformity his spiritual inferiority is also suggested by Prosperorsquos claim that his birth resulted from the union between his mother the witch Sycorax and the devil

Calibanrsquos ParentageWhen the play opens Caliban is twenty four years of age having been born on the island twelve years before the coming of Prospero His mother was the foul witch Sycorax who was banished from Algiers for ldquomischiefs manifold and sorceries terrible to enter human hearingrdquo (Act I Sc 2 Line 264) and the father was the Devil himself Thus

Caliban is a monster of evil and brute nature ugly deformed and stinking

Calibanrsquos Savage and Malignant Natureo Caliban is entirely a creature of the earth ndash gross brutal and savage He regards himself as the rightful possessor

of the island and Prospero as a usurper

o In his young age he was on good terms with Prospero He had consented to be received by Prospero at his house and to be educated by him He has learnt human language only to curse his master whom he abhors

o His beastly nature soon breaks out and ends in a vicious attack on Miranda This opens the eye of Prospero who becomes severe to him and enforces his service by threats and violence

o Prospero uses him to make dams for fish to fetch firewood scraper trenches wash dishes and keep his cell clean

Calibanrsquos Hatred for ProsperoA profound hatred for Prospero has taken hold of Caliban It springs from a sense of his being dispossessed and ill-treated He would kill Prospero if he could but he knows the power of Prosperorsquos lsquobookrsquo Hence he transfers his allegiance to Stephano who seems like a god to him He also incites the two drunken associates to batter the skull of Prospero when he sleeps in the afternoon

Caliban Shows Considerable Intelligenceo He has learnt Prosperorsquos language

ldquoYou taught me language and my profit onrsquot (Act II Sc 2 Lines 86-89)Is I know how to curserdquo

o He is well aware of the futility of arguing with one who has more power than he has

ldquoI must obey his art is such power (Act I Sc 2 Lines 373-376)It would control my damrsquos god SetebosAnd make a vassal of himrdquo

o He realizes the importance of Prosperorsquos books

ldquoRemember (Act III Sc 2 Lines 89-92)First to possess his books for without themHersquos but a sot as I am nor hath notOne spirit to commandrdquo

o He knows the value of stealth when attacking the enemy

ldquoPray you tread softly that the blind mole may not (Act IV Sc 1 Lines 194-195)Hear a foot fall we now are near his cellrdquo

o Caliban has a better set of values than Stephano and Trinculo They are distracted from their plan by their greed for Prosperorsquos rich garments Only Caliban realizes that such a finery is unimportant

ldquoLeave it alone thou fool it is but trashrdquo (Act IV Sc 1 Lines 224)

Caliban is not a good judge of characterCaliban is not a good judge of character He decides for example that Stephano is a god because he dispenses lsquocelestial liquorrsquo (Act II Sc 2 Line 115) but then it must be remembered that he has only known his mother Sycorax Prospero Miranda and the spirits that torture him However he quickly discovers his error of judgementrdquo

ldquoWhat a thrice-double ass (Act V Sc 1 Lines 295-297)Was I to take this drunkard for a godAnd worship this dull foolrdquo

Calibanrsquos Imaginative NatureIf Caliban is sub-human in what has been said above he is human in the respect of the poetic side of his character He listens to music with rapture He tells of the beautiful dreams in which heaven rains treasures upon him and which upon waking he yearns to renew One of the most poetic passages in whole play is Calibanrsquos description of the island

to Stephano and Trinculo

ldquoBe not afeard The isle is full of noises (Act III Sc 2 Lines 135-143)Sounds and sweet airs that give delight and hurt notSometimes a thousand twangling instrumentsWill hum about mine ears and sometime voicesThat if I then had waked after long sleepWill make me sleep again and then in dreamingThe clouds methought would open and show richesReady to drop upon me that when I wakedI cried to dream againrdquo

Caliban - Less Ignoble Than Some OthersCalibanrsquos motive for murder is less dishonourable than that of Antonio and Sebastian They plan to kill Alonso to gain his power and wealth Caliban merely wants revenge and the return of lsquohisrsquo island

Conclusiono Calibanrsquos character is not portrayed very clearly in the play and hence we cannot decide whether he is a poor

savage being grossly maltreated by Prospero or whether he is evil and must therefore be kept in bondage or enslavement

o Caliban is contrasted with Ariel who is a spirit and thus swift and uninterested in physical activitieso Caliban is also contrasted with Prospero who is the all-powerful master of the island and of the destiny of all

those on the islando Caliban is also contrasted with civilized man showing him to be less evil than Antonio and Stephano and less

materialistic than Stephano and Trinculoo Caliban has suffered at the hands of Prospero and he has learnt to curse by listening to Prosperorsquos abuse He

certainly believes that Prospero has deprived him of his birthrighto Finally the character Caliban is thought to be one of Shakespearersquos masterpieces The complexity of the character

is reflected in the large volume of critical discussion that has grown around it

ECO ndash12 Topic-Forms of market

MonopolyMonopoly is a market structure in which there is a single seller there are no close substitutes for the commodity produced by the firm and there are barriers to entry Example Indian Railways which is operated under government of India Monopoly also implies absence of competitionFeatures of Monopoly Monopoly is characterized by1 Single Seller In monopoly there is only one firm producing the product The whole industry consists of this single firm Thus under monopoly there is no distinction between firm and industry Being the only firm there is significant control of the firm over supply and price Thus under monopoly buyers do not have the option of buying the commodity from any other seller They have to buy the product from the firm or they can go without the commodity This fact gives immense control to the monopolist over the market

2No Close Substitute There are no close substitutes of the product produced by the monopolist firm If there are close substitutes of the product in the market it implies presence of more than one firm and hence no monopoly In order to ensure a total of control over the market by the monopolist firm it is assumed that there are no close substitutes of the product

3 No Entry amp Exit Monopoly can only exist when there is strong barriers before a new firm to enter the market In fact once a monopoly firm starts producing the product no other firm can produce the same One reason for this is the ability of the

monopolist to produce the product at a lower cost than any new firm who thinks to enter the market If a new firm who knows that it cannot produce at a lower cost than the monopolist then that firm will never enter the market for fear of losing out in competition Similarly the monopolist who is operating for a long time may be enjoying reputation among its customers and is in a better position to use the situation in its own benefit A new firm has to take long time to achieve this and so may not be interested to enter the market

4 Price Maker Being the single seller of the product the monopolist has full control over the pricing of the product On the other hand if there is a large number of buyers in the market so no single buyer exercises any significant influence over price determination Thus it is a sellerrsquos market So monopoly firm is a price maker

5 Price Discrimination Having considerable control over the market on account of being single seller with no entry of other firms the monopolist can exercise policy of price discrimination it means that the monopolist can sell different quantities of the same product to a consumer at different price or same quantity to different consumers at different prices by adjudging the standard of living of the consumer

6 Shape of Demand Curve Since a monopolist has full control over the price therefore he can sell more by lowering the price This makes the demand curve downward sloping

Subject Ac-12 290620 Topic- retirement Model sumThe Balance Sheet of Rohit Nisha and Sunil who are partners in a firm sharing profits according to their capitals as on 31st March 2014 was as under

Liabilities Amount Assets Amount (Rs) (` Rs)

Creditors 25000 Machinery 40000Bills Payable 13000 Building 90000General Reserve 22000 Debtors 30000Capital Less Provision for Rohit 60000 Bad debts 1000

29000 Nisha 40000 Stocks 23000 Sunil 40000 140000 Cash at Bank 18000

200000 200000

On the date of Balance Sheet Nisha retired from the firm and following adjustments were made(i) Building is appreciated by 20(ii) Provision for bad debts is increased to 5 on Debtors(iii) Machinery is depreciated by 10(iv) Goodwill of the firm is valued at Rs 56000 and the retiring partnerrsquos share is adjusted

(v) The capital of the new firm is fixed at Rs120000 Prepare Revaluation Account Capital Accounts of the partner and Balance Sheet of the new firm after Nisharsquos retirement Revaluation AccountDr Cr

Particulars Amount Particulars Amount (`Rs) (Rs`)

Provision for Bad debt Ac 500 Building Ac 18000Machinery Ac 4000Profit transferred toCapital Accounts (3 2 2)Rohit 5786Nisha 3857Sunil 3857

13500

18000 18000

Capital Account

Dr Cr

Particulars Rohit Nisha Sunil Particulars Rohit Nisha Sunil (Rs`) (Rs`) (`Rs) (Rs`) (Rs`) (Rs`)

Sunilrsquos Capital ac 9600 mdash 6400 Balance bd 60000 40000 40000Bank - 66143 - General Reserve 9428 6286 6286Balance cd 72000 mdash 48000 Revaluation (Profi 5786 3857 3857 Rohitrsquos Capital Ac mdash 9600 mdash

Sunilrsquos Capital Ac 6400 Bank 6386 - 4257

81600 66143 54400 81600 66143 54400

Balance Sheet as at 31st March 2014

Liabilities Amount Assets Amount (Rs`) (Rs`)

Creditors 25000 Building 108000Bank overdraft 37500 Machinery 36000

Bills Payable 13000 Debtors 30000Capital Less ProvisionRohit 72000 for Bad debts 1500 28500Sunil 48000 120000 Stock 23000

195500 195500

Working Notes (i) (a) Profit sharing ratio is 60000 40000 40000 ie = 3 2 2(b) Gaining Ratio Rohit = 35 ndash 37 = 2135 ndash 1535 = 635Sunil = 25-27 = 1435 ndash 1035 = 435= 635 435= 6 4 = 3 2(c) Nisha Share of Goodwill = Rs 56000 times 27 = Rs16000Share of Goodwill in the gaining ratio by the existing partner ieRohit = Rs16000 times 35 = Rs 9600Sunil = Rs 16000 times 25 = Rs 6400

The journal entry isRohitrsquos Capital Ac Dr 9600Sunilrsquos Capital Ac Dr 6400 To Nisharsquos Capital Ac 16000(Share of Goodwill divided into gaining ratio)

  • 1 Static Friction
  • The frictional force that acts between the surfaces when they are at rest with respect to each other is called Static Friction
    • Static Friction Examples
      • 2 Sliding Friction
        • Examples Of Sliding Friction
          • 3 Rolling Friction
            • Examples Of Rolling Friction
              • Objects and Reasons of the Forest Conservation Act
Page 25:  · Web viewSubject . Topic . Summary . Execution . English 1 . Sounds of animals . Hens –cackle Horses –neigh Lions –roar Owls –hoots Snake –hiss. English 2 . Mother’s

MAT

HEM

ATIC

S

Ch 1

1Al

gebr

ic E

xpre

ssio

n

1 Constant A symbol which has fixed value is called a constant[eg 8 23 -15 radic3 etc]

2 VariableA symbol which does not have any fixed value but may be assigned value (values) according to the requirement is called variable or literal[eg x y p q etc]

3 TermsA term is a number (constant) a variable a combination (product or quotient) of numbers and variables[eg 7 x 5x etc]

4 Algebric expressionA single term or acombination of two or more terms connected by plus (+) or minus (-) sign forms an algebraic expression[eg 5-y 3x2-5x xy-6z+4 etc]

5 PolynomialAn algebraic expression which contains more than one term is called a polynomial (multinomial)[eg x2-5x 5y+xy+x2y etc]

6 Degree of polynomial(a) When the polynomial contains only one variable the highest power of the variable is the degree of the polynomialeg the degree of the polynomial of 4x-7x5+8 is 5(b) When the polynomial contains two or more variablesStep (i) Find the powers of the variables in each term (ii) The highest sum of the powers is taken to be the degree of the polynomialeg the degree of the polynomial 5x2y-4x3y5+6 is = 3+5 = 8Remember An algebraic expression is a polynomial if degree of each term used in it is a non-negative integer

Exercise ndash 11(A)

1 Separate the constants and variables from the following

-7 7+x 7x+yz radic5 radic xy 3 yz

8 45y -3x

Solution Constant Variables-7 radic5 7+x 7x+yz radic xy

3 yz8

45y -3x

2 Write the number of terms in each of the following polynomials(i) 5x2+3timesax (ii) axdivide4-7 (iii) ax-by+ytimesz (iv) 23+atimesbdivide2

Solution Polynomials Number of terms(i) 5x2+3timesax 2(ii) axdivide4-7 2(iii) ax-by+ytimesz 3(iv) 23+atimesbdivide2 2

4 Write the degree of the each polynomials(i) xy+7z (ii) x2-6x3+8 (iii) y-6y2+5y8 (iv) xyz-3 (vi) x5y7-8x3y8+10x4y4z4

Solution Polynomials Degree(i) xy+7z 2(ii) x2-6x3+8 3(iii) y-6y2+5y8 8(iv) xyz-3 3(vi)x5y7-8x3y8+10x4y4z4 12

5Write the coefficient of(i) ab in 7abx (iv) 8 in a2-8ax+a (v) 4xy in x2-4xy+y2

SolutionCoefficient

(i) ab in 7abx 7x(iv) 8 in a2-8ax+a -ax(v) 4xy in x2-4xy+y2 -1

7 CoefficientAny factor of an algebraic quantity is called the coefficient of the remaining quantityeg in the algebraic term 7xyz 7 is coefficient of xyz 7x is coefficient of yz and so on

8 Like term The terms having the same literal coefficient are called like terms and those having different literal coefficients are called unlike terms

eg (i) 5xyz 8xyz -6xyz and 23xyz are like

terms(ii) 7xy2 8x2yz and -15xyz2 are unlike terms

6 in 57xy2z3 write the coefficient of

(i) 5 (vii) 5xy2 (viii) 17yz (xi) 5xyz

Solution Coefficient

(i) 5 17

xy2z3

(vii) 5xy2 17z3

(viii) 17yz

5xyzsup2

(xi) 5xyz 17yz2

7 In polynomial given below separate the like terms(ii) y2z3 xy2z3 -58x2yz -4y2z3 -8xz3y2 3x2yz and 2z3y2

Solution y2z3 -4y2z3 2z3y2 are like terms

xy2z3 -8xz3y2 are like terms

-58x2yz 3x2yz are like terms

Class IXSubject Topic Summary Execution

Bengali (2nd language)

বাগzwnjধারাzwnj বা ধারা-বা ধারা ল হিবকেশষ পরকার বাক -হিb -াকেবর এক হিবকেশষ পরকাশরীহিত াকেক কতগকেলা কার সমষটির মকেধয এগহিলকেক বা ধারা বকেল আবার কতগকেলা শকেবদর বাধাধরা যকান রীহিত যনই য-াকেব চকেল আসকে যসই -াকেবই চকেল আসকে তখন যসই শবদগহিল খন একক -াকেব অG পরকাশ ককের তখন একের বা ধারা বকেল বা ধারার পরকেয়া -াষাকেক আরও সFর ককের যতাকেল

অকাল পকক(অপহিরনত বয়কেস পাকাহিম)-মাতর শ বর বয়কেস যমকেয়টির া মকেখর কা তাকেত অকালপককতা ধরা পকে

অককা পাওয়া( মারা াওয়া) ndash পকেকIমারটি পকেকIমারকেত হিকেয় বাসাতরীকের াকেত মার যখকেত যখকেত অককা যপল

অহি| পরীকষা ( কঠিন ও পরকত পরীকষা)- যকেলটির আজ ডাকতাহির যরজালট যবকেরাকেব এIাই তার জীবকেনর ব অহি| পরীকষা

অষটরমভা (ফাহিক) ndash রীতা মকেখই বকো বকো কা বকেল আর কাকেজর যবলায় অষটরমভা

অকমGার ধাী (অপাG) ndash সমনকেক হিনকেয় যকান ান কেব না ও একেকবাকেরই অকমGার ধাী

অকেনধর ষটি (অসাকেয়র সায়)- আহিশ বকেরর বকোর নাহিত ল অকেনধর ষটি তাকেক াা বকোর একম চকেল না

আকেককল গড়ম (তবহিদধ)- ার তহিম উপকার করকেল যসই যতামার হিবরকেদধ সাকষয হিকেয়কে শকেনই আমার আকেককল গড়ম

আষাকে লপ( অবাসতব লপ) ndashIাকা এখন যকেব না এIা বলকেলই ত এমন আষাকে লপ ফাার যকান রকার হিল না

Hindi- महायजञ ा इस हानी म लख न या बतान ा परयास किया ह कि किसी भी अचछ

2nd language

परसार(यशपाल ाय या पणय न ा फल अवशय मिमलता ह ोई भी परोपार अथवा पणय लिलए किया गया ाय बार नही 0ाता वह ए परार ा यजञ हए धनी सठ थ धम परायण और किवनमर सठ न आन ी यजञ किए थ और दान म न 0ान कितना धन दिदन दखिखयो म बात दिदया थादिदन पलट और सठ यहा गरीबी आ गई उन दिदनो यजञ बचन ी परथा थी सठ भी अपनी 0गह बचन लिलए डलपर ए सट यहा चलन ो तयार हए सठानी रासत लिलए रोटी पड म बाधर सठ ो द दी रासत म ए भख R ो दखर सठ न चारो रोटी उसो खिखला दी खर वह सठ यहा डलपर पहच तो उनी सठानी न उस महायजञ बचन ो हा यदिद बचन आए सठ न R ो रोटी खिखलान ो महायजञ नही समझा और वापस लौट आया घर आर शाम ो उसी घर म उस ए बडा ख0ाना मिमला 0ो उस दवारा किए गएrsquo महायजञrsquo ा परसार था

English language

Letter formal The heading the name and address of the person you are writing to must be included beneath your own address In formal letters ldquoblock stylerdquo of address is preferred

Subject complain in brief

Salutation If the person you are writing to is known to you you may begin ldquoDear MrrdquoOr ldquoDear Mrsrdquo In all other instances you should begin ldquoDear Sirrdquo or ldquoDear Madamrdquo Or ldquoSirsrdquo

The body A formal or business letter has four partsReference The letter should begin by referring to a letter you have received an advertisement or the reason that has prompted you to writeInformation In the second paragraph it is necessary to supply more detailed information that is related to the referencePurpose Here you must give the reason why you are writing the letter This must be stated clearly and ensure that it is relevant to the question that has been setConclusion round off the letter with some polite remarkThe subscription when a letter has begun with dear sir sirs Madam you should end with Yours faithfully or yours truly When however you address a person by name you must conclude with the words ldquoYours sincerelyrdquo

1 A park in your locality is slowly being used as a rubbish dump Write a letter to the Mayor of your city pointing out the nuisance and danger of this Request that action be taken to stop this immediately

Or2 You being a boarder ordered a set of lab manuals from a famous book shop in the town They sent you a wrong set of books Write a letter to the manager of the book shop

Chemistry Chapter-1 1)CHEMICAL FORMULA- Q What is the Significance of

L-2The Language of Chemistrybull Chemical Formula

Itrsquos a symbolic representation of a chemical substance eg ndash The formula of Sulphuric acid is H2SO4

2) Steps of writing Chemical Formula of a given substance-

1 Write the symbols of the constituent atoms or radicals side by side Keep the basic radical on LHS and acid radical on the RHS ( Na+Cl- )2 In case of a radical having more than one atom( compound radical) enclose the radical in a bracket eg (SO4-)3 Write the valencies of each radical on its right hand top4 If the valencies of the two radicals are divisible by a common factor then divide the valencies by the common factor5 Invert (criss-cross) the valency number ie write the valency of one atom below the second atom and vice versa 6 On interchanging if valency number is lsquoone the figure lsquoonersquo is never writtenFor Example- Compound -Calcium Nitrate1 Writing the symbols- Ca(NO3)2 Writing the valencies on their right hand top- Ca2(NO3)1

3 Valency numeral in simple ratio- Ca2(NO3)1

4 Criss-cross- Ca 2NO3 1

5 Writing the formula of the compound- Ca(NO3)2

Chemical formula

A The formula of a substance conveys the following information regarding a substance 1 The name of the substance (qualitative)2 The elements constituting the substance (qualitative)3 The number of various atoms present in a molecule of the substance (quantitative)4 Molecular weight of the substance and the relative weights of different elements present in it (qualitative)

Q What are the limitations of Chemical Formula

A The chemical formula suffers from the following limitations-I It fails to convey whether the elements in a molecule are present in the form of atoms or ionsFor example the formula KBr fails to tell us whether Potassium and Bromine are present in the form of ions II It does not tell anything about the binding force that holds atom in a molecule togetherIII It does not tell us about the arrangement of various atoms with respect to one another within the molecule

Q Examples of Some Chemicals with their Formula Chemical name and Common Name-

A Given in the class notesCommercial Studies

Joint Stock Company

Let us discuss about the demerits of Joint Stock CompanyDespite so many advantages it has got many disadvantages which are as follows

Difficulty in FormationDelay in Decision makingExcessive Government ControlLack of Secrecy

Company can be classified into several categories based on incorporation

QuestionExplain the demerits of Joint Stock CompanyAnswer) 1 Difficulty in Formation The legal requirements and formalities required to be completed are so many The cost involved is quite heavy It has to approach large number of people for its capital It cannot start its business unless certificate of incorporation has been obtained This is granted after a long time when all the formalities are completed

Chartered CompanyStatutory CompanyRegistered Company

Delay in Decision making In this form of organization decisions are not made by single individual All important decisions are taken by the Board of Directors Decision-making process is time-consuming So many opportunities may be costly because of delay in decision-making Promptness of decisions which is a common feature of sole trader ship and partnership is not found in a company

Excessive Government ControlA company and the management have to function well within the law and the provisions of Companies Act are quite elaborate and complex At every step it is necessary to comply with its provisions lest the company and the management should be penalized The penalties are quite heavy and in several cases officers in default can be punished with imprisonment This hampers the proper functioning of the company

Lack of Secrecy The management of companies remains in the hands of many persons Every important thing is discussed in the meetings of Board of Directors Hence secrets of the business cannot be maintained In case of sole proprietorship and partnership forms of organisation such secrecy is possible because a few persons are involved in the management

2 Define the following

Chartered Company- The crown in exercise of the royal prerogative has power to create a corporation by the grant of a charter to persons assenting to be incorporated Such companies or corporations are known as chartered companies Examples of this type of companies are Bank of England (1694) East India Company (1600) The powers and the nature of business of a chartered company are defined by the charter which incorporates it After the country attained independence these types of companies do not exist

in IndiaStatutory Company- A company may be incorporated by means of a special Act of the Parliament or any state legislature Such companies are called statutory companies Instances of statutory companies in India are Reserve Bank of India the Life Insurance Corporation of India the Food Corporation of India etc The provisions of the Companies Act 1956 apply to statutory companies except where the said provisions are inconsistent with the provisions of the Act creating them Statutory companies are mostly invested with compulsory powersRegistered companiesCompanies registered under the Companies Act 1956 or earlier Companies Acts are called registered companies Such companies come into existence when they are registered under the Companies Act and a certificate of incorporation is granted to them by the Registrar

Economics

Chapter-4Basic problems of Economy

Today let us discuss with the topic Production Possibility curve

QuestionExplain the concept of Production Possibility Curve with the help of diagram

Answer) Production Possibility curve is a locus of all possible combinations of two commodities which can be produced in a country with its given resources and technology

The above diagram shows that with the given resources and technology the economy can produce maximum either 5 thousand meters of cloth or 15 thousand quintals of wheat or any other combination of the two goods like B( 1 thousand meters of cloth and 14 thousand quintals of wheat C ( 2 thousands meters of cloth and 12 thousand quintals of wheat) etcProduction Possibility curve is also called production possibility boundary or frontier as it sets the maximum limit of what it is possible to produce with given resources

Geography

Rotationand Revolution

SUNrsquoS POSITION AND SEASONAL CHANGES EQUINOXES ndash SPRING AND AUTUMN

Q1 What is Spring EquinoxA1 On 21st March sunrays fall directly on the equator On that day

As the Equator divides the Earth into two equal halves the sun rays fall directly on the equator twice in a year Equinoxes means equal Spring EquinoxOn 21st March sunrays fall directly on the equator On that day the duration of day and night both are equal ( 12 hours day and 12 hours night) on every places located on equator This day is called as Spring EquinoxAutumn EquinoxOn 23rd September sunrays fall directly on the equator On that day the duration of day and night both are equal ( 12 hours day and 12 hours night) on every places located on equator This day is called as Autumn Equinox

SOLSTICES ndash SUMMER AND WINTERDue to inclination of the Earth on its axis and the apparent movement of the sun the sun rays fall directly on both tropics once in a year Solstice is a Latin word which mean ldquothe Sun standing stillrdquoSummer SolsticesAfter 21st March there is an apparent movement of the Sun to the north of the equator The apparent northward movement up to 21st June when the Sun appears overhead at the Tropic of Cancer (22frac12degN) The sun appears to stand still at this position and then moves southwards towards the equator This position of the Sun on 21st June is known as Summer Solstices On that day the duration of day and night both are equal ( 12 hours day and 12 hours night) on every places located on Tropic of Cancer (22frac12degN)Winter solstices The apparent southward movement of the Sun continues beyond the equator till 22nd

December On this day the Sun is overhead at the Tropic of Capricorn

the duration of day and night both are equal ( 12 hours day and 12 hours night) on every places located on equator This day is called as Spring Equinox

Q2 What do you mean by EquinoxA2 Equinoxes means equal It is use to explain the equal duration of day and night ( 12 hours day and 12 hours night) on the Earth

Q3 On which date the longest day in Tropic of CancerA3 21st June

Q4 What is the meaning of SolsticeA4 Solstice is a Latin word which mean ldquothe Sun standing stillrdquo

Q5 Which is the longest day in southern hemisphereA5 22nd December

Q6 On what date does the Arctic Circle experience the lsquoMidnight SunrsquoA6 On 21 June the Arctic Circle experiences the lsquoMidnight Sunrsquo

Q7 What is cause of Midnight Sun in NorwayA7 During the summer solstice (21 June) the North Pole is inclined towards the Sun Therefore the duration of sunlight or daytime increases from 12 hours at the Equator to 24 hours at the Arctic Circle and beyond Thatrsquos why The region beyond the Arctic Circle especially Norway is known as the Land of the Midnight Sun because there the Sun does not rise or set on 21 June

Q8 Match the column A with BA B

Summer Solstice 21st March

Autumn Equinox 23rd

September

Winter Solstice 21st June

(22frac12degS) This position of the Sun is referred to as the Winter Solstice because it marks the winter season in the Northern Hemisphere On that day the duration of day and night both are equal ( 12 hours day and 12 hours night) on every places located on Tropic of Capricorn (22frac12degS)SEASONS AND DURATION OF DAY AND NIGHT During the equinoxes all places on the Earth have 12 hours of day and 12 hours of night Due to the revolution of the Earth round the Sun on an inclined axis the duration of day and night varies according to seasons and the latitude of a placeDuring the summer solstice (21 June) the North Pole is inclined towards the Sun Therefore the duration of sunlight or daytime increases from 12 hours at the Equator to 24 hours at the Arctic Circle and beyondThe region beyond the Arctic Circle especially Norway is known as the Land of the Midnight Sun because there the Sun does not rise or set on 21 JuneAt the North Pole there will be six months of daylight The Sun will be seen always above the horizon at a low angle At 66degN 24 hours of sunlight can be seen only on 21 June Hammerfest in northern Norway is a place of tourist attraction for observing the phenomenon of the Midnight Sun This place has continuous daylight from 13 May to 29 July This place is easily accessible to tourists and has hotels and other facilities The view of the midnight Sun from here is enthrallingIn the Southern Hemisphere the duration of daylight decreases from 12 hours at the equator to 0 hours beyond the Antarctic Circle In the South Polar Region there is 24 hours of darkness The Sun is always below the horizon In the Southern Hemisphere which experiences winter the duration of night-time is longer than the duration of daylight

Spring Equinox 22nd

December

A8 A B

Summer Solstice 21st June

Autumn Equinox 23rd

September

Winter Solstice 22nd

December

Spring Equinox 21st March

During winter solstice (22 December) the South Pole is inclined towards the Sun The Southern Hemisphere experiences summer and the Northern Hemisphere has winter Therefore the duration of daylight or sunlight is greater in the Southern Hemisphere than in the Northern HemisphereThe duration of daylight increases from 12 hours at the equator to 24 hours beyond the Antarctic Circle The South Polar Region has 24 hours of sunlight for many days continuously At the South Pole there will be six months of sunlight The Sun will always be seen at a low angle above the horizon In the Northern Hemisphere the duration of daylight will decrease from 12 hours at the equator to 0 hours at the Arctic Circle There are 24 hours of darkness in the North Polar region The duration of night is greater than the duration of daylight as one move northwards from the Equator It is evident from the above table that the duration of daylight is 12 hours throughout the year at the equator only As one moves away from the equator the seasonal variations in the duration of daylight increase The seasonal variations in the duration of daylight are maximum at the Polar Regions

Subject Eng Literature (The Merchant of Venice ndash William Shakespeare)Topic Act II Scene 7 Lines 36 to 80 (End of scene ) [Students should read the original play and also the paraphrase provided]

Summary Questions amp AnswersThe Prince then examines the inscription on the silver casket which says ldquoWho chooseth me shall get as much as he deservesrdquo The Prince says that he deserves Portia more than anybody else because of his high rank his noble birth and his great wealth and power But then he argues that silver is ten times

(1) (Act II Sc 7 L 39-47)

From the four corners of the earth they come

To kiss this shrine this mortal breathing saint

The Hyrcanian deserts and the vasty wildsOf wide Arabia are as through-fares now

inferior to gold and therefore he cannot believe that the portrait of such a beautiful lady as Portia can be contained in the silver casket He decides to see the inscription on the golden casket before making his decision

The Prince goes to examine the inscription on the golden casket which says ldquoWho chooseth me shall get what many men desirerdquo The Prince believes that the whole world desires to possess Portia otherwise so many suitors would not have come from all corners of the world for winning Portia Some of them have come from the distant lands of Persia and Arabia The deserts of Persia (Hyrcanian deserts) and the boundless desolate lands of Arabia have been crossed by the Princes seeking the hand of Portia He contrasts this casket containing Portiarsquos portrait with the old English gold coin bearing the image of the archangel (angel of the highest rank) He goes on to remark that while the figure of the archangel is engraved (Insculped) upon the English coin the picture of Portia who is beautiful as an angel lies hidden inside one of the caskets namely the Golden Casket (Golden Bed)

On the basis of his assessment of the inscription on the golden casket the Prince decides to choose the golden casket He asks for the key and opens the golden casket only to find therein an empty human skull holding a roll of

For princes to come view fair PortiaThe watery kingdom whose ambitious headSpets in the face of heaven is no barTo stop the foreign spirits but they comeAs orsquoer a brook to see fair Portia

(i) Explain the occasion for the above mentioned speech

These are the comments of the Prince of Morocco after he reads the inscription on the golden casket His mental process is revealed to us in these words We find him debating within himself as to which casket he should choose

(ii) What light does the above speech throw on the personality of Prince of Morocco

From the above mentioned speech we come to know that the Prince of Morocco is keen to marry Portia He is the type of person who is easily taken away by outward appearance He is in love with Portia because of her beauty

(iii) What information can you gather about Portia from the above mentioned lines

The given speech shows that Portia is a very beautiful lady She must be possessed of good qualities because many suitors come to her place from all over the world with a desire to get married to her The Prince of Morocco is so impressed by her beauty that he calls her a saint According to him the whole world is desirous of having her

(iv) Elucidate the significance of the first two lines

In these lines the Prince of Morocco pays a compliment to Portia These lines show his admiration for her He says that people come from all parts of the world to see fair Portia

(v) Explain the meaning of the last four lines of the

passage

In these lines the Prince of Morocco says that even the vast oceans which throw a challenge at the sky are unable to prevent men from coming to Portiarsquos place to have a glimpse of her These lines are also a tribute to Portiarsquos beauty and good qualities Many men voyage across the ocean treating it as a mere stream to see the beautiful Portia

paper in which is written that whoever happens to be guided by the glitter of things is invariably deceived

On reading the scroll the Prince says that he is too sad at heart to speak a more formal farewell and leaves with his followers amidst a sound of trumpets

After the Prince of Morocco leaves Portia remarks that the Prince is a gentle fellow but she is rid of him May all persons of his nature make a similar choice

IMPORTANT PASSAGES EXPLAINED

(Act II Sc 7 L 39-43)From the four corners of the earth they come

To kiss this shrine this mortal breathing saintThe Hyrcanian deserts and the vasty wildsOf wide Arabia are as through-fares nowFor princes to come view fair Portia

Context

This passage occurs in Act II Scene 7 in The Merchant of Venice This is part of the speech made by the Prince of Morocco

(2)

(Act II Sc 7 L 48-53)

MOROCCO One of these three contains her heavenly pictureIst like that lead contains her

Twere damnation To think so base a thought it were too grossTo rib her cerecloth in the obscure graveOr shall I think in silver shes immurdBeing ten times undervalued to tried gold

(i) What meaning does the Prince of Morocco find out of the inscription of the golden casket What have Belmont and Portiarsquos house been called and why

The inscription on the golden casket is ldquoWho chooseth me shall gain what many men desirerdquo The Prince finds out that it means that the chooser of the golden casket will get Portia because many men desire her In fact the entire world desires her Because of the coming of many suitors to Belmont from different countries in order to win Portiarsquos hand Belmont has become a centre of pilgrimage and her house is the shrine where saintly Portia is installed

(ii) What does the Prince of Morocco do before making the final choice of the casket Which is the correct casket and who will win Portiarsquos hand

The Prince of Morocco surveys and analyses the inscriptions on the casket of lead silver and gold Before making the final choice like a very systematic and methodical person he once again considers the claims of the caskets The casket containing Portiarsquos picture is the correct casket and the person choosing it will win Portiarsquos hand

Explanation

While praising Portia the Prince of Morocco conceives Portia as a goddess whose image is placed inside one of the caskets Many suitors are coming from far and wide the north and the south the east and the west (Four corners) in order to try their luck Some of them have come from the distant land of Persia and Arabia The deserts of Persia (Hyrcanian deserts) and the boundless desolate lands of Arabia have been crossed by the Princes seeking the hand of Portia All this shows that Portia is indeed the most beautiful lady of the world

(iii) What does the Prince of Morocco say in his estimation while examining the motto on the silver casket What does he find in the golden casket

While examining the motto on the silver casket which says ldquoWho chooseth me shall get as much as he deservesrdquo Morocco says that in his own estimation he surely deserves Portia in all respects ndash rank birth wealth etc

He chooses the golden casket When he opens it he finds an empty human skull holding a scroll in which it is written that those who are attracted by the glittering outside of things are always deceived as Morocco has been deceived

(iv) What kind of nature does the Prince of Morocco have

The Prince of Morocco has a simple nature who does not look deeply into the inner meaning of things but is dazzled by the outward appearance of gold He is inclined to over-estimate his own value and does not realize that it is a duty to ldquogive and hazardrdquo To say that he will not hazard for lead shows that he misreads the true meaning of the inscription which is that he should be prepared to ldquohazard all he hathrdquo for Portia So his feeling is only one of fascination and romantic attraction

(v) Do you think that the lottery of the caskets is not a matter that will be determined by chance

In fact the lottery of the casket is not a matter that will be determined by mere chance but that it is a true test of character and of sincerity which is amply proved not only by Moroccorsquos choice but also by the arguments which he uses to help him in his choice

(Act II Sc 7 L 55-59)

They have in England

A coin that bears the figure of an angelStamped in gold but thats insculpd uponBut here an angel in a golden bedLies all within

Context

(3)

(Act II Sc 7 L 63-77)A carrion Death within whose empty eye

There is a written scroll Ill read the writing

All that glisters is not goldOften have you heard that toldMany a man his life hath soldBut my outside to beholdGilded tombs do worms infoldHad you been as wise as boldYoung in limbs in judgment oldYour answer had not been inscrolld

This passage occurs in Act II Scene 7 in The Merchant of Venice This is part of the speech made by the Prince of Morocco

Explanation

In this passage the Prince of Morocco bestows high praise on Portia whose hand he is seeking He contrasts this casket containing Portiarsquos portrait with the old English gold coin bearing the image of the archangel (angel of the highest rank) He goes on to remark that while the figure of the archangel is engraved (Insculped) upon the English coin the picture of Portia who is beautiful as an angel lies hidden inside one of the caskets namely the Golden Casket (Golden Bed) In the day of Elizabeth silver was ten times inferior in value to gold Therefore the Prince of Morocco believing that Portiarsquos portrait is contained in the Golden Casket decides to choose the Golden Casket

Fare you well your suit is coldCold indeed and labour lostThen farewell heat and welcome frostmdashPortia adieu I have too grievd a heartTo take a tedious leave Thus losers part

(i) What reward does the Prince of Morocco get after making a wrong choice of the Casket How does he feel

After making the wrong choice in selecting the casket of gold the Prince of Morocco as a reward earns a rebuke in the form of a scroll tucked in the empty eye-socket of a skull kept in the casket of gold The Prince is shocked and disappointed He becomes all the more sad and dejected when he reads the scroll which points to his foolishness in being misled by the appearance and outward show as indicative of its worth

(ii) How does the Prince respond after reading the scroll

After reading the scroll the Prince though upset accepts the result with good grace and decorum befitting a royal suitor and true sportsman He says that his love-suit is really cold otherwise he would have chosen correctly but now his efforts have been in vain So he bids farewell to Portia to the warmth and enthusiasm of love and welcomes the cold and bitterness of dejection and misery of life which lies ahead

(iii) What request does he make to Portia and why

After being failure in his mission he requests Portia to give him permission to leave at once because he is too sad to undergo the tediousness of a formal leave-taking He tells that it is the manner in which defeated persons part unceremoniously

(iv) Explain the following lines

ldquoAll that glisters is not goldOften have you heard that toldMany a man his life hath soldBut my outside to beholdGilded tombs do worms infoldrdquo

Mere glitter does not make a metal to be gold Man has often been warned against appearance but it has been of no use Many people have sacrificed their lives only to seek the outer appearance of gold Worms are found inside the gilded

monuments

Class XSubject Topic Summary Execution

Hindi 2ndlang

नया रासता भाग 6 मायाराम 0ी घर म धनी मल 0ी और उनी बटी सरिरता ी ही चचा बनी रहती थी अमिमत ो इसम ोई रलिच ना थी वह धनी घर ी लडी स शादी र सवय ो बचना नही चाहता था उसा भी सवाणिभमान ह ईशवर ी पा

स उस पास पस ी ोई मी नही थी अभी उसन फकटरी ही लगाई थी उसी समझ बाहर था कि उस घर वालो ा झाव पस ी तरफ कयो

ह उसन मा स सवाल किया कि मा तम सरिरता स मरी शादी कयो रना चाहती हो मा न उस समझाया कि वह दखन म बरी नही ह और किफर खानदान अचछा

ह वह ए शल गरहणी रप म घर सभाल सगी अमिमत न मा ो इस बात ा एहसास राया कि मीन सबध लिलए मना रन पर उस दिदल

पर कया बीती होगी मा और अमिमत ी लडी बार म ाफी बात हईमा ा झाव सरिरता ी तरफ था कयोकि वह घर पर अचछा दह0 लर आ रही

थी अमिमत न अपनी मौसी ी बरी हालत बार म बताया कि किस तरह वह बड घर ी खानदानी बटी लाई थी और आ0 उसी हालत कितनी खराब ह लाई थी बहकलब 0ाती ह और बचचो ो भी नही दखती ह बात चल ही रही

थी कि तभी ए ार बाहर आर री धनी मल0ी घर अदर आए और पीछ स डराइवर फल ी ए टोरी लर आया अदर आए और पीछ स

डराइवर ए टोरी फल ी लर आया अमिमत ो फल ी पटी बरी लग रही थी अमिमत न पछ लिलया यह फल कयो ल आए ह प इन सब ी कया

0ररत थी उनो न 0वाब दिदया कि 4 पटी शमीर स मगाए थ अमिमत ो या सनर करोध आ गया तभी उस किपता 0ी आ गए उन आत ही अमिमत उठर बाहर चला गया वहा वहा मा पास आर बठ गया और बोला

अभी रिरशता तय नही हआ और धनी मल 0ी धनी मल 0ी फल ी पटी लर चलआय मा न समझाया कि 0ब सबध 0ड 0ाता ह तो खाली हाथ नही

आत अमिमत न मा स हा कि तम सबन सरिरता ो इस घर म लान ी ठान रखी ह धनीमल 0ी उस दिदन सरिरता ो दखन ी तारीख तय रन आय थ

Commercial Studies

Banking Nowadays Bank provide easy and quick services through internet facilities methods of Banking is called internet bankingIn order to save the time and money involved in visiting Bank branches people increasingly prefer to have internet banking

There are different modes of doing internet banking or transferring money through online They areReal Time Gross Settlement (RTGS)National Electronic Fund Transfers (NEFT)

1

Question

1) Explain the term RTGS Write the features of RTGS

Answer)The acronym RTGS stands for Real Time Gross Settlement which may be defined as the continuous real time settlement of funds transfer individually on and order by order basis without netting lsquoReal timersquo may be defined as the processing of instructions at the time they are received rather than at some letter time lsquoGross settlementrsquo may be defined as the settlement of transfer instructions which occurs

individually

Features of RTGS1It is the continuous settlement of

funds transfer individually on an order by order basis

2RTGS facility is provided only by CBS core banking solution enabled Bank branches

3Amount charged from the customer for RTGS transactions vary from bank to bank

2) Explain the term NEFT Write the features of NEFT

Answer) National electronic funds transfer may be defined as a nationwide system that facilitates individuals Farms and copper operates to electronically transfer funds from any bank branch to any individual farm or corporate having an account with any other bank branch in the country

Features of NEFT2 Transfer can be made 7 times on

weekdays and 6 times on Saturday

3 NEFT cannot be used to receive foreign remittances

4 NEFT transaction takes place in batches

5 A bank branch must be NEFT enabled to become a part of NEFT fund transfer network

6 There is no maximum or minimum amount that can be transferred through NEFT when one bank has a bank account

English Language

CompositionEssay

A composition is an art of creating a piece of writing on any topic or subject It is the writing correctly beautifully and clearly in order to make some interesting reading Structure of the composition

Introduction ( you lay the foundation for your composition)

Body (it constitutes the main part of the essay)

Conclusion (final statement that leaves a lasting impression)

Kinds of essays1 The Narrative essay2 The descriptive essay3 The reflective essay4 The argumentative essay

Write a composition on any one of the following topics (350- 400 words)

1 Friendship Or2 The first day of your school

Subject Eng Literature (The Merchant of Venice ndash William Shakespeare)Topic Act V Scene 1 Lines 127 to 158 (Nerissa helliphellip The clerk will nersquoer wear hair onrsquos face that had it) [Students should read the original play and also the paraphrase given in the school prescribed textbook]

Summary Revision Questions o Soon thereafter Bassanio Gratiano

and Antonio arrive

o Bassanio tells Portia that he is feeling as if it is morning because of the presence of Portia who is shining like the sun When Antonio is introduced by Bassanio to Portia she tells Bassanio that he should be grateful to Antonio who took so much trouble on his account even to the extent of risking his life

o Nerissa starts quarrelling with Gratiano and demands that he show her the ring she had presented to him and which she had warned him not to lose She suspects that Gratiano must have presented the ring to some young woman and not to the lawyerrsquos clerk as he repeatedly says and assures

Answer the following questions to check your preparation of Act IV Scenes 1 and 2

You must attempt only after you have completed your preparation of Act IV The answers must be in complete sentences using textual evidence (with citation) when necessary

[It would be in your own interest to attempt the above questions honestly totally refraining from consulting your textbook or your notes during answering After completion you should correct the paper yourself consulting the textbooknotes etc and award marks as specified Please let me know the marks you scored through WhatsApp in the group or to my personal WhatsApp]

Act IV Scene 1 (each question carries 2 marks)

1 What did the Duke try to do for Antonio

2 Why does Shylock refuse to show mercy How does he justify his stance

3 Why does Antonio say he is ready to die 4 What information is contained in Bellariorsquos letter

5 Why does Portia (as Balthazar) assert that Shylock must show mercy How does he respond

6 What offers are made to Shylock to get him to spare Antonio How are they received

7 What does Antoniorsquos speech as he faces the prospect of Shylockrsquos knife tell you about his character

8 How do Bassanio and Gratiano react to the looming prospect of Antoniorsquos demise

9 How does Portia (as Balthazar) use the law to turn the tables on Shylock

10 What does the Duke decree should happen to Shylock Why What happens to Shylockrsquos estate

11 What does Portia ask Bassanio as payment for her ldquoservicesrdquo What is his initial response What makes him change his mind

Act IV Scene 2 (each question carries 1frac12 marks)

1 What does Gratiano bring to Portia (Balthazar)

2 What does Nerissa plan on getting from Gratiano What does Portiarsquos comment suggest about men

ECO-10 280620 Topic-Supply AnalysisSHIFTING OF SUPPLY

But if there is change in factors other than the price of the commodity then either more is supplied at the same price or less supplied at the same price In such cases the price of the commodity remains constant but there is a change in other factors like change in the price of inputs change in technology of production change in price of other related goods change in taxation policy of the government etc For example there is an improvement in the technology of production of the commodity in question It leads to decrease in per unit of cost production of the commodity The firm is willing to sell more quantity of the commodity at the same price So the supply other commodity increases at the same price This increase in supply is shown by rightward shift of supply curve On the other hand if the firm uses inferior technology of production the cost of production per unit of the commodity increases The firm is willing to sell less quantity at the same price So the supply of the commodity decreases at the same price This decrease in supply is shown by leftward shift of the supply curve The above cases of increase and decrease in supply can be shown with the help of the following figures

Y INCREASE OF SUPPLY Price (Rs) s

P A s1

B

s

X` O s1 X

q q1

Y` Quantity demanded (in units)

Y DECREASE IN SUPPLY s2

s

price (Rs)

C

p A

s2

s

X` o X

q2 q

Y` Quantity demanded ( in units)

Main factors causing increase in supply or rightward shift of supply Curve(i) Fall in the price of other related goods

(ii) Fall in the price of inputsfactors(iii) Use of better technology in production(iv) Decrease in the rate of excise duty by government(v) If the objective of producer changes from profit maximization to salesMaximization

Main factors causing decrease in supply or leftward shift of supply curve(i) Increase in the price of other related goods(ii) Rise in the price of inputsfactors(iii) Use of inferior technology in production(iv) Increase in the rate of excise duty by the government(v) If the objective

Subject - Biology Topic ndash Chapter mdash6 PhotosynthesisSummary Execution

Today we will know about photosynthesis and its stages

Q1 What do you mean by photosynthesis The process by which living plants containing chlorophyll produce food

substances from carbon-di- oxide and water by using light energy Sunlight

6CO2 +12 H2O----------------------- C6 H12O6 + 6H2O + 6O2

Chlorophyll

Q2 What are the importance of photosynthesis I) Food for all Green plants trap solar energy by photosynthesis

process and supply food and energy for all living organisms either directly or indirectly

Ii) Oxygen to breathe in by product of photosynthesis is oxygen which is essential for all living organisms respiration

Q3 Write about two main phases of photosynthesis A Light dependent phase This phase occur in grana of chloroplast I) The chlorophyll on exposure to light energy becomes activated by

absorbing photons Ii) The absorbed energy is used in splitting the water molecules (H2O)

into its two components (H+ and OH- ) and releasing electron s 2H2O------------------------- 4H+ + 4e- +O2

Energy of 4 photons This reaction is known as photolysis

End products are H+ and oxygen water

B Light independent (Dark ) phase The reactions in this phase require no light energy

Here CO2 combine with H+ and produce glucose

Class XI

Subject Topic Summary ExecutionEVS Chapter-4 Legal

regimes for sustainable development

Environmental legislationEnvironmental legislation is the collection of laws and regulations pertaining to air quality water quality the wilderness endangered wildlife and other environmental factors The act ensures that matters important to the environment are thoroughly

Learn -The Forest (Conservation) Act 1980

considered in any decisions made by federal agencies

The Forest (Conservation) Act 1980 The Forest (Conservation) Act 1980 an Act of the Parliament of India to provide for the conservation of forests and for matters connected therewith or ancillary or incidental thereto It was further amended in 1988 This law extends to the whole of IndiaObjects and Reasons of the Forest Conservation Act

Deforestation causes ecological imbalance and leads to environmental deterioration Deforestation had been taking place on a large scale in the country and it had caused widespread concern The act seeks to check upon deforestation and de-reservation of forests

Subject Eng Literature (The Tempest ndash William Shakespeare) Topic Act II Scene 1 Lines 314 to 329 (End of scene)

[Students should read the original play and also the paraphrase given in the school prescribed textbook]Summary Questions amp Answers

Conspiracy of Antonio and Sebastian (Contd)

o As they approach Ariel appears again and wakes up Gonzalo by singing a tune in his ear Alonso also wakes up and they see both Sebastian and Antonio with drawn swords On being caught off guard they make up a story saying that they had heard a bellowing of bulls or lions

o They then moved to another part of the island

o Ariel at once rushes to Prospero to inform him of this development

SUMMING-UP of ACT-2 SCENE-1

(i) Among the survivors Ferdinand is separated from the rest which results in the disconsolate grief of Alonso as he took him for dead

(ii) The villainy of Antonio is confirmed

(iii) The supremacy of Prosperorsquos magic which resulted in the failure of the human conspiracy

(1)

(Act II Sc 1 L 311-325)SEBASTIAN Whiles we stood here securing your repose

Even now we heard a hollow burst of bellowing Like bulls or rather lions Didt not wake youIt struck mine ear most terribly

ALONSO I heard nothingANTONIO O rsquotwas a din to fright a monsters ear

To make an earthquake Sure it was the roarOf a whole herd of lions

ALONSO Heard you this GonzaloGONZALO Upon mine honour sir I heard a humming

And that a strange one too which did awake meI shaked you sir and cried As mine eyes opened I saw their weapons drawn There was a noiseThats verily rsquoTis best we stand upon our guardOr that we quit this place Lets draw our weapons

(i) Why has Prospero sent Ariel to Gonzalo and Alonso What does Ariel do to awaken Gonzalo

Prospero has already come to know by his magic powers the danger which threatens Gonzalo who had been Prosperorsquos friend and so he sent Ariel to preserve the lives of both Gonzalo and Alonso Prospero does not want that his scheme should remain unfulfilled Ariel begins to sing a song in Gonzalorsquos ears to awaken him(ii) Who are ready to carry out their plan Who takes steps to stop them Why does Gonzalo feel surprised after being awakened

Sebastian and Antonio are ready to carry out their plans They are standing with their swords drawn to kill Alonso and

(iv) We see two sets of contrasting characters Gonzalo-Adrian against Antonio-Sebastian

(v) The grief that works in Alonso can be perceived to his repentance for his association in Antoniorsquos crime against Prospero

Gonzalo Ariel takes steps to stop them from carrying out their nefarious scheme When Gonzalo is awakened by the song sung by Ariel into his ears he (Gonzalo) feels surprised because he sees Sebastian and Antonio standing with their swords drawn(iii) What reason do Sebastian and Antonio tell of drawing their swords when they are suspected by Alonso and Gonzalo

When Sebastian and Antonio are seen with their swords drawn they are looked with suspicion by Gonzalo and Alonso At first Sebastian tells them that as they stood here to guard them during their sleep they heard only a little before a sudden loud noise very much like the roaring of bulls or more probably that of lions Then Antonio follows him saying that this was a noise so terrible as to frighten even a monsterrsquos ears and this noise could even have shaken the earth and it was surely like the roaring of a multitude of lions Then seeing the danger they have drawn their swords Perhaps after hearing the terrible noise they (Gonzalo and Alonso) woke up from their sound sleep

(iv) What does Gonzalo tell Alonso about the strange noise What did he see on opening his eyes Gonzalo tells Alonso that he did not hear the sound of roaring but he heard a humming sound which was strange and which woke him up After waking up he gave him (Alonso) a shaking and a loud cry On opening his eyes he saw these two gentlemen standing with their swords drawn(v) What does Gonzalo suggest

Gonzalo suggests that there was a noise indeed and of that he has no doubt at all and suggests that the best course for them would be to remain alert and vigilant against any possible danger to their lives or to leave this place and move to some other part of the island

Class XIISubject Topic Summary Execution

Commerce

Chapter- Management

Today we will discuss about LEVELS OF MANAGEMENT

Levels of management is a series or chain of managerial positions from top to bottom It helps individuals to know their authority responsibilities and superior-subordinate relations among themselves There are mainly three levels of Management TOP LEVEL MANAGEMENTMIDDLE LEVEL MANAGEMENTLOWER LEVEL MANAGEMENT

Top level managementIt consists of members at the highest level in the management hierarchy This level includes Board Of Directors Chief Executive Managing Directors Chairman President Vice President

Rolefunctions of the top levelmanagement1To analyse evaluate and deal

with theexternal environment2 To determine the objectives and

policies of the business3 To strive for welfare and survival

of business

4 To create an organisational Framework consisting of authority responsibility relationship

Middle level management Congress of members or groups who are concerned with implementation of the policies let down by the top managementThis level includes head of the department such as finance manager marketing manager branch and regional managers departmental and divisional heads plant superintendent etc

Role of functions of the middle level management

1 To interpret the policies framed by top management

2 To assign duties and responsibilities to lower level managers

3 To select and appoint employees for middle and supervisory level and evaluate their performance

4 To co-operate with other departments for smooth functioning

Operational or supervisory level managementIt refers to the group are members who are concerned with execution of the work They are also known as fast line managers This level includes supervisor 4 men Section Officer clerk Inspector etc

Role of functions of the lower level management1 To plan and execute day-to-

day operations2 To supervise and control the workers3 To arrange materials and

tools to start the process and make arrangements for training

4 Today present workers grievance and suggestions before the management and

ensure safe and proper working conditions in the factory

Business Studies

Staff Appraisal Chapter- 10 Today let us start with a new chapter

Staff Appraisal

Meaning of Performance Appraisal

Performance Appraisal is the systematic evaluation of the performance of employees and to understand the abilities of a person for further growth and developmentThe supervisors measure the pay of employees and compare it with targets and plansThe supervisor analyses the factors behind work performances of employeesThe employers are in position to guide the employees for a better performance

Objectives of Performance Appraisal

Following are the objectives of Performance Appraisal

To maintain records in order to determine compensation packages wage structure salaries raises etc

To identify the strengths and weaknesses of employees to place right men on right job

To maintain and assess the potential present in a person for further growth and development

To provide a feedback to employees regarding their performance and related status

To provide a feedback to employees regarding their performance and related status

Importance of Performance Appraisal

Performance appraisal provides important and useful information for the assessment of employees skill

knowledge ability and overall job performance The following are the points which indicate the importance of performance appraisal in an organization

1 Performance appraisal helps supervisors to assess the work performance of their subordinates

2 Performance appraisal helps to assess the training and development needs of employees

3 Performance appraisal provides grounds for employees to correct their mistakes and it also provides proper guidance and criticism for employees development4 Performance appraisal provides reward for better performance

5 Performance appraisal helps to improve the communication system of the organization

6 Performance appraisal evaluates whether human resource programs being implemented in the organization have been effective

7 Performance appraisal helps to prepare pay structure for each employee working in the organization

8 Performance appraisal helps to review the potentiality of employees so that their future capability is anticipated

Geography

DRIANAGE The SubarnarekhaThe Subarnarekha and the Brahmaniinterposed between the Ganga and the Mahanadi deltas drain an area of 19300 sq kmand 39033 sq km respectively The drainage basins of these streams are shared byJharkhand Odisha west Bengal and Chhattisgarh The Brahmani is known as southKoel in its upper reaches in Jharkhand

The NarmadaThe Narmada rises in the Amarkantak hills of MadhyaPradesh It flows towards the West in a rift valleyformed due to a geological fault The total length of it is 1300 km All the tributaries of the

Q1 Name the two westward flowing rivers in the peninsular plateauA1 Narmada and Tapi are the only westward flowing rivers of the peninsular plateau

Q2 Differentiate between east-flowing rivers and west-flowing riversA2

East-flowing rivers

West-flowing rivers

Narmada are very short inlength Most of its tributaries join the main streamright anglesThe Narmada basin covers parts of Madhya Pradesh and Gujarat

The Tapi The Tapi rises in the Satpura ranges in the Betul listrictof Madhya Pradesh It flows in a rift valley parallel tothe Narmada but it is much shorter in length It coversparts of Madhya Pradesh Gujarat and MaharashtraThe length is about 724 km

The Sabarmati and the MahiThe Sabarmati rises in the Aravali hills and flows south-south-westwards for a distance of 300 kilometres to the Arabian Sea The Sabarmatibasin extends over an area of 21674 sq km in Rajasthan and Gujarat The Mahi rises inthe east of Udaipur and drains an area of 34842 sq km lying in Madhya PradeshRajasthan and Gujarat It flows south-westwards for a distance of 533 km before it fallsinto the Gulf of Khambhat

The ChambalThe Chambal rises near Mhow in the Vindhya Range and flows towards the northgenerally in a gorge upto Kota Below Kota it turns to the north-east direction and afterreaching Pinahat it turns to the east and runs nearly parallel to the Yamuna beforejoining it in the southern part of the Etawah district in Uttar PradeshMajor Rivers of India with their basin area (Sqkm)

Himalayan System Indus 321290Ganga 861404

Brahmaputra 187110Indus System

Jhelum 34775Beas 20303

Ganga System Yamuna 366223Ghaghra 127950

Peninsular RiversNarmada 98796

Tapi 65145Mahanadi 141600

Subarnarekha 19300Sabarmati 21674

Mahi 34842Godavari 312812

Godavari Krishna Kaveri Mahanadi are the east-flowing rivers

Narmada Tapi west-flowing rivers

They fall into the Bay of Bengal

They fall into Arabian Sea

These rivers form big deltas

These rivers form comparativelysmall deltas

Catchment areas of these rivers are larger

Catchment areas of these rivers are smaller

Krishna 2589488Cauveri 87900

Subject ndashBiology Topic ndashChapter -5 Inheritance amp Variations Summary ExecutionToday we will discussabout linkage and its classification

LINKAGE The tendency of the genes located on the same chromosome to stay together is

hereditary transmission Linked genes the genes responsible for this Genes that exhibit the process of linkage locates in the same chromosome The distance between the linked genes in a chromosome determines the strength

of linkage i e genes that are located close to each other show stronger linkage than that are located far from each other

COMPLETE LINKAGE It is the type of linkage showed by the genes that are closely located or are tightly

linked with each other as they have no chance of separatingby crossing over These genes are always transmitted together to the same gamete and the same

offspring In such condition only parental or non cross over type of gametes are formedINCOMPLETE KINKAGE It is type of linkage showed by the genes that are distantly located orare loosely

linked with each other because they have chance of separating by crossing over

SIGNIFICANCE i) It helps in holding the parental character togetherii) It checks the appearance of new recombination and helps in bringing the

hybrid population which resembles the original parents iii) Linked genes dilute the effects of undesirable traits

Subject Eng Literature (The Tempest ndash William Shakespeare) Topic Essay Questions (EQ-3)Question No 3

Give a character sketch of CalibanAnswer

The character of Caliban has been wonderfully conceived by Shakespeare as the manifestation of all that is gross and earthy ndash a sort of creature of the earth as Ariel is a sort of creature of the air

Calibanrsquos Physical Appearanceo Caliban is lsquofreckledrsquo a lsquomisshapen knaversquo not honoured with human shape

o Prospero calls him lsquothou tortoisersquo (Act I Sc 2 Line 317) Trinculo stumbling upon him describes him as ldquoA strange fish hellip Legged like a man And his fins like armsrdquo He ldquosmells like a fishrdquo (Act II Sc 2 Line 25)

o Prospero also calls him a ldquobeastrdquo (Act IV Sc 1 Line 140) and ldquoThis misshapen knaverdquo (Act V Sc 1 Line 268)

o Further it appears that in addition to his physical deformity his spiritual inferiority is also suggested by Prosperorsquos claim that his birth resulted from the union between his mother the witch Sycorax and the devil

Calibanrsquos ParentageWhen the play opens Caliban is twenty four years of age having been born on the island twelve years before the coming of Prospero His mother was the foul witch Sycorax who was banished from Algiers for ldquomischiefs manifold and sorceries terrible to enter human hearingrdquo (Act I Sc 2 Line 264) and the father was the Devil himself Thus

Caliban is a monster of evil and brute nature ugly deformed and stinking

Calibanrsquos Savage and Malignant Natureo Caliban is entirely a creature of the earth ndash gross brutal and savage He regards himself as the rightful possessor

of the island and Prospero as a usurper

o In his young age he was on good terms with Prospero He had consented to be received by Prospero at his house and to be educated by him He has learnt human language only to curse his master whom he abhors

o His beastly nature soon breaks out and ends in a vicious attack on Miranda This opens the eye of Prospero who becomes severe to him and enforces his service by threats and violence

o Prospero uses him to make dams for fish to fetch firewood scraper trenches wash dishes and keep his cell clean

Calibanrsquos Hatred for ProsperoA profound hatred for Prospero has taken hold of Caliban It springs from a sense of his being dispossessed and ill-treated He would kill Prospero if he could but he knows the power of Prosperorsquos lsquobookrsquo Hence he transfers his allegiance to Stephano who seems like a god to him He also incites the two drunken associates to batter the skull of Prospero when he sleeps in the afternoon

Caliban Shows Considerable Intelligenceo He has learnt Prosperorsquos language

ldquoYou taught me language and my profit onrsquot (Act II Sc 2 Lines 86-89)Is I know how to curserdquo

o He is well aware of the futility of arguing with one who has more power than he has

ldquoI must obey his art is such power (Act I Sc 2 Lines 373-376)It would control my damrsquos god SetebosAnd make a vassal of himrdquo

o He realizes the importance of Prosperorsquos books

ldquoRemember (Act III Sc 2 Lines 89-92)First to possess his books for without themHersquos but a sot as I am nor hath notOne spirit to commandrdquo

o He knows the value of stealth when attacking the enemy

ldquoPray you tread softly that the blind mole may not (Act IV Sc 1 Lines 194-195)Hear a foot fall we now are near his cellrdquo

o Caliban has a better set of values than Stephano and Trinculo They are distracted from their plan by their greed for Prosperorsquos rich garments Only Caliban realizes that such a finery is unimportant

ldquoLeave it alone thou fool it is but trashrdquo (Act IV Sc 1 Lines 224)

Caliban is not a good judge of characterCaliban is not a good judge of character He decides for example that Stephano is a god because he dispenses lsquocelestial liquorrsquo (Act II Sc 2 Line 115) but then it must be remembered that he has only known his mother Sycorax Prospero Miranda and the spirits that torture him However he quickly discovers his error of judgementrdquo

ldquoWhat a thrice-double ass (Act V Sc 1 Lines 295-297)Was I to take this drunkard for a godAnd worship this dull foolrdquo

Calibanrsquos Imaginative NatureIf Caliban is sub-human in what has been said above he is human in the respect of the poetic side of his character He listens to music with rapture He tells of the beautiful dreams in which heaven rains treasures upon him and which upon waking he yearns to renew One of the most poetic passages in whole play is Calibanrsquos description of the island

to Stephano and Trinculo

ldquoBe not afeard The isle is full of noises (Act III Sc 2 Lines 135-143)Sounds and sweet airs that give delight and hurt notSometimes a thousand twangling instrumentsWill hum about mine ears and sometime voicesThat if I then had waked after long sleepWill make me sleep again and then in dreamingThe clouds methought would open and show richesReady to drop upon me that when I wakedI cried to dream againrdquo

Caliban - Less Ignoble Than Some OthersCalibanrsquos motive for murder is less dishonourable than that of Antonio and Sebastian They plan to kill Alonso to gain his power and wealth Caliban merely wants revenge and the return of lsquohisrsquo island

Conclusiono Calibanrsquos character is not portrayed very clearly in the play and hence we cannot decide whether he is a poor

savage being grossly maltreated by Prospero or whether he is evil and must therefore be kept in bondage or enslavement

o Caliban is contrasted with Ariel who is a spirit and thus swift and uninterested in physical activitieso Caliban is also contrasted with Prospero who is the all-powerful master of the island and of the destiny of all

those on the islando Caliban is also contrasted with civilized man showing him to be less evil than Antonio and Stephano and less

materialistic than Stephano and Trinculoo Caliban has suffered at the hands of Prospero and he has learnt to curse by listening to Prosperorsquos abuse He

certainly believes that Prospero has deprived him of his birthrighto Finally the character Caliban is thought to be one of Shakespearersquos masterpieces The complexity of the character

is reflected in the large volume of critical discussion that has grown around it

ECO ndash12 Topic-Forms of market

MonopolyMonopoly is a market structure in which there is a single seller there are no close substitutes for the commodity produced by the firm and there are barriers to entry Example Indian Railways which is operated under government of India Monopoly also implies absence of competitionFeatures of Monopoly Monopoly is characterized by1 Single Seller In monopoly there is only one firm producing the product The whole industry consists of this single firm Thus under monopoly there is no distinction between firm and industry Being the only firm there is significant control of the firm over supply and price Thus under monopoly buyers do not have the option of buying the commodity from any other seller They have to buy the product from the firm or they can go without the commodity This fact gives immense control to the monopolist over the market

2No Close Substitute There are no close substitutes of the product produced by the monopolist firm If there are close substitutes of the product in the market it implies presence of more than one firm and hence no monopoly In order to ensure a total of control over the market by the monopolist firm it is assumed that there are no close substitutes of the product

3 No Entry amp Exit Monopoly can only exist when there is strong barriers before a new firm to enter the market In fact once a monopoly firm starts producing the product no other firm can produce the same One reason for this is the ability of the

monopolist to produce the product at a lower cost than any new firm who thinks to enter the market If a new firm who knows that it cannot produce at a lower cost than the monopolist then that firm will never enter the market for fear of losing out in competition Similarly the monopolist who is operating for a long time may be enjoying reputation among its customers and is in a better position to use the situation in its own benefit A new firm has to take long time to achieve this and so may not be interested to enter the market

4 Price Maker Being the single seller of the product the monopolist has full control over the pricing of the product On the other hand if there is a large number of buyers in the market so no single buyer exercises any significant influence over price determination Thus it is a sellerrsquos market So monopoly firm is a price maker

5 Price Discrimination Having considerable control over the market on account of being single seller with no entry of other firms the monopolist can exercise policy of price discrimination it means that the monopolist can sell different quantities of the same product to a consumer at different price or same quantity to different consumers at different prices by adjudging the standard of living of the consumer

6 Shape of Demand Curve Since a monopolist has full control over the price therefore he can sell more by lowering the price This makes the demand curve downward sloping

Subject Ac-12 290620 Topic- retirement Model sumThe Balance Sheet of Rohit Nisha and Sunil who are partners in a firm sharing profits according to their capitals as on 31st March 2014 was as under

Liabilities Amount Assets Amount (Rs) (` Rs)

Creditors 25000 Machinery 40000Bills Payable 13000 Building 90000General Reserve 22000 Debtors 30000Capital Less Provision for Rohit 60000 Bad debts 1000

29000 Nisha 40000 Stocks 23000 Sunil 40000 140000 Cash at Bank 18000

200000 200000

On the date of Balance Sheet Nisha retired from the firm and following adjustments were made(i) Building is appreciated by 20(ii) Provision for bad debts is increased to 5 on Debtors(iii) Machinery is depreciated by 10(iv) Goodwill of the firm is valued at Rs 56000 and the retiring partnerrsquos share is adjusted

(v) The capital of the new firm is fixed at Rs120000 Prepare Revaluation Account Capital Accounts of the partner and Balance Sheet of the new firm after Nisharsquos retirement Revaluation AccountDr Cr

Particulars Amount Particulars Amount (`Rs) (Rs`)

Provision for Bad debt Ac 500 Building Ac 18000Machinery Ac 4000Profit transferred toCapital Accounts (3 2 2)Rohit 5786Nisha 3857Sunil 3857

13500

18000 18000

Capital Account

Dr Cr

Particulars Rohit Nisha Sunil Particulars Rohit Nisha Sunil (Rs`) (Rs`) (`Rs) (Rs`) (Rs`) (Rs`)

Sunilrsquos Capital ac 9600 mdash 6400 Balance bd 60000 40000 40000Bank - 66143 - General Reserve 9428 6286 6286Balance cd 72000 mdash 48000 Revaluation (Profi 5786 3857 3857 Rohitrsquos Capital Ac mdash 9600 mdash

Sunilrsquos Capital Ac 6400 Bank 6386 - 4257

81600 66143 54400 81600 66143 54400

Balance Sheet as at 31st March 2014

Liabilities Amount Assets Amount (Rs`) (Rs`)

Creditors 25000 Building 108000Bank overdraft 37500 Machinery 36000

Bills Payable 13000 Debtors 30000Capital Less ProvisionRohit 72000 for Bad debts 1500 28500Sunil 48000 120000 Stock 23000

195500 195500

Working Notes (i) (a) Profit sharing ratio is 60000 40000 40000 ie = 3 2 2(b) Gaining Ratio Rohit = 35 ndash 37 = 2135 ndash 1535 = 635Sunil = 25-27 = 1435 ndash 1035 = 435= 635 435= 6 4 = 3 2(c) Nisha Share of Goodwill = Rs 56000 times 27 = Rs16000Share of Goodwill in the gaining ratio by the existing partner ieRohit = Rs16000 times 35 = Rs 9600Sunil = Rs 16000 times 25 = Rs 6400

The journal entry isRohitrsquos Capital Ac Dr 9600Sunilrsquos Capital Ac Dr 6400 To Nisharsquos Capital Ac 16000(Share of Goodwill divided into gaining ratio)

  • 1 Static Friction
  • The frictional force that acts between the surfaces when they are at rest with respect to each other is called Static Friction
    • Static Friction Examples
      • 2 Sliding Friction
        • Examples Of Sliding Friction
          • 3 Rolling Friction
            • Examples Of Rolling Friction
              • Objects and Reasons of the Forest Conservation Act
Page 26:  · Web viewSubject . Topic . Summary . Execution . English 1 . Sounds of animals . Hens –cackle Horses –neigh Lions –roar Owls –hoots Snake –hiss. English 2 . Mother’s

7 CoefficientAny factor of an algebraic quantity is called the coefficient of the remaining quantityeg in the algebraic term 7xyz 7 is coefficient of xyz 7x is coefficient of yz and so on

8 Like term The terms having the same literal coefficient are called like terms and those having different literal coefficients are called unlike terms

eg (i) 5xyz 8xyz -6xyz and 23xyz are like

terms(ii) 7xy2 8x2yz and -15xyz2 are unlike terms

6 in 57xy2z3 write the coefficient of

(i) 5 (vii) 5xy2 (viii) 17yz (xi) 5xyz

Solution Coefficient

(i) 5 17

xy2z3

(vii) 5xy2 17z3

(viii) 17yz

5xyzsup2

(xi) 5xyz 17yz2

7 In polynomial given below separate the like terms(ii) y2z3 xy2z3 -58x2yz -4y2z3 -8xz3y2 3x2yz and 2z3y2

Solution y2z3 -4y2z3 2z3y2 are like terms

xy2z3 -8xz3y2 are like terms

-58x2yz 3x2yz are like terms

Class IXSubject Topic Summary Execution

Bengali (2nd language)

বাগzwnjধারাzwnj বা ধারা-বা ধারা ল হিবকেশষ পরকার বাক -হিb -াকেবর এক হিবকেশষ পরকাশরীহিত াকেক কতগকেলা কার সমষটির মকেধয এগহিলকেক বা ধারা বকেল আবার কতগকেলা শকেবদর বাধাধরা যকান রীহিত যনই য-াকেব চকেল আসকে যসই -াকেবই চকেল আসকে তখন যসই শবদগহিল খন একক -াকেব অG পরকাশ ককের তখন একের বা ধারা বকেল বা ধারার পরকেয়া -াষাকেক আরও সFর ককের যতাকেল

অকাল পকক(অপহিরনত বয়কেস পাকাহিম)-মাতর শ বর বয়কেস যমকেয়টির া মকেখর কা তাকেত অকালপককতা ধরা পকে

অককা পাওয়া( মারা াওয়া) ndash পকেকIমারটি পকেকIমারকেত হিকেয় বাসাতরীকের াকেত মার যখকেত যখকেত অককা যপল

অহি| পরীকষা ( কঠিন ও পরকত পরীকষা)- যকেলটির আজ ডাকতাহির যরজালট যবকেরাকেব এIাই তার জীবকেনর ব অহি| পরীকষা

অষটরমভা (ফাহিক) ndash রীতা মকেখই বকো বকো কা বকেল আর কাকেজর যবলায় অষটরমভা

অকমGার ধাী (অপাG) ndash সমনকেক হিনকেয় যকান ান কেব না ও একেকবাকেরই অকমGার ধাী

অকেনধর ষটি (অসাকেয়র সায়)- আহিশ বকেরর বকোর নাহিত ল অকেনধর ষটি তাকেক াা বকোর একম চকেল না

আকেককল গড়ম (তবহিদধ)- ার তহিম উপকার করকেল যসই যতামার হিবরকেদধ সাকষয হিকেয়কে শকেনই আমার আকেককল গড়ম

আষাকে লপ( অবাসতব লপ) ndashIাকা এখন যকেব না এIা বলকেলই ত এমন আষাকে লপ ফাার যকান রকার হিল না

Hindi- महायजञ ा इस हानी म लख न या बतान ा परयास किया ह कि किसी भी अचछ

2nd language

परसार(यशपाल ाय या पणय न ा फल अवशय मिमलता ह ोई भी परोपार अथवा पणय लिलए किया गया ाय बार नही 0ाता वह ए परार ा यजञ हए धनी सठ थ धम परायण और किवनमर सठ न आन ी यजञ किए थ और दान म न 0ान कितना धन दिदन दखिखयो म बात दिदया थादिदन पलट और सठ यहा गरीबी आ गई उन दिदनो यजञ बचन ी परथा थी सठ भी अपनी 0गह बचन लिलए डलपर ए सट यहा चलन ो तयार हए सठानी रासत लिलए रोटी पड म बाधर सठ ो द दी रासत म ए भख R ो दखर सठ न चारो रोटी उसो खिखला दी खर वह सठ यहा डलपर पहच तो उनी सठानी न उस महायजञ बचन ो हा यदिद बचन आए सठ न R ो रोटी खिखलान ो महायजञ नही समझा और वापस लौट आया घर आर शाम ो उसी घर म उस ए बडा ख0ाना मिमला 0ो उस दवारा किए गएrsquo महायजञrsquo ा परसार था

English language

Letter formal The heading the name and address of the person you are writing to must be included beneath your own address In formal letters ldquoblock stylerdquo of address is preferred

Subject complain in brief

Salutation If the person you are writing to is known to you you may begin ldquoDear MrrdquoOr ldquoDear Mrsrdquo In all other instances you should begin ldquoDear Sirrdquo or ldquoDear Madamrdquo Or ldquoSirsrdquo

The body A formal or business letter has four partsReference The letter should begin by referring to a letter you have received an advertisement or the reason that has prompted you to writeInformation In the second paragraph it is necessary to supply more detailed information that is related to the referencePurpose Here you must give the reason why you are writing the letter This must be stated clearly and ensure that it is relevant to the question that has been setConclusion round off the letter with some polite remarkThe subscription when a letter has begun with dear sir sirs Madam you should end with Yours faithfully or yours truly When however you address a person by name you must conclude with the words ldquoYours sincerelyrdquo

1 A park in your locality is slowly being used as a rubbish dump Write a letter to the Mayor of your city pointing out the nuisance and danger of this Request that action be taken to stop this immediately

Or2 You being a boarder ordered a set of lab manuals from a famous book shop in the town They sent you a wrong set of books Write a letter to the manager of the book shop

Chemistry Chapter-1 1)CHEMICAL FORMULA- Q What is the Significance of

L-2The Language of Chemistrybull Chemical Formula

Itrsquos a symbolic representation of a chemical substance eg ndash The formula of Sulphuric acid is H2SO4

2) Steps of writing Chemical Formula of a given substance-

1 Write the symbols of the constituent atoms or radicals side by side Keep the basic radical on LHS and acid radical on the RHS ( Na+Cl- )2 In case of a radical having more than one atom( compound radical) enclose the radical in a bracket eg (SO4-)3 Write the valencies of each radical on its right hand top4 If the valencies of the two radicals are divisible by a common factor then divide the valencies by the common factor5 Invert (criss-cross) the valency number ie write the valency of one atom below the second atom and vice versa 6 On interchanging if valency number is lsquoone the figure lsquoonersquo is never writtenFor Example- Compound -Calcium Nitrate1 Writing the symbols- Ca(NO3)2 Writing the valencies on their right hand top- Ca2(NO3)1

3 Valency numeral in simple ratio- Ca2(NO3)1

4 Criss-cross- Ca 2NO3 1

5 Writing the formula of the compound- Ca(NO3)2

Chemical formula

A The formula of a substance conveys the following information regarding a substance 1 The name of the substance (qualitative)2 The elements constituting the substance (qualitative)3 The number of various atoms present in a molecule of the substance (quantitative)4 Molecular weight of the substance and the relative weights of different elements present in it (qualitative)

Q What are the limitations of Chemical Formula

A The chemical formula suffers from the following limitations-I It fails to convey whether the elements in a molecule are present in the form of atoms or ionsFor example the formula KBr fails to tell us whether Potassium and Bromine are present in the form of ions II It does not tell anything about the binding force that holds atom in a molecule togetherIII It does not tell us about the arrangement of various atoms with respect to one another within the molecule

Q Examples of Some Chemicals with their Formula Chemical name and Common Name-

A Given in the class notesCommercial Studies

Joint Stock Company

Let us discuss about the demerits of Joint Stock CompanyDespite so many advantages it has got many disadvantages which are as follows

Difficulty in FormationDelay in Decision makingExcessive Government ControlLack of Secrecy

Company can be classified into several categories based on incorporation

QuestionExplain the demerits of Joint Stock CompanyAnswer) 1 Difficulty in Formation The legal requirements and formalities required to be completed are so many The cost involved is quite heavy It has to approach large number of people for its capital It cannot start its business unless certificate of incorporation has been obtained This is granted after a long time when all the formalities are completed

Chartered CompanyStatutory CompanyRegistered Company

Delay in Decision making In this form of organization decisions are not made by single individual All important decisions are taken by the Board of Directors Decision-making process is time-consuming So many opportunities may be costly because of delay in decision-making Promptness of decisions which is a common feature of sole trader ship and partnership is not found in a company

Excessive Government ControlA company and the management have to function well within the law and the provisions of Companies Act are quite elaborate and complex At every step it is necessary to comply with its provisions lest the company and the management should be penalized The penalties are quite heavy and in several cases officers in default can be punished with imprisonment This hampers the proper functioning of the company

Lack of Secrecy The management of companies remains in the hands of many persons Every important thing is discussed in the meetings of Board of Directors Hence secrets of the business cannot be maintained In case of sole proprietorship and partnership forms of organisation such secrecy is possible because a few persons are involved in the management

2 Define the following

Chartered Company- The crown in exercise of the royal prerogative has power to create a corporation by the grant of a charter to persons assenting to be incorporated Such companies or corporations are known as chartered companies Examples of this type of companies are Bank of England (1694) East India Company (1600) The powers and the nature of business of a chartered company are defined by the charter which incorporates it After the country attained independence these types of companies do not exist

in IndiaStatutory Company- A company may be incorporated by means of a special Act of the Parliament or any state legislature Such companies are called statutory companies Instances of statutory companies in India are Reserve Bank of India the Life Insurance Corporation of India the Food Corporation of India etc The provisions of the Companies Act 1956 apply to statutory companies except where the said provisions are inconsistent with the provisions of the Act creating them Statutory companies are mostly invested with compulsory powersRegistered companiesCompanies registered under the Companies Act 1956 or earlier Companies Acts are called registered companies Such companies come into existence when they are registered under the Companies Act and a certificate of incorporation is granted to them by the Registrar

Economics

Chapter-4Basic problems of Economy

Today let us discuss with the topic Production Possibility curve

QuestionExplain the concept of Production Possibility Curve with the help of diagram

Answer) Production Possibility curve is a locus of all possible combinations of two commodities which can be produced in a country with its given resources and technology

The above diagram shows that with the given resources and technology the economy can produce maximum either 5 thousand meters of cloth or 15 thousand quintals of wheat or any other combination of the two goods like B( 1 thousand meters of cloth and 14 thousand quintals of wheat C ( 2 thousands meters of cloth and 12 thousand quintals of wheat) etcProduction Possibility curve is also called production possibility boundary or frontier as it sets the maximum limit of what it is possible to produce with given resources

Geography

Rotationand Revolution

SUNrsquoS POSITION AND SEASONAL CHANGES EQUINOXES ndash SPRING AND AUTUMN

Q1 What is Spring EquinoxA1 On 21st March sunrays fall directly on the equator On that day

As the Equator divides the Earth into two equal halves the sun rays fall directly on the equator twice in a year Equinoxes means equal Spring EquinoxOn 21st March sunrays fall directly on the equator On that day the duration of day and night both are equal ( 12 hours day and 12 hours night) on every places located on equator This day is called as Spring EquinoxAutumn EquinoxOn 23rd September sunrays fall directly on the equator On that day the duration of day and night both are equal ( 12 hours day and 12 hours night) on every places located on equator This day is called as Autumn Equinox

SOLSTICES ndash SUMMER AND WINTERDue to inclination of the Earth on its axis and the apparent movement of the sun the sun rays fall directly on both tropics once in a year Solstice is a Latin word which mean ldquothe Sun standing stillrdquoSummer SolsticesAfter 21st March there is an apparent movement of the Sun to the north of the equator The apparent northward movement up to 21st June when the Sun appears overhead at the Tropic of Cancer (22frac12degN) The sun appears to stand still at this position and then moves southwards towards the equator This position of the Sun on 21st June is known as Summer Solstices On that day the duration of day and night both are equal ( 12 hours day and 12 hours night) on every places located on Tropic of Cancer (22frac12degN)Winter solstices The apparent southward movement of the Sun continues beyond the equator till 22nd

December On this day the Sun is overhead at the Tropic of Capricorn

the duration of day and night both are equal ( 12 hours day and 12 hours night) on every places located on equator This day is called as Spring Equinox

Q2 What do you mean by EquinoxA2 Equinoxes means equal It is use to explain the equal duration of day and night ( 12 hours day and 12 hours night) on the Earth

Q3 On which date the longest day in Tropic of CancerA3 21st June

Q4 What is the meaning of SolsticeA4 Solstice is a Latin word which mean ldquothe Sun standing stillrdquo

Q5 Which is the longest day in southern hemisphereA5 22nd December

Q6 On what date does the Arctic Circle experience the lsquoMidnight SunrsquoA6 On 21 June the Arctic Circle experiences the lsquoMidnight Sunrsquo

Q7 What is cause of Midnight Sun in NorwayA7 During the summer solstice (21 June) the North Pole is inclined towards the Sun Therefore the duration of sunlight or daytime increases from 12 hours at the Equator to 24 hours at the Arctic Circle and beyond Thatrsquos why The region beyond the Arctic Circle especially Norway is known as the Land of the Midnight Sun because there the Sun does not rise or set on 21 June

Q8 Match the column A with BA B

Summer Solstice 21st March

Autumn Equinox 23rd

September

Winter Solstice 21st June

(22frac12degS) This position of the Sun is referred to as the Winter Solstice because it marks the winter season in the Northern Hemisphere On that day the duration of day and night both are equal ( 12 hours day and 12 hours night) on every places located on Tropic of Capricorn (22frac12degS)SEASONS AND DURATION OF DAY AND NIGHT During the equinoxes all places on the Earth have 12 hours of day and 12 hours of night Due to the revolution of the Earth round the Sun on an inclined axis the duration of day and night varies according to seasons and the latitude of a placeDuring the summer solstice (21 June) the North Pole is inclined towards the Sun Therefore the duration of sunlight or daytime increases from 12 hours at the Equator to 24 hours at the Arctic Circle and beyondThe region beyond the Arctic Circle especially Norway is known as the Land of the Midnight Sun because there the Sun does not rise or set on 21 JuneAt the North Pole there will be six months of daylight The Sun will be seen always above the horizon at a low angle At 66degN 24 hours of sunlight can be seen only on 21 June Hammerfest in northern Norway is a place of tourist attraction for observing the phenomenon of the Midnight Sun This place has continuous daylight from 13 May to 29 July This place is easily accessible to tourists and has hotels and other facilities The view of the midnight Sun from here is enthrallingIn the Southern Hemisphere the duration of daylight decreases from 12 hours at the equator to 0 hours beyond the Antarctic Circle In the South Polar Region there is 24 hours of darkness The Sun is always below the horizon In the Southern Hemisphere which experiences winter the duration of night-time is longer than the duration of daylight

Spring Equinox 22nd

December

A8 A B

Summer Solstice 21st June

Autumn Equinox 23rd

September

Winter Solstice 22nd

December

Spring Equinox 21st March

During winter solstice (22 December) the South Pole is inclined towards the Sun The Southern Hemisphere experiences summer and the Northern Hemisphere has winter Therefore the duration of daylight or sunlight is greater in the Southern Hemisphere than in the Northern HemisphereThe duration of daylight increases from 12 hours at the equator to 24 hours beyond the Antarctic Circle The South Polar Region has 24 hours of sunlight for many days continuously At the South Pole there will be six months of sunlight The Sun will always be seen at a low angle above the horizon In the Northern Hemisphere the duration of daylight will decrease from 12 hours at the equator to 0 hours at the Arctic Circle There are 24 hours of darkness in the North Polar region The duration of night is greater than the duration of daylight as one move northwards from the Equator It is evident from the above table that the duration of daylight is 12 hours throughout the year at the equator only As one moves away from the equator the seasonal variations in the duration of daylight increase The seasonal variations in the duration of daylight are maximum at the Polar Regions

Subject Eng Literature (The Merchant of Venice ndash William Shakespeare)Topic Act II Scene 7 Lines 36 to 80 (End of scene ) [Students should read the original play and also the paraphrase provided]

Summary Questions amp AnswersThe Prince then examines the inscription on the silver casket which says ldquoWho chooseth me shall get as much as he deservesrdquo The Prince says that he deserves Portia more than anybody else because of his high rank his noble birth and his great wealth and power But then he argues that silver is ten times

(1) (Act II Sc 7 L 39-47)

From the four corners of the earth they come

To kiss this shrine this mortal breathing saint

The Hyrcanian deserts and the vasty wildsOf wide Arabia are as through-fares now

inferior to gold and therefore he cannot believe that the portrait of such a beautiful lady as Portia can be contained in the silver casket He decides to see the inscription on the golden casket before making his decision

The Prince goes to examine the inscription on the golden casket which says ldquoWho chooseth me shall get what many men desirerdquo The Prince believes that the whole world desires to possess Portia otherwise so many suitors would not have come from all corners of the world for winning Portia Some of them have come from the distant lands of Persia and Arabia The deserts of Persia (Hyrcanian deserts) and the boundless desolate lands of Arabia have been crossed by the Princes seeking the hand of Portia He contrasts this casket containing Portiarsquos portrait with the old English gold coin bearing the image of the archangel (angel of the highest rank) He goes on to remark that while the figure of the archangel is engraved (Insculped) upon the English coin the picture of Portia who is beautiful as an angel lies hidden inside one of the caskets namely the Golden Casket (Golden Bed)

On the basis of his assessment of the inscription on the golden casket the Prince decides to choose the golden casket He asks for the key and opens the golden casket only to find therein an empty human skull holding a roll of

For princes to come view fair PortiaThe watery kingdom whose ambitious headSpets in the face of heaven is no barTo stop the foreign spirits but they comeAs orsquoer a brook to see fair Portia

(i) Explain the occasion for the above mentioned speech

These are the comments of the Prince of Morocco after he reads the inscription on the golden casket His mental process is revealed to us in these words We find him debating within himself as to which casket he should choose

(ii) What light does the above speech throw on the personality of Prince of Morocco

From the above mentioned speech we come to know that the Prince of Morocco is keen to marry Portia He is the type of person who is easily taken away by outward appearance He is in love with Portia because of her beauty

(iii) What information can you gather about Portia from the above mentioned lines

The given speech shows that Portia is a very beautiful lady She must be possessed of good qualities because many suitors come to her place from all over the world with a desire to get married to her The Prince of Morocco is so impressed by her beauty that he calls her a saint According to him the whole world is desirous of having her

(iv) Elucidate the significance of the first two lines

In these lines the Prince of Morocco pays a compliment to Portia These lines show his admiration for her He says that people come from all parts of the world to see fair Portia

(v) Explain the meaning of the last four lines of the

passage

In these lines the Prince of Morocco says that even the vast oceans which throw a challenge at the sky are unable to prevent men from coming to Portiarsquos place to have a glimpse of her These lines are also a tribute to Portiarsquos beauty and good qualities Many men voyage across the ocean treating it as a mere stream to see the beautiful Portia

paper in which is written that whoever happens to be guided by the glitter of things is invariably deceived

On reading the scroll the Prince says that he is too sad at heart to speak a more formal farewell and leaves with his followers amidst a sound of trumpets

After the Prince of Morocco leaves Portia remarks that the Prince is a gentle fellow but she is rid of him May all persons of his nature make a similar choice

IMPORTANT PASSAGES EXPLAINED

(Act II Sc 7 L 39-43)From the four corners of the earth they come

To kiss this shrine this mortal breathing saintThe Hyrcanian deserts and the vasty wildsOf wide Arabia are as through-fares nowFor princes to come view fair Portia

Context

This passage occurs in Act II Scene 7 in The Merchant of Venice This is part of the speech made by the Prince of Morocco

(2)

(Act II Sc 7 L 48-53)

MOROCCO One of these three contains her heavenly pictureIst like that lead contains her

Twere damnation To think so base a thought it were too grossTo rib her cerecloth in the obscure graveOr shall I think in silver shes immurdBeing ten times undervalued to tried gold

(i) What meaning does the Prince of Morocco find out of the inscription of the golden casket What have Belmont and Portiarsquos house been called and why

The inscription on the golden casket is ldquoWho chooseth me shall gain what many men desirerdquo The Prince finds out that it means that the chooser of the golden casket will get Portia because many men desire her In fact the entire world desires her Because of the coming of many suitors to Belmont from different countries in order to win Portiarsquos hand Belmont has become a centre of pilgrimage and her house is the shrine where saintly Portia is installed

(ii) What does the Prince of Morocco do before making the final choice of the casket Which is the correct casket and who will win Portiarsquos hand

The Prince of Morocco surveys and analyses the inscriptions on the casket of lead silver and gold Before making the final choice like a very systematic and methodical person he once again considers the claims of the caskets The casket containing Portiarsquos picture is the correct casket and the person choosing it will win Portiarsquos hand

Explanation

While praising Portia the Prince of Morocco conceives Portia as a goddess whose image is placed inside one of the caskets Many suitors are coming from far and wide the north and the south the east and the west (Four corners) in order to try their luck Some of them have come from the distant land of Persia and Arabia The deserts of Persia (Hyrcanian deserts) and the boundless desolate lands of Arabia have been crossed by the Princes seeking the hand of Portia All this shows that Portia is indeed the most beautiful lady of the world

(iii) What does the Prince of Morocco say in his estimation while examining the motto on the silver casket What does he find in the golden casket

While examining the motto on the silver casket which says ldquoWho chooseth me shall get as much as he deservesrdquo Morocco says that in his own estimation he surely deserves Portia in all respects ndash rank birth wealth etc

He chooses the golden casket When he opens it he finds an empty human skull holding a scroll in which it is written that those who are attracted by the glittering outside of things are always deceived as Morocco has been deceived

(iv) What kind of nature does the Prince of Morocco have

The Prince of Morocco has a simple nature who does not look deeply into the inner meaning of things but is dazzled by the outward appearance of gold He is inclined to over-estimate his own value and does not realize that it is a duty to ldquogive and hazardrdquo To say that he will not hazard for lead shows that he misreads the true meaning of the inscription which is that he should be prepared to ldquohazard all he hathrdquo for Portia So his feeling is only one of fascination and romantic attraction

(v) Do you think that the lottery of the caskets is not a matter that will be determined by chance

In fact the lottery of the casket is not a matter that will be determined by mere chance but that it is a true test of character and of sincerity which is amply proved not only by Moroccorsquos choice but also by the arguments which he uses to help him in his choice

(Act II Sc 7 L 55-59)

They have in England

A coin that bears the figure of an angelStamped in gold but thats insculpd uponBut here an angel in a golden bedLies all within

Context

(3)

(Act II Sc 7 L 63-77)A carrion Death within whose empty eye

There is a written scroll Ill read the writing

All that glisters is not goldOften have you heard that toldMany a man his life hath soldBut my outside to beholdGilded tombs do worms infoldHad you been as wise as boldYoung in limbs in judgment oldYour answer had not been inscrolld

This passage occurs in Act II Scene 7 in The Merchant of Venice This is part of the speech made by the Prince of Morocco

Explanation

In this passage the Prince of Morocco bestows high praise on Portia whose hand he is seeking He contrasts this casket containing Portiarsquos portrait with the old English gold coin bearing the image of the archangel (angel of the highest rank) He goes on to remark that while the figure of the archangel is engraved (Insculped) upon the English coin the picture of Portia who is beautiful as an angel lies hidden inside one of the caskets namely the Golden Casket (Golden Bed) In the day of Elizabeth silver was ten times inferior in value to gold Therefore the Prince of Morocco believing that Portiarsquos portrait is contained in the Golden Casket decides to choose the Golden Casket

Fare you well your suit is coldCold indeed and labour lostThen farewell heat and welcome frostmdashPortia adieu I have too grievd a heartTo take a tedious leave Thus losers part

(i) What reward does the Prince of Morocco get after making a wrong choice of the Casket How does he feel

After making the wrong choice in selecting the casket of gold the Prince of Morocco as a reward earns a rebuke in the form of a scroll tucked in the empty eye-socket of a skull kept in the casket of gold The Prince is shocked and disappointed He becomes all the more sad and dejected when he reads the scroll which points to his foolishness in being misled by the appearance and outward show as indicative of its worth

(ii) How does the Prince respond after reading the scroll

After reading the scroll the Prince though upset accepts the result with good grace and decorum befitting a royal suitor and true sportsman He says that his love-suit is really cold otherwise he would have chosen correctly but now his efforts have been in vain So he bids farewell to Portia to the warmth and enthusiasm of love and welcomes the cold and bitterness of dejection and misery of life which lies ahead

(iii) What request does he make to Portia and why

After being failure in his mission he requests Portia to give him permission to leave at once because he is too sad to undergo the tediousness of a formal leave-taking He tells that it is the manner in which defeated persons part unceremoniously

(iv) Explain the following lines

ldquoAll that glisters is not goldOften have you heard that toldMany a man his life hath soldBut my outside to beholdGilded tombs do worms infoldrdquo

Mere glitter does not make a metal to be gold Man has often been warned against appearance but it has been of no use Many people have sacrificed their lives only to seek the outer appearance of gold Worms are found inside the gilded

monuments

Class XSubject Topic Summary Execution

Hindi 2ndlang

नया रासता भाग 6 मायाराम 0ी घर म धनी मल 0ी और उनी बटी सरिरता ी ही चचा बनी रहती थी अमिमत ो इसम ोई रलिच ना थी वह धनी घर ी लडी स शादी र सवय ो बचना नही चाहता था उसा भी सवाणिभमान ह ईशवर ी पा

स उस पास पस ी ोई मी नही थी अभी उसन फकटरी ही लगाई थी उसी समझ बाहर था कि उस घर वालो ा झाव पस ी तरफ कयो

ह उसन मा स सवाल किया कि मा तम सरिरता स मरी शादी कयो रना चाहती हो मा न उस समझाया कि वह दखन म बरी नही ह और किफर खानदान अचछा

ह वह ए शल गरहणी रप म घर सभाल सगी अमिमत न मा ो इस बात ा एहसास राया कि मीन सबध लिलए मना रन पर उस दिदल

पर कया बीती होगी मा और अमिमत ी लडी बार म ाफी बात हईमा ा झाव सरिरता ी तरफ था कयोकि वह घर पर अचछा दह0 लर आ रही

थी अमिमत न अपनी मौसी ी बरी हालत बार म बताया कि किस तरह वह बड घर ी खानदानी बटी लाई थी और आ0 उसी हालत कितनी खराब ह लाई थी बहकलब 0ाती ह और बचचो ो भी नही दखती ह बात चल ही रही

थी कि तभी ए ार बाहर आर री धनी मल0ी घर अदर आए और पीछ स डराइवर फल ी ए टोरी लर आया अदर आए और पीछ स

डराइवर ए टोरी फल ी लर आया अमिमत ो फल ी पटी बरी लग रही थी अमिमत न पछ लिलया यह फल कयो ल आए ह प इन सब ी कया

0ररत थी उनो न 0वाब दिदया कि 4 पटी शमीर स मगाए थ अमिमत ो या सनर करोध आ गया तभी उस किपता 0ी आ गए उन आत ही अमिमत उठर बाहर चला गया वहा वहा मा पास आर बठ गया और बोला

अभी रिरशता तय नही हआ और धनी मल 0ी धनी मल 0ी फल ी पटी लर चलआय मा न समझाया कि 0ब सबध 0ड 0ाता ह तो खाली हाथ नही

आत अमिमत न मा स हा कि तम सबन सरिरता ो इस घर म लान ी ठान रखी ह धनीमल 0ी उस दिदन सरिरता ो दखन ी तारीख तय रन आय थ

Commercial Studies

Banking Nowadays Bank provide easy and quick services through internet facilities methods of Banking is called internet bankingIn order to save the time and money involved in visiting Bank branches people increasingly prefer to have internet banking

There are different modes of doing internet banking or transferring money through online They areReal Time Gross Settlement (RTGS)National Electronic Fund Transfers (NEFT)

1

Question

1) Explain the term RTGS Write the features of RTGS

Answer)The acronym RTGS stands for Real Time Gross Settlement which may be defined as the continuous real time settlement of funds transfer individually on and order by order basis without netting lsquoReal timersquo may be defined as the processing of instructions at the time they are received rather than at some letter time lsquoGross settlementrsquo may be defined as the settlement of transfer instructions which occurs

individually

Features of RTGS1It is the continuous settlement of

funds transfer individually on an order by order basis

2RTGS facility is provided only by CBS core banking solution enabled Bank branches

3Amount charged from the customer for RTGS transactions vary from bank to bank

2) Explain the term NEFT Write the features of NEFT

Answer) National electronic funds transfer may be defined as a nationwide system that facilitates individuals Farms and copper operates to electronically transfer funds from any bank branch to any individual farm or corporate having an account with any other bank branch in the country

Features of NEFT2 Transfer can be made 7 times on

weekdays and 6 times on Saturday

3 NEFT cannot be used to receive foreign remittances

4 NEFT transaction takes place in batches

5 A bank branch must be NEFT enabled to become a part of NEFT fund transfer network

6 There is no maximum or minimum amount that can be transferred through NEFT when one bank has a bank account

English Language

CompositionEssay

A composition is an art of creating a piece of writing on any topic or subject It is the writing correctly beautifully and clearly in order to make some interesting reading Structure of the composition

Introduction ( you lay the foundation for your composition)

Body (it constitutes the main part of the essay)

Conclusion (final statement that leaves a lasting impression)

Kinds of essays1 The Narrative essay2 The descriptive essay3 The reflective essay4 The argumentative essay

Write a composition on any one of the following topics (350- 400 words)

1 Friendship Or2 The first day of your school

Subject Eng Literature (The Merchant of Venice ndash William Shakespeare)Topic Act V Scene 1 Lines 127 to 158 (Nerissa helliphellip The clerk will nersquoer wear hair onrsquos face that had it) [Students should read the original play and also the paraphrase given in the school prescribed textbook]

Summary Revision Questions o Soon thereafter Bassanio Gratiano

and Antonio arrive

o Bassanio tells Portia that he is feeling as if it is morning because of the presence of Portia who is shining like the sun When Antonio is introduced by Bassanio to Portia she tells Bassanio that he should be grateful to Antonio who took so much trouble on his account even to the extent of risking his life

o Nerissa starts quarrelling with Gratiano and demands that he show her the ring she had presented to him and which she had warned him not to lose She suspects that Gratiano must have presented the ring to some young woman and not to the lawyerrsquos clerk as he repeatedly says and assures

Answer the following questions to check your preparation of Act IV Scenes 1 and 2

You must attempt only after you have completed your preparation of Act IV The answers must be in complete sentences using textual evidence (with citation) when necessary

[It would be in your own interest to attempt the above questions honestly totally refraining from consulting your textbook or your notes during answering After completion you should correct the paper yourself consulting the textbooknotes etc and award marks as specified Please let me know the marks you scored through WhatsApp in the group or to my personal WhatsApp]

Act IV Scene 1 (each question carries 2 marks)

1 What did the Duke try to do for Antonio

2 Why does Shylock refuse to show mercy How does he justify his stance

3 Why does Antonio say he is ready to die 4 What information is contained in Bellariorsquos letter

5 Why does Portia (as Balthazar) assert that Shylock must show mercy How does he respond

6 What offers are made to Shylock to get him to spare Antonio How are they received

7 What does Antoniorsquos speech as he faces the prospect of Shylockrsquos knife tell you about his character

8 How do Bassanio and Gratiano react to the looming prospect of Antoniorsquos demise

9 How does Portia (as Balthazar) use the law to turn the tables on Shylock

10 What does the Duke decree should happen to Shylock Why What happens to Shylockrsquos estate

11 What does Portia ask Bassanio as payment for her ldquoservicesrdquo What is his initial response What makes him change his mind

Act IV Scene 2 (each question carries 1frac12 marks)

1 What does Gratiano bring to Portia (Balthazar)

2 What does Nerissa plan on getting from Gratiano What does Portiarsquos comment suggest about men

ECO-10 280620 Topic-Supply AnalysisSHIFTING OF SUPPLY

But if there is change in factors other than the price of the commodity then either more is supplied at the same price or less supplied at the same price In such cases the price of the commodity remains constant but there is a change in other factors like change in the price of inputs change in technology of production change in price of other related goods change in taxation policy of the government etc For example there is an improvement in the technology of production of the commodity in question It leads to decrease in per unit of cost production of the commodity The firm is willing to sell more quantity of the commodity at the same price So the supply other commodity increases at the same price This increase in supply is shown by rightward shift of supply curve On the other hand if the firm uses inferior technology of production the cost of production per unit of the commodity increases The firm is willing to sell less quantity at the same price So the supply of the commodity decreases at the same price This decrease in supply is shown by leftward shift of the supply curve The above cases of increase and decrease in supply can be shown with the help of the following figures

Y INCREASE OF SUPPLY Price (Rs) s

P A s1

B

s

X` O s1 X

q q1

Y` Quantity demanded (in units)

Y DECREASE IN SUPPLY s2

s

price (Rs)

C

p A

s2

s

X` o X

q2 q

Y` Quantity demanded ( in units)

Main factors causing increase in supply or rightward shift of supply Curve(i) Fall in the price of other related goods

(ii) Fall in the price of inputsfactors(iii) Use of better technology in production(iv) Decrease in the rate of excise duty by government(v) If the objective of producer changes from profit maximization to salesMaximization

Main factors causing decrease in supply or leftward shift of supply curve(i) Increase in the price of other related goods(ii) Rise in the price of inputsfactors(iii) Use of inferior technology in production(iv) Increase in the rate of excise duty by the government(v) If the objective

Subject - Biology Topic ndash Chapter mdash6 PhotosynthesisSummary Execution

Today we will know about photosynthesis and its stages

Q1 What do you mean by photosynthesis The process by which living plants containing chlorophyll produce food

substances from carbon-di- oxide and water by using light energy Sunlight

6CO2 +12 H2O----------------------- C6 H12O6 + 6H2O + 6O2

Chlorophyll

Q2 What are the importance of photosynthesis I) Food for all Green plants trap solar energy by photosynthesis

process and supply food and energy for all living organisms either directly or indirectly

Ii) Oxygen to breathe in by product of photosynthesis is oxygen which is essential for all living organisms respiration

Q3 Write about two main phases of photosynthesis A Light dependent phase This phase occur in grana of chloroplast I) The chlorophyll on exposure to light energy becomes activated by

absorbing photons Ii) The absorbed energy is used in splitting the water molecules (H2O)

into its two components (H+ and OH- ) and releasing electron s 2H2O------------------------- 4H+ + 4e- +O2

Energy of 4 photons This reaction is known as photolysis

End products are H+ and oxygen water

B Light independent (Dark ) phase The reactions in this phase require no light energy

Here CO2 combine with H+ and produce glucose

Class XI

Subject Topic Summary ExecutionEVS Chapter-4 Legal

regimes for sustainable development

Environmental legislationEnvironmental legislation is the collection of laws and regulations pertaining to air quality water quality the wilderness endangered wildlife and other environmental factors The act ensures that matters important to the environment are thoroughly

Learn -The Forest (Conservation) Act 1980

considered in any decisions made by federal agencies

The Forest (Conservation) Act 1980 The Forest (Conservation) Act 1980 an Act of the Parliament of India to provide for the conservation of forests and for matters connected therewith or ancillary or incidental thereto It was further amended in 1988 This law extends to the whole of IndiaObjects and Reasons of the Forest Conservation Act

Deforestation causes ecological imbalance and leads to environmental deterioration Deforestation had been taking place on a large scale in the country and it had caused widespread concern The act seeks to check upon deforestation and de-reservation of forests

Subject Eng Literature (The Tempest ndash William Shakespeare) Topic Act II Scene 1 Lines 314 to 329 (End of scene)

[Students should read the original play and also the paraphrase given in the school prescribed textbook]Summary Questions amp Answers

Conspiracy of Antonio and Sebastian (Contd)

o As they approach Ariel appears again and wakes up Gonzalo by singing a tune in his ear Alonso also wakes up and they see both Sebastian and Antonio with drawn swords On being caught off guard they make up a story saying that they had heard a bellowing of bulls or lions

o They then moved to another part of the island

o Ariel at once rushes to Prospero to inform him of this development

SUMMING-UP of ACT-2 SCENE-1

(i) Among the survivors Ferdinand is separated from the rest which results in the disconsolate grief of Alonso as he took him for dead

(ii) The villainy of Antonio is confirmed

(iii) The supremacy of Prosperorsquos magic which resulted in the failure of the human conspiracy

(1)

(Act II Sc 1 L 311-325)SEBASTIAN Whiles we stood here securing your repose

Even now we heard a hollow burst of bellowing Like bulls or rather lions Didt not wake youIt struck mine ear most terribly

ALONSO I heard nothingANTONIO O rsquotwas a din to fright a monsters ear

To make an earthquake Sure it was the roarOf a whole herd of lions

ALONSO Heard you this GonzaloGONZALO Upon mine honour sir I heard a humming

And that a strange one too which did awake meI shaked you sir and cried As mine eyes opened I saw their weapons drawn There was a noiseThats verily rsquoTis best we stand upon our guardOr that we quit this place Lets draw our weapons

(i) Why has Prospero sent Ariel to Gonzalo and Alonso What does Ariel do to awaken Gonzalo

Prospero has already come to know by his magic powers the danger which threatens Gonzalo who had been Prosperorsquos friend and so he sent Ariel to preserve the lives of both Gonzalo and Alonso Prospero does not want that his scheme should remain unfulfilled Ariel begins to sing a song in Gonzalorsquos ears to awaken him(ii) Who are ready to carry out their plan Who takes steps to stop them Why does Gonzalo feel surprised after being awakened

Sebastian and Antonio are ready to carry out their plans They are standing with their swords drawn to kill Alonso and

(iv) We see two sets of contrasting characters Gonzalo-Adrian against Antonio-Sebastian

(v) The grief that works in Alonso can be perceived to his repentance for his association in Antoniorsquos crime against Prospero

Gonzalo Ariel takes steps to stop them from carrying out their nefarious scheme When Gonzalo is awakened by the song sung by Ariel into his ears he (Gonzalo) feels surprised because he sees Sebastian and Antonio standing with their swords drawn(iii) What reason do Sebastian and Antonio tell of drawing their swords when they are suspected by Alonso and Gonzalo

When Sebastian and Antonio are seen with their swords drawn they are looked with suspicion by Gonzalo and Alonso At first Sebastian tells them that as they stood here to guard them during their sleep they heard only a little before a sudden loud noise very much like the roaring of bulls or more probably that of lions Then Antonio follows him saying that this was a noise so terrible as to frighten even a monsterrsquos ears and this noise could even have shaken the earth and it was surely like the roaring of a multitude of lions Then seeing the danger they have drawn their swords Perhaps after hearing the terrible noise they (Gonzalo and Alonso) woke up from their sound sleep

(iv) What does Gonzalo tell Alonso about the strange noise What did he see on opening his eyes Gonzalo tells Alonso that he did not hear the sound of roaring but he heard a humming sound which was strange and which woke him up After waking up he gave him (Alonso) a shaking and a loud cry On opening his eyes he saw these two gentlemen standing with their swords drawn(v) What does Gonzalo suggest

Gonzalo suggests that there was a noise indeed and of that he has no doubt at all and suggests that the best course for them would be to remain alert and vigilant against any possible danger to their lives or to leave this place and move to some other part of the island

Class XIISubject Topic Summary Execution

Commerce

Chapter- Management

Today we will discuss about LEVELS OF MANAGEMENT

Levels of management is a series or chain of managerial positions from top to bottom It helps individuals to know their authority responsibilities and superior-subordinate relations among themselves There are mainly three levels of Management TOP LEVEL MANAGEMENTMIDDLE LEVEL MANAGEMENTLOWER LEVEL MANAGEMENT

Top level managementIt consists of members at the highest level in the management hierarchy This level includes Board Of Directors Chief Executive Managing Directors Chairman President Vice President

Rolefunctions of the top levelmanagement1To analyse evaluate and deal

with theexternal environment2 To determine the objectives and

policies of the business3 To strive for welfare and survival

of business

4 To create an organisational Framework consisting of authority responsibility relationship

Middle level management Congress of members or groups who are concerned with implementation of the policies let down by the top managementThis level includes head of the department such as finance manager marketing manager branch and regional managers departmental and divisional heads plant superintendent etc

Role of functions of the middle level management

1 To interpret the policies framed by top management

2 To assign duties and responsibilities to lower level managers

3 To select and appoint employees for middle and supervisory level and evaluate their performance

4 To co-operate with other departments for smooth functioning

Operational or supervisory level managementIt refers to the group are members who are concerned with execution of the work They are also known as fast line managers This level includes supervisor 4 men Section Officer clerk Inspector etc

Role of functions of the lower level management1 To plan and execute day-to-

day operations2 To supervise and control the workers3 To arrange materials and

tools to start the process and make arrangements for training

4 Today present workers grievance and suggestions before the management and

ensure safe and proper working conditions in the factory

Business Studies

Staff Appraisal Chapter- 10 Today let us start with a new chapter

Staff Appraisal

Meaning of Performance Appraisal

Performance Appraisal is the systematic evaluation of the performance of employees and to understand the abilities of a person for further growth and developmentThe supervisors measure the pay of employees and compare it with targets and plansThe supervisor analyses the factors behind work performances of employeesThe employers are in position to guide the employees for a better performance

Objectives of Performance Appraisal

Following are the objectives of Performance Appraisal

To maintain records in order to determine compensation packages wage structure salaries raises etc

To identify the strengths and weaknesses of employees to place right men on right job

To maintain and assess the potential present in a person for further growth and development

To provide a feedback to employees regarding their performance and related status

To provide a feedback to employees regarding their performance and related status

Importance of Performance Appraisal

Performance appraisal provides important and useful information for the assessment of employees skill

knowledge ability and overall job performance The following are the points which indicate the importance of performance appraisal in an organization

1 Performance appraisal helps supervisors to assess the work performance of their subordinates

2 Performance appraisal helps to assess the training and development needs of employees

3 Performance appraisal provides grounds for employees to correct their mistakes and it also provides proper guidance and criticism for employees development4 Performance appraisal provides reward for better performance

5 Performance appraisal helps to improve the communication system of the organization

6 Performance appraisal evaluates whether human resource programs being implemented in the organization have been effective

7 Performance appraisal helps to prepare pay structure for each employee working in the organization

8 Performance appraisal helps to review the potentiality of employees so that their future capability is anticipated

Geography

DRIANAGE The SubarnarekhaThe Subarnarekha and the Brahmaniinterposed between the Ganga and the Mahanadi deltas drain an area of 19300 sq kmand 39033 sq km respectively The drainage basins of these streams are shared byJharkhand Odisha west Bengal and Chhattisgarh The Brahmani is known as southKoel in its upper reaches in Jharkhand

The NarmadaThe Narmada rises in the Amarkantak hills of MadhyaPradesh It flows towards the West in a rift valleyformed due to a geological fault The total length of it is 1300 km All the tributaries of the

Q1 Name the two westward flowing rivers in the peninsular plateauA1 Narmada and Tapi are the only westward flowing rivers of the peninsular plateau

Q2 Differentiate between east-flowing rivers and west-flowing riversA2

East-flowing rivers

West-flowing rivers

Narmada are very short inlength Most of its tributaries join the main streamright anglesThe Narmada basin covers parts of Madhya Pradesh and Gujarat

The Tapi The Tapi rises in the Satpura ranges in the Betul listrictof Madhya Pradesh It flows in a rift valley parallel tothe Narmada but it is much shorter in length It coversparts of Madhya Pradesh Gujarat and MaharashtraThe length is about 724 km

The Sabarmati and the MahiThe Sabarmati rises in the Aravali hills and flows south-south-westwards for a distance of 300 kilometres to the Arabian Sea The Sabarmatibasin extends over an area of 21674 sq km in Rajasthan and Gujarat The Mahi rises inthe east of Udaipur and drains an area of 34842 sq km lying in Madhya PradeshRajasthan and Gujarat It flows south-westwards for a distance of 533 km before it fallsinto the Gulf of Khambhat

The ChambalThe Chambal rises near Mhow in the Vindhya Range and flows towards the northgenerally in a gorge upto Kota Below Kota it turns to the north-east direction and afterreaching Pinahat it turns to the east and runs nearly parallel to the Yamuna beforejoining it in the southern part of the Etawah district in Uttar PradeshMajor Rivers of India with their basin area (Sqkm)

Himalayan System Indus 321290Ganga 861404

Brahmaputra 187110Indus System

Jhelum 34775Beas 20303

Ganga System Yamuna 366223Ghaghra 127950

Peninsular RiversNarmada 98796

Tapi 65145Mahanadi 141600

Subarnarekha 19300Sabarmati 21674

Mahi 34842Godavari 312812

Godavari Krishna Kaveri Mahanadi are the east-flowing rivers

Narmada Tapi west-flowing rivers

They fall into the Bay of Bengal

They fall into Arabian Sea

These rivers form big deltas

These rivers form comparativelysmall deltas

Catchment areas of these rivers are larger

Catchment areas of these rivers are smaller

Krishna 2589488Cauveri 87900

Subject ndashBiology Topic ndashChapter -5 Inheritance amp Variations Summary ExecutionToday we will discussabout linkage and its classification

LINKAGE The tendency of the genes located on the same chromosome to stay together is

hereditary transmission Linked genes the genes responsible for this Genes that exhibit the process of linkage locates in the same chromosome The distance between the linked genes in a chromosome determines the strength

of linkage i e genes that are located close to each other show stronger linkage than that are located far from each other

COMPLETE LINKAGE It is the type of linkage showed by the genes that are closely located or are tightly

linked with each other as they have no chance of separatingby crossing over These genes are always transmitted together to the same gamete and the same

offspring In such condition only parental or non cross over type of gametes are formedINCOMPLETE KINKAGE It is type of linkage showed by the genes that are distantly located orare loosely

linked with each other because they have chance of separating by crossing over

SIGNIFICANCE i) It helps in holding the parental character togetherii) It checks the appearance of new recombination and helps in bringing the

hybrid population which resembles the original parents iii) Linked genes dilute the effects of undesirable traits

Subject Eng Literature (The Tempest ndash William Shakespeare) Topic Essay Questions (EQ-3)Question No 3

Give a character sketch of CalibanAnswer

The character of Caliban has been wonderfully conceived by Shakespeare as the manifestation of all that is gross and earthy ndash a sort of creature of the earth as Ariel is a sort of creature of the air

Calibanrsquos Physical Appearanceo Caliban is lsquofreckledrsquo a lsquomisshapen knaversquo not honoured with human shape

o Prospero calls him lsquothou tortoisersquo (Act I Sc 2 Line 317) Trinculo stumbling upon him describes him as ldquoA strange fish hellip Legged like a man And his fins like armsrdquo He ldquosmells like a fishrdquo (Act II Sc 2 Line 25)

o Prospero also calls him a ldquobeastrdquo (Act IV Sc 1 Line 140) and ldquoThis misshapen knaverdquo (Act V Sc 1 Line 268)

o Further it appears that in addition to his physical deformity his spiritual inferiority is also suggested by Prosperorsquos claim that his birth resulted from the union between his mother the witch Sycorax and the devil

Calibanrsquos ParentageWhen the play opens Caliban is twenty four years of age having been born on the island twelve years before the coming of Prospero His mother was the foul witch Sycorax who was banished from Algiers for ldquomischiefs manifold and sorceries terrible to enter human hearingrdquo (Act I Sc 2 Line 264) and the father was the Devil himself Thus

Caliban is a monster of evil and brute nature ugly deformed and stinking

Calibanrsquos Savage and Malignant Natureo Caliban is entirely a creature of the earth ndash gross brutal and savage He regards himself as the rightful possessor

of the island and Prospero as a usurper

o In his young age he was on good terms with Prospero He had consented to be received by Prospero at his house and to be educated by him He has learnt human language only to curse his master whom he abhors

o His beastly nature soon breaks out and ends in a vicious attack on Miranda This opens the eye of Prospero who becomes severe to him and enforces his service by threats and violence

o Prospero uses him to make dams for fish to fetch firewood scraper trenches wash dishes and keep his cell clean

Calibanrsquos Hatred for ProsperoA profound hatred for Prospero has taken hold of Caliban It springs from a sense of his being dispossessed and ill-treated He would kill Prospero if he could but he knows the power of Prosperorsquos lsquobookrsquo Hence he transfers his allegiance to Stephano who seems like a god to him He also incites the two drunken associates to batter the skull of Prospero when he sleeps in the afternoon

Caliban Shows Considerable Intelligenceo He has learnt Prosperorsquos language

ldquoYou taught me language and my profit onrsquot (Act II Sc 2 Lines 86-89)Is I know how to curserdquo

o He is well aware of the futility of arguing with one who has more power than he has

ldquoI must obey his art is such power (Act I Sc 2 Lines 373-376)It would control my damrsquos god SetebosAnd make a vassal of himrdquo

o He realizes the importance of Prosperorsquos books

ldquoRemember (Act III Sc 2 Lines 89-92)First to possess his books for without themHersquos but a sot as I am nor hath notOne spirit to commandrdquo

o He knows the value of stealth when attacking the enemy

ldquoPray you tread softly that the blind mole may not (Act IV Sc 1 Lines 194-195)Hear a foot fall we now are near his cellrdquo

o Caliban has a better set of values than Stephano and Trinculo They are distracted from their plan by their greed for Prosperorsquos rich garments Only Caliban realizes that such a finery is unimportant

ldquoLeave it alone thou fool it is but trashrdquo (Act IV Sc 1 Lines 224)

Caliban is not a good judge of characterCaliban is not a good judge of character He decides for example that Stephano is a god because he dispenses lsquocelestial liquorrsquo (Act II Sc 2 Line 115) but then it must be remembered that he has only known his mother Sycorax Prospero Miranda and the spirits that torture him However he quickly discovers his error of judgementrdquo

ldquoWhat a thrice-double ass (Act V Sc 1 Lines 295-297)Was I to take this drunkard for a godAnd worship this dull foolrdquo

Calibanrsquos Imaginative NatureIf Caliban is sub-human in what has been said above he is human in the respect of the poetic side of his character He listens to music with rapture He tells of the beautiful dreams in which heaven rains treasures upon him and which upon waking he yearns to renew One of the most poetic passages in whole play is Calibanrsquos description of the island

to Stephano and Trinculo

ldquoBe not afeard The isle is full of noises (Act III Sc 2 Lines 135-143)Sounds and sweet airs that give delight and hurt notSometimes a thousand twangling instrumentsWill hum about mine ears and sometime voicesThat if I then had waked after long sleepWill make me sleep again and then in dreamingThe clouds methought would open and show richesReady to drop upon me that when I wakedI cried to dream againrdquo

Caliban - Less Ignoble Than Some OthersCalibanrsquos motive for murder is less dishonourable than that of Antonio and Sebastian They plan to kill Alonso to gain his power and wealth Caliban merely wants revenge and the return of lsquohisrsquo island

Conclusiono Calibanrsquos character is not portrayed very clearly in the play and hence we cannot decide whether he is a poor

savage being grossly maltreated by Prospero or whether he is evil and must therefore be kept in bondage or enslavement

o Caliban is contrasted with Ariel who is a spirit and thus swift and uninterested in physical activitieso Caliban is also contrasted with Prospero who is the all-powerful master of the island and of the destiny of all

those on the islando Caliban is also contrasted with civilized man showing him to be less evil than Antonio and Stephano and less

materialistic than Stephano and Trinculoo Caliban has suffered at the hands of Prospero and he has learnt to curse by listening to Prosperorsquos abuse He

certainly believes that Prospero has deprived him of his birthrighto Finally the character Caliban is thought to be one of Shakespearersquos masterpieces The complexity of the character

is reflected in the large volume of critical discussion that has grown around it

ECO ndash12 Topic-Forms of market

MonopolyMonopoly is a market structure in which there is a single seller there are no close substitutes for the commodity produced by the firm and there are barriers to entry Example Indian Railways which is operated under government of India Monopoly also implies absence of competitionFeatures of Monopoly Monopoly is characterized by1 Single Seller In monopoly there is only one firm producing the product The whole industry consists of this single firm Thus under monopoly there is no distinction between firm and industry Being the only firm there is significant control of the firm over supply and price Thus under monopoly buyers do not have the option of buying the commodity from any other seller They have to buy the product from the firm or they can go without the commodity This fact gives immense control to the monopolist over the market

2No Close Substitute There are no close substitutes of the product produced by the monopolist firm If there are close substitutes of the product in the market it implies presence of more than one firm and hence no monopoly In order to ensure a total of control over the market by the monopolist firm it is assumed that there are no close substitutes of the product

3 No Entry amp Exit Monopoly can only exist when there is strong barriers before a new firm to enter the market In fact once a monopoly firm starts producing the product no other firm can produce the same One reason for this is the ability of the

monopolist to produce the product at a lower cost than any new firm who thinks to enter the market If a new firm who knows that it cannot produce at a lower cost than the monopolist then that firm will never enter the market for fear of losing out in competition Similarly the monopolist who is operating for a long time may be enjoying reputation among its customers and is in a better position to use the situation in its own benefit A new firm has to take long time to achieve this and so may not be interested to enter the market

4 Price Maker Being the single seller of the product the monopolist has full control over the pricing of the product On the other hand if there is a large number of buyers in the market so no single buyer exercises any significant influence over price determination Thus it is a sellerrsquos market So monopoly firm is a price maker

5 Price Discrimination Having considerable control over the market on account of being single seller with no entry of other firms the monopolist can exercise policy of price discrimination it means that the monopolist can sell different quantities of the same product to a consumer at different price or same quantity to different consumers at different prices by adjudging the standard of living of the consumer

6 Shape of Demand Curve Since a monopolist has full control over the price therefore he can sell more by lowering the price This makes the demand curve downward sloping

Subject Ac-12 290620 Topic- retirement Model sumThe Balance Sheet of Rohit Nisha and Sunil who are partners in a firm sharing profits according to their capitals as on 31st March 2014 was as under

Liabilities Amount Assets Amount (Rs) (` Rs)

Creditors 25000 Machinery 40000Bills Payable 13000 Building 90000General Reserve 22000 Debtors 30000Capital Less Provision for Rohit 60000 Bad debts 1000

29000 Nisha 40000 Stocks 23000 Sunil 40000 140000 Cash at Bank 18000

200000 200000

On the date of Balance Sheet Nisha retired from the firm and following adjustments were made(i) Building is appreciated by 20(ii) Provision for bad debts is increased to 5 on Debtors(iii) Machinery is depreciated by 10(iv) Goodwill of the firm is valued at Rs 56000 and the retiring partnerrsquos share is adjusted

(v) The capital of the new firm is fixed at Rs120000 Prepare Revaluation Account Capital Accounts of the partner and Balance Sheet of the new firm after Nisharsquos retirement Revaluation AccountDr Cr

Particulars Amount Particulars Amount (`Rs) (Rs`)

Provision for Bad debt Ac 500 Building Ac 18000Machinery Ac 4000Profit transferred toCapital Accounts (3 2 2)Rohit 5786Nisha 3857Sunil 3857

13500

18000 18000

Capital Account

Dr Cr

Particulars Rohit Nisha Sunil Particulars Rohit Nisha Sunil (Rs`) (Rs`) (`Rs) (Rs`) (Rs`) (Rs`)

Sunilrsquos Capital ac 9600 mdash 6400 Balance bd 60000 40000 40000Bank - 66143 - General Reserve 9428 6286 6286Balance cd 72000 mdash 48000 Revaluation (Profi 5786 3857 3857 Rohitrsquos Capital Ac mdash 9600 mdash

Sunilrsquos Capital Ac 6400 Bank 6386 - 4257

81600 66143 54400 81600 66143 54400

Balance Sheet as at 31st March 2014

Liabilities Amount Assets Amount (Rs`) (Rs`)

Creditors 25000 Building 108000Bank overdraft 37500 Machinery 36000

Bills Payable 13000 Debtors 30000Capital Less ProvisionRohit 72000 for Bad debts 1500 28500Sunil 48000 120000 Stock 23000

195500 195500

Working Notes (i) (a) Profit sharing ratio is 60000 40000 40000 ie = 3 2 2(b) Gaining Ratio Rohit = 35 ndash 37 = 2135 ndash 1535 = 635Sunil = 25-27 = 1435 ndash 1035 = 435= 635 435= 6 4 = 3 2(c) Nisha Share of Goodwill = Rs 56000 times 27 = Rs16000Share of Goodwill in the gaining ratio by the existing partner ieRohit = Rs16000 times 35 = Rs 9600Sunil = Rs 16000 times 25 = Rs 6400

The journal entry isRohitrsquos Capital Ac Dr 9600Sunilrsquos Capital Ac Dr 6400 To Nisharsquos Capital Ac 16000(Share of Goodwill divided into gaining ratio)

  • 1 Static Friction
  • The frictional force that acts between the surfaces when they are at rest with respect to each other is called Static Friction
    • Static Friction Examples
      • 2 Sliding Friction
        • Examples Of Sliding Friction
          • 3 Rolling Friction
            • Examples Of Rolling Friction
              • Objects and Reasons of the Forest Conservation Act
Page 27:  · Web viewSubject . Topic . Summary . Execution . English 1 . Sounds of animals . Hens –cackle Horses –neigh Lions –roar Owls –hoots Snake –hiss. English 2 . Mother’s

2nd language

परसार(यशपाल ाय या पणय न ा फल अवशय मिमलता ह ोई भी परोपार अथवा पणय लिलए किया गया ाय बार नही 0ाता वह ए परार ा यजञ हए धनी सठ थ धम परायण और किवनमर सठ न आन ी यजञ किए थ और दान म न 0ान कितना धन दिदन दखिखयो म बात दिदया थादिदन पलट और सठ यहा गरीबी आ गई उन दिदनो यजञ बचन ी परथा थी सठ भी अपनी 0गह बचन लिलए डलपर ए सट यहा चलन ो तयार हए सठानी रासत लिलए रोटी पड म बाधर सठ ो द दी रासत म ए भख R ो दखर सठ न चारो रोटी उसो खिखला दी खर वह सठ यहा डलपर पहच तो उनी सठानी न उस महायजञ बचन ो हा यदिद बचन आए सठ न R ो रोटी खिखलान ो महायजञ नही समझा और वापस लौट आया घर आर शाम ो उसी घर म उस ए बडा ख0ाना मिमला 0ो उस दवारा किए गएrsquo महायजञrsquo ा परसार था

English language

Letter formal The heading the name and address of the person you are writing to must be included beneath your own address In formal letters ldquoblock stylerdquo of address is preferred

Subject complain in brief

Salutation If the person you are writing to is known to you you may begin ldquoDear MrrdquoOr ldquoDear Mrsrdquo In all other instances you should begin ldquoDear Sirrdquo or ldquoDear Madamrdquo Or ldquoSirsrdquo

The body A formal or business letter has four partsReference The letter should begin by referring to a letter you have received an advertisement or the reason that has prompted you to writeInformation In the second paragraph it is necessary to supply more detailed information that is related to the referencePurpose Here you must give the reason why you are writing the letter This must be stated clearly and ensure that it is relevant to the question that has been setConclusion round off the letter with some polite remarkThe subscription when a letter has begun with dear sir sirs Madam you should end with Yours faithfully or yours truly When however you address a person by name you must conclude with the words ldquoYours sincerelyrdquo

1 A park in your locality is slowly being used as a rubbish dump Write a letter to the Mayor of your city pointing out the nuisance and danger of this Request that action be taken to stop this immediately

Or2 You being a boarder ordered a set of lab manuals from a famous book shop in the town They sent you a wrong set of books Write a letter to the manager of the book shop

Chemistry Chapter-1 1)CHEMICAL FORMULA- Q What is the Significance of

L-2The Language of Chemistrybull Chemical Formula

Itrsquos a symbolic representation of a chemical substance eg ndash The formula of Sulphuric acid is H2SO4

2) Steps of writing Chemical Formula of a given substance-

1 Write the symbols of the constituent atoms or radicals side by side Keep the basic radical on LHS and acid radical on the RHS ( Na+Cl- )2 In case of a radical having more than one atom( compound radical) enclose the radical in a bracket eg (SO4-)3 Write the valencies of each radical on its right hand top4 If the valencies of the two radicals are divisible by a common factor then divide the valencies by the common factor5 Invert (criss-cross) the valency number ie write the valency of one atom below the second atom and vice versa 6 On interchanging if valency number is lsquoone the figure lsquoonersquo is never writtenFor Example- Compound -Calcium Nitrate1 Writing the symbols- Ca(NO3)2 Writing the valencies on their right hand top- Ca2(NO3)1

3 Valency numeral in simple ratio- Ca2(NO3)1

4 Criss-cross- Ca 2NO3 1

5 Writing the formula of the compound- Ca(NO3)2

Chemical formula

A The formula of a substance conveys the following information regarding a substance 1 The name of the substance (qualitative)2 The elements constituting the substance (qualitative)3 The number of various atoms present in a molecule of the substance (quantitative)4 Molecular weight of the substance and the relative weights of different elements present in it (qualitative)

Q What are the limitations of Chemical Formula

A The chemical formula suffers from the following limitations-I It fails to convey whether the elements in a molecule are present in the form of atoms or ionsFor example the formula KBr fails to tell us whether Potassium and Bromine are present in the form of ions II It does not tell anything about the binding force that holds atom in a molecule togetherIII It does not tell us about the arrangement of various atoms with respect to one another within the molecule

Q Examples of Some Chemicals with their Formula Chemical name and Common Name-

A Given in the class notesCommercial Studies

Joint Stock Company

Let us discuss about the demerits of Joint Stock CompanyDespite so many advantages it has got many disadvantages which are as follows

Difficulty in FormationDelay in Decision makingExcessive Government ControlLack of Secrecy

Company can be classified into several categories based on incorporation

QuestionExplain the demerits of Joint Stock CompanyAnswer) 1 Difficulty in Formation The legal requirements and formalities required to be completed are so many The cost involved is quite heavy It has to approach large number of people for its capital It cannot start its business unless certificate of incorporation has been obtained This is granted after a long time when all the formalities are completed

Chartered CompanyStatutory CompanyRegistered Company

Delay in Decision making In this form of organization decisions are not made by single individual All important decisions are taken by the Board of Directors Decision-making process is time-consuming So many opportunities may be costly because of delay in decision-making Promptness of decisions which is a common feature of sole trader ship and partnership is not found in a company

Excessive Government ControlA company and the management have to function well within the law and the provisions of Companies Act are quite elaborate and complex At every step it is necessary to comply with its provisions lest the company and the management should be penalized The penalties are quite heavy and in several cases officers in default can be punished with imprisonment This hampers the proper functioning of the company

Lack of Secrecy The management of companies remains in the hands of many persons Every important thing is discussed in the meetings of Board of Directors Hence secrets of the business cannot be maintained In case of sole proprietorship and partnership forms of organisation such secrecy is possible because a few persons are involved in the management

2 Define the following

Chartered Company- The crown in exercise of the royal prerogative has power to create a corporation by the grant of a charter to persons assenting to be incorporated Such companies or corporations are known as chartered companies Examples of this type of companies are Bank of England (1694) East India Company (1600) The powers and the nature of business of a chartered company are defined by the charter which incorporates it After the country attained independence these types of companies do not exist

in IndiaStatutory Company- A company may be incorporated by means of a special Act of the Parliament or any state legislature Such companies are called statutory companies Instances of statutory companies in India are Reserve Bank of India the Life Insurance Corporation of India the Food Corporation of India etc The provisions of the Companies Act 1956 apply to statutory companies except where the said provisions are inconsistent with the provisions of the Act creating them Statutory companies are mostly invested with compulsory powersRegistered companiesCompanies registered under the Companies Act 1956 or earlier Companies Acts are called registered companies Such companies come into existence when they are registered under the Companies Act and a certificate of incorporation is granted to them by the Registrar

Economics

Chapter-4Basic problems of Economy

Today let us discuss with the topic Production Possibility curve

QuestionExplain the concept of Production Possibility Curve with the help of diagram

Answer) Production Possibility curve is a locus of all possible combinations of two commodities which can be produced in a country with its given resources and technology

The above diagram shows that with the given resources and technology the economy can produce maximum either 5 thousand meters of cloth or 15 thousand quintals of wheat or any other combination of the two goods like B( 1 thousand meters of cloth and 14 thousand quintals of wheat C ( 2 thousands meters of cloth and 12 thousand quintals of wheat) etcProduction Possibility curve is also called production possibility boundary or frontier as it sets the maximum limit of what it is possible to produce with given resources

Geography

Rotationand Revolution

SUNrsquoS POSITION AND SEASONAL CHANGES EQUINOXES ndash SPRING AND AUTUMN

Q1 What is Spring EquinoxA1 On 21st March sunrays fall directly on the equator On that day

As the Equator divides the Earth into two equal halves the sun rays fall directly on the equator twice in a year Equinoxes means equal Spring EquinoxOn 21st March sunrays fall directly on the equator On that day the duration of day and night both are equal ( 12 hours day and 12 hours night) on every places located on equator This day is called as Spring EquinoxAutumn EquinoxOn 23rd September sunrays fall directly on the equator On that day the duration of day and night both are equal ( 12 hours day and 12 hours night) on every places located on equator This day is called as Autumn Equinox

SOLSTICES ndash SUMMER AND WINTERDue to inclination of the Earth on its axis and the apparent movement of the sun the sun rays fall directly on both tropics once in a year Solstice is a Latin word which mean ldquothe Sun standing stillrdquoSummer SolsticesAfter 21st March there is an apparent movement of the Sun to the north of the equator The apparent northward movement up to 21st June when the Sun appears overhead at the Tropic of Cancer (22frac12degN) The sun appears to stand still at this position and then moves southwards towards the equator This position of the Sun on 21st June is known as Summer Solstices On that day the duration of day and night both are equal ( 12 hours day and 12 hours night) on every places located on Tropic of Cancer (22frac12degN)Winter solstices The apparent southward movement of the Sun continues beyond the equator till 22nd

December On this day the Sun is overhead at the Tropic of Capricorn

the duration of day and night both are equal ( 12 hours day and 12 hours night) on every places located on equator This day is called as Spring Equinox

Q2 What do you mean by EquinoxA2 Equinoxes means equal It is use to explain the equal duration of day and night ( 12 hours day and 12 hours night) on the Earth

Q3 On which date the longest day in Tropic of CancerA3 21st June

Q4 What is the meaning of SolsticeA4 Solstice is a Latin word which mean ldquothe Sun standing stillrdquo

Q5 Which is the longest day in southern hemisphereA5 22nd December

Q6 On what date does the Arctic Circle experience the lsquoMidnight SunrsquoA6 On 21 June the Arctic Circle experiences the lsquoMidnight Sunrsquo

Q7 What is cause of Midnight Sun in NorwayA7 During the summer solstice (21 June) the North Pole is inclined towards the Sun Therefore the duration of sunlight or daytime increases from 12 hours at the Equator to 24 hours at the Arctic Circle and beyond Thatrsquos why The region beyond the Arctic Circle especially Norway is known as the Land of the Midnight Sun because there the Sun does not rise or set on 21 June

Q8 Match the column A with BA B

Summer Solstice 21st March

Autumn Equinox 23rd

September

Winter Solstice 21st June

(22frac12degS) This position of the Sun is referred to as the Winter Solstice because it marks the winter season in the Northern Hemisphere On that day the duration of day and night both are equal ( 12 hours day and 12 hours night) on every places located on Tropic of Capricorn (22frac12degS)SEASONS AND DURATION OF DAY AND NIGHT During the equinoxes all places on the Earth have 12 hours of day and 12 hours of night Due to the revolution of the Earth round the Sun on an inclined axis the duration of day and night varies according to seasons and the latitude of a placeDuring the summer solstice (21 June) the North Pole is inclined towards the Sun Therefore the duration of sunlight or daytime increases from 12 hours at the Equator to 24 hours at the Arctic Circle and beyondThe region beyond the Arctic Circle especially Norway is known as the Land of the Midnight Sun because there the Sun does not rise or set on 21 JuneAt the North Pole there will be six months of daylight The Sun will be seen always above the horizon at a low angle At 66degN 24 hours of sunlight can be seen only on 21 June Hammerfest in northern Norway is a place of tourist attraction for observing the phenomenon of the Midnight Sun This place has continuous daylight from 13 May to 29 July This place is easily accessible to tourists and has hotels and other facilities The view of the midnight Sun from here is enthrallingIn the Southern Hemisphere the duration of daylight decreases from 12 hours at the equator to 0 hours beyond the Antarctic Circle In the South Polar Region there is 24 hours of darkness The Sun is always below the horizon In the Southern Hemisphere which experiences winter the duration of night-time is longer than the duration of daylight

Spring Equinox 22nd

December

A8 A B

Summer Solstice 21st June

Autumn Equinox 23rd

September

Winter Solstice 22nd

December

Spring Equinox 21st March

During winter solstice (22 December) the South Pole is inclined towards the Sun The Southern Hemisphere experiences summer and the Northern Hemisphere has winter Therefore the duration of daylight or sunlight is greater in the Southern Hemisphere than in the Northern HemisphereThe duration of daylight increases from 12 hours at the equator to 24 hours beyond the Antarctic Circle The South Polar Region has 24 hours of sunlight for many days continuously At the South Pole there will be six months of sunlight The Sun will always be seen at a low angle above the horizon In the Northern Hemisphere the duration of daylight will decrease from 12 hours at the equator to 0 hours at the Arctic Circle There are 24 hours of darkness in the North Polar region The duration of night is greater than the duration of daylight as one move northwards from the Equator It is evident from the above table that the duration of daylight is 12 hours throughout the year at the equator only As one moves away from the equator the seasonal variations in the duration of daylight increase The seasonal variations in the duration of daylight are maximum at the Polar Regions

Subject Eng Literature (The Merchant of Venice ndash William Shakespeare)Topic Act II Scene 7 Lines 36 to 80 (End of scene ) [Students should read the original play and also the paraphrase provided]

Summary Questions amp AnswersThe Prince then examines the inscription on the silver casket which says ldquoWho chooseth me shall get as much as he deservesrdquo The Prince says that he deserves Portia more than anybody else because of his high rank his noble birth and his great wealth and power But then he argues that silver is ten times

(1) (Act II Sc 7 L 39-47)

From the four corners of the earth they come

To kiss this shrine this mortal breathing saint

The Hyrcanian deserts and the vasty wildsOf wide Arabia are as through-fares now

inferior to gold and therefore he cannot believe that the portrait of such a beautiful lady as Portia can be contained in the silver casket He decides to see the inscription on the golden casket before making his decision

The Prince goes to examine the inscription on the golden casket which says ldquoWho chooseth me shall get what many men desirerdquo The Prince believes that the whole world desires to possess Portia otherwise so many suitors would not have come from all corners of the world for winning Portia Some of them have come from the distant lands of Persia and Arabia The deserts of Persia (Hyrcanian deserts) and the boundless desolate lands of Arabia have been crossed by the Princes seeking the hand of Portia He contrasts this casket containing Portiarsquos portrait with the old English gold coin bearing the image of the archangel (angel of the highest rank) He goes on to remark that while the figure of the archangel is engraved (Insculped) upon the English coin the picture of Portia who is beautiful as an angel lies hidden inside one of the caskets namely the Golden Casket (Golden Bed)

On the basis of his assessment of the inscription on the golden casket the Prince decides to choose the golden casket He asks for the key and opens the golden casket only to find therein an empty human skull holding a roll of

For princes to come view fair PortiaThe watery kingdom whose ambitious headSpets in the face of heaven is no barTo stop the foreign spirits but they comeAs orsquoer a brook to see fair Portia

(i) Explain the occasion for the above mentioned speech

These are the comments of the Prince of Morocco after he reads the inscription on the golden casket His mental process is revealed to us in these words We find him debating within himself as to which casket he should choose

(ii) What light does the above speech throw on the personality of Prince of Morocco

From the above mentioned speech we come to know that the Prince of Morocco is keen to marry Portia He is the type of person who is easily taken away by outward appearance He is in love with Portia because of her beauty

(iii) What information can you gather about Portia from the above mentioned lines

The given speech shows that Portia is a very beautiful lady She must be possessed of good qualities because many suitors come to her place from all over the world with a desire to get married to her The Prince of Morocco is so impressed by her beauty that he calls her a saint According to him the whole world is desirous of having her

(iv) Elucidate the significance of the first two lines

In these lines the Prince of Morocco pays a compliment to Portia These lines show his admiration for her He says that people come from all parts of the world to see fair Portia

(v) Explain the meaning of the last four lines of the

passage

In these lines the Prince of Morocco says that even the vast oceans which throw a challenge at the sky are unable to prevent men from coming to Portiarsquos place to have a glimpse of her These lines are also a tribute to Portiarsquos beauty and good qualities Many men voyage across the ocean treating it as a mere stream to see the beautiful Portia

paper in which is written that whoever happens to be guided by the glitter of things is invariably deceived

On reading the scroll the Prince says that he is too sad at heart to speak a more formal farewell and leaves with his followers amidst a sound of trumpets

After the Prince of Morocco leaves Portia remarks that the Prince is a gentle fellow but she is rid of him May all persons of his nature make a similar choice

IMPORTANT PASSAGES EXPLAINED

(Act II Sc 7 L 39-43)From the four corners of the earth they come

To kiss this shrine this mortal breathing saintThe Hyrcanian deserts and the vasty wildsOf wide Arabia are as through-fares nowFor princes to come view fair Portia

Context

This passage occurs in Act II Scene 7 in The Merchant of Venice This is part of the speech made by the Prince of Morocco

(2)

(Act II Sc 7 L 48-53)

MOROCCO One of these three contains her heavenly pictureIst like that lead contains her

Twere damnation To think so base a thought it were too grossTo rib her cerecloth in the obscure graveOr shall I think in silver shes immurdBeing ten times undervalued to tried gold

(i) What meaning does the Prince of Morocco find out of the inscription of the golden casket What have Belmont and Portiarsquos house been called and why

The inscription on the golden casket is ldquoWho chooseth me shall gain what many men desirerdquo The Prince finds out that it means that the chooser of the golden casket will get Portia because many men desire her In fact the entire world desires her Because of the coming of many suitors to Belmont from different countries in order to win Portiarsquos hand Belmont has become a centre of pilgrimage and her house is the shrine where saintly Portia is installed

(ii) What does the Prince of Morocco do before making the final choice of the casket Which is the correct casket and who will win Portiarsquos hand

The Prince of Morocco surveys and analyses the inscriptions on the casket of lead silver and gold Before making the final choice like a very systematic and methodical person he once again considers the claims of the caskets The casket containing Portiarsquos picture is the correct casket and the person choosing it will win Portiarsquos hand

Explanation

While praising Portia the Prince of Morocco conceives Portia as a goddess whose image is placed inside one of the caskets Many suitors are coming from far and wide the north and the south the east and the west (Four corners) in order to try their luck Some of them have come from the distant land of Persia and Arabia The deserts of Persia (Hyrcanian deserts) and the boundless desolate lands of Arabia have been crossed by the Princes seeking the hand of Portia All this shows that Portia is indeed the most beautiful lady of the world

(iii) What does the Prince of Morocco say in his estimation while examining the motto on the silver casket What does he find in the golden casket

While examining the motto on the silver casket which says ldquoWho chooseth me shall get as much as he deservesrdquo Morocco says that in his own estimation he surely deserves Portia in all respects ndash rank birth wealth etc

He chooses the golden casket When he opens it he finds an empty human skull holding a scroll in which it is written that those who are attracted by the glittering outside of things are always deceived as Morocco has been deceived

(iv) What kind of nature does the Prince of Morocco have

The Prince of Morocco has a simple nature who does not look deeply into the inner meaning of things but is dazzled by the outward appearance of gold He is inclined to over-estimate his own value and does not realize that it is a duty to ldquogive and hazardrdquo To say that he will not hazard for lead shows that he misreads the true meaning of the inscription which is that he should be prepared to ldquohazard all he hathrdquo for Portia So his feeling is only one of fascination and romantic attraction

(v) Do you think that the lottery of the caskets is not a matter that will be determined by chance

In fact the lottery of the casket is not a matter that will be determined by mere chance but that it is a true test of character and of sincerity which is amply proved not only by Moroccorsquos choice but also by the arguments which he uses to help him in his choice

(Act II Sc 7 L 55-59)

They have in England

A coin that bears the figure of an angelStamped in gold but thats insculpd uponBut here an angel in a golden bedLies all within

Context

(3)

(Act II Sc 7 L 63-77)A carrion Death within whose empty eye

There is a written scroll Ill read the writing

All that glisters is not goldOften have you heard that toldMany a man his life hath soldBut my outside to beholdGilded tombs do worms infoldHad you been as wise as boldYoung in limbs in judgment oldYour answer had not been inscrolld

This passage occurs in Act II Scene 7 in The Merchant of Venice This is part of the speech made by the Prince of Morocco

Explanation

In this passage the Prince of Morocco bestows high praise on Portia whose hand he is seeking He contrasts this casket containing Portiarsquos portrait with the old English gold coin bearing the image of the archangel (angel of the highest rank) He goes on to remark that while the figure of the archangel is engraved (Insculped) upon the English coin the picture of Portia who is beautiful as an angel lies hidden inside one of the caskets namely the Golden Casket (Golden Bed) In the day of Elizabeth silver was ten times inferior in value to gold Therefore the Prince of Morocco believing that Portiarsquos portrait is contained in the Golden Casket decides to choose the Golden Casket

Fare you well your suit is coldCold indeed and labour lostThen farewell heat and welcome frostmdashPortia adieu I have too grievd a heartTo take a tedious leave Thus losers part

(i) What reward does the Prince of Morocco get after making a wrong choice of the Casket How does he feel

After making the wrong choice in selecting the casket of gold the Prince of Morocco as a reward earns a rebuke in the form of a scroll tucked in the empty eye-socket of a skull kept in the casket of gold The Prince is shocked and disappointed He becomes all the more sad and dejected when he reads the scroll which points to his foolishness in being misled by the appearance and outward show as indicative of its worth

(ii) How does the Prince respond after reading the scroll

After reading the scroll the Prince though upset accepts the result with good grace and decorum befitting a royal suitor and true sportsman He says that his love-suit is really cold otherwise he would have chosen correctly but now his efforts have been in vain So he bids farewell to Portia to the warmth and enthusiasm of love and welcomes the cold and bitterness of dejection and misery of life which lies ahead

(iii) What request does he make to Portia and why

After being failure in his mission he requests Portia to give him permission to leave at once because he is too sad to undergo the tediousness of a formal leave-taking He tells that it is the manner in which defeated persons part unceremoniously

(iv) Explain the following lines

ldquoAll that glisters is not goldOften have you heard that toldMany a man his life hath soldBut my outside to beholdGilded tombs do worms infoldrdquo

Mere glitter does not make a metal to be gold Man has often been warned against appearance but it has been of no use Many people have sacrificed their lives only to seek the outer appearance of gold Worms are found inside the gilded

monuments

Class XSubject Topic Summary Execution

Hindi 2ndlang

नया रासता भाग 6 मायाराम 0ी घर म धनी मल 0ी और उनी बटी सरिरता ी ही चचा बनी रहती थी अमिमत ो इसम ोई रलिच ना थी वह धनी घर ी लडी स शादी र सवय ो बचना नही चाहता था उसा भी सवाणिभमान ह ईशवर ी पा

स उस पास पस ी ोई मी नही थी अभी उसन फकटरी ही लगाई थी उसी समझ बाहर था कि उस घर वालो ा झाव पस ी तरफ कयो

ह उसन मा स सवाल किया कि मा तम सरिरता स मरी शादी कयो रना चाहती हो मा न उस समझाया कि वह दखन म बरी नही ह और किफर खानदान अचछा

ह वह ए शल गरहणी रप म घर सभाल सगी अमिमत न मा ो इस बात ा एहसास राया कि मीन सबध लिलए मना रन पर उस दिदल

पर कया बीती होगी मा और अमिमत ी लडी बार म ाफी बात हईमा ा झाव सरिरता ी तरफ था कयोकि वह घर पर अचछा दह0 लर आ रही

थी अमिमत न अपनी मौसी ी बरी हालत बार म बताया कि किस तरह वह बड घर ी खानदानी बटी लाई थी और आ0 उसी हालत कितनी खराब ह लाई थी बहकलब 0ाती ह और बचचो ो भी नही दखती ह बात चल ही रही

थी कि तभी ए ार बाहर आर री धनी मल0ी घर अदर आए और पीछ स डराइवर फल ी ए टोरी लर आया अदर आए और पीछ स

डराइवर ए टोरी फल ी लर आया अमिमत ो फल ी पटी बरी लग रही थी अमिमत न पछ लिलया यह फल कयो ल आए ह प इन सब ी कया

0ररत थी उनो न 0वाब दिदया कि 4 पटी शमीर स मगाए थ अमिमत ो या सनर करोध आ गया तभी उस किपता 0ी आ गए उन आत ही अमिमत उठर बाहर चला गया वहा वहा मा पास आर बठ गया और बोला

अभी रिरशता तय नही हआ और धनी मल 0ी धनी मल 0ी फल ी पटी लर चलआय मा न समझाया कि 0ब सबध 0ड 0ाता ह तो खाली हाथ नही

आत अमिमत न मा स हा कि तम सबन सरिरता ो इस घर म लान ी ठान रखी ह धनीमल 0ी उस दिदन सरिरता ो दखन ी तारीख तय रन आय थ

Commercial Studies

Banking Nowadays Bank provide easy and quick services through internet facilities methods of Banking is called internet bankingIn order to save the time and money involved in visiting Bank branches people increasingly prefer to have internet banking

There are different modes of doing internet banking or transferring money through online They areReal Time Gross Settlement (RTGS)National Electronic Fund Transfers (NEFT)

1

Question

1) Explain the term RTGS Write the features of RTGS

Answer)The acronym RTGS stands for Real Time Gross Settlement which may be defined as the continuous real time settlement of funds transfer individually on and order by order basis without netting lsquoReal timersquo may be defined as the processing of instructions at the time they are received rather than at some letter time lsquoGross settlementrsquo may be defined as the settlement of transfer instructions which occurs

individually

Features of RTGS1It is the continuous settlement of

funds transfer individually on an order by order basis

2RTGS facility is provided only by CBS core banking solution enabled Bank branches

3Amount charged from the customer for RTGS transactions vary from bank to bank

2) Explain the term NEFT Write the features of NEFT

Answer) National electronic funds transfer may be defined as a nationwide system that facilitates individuals Farms and copper operates to electronically transfer funds from any bank branch to any individual farm or corporate having an account with any other bank branch in the country

Features of NEFT2 Transfer can be made 7 times on

weekdays and 6 times on Saturday

3 NEFT cannot be used to receive foreign remittances

4 NEFT transaction takes place in batches

5 A bank branch must be NEFT enabled to become a part of NEFT fund transfer network

6 There is no maximum or minimum amount that can be transferred through NEFT when one bank has a bank account

English Language

CompositionEssay

A composition is an art of creating a piece of writing on any topic or subject It is the writing correctly beautifully and clearly in order to make some interesting reading Structure of the composition

Introduction ( you lay the foundation for your composition)

Body (it constitutes the main part of the essay)

Conclusion (final statement that leaves a lasting impression)

Kinds of essays1 The Narrative essay2 The descriptive essay3 The reflective essay4 The argumentative essay

Write a composition on any one of the following topics (350- 400 words)

1 Friendship Or2 The first day of your school

Subject Eng Literature (The Merchant of Venice ndash William Shakespeare)Topic Act V Scene 1 Lines 127 to 158 (Nerissa helliphellip The clerk will nersquoer wear hair onrsquos face that had it) [Students should read the original play and also the paraphrase given in the school prescribed textbook]

Summary Revision Questions o Soon thereafter Bassanio Gratiano

and Antonio arrive

o Bassanio tells Portia that he is feeling as if it is morning because of the presence of Portia who is shining like the sun When Antonio is introduced by Bassanio to Portia she tells Bassanio that he should be grateful to Antonio who took so much trouble on his account even to the extent of risking his life

o Nerissa starts quarrelling with Gratiano and demands that he show her the ring she had presented to him and which she had warned him not to lose She suspects that Gratiano must have presented the ring to some young woman and not to the lawyerrsquos clerk as he repeatedly says and assures

Answer the following questions to check your preparation of Act IV Scenes 1 and 2

You must attempt only after you have completed your preparation of Act IV The answers must be in complete sentences using textual evidence (with citation) when necessary

[It would be in your own interest to attempt the above questions honestly totally refraining from consulting your textbook or your notes during answering After completion you should correct the paper yourself consulting the textbooknotes etc and award marks as specified Please let me know the marks you scored through WhatsApp in the group or to my personal WhatsApp]

Act IV Scene 1 (each question carries 2 marks)

1 What did the Duke try to do for Antonio

2 Why does Shylock refuse to show mercy How does he justify his stance

3 Why does Antonio say he is ready to die 4 What information is contained in Bellariorsquos letter

5 Why does Portia (as Balthazar) assert that Shylock must show mercy How does he respond

6 What offers are made to Shylock to get him to spare Antonio How are they received

7 What does Antoniorsquos speech as he faces the prospect of Shylockrsquos knife tell you about his character

8 How do Bassanio and Gratiano react to the looming prospect of Antoniorsquos demise

9 How does Portia (as Balthazar) use the law to turn the tables on Shylock

10 What does the Duke decree should happen to Shylock Why What happens to Shylockrsquos estate

11 What does Portia ask Bassanio as payment for her ldquoservicesrdquo What is his initial response What makes him change his mind

Act IV Scene 2 (each question carries 1frac12 marks)

1 What does Gratiano bring to Portia (Balthazar)

2 What does Nerissa plan on getting from Gratiano What does Portiarsquos comment suggest about men

ECO-10 280620 Topic-Supply AnalysisSHIFTING OF SUPPLY

But if there is change in factors other than the price of the commodity then either more is supplied at the same price or less supplied at the same price In such cases the price of the commodity remains constant but there is a change in other factors like change in the price of inputs change in technology of production change in price of other related goods change in taxation policy of the government etc For example there is an improvement in the technology of production of the commodity in question It leads to decrease in per unit of cost production of the commodity The firm is willing to sell more quantity of the commodity at the same price So the supply other commodity increases at the same price This increase in supply is shown by rightward shift of supply curve On the other hand if the firm uses inferior technology of production the cost of production per unit of the commodity increases The firm is willing to sell less quantity at the same price So the supply of the commodity decreases at the same price This decrease in supply is shown by leftward shift of the supply curve The above cases of increase and decrease in supply can be shown with the help of the following figures

Y INCREASE OF SUPPLY Price (Rs) s

P A s1

B

s

X` O s1 X

q q1

Y` Quantity demanded (in units)

Y DECREASE IN SUPPLY s2

s

price (Rs)

C

p A

s2

s

X` o X

q2 q

Y` Quantity demanded ( in units)

Main factors causing increase in supply or rightward shift of supply Curve(i) Fall in the price of other related goods

(ii) Fall in the price of inputsfactors(iii) Use of better technology in production(iv) Decrease in the rate of excise duty by government(v) If the objective of producer changes from profit maximization to salesMaximization

Main factors causing decrease in supply or leftward shift of supply curve(i) Increase in the price of other related goods(ii) Rise in the price of inputsfactors(iii) Use of inferior technology in production(iv) Increase in the rate of excise duty by the government(v) If the objective

Subject - Biology Topic ndash Chapter mdash6 PhotosynthesisSummary Execution

Today we will know about photosynthesis and its stages

Q1 What do you mean by photosynthesis The process by which living plants containing chlorophyll produce food

substances from carbon-di- oxide and water by using light energy Sunlight

6CO2 +12 H2O----------------------- C6 H12O6 + 6H2O + 6O2

Chlorophyll

Q2 What are the importance of photosynthesis I) Food for all Green plants trap solar energy by photosynthesis

process and supply food and energy for all living organisms either directly or indirectly

Ii) Oxygen to breathe in by product of photosynthesis is oxygen which is essential for all living organisms respiration

Q3 Write about two main phases of photosynthesis A Light dependent phase This phase occur in grana of chloroplast I) The chlorophyll on exposure to light energy becomes activated by

absorbing photons Ii) The absorbed energy is used in splitting the water molecules (H2O)

into its two components (H+ and OH- ) and releasing electron s 2H2O------------------------- 4H+ + 4e- +O2

Energy of 4 photons This reaction is known as photolysis

End products are H+ and oxygen water

B Light independent (Dark ) phase The reactions in this phase require no light energy

Here CO2 combine with H+ and produce glucose

Class XI

Subject Topic Summary ExecutionEVS Chapter-4 Legal

regimes for sustainable development

Environmental legislationEnvironmental legislation is the collection of laws and regulations pertaining to air quality water quality the wilderness endangered wildlife and other environmental factors The act ensures that matters important to the environment are thoroughly

Learn -The Forest (Conservation) Act 1980

considered in any decisions made by federal agencies

The Forest (Conservation) Act 1980 The Forest (Conservation) Act 1980 an Act of the Parliament of India to provide for the conservation of forests and for matters connected therewith or ancillary or incidental thereto It was further amended in 1988 This law extends to the whole of IndiaObjects and Reasons of the Forest Conservation Act

Deforestation causes ecological imbalance and leads to environmental deterioration Deforestation had been taking place on a large scale in the country and it had caused widespread concern The act seeks to check upon deforestation and de-reservation of forests

Subject Eng Literature (The Tempest ndash William Shakespeare) Topic Act II Scene 1 Lines 314 to 329 (End of scene)

[Students should read the original play and also the paraphrase given in the school prescribed textbook]Summary Questions amp Answers

Conspiracy of Antonio and Sebastian (Contd)

o As they approach Ariel appears again and wakes up Gonzalo by singing a tune in his ear Alonso also wakes up and they see both Sebastian and Antonio with drawn swords On being caught off guard they make up a story saying that they had heard a bellowing of bulls or lions

o They then moved to another part of the island

o Ariel at once rushes to Prospero to inform him of this development

SUMMING-UP of ACT-2 SCENE-1

(i) Among the survivors Ferdinand is separated from the rest which results in the disconsolate grief of Alonso as he took him for dead

(ii) The villainy of Antonio is confirmed

(iii) The supremacy of Prosperorsquos magic which resulted in the failure of the human conspiracy

(1)

(Act II Sc 1 L 311-325)SEBASTIAN Whiles we stood here securing your repose

Even now we heard a hollow burst of bellowing Like bulls or rather lions Didt not wake youIt struck mine ear most terribly

ALONSO I heard nothingANTONIO O rsquotwas a din to fright a monsters ear

To make an earthquake Sure it was the roarOf a whole herd of lions

ALONSO Heard you this GonzaloGONZALO Upon mine honour sir I heard a humming

And that a strange one too which did awake meI shaked you sir and cried As mine eyes opened I saw their weapons drawn There was a noiseThats verily rsquoTis best we stand upon our guardOr that we quit this place Lets draw our weapons

(i) Why has Prospero sent Ariel to Gonzalo and Alonso What does Ariel do to awaken Gonzalo

Prospero has already come to know by his magic powers the danger which threatens Gonzalo who had been Prosperorsquos friend and so he sent Ariel to preserve the lives of both Gonzalo and Alonso Prospero does not want that his scheme should remain unfulfilled Ariel begins to sing a song in Gonzalorsquos ears to awaken him(ii) Who are ready to carry out their plan Who takes steps to stop them Why does Gonzalo feel surprised after being awakened

Sebastian and Antonio are ready to carry out their plans They are standing with their swords drawn to kill Alonso and

(iv) We see two sets of contrasting characters Gonzalo-Adrian against Antonio-Sebastian

(v) The grief that works in Alonso can be perceived to his repentance for his association in Antoniorsquos crime against Prospero

Gonzalo Ariel takes steps to stop them from carrying out their nefarious scheme When Gonzalo is awakened by the song sung by Ariel into his ears he (Gonzalo) feels surprised because he sees Sebastian and Antonio standing with their swords drawn(iii) What reason do Sebastian and Antonio tell of drawing their swords when they are suspected by Alonso and Gonzalo

When Sebastian and Antonio are seen with their swords drawn they are looked with suspicion by Gonzalo and Alonso At first Sebastian tells them that as they stood here to guard them during their sleep they heard only a little before a sudden loud noise very much like the roaring of bulls or more probably that of lions Then Antonio follows him saying that this was a noise so terrible as to frighten even a monsterrsquos ears and this noise could even have shaken the earth and it was surely like the roaring of a multitude of lions Then seeing the danger they have drawn their swords Perhaps after hearing the terrible noise they (Gonzalo and Alonso) woke up from their sound sleep

(iv) What does Gonzalo tell Alonso about the strange noise What did he see on opening his eyes Gonzalo tells Alonso that he did not hear the sound of roaring but he heard a humming sound which was strange and which woke him up After waking up he gave him (Alonso) a shaking and a loud cry On opening his eyes he saw these two gentlemen standing with their swords drawn(v) What does Gonzalo suggest

Gonzalo suggests that there was a noise indeed and of that he has no doubt at all and suggests that the best course for them would be to remain alert and vigilant against any possible danger to their lives or to leave this place and move to some other part of the island

Class XIISubject Topic Summary Execution

Commerce

Chapter- Management

Today we will discuss about LEVELS OF MANAGEMENT

Levels of management is a series or chain of managerial positions from top to bottom It helps individuals to know their authority responsibilities and superior-subordinate relations among themselves There are mainly three levels of Management TOP LEVEL MANAGEMENTMIDDLE LEVEL MANAGEMENTLOWER LEVEL MANAGEMENT

Top level managementIt consists of members at the highest level in the management hierarchy This level includes Board Of Directors Chief Executive Managing Directors Chairman President Vice President

Rolefunctions of the top levelmanagement1To analyse evaluate and deal

with theexternal environment2 To determine the objectives and

policies of the business3 To strive for welfare and survival

of business

4 To create an organisational Framework consisting of authority responsibility relationship

Middle level management Congress of members or groups who are concerned with implementation of the policies let down by the top managementThis level includes head of the department such as finance manager marketing manager branch and regional managers departmental and divisional heads plant superintendent etc

Role of functions of the middle level management

1 To interpret the policies framed by top management

2 To assign duties and responsibilities to lower level managers

3 To select and appoint employees for middle and supervisory level and evaluate their performance

4 To co-operate with other departments for smooth functioning

Operational or supervisory level managementIt refers to the group are members who are concerned with execution of the work They are also known as fast line managers This level includes supervisor 4 men Section Officer clerk Inspector etc

Role of functions of the lower level management1 To plan and execute day-to-

day operations2 To supervise and control the workers3 To arrange materials and

tools to start the process and make arrangements for training

4 Today present workers grievance and suggestions before the management and

ensure safe and proper working conditions in the factory

Business Studies

Staff Appraisal Chapter- 10 Today let us start with a new chapter

Staff Appraisal

Meaning of Performance Appraisal

Performance Appraisal is the systematic evaluation of the performance of employees and to understand the abilities of a person for further growth and developmentThe supervisors measure the pay of employees and compare it with targets and plansThe supervisor analyses the factors behind work performances of employeesThe employers are in position to guide the employees for a better performance

Objectives of Performance Appraisal

Following are the objectives of Performance Appraisal

To maintain records in order to determine compensation packages wage structure salaries raises etc

To identify the strengths and weaknesses of employees to place right men on right job

To maintain and assess the potential present in a person for further growth and development

To provide a feedback to employees regarding their performance and related status

To provide a feedback to employees regarding their performance and related status

Importance of Performance Appraisal

Performance appraisal provides important and useful information for the assessment of employees skill

knowledge ability and overall job performance The following are the points which indicate the importance of performance appraisal in an organization

1 Performance appraisal helps supervisors to assess the work performance of their subordinates

2 Performance appraisal helps to assess the training and development needs of employees

3 Performance appraisal provides grounds for employees to correct their mistakes and it also provides proper guidance and criticism for employees development4 Performance appraisal provides reward for better performance

5 Performance appraisal helps to improve the communication system of the organization

6 Performance appraisal evaluates whether human resource programs being implemented in the organization have been effective

7 Performance appraisal helps to prepare pay structure for each employee working in the organization

8 Performance appraisal helps to review the potentiality of employees so that their future capability is anticipated

Geography

DRIANAGE The SubarnarekhaThe Subarnarekha and the Brahmaniinterposed between the Ganga and the Mahanadi deltas drain an area of 19300 sq kmand 39033 sq km respectively The drainage basins of these streams are shared byJharkhand Odisha west Bengal and Chhattisgarh The Brahmani is known as southKoel in its upper reaches in Jharkhand

The NarmadaThe Narmada rises in the Amarkantak hills of MadhyaPradesh It flows towards the West in a rift valleyformed due to a geological fault The total length of it is 1300 km All the tributaries of the

Q1 Name the two westward flowing rivers in the peninsular plateauA1 Narmada and Tapi are the only westward flowing rivers of the peninsular plateau

Q2 Differentiate between east-flowing rivers and west-flowing riversA2

East-flowing rivers

West-flowing rivers

Narmada are very short inlength Most of its tributaries join the main streamright anglesThe Narmada basin covers parts of Madhya Pradesh and Gujarat

The Tapi The Tapi rises in the Satpura ranges in the Betul listrictof Madhya Pradesh It flows in a rift valley parallel tothe Narmada but it is much shorter in length It coversparts of Madhya Pradesh Gujarat and MaharashtraThe length is about 724 km

The Sabarmati and the MahiThe Sabarmati rises in the Aravali hills and flows south-south-westwards for a distance of 300 kilometres to the Arabian Sea The Sabarmatibasin extends over an area of 21674 sq km in Rajasthan and Gujarat The Mahi rises inthe east of Udaipur and drains an area of 34842 sq km lying in Madhya PradeshRajasthan and Gujarat It flows south-westwards for a distance of 533 km before it fallsinto the Gulf of Khambhat

The ChambalThe Chambal rises near Mhow in the Vindhya Range and flows towards the northgenerally in a gorge upto Kota Below Kota it turns to the north-east direction and afterreaching Pinahat it turns to the east and runs nearly parallel to the Yamuna beforejoining it in the southern part of the Etawah district in Uttar PradeshMajor Rivers of India with their basin area (Sqkm)

Himalayan System Indus 321290Ganga 861404

Brahmaputra 187110Indus System

Jhelum 34775Beas 20303

Ganga System Yamuna 366223Ghaghra 127950

Peninsular RiversNarmada 98796

Tapi 65145Mahanadi 141600

Subarnarekha 19300Sabarmati 21674

Mahi 34842Godavari 312812

Godavari Krishna Kaveri Mahanadi are the east-flowing rivers

Narmada Tapi west-flowing rivers

They fall into the Bay of Bengal

They fall into Arabian Sea

These rivers form big deltas

These rivers form comparativelysmall deltas

Catchment areas of these rivers are larger

Catchment areas of these rivers are smaller

Krishna 2589488Cauveri 87900

Subject ndashBiology Topic ndashChapter -5 Inheritance amp Variations Summary ExecutionToday we will discussabout linkage and its classification

LINKAGE The tendency of the genes located on the same chromosome to stay together is

hereditary transmission Linked genes the genes responsible for this Genes that exhibit the process of linkage locates in the same chromosome The distance between the linked genes in a chromosome determines the strength

of linkage i e genes that are located close to each other show stronger linkage than that are located far from each other

COMPLETE LINKAGE It is the type of linkage showed by the genes that are closely located or are tightly

linked with each other as they have no chance of separatingby crossing over These genes are always transmitted together to the same gamete and the same

offspring In such condition only parental or non cross over type of gametes are formedINCOMPLETE KINKAGE It is type of linkage showed by the genes that are distantly located orare loosely

linked with each other because they have chance of separating by crossing over

SIGNIFICANCE i) It helps in holding the parental character togetherii) It checks the appearance of new recombination and helps in bringing the

hybrid population which resembles the original parents iii) Linked genes dilute the effects of undesirable traits

Subject Eng Literature (The Tempest ndash William Shakespeare) Topic Essay Questions (EQ-3)Question No 3

Give a character sketch of CalibanAnswer

The character of Caliban has been wonderfully conceived by Shakespeare as the manifestation of all that is gross and earthy ndash a sort of creature of the earth as Ariel is a sort of creature of the air

Calibanrsquos Physical Appearanceo Caliban is lsquofreckledrsquo a lsquomisshapen knaversquo not honoured with human shape

o Prospero calls him lsquothou tortoisersquo (Act I Sc 2 Line 317) Trinculo stumbling upon him describes him as ldquoA strange fish hellip Legged like a man And his fins like armsrdquo He ldquosmells like a fishrdquo (Act II Sc 2 Line 25)

o Prospero also calls him a ldquobeastrdquo (Act IV Sc 1 Line 140) and ldquoThis misshapen knaverdquo (Act V Sc 1 Line 268)

o Further it appears that in addition to his physical deformity his spiritual inferiority is also suggested by Prosperorsquos claim that his birth resulted from the union between his mother the witch Sycorax and the devil

Calibanrsquos ParentageWhen the play opens Caliban is twenty four years of age having been born on the island twelve years before the coming of Prospero His mother was the foul witch Sycorax who was banished from Algiers for ldquomischiefs manifold and sorceries terrible to enter human hearingrdquo (Act I Sc 2 Line 264) and the father was the Devil himself Thus

Caliban is a monster of evil and brute nature ugly deformed and stinking

Calibanrsquos Savage and Malignant Natureo Caliban is entirely a creature of the earth ndash gross brutal and savage He regards himself as the rightful possessor

of the island and Prospero as a usurper

o In his young age he was on good terms with Prospero He had consented to be received by Prospero at his house and to be educated by him He has learnt human language only to curse his master whom he abhors

o His beastly nature soon breaks out and ends in a vicious attack on Miranda This opens the eye of Prospero who becomes severe to him and enforces his service by threats and violence

o Prospero uses him to make dams for fish to fetch firewood scraper trenches wash dishes and keep his cell clean

Calibanrsquos Hatred for ProsperoA profound hatred for Prospero has taken hold of Caliban It springs from a sense of his being dispossessed and ill-treated He would kill Prospero if he could but he knows the power of Prosperorsquos lsquobookrsquo Hence he transfers his allegiance to Stephano who seems like a god to him He also incites the two drunken associates to batter the skull of Prospero when he sleeps in the afternoon

Caliban Shows Considerable Intelligenceo He has learnt Prosperorsquos language

ldquoYou taught me language and my profit onrsquot (Act II Sc 2 Lines 86-89)Is I know how to curserdquo

o He is well aware of the futility of arguing with one who has more power than he has

ldquoI must obey his art is such power (Act I Sc 2 Lines 373-376)It would control my damrsquos god SetebosAnd make a vassal of himrdquo

o He realizes the importance of Prosperorsquos books

ldquoRemember (Act III Sc 2 Lines 89-92)First to possess his books for without themHersquos but a sot as I am nor hath notOne spirit to commandrdquo

o He knows the value of stealth when attacking the enemy

ldquoPray you tread softly that the blind mole may not (Act IV Sc 1 Lines 194-195)Hear a foot fall we now are near his cellrdquo

o Caliban has a better set of values than Stephano and Trinculo They are distracted from their plan by their greed for Prosperorsquos rich garments Only Caliban realizes that such a finery is unimportant

ldquoLeave it alone thou fool it is but trashrdquo (Act IV Sc 1 Lines 224)

Caliban is not a good judge of characterCaliban is not a good judge of character He decides for example that Stephano is a god because he dispenses lsquocelestial liquorrsquo (Act II Sc 2 Line 115) but then it must be remembered that he has only known his mother Sycorax Prospero Miranda and the spirits that torture him However he quickly discovers his error of judgementrdquo

ldquoWhat a thrice-double ass (Act V Sc 1 Lines 295-297)Was I to take this drunkard for a godAnd worship this dull foolrdquo

Calibanrsquos Imaginative NatureIf Caliban is sub-human in what has been said above he is human in the respect of the poetic side of his character He listens to music with rapture He tells of the beautiful dreams in which heaven rains treasures upon him and which upon waking he yearns to renew One of the most poetic passages in whole play is Calibanrsquos description of the island

to Stephano and Trinculo

ldquoBe not afeard The isle is full of noises (Act III Sc 2 Lines 135-143)Sounds and sweet airs that give delight and hurt notSometimes a thousand twangling instrumentsWill hum about mine ears and sometime voicesThat if I then had waked after long sleepWill make me sleep again and then in dreamingThe clouds methought would open and show richesReady to drop upon me that when I wakedI cried to dream againrdquo

Caliban - Less Ignoble Than Some OthersCalibanrsquos motive for murder is less dishonourable than that of Antonio and Sebastian They plan to kill Alonso to gain his power and wealth Caliban merely wants revenge and the return of lsquohisrsquo island

Conclusiono Calibanrsquos character is not portrayed very clearly in the play and hence we cannot decide whether he is a poor

savage being grossly maltreated by Prospero or whether he is evil and must therefore be kept in bondage or enslavement

o Caliban is contrasted with Ariel who is a spirit and thus swift and uninterested in physical activitieso Caliban is also contrasted with Prospero who is the all-powerful master of the island and of the destiny of all

those on the islando Caliban is also contrasted with civilized man showing him to be less evil than Antonio and Stephano and less

materialistic than Stephano and Trinculoo Caliban has suffered at the hands of Prospero and he has learnt to curse by listening to Prosperorsquos abuse He

certainly believes that Prospero has deprived him of his birthrighto Finally the character Caliban is thought to be one of Shakespearersquos masterpieces The complexity of the character

is reflected in the large volume of critical discussion that has grown around it

ECO ndash12 Topic-Forms of market

MonopolyMonopoly is a market structure in which there is a single seller there are no close substitutes for the commodity produced by the firm and there are barriers to entry Example Indian Railways which is operated under government of India Monopoly also implies absence of competitionFeatures of Monopoly Monopoly is characterized by1 Single Seller In monopoly there is only one firm producing the product The whole industry consists of this single firm Thus under monopoly there is no distinction between firm and industry Being the only firm there is significant control of the firm over supply and price Thus under monopoly buyers do not have the option of buying the commodity from any other seller They have to buy the product from the firm or they can go without the commodity This fact gives immense control to the monopolist over the market

2No Close Substitute There are no close substitutes of the product produced by the monopolist firm If there are close substitutes of the product in the market it implies presence of more than one firm and hence no monopoly In order to ensure a total of control over the market by the monopolist firm it is assumed that there are no close substitutes of the product

3 No Entry amp Exit Monopoly can only exist when there is strong barriers before a new firm to enter the market In fact once a monopoly firm starts producing the product no other firm can produce the same One reason for this is the ability of the

monopolist to produce the product at a lower cost than any new firm who thinks to enter the market If a new firm who knows that it cannot produce at a lower cost than the monopolist then that firm will never enter the market for fear of losing out in competition Similarly the monopolist who is operating for a long time may be enjoying reputation among its customers and is in a better position to use the situation in its own benefit A new firm has to take long time to achieve this and so may not be interested to enter the market

4 Price Maker Being the single seller of the product the monopolist has full control over the pricing of the product On the other hand if there is a large number of buyers in the market so no single buyer exercises any significant influence over price determination Thus it is a sellerrsquos market So monopoly firm is a price maker

5 Price Discrimination Having considerable control over the market on account of being single seller with no entry of other firms the monopolist can exercise policy of price discrimination it means that the monopolist can sell different quantities of the same product to a consumer at different price or same quantity to different consumers at different prices by adjudging the standard of living of the consumer

6 Shape of Demand Curve Since a monopolist has full control over the price therefore he can sell more by lowering the price This makes the demand curve downward sloping

Subject Ac-12 290620 Topic- retirement Model sumThe Balance Sheet of Rohit Nisha and Sunil who are partners in a firm sharing profits according to their capitals as on 31st March 2014 was as under

Liabilities Amount Assets Amount (Rs) (` Rs)

Creditors 25000 Machinery 40000Bills Payable 13000 Building 90000General Reserve 22000 Debtors 30000Capital Less Provision for Rohit 60000 Bad debts 1000

29000 Nisha 40000 Stocks 23000 Sunil 40000 140000 Cash at Bank 18000

200000 200000

On the date of Balance Sheet Nisha retired from the firm and following adjustments were made(i) Building is appreciated by 20(ii) Provision for bad debts is increased to 5 on Debtors(iii) Machinery is depreciated by 10(iv) Goodwill of the firm is valued at Rs 56000 and the retiring partnerrsquos share is adjusted

(v) The capital of the new firm is fixed at Rs120000 Prepare Revaluation Account Capital Accounts of the partner and Balance Sheet of the new firm after Nisharsquos retirement Revaluation AccountDr Cr

Particulars Amount Particulars Amount (`Rs) (Rs`)

Provision for Bad debt Ac 500 Building Ac 18000Machinery Ac 4000Profit transferred toCapital Accounts (3 2 2)Rohit 5786Nisha 3857Sunil 3857

13500

18000 18000

Capital Account

Dr Cr

Particulars Rohit Nisha Sunil Particulars Rohit Nisha Sunil (Rs`) (Rs`) (`Rs) (Rs`) (Rs`) (Rs`)

Sunilrsquos Capital ac 9600 mdash 6400 Balance bd 60000 40000 40000Bank - 66143 - General Reserve 9428 6286 6286Balance cd 72000 mdash 48000 Revaluation (Profi 5786 3857 3857 Rohitrsquos Capital Ac mdash 9600 mdash

Sunilrsquos Capital Ac 6400 Bank 6386 - 4257

81600 66143 54400 81600 66143 54400

Balance Sheet as at 31st March 2014

Liabilities Amount Assets Amount (Rs`) (Rs`)

Creditors 25000 Building 108000Bank overdraft 37500 Machinery 36000

Bills Payable 13000 Debtors 30000Capital Less ProvisionRohit 72000 for Bad debts 1500 28500Sunil 48000 120000 Stock 23000

195500 195500

Working Notes (i) (a) Profit sharing ratio is 60000 40000 40000 ie = 3 2 2(b) Gaining Ratio Rohit = 35 ndash 37 = 2135 ndash 1535 = 635Sunil = 25-27 = 1435 ndash 1035 = 435= 635 435= 6 4 = 3 2(c) Nisha Share of Goodwill = Rs 56000 times 27 = Rs16000Share of Goodwill in the gaining ratio by the existing partner ieRohit = Rs16000 times 35 = Rs 9600Sunil = Rs 16000 times 25 = Rs 6400

The journal entry isRohitrsquos Capital Ac Dr 9600Sunilrsquos Capital Ac Dr 6400 To Nisharsquos Capital Ac 16000(Share of Goodwill divided into gaining ratio)

  • 1 Static Friction
  • The frictional force that acts between the surfaces when they are at rest with respect to each other is called Static Friction
    • Static Friction Examples
      • 2 Sliding Friction
        • Examples Of Sliding Friction
          • 3 Rolling Friction
            • Examples Of Rolling Friction
              • Objects and Reasons of the Forest Conservation Act
Page 28:  · Web viewSubject . Topic . Summary . Execution . English 1 . Sounds of animals . Hens –cackle Horses –neigh Lions –roar Owls –hoots Snake –hiss. English 2 . Mother’s

L-2The Language of Chemistrybull Chemical Formula

Itrsquos a symbolic representation of a chemical substance eg ndash The formula of Sulphuric acid is H2SO4

2) Steps of writing Chemical Formula of a given substance-

1 Write the symbols of the constituent atoms or radicals side by side Keep the basic radical on LHS and acid radical on the RHS ( Na+Cl- )2 In case of a radical having more than one atom( compound radical) enclose the radical in a bracket eg (SO4-)3 Write the valencies of each radical on its right hand top4 If the valencies of the two radicals are divisible by a common factor then divide the valencies by the common factor5 Invert (criss-cross) the valency number ie write the valency of one atom below the second atom and vice versa 6 On interchanging if valency number is lsquoone the figure lsquoonersquo is never writtenFor Example- Compound -Calcium Nitrate1 Writing the symbols- Ca(NO3)2 Writing the valencies on their right hand top- Ca2(NO3)1

3 Valency numeral in simple ratio- Ca2(NO3)1

4 Criss-cross- Ca 2NO3 1

5 Writing the formula of the compound- Ca(NO3)2

Chemical formula

A The formula of a substance conveys the following information regarding a substance 1 The name of the substance (qualitative)2 The elements constituting the substance (qualitative)3 The number of various atoms present in a molecule of the substance (quantitative)4 Molecular weight of the substance and the relative weights of different elements present in it (qualitative)

Q What are the limitations of Chemical Formula

A The chemical formula suffers from the following limitations-I It fails to convey whether the elements in a molecule are present in the form of atoms or ionsFor example the formula KBr fails to tell us whether Potassium and Bromine are present in the form of ions II It does not tell anything about the binding force that holds atom in a molecule togetherIII It does not tell us about the arrangement of various atoms with respect to one another within the molecule

Q Examples of Some Chemicals with their Formula Chemical name and Common Name-

A Given in the class notesCommercial Studies

Joint Stock Company

Let us discuss about the demerits of Joint Stock CompanyDespite so many advantages it has got many disadvantages which are as follows

Difficulty in FormationDelay in Decision makingExcessive Government ControlLack of Secrecy

Company can be classified into several categories based on incorporation

QuestionExplain the demerits of Joint Stock CompanyAnswer) 1 Difficulty in Formation The legal requirements and formalities required to be completed are so many The cost involved is quite heavy It has to approach large number of people for its capital It cannot start its business unless certificate of incorporation has been obtained This is granted after a long time when all the formalities are completed

Chartered CompanyStatutory CompanyRegistered Company

Delay in Decision making In this form of organization decisions are not made by single individual All important decisions are taken by the Board of Directors Decision-making process is time-consuming So many opportunities may be costly because of delay in decision-making Promptness of decisions which is a common feature of sole trader ship and partnership is not found in a company

Excessive Government ControlA company and the management have to function well within the law and the provisions of Companies Act are quite elaborate and complex At every step it is necessary to comply with its provisions lest the company and the management should be penalized The penalties are quite heavy and in several cases officers in default can be punished with imprisonment This hampers the proper functioning of the company

Lack of Secrecy The management of companies remains in the hands of many persons Every important thing is discussed in the meetings of Board of Directors Hence secrets of the business cannot be maintained In case of sole proprietorship and partnership forms of organisation such secrecy is possible because a few persons are involved in the management

2 Define the following

Chartered Company- The crown in exercise of the royal prerogative has power to create a corporation by the grant of a charter to persons assenting to be incorporated Such companies or corporations are known as chartered companies Examples of this type of companies are Bank of England (1694) East India Company (1600) The powers and the nature of business of a chartered company are defined by the charter which incorporates it After the country attained independence these types of companies do not exist

in IndiaStatutory Company- A company may be incorporated by means of a special Act of the Parliament or any state legislature Such companies are called statutory companies Instances of statutory companies in India are Reserve Bank of India the Life Insurance Corporation of India the Food Corporation of India etc The provisions of the Companies Act 1956 apply to statutory companies except where the said provisions are inconsistent with the provisions of the Act creating them Statutory companies are mostly invested with compulsory powersRegistered companiesCompanies registered under the Companies Act 1956 or earlier Companies Acts are called registered companies Such companies come into existence when they are registered under the Companies Act and a certificate of incorporation is granted to them by the Registrar

Economics

Chapter-4Basic problems of Economy

Today let us discuss with the topic Production Possibility curve

QuestionExplain the concept of Production Possibility Curve with the help of diagram

Answer) Production Possibility curve is a locus of all possible combinations of two commodities which can be produced in a country with its given resources and technology

The above diagram shows that with the given resources and technology the economy can produce maximum either 5 thousand meters of cloth or 15 thousand quintals of wheat or any other combination of the two goods like B( 1 thousand meters of cloth and 14 thousand quintals of wheat C ( 2 thousands meters of cloth and 12 thousand quintals of wheat) etcProduction Possibility curve is also called production possibility boundary or frontier as it sets the maximum limit of what it is possible to produce with given resources

Geography

Rotationand Revolution

SUNrsquoS POSITION AND SEASONAL CHANGES EQUINOXES ndash SPRING AND AUTUMN

Q1 What is Spring EquinoxA1 On 21st March sunrays fall directly on the equator On that day

As the Equator divides the Earth into two equal halves the sun rays fall directly on the equator twice in a year Equinoxes means equal Spring EquinoxOn 21st March sunrays fall directly on the equator On that day the duration of day and night both are equal ( 12 hours day and 12 hours night) on every places located on equator This day is called as Spring EquinoxAutumn EquinoxOn 23rd September sunrays fall directly on the equator On that day the duration of day and night both are equal ( 12 hours day and 12 hours night) on every places located on equator This day is called as Autumn Equinox

SOLSTICES ndash SUMMER AND WINTERDue to inclination of the Earth on its axis and the apparent movement of the sun the sun rays fall directly on both tropics once in a year Solstice is a Latin word which mean ldquothe Sun standing stillrdquoSummer SolsticesAfter 21st March there is an apparent movement of the Sun to the north of the equator The apparent northward movement up to 21st June when the Sun appears overhead at the Tropic of Cancer (22frac12degN) The sun appears to stand still at this position and then moves southwards towards the equator This position of the Sun on 21st June is known as Summer Solstices On that day the duration of day and night both are equal ( 12 hours day and 12 hours night) on every places located on Tropic of Cancer (22frac12degN)Winter solstices The apparent southward movement of the Sun continues beyond the equator till 22nd

December On this day the Sun is overhead at the Tropic of Capricorn

the duration of day and night both are equal ( 12 hours day and 12 hours night) on every places located on equator This day is called as Spring Equinox

Q2 What do you mean by EquinoxA2 Equinoxes means equal It is use to explain the equal duration of day and night ( 12 hours day and 12 hours night) on the Earth

Q3 On which date the longest day in Tropic of CancerA3 21st June

Q4 What is the meaning of SolsticeA4 Solstice is a Latin word which mean ldquothe Sun standing stillrdquo

Q5 Which is the longest day in southern hemisphereA5 22nd December

Q6 On what date does the Arctic Circle experience the lsquoMidnight SunrsquoA6 On 21 June the Arctic Circle experiences the lsquoMidnight Sunrsquo

Q7 What is cause of Midnight Sun in NorwayA7 During the summer solstice (21 June) the North Pole is inclined towards the Sun Therefore the duration of sunlight or daytime increases from 12 hours at the Equator to 24 hours at the Arctic Circle and beyond Thatrsquos why The region beyond the Arctic Circle especially Norway is known as the Land of the Midnight Sun because there the Sun does not rise or set on 21 June

Q8 Match the column A with BA B

Summer Solstice 21st March

Autumn Equinox 23rd

September

Winter Solstice 21st June

(22frac12degS) This position of the Sun is referred to as the Winter Solstice because it marks the winter season in the Northern Hemisphere On that day the duration of day and night both are equal ( 12 hours day and 12 hours night) on every places located on Tropic of Capricorn (22frac12degS)SEASONS AND DURATION OF DAY AND NIGHT During the equinoxes all places on the Earth have 12 hours of day and 12 hours of night Due to the revolution of the Earth round the Sun on an inclined axis the duration of day and night varies according to seasons and the latitude of a placeDuring the summer solstice (21 June) the North Pole is inclined towards the Sun Therefore the duration of sunlight or daytime increases from 12 hours at the Equator to 24 hours at the Arctic Circle and beyondThe region beyond the Arctic Circle especially Norway is known as the Land of the Midnight Sun because there the Sun does not rise or set on 21 JuneAt the North Pole there will be six months of daylight The Sun will be seen always above the horizon at a low angle At 66degN 24 hours of sunlight can be seen only on 21 June Hammerfest in northern Norway is a place of tourist attraction for observing the phenomenon of the Midnight Sun This place has continuous daylight from 13 May to 29 July This place is easily accessible to tourists and has hotels and other facilities The view of the midnight Sun from here is enthrallingIn the Southern Hemisphere the duration of daylight decreases from 12 hours at the equator to 0 hours beyond the Antarctic Circle In the South Polar Region there is 24 hours of darkness The Sun is always below the horizon In the Southern Hemisphere which experiences winter the duration of night-time is longer than the duration of daylight

Spring Equinox 22nd

December

A8 A B

Summer Solstice 21st June

Autumn Equinox 23rd

September

Winter Solstice 22nd

December

Spring Equinox 21st March

During winter solstice (22 December) the South Pole is inclined towards the Sun The Southern Hemisphere experiences summer and the Northern Hemisphere has winter Therefore the duration of daylight or sunlight is greater in the Southern Hemisphere than in the Northern HemisphereThe duration of daylight increases from 12 hours at the equator to 24 hours beyond the Antarctic Circle The South Polar Region has 24 hours of sunlight for many days continuously At the South Pole there will be six months of sunlight The Sun will always be seen at a low angle above the horizon In the Northern Hemisphere the duration of daylight will decrease from 12 hours at the equator to 0 hours at the Arctic Circle There are 24 hours of darkness in the North Polar region The duration of night is greater than the duration of daylight as one move northwards from the Equator It is evident from the above table that the duration of daylight is 12 hours throughout the year at the equator only As one moves away from the equator the seasonal variations in the duration of daylight increase The seasonal variations in the duration of daylight are maximum at the Polar Regions

Subject Eng Literature (The Merchant of Venice ndash William Shakespeare)Topic Act II Scene 7 Lines 36 to 80 (End of scene ) [Students should read the original play and also the paraphrase provided]

Summary Questions amp AnswersThe Prince then examines the inscription on the silver casket which says ldquoWho chooseth me shall get as much as he deservesrdquo The Prince says that he deserves Portia more than anybody else because of his high rank his noble birth and his great wealth and power But then he argues that silver is ten times

(1) (Act II Sc 7 L 39-47)

From the four corners of the earth they come

To kiss this shrine this mortal breathing saint

The Hyrcanian deserts and the vasty wildsOf wide Arabia are as through-fares now

inferior to gold and therefore he cannot believe that the portrait of such a beautiful lady as Portia can be contained in the silver casket He decides to see the inscription on the golden casket before making his decision

The Prince goes to examine the inscription on the golden casket which says ldquoWho chooseth me shall get what many men desirerdquo The Prince believes that the whole world desires to possess Portia otherwise so many suitors would not have come from all corners of the world for winning Portia Some of them have come from the distant lands of Persia and Arabia The deserts of Persia (Hyrcanian deserts) and the boundless desolate lands of Arabia have been crossed by the Princes seeking the hand of Portia He contrasts this casket containing Portiarsquos portrait with the old English gold coin bearing the image of the archangel (angel of the highest rank) He goes on to remark that while the figure of the archangel is engraved (Insculped) upon the English coin the picture of Portia who is beautiful as an angel lies hidden inside one of the caskets namely the Golden Casket (Golden Bed)

On the basis of his assessment of the inscription on the golden casket the Prince decides to choose the golden casket He asks for the key and opens the golden casket only to find therein an empty human skull holding a roll of

For princes to come view fair PortiaThe watery kingdom whose ambitious headSpets in the face of heaven is no barTo stop the foreign spirits but they comeAs orsquoer a brook to see fair Portia

(i) Explain the occasion for the above mentioned speech

These are the comments of the Prince of Morocco after he reads the inscription on the golden casket His mental process is revealed to us in these words We find him debating within himself as to which casket he should choose

(ii) What light does the above speech throw on the personality of Prince of Morocco

From the above mentioned speech we come to know that the Prince of Morocco is keen to marry Portia He is the type of person who is easily taken away by outward appearance He is in love with Portia because of her beauty

(iii) What information can you gather about Portia from the above mentioned lines

The given speech shows that Portia is a very beautiful lady She must be possessed of good qualities because many suitors come to her place from all over the world with a desire to get married to her The Prince of Morocco is so impressed by her beauty that he calls her a saint According to him the whole world is desirous of having her

(iv) Elucidate the significance of the first two lines

In these lines the Prince of Morocco pays a compliment to Portia These lines show his admiration for her He says that people come from all parts of the world to see fair Portia

(v) Explain the meaning of the last four lines of the

passage

In these lines the Prince of Morocco says that even the vast oceans which throw a challenge at the sky are unable to prevent men from coming to Portiarsquos place to have a glimpse of her These lines are also a tribute to Portiarsquos beauty and good qualities Many men voyage across the ocean treating it as a mere stream to see the beautiful Portia

paper in which is written that whoever happens to be guided by the glitter of things is invariably deceived

On reading the scroll the Prince says that he is too sad at heart to speak a more formal farewell and leaves with his followers amidst a sound of trumpets

After the Prince of Morocco leaves Portia remarks that the Prince is a gentle fellow but she is rid of him May all persons of his nature make a similar choice

IMPORTANT PASSAGES EXPLAINED

(Act II Sc 7 L 39-43)From the four corners of the earth they come

To kiss this shrine this mortal breathing saintThe Hyrcanian deserts and the vasty wildsOf wide Arabia are as through-fares nowFor princes to come view fair Portia

Context

This passage occurs in Act II Scene 7 in The Merchant of Venice This is part of the speech made by the Prince of Morocco

(2)

(Act II Sc 7 L 48-53)

MOROCCO One of these three contains her heavenly pictureIst like that lead contains her

Twere damnation To think so base a thought it were too grossTo rib her cerecloth in the obscure graveOr shall I think in silver shes immurdBeing ten times undervalued to tried gold

(i) What meaning does the Prince of Morocco find out of the inscription of the golden casket What have Belmont and Portiarsquos house been called and why

The inscription on the golden casket is ldquoWho chooseth me shall gain what many men desirerdquo The Prince finds out that it means that the chooser of the golden casket will get Portia because many men desire her In fact the entire world desires her Because of the coming of many suitors to Belmont from different countries in order to win Portiarsquos hand Belmont has become a centre of pilgrimage and her house is the shrine where saintly Portia is installed

(ii) What does the Prince of Morocco do before making the final choice of the casket Which is the correct casket and who will win Portiarsquos hand

The Prince of Morocco surveys and analyses the inscriptions on the casket of lead silver and gold Before making the final choice like a very systematic and methodical person he once again considers the claims of the caskets The casket containing Portiarsquos picture is the correct casket and the person choosing it will win Portiarsquos hand

Explanation

While praising Portia the Prince of Morocco conceives Portia as a goddess whose image is placed inside one of the caskets Many suitors are coming from far and wide the north and the south the east and the west (Four corners) in order to try their luck Some of them have come from the distant land of Persia and Arabia The deserts of Persia (Hyrcanian deserts) and the boundless desolate lands of Arabia have been crossed by the Princes seeking the hand of Portia All this shows that Portia is indeed the most beautiful lady of the world

(iii) What does the Prince of Morocco say in his estimation while examining the motto on the silver casket What does he find in the golden casket

While examining the motto on the silver casket which says ldquoWho chooseth me shall get as much as he deservesrdquo Morocco says that in his own estimation he surely deserves Portia in all respects ndash rank birth wealth etc

He chooses the golden casket When he opens it he finds an empty human skull holding a scroll in which it is written that those who are attracted by the glittering outside of things are always deceived as Morocco has been deceived

(iv) What kind of nature does the Prince of Morocco have

The Prince of Morocco has a simple nature who does not look deeply into the inner meaning of things but is dazzled by the outward appearance of gold He is inclined to over-estimate his own value and does not realize that it is a duty to ldquogive and hazardrdquo To say that he will not hazard for lead shows that he misreads the true meaning of the inscription which is that he should be prepared to ldquohazard all he hathrdquo for Portia So his feeling is only one of fascination and romantic attraction

(v) Do you think that the lottery of the caskets is not a matter that will be determined by chance

In fact the lottery of the casket is not a matter that will be determined by mere chance but that it is a true test of character and of sincerity which is amply proved not only by Moroccorsquos choice but also by the arguments which he uses to help him in his choice

(Act II Sc 7 L 55-59)

They have in England

A coin that bears the figure of an angelStamped in gold but thats insculpd uponBut here an angel in a golden bedLies all within

Context

(3)

(Act II Sc 7 L 63-77)A carrion Death within whose empty eye

There is a written scroll Ill read the writing

All that glisters is not goldOften have you heard that toldMany a man his life hath soldBut my outside to beholdGilded tombs do worms infoldHad you been as wise as boldYoung in limbs in judgment oldYour answer had not been inscrolld

This passage occurs in Act II Scene 7 in The Merchant of Venice This is part of the speech made by the Prince of Morocco

Explanation

In this passage the Prince of Morocco bestows high praise on Portia whose hand he is seeking He contrasts this casket containing Portiarsquos portrait with the old English gold coin bearing the image of the archangel (angel of the highest rank) He goes on to remark that while the figure of the archangel is engraved (Insculped) upon the English coin the picture of Portia who is beautiful as an angel lies hidden inside one of the caskets namely the Golden Casket (Golden Bed) In the day of Elizabeth silver was ten times inferior in value to gold Therefore the Prince of Morocco believing that Portiarsquos portrait is contained in the Golden Casket decides to choose the Golden Casket

Fare you well your suit is coldCold indeed and labour lostThen farewell heat and welcome frostmdashPortia adieu I have too grievd a heartTo take a tedious leave Thus losers part

(i) What reward does the Prince of Morocco get after making a wrong choice of the Casket How does he feel

After making the wrong choice in selecting the casket of gold the Prince of Morocco as a reward earns a rebuke in the form of a scroll tucked in the empty eye-socket of a skull kept in the casket of gold The Prince is shocked and disappointed He becomes all the more sad and dejected when he reads the scroll which points to his foolishness in being misled by the appearance and outward show as indicative of its worth

(ii) How does the Prince respond after reading the scroll

After reading the scroll the Prince though upset accepts the result with good grace and decorum befitting a royal suitor and true sportsman He says that his love-suit is really cold otherwise he would have chosen correctly but now his efforts have been in vain So he bids farewell to Portia to the warmth and enthusiasm of love and welcomes the cold and bitterness of dejection and misery of life which lies ahead

(iii) What request does he make to Portia and why

After being failure in his mission he requests Portia to give him permission to leave at once because he is too sad to undergo the tediousness of a formal leave-taking He tells that it is the manner in which defeated persons part unceremoniously

(iv) Explain the following lines

ldquoAll that glisters is not goldOften have you heard that toldMany a man his life hath soldBut my outside to beholdGilded tombs do worms infoldrdquo

Mere glitter does not make a metal to be gold Man has often been warned against appearance but it has been of no use Many people have sacrificed their lives only to seek the outer appearance of gold Worms are found inside the gilded

monuments

Class XSubject Topic Summary Execution

Hindi 2ndlang

नया रासता भाग 6 मायाराम 0ी घर म धनी मल 0ी और उनी बटी सरिरता ी ही चचा बनी रहती थी अमिमत ो इसम ोई रलिच ना थी वह धनी घर ी लडी स शादी र सवय ो बचना नही चाहता था उसा भी सवाणिभमान ह ईशवर ी पा

स उस पास पस ी ोई मी नही थी अभी उसन फकटरी ही लगाई थी उसी समझ बाहर था कि उस घर वालो ा झाव पस ी तरफ कयो

ह उसन मा स सवाल किया कि मा तम सरिरता स मरी शादी कयो रना चाहती हो मा न उस समझाया कि वह दखन म बरी नही ह और किफर खानदान अचछा

ह वह ए शल गरहणी रप म घर सभाल सगी अमिमत न मा ो इस बात ा एहसास राया कि मीन सबध लिलए मना रन पर उस दिदल

पर कया बीती होगी मा और अमिमत ी लडी बार म ाफी बात हईमा ा झाव सरिरता ी तरफ था कयोकि वह घर पर अचछा दह0 लर आ रही

थी अमिमत न अपनी मौसी ी बरी हालत बार म बताया कि किस तरह वह बड घर ी खानदानी बटी लाई थी और आ0 उसी हालत कितनी खराब ह लाई थी बहकलब 0ाती ह और बचचो ो भी नही दखती ह बात चल ही रही

थी कि तभी ए ार बाहर आर री धनी मल0ी घर अदर आए और पीछ स डराइवर फल ी ए टोरी लर आया अदर आए और पीछ स

डराइवर ए टोरी फल ी लर आया अमिमत ो फल ी पटी बरी लग रही थी अमिमत न पछ लिलया यह फल कयो ल आए ह प इन सब ी कया

0ररत थी उनो न 0वाब दिदया कि 4 पटी शमीर स मगाए थ अमिमत ो या सनर करोध आ गया तभी उस किपता 0ी आ गए उन आत ही अमिमत उठर बाहर चला गया वहा वहा मा पास आर बठ गया और बोला

अभी रिरशता तय नही हआ और धनी मल 0ी धनी मल 0ी फल ी पटी लर चलआय मा न समझाया कि 0ब सबध 0ड 0ाता ह तो खाली हाथ नही

आत अमिमत न मा स हा कि तम सबन सरिरता ो इस घर म लान ी ठान रखी ह धनीमल 0ी उस दिदन सरिरता ो दखन ी तारीख तय रन आय थ

Commercial Studies

Banking Nowadays Bank provide easy and quick services through internet facilities methods of Banking is called internet bankingIn order to save the time and money involved in visiting Bank branches people increasingly prefer to have internet banking

There are different modes of doing internet banking or transferring money through online They areReal Time Gross Settlement (RTGS)National Electronic Fund Transfers (NEFT)

1

Question

1) Explain the term RTGS Write the features of RTGS

Answer)The acronym RTGS stands for Real Time Gross Settlement which may be defined as the continuous real time settlement of funds transfer individually on and order by order basis without netting lsquoReal timersquo may be defined as the processing of instructions at the time they are received rather than at some letter time lsquoGross settlementrsquo may be defined as the settlement of transfer instructions which occurs

individually

Features of RTGS1It is the continuous settlement of

funds transfer individually on an order by order basis

2RTGS facility is provided only by CBS core banking solution enabled Bank branches

3Amount charged from the customer for RTGS transactions vary from bank to bank

2) Explain the term NEFT Write the features of NEFT

Answer) National electronic funds transfer may be defined as a nationwide system that facilitates individuals Farms and copper operates to electronically transfer funds from any bank branch to any individual farm or corporate having an account with any other bank branch in the country

Features of NEFT2 Transfer can be made 7 times on

weekdays and 6 times on Saturday

3 NEFT cannot be used to receive foreign remittances

4 NEFT transaction takes place in batches

5 A bank branch must be NEFT enabled to become a part of NEFT fund transfer network

6 There is no maximum or minimum amount that can be transferred through NEFT when one bank has a bank account

English Language

CompositionEssay

A composition is an art of creating a piece of writing on any topic or subject It is the writing correctly beautifully and clearly in order to make some interesting reading Structure of the composition

Introduction ( you lay the foundation for your composition)

Body (it constitutes the main part of the essay)

Conclusion (final statement that leaves a lasting impression)

Kinds of essays1 The Narrative essay2 The descriptive essay3 The reflective essay4 The argumentative essay

Write a composition on any one of the following topics (350- 400 words)

1 Friendship Or2 The first day of your school

Subject Eng Literature (The Merchant of Venice ndash William Shakespeare)Topic Act V Scene 1 Lines 127 to 158 (Nerissa helliphellip The clerk will nersquoer wear hair onrsquos face that had it) [Students should read the original play and also the paraphrase given in the school prescribed textbook]

Summary Revision Questions o Soon thereafter Bassanio Gratiano

and Antonio arrive

o Bassanio tells Portia that he is feeling as if it is morning because of the presence of Portia who is shining like the sun When Antonio is introduced by Bassanio to Portia she tells Bassanio that he should be grateful to Antonio who took so much trouble on his account even to the extent of risking his life

o Nerissa starts quarrelling with Gratiano and demands that he show her the ring she had presented to him and which she had warned him not to lose She suspects that Gratiano must have presented the ring to some young woman and not to the lawyerrsquos clerk as he repeatedly says and assures

Answer the following questions to check your preparation of Act IV Scenes 1 and 2

You must attempt only after you have completed your preparation of Act IV The answers must be in complete sentences using textual evidence (with citation) when necessary

[It would be in your own interest to attempt the above questions honestly totally refraining from consulting your textbook or your notes during answering After completion you should correct the paper yourself consulting the textbooknotes etc and award marks as specified Please let me know the marks you scored through WhatsApp in the group or to my personal WhatsApp]

Act IV Scene 1 (each question carries 2 marks)

1 What did the Duke try to do for Antonio

2 Why does Shylock refuse to show mercy How does he justify his stance

3 Why does Antonio say he is ready to die 4 What information is contained in Bellariorsquos letter

5 Why does Portia (as Balthazar) assert that Shylock must show mercy How does he respond

6 What offers are made to Shylock to get him to spare Antonio How are they received

7 What does Antoniorsquos speech as he faces the prospect of Shylockrsquos knife tell you about his character

8 How do Bassanio and Gratiano react to the looming prospect of Antoniorsquos demise

9 How does Portia (as Balthazar) use the law to turn the tables on Shylock

10 What does the Duke decree should happen to Shylock Why What happens to Shylockrsquos estate

11 What does Portia ask Bassanio as payment for her ldquoservicesrdquo What is his initial response What makes him change his mind

Act IV Scene 2 (each question carries 1frac12 marks)

1 What does Gratiano bring to Portia (Balthazar)

2 What does Nerissa plan on getting from Gratiano What does Portiarsquos comment suggest about men

ECO-10 280620 Topic-Supply AnalysisSHIFTING OF SUPPLY

But if there is change in factors other than the price of the commodity then either more is supplied at the same price or less supplied at the same price In such cases the price of the commodity remains constant but there is a change in other factors like change in the price of inputs change in technology of production change in price of other related goods change in taxation policy of the government etc For example there is an improvement in the technology of production of the commodity in question It leads to decrease in per unit of cost production of the commodity The firm is willing to sell more quantity of the commodity at the same price So the supply other commodity increases at the same price This increase in supply is shown by rightward shift of supply curve On the other hand if the firm uses inferior technology of production the cost of production per unit of the commodity increases The firm is willing to sell less quantity at the same price So the supply of the commodity decreases at the same price This decrease in supply is shown by leftward shift of the supply curve The above cases of increase and decrease in supply can be shown with the help of the following figures

Y INCREASE OF SUPPLY Price (Rs) s

P A s1

B

s

X` O s1 X

q q1

Y` Quantity demanded (in units)

Y DECREASE IN SUPPLY s2

s

price (Rs)

C

p A

s2

s

X` o X

q2 q

Y` Quantity demanded ( in units)

Main factors causing increase in supply or rightward shift of supply Curve(i) Fall in the price of other related goods

(ii) Fall in the price of inputsfactors(iii) Use of better technology in production(iv) Decrease in the rate of excise duty by government(v) If the objective of producer changes from profit maximization to salesMaximization

Main factors causing decrease in supply or leftward shift of supply curve(i) Increase in the price of other related goods(ii) Rise in the price of inputsfactors(iii) Use of inferior technology in production(iv) Increase in the rate of excise duty by the government(v) If the objective

Subject - Biology Topic ndash Chapter mdash6 PhotosynthesisSummary Execution

Today we will know about photosynthesis and its stages

Q1 What do you mean by photosynthesis The process by which living plants containing chlorophyll produce food

substances from carbon-di- oxide and water by using light energy Sunlight

6CO2 +12 H2O----------------------- C6 H12O6 + 6H2O + 6O2

Chlorophyll

Q2 What are the importance of photosynthesis I) Food for all Green plants trap solar energy by photosynthesis

process and supply food and energy for all living organisms either directly or indirectly

Ii) Oxygen to breathe in by product of photosynthesis is oxygen which is essential for all living organisms respiration

Q3 Write about two main phases of photosynthesis A Light dependent phase This phase occur in grana of chloroplast I) The chlorophyll on exposure to light energy becomes activated by

absorbing photons Ii) The absorbed energy is used in splitting the water molecules (H2O)

into its two components (H+ and OH- ) and releasing electron s 2H2O------------------------- 4H+ + 4e- +O2

Energy of 4 photons This reaction is known as photolysis

End products are H+ and oxygen water

B Light independent (Dark ) phase The reactions in this phase require no light energy

Here CO2 combine with H+ and produce glucose

Class XI

Subject Topic Summary ExecutionEVS Chapter-4 Legal

regimes for sustainable development

Environmental legislationEnvironmental legislation is the collection of laws and regulations pertaining to air quality water quality the wilderness endangered wildlife and other environmental factors The act ensures that matters important to the environment are thoroughly

Learn -The Forest (Conservation) Act 1980

considered in any decisions made by federal agencies

The Forest (Conservation) Act 1980 The Forest (Conservation) Act 1980 an Act of the Parliament of India to provide for the conservation of forests and for matters connected therewith or ancillary or incidental thereto It was further amended in 1988 This law extends to the whole of IndiaObjects and Reasons of the Forest Conservation Act

Deforestation causes ecological imbalance and leads to environmental deterioration Deforestation had been taking place on a large scale in the country and it had caused widespread concern The act seeks to check upon deforestation and de-reservation of forests

Subject Eng Literature (The Tempest ndash William Shakespeare) Topic Act II Scene 1 Lines 314 to 329 (End of scene)

[Students should read the original play and also the paraphrase given in the school prescribed textbook]Summary Questions amp Answers

Conspiracy of Antonio and Sebastian (Contd)

o As they approach Ariel appears again and wakes up Gonzalo by singing a tune in his ear Alonso also wakes up and they see both Sebastian and Antonio with drawn swords On being caught off guard they make up a story saying that they had heard a bellowing of bulls or lions

o They then moved to another part of the island

o Ariel at once rushes to Prospero to inform him of this development

SUMMING-UP of ACT-2 SCENE-1

(i) Among the survivors Ferdinand is separated from the rest which results in the disconsolate grief of Alonso as he took him for dead

(ii) The villainy of Antonio is confirmed

(iii) The supremacy of Prosperorsquos magic which resulted in the failure of the human conspiracy

(1)

(Act II Sc 1 L 311-325)SEBASTIAN Whiles we stood here securing your repose

Even now we heard a hollow burst of bellowing Like bulls or rather lions Didt not wake youIt struck mine ear most terribly

ALONSO I heard nothingANTONIO O rsquotwas a din to fright a monsters ear

To make an earthquake Sure it was the roarOf a whole herd of lions

ALONSO Heard you this GonzaloGONZALO Upon mine honour sir I heard a humming

And that a strange one too which did awake meI shaked you sir and cried As mine eyes opened I saw their weapons drawn There was a noiseThats verily rsquoTis best we stand upon our guardOr that we quit this place Lets draw our weapons

(i) Why has Prospero sent Ariel to Gonzalo and Alonso What does Ariel do to awaken Gonzalo

Prospero has already come to know by his magic powers the danger which threatens Gonzalo who had been Prosperorsquos friend and so he sent Ariel to preserve the lives of both Gonzalo and Alonso Prospero does not want that his scheme should remain unfulfilled Ariel begins to sing a song in Gonzalorsquos ears to awaken him(ii) Who are ready to carry out their plan Who takes steps to stop them Why does Gonzalo feel surprised after being awakened

Sebastian and Antonio are ready to carry out their plans They are standing with their swords drawn to kill Alonso and

(iv) We see two sets of contrasting characters Gonzalo-Adrian against Antonio-Sebastian

(v) The grief that works in Alonso can be perceived to his repentance for his association in Antoniorsquos crime against Prospero

Gonzalo Ariel takes steps to stop them from carrying out their nefarious scheme When Gonzalo is awakened by the song sung by Ariel into his ears he (Gonzalo) feels surprised because he sees Sebastian and Antonio standing with their swords drawn(iii) What reason do Sebastian and Antonio tell of drawing their swords when they are suspected by Alonso and Gonzalo

When Sebastian and Antonio are seen with their swords drawn they are looked with suspicion by Gonzalo and Alonso At first Sebastian tells them that as they stood here to guard them during their sleep they heard only a little before a sudden loud noise very much like the roaring of bulls or more probably that of lions Then Antonio follows him saying that this was a noise so terrible as to frighten even a monsterrsquos ears and this noise could even have shaken the earth and it was surely like the roaring of a multitude of lions Then seeing the danger they have drawn their swords Perhaps after hearing the terrible noise they (Gonzalo and Alonso) woke up from their sound sleep

(iv) What does Gonzalo tell Alonso about the strange noise What did he see on opening his eyes Gonzalo tells Alonso that he did not hear the sound of roaring but he heard a humming sound which was strange and which woke him up After waking up he gave him (Alonso) a shaking and a loud cry On opening his eyes he saw these two gentlemen standing with their swords drawn(v) What does Gonzalo suggest

Gonzalo suggests that there was a noise indeed and of that he has no doubt at all and suggests that the best course for them would be to remain alert and vigilant against any possible danger to their lives or to leave this place and move to some other part of the island

Class XIISubject Topic Summary Execution

Commerce

Chapter- Management

Today we will discuss about LEVELS OF MANAGEMENT

Levels of management is a series or chain of managerial positions from top to bottom It helps individuals to know their authority responsibilities and superior-subordinate relations among themselves There are mainly three levels of Management TOP LEVEL MANAGEMENTMIDDLE LEVEL MANAGEMENTLOWER LEVEL MANAGEMENT

Top level managementIt consists of members at the highest level in the management hierarchy This level includes Board Of Directors Chief Executive Managing Directors Chairman President Vice President

Rolefunctions of the top levelmanagement1To analyse evaluate and deal

with theexternal environment2 To determine the objectives and

policies of the business3 To strive for welfare and survival

of business

4 To create an organisational Framework consisting of authority responsibility relationship

Middle level management Congress of members or groups who are concerned with implementation of the policies let down by the top managementThis level includes head of the department such as finance manager marketing manager branch and regional managers departmental and divisional heads plant superintendent etc

Role of functions of the middle level management

1 To interpret the policies framed by top management

2 To assign duties and responsibilities to lower level managers

3 To select and appoint employees for middle and supervisory level and evaluate their performance

4 To co-operate with other departments for smooth functioning

Operational or supervisory level managementIt refers to the group are members who are concerned with execution of the work They are also known as fast line managers This level includes supervisor 4 men Section Officer clerk Inspector etc

Role of functions of the lower level management1 To plan and execute day-to-

day operations2 To supervise and control the workers3 To arrange materials and

tools to start the process and make arrangements for training

4 Today present workers grievance and suggestions before the management and

ensure safe and proper working conditions in the factory

Business Studies

Staff Appraisal Chapter- 10 Today let us start with a new chapter

Staff Appraisal

Meaning of Performance Appraisal

Performance Appraisal is the systematic evaluation of the performance of employees and to understand the abilities of a person for further growth and developmentThe supervisors measure the pay of employees and compare it with targets and plansThe supervisor analyses the factors behind work performances of employeesThe employers are in position to guide the employees for a better performance

Objectives of Performance Appraisal

Following are the objectives of Performance Appraisal

To maintain records in order to determine compensation packages wage structure salaries raises etc

To identify the strengths and weaknesses of employees to place right men on right job

To maintain and assess the potential present in a person for further growth and development

To provide a feedback to employees regarding their performance and related status

To provide a feedback to employees regarding their performance and related status

Importance of Performance Appraisal

Performance appraisal provides important and useful information for the assessment of employees skill

knowledge ability and overall job performance The following are the points which indicate the importance of performance appraisal in an organization

1 Performance appraisal helps supervisors to assess the work performance of their subordinates

2 Performance appraisal helps to assess the training and development needs of employees

3 Performance appraisal provides grounds for employees to correct their mistakes and it also provides proper guidance and criticism for employees development4 Performance appraisal provides reward for better performance

5 Performance appraisal helps to improve the communication system of the organization

6 Performance appraisal evaluates whether human resource programs being implemented in the organization have been effective

7 Performance appraisal helps to prepare pay structure for each employee working in the organization

8 Performance appraisal helps to review the potentiality of employees so that their future capability is anticipated

Geography

DRIANAGE The SubarnarekhaThe Subarnarekha and the Brahmaniinterposed between the Ganga and the Mahanadi deltas drain an area of 19300 sq kmand 39033 sq km respectively The drainage basins of these streams are shared byJharkhand Odisha west Bengal and Chhattisgarh The Brahmani is known as southKoel in its upper reaches in Jharkhand

The NarmadaThe Narmada rises in the Amarkantak hills of MadhyaPradesh It flows towards the West in a rift valleyformed due to a geological fault The total length of it is 1300 km All the tributaries of the

Q1 Name the two westward flowing rivers in the peninsular plateauA1 Narmada and Tapi are the only westward flowing rivers of the peninsular plateau

Q2 Differentiate between east-flowing rivers and west-flowing riversA2

East-flowing rivers

West-flowing rivers

Narmada are very short inlength Most of its tributaries join the main streamright anglesThe Narmada basin covers parts of Madhya Pradesh and Gujarat

The Tapi The Tapi rises in the Satpura ranges in the Betul listrictof Madhya Pradesh It flows in a rift valley parallel tothe Narmada but it is much shorter in length It coversparts of Madhya Pradesh Gujarat and MaharashtraThe length is about 724 km

The Sabarmati and the MahiThe Sabarmati rises in the Aravali hills and flows south-south-westwards for a distance of 300 kilometres to the Arabian Sea The Sabarmatibasin extends over an area of 21674 sq km in Rajasthan and Gujarat The Mahi rises inthe east of Udaipur and drains an area of 34842 sq km lying in Madhya PradeshRajasthan and Gujarat It flows south-westwards for a distance of 533 km before it fallsinto the Gulf of Khambhat

The ChambalThe Chambal rises near Mhow in the Vindhya Range and flows towards the northgenerally in a gorge upto Kota Below Kota it turns to the north-east direction and afterreaching Pinahat it turns to the east and runs nearly parallel to the Yamuna beforejoining it in the southern part of the Etawah district in Uttar PradeshMajor Rivers of India with their basin area (Sqkm)

Himalayan System Indus 321290Ganga 861404

Brahmaputra 187110Indus System

Jhelum 34775Beas 20303

Ganga System Yamuna 366223Ghaghra 127950

Peninsular RiversNarmada 98796

Tapi 65145Mahanadi 141600

Subarnarekha 19300Sabarmati 21674

Mahi 34842Godavari 312812

Godavari Krishna Kaveri Mahanadi are the east-flowing rivers

Narmada Tapi west-flowing rivers

They fall into the Bay of Bengal

They fall into Arabian Sea

These rivers form big deltas

These rivers form comparativelysmall deltas

Catchment areas of these rivers are larger

Catchment areas of these rivers are smaller

Krishna 2589488Cauveri 87900

Subject ndashBiology Topic ndashChapter -5 Inheritance amp Variations Summary ExecutionToday we will discussabout linkage and its classification

LINKAGE The tendency of the genes located on the same chromosome to stay together is

hereditary transmission Linked genes the genes responsible for this Genes that exhibit the process of linkage locates in the same chromosome The distance between the linked genes in a chromosome determines the strength

of linkage i e genes that are located close to each other show stronger linkage than that are located far from each other

COMPLETE LINKAGE It is the type of linkage showed by the genes that are closely located or are tightly

linked with each other as they have no chance of separatingby crossing over These genes are always transmitted together to the same gamete and the same

offspring In such condition only parental or non cross over type of gametes are formedINCOMPLETE KINKAGE It is type of linkage showed by the genes that are distantly located orare loosely

linked with each other because they have chance of separating by crossing over

SIGNIFICANCE i) It helps in holding the parental character togetherii) It checks the appearance of new recombination and helps in bringing the

hybrid population which resembles the original parents iii) Linked genes dilute the effects of undesirable traits

Subject Eng Literature (The Tempest ndash William Shakespeare) Topic Essay Questions (EQ-3)Question No 3

Give a character sketch of CalibanAnswer

The character of Caliban has been wonderfully conceived by Shakespeare as the manifestation of all that is gross and earthy ndash a sort of creature of the earth as Ariel is a sort of creature of the air

Calibanrsquos Physical Appearanceo Caliban is lsquofreckledrsquo a lsquomisshapen knaversquo not honoured with human shape

o Prospero calls him lsquothou tortoisersquo (Act I Sc 2 Line 317) Trinculo stumbling upon him describes him as ldquoA strange fish hellip Legged like a man And his fins like armsrdquo He ldquosmells like a fishrdquo (Act II Sc 2 Line 25)

o Prospero also calls him a ldquobeastrdquo (Act IV Sc 1 Line 140) and ldquoThis misshapen knaverdquo (Act V Sc 1 Line 268)

o Further it appears that in addition to his physical deformity his spiritual inferiority is also suggested by Prosperorsquos claim that his birth resulted from the union between his mother the witch Sycorax and the devil

Calibanrsquos ParentageWhen the play opens Caliban is twenty four years of age having been born on the island twelve years before the coming of Prospero His mother was the foul witch Sycorax who was banished from Algiers for ldquomischiefs manifold and sorceries terrible to enter human hearingrdquo (Act I Sc 2 Line 264) and the father was the Devil himself Thus

Caliban is a monster of evil and brute nature ugly deformed and stinking

Calibanrsquos Savage and Malignant Natureo Caliban is entirely a creature of the earth ndash gross brutal and savage He regards himself as the rightful possessor

of the island and Prospero as a usurper

o In his young age he was on good terms with Prospero He had consented to be received by Prospero at his house and to be educated by him He has learnt human language only to curse his master whom he abhors

o His beastly nature soon breaks out and ends in a vicious attack on Miranda This opens the eye of Prospero who becomes severe to him and enforces his service by threats and violence

o Prospero uses him to make dams for fish to fetch firewood scraper trenches wash dishes and keep his cell clean

Calibanrsquos Hatred for ProsperoA profound hatred for Prospero has taken hold of Caliban It springs from a sense of his being dispossessed and ill-treated He would kill Prospero if he could but he knows the power of Prosperorsquos lsquobookrsquo Hence he transfers his allegiance to Stephano who seems like a god to him He also incites the two drunken associates to batter the skull of Prospero when he sleeps in the afternoon

Caliban Shows Considerable Intelligenceo He has learnt Prosperorsquos language

ldquoYou taught me language and my profit onrsquot (Act II Sc 2 Lines 86-89)Is I know how to curserdquo

o He is well aware of the futility of arguing with one who has more power than he has

ldquoI must obey his art is such power (Act I Sc 2 Lines 373-376)It would control my damrsquos god SetebosAnd make a vassal of himrdquo

o He realizes the importance of Prosperorsquos books

ldquoRemember (Act III Sc 2 Lines 89-92)First to possess his books for without themHersquos but a sot as I am nor hath notOne spirit to commandrdquo

o He knows the value of stealth when attacking the enemy

ldquoPray you tread softly that the blind mole may not (Act IV Sc 1 Lines 194-195)Hear a foot fall we now are near his cellrdquo

o Caliban has a better set of values than Stephano and Trinculo They are distracted from their plan by their greed for Prosperorsquos rich garments Only Caliban realizes that such a finery is unimportant

ldquoLeave it alone thou fool it is but trashrdquo (Act IV Sc 1 Lines 224)

Caliban is not a good judge of characterCaliban is not a good judge of character He decides for example that Stephano is a god because he dispenses lsquocelestial liquorrsquo (Act II Sc 2 Line 115) but then it must be remembered that he has only known his mother Sycorax Prospero Miranda and the spirits that torture him However he quickly discovers his error of judgementrdquo

ldquoWhat a thrice-double ass (Act V Sc 1 Lines 295-297)Was I to take this drunkard for a godAnd worship this dull foolrdquo

Calibanrsquos Imaginative NatureIf Caliban is sub-human in what has been said above he is human in the respect of the poetic side of his character He listens to music with rapture He tells of the beautiful dreams in which heaven rains treasures upon him and which upon waking he yearns to renew One of the most poetic passages in whole play is Calibanrsquos description of the island

to Stephano and Trinculo

ldquoBe not afeard The isle is full of noises (Act III Sc 2 Lines 135-143)Sounds and sweet airs that give delight and hurt notSometimes a thousand twangling instrumentsWill hum about mine ears and sometime voicesThat if I then had waked after long sleepWill make me sleep again and then in dreamingThe clouds methought would open and show richesReady to drop upon me that when I wakedI cried to dream againrdquo

Caliban - Less Ignoble Than Some OthersCalibanrsquos motive for murder is less dishonourable than that of Antonio and Sebastian They plan to kill Alonso to gain his power and wealth Caliban merely wants revenge and the return of lsquohisrsquo island

Conclusiono Calibanrsquos character is not portrayed very clearly in the play and hence we cannot decide whether he is a poor

savage being grossly maltreated by Prospero or whether he is evil and must therefore be kept in bondage or enslavement

o Caliban is contrasted with Ariel who is a spirit and thus swift and uninterested in physical activitieso Caliban is also contrasted with Prospero who is the all-powerful master of the island and of the destiny of all

those on the islando Caliban is also contrasted with civilized man showing him to be less evil than Antonio and Stephano and less

materialistic than Stephano and Trinculoo Caliban has suffered at the hands of Prospero and he has learnt to curse by listening to Prosperorsquos abuse He

certainly believes that Prospero has deprived him of his birthrighto Finally the character Caliban is thought to be one of Shakespearersquos masterpieces The complexity of the character

is reflected in the large volume of critical discussion that has grown around it

ECO ndash12 Topic-Forms of market

MonopolyMonopoly is a market structure in which there is a single seller there are no close substitutes for the commodity produced by the firm and there are barriers to entry Example Indian Railways which is operated under government of India Monopoly also implies absence of competitionFeatures of Monopoly Monopoly is characterized by1 Single Seller In monopoly there is only one firm producing the product The whole industry consists of this single firm Thus under monopoly there is no distinction between firm and industry Being the only firm there is significant control of the firm over supply and price Thus under monopoly buyers do not have the option of buying the commodity from any other seller They have to buy the product from the firm or they can go without the commodity This fact gives immense control to the monopolist over the market

2No Close Substitute There are no close substitutes of the product produced by the monopolist firm If there are close substitutes of the product in the market it implies presence of more than one firm and hence no monopoly In order to ensure a total of control over the market by the monopolist firm it is assumed that there are no close substitutes of the product

3 No Entry amp Exit Monopoly can only exist when there is strong barriers before a new firm to enter the market In fact once a monopoly firm starts producing the product no other firm can produce the same One reason for this is the ability of the

monopolist to produce the product at a lower cost than any new firm who thinks to enter the market If a new firm who knows that it cannot produce at a lower cost than the monopolist then that firm will never enter the market for fear of losing out in competition Similarly the monopolist who is operating for a long time may be enjoying reputation among its customers and is in a better position to use the situation in its own benefit A new firm has to take long time to achieve this and so may not be interested to enter the market

4 Price Maker Being the single seller of the product the monopolist has full control over the pricing of the product On the other hand if there is a large number of buyers in the market so no single buyer exercises any significant influence over price determination Thus it is a sellerrsquos market So monopoly firm is a price maker

5 Price Discrimination Having considerable control over the market on account of being single seller with no entry of other firms the monopolist can exercise policy of price discrimination it means that the monopolist can sell different quantities of the same product to a consumer at different price or same quantity to different consumers at different prices by adjudging the standard of living of the consumer

6 Shape of Demand Curve Since a monopolist has full control over the price therefore he can sell more by lowering the price This makes the demand curve downward sloping

Subject Ac-12 290620 Topic- retirement Model sumThe Balance Sheet of Rohit Nisha and Sunil who are partners in a firm sharing profits according to their capitals as on 31st March 2014 was as under

Liabilities Amount Assets Amount (Rs) (` Rs)

Creditors 25000 Machinery 40000Bills Payable 13000 Building 90000General Reserve 22000 Debtors 30000Capital Less Provision for Rohit 60000 Bad debts 1000

29000 Nisha 40000 Stocks 23000 Sunil 40000 140000 Cash at Bank 18000

200000 200000

On the date of Balance Sheet Nisha retired from the firm and following adjustments were made(i) Building is appreciated by 20(ii) Provision for bad debts is increased to 5 on Debtors(iii) Machinery is depreciated by 10(iv) Goodwill of the firm is valued at Rs 56000 and the retiring partnerrsquos share is adjusted

(v) The capital of the new firm is fixed at Rs120000 Prepare Revaluation Account Capital Accounts of the partner and Balance Sheet of the new firm after Nisharsquos retirement Revaluation AccountDr Cr

Particulars Amount Particulars Amount (`Rs) (Rs`)

Provision for Bad debt Ac 500 Building Ac 18000Machinery Ac 4000Profit transferred toCapital Accounts (3 2 2)Rohit 5786Nisha 3857Sunil 3857

13500

18000 18000

Capital Account

Dr Cr

Particulars Rohit Nisha Sunil Particulars Rohit Nisha Sunil (Rs`) (Rs`) (`Rs) (Rs`) (Rs`) (Rs`)

Sunilrsquos Capital ac 9600 mdash 6400 Balance bd 60000 40000 40000Bank - 66143 - General Reserve 9428 6286 6286Balance cd 72000 mdash 48000 Revaluation (Profi 5786 3857 3857 Rohitrsquos Capital Ac mdash 9600 mdash

Sunilrsquos Capital Ac 6400 Bank 6386 - 4257

81600 66143 54400 81600 66143 54400

Balance Sheet as at 31st March 2014

Liabilities Amount Assets Amount (Rs`) (Rs`)

Creditors 25000 Building 108000Bank overdraft 37500 Machinery 36000

Bills Payable 13000 Debtors 30000Capital Less ProvisionRohit 72000 for Bad debts 1500 28500Sunil 48000 120000 Stock 23000

195500 195500

Working Notes (i) (a) Profit sharing ratio is 60000 40000 40000 ie = 3 2 2(b) Gaining Ratio Rohit = 35 ndash 37 = 2135 ndash 1535 = 635Sunil = 25-27 = 1435 ndash 1035 = 435= 635 435= 6 4 = 3 2(c) Nisha Share of Goodwill = Rs 56000 times 27 = Rs16000Share of Goodwill in the gaining ratio by the existing partner ieRohit = Rs16000 times 35 = Rs 9600Sunil = Rs 16000 times 25 = Rs 6400

The journal entry isRohitrsquos Capital Ac Dr 9600Sunilrsquos Capital Ac Dr 6400 To Nisharsquos Capital Ac 16000(Share of Goodwill divided into gaining ratio)

  • 1 Static Friction
  • The frictional force that acts between the surfaces when they are at rest with respect to each other is called Static Friction
    • Static Friction Examples
      • 2 Sliding Friction
        • Examples Of Sliding Friction
          • 3 Rolling Friction
            • Examples Of Rolling Friction
              • Objects and Reasons of the Forest Conservation Act
Page 29:  · Web viewSubject . Topic . Summary . Execution . English 1 . Sounds of animals . Hens –cackle Horses –neigh Lions –roar Owls –hoots Snake –hiss. English 2 . Mother’s

Chartered CompanyStatutory CompanyRegistered Company

Delay in Decision making In this form of organization decisions are not made by single individual All important decisions are taken by the Board of Directors Decision-making process is time-consuming So many opportunities may be costly because of delay in decision-making Promptness of decisions which is a common feature of sole trader ship and partnership is not found in a company

Excessive Government ControlA company and the management have to function well within the law and the provisions of Companies Act are quite elaborate and complex At every step it is necessary to comply with its provisions lest the company and the management should be penalized The penalties are quite heavy and in several cases officers in default can be punished with imprisonment This hampers the proper functioning of the company

Lack of Secrecy The management of companies remains in the hands of many persons Every important thing is discussed in the meetings of Board of Directors Hence secrets of the business cannot be maintained In case of sole proprietorship and partnership forms of organisation such secrecy is possible because a few persons are involved in the management

2 Define the following

Chartered Company- The crown in exercise of the royal prerogative has power to create a corporation by the grant of a charter to persons assenting to be incorporated Such companies or corporations are known as chartered companies Examples of this type of companies are Bank of England (1694) East India Company (1600) The powers and the nature of business of a chartered company are defined by the charter which incorporates it After the country attained independence these types of companies do not exist

in IndiaStatutory Company- A company may be incorporated by means of a special Act of the Parliament or any state legislature Such companies are called statutory companies Instances of statutory companies in India are Reserve Bank of India the Life Insurance Corporation of India the Food Corporation of India etc The provisions of the Companies Act 1956 apply to statutory companies except where the said provisions are inconsistent with the provisions of the Act creating them Statutory companies are mostly invested with compulsory powersRegistered companiesCompanies registered under the Companies Act 1956 or earlier Companies Acts are called registered companies Such companies come into existence when they are registered under the Companies Act and a certificate of incorporation is granted to them by the Registrar

Economics

Chapter-4Basic problems of Economy

Today let us discuss with the topic Production Possibility curve

QuestionExplain the concept of Production Possibility Curve with the help of diagram

Answer) Production Possibility curve is a locus of all possible combinations of two commodities which can be produced in a country with its given resources and technology

The above diagram shows that with the given resources and technology the economy can produce maximum either 5 thousand meters of cloth or 15 thousand quintals of wheat or any other combination of the two goods like B( 1 thousand meters of cloth and 14 thousand quintals of wheat C ( 2 thousands meters of cloth and 12 thousand quintals of wheat) etcProduction Possibility curve is also called production possibility boundary or frontier as it sets the maximum limit of what it is possible to produce with given resources

Geography

Rotationand Revolution

SUNrsquoS POSITION AND SEASONAL CHANGES EQUINOXES ndash SPRING AND AUTUMN

Q1 What is Spring EquinoxA1 On 21st March sunrays fall directly on the equator On that day

As the Equator divides the Earth into two equal halves the sun rays fall directly on the equator twice in a year Equinoxes means equal Spring EquinoxOn 21st March sunrays fall directly on the equator On that day the duration of day and night both are equal ( 12 hours day and 12 hours night) on every places located on equator This day is called as Spring EquinoxAutumn EquinoxOn 23rd September sunrays fall directly on the equator On that day the duration of day and night both are equal ( 12 hours day and 12 hours night) on every places located on equator This day is called as Autumn Equinox

SOLSTICES ndash SUMMER AND WINTERDue to inclination of the Earth on its axis and the apparent movement of the sun the sun rays fall directly on both tropics once in a year Solstice is a Latin word which mean ldquothe Sun standing stillrdquoSummer SolsticesAfter 21st March there is an apparent movement of the Sun to the north of the equator The apparent northward movement up to 21st June when the Sun appears overhead at the Tropic of Cancer (22frac12degN) The sun appears to stand still at this position and then moves southwards towards the equator This position of the Sun on 21st June is known as Summer Solstices On that day the duration of day and night both are equal ( 12 hours day and 12 hours night) on every places located on Tropic of Cancer (22frac12degN)Winter solstices The apparent southward movement of the Sun continues beyond the equator till 22nd

December On this day the Sun is overhead at the Tropic of Capricorn

the duration of day and night both are equal ( 12 hours day and 12 hours night) on every places located on equator This day is called as Spring Equinox

Q2 What do you mean by EquinoxA2 Equinoxes means equal It is use to explain the equal duration of day and night ( 12 hours day and 12 hours night) on the Earth

Q3 On which date the longest day in Tropic of CancerA3 21st June

Q4 What is the meaning of SolsticeA4 Solstice is a Latin word which mean ldquothe Sun standing stillrdquo

Q5 Which is the longest day in southern hemisphereA5 22nd December

Q6 On what date does the Arctic Circle experience the lsquoMidnight SunrsquoA6 On 21 June the Arctic Circle experiences the lsquoMidnight Sunrsquo

Q7 What is cause of Midnight Sun in NorwayA7 During the summer solstice (21 June) the North Pole is inclined towards the Sun Therefore the duration of sunlight or daytime increases from 12 hours at the Equator to 24 hours at the Arctic Circle and beyond Thatrsquos why The region beyond the Arctic Circle especially Norway is known as the Land of the Midnight Sun because there the Sun does not rise or set on 21 June

Q8 Match the column A with BA B

Summer Solstice 21st March

Autumn Equinox 23rd

September

Winter Solstice 21st June

(22frac12degS) This position of the Sun is referred to as the Winter Solstice because it marks the winter season in the Northern Hemisphere On that day the duration of day and night both are equal ( 12 hours day and 12 hours night) on every places located on Tropic of Capricorn (22frac12degS)SEASONS AND DURATION OF DAY AND NIGHT During the equinoxes all places on the Earth have 12 hours of day and 12 hours of night Due to the revolution of the Earth round the Sun on an inclined axis the duration of day and night varies according to seasons and the latitude of a placeDuring the summer solstice (21 June) the North Pole is inclined towards the Sun Therefore the duration of sunlight or daytime increases from 12 hours at the Equator to 24 hours at the Arctic Circle and beyondThe region beyond the Arctic Circle especially Norway is known as the Land of the Midnight Sun because there the Sun does not rise or set on 21 JuneAt the North Pole there will be six months of daylight The Sun will be seen always above the horizon at a low angle At 66degN 24 hours of sunlight can be seen only on 21 June Hammerfest in northern Norway is a place of tourist attraction for observing the phenomenon of the Midnight Sun This place has continuous daylight from 13 May to 29 July This place is easily accessible to tourists and has hotels and other facilities The view of the midnight Sun from here is enthrallingIn the Southern Hemisphere the duration of daylight decreases from 12 hours at the equator to 0 hours beyond the Antarctic Circle In the South Polar Region there is 24 hours of darkness The Sun is always below the horizon In the Southern Hemisphere which experiences winter the duration of night-time is longer than the duration of daylight

Spring Equinox 22nd

December

A8 A B

Summer Solstice 21st June

Autumn Equinox 23rd

September

Winter Solstice 22nd

December

Spring Equinox 21st March

During winter solstice (22 December) the South Pole is inclined towards the Sun The Southern Hemisphere experiences summer and the Northern Hemisphere has winter Therefore the duration of daylight or sunlight is greater in the Southern Hemisphere than in the Northern HemisphereThe duration of daylight increases from 12 hours at the equator to 24 hours beyond the Antarctic Circle The South Polar Region has 24 hours of sunlight for many days continuously At the South Pole there will be six months of sunlight The Sun will always be seen at a low angle above the horizon In the Northern Hemisphere the duration of daylight will decrease from 12 hours at the equator to 0 hours at the Arctic Circle There are 24 hours of darkness in the North Polar region The duration of night is greater than the duration of daylight as one move northwards from the Equator It is evident from the above table that the duration of daylight is 12 hours throughout the year at the equator only As one moves away from the equator the seasonal variations in the duration of daylight increase The seasonal variations in the duration of daylight are maximum at the Polar Regions

Subject Eng Literature (The Merchant of Venice ndash William Shakespeare)Topic Act II Scene 7 Lines 36 to 80 (End of scene ) [Students should read the original play and also the paraphrase provided]

Summary Questions amp AnswersThe Prince then examines the inscription on the silver casket which says ldquoWho chooseth me shall get as much as he deservesrdquo The Prince says that he deserves Portia more than anybody else because of his high rank his noble birth and his great wealth and power But then he argues that silver is ten times

(1) (Act II Sc 7 L 39-47)

From the four corners of the earth they come

To kiss this shrine this mortal breathing saint

The Hyrcanian deserts and the vasty wildsOf wide Arabia are as through-fares now

inferior to gold and therefore he cannot believe that the portrait of such a beautiful lady as Portia can be contained in the silver casket He decides to see the inscription on the golden casket before making his decision

The Prince goes to examine the inscription on the golden casket which says ldquoWho chooseth me shall get what many men desirerdquo The Prince believes that the whole world desires to possess Portia otherwise so many suitors would not have come from all corners of the world for winning Portia Some of them have come from the distant lands of Persia and Arabia The deserts of Persia (Hyrcanian deserts) and the boundless desolate lands of Arabia have been crossed by the Princes seeking the hand of Portia He contrasts this casket containing Portiarsquos portrait with the old English gold coin bearing the image of the archangel (angel of the highest rank) He goes on to remark that while the figure of the archangel is engraved (Insculped) upon the English coin the picture of Portia who is beautiful as an angel lies hidden inside one of the caskets namely the Golden Casket (Golden Bed)

On the basis of his assessment of the inscription on the golden casket the Prince decides to choose the golden casket He asks for the key and opens the golden casket only to find therein an empty human skull holding a roll of

For princes to come view fair PortiaThe watery kingdom whose ambitious headSpets in the face of heaven is no barTo stop the foreign spirits but they comeAs orsquoer a brook to see fair Portia

(i) Explain the occasion for the above mentioned speech

These are the comments of the Prince of Morocco after he reads the inscription on the golden casket His mental process is revealed to us in these words We find him debating within himself as to which casket he should choose

(ii) What light does the above speech throw on the personality of Prince of Morocco

From the above mentioned speech we come to know that the Prince of Morocco is keen to marry Portia He is the type of person who is easily taken away by outward appearance He is in love with Portia because of her beauty

(iii) What information can you gather about Portia from the above mentioned lines

The given speech shows that Portia is a very beautiful lady She must be possessed of good qualities because many suitors come to her place from all over the world with a desire to get married to her The Prince of Morocco is so impressed by her beauty that he calls her a saint According to him the whole world is desirous of having her

(iv) Elucidate the significance of the first two lines

In these lines the Prince of Morocco pays a compliment to Portia These lines show his admiration for her He says that people come from all parts of the world to see fair Portia

(v) Explain the meaning of the last four lines of the

passage

In these lines the Prince of Morocco says that even the vast oceans which throw a challenge at the sky are unable to prevent men from coming to Portiarsquos place to have a glimpse of her These lines are also a tribute to Portiarsquos beauty and good qualities Many men voyage across the ocean treating it as a mere stream to see the beautiful Portia

paper in which is written that whoever happens to be guided by the glitter of things is invariably deceived

On reading the scroll the Prince says that he is too sad at heart to speak a more formal farewell and leaves with his followers amidst a sound of trumpets

After the Prince of Morocco leaves Portia remarks that the Prince is a gentle fellow but she is rid of him May all persons of his nature make a similar choice

IMPORTANT PASSAGES EXPLAINED

(Act II Sc 7 L 39-43)From the four corners of the earth they come

To kiss this shrine this mortal breathing saintThe Hyrcanian deserts and the vasty wildsOf wide Arabia are as through-fares nowFor princes to come view fair Portia

Context

This passage occurs in Act II Scene 7 in The Merchant of Venice This is part of the speech made by the Prince of Morocco

(2)

(Act II Sc 7 L 48-53)

MOROCCO One of these three contains her heavenly pictureIst like that lead contains her

Twere damnation To think so base a thought it were too grossTo rib her cerecloth in the obscure graveOr shall I think in silver shes immurdBeing ten times undervalued to tried gold

(i) What meaning does the Prince of Morocco find out of the inscription of the golden casket What have Belmont and Portiarsquos house been called and why

The inscription on the golden casket is ldquoWho chooseth me shall gain what many men desirerdquo The Prince finds out that it means that the chooser of the golden casket will get Portia because many men desire her In fact the entire world desires her Because of the coming of many suitors to Belmont from different countries in order to win Portiarsquos hand Belmont has become a centre of pilgrimage and her house is the shrine where saintly Portia is installed

(ii) What does the Prince of Morocco do before making the final choice of the casket Which is the correct casket and who will win Portiarsquos hand

The Prince of Morocco surveys and analyses the inscriptions on the casket of lead silver and gold Before making the final choice like a very systematic and methodical person he once again considers the claims of the caskets The casket containing Portiarsquos picture is the correct casket and the person choosing it will win Portiarsquos hand

Explanation

While praising Portia the Prince of Morocco conceives Portia as a goddess whose image is placed inside one of the caskets Many suitors are coming from far and wide the north and the south the east and the west (Four corners) in order to try their luck Some of them have come from the distant land of Persia and Arabia The deserts of Persia (Hyrcanian deserts) and the boundless desolate lands of Arabia have been crossed by the Princes seeking the hand of Portia All this shows that Portia is indeed the most beautiful lady of the world

(iii) What does the Prince of Morocco say in his estimation while examining the motto on the silver casket What does he find in the golden casket

While examining the motto on the silver casket which says ldquoWho chooseth me shall get as much as he deservesrdquo Morocco says that in his own estimation he surely deserves Portia in all respects ndash rank birth wealth etc

He chooses the golden casket When he opens it he finds an empty human skull holding a scroll in which it is written that those who are attracted by the glittering outside of things are always deceived as Morocco has been deceived

(iv) What kind of nature does the Prince of Morocco have

The Prince of Morocco has a simple nature who does not look deeply into the inner meaning of things but is dazzled by the outward appearance of gold He is inclined to over-estimate his own value and does not realize that it is a duty to ldquogive and hazardrdquo To say that he will not hazard for lead shows that he misreads the true meaning of the inscription which is that he should be prepared to ldquohazard all he hathrdquo for Portia So his feeling is only one of fascination and romantic attraction

(v) Do you think that the lottery of the caskets is not a matter that will be determined by chance

In fact the lottery of the casket is not a matter that will be determined by mere chance but that it is a true test of character and of sincerity which is amply proved not only by Moroccorsquos choice but also by the arguments which he uses to help him in his choice

(Act II Sc 7 L 55-59)

They have in England

A coin that bears the figure of an angelStamped in gold but thats insculpd uponBut here an angel in a golden bedLies all within

Context

(3)

(Act II Sc 7 L 63-77)A carrion Death within whose empty eye

There is a written scroll Ill read the writing

All that glisters is not goldOften have you heard that toldMany a man his life hath soldBut my outside to beholdGilded tombs do worms infoldHad you been as wise as boldYoung in limbs in judgment oldYour answer had not been inscrolld

This passage occurs in Act II Scene 7 in The Merchant of Venice This is part of the speech made by the Prince of Morocco

Explanation

In this passage the Prince of Morocco bestows high praise on Portia whose hand he is seeking He contrasts this casket containing Portiarsquos portrait with the old English gold coin bearing the image of the archangel (angel of the highest rank) He goes on to remark that while the figure of the archangel is engraved (Insculped) upon the English coin the picture of Portia who is beautiful as an angel lies hidden inside one of the caskets namely the Golden Casket (Golden Bed) In the day of Elizabeth silver was ten times inferior in value to gold Therefore the Prince of Morocco believing that Portiarsquos portrait is contained in the Golden Casket decides to choose the Golden Casket

Fare you well your suit is coldCold indeed and labour lostThen farewell heat and welcome frostmdashPortia adieu I have too grievd a heartTo take a tedious leave Thus losers part

(i) What reward does the Prince of Morocco get after making a wrong choice of the Casket How does he feel

After making the wrong choice in selecting the casket of gold the Prince of Morocco as a reward earns a rebuke in the form of a scroll tucked in the empty eye-socket of a skull kept in the casket of gold The Prince is shocked and disappointed He becomes all the more sad and dejected when he reads the scroll which points to his foolishness in being misled by the appearance and outward show as indicative of its worth

(ii) How does the Prince respond after reading the scroll

After reading the scroll the Prince though upset accepts the result with good grace and decorum befitting a royal suitor and true sportsman He says that his love-suit is really cold otherwise he would have chosen correctly but now his efforts have been in vain So he bids farewell to Portia to the warmth and enthusiasm of love and welcomes the cold and bitterness of dejection and misery of life which lies ahead

(iii) What request does he make to Portia and why

After being failure in his mission he requests Portia to give him permission to leave at once because he is too sad to undergo the tediousness of a formal leave-taking He tells that it is the manner in which defeated persons part unceremoniously

(iv) Explain the following lines

ldquoAll that glisters is not goldOften have you heard that toldMany a man his life hath soldBut my outside to beholdGilded tombs do worms infoldrdquo

Mere glitter does not make a metal to be gold Man has often been warned against appearance but it has been of no use Many people have sacrificed their lives only to seek the outer appearance of gold Worms are found inside the gilded

monuments

Class XSubject Topic Summary Execution

Hindi 2ndlang

नया रासता भाग 6 मायाराम 0ी घर म धनी मल 0ी और उनी बटी सरिरता ी ही चचा बनी रहती थी अमिमत ो इसम ोई रलिच ना थी वह धनी घर ी लडी स शादी र सवय ो बचना नही चाहता था उसा भी सवाणिभमान ह ईशवर ी पा

स उस पास पस ी ोई मी नही थी अभी उसन फकटरी ही लगाई थी उसी समझ बाहर था कि उस घर वालो ा झाव पस ी तरफ कयो

ह उसन मा स सवाल किया कि मा तम सरिरता स मरी शादी कयो रना चाहती हो मा न उस समझाया कि वह दखन म बरी नही ह और किफर खानदान अचछा

ह वह ए शल गरहणी रप म घर सभाल सगी अमिमत न मा ो इस बात ा एहसास राया कि मीन सबध लिलए मना रन पर उस दिदल

पर कया बीती होगी मा और अमिमत ी लडी बार म ाफी बात हईमा ा झाव सरिरता ी तरफ था कयोकि वह घर पर अचछा दह0 लर आ रही

थी अमिमत न अपनी मौसी ी बरी हालत बार म बताया कि किस तरह वह बड घर ी खानदानी बटी लाई थी और आ0 उसी हालत कितनी खराब ह लाई थी बहकलब 0ाती ह और बचचो ो भी नही दखती ह बात चल ही रही

थी कि तभी ए ार बाहर आर री धनी मल0ी घर अदर आए और पीछ स डराइवर फल ी ए टोरी लर आया अदर आए और पीछ स

डराइवर ए टोरी फल ी लर आया अमिमत ो फल ी पटी बरी लग रही थी अमिमत न पछ लिलया यह फल कयो ल आए ह प इन सब ी कया

0ररत थी उनो न 0वाब दिदया कि 4 पटी शमीर स मगाए थ अमिमत ो या सनर करोध आ गया तभी उस किपता 0ी आ गए उन आत ही अमिमत उठर बाहर चला गया वहा वहा मा पास आर बठ गया और बोला

अभी रिरशता तय नही हआ और धनी मल 0ी धनी मल 0ी फल ी पटी लर चलआय मा न समझाया कि 0ब सबध 0ड 0ाता ह तो खाली हाथ नही

आत अमिमत न मा स हा कि तम सबन सरिरता ो इस घर म लान ी ठान रखी ह धनीमल 0ी उस दिदन सरिरता ो दखन ी तारीख तय रन आय थ

Commercial Studies

Banking Nowadays Bank provide easy and quick services through internet facilities methods of Banking is called internet bankingIn order to save the time and money involved in visiting Bank branches people increasingly prefer to have internet banking

There are different modes of doing internet banking or transferring money through online They areReal Time Gross Settlement (RTGS)National Electronic Fund Transfers (NEFT)

1

Question

1) Explain the term RTGS Write the features of RTGS

Answer)The acronym RTGS stands for Real Time Gross Settlement which may be defined as the continuous real time settlement of funds transfer individually on and order by order basis without netting lsquoReal timersquo may be defined as the processing of instructions at the time they are received rather than at some letter time lsquoGross settlementrsquo may be defined as the settlement of transfer instructions which occurs

individually

Features of RTGS1It is the continuous settlement of

funds transfer individually on an order by order basis

2RTGS facility is provided only by CBS core banking solution enabled Bank branches

3Amount charged from the customer for RTGS transactions vary from bank to bank

2) Explain the term NEFT Write the features of NEFT

Answer) National electronic funds transfer may be defined as a nationwide system that facilitates individuals Farms and copper operates to electronically transfer funds from any bank branch to any individual farm or corporate having an account with any other bank branch in the country

Features of NEFT2 Transfer can be made 7 times on

weekdays and 6 times on Saturday

3 NEFT cannot be used to receive foreign remittances

4 NEFT transaction takes place in batches

5 A bank branch must be NEFT enabled to become a part of NEFT fund transfer network

6 There is no maximum or minimum amount that can be transferred through NEFT when one bank has a bank account

English Language

CompositionEssay

A composition is an art of creating a piece of writing on any topic or subject It is the writing correctly beautifully and clearly in order to make some interesting reading Structure of the composition

Introduction ( you lay the foundation for your composition)

Body (it constitutes the main part of the essay)

Conclusion (final statement that leaves a lasting impression)

Kinds of essays1 The Narrative essay2 The descriptive essay3 The reflective essay4 The argumentative essay

Write a composition on any one of the following topics (350- 400 words)

1 Friendship Or2 The first day of your school

Subject Eng Literature (The Merchant of Venice ndash William Shakespeare)Topic Act V Scene 1 Lines 127 to 158 (Nerissa helliphellip The clerk will nersquoer wear hair onrsquos face that had it) [Students should read the original play and also the paraphrase given in the school prescribed textbook]

Summary Revision Questions o Soon thereafter Bassanio Gratiano

and Antonio arrive

o Bassanio tells Portia that he is feeling as if it is morning because of the presence of Portia who is shining like the sun When Antonio is introduced by Bassanio to Portia she tells Bassanio that he should be grateful to Antonio who took so much trouble on his account even to the extent of risking his life

o Nerissa starts quarrelling with Gratiano and demands that he show her the ring she had presented to him and which she had warned him not to lose She suspects that Gratiano must have presented the ring to some young woman and not to the lawyerrsquos clerk as he repeatedly says and assures

Answer the following questions to check your preparation of Act IV Scenes 1 and 2

You must attempt only after you have completed your preparation of Act IV The answers must be in complete sentences using textual evidence (with citation) when necessary

[It would be in your own interest to attempt the above questions honestly totally refraining from consulting your textbook or your notes during answering After completion you should correct the paper yourself consulting the textbooknotes etc and award marks as specified Please let me know the marks you scored through WhatsApp in the group or to my personal WhatsApp]

Act IV Scene 1 (each question carries 2 marks)

1 What did the Duke try to do for Antonio

2 Why does Shylock refuse to show mercy How does he justify his stance

3 Why does Antonio say he is ready to die 4 What information is contained in Bellariorsquos letter

5 Why does Portia (as Balthazar) assert that Shylock must show mercy How does he respond

6 What offers are made to Shylock to get him to spare Antonio How are they received

7 What does Antoniorsquos speech as he faces the prospect of Shylockrsquos knife tell you about his character

8 How do Bassanio and Gratiano react to the looming prospect of Antoniorsquos demise

9 How does Portia (as Balthazar) use the law to turn the tables on Shylock

10 What does the Duke decree should happen to Shylock Why What happens to Shylockrsquos estate

11 What does Portia ask Bassanio as payment for her ldquoservicesrdquo What is his initial response What makes him change his mind

Act IV Scene 2 (each question carries 1frac12 marks)

1 What does Gratiano bring to Portia (Balthazar)

2 What does Nerissa plan on getting from Gratiano What does Portiarsquos comment suggest about men

ECO-10 280620 Topic-Supply AnalysisSHIFTING OF SUPPLY

But if there is change in factors other than the price of the commodity then either more is supplied at the same price or less supplied at the same price In such cases the price of the commodity remains constant but there is a change in other factors like change in the price of inputs change in technology of production change in price of other related goods change in taxation policy of the government etc For example there is an improvement in the technology of production of the commodity in question It leads to decrease in per unit of cost production of the commodity The firm is willing to sell more quantity of the commodity at the same price So the supply other commodity increases at the same price This increase in supply is shown by rightward shift of supply curve On the other hand if the firm uses inferior technology of production the cost of production per unit of the commodity increases The firm is willing to sell less quantity at the same price So the supply of the commodity decreases at the same price This decrease in supply is shown by leftward shift of the supply curve The above cases of increase and decrease in supply can be shown with the help of the following figures

Y INCREASE OF SUPPLY Price (Rs) s

P A s1

B

s

X` O s1 X

q q1

Y` Quantity demanded (in units)

Y DECREASE IN SUPPLY s2

s

price (Rs)

C

p A

s2

s

X` o X

q2 q

Y` Quantity demanded ( in units)

Main factors causing increase in supply or rightward shift of supply Curve(i) Fall in the price of other related goods

(ii) Fall in the price of inputsfactors(iii) Use of better technology in production(iv) Decrease in the rate of excise duty by government(v) If the objective of producer changes from profit maximization to salesMaximization

Main factors causing decrease in supply or leftward shift of supply curve(i) Increase in the price of other related goods(ii) Rise in the price of inputsfactors(iii) Use of inferior technology in production(iv) Increase in the rate of excise duty by the government(v) If the objective

Subject - Biology Topic ndash Chapter mdash6 PhotosynthesisSummary Execution

Today we will know about photosynthesis and its stages

Q1 What do you mean by photosynthesis The process by which living plants containing chlorophyll produce food

substances from carbon-di- oxide and water by using light energy Sunlight

6CO2 +12 H2O----------------------- C6 H12O6 + 6H2O + 6O2

Chlorophyll

Q2 What are the importance of photosynthesis I) Food for all Green plants trap solar energy by photosynthesis

process and supply food and energy for all living organisms either directly or indirectly

Ii) Oxygen to breathe in by product of photosynthesis is oxygen which is essential for all living organisms respiration

Q3 Write about two main phases of photosynthesis A Light dependent phase This phase occur in grana of chloroplast I) The chlorophyll on exposure to light energy becomes activated by

absorbing photons Ii) The absorbed energy is used in splitting the water molecules (H2O)

into its two components (H+ and OH- ) and releasing electron s 2H2O------------------------- 4H+ + 4e- +O2

Energy of 4 photons This reaction is known as photolysis

End products are H+ and oxygen water

B Light independent (Dark ) phase The reactions in this phase require no light energy

Here CO2 combine with H+ and produce glucose

Class XI

Subject Topic Summary ExecutionEVS Chapter-4 Legal

regimes for sustainable development

Environmental legislationEnvironmental legislation is the collection of laws and regulations pertaining to air quality water quality the wilderness endangered wildlife and other environmental factors The act ensures that matters important to the environment are thoroughly

Learn -The Forest (Conservation) Act 1980

considered in any decisions made by federal agencies

The Forest (Conservation) Act 1980 The Forest (Conservation) Act 1980 an Act of the Parliament of India to provide for the conservation of forests and for matters connected therewith or ancillary or incidental thereto It was further amended in 1988 This law extends to the whole of IndiaObjects and Reasons of the Forest Conservation Act

Deforestation causes ecological imbalance and leads to environmental deterioration Deforestation had been taking place on a large scale in the country and it had caused widespread concern The act seeks to check upon deforestation and de-reservation of forests

Subject Eng Literature (The Tempest ndash William Shakespeare) Topic Act II Scene 1 Lines 314 to 329 (End of scene)

[Students should read the original play and also the paraphrase given in the school prescribed textbook]Summary Questions amp Answers

Conspiracy of Antonio and Sebastian (Contd)

o As they approach Ariel appears again and wakes up Gonzalo by singing a tune in his ear Alonso also wakes up and they see both Sebastian and Antonio with drawn swords On being caught off guard they make up a story saying that they had heard a bellowing of bulls or lions

o They then moved to another part of the island

o Ariel at once rushes to Prospero to inform him of this development

SUMMING-UP of ACT-2 SCENE-1

(i) Among the survivors Ferdinand is separated from the rest which results in the disconsolate grief of Alonso as he took him for dead

(ii) The villainy of Antonio is confirmed

(iii) The supremacy of Prosperorsquos magic which resulted in the failure of the human conspiracy

(1)

(Act II Sc 1 L 311-325)SEBASTIAN Whiles we stood here securing your repose

Even now we heard a hollow burst of bellowing Like bulls or rather lions Didt not wake youIt struck mine ear most terribly

ALONSO I heard nothingANTONIO O rsquotwas a din to fright a monsters ear

To make an earthquake Sure it was the roarOf a whole herd of lions

ALONSO Heard you this GonzaloGONZALO Upon mine honour sir I heard a humming

And that a strange one too which did awake meI shaked you sir and cried As mine eyes opened I saw their weapons drawn There was a noiseThats verily rsquoTis best we stand upon our guardOr that we quit this place Lets draw our weapons

(i) Why has Prospero sent Ariel to Gonzalo and Alonso What does Ariel do to awaken Gonzalo

Prospero has already come to know by his magic powers the danger which threatens Gonzalo who had been Prosperorsquos friend and so he sent Ariel to preserve the lives of both Gonzalo and Alonso Prospero does not want that his scheme should remain unfulfilled Ariel begins to sing a song in Gonzalorsquos ears to awaken him(ii) Who are ready to carry out their plan Who takes steps to stop them Why does Gonzalo feel surprised after being awakened

Sebastian and Antonio are ready to carry out their plans They are standing with their swords drawn to kill Alonso and

(iv) We see two sets of contrasting characters Gonzalo-Adrian against Antonio-Sebastian

(v) The grief that works in Alonso can be perceived to his repentance for his association in Antoniorsquos crime against Prospero

Gonzalo Ariel takes steps to stop them from carrying out their nefarious scheme When Gonzalo is awakened by the song sung by Ariel into his ears he (Gonzalo) feels surprised because he sees Sebastian and Antonio standing with their swords drawn(iii) What reason do Sebastian and Antonio tell of drawing their swords when they are suspected by Alonso and Gonzalo

When Sebastian and Antonio are seen with their swords drawn they are looked with suspicion by Gonzalo and Alonso At first Sebastian tells them that as they stood here to guard them during their sleep they heard only a little before a sudden loud noise very much like the roaring of bulls or more probably that of lions Then Antonio follows him saying that this was a noise so terrible as to frighten even a monsterrsquos ears and this noise could even have shaken the earth and it was surely like the roaring of a multitude of lions Then seeing the danger they have drawn their swords Perhaps after hearing the terrible noise they (Gonzalo and Alonso) woke up from their sound sleep

(iv) What does Gonzalo tell Alonso about the strange noise What did he see on opening his eyes Gonzalo tells Alonso that he did not hear the sound of roaring but he heard a humming sound which was strange and which woke him up After waking up he gave him (Alonso) a shaking and a loud cry On opening his eyes he saw these two gentlemen standing with their swords drawn(v) What does Gonzalo suggest

Gonzalo suggests that there was a noise indeed and of that he has no doubt at all and suggests that the best course for them would be to remain alert and vigilant against any possible danger to their lives or to leave this place and move to some other part of the island

Class XIISubject Topic Summary Execution

Commerce

Chapter- Management

Today we will discuss about LEVELS OF MANAGEMENT

Levels of management is a series or chain of managerial positions from top to bottom It helps individuals to know their authority responsibilities and superior-subordinate relations among themselves There are mainly three levels of Management TOP LEVEL MANAGEMENTMIDDLE LEVEL MANAGEMENTLOWER LEVEL MANAGEMENT

Top level managementIt consists of members at the highest level in the management hierarchy This level includes Board Of Directors Chief Executive Managing Directors Chairman President Vice President

Rolefunctions of the top levelmanagement1To analyse evaluate and deal

with theexternal environment2 To determine the objectives and

policies of the business3 To strive for welfare and survival

of business

4 To create an organisational Framework consisting of authority responsibility relationship

Middle level management Congress of members or groups who are concerned with implementation of the policies let down by the top managementThis level includes head of the department such as finance manager marketing manager branch and regional managers departmental and divisional heads plant superintendent etc

Role of functions of the middle level management

1 To interpret the policies framed by top management

2 To assign duties and responsibilities to lower level managers

3 To select and appoint employees for middle and supervisory level and evaluate their performance

4 To co-operate with other departments for smooth functioning

Operational or supervisory level managementIt refers to the group are members who are concerned with execution of the work They are also known as fast line managers This level includes supervisor 4 men Section Officer clerk Inspector etc

Role of functions of the lower level management1 To plan and execute day-to-

day operations2 To supervise and control the workers3 To arrange materials and

tools to start the process and make arrangements for training

4 Today present workers grievance and suggestions before the management and

ensure safe and proper working conditions in the factory

Business Studies

Staff Appraisal Chapter- 10 Today let us start with a new chapter

Staff Appraisal

Meaning of Performance Appraisal

Performance Appraisal is the systematic evaluation of the performance of employees and to understand the abilities of a person for further growth and developmentThe supervisors measure the pay of employees and compare it with targets and plansThe supervisor analyses the factors behind work performances of employeesThe employers are in position to guide the employees for a better performance

Objectives of Performance Appraisal

Following are the objectives of Performance Appraisal

To maintain records in order to determine compensation packages wage structure salaries raises etc

To identify the strengths and weaknesses of employees to place right men on right job

To maintain and assess the potential present in a person for further growth and development

To provide a feedback to employees regarding their performance and related status

To provide a feedback to employees regarding their performance and related status

Importance of Performance Appraisal

Performance appraisal provides important and useful information for the assessment of employees skill

knowledge ability and overall job performance The following are the points which indicate the importance of performance appraisal in an organization

1 Performance appraisal helps supervisors to assess the work performance of their subordinates

2 Performance appraisal helps to assess the training and development needs of employees

3 Performance appraisal provides grounds for employees to correct their mistakes and it also provides proper guidance and criticism for employees development4 Performance appraisal provides reward for better performance

5 Performance appraisal helps to improve the communication system of the organization

6 Performance appraisal evaluates whether human resource programs being implemented in the organization have been effective

7 Performance appraisal helps to prepare pay structure for each employee working in the organization

8 Performance appraisal helps to review the potentiality of employees so that their future capability is anticipated

Geography

DRIANAGE The SubarnarekhaThe Subarnarekha and the Brahmaniinterposed between the Ganga and the Mahanadi deltas drain an area of 19300 sq kmand 39033 sq km respectively The drainage basins of these streams are shared byJharkhand Odisha west Bengal and Chhattisgarh The Brahmani is known as southKoel in its upper reaches in Jharkhand

The NarmadaThe Narmada rises in the Amarkantak hills of MadhyaPradesh It flows towards the West in a rift valleyformed due to a geological fault The total length of it is 1300 km All the tributaries of the

Q1 Name the two westward flowing rivers in the peninsular plateauA1 Narmada and Tapi are the only westward flowing rivers of the peninsular plateau

Q2 Differentiate between east-flowing rivers and west-flowing riversA2

East-flowing rivers

West-flowing rivers

Narmada are very short inlength Most of its tributaries join the main streamright anglesThe Narmada basin covers parts of Madhya Pradesh and Gujarat

The Tapi The Tapi rises in the Satpura ranges in the Betul listrictof Madhya Pradesh It flows in a rift valley parallel tothe Narmada but it is much shorter in length It coversparts of Madhya Pradesh Gujarat and MaharashtraThe length is about 724 km

The Sabarmati and the MahiThe Sabarmati rises in the Aravali hills and flows south-south-westwards for a distance of 300 kilometres to the Arabian Sea The Sabarmatibasin extends over an area of 21674 sq km in Rajasthan and Gujarat The Mahi rises inthe east of Udaipur and drains an area of 34842 sq km lying in Madhya PradeshRajasthan and Gujarat It flows south-westwards for a distance of 533 km before it fallsinto the Gulf of Khambhat

The ChambalThe Chambal rises near Mhow in the Vindhya Range and flows towards the northgenerally in a gorge upto Kota Below Kota it turns to the north-east direction and afterreaching Pinahat it turns to the east and runs nearly parallel to the Yamuna beforejoining it in the southern part of the Etawah district in Uttar PradeshMajor Rivers of India with their basin area (Sqkm)

Himalayan System Indus 321290Ganga 861404

Brahmaputra 187110Indus System

Jhelum 34775Beas 20303

Ganga System Yamuna 366223Ghaghra 127950

Peninsular RiversNarmada 98796

Tapi 65145Mahanadi 141600

Subarnarekha 19300Sabarmati 21674

Mahi 34842Godavari 312812

Godavari Krishna Kaveri Mahanadi are the east-flowing rivers

Narmada Tapi west-flowing rivers

They fall into the Bay of Bengal

They fall into Arabian Sea

These rivers form big deltas

These rivers form comparativelysmall deltas

Catchment areas of these rivers are larger

Catchment areas of these rivers are smaller

Krishna 2589488Cauveri 87900

Subject ndashBiology Topic ndashChapter -5 Inheritance amp Variations Summary ExecutionToday we will discussabout linkage and its classification

LINKAGE The tendency of the genes located on the same chromosome to stay together is

hereditary transmission Linked genes the genes responsible for this Genes that exhibit the process of linkage locates in the same chromosome The distance between the linked genes in a chromosome determines the strength

of linkage i e genes that are located close to each other show stronger linkage than that are located far from each other

COMPLETE LINKAGE It is the type of linkage showed by the genes that are closely located or are tightly

linked with each other as they have no chance of separatingby crossing over These genes are always transmitted together to the same gamete and the same

offspring In such condition only parental or non cross over type of gametes are formedINCOMPLETE KINKAGE It is type of linkage showed by the genes that are distantly located orare loosely

linked with each other because they have chance of separating by crossing over

SIGNIFICANCE i) It helps in holding the parental character togetherii) It checks the appearance of new recombination and helps in bringing the

hybrid population which resembles the original parents iii) Linked genes dilute the effects of undesirable traits

Subject Eng Literature (The Tempest ndash William Shakespeare) Topic Essay Questions (EQ-3)Question No 3

Give a character sketch of CalibanAnswer

The character of Caliban has been wonderfully conceived by Shakespeare as the manifestation of all that is gross and earthy ndash a sort of creature of the earth as Ariel is a sort of creature of the air

Calibanrsquos Physical Appearanceo Caliban is lsquofreckledrsquo a lsquomisshapen knaversquo not honoured with human shape

o Prospero calls him lsquothou tortoisersquo (Act I Sc 2 Line 317) Trinculo stumbling upon him describes him as ldquoA strange fish hellip Legged like a man And his fins like armsrdquo He ldquosmells like a fishrdquo (Act II Sc 2 Line 25)

o Prospero also calls him a ldquobeastrdquo (Act IV Sc 1 Line 140) and ldquoThis misshapen knaverdquo (Act V Sc 1 Line 268)

o Further it appears that in addition to his physical deformity his spiritual inferiority is also suggested by Prosperorsquos claim that his birth resulted from the union between his mother the witch Sycorax and the devil

Calibanrsquos ParentageWhen the play opens Caliban is twenty four years of age having been born on the island twelve years before the coming of Prospero His mother was the foul witch Sycorax who was banished from Algiers for ldquomischiefs manifold and sorceries terrible to enter human hearingrdquo (Act I Sc 2 Line 264) and the father was the Devil himself Thus

Caliban is a monster of evil and brute nature ugly deformed and stinking

Calibanrsquos Savage and Malignant Natureo Caliban is entirely a creature of the earth ndash gross brutal and savage He regards himself as the rightful possessor

of the island and Prospero as a usurper

o In his young age he was on good terms with Prospero He had consented to be received by Prospero at his house and to be educated by him He has learnt human language only to curse his master whom he abhors

o His beastly nature soon breaks out and ends in a vicious attack on Miranda This opens the eye of Prospero who becomes severe to him and enforces his service by threats and violence

o Prospero uses him to make dams for fish to fetch firewood scraper trenches wash dishes and keep his cell clean

Calibanrsquos Hatred for ProsperoA profound hatred for Prospero has taken hold of Caliban It springs from a sense of his being dispossessed and ill-treated He would kill Prospero if he could but he knows the power of Prosperorsquos lsquobookrsquo Hence he transfers his allegiance to Stephano who seems like a god to him He also incites the two drunken associates to batter the skull of Prospero when he sleeps in the afternoon

Caliban Shows Considerable Intelligenceo He has learnt Prosperorsquos language

ldquoYou taught me language and my profit onrsquot (Act II Sc 2 Lines 86-89)Is I know how to curserdquo

o He is well aware of the futility of arguing with one who has more power than he has

ldquoI must obey his art is such power (Act I Sc 2 Lines 373-376)It would control my damrsquos god SetebosAnd make a vassal of himrdquo

o He realizes the importance of Prosperorsquos books

ldquoRemember (Act III Sc 2 Lines 89-92)First to possess his books for without themHersquos but a sot as I am nor hath notOne spirit to commandrdquo

o He knows the value of stealth when attacking the enemy

ldquoPray you tread softly that the blind mole may not (Act IV Sc 1 Lines 194-195)Hear a foot fall we now are near his cellrdquo

o Caliban has a better set of values than Stephano and Trinculo They are distracted from their plan by their greed for Prosperorsquos rich garments Only Caliban realizes that such a finery is unimportant

ldquoLeave it alone thou fool it is but trashrdquo (Act IV Sc 1 Lines 224)

Caliban is not a good judge of characterCaliban is not a good judge of character He decides for example that Stephano is a god because he dispenses lsquocelestial liquorrsquo (Act II Sc 2 Line 115) but then it must be remembered that he has only known his mother Sycorax Prospero Miranda and the spirits that torture him However he quickly discovers his error of judgementrdquo

ldquoWhat a thrice-double ass (Act V Sc 1 Lines 295-297)Was I to take this drunkard for a godAnd worship this dull foolrdquo

Calibanrsquos Imaginative NatureIf Caliban is sub-human in what has been said above he is human in the respect of the poetic side of his character He listens to music with rapture He tells of the beautiful dreams in which heaven rains treasures upon him and which upon waking he yearns to renew One of the most poetic passages in whole play is Calibanrsquos description of the island

to Stephano and Trinculo

ldquoBe not afeard The isle is full of noises (Act III Sc 2 Lines 135-143)Sounds and sweet airs that give delight and hurt notSometimes a thousand twangling instrumentsWill hum about mine ears and sometime voicesThat if I then had waked after long sleepWill make me sleep again and then in dreamingThe clouds methought would open and show richesReady to drop upon me that when I wakedI cried to dream againrdquo

Caliban - Less Ignoble Than Some OthersCalibanrsquos motive for murder is less dishonourable than that of Antonio and Sebastian They plan to kill Alonso to gain his power and wealth Caliban merely wants revenge and the return of lsquohisrsquo island

Conclusiono Calibanrsquos character is not portrayed very clearly in the play and hence we cannot decide whether he is a poor

savage being grossly maltreated by Prospero or whether he is evil and must therefore be kept in bondage or enslavement

o Caliban is contrasted with Ariel who is a spirit and thus swift and uninterested in physical activitieso Caliban is also contrasted with Prospero who is the all-powerful master of the island and of the destiny of all

those on the islando Caliban is also contrasted with civilized man showing him to be less evil than Antonio and Stephano and less

materialistic than Stephano and Trinculoo Caliban has suffered at the hands of Prospero and he has learnt to curse by listening to Prosperorsquos abuse He

certainly believes that Prospero has deprived him of his birthrighto Finally the character Caliban is thought to be one of Shakespearersquos masterpieces The complexity of the character

is reflected in the large volume of critical discussion that has grown around it

ECO ndash12 Topic-Forms of market

MonopolyMonopoly is a market structure in which there is a single seller there are no close substitutes for the commodity produced by the firm and there are barriers to entry Example Indian Railways which is operated under government of India Monopoly also implies absence of competitionFeatures of Monopoly Monopoly is characterized by1 Single Seller In monopoly there is only one firm producing the product The whole industry consists of this single firm Thus under monopoly there is no distinction between firm and industry Being the only firm there is significant control of the firm over supply and price Thus under monopoly buyers do not have the option of buying the commodity from any other seller They have to buy the product from the firm or they can go without the commodity This fact gives immense control to the monopolist over the market

2No Close Substitute There are no close substitutes of the product produced by the monopolist firm If there are close substitutes of the product in the market it implies presence of more than one firm and hence no monopoly In order to ensure a total of control over the market by the monopolist firm it is assumed that there are no close substitutes of the product

3 No Entry amp Exit Monopoly can only exist when there is strong barriers before a new firm to enter the market In fact once a monopoly firm starts producing the product no other firm can produce the same One reason for this is the ability of the

monopolist to produce the product at a lower cost than any new firm who thinks to enter the market If a new firm who knows that it cannot produce at a lower cost than the monopolist then that firm will never enter the market for fear of losing out in competition Similarly the monopolist who is operating for a long time may be enjoying reputation among its customers and is in a better position to use the situation in its own benefit A new firm has to take long time to achieve this and so may not be interested to enter the market

4 Price Maker Being the single seller of the product the monopolist has full control over the pricing of the product On the other hand if there is a large number of buyers in the market so no single buyer exercises any significant influence over price determination Thus it is a sellerrsquos market So monopoly firm is a price maker

5 Price Discrimination Having considerable control over the market on account of being single seller with no entry of other firms the monopolist can exercise policy of price discrimination it means that the monopolist can sell different quantities of the same product to a consumer at different price or same quantity to different consumers at different prices by adjudging the standard of living of the consumer

6 Shape of Demand Curve Since a monopolist has full control over the price therefore he can sell more by lowering the price This makes the demand curve downward sloping

Subject Ac-12 290620 Topic- retirement Model sumThe Balance Sheet of Rohit Nisha and Sunil who are partners in a firm sharing profits according to their capitals as on 31st March 2014 was as under

Liabilities Amount Assets Amount (Rs) (` Rs)

Creditors 25000 Machinery 40000Bills Payable 13000 Building 90000General Reserve 22000 Debtors 30000Capital Less Provision for Rohit 60000 Bad debts 1000

29000 Nisha 40000 Stocks 23000 Sunil 40000 140000 Cash at Bank 18000

200000 200000

On the date of Balance Sheet Nisha retired from the firm and following adjustments were made(i) Building is appreciated by 20(ii) Provision for bad debts is increased to 5 on Debtors(iii) Machinery is depreciated by 10(iv) Goodwill of the firm is valued at Rs 56000 and the retiring partnerrsquos share is adjusted

(v) The capital of the new firm is fixed at Rs120000 Prepare Revaluation Account Capital Accounts of the partner and Balance Sheet of the new firm after Nisharsquos retirement Revaluation AccountDr Cr

Particulars Amount Particulars Amount (`Rs) (Rs`)

Provision for Bad debt Ac 500 Building Ac 18000Machinery Ac 4000Profit transferred toCapital Accounts (3 2 2)Rohit 5786Nisha 3857Sunil 3857

13500

18000 18000

Capital Account

Dr Cr

Particulars Rohit Nisha Sunil Particulars Rohit Nisha Sunil (Rs`) (Rs`) (`Rs) (Rs`) (Rs`) (Rs`)

Sunilrsquos Capital ac 9600 mdash 6400 Balance bd 60000 40000 40000Bank - 66143 - General Reserve 9428 6286 6286Balance cd 72000 mdash 48000 Revaluation (Profi 5786 3857 3857 Rohitrsquos Capital Ac mdash 9600 mdash

Sunilrsquos Capital Ac 6400 Bank 6386 - 4257

81600 66143 54400 81600 66143 54400

Balance Sheet as at 31st March 2014

Liabilities Amount Assets Amount (Rs`) (Rs`)

Creditors 25000 Building 108000Bank overdraft 37500 Machinery 36000

Bills Payable 13000 Debtors 30000Capital Less ProvisionRohit 72000 for Bad debts 1500 28500Sunil 48000 120000 Stock 23000

195500 195500

Working Notes (i) (a) Profit sharing ratio is 60000 40000 40000 ie = 3 2 2(b) Gaining Ratio Rohit = 35 ndash 37 = 2135 ndash 1535 = 635Sunil = 25-27 = 1435 ndash 1035 = 435= 635 435= 6 4 = 3 2(c) Nisha Share of Goodwill = Rs 56000 times 27 = Rs16000Share of Goodwill in the gaining ratio by the existing partner ieRohit = Rs16000 times 35 = Rs 9600Sunil = Rs 16000 times 25 = Rs 6400

The journal entry isRohitrsquos Capital Ac Dr 9600Sunilrsquos Capital Ac Dr 6400 To Nisharsquos Capital Ac 16000(Share of Goodwill divided into gaining ratio)

  • 1 Static Friction
  • The frictional force that acts between the surfaces when they are at rest with respect to each other is called Static Friction
    • Static Friction Examples
      • 2 Sliding Friction
        • Examples Of Sliding Friction
          • 3 Rolling Friction
            • Examples Of Rolling Friction
              • Objects and Reasons of the Forest Conservation Act
Page 30:  · Web viewSubject . Topic . Summary . Execution . English 1 . Sounds of animals . Hens –cackle Horses –neigh Lions –roar Owls –hoots Snake –hiss. English 2 . Mother’s

in IndiaStatutory Company- A company may be incorporated by means of a special Act of the Parliament or any state legislature Such companies are called statutory companies Instances of statutory companies in India are Reserve Bank of India the Life Insurance Corporation of India the Food Corporation of India etc The provisions of the Companies Act 1956 apply to statutory companies except where the said provisions are inconsistent with the provisions of the Act creating them Statutory companies are mostly invested with compulsory powersRegistered companiesCompanies registered under the Companies Act 1956 or earlier Companies Acts are called registered companies Such companies come into existence when they are registered under the Companies Act and a certificate of incorporation is granted to them by the Registrar

Economics

Chapter-4Basic problems of Economy

Today let us discuss with the topic Production Possibility curve

QuestionExplain the concept of Production Possibility Curve with the help of diagram

Answer) Production Possibility curve is a locus of all possible combinations of two commodities which can be produced in a country with its given resources and technology

The above diagram shows that with the given resources and technology the economy can produce maximum either 5 thousand meters of cloth or 15 thousand quintals of wheat or any other combination of the two goods like B( 1 thousand meters of cloth and 14 thousand quintals of wheat C ( 2 thousands meters of cloth and 12 thousand quintals of wheat) etcProduction Possibility curve is also called production possibility boundary or frontier as it sets the maximum limit of what it is possible to produce with given resources

Geography

Rotationand Revolution

SUNrsquoS POSITION AND SEASONAL CHANGES EQUINOXES ndash SPRING AND AUTUMN

Q1 What is Spring EquinoxA1 On 21st March sunrays fall directly on the equator On that day

As the Equator divides the Earth into two equal halves the sun rays fall directly on the equator twice in a year Equinoxes means equal Spring EquinoxOn 21st March sunrays fall directly on the equator On that day the duration of day and night both are equal ( 12 hours day and 12 hours night) on every places located on equator This day is called as Spring EquinoxAutumn EquinoxOn 23rd September sunrays fall directly on the equator On that day the duration of day and night both are equal ( 12 hours day and 12 hours night) on every places located on equator This day is called as Autumn Equinox

SOLSTICES ndash SUMMER AND WINTERDue to inclination of the Earth on its axis and the apparent movement of the sun the sun rays fall directly on both tropics once in a year Solstice is a Latin word which mean ldquothe Sun standing stillrdquoSummer SolsticesAfter 21st March there is an apparent movement of the Sun to the north of the equator The apparent northward movement up to 21st June when the Sun appears overhead at the Tropic of Cancer (22frac12degN) The sun appears to stand still at this position and then moves southwards towards the equator This position of the Sun on 21st June is known as Summer Solstices On that day the duration of day and night both are equal ( 12 hours day and 12 hours night) on every places located on Tropic of Cancer (22frac12degN)Winter solstices The apparent southward movement of the Sun continues beyond the equator till 22nd

December On this day the Sun is overhead at the Tropic of Capricorn

the duration of day and night both are equal ( 12 hours day and 12 hours night) on every places located on equator This day is called as Spring Equinox

Q2 What do you mean by EquinoxA2 Equinoxes means equal It is use to explain the equal duration of day and night ( 12 hours day and 12 hours night) on the Earth

Q3 On which date the longest day in Tropic of CancerA3 21st June

Q4 What is the meaning of SolsticeA4 Solstice is a Latin word which mean ldquothe Sun standing stillrdquo

Q5 Which is the longest day in southern hemisphereA5 22nd December

Q6 On what date does the Arctic Circle experience the lsquoMidnight SunrsquoA6 On 21 June the Arctic Circle experiences the lsquoMidnight Sunrsquo

Q7 What is cause of Midnight Sun in NorwayA7 During the summer solstice (21 June) the North Pole is inclined towards the Sun Therefore the duration of sunlight or daytime increases from 12 hours at the Equator to 24 hours at the Arctic Circle and beyond Thatrsquos why The region beyond the Arctic Circle especially Norway is known as the Land of the Midnight Sun because there the Sun does not rise or set on 21 June

Q8 Match the column A with BA B

Summer Solstice 21st March

Autumn Equinox 23rd

September

Winter Solstice 21st June

(22frac12degS) This position of the Sun is referred to as the Winter Solstice because it marks the winter season in the Northern Hemisphere On that day the duration of day and night both are equal ( 12 hours day and 12 hours night) on every places located on Tropic of Capricorn (22frac12degS)SEASONS AND DURATION OF DAY AND NIGHT During the equinoxes all places on the Earth have 12 hours of day and 12 hours of night Due to the revolution of the Earth round the Sun on an inclined axis the duration of day and night varies according to seasons and the latitude of a placeDuring the summer solstice (21 June) the North Pole is inclined towards the Sun Therefore the duration of sunlight or daytime increases from 12 hours at the Equator to 24 hours at the Arctic Circle and beyondThe region beyond the Arctic Circle especially Norway is known as the Land of the Midnight Sun because there the Sun does not rise or set on 21 JuneAt the North Pole there will be six months of daylight The Sun will be seen always above the horizon at a low angle At 66degN 24 hours of sunlight can be seen only on 21 June Hammerfest in northern Norway is a place of tourist attraction for observing the phenomenon of the Midnight Sun This place has continuous daylight from 13 May to 29 July This place is easily accessible to tourists and has hotels and other facilities The view of the midnight Sun from here is enthrallingIn the Southern Hemisphere the duration of daylight decreases from 12 hours at the equator to 0 hours beyond the Antarctic Circle In the South Polar Region there is 24 hours of darkness The Sun is always below the horizon In the Southern Hemisphere which experiences winter the duration of night-time is longer than the duration of daylight

Spring Equinox 22nd

December

A8 A B

Summer Solstice 21st June

Autumn Equinox 23rd

September

Winter Solstice 22nd

December

Spring Equinox 21st March

During winter solstice (22 December) the South Pole is inclined towards the Sun The Southern Hemisphere experiences summer and the Northern Hemisphere has winter Therefore the duration of daylight or sunlight is greater in the Southern Hemisphere than in the Northern HemisphereThe duration of daylight increases from 12 hours at the equator to 24 hours beyond the Antarctic Circle The South Polar Region has 24 hours of sunlight for many days continuously At the South Pole there will be six months of sunlight The Sun will always be seen at a low angle above the horizon In the Northern Hemisphere the duration of daylight will decrease from 12 hours at the equator to 0 hours at the Arctic Circle There are 24 hours of darkness in the North Polar region The duration of night is greater than the duration of daylight as one move northwards from the Equator It is evident from the above table that the duration of daylight is 12 hours throughout the year at the equator only As one moves away from the equator the seasonal variations in the duration of daylight increase The seasonal variations in the duration of daylight are maximum at the Polar Regions

Subject Eng Literature (The Merchant of Venice ndash William Shakespeare)Topic Act II Scene 7 Lines 36 to 80 (End of scene ) [Students should read the original play and also the paraphrase provided]

Summary Questions amp AnswersThe Prince then examines the inscription on the silver casket which says ldquoWho chooseth me shall get as much as he deservesrdquo The Prince says that he deserves Portia more than anybody else because of his high rank his noble birth and his great wealth and power But then he argues that silver is ten times

(1) (Act II Sc 7 L 39-47)

From the four corners of the earth they come

To kiss this shrine this mortal breathing saint

The Hyrcanian deserts and the vasty wildsOf wide Arabia are as through-fares now

inferior to gold and therefore he cannot believe that the portrait of such a beautiful lady as Portia can be contained in the silver casket He decides to see the inscription on the golden casket before making his decision

The Prince goes to examine the inscription on the golden casket which says ldquoWho chooseth me shall get what many men desirerdquo The Prince believes that the whole world desires to possess Portia otherwise so many suitors would not have come from all corners of the world for winning Portia Some of them have come from the distant lands of Persia and Arabia The deserts of Persia (Hyrcanian deserts) and the boundless desolate lands of Arabia have been crossed by the Princes seeking the hand of Portia He contrasts this casket containing Portiarsquos portrait with the old English gold coin bearing the image of the archangel (angel of the highest rank) He goes on to remark that while the figure of the archangel is engraved (Insculped) upon the English coin the picture of Portia who is beautiful as an angel lies hidden inside one of the caskets namely the Golden Casket (Golden Bed)

On the basis of his assessment of the inscription on the golden casket the Prince decides to choose the golden casket He asks for the key and opens the golden casket only to find therein an empty human skull holding a roll of

For princes to come view fair PortiaThe watery kingdom whose ambitious headSpets in the face of heaven is no barTo stop the foreign spirits but they comeAs orsquoer a brook to see fair Portia

(i) Explain the occasion for the above mentioned speech

These are the comments of the Prince of Morocco after he reads the inscription on the golden casket His mental process is revealed to us in these words We find him debating within himself as to which casket he should choose

(ii) What light does the above speech throw on the personality of Prince of Morocco

From the above mentioned speech we come to know that the Prince of Morocco is keen to marry Portia He is the type of person who is easily taken away by outward appearance He is in love with Portia because of her beauty

(iii) What information can you gather about Portia from the above mentioned lines

The given speech shows that Portia is a very beautiful lady She must be possessed of good qualities because many suitors come to her place from all over the world with a desire to get married to her The Prince of Morocco is so impressed by her beauty that he calls her a saint According to him the whole world is desirous of having her

(iv) Elucidate the significance of the first two lines

In these lines the Prince of Morocco pays a compliment to Portia These lines show his admiration for her He says that people come from all parts of the world to see fair Portia

(v) Explain the meaning of the last four lines of the

passage

In these lines the Prince of Morocco says that even the vast oceans which throw a challenge at the sky are unable to prevent men from coming to Portiarsquos place to have a glimpse of her These lines are also a tribute to Portiarsquos beauty and good qualities Many men voyage across the ocean treating it as a mere stream to see the beautiful Portia

paper in which is written that whoever happens to be guided by the glitter of things is invariably deceived

On reading the scroll the Prince says that he is too sad at heart to speak a more formal farewell and leaves with his followers amidst a sound of trumpets

After the Prince of Morocco leaves Portia remarks that the Prince is a gentle fellow but she is rid of him May all persons of his nature make a similar choice

IMPORTANT PASSAGES EXPLAINED

(Act II Sc 7 L 39-43)From the four corners of the earth they come

To kiss this shrine this mortal breathing saintThe Hyrcanian deserts and the vasty wildsOf wide Arabia are as through-fares nowFor princes to come view fair Portia

Context

This passage occurs in Act II Scene 7 in The Merchant of Venice This is part of the speech made by the Prince of Morocco

(2)

(Act II Sc 7 L 48-53)

MOROCCO One of these three contains her heavenly pictureIst like that lead contains her

Twere damnation To think so base a thought it were too grossTo rib her cerecloth in the obscure graveOr shall I think in silver shes immurdBeing ten times undervalued to tried gold

(i) What meaning does the Prince of Morocco find out of the inscription of the golden casket What have Belmont and Portiarsquos house been called and why

The inscription on the golden casket is ldquoWho chooseth me shall gain what many men desirerdquo The Prince finds out that it means that the chooser of the golden casket will get Portia because many men desire her In fact the entire world desires her Because of the coming of many suitors to Belmont from different countries in order to win Portiarsquos hand Belmont has become a centre of pilgrimage and her house is the shrine where saintly Portia is installed

(ii) What does the Prince of Morocco do before making the final choice of the casket Which is the correct casket and who will win Portiarsquos hand

The Prince of Morocco surveys and analyses the inscriptions on the casket of lead silver and gold Before making the final choice like a very systematic and methodical person he once again considers the claims of the caskets The casket containing Portiarsquos picture is the correct casket and the person choosing it will win Portiarsquos hand

Explanation

While praising Portia the Prince of Morocco conceives Portia as a goddess whose image is placed inside one of the caskets Many suitors are coming from far and wide the north and the south the east and the west (Four corners) in order to try their luck Some of them have come from the distant land of Persia and Arabia The deserts of Persia (Hyrcanian deserts) and the boundless desolate lands of Arabia have been crossed by the Princes seeking the hand of Portia All this shows that Portia is indeed the most beautiful lady of the world

(iii) What does the Prince of Morocco say in his estimation while examining the motto on the silver casket What does he find in the golden casket

While examining the motto on the silver casket which says ldquoWho chooseth me shall get as much as he deservesrdquo Morocco says that in his own estimation he surely deserves Portia in all respects ndash rank birth wealth etc

He chooses the golden casket When he opens it he finds an empty human skull holding a scroll in which it is written that those who are attracted by the glittering outside of things are always deceived as Morocco has been deceived

(iv) What kind of nature does the Prince of Morocco have

The Prince of Morocco has a simple nature who does not look deeply into the inner meaning of things but is dazzled by the outward appearance of gold He is inclined to over-estimate his own value and does not realize that it is a duty to ldquogive and hazardrdquo To say that he will not hazard for lead shows that he misreads the true meaning of the inscription which is that he should be prepared to ldquohazard all he hathrdquo for Portia So his feeling is only one of fascination and romantic attraction

(v) Do you think that the lottery of the caskets is not a matter that will be determined by chance

In fact the lottery of the casket is not a matter that will be determined by mere chance but that it is a true test of character and of sincerity which is amply proved not only by Moroccorsquos choice but also by the arguments which he uses to help him in his choice

(Act II Sc 7 L 55-59)

They have in England

A coin that bears the figure of an angelStamped in gold but thats insculpd uponBut here an angel in a golden bedLies all within

Context

(3)

(Act II Sc 7 L 63-77)A carrion Death within whose empty eye

There is a written scroll Ill read the writing

All that glisters is not goldOften have you heard that toldMany a man his life hath soldBut my outside to beholdGilded tombs do worms infoldHad you been as wise as boldYoung in limbs in judgment oldYour answer had not been inscrolld

This passage occurs in Act II Scene 7 in The Merchant of Venice This is part of the speech made by the Prince of Morocco

Explanation

In this passage the Prince of Morocco bestows high praise on Portia whose hand he is seeking He contrasts this casket containing Portiarsquos portrait with the old English gold coin bearing the image of the archangel (angel of the highest rank) He goes on to remark that while the figure of the archangel is engraved (Insculped) upon the English coin the picture of Portia who is beautiful as an angel lies hidden inside one of the caskets namely the Golden Casket (Golden Bed) In the day of Elizabeth silver was ten times inferior in value to gold Therefore the Prince of Morocco believing that Portiarsquos portrait is contained in the Golden Casket decides to choose the Golden Casket

Fare you well your suit is coldCold indeed and labour lostThen farewell heat and welcome frostmdashPortia adieu I have too grievd a heartTo take a tedious leave Thus losers part

(i) What reward does the Prince of Morocco get after making a wrong choice of the Casket How does he feel

After making the wrong choice in selecting the casket of gold the Prince of Morocco as a reward earns a rebuke in the form of a scroll tucked in the empty eye-socket of a skull kept in the casket of gold The Prince is shocked and disappointed He becomes all the more sad and dejected when he reads the scroll which points to his foolishness in being misled by the appearance and outward show as indicative of its worth

(ii) How does the Prince respond after reading the scroll

After reading the scroll the Prince though upset accepts the result with good grace and decorum befitting a royal suitor and true sportsman He says that his love-suit is really cold otherwise he would have chosen correctly but now his efforts have been in vain So he bids farewell to Portia to the warmth and enthusiasm of love and welcomes the cold and bitterness of dejection and misery of life which lies ahead

(iii) What request does he make to Portia and why

After being failure in his mission he requests Portia to give him permission to leave at once because he is too sad to undergo the tediousness of a formal leave-taking He tells that it is the manner in which defeated persons part unceremoniously

(iv) Explain the following lines

ldquoAll that glisters is not goldOften have you heard that toldMany a man his life hath soldBut my outside to beholdGilded tombs do worms infoldrdquo

Mere glitter does not make a metal to be gold Man has often been warned against appearance but it has been of no use Many people have sacrificed their lives only to seek the outer appearance of gold Worms are found inside the gilded

monuments

Class XSubject Topic Summary Execution

Hindi 2ndlang

नया रासता भाग 6 मायाराम 0ी घर म धनी मल 0ी और उनी बटी सरिरता ी ही चचा बनी रहती थी अमिमत ो इसम ोई रलिच ना थी वह धनी घर ी लडी स शादी र सवय ो बचना नही चाहता था उसा भी सवाणिभमान ह ईशवर ी पा

स उस पास पस ी ोई मी नही थी अभी उसन फकटरी ही लगाई थी उसी समझ बाहर था कि उस घर वालो ा झाव पस ी तरफ कयो

ह उसन मा स सवाल किया कि मा तम सरिरता स मरी शादी कयो रना चाहती हो मा न उस समझाया कि वह दखन म बरी नही ह और किफर खानदान अचछा

ह वह ए शल गरहणी रप म घर सभाल सगी अमिमत न मा ो इस बात ा एहसास राया कि मीन सबध लिलए मना रन पर उस दिदल

पर कया बीती होगी मा और अमिमत ी लडी बार म ाफी बात हईमा ा झाव सरिरता ी तरफ था कयोकि वह घर पर अचछा दह0 लर आ रही

थी अमिमत न अपनी मौसी ी बरी हालत बार म बताया कि किस तरह वह बड घर ी खानदानी बटी लाई थी और आ0 उसी हालत कितनी खराब ह लाई थी बहकलब 0ाती ह और बचचो ो भी नही दखती ह बात चल ही रही

थी कि तभी ए ार बाहर आर री धनी मल0ी घर अदर आए और पीछ स डराइवर फल ी ए टोरी लर आया अदर आए और पीछ स

डराइवर ए टोरी फल ी लर आया अमिमत ो फल ी पटी बरी लग रही थी अमिमत न पछ लिलया यह फल कयो ल आए ह प इन सब ी कया

0ररत थी उनो न 0वाब दिदया कि 4 पटी शमीर स मगाए थ अमिमत ो या सनर करोध आ गया तभी उस किपता 0ी आ गए उन आत ही अमिमत उठर बाहर चला गया वहा वहा मा पास आर बठ गया और बोला

अभी रिरशता तय नही हआ और धनी मल 0ी धनी मल 0ी फल ी पटी लर चलआय मा न समझाया कि 0ब सबध 0ड 0ाता ह तो खाली हाथ नही

आत अमिमत न मा स हा कि तम सबन सरिरता ो इस घर म लान ी ठान रखी ह धनीमल 0ी उस दिदन सरिरता ो दखन ी तारीख तय रन आय थ

Commercial Studies

Banking Nowadays Bank provide easy and quick services through internet facilities methods of Banking is called internet bankingIn order to save the time and money involved in visiting Bank branches people increasingly prefer to have internet banking

There are different modes of doing internet banking or transferring money through online They areReal Time Gross Settlement (RTGS)National Electronic Fund Transfers (NEFT)

1

Question

1) Explain the term RTGS Write the features of RTGS

Answer)The acronym RTGS stands for Real Time Gross Settlement which may be defined as the continuous real time settlement of funds transfer individually on and order by order basis without netting lsquoReal timersquo may be defined as the processing of instructions at the time they are received rather than at some letter time lsquoGross settlementrsquo may be defined as the settlement of transfer instructions which occurs

individually

Features of RTGS1It is the continuous settlement of

funds transfer individually on an order by order basis

2RTGS facility is provided only by CBS core banking solution enabled Bank branches

3Amount charged from the customer for RTGS transactions vary from bank to bank

2) Explain the term NEFT Write the features of NEFT

Answer) National electronic funds transfer may be defined as a nationwide system that facilitates individuals Farms and copper operates to electronically transfer funds from any bank branch to any individual farm or corporate having an account with any other bank branch in the country

Features of NEFT2 Transfer can be made 7 times on

weekdays and 6 times on Saturday

3 NEFT cannot be used to receive foreign remittances

4 NEFT transaction takes place in batches

5 A bank branch must be NEFT enabled to become a part of NEFT fund transfer network

6 There is no maximum or minimum amount that can be transferred through NEFT when one bank has a bank account

English Language

CompositionEssay

A composition is an art of creating a piece of writing on any topic or subject It is the writing correctly beautifully and clearly in order to make some interesting reading Structure of the composition

Introduction ( you lay the foundation for your composition)

Body (it constitutes the main part of the essay)

Conclusion (final statement that leaves a lasting impression)

Kinds of essays1 The Narrative essay2 The descriptive essay3 The reflective essay4 The argumentative essay

Write a composition on any one of the following topics (350- 400 words)

1 Friendship Or2 The first day of your school

Subject Eng Literature (The Merchant of Venice ndash William Shakespeare)Topic Act V Scene 1 Lines 127 to 158 (Nerissa helliphellip The clerk will nersquoer wear hair onrsquos face that had it) [Students should read the original play and also the paraphrase given in the school prescribed textbook]

Summary Revision Questions o Soon thereafter Bassanio Gratiano

and Antonio arrive

o Bassanio tells Portia that he is feeling as if it is morning because of the presence of Portia who is shining like the sun When Antonio is introduced by Bassanio to Portia she tells Bassanio that he should be grateful to Antonio who took so much trouble on his account even to the extent of risking his life

o Nerissa starts quarrelling with Gratiano and demands that he show her the ring she had presented to him and which she had warned him not to lose She suspects that Gratiano must have presented the ring to some young woman and not to the lawyerrsquos clerk as he repeatedly says and assures

Answer the following questions to check your preparation of Act IV Scenes 1 and 2

You must attempt only after you have completed your preparation of Act IV The answers must be in complete sentences using textual evidence (with citation) when necessary

[It would be in your own interest to attempt the above questions honestly totally refraining from consulting your textbook or your notes during answering After completion you should correct the paper yourself consulting the textbooknotes etc and award marks as specified Please let me know the marks you scored through WhatsApp in the group or to my personal WhatsApp]

Act IV Scene 1 (each question carries 2 marks)

1 What did the Duke try to do for Antonio

2 Why does Shylock refuse to show mercy How does he justify his stance

3 Why does Antonio say he is ready to die 4 What information is contained in Bellariorsquos letter

5 Why does Portia (as Balthazar) assert that Shylock must show mercy How does he respond

6 What offers are made to Shylock to get him to spare Antonio How are they received

7 What does Antoniorsquos speech as he faces the prospect of Shylockrsquos knife tell you about his character

8 How do Bassanio and Gratiano react to the looming prospect of Antoniorsquos demise

9 How does Portia (as Balthazar) use the law to turn the tables on Shylock

10 What does the Duke decree should happen to Shylock Why What happens to Shylockrsquos estate

11 What does Portia ask Bassanio as payment for her ldquoservicesrdquo What is his initial response What makes him change his mind

Act IV Scene 2 (each question carries 1frac12 marks)

1 What does Gratiano bring to Portia (Balthazar)

2 What does Nerissa plan on getting from Gratiano What does Portiarsquos comment suggest about men

ECO-10 280620 Topic-Supply AnalysisSHIFTING OF SUPPLY

But if there is change in factors other than the price of the commodity then either more is supplied at the same price or less supplied at the same price In such cases the price of the commodity remains constant but there is a change in other factors like change in the price of inputs change in technology of production change in price of other related goods change in taxation policy of the government etc For example there is an improvement in the technology of production of the commodity in question It leads to decrease in per unit of cost production of the commodity The firm is willing to sell more quantity of the commodity at the same price So the supply other commodity increases at the same price This increase in supply is shown by rightward shift of supply curve On the other hand if the firm uses inferior technology of production the cost of production per unit of the commodity increases The firm is willing to sell less quantity at the same price So the supply of the commodity decreases at the same price This decrease in supply is shown by leftward shift of the supply curve The above cases of increase and decrease in supply can be shown with the help of the following figures

Y INCREASE OF SUPPLY Price (Rs) s

P A s1

B

s

X` O s1 X

q q1

Y` Quantity demanded (in units)

Y DECREASE IN SUPPLY s2

s

price (Rs)

C

p A

s2

s

X` o X

q2 q

Y` Quantity demanded ( in units)

Main factors causing increase in supply or rightward shift of supply Curve(i) Fall in the price of other related goods

(ii) Fall in the price of inputsfactors(iii) Use of better technology in production(iv) Decrease in the rate of excise duty by government(v) If the objective of producer changes from profit maximization to salesMaximization

Main factors causing decrease in supply or leftward shift of supply curve(i) Increase in the price of other related goods(ii) Rise in the price of inputsfactors(iii) Use of inferior technology in production(iv) Increase in the rate of excise duty by the government(v) If the objective

Subject - Biology Topic ndash Chapter mdash6 PhotosynthesisSummary Execution

Today we will know about photosynthesis and its stages

Q1 What do you mean by photosynthesis The process by which living plants containing chlorophyll produce food

substances from carbon-di- oxide and water by using light energy Sunlight

6CO2 +12 H2O----------------------- C6 H12O6 + 6H2O + 6O2

Chlorophyll

Q2 What are the importance of photosynthesis I) Food for all Green plants trap solar energy by photosynthesis

process and supply food and energy for all living organisms either directly or indirectly

Ii) Oxygen to breathe in by product of photosynthesis is oxygen which is essential for all living organisms respiration

Q3 Write about two main phases of photosynthesis A Light dependent phase This phase occur in grana of chloroplast I) The chlorophyll on exposure to light energy becomes activated by

absorbing photons Ii) The absorbed energy is used in splitting the water molecules (H2O)

into its two components (H+ and OH- ) and releasing electron s 2H2O------------------------- 4H+ + 4e- +O2

Energy of 4 photons This reaction is known as photolysis

End products are H+ and oxygen water

B Light independent (Dark ) phase The reactions in this phase require no light energy

Here CO2 combine with H+ and produce glucose

Class XI

Subject Topic Summary ExecutionEVS Chapter-4 Legal

regimes for sustainable development

Environmental legislationEnvironmental legislation is the collection of laws and regulations pertaining to air quality water quality the wilderness endangered wildlife and other environmental factors The act ensures that matters important to the environment are thoroughly

Learn -The Forest (Conservation) Act 1980

considered in any decisions made by federal agencies

The Forest (Conservation) Act 1980 The Forest (Conservation) Act 1980 an Act of the Parliament of India to provide for the conservation of forests and for matters connected therewith or ancillary or incidental thereto It was further amended in 1988 This law extends to the whole of IndiaObjects and Reasons of the Forest Conservation Act

Deforestation causes ecological imbalance and leads to environmental deterioration Deforestation had been taking place on a large scale in the country and it had caused widespread concern The act seeks to check upon deforestation and de-reservation of forests

Subject Eng Literature (The Tempest ndash William Shakespeare) Topic Act II Scene 1 Lines 314 to 329 (End of scene)

[Students should read the original play and also the paraphrase given in the school prescribed textbook]Summary Questions amp Answers

Conspiracy of Antonio and Sebastian (Contd)

o As they approach Ariel appears again and wakes up Gonzalo by singing a tune in his ear Alonso also wakes up and they see both Sebastian and Antonio with drawn swords On being caught off guard they make up a story saying that they had heard a bellowing of bulls or lions

o They then moved to another part of the island

o Ariel at once rushes to Prospero to inform him of this development

SUMMING-UP of ACT-2 SCENE-1

(i) Among the survivors Ferdinand is separated from the rest which results in the disconsolate grief of Alonso as he took him for dead

(ii) The villainy of Antonio is confirmed

(iii) The supremacy of Prosperorsquos magic which resulted in the failure of the human conspiracy

(1)

(Act II Sc 1 L 311-325)SEBASTIAN Whiles we stood here securing your repose

Even now we heard a hollow burst of bellowing Like bulls or rather lions Didt not wake youIt struck mine ear most terribly

ALONSO I heard nothingANTONIO O rsquotwas a din to fright a monsters ear

To make an earthquake Sure it was the roarOf a whole herd of lions

ALONSO Heard you this GonzaloGONZALO Upon mine honour sir I heard a humming

And that a strange one too which did awake meI shaked you sir and cried As mine eyes opened I saw their weapons drawn There was a noiseThats verily rsquoTis best we stand upon our guardOr that we quit this place Lets draw our weapons

(i) Why has Prospero sent Ariel to Gonzalo and Alonso What does Ariel do to awaken Gonzalo

Prospero has already come to know by his magic powers the danger which threatens Gonzalo who had been Prosperorsquos friend and so he sent Ariel to preserve the lives of both Gonzalo and Alonso Prospero does not want that his scheme should remain unfulfilled Ariel begins to sing a song in Gonzalorsquos ears to awaken him(ii) Who are ready to carry out their plan Who takes steps to stop them Why does Gonzalo feel surprised after being awakened

Sebastian and Antonio are ready to carry out their plans They are standing with their swords drawn to kill Alonso and

(iv) We see two sets of contrasting characters Gonzalo-Adrian against Antonio-Sebastian

(v) The grief that works in Alonso can be perceived to his repentance for his association in Antoniorsquos crime against Prospero

Gonzalo Ariel takes steps to stop them from carrying out their nefarious scheme When Gonzalo is awakened by the song sung by Ariel into his ears he (Gonzalo) feels surprised because he sees Sebastian and Antonio standing with their swords drawn(iii) What reason do Sebastian and Antonio tell of drawing their swords when they are suspected by Alonso and Gonzalo

When Sebastian and Antonio are seen with their swords drawn they are looked with suspicion by Gonzalo and Alonso At first Sebastian tells them that as they stood here to guard them during their sleep they heard only a little before a sudden loud noise very much like the roaring of bulls or more probably that of lions Then Antonio follows him saying that this was a noise so terrible as to frighten even a monsterrsquos ears and this noise could even have shaken the earth and it was surely like the roaring of a multitude of lions Then seeing the danger they have drawn their swords Perhaps after hearing the terrible noise they (Gonzalo and Alonso) woke up from their sound sleep

(iv) What does Gonzalo tell Alonso about the strange noise What did he see on opening his eyes Gonzalo tells Alonso that he did not hear the sound of roaring but he heard a humming sound which was strange and which woke him up After waking up he gave him (Alonso) a shaking and a loud cry On opening his eyes he saw these two gentlemen standing with their swords drawn(v) What does Gonzalo suggest

Gonzalo suggests that there was a noise indeed and of that he has no doubt at all and suggests that the best course for them would be to remain alert and vigilant against any possible danger to their lives or to leave this place and move to some other part of the island

Class XIISubject Topic Summary Execution

Commerce

Chapter- Management

Today we will discuss about LEVELS OF MANAGEMENT

Levels of management is a series or chain of managerial positions from top to bottom It helps individuals to know their authority responsibilities and superior-subordinate relations among themselves There are mainly three levels of Management TOP LEVEL MANAGEMENTMIDDLE LEVEL MANAGEMENTLOWER LEVEL MANAGEMENT

Top level managementIt consists of members at the highest level in the management hierarchy This level includes Board Of Directors Chief Executive Managing Directors Chairman President Vice President

Rolefunctions of the top levelmanagement1To analyse evaluate and deal

with theexternal environment2 To determine the objectives and

policies of the business3 To strive for welfare and survival

of business

4 To create an organisational Framework consisting of authority responsibility relationship

Middle level management Congress of members or groups who are concerned with implementation of the policies let down by the top managementThis level includes head of the department such as finance manager marketing manager branch and regional managers departmental and divisional heads plant superintendent etc

Role of functions of the middle level management

1 To interpret the policies framed by top management

2 To assign duties and responsibilities to lower level managers

3 To select and appoint employees for middle and supervisory level and evaluate their performance

4 To co-operate with other departments for smooth functioning

Operational or supervisory level managementIt refers to the group are members who are concerned with execution of the work They are also known as fast line managers This level includes supervisor 4 men Section Officer clerk Inspector etc

Role of functions of the lower level management1 To plan and execute day-to-

day operations2 To supervise and control the workers3 To arrange materials and

tools to start the process and make arrangements for training

4 Today present workers grievance and suggestions before the management and

ensure safe and proper working conditions in the factory

Business Studies

Staff Appraisal Chapter- 10 Today let us start with a new chapter

Staff Appraisal

Meaning of Performance Appraisal

Performance Appraisal is the systematic evaluation of the performance of employees and to understand the abilities of a person for further growth and developmentThe supervisors measure the pay of employees and compare it with targets and plansThe supervisor analyses the factors behind work performances of employeesThe employers are in position to guide the employees for a better performance

Objectives of Performance Appraisal

Following are the objectives of Performance Appraisal

To maintain records in order to determine compensation packages wage structure salaries raises etc

To identify the strengths and weaknesses of employees to place right men on right job

To maintain and assess the potential present in a person for further growth and development

To provide a feedback to employees regarding their performance and related status

To provide a feedback to employees regarding their performance and related status

Importance of Performance Appraisal

Performance appraisal provides important and useful information for the assessment of employees skill

knowledge ability and overall job performance The following are the points which indicate the importance of performance appraisal in an organization

1 Performance appraisal helps supervisors to assess the work performance of their subordinates

2 Performance appraisal helps to assess the training and development needs of employees

3 Performance appraisal provides grounds for employees to correct their mistakes and it also provides proper guidance and criticism for employees development4 Performance appraisal provides reward for better performance

5 Performance appraisal helps to improve the communication system of the organization

6 Performance appraisal evaluates whether human resource programs being implemented in the organization have been effective

7 Performance appraisal helps to prepare pay structure for each employee working in the organization

8 Performance appraisal helps to review the potentiality of employees so that their future capability is anticipated

Geography

DRIANAGE The SubarnarekhaThe Subarnarekha and the Brahmaniinterposed between the Ganga and the Mahanadi deltas drain an area of 19300 sq kmand 39033 sq km respectively The drainage basins of these streams are shared byJharkhand Odisha west Bengal and Chhattisgarh The Brahmani is known as southKoel in its upper reaches in Jharkhand

The NarmadaThe Narmada rises in the Amarkantak hills of MadhyaPradesh It flows towards the West in a rift valleyformed due to a geological fault The total length of it is 1300 km All the tributaries of the

Q1 Name the two westward flowing rivers in the peninsular plateauA1 Narmada and Tapi are the only westward flowing rivers of the peninsular plateau

Q2 Differentiate between east-flowing rivers and west-flowing riversA2

East-flowing rivers

West-flowing rivers

Narmada are very short inlength Most of its tributaries join the main streamright anglesThe Narmada basin covers parts of Madhya Pradesh and Gujarat

The Tapi The Tapi rises in the Satpura ranges in the Betul listrictof Madhya Pradesh It flows in a rift valley parallel tothe Narmada but it is much shorter in length It coversparts of Madhya Pradesh Gujarat and MaharashtraThe length is about 724 km

The Sabarmati and the MahiThe Sabarmati rises in the Aravali hills and flows south-south-westwards for a distance of 300 kilometres to the Arabian Sea The Sabarmatibasin extends over an area of 21674 sq km in Rajasthan and Gujarat The Mahi rises inthe east of Udaipur and drains an area of 34842 sq km lying in Madhya PradeshRajasthan and Gujarat It flows south-westwards for a distance of 533 km before it fallsinto the Gulf of Khambhat

The ChambalThe Chambal rises near Mhow in the Vindhya Range and flows towards the northgenerally in a gorge upto Kota Below Kota it turns to the north-east direction and afterreaching Pinahat it turns to the east and runs nearly parallel to the Yamuna beforejoining it in the southern part of the Etawah district in Uttar PradeshMajor Rivers of India with their basin area (Sqkm)

Himalayan System Indus 321290Ganga 861404

Brahmaputra 187110Indus System

Jhelum 34775Beas 20303

Ganga System Yamuna 366223Ghaghra 127950

Peninsular RiversNarmada 98796

Tapi 65145Mahanadi 141600

Subarnarekha 19300Sabarmati 21674

Mahi 34842Godavari 312812

Godavari Krishna Kaveri Mahanadi are the east-flowing rivers

Narmada Tapi west-flowing rivers

They fall into the Bay of Bengal

They fall into Arabian Sea

These rivers form big deltas

These rivers form comparativelysmall deltas

Catchment areas of these rivers are larger

Catchment areas of these rivers are smaller

Krishna 2589488Cauveri 87900

Subject ndashBiology Topic ndashChapter -5 Inheritance amp Variations Summary ExecutionToday we will discussabout linkage and its classification

LINKAGE The tendency of the genes located on the same chromosome to stay together is

hereditary transmission Linked genes the genes responsible for this Genes that exhibit the process of linkage locates in the same chromosome The distance between the linked genes in a chromosome determines the strength

of linkage i e genes that are located close to each other show stronger linkage than that are located far from each other

COMPLETE LINKAGE It is the type of linkage showed by the genes that are closely located or are tightly

linked with each other as they have no chance of separatingby crossing over These genes are always transmitted together to the same gamete and the same

offspring In such condition only parental or non cross over type of gametes are formedINCOMPLETE KINKAGE It is type of linkage showed by the genes that are distantly located orare loosely

linked with each other because they have chance of separating by crossing over

SIGNIFICANCE i) It helps in holding the parental character togetherii) It checks the appearance of new recombination and helps in bringing the

hybrid population which resembles the original parents iii) Linked genes dilute the effects of undesirable traits

Subject Eng Literature (The Tempest ndash William Shakespeare) Topic Essay Questions (EQ-3)Question No 3

Give a character sketch of CalibanAnswer

The character of Caliban has been wonderfully conceived by Shakespeare as the manifestation of all that is gross and earthy ndash a sort of creature of the earth as Ariel is a sort of creature of the air

Calibanrsquos Physical Appearanceo Caliban is lsquofreckledrsquo a lsquomisshapen knaversquo not honoured with human shape

o Prospero calls him lsquothou tortoisersquo (Act I Sc 2 Line 317) Trinculo stumbling upon him describes him as ldquoA strange fish hellip Legged like a man And his fins like armsrdquo He ldquosmells like a fishrdquo (Act II Sc 2 Line 25)

o Prospero also calls him a ldquobeastrdquo (Act IV Sc 1 Line 140) and ldquoThis misshapen knaverdquo (Act V Sc 1 Line 268)

o Further it appears that in addition to his physical deformity his spiritual inferiority is also suggested by Prosperorsquos claim that his birth resulted from the union between his mother the witch Sycorax and the devil

Calibanrsquos ParentageWhen the play opens Caliban is twenty four years of age having been born on the island twelve years before the coming of Prospero His mother was the foul witch Sycorax who was banished from Algiers for ldquomischiefs manifold and sorceries terrible to enter human hearingrdquo (Act I Sc 2 Line 264) and the father was the Devil himself Thus

Caliban is a monster of evil and brute nature ugly deformed and stinking

Calibanrsquos Savage and Malignant Natureo Caliban is entirely a creature of the earth ndash gross brutal and savage He regards himself as the rightful possessor

of the island and Prospero as a usurper

o In his young age he was on good terms with Prospero He had consented to be received by Prospero at his house and to be educated by him He has learnt human language only to curse his master whom he abhors

o His beastly nature soon breaks out and ends in a vicious attack on Miranda This opens the eye of Prospero who becomes severe to him and enforces his service by threats and violence

o Prospero uses him to make dams for fish to fetch firewood scraper trenches wash dishes and keep his cell clean

Calibanrsquos Hatred for ProsperoA profound hatred for Prospero has taken hold of Caliban It springs from a sense of his being dispossessed and ill-treated He would kill Prospero if he could but he knows the power of Prosperorsquos lsquobookrsquo Hence he transfers his allegiance to Stephano who seems like a god to him He also incites the two drunken associates to batter the skull of Prospero when he sleeps in the afternoon

Caliban Shows Considerable Intelligenceo He has learnt Prosperorsquos language

ldquoYou taught me language and my profit onrsquot (Act II Sc 2 Lines 86-89)Is I know how to curserdquo

o He is well aware of the futility of arguing with one who has more power than he has

ldquoI must obey his art is such power (Act I Sc 2 Lines 373-376)It would control my damrsquos god SetebosAnd make a vassal of himrdquo

o He realizes the importance of Prosperorsquos books

ldquoRemember (Act III Sc 2 Lines 89-92)First to possess his books for without themHersquos but a sot as I am nor hath notOne spirit to commandrdquo

o He knows the value of stealth when attacking the enemy

ldquoPray you tread softly that the blind mole may not (Act IV Sc 1 Lines 194-195)Hear a foot fall we now are near his cellrdquo

o Caliban has a better set of values than Stephano and Trinculo They are distracted from their plan by their greed for Prosperorsquos rich garments Only Caliban realizes that such a finery is unimportant

ldquoLeave it alone thou fool it is but trashrdquo (Act IV Sc 1 Lines 224)

Caliban is not a good judge of characterCaliban is not a good judge of character He decides for example that Stephano is a god because he dispenses lsquocelestial liquorrsquo (Act II Sc 2 Line 115) but then it must be remembered that he has only known his mother Sycorax Prospero Miranda and the spirits that torture him However he quickly discovers his error of judgementrdquo

ldquoWhat a thrice-double ass (Act V Sc 1 Lines 295-297)Was I to take this drunkard for a godAnd worship this dull foolrdquo

Calibanrsquos Imaginative NatureIf Caliban is sub-human in what has been said above he is human in the respect of the poetic side of his character He listens to music with rapture He tells of the beautiful dreams in which heaven rains treasures upon him and which upon waking he yearns to renew One of the most poetic passages in whole play is Calibanrsquos description of the island

to Stephano and Trinculo

ldquoBe not afeard The isle is full of noises (Act III Sc 2 Lines 135-143)Sounds and sweet airs that give delight and hurt notSometimes a thousand twangling instrumentsWill hum about mine ears and sometime voicesThat if I then had waked after long sleepWill make me sleep again and then in dreamingThe clouds methought would open and show richesReady to drop upon me that when I wakedI cried to dream againrdquo

Caliban - Less Ignoble Than Some OthersCalibanrsquos motive for murder is less dishonourable than that of Antonio and Sebastian They plan to kill Alonso to gain his power and wealth Caliban merely wants revenge and the return of lsquohisrsquo island

Conclusiono Calibanrsquos character is not portrayed very clearly in the play and hence we cannot decide whether he is a poor

savage being grossly maltreated by Prospero or whether he is evil and must therefore be kept in bondage or enslavement

o Caliban is contrasted with Ariel who is a spirit and thus swift and uninterested in physical activitieso Caliban is also contrasted with Prospero who is the all-powerful master of the island and of the destiny of all

those on the islando Caliban is also contrasted with civilized man showing him to be less evil than Antonio and Stephano and less

materialistic than Stephano and Trinculoo Caliban has suffered at the hands of Prospero and he has learnt to curse by listening to Prosperorsquos abuse He

certainly believes that Prospero has deprived him of his birthrighto Finally the character Caliban is thought to be one of Shakespearersquos masterpieces The complexity of the character

is reflected in the large volume of critical discussion that has grown around it

ECO ndash12 Topic-Forms of market

MonopolyMonopoly is a market structure in which there is a single seller there are no close substitutes for the commodity produced by the firm and there are barriers to entry Example Indian Railways which is operated under government of India Monopoly also implies absence of competitionFeatures of Monopoly Monopoly is characterized by1 Single Seller In monopoly there is only one firm producing the product The whole industry consists of this single firm Thus under monopoly there is no distinction between firm and industry Being the only firm there is significant control of the firm over supply and price Thus under monopoly buyers do not have the option of buying the commodity from any other seller They have to buy the product from the firm or they can go without the commodity This fact gives immense control to the monopolist over the market

2No Close Substitute There are no close substitutes of the product produced by the monopolist firm If there are close substitutes of the product in the market it implies presence of more than one firm and hence no monopoly In order to ensure a total of control over the market by the monopolist firm it is assumed that there are no close substitutes of the product

3 No Entry amp Exit Monopoly can only exist when there is strong barriers before a new firm to enter the market In fact once a monopoly firm starts producing the product no other firm can produce the same One reason for this is the ability of the

monopolist to produce the product at a lower cost than any new firm who thinks to enter the market If a new firm who knows that it cannot produce at a lower cost than the monopolist then that firm will never enter the market for fear of losing out in competition Similarly the monopolist who is operating for a long time may be enjoying reputation among its customers and is in a better position to use the situation in its own benefit A new firm has to take long time to achieve this and so may not be interested to enter the market

4 Price Maker Being the single seller of the product the monopolist has full control over the pricing of the product On the other hand if there is a large number of buyers in the market so no single buyer exercises any significant influence over price determination Thus it is a sellerrsquos market So monopoly firm is a price maker

5 Price Discrimination Having considerable control over the market on account of being single seller with no entry of other firms the monopolist can exercise policy of price discrimination it means that the monopolist can sell different quantities of the same product to a consumer at different price or same quantity to different consumers at different prices by adjudging the standard of living of the consumer

6 Shape of Demand Curve Since a monopolist has full control over the price therefore he can sell more by lowering the price This makes the demand curve downward sloping

Subject Ac-12 290620 Topic- retirement Model sumThe Balance Sheet of Rohit Nisha and Sunil who are partners in a firm sharing profits according to their capitals as on 31st March 2014 was as under

Liabilities Amount Assets Amount (Rs) (` Rs)

Creditors 25000 Machinery 40000Bills Payable 13000 Building 90000General Reserve 22000 Debtors 30000Capital Less Provision for Rohit 60000 Bad debts 1000

29000 Nisha 40000 Stocks 23000 Sunil 40000 140000 Cash at Bank 18000

200000 200000

On the date of Balance Sheet Nisha retired from the firm and following adjustments were made(i) Building is appreciated by 20(ii) Provision for bad debts is increased to 5 on Debtors(iii) Machinery is depreciated by 10(iv) Goodwill of the firm is valued at Rs 56000 and the retiring partnerrsquos share is adjusted

(v) The capital of the new firm is fixed at Rs120000 Prepare Revaluation Account Capital Accounts of the partner and Balance Sheet of the new firm after Nisharsquos retirement Revaluation AccountDr Cr

Particulars Amount Particulars Amount (`Rs) (Rs`)

Provision for Bad debt Ac 500 Building Ac 18000Machinery Ac 4000Profit transferred toCapital Accounts (3 2 2)Rohit 5786Nisha 3857Sunil 3857

13500

18000 18000

Capital Account

Dr Cr

Particulars Rohit Nisha Sunil Particulars Rohit Nisha Sunil (Rs`) (Rs`) (`Rs) (Rs`) (Rs`) (Rs`)

Sunilrsquos Capital ac 9600 mdash 6400 Balance bd 60000 40000 40000Bank - 66143 - General Reserve 9428 6286 6286Balance cd 72000 mdash 48000 Revaluation (Profi 5786 3857 3857 Rohitrsquos Capital Ac mdash 9600 mdash

Sunilrsquos Capital Ac 6400 Bank 6386 - 4257

81600 66143 54400 81600 66143 54400

Balance Sheet as at 31st March 2014

Liabilities Amount Assets Amount (Rs`) (Rs`)

Creditors 25000 Building 108000Bank overdraft 37500 Machinery 36000

Bills Payable 13000 Debtors 30000Capital Less ProvisionRohit 72000 for Bad debts 1500 28500Sunil 48000 120000 Stock 23000

195500 195500

Working Notes (i) (a) Profit sharing ratio is 60000 40000 40000 ie = 3 2 2(b) Gaining Ratio Rohit = 35 ndash 37 = 2135 ndash 1535 = 635Sunil = 25-27 = 1435 ndash 1035 = 435= 635 435= 6 4 = 3 2(c) Nisha Share of Goodwill = Rs 56000 times 27 = Rs16000Share of Goodwill in the gaining ratio by the existing partner ieRohit = Rs16000 times 35 = Rs 9600Sunil = Rs 16000 times 25 = Rs 6400

The journal entry isRohitrsquos Capital Ac Dr 9600Sunilrsquos Capital Ac Dr 6400 To Nisharsquos Capital Ac 16000(Share of Goodwill divided into gaining ratio)

  • 1 Static Friction
  • The frictional force that acts between the surfaces when they are at rest with respect to each other is called Static Friction
    • Static Friction Examples
      • 2 Sliding Friction
        • Examples Of Sliding Friction
          • 3 Rolling Friction
            • Examples Of Rolling Friction
              • Objects and Reasons of the Forest Conservation Act
Page 31:  · Web viewSubject . Topic . Summary . Execution . English 1 . Sounds of animals . Hens –cackle Horses –neigh Lions –roar Owls –hoots Snake –hiss. English 2 . Mother’s

As the Equator divides the Earth into two equal halves the sun rays fall directly on the equator twice in a year Equinoxes means equal Spring EquinoxOn 21st March sunrays fall directly on the equator On that day the duration of day and night both are equal ( 12 hours day and 12 hours night) on every places located on equator This day is called as Spring EquinoxAutumn EquinoxOn 23rd September sunrays fall directly on the equator On that day the duration of day and night both are equal ( 12 hours day and 12 hours night) on every places located on equator This day is called as Autumn Equinox

SOLSTICES ndash SUMMER AND WINTERDue to inclination of the Earth on its axis and the apparent movement of the sun the sun rays fall directly on both tropics once in a year Solstice is a Latin word which mean ldquothe Sun standing stillrdquoSummer SolsticesAfter 21st March there is an apparent movement of the Sun to the north of the equator The apparent northward movement up to 21st June when the Sun appears overhead at the Tropic of Cancer (22frac12degN) The sun appears to stand still at this position and then moves southwards towards the equator This position of the Sun on 21st June is known as Summer Solstices On that day the duration of day and night both are equal ( 12 hours day and 12 hours night) on every places located on Tropic of Cancer (22frac12degN)Winter solstices The apparent southward movement of the Sun continues beyond the equator till 22nd

December On this day the Sun is overhead at the Tropic of Capricorn

the duration of day and night both are equal ( 12 hours day and 12 hours night) on every places located on equator This day is called as Spring Equinox

Q2 What do you mean by EquinoxA2 Equinoxes means equal It is use to explain the equal duration of day and night ( 12 hours day and 12 hours night) on the Earth

Q3 On which date the longest day in Tropic of CancerA3 21st June

Q4 What is the meaning of SolsticeA4 Solstice is a Latin word which mean ldquothe Sun standing stillrdquo

Q5 Which is the longest day in southern hemisphereA5 22nd December

Q6 On what date does the Arctic Circle experience the lsquoMidnight SunrsquoA6 On 21 June the Arctic Circle experiences the lsquoMidnight Sunrsquo

Q7 What is cause of Midnight Sun in NorwayA7 During the summer solstice (21 June) the North Pole is inclined towards the Sun Therefore the duration of sunlight or daytime increases from 12 hours at the Equator to 24 hours at the Arctic Circle and beyond Thatrsquos why The region beyond the Arctic Circle especially Norway is known as the Land of the Midnight Sun because there the Sun does not rise or set on 21 June

Q8 Match the column A with BA B

Summer Solstice 21st March

Autumn Equinox 23rd

September

Winter Solstice 21st June

(22frac12degS) This position of the Sun is referred to as the Winter Solstice because it marks the winter season in the Northern Hemisphere On that day the duration of day and night both are equal ( 12 hours day and 12 hours night) on every places located on Tropic of Capricorn (22frac12degS)SEASONS AND DURATION OF DAY AND NIGHT During the equinoxes all places on the Earth have 12 hours of day and 12 hours of night Due to the revolution of the Earth round the Sun on an inclined axis the duration of day and night varies according to seasons and the latitude of a placeDuring the summer solstice (21 June) the North Pole is inclined towards the Sun Therefore the duration of sunlight or daytime increases from 12 hours at the Equator to 24 hours at the Arctic Circle and beyondThe region beyond the Arctic Circle especially Norway is known as the Land of the Midnight Sun because there the Sun does not rise or set on 21 JuneAt the North Pole there will be six months of daylight The Sun will be seen always above the horizon at a low angle At 66degN 24 hours of sunlight can be seen only on 21 June Hammerfest in northern Norway is a place of tourist attraction for observing the phenomenon of the Midnight Sun This place has continuous daylight from 13 May to 29 July This place is easily accessible to tourists and has hotels and other facilities The view of the midnight Sun from here is enthrallingIn the Southern Hemisphere the duration of daylight decreases from 12 hours at the equator to 0 hours beyond the Antarctic Circle In the South Polar Region there is 24 hours of darkness The Sun is always below the horizon In the Southern Hemisphere which experiences winter the duration of night-time is longer than the duration of daylight

Spring Equinox 22nd

December

A8 A B

Summer Solstice 21st June

Autumn Equinox 23rd

September

Winter Solstice 22nd

December

Spring Equinox 21st March

During winter solstice (22 December) the South Pole is inclined towards the Sun The Southern Hemisphere experiences summer and the Northern Hemisphere has winter Therefore the duration of daylight or sunlight is greater in the Southern Hemisphere than in the Northern HemisphereThe duration of daylight increases from 12 hours at the equator to 24 hours beyond the Antarctic Circle The South Polar Region has 24 hours of sunlight for many days continuously At the South Pole there will be six months of sunlight The Sun will always be seen at a low angle above the horizon In the Northern Hemisphere the duration of daylight will decrease from 12 hours at the equator to 0 hours at the Arctic Circle There are 24 hours of darkness in the North Polar region The duration of night is greater than the duration of daylight as one move northwards from the Equator It is evident from the above table that the duration of daylight is 12 hours throughout the year at the equator only As one moves away from the equator the seasonal variations in the duration of daylight increase The seasonal variations in the duration of daylight are maximum at the Polar Regions

Subject Eng Literature (The Merchant of Venice ndash William Shakespeare)Topic Act II Scene 7 Lines 36 to 80 (End of scene ) [Students should read the original play and also the paraphrase provided]

Summary Questions amp AnswersThe Prince then examines the inscription on the silver casket which says ldquoWho chooseth me shall get as much as he deservesrdquo The Prince says that he deserves Portia more than anybody else because of his high rank his noble birth and his great wealth and power But then he argues that silver is ten times

(1) (Act II Sc 7 L 39-47)

From the four corners of the earth they come

To kiss this shrine this mortal breathing saint

The Hyrcanian deserts and the vasty wildsOf wide Arabia are as through-fares now

inferior to gold and therefore he cannot believe that the portrait of such a beautiful lady as Portia can be contained in the silver casket He decides to see the inscription on the golden casket before making his decision

The Prince goes to examine the inscription on the golden casket which says ldquoWho chooseth me shall get what many men desirerdquo The Prince believes that the whole world desires to possess Portia otherwise so many suitors would not have come from all corners of the world for winning Portia Some of them have come from the distant lands of Persia and Arabia The deserts of Persia (Hyrcanian deserts) and the boundless desolate lands of Arabia have been crossed by the Princes seeking the hand of Portia He contrasts this casket containing Portiarsquos portrait with the old English gold coin bearing the image of the archangel (angel of the highest rank) He goes on to remark that while the figure of the archangel is engraved (Insculped) upon the English coin the picture of Portia who is beautiful as an angel lies hidden inside one of the caskets namely the Golden Casket (Golden Bed)

On the basis of his assessment of the inscription on the golden casket the Prince decides to choose the golden casket He asks for the key and opens the golden casket only to find therein an empty human skull holding a roll of

For princes to come view fair PortiaThe watery kingdom whose ambitious headSpets in the face of heaven is no barTo stop the foreign spirits but they comeAs orsquoer a brook to see fair Portia

(i) Explain the occasion for the above mentioned speech

These are the comments of the Prince of Morocco after he reads the inscription on the golden casket His mental process is revealed to us in these words We find him debating within himself as to which casket he should choose

(ii) What light does the above speech throw on the personality of Prince of Morocco

From the above mentioned speech we come to know that the Prince of Morocco is keen to marry Portia He is the type of person who is easily taken away by outward appearance He is in love with Portia because of her beauty

(iii) What information can you gather about Portia from the above mentioned lines

The given speech shows that Portia is a very beautiful lady She must be possessed of good qualities because many suitors come to her place from all over the world with a desire to get married to her The Prince of Morocco is so impressed by her beauty that he calls her a saint According to him the whole world is desirous of having her

(iv) Elucidate the significance of the first two lines

In these lines the Prince of Morocco pays a compliment to Portia These lines show his admiration for her He says that people come from all parts of the world to see fair Portia

(v) Explain the meaning of the last four lines of the

passage

In these lines the Prince of Morocco says that even the vast oceans which throw a challenge at the sky are unable to prevent men from coming to Portiarsquos place to have a glimpse of her These lines are also a tribute to Portiarsquos beauty and good qualities Many men voyage across the ocean treating it as a mere stream to see the beautiful Portia

paper in which is written that whoever happens to be guided by the glitter of things is invariably deceived

On reading the scroll the Prince says that he is too sad at heart to speak a more formal farewell and leaves with his followers amidst a sound of trumpets

After the Prince of Morocco leaves Portia remarks that the Prince is a gentle fellow but she is rid of him May all persons of his nature make a similar choice

IMPORTANT PASSAGES EXPLAINED

(Act II Sc 7 L 39-43)From the four corners of the earth they come

To kiss this shrine this mortal breathing saintThe Hyrcanian deserts and the vasty wildsOf wide Arabia are as through-fares nowFor princes to come view fair Portia

Context

This passage occurs in Act II Scene 7 in The Merchant of Venice This is part of the speech made by the Prince of Morocco

(2)

(Act II Sc 7 L 48-53)

MOROCCO One of these three contains her heavenly pictureIst like that lead contains her

Twere damnation To think so base a thought it were too grossTo rib her cerecloth in the obscure graveOr shall I think in silver shes immurdBeing ten times undervalued to tried gold

(i) What meaning does the Prince of Morocco find out of the inscription of the golden casket What have Belmont and Portiarsquos house been called and why

The inscription on the golden casket is ldquoWho chooseth me shall gain what many men desirerdquo The Prince finds out that it means that the chooser of the golden casket will get Portia because many men desire her In fact the entire world desires her Because of the coming of many suitors to Belmont from different countries in order to win Portiarsquos hand Belmont has become a centre of pilgrimage and her house is the shrine where saintly Portia is installed

(ii) What does the Prince of Morocco do before making the final choice of the casket Which is the correct casket and who will win Portiarsquos hand

The Prince of Morocco surveys and analyses the inscriptions on the casket of lead silver and gold Before making the final choice like a very systematic and methodical person he once again considers the claims of the caskets The casket containing Portiarsquos picture is the correct casket and the person choosing it will win Portiarsquos hand

Explanation

While praising Portia the Prince of Morocco conceives Portia as a goddess whose image is placed inside one of the caskets Many suitors are coming from far and wide the north and the south the east and the west (Four corners) in order to try their luck Some of them have come from the distant land of Persia and Arabia The deserts of Persia (Hyrcanian deserts) and the boundless desolate lands of Arabia have been crossed by the Princes seeking the hand of Portia All this shows that Portia is indeed the most beautiful lady of the world

(iii) What does the Prince of Morocco say in his estimation while examining the motto on the silver casket What does he find in the golden casket

While examining the motto on the silver casket which says ldquoWho chooseth me shall get as much as he deservesrdquo Morocco says that in his own estimation he surely deserves Portia in all respects ndash rank birth wealth etc

He chooses the golden casket When he opens it he finds an empty human skull holding a scroll in which it is written that those who are attracted by the glittering outside of things are always deceived as Morocco has been deceived

(iv) What kind of nature does the Prince of Morocco have

The Prince of Morocco has a simple nature who does not look deeply into the inner meaning of things but is dazzled by the outward appearance of gold He is inclined to over-estimate his own value and does not realize that it is a duty to ldquogive and hazardrdquo To say that he will not hazard for lead shows that he misreads the true meaning of the inscription which is that he should be prepared to ldquohazard all he hathrdquo for Portia So his feeling is only one of fascination and romantic attraction

(v) Do you think that the lottery of the caskets is not a matter that will be determined by chance

In fact the lottery of the casket is not a matter that will be determined by mere chance but that it is a true test of character and of sincerity which is amply proved not only by Moroccorsquos choice but also by the arguments which he uses to help him in his choice

(Act II Sc 7 L 55-59)

They have in England

A coin that bears the figure of an angelStamped in gold but thats insculpd uponBut here an angel in a golden bedLies all within

Context

(3)

(Act II Sc 7 L 63-77)A carrion Death within whose empty eye

There is a written scroll Ill read the writing

All that glisters is not goldOften have you heard that toldMany a man his life hath soldBut my outside to beholdGilded tombs do worms infoldHad you been as wise as boldYoung in limbs in judgment oldYour answer had not been inscrolld

This passage occurs in Act II Scene 7 in The Merchant of Venice This is part of the speech made by the Prince of Morocco

Explanation

In this passage the Prince of Morocco bestows high praise on Portia whose hand he is seeking He contrasts this casket containing Portiarsquos portrait with the old English gold coin bearing the image of the archangel (angel of the highest rank) He goes on to remark that while the figure of the archangel is engraved (Insculped) upon the English coin the picture of Portia who is beautiful as an angel lies hidden inside one of the caskets namely the Golden Casket (Golden Bed) In the day of Elizabeth silver was ten times inferior in value to gold Therefore the Prince of Morocco believing that Portiarsquos portrait is contained in the Golden Casket decides to choose the Golden Casket

Fare you well your suit is coldCold indeed and labour lostThen farewell heat and welcome frostmdashPortia adieu I have too grievd a heartTo take a tedious leave Thus losers part

(i) What reward does the Prince of Morocco get after making a wrong choice of the Casket How does he feel

After making the wrong choice in selecting the casket of gold the Prince of Morocco as a reward earns a rebuke in the form of a scroll tucked in the empty eye-socket of a skull kept in the casket of gold The Prince is shocked and disappointed He becomes all the more sad and dejected when he reads the scroll which points to his foolishness in being misled by the appearance and outward show as indicative of its worth

(ii) How does the Prince respond after reading the scroll

After reading the scroll the Prince though upset accepts the result with good grace and decorum befitting a royal suitor and true sportsman He says that his love-suit is really cold otherwise he would have chosen correctly but now his efforts have been in vain So he bids farewell to Portia to the warmth and enthusiasm of love and welcomes the cold and bitterness of dejection and misery of life which lies ahead

(iii) What request does he make to Portia and why

After being failure in his mission he requests Portia to give him permission to leave at once because he is too sad to undergo the tediousness of a formal leave-taking He tells that it is the manner in which defeated persons part unceremoniously

(iv) Explain the following lines

ldquoAll that glisters is not goldOften have you heard that toldMany a man his life hath soldBut my outside to beholdGilded tombs do worms infoldrdquo

Mere glitter does not make a metal to be gold Man has often been warned against appearance but it has been of no use Many people have sacrificed their lives only to seek the outer appearance of gold Worms are found inside the gilded

monuments

Class XSubject Topic Summary Execution

Hindi 2ndlang

नया रासता भाग 6 मायाराम 0ी घर म धनी मल 0ी और उनी बटी सरिरता ी ही चचा बनी रहती थी अमिमत ो इसम ोई रलिच ना थी वह धनी घर ी लडी स शादी र सवय ो बचना नही चाहता था उसा भी सवाणिभमान ह ईशवर ी पा

स उस पास पस ी ोई मी नही थी अभी उसन फकटरी ही लगाई थी उसी समझ बाहर था कि उस घर वालो ा झाव पस ी तरफ कयो

ह उसन मा स सवाल किया कि मा तम सरिरता स मरी शादी कयो रना चाहती हो मा न उस समझाया कि वह दखन म बरी नही ह और किफर खानदान अचछा

ह वह ए शल गरहणी रप म घर सभाल सगी अमिमत न मा ो इस बात ा एहसास राया कि मीन सबध लिलए मना रन पर उस दिदल

पर कया बीती होगी मा और अमिमत ी लडी बार म ाफी बात हईमा ा झाव सरिरता ी तरफ था कयोकि वह घर पर अचछा दह0 लर आ रही

थी अमिमत न अपनी मौसी ी बरी हालत बार म बताया कि किस तरह वह बड घर ी खानदानी बटी लाई थी और आ0 उसी हालत कितनी खराब ह लाई थी बहकलब 0ाती ह और बचचो ो भी नही दखती ह बात चल ही रही

थी कि तभी ए ार बाहर आर री धनी मल0ी घर अदर आए और पीछ स डराइवर फल ी ए टोरी लर आया अदर आए और पीछ स

डराइवर ए टोरी फल ी लर आया अमिमत ो फल ी पटी बरी लग रही थी अमिमत न पछ लिलया यह फल कयो ल आए ह प इन सब ी कया

0ररत थी उनो न 0वाब दिदया कि 4 पटी शमीर स मगाए थ अमिमत ो या सनर करोध आ गया तभी उस किपता 0ी आ गए उन आत ही अमिमत उठर बाहर चला गया वहा वहा मा पास आर बठ गया और बोला

अभी रिरशता तय नही हआ और धनी मल 0ी धनी मल 0ी फल ी पटी लर चलआय मा न समझाया कि 0ब सबध 0ड 0ाता ह तो खाली हाथ नही

आत अमिमत न मा स हा कि तम सबन सरिरता ो इस घर म लान ी ठान रखी ह धनीमल 0ी उस दिदन सरिरता ो दखन ी तारीख तय रन आय थ

Commercial Studies

Banking Nowadays Bank provide easy and quick services through internet facilities methods of Banking is called internet bankingIn order to save the time and money involved in visiting Bank branches people increasingly prefer to have internet banking

There are different modes of doing internet banking or transferring money through online They areReal Time Gross Settlement (RTGS)National Electronic Fund Transfers (NEFT)

1

Question

1) Explain the term RTGS Write the features of RTGS

Answer)The acronym RTGS stands for Real Time Gross Settlement which may be defined as the continuous real time settlement of funds transfer individually on and order by order basis without netting lsquoReal timersquo may be defined as the processing of instructions at the time they are received rather than at some letter time lsquoGross settlementrsquo may be defined as the settlement of transfer instructions which occurs

individually

Features of RTGS1It is the continuous settlement of

funds transfer individually on an order by order basis

2RTGS facility is provided only by CBS core banking solution enabled Bank branches

3Amount charged from the customer for RTGS transactions vary from bank to bank

2) Explain the term NEFT Write the features of NEFT

Answer) National electronic funds transfer may be defined as a nationwide system that facilitates individuals Farms and copper operates to electronically transfer funds from any bank branch to any individual farm or corporate having an account with any other bank branch in the country

Features of NEFT2 Transfer can be made 7 times on

weekdays and 6 times on Saturday

3 NEFT cannot be used to receive foreign remittances

4 NEFT transaction takes place in batches

5 A bank branch must be NEFT enabled to become a part of NEFT fund transfer network

6 There is no maximum or minimum amount that can be transferred through NEFT when one bank has a bank account

English Language

CompositionEssay

A composition is an art of creating a piece of writing on any topic or subject It is the writing correctly beautifully and clearly in order to make some interesting reading Structure of the composition

Introduction ( you lay the foundation for your composition)

Body (it constitutes the main part of the essay)

Conclusion (final statement that leaves a lasting impression)

Kinds of essays1 The Narrative essay2 The descriptive essay3 The reflective essay4 The argumentative essay

Write a composition on any one of the following topics (350- 400 words)

1 Friendship Or2 The first day of your school

Subject Eng Literature (The Merchant of Venice ndash William Shakespeare)Topic Act V Scene 1 Lines 127 to 158 (Nerissa helliphellip The clerk will nersquoer wear hair onrsquos face that had it) [Students should read the original play and also the paraphrase given in the school prescribed textbook]

Summary Revision Questions o Soon thereafter Bassanio Gratiano

and Antonio arrive

o Bassanio tells Portia that he is feeling as if it is morning because of the presence of Portia who is shining like the sun When Antonio is introduced by Bassanio to Portia she tells Bassanio that he should be grateful to Antonio who took so much trouble on his account even to the extent of risking his life

o Nerissa starts quarrelling with Gratiano and demands that he show her the ring she had presented to him and which she had warned him not to lose She suspects that Gratiano must have presented the ring to some young woman and not to the lawyerrsquos clerk as he repeatedly says and assures

Answer the following questions to check your preparation of Act IV Scenes 1 and 2

You must attempt only after you have completed your preparation of Act IV The answers must be in complete sentences using textual evidence (with citation) when necessary

[It would be in your own interest to attempt the above questions honestly totally refraining from consulting your textbook or your notes during answering After completion you should correct the paper yourself consulting the textbooknotes etc and award marks as specified Please let me know the marks you scored through WhatsApp in the group or to my personal WhatsApp]

Act IV Scene 1 (each question carries 2 marks)

1 What did the Duke try to do for Antonio

2 Why does Shylock refuse to show mercy How does he justify his stance

3 Why does Antonio say he is ready to die 4 What information is contained in Bellariorsquos letter

5 Why does Portia (as Balthazar) assert that Shylock must show mercy How does he respond

6 What offers are made to Shylock to get him to spare Antonio How are they received

7 What does Antoniorsquos speech as he faces the prospect of Shylockrsquos knife tell you about his character

8 How do Bassanio and Gratiano react to the looming prospect of Antoniorsquos demise

9 How does Portia (as Balthazar) use the law to turn the tables on Shylock

10 What does the Duke decree should happen to Shylock Why What happens to Shylockrsquos estate

11 What does Portia ask Bassanio as payment for her ldquoservicesrdquo What is his initial response What makes him change his mind

Act IV Scene 2 (each question carries 1frac12 marks)

1 What does Gratiano bring to Portia (Balthazar)

2 What does Nerissa plan on getting from Gratiano What does Portiarsquos comment suggest about men

ECO-10 280620 Topic-Supply AnalysisSHIFTING OF SUPPLY

But if there is change in factors other than the price of the commodity then either more is supplied at the same price or less supplied at the same price In such cases the price of the commodity remains constant but there is a change in other factors like change in the price of inputs change in technology of production change in price of other related goods change in taxation policy of the government etc For example there is an improvement in the technology of production of the commodity in question It leads to decrease in per unit of cost production of the commodity The firm is willing to sell more quantity of the commodity at the same price So the supply other commodity increases at the same price This increase in supply is shown by rightward shift of supply curve On the other hand if the firm uses inferior technology of production the cost of production per unit of the commodity increases The firm is willing to sell less quantity at the same price So the supply of the commodity decreases at the same price This decrease in supply is shown by leftward shift of the supply curve The above cases of increase and decrease in supply can be shown with the help of the following figures

Y INCREASE OF SUPPLY Price (Rs) s

P A s1

B

s

X` O s1 X

q q1

Y` Quantity demanded (in units)

Y DECREASE IN SUPPLY s2

s

price (Rs)

C

p A

s2

s

X` o X

q2 q

Y` Quantity demanded ( in units)

Main factors causing increase in supply or rightward shift of supply Curve(i) Fall in the price of other related goods

(ii) Fall in the price of inputsfactors(iii) Use of better technology in production(iv) Decrease in the rate of excise duty by government(v) If the objective of producer changes from profit maximization to salesMaximization

Main factors causing decrease in supply or leftward shift of supply curve(i) Increase in the price of other related goods(ii) Rise in the price of inputsfactors(iii) Use of inferior technology in production(iv) Increase in the rate of excise duty by the government(v) If the objective

Subject - Biology Topic ndash Chapter mdash6 PhotosynthesisSummary Execution

Today we will know about photosynthesis and its stages

Q1 What do you mean by photosynthesis The process by which living plants containing chlorophyll produce food

substances from carbon-di- oxide and water by using light energy Sunlight

6CO2 +12 H2O----------------------- C6 H12O6 + 6H2O + 6O2

Chlorophyll

Q2 What are the importance of photosynthesis I) Food for all Green plants trap solar energy by photosynthesis

process and supply food and energy for all living organisms either directly or indirectly

Ii) Oxygen to breathe in by product of photosynthesis is oxygen which is essential for all living organisms respiration

Q3 Write about two main phases of photosynthesis A Light dependent phase This phase occur in grana of chloroplast I) The chlorophyll on exposure to light energy becomes activated by

absorbing photons Ii) The absorbed energy is used in splitting the water molecules (H2O)

into its two components (H+ and OH- ) and releasing electron s 2H2O------------------------- 4H+ + 4e- +O2

Energy of 4 photons This reaction is known as photolysis

End products are H+ and oxygen water

B Light independent (Dark ) phase The reactions in this phase require no light energy

Here CO2 combine with H+ and produce glucose

Class XI

Subject Topic Summary ExecutionEVS Chapter-4 Legal

regimes for sustainable development

Environmental legislationEnvironmental legislation is the collection of laws and regulations pertaining to air quality water quality the wilderness endangered wildlife and other environmental factors The act ensures that matters important to the environment are thoroughly

Learn -The Forest (Conservation) Act 1980

considered in any decisions made by federal agencies

The Forest (Conservation) Act 1980 The Forest (Conservation) Act 1980 an Act of the Parliament of India to provide for the conservation of forests and for matters connected therewith or ancillary or incidental thereto It was further amended in 1988 This law extends to the whole of IndiaObjects and Reasons of the Forest Conservation Act

Deforestation causes ecological imbalance and leads to environmental deterioration Deforestation had been taking place on a large scale in the country and it had caused widespread concern The act seeks to check upon deforestation and de-reservation of forests

Subject Eng Literature (The Tempest ndash William Shakespeare) Topic Act II Scene 1 Lines 314 to 329 (End of scene)

[Students should read the original play and also the paraphrase given in the school prescribed textbook]Summary Questions amp Answers

Conspiracy of Antonio and Sebastian (Contd)

o As they approach Ariel appears again and wakes up Gonzalo by singing a tune in his ear Alonso also wakes up and they see both Sebastian and Antonio with drawn swords On being caught off guard they make up a story saying that they had heard a bellowing of bulls or lions

o They then moved to another part of the island

o Ariel at once rushes to Prospero to inform him of this development

SUMMING-UP of ACT-2 SCENE-1

(i) Among the survivors Ferdinand is separated from the rest which results in the disconsolate grief of Alonso as he took him for dead

(ii) The villainy of Antonio is confirmed

(iii) The supremacy of Prosperorsquos magic which resulted in the failure of the human conspiracy

(1)

(Act II Sc 1 L 311-325)SEBASTIAN Whiles we stood here securing your repose

Even now we heard a hollow burst of bellowing Like bulls or rather lions Didt not wake youIt struck mine ear most terribly

ALONSO I heard nothingANTONIO O rsquotwas a din to fright a monsters ear

To make an earthquake Sure it was the roarOf a whole herd of lions

ALONSO Heard you this GonzaloGONZALO Upon mine honour sir I heard a humming

And that a strange one too which did awake meI shaked you sir and cried As mine eyes opened I saw their weapons drawn There was a noiseThats verily rsquoTis best we stand upon our guardOr that we quit this place Lets draw our weapons

(i) Why has Prospero sent Ariel to Gonzalo and Alonso What does Ariel do to awaken Gonzalo

Prospero has already come to know by his magic powers the danger which threatens Gonzalo who had been Prosperorsquos friend and so he sent Ariel to preserve the lives of both Gonzalo and Alonso Prospero does not want that his scheme should remain unfulfilled Ariel begins to sing a song in Gonzalorsquos ears to awaken him(ii) Who are ready to carry out their plan Who takes steps to stop them Why does Gonzalo feel surprised after being awakened

Sebastian and Antonio are ready to carry out their plans They are standing with their swords drawn to kill Alonso and

(iv) We see two sets of contrasting characters Gonzalo-Adrian against Antonio-Sebastian

(v) The grief that works in Alonso can be perceived to his repentance for his association in Antoniorsquos crime against Prospero

Gonzalo Ariel takes steps to stop them from carrying out their nefarious scheme When Gonzalo is awakened by the song sung by Ariel into his ears he (Gonzalo) feels surprised because he sees Sebastian and Antonio standing with their swords drawn(iii) What reason do Sebastian and Antonio tell of drawing their swords when they are suspected by Alonso and Gonzalo

When Sebastian and Antonio are seen with their swords drawn they are looked with suspicion by Gonzalo and Alonso At first Sebastian tells them that as they stood here to guard them during their sleep they heard only a little before a sudden loud noise very much like the roaring of bulls or more probably that of lions Then Antonio follows him saying that this was a noise so terrible as to frighten even a monsterrsquos ears and this noise could even have shaken the earth and it was surely like the roaring of a multitude of lions Then seeing the danger they have drawn their swords Perhaps after hearing the terrible noise they (Gonzalo and Alonso) woke up from their sound sleep

(iv) What does Gonzalo tell Alonso about the strange noise What did he see on opening his eyes Gonzalo tells Alonso that he did not hear the sound of roaring but he heard a humming sound which was strange and which woke him up After waking up he gave him (Alonso) a shaking and a loud cry On opening his eyes he saw these two gentlemen standing with their swords drawn(v) What does Gonzalo suggest

Gonzalo suggests that there was a noise indeed and of that he has no doubt at all and suggests that the best course for them would be to remain alert and vigilant against any possible danger to their lives or to leave this place and move to some other part of the island

Class XIISubject Topic Summary Execution

Commerce

Chapter- Management

Today we will discuss about LEVELS OF MANAGEMENT

Levels of management is a series or chain of managerial positions from top to bottom It helps individuals to know their authority responsibilities and superior-subordinate relations among themselves There are mainly three levels of Management TOP LEVEL MANAGEMENTMIDDLE LEVEL MANAGEMENTLOWER LEVEL MANAGEMENT

Top level managementIt consists of members at the highest level in the management hierarchy This level includes Board Of Directors Chief Executive Managing Directors Chairman President Vice President

Rolefunctions of the top levelmanagement1To analyse evaluate and deal

with theexternal environment2 To determine the objectives and

policies of the business3 To strive for welfare and survival

of business

4 To create an organisational Framework consisting of authority responsibility relationship

Middle level management Congress of members or groups who are concerned with implementation of the policies let down by the top managementThis level includes head of the department such as finance manager marketing manager branch and regional managers departmental and divisional heads plant superintendent etc

Role of functions of the middle level management

1 To interpret the policies framed by top management

2 To assign duties and responsibilities to lower level managers

3 To select and appoint employees for middle and supervisory level and evaluate their performance

4 To co-operate with other departments for smooth functioning

Operational or supervisory level managementIt refers to the group are members who are concerned with execution of the work They are also known as fast line managers This level includes supervisor 4 men Section Officer clerk Inspector etc

Role of functions of the lower level management1 To plan and execute day-to-

day operations2 To supervise and control the workers3 To arrange materials and

tools to start the process and make arrangements for training

4 Today present workers grievance and suggestions before the management and

ensure safe and proper working conditions in the factory

Business Studies

Staff Appraisal Chapter- 10 Today let us start with a new chapter

Staff Appraisal

Meaning of Performance Appraisal

Performance Appraisal is the systematic evaluation of the performance of employees and to understand the abilities of a person for further growth and developmentThe supervisors measure the pay of employees and compare it with targets and plansThe supervisor analyses the factors behind work performances of employeesThe employers are in position to guide the employees for a better performance

Objectives of Performance Appraisal

Following are the objectives of Performance Appraisal

To maintain records in order to determine compensation packages wage structure salaries raises etc

To identify the strengths and weaknesses of employees to place right men on right job

To maintain and assess the potential present in a person for further growth and development

To provide a feedback to employees regarding their performance and related status

To provide a feedback to employees regarding their performance and related status

Importance of Performance Appraisal

Performance appraisal provides important and useful information for the assessment of employees skill

knowledge ability and overall job performance The following are the points which indicate the importance of performance appraisal in an organization

1 Performance appraisal helps supervisors to assess the work performance of their subordinates

2 Performance appraisal helps to assess the training and development needs of employees

3 Performance appraisal provides grounds for employees to correct their mistakes and it also provides proper guidance and criticism for employees development4 Performance appraisal provides reward for better performance

5 Performance appraisal helps to improve the communication system of the organization

6 Performance appraisal evaluates whether human resource programs being implemented in the organization have been effective

7 Performance appraisal helps to prepare pay structure for each employee working in the organization

8 Performance appraisal helps to review the potentiality of employees so that their future capability is anticipated

Geography

DRIANAGE The SubarnarekhaThe Subarnarekha and the Brahmaniinterposed between the Ganga and the Mahanadi deltas drain an area of 19300 sq kmand 39033 sq km respectively The drainage basins of these streams are shared byJharkhand Odisha west Bengal and Chhattisgarh The Brahmani is known as southKoel in its upper reaches in Jharkhand

The NarmadaThe Narmada rises in the Amarkantak hills of MadhyaPradesh It flows towards the West in a rift valleyformed due to a geological fault The total length of it is 1300 km All the tributaries of the

Q1 Name the two westward flowing rivers in the peninsular plateauA1 Narmada and Tapi are the only westward flowing rivers of the peninsular plateau

Q2 Differentiate between east-flowing rivers and west-flowing riversA2

East-flowing rivers

West-flowing rivers

Narmada are very short inlength Most of its tributaries join the main streamright anglesThe Narmada basin covers parts of Madhya Pradesh and Gujarat

The Tapi The Tapi rises in the Satpura ranges in the Betul listrictof Madhya Pradesh It flows in a rift valley parallel tothe Narmada but it is much shorter in length It coversparts of Madhya Pradesh Gujarat and MaharashtraThe length is about 724 km

The Sabarmati and the MahiThe Sabarmati rises in the Aravali hills and flows south-south-westwards for a distance of 300 kilometres to the Arabian Sea The Sabarmatibasin extends over an area of 21674 sq km in Rajasthan and Gujarat The Mahi rises inthe east of Udaipur and drains an area of 34842 sq km lying in Madhya PradeshRajasthan and Gujarat It flows south-westwards for a distance of 533 km before it fallsinto the Gulf of Khambhat

The ChambalThe Chambal rises near Mhow in the Vindhya Range and flows towards the northgenerally in a gorge upto Kota Below Kota it turns to the north-east direction and afterreaching Pinahat it turns to the east and runs nearly parallel to the Yamuna beforejoining it in the southern part of the Etawah district in Uttar PradeshMajor Rivers of India with their basin area (Sqkm)

Himalayan System Indus 321290Ganga 861404

Brahmaputra 187110Indus System

Jhelum 34775Beas 20303

Ganga System Yamuna 366223Ghaghra 127950

Peninsular RiversNarmada 98796

Tapi 65145Mahanadi 141600

Subarnarekha 19300Sabarmati 21674

Mahi 34842Godavari 312812

Godavari Krishna Kaveri Mahanadi are the east-flowing rivers

Narmada Tapi west-flowing rivers

They fall into the Bay of Bengal

They fall into Arabian Sea

These rivers form big deltas

These rivers form comparativelysmall deltas

Catchment areas of these rivers are larger

Catchment areas of these rivers are smaller

Krishna 2589488Cauveri 87900

Subject ndashBiology Topic ndashChapter -5 Inheritance amp Variations Summary ExecutionToday we will discussabout linkage and its classification

LINKAGE The tendency of the genes located on the same chromosome to stay together is

hereditary transmission Linked genes the genes responsible for this Genes that exhibit the process of linkage locates in the same chromosome The distance between the linked genes in a chromosome determines the strength

of linkage i e genes that are located close to each other show stronger linkage than that are located far from each other

COMPLETE LINKAGE It is the type of linkage showed by the genes that are closely located or are tightly

linked with each other as they have no chance of separatingby crossing over These genes are always transmitted together to the same gamete and the same

offspring In such condition only parental or non cross over type of gametes are formedINCOMPLETE KINKAGE It is type of linkage showed by the genes that are distantly located orare loosely

linked with each other because they have chance of separating by crossing over

SIGNIFICANCE i) It helps in holding the parental character togetherii) It checks the appearance of new recombination and helps in bringing the

hybrid population which resembles the original parents iii) Linked genes dilute the effects of undesirable traits

Subject Eng Literature (The Tempest ndash William Shakespeare) Topic Essay Questions (EQ-3)Question No 3

Give a character sketch of CalibanAnswer

The character of Caliban has been wonderfully conceived by Shakespeare as the manifestation of all that is gross and earthy ndash a sort of creature of the earth as Ariel is a sort of creature of the air

Calibanrsquos Physical Appearanceo Caliban is lsquofreckledrsquo a lsquomisshapen knaversquo not honoured with human shape

o Prospero calls him lsquothou tortoisersquo (Act I Sc 2 Line 317) Trinculo stumbling upon him describes him as ldquoA strange fish hellip Legged like a man And his fins like armsrdquo He ldquosmells like a fishrdquo (Act II Sc 2 Line 25)

o Prospero also calls him a ldquobeastrdquo (Act IV Sc 1 Line 140) and ldquoThis misshapen knaverdquo (Act V Sc 1 Line 268)

o Further it appears that in addition to his physical deformity his spiritual inferiority is also suggested by Prosperorsquos claim that his birth resulted from the union between his mother the witch Sycorax and the devil

Calibanrsquos ParentageWhen the play opens Caliban is twenty four years of age having been born on the island twelve years before the coming of Prospero His mother was the foul witch Sycorax who was banished from Algiers for ldquomischiefs manifold and sorceries terrible to enter human hearingrdquo (Act I Sc 2 Line 264) and the father was the Devil himself Thus

Caliban is a monster of evil and brute nature ugly deformed and stinking

Calibanrsquos Savage and Malignant Natureo Caliban is entirely a creature of the earth ndash gross brutal and savage He regards himself as the rightful possessor

of the island and Prospero as a usurper

o In his young age he was on good terms with Prospero He had consented to be received by Prospero at his house and to be educated by him He has learnt human language only to curse his master whom he abhors

o His beastly nature soon breaks out and ends in a vicious attack on Miranda This opens the eye of Prospero who becomes severe to him and enforces his service by threats and violence

o Prospero uses him to make dams for fish to fetch firewood scraper trenches wash dishes and keep his cell clean

Calibanrsquos Hatred for ProsperoA profound hatred for Prospero has taken hold of Caliban It springs from a sense of his being dispossessed and ill-treated He would kill Prospero if he could but he knows the power of Prosperorsquos lsquobookrsquo Hence he transfers his allegiance to Stephano who seems like a god to him He also incites the two drunken associates to batter the skull of Prospero when he sleeps in the afternoon

Caliban Shows Considerable Intelligenceo He has learnt Prosperorsquos language

ldquoYou taught me language and my profit onrsquot (Act II Sc 2 Lines 86-89)Is I know how to curserdquo

o He is well aware of the futility of arguing with one who has more power than he has

ldquoI must obey his art is such power (Act I Sc 2 Lines 373-376)It would control my damrsquos god SetebosAnd make a vassal of himrdquo

o He realizes the importance of Prosperorsquos books

ldquoRemember (Act III Sc 2 Lines 89-92)First to possess his books for without themHersquos but a sot as I am nor hath notOne spirit to commandrdquo

o He knows the value of stealth when attacking the enemy

ldquoPray you tread softly that the blind mole may not (Act IV Sc 1 Lines 194-195)Hear a foot fall we now are near his cellrdquo

o Caliban has a better set of values than Stephano and Trinculo They are distracted from their plan by their greed for Prosperorsquos rich garments Only Caliban realizes that such a finery is unimportant

ldquoLeave it alone thou fool it is but trashrdquo (Act IV Sc 1 Lines 224)

Caliban is not a good judge of characterCaliban is not a good judge of character He decides for example that Stephano is a god because he dispenses lsquocelestial liquorrsquo (Act II Sc 2 Line 115) but then it must be remembered that he has only known his mother Sycorax Prospero Miranda and the spirits that torture him However he quickly discovers his error of judgementrdquo

ldquoWhat a thrice-double ass (Act V Sc 1 Lines 295-297)Was I to take this drunkard for a godAnd worship this dull foolrdquo

Calibanrsquos Imaginative NatureIf Caliban is sub-human in what has been said above he is human in the respect of the poetic side of his character He listens to music with rapture He tells of the beautiful dreams in which heaven rains treasures upon him and which upon waking he yearns to renew One of the most poetic passages in whole play is Calibanrsquos description of the island

to Stephano and Trinculo

ldquoBe not afeard The isle is full of noises (Act III Sc 2 Lines 135-143)Sounds and sweet airs that give delight and hurt notSometimes a thousand twangling instrumentsWill hum about mine ears and sometime voicesThat if I then had waked after long sleepWill make me sleep again and then in dreamingThe clouds methought would open and show richesReady to drop upon me that when I wakedI cried to dream againrdquo

Caliban - Less Ignoble Than Some OthersCalibanrsquos motive for murder is less dishonourable than that of Antonio and Sebastian They plan to kill Alonso to gain his power and wealth Caliban merely wants revenge and the return of lsquohisrsquo island

Conclusiono Calibanrsquos character is not portrayed very clearly in the play and hence we cannot decide whether he is a poor

savage being grossly maltreated by Prospero or whether he is evil and must therefore be kept in bondage or enslavement

o Caliban is contrasted with Ariel who is a spirit and thus swift and uninterested in physical activitieso Caliban is also contrasted with Prospero who is the all-powerful master of the island and of the destiny of all

those on the islando Caliban is also contrasted with civilized man showing him to be less evil than Antonio and Stephano and less

materialistic than Stephano and Trinculoo Caliban has suffered at the hands of Prospero and he has learnt to curse by listening to Prosperorsquos abuse He

certainly believes that Prospero has deprived him of his birthrighto Finally the character Caliban is thought to be one of Shakespearersquos masterpieces The complexity of the character

is reflected in the large volume of critical discussion that has grown around it

ECO ndash12 Topic-Forms of market

MonopolyMonopoly is a market structure in which there is a single seller there are no close substitutes for the commodity produced by the firm and there are barriers to entry Example Indian Railways which is operated under government of India Monopoly also implies absence of competitionFeatures of Monopoly Monopoly is characterized by1 Single Seller In monopoly there is only one firm producing the product The whole industry consists of this single firm Thus under monopoly there is no distinction between firm and industry Being the only firm there is significant control of the firm over supply and price Thus under monopoly buyers do not have the option of buying the commodity from any other seller They have to buy the product from the firm or they can go without the commodity This fact gives immense control to the monopolist over the market

2No Close Substitute There are no close substitutes of the product produced by the monopolist firm If there are close substitutes of the product in the market it implies presence of more than one firm and hence no monopoly In order to ensure a total of control over the market by the monopolist firm it is assumed that there are no close substitutes of the product

3 No Entry amp Exit Monopoly can only exist when there is strong barriers before a new firm to enter the market In fact once a monopoly firm starts producing the product no other firm can produce the same One reason for this is the ability of the

monopolist to produce the product at a lower cost than any new firm who thinks to enter the market If a new firm who knows that it cannot produce at a lower cost than the monopolist then that firm will never enter the market for fear of losing out in competition Similarly the monopolist who is operating for a long time may be enjoying reputation among its customers and is in a better position to use the situation in its own benefit A new firm has to take long time to achieve this and so may not be interested to enter the market

4 Price Maker Being the single seller of the product the monopolist has full control over the pricing of the product On the other hand if there is a large number of buyers in the market so no single buyer exercises any significant influence over price determination Thus it is a sellerrsquos market So monopoly firm is a price maker

5 Price Discrimination Having considerable control over the market on account of being single seller with no entry of other firms the monopolist can exercise policy of price discrimination it means that the monopolist can sell different quantities of the same product to a consumer at different price or same quantity to different consumers at different prices by adjudging the standard of living of the consumer

6 Shape of Demand Curve Since a monopolist has full control over the price therefore he can sell more by lowering the price This makes the demand curve downward sloping

Subject Ac-12 290620 Topic- retirement Model sumThe Balance Sheet of Rohit Nisha and Sunil who are partners in a firm sharing profits according to their capitals as on 31st March 2014 was as under

Liabilities Amount Assets Amount (Rs) (` Rs)

Creditors 25000 Machinery 40000Bills Payable 13000 Building 90000General Reserve 22000 Debtors 30000Capital Less Provision for Rohit 60000 Bad debts 1000

29000 Nisha 40000 Stocks 23000 Sunil 40000 140000 Cash at Bank 18000

200000 200000

On the date of Balance Sheet Nisha retired from the firm and following adjustments were made(i) Building is appreciated by 20(ii) Provision for bad debts is increased to 5 on Debtors(iii) Machinery is depreciated by 10(iv) Goodwill of the firm is valued at Rs 56000 and the retiring partnerrsquos share is adjusted

(v) The capital of the new firm is fixed at Rs120000 Prepare Revaluation Account Capital Accounts of the partner and Balance Sheet of the new firm after Nisharsquos retirement Revaluation AccountDr Cr

Particulars Amount Particulars Amount (`Rs) (Rs`)

Provision for Bad debt Ac 500 Building Ac 18000Machinery Ac 4000Profit transferred toCapital Accounts (3 2 2)Rohit 5786Nisha 3857Sunil 3857

13500

18000 18000

Capital Account

Dr Cr

Particulars Rohit Nisha Sunil Particulars Rohit Nisha Sunil (Rs`) (Rs`) (`Rs) (Rs`) (Rs`) (Rs`)

Sunilrsquos Capital ac 9600 mdash 6400 Balance bd 60000 40000 40000Bank - 66143 - General Reserve 9428 6286 6286Balance cd 72000 mdash 48000 Revaluation (Profi 5786 3857 3857 Rohitrsquos Capital Ac mdash 9600 mdash

Sunilrsquos Capital Ac 6400 Bank 6386 - 4257

81600 66143 54400 81600 66143 54400

Balance Sheet as at 31st March 2014

Liabilities Amount Assets Amount (Rs`) (Rs`)

Creditors 25000 Building 108000Bank overdraft 37500 Machinery 36000

Bills Payable 13000 Debtors 30000Capital Less ProvisionRohit 72000 for Bad debts 1500 28500Sunil 48000 120000 Stock 23000

195500 195500

Working Notes (i) (a) Profit sharing ratio is 60000 40000 40000 ie = 3 2 2(b) Gaining Ratio Rohit = 35 ndash 37 = 2135 ndash 1535 = 635Sunil = 25-27 = 1435 ndash 1035 = 435= 635 435= 6 4 = 3 2(c) Nisha Share of Goodwill = Rs 56000 times 27 = Rs16000Share of Goodwill in the gaining ratio by the existing partner ieRohit = Rs16000 times 35 = Rs 9600Sunil = Rs 16000 times 25 = Rs 6400

The journal entry isRohitrsquos Capital Ac Dr 9600Sunilrsquos Capital Ac Dr 6400 To Nisharsquos Capital Ac 16000(Share of Goodwill divided into gaining ratio)

  • 1 Static Friction
  • The frictional force that acts between the surfaces when they are at rest with respect to each other is called Static Friction
    • Static Friction Examples
      • 2 Sliding Friction
        • Examples Of Sliding Friction
          • 3 Rolling Friction
            • Examples Of Rolling Friction
              • Objects and Reasons of the Forest Conservation Act
Page 32:  · Web viewSubject . Topic . Summary . Execution . English 1 . Sounds of animals . Hens –cackle Horses –neigh Lions –roar Owls –hoots Snake –hiss. English 2 . Mother’s

(22frac12degS) This position of the Sun is referred to as the Winter Solstice because it marks the winter season in the Northern Hemisphere On that day the duration of day and night both are equal ( 12 hours day and 12 hours night) on every places located on Tropic of Capricorn (22frac12degS)SEASONS AND DURATION OF DAY AND NIGHT During the equinoxes all places on the Earth have 12 hours of day and 12 hours of night Due to the revolution of the Earth round the Sun on an inclined axis the duration of day and night varies according to seasons and the latitude of a placeDuring the summer solstice (21 June) the North Pole is inclined towards the Sun Therefore the duration of sunlight or daytime increases from 12 hours at the Equator to 24 hours at the Arctic Circle and beyondThe region beyond the Arctic Circle especially Norway is known as the Land of the Midnight Sun because there the Sun does not rise or set on 21 JuneAt the North Pole there will be six months of daylight The Sun will be seen always above the horizon at a low angle At 66degN 24 hours of sunlight can be seen only on 21 June Hammerfest in northern Norway is a place of tourist attraction for observing the phenomenon of the Midnight Sun This place has continuous daylight from 13 May to 29 July This place is easily accessible to tourists and has hotels and other facilities The view of the midnight Sun from here is enthrallingIn the Southern Hemisphere the duration of daylight decreases from 12 hours at the equator to 0 hours beyond the Antarctic Circle In the South Polar Region there is 24 hours of darkness The Sun is always below the horizon In the Southern Hemisphere which experiences winter the duration of night-time is longer than the duration of daylight

Spring Equinox 22nd

December

A8 A B

Summer Solstice 21st June

Autumn Equinox 23rd

September

Winter Solstice 22nd

December

Spring Equinox 21st March

During winter solstice (22 December) the South Pole is inclined towards the Sun The Southern Hemisphere experiences summer and the Northern Hemisphere has winter Therefore the duration of daylight or sunlight is greater in the Southern Hemisphere than in the Northern HemisphereThe duration of daylight increases from 12 hours at the equator to 24 hours beyond the Antarctic Circle The South Polar Region has 24 hours of sunlight for many days continuously At the South Pole there will be six months of sunlight The Sun will always be seen at a low angle above the horizon In the Northern Hemisphere the duration of daylight will decrease from 12 hours at the equator to 0 hours at the Arctic Circle There are 24 hours of darkness in the North Polar region The duration of night is greater than the duration of daylight as one move northwards from the Equator It is evident from the above table that the duration of daylight is 12 hours throughout the year at the equator only As one moves away from the equator the seasonal variations in the duration of daylight increase The seasonal variations in the duration of daylight are maximum at the Polar Regions

Subject Eng Literature (The Merchant of Venice ndash William Shakespeare)Topic Act II Scene 7 Lines 36 to 80 (End of scene ) [Students should read the original play and also the paraphrase provided]

Summary Questions amp AnswersThe Prince then examines the inscription on the silver casket which says ldquoWho chooseth me shall get as much as he deservesrdquo The Prince says that he deserves Portia more than anybody else because of his high rank his noble birth and his great wealth and power But then he argues that silver is ten times

(1) (Act II Sc 7 L 39-47)

From the four corners of the earth they come

To kiss this shrine this mortal breathing saint

The Hyrcanian deserts and the vasty wildsOf wide Arabia are as through-fares now

inferior to gold and therefore he cannot believe that the portrait of such a beautiful lady as Portia can be contained in the silver casket He decides to see the inscription on the golden casket before making his decision

The Prince goes to examine the inscription on the golden casket which says ldquoWho chooseth me shall get what many men desirerdquo The Prince believes that the whole world desires to possess Portia otherwise so many suitors would not have come from all corners of the world for winning Portia Some of them have come from the distant lands of Persia and Arabia The deserts of Persia (Hyrcanian deserts) and the boundless desolate lands of Arabia have been crossed by the Princes seeking the hand of Portia He contrasts this casket containing Portiarsquos portrait with the old English gold coin bearing the image of the archangel (angel of the highest rank) He goes on to remark that while the figure of the archangel is engraved (Insculped) upon the English coin the picture of Portia who is beautiful as an angel lies hidden inside one of the caskets namely the Golden Casket (Golden Bed)

On the basis of his assessment of the inscription on the golden casket the Prince decides to choose the golden casket He asks for the key and opens the golden casket only to find therein an empty human skull holding a roll of

For princes to come view fair PortiaThe watery kingdom whose ambitious headSpets in the face of heaven is no barTo stop the foreign spirits but they comeAs orsquoer a brook to see fair Portia

(i) Explain the occasion for the above mentioned speech

These are the comments of the Prince of Morocco after he reads the inscription on the golden casket His mental process is revealed to us in these words We find him debating within himself as to which casket he should choose

(ii) What light does the above speech throw on the personality of Prince of Morocco

From the above mentioned speech we come to know that the Prince of Morocco is keen to marry Portia He is the type of person who is easily taken away by outward appearance He is in love with Portia because of her beauty

(iii) What information can you gather about Portia from the above mentioned lines

The given speech shows that Portia is a very beautiful lady She must be possessed of good qualities because many suitors come to her place from all over the world with a desire to get married to her The Prince of Morocco is so impressed by her beauty that he calls her a saint According to him the whole world is desirous of having her

(iv) Elucidate the significance of the first two lines

In these lines the Prince of Morocco pays a compliment to Portia These lines show his admiration for her He says that people come from all parts of the world to see fair Portia

(v) Explain the meaning of the last four lines of the

passage

In these lines the Prince of Morocco says that even the vast oceans which throw a challenge at the sky are unable to prevent men from coming to Portiarsquos place to have a glimpse of her These lines are also a tribute to Portiarsquos beauty and good qualities Many men voyage across the ocean treating it as a mere stream to see the beautiful Portia

paper in which is written that whoever happens to be guided by the glitter of things is invariably deceived

On reading the scroll the Prince says that he is too sad at heart to speak a more formal farewell and leaves with his followers amidst a sound of trumpets

After the Prince of Morocco leaves Portia remarks that the Prince is a gentle fellow but she is rid of him May all persons of his nature make a similar choice

IMPORTANT PASSAGES EXPLAINED

(Act II Sc 7 L 39-43)From the four corners of the earth they come

To kiss this shrine this mortal breathing saintThe Hyrcanian deserts and the vasty wildsOf wide Arabia are as through-fares nowFor princes to come view fair Portia

Context

This passage occurs in Act II Scene 7 in The Merchant of Venice This is part of the speech made by the Prince of Morocco

(2)

(Act II Sc 7 L 48-53)

MOROCCO One of these three contains her heavenly pictureIst like that lead contains her

Twere damnation To think so base a thought it were too grossTo rib her cerecloth in the obscure graveOr shall I think in silver shes immurdBeing ten times undervalued to tried gold

(i) What meaning does the Prince of Morocco find out of the inscription of the golden casket What have Belmont and Portiarsquos house been called and why

The inscription on the golden casket is ldquoWho chooseth me shall gain what many men desirerdquo The Prince finds out that it means that the chooser of the golden casket will get Portia because many men desire her In fact the entire world desires her Because of the coming of many suitors to Belmont from different countries in order to win Portiarsquos hand Belmont has become a centre of pilgrimage and her house is the shrine where saintly Portia is installed

(ii) What does the Prince of Morocco do before making the final choice of the casket Which is the correct casket and who will win Portiarsquos hand

The Prince of Morocco surveys and analyses the inscriptions on the casket of lead silver and gold Before making the final choice like a very systematic and methodical person he once again considers the claims of the caskets The casket containing Portiarsquos picture is the correct casket and the person choosing it will win Portiarsquos hand

Explanation

While praising Portia the Prince of Morocco conceives Portia as a goddess whose image is placed inside one of the caskets Many suitors are coming from far and wide the north and the south the east and the west (Four corners) in order to try their luck Some of them have come from the distant land of Persia and Arabia The deserts of Persia (Hyrcanian deserts) and the boundless desolate lands of Arabia have been crossed by the Princes seeking the hand of Portia All this shows that Portia is indeed the most beautiful lady of the world

(iii) What does the Prince of Morocco say in his estimation while examining the motto on the silver casket What does he find in the golden casket

While examining the motto on the silver casket which says ldquoWho chooseth me shall get as much as he deservesrdquo Morocco says that in his own estimation he surely deserves Portia in all respects ndash rank birth wealth etc

He chooses the golden casket When he opens it he finds an empty human skull holding a scroll in which it is written that those who are attracted by the glittering outside of things are always deceived as Morocco has been deceived

(iv) What kind of nature does the Prince of Morocco have

The Prince of Morocco has a simple nature who does not look deeply into the inner meaning of things but is dazzled by the outward appearance of gold He is inclined to over-estimate his own value and does not realize that it is a duty to ldquogive and hazardrdquo To say that he will not hazard for lead shows that he misreads the true meaning of the inscription which is that he should be prepared to ldquohazard all he hathrdquo for Portia So his feeling is only one of fascination and romantic attraction

(v) Do you think that the lottery of the caskets is not a matter that will be determined by chance

In fact the lottery of the casket is not a matter that will be determined by mere chance but that it is a true test of character and of sincerity which is amply proved not only by Moroccorsquos choice but also by the arguments which he uses to help him in his choice

(Act II Sc 7 L 55-59)

They have in England

A coin that bears the figure of an angelStamped in gold but thats insculpd uponBut here an angel in a golden bedLies all within

Context

(3)

(Act II Sc 7 L 63-77)A carrion Death within whose empty eye

There is a written scroll Ill read the writing

All that glisters is not goldOften have you heard that toldMany a man his life hath soldBut my outside to beholdGilded tombs do worms infoldHad you been as wise as boldYoung in limbs in judgment oldYour answer had not been inscrolld

This passage occurs in Act II Scene 7 in The Merchant of Venice This is part of the speech made by the Prince of Morocco

Explanation

In this passage the Prince of Morocco bestows high praise on Portia whose hand he is seeking He contrasts this casket containing Portiarsquos portrait with the old English gold coin bearing the image of the archangel (angel of the highest rank) He goes on to remark that while the figure of the archangel is engraved (Insculped) upon the English coin the picture of Portia who is beautiful as an angel lies hidden inside one of the caskets namely the Golden Casket (Golden Bed) In the day of Elizabeth silver was ten times inferior in value to gold Therefore the Prince of Morocco believing that Portiarsquos portrait is contained in the Golden Casket decides to choose the Golden Casket

Fare you well your suit is coldCold indeed and labour lostThen farewell heat and welcome frostmdashPortia adieu I have too grievd a heartTo take a tedious leave Thus losers part

(i) What reward does the Prince of Morocco get after making a wrong choice of the Casket How does he feel

After making the wrong choice in selecting the casket of gold the Prince of Morocco as a reward earns a rebuke in the form of a scroll tucked in the empty eye-socket of a skull kept in the casket of gold The Prince is shocked and disappointed He becomes all the more sad and dejected when he reads the scroll which points to his foolishness in being misled by the appearance and outward show as indicative of its worth

(ii) How does the Prince respond after reading the scroll

After reading the scroll the Prince though upset accepts the result with good grace and decorum befitting a royal suitor and true sportsman He says that his love-suit is really cold otherwise he would have chosen correctly but now his efforts have been in vain So he bids farewell to Portia to the warmth and enthusiasm of love and welcomes the cold and bitterness of dejection and misery of life which lies ahead

(iii) What request does he make to Portia and why

After being failure in his mission he requests Portia to give him permission to leave at once because he is too sad to undergo the tediousness of a formal leave-taking He tells that it is the manner in which defeated persons part unceremoniously

(iv) Explain the following lines

ldquoAll that glisters is not goldOften have you heard that toldMany a man his life hath soldBut my outside to beholdGilded tombs do worms infoldrdquo

Mere glitter does not make a metal to be gold Man has often been warned against appearance but it has been of no use Many people have sacrificed their lives only to seek the outer appearance of gold Worms are found inside the gilded

monuments

Class XSubject Topic Summary Execution

Hindi 2ndlang

नया रासता भाग 6 मायाराम 0ी घर म धनी मल 0ी और उनी बटी सरिरता ी ही चचा बनी रहती थी अमिमत ो इसम ोई रलिच ना थी वह धनी घर ी लडी स शादी र सवय ो बचना नही चाहता था उसा भी सवाणिभमान ह ईशवर ी पा

स उस पास पस ी ोई मी नही थी अभी उसन फकटरी ही लगाई थी उसी समझ बाहर था कि उस घर वालो ा झाव पस ी तरफ कयो

ह उसन मा स सवाल किया कि मा तम सरिरता स मरी शादी कयो रना चाहती हो मा न उस समझाया कि वह दखन म बरी नही ह और किफर खानदान अचछा

ह वह ए शल गरहणी रप म घर सभाल सगी अमिमत न मा ो इस बात ा एहसास राया कि मीन सबध लिलए मना रन पर उस दिदल

पर कया बीती होगी मा और अमिमत ी लडी बार म ाफी बात हईमा ा झाव सरिरता ी तरफ था कयोकि वह घर पर अचछा दह0 लर आ रही

थी अमिमत न अपनी मौसी ी बरी हालत बार म बताया कि किस तरह वह बड घर ी खानदानी बटी लाई थी और आ0 उसी हालत कितनी खराब ह लाई थी बहकलब 0ाती ह और बचचो ो भी नही दखती ह बात चल ही रही

थी कि तभी ए ार बाहर आर री धनी मल0ी घर अदर आए और पीछ स डराइवर फल ी ए टोरी लर आया अदर आए और पीछ स

डराइवर ए टोरी फल ी लर आया अमिमत ो फल ी पटी बरी लग रही थी अमिमत न पछ लिलया यह फल कयो ल आए ह प इन सब ी कया

0ररत थी उनो न 0वाब दिदया कि 4 पटी शमीर स मगाए थ अमिमत ो या सनर करोध आ गया तभी उस किपता 0ी आ गए उन आत ही अमिमत उठर बाहर चला गया वहा वहा मा पास आर बठ गया और बोला

अभी रिरशता तय नही हआ और धनी मल 0ी धनी मल 0ी फल ी पटी लर चलआय मा न समझाया कि 0ब सबध 0ड 0ाता ह तो खाली हाथ नही

आत अमिमत न मा स हा कि तम सबन सरिरता ो इस घर म लान ी ठान रखी ह धनीमल 0ी उस दिदन सरिरता ो दखन ी तारीख तय रन आय थ

Commercial Studies

Banking Nowadays Bank provide easy and quick services through internet facilities methods of Banking is called internet bankingIn order to save the time and money involved in visiting Bank branches people increasingly prefer to have internet banking

There are different modes of doing internet banking or transferring money through online They areReal Time Gross Settlement (RTGS)National Electronic Fund Transfers (NEFT)

1

Question

1) Explain the term RTGS Write the features of RTGS

Answer)The acronym RTGS stands for Real Time Gross Settlement which may be defined as the continuous real time settlement of funds transfer individually on and order by order basis without netting lsquoReal timersquo may be defined as the processing of instructions at the time they are received rather than at some letter time lsquoGross settlementrsquo may be defined as the settlement of transfer instructions which occurs

individually

Features of RTGS1It is the continuous settlement of

funds transfer individually on an order by order basis

2RTGS facility is provided only by CBS core banking solution enabled Bank branches

3Amount charged from the customer for RTGS transactions vary from bank to bank

2) Explain the term NEFT Write the features of NEFT

Answer) National electronic funds transfer may be defined as a nationwide system that facilitates individuals Farms and copper operates to electronically transfer funds from any bank branch to any individual farm or corporate having an account with any other bank branch in the country

Features of NEFT2 Transfer can be made 7 times on

weekdays and 6 times on Saturday

3 NEFT cannot be used to receive foreign remittances

4 NEFT transaction takes place in batches

5 A bank branch must be NEFT enabled to become a part of NEFT fund transfer network

6 There is no maximum or minimum amount that can be transferred through NEFT when one bank has a bank account

English Language

CompositionEssay

A composition is an art of creating a piece of writing on any topic or subject It is the writing correctly beautifully and clearly in order to make some interesting reading Structure of the composition

Introduction ( you lay the foundation for your composition)

Body (it constitutes the main part of the essay)

Conclusion (final statement that leaves a lasting impression)

Kinds of essays1 The Narrative essay2 The descriptive essay3 The reflective essay4 The argumentative essay

Write a composition on any one of the following topics (350- 400 words)

1 Friendship Or2 The first day of your school

Subject Eng Literature (The Merchant of Venice ndash William Shakespeare)Topic Act V Scene 1 Lines 127 to 158 (Nerissa helliphellip The clerk will nersquoer wear hair onrsquos face that had it) [Students should read the original play and also the paraphrase given in the school prescribed textbook]

Summary Revision Questions o Soon thereafter Bassanio Gratiano

and Antonio arrive

o Bassanio tells Portia that he is feeling as if it is morning because of the presence of Portia who is shining like the sun When Antonio is introduced by Bassanio to Portia she tells Bassanio that he should be grateful to Antonio who took so much trouble on his account even to the extent of risking his life

o Nerissa starts quarrelling with Gratiano and demands that he show her the ring she had presented to him and which she had warned him not to lose She suspects that Gratiano must have presented the ring to some young woman and not to the lawyerrsquos clerk as he repeatedly says and assures

Answer the following questions to check your preparation of Act IV Scenes 1 and 2

You must attempt only after you have completed your preparation of Act IV The answers must be in complete sentences using textual evidence (with citation) when necessary

[It would be in your own interest to attempt the above questions honestly totally refraining from consulting your textbook or your notes during answering After completion you should correct the paper yourself consulting the textbooknotes etc and award marks as specified Please let me know the marks you scored through WhatsApp in the group or to my personal WhatsApp]

Act IV Scene 1 (each question carries 2 marks)

1 What did the Duke try to do for Antonio

2 Why does Shylock refuse to show mercy How does he justify his stance

3 Why does Antonio say he is ready to die 4 What information is contained in Bellariorsquos letter

5 Why does Portia (as Balthazar) assert that Shylock must show mercy How does he respond

6 What offers are made to Shylock to get him to spare Antonio How are they received

7 What does Antoniorsquos speech as he faces the prospect of Shylockrsquos knife tell you about his character

8 How do Bassanio and Gratiano react to the looming prospect of Antoniorsquos demise

9 How does Portia (as Balthazar) use the law to turn the tables on Shylock

10 What does the Duke decree should happen to Shylock Why What happens to Shylockrsquos estate

11 What does Portia ask Bassanio as payment for her ldquoservicesrdquo What is his initial response What makes him change his mind

Act IV Scene 2 (each question carries 1frac12 marks)

1 What does Gratiano bring to Portia (Balthazar)

2 What does Nerissa plan on getting from Gratiano What does Portiarsquos comment suggest about men

ECO-10 280620 Topic-Supply AnalysisSHIFTING OF SUPPLY

But if there is change in factors other than the price of the commodity then either more is supplied at the same price or less supplied at the same price In such cases the price of the commodity remains constant but there is a change in other factors like change in the price of inputs change in technology of production change in price of other related goods change in taxation policy of the government etc For example there is an improvement in the technology of production of the commodity in question It leads to decrease in per unit of cost production of the commodity The firm is willing to sell more quantity of the commodity at the same price So the supply other commodity increases at the same price This increase in supply is shown by rightward shift of supply curve On the other hand if the firm uses inferior technology of production the cost of production per unit of the commodity increases The firm is willing to sell less quantity at the same price So the supply of the commodity decreases at the same price This decrease in supply is shown by leftward shift of the supply curve The above cases of increase and decrease in supply can be shown with the help of the following figures

Y INCREASE OF SUPPLY Price (Rs) s

P A s1

B

s

X` O s1 X

q q1

Y` Quantity demanded (in units)

Y DECREASE IN SUPPLY s2

s

price (Rs)

C

p A

s2

s

X` o X

q2 q

Y` Quantity demanded ( in units)

Main factors causing increase in supply or rightward shift of supply Curve(i) Fall in the price of other related goods

(ii) Fall in the price of inputsfactors(iii) Use of better technology in production(iv) Decrease in the rate of excise duty by government(v) If the objective of producer changes from profit maximization to salesMaximization

Main factors causing decrease in supply or leftward shift of supply curve(i) Increase in the price of other related goods(ii) Rise in the price of inputsfactors(iii) Use of inferior technology in production(iv) Increase in the rate of excise duty by the government(v) If the objective

Subject - Biology Topic ndash Chapter mdash6 PhotosynthesisSummary Execution

Today we will know about photosynthesis and its stages

Q1 What do you mean by photosynthesis The process by which living plants containing chlorophyll produce food

substances from carbon-di- oxide and water by using light energy Sunlight

6CO2 +12 H2O----------------------- C6 H12O6 + 6H2O + 6O2

Chlorophyll

Q2 What are the importance of photosynthesis I) Food for all Green plants trap solar energy by photosynthesis

process and supply food and energy for all living organisms either directly or indirectly

Ii) Oxygen to breathe in by product of photosynthesis is oxygen which is essential for all living organisms respiration

Q3 Write about two main phases of photosynthesis A Light dependent phase This phase occur in grana of chloroplast I) The chlorophyll on exposure to light energy becomes activated by

absorbing photons Ii) The absorbed energy is used in splitting the water molecules (H2O)

into its two components (H+ and OH- ) and releasing electron s 2H2O------------------------- 4H+ + 4e- +O2

Energy of 4 photons This reaction is known as photolysis

End products are H+ and oxygen water

B Light independent (Dark ) phase The reactions in this phase require no light energy

Here CO2 combine with H+ and produce glucose

Class XI

Subject Topic Summary ExecutionEVS Chapter-4 Legal

regimes for sustainable development

Environmental legislationEnvironmental legislation is the collection of laws and regulations pertaining to air quality water quality the wilderness endangered wildlife and other environmental factors The act ensures that matters important to the environment are thoroughly

Learn -The Forest (Conservation) Act 1980

considered in any decisions made by federal agencies

The Forest (Conservation) Act 1980 The Forest (Conservation) Act 1980 an Act of the Parliament of India to provide for the conservation of forests and for matters connected therewith or ancillary or incidental thereto It was further amended in 1988 This law extends to the whole of IndiaObjects and Reasons of the Forest Conservation Act

Deforestation causes ecological imbalance and leads to environmental deterioration Deforestation had been taking place on a large scale in the country and it had caused widespread concern The act seeks to check upon deforestation and de-reservation of forests

Subject Eng Literature (The Tempest ndash William Shakespeare) Topic Act II Scene 1 Lines 314 to 329 (End of scene)

[Students should read the original play and also the paraphrase given in the school prescribed textbook]Summary Questions amp Answers

Conspiracy of Antonio and Sebastian (Contd)

o As they approach Ariel appears again and wakes up Gonzalo by singing a tune in his ear Alonso also wakes up and they see both Sebastian and Antonio with drawn swords On being caught off guard they make up a story saying that they had heard a bellowing of bulls or lions

o They then moved to another part of the island

o Ariel at once rushes to Prospero to inform him of this development

SUMMING-UP of ACT-2 SCENE-1

(i) Among the survivors Ferdinand is separated from the rest which results in the disconsolate grief of Alonso as he took him for dead

(ii) The villainy of Antonio is confirmed

(iii) The supremacy of Prosperorsquos magic which resulted in the failure of the human conspiracy

(1)

(Act II Sc 1 L 311-325)SEBASTIAN Whiles we stood here securing your repose

Even now we heard a hollow burst of bellowing Like bulls or rather lions Didt not wake youIt struck mine ear most terribly

ALONSO I heard nothingANTONIO O rsquotwas a din to fright a monsters ear

To make an earthquake Sure it was the roarOf a whole herd of lions

ALONSO Heard you this GonzaloGONZALO Upon mine honour sir I heard a humming

And that a strange one too which did awake meI shaked you sir and cried As mine eyes opened I saw their weapons drawn There was a noiseThats verily rsquoTis best we stand upon our guardOr that we quit this place Lets draw our weapons

(i) Why has Prospero sent Ariel to Gonzalo and Alonso What does Ariel do to awaken Gonzalo

Prospero has already come to know by his magic powers the danger which threatens Gonzalo who had been Prosperorsquos friend and so he sent Ariel to preserve the lives of both Gonzalo and Alonso Prospero does not want that his scheme should remain unfulfilled Ariel begins to sing a song in Gonzalorsquos ears to awaken him(ii) Who are ready to carry out their plan Who takes steps to stop them Why does Gonzalo feel surprised after being awakened

Sebastian and Antonio are ready to carry out their plans They are standing with their swords drawn to kill Alonso and

(iv) We see two sets of contrasting characters Gonzalo-Adrian against Antonio-Sebastian

(v) The grief that works in Alonso can be perceived to his repentance for his association in Antoniorsquos crime against Prospero

Gonzalo Ariel takes steps to stop them from carrying out their nefarious scheme When Gonzalo is awakened by the song sung by Ariel into his ears he (Gonzalo) feels surprised because he sees Sebastian and Antonio standing with their swords drawn(iii) What reason do Sebastian and Antonio tell of drawing their swords when they are suspected by Alonso and Gonzalo

When Sebastian and Antonio are seen with their swords drawn they are looked with suspicion by Gonzalo and Alonso At first Sebastian tells them that as they stood here to guard them during their sleep they heard only a little before a sudden loud noise very much like the roaring of bulls or more probably that of lions Then Antonio follows him saying that this was a noise so terrible as to frighten even a monsterrsquos ears and this noise could even have shaken the earth and it was surely like the roaring of a multitude of lions Then seeing the danger they have drawn their swords Perhaps after hearing the terrible noise they (Gonzalo and Alonso) woke up from their sound sleep

(iv) What does Gonzalo tell Alonso about the strange noise What did he see on opening his eyes Gonzalo tells Alonso that he did not hear the sound of roaring but he heard a humming sound which was strange and which woke him up After waking up he gave him (Alonso) a shaking and a loud cry On opening his eyes he saw these two gentlemen standing with their swords drawn(v) What does Gonzalo suggest

Gonzalo suggests that there was a noise indeed and of that he has no doubt at all and suggests that the best course for them would be to remain alert and vigilant against any possible danger to their lives or to leave this place and move to some other part of the island

Class XIISubject Topic Summary Execution

Commerce

Chapter- Management

Today we will discuss about LEVELS OF MANAGEMENT

Levels of management is a series or chain of managerial positions from top to bottom It helps individuals to know their authority responsibilities and superior-subordinate relations among themselves There are mainly three levels of Management TOP LEVEL MANAGEMENTMIDDLE LEVEL MANAGEMENTLOWER LEVEL MANAGEMENT

Top level managementIt consists of members at the highest level in the management hierarchy This level includes Board Of Directors Chief Executive Managing Directors Chairman President Vice President

Rolefunctions of the top levelmanagement1To analyse evaluate and deal

with theexternal environment2 To determine the objectives and

policies of the business3 To strive for welfare and survival

of business

4 To create an organisational Framework consisting of authority responsibility relationship

Middle level management Congress of members or groups who are concerned with implementation of the policies let down by the top managementThis level includes head of the department such as finance manager marketing manager branch and regional managers departmental and divisional heads plant superintendent etc

Role of functions of the middle level management

1 To interpret the policies framed by top management

2 To assign duties and responsibilities to lower level managers

3 To select and appoint employees for middle and supervisory level and evaluate their performance

4 To co-operate with other departments for smooth functioning

Operational or supervisory level managementIt refers to the group are members who are concerned with execution of the work They are also known as fast line managers This level includes supervisor 4 men Section Officer clerk Inspector etc

Role of functions of the lower level management1 To plan and execute day-to-

day operations2 To supervise and control the workers3 To arrange materials and

tools to start the process and make arrangements for training

4 Today present workers grievance and suggestions before the management and

ensure safe and proper working conditions in the factory

Business Studies

Staff Appraisal Chapter- 10 Today let us start with a new chapter

Staff Appraisal

Meaning of Performance Appraisal

Performance Appraisal is the systematic evaluation of the performance of employees and to understand the abilities of a person for further growth and developmentThe supervisors measure the pay of employees and compare it with targets and plansThe supervisor analyses the factors behind work performances of employeesThe employers are in position to guide the employees for a better performance

Objectives of Performance Appraisal

Following are the objectives of Performance Appraisal

To maintain records in order to determine compensation packages wage structure salaries raises etc

To identify the strengths and weaknesses of employees to place right men on right job

To maintain and assess the potential present in a person for further growth and development

To provide a feedback to employees regarding their performance and related status

To provide a feedback to employees regarding their performance and related status

Importance of Performance Appraisal

Performance appraisal provides important and useful information for the assessment of employees skill

knowledge ability and overall job performance The following are the points which indicate the importance of performance appraisal in an organization

1 Performance appraisal helps supervisors to assess the work performance of their subordinates

2 Performance appraisal helps to assess the training and development needs of employees

3 Performance appraisal provides grounds for employees to correct their mistakes and it also provides proper guidance and criticism for employees development4 Performance appraisal provides reward for better performance

5 Performance appraisal helps to improve the communication system of the organization

6 Performance appraisal evaluates whether human resource programs being implemented in the organization have been effective

7 Performance appraisal helps to prepare pay structure for each employee working in the organization

8 Performance appraisal helps to review the potentiality of employees so that their future capability is anticipated

Geography

DRIANAGE The SubarnarekhaThe Subarnarekha and the Brahmaniinterposed between the Ganga and the Mahanadi deltas drain an area of 19300 sq kmand 39033 sq km respectively The drainage basins of these streams are shared byJharkhand Odisha west Bengal and Chhattisgarh The Brahmani is known as southKoel in its upper reaches in Jharkhand

The NarmadaThe Narmada rises in the Amarkantak hills of MadhyaPradesh It flows towards the West in a rift valleyformed due to a geological fault The total length of it is 1300 km All the tributaries of the

Q1 Name the two westward flowing rivers in the peninsular plateauA1 Narmada and Tapi are the only westward flowing rivers of the peninsular plateau

Q2 Differentiate between east-flowing rivers and west-flowing riversA2

East-flowing rivers

West-flowing rivers

Narmada are very short inlength Most of its tributaries join the main streamright anglesThe Narmada basin covers parts of Madhya Pradesh and Gujarat

The Tapi The Tapi rises in the Satpura ranges in the Betul listrictof Madhya Pradesh It flows in a rift valley parallel tothe Narmada but it is much shorter in length It coversparts of Madhya Pradesh Gujarat and MaharashtraThe length is about 724 km

The Sabarmati and the MahiThe Sabarmati rises in the Aravali hills and flows south-south-westwards for a distance of 300 kilometres to the Arabian Sea The Sabarmatibasin extends over an area of 21674 sq km in Rajasthan and Gujarat The Mahi rises inthe east of Udaipur and drains an area of 34842 sq km lying in Madhya PradeshRajasthan and Gujarat It flows south-westwards for a distance of 533 km before it fallsinto the Gulf of Khambhat

The ChambalThe Chambal rises near Mhow in the Vindhya Range and flows towards the northgenerally in a gorge upto Kota Below Kota it turns to the north-east direction and afterreaching Pinahat it turns to the east and runs nearly parallel to the Yamuna beforejoining it in the southern part of the Etawah district in Uttar PradeshMajor Rivers of India with their basin area (Sqkm)

Himalayan System Indus 321290Ganga 861404

Brahmaputra 187110Indus System

Jhelum 34775Beas 20303

Ganga System Yamuna 366223Ghaghra 127950

Peninsular RiversNarmada 98796

Tapi 65145Mahanadi 141600

Subarnarekha 19300Sabarmati 21674

Mahi 34842Godavari 312812

Godavari Krishna Kaveri Mahanadi are the east-flowing rivers

Narmada Tapi west-flowing rivers

They fall into the Bay of Bengal

They fall into Arabian Sea

These rivers form big deltas

These rivers form comparativelysmall deltas

Catchment areas of these rivers are larger

Catchment areas of these rivers are smaller

Krishna 2589488Cauveri 87900

Subject ndashBiology Topic ndashChapter -5 Inheritance amp Variations Summary ExecutionToday we will discussabout linkage and its classification

LINKAGE The tendency of the genes located on the same chromosome to stay together is

hereditary transmission Linked genes the genes responsible for this Genes that exhibit the process of linkage locates in the same chromosome The distance between the linked genes in a chromosome determines the strength

of linkage i e genes that are located close to each other show stronger linkage than that are located far from each other

COMPLETE LINKAGE It is the type of linkage showed by the genes that are closely located or are tightly

linked with each other as they have no chance of separatingby crossing over These genes are always transmitted together to the same gamete and the same

offspring In such condition only parental or non cross over type of gametes are formedINCOMPLETE KINKAGE It is type of linkage showed by the genes that are distantly located orare loosely

linked with each other because they have chance of separating by crossing over

SIGNIFICANCE i) It helps in holding the parental character togetherii) It checks the appearance of new recombination and helps in bringing the

hybrid population which resembles the original parents iii) Linked genes dilute the effects of undesirable traits

Subject Eng Literature (The Tempest ndash William Shakespeare) Topic Essay Questions (EQ-3)Question No 3

Give a character sketch of CalibanAnswer

The character of Caliban has been wonderfully conceived by Shakespeare as the manifestation of all that is gross and earthy ndash a sort of creature of the earth as Ariel is a sort of creature of the air

Calibanrsquos Physical Appearanceo Caliban is lsquofreckledrsquo a lsquomisshapen knaversquo not honoured with human shape

o Prospero calls him lsquothou tortoisersquo (Act I Sc 2 Line 317) Trinculo stumbling upon him describes him as ldquoA strange fish hellip Legged like a man And his fins like armsrdquo He ldquosmells like a fishrdquo (Act II Sc 2 Line 25)

o Prospero also calls him a ldquobeastrdquo (Act IV Sc 1 Line 140) and ldquoThis misshapen knaverdquo (Act V Sc 1 Line 268)

o Further it appears that in addition to his physical deformity his spiritual inferiority is also suggested by Prosperorsquos claim that his birth resulted from the union between his mother the witch Sycorax and the devil

Calibanrsquos ParentageWhen the play opens Caliban is twenty four years of age having been born on the island twelve years before the coming of Prospero His mother was the foul witch Sycorax who was banished from Algiers for ldquomischiefs manifold and sorceries terrible to enter human hearingrdquo (Act I Sc 2 Line 264) and the father was the Devil himself Thus

Caliban is a monster of evil and brute nature ugly deformed and stinking

Calibanrsquos Savage and Malignant Natureo Caliban is entirely a creature of the earth ndash gross brutal and savage He regards himself as the rightful possessor

of the island and Prospero as a usurper

o In his young age he was on good terms with Prospero He had consented to be received by Prospero at his house and to be educated by him He has learnt human language only to curse his master whom he abhors

o His beastly nature soon breaks out and ends in a vicious attack on Miranda This opens the eye of Prospero who becomes severe to him and enforces his service by threats and violence

o Prospero uses him to make dams for fish to fetch firewood scraper trenches wash dishes and keep his cell clean

Calibanrsquos Hatred for ProsperoA profound hatred for Prospero has taken hold of Caliban It springs from a sense of his being dispossessed and ill-treated He would kill Prospero if he could but he knows the power of Prosperorsquos lsquobookrsquo Hence he transfers his allegiance to Stephano who seems like a god to him He also incites the two drunken associates to batter the skull of Prospero when he sleeps in the afternoon

Caliban Shows Considerable Intelligenceo He has learnt Prosperorsquos language

ldquoYou taught me language and my profit onrsquot (Act II Sc 2 Lines 86-89)Is I know how to curserdquo

o He is well aware of the futility of arguing with one who has more power than he has

ldquoI must obey his art is such power (Act I Sc 2 Lines 373-376)It would control my damrsquos god SetebosAnd make a vassal of himrdquo

o He realizes the importance of Prosperorsquos books

ldquoRemember (Act III Sc 2 Lines 89-92)First to possess his books for without themHersquos but a sot as I am nor hath notOne spirit to commandrdquo

o He knows the value of stealth when attacking the enemy

ldquoPray you tread softly that the blind mole may not (Act IV Sc 1 Lines 194-195)Hear a foot fall we now are near his cellrdquo

o Caliban has a better set of values than Stephano and Trinculo They are distracted from their plan by their greed for Prosperorsquos rich garments Only Caliban realizes that such a finery is unimportant

ldquoLeave it alone thou fool it is but trashrdquo (Act IV Sc 1 Lines 224)

Caliban is not a good judge of characterCaliban is not a good judge of character He decides for example that Stephano is a god because he dispenses lsquocelestial liquorrsquo (Act II Sc 2 Line 115) but then it must be remembered that he has only known his mother Sycorax Prospero Miranda and the spirits that torture him However he quickly discovers his error of judgementrdquo

ldquoWhat a thrice-double ass (Act V Sc 1 Lines 295-297)Was I to take this drunkard for a godAnd worship this dull foolrdquo

Calibanrsquos Imaginative NatureIf Caliban is sub-human in what has been said above he is human in the respect of the poetic side of his character He listens to music with rapture He tells of the beautiful dreams in which heaven rains treasures upon him and which upon waking he yearns to renew One of the most poetic passages in whole play is Calibanrsquos description of the island

to Stephano and Trinculo

ldquoBe not afeard The isle is full of noises (Act III Sc 2 Lines 135-143)Sounds and sweet airs that give delight and hurt notSometimes a thousand twangling instrumentsWill hum about mine ears and sometime voicesThat if I then had waked after long sleepWill make me sleep again and then in dreamingThe clouds methought would open and show richesReady to drop upon me that when I wakedI cried to dream againrdquo

Caliban - Less Ignoble Than Some OthersCalibanrsquos motive for murder is less dishonourable than that of Antonio and Sebastian They plan to kill Alonso to gain his power and wealth Caliban merely wants revenge and the return of lsquohisrsquo island

Conclusiono Calibanrsquos character is not portrayed very clearly in the play and hence we cannot decide whether he is a poor

savage being grossly maltreated by Prospero or whether he is evil and must therefore be kept in bondage or enslavement

o Caliban is contrasted with Ariel who is a spirit and thus swift and uninterested in physical activitieso Caliban is also contrasted with Prospero who is the all-powerful master of the island and of the destiny of all

those on the islando Caliban is also contrasted with civilized man showing him to be less evil than Antonio and Stephano and less

materialistic than Stephano and Trinculoo Caliban has suffered at the hands of Prospero and he has learnt to curse by listening to Prosperorsquos abuse He

certainly believes that Prospero has deprived him of his birthrighto Finally the character Caliban is thought to be one of Shakespearersquos masterpieces The complexity of the character

is reflected in the large volume of critical discussion that has grown around it

ECO ndash12 Topic-Forms of market

MonopolyMonopoly is a market structure in which there is a single seller there are no close substitutes for the commodity produced by the firm and there are barriers to entry Example Indian Railways which is operated under government of India Monopoly also implies absence of competitionFeatures of Monopoly Monopoly is characterized by1 Single Seller In monopoly there is only one firm producing the product The whole industry consists of this single firm Thus under monopoly there is no distinction between firm and industry Being the only firm there is significant control of the firm over supply and price Thus under monopoly buyers do not have the option of buying the commodity from any other seller They have to buy the product from the firm or they can go without the commodity This fact gives immense control to the monopolist over the market

2No Close Substitute There are no close substitutes of the product produced by the monopolist firm If there are close substitutes of the product in the market it implies presence of more than one firm and hence no monopoly In order to ensure a total of control over the market by the monopolist firm it is assumed that there are no close substitutes of the product

3 No Entry amp Exit Monopoly can only exist when there is strong barriers before a new firm to enter the market In fact once a monopoly firm starts producing the product no other firm can produce the same One reason for this is the ability of the

monopolist to produce the product at a lower cost than any new firm who thinks to enter the market If a new firm who knows that it cannot produce at a lower cost than the monopolist then that firm will never enter the market for fear of losing out in competition Similarly the monopolist who is operating for a long time may be enjoying reputation among its customers and is in a better position to use the situation in its own benefit A new firm has to take long time to achieve this and so may not be interested to enter the market

4 Price Maker Being the single seller of the product the monopolist has full control over the pricing of the product On the other hand if there is a large number of buyers in the market so no single buyer exercises any significant influence over price determination Thus it is a sellerrsquos market So monopoly firm is a price maker

5 Price Discrimination Having considerable control over the market on account of being single seller with no entry of other firms the monopolist can exercise policy of price discrimination it means that the monopolist can sell different quantities of the same product to a consumer at different price or same quantity to different consumers at different prices by adjudging the standard of living of the consumer

6 Shape of Demand Curve Since a monopolist has full control over the price therefore he can sell more by lowering the price This makes the demand curve downward sloping

Subject Ac-12 290620 Topic- retirement Model sumThe Balance Sheet of Rohit Nisha and Sunil who are partners in a firm sharing profits according to their capitals as on 31st March 2014 was as under

Liabilities Amount Assets Amount (Rs) (` Rs)

Creditors 25000 Machinery 40000Bills Payable 13000 Building 90000General Reserve 22000 Debtors 30000Capital Less Provision for Rohit 60000 Bad debts 1000

29000 Nisha 40000 Stocks 23000 Sunil 40000 140000 Cash at Bank 18000

200000 200000

On the date of Balance Sheet Nisha retired from the firm and following adjustments were made(i) Building is appreciated by 20(ii) Provision for bad debts is increased to 5 on Debtors(iii) Machinery is depreciated by 10(iv) Goodwill of the firm is valued at Rs 56000 and the retiring partnerrsquos share is adjusted

(v) The capital of the new firm is fixed at Rs120000 Prepare Revaluation Account Capital Accounts of the partner and Balance Sheet of the new firm after Nisharsquos retirement Revaluation AccountDr Cr

Particulars Amount Particulars Amount (`Rs) (Rs`)

Provision for Bad debt Ac 500 Building Ac 18000Machinery Ac 4000Profit transferred toCapital Accounts (3 2 2)Rohit 5786Nisha 3857Sunil 3857

13500

18000 18000

Capital Account

Dr Cr

Particulars Rohit Nisha Sunil Particulars Rohit Nisha Sunil (Rs`) (Rs`) (`Rs) (Rs`) (Rs`) (Rs`)

Sunilrsquos Capital ac 9600 mdash 6400 Balance bd 60000 40000 40000Bank - 66143 - General Reserve 9428 6286 6286Balance cd 72000 mdash 48000 Revaluation (Profi 5786 3857 3857 Rohitrsquos Capital Ac mdash 9600 mdash

Sunilrsquos Capital Ac 6400 Bank 6386 - 4257

81600 66143 54400 81600 66143 54400

Balance Sheet as at 31st March 2014

Liabilities Amount Assets Amount (Rs`) (Rs`)

Creditors 25000 Building 108000Bank overdraft 37500 Machinery 36000

Bills Payable 13000 Debtors 30000Capital Less ProvisionRohit 72000 for Bad debts 1500 28500Sunil 48000 120000 Stock 23000

195500 195500

Working Notes (i) (a) Profit sharing ratio is 60000 40000 40000 ie = 3 2 2(b) Gaining Ratio Rohit = 35 ndash 37 = 2135 ndash 1535 = 635Sunil = 25-27 = 1435 ndash 1035 = 435= 635 435= 6 4 = 3 2(c) Nisha Share of Goodwill = Rs 56000 times 27 = Rs16000Share of Goodwill in the gaining ratio by the existing partner ieRohit = Rs16000 times 35 = Rs 9600Sunil = Rs 16000 times 25 = Rs 6400

The journal entry isRohitrsquos Capital Ac Dr 9600Sunilrsquos Capital Ac Dr 6400 To Nisharsquos Capital Ac 16000(Share of Goodwill divided into gaining ratio)

  • 1 Static Friction
  • The frictional force that acts between the surfaces when they are at rest with respect to each other is called Static Friction
    • Static Friction Examples
      • 2 Sliding Friction
        • Examples Of Sliding Friction
          • 3 Rolling Friction
            • Examples Of Rolling Friction
              • Objects and Reasons of the Forest Conservation Act
Page 33:  · Web viewSubject . Topic . Summary . Execution . English 1 . Sounds of animals . Hens –cackle Horses –neigh Lions –roar Owls –hoots Snake –hiss. English 2 . Mother’s

During winter solstice (22 December) the South Pole is inclined towards the Sun The Southern Hemisphere experiences summer and the Northern Hemisphere has winter Therefore the duration of daylight or sunlight is greater in the Southern Hemisphere than in the Northern HemisphereThe duration of daylight increases from 12 hours at the equator to 24 hours beyond the Antarctic Circle The South Polar Region has 24 hours of sunlight for many days continuously At the South Pole there will be six months of sunlight The Sun will always be seen at a low angle above the horizon In the Northern Hemisphere the duration of daylight will decrease from 12 hours at the equator to 0 hours at the Arctic Circle There are 24 hours of darkness in the North Polar region The duration of night is greater than the duration of daylight as one move northwards from the Equator It is evident from the above table that the duration of daylight is 12 hours throughout the year at the equator only As one moves away from the equator the seasonal variations in the duration of daylight increase The seasonal variations in the duration of daylight are maximum at the Polar Regions

Subject Eng Literature (The Merchant of Venice ndash William Shakespeare)Topic Act II Scene 7 Lines 36 to 80 (End of scene ) [Students should read the original play and also the paraphrase provided]

Summary Questions amp AnswersThe Prince then examines the inscription on the silver casket which says ldquoWho chooseth me shall get as much as he deservesrdquo The Prince says that he deserves Portia more than anybody else because of his high rank his noble birth and his great wealth and power But then he argues that silver is ten times

(1) (Act II Sc 7 L 39-47)

From the four corners of the earth they come

To kiss this shrine this mortal breathing saint

The Hyrcanian deserts and the vasty wildsOf wide Arabia are as through-fares now

inferior to gold and therefore he cannot believe that the portrait of such a beautiful lady as Portia can be contained in the silver casket He decides to see the inscription on the golden casket before making his decision

The Prince goes to examine the inscription on the golden casket which says ldquoWho chooseth me shall get what many men desirerdquo The Prince believes that the whole world desires to possess Portia otherwise so many suitors would not have come from all corners of the world for winning Portia Some of them have come from the distant lands of Persia and Arabia The deserts of Persia (Hyrcanian deserts) and the boundless desolate lands of Arabia have been crossed by the Princes seeking the hand of Portia He contrasts this casket containing Portiarsquos portrait with the old English gold coin bearing the image of the archangel (angel of the highest rank) He goes on to remark that while the figure of the archangel is engraved (Insculped) upon the English coin the picture of Portia who is beautiful as an angel lies hidden inside one of the caskets namely the Golden Casket (Golden Bed)

On the basis of his assessment of the inscription on the golden casket the Prince decides to choose the golden casket He asks for the key and opens the golden casket only to find therein an empty human skull holding a roll of

For princes to come view fair PortiaThe watery kingdom whose ambitious headSpets in the face of heaven is no barTo stop the foreign spirits but they comeAs orsquoer a brook to see fair Portia

(i) Explain the occasion for the above mentioned speech

These are the comments of the Prince of Morocco after he reads the inscription on the golden casket His mental process is revealed to us in these words We find him debating within himself as to which casket he should choose

(ii) What light does the above speech throw on the personality of Prince of Morocco

From the above mentioned speech we come to know that the Prince of Morocco is keen to marry Portia He is the type of person who is easily taken away by outward appearance He is in love with Portia because of her beauty

(iii) What information can you gather about Portia from the above mentioned lines

The given speech shows that Portia is a very beautiful lady She must be possessed of good qualities because many suitors come to her place from all over the world with a desire to get married to her The Prince of Morocco is so impressed by her beauty that he calls her a saint According to him the whole world is desirous of having her

(iv) Elucidate the significance of the first two lines

In these lines the Prince of Morocco pays a compliment to Portia These lines show his admiration for her He says that people come from all parts of the world to see fair Portia

(v) Explain the meaning of the last four lines of the

passage

In these lines the Prince of Morocco says that even the vast oceans which throw a challenge at the sky are unable to prevent men from coming to Portiarsquos place to have a glimpse of her These lines are also a tribute to Portiarsquos beauty and good qualities Many men voyage across the ocean treating it as a mere stream to see the beautiful Portia

paper in which is written that whoever happens to be guided by the glitter of things is invariably deceived

On reading the scroll the Prince says that he is too sad at heart to speak a more formal farewell and leaves with his followers amidst a sound of trumpets

After the Prince of Morocco leaves Portia remarks that the Prince is a gentle fellow but she is rid of him May all persons of his nature make a similar choice

IMPORTANT PASSAGES EXPLAINED

(Act II Sc 7 L 39-43)From the four corners of the earth they come

To kiss this shrine this mortal breathing saintThe Hyrcanian deserts and the vasty wildsOf wide Arabia are as through-fares nowFor princes to come view fair Portia

Context

This passage occurs in Act II Scene 7 in The Merchant of Venice This is part of the speech made by the Prince of Morocco

(2)

(Act II Sc 7 L 48-53)

MOROCCO One of these three contains her heavenly pictureIst like that lead contains her

Twere damnation To think so base a thought it were too grossTo rib her cerecloth in the obscure graveOr shall I think in silver shes immurdBeing ten times undervalued to tried gold

(i) What meaning does the Prince of Morocco find out of the inscription of the golden casket What have Belmont and Portiarsquos house been called and why

The inscription on the golden casket is ldquoWho chooseth me shall gain what many men desirerdquo The Prince finds out that it means that the chooser of the golden casket will get Portia because many men desire her In fact the entire world desires her Because of the coming of many suitors to Belmont from different countries in order to win Portiarsquos hand Belmont has become a centre of pilgrimage and her house is the shrine where saintly Portia is installed

(ii) What does the Prince of Morocco do before making the final choice of the casket Which is the correct casket and who will win Portiarsquos hand

The Prince of Morocco surveys and analyses the inscriptions on the casket of lead silver and gold Before making the final choice like a very systematic and methodical person he once again considers the claims of the caskets The casket containing Portiarsquos picture is the correct casket and the person choosing it will win Portiarsquos hand

Explanation

While praising Portia the Prince of Morocco conceives Portia as a goddess whose image is placed inside one of the caskets Many suitors are coming from far and wide the north and the south the east and the west (Four corners) in order to try their luck Some of them have come from the distant land of Persia and Arabia The deserts of Persia (Hyrcanian deserts) and the boundless desolate lands of Arabia have been crossed by the Princes seeking the hand of Portia All this shows that Portia is indeed the most beautiful lady of the world

(iii) What does the Prince of Morocco say in his estimation while examining the motto on the silver casket What does he find in the golden casket

While examining the motto on the silver casket which says ldquoWho chooseth me shall get as much as he deservesrdquo Morocco says that in his own estimation he surely deserves Portia in all respects ndash rank birth wealth etc

He chooses the golden casket When he opens it he finds an empty human skull holding a scroll in which it is written that those who are attracted by the glittering outside of things are always deceived as Morocco has been deceived

(iv) What kind of nature does the Prince of Morocco have

The Prince of Morocco has a simple nature who does not look deeply into the inner meaning of things but is dazzled by the outward appearance of gold He is inclined to over-estimate his own value and does not realize that it is a duty to ldquogive and hazardrdquo To say that he will not hazard for lead shows that he misreads the true meaning of the inscription which is that he should be prepared to ldquohazard all he hathrdquo for Portia So his feeling is only one of fascination and romantic attraction

(v) Do you think that the lottery of the caskets is not a matter that will be determined by chance

In fact the lottery of the casket is not a matter that will be determined by mere chance but that it is a true test of character and of sincerity which is amply proved not only by Moroccorsquos choice but also by the arguments which he uses to help him in his choice

(Act II Sc 7 L 55-59)

They have in England

A coin that bears the figure of an angelStamped in gold but thats insculpd uponBut here an angel in a golden bedLies all within

Context

(3)

(Act II Sc 7 L 63-77)A carrion Death within whose empty eye

There is a written scroll Ill read the writing

All that glisters is not goldOften have you heard that toldMany a man his life hath soldBut my outside to beholdGilded tombs do worms infoldHad you been as wise as boldYoung in limbs in judgment oldYour answer had not been inscrolld

This passage occurs in Act II Scene 7 in The Merchant of Venice This is part of the speech made by the Prince of Morocco

Explanation

In this passage the Prince of Morocco bestows high praise on Portia whose hand he is seeking He contrasts this casket containing Portiarsquos portrait with the old English gold coin bearing the image of the archangel (angel of the highest rank) He goes on to remark that while the figure of the archangel is engraved (Insculped) upon the English coin the picture of Portia who is beautiful as an angel lies hidden inside one of the caskets namely the Golden Casket (Golden Bed) In the day of Elizabeth silver was ten times inferior in value to gold Therefore the Prince of Morocco believing that Portiarsquos portrait is contained in the Golden Casket decides to choose the Golden Casket

Fare you well your suit is coldCold indeed and labour lostThen farewell heat and welcome frostmdashPortia adieu I have too grievd a heartTo take a tedious leave Thus losers part

(i) What reward does the Prince of Morocco get after making a wrong choice of the Casket How does he feel

After making the wrong choice in selecting the casket of gold the Prince of Morocco as a reward earns a rebuke in the form of a scroll tucked in the empty eye-socket of a skull kept in the casket of gold The Prince is shocked and disappointed He becomes all the more sad and dejected when he reads the scroll which points to his foolishness in being misled by the appearance and outward show as indicative of its worth

(ii) How does the Prince respond after reading the scroll

After reading the scroll the Prince though upset accepts the result with good grace and decorum befitting a royal suitor and true sportsman He says that his love-suit is really cold otherwise he would have chosen correctly but now his efforts have been in vain So he bids farewell to Portia to the warmth and enthusiasm of love and welcomes the cold and bitterness of dejection and misery of life which lies ahead

(iii) What request does he make to Portia and why

After being failure in his mission he requests Portia to give him permission to leave at once because he is too sad to undergo the tediousness of a formal leave-taking He tells that it is the manner in which defeated persons part unceremoniously

(iv) Explain the following lines

ldquoAll that glisters is not goldOften have you heard that toldMany a man his life hath soldBut my outside to beholdGilded tombs do worms infoldrdquo

Mere glitter does not make a metal to be gold Man has often been warned against appearance but it has been of no use Many people have sacrificed their lives only to seek the outer appearance of gold Worms are found inside the gilded

monuments

Class XSubject Topic Summary Execution

Hindi 2ndlang

नया रासता भाग 6 मायाराम 0ी घर म धनी मल 0ी और उनी बटी सरिरता ी ही चचा बनी रहती थी अमिमत ो इसम ोई रलिच ना थी वह धनी घर ी लडी स शादी र सवय ो बचना नही चाहता था उसा भी सवाणिभमान ह ईशवर ी पा

स उस पास पस ी ोई मी नही थी अभी उसन फकटरी ही लगाई थी उसी समझ बाहर था कि उस घर वालो ा झाव पस ी तरफ कयो

ह उसन मा स सवाल किया कि मा तम सरिरता स मरी शादी कयो रना चाहती हो मा न उस समझाया कि वह दखन म बरी नही ह और किफर खानदान अचछा

ह वह ए शल गरहणी रप म घर सभाल सगी अमिमत न मा ो इस बात ा एहसास राया कि मीन सबध लिलए मना रन पर उस दिदल

पर कया बीती होगी मा और अमिमत ी लडी बार म ाफी बात हईमा ा झाव सरिरता ी तरफ था कयोकि वह घर पर अचछा दह0 लर आ रही

थी अमिमत न अपनी मौसी ी बरी हालत बार म बताया कि किस तरह वह बड घर ी खानदानी बटी लाई थी और आ0 उसी हालत कितनी खराब ह लाई थी बहकलब 0ाती ह और बचचो ो भी नही दखती ह बात चल ही रही

थी कि तभी ए ार बाहर आर री धनी मल0ी घर अदर आए और पीछ स डराइवर फल ी ए टोरी लर आया अदर आए और पीछ स

डराइवर ए टोरी फल ी लर आया अमिमत ो फल ी पटी बरी लग रही थी अमिमत न पछ लिलया यह फल कयो ल आए ह प इन सब ी कया

0ररत थी उनो न 0वाब दिदया कि 4 पटी शमीर स मगाए थ अमिमत ो या सनर करोध आ गया तभी उस किपता 0ी आ गए उन आत ही अमिमत उठर बाहर चला गया वहा वहा मा पास आर बठ गया और बोला

अभी रिरशता तय नही हआ और धनी मल 0ी धनी मल 0ी फल ी पटी लर चलआय मा न समझाया कि 0ब सबध 0ड 0ाता ह तो खाली हाथ नही

आत अमिमत न मा स हा कि तम सबन सरिरता ो इस घर म लान ी ठान रखी ह धनीमल 0ी उस दिदन सरिरता ो दखन ी तारीख तय रन आय थ

Commercial Studies

Banking Nowadays Bank provide easy and quick services through internet facilities methods of Banking is called internet bankingIn order to save the time and money involved in visiting Bank branches people increasingly prefer to have internet banking

There are different modes of doing internet banking or transferring money through online They areReal Time Gross Settlement (RTGS)National Electronic Fund Transfers (NEFT)

1

Question

1) Explain the term RTGS Write the features of RTGS

Answer)The acronym RTGS stands for Real Time Gross Settlement which may be defined as the continuous real time settlement of funds transfer individually on and order by order basis without netting lsquoReal timersquo may be defined as the processing of instructions at the time they are received rather than at some letter time lsquoGross settlementrsquo may be defined as the settlement of transfer instructions which occurs

individually

Features of RTGS1It is the continuous settlement of

funds transfer individually on an order by order basis

2RTGS facility is provided only by CBS core banking solution enabled Bank branches

3Amount charged from the customer for RTGS transactions vary from bank to bank

2) Explain the term NEFT Write the features of NEFT

Answer) National electronic funds transfer may be defined as a nationwide system that facilitates individuals Farms and copper operates to electronically transfer funds from any bank branch to any individual farm or corporate having an account with any other bank branch in the country

Features of NEFT2 Transfer can be made 7 times on

weekdays and 6 times on Saturday

3 NEFT cannot be used to receive foreign remittances

4 NEFT transaction takes place in batches

5 A bank branch must be NEFT enabled to become a part of NEFT fund transfer network

6 There is no maximum or minimum amount that can be transferred through NEFT when one bank has a bank account

English Language

CompositionEssay

A composition is an art of creating a piece of writing on any topic or subject It is the writing correctly beautifully and clearly in order to make some interesting reading Structure of the composition

Introduction ( you lay the foundation for your composition)

Body (it constitutes the main part of the essay)

Conclusion (final statement that leaves a lasting impression)

Kinds of essays1 The Narrative essay2 The descriptive essay3 The reflective essay4 The argumentative essay

Write a composition on any one of the following topics (350- 400 words)

1 Friendship Or2 The first day of your school

Subject Eng Literature (The Merchant of Venice ndash William Shakespeare)Topic Act V Scene 1 Lines 127 to 158 (Nerissa helliphellip The clerk will nersquoer wear hair onrsquos face that had it) [Students should read the original play and also the paraphrase given in the school prescribed textbook]

Summary Revision Questions o Soon thereafter Bassanio Gratiano

and Antonio arrive

o Bassanio tells Portia that he is feeling as if it is morning because of the presence of Portia who is shining like the sun When Antonio is introduced by Bassanio to Portia she tells Bassanio that he should be grateful to Antonio who took so much trouble on his account even to the extent of risking his life

o Nerissa starts quarrelling with Gratiano and demands that he show her the ring she had presented to him and which she had warned him not to lose She suspects that Gratiano must have presented the ring to some young woman and not to the lawyerrsquos clerk as he repeatedly says and assures

Answer the following questions to check your preparation of Act IV Scenes 1 and 2

You must attempt only after you have completed your preparation of Act IV The answers must be in complete sentences using textual evidence (with citation) when necessary

[It would be in your own interest to attempt the above questions honestly totally refraining from consulting your textbook or your notes during answering After completion you should correct the paper yourself consulting the textbooknotes etc and award marks as specified Please let me know the marks you scored through WhatsApp in the group or to my personal WhatsApp]

Act IV Scene 1 (each question carries 2 marks)

1 What did the Duke try to do for Antonio

2 Why does Shylock refuse to show mercy How does he justify his stance

3 Why does Antonio say he is ready to die 4 What information is contained in Bellariorsquos letter

5 Why does Portia (as Balthazar) assert that Shylock must show mercy How does he respond

6 What offers are made to Shylock to get him to spare Antonio How are they received

7 What does Antoniorsquos speech as he faces the prospect of Shylockrsquos knife tell you about his character

8 How do Bassanio and Gratiano react to the looming prospect of Antoniorsquos demise

9 How does Portia (as Balthazar) use the law to turn the tables on Shylock

10 What does the Duke decree should happen to Shylock Why What happens to Shylockrsquos estate

11 What does Portia ask Bassanio as payment for her ldquoservicesrdquo What is his initial response What makes him change his mind

Act IV Scene 2 (each question carries 1frac12 marks)

1 What does Gratiano bring to Portia (Balthazar)

2 What does Nerissa plan on getting from Gratiano What does Portiarsquos comment suggest about men

ECO-10 280620 Topic-Supply AnalysisSHIFTING OF SUPPLY

But if there is change in factors other than the price of the commodity then either more is supplied at the same price or less supplied at the same price In such cases the price of the commodity remains constant but there is a change in other factors like change in the price of inputs change in technology of production change in price of other related goods change in taxation policy of the government etc For example there is an improvement in the technology of production of the commodity in question It leads to decrease in per unit of cost production of the commodity The firm is willing to sell more quantity of the commodity at the same price So the supply other commodity increases at the same price This increase in supply is shown by rightward shift of supply curve On the other hand if the firm uses inferior technology of production the cost of production per unit of the commodity increases The firm is willing to sell less quantity at the same price So the supply of the commodity decreases at the same price This decrease in supply is shown by leftward shift of the supply curve The above cases of increase and decrease in supply can be shown with the help of the following figures

Y INCREASE OF SUPPLY Price (Rs) s

P A s1

B

s

X` O s1 X

q q1

Y` Quantity demanded (in units)

Y DECREASE IN SUPPLY s2

s

price (Rs)

C

p A

s2

s

X` o X

q2 q

Y` Quantity demanded ( in units)

Main factors causing increase in supply or rightward shift of supply Curve(i) Fall in the price of other related goods

(ii) Fall in the price of inputsfactors(iii) Use of better technology in production(iv) Decrease in the rate of excise duty by government(v) If the objective of producer changes from profit maximization to salesMaximization

Main factors causing decrease in supply or leftward shift of supply curve(i) Increase in the price of other related goods(ii) Rise in the price of inputsfactors(iii) Use of inferior technology in production(iv) Increase in the rate of excise duty by the government(v) If the objective

Subject - Biology Topic ndash Chapter mdash6 PhotosynthesisSummary Execution

Today we will know about photosynthesis and its stages

Q1 What do you mean by photosynthesis The process by which living plants containing chlorophyll produce food

substances from carbon-di- oxide and water by using light energy Sunlight

6CO2 +12 H2O----------------------- C6 H12O6 + 6H2O + 6O2

Chlorophyll

Q2 What are the importance of photosynthesis I) Food for all Green plants trap solar energy by photosynthesis

process and supply food and energy for all living organisms either directly or indirectly

Ii) Oxygen to breathe in by product of photosynthesis is oxygen which is essential for all living organisms respiration

Q3 Write about two main phases of photosynthesis A Light dependent phase This phase occur in grana of chloroplast I) The chlorophyll on exposure to light energy becomes activated by

absorbing photons Ii) The absorbed energy is used in splitting the water molecules (H2O)

into its two components (H+ and OH- ) and releasing electron s 2H2O------------------------- 4H+ + 4e- +O2

Energy of 4 photons This reaction is known as photolysis

End products are H+ and oxygen water

B Light independent (Dark ) phase The reactions in this phase require no light energy

Here CO2 combine with H+ and produce glucose

Class XI

Subject Topic Summary ExecutionEVS Chapter-4 Legal

regimes for sustainable development

Environmental legislationEnvironmental legislation is the collection of laws and regulations pertaining to air quality water quality the wilderness endangered wildlife and other environmental factors The act ensures that matters important to the environment are thoroughly

Learn -The Forest (Conservation) Act 1980

considered in any decisions made by federal agencies

The Forest (Conservation) Act 1980 The Forest (Conservation) Act 1980 an Act of the Parliament of India to provide for the conservation of forests and for matters connected therewith or ancillary or incidental thereto It was further amended in 1988 This law extends to the whole of IndiaObjects and Reasons of the Forest Conservation Act

Deforestation causes ecological imbalance and leads to environmental deterioration Deforestation had been taking place on a large scale in the country and it had caused widespread concern The act seeks to check upon deforestation and de-reservation of forests

Subject Eng Literature (The Tempest ndash William Shakespeare) Topic Act II Scene 1 Lines 314 to 329 (End of scene)

[Students should read the original play and also the paraphrase given in the school prescribed textbook]Summary Questions amp Answers

Conspiracy of Antonio and Sebastian (Contd)

o As they approach Ariel appears again and wakes up Gonzalo by singing a tune in his ear Alonso also wakes up and they see both Sebastian and Antonio with drawn swords On being caught off guard they make up a story saying that they had heard a bellowing of bulls or lions

o They then moved to another part of the island

o Ariel at once rushes to Prospero to inform him of this development

SUMMING-UP of ACT-2 SCENE-1

(i) Among the survivors Ferdinand is separated from the rest which results in the disconsolate grief of Alonso as he took him for dead

(ii) The villainy of Antonio is confirmed

(iii) The supremacy of Prosperorsquos magic which resulted in the failure of the human conspiracy

(1)

(Act II Sc 1 L 311-325)SEBASTIAN Whiles we stood here securing your repose

Even now we heard a hollow burst of bellowing Like bulls or rather lions Didt not wake youIt struck mine ear most terribly

ALONSO I heard nothingANTONIO O rsquotwas a din to fright a monsters ear

To make an earthquake Sure it was the roarOf a whole herd of lions

ALONSO Heard you this GonzaloGONZALO Upon mine honour sir I heard a humming

And that a strange one too which did awake meI shaked you sir and cried As mine eyes opened I saw their weapons drawn There was a noiseThats verily rsquoTis best we stand upon our guardOr that we quit this place Lets draw our weapons

(i) Why has Prospero sent Ariel to Gonzalo and Alonso What does Ariel do to awaken Gonzalo

Prospero has already come to know by his magic powers the danger which threatens Gonzalo who had been Prosperorsquos friend and so he sent Ariel to preserve the lives of both Gonzalo and Alonso Prospero does not want that his scheme should remain unfulfilled Ariel begins to sing a song in Gonzalorsquos ears to awaken him(ii) Who are ready to carry out their plan Who takes steps to stop them Why does Gonzalo feel surprised after being awakened

Sebastian and Antonio are ready to carry out their plans They are standing with their swords drawn to kill Alonso and

(iv) We see two sets of contrasting characters Gonzalo-Adrian against Antonio-Sebastian

(v) The grief that works in Alonso can be perceived to his repentance for his association in Antoniorsquos crime against Prospero

Gonzalo Ariel takes steps to stop them from carrying out their nefarious scheme When Gonzalo is awakened by the song sung by Ariel into his ears he (Gonzalo) feels surprised because he sees Sebastian and Antonio standing with their swords drawn(iii) What reason do Sebastian and Antonio tell of drawing their swords when they are suspected by Alonso and Gonzalo

When Sebastian and Antonio are seen with their swords drawn they are looked with suspicion by Gonzalo and Alonso At first Sebastian tells them that as they stood here to guard them during their sleep they heard only a little before a sudden loud noise very much like the roaring of bulls or more probably that of lions Then Antonio follows him saying that this was a noise so terrible as to frighten even a monsterrsquos ears and this noise could even have shaken the earth and it was surely like the roaring of a multitude of lions Then seeing the danger they have drawn their swords Perhaps after hearing the terrible noise they (Gonzalo and Alonso) woke up from their sound sleep

(iv) What does Gonzalo tell Alonso about the strange noise What did he see on opening his eyes Gonzalo tells Alonso that he did not hear the sound of roaring but he heard a humming sound which was strange and which woke him up After waking up he gave him (Alonso) a shaking and a loud cry On opening his eyes he saw these two gentlemen standing with their swords drawn(v) What does Gonzalo suggest

Gonzalo suggests that there was a noise indeed and of that he has no doubt at all and suggests that the best course for them would be to remain alert and vigilant against any possible danger to their lives or to leave this place and move to some other part of the island

Class XIISubject Topic Summary Execution

Commerce

Chapter- Management

Today we will discuss about LEVELS OF MANAGEMENT

Levels of management is a series or chain of managerial positions from top to bottom It helps individuals to know their authority responsibilities and superior-subordinate relations among themselves There are mainly three levels of Management TOP LEVEL MANAGEMENTMIDDLE LEVEL MANAGEMENTLOWER LEVEL MANAGEMENT

Top level managementIt consists of members at the highest level in the management hierarchy This level includes Board Of Directors Chief Executive Managing Directors Chairman President Vice President

Rolefunctions of the top levelmanagement1To analyse evaluate and deal

with theexternal environment2 To determine the objectives and

policies of the business3 To strive for welfare and survival

of business

4 To create an organisational Framework consisting of authority responsibility relationship

Middle level management Congress of members or groups who are concerned with implementation of the policies let down by the top managementThis level includes head of the department such as finance manager marketing manager branch and regional managers departmental and divisional heads plant superintendent etc

Role of functions of the middle level management

1 To interpret the policies framed by top management

2 To assign duties and responsibilities to lower level managers

3 To select and appoint employees for middle and supervisory level and evaluate their performance

4 To co-operate with other departments for smooth functioning

Operational or supervisory level managementIt refers to the group are members who are concerned with execution of the work They are also known as fast line managers This level includes supervisor 4 men Section Officer clerk Inspector etc

Role of functions of the lower level management1 To plan and execute day-to-

day operations2 To supervise and control the workers3 To arrange materials and

tools to start the process and make arrangements for training

4 Today present workers grievance and suggestions before the management and

ensure safe and proper working conditions in the factory

Business Studies

Staff Appraisal Chapter- 10 Today let us start with a new chapter

Staff Appraisal

Meaning of Performance Appraisal

Performance Appraisal is the systematic evaluation of the performance of employees and to understand the abilities of a person for further growth and developmentThe supervisors measure the pay of employees and compare it with targets and plansThe supervisor analyses the factors behind work performances of employeesThe employers are in position to guide the employees for a better performance

Objectives of Performance Appraisal

Following are the objectives of Performance Appraisal

To maintain records in order to determine compensation packages wage structure salaries raises etc

To identify the strengths and weaknesses of employees to place right men on right job

To maintain and assess the potential present in a person for further growth and development

To provide a feedback to employees regarding their performance and related status

To provide a feedback to employees regarding their performance and related status

Importance of Performance Appraisal

Performance appraisal provides important and useful information for the assessment of employees skill

knowledge ability and overall job performance The following are the points which indicate the importance of performance appraisal in an organization

1 Performance appraisal helps supervisors to assess the work performance of their subordinates

2 Performance appraisal helps to assess the training and development needs of employees

3 Performance appraisal provides grounds for employees to correct their mistakes and it also provides proper guidance and criticism for employees development4 Performance appraisal provides reward for better performance

5 Performance appraisal helps to improve the communication system of the organization

6 Performance appraisal evaluates whether human resource programs being implemented in the organization have been effective

7 Performance appraisal helps to prepare pay structure for each employee working in the organization

8 Performance appraisal helps to review the potentiality of employees so that their future capability is anticipated

Geography

DRIANAGE The SubarnarekhaThe Subarnarekha and the Brahmaniinterposed between the Ganga and the Mahanadi deltas drain an area of 19300 sq kmand 39033 sq km respectively The drainage basins of these streams are shared byJharkhand Odisha west Bengal and Chhattisgarh The Brahmani is known as southKoel in its upper reaches in Jharkhand

The NarmadaThe Narmada rises in the Amarkantak hills of MadhyaPradesh It flows towards the West in a rift valleyformed due to a geological fault The total length of it is 1300 km All the tributaries of the

Q1 Name the two westward flowing rivers in the peninsular plateauA1 Narmada and Tapi are the only westward flowing rivers of the peninsular plateau

Q2 Differentiate between east-flowing rivers and west-flowing riversA2

East-flowing rivers

West-flowing rivers

Narmada are very short inlength Most of its tributaries join the main streamright anglesThe Narmada basin covers parts of Madhya Pradesh and Gujarat

The Tapi The Tapi rises in the Satpura ranges in the Betul listrictof Madhya Pradesh It flows in a rift valley parallel tothe Narmada but it is much shorter in length It coversparts of Madhya Pradesh Gujarat and MaharashtraThe length is about 724 km

The Sabarmati and the MahiThe Sabarmati rises in the Aravali hills and flows south-south-westwards for a distance of 300 kilometres to the Arabian Sea The Sabarmatibasin extends over an area of 21674 sq km in Rajasthan and Gujarat The Mahi rises inthe east of Udaipur and drains an area of 34842 sq km lying in Madhya PradeshRajasthan and Gujarat It flows south-westwards for a distance of 533 km before it fallsinto the Gulf of Khambhat

The ChambalThe Chambal rises near Mhow in the Vindhya Range and flows towards the northgenerally in a gorge upto Kota Below Kota it turns to the north-east direction and afterreaching Pinahat it turns to the east and runs nearly parallel to the Yamuna beforejoining it in the southern part of the Etawah district in Uttar PradeshMajor Rivers of India with their basin area (Sqkm)

Himalayan System Indus 321290Ganga 861404

Brahmaputra 187110Indus System

Jhelum 34775Beas 20303

Ganga System Yamuna 366223Ghaghra 127950

Peninsular RiversNarmada 98796

Tapi 65145Mahanadi 141600

Subarnarekha 19300Sabarmati 21674

Mahi 34842Godavari 312812

Godavari Krishna Kaveri Mahanadi are the east-flowing rivers

Narmada Tapi west-flowing rivers

They fall into the Bay of Bengal

They fall into Arabian Sea

These rivers form big deltas

These rivers form comparativelysmall deltas

Catchment areas of these rivers are larger

Catchment areas of these rivers are smaller

Krishna 2589488Cauveri 87900

Subject ndashBiology Topic ndashChapter -5 Inheritance amp Variations Summary ExecutionToday we will discussabout linkage and its classification

LINKAGE The tendency of the genes located on the same chromosome to stay together is

hereditary transmission Linked genes the genes responsible for this Genes that exhibit the process of linkage locates in the same chromosome The distance between the linked genes in a chromosome determines the strength

of linkage i e genes that are located close to each other show stronger linkage than that are located far from each other

COMPLETE LINKAGE It is the type of linkage showed by the genes that are closely located or are tightly

linked with each other as they have no chance of separatingby crossing over These genes are always transmitted together to the same gamete and the same

offspring In such condition only parental or non cross over type of gametes are formedINCOMPLETE KINKAGE It is type of linkage showed by the genes that are distantly located orare loosely

linked with each other because they have chance of separating by crossing over

SIGNIFICANCE i) It helps in holding the parental character togetherii) It checks the appearance of new recombination and helps in bringing the

hybrid population which resembles the original parents iii) Linked genes dilute the effects of undesirable traits

Subject Eng Literature (The Tempest ndash William Shakespeare) Topic Essay Questions (EQ-3)Question No 3

Give a character sketch of CalibanAnswer

The character of Caliban has been wonderfully conceived by Shakespeare as the manifestation of all that is gross and earthy ndash a sort of creature of the earth as Ariel is a sort of creature of the air

Calibanrsquos Physical Appearanceo Caliban is lsquofreckledrsquo a lsquomisshapen knaversquo not honoured with human shape

o Prospero calls him lsquothou tortoisersquo (Act I Sc 2 Line 317) Trinculo stumbling upon him describes him as ldquoA strange fish hellip Legged like a man And his fins like armsrdquo He ldquosmells like a fishrdquo (Act II Sc 2 Line 25)

o Prospero also calls him a ldquobeastrdquo (Act IV Sc 1 Line 140) and ldquoThis misshapen knaverdquo (Act V Sc 1 Line 268)

o Further it appears that in addition to his physical deformity his spiritual inferiority is also suggested by Prosperorsquos claim that his birth resulted from the union between his mother the witch Sycorax and the devil

Calibanrsquos ParentageWhen the play opens Caliban is twenty four years of age having been born on the island twelve years before the coming of Prospero His mother was the foul witch Sycorax who was banished from Algiers for ldquomischiefs manifold and sorceries terrible to enter human hearingrdquo (Act I Sc 2 Line 264) and the father was the Devil himself Thus

Caliban is a monster of evil and brute nature ugly deformed and stinking

Calibanrsquos Savage and Malignant Natureo Caliban is entirely a creature of the earth ndash gross brutal and savage He regards himself as the rightful possessor

of the island and Prospero as a usurper

o In his young age he was on good terms with Prospero He had consented to be received by Prospero at his house and to be educated by him He has learnt human language only to curse his master whom he abhors

o His beastly nature soon breaks out and ends in a vicious attack on Miranda This opens the eye of Prospero who becomes severe to him and enforces his service by threats and violence

o Prospero uses him to make dams for fish to fetch firewood scraper trenches wash dishes and keep his cell clean

Calibanrsquos Hatred for ProsperoA profound hatred for Prospero has taken hold of Caliban It springs from a sense of his being dispossessed and ill-treated He would kill Prospero if he could but he knows the power of Prosperorsquos lsquobookrsquo Hence he transfers his allegiance to Stephano who seems like a god to him He also incites the two drunken associates to batter the skull of Prospero when he sleeps in the afternoon

Caliban Shows Considerable Intelligenceo He has learnt Prosperorsquos language

ldquoYou taught me language and my profit onrsquot (Act II Sc 2 Lines 86-89)Is I know how to curserdquo

o He is well aware of the futility of arguing with one who has more power than he has

ldquoI must obey his art is such power (Act I Sc 2 Lines 373-376)It would control my damrsquos god SetebosAnd make a vassal of himrdquo

o He realizes the importance of Prosperorsquos books

ldquoRemember (Act III Sc 2 Lines 89-92)First to possess his books for without themHersquos but a sot as I am nor hath notOne spirit to commandrdquo

o He knows the value of stealth when attacking the enemy

ldquoPray you tread softly that the blind mole may not (Act IV Sc 1 Lines 194-195)Hear a foot fall we now are near his cellrdquo

o Caliban has a better set of values than Stephano and Trinculo They are distracted from their plan by their greed for Prosperorsquos rich garments Only Caliban realizes that such a finery is unimportant

ldquoLeave it alone thou fool it is but trashrdquo (Act IV Sc 1 Lines 224)

Caliban is not a good judge of characterCaliban is not a good judge of character He decides for example that Stephano is a god because he dispenses lsquocelestial liquorrsquo (Act II Sc 2 Line 115) but then it must be remembered that he has only known his mother Sycorax Prospero Miranda and the spirits that torture him However he quickly discovers his error of judgementrdquo

ldquoWhat a thrice-double ass (Act V Sc 1 Lines 295-297)Was I to take this drunkard for a godAnd worship this dull foolrdquo

Calibanrsquos Imaginative NatureIf Caliban is sub-human in what has been said above he is human in the respect of the poetic side of his character He listens to music with rapture He tells of the beautiful dreams in which heaven rains treasures upon him and which upon waking he yearns to renew One of the most poetic passages in whole play is Calibanrsquos description of the island

to Stephano and Trinculo

ldquoBe not afeard The isle is full of noises (Act III Sc 2 Lines 135-143)Sounds and sweet airs that give delight and hurt notSometimes a thousand twangling instrumentsWill hum about mine ears and sometime voicesThat if I then had waked after long sleepWill make me sleep again and then in dreamingThe clouds methought would open and show richesReady to drop upon me that when I wakedI cried to dream againrdquo

Caliban - Less Ignoble Than Some OthersCalibanrsquos motive for murder is less dishonourable than that of Antonio and Sebastian They plan to kill Alonso to gain his power and wealth Caliban merely wants revenge and the return of lsquohisrsquo island

Conclusiono Calibanrsquos character is not portrayed very clearly in the play and hence we cannot decide whether he is a poor

savage being grossly maltreated by Prospero or whether he is evil and must therefore be kept in bondage or enslavement

o Caliban is contrasted with Ariel who is a spirit and thus swift and uninterested in physical activitieso Caliban is also contrasted with Prospero who is the all-powerful master of the island and of the destiny of all

those on the islando Caliban is also contrasted with civilized man showing him to be less evil than Antonio and Stephano and less

materialistic than Stephano and Trinculoo Caliban has suffered at the hands of Prospero and he has learnt to curse by listening to Prosperorsquos abuse He

certainly believes that Prospero has deprived him of his birthrighto Finally the character Caliban is thought to be one of Shakespearersquos masterpieces The complexity of the character

is reflected in the large volume of critical discussion that has grown around it

ECO ndash12 Topic-Forms of market

MonopolyMonopoly is a market structure in which there is a single seller there are no close substitutes for the commodity produced by the firm and there are barriers to entry Example Indian Railways which is operated under government of India Monopoly also implies absence of competitionFeatures of Monopoly Monopoly is characterized by1 Single Seller In monopoly there is only one firm producing the product The whole industry consists of this single firm Thus under monopoly there is no distinction between firm and industry Being the only firm there is significant control of the firm over supply and price Thus under monopoly buyers do not have the option of buying the commodity from any other seller They have to buy the product from the firm or they can go without the commodity This fact gives immense control to the monopolist over the market

2No Close Substitute There are no close substitutes of the product produced by the monopolist firm If there are close substitutes of the product in the market it implies presence of more than one firm and hence no monopoly In order to ensure a total of control over the market by the monopolist firm it is assumed that there are no close substitutes of the product

3 No Entry amp Exit Monopoly can only exist when there is strong barriers before a new firm to enter the market In fact once a monopoly firm starts producing the product no other firm can produce the same One reason for this is the ability of the

monopolist to produce the product at a lower cost than any new firm who thinks to enter the market If a new firm who knows that it cannot produce at a lower cost than the monopolist then that firm will never enter the market for fear of losing out in competition Similarly the monopolist who is operating for a long time may be enjoying reputation among its customers and is in a better position to use the situation in its own benefit A new firm has to take long time to achieve this and so may not be interested to enter the market

4 Price Maker Being the single seller of the product the monopolist has full control over the pricing of the product On the other hand if there is a large number of buyers in the market so no single buyer exercises any significant influence over price determination Thus it is a sellerrsquos market So monopoly firm is a price maker

5 Price Discrimination Having considerable control over the market on account of being single seller with no entry of other firms the monopolist can exercise policy of price discrimination it means that the monopolist can sell different quantities of the same product to a consumer at different price or same quantity to different consumers at different prices by adjudging the standard of living of the consumer

6 Shape of Demand Curve Since a monopolist has full control over the price therefore he can sell more by lowering the price This makes the demand curve downward sloping

Subject Ac-12 290620 Topic- retirement Model sumThe Balance Sheet of Rohit Nisha and Sunil who are partners in a firm sharing profits according to their capitals as on 31st March 2014 was as under

Liabilities Amount Assets Amount (Rs) (` Rs)

Creditors 25000 Machinery 40000Bills Payable 13000 Building 90000General Reserve 22000 Debtors 30000Capital Less Provision for Rohit 60000 Bad debts 1000

29000 Nisha 40000 Stocks 23000 Sunil 40000 140000 Cash at Bank 18000

200000 200000

On the date of Balance Sheet Nisha retired from the firm and following adjustments were made(i) Building is appreciated by 20(ii) Provision for bad debts is increased to 5 on Debtors(iii) Machinery is depreciated by 10(iv) Goodwill of the firm is valued at Rs 56000 and the retiring partnerrsquos share is adjusted

(v) The capital of the new firm is fixed at Rs120000 Prepare Revaluation Account Capital Accounts of the partner and Balance Sheet of the new firm after Nisharsquos retirement Revaluation AccountDr Cr

Particulars Amount Particulars Amount (`Rs) (Rs`)

Provision for Bad debt Ac 500 Building Ac 18000Machinery Ac 4000Profit transferred toCapital Accounts (3 2 2)Rohit 5786Nisha 3857Sunil 3857

13500

18000 18000

Capital Account

Dr Cr

Particulars Rohit Nisha Sunil Particulars Rohit Nisha Sunil (Rs`) (Rs`) (`Rs) (Rs`) (Rs`) (Rs`)

Sunilrsquos Capital ac 9600 mdash 6400 Balance bd 60000 40000 40000Bank - 66143 - General Reserve 9428 6286 6286Balance cd 72000 mdash 48000 Revaluation (Profi 5786 3857 3857 Rohitrsquos Capital Ac mdash 9600 mdash

Sunilrsquos Capital Ac 6400 Bank 6386 - 4257

81600 66143 54400 81600 66143 54400

Balance Sheet as at 31st March 2014

Liabilities Amount Assets Amount (Rs`) (Rs`)

Creditors 25000 Building 108000Bank overdraft 37500 Machinery 36000

Bills Payable 13000 Debtors 30000Capital Less ProvisionRohit 72000 for Bad debts 1500 28500Sunil 48000 120000 Stock 23000

195500 195500

Working Notes (i) (a) Profit sharing ratio is 60000 40000 40000 ie = 3 2 2(b) Gaining Ratio Rohit = 35 ndash 37 = 2135 ndash 1535 = 635Sunil = 25-27 = 1435 ndash 1035 = 435= 635 435= 6 4 = 3 2(c) Nisha Share of Goodwill = Rs 56000 times 27 = Rs16000Share of Goodwill in the gaining ratio by the existing partner ieRohit = Rs16000 times 35 = Rs 9600Sunil = Rs 16000 times 25 = Rs 6400

The journal entry isRohitrsquos Capital Ac Dr 9600Sunilrsquos Capital Ac Dr 6400 To Nisharsquos Capital Ac 16000(Share of Goodwill divided into gaining ratio)

  • 1 Static Friction
  • The frictional force that acts between the surfaces when they are at rest with respect to each other is called Static Friction
    • Static Friction Examples
      • 2 Sliding Friction
        • Examples Of Sliding Friction
          • 3 Rolling Friction
            • Examples Of Rolling Friction
              • Objects and Reasons of the Forest Conservation Act
Page 34:  · Web viewSubject . Topic . Summary . Execution . English 1 . Sounds of animals . Hens –cackle Horses –neigh Lions –roar Owls –hoots Snake –hiss. English 2 . Mother’s

inferior to gold and therefore he cannot believe that the portrait of such a beautiful lady as Portia can be contained in the silver casket He decides to see the inscription on the golden casket before making his decision

The Prince goes to examine the inscription on the golden casket which says ldquoWho chooseth me shall get what many men desirerdquo The Prince believes that the whole world desires to possess Portia otherwise so many suitors would not have come from all corners of the world for winning Portia Some of them have come from the distant lands of Persia and Arabia The deserts of Persia (Hyrcanian deserts) and the boundless desolate lands of Arabia have been crossed by the Princes seeking the hand of Portia He contrasts this casket containing Portiarsquos portrait with the old English gold coin bearing the image of the archangel (angel of the highest rank) He goes on to remark that while the figure of the archangel is engraved (Insculped) upon the English coin the picture of Portia who is beautiful as an angel lies hidden inside one of the caskets namely the Golden Casket (Golden Bed)

On the basis of his assessment of the inscription on the golden casket the Prince decides to choose the golden casket He asks for the key and opens the golden casket only to find therein an empty human skull holding a roll of

For princes to come view fair PortiaThe watery kingdom whose ambitious headSpets in the face of heaven is no barTo stop the foreign spirits but they comeAs orsquoer a brook to see fair Portia

(i) Explain the occasion for the above mentioned speech

These are the comments of the Prince of Morocco after he reads the inscription on the golden casket His mental process is revealed to us in these words We find him debating within himself as to which casket he should choose

(ii) What light does the above speech throw on the personality of Prince of Morocco

From the above mentioned speech we come to know that the Prince of Morocco is keen to marry Portia He is the type of person who is easily taken away by outward appearance He is in love with Portia because of her beauty

(iii) What information can you gather about Portia from the above mentioned lines

The given speech shows that Portia is a very beautiful lady She must be possessed of good qualities because many suitors come to her place from all over the world with a desire to get married to her The Prince of Morocco is so impressed by her beauty that he calls her a saint According to him the whole world is desirous of having her

(iv) Elucidate the significance of the first two lines

In these lines the Prince of Morocco pays a compliment to Portia These lines show his admiration for her He says that people come from all parts of the world to see fair Portia

(v) Explain the meaning of the last four lines of the

passage

In these lines the Prince of Morocco says that even the vast oceans which throw a challenge at the sky are unable to prevent men from coming to Portiarsquos place to have a glimpse of her These lines are also a tribute to Portiarsquos beauty and good qualities Many men voyage across the ocean treating it as a mere stream to see the beautiful Portia

paper in which is written that whoever happens to be guided by the glitter of things is invariably deceived

On reading the scroll the Prince says that he is too sad at heart to speak a more formal farewell and leaves with his followers amidst a sound of trumpets

After the Prince of Morocco leaves Portia remarks that the Prince is a gentle fellow but she is rid of him May all persons of his nature make a similar choice

IMPORTANT PASSAGES EXPLAINED

(Act II Sc 7 L 39-43)From the four corners of the earth they come

To kiss this shrine this mortal breathing saintThe Hyrcanian deserts and the vasty wildsOf wide Arabia are as through-fares nowFor princes to come view fair Portia

Context

This passage occurs in Act II Scene 7 in The Merchant of Venice This is part of the speech made by the Prince of Morocco

(2)

(Act II Sc 7 L 48-53)

MOROCCO One of these three contains her heavenly pictureIst like that lead contains her

Twere damnation To think so base a thought it were too grossTo rib her cerecloth in the obscure graveOr shall I think in silver shes immurdBeing ten times undervalued to tried gold

(i) What meaning does the Prince of Morocco find out of the inscription of the golden casket What have Belmont and Portiarsquos house been called and why

The inscription on the golden casket is ldquoWho chooseth me shall gain what many men desirerdquo The Prince finds out that it means that the chooser of the golden casket will get Portia because many men desire her In fact the entire world desires her Because of the coming of many suitors to Belmont from different countries in order to win Portiarsquos hand Belmont has become a centre of pilgrimage and her house is the shrine where saintly Portia is installed

(ii) What does the Prince of Morocco do before making the final choice of the casket Which is the correct casket and who will win Portiarsquos hand

The Prince of Morocco surveys and analyses the inscriptions on the casket of lead silver and gold Before making the final choice like a very systematic and methodical person he once again considers the claims of the caskets The casket containing Portiarsquos picture is the correct casket and the person choosing it will win Portiarsquos hand

Explanation

While praising Portia the Prince of Morocco conceives Portia as a goddess whose image is placed inside one of the caskets Many suitors are coming from far and wide the north and the south the east and the west (Four corners) in order to try their luck Some of them have come from the distant land of Persia and Arabia The deserts of Persia (Hyrcanian deserts) and the boundless desolate lands of Arabia have been crossed by the Princes seeking the hand of Portia All this shows that Portia is indeed the most beautiful lady of the world

(iii) What does the Prince of Morocco say in his estimation while examining the motto on the silver casket What does he find in the golden casket

While examining the motto on the silver casket which says ldquoWho chooseth me shall get as much as he deservesrdquo Morocco says that in his own estimation he surely deserves Portia in all respects ndash rank birth wealth etc

He chooses the golden casket When he opens it he finds an empty human skull holding a scroll in which it is written that those who are attracted by the glittering outside of things are always deceived as Morocco has been deceived

(iv) What kind of nature does the Prince of Morocco have

The Prince of Morocco has a simple nature who does not look deeply into the inner meaning of things but is dazzled by the outward appearance of gold He is inclined to over-estimate his own value and does not realize that it is a duty to ldquogive and hazardrdquo To say that he will not hazard for lead shows that he misreads the true meaning of the inscription which is that he should be prepared to ldquohazard all he hathrdquo for Portia So his feeling is only one of fascination and romantic attraction

(v) Do you think that the lottery of the caskets is not a matter that will be determined by chance

In fact the lottery of the casket is not a matter that will be determined by mere chance but that it is a true test of character and of sincerity which is amply proved not only by Moroccorsquos choice but also by the arguments which he uses to help him in his choice

(Act II Sc 7 L 55-59)

They have in England

A coin that bears the figure of an angelStamped in gold but thats insculpd uponBut here an angel in a golden bedLies all within

Context

(3)

(Act II Sc 7 L 63-77)A carrion Death within whose empty eye

There is a written scroll Ill read the writing

All that glisters is not goldOften have you heard that toldMany a man his life hath soldBut my outside to beholdGilded tombs do worms infoldHad you been as wise as boldYoung in limbs in judgment oldYour answer had not been inscrolld

This passage occurs in Act II Scene 7 in The Merchant of Venice This is part of the speech made by the Prince of Morocco

Explanation

In this passage the Prince of Morocco bestows high praise on Portia whose hand he is seeking He contrasts this casket containing Portiarsquos portrait with the old English gold coin bearing the image of the archangel (angel of the highest rank) He goes on to remark that while the figure of the archangel is engraved (Insculped) upon the English coin the picture of Portia who is beautiful as an angel lies hidden inside one of the caskets namely the Golden Casket (Golden Bed) In the day of Elizabeth silver was ten times inferior in value to gold Therefore the Prince of Morocco believing that Portiarsquos portrait is contained in the Golden Casket decides to choose the Golden Casket

Fare you well your suit is coldCold indeed and labour lostThen farewell heat and welcome frostmdashPortia adieu I have too grievd a heartTo take a tedious leave Thus losers part

(i) What reward does the Prince of Morocco get after making a wrong choice of the Casket How does he feel

After making the wrong choice in selecting the casket of gold the Prince of Morocco as a reward earns a rebuke in the form of a scroll tucked in the empty eye-socket of a skull kept in the casket of gold The Prince is shocked and disappointed He becomes all the more sad and dejected when he reads the scroll which points to his foolishness in being misled by the appearance and outward show as indicative of its worth

(ii) How does the Prince respond after reading the scroll

After reading the scroll the Prince though upset accepts the result with good grace and decorum befitting a royal suitor and true sportsman He says that his love-suit is really cold otherwise he would have chosen correctly but now his efforts have been in vain So he bids farewell to Portia to the warmth and enthusiasm of love and welcomes the cold and bitterness of dejection and misery of life which lies ahead

(iii) What request does he make to Portia and why

After being failure in his mission he requests Portia to give him permission to leave at once because he is too sad to undergo the tediousness of a formal leave-taking He tells that it is the manner in which defeated persons part unceremoniously

(iv) Explain the following lines

ldquoAll that glisters is not goldOften have you heard that toldMany a man his life hath soldBut my outside to beholdGilded tombs do worms infoldrdquo

Mere glitter does not make a metal to be gold Man has often been warned against appearance but it has been of no use Many people have sacrificed their lives only to seek the outer appearance of gold Worms are found inside the gilded

monuments

Class XSubject Topic Summary Execution

Hindi 2ndlang

नया रासता भाग 6 मायाराम 0ी घर म धनी मल 0ी और उनी बटी सरिरता ी ही चचा बनी रहती थी अमिमत ो इसम ोई रलिच ना थी वह धनी घर ी लडी स शादी र सवय ो बचना नही चाहता था उसा भी सवाणिभमान ह ईशवर ी पा

स उस पास पस ी ोई मी नही थी अभी उसन फकटरी ही लगाई थी उसी समझ बाहर था कि उस घर वालो ा झाव पस ी तरफ कयो

ह उसन मा स सवाल किया कि मा तम सरिरता स मरी शादी कयो रना चाहती हो मा न उस समझाया कि वह दखन म बरी नही ह और किफर खानदान अचछा

ह वह ए शल गरहणी रप म घर सभाल सगी अमिमत न मा ो इस बात ा एहसास राया कि मीन सबध लिलए मना रन पर उस दिदल

पर कया बीती होगी मा और अमिमत ी लडी बार म ाफी बात हईमा ा झाव सरिरता ी तरफ था कयोकि वह घर पर अचछा दह0 लर आ रही

थी अमिमत न अपनी मौसी ी बरी हालत बार म बताया कि किस तरह वह बड घर ी खानदानी बटी लाई थी और आ0 उसी हालत कितनी खराब ह लाई थी बहकलब 0ाती ह और बचचो ो भी नही दखती ह बात चल ही रही

थी कि तभी ए ार बाहर आर री धनी मल0ी घर अदर आए और पीछ स डराइवर फल ी ए टोरी लर आया अदर आए और पीछ स

डराइवर ए टोरी फल ी लर आया अमिमत ो फल ी पटी बरी लग रही थी अमिमत न पछ लिलया यह फल कयो ल आए ह प इन सब ी कया

0ररत थी उनो न 0वाब दिदया कि 4 पटी शमीर स मगाए थ अमिमत ो या सनर करोध आ गया तभी उस किपता 0ी आ गए उन आत ही अमिमत उठर बाहर चला गया वहा वहा मा पास आर बठ गया और बोला

अभी रिरशता तय नही हआ और धनी मल 0ी धनी मल 0ी फल ी पटी लर चलआय मा न समझाया कि 0ब सबध 0ड 0ाता ह तो खाली हाथ नही

आत अमिमत न मा स हा कि तम सबन सरिरता ो इस घर म लान ी ठान रखी ह धनीमल 0ी उस दिदन सरिरता ो दखन ी तारीख तय रन आय थ

Commercial Studies

Banking Nowadays Bank provide easy and quick services through internet facilities methods of Banking is called internet bankingIn order to save the time and money involved in visiting Bank branches people increasingly prefer to have internet banking

There are different modes of doing internet banking or transferring money through online They areReal Time Gross Settlement (RTGS)National Electronic Fund Transfers (NEFT)

1

Question

1) Explain the term RTGS Write the features of RTGS

Answer)The acronym RTGS stands for Real Time Gross Settlement which may be defined as the continuous real time settlement of funds transfer individually on and order by order basis without netting lsquoReal timersquo may be defined as the processing of instructions at the time they are received rather than at some letter time lsquoGross settlementrsquo may be defined as the settlement of transfer instructions which occurs

individually

Features of RTGS1It is the continuous settlement of

funds transfer individually on an order by order basis

2RTGS facility is provided only by CBS core banking solution enabled Bank branches

3Amount charged from the customer for RTGS transactions vary from bank to bank

2) Explain the term NEFT Write the features of NEFT

Answer) National electronic funds transfer may be defined as a nationwide system that facilitates individuals Farms and copper operates to electronically transfer funds from any bank branch to any individual farm or corporate having an account with any other bank branch in the country

Features of NEFT2 Transfer can be made 7 times on

weekdays and 6 times on Saturday

3 NEFT cannot be used to receive foreign remittances

4 NEFT transaction takes place in batches

5 A bank branch must be NEFT enabled to become a part of NEFT fund transfer network

6 There is no maximum or minimum amount that can be transferred through NEFT when one bank has a bank account

English Language

CompositionEssay

A composition is an art of creating a piece of writing on any topic or subject It is the writing correctly beautifully and clearly in order to make some interesting reading Structure of the composition

Introduction ( you lay the foundation for your composition)

Body (it constitutes the main part of the essay)

Conclusion (final statement that leaves a lasting impression)

Kinds of essays1 The Narrative essay2 The descriptive essay3 The reflective essay4 The argumentative essay

Write a composition on any one of the following topics (350- 400 words)

1 Friendship Or2 The first day of your school

Subject Eng Literature (The Merchant of Venice ndash William Shakespeare)Topic Act V Scene 1 Lines 127 to 158 (Nerissa helliphellip The clerk will nersquoer wear hair onrsquos face that had it) [Students should read the original play and also the paraphrase given in the school prescribed textbook]

Summary Revision Questions o Soon thereafter Bassanio Gratiano

and Antonio arrive

o Bassanio tells Portia that he is feeling as if it is morning because of the presence of Portia who is shining like the sun When Antonio is introduced by Bassanio to Portia she tells Bassanio that he should be grateful to Antonio who took so much trouble on his account even to the extent of risking his life

o Nerissa starts quarrelling with Gratiano and demands that he show her the ring she had presented to him and which she had warned him not to lose She suspects that Gratiano must have presented the ring to some young woman and not to the lawyerrsquos clerk as he repeatedly says and assures

Answer the following questions to check your preparation of Act IV Scenes 1 and 2

You must attempt only after you have completed your preparation of Act IV The answers must be in complete sentences using textual evidence (with citation) when necessary

[It would be in your own interest to attempt the above questions honestly totally refraining from consulting your textbook or your notes during answering After completion you should correct the paper yourself consulting the textbooknotes etc and award marks as specified Please let me know the marks you scored through WhatsApp in the group or to my personal WhatsApp]

Act IV Scene 1 (each question carries 2 marks)

1 What did the Duke try to do for Antonio

2 Why does Shylock refuse to show mercy How does he justify his stance

3 Why does Antonio say he is ready to die 4 What information is contained in Bellariorsquos letter

5 Why does Portia (as Balthazar) assert that Shylock must show mercy How does he respond

6 What offers are made to Shylock to get him to spare Antonio How are they received

7 What does Antoniorsquos speech as he faces the prospect of Shylockrsquos knife tell you about his character

8 How do Bassanio and Gratiano react to the looming prospect of Antoniorsquos demise

9 How does Portia (as Balthazar) use the law to turn the tables on Shylock

10 What does the Duke decree should happen to Shylock Why What happens to Shylockrsquos estate

11 What does Portia ask Bassanio as payment for her ldquoservicesrdquo What is his initial response What makes him change his mind

Act IV Scene 2 (each question carries 1frac12 marks)

1 What does Gratiano bring to Portia (Balthazar)

2 What does Nerissa plan on getting from Gratiano What does Portiarsquos comment suggest about men

ECO-10 280620 Topic-Supply AnalysisSHIFTING OF SUPPLY

But if there is change in factors other than the price of the commodity then either more is supplied at the same price or less supplied at the same price In such cases the price of the commodity remains constant but there is a change in other factors like change in the price of inputs change in technology of production change in price of other related goods change in taxation policy of the government etc For example there is an improvement in the technology of production of the commodity in question It leads to decrease in per unit of cost production of the commodity The firm is willing to sell more quantity of the commodity at the same price So the supply other commodity increases at the same price This increase in supply is shown by rightward shift of supply curve On the other hand if the firm uses inferior technology of production the cost of production per unit of the commodity increases The firm is willing to sell less quantity at the same price So the supply of the commodity decreases at the same price This decrease in supply is shown by leftward shift of the supply curve The above cases of increase and decrease in supply can be shown with the help of the following figures

Y INCREASE OF SUPPLY Price (Rs) s

P A s1

B

s

X` O s1 X

q q1

Y` Quantity demanded (in units)

Y DECREASE IN SUPPLY s2

s

price (Rs)

C

p A

s2

s

X` o X

q2 q

Y` Quantity demanded ( in units)

Main factors causing increase in supply or rightward shift of supply Curve(i) Fall in the price of other related goods

(ii) Fall in the price of inputsfactors(iii) Use of better technology in production(iv) Decrease in the rate of excise duty by government(v) If the objective of producer changes from profit maximization to salesMaximization

Main factors causing decrease in supply or leftward shift of supply curve(i) Increase in the price of other related goods(ii) Rise in the price of inputsfactors(iii) Use of inferior technology in production(iv) Increase in the rate of excise duty by the government(v) If the objective

Subject - Biology Topic ndash Chapter mdash6 PhotosynthesisSummary Execution

Today we will know about photosynthesis and its stages

Q1 What do you mean by photosynthesis The process by which living plants containing chlorophyll produce food

substances from carbon-di- oxide and water by using light energy Sunlight

6CO2 +12 H2O----------------------- C6 H12O6 + 6H2O + 6O2

Chlorophyll

Q2 What are the importance of photosynthesis I) Food for all Green plants trap solar energy by photosynthesis

process and supply food and energy for all living organisms either directly or indirectly

Ii) Oxygen to breathe in by product of photosynthesis is oxygen which is essential for all living organisms respiration

Q3 Write about two main phases of photosynthesis A Light dependent phase This phase occur in grana of chloroplast I) The chlorophyll on exposure to light energy becomes activated by

absorbing photons Ii) The absorbed energy is used in splitting the water molecules (H2O)

into its two components (H+ and OH- ) and releasing electron s 2H2O------------------------- 4H+ + 4e- +O2

Energy of 4 photons This reaction is known as photolysis

End products are H+ and oxygen water

B Light independent (Dark ) phase The reactions in this phase require no light energy

Here CO2 combine with H+ and produce glucose

Class XI

Subject Topic Summary ExecutionEVS Chapter-4 Legal

regimes for sustainable development

Environmental legislationEnvironmental legislation is the collection of laws and regulations pertaining to air quality water quality the wilderness endangered wildlife and other environmental factors The act ensures that matters important to the environment are thoroughly

Learn -The Forest (Conservation) Act 1980

considered in any decisions made by federal agencies

The Forest (Conservation) Act 1980 The Forest (Conservation) Act 1980 an Act of the Parliament of India to provide for the conservation of forests and for matters connected therewith or ancillary or incidental thereto It was further amended in 1988 This law extends to the whole of IndiaObjects and Reasons of the Forest Conservation Act

Deforestation causes ecological imbalance and leads to environmental deterioration Deforestation had been taking place on a large scale in the country and it had caused widespread concern The act seeks to check upon deforestation and de-reservation of forests

Subject Eng Literature (The Tempest ndash William Shakespeare) Topic Act II Scene 1 Lines 314 to 329 (End of scene)

[Students should read the original play and also the paraphrase given in the school prescribed textbook]Summary Questions amp Answers

Conspiracy of Antonio and Sebastian (Contd)

o As they approach Ariel appears again and wakes up Gonzalo by singing a tune in his ear Alonso also wakes up and they see both Sebastian and Antonio with drawn swords On being caught off guard they make up a story saying that they had heard a bellowing of bulls or lions

o They then moved to another part of the island

o Ariel at once rushes to Prospero to inform him of this development

SUMMING-UP of ACT-2 SCENE-1

(i) Among the survivors Ferdinand is separated from the rest which results in the disconsolate grief of Alonso as he took him for dead

(ii) The villainy of Antonio is confirmed

(iii) The supremacy of Prosperorsquos magic which resulted in the failure of the human conspiracy

(1)

(Act II Sc 1 L 311-325)SEBASTIAN Whiles we stood here securing your repose

Even now we heard a hollow burst of bellowing Like bulls or rather lions Didt not wake youIt struck mine ear most terribly

ALONSO I heard nothingANTONIO O rsquotwas a din to fright a monsters ear

To make an earthquake Sure it was the roarOf a whole herd of lions

ALONSO Heard you this GonzaloGONZALO Upon mine honour sir I heard a humming

And that a strange one too which did awake meI shaked you sir and cried As mine eyes opened I saw their weapons drawn There was a noiseThats verily rsquoTis best we stand upon our guardOr that we quit this place Lets draw our weapons

(i) Why has Prospero sent Ariel to Gonzalo and Alonso What does Ariel do to awaken Gonzalo

Prospero has already come to know by his magic powers the danger which threatens Gonzalo who had been Prosperorsquos friend and so he sent Ariel to preserve the lives of both Gonzalo and Alonso Prospero does not want that his scheme should remain unfulfilled Ariel begins to sing a song in Gonzalorsquos ears to awaken him(ii) Who are ready to carry out their plan Who takes steps to stop them Why does Gonzalo feel surprised after being awakened

Sebastian and Antonio are ready to carry out their plans They are standing with their swords drawn to kill Alonso and

(iv) We see two sets of contrasting characters Gonzalo-Adrian against Antonio-Sebastian

(v) The grief that works in Alonso can be perceived to his repentance for his association in Antoniorsquos crime against Prospero

Gonzalo Ariel takes steps to stop them from carrying out their nefarious scheme When Gonzalo is awakened by the song sung by Ariel into his ears he (Gonzalo) feels surprised because he sees Sebastian and Antonio standing with their swords drawn(iii) What reason do Sebastian and Antonio tell of drawing their swords when they are suspected by Alonso and Gonzalo

When Sebastian and Antonio are seen with their swords drawn they are looked with suspicion by Gonzalo and Alonso At first Sebastian tells them that as they stood here to guard them during their sleep they heard only a little before a sudden loud noise very much like the roaring of bulls or more probably that of lions Then Antonio follows him saying that this was a noise so terrible as to frighten even a monsterrsquos ears and this noise could even have shaken the earth and it was surely like the roaring of a multitude of lions Then seeing the danger they have drawn their swords Perhaps after hearing the terrible noise they (Gonzalo and Alonso) woke up from their sound sleep

(iv) What does Gonzalo tell Alonso about the strange noise What did he see on opening his eyes Gonzalo tells Alonso that he did not hear the sound of roaring but he heard a humming sound which was strange and which woke him up After waking up he gave him (Alonso) a shaking and a loud cry On opening his eyes he saw these two gentlemen standing with their swords drawn(v) What does Gonzalo suggest

Gonzalo suggests that there was a noise indeed and of that he has no doubt at all and suggests that the best course for them would be to remain alert and vigilant against any possible danger to their lives or to leave this place and move to some other part of the island

Class XIISubject Topic Summary Execution

Commerce

Chapter- Management

Today we will discuss about LEVELS OF MANAGEMENT

Levels of management is a series or chain of managerial positions from top to bottom It helps individuals to know their authority responsibilities and superior-subordinate relations among themselves There are mainly three levels of Management TOP LEVEL MANAGEMENTMIDDLE LEVEL MANAGEMENTLOWER LEVEL MANAGEMENT

Top level managementIt consists of members at the highest level in the management hierarchy This level includes Board Of Directors Chief Executive Managing Directors Chairman President Vice President

Rolefunctions of the top levelmanagement1To analyse evaluate and deal

with theexternal environment2 To determine the objectives and

policies of the business3 To strive for welfare and survival

of business

4 To create an organisational Framework consisting of authority responsibility relationship

Middle level management Congress of members or groups who are concerned with implementation of the policies let down by the top managementThis level includes head of the department such as finance manager marketing manager branch and regional managers departmental and divisional heads plant superintendent etc

Role of functions of the middle level management

1 To interpret the policies framed by top management

2 To assign duties and responsibilities to lower level managers

3 To select and appoint employees for middle and supervisory level and evaluate their performance

4 To co-operate with other departments for smooth functioning

Operational or supervisory level managementIt refers to the group are members who are concerned with execution of the work They are also known as fast line managers This level includes supervisor 4 men Section Officer clerk Inspector etc

Role of functions of the lower level management1 To plan and execute day-to-

day operations2 To supervise and control the workers3 To arrange materials and

tools to start the process and make arrangements for training

4 Today present workers grievance and suggestions before the management and

ensure safe and proper working conditions in the factory

Business Studies

Staff Appraisal Chapter- 10 Today let us start with a new chapter

Staff Appraisal

Meaning of Performance Appraisal

Performance Appraisal is the systematic evaluation of the performance of employees and to understand the abilities of a person for further growth and developmentThe supervisors measure the pay of employees and compare it with targets and plansThe supervisor analyses the factors behind work performances of employeesThe employers are in position to guide the employees for a better performance

Objectives of Performance Appraisal

Following are the objectives of Performance Appraisal

To maintain records in order to determine compensation packages wage structure salaries raises etc

To identify the strengths and weaknesses of employees to place right men on right job

To maintain and assess the potential present in a person for further growth and development

To provide a feedback to employees regarding their performance and related status

To provide a feedback to employees regarding their performance and related status

Importance of Performance Appraisal

Performance appraisal provides important and useful information for the assessment of employees skill

knowledge ability and overall job performance The following are the points which indicate the importance of performance appraisal in an organization

1 Performance appraisal helps supervisors to assess the work performance of their subordinates

2 Performance appraisal helps to assess the training and development needs of employees

3 Performance appraisal provides grounds for employees to correct their mistakes and it also provides proper guidance and criticism for employees development4 Performance appraisal provides reward for better performance

5 Performance appraisal helps to improve the communication system of the organization

6 Performance appraisal evaluates whether human resource programs being implemented in the organization have been effective

7 Performance appraisal helps to prepare pay structure for each employee working in the organization

8 Performance appraisal helps to review the potentiality of employees so that their future capability is anticipated

Geography

DRIANAGE The SubarnarekhaThe Subarnarekha and the Brahmaniinterposed between the Ganga and the Mahanadi deltas drain an area of 19300 sq kmand 39033 sq km respectively The drainage basins of these streams are shared byJharkhand Odisha west Bengal and Chhattisgarh The Brahmani is known as southKoel in its upper reaches in Jharkhand

The NarmadaThe Narmada rises in the Amarkantak hills of MadhyaPradesh It flows towards the West in a rift valleyformed due to a geological fault The total length of it is 1300 km All the tributaries of the

Q1 Name the two westward flowing rivers in the peninsular plateauA1 Narmada and Tapi are the only westward flowing rivers of the peninsular plateau

Q2 Differentiate between east-flowing rivers and west-flowing riversA2

East-flowing rivers

West-flowing rivers

Narmada are very short inlength Most of its tributaries join the main streamright anglesThe Narmada basin covers parts of Madhya Pradesh and Gujarat

The Tapi The Tapi rises in the Satpura ranges in the Betul listrictof Madhya Pradesh It flows in a rift valley parallel tothe Narmada but it is much shorter in length It coversparts of Madhya Pradesh Gujarat and MaharashtraThe length is about 724 km

The Sabarmati and the MahiThe Sabarmati rises in the Aravali hills and flows south-south-westwards for a distance of 300 kilometres to the Arabian Sea The Sabarmatibasin extends over an area of 21674 sq km in Rajasthan and Gujarat The Mahi rises inthe east of Udaipur and drains an area of 34842 sq km lying in Madhya PradeshRajasthan and Gujarat It flows south-westwards for a distance of 533 km before it fallsinto the Gulf of Khambhat

The ChambalThe Chambal rises near Mhow in the Vindhya Range and flows towards the northgenerally in a gorge upto Kota Below Kota it turns to the north-east direction and afterreaching Pinahat it turns to the east and runs nearly parallel to the Yamuna beforejoining it in the southern part of the Etawah district in Uttar PradeshMajor Rivers of India with their basin area (Sqkm)

Himalayan System Indus 321290Ganga 861404

Brahmaputra 187110Indus System

Jhelum 34775Beas 20303

Ganga System Yamuna 366223Ghaghra 127950

Peninsular RiversNarmada 98796

Tapi 65145Mahanadi 141600

Subarnarekha 19300Sabarmati 21674

Mahi 34842Godavari 312812

Godavari Krishna Kaveri Mahanadi are the east-flowing rivers

Narmada Tapi west-flowing rivers

They fall into the Bay of Bengal

They fall into Arabian Sea

These rivers form big deltas

These rivers form comparativelysmall deltas

Catchment areas of these rivers are larger

Catchment areas of these rivers are smaller

Krishna 2589488Cauveri 87900

Subject ndashBiology Topic ndashChapter -5 Inheritance amp Variations Summary ExecutionToday we will discussabout linkage and its classification

LINKAGE The tendency of the genes located on the same chromosome to stay together is

hereditary transmission Linked genes the genes responsible for this Genes that exhibit the process of linkage locates in the same chromosome The distance between the linked genes in a chromosome determines the strength

of linkage i e genes that are located close to each other show stronger linkage than that are located far from each other

COMPLETE LINKAGE It is the type of linkage showed by the genes that are closely located or are tightly

linked with each other as they have no chance of separatingby crossing over These genes are always transmitted together to the same gamete and the same

offspring In such condition only parental or non cross over type of gametes are formedINCOMPLETE KINKAGE It is type of linkage showed by the genes that are distantly located orare loosely

linked with each other because they have chance of separating by crossing over

SIGNIFICANCE i) It helps in holding the parental character togetherii) It checks the appearance of new recombination and helps in bringing the

hybrid population which resembles the original parents iii) Linked genes dilute the effects of undesirable traits

Subject Eng Literature (The Tempest ndash William Shakespeare) Topic Essay Questions (EQ-3)Question No 3

Give a character sketch of CalibanAnswer

The character of Caliban has been wonderfully conceived by Shakespeare as the manifestation of all that is gross and earthy ndash a sort of creature of the earth as Ariel is a sort of creature of the air

Calibanrsquos Physical Appearanceo Caliban is lsquofreckledrsquo a lsquomisshapen knaversquo not honoured with human shape

o Prospero calls him lsquothou tortoisersquo (Act I Sc 2 Line 317) Trinculo stumbling upon him describes him as ldquoA strange fish hellip Legged like a man And his fins like armsrdquo He ldquosmells like a fishrdquo (Act II Sc 2 Line 25)

o Prospero also calls him a ldquobeastrdquo (Act IV Sc 1 Line 140) and ldquoThis misshapen knaverdquo (Act V Sc 1 Line 268)

o Further it appears that in addition to his physical deformity his spiritual inferiority is also suggested by Prosperorsquos claim that his birth resulted from the union between his mother the witch Sycorax and the devil

Calibanrsquos ParentageWhen the play opens Caliban is twenty four years of age having been born on the island twelve years before the coming of Prospero His mother was the foul witch Sycorax who was banished from Algiers for ldquomischiefs manifold and sorceries terrible to enter human hearingrdquo (Act I Sc 2 Line 264) and the father was the Devil himself Thus

Caliban is a monster of evil and brute nature ugly deformed and stinking

Calibanrsquos Savage and Malignant Natureo Caliban is entirely a creature of the earth ndash gross brutal and savage He regards himself as the rightful possessor

of the island and Prospero as a usurper

o In his young age he was on good terms with Prospero He had consented to be received by Prospero at his house and to be educated by him He has learnt human language only to curse his master whom he abhors

o His beastly nature soon breaks out and ends in a vicious attack on Miranda This opens the eye of Prospero who becomes severe to him and enforces his service by threats and violence

o Prospero uses him to make dams for fish to fetch firewood scraper trenches wash dishes and keep his cell clean

Calibanrsquos Hatred for ProsperoA profound hatred for Prospero has taken hold of Caliban It springs from a sense of his being dispossessed and ill-treated He would kill Prospero if he could but he knows the power of Prosperorsquos lsquobookrsquo Hence he transfers his allegiance to Stephano who seems like a god to him He also incites the two drunken associates to batter the skull of Prospero when he sleeps in the afternoon

Caliban Shows Considerable Intelligenceo He has learnt Prosperorsquos language

ldquoYou taught me language and my profit onrsquot (Act II Sc 2 Lines 86-89)Is I know how to curserdquo

o He is well aware of the futility of arguing with one who has more power than he has

ldquoI must obey his art is such power (Act I Sc 2 Lines 373-376)It would control my damrsquos god SetebosAnd make a vassal of himrdquo

o He realizes the importance of Prosperorsquos books

ldquoRemember (Act III Sc 2 Lines 89-92)First to possess his books for without themHersquos but a sot as I am nor hath notOne spirit to commandrdquo

o He knows the value of stealth when attacking the enemy

ldquoPray you tread softly that the blind mole may not (Act IV Sc 1 Lines 194-195)Hear a foot fall we now are near his cellrdquo

o Caliban has a better set of values than Stephano and Trinculo They are distracted from their plan by their greed for Prosperorsquos rich garments Only Caliban realizes that such a finery is unimportant

ldquoLeave it alone thou fool it is but trashrdquo (Act IV Sc 1 Lines 224)

Caliban is not a good judge of characterCaliban is not a good judge of character He decides for example that Stephano is a god because he dispenses lsquocelestial liquorrsquo (Act II Sc 2 Line 115) but then it must be remembered that he has only known his mother Sycorax Prospero Miranda and the spirits that torture him However he quickly discovers his error of judgementrdquo

ldquoWhat a thrice-double ass (Act V Sc 1 Lines 295-297)Was I to take this drunkard for a godAnd worship this dull foolrdquo

Calibanrsquos Imaginative NatureIf Caliban is sub-human in what has been said above he is human in the respect of the poetic side of his character He listens to music with rapture He tells of the beautiful dreams in which heaven rains treasures upon him and which upon waking he yearns to renew One of the most poetic passages in whole play is Calibanrsquos description of the island

to Stephano and Trinculo

ldquoBe not afeard The isle is full of noises (Act III Sc 2 Lines 135-143)Sounds and sweet airs that give delight and hurt notSometimes a thousand twangling instrumentsWill hum about mine ears and sometime voicesThat if I then had waked after long sleepWill make me sleep again and then in dreamingThe clouds methought would open and show richesReady to drop upon me that when I wakedI cried to dream againrdquo

Caliban - Less Ignoble Than Some OthersCalibanrsquos motive for murder is less dishonourable than that of Antonio and Sebastian They plan to kill Alonso to gain his power and wealth Caliban merely wants revenge and the return of lsquohisrsquo island

Conclusiono Calibanrsquos character is not portrayed very clearly in the play and hence we cannot decide whether he is a poor

savage being grossly maltreated by Prospero or whether he is evil and must therefore be kept in bondage or enslavement

o Caliban is contrasted with Ariel who is a spirit and thus swift and uninterested in physical activitieso Caliban is also contrasted with Prospero who is the all-powerful master of the island and of the destiny of all

those on the islando Caliban is also contrasted with civilized man showing him to be less evil than Antonio and Stephano and less

materialistic than Stephano and Trinculoo Caliban has suffered at the hands of Prospero and he has learnt to curse by listening to Prosperorsquos abuse He

certainly believes that Prospero has deprived him of his birthrighto Finally the character Caliban is thought to be one of Shakespearersquos masterpieces The complexity of the character

is reflected in the large volume of critical discussion that has grown around it

ECO ndash12 Topic-Forms of market

MonopolyMonopoly is a market structure in which there is a single seller there are no close substitutes for the commodity produced by the firm and there are barriers to entry Example Indian Railways which is operated under government of India Monopoly also implies absence of competitionFeatures of Monopoly Monopoly is characterized by1 Single Seller In monopoly there is only one firm producing the product The whole industry consists of this single firm Thus under monopoly there is no distinction between firm and industry Being the only firm there is significant control of the firm over supply and price Thus under monopoly buyers do not have the option of buying the commodity from any other seller They have to buy the product from the firm or they can go without the commodity This fact gives immense control to the monopolist over the market

2No Close Substitute There are no close substitutes of the product produced by the monopolist firm If there are close substitutes of the product in the market it implies presence of more than one firm and hence no monopoly In order to ensure a total of control over the market by the monopolist firm it is assumed that there are no close substitutes of the product

3 No Entry amp Exit Monopoly can only exist when there is strong barriers before a new firm to enter the market In fact once a monopoly firm starts producing the product no other firm can produce the same One reason for this is the ability of the

monopolist to produce the product at a lower cost than any new firm who thinks to enter the market If a new firm who knows that it cannot produce at a lower cost than the monopolist then that firm will never enter the market for fear of losing out in competition Similarly the monopolist who is operating for a long time may be enjoying reputation among its customers and is in a better position to use the situation in its own benefit A new firm has to take long time to achieve this and so may not be interested to enter the market

4 Price Maker Being the single seller of the product the monopolist has full control over the pricing of the product On the other hand if there is a large number of buyers in the market so no single buyer exercises any significant influence over price determination Thus it is a sellerrsquos market So monopoly firm is a price maker

5 Price Discrimination Having considerable control over the market on account of being single seller with no entry of other firms the monopolist can exercise policy of price discrimination it means that the monopolist can sell different quantities of the same product to a consumer at different price or same quantity to different consumers at different prices by adjudging the standard of living of the consumer

6 Shape of Demand Curve Since a monopolist has full control over the price therefore he can sell more by lowering the price This makes the demand curve downward sloping

Subject Ac-12 290620 Topic- retirement Model sumThe Balance Sheet of Rohit Nisha and Sunil who are partners in a firm sharing profits according to their capitals as on 31st March 2014 was as under

Liabilities Amount Assets Amount (Rs) (` Rs)

Creditors 25000 Machinery 40000Bills Payable 13000 Building 90000General Reserve 22000 Debtors 30000Capital Less Provision for Rohit 60000 Bad debts 1000

29000 Nisha 40000 Stocks 23000 Sunil 40000 140000 Cash at Bank 18000

200000 200000

On the date of Balance Sheet Nisha retired from the firm and following adjustments were made(i) Building is appreciated by 20(ii) Provision for bad debts is increased to 5 on Debtors(iii) Machinery is depreciated by 10(iv) Goodwill of the firm is valued at Rs 56000 and the retiring partnerrsquos share is adjusted

(v) The capital of the new firm is fixed at Rs120000 Prepare Revaluation Account Capital Accounts of the partner and Balance Sheet of the new firm after Nisharsquos retirement Revaluation AccountDr Cr

Particulars Amount Particulars Amount (`Rs) (Rs`)

Provision for Bad debt Ac 500 Building Ac 18000Machinery Ac 4000Profit transferred toCapital Accounts (3 2 2)Rohit 5786Nisha 3857Sunil 3857

13500

18000 18000

Capital Account

Dr Cr

Particulars Rohit Nisha Sunil Particulars Rohit Nisha Sunil (Rs`) (Rs`) (`Rs) (Rs`) (Rs`) (Rs`)

Sunilrsquos Capital ac 9600 mdash 6400 Balance bd 60000 40000 40000Bank - 66143 - General Reserve 9428 6286 6286Balance cd 72000 mdash 48000 Revaluation (Profi 5786 3857 3857 Rohitrsquos Capital Ac mdash 9600 mdash

Sunilrsquos Capital Ac 6400 Bank 6386 - 4257

81600 66143 54400 81600 66143 54400

Balance Sheet as at 31st March 2014

Liabilities Amount Assets Amount (Rs`) (Rs`)

Creditors 25000 Building 108000Bank overdraft 37500 Machinery 36000

Bills Payable 13000 Debtors 30000Capital Less ProvisionRohit 72000 for Bad debts 1500 28500Sunil 48000 120000 Stock 23000

195500 195500

Working Notes (i) (a) Profit sharing ratio is 60000 40000 40000 ie = 3 2 2(b) Gaining Ratio Rohit = 35 ndash 37 = 2135 ndash 1535 = 635Sunil = 25-27 = 1435 ndash 1035 = 435= 635 435= 6 4 = 3 2(c) Nisha Share of Goodwill = Rs 56000 times 27 = Rs16000Share of Goodwill in the gaining ratio by the existing partner ieRohit = Rs16000 times 35 = Rs 9600Sunil = Rs 16000 times 25 = Rs 6400

The journal entry isRohitrsquos Capital Ac Dr 9600Sunilrsquos Capital Ac Dr 6400 To Nisharsquos Capital Ac 16000(Share of Goodwill divided into gaining ratio)

  • 1 Static Friction
  • The frictional force that acts between the surfaces when they are at rest with respect to each other is called Static Friction
    • Static Friction Examples
      • 2 Sliding Friction
        • Examples Of Sliding Friction
          • 3 Rolling Friction
            • Examples Of Rolling Friction
              • Objects and Reasons of the Forest Conservation Act
Page 35:  · Web viewSubject . Topic . Summary . Execution . English 1 . Sounds of animals . Hens –cackle Horses –neigh Lions –roar Owls –hoots Snake –hiss. English 2 . Mother’s

passage

In these lines the Prince of Morocco says that even the vast oceans which throw a challenge at the sky are unable to prevent men from coming to Portiarsquos place to have a glimpse of her These lines are also a tribute to Portiarsquos beauty and good qualities Many men voyage across the ocean treating it as a mere stream to see the beautiful Portia

paper in which is written that whoever happens to be guided by the glitter of things is invariably deceived

On reading the scroll the Prince says that he is too sad at heart to speak a more formal farewell and leaves with his followers amidst a sound of trumpets

After the Prince of Morocco leaves Portia remarks that the Prince is a gentle fellow but she is rid of him May all persons of his nature make a similar choice

IMPORTANT PASSAGES EXPLAINED

(Act II Sc 7 L 39-43)From the four corners of the earth they come

To kiss this shrine this mortal breathing saintThe Hyrcanian deserts and the vasty wildsOf wide Arabia are as through-fares nowFor princes to come view fair Portia

Context

This passage occurs in Act II Scene 7 in The Merchant of Venice This is part of the speech made by the Prince of Morocco

(2)

(Act II Sc 7 L 48-53)

MOROCCO One of these three contains her heavenly pictureIst like that lead contains her

Twere damnation To think so base a thought it were too grossTo rib her cerecloth in the obscure graveOr shall I think in silver shes immurdBeing ten times undervalued to tried gold

(i) What meaning does the Prince of Morocco find out of the inscription of the golden casket What have Belmont and Portiarsquos house been called and why

The inscription on the golden casket is ldquoWho chooseth me shall gain what many men desirerdquo The Prince finds out that it means that the chooser of the golden casket will get Portia because many men desire her In fact the entire world desires her Because of the coming of many suitors to Belmont from different countries in order to win Portiarsquos hand Belmont has become a centre of pilgrimage and her house is the shrine where saintly Portia is installed

(ii) What does the Prince of Morocco do before making the final choice of the casket Which is the correct casket and who will win Portiarsquos hand

The Prince of Morocco surveys and analyses the inscriptions on the casket of lead silver and gold Before making the final choice like a very systematic and methodical person he once again considers the claims of the caskets The casket containing Portiarsquos picture is the correct casket and the person choosing it will win Portiarsquos hand

Explanation

While praising Portia the Prince of Morocco conceives Portia as a goddess whose image is placed inside one of the caskets Many suitors are coming from far and wide the north and the south the east and the west (Four corners) in order to try their luck Some of them have come from the distant land of Persia and Arabia The deserts of Persia (Hyrcanian deserts) and the boundless desolate lands of Arabia have been crossed by the Princes seeking the hand of Portia All this shows that Portia is indeed the most beautiful lady of the world

(iii) What does the Prince of Morocco say in his estimation while examining the motto on the silver casket What does he find in the golden casket

While examining the motto on the silver casket which says ldquoWho chooseth me shall get as much as he deservesrdquo Morocco says that in his own estimation he surely deserves Portia in all respects ndash rank birth wealth etc

He chooses the golden casket When he opens it he finds an empty human skull holding a scroll in which it is written that those who are attracted by the glittering outside of things are always deceived as Morocco has been deceived

(iv) What kind of nature does the Prince of Morocco have

The Prince of Morocco has a simple nature who does not look deeply into the inner meaning of things but is dazzled by the outward appearance of gold He is inclined to over-estimate his own value and does not realize that it is a duty to ldquogive and hazardrdquo To say that he will not hazard for lead shows that he misreads the true meaning of the inscription which is that he should be prepared to ldquohazard all he hathrdquo for Portia So his feeling is only one of fascination and romantic attraction

(v) Do you think that the lottery of the caskets is not a matter that will be determined by chance

In fact the lottery of the casket is not a matter that will be determined by mere chance but that it is a true test of character and of sincerity which is amply proved not only by Moroccorsquos choice but also by the arguments which he uses to help him in his choice

(Act II Sc 7 L 55-59)

They have in England

A coin that bears the figure of an angelStamped in gold but thats insculpd uponBut here an angel in a golden bedLies all within

Context

(3)

(Act II Sc 7 L 63-77)A carrion Death within whose empty eye

There is a written scroll Ill read the writing

All that glisters is not goldOften have you heard that toldMany a man his life hath soldBut my outside to beholdGilded tombs do worms infoldHad you been as wise as boldYoung in limbs in judgment oldYour answer had not been inscrolld

This passage occurs in Act II Scene 7 in The Merchant of Venice This is part of the speech made by the Prince of Morocco

Explanation

In this passage the Prince of Morocco bestows high praise on Portia whose hand he is seeking He contrasts this casket containing Portiarsquos portrait with the old English gold coin bearing the image of the archangel (angel of the highest rank) He goes on to remark that while the figure of the archangel is engraved (Insculped) upon the English coin the picture of Portia who is beautiful as an angel lies hidden inside one of the caskets namely the Golden Casket (Golden Bed) In the day of Elizabeth silver was ten times inferior in value to gold Therefore the Prince of Morocco believing that Portiarsquos portrait is contained in the Golden Casket decides to choose the Golden Casket

Fare you well your suit is coldCold indeed and labour lostThen farewell heat and welcome frostmdashPortia adieu I have too grievd a heartTo take a tedious leave Thus losers part

(i) What reward does the Prince of Morocco get after making a wrong choice of the Casket How does he feel

After making the wrong choice in selecting the casket of gold the Prince of Morocco as a reward earns a rebuke in the form of a scroll tucked in the empty eye-socket of a skull kept in the casket of gold The Prince is shocked and disappointed He becomes all the more sad and dejected when he reads the scroll which points to his foolishness in being misled by the appearance and outward show as indicative of its worth

(ii) How does the Prince respond after reading the scroll

After reading the scroll the Prince though upset accepts the result with good grace and decorum befitting a royal suitor and true sportsman He says that his love-suit is really cold otherwise he would have chosen correctly but now his efforts have been in vain So he bids farewell to Portia to the warmth and enthusiasm of love and welcomes the cold and bitterness of dejection and misery of life which lies ahead

(iii) What request does he make to Portia and why

After being failure in his mission he requests Portia to give him permission to leave at once because he is too sad to undergo the tediousness of a formal leave-taking He tells that it is the manner in which defeated persons part unceremoniously

(iv) Explain the following lines

ldquoAll that glisters is not goldOften have you heard that toldMany a man his life hath soldBut my outside to beholdGilded tombs do worms infoldrdquo

Mere glitter does not make a metal to be gold Man has often been warned against appearance but it has been of no use Many people have sacrificed their lives only to seek the outer appearance of gold Worms are found inside the gilded

monuments

Class XSubject Topic Summary Execution

Hindi 2ndlang

नया रासता भाग 6 मायाराम 0ी घर म धनी मल 0ी और उनी बटी सरिरता ी ही चचा बनी रहती थी अमिमत ो इसम ोई रलिच ना थी वह धनी घर ी लडी स शादी र सवय ो बचना नही चाहता था उसा भी सवाणिभमान ह ईशवर ी पा

स उस पास पस ी ोई मी नही थी अभी उसन फकटरी ही लगाई थी उसी समझ बाहर था कि उस घर वालो ा झाव पस ी तरफ कयो

ह उसन मा स सवाल किया कि मा तम सरिरता स मरी शादी कयो रना चाहती हो मा न उस समझाया कि वह दखन म बरी नही ह और किफर खानदान अचछा

ह वह ए शल गरहणी रप म घर सभाल सगी अमिमत न मा ो इस बात ा एहसास राया कि मीन सबध लिलए मना रन पर उस दिदल

पर कया बीती होगी मा और अमिमत ी लडी बार म ाफी बात हईमा ा झाव सरिरता ी तरफ था कयोकि वह घर पर अचछा दह0 लर आ रही

थी अमिमत न अपनी मौसी ी बरी हालत बार म बताया कि किस तरह वह बड घर ी खानदानी बटी लाई थी और आ0 उसी हालत कितनी खराब ह लाई थी बहकलब 0ाती ह और बचचो ो भी नही दखती ह बात चल ही रही

थी कि तभी ए ार बाहर आर री धनी मल0ी घर अदर आए और पीछ स डराइवर फल ी ए टोरी लर आया अदर आए और पीछ स

डराइवर ए टोरी फल ी लर आया अमिमत ो फल ी पटी बरी लग रही थी अमिमत न पछ लिलया यह फल कयो ल आए ह प इन सब ी कया

0ररत थी उनो न 0वाब दिदया कि 4 पटी शमीर स मगाए थ अमिमत ो या सनर करोध आ गया तभी उस किपता 0ी आ गए उन आत ही अमिमत उठर बाहर चला गया वहा वहा मा पास आर बठ गया और बोला

अभी रिरशता तय नही हआ और धनी मल 0ी धनी मल 0ी फल ी पटी लर चलआय मा न समझाया कि 0ब सबध 0ड 0ाता ह तो खाली हाथ नही

आत अमिमत न मा स हा कि तम सबन सरिरता ो इस घर म लान ी ठान रखी ह धनीमल 0ी उस दिदन सरिरता ो दखन ी तारीख तय रन आय थ

Commercial Studies

Banking Nowadays Bank provide easy and quick services through internet facilities methods of Banking is called internet bankingIn order to save the time and money involved in visiting Bank branches people increasingly prefer to have internet banking

There are different modes of doing internet banking or transferring money through online They areReal Time Gross Settlement (RTGS)National Electronic Fund Transfers (NEFT)

1

Question

1) Explain the term RTGS Write the features of RTGS

Answer)The acronym RTGS stands for Real Time Gross Settlement which may be defined as the continuous real time settlement of funds transfer individually on and order by order basis without netting lsquoReal timersquo may be defined as the processing of instructions at the time they are received rather than at some letter time lsquoGross settlementrsquo may be defined as the settlement of transfer instructions which occurs

individually

Features of RTGS1It is the continuous settlement of

funds transfer individually on an order by order basis

2RTGS facility is provided only by CBS core banking solution enabled Bank branches

3Amount charged from the customer for RTGS transactions vary from bank to bank

2) Explain the term NEFT Write the features of NEFT

Answer) National electronic funds transfer may be defined as a nationwide system that facilitates individuals Farms and copper operates to electronically transfer funds from any bank branch to any individual farm or corporate having an account with any other bank branch in the country

Features of NEFT2 Transfer can be made 7 times on

weekdays and 6 times on Saturday

3 NEFT cannot be used to receive foreign remittances

4 NEFT transaction takes place in batches

5 A bank branch must be NEFT enabled to become a part of NEFT fund transfer network

6 There is no maximum or minimum amount that can be transferred through NEFT when one bank has a bank account

English Language

CompositionEssay

A composition is an art of creating a piece of writing on any topic or subject It is the writing correctly beautifully and clearly in order to make some interesting reading Structure of the composition

Introduction ( you lay the foundation for your composition)

Body (it constitutes the main part of the essay)

Conclusion (final statement that leaves a lasting impression)

Kinds of essays1 The Narrative essay2 The descriptive essay3 The reflective essay4 The argumentative essay

Write a composition on any one of the following topics (350- 400 words)

1 Friendship Or2 The first day of your school

Subject Eng Literature (The Merchant of Venice ndash William Shakespeare)Topic Act V Scene 1 Lines 127 to 158 (Nerissa helliphellip The clerk will nersquoer wear hair onrsquos face that had it) [Students should read the original play and also the paraphrase given in the school prescribed textbook]

Summary Revision Questions o Soon thereafter Bassanio Gratiano

and Antonio arrive

o Bassanio tells Portia that he is feeling as if it is morning because of the presence of Portia who is shining like the sun When Antonio is introduced by Bassanio to Portia she tells Bassanio that he should be grateful to Antonio who took so much trouble on his account even to the extent of risking his life

o Nerissa starts quarrelling with Gratiano and demands that he show her the ring she had presented to him and which she had warned him not to lose She suspects that Gratiano must have presented the ring to some young woman and not to the lawyerrsquos clerk as he repeatedly says and assures

Answer the following questions to check your preparation of Act IV Scenes 1 and 2

You must attempt only after you have completed your preparation of Act IV The answers must be in complete sentences using textual evidence (with citation) when necessary

[It would be in your own interest to attempt the above questions honestly totally refraining from consulting your textbook or your notes during answering After completion you should correct the paper yourself consulting the textbooknotes etc and award marks as specified Please let me know the marks you scored through WhatsApp in the group or to my personal WhatsApp]

Act IV Scene 1 (each question carries 2 marks)

1 What did the Duke try to do for Antonio

2 Why does Shylock refuse to show mercy How does he justify his stance

3 Why does Antonio say he is ready to die 4 What information is contained in Bellariorsquos letter

5 Why does Portia (as Balthazar) assert that Shylock must show mercy How does he respond

6 What offers are made to Shylock to get him to spare Antonio How are they received

7 What does Antoniorsquos speech as he faces the prospect of Shylockrsquos knife tell you about his character

8 How do Bassanio and Gratiano react to the looming prospect of Antoniorsquos demise

9 How does Portia (as Balthazar) use the law to turn the tables on Shylock

10 What does the Duke decree should happen to Shylock Why What happens to Shylockrsquos estate

11 What does Portia ask Bassanio as payment for her ldquoservicesrdquo What is his initial response What makes him change his mind

Act IV Scene 2 (each question carries 1frac12 marks)

1 What does Gratiano bring to Portia (Balthazar)

2 What does Nerissa plan on getting from Gratiano What does Portiarsquos comment suggest about men

ECO-10 280620 Topic-Supply AnalysisSHIFTING OF SUPPLY

But if there is change in factors other than the price of the commodity then either more is supplied at the same price or less supplied at the same price In such cases the price of the commodity remains constant but there is a change in other factors like change in the price of inputs change in technology of production change in price of other related goods change in taxation policy of the government etc For example there is an improvement in the technology of production of the commodity in question It leads to decrease in per unit of cost production of the commodity The firm is willing to sell more quantity of the commodity at the same price So the supply other commodity increases at the same price This increase in supply is shown by rightward shift of supply curve On the other hand if the firm uses inferior technology of production the cost of production per unit of the commodity increases The firm is willing to sell less quantity at the same price So the supply of the commodity decreases at the same price This decrease in supply is shown by leftward shift of the supply curve The above cases of increase and decrease in supply can be shown with the help of the following figures

Y INCREASE OF SUPPLY Price (Rs) s

P A s1

B

s

X` O s1 X

q q1

Y` Quantity demanded (in units)

Y DECREASE IN SUPPLY s2

s

price (Rs)

C

p A

s2

s

X` o X

q2 q

Y` Quantity demanded ( in units)

Main factors causing increase in supply or rightward shift of supply Curve(i) Fall in the price of other related goods

(ii) Fall in the price of inputsfactors(iii) Use of better technology in production(iv) Decrease in the rate of excise duty by government(v) If the objective of producer changes from profit maximization to salesMaximization

Main factors causing decrease in supply or leftward shift of supply curve(i) Increase in the price of other related goods(ii) Rise in the price of inputsfactors(iii) Use of inferior technology in production(iv) Increase in the rate of excise duty by the government(v) If the objective

Subject - Biology Topic ndash Chapter mdash6 PhotosynthesisSummary Execution

Today we will know about photosynthesis and its stages

Q1 What do you mean by photosynthesis The process by which living plants containing chlorophyll produce food

substances from carbon-di- oxide and water by using light energy Sunlight

6CO2 +12 H2O----------------------- C6 H12O6 + 6H2O + 6O2

Chlorophyll

Q2 What are the importance of photosynthesis I) Food for all Green plants trap solar energy by photosynthesis

process and supply food and energy for all living organisms either directly or indirectly

Ii) Oxygen to breathe in by product of photosynthesis is oxygen which is essential for all living organisms respiration

Q3 Write about two main phases of photosynthesis A Light dependent phase This phase occur in grana of chloroplast I) The chlorophyll on exposure to light energy becomes activated by

absorbing photons Ii) The absorbed energy is used in splitting the water molecules (H2O)

into its two components (H+ and OH- ) and releasing electron s 2H2O------------------------- 4H+ + 4e- +O2

Energy of 4 photons This reaction is known as photolysis

End products are H+ and oxygen water

B Light independent (Dark ) phase The reactions in this phase require no light energy

Here CO2 combine with H+ and produce glucose

Class XI

Subject Topic Summary ExecutionEVS Chapter-4 Legal

regimes for sustainable development

Environmental legislationEnvironmental legislation is the collection of laws and regulations pertaining to air quality water quality the wilderness endangered wildlife and other environmental factors The act ensures that matters important to the environment are thoroughly

Learn -The Forest (Conservation) Act 1980

considered in any decisions made by federal agencies

The Forest (Conservation) Act 1980 The Forest (Conservation) Act 1980 an Act of the Parliament of India to provide for the conservation of forests and for matters connected therewith or ancillary or incidental thereto It was further amended in 1988 This law extends to the whole of IndiaObjects and Reasons of the Forest Conservation Act

Deforestation causes ecological imbalance and leads to environmental deterioration Deforestation had been taking place on a large scale in the country and it had caused widespread concern The act seeks to check upon deforestation and de-reservation of forests

Subject Eng Literature (The Tempest ndash William Shakespeare) Topic Act II Scene 1 Lines 314 to 329 (End of scene)

[Students should read the original play and also the paraphrase given in the school prescribed textbook]Summary Questions amp Answers

Conspiracy of Antonio and Sebastian (Contd)

o As they approach Ariel appears again and wakes up Gonzalo by singing a tune in his ear Alonso also wakes up and they see both Sebastian and Antonio with drawn swords On being caught off guard they make up a story saying that they had heard a bellowing of bulls or lions

o They then moved to another part of the island

o Ariel at once rushes to Prospero to inform him of this development

SUMMING-UP of ACT-2 SCENE-1

(i) Among the survivors Ferdinand is separated from the rest which results in the disconsolate grief of Alonso as he took him for dead

(ii) The villainy of Antonio is confirmed

(iii) The supremacy of Prosperorsquos magic which resulted in the failure of the human conspiracy

(1)

(Act II Sc 1 L 311-325)SEBASTIAN Whiles we stood here securing your repose

Even now we heard a hollow burst of bellowing Like bulls or rather lions Didt not wake youIt struck mine ear most terribly

ALONSO I heard nothingANTONIO O rsquotwas a din to fright a monsters ear

To make an earthquake Sure it was the roarOf a whole herd of lions

ALONSO Heard you this GonzaloGONZALO Upon mine honour sir I heard a humming

And that a strange one too which did awake meI shaked you sir and cried As mine eyes opened I saw their weapons drawn There was a noiseThats verily rsquoTis best we stand upon our guardOr that we quit this place Lets draw our weapons

(i) Why has Prospero sent Ariel to Gonzalo and Alonso What does Ariel do to awaken Gonzalo

Prospero has already come to know by his magic powers the danger which threatens Gonzalo who had been Prosperorsquos friend and so he sent Ariel to preserve the lives of both Gonzalo and Alonso Prospero does not want that his scheme should remain unfulfilled Ariel begins to sing a song in Gonzalorsquos ears to awaken him(ii) Who are ready to carry out their plan Who takes steps to stop them Why does Gonzalo feel surprised after being awakened

Sebastian and Antonio are ready to carry out their plans They are standing with their swords drawn to kill Alonso and

(iv) We see two sets of contrasting characters Gonzalo-Adrian against Antonio-Sebastian

(v) The grief that works in Alonso can be perceived to his repentance for his association in Antoniorsquos crime against Prospero

Gonzalo Ariel takes steps to stop them from carrying out their nefarious scheme When Gonzalo is awakened by the song sung by Ariel into his ears he (Gonzalo) feels surprised because he sees Sebastian and Antonio standing with their swords drawn(iii) What reason do Sebastian and Antonio tell of drawing their swords when they are suspected by Alonso and Gonzalo

When Sebastian and Antonio are seen with their swords drawn they are looked with suspicion by Gonzalo and Alonso At first Sebastian tells them that as they stood here to guard them during their sleep they heard only a little before a sudden loud noise very much like the roaring of bulls or more probably that of lions Then Antonio follows him saying that this was a noise so terrible as to frighten even a monsterrsquos ears and this noise could even have shaken the earth and it was surely like the roaring of a multitude of lions Then seeing the danger they have drawn their swords Perhaps after hearing the terrible noise they (Gonzalo and Alonso) woke up from their sound sleep

(iv) What does Gonzalo tell Alonso about the strange noise What did he see on opening his eyes Gonzalo tells Alonso that he did not hear the sound of roaring but he heard a humming sound which was strange and which woke him up After waking up he gave him (Alonso) a shaking and a loud cry On opening his eyes he saw these two gentlemen standing with their swords drawn(v) What does Gonzalo suggest

Gonzalo suggests that there was a noise indeed and of that he has no doubt at all and suggests that the best course for them would be to remain alert and vigilant against any possible danger to their lives or to leave this place and move to some other part of the island

Class XIISubject Topic Summary Execution

Commerce

Chapter- Management

Today we will discuss about LEVELS OF MANAGEMENT

Levels of management is a series or chain of managerial positions from top to bottom It helps individuals to know their authority responsibilities and superior-subordinate relations among themselves There are mainly three levels of Management TOP LEVEL MANAGEMENTMIDDLE LEVEL MANAGEMENTLOWER LEVEL MANAGEMENT

Top level managementIt consists of members at the highest level in the management hierarchy This level includes Board Of Directors Chief Executive Managing Directors Chairman President Vice President

Rolefunctions of the top levelmanagement1To analyse evaluate and deal

with theexternal environment2 To determine the objectives and

policies of the business3 To strive for welfare and survival

of business

4 To create an organisational Framework consisting of authority responsibility relationship

Middle level management Congress of members or groups who are concerned with implementation of the policies let down by the top managementThis level includes head of the department such as finance manager marketing manager branch and regional managers departmental and divisional heads plant superintendent etc

Role of functions of the middle level management

1 To interpret the policies framed by top management

2 To assign duties and responsibilities to lower level managers

3 To select and appoint employees for middle and supervisory level and evaluate their performance

4 To co-operate with other departments for smooth functioning

Operational or supervisory level managementIt refers to the group are members who are concerned with execution of the work They are also known as fast line managers This level includes supervisor 4 men Section Officer clerk Inspector etc

Role of functions of the lower level management1 To plan and execute day-to-

day operations2 To supervise and control the workers3 To arrange materials and

tools to start the process and make arrangements for training

4 Today present workers grievance and suggestions before the management and

ensure safe and proper working conditions in the factory

Business Studies

Staff Appraisal Chapter- 10 Today let us start with a new chapter

Staff Appraisal

Meaning of Performance Appraisal

Performance Appraisal is the systematic evaluation of the performance of employees and to understand the abilities of a person for further growth and developmentThe supervisors measure the pay of employees and compare it with targets and plansThe supervisor analyses the factors behind work performances of employeesThe employers are in position to guide the employees for a better performance

Objectives of Performance Appraisal

Following are the objectives of Performance Appraisal

To maintain records in order to determine compensation packages wage structure salaries raises etc

To identify the strengths and weaknesses of employees to place right men on right job

To maintain and assess the potential present in a person for further growth and development

To provide a feedback to employees regarding their performance and related status

To provide a feedback to employees regarding their performance and related status

Importance of Performance Appraisal

Performance appraisal provides important and useful information for the assessment of employees skill

knowledge ability and overall job performance The following are the points which indicate the importance of performance appraisal in an organization

1 Performance appraisal helps supervisors to assess the work performance of their subordinates

2 Performance appraisal helps to assess the training and development needs of employees

3 Performance appraisal provides grounds for employees to correct their mistakes and it also provides proper guidance and criticism for employees development4 Performance appraisal provides reward for better performance

5 Performance appraisal helps to improve the communication system of the organization

6 Performance appraisal evaluates whether human resource programs being implemented in the organization have been effective

7 Performance appraisal helps to prepare pay structure for each employee working in the organization

8 Performance appraisal helps to review the potentiality of employees so that their future capability is anticipated

Geography

DRIANAGE The SubarnarekhaThe Subarnarekha and the Brahmaniinterposed between the Ganga and the Mahanadi deltas drain an area of 19300 sq kmand 39033 sq km respectively The drainage basins of these streams are shared byJharkhand Odisha west Bengal and Chhattisgarh The Brahmani is known as southKoel in its upper reaches in Jharkhand

The NarmadaThe Narmada rises in the Amarkantak hills of MadhyaPradesh It flows towards the West in a rift valleyformed due to a geological fault The total length of it is 1300 km All the tributaries of the

Q1 Name the two westward flowing rivers in the peninsular plateauA1 Narmada and Tapi are the only westward flowing rivers of the peninsular plateau

Q2 Differentiate between east-flowing rivers and west-flowing riversA2

East-flowing rivers

West-flowing rivers

Narmada are very short inlength Most of its tributaries join the main streamright anglesThe Narmada basin covers parts of Madhya Pradesh and Gujarat

The Tapi The Tapi rises in the Satpura ranges in the Betul listrictof Madhya Pradesh It flows in a rift valley parallel tothe Narmada but it is much shorter in length It coversparts of Madhya Pradesh Gujarat and MaharashtraThe length is about 724 km

The Sabarmati and the MahiThe Sabarmati rises in the Aravali hills and flows south-south-westwards for a distance of 300 kilometres to the Arabian Sea The Sabarmatibasin extends over an area of 21674 sq km in Rajasthan and Gujarat The Mahi rises inthe east of Udaipur and drains an area of 34842 sq km lying in Madhya PradeshRajasthan and Gujarat It flows south-westwards for a distance of 533 km before it fallsinto the Gulf of Khambhat

The ChambalThe Chambal rises near Mhow in the Vindhya Range and flows towards the northgenerally in a gorge upto Kota Below Kota it turns to the north-east direction and afterreaching Pinahat it turns to the east and runs nearly parallel to the Yamuna beforejoining it in the southern part of the Etawah district in Uttar PradeshMajor Rivers of India with their basin area (Sqkm)

Himalayan System Indus 321290Ganga 861404

Brahmaputra 187110Indus System

Jhelum 34775Beas 20303

Ganga System Yamuna 366223Ghaghra 127950

Peninsular RiversNarmada 98796

Tapi 65145Mahanadi 141600

Subarnarekha 19300Sabarmati 21674

Mahi 34842Godavari 312812

Godavari Krishna Kaveri Mahanadi are the east-flowing rivers

Narmada Tapi west-flowing rivers

They fall into the Bay of Bengal

They fall into Arabian Sea

These rivers form big deltas

These rivers form comparativelysmall deltas

Catchment areas of these rivers are larger

Catchment areas of these rivers are smaller

Krishna 2589488Cauveri 87900

Subject ndashBiology Topic ndashChapter -5 Inheritance amp Variations Summary ExecutionToday we will discussabout linkage and its classification

LINKAGE The tendency of the genes located on the same chromosome to stay together is

hereditary transmission Linked genes the genes responsible for this Genes that exhibit the process of linkage locates in the same chromosome The distance between the linked genes in a chromosome determines the strength

of linkage i e genes that are located close to each other show stronger linkage than that are located far from each other

COMPLETE LINKAGE It is the type of linkage showed by the genes that are closely located or are tightly

linked with each other as they have no chance of separatingby crossing over These genes are always transmitted together to the same gamete and the same

offspring In such condition only parental or non cross over type of gametes are formedINCOMPLETE KINKAGE It is type of linkage showed by the genes that are distantly located orare loosely

linked with each other because they have chance of separating by crossing over

SIGNIFICANCE i) It helps in holding the parental character togetherii) It checks the appearance of new recombination and helps in bringing the

hybrid population which resembles the original parents iii) Linked genes dilute the effects of undesirable traits

Subject Eng Literature (The Tempest ndash William Shakespeare) Topic Essay Questions (EQ-3)Question No 3

Give a character sketch of CalibanAnswer

The character of Caliban has been wonderfully conceived by Shakespeare as the manifestation of all that is gross and earthy ndash a sort of creature of the earth as Ariel is a sort of creature of the air

Calibanrsquos Physical Appearanceo Caliban is lsquofreckledrsquo a lsquomisshapen knaversquo not honoured with human shape

o Prospero calls him lsquothou tortoisersquo (Act I Sc 2 Line 317) Trinculo stumbling upon him describes him as ldquoA strange fish hellip Legged like a man And his fins like armsrdquo He ldquosmells like a fishrdquo (Act II Sc 2 Line 25)

o Prospero also calls him a ldquobeastrdquo (Act IV Sc 1 Line 140) and ldquoThis misshapen knaverdquo (Act V Sc 1 Line 268)

o Further it appears that in addition to his physical deformity his spiritual inferiority is also suggested by Prosperorsquos claim that his birth resulted from the union between his mother the witch Sycorax and the devil

Calibanrsquos ParentageWhen the play opens Caliban is twenty four years of age having been born on the island twelve years before the coming of Prospero His mother was the foul witch Sycorax who was banished from Algiers for ldquomischiefs manifold and sorceries terrible to enter human hearingrdquo (Act I Sc 2 Line 264) and the father was the Devil himself Thus

Caliban is a monster of evil and brute nature ugly deformed and stinking

Calibanrsquos Savage and Malignant Natureo Caliban is entirely a creature of the earth ndash gross brutal and savage He regards himself as the rightful possessor

of the island and Prospero as a usurper

o In his young age he was on good terms with Prospero He had consented to be received by Prospero at his house and to be educated by him He has learnt human language only to curse his master whom he abhors

o His beastly nature soon breaks out and ends in a vicious attack on Miranda This opens the eye of Prospero who becomes severe to him and enforces his service by threats and violence

o Prospero uses him to make dams for fish to fetch firewood scraper trenches wash dishes and keep his cell clean

Calibanrsquos Hatred for ProsperoA profound hatred for Prospero has taken hold of Caliban It springs from a sense of his being dispossessed and ill-treated He would kill Prospero if he could but he knows the power of Prosperorsquos lsquobookrsquo Hence he transfers his allegiance to Stephano who seems like a god to him He also incites the two drunken associates to batter the skull of Prospero when he sleeps in the afternoon

Caliban Shows Considerable Intelligenceo He has learnt Prosperorsquos language

ldquoYou taught me language and my profit onrsquot (Act II Sc 2 Lines 86-89)Is I know how to curserdquo

o He is well aware of the futility of arguing with one who has more power than he has

ldquoI must obey his art is such power (Act I Sc 2 Lines 373-376)It would control my damrsquos god SetebosAnd make a vassal of himrdquo

o He realizes the importance of Prosperorsquos books

ldquoRemember (Act III Sc 2 Lines 89-92)First to possess his books for without themHersquos but a sot as I am nor hath notOne spirit to commandrdquo

o He knows the value of stealth when attacking the enemy

ldquoPray you tread softly that the blind mole may not (Act IV Sc 1 Lines 194-195)Hear a foot fall we now are near his cellrdquo

o Caliban has a better set of values than Stephano and Trinculo They are distracted from their plan by their greed for Prosperorsquos rich garments Only Caliban realizes that such a finery is unimportant

ldquoLeave it alone thou fool it is but trashrdquo (Act IV Sc 1 Lines 224)

Caliban is not a good judge of characterCaliban is not a good judge of character He decides for example that Stephano is a god because he dispenses lsquocelestial liquorrsquo (Act II Sc 2 Line 115) but then it must be remembered that he has only known his mother Sycorax Prospero Miranda and the spirits that torture him However he quickly discovers his error of judgementrdquo

ldquoWhat a thrice-double ass (Act V Sc 1 Lines 295-297)Was I to take this drunkard for a godAnd worship this dull foolrdquo

Calibanrsquos Imaginative NatureIf Caliban is sub-human in what has been said above he is human in the respect of the poetic side of his character He listens to music with rapture He tells of the beautiful dreams in which heaven rains treasures upon him and which upon waking he yearns to renew One of the most poetic passages in whole play is Calibanrsquos description of the island

to Stephano and Trinculo

ldquoBe not afeard The isle is full of noises (Act III Sc 2 Lines 135-143)Sounds and sweet airs that give delight and hurt notSometimes a thousand twangling instrumentsWill hum about mine ears and sometime voicesThat if I then had waked after long sleepWill make me sleep again and then in dreamingThe clouds methought would open and show richesReady to drop upon me that when I wakedI cried to dream againrdquo

Caliban - Less Ignoble Than Some OthersCalibanrsquos motive for murder is less dishonourable than that of Antonio and Sebastian They plan to kill Alonso to gain his power and wealth Caliban merely wants revenge and the return of lsquohisrsquo island

Conclusiono Calibanrsquos character is not portrayed very clearly in the play and hence we cannot decide whether he is a poor

savage being grossly maltreated by Prospero or whether he is evil and must therefore be kept in bondage or enslavement

o Caliban is contrasted with Ariel who is a spirit and thus swift and uninterested in physical activitieso Caliban is also contrasted with Prospero who is the all-powerful master of the island and of the destiny of all

those on the islando Caliban is also contrasted with civilized man showing him to be less evil than Antonio and Stephano and less

materialistic than Stephano and Trinculoo Caliban has suffered at the hands of Prospero and he has learnt to curse by listening to Prosperorsquos abuse He

certainly believes that Prospero has deprived him of his birthrighto Finally the character Caliban is thought to be one of Shakespearersquos masterpieces The complexity of the character

is reflected in the large volume of critical discussion that has grown around it

ECO ndash12 Topic-Forms of market

MonopolyMonopoly is a market structure in which there is a single seller there are no close substitutes for the commodity produced by the firm and there are barriers to entry Example Indian Railways which is operated under government of India Monopoly also implies absence of competitionFeatures of Monopoly Monopoly is characterized by1 Single Seller In monopoly there is only one firm producing the product The whole industry consists of this single firm Thus under monopoly there is no distinction between firm and industry Being the only firm there is significant control of the firm over supply and price Thus under monopoly buyers do not have the option of buying the commodity from any other seller They have to buy the product from the firm or they can go without the commodity This fact gives immense control to the monopolist over the market

2No Close Substitute There are no close substitutes of the product produced by the monopolist firm If there are close substitutes of the product in the market it implies presence of more than one firm and hence no monopoly In order to ensure a total of control over the market by the monopolist firm it is assumed that there are no close substitutes of the product

3 No Entry amp Exit Monopoly can only exist when there is strong barriers before a new firm to enter the market In fact once a monopoly firm starts producing the product no other firm can produce the same One reason for this is the ability of the

monopolist to produce the product at a lower cost than any new firm who thinks to enter the market If a new firm who knows that it cannot produce at a lower cost than the monopolist then that firm will never enter the market for fear of losing out in competition Similarly the monopolist who is operating for a long time may be enjoying reputation among its customers and is in a better position to use the situation in its own benefit A new firm has to take long time to achieve this and so may not be interested to enter the market

4 Price Maker Being the single seller of the product the monopolist has full control over the pricing of the product On the other hand if there is a large number of buyers in the market so no single buyer exercises any significant influence over price determination Thus it is a sellerrsquos market So monopoly firm is a price maker

5 Price Discrimination Having considerable control over the market on account of being single seller with no entry of other firms the monopolist can exercise policy of price discrimination it means that the monopolist can sell different quantities of the same product to a consumer at different price or same quantity to different consumers at different prices by adjudging the standard of living of the consumer

6 Shape of Demand Curve Since a monopolist has full control over the price therefore he can sell more by lowering the price This makes the demand curve downward sloping

Subject Ac-12 290620 Topic- retirement Model sumThe Balance Sheet of Rohit Nisha and Sunil who are partners in a firm sharing profits according to their capitals as on 31st March 2014 was as under

Liabilities Amount Assets Amount (Rs) (` Rs)

Creditors 25000 Machinery 40000Bills Payable 13000 Building 90000General Reserve 22000 Debtors 30000Capital Less Provision for Rohit 60000 Bad debts 1000

29000 Nisha 40000 Stocks 23000 Sunil 40000 140000 Cash at Bank 18000

200000 200000

On the date of Balance Sheet Nisha retired from the firm and following adjustments were made(i) Building is appreciated by 20(ii) Provision for bad debts is increased to 5 on Debtors(iii) Machinery is depreciated by 10(iv) Goodwill of the firm is valued at Rs 56000 and the retiring partnerrsquos share is adjusted

(v) The capital of the new firm is fixed at Rs120000 Prepare Revaluation Account Capital Accounts of the partner and Balance Sheet of the new firm after Nisharsquos retirement Revaluation AccountDr Cr

Particulars Amount Particulars Amount (`Rs) (Rs`)

Provision for Bad debt Ac 500 Building Ac 18000Machinery Ac 4000Profit transferred toCapital Accounts (3 2 2)Rohit 5786Nisha 3857Sunil 3857

13500

18000 18000

Capital Account

Dr Cr

Particulars Rohit Nisha Sunil Particulars Rohit Nisha Sunil (Rs`) (Rs`) (`Rs) (Rs`) (Rs`) (Rs`)

Sunilrsquos Capital ac 9600 mdash 6400 Balance bd 60000 40000 40000Bank - 66143 - General Reserve 9428 6286 6286Balance cd 72000 mdash 48000 Revaluation (Profi 5786 3857 3857 Rohitrsquos Capital Ac mdash 9600 mdash

Sunilrsquos Capital Ac 6400 Bank 6386 - 4257

81600 66143 54400 81600 66143 54400

Balance Sheet as at 31st March 2014

Liabilities Amount Assets Amount (Rs`) (Rs`)

Creditors 25000 Building 108000Bank overdraft 37500 Machinery 36000

Bills Payable 13000 Debtors 30000Capital Less ProvisionRohit 72000 for Bad debts 1500 28500Sunil 48000 120000 Stock 23000

195500 195500

Working Notes (i) (a) Profit sharing ratio is 60000 40000 40000 ie = 3 2 2(b) Gaining Ratio Rohit = 35 ndash 37 = 2135 ndash 1535 = 635Sunil = 25-27 = 1435 ndash 1035 = 435= 635 435= 6 4 = 3 2(c) Nisha Share of Goodwill = Rs 56000 times 27 = Rs16000Share of Goodwill in the gaining ratio by the existing partner ieRohit = Rs16000 times 35 = Rs 9600Sunil = Rs 16000 times 25 = Rs 6400

The journal entry isRohitrsquos Capital Ac Dr 9600Sunilrsquos Capital Ac Dr 6400 To Nisharsquos Capital Ac 16000(Share of Goodwill divided into gaining ratio)

  • 1 Static Friction
  • The frictional force that acts between the surfaces when they are at rest with respect to each other is called Static Friction
    • Static Friction Examples
      • 2 Sliding Friction
        • Examples Of Sliding Friction
          • 3 Rolling Friction
            • Examples Of Rolling Friction
              • Objects and Reasons of the Forest Conservation Act
Page 36:  · Web viewSubject . Topic . Summary . Execution . English 1 . Sounds of animals . Hens –cackle Horses –neigh Lions –roar Owls –hoots Snake –hiss. English 2 . Mother’s

Explanation

While praising Portia the Prince of Morocco conceives Portia as a goddess whose image is placed inside one of the caskets Many suitors are coming from far and wide the north and the south the east and the west (Four corners) in order to try their luck Some of them have come from the distant land of Persia and Arabia The deserts of Persia (Hyrcanian deserts) and the boundless desolate lands of Arabia have been crossed by the Princes seeking the hand of Portia All this shows that Portia is indeed the most beautiful lady of the world

(iii) What does the Prince of Morocco say in his estimation while examining the motto on the silver casket What does he find in the golden casket

While examining the motto on the silver casket which says ldquoWho chooseth me shall get as much as he deservesrdquo Morocco says that in his own estimation he surely deserves Portia in all respects ndash rank birth wealth etc

He chooses the golden casket When he opens it he finds an empty human skull holding a scroll in which it is written that those who are attracted by the glittering outside of things are always deceived as Morocco has been deceived

(iv) What kind of nature does the Prince of Morocco have

The Prince of Morocco has a simple nature who does not look deeply into the inner meaning of things but is dazzled by the outward appearance of gold He is inclined to over-estimate his own value and does not realize that it is a duty to ldquogive and hazardrdquo To say that he will not hazard for lead shows that he misreads the true meaning of the inscription which is that he should be prepared to ldquohazard all he hathrdquo for Portia So his feeling is only one of fascination and romantic attraction

(v) Do you think that the lottery of the caskets is not a matter that will be determined by chance

In fact the lottery of the casket is not a matter that will be determined by mere chance but that it is a true test of character and of sincerity which is amply proved not only by Moroccorsquos choice but also by the arguments which he uses to help him in his choice

(Act II Sc 7 L 55-59)

They have in England

A coin that bears the figure of an angelStamped in gold but thats insculpd uponBut here an angel in a golden bedLies all within

Context

(3)

(Act II Sc 7 L 63-77)A carrion Death within whose empty eye

There is a written scroll Ill read the writing

All that glisters is not goldOften have you heard that toldMany a man his life hath soldBut my outside to beholdGilded tombs do worms infoldHad you been as wise as boldYoung in limbs in judgment oldYour answer had not been inscrolld

This passage occurs in Act II Scene 7 in The Merchant of Venice This is part of the speech made by the Prince of Morocco

Explanation

In this passage the Prince of Morocco bestows high praise on Portia whose hand he is seeking He contrasts this casket containing Portiarsquos portrait with the old English gold coin bearing the image of the archangel (angel of the highest rank) He goes on to remark that while the figure of the archangel is engraved (Insculped) upon the English coin the picture of Portia who is beautiful as an angel lies hidden inside one of the caskets namely the Golden Casket (Golden Bed) In the day of Elizabeth silver was ten times inferior in value to gold Therefore the Prince of Morocco believing that Portiarsquos portrait is contained in the Golden Casket decides to choose the Golden Casket

Fare you well your suit is coldCold indeed and labour lostThen farewell heat and welcome frostmdashPortia adieu I have too grievd a heartTo take a tedious leave Thus losers part

(i) What reward does the Prince of Morocco get after making a wrong choice of the Casket How does he feel

After making the wrong choice in selecting the casket of gold the Prince of Morocco as a reward earns a rebuke in the form of a scroll tucked in the empty eye-socket of a skull kept in the casket of gold The Prince is shocked and disappointed He becomes all the more sad and dejected when he reads the scroll which points to his foolishness in being misled by the appearance and outward show as indicative of its worth

(ii) How does the Prince respond after reading the scroll

After reading the scroll the Prince though upset accepts the result with good grace and decorum befitting a royal suitor and true sportsman He says that his love-suit is really cold otherwise he would have chosen correctly but now his efforts have been in vain So he bids farewell to Portia to the warmth and enthusiasm of love and welcomes the cold and bitterness of dejection and misery of life which lies ahead

(iii) What request does he make to Portia and why

After being failure in his mission he requests Portia to give him permission to leave at once because he is too sad to undergo the tediousness of a formal leave-taking He tells that it is the manner in which defeated persons part unceremoniously

(iv) Explain the following lines

ldquoAll that glisters is not goldOften have you heard that toldMany a man his life hath soldBut my outside to beholdGilded tombs do worms infoldrdquo

Mere glitter does not make a metal to be gold Man has often been warned against appearance but it has been of no use Many people have sacrificed their lives only to seek the outer appearance of gold Worms are found inside the gilded

monuments

Class XSubject Topic Summary Execution

Hindi 2ndlang

नया रासता भाग 6 मायाराम 0ी घर म धनी मल 0ी और उनी बटी सरिरता ी ही चचा बनी रहती थी अमिमत ो इसम ोई रलिच ना थी वह धनी घर ी लडी स शादी र सवय ो बचना नही चाहता था उसा भी सवाणिभमान ह ईशवर ी पा

स उस पास पस ी ोई मी नही थी अभी उसन फकटरी ही लगाई थी उसी समझ बाहर था कि उस घर वालो ा झाव पस ी तरफ कयो

ह उसन मा स सवाल किया कि मा तम सरिरता स मरी शादी कयो रना चाहती हो मा न उस समझाया कि वह दखन म बरी नही ह और किफर खानदान अचछा

ह वह ए शल गरहणी रप म घर सभाल सगी अमिमत न मा ो इस बात ा एहसास राया कि मीन सबध लिलए मना रन पर उस दिदल

पर कया बीती होगी मा और अमिमत ी लडी बार म ाफी बात हईमा ा झाव सरिरता ी तरफ था कयोकि वह घर पर अचछा दह0 लर आ रही

थी अमिमत न अपनी मौसी ी बरी हालत बार म बताया कि किस तरह वह बड घर ी खानदानी बटी लाई थी और आ0 उसी हालत कितनी खराब ह लाई थी बहकलब 0ाती ह और बचचो ो भी नही दखती ह बात चल ही रही

थी कि तभी ए ार बाहर आर री धनी मल0ी घर अदर आए और पीछ स डराइवर फल ी ए टोरी लर आया अदर आए और पीछ स

डराइवर ए टोरी फल ी लर आया अमिमत ो फल ी पटी बरी लग रही थी अमिमत न पछ लिलया यह फल कयो ल आए ह प इन सब ी कया

0ररत थी उनो न 0वाब दिदया कि 4 पटी शमीर स मगाए थ अमिमत ो या सनर करोध आ गया तभी उस किपता 0ी आ गए उन आत ही अमिमत उठर बाहर चला गया वहा वहा मा पास आर बठ गया और बोला

अभी रिरशता तय नही हआ और धनी मल 0ी धनी मल 0ी फल ी पटी लर चलआय मा न समझाया कि 0ब सबध 0ड 0ाता ह तो खाली हाथ नही

आत अमिमत न मा स हा कि तम सबन सरिरता ो इस घर म लान ी ठान रखी ह धनीमल 0ी उस दिदन सरिरता ो दखन ी तारीख तय रन आय थ

Commercial Studies

Banking Nowadays Bank provide easy and quick services through internet facilities methods of Banking is called internet bankingIn order to save the time and money involved in visiting Bank branches people increasingly prefer to have internet banking

There are different modes of doing internet banking or transferring money through online They areReal Time Gross Settlement (RTGS)National Electronic Fund Transfers (NEFT)

1

Question

1) Explain the term RTGS Write the features of RTGS

Answer)The acronym RTGS stands for Real Time Gross Settlement which may be defined as the continuous real time settlement of funds transfer individually on and order by order basis without netting lsquoReal timersquo may be defined as the processing of instructions at the time they are received rather than at some letter time lsquoGross settlementrsquo may be defined as the settlement of transfer instructions which occurs

individually

Features of RTGS1It is the continuous settlement of

funds transfer individually on an order by order basis

2RTGS facility is provided only by CBS core banking solution enabled Bank branches

3Amount charged from the customer for RTGS transactions vary from bank to bank

2) Explain the term NEFT Write the features of NEFT

Answer) National electronic funds transfer may be defined as a nationwide system that facilitates individuals Farms and copper operates to electronically transfer funds from any bank branch to any individual farm or corporate having an account with any other bank branch in the country

Features of NEFT2 Transfer can be made 7 times on

weekdays and 6 times on Saturday

3 NEFT cannot be used to receive foreign remittances

4 NEFT transaction takes place in batches

5 A bank branch must be NEFT enabled to become a part of NEFT fund transfer network

6 There is no maximum or minimum amount that can be transferred through NEFT when one bank has a bank account

English Language

CompositionEssay

A composition is an art of creating a piece of writing on any topic or subject It is the writing correctly beautifully and clearly in order to make some interesting reading Structure of the composition

Introduction ( you lay the foundation for your composition)

Body (it constitutes the main part of the essay)

Conclusion (final statement that leaves a lasting impression)

Kinds of essays1 The Narrative essay2 The descriptive essay3 The reflective essay4 The argumentative essay

Write a composition on any one of the following topics (350- 400 words)

1 Friendship Or2 The first day of your school

Subject Eng Literature (The Merchant of Venice ndash William Shakespeare)Topic Act V Scene 1 Lines 127 to 158 (Nerissa helliphellip The clerk will nersquoer wear hair onrsquos face that had it) [Students should read the original play and also the paraphrase given in the school prescribed textbook]

Summary Revision Questions o Soon thereafter Bassanio Gratiano

and Antonio arrive

o Bassanio tells Portia that he is feeling as if it is morning because of the presence of Portia who is shining like the sun When Antonio is introduced by Bassanio to Portia she tells Bassanio that he should be grateful to Antonio who took so much trouble on his account even to the extent of risking his life

o Nerissa starts quarrelling with Gratiano and demands that he show her the ring she had presented to him and which she had warned him not to lose She suspects that Gratiano must have presented the ring to some young woman and not to the lawyerrsquos clerk as he repeatedly says and assures

Answer the following questions to check your preparation of Act IV Scenes 1 and 2

You must attempt only after you have completed your preparation of Act IV The answers must be in complete sentences using textual evidence (with citation) when necessary

[It would be in your own interest to attempt the above questions honestly totally refraining from consulting your textbook or your notes during answering After completion you should correct the paper yourself consulting the textbooknotes etc and award marks as specified Please let me know the marks you scored through WhatsApp in the group or to my personal WhatsApp]

Act IV Scene 1 (each question carries 2 marks)

1 What did the Duke try to do for Antonio

2 Why does Shylock refuse to show mercy How does he justify his stance

3 Why does Antonio say he is ready to die 4 What information is contained in Bellariorsquos letter

5 Why does Portia (as Balthazar) assert that Shylock must show mercy How does he respond

6 What offers are made to Shylock to get him to spare Antonio How are they received

7 What does Antoniorsquos speech as he faces the prospect of Shylockrsquos knife tell you about his character

8 How do Bassanio and Gratiano react to the looming prospect of Antoniorsquos demise

9 How does Portia (as Balthazar) use the law to turn the tables on Shylock

10 What does the Duke decree should happen to Shylock Why What happens to Shylockrsquos estate

11 What does Portia ask Bassanio as payment for her ldquoservicesrdquo What is his initial response What makes him change his mind

Act IV Scene 2 (each question carries 1frac12 marks)

1 What does Gratiano bring to Portia (Balthazar)

2 What does Nerissa plan on getting from Gratiano What does Portiarsquos comment suggest about men

ECO-10 280620 Topic-Supply AnalysisSHIFTING OF SUPPLY

But if there is change in factors other than the price of the commodity then either more is supplied at the same price or less supplied at the same price In such cases the price of the commodity remains constant but there is a change in other factors like change in the price of inputs change in technology of production change in price of other related goods change in taxation policy of the government etc For example there is an improvement in the technology of production of the commodity in question It leads to decrease in per unit of cost production of the commodity The firm is willing to sell more quantity of the commodity at the same price So the supply other commodity increases at the same price This increase in supply is shown by rightward shift of supply curve On the other hand if the firm uses inferior technology of production the cost of production per unit of the commodity increases The firm is willing to sell less quantity at the same price So the supply of the commodity decreases at the same price This decrease in supply is shown by leftward shift of the supply curve The above cases of increase and decrease in supply can be shown with the help of the following figures

Y INCREASE OF SUPPLY Price (Rs) s

P A s1

B

s

X` O s1 X

q q1

Y` Quantity demanded (in units)

Y DECREASE IN SUPPLY s2

s

price (Rs)

C

p A

s2

s

X` o X

q2 q

Y` Quantity demanded ( in units)

Main factors causing increase in supply or rightward shift of supply Curve(i) Fall in the price of other related goods

(ii) Fall in the price of inputsfactors(iii) Use of better technology in production(iv) Decrease in the rate of excise duty by government(v) If the objective of producer changes from profit maximization to salesMaximization

Main factors causing decrease in supply or leftward shift of supply curve(i) Increase in the price of other related goods(ii) Rise in the price of inputsfactors(iii) Use of inferior technology in production(iv) Increase in the rate of excise duty by the government(v) If the objective

Subject - Biology Topic ndash Chapter mdash6 PhotosynthesisSummary Execution

Today we will know about photosynthesis and its stages

Q1 What do you mean by photosynthesis The process by which living plants containing chlorophyll produce food

substances from carbon-di- oxide and water by using light energy Sunlight

6CO2 +12 H2O----------------------- C6 H12O6 + 6H2O + 6O2

Chlorophyll

Q2 What are the importance of photosynthesis I) Food for all Green plants trap solar energy by photosynthesis

process and supply food and energy for all living organisms either directly or indirectly

Ii) Oxygen to breathe in by product of photosynthesis is oxygen which is essential for all living organisms respiration

Q3 Write about two main phases of photosynthesis A Light dependent phase This phase occur in grana of chloroplast I) The chlorophyll on exposure to light energy becomes activated by

absorbing photons Ii) The absorbed energy is used in splitting the water molecules (H2O)

into its two components (H+ and OH- ) and releasing electron s 2H2O------------------------- 4H+ + 4e- +O2

Energy of 4 photons This reaction is known as photolysis

End products are H+ and oxygen water

B Light independent (Dark ) phase The reactions in this phase require no light energy

Here CO2 combine with H+ and produce glucose

Class XI

Subject Topic Summary ExecutionEVS Chapter-4 Legal

regimes for sustainable development

Environmental legislationEnvironmental legislation is the collection of laws and regulations pertaining to air quality water quality the wilderness endangered wildlife and other environmental factors The act ensures that matters important to the environment are thoroughly

Learn -The Forest (Conservation) Act 1980

considered in any decisions made by federal agencies

The Forest (Conservation) Act 1980 The Forest (Conservation) Act 1980 an Act of the Parliament of India to provide for the conservation of forests and for matters connected therewith or ancillary or incidental thereto It was further amended in 1988 This law extends to the whole of IndiaObjects and Reasons of the Forest Conservation Act

Deforestation causes ecological imbalance and leads to environmental deterioration Deforestation had been taking place on a large scale in the country and it had caused widespread concern The act seeks to check upon deforestation and de-reservation of forests

Subject Eng Literature (The Tempest ndash William Shakespeare) Topic Act II Scene 1 Lines 314 to 329 (End of scene)

[Students should read the original play and also the paraphrase given in the school prescribed textbook]Summary Questions amp Answers

Conspiracy of Antonio and Sebastian (Contd)

o As they approach Ariel appears again and wakes up Gonzalo by singing a tune in his ear Alonso also wakes up and they see both Sebastian and Antonio with drawn swords On being caught off guard they make up a story saying that they had heard a bellowing of bulls or lions

o They then moved to another part of the island

o Ariel at once rushes to Prospero to inform him of this development

SUMMING-UP of ACT-2 SCENE-1

(i) Among the survivors Ferdinand is separated from the rest which results in the disconsolate grief of Alonso as he took him for dead

(ii) The villainy of Antonio is confirmed

(iii) The supremacy of Prosperorsquos magic which resulted in the failure of the human conspiracy

(1)

(Act II Sc 1 L 311-325)SEBASTIAN Whiles we stood here securing your repose

Even now we heard a hollow burst of bellowing Like bulls or rather lions Didt not wake youIt struck mine ear most terribly

ALONSO I heard nothingANTONIO O rsquotwas a din to fright a monsters ear

To make an earthquake Sure it was the roarOf a whole herd of lions

ALONSO Heard you this GonzaloGONZALO Upon mine honour sir I heard a humming

And that a strange one too which did awake meI shaked you sir and cried As mine eyes opened I saw their weapons drawn There was a noiseThats verily rsquoTis best we stand upon our guardOr that we quit this place Lets draw our weapons

(i) Why has Prospero sent Ariel to Gonzalo and Alonso What does Ariel do to awaken Gonzalo

Prospero has already come to know by his magic powers the danger which threatens Gonzalo who had been Prosperorsquos friend and so he sent Ariel to preserve the lives of both Gonzalo and Alonso Prospero does not want that his scheme should remain unfulfilled Ariel begins to sing a song in Gonzalorsquos ears to awaken him(ii) Who are ready to carry out their plan Who takes steps to stop them Why does Gonzalo feel surprised after being awakened

Sebastian and Antonio are ready to carry out their plans They are standing with their swords drawn to kill Alonso and

(iv) We see two sets of contrasting characters Gonzalo-Adrian against Antonio-Sebastian

(v) The grief that works in Alonso can be perceived to his repentance for his association in Antoniorsquos crime against Prospero

Gonzalo Ariel takes steps to stop them from carrying out their nefarious scheme When Gonzalo is awakened by the song sung by Ariel into his ears he (Gonzalo) feels surprised because he sees Sebastian and Antonio standing with their swords drawn(iii) What reason do Sebastian and Antonio tell of drawing their swords when they are suspected by Alonso and Gonzalo

When Sebastian and Antonio are seen with their swords drawn they are looked with suspicion by Gonzalo and Alonso At first Sebastian tells them that as they stood here to guard them during their sleep they heard only a little before a sudden loud noise very much like the roaring of bulls or more probably that of lions Then Antonio follows him saying that this was a noise so terrible as to frighten even a monsterrsquos ears and this noise could even have shaken the earth and it was surely like the roaring of a multitude of lions Then seeing the danger they have drawn their swords Perhaps after hearing the terrible noise they (Gonzalo and Alonso) woke up from their sound sleep

(iv) What does Gonzalo tell Alonso about the strange noise What did he see on opening his eyes Gonzalo tells Alonso that he did not hear the sound of roaring but he heard a humming sound which was strange and which woke him up After waking up he gave him (Alonso) a shaking and a loud cry On opening his eyes he saw these two gentlemen standing with their swords drawn(v) What does Gonzalo suggest

Gonzalo suggests that there was a noise indeed and of that he has no doubt at all and suggests that the best course for them would be to remain alert and vigilant against any possible danger to their lives or to leave this place and move to some other part of the island

Class XIISubject Topic Summary Execution

Commerce

Chapter- Management

Today we will discuss about LEVELS OF MANAGEMENT

Levels of management is a series or chain of managerial positions from top to bottom It helps individuals to know their authority responsibilities and superior-subordinate relations among themselves There are mainly three levels of Management TOP LEVEL MANAGEMENTMIDDLE LEVEL MANAGEMENTLOWER LEVEL MANAGEMENT

Top level managementIt consists of members at the highest level in the management hierarchy This level includes Board Of Directors Chief Executive Managing Directors Chairman President Vice President

Rolefunctions of the top levelmanagement1To analyse evaluate and deal

with theexternal environment2 To determine the objectives and

policies of the business3 To strive for welfare and survival

of business

4 To create an organisational Framework consisting of authority responsibility relationship

Middle level management Congress of members or groups who are concerned with implementation of the policies let down by the top managementThis level includes head of the department such as finance manager marketing manager branch and regional managers departmental and divisional heads plant superintendent etc

Role of functions of the middle level management

1 To interpret the policies framed by top management

2 To assign duties and responsibilities to lower level managers

3 To select and appoint employees for middle and supervisory level and evaluate their performance

4 To co-operate with other departments for smooth functioning

Operational or supervisory level managementIt refers to the group are members who are concerned with execution of the work They are also known as fast line managers This level includes supervisor 4 men Section Officer clerk Inspector etc

Role of functions of the lower level management1 To plan and execute day-to-

day operations2 To supervise and control the workers3 To arrange materials and

tools to start the process and make arrangements for training

4 Today present workers grievance and suggestions before the management and

ensure safe and proper working conditions in the factory

Business Studies

Staff Appraisal Chapter- 10 Today let us start with a new chapter

Staff Appraisal

Meaning of Performance Appraisal

Performance Appraisal is the systematic evaluation of the performance of employees and to understand the abilities of a person for further growth and developmentThe supervisors measure the pay of employees and compare it with targets and plansThe supervisor analyses the factors behind work performances of employeesThe employers are in position to guide the employees for a better performance

Objectives of Performance Appraisal

Following are the objectives of Performance Appraisal

To maintain records in order to determine compensation packages wage structure salaries raises etc

To identify the strengths and weaknesses of employees to place right men on right job

To maintain and assess the potential present in a person for further growth and development

To provide a feedback to employees regarding their performance and related status

To provide a feedback to employees regarding their performance and related status

Importance of Performance Appraisal

Performance appraisal provides important and useful information for the assessment of employees skill

knowledge ability and overall job performance The following are the points which indicate the importance of performance appraisal in an organization

1 Performance appraisal helps supervisors to assess the work performance of their subordinates

2 Performance appraisal helps to assess the training and development needs of employees

3 Performance appraisal provides grounds for employees to correct their mistakes and it also provides proper guidance and criticism for employees development4 Performance appraisal provides reward for better performance

5 Performance appraisal helps to improve the communication system of the organization

6 Performance appraisal evaluates whether human resource programs being implemented in the organization have been effective

7 Performance appraisal helps to prepare pay structure for each employee working in the organization

8 Performance appraisal helps to review the potentiality of employees so that their future capability is anticipated

Geography

DRIANAGE The SubarnarekhaThe Subarnarekha and the Brahmaniinterposed between the Ganga and the Mahanadi deltas drain an area of 19300 sq kmand 39033 sq km respectively The drainage basins of these streams are shared byJharkhand Odisha west Bengal and Chhattisgarh The Brahmani is known as southKoel in its upper reaches in Jharkhand

The NarmadaThe Narmada rises in the Amarkantak hills of MadhyaPradesh It flows towards the West in a rift valleyformed due to a geological fault The total length of it is 1300 km All the tributaries of the

Q1 Name the two westward flowing rivers in the peninsular plateauA1 Narmada and Tapi are the only westward flowing rivers of the peninsular plateau

Q2 Differentiate between east-flowing rivers and west-flowing riversA2

East-flowing rivers

West-flowing rivers

Narmada are very short inlength Most of its tributaries join the main streamright anglesThe Narmada basin covers parts of Madhya Pradesh and Gujarat

The Tapi The Tapi rises in the Satpura ranges in the Betul listrictof Madhya Pradesh It flows in a rift valley parallel tothe Narmada but it is much shorter in length It coversparts of Madhya Pradesh Gujarat and MaharashtraThe length is about 724 km

The Sabarmati and the MahiThe Sabarmati rises in the Aravali hills and flows south-south-westwards for a distance of 300 kilometres to the Arabian Sea The Sabarmatibasin extends over an area of 21674 sq km in Rajasthan and Gujarat The Mahi rises inthe east of Udaipur and drains an area of 34842 sq km lying in Madhya PradeshRajasthan and Gujarat It flows south-westwards for a distance of 533 km before it fallsinto the Gulf of Khambhat

The ChambalThe Chambal rises near Mhow in the Vindhya Range and flows towards the northgenerally in a gorge upto Kota Below Kota it turns to the north-east direction and afterreaching Pinahat it turns to the east and runs nearly parallel to the Yamuna beforejoining it in the southern part of the Etawah district in Uttar PradeshMajor Rivers of India with their basin area (Sqkm)

Himalayan System Indus 321290Ganga 861404

Brahmaputra 187110Indus System

Jhelum 34775Beas 20303

Ganga System Yamuna 366223Ghaghra 127950

Peninsular RiversNarmada 98796

Tapi 65145Mahanadi 141600

Subarnarekha 19300Sabarmati 21674

Mahi 34842Godavari 312812

Godavari Krishna Kaveri Mahanadi are the east-flowing rivers

Narmada Tapi west-flowing rivers

They fall into the Bay of Bengal

They fall into Arabian Sea

These rivers form big deltas

These rivers form comparativelysmall deltas

Catchment areas of these rivers are larger

Catchment areas of these rivers are smaller

Krishna 2589488Cauveri 87900

Subject ndashBiology Topic ndashChapter -5 Inheritance amp Variations Summary ExecutionToday we will discussabout linkage and its classification

LINKAGE The tendency of the genes located on the same chromosome to stay together is

hereditary transmission Linked genes the genes responsible for this Genes that exhibit the process of linkage locates in the same chromosome The distance between the linked genes in a chromosome determines the strength

of linkage i e genes that are located close to each other show stronger linkage than that are located far from each other

COMPLETE LINKAGE It is the type of linkage showed by the genes that are closely located or are tightly

linked with each other as they have no chance of separatingby crossing over These genes are always transmitted together to the same gamete and the same

offspring In such condition only parental or non cross over type of gametes are formedINCOMPLETE KINKAGE It is type of linkage showed by the genes that are distantly located orare loosely

linked with each other because they have chance of separating by crossing over

SIGNIFICANCE i) It helps in holding the parental character togetherii) It checks the appearance of new recombination and helps in bringing the

hybrid population which resembles the original parents iii) Linked genes dilute the effects of undesirable traits

Subject Eng Literature (The Tempest ndash William Shakespeare) Topic Essay Questions (EQ-3)Question No 3

Give a character sketch of CalibanAnswer

The character of Caliban has been wonderfully conceived by Shakespeare as the manifestation of all that is gross and earthy ndash a sort of creature of the earth as Ariel is a sort of creature of the air

Calibanrsquos Physical Appearanceo Caliban is lsquofreckledrsquo a lsquomisshapen knaversquo not honoured with human shape

o Prospero calls him lsquothou tortoisersquo (Act I Sc 2 Line 317) Trinculo stumbling upon him describes him as ldquoA strange fish hellip Legged like a man And his fins like armsrdquo He ldquosmells like a fishrdquo (Act II Sc 2 Line 25)

o Prospero also calls him a ldquobeastrdquo (Act IV Sc 1 Line 140) and ldquoThis misshapen knaverdquo (Act V Sc 1 Line 268)

o Further it appears that in addition to his physical deformity his spiritual inferiority is also suggested by Prosperorsquos claim that his birth resulted from the union between his mother the witch Sycorax and the devil

Calibanrsquos ParentageWhen the play opens Caliban is twenty four years of age having been born on the island twelve years before the coming of Prospero His mother was the foul witch Sycorax who was banished from Algiers for ldquomischiefs manifold and sorceries terrible to enter human hearingrdquo (Act I Sc 2 Line 264) and the father was the Devil himself Thus

Caliban is a monster of evil and brute nature ugly deformed and stinking

Calibanrsquos Savage and Malignant Natureo Caliban is entirely a creature of the earth ndash gross brutal and savage He regards himself as the rightful possessor

of the island and Prospero as a usurper

o In his young age he was on good terms with Prospero He had consented to be received by Prospero at his house and to be educated by him He has learnt human language only to curse his master whom he abhors

o His beastly nature soon breaks out and ends in a vicious attack on Miranda This opens the eye of Prospero who becomes severe to him and enforces his service by threats and violence

o Prospero uses him to make dams for fish to fetch firewood scraper trenches wash dishes and keep his cell clean

Calibanrsquos Hatred for ProsperoA profound hatred for Prospero has taken hold of Caliban It springs from a sense of his being dispossessed and ill-treated He would kill Prospero if he could but he knows the power of Prosperorsquos lsquobookrsquo Hence he transfers his allegiance to Stephano who seems like a god to him He also incites the two drunken associates to batter the skull of Prospero when he sleeps in the afternoon

Caliban Shows Considerable Intelligenceo He has learnt Prosperorsquos language

ldquoYou taught me language and my profit onrsquot (Act II Sc 2 Lines 86-89)Is I know how to curserdquo

o He is well aware of the futility of arguing with one who has more power than he has

ldquoI must obey his art is such power (Act I Sc 2 Lines 373-376)It would control my damrsquos god SetebosAnd make a vassal of himrdquo

o He realizes the importance of Prosperorsquos books

ldquoRemember (Act III Sc 2 Lines 89-92)First to possess his books for without themHersquos but a sot as I am nor hath notOne spirit to commandrdquo

o He knows the value of stealth when attacking the enemy

ldquoPray you tread softly that the blind mole may not (Act IV Sc 1 Lines 194-195)Hear a foot fall we now are near his cellrdquo

o Caliban has a better set of values than Stephano and Trinculo They are distracted from their plan by their greed for Prosperorsquos rich garments Only Caliban realizes that such a finery is unimportant

ldquoLeave it alone thou fool it is but trashrdquo (Act IV Sc 1 Lines 224)

Caliban is not a good judge of characterCaliban is not a good judge of character He decides for example that Stephano is a god because he dispenses lsquocelestial liquorrsquo (Act II Sc 2 Line 115) but then it must be remembered that he has only known his mother Sycorax Prospero Miranda and the spirits that torture him However he quickly discovers his error of judgementrdquo

ldquoWhat a thrice-double ass (Act V Sc 1 Lines 295-297)Was I to take this drunkard for a godAnd worship this dull foolrdquo

Calibanrsquos Imaginative NatureIf Caliban is sub-human in what has been said above he is human in the respect of the poetic side of his character He listens to music with rapture He tells of the beautiful dreams in which heaven rains treasures upon him and which upon waking he yearns to renew One of the most poetic passages in whole play is Calibanrsquos description of the island

to Stephano and Trinculo

ldquoBe not afeard The isle is full of noises (Act III Sc 2 Lines 135-143)Sounds and sweet airs that give delight and hurt notSometimes a thousand twangling instrumentsWill hum about mine ears and sometime voicesThat if I then had waked after long sleepWill make me sleep again and then in dreamingThe clouds methought would open and show richesReady to drop upon me that when I wakedI cried to dream againrdquo

Caliban - Less Ignoble Than Some OthersCalibanrsquos motive for murder is less dishonourable than that of Antonio and Sebastian They plan to kill Alonso to gain his power and wealth Caliban merely wants revenge and the return of lsquohisrsquo island

Conclusiono Calibanrsquos character is not portrayed very clearly in the play and hence we cannot decide whether he is a poor

savage being grossly maltreated by Prospero or whether he is evil and must therefore be kept in bondage or enslavement

o Caliban is contrasted with Ariel who is a spirit and thus swift and uninterested in physical activitieso Caliban is also contrasted with Prospero who is the all-powerful master of the island and of the destiny of all

those on the islando Caliban is also contrasted with civilized man showing him to be less evil than Antonio and Stephano and less

materialistic than Stephano and Trinculoo Caliban has suffered at the hands of Prospero and he has learnt to curse by listening to Prosperorsquos abuse He

certainly believes that Prospero has deprived him of his birthrighto Finally the character Caliban is thought to be one of Shakespearersquos masterpieces The complexity of the character

is reflected in the large volume of critical discussion that has grown around it

ECO ndash12 Topic-Forms of market

MonopolyMonopoly is a market structure in which there is a single seller there are no close substitutes for the commodity produced by the firm and there are barriers to entry Example Indian Railways which is operated under government of India Monopoly also implies absence of competitionFeatures of Monopoly Monopoly is characterized by1 Single Seller In monopoly there is only one firm producing the product The whole industry consists of this single firm Thus under monopoly there is no distinction between firm and industry Being the only firm there is significant control of the firm over supply and price Thus under monopoly buyers do not have the option of buying the commodity from any other seller They have to buy the product from the firm or they can go without the commodity This fact gives immense control to the monopolist over the market

2No Close Substitute There are no close substitutes of the product produced by the monopolist firm If there are close substitutes of the product in the market it implies presence of more than one firm and hence no monopoly In order to ensure a total of control over the market by the monopolist firm it is assumed that there are no close substitutes of the product

3 No Entry amp Exit Monopoly can only exist when there is strong barriers before a new firm to enter the market In fact once a monopoly firm starts producing the product no other firm can produce the same One reason for this is the ability of the

monopolist to produce the product at a lower cost than any new firm who thinks to enter the market If a new firm who knows that it cannot produce at a lower cost than the monopolist then that firm will never enter the market for fear of losing out in competition Similarly the monopolist who is operating for a long time may be enjoying reputation among its customers and is in a better position to use the situation in its own benefit A new firm has to take long time to achieve this and so may not be interested to enter the market

4 Price Maker Being the single seller of the product the monopolist has full control over the pricing of the product On the other hand if there is a large number of buyers in the market so no single buyer exercises any significant influence over price determination Thus it is a sellerrsquos market So monopoly firm is a price maker

5 Price Discrimination Having considerable control over the market on account of being single seller with no entry of other firms the monopolist can exercise policy of price discrimination it means that the monopolist can sell different quantities of the same product to a consumer at different price or same quantity to different consumers at different prices by adjudging the standard of living of the consumer

6 Shape of Demand Curve Since a monopolist has full control over the price therefore he can sell more by lowering the price This makes the demand curve downward sloping

Subject Ac-12 290620 Topic- retirement Model sumThe Balance Sheet of Rohit Nisha and Sunil who are partners in a firm sharing profits according to their capitals as on 31st March 2014 was as under

Liabilities Amount Assets Amount (Rs) (` Rs)

Creditors 25000 Machinery 40000Bills Payable 13000 Building 90000General Reserve 22000 Debtors 30000Capital Less Provision for Rohit 60000 Bad debts 1000

29000 Nisha 40000 Stocks 23000 Sunil 40000 140000 Cash at Bank 18000

200000 200000

On the date of Balance Sheet Nisha retired from the firm and following adjustments were made(i) Building is appreciated by 20(ii) Provision for bad debts is increased to 5 on Debtors(iii) Machinery is depreciated by 10(iv) Goodwill of the firm is valued at Rs 56000 and the retiring partnerrsquos share is adjusted

(v) The capital of the new firm is fixed at Rs120000 Prepare Revaluation Account Capital Accounts of the partner and Balance Sheet of the new firm after Nisharsquos retirement Revaluation AccountDr Cr

Particulars Amount Particulars Amount (`Rs) (Rs`)

Provision for Bad debt Ac 500 Building Ac 18000Machinery Ac 4000Profit transferred toCapital Accounts (3 2 2)Rohit 5786Nisha 3857Sunil 3857

13500

18000 18000

Capital Account

Dr Cr

Particulars Rohit Nisha Sunil Particulars Rohit Nisha Sunil (Rs`) (Rs`) (`Rs) (Rs`) (Rs`) (Rs`)

Sunilrsquos Capital ac 9600 mdash 6400 Balance bd 60000 40000 40000Bank - 66143 - General Reserve 9428 6286 6286Balance cd 72000 mdash 48000 Revaluation (Profi 5786 3857 3857 Rohitrsquos Capital Ac mdash 9600 mdash

Sunilrsquos Capital Ac 6400 Bank 6386 - 4257

81600 66143 54400 81600 66143 54400

Balance Sheet as at 31st March 2014

Liabilities Amount Assets Amount (Rs`) (Rs`)

Creditors 25000 Building 108000Bank overdraft 37500 Machinery 36000

Bills Payable 13000 Debtors 30000Capital Less ProvisionRohit 72000 for Bad debts 1500 28500Sunil 48000 120000 Stock 23000

195500 195500

Working Notes (i) (a) Profit sharing ratio is 60000 40000 40000 ie = 3 2 2(b) Gaining Ratio Rohit = 35 ndash 37 = 2135 ndash 1535 = 635Sunil = 25-27 = 1435 ndash 1035 = 435= 635 435= 6 4 = 3 2(c) Nisha Share of Goodwill = Rs 56000 times 27 = Rs16000Share of Goodwill in the gaining ratio by the existing partner ieRohit = Rs16000 times 35 = Rs 9600Sunil = Rs 16000 times 25 = Rs 6400

The journal entry isRohitrsquos Capital Ac Dr 9600Sunilrsquos Capital Ac Dr 6400 To Nisharsquos Capital Ac 16000(Share of Goodwill divided into gaining ratio)

  • 1 Static Friction
  • The frictional force that acts between the surfaces when they are at rest with respect to each other is called Static Friction
    • Static Friction Examples
      • 2 Sliding Friction
        • Examples Of Sliding Friction
          • 3 Rolling Friction
            • Examples Of Rolling Friction
              • Objects and Reasons of the Forest Conservation Act
Page 37:  · Web viewSubject . Topic . Summary . Execution . English 1 . Sounds of animals . Hens –cackle Horses –neigh Lions –roar Owls –hoots Snake –hiss. English 2 . Mother’s

This passage occurs in Act II Scene 7 in The Merchant of Venice This is part of the speech made by the Prince of Morocco

Explanation

In this passage the Prince of Morocco bestows high praise on Portia whose hand he is seeking He contrasts this casket containing Portiarsquos portrait with the old English gold coin bearing the image of the archangel (angel of the highest rank) He goes on to remark that while the figure of the archangel is engraved (Insculped) upon the English coin the picture of Portia who is beautiful as an angel lies hidden inside one of the caskets namely the Golden Casket (Golden Bed) In the day of Elizabeth silver was ten times inferior in value to gold Therefore the Prince of Morocco believing that Portiarsquos portrait is contained in the Golden Casket decides to choose the Golden Casket

Fare you well your suit is coldCold indeed and labour lostThen farewell heat and welcome frostmdashPortia adieu I have too grievd a heartTo take a tedious leave Thus losers part

(i) What reward does the Prince of Morocco get after making a wrong choice of the Casket How does he feel

After making the wrong choice in selecting the casket of gold the Prince of Morocco as a reward earns a rebuke in the form of a scroll tucked in the empty eye-socket of a skull kept in the casket of gold The Prince is shocked and disappointed He becomes all the more sad and dejected when he reads the scroll which points to his foolishness in being misled by the appearance and outward show as indicative of its worth

(ii) How does the Prince respond after reading the scroll

After reading the scroll the Prince though upset accepts the result with good grace and decorum befitting a royal suitor and true sportsman He says that his love-suit is really cold otherwise he would have chosen correctly but now his efforts have been in vain So he bids farewell to Portia to the warmth and enthusiasm of love and welcomes the cold and bitterness of dejection and misery of life which lies ahead

(iii) What request does he make to Portia and why

After being failure in his mission he requests Portia to give him permission to leave at once because he is too sad to undergo the tediousness of a formal leave-taking He tells that it is the manner in which defeated persons part unceremoniously

(iv) Explain the following lines

ldquoAll that glisters is not goldOften have you heard that toldMany a man his life hath soldBut my outside to beholdGilded tombs do worms infoldrdquo

Mere glitter does not make a metal to be gold Man has often been warned against appearance but it has been of no use Many people have sacrificed their lives only to seek the outer appearance of gold Worms are found inside the gilded

monuments

Class XSubject Topic Summary Execution

Hindi 2ndlang

नया रासता भाग 6 मायाराम 0ी घर म धनी मल 0ी और उनी बटी सरिरता ी ही चचा बनी रहती थी अमिमत ो इसम ोई रलिच ना थी वह धनी घर ी लडी स शादी र सवय ो बचना नही चाहता था उसा भी सवाणिभमान ह ईशवर ी पा

स उस पास पस ी ोई मी नही थी अभी उसन फकटरी ही लगाई थी उसी समझ बाहर था कि उस घर वालो ा झाव पस ी तरफ कयो

ह उसन मा स सवाल किया कि मा तम सरिरता स मरी शादी कयो रना चाहती हो मा न उस समझाया कि वह दखन म बरी नही ह और किफर खानदान अचछा

ह वह ए शल गरहणी रप म घर सभाल सगी अमिमत न मा ो इस बात ा एहसास राया कि मीन सबध लिलए मना रन पर उस दिदल

पर कया बीती होगी मा और अमिमत ी लडी बार म ाफी बात हईमा ा झाव सरिरता ी तरफ था कयोकि वह घर पर अचछा दह0 लर आ रही

थी अमिमत न अपनी मौसी ी बरी हालत बार म बताया कि किस तरह वह बड घर ी खानदानी बटी लाई थी और आ0 उसी हालत कितनी खराब ह लाई थी बहकलब 0ाती ह और बचचो ो भी नही दखती ह बात चल ही रही

थी कि तभी ए ार बाहर आर री धनी मल0ी घर अदर आए और पीछ स डराइवर फल ी ए टोरी लर आया अदर आए और पीछ स

डराइवर ए टोरी फल ी लर आया अमिमत ो फल ी पटी बरी लग रही थी अमिमत न पछ लिलया यह फल कयो ल आए ह प इन सब ी कया

0ररत थी उनो न 0वाब दिदया कि 4 पटी शमीर स मगाए थ अमिमत ो या सनर करोध आ गया तभी उस किपता 0ी आ गए उन आत ही अमिमत उठर बाहर चला गया वहा वहा मा पास आर बठ गया और बोला

अभी रिरशता तय नही हआ और धनी मल 0ी धनी मल 0ी फल ी पटी लर चलआय मा न समझाया कि 0ब सबध 0ड 0ाता ह तो खाली हाथ नही

आत अमिमत न मा स हा कि तम सबन सरिरता ो इस घर म लान ी ठान रखी ह धनीमल 0ी उस दिदन सरिरता ो दखन ी तारीख तय रन आय थ

Commercial Studies

Banking Nowadays Bank provide easy and quick services through internet facilities methods of Banking is called internet bankingIn order to save the time and money involved in visiting Bank branches people increasingly prefer to have internet banking

There are different modes of doing internet banking or transferring money through online They areReal Time Gross Settlement (RTGS)National Electronic Fund Transfers (NEFT)

1

Question

1) Explain the term RTGS Write the features of RTGS

Answer)The acronym RTGS stands for Real Time Gross Settlement which may be defined as the continuous real time settlement of funds transfer individually on and order by order basis without netting lsquoReal timersquo may be defined as the processing of instructions at the time they are received rather than at some letter time lsquoGross settlementrsquo may be defined as the settlement of transfer instructions which occurs

individually

Features of RTGS1It is the continuous settlement of

funds transfer individually on an order by order basis

2RTGS facility is provided only by CBS core banking solution enabled Bank branches

3Amount charged from the customer for RTGS transactions vary from bank to bank

2) Explain the term NEFT Write the features of NEFT

Answer) National electronic funds transfer may be defined as a nationwide system that facilitates individuals Farms and copper operates to electronically transfer funds from any bank branch to any individual farm or corporate having an account with any other bank branch in the country

Features of NEFT2 Transfer can be made 7 times on

weekdays and 6 times on Saturday

3 NEFT cannot be used to receive foreign remittances

4 NEFT transaction takes place in batches

5 A bank branch must be NEFT enabled to become a part of NEFT fund transfer network

6 There is no maximum or minimum amount that can be transferred through NEFT when one bank has a bank account

English Language

CompositionEssay

A composition is an art of creating a piece of writing on any topic or subject It is the writing correctly beautifully and clearly in order to make some interesting reading Structure of the composition

Introduction ( you lay the foundation for your composition)

Body (it constitutes the main part of the essay)

Conclusion (final statement that leaves a lasting impression)

Kinds of essays1 The Narrative essay2 The descriptive essay3 The reflective essay4 The argumentative essay

Write a composition on any one of the following topics (350- 400 words)

1 Friendship Or2 The first day of your school

Subject Eng Literature (The Merchant of Venice ndash William Shakespeare)Topic Act V Scene 1 Lines 127 to 158 (Nerissa helliphellip The clerk will nersquoer wear hair onrsquos face that had it) [Students should read the original play and also the paraphrase given in the school prescribed textbook]

Summary Revision Questions o Soon thereafter Bassanio Gratiano

and Antonio arrive

o Bassanio tells Portia that he is feeling as if it is morning because of the presence of Portia who is shining like the sun When Antonio is introduced by Bassanio to Portia she tells Bassanio that he should be grateful to Antonio who took so much trouble on his account even to the extent of risking his life

o Nerissa starts quarrelling with Gratiano and demands that he show her the ring she had presented to him and which she had warned him not to lose She suspects that Gratiano must have presented the ring to some young woman and not to the lawyerrsquos clerk as he repeatedly says and assures

Answer the following questions to check your preparation of Act IV Scenes 1 and 2

You must attempt only after you have completed your preparation of Act IV The answers must be in complete sentences using textual evidence (with citation) when necessary

[It would be in your own interest to attempt the above questions honestly totally refraining from consulting your textbook or your notes during answering After completion you should correct the paper yourself consulting the textbooknotes etc and award marks as specified Please let me know the marks you scored through WhatsApp in the group or to my personal WhatsApp]

Act IV Scene 1 (each question carries 2 marks)

1 What did the Duke try to do for Antonio

2 Why does Shylock refuse to show mercy How does he justify his stance

3 Why does Antonio say he is ready to die 4 What information is contained in Bellariorsquos letter

5 Why does Portia (as Balthazar) assert that Shylock must show mercy How does he respond

6 What offers are made to Shylock to get him to spare Antonio How are they received

7 What does Antoniorsquos speech as he faces the prospect of Shylockrsquos knife tell you about his character

8 How do Bassanio and Gratiano react to the looming prospect of Antoniorsquos demise

9 How does Portia (as Balthazar) use the law to turn the tables on Shylock

10 What does the Duke decree should happen to Shylock Why What happens to Shylockrsquos estate

11 What does Portia ask Bassanio as payment for her ldquoservicesrdquo What is his initial response What makes him change his mind

Act IV Scene 2 (each question carries 1frac12 marks)

1 What does Gratiano bring to Portia (Balthazar)

2 What does Nerissa plan on getting from Gratiano What does Portiarsquos comment suggest about men

ECO-10 280620 Topic-Supply AnalysisSHIFTING OF SUPPLY

But if there is change in factors other than the price of the commodity then either more is supplied at the same price or less supplied at the same price In such cases the price of the commodity remains constant but there is a change in other factors like change in the price of inputs change in technology of production change in price of other related goods change in taxation policy of the government etc For example there is an improvement in the technology of production of the commodity in question It leads to decrease in per unit of cost production of the commodity The firm is willing to sell more quantity of the commodity at the same price So the supply other commodity increases at the same price This increase in supply is shown by rightward shift of supply curve On the other hand if the firm uses inferior technology of production the cost of production per unit of the commodity increases The firm is willing to sell less quantity at the same price So the supply of the commodity decreases at the same price This decrease in supply is shown by leftward shift of the supply curve The above cases of increase and decrease in supply can be shown with the help of the following figures

Y INCREASE OF SUPPLY Price (Rs) s

P A s1

B

s

X` O s1 X

q q1

Y` Quantity demanded (in units)

Y DECREASE IN SUPPLY s2

s

price (Rs)

C

p A

s2

s

X` o X

q2 q

Y` Quantity demanded ( in units)

Main factors causing increase in supply or rightward shift of supply Curve(i) Fall in the price of other related goods

(ii) Fall in the price of inputsfactors(iii) Use of better technology in production(iv) Decrease in the rate of excise duty by government(v) If the objective of producer changes from profit maximization to salesMaximization

Main factors causing decrease in supply or leftward shift of supply curve(i) Increase in the price of other related goods(ii) Rise in the price of inputsfactors(iii) Use of inferior technology in production(iv) Increase in the rate of excise duty by the government(v) If the objective

Subject - Biology Topic ndash Chapter mdash6 PhotosynthesisSummary Execution

Today we will know about photosynthesis and its stages

Q1 What do you mean by photosynthesis The process by which living plants containing chlorophyll produce food

substances from carbon-di- oxide and water by using light energy Sunlight

6CO2 +12 H2O----------------------- C6 H12O6 + 6H2O + 6O2

Chlorophyll

Q2 What are the importance of photosynthesis I) Food for all Green plants trap solar energy by photosynthesis

process and supply food and energy for all living organisms either directly or indirectly

Ii) Oxygen to breathe in by product of photosynthesis is oxygen which is essential for all living organisms respiration

Q3 Write about two main phases of photosynthesis A Light dependent phase This phase occur in grana of chloroplast I) The chlorophyll on exposure to light energy becomes activated by

absorbing photons Ii) The absorbed energy is used in splitting the water molecules (H2O)

into its two components (H+ and OH- ) and releasing electron s 2H2O------------------------- 4H+ + 4e- +O2

Energy of 4 photons This reaction is known as photolysis

End products are H+ and oxygen water

B Light independent (Dark ) phase The reactions in this phase require no light energy

Here CO2 combine with H+ and produce glucose

Class XI

Subject Topic Summary ExecutionEVS Chapter-4 Legal

regimes for sustainable development

Environmental legislationEnvironmental legislation is the collection of laws and regulations pertaining to air quality water quality the wilderness endangered wildlife and other environmental factors The act ensures that matters important to the environment are thoroughly

Learn -The Forest (Conservation) Act 1980

considered in any decisions made by federal agencies

The Forest (Conservation) Act 1980 The Forest (Conservation) Act 1980 an Act of the Parliament of India to provide for the conservation of forests and for matters connected therewith or ancillary or incidental thereto It was further amended in 1988 This law extends to the whole of IndiaObjects and Reasons of the Forest Conservation Act

Deforestation causes ecological imbalance and leads to environmental deterioration Deforestation had been taking place on a large scale in the country and it had caused widespread concern The act seeks to check upon deforestation and de-reservation of forests

Subject Eng Literature (The Tempest ndash William Shakespeare) Topic Act II Scene 1 Lines 314 to 329 (End of scene)

[Students should read the original play and also the paraphrase given in the school prescribed textbook]Summary Questions amp Answers

Conspiracy of Antonio and Sebastian (Contd)

o As they approach Ariel appears again and wakes up Gonzalo by singing a tune in his ear Alonso also wakes up and they see both Sebastian and Antonio with drawn swords On being caught off guard they make up a story saying that they had heard a bellowing of bulls or lions

o They then moved to another part of the island

o Ariel at once rushes to Prospero to inform him of this development

SUMMING-UP of ACT-2 SCENE-1

(i) Among the survivors Ferdinand is separated from the rest which results in the disconsolate grief of Alonso as he took him for dead

(ii) The villainy of Antonio is confirmed

(iii) The supremacy of Prosperorsquos magic which resulted in the failure of the human conspiracy

(1)

(Act II Sc 1 L 311-325)SEBASTIAN Whiles we stood here securing your repose

Even now we heard a hollow burst of bellowing Like bulls or rather lions Didt not wake youIt struck mine ear most terribly

ALONSO I heard nothingANTONIO O rsquotwas a din to fright a monsters ear

To make an earthquake Sure it was the roarOf a whole herd of lions

ALONSO Heard you this GonzaloGONZALO Upon mine honour sir I heard a humming

And that a strange one too which did awake meI shaked you sir and cried As mine eyes opened I saw their weapons drawn There was a noiseThats verily rsquoTis best we stand upon our guardOr that we quit this place Lets draw our weapons

(i) Why has Prospero sent Ariel to Gonzalo and Alonso What does Ariel do to awaken Gonzalo

Prospero has already come to know by his magic powers the danger which threatens Gonzalo who had been Prosperorsquos friend and so he sent Ariel to preserve the lives of both Gonzalo and Alonso Prospero does not want that his scheme should remain unfulfilled Ariel begins to sing a song in Gonzalorsquos ears to awaken him(ii) Who are ready to carry out their plan Who takes steps to stop them Why does Gonzalo feel surprised after being awakened

Sebastian and Antonio are ready to carry out their plans They are standing with their swords drawn to kill Alonso and

(iv) We see two sets of contrasting characters Gonzalo-Adrian against Antonio-Sebastian

(v) The grief that works in Alonso can be perceived to his repentance for his association in Antoniorsquos crime against Prospero

Gonzalo Ariel takes steps to stop them from carrying out their nefarious scheme When Gonzalo is awakened by the song sung by Ariel into his ears he (Gonzalo) feels surprised because he sees Sebastian and Antonio standing with their swords drawn(iii) What reason do Sebastian and Antonio tell of drawing their swords when they are suspected by Alonso and Gonzalo

When Sebastian and Antonio are seen with their swords drawn they are looked with suspicion by Gonzalo and Alonso At first Sebastian tells them that as they stood here to guard them during their sleep they heard only a little before a sudden loud noise very much like the roaring of bulls or more probably that of lions Then Antonio follows him saying that this was a noise so terrible as to frighten even a monsterrsquos ears and this noise could even have shaken the earth and it was surely like the roaring of a multitude of lions Then seeing the danger they have drawn their swords Perhaps after hearing the terrible noise they (Gonzalo and Alonso) woke up from their sound sleep

(iv) What does Gonzalo tell Alonso about the strange noise What did he see on opening his eyes Gonzalo tells Alonso that he did not hear the sound of roaring but he heard a humming sound which was strange and which woke him up After waking up he gave him (Alonso) a shaking and a loud cry On opening his eyes he saw these two gentlemen standing with their swords drawn(v) What does Gonzalo suggest

Gonzalo suggests that there was a noise indeed and of that he has no doubt at all and suggests that the best course for them would be to remain alert and vigilant against any possible danger to their lives or to leave this place and move to some other part of the island

Class XIISubject Topic Summary Execution

Commerce

Chapter- Management

Today we will discuss about LEVELS OF MANAGEMENT

Levels of management is a series or chain of managerial positions from top to bottom It helps individuals to know their authority responsibilities and superior-subordinate relations among themselves There are mainly three levels of Management TOP LEVEL MANAGEMENTMIDDLE LEVEL MANAGEMENTLOWER LEVEL MANAGEMENT

Top level managementIt consists of members at the highest level in the management hierarchy This level includes Board Of Directors Chief Executive Managing Directors Chairman President Vice President

Rolefunctions of the top levelmanagement1To analyse evaluate and deal

with theexternal environment2 To determine the objectives and

policies of the business3 To strive for welfare and survival

of business

4 To create an organisational Framework consisting of authority responsibility relationship

Middle level management Congress of members or groups who are concerned with implementation of the policies let down by the top managementThis level includes head of the department such as finance manager marketing manager branch and regional managers departmental and divisional heads plant superintendent etc

Role of functions of the middle level management

1 To interpret the policies framed by top management

2 To assign duties and responsibilities to lower level managers

3 To select and appoint employees for middle and supervisory level and evaluate their performance

4 To co-operate with other departments for smooth functioning

Operational or supervisory level managementIt refers to the group are members who are concerned with execution of the work They are also known as fast line managers This level includes supervisor 4 men Section Officer clerk Inspector etc

Role of functions of the lower level management1 To plan and execute day-to-

day operations2 To supervise and control the workers3 To arrange materials and

tools to start the process and make arrangements for training

4 Today present workers grievance and suggestions before the management and

ensure safe and proper working conditions in the factory

Business Studies

Staff Appraisal Chapter- 10 Today let us start with a new chapter

Staff Appraisal

Meaning of Performance Appraisal

Performance Appraisal is the systematic evaluation of the performance of employees and to understand the abilities of a person for further growth and developmentThe supervisors measure the pay of employees and compare it with targets and plansThe supervisor analyses the factors behind work performances of employeesThe employers are in position to guide the employees for a better performance

Objectives of Performance Appraisal

Following are the objectives of Performance Appraisal

To maintain records in order to determine compensation packages wage structure salaries raises etc

To identify the strengths and weaknesses of employees to place right men on right job

To maintain and assess the potential present in a person for further growth and development

To provide a feedback to employees regarding their performance and related status

To provide a feedback to employees regarding their performance and related status

Importance of Performance Appraisal

Performance appraisal provides important and useful information for the assessment of employees skill

knowledge ability and overall job performance The following are the points which indicate the importance of performance appraisal in an organization

1 Performance appraisal helps supervisors to assess the work performance of their subordinates

2 Performance appraisal helps to assess the training and development needs of employees

3 Performance appraisal provides grounds for employees to correct their mistakes and it also provides proper guidance and criticism for employees development4 Performance appraisal provides reward for better performance

5 Performance appraisal helps to improve the communication system of the organization

6 Performance appraisal evaluates whether human resource programs being implemented in the organization have been effective

7 Performance appraisal helps to prepare pay structure for each employee working in the organization

8 Performance appraisal helps to review the potentiality of employees so that their future capability is anticipated

Geography

DRIANAGE The SubarnarekhaThe Subarnarekha and the Brahmaniinterposed between the Ganga and the Mahanadi deltas drain an area of 19300 sq kmand 39033 sq km respectively The drainage basins of these streams are shared byJharkhand Odisha west Bengal and Chhattisgarh The Brahmani is known as southKoel in its upper reaches in Jharkhand

The NarmadaThe Narmada rises in the Amarkantak hills of MadhyaPradesh It flows towards the West in a rift valleyformed due to a geological fault The total length of it is 1300 km All the tributaries of the

Q1 Name the two westward flowing rivers in the peninsular plateauA1 Narmada and Tapi are the only westward flowing rivers of the peninsular plateau

Q2 Differentiate between east-flowing rivers and west-flowing riversA2

East-flowing rivers

West-flowing rivers

Narmada are very short inlength Most of its tributaries join the main streamright anglesThe Narmada basin covers parts of Madhya Pradesh and Gujarat

The Tapi The Tapi rises in the Satpura ranges in the Betul listrictof Madhya Pradesh It flows in a rift valley parallel tothe Narmada but it is much shorter in length It coversparts of Madhya Pradesh Gujarat and MaharashtraThe length is about 724 km

The Sabarmati and the MahiThe Sabarmati rises in the Aravali hills and flows south-south-westwards for a distance of 300 kilometres to the Arabian Sea The Sabarmatibasin extends over an area of 21674 sq km in Rajasthan and Gujarat The Mahi rises inthe east of Udaipur and drains an area of 34842 sq km lying in Madhya PradeshRajasthan and Gujarat It flows south-westwards for a distance of 533 km before it fallsinto the Gulf of Khambhat

The ChambalThe Chambal rises near Mhow in the Vindhya Range and flows towards the northgenerally in a gorge upto Kota Below Kota it turns to the north-east direction and afterreaching Pinahat it turns to the east and runs nearly parallel to the Yamuna beforejoining it in the southern part of the Etawah district in Uttar PradeshMajor Rivers of India with their basin area (Sqkm)

Himalayan System Indus 321290Ganga 861404

Brahmaputra 187110Indus System

Jhelum 34775Beas 20303

Ganga System Yamuna 366223Ghaghra 127950

Peninsular RiversNarmada 98796

Tapi 65145Mahanadi 141600

Subarnarekha 19300Sabarmati 21674

Mahi 34842Godavari 312812

Godavari Krishna Kaveri Mahanadi are the east-flowing rivers

Narmada Tapi west-flowing rivers

They fall into the Bay of Bengal

They fall into Arabian Sea

These rivers form big deltas

These rivers form comparativelysmall deltas

Catchment areas of these rivers are larger

Catchment areas of these rivers are smaller

Krishna 2589488Cauveri 87900

Subject ndashBiology Topic ndashChapter -5 Inheritance amp Variations Summary ExecutionToday we will discussabout linkage and its classification

LINKAGE The tendency of the genes located on the same chromosome to stay together is

hereditary transmission Linked genes the genes responsible for this Genes that exhibit the process of linkage locates in the same chromosome The distance between the linked genes in a chromosome determines the strength

of linkage i e genes that are located close to each other show stronger linkage than that are located far from each other

COMPLETE LINKAGE It is the type of linkage showed by the genes that are closely located or are tightly

linked with each other as they have no chance of separatingby crossing over These genes are always transmitted together to the same gamete and the same

offspring In such condition only parental or non cross over type of gametes are formedINCOMPLETE KINKAGE It is type of linkage showed by the genes that are distantly located orare loosely

linked with each other because they have chance of separating by crossing over

SIGNIFICANCE i) It helps in holding the parental character togetherii) It checks the appearance of new recombination and helps in bringing the

hybrid population which resembles the original parents iii) Linked genes dilute the effects of undesirable traits

Subject Eng Literature (The Tempest ndash William Shakespeare) Topic Essay Questions (EQ-3)Question No 3

Give a character sketch of CalibanAnswer

The character of Caliban has been wonderfully conceived by Shakespeare as the manifestation of all that is gross and earthy ndash a sort of creature of the earth as Ariel is a sort of creature of the air

Calibanrsquos Physical Appearanceo Caliban is lsquofreckledrsquo a lsquomisshapen knaversquo not honoured with human shape

o Prospero calls him lsquothou tortoisersquo (Act I Sc 2 Line 317) Trinculo stumbling upon him describes him as ldquoA strange fish hellip Legged like a man And his fins like armsrdquo He ldquosmells like a fishrdquo (Act II Sc 2 Line 25)

o Prospero also calls him a ldquobeastrdquo (Act IV Sc 1 Line 140) and ldquoThis misshapen knaverdquo (Act V Sc 1 Line 268)

o Further it appears that in addition to his physical deformity his spiritual inferiority is also suggested by Prosperorsquos claim that his birth resulted from the union between his mother the witch Sycorax and the devil

Calibanrsquos ParentageWhen the play opens Caliban is twenty four years of age having been born on the island twelve years before the coming of Prospero His mother was the foul witch Sycorax who was banished from Algiers for ldquomischiefs manifold and sorceries terrible to enter human hearingrdquo (Act I Sc 2 Line 264) and the father was the Devil himself Thus

Caliban is a monster of evil and brute nature ugly deformed and stinking

Calibanrsquos Savage and Malignant Natureo Caliban is entirely a creature of the earth ndash gross brutal and savage He regards himself as the rightful possessor

of the island and Prospero as a usurper

o In his young age he was on good terms with Prospero He had consented to be received by Prospero at his house and to be educated by him He has learnt human language only to curse his master whom he abhors

o His beastly nature soon breaks out and ends in a vicious attack on Miranda This opens the eye of Prospero who becomes severe to him and enforces his service by threats and violence

o Prospero uses him to make dams for fish to fetch firewood scraper trenches wash dishes and keep his cell clean

Calibanrsquos Hatred for ProsperoA profound hatred for Prospero has taken hold of Caliban It springs from a sense of his being dispossessed and ill-treated He would kill Prospero if he could but he knows the power of Prosperorsquos lsquobookrsquo Hence he transfers his allegiance to Stephano who seems like a god to him He also incites the two drunken associates to batter the skull of Prospero when he sleeps in the afternoon

Caliban Shows Considerable Intelligenceo He has learnt Prosperorsquos language

ldquoYou taught me language and my profit onrsquot (Act II Sc 2 Lines 86-89)Is I know how to curserdquo

o He is well aware of the futility of arguing with one who has more power than he has

ldquoI must obey his art is such power (Act I Sc 2 Lines 373-376)It would control my damrsquos god SetebosAnd make a vassal of himrdquo

o He realizes the importance of Prosperorsquos books

ldquoRemember (Act III Sc 2 Lines 89-92)First to possess his books for without themHersquos but a sot as I am nor hath notOne spirit to commandrdquo

o He knows the value of stealth when attacking the enemy

ldquoPray you tread softly that the blind mole may not (Act IV Sc 1 Lines 194-195)Hear a foot fall we now are near his cellrdquo

o Caliban has a better set of values than Stephano and Trinculo They are distracted from their plan by their greed for Prosperorsquos rich garments Only Caliban realizes that such a finery is unimportant

ldquoLeave it alone thou fool it is but trashrdquo (Act IV Sc 1 Lines 224)

Caliban is not a good judge of characterCaliban is not a good judge of character He decides for example that Stephano is a god because he dispenses lsquocelestial liquorrsquo (Act II Sc 2 Line 115) but then it must be remembered that he has only known his mother Sycorax Prospero Miranda and the spirits that torture him However he quickly discovers his error of judgementrdquo

ldquoWhat a thrice-double ass (Act V Sc 1 Lines 295-297)Was I to take this drunkard for a godAnd worship this dull foolrdquo

Calibanrsquos Imaginative NatureIf Caliban is sub-human in what has been said above he is human in the respect of the poetic side of his character He listens to music with rapture He tells of the beautiful dreams in which heaven rains treasures upon him and which upon waking he yearns to renew One of the most poetic passages in whole play is Calibanrsquos description of the island

to Stephano and Trinculo

ldquoBe not afeard The isle is full of noises (Act III Sc 2 Lines 135-143)Sounds and sweet airs that give delight and hurt notSometimes a thousand twangling instrumentsWill hum about mine ears and sometime voicesThat if I then had waked after long sleepWill make me sleep again and then in dreamingThe clouds methought would open and show richesReady to drop upon me that when I wakedI cried to dream againrdquo

Caliban - Less Ignoble Than Some OthersCalibanrsquos motive for murder is less dishonourable than that of Antonio and Sebastian They plan to kill Alonso to gain his power and wealth Caliban merely wants revenge and the return of lsquohisrsquo island

Conclusiono Calibanrsquos character is not portrayed very clearly in the play and hence we cannot decide whether he is a poor

savage being grossly maltreated by Prospero or whether he is evil and must therefore be kept in bondage or enslavement

o Caliban is contrasted with Ariel who is a spirit and thus swift and uninterested in physical activitieso Caliban is also contrasted with Prospero who is the all-powerful master of the island and of the destiny of all

those on the islando Caliban is also contrasted with civilized man showing him to be less evil than Antonio and Stephano and less

materialistic than Stephano and Trinculoo Caliban has suffered at the hands of Prospero and he has learnt to curse by listening to Prosperorsquos abuse He

certainly believes that Prospero has deprived him of his birthrighto Finally the character Caliban is thought to be one of Shakespearersquos masterpieces The complexity of the character

is reflected in the large volume of critical discussion that has grown around it

ECO ndash12 Topic-Forms of market

MonopolyMonopoly is a market structure in which there is a single seller there are no close substitutes for the commodity produced by the firm and there are barriers to entry Example Indian Railways which is operated under government of India Monopoly also implies absence of competitionFeatures of Monopoly Monopoly is characterized by1 Single Seller In monopoly there is only one firm producing the product The whole industry consists of this single firm Thus under monopoly there is no distinction between firm and industry Being the only firm there is significant control of the firm over supply and price Thus under monopoly buyers do not have the option of buying the commodity from any other seller They have to buy the product from the firm or they can go without the commodity This fact gives immense control to the monopolist over the market

2No Close Substitute There are no close substitutes of the product produced by the monopolist firm If there are close substitutes of the product in the market it implies presence of more than one firm and hence no monopoly In order to ensure a total of control over the market by the monopolist firm it is assumed that there are no close substitutes of the product

3 No Entry amp Exit Monopoly can only exist when there is strong barriers before a new firm to enter the market In fact once a monopoly firm starts producing the product no other firm can produce the same One reason for this is the ability of the

monopolist to produce the product at a lower cost than any new firm who thinks to enter the market If a new firm who knows that it cannot produce at a lower cost than the monopolist then that firm will never enter the market for fear of losing out in competition Similarly the monopolist who is operating for a long time may be enjoying reputation among its customers and is in a better position to use the situation in its own benefit A new firm has to take long time to achieve this and so may not be interested to enter the market

4 Price Maker Being the single seller of the product the monopolist has full control over the pricing of the product On the other hand if there is a large number of buyers in the market so no single buyer exercises any significant influence over price determination Thus it is a sellerrsquos market So monopoly firm is a price maker

5 Price Discrimination Having considerable control over the market on account of being single seller with no entry of other firms the monopolist can exercise policy of price discrimination it means that the monopolist can sell different quantities of the same product to a consumer at different price or same quantity to different consumers at different prices by adjudging the standard of living of the consumer

6 Shape of Demand Curve Since a monopolist has full control over the price therefore he can sell more by lowering the price This makes the demand curve downward sloping

Subject Ac-12 290620 Topic- retirement Model sumThe Balance Sheet of Rohit Nisha and Sunil who are partners in a firm sharing profits according to their capitals as on 31st March 2014 was as under

Liabilities Amount Assets Amount (Rs) (` Rs)

Creditors 25000 Machinery 40000Bills Payable 13000 Building 90000General Reserve 22000 Debtors 30000Capital Less Provision for Rohit 60000 Bad debts 1000

29000 Nisha 40000 Stocks 23000 Sunil 40000 140000 Cash at Bank 18000

200000 200000

On the date of Balance Sheet Nisha retired from the firm and following adjustments were made(i) Building is appreciated by 20(ii) Provision for bad debts is increased to 5 on Debtors(iii) Machinery is depreciated by 10(iv) Goodwill of the firm is valued at Rs 56000 and the retiring partnerrsquos share is adjusted

(v) The capital of the new firm is fixed at Rs120000 Prepare Revaluation Account Capital Accounts of the partner and Balance Sheet of the new firm after Nisharsquos retirement Revaluation AccountDr Cr

Particulars Amount Particulars Amount (`Rs) (Rs`)

Provision for Bad debt Ac 500 Building Ac 18000Machinery Ac 4000Profit transferred toCapital Accounts (3 2 2)Rohit 5786Nisha 3857Sunil 3857

13500

18000 18000

Capital Account

Dr Cr

Particulars Rohit Nisha Sunil Particulars Rohit Nisha Sunil (Rs`) (Rs`) (`Rs) (Rs`) (Rs`) (Rs`)

Sunilrsquos Capital ac 9600 mdash 6400 Balance bd 60000 40000 40000Bank - 66143 - General Reserve 9428 6286 6286Balance cd 72000 mdash 48000 Revaluation (Profi 5786 3857 3857 Rohitrsquos Capital Ac mdash 9600 mdash

Sunilrsquos Capital Ac 6400 Bank 6386 - 4257

81600 66143 54400 81600 66143 54400

Balance Sheet as at 31st March 2014

Liabilities Amount Assets Amount (Rs`) (Rs`)

Creditors 25000 Building 108000Bank overdraft 37500 Machinery 36000

Bills Payable 13000 Debtors 30000Capital Less ProvisionRohit 72000 for Bad debts 1500 28500Sunil 48000 120000 Stock 23000

195500 195500

Working Notes (i) (a) Profit sharing ratio is 60000 40000 40000 ie = 3 2 2(b) Gaining Ratio Rohit = 35 ndash 37 = 2135 ndash 1535 = 635Sunil = 25-27 = 1435 ndash 1035 = 435= 635 435= 6 4 = 3 2(c) Nisha Share of Goodwill = Rs 56000 times 27 = Rs16000Share of Goodwill in the gaining ratio by the existing partner ieRohit = Rs16000 times 35 = Rs 9600Sunil = Rs 16000 times 25 = Rs 6400

The journal entry isRohitrsquos Capital Ac Dr 9600Sunilrsquos Capital Ac Dr 6400 To Nisharsquos Capital Ac 16000(Share of Goodwill divided into gaining ratio)

  • 1 Static Friction
  • The frictional force that acts between the surfaces when they are at rest with respect to each other is called Static Friction
    • Static Friction Examples
      • 2 Sliding Friction
        • Examples Of Sliding Friction
          • 3 Rolling Friction
            • Examples Of Rolling Friction
              • Objects and Reasons of the Forest Conservation Act
Page 38:  · Web viewSubject . Topic . Summary . Execution . English 1 . Sounds of animals . Hens –cackle Horses –neigh Lions –roar Owls –hoots Snake –hiss. English 2 . Mother’s

monuments

Class XSubject Topic Summary Execution

Hindi 2ndlang

नया रासता भाग 6 मायाराम 0ी घर म धनी मल 0ी और उनी बटी सरिरता ी ही चचा बनी रहती थी अमिमत ो इसम ोई रलिच ना थी वह धनी घर ी लडी स शादी र सवय ो बचना नही चाहता था उसा भी सवाणिभमान ह ईशवर ी पा

स उस पास पस ी ोई मी नही थी अभी उसन फकटरी ही लगाई थी उसी समझ बाहर था कि उस घर वालो ा झाव पस ी तरफ कयो

ह उसन मा स सवाल किया कि मा तम सरिरता स मरी शादी कयो रना चाहती हो मा न उस समझाया कि वह दखन म बरी नही ह और किफर खानदान अचछा

ह वह ए शल गरहणी रप म घर सभाल सगी अमिमत न मा ो इस बात ा एहसास राया कि मीन सबध लिलए मना रन पर उस दिदल

पर कया बीती होगी मा और अमिमत ी लडी बार म ाफी बात हईमा ा झाव सरिरता ी तरफ था कयोकि वह घर पर अचछा दह0 लर आ रही

थी अमिमत न अपनी मौसी ी बरी हालत बार म बताया कि किस तरह वह बड घर ी खानदानी बटी लाई थी और आ0 उसी हालत कितनी खराब ह लाई थी बहकलब 0ाती ह और बचचो ो भी नही दखती ह बात चल ही रही

थी कि तभी ए ार बाहर आर री धनी मल0ी घर अदर आए और पीछ स डराइवर फल ी ए टोरी लर आया अदर आए और पीछ स

डराइवर ए टोरी फल ी लर आया अमिमत ो फल ी पटी बरी लग रही थी अमिमत न पछ लिलया यह फल कयो ल आए ह प इन सब ी कया

0ररत थी उनो न 0वाब दिदया कि 4 पटी शमीर स मगाए थ अमिमत ो या सनर करोध आ गया तभी उस किपता 0ी आ गए उन आत ही अमिमत उठर बाहर चला गया वहा वहा मा पास आर बठ गया और बोला

अभी रिरशता तय नही हआ और धनी मल 0ी धनी मल 0ी फल ी पटी लर चलआय मा न समझाया कि 0ब सबध 0ड 0ाता ह तो खाली हाथ नही

आत अमिमत न मा स हा कि तम सबन सरिरता ो इस घर म लान ी ठान रखी ह धनीमल 0ी उस दिदन सरिरता ो दखन ी तारीख तय रन आय थ

Commercial Studies

Banking Nowadays Bank provide easy and quick services through internet facilities methods of Banking is called internet bankingIn order to save the time and money involved in visiting Bank branches people increasingly prefer to have internet banking

There are different modes of doing internet banking or transferring money through online They areReal Time Gross Settlement (RTGS)National Electronic Fund Transfers (NEFT)

1

Question

1) Explain the term RTGS Write the features of RTGS

Answer)The acronym RTGS stands for Real Time Gross Settlement which may be defined as the continuous real time settlement of funds transfer individually on and order by order basis without netting lsquoReal timersquo may be defined as the processing of instructions at the time they are received rather than at some letter time lsquoGross settlementrsquo may be defined as the settlement of transfer instructions which occurs

individually

Features of RTGS1It is the continuous settlement of

funds transfer individually on an order by order basis

2RTGS facility is provided only by CBS core banking solution enabled Bank branches

3Amount charged from the customer for RTGS transactions vary from bank to bank

2) Explain the term NEFT Write the features of NEFT

Answer) National electronic funds transfer may be defined as a nationwide system that facilitates individuals Farms and copper operates to electronically transfer funds from any bank branch to any individual farm or corporate having an account with any other bank branch in the country

Features of NEFT2 Transfer can be made 7 times on

weekdays and 6 times on Saturday

3 NEFT cannot be used to receive foreign remittances

4 NEFT transaction takes place in batches

5 A bank branch must be NEFT enabled to become a part of NEFT fund transfer network

6 There is no maximum or minimum amount that can be transferred through NEFT when one bank has a bank account

English Language

CompositionEssay

A composition is an art of creating a piece of writing on any topic or subject It is the writing correctly beautifully and clearly in order to make some interesting reading Structure of the composition

Introduction ( you lay the foundation for your composition)

Body (it constitutes the main part of the essay)

Conclusion (final statement that leaves a lasting impression)

Kinds of essays1 The Narrative essay2 The descriptive essay3 The reflective essay4 The argumentative essay

Write a composition on any one of the following topics (350- 400 words)

1 Friendship Or2 The first day of your school

Subject Eng Literature (The Merchant of Venice ndash William Shakespeare)Topic Act V Scene 1 Lines 127 to 158 (Nerissa helliphellip The clerk will nersquoer wear hair onrsquos face that had it) [Students should read the original play and also the paraphrase given in the school prescribed textbook]

Summary Revision Questions o Soon thereafter Bassanio Gratiano

and Antonio arrive

o Bassanio tells Portia that he is feeling as if it is morning because of the presence of Portia who is shining like the sun When Antonio is introduced by Bassanio to Portia she tells Bassanio that he should be grateful to Antonio who took so much trouble on his account even to the extent of risking his life

o Nerissa starts quarrelling with Gratiano and demands that he show her the ring she had presented to him and which she had warned him not to lose She suspects that Gratiano must have presented the ring to some young woman and not to the lawyerrsquos clerk as he repeatedly says and assures

Answer the following questions to check your preparation of Act IV Scenes 1 and 2

You must attempt only after you have completed your preparation of Act IV The answers must be in complete sentences using textual evidence (with citation) when necessary

[It would be in your own interest to attempt the above questions honestly totally refraining from consulting your textbook or your notes during answering After completion you should correct the paper yourself consulting the textbooknotes etc and award marks as specified Please let me know the marks you scored through WhatsApp in the group or to my personal WhatsApp]

Act IV Scene 1 (each question carries 2 marks)

1 What did the Duke try to do for Antonio

2 Why does Shylock refuse to show mercy How does he justify his stance

3 Why does Antonio say he is ready to die 4 What information is contained in Bellariorsquos letter

5 Why does Portia (as Balthazar) assert that Shylock must show mercy How does he respond

6 What offers are made to Shylock to get him to spare Antonio How are they received

7 What does Antoniorsquos speech as he faces the prospect of Shylockrsquos knife tell you about his character

8 How do Bassanio and Gratiano react to the looming prospect of Antoniorsquos demise

9 How does Portia (as Balthazar) use the law to turn the tables on Shylock

10 What does the Duke decree should happen to Shylock Why What happens to Shylockrsquos estate

11 What does Portia ask Bassanio as payment for her ldquoservicesrdquo What is his initial response What makes him change his mind

Act IV Scene 2 (each question carries 1frac12 marks)

1 What does Gratiano bring to Portia (Balthazar)

2 What does Nerissa plan on getting from Gratiano What does Portiarsquos comment suggest about men

ECO-10 280620 Topic-Supply AnalysisSHIFTING OF SUPPLY

But if there is change in factors other than the price of the commodity then either more is supplied at the same price or less supplied at the same price In such cases the price of the commodity remains constant but there is a change in other factors like change in the price of inputs change in technology of production change in price of other related goods change in taxation policy of the government etc For example there is an improvement in the technology of production of the commodity in question It leads to decrease in per unit of cost production of the commodity The firm is willing to sell more quantity of the commodity at the same price So the supply other commodity increases at the same price This increase in supply is shown by rightward shift of supply curve On the other hand if the firm uses inferior technology of production the cost of production per unit of the commodity increases The firm is willing to sell less quantity at the same price So the supply of the commodity decreases at the same price This decrease in supply is shown by leftward shift of the supply curve The above cases of increase and decrease in supply can be shown with the help of the following figures

Y INCREASE OF SUPPLY Price (Rs) s

P A s1

B

s

X` O s1 X

q q1

Y` Quantity demanded (in units)

Y DECREASE IN SUPPLY s2

s

price (Rs)

C

p A

s2

s

X` o X

q2 q

Y` Quantity demanded ( in units)

Main factors causing increase in supply or rightward shift of supply Curve(i) Fall in the price of other related goods

(ii) Fall in the price of inputsfactors(iii) Use of better technology in production(iv) Decrease in the rate of excise duty by government(v) If the objective of producer changes from profit maximization to salesMaximization

Main factors causing decrease in supply or leftward shift of supply curve(i) Increase in the price of other related goods(ii) Rise in the price of inputsfactors(iii) Use of inferior technology in production(iv) Increase in the rate of excise duty by the government(v) If the objective

Subject - Biology Topic ndash Chapter mdash6 PhotosynthesisSummary Execution

Today we will know about photosynthesis and its stages

Q1 What do you mean by photosynthesis The process by which living plants containing chlorophyll produce food

substances from carbon-di- oxide and water by using light energy Sunlight

6CO2 +12 H2O----------------------- C6 H12O6 + 6H2O + 6O2

Chlorophyll

Q2 What are the importance of photosynthesis I) Food for all Green plants trap solar energy by photosynthesis

process and supply food and energy for all living organisms either directly or indirectly

Ii) Oxygen to breathe in by product of photosynthesis is oxygen which is essential for all living organisms respiration

Q3 Write about two main phases of photosynthesis A Light dependent phase This phase occur in grana of chloroplast I) The chlorophyll on exposure to light energy becomes activated by

absorbing photons Ii) The absorbed energy is used in splitting the water molecules (H2O)

into its two components (H+ and OH- ) and releasing electron s 2H2O------------------------- 4H+ + 4e- +O2

Energy of 4 photons This reaction is known as photolysis

End products are H+ and oxygen water

B Light independent (Dark ) phase The reactions in this phase require no light energy

Here CO2 combine with H+ and produce glucose

Class XI

Subject Topic Summary ExecutionEVS Chapter-4 Legal

regimes for sustainable development

Environmental legislationEnvironmental legislation is the collection of laws and regulations pertaining to air quality water quality the wilderness endangered wildlife and other environmental factors The act ensures that matters important to the environment are thoroughly

Learn -The Forest (Conservation) Act 1980

considered in any decisions made by federal agencies

The Forest (Conservation) Act 1980 The Forest (Conservation) Act 1980 an Act of the Parliament of India to provide for the conservation of forests and for matters connected therewith or ancillary or incidental thereto It was further amended in 1988 This law extends to the whole of IndiaObjects and Reasons of the Forest Conservation Act

Deforestation causes ecological imbalance and leads to environmental deterioration Deforestation had been taking place on a large scale in the country and it had caused widespread concern The act seeks to check upon deforestation and de-reservation of forests

Subject Eng Literature (The Tempest ndash William Shakespeare) Topic Act II Scene 1 Lines 314 to 329 (End of scene)

[Students should read the original play and also the paraphrase given in the school prescribed textbook]Summary Questions amp Answers

Conspiracy of Antonio and Sebastian (Contd)

o As they approach Ariel appears again and wakes up Gonzalo by singing a tune in his ear Alonso also wakes up and they see both Sebastian and Antonio with drawn swords On being caught off guard they make up a story saying that they had heard a bellowing of bulls or lions

o They then moved to another part of the island

o Ariel at once rushes to Prospero to inform him of this development

SUMMING-UP of ACT-2 SCENE-1

(i) Among the survivors Ferdinand is separated from the rest which results in the disconsolate grief of Alonso as he took him for dead

(ii) The villainy of Antonio is confirmed

(iii) The supremacy of Prosperorsquos magic which resulted in the failure of the human conspiracy

(1)

(Act II Sc 1 L 311-325)SEBASTIAN Whiles we stood here securing your repose

Even now we heard a hollow burst of bellowing Like bulls or rather lions Didt not wake youIt struck mine ear most terribly

ALONSO I heard nothingANTONIO O rsquotwas a din to fright a monsters ear

To make an earthquake Sure it was the roarOf a whole herd of lions

ALONSO Heard you this GonzaloGONZALO Upon mine honour sir I heard a humming

And that a strange one too which did awake meI shaked you sir and cried As mine eyes opened I saw their weapons drawn There was a noiseThats verily rsquoTis best we stand upon our guardOr that we quit this place Lets draw our weapons

(i) Why has Prospero sent Ariel to Gonzalo and Alonso What does Ariel do to awaken Gonzalo

Prospero has already come to know by his magic powers the danger which threatens Gonzalo who had been Prosperorsquos friend and so he sent Ariel to preserve the lives of both Gonzalo and Alonso Prospero does not want that his scheme should remain unfulfilled Ariel begins to sing a song in Gonzalorsquos ears to awaken him(ii) Who are ready to carry out their plan Who takes steps to stop them Why does Gonzalo feel surprised after being awakened

Sebastian and Antonio are ready to carry out their plans They are standing with their swords drawn to kill Alonso and

(iv) We see two sets of contrasting characters Gonzalo-Adrian against Antonio-Sebastian

(v) The grief that works in Alonso can be perceived to his repentance for his association in Antoniorsquos crime against Prospero

Gonzalo Ariel takes steps to stop them from carrying out their nefarious scheme When Gonzalo is awakened by the song sung by Ariel into his ears he (Gonzalo) feels surprised because he sees Sebastian and Antonio standing with their swords drawn(iii) What reason do Sebastian and Antonio tell of drawing their swords when they are suspected by Alonso and Gonzalo

When Sebastian and Antonio are seen with their swords drawn they are looked with suspicion by Gonzalo and Alonso At first Sebastian tells them that as they stood here to guard them during their sleep they heard only a little before a sudden loud noise very much like the roaring of bulls or more probably that of lions Then Antonio follows him saying that this was a noise so terrible as to frighten even a monsterrsquos ears and this noise could even have shaken the earth and it was surely like the roaring of a multitude of lions Then seeing the danger they have drawn their swords Perhaps after hearing the terrible noise they (Gonzalo and Alonso) woke up from their sound sleep

(iv) What does Gonzalo tell Alonso about the strange noise What did he see on opening his eyes Gonzalo tells Alonso that he did not hear the sound of roaring but he heard a humming sound which was strange and which woke him up After waking up he gave him (Alonso) a shaking and a loud cry On opening his eyes he saw these two gentlemen standing with their swords drawn(v) What does Gonzalo suggest

Gonzalo suggests that there was a noise indeed and of that he has no doubt at all and suggests that the best course for them would be to remain alert and vigilant against any possible danger to their lives or to leave this place and move to some other part of the island

Class XIISubject Topic Summary Execution

Commerce

Chapter- Management

Today we will discuss about LEVELS OF MANAGEMENT

Levels of management is a series or chain of managerial positions from top to bottom It helps individuals to know their authority responsibilities and superior-subordinate relations among themselves There are mainly three levels of Management TOP LEVEL MANAGEMENTMIDDLE LEVEL MANAGEMENTLOWER LEVEL MANAGEMENT

Top level managementIt consists of members at the highest level in the management hierarchy This level includes Board Of Directors Chief Executive Managing Directors Chairman President Vice President

Rolefunctions of the top levelmanagement1To analyse evaluate and deal

with theexternal environment2 To determine the objectives and

policies of the business3 To strive for welfare and survival

of business

4 To create an organisational Framework consisting of authority responsibility relationship

Middle level management Congress of members or groups who are concerned with implementation of the policies let down by the top managementThis level includes head of the department such as finance manager marketing manager branch and regional managers departmental and divisional heads plant superintendent etc

Role of functions of the middle level management

1 To interpret the policies framed by top management

2 To assign duties and responsibilities to lower level managers

3 To select and appoint employees for middle and supervisory level and evaluate their performance

4 To co-operate with other departments for smooth functioning

Operational or supervisory level managementIt refers to the group are members who are concerned with execution of the work They are also known as fast line managers This level includes supervisor 4 men Section Officer clerk Inspector etc

Role of functions of the lower level management1 To plan and execute day-to-

day operations2 To supervise and control the workers3 To arrange materials and

tools to start the process and make arrangements for training

4 Today present workers grievance and suggestions before the management and

ensure safe and proper working conditions in the factory

Business Studies

Staff Appraisal Chapter- 10 Today let us start with a new chapter

Staff Appraisal

Meaning of Performance Appraisal

Performance Appraisal is the systematic evaluation of the performance of employees and to understand the abilities of a person for further growth and developmentThe supervisors measure the pay of employees and compare it with targets and plansThe supervisor analyses the factors behind work performances of employeesThe employers are in position to guide the employees for a better performance

Objectives of Performance Appraisal

Following are the objectives of Performance Appraisal

To maintain records in order to determine compensation packages wage structure salaries raises etc

To identify the strengths and weaknesses of employees to place right men on right job

To maintain and assess the potential present in a person for further growth and development

To provide a feedback to employees regarding their performance and related status

To provide a feedback to employees regarding their performance and related status

Importance of Performance Appraisal

Performance appraisal provides important and useful information for the assessment of employees skill

knowledge ability and overall job performance The following are the points which indicate the importance of performance appraisal in an organization

1 Performance appraisal helps supervisors to assess the work performance of their subordinates

2 Performance appraisal helps to assess the training and development needs of employees

3 Performance appraisal provides grounds for employees to correct their mistakes and it also provides proper guidance and criticism for employees development4 Performance appraisal provides reward for better performance

5 Performance appraisal helps to improve the communication system of the organization

6 Performance appraisal evaluates whether human resource programs being implemented in the organization have been effective

7 Performance appraisal helps to prepare pay structure for each employee working in the organization

8 Performance appraisal helps to review the potentiality of employees so that their future capability is anticipated

Geography

DRIANAGE The SubarnarekhaThe Subarnarekha and the Brahmaniinterposed between the Ganga and the Mahanadi deltas drain an area of 19300 sq kmand 39033 sq km respectively The drainage basins of these streams are shared byJharkhand Odisha west Bengal and Chhattisgarh The Brahmani is known as southKoel in its upper reaches in Jharkhand

The NarmadaThe Narmada rises in the Amarkantak hills of MadhyaPradesh It flows towards the West in a rift valleyformed due to a geological fault The total length of it is 1300 km All the tributaries of the

Q1 Name the two westward flowing rivers in the peninsular plateauA1 Narmada and Tapi are the only westward flowing rivers of the peninsular plateau

Q2 Differentiate between east-flowing rivers and west-flowing riversA2

East-flowing rivers

West-flowing rivers

Narmada are very short inlength Most of its tributaries join the main streamright anglesThe Narmada basin covers parts of Madhya Pradesh and Gujarat

The Tapi The Tapi rises in the Satpura ranges in the Betul listrictof Madhya Pradesh It flows in a rift valley parallel tothe Narmada but it is much shorter in length It coversparts of Madhya Pradesh Gujarat and MaharashtraThe length is about 724 km

The Sabarmati and the MahiThe Sabarmati rises in the Aravali hills and flows south-south-westwards for a distance of 300 kilometres to the Arabian Sea The Sabarmatibasin extends over an area of 21674 sq km in Rajasthan and Gujarat The Mahi rises inthe east of Udaipur and drains an area of 34842 sq km lying in Madhya PradeshRajasthan and Gujarat It flows south-westwards for a distance of 533 km before it fallsinto the Gulf of Khambhat

The ChambalThe Chambal rises near Mhow in the Vindhya Range and flows towards the northgenerally in a gorge upto Kota Below Kota it turns to the north-east direction and afterreaching Pinahat it turns to the east and runs nearly parallel to the Yamuna beforejoining it in the southern part of the Etawah district in Uttar PradeshMajor Rivers of India with their basin area (Sqkm)

Himalayan System Indus 321290Ganga 861404

Brahmaputra 187110Indus System

Jhelum 34775Beas 20303

Ganga System Yamuna 366223Ghaghra 127950

Peninsular RiversNarmada 98796

Tapi 65145Mahanadi 141600

Subarnarekha 19300Sabarmati 21674

Mahi 34842Godavari 312812

Godavari Krishna Kaveri Mahanadi are the east-flowing rivers

Narmada Tapi west-flowing rivers

They fall into the Bay of Bengal

They fall into Arabian Sea

These rivers form big deltas

These rivers form comparativelysmall deltas

Catchment areas of these rivers are larger

Catchment areas of these rivers are smaller

Krishna 2589488Cauveri 87900

Subject ndashBiology Topic ndashChapter -5 Inheritance amp Variations Summary ExecutionToday we will discussabout linkage and its classification

LINKAGE The tendency of the genes located on the same chromosome to stay together is

hereditary transmission Linked genes the genes responsible for this Genes that exhibit the process of linkage locates in the same chromosome The distance between the linked genes in a chromosome determines the strength

of linkage i e genes that are located close to each other show stronger linkage than that are located far from each other

COMPLETE LINKAGE It is the type of linkage showed by the genes that are closely located or are tightly

linked with each other as they have no chance of separatingby crossing over These genes are always transmitted together to the same gamete and the same

offspring In such condition only parental or non cross over type of gametes are formedINCOMPLETE KINKAGE It is type of linkage showed by the genes that are distantly located orare loosely

linked with each other because they have chance of separating by crossing over

SIGNIFICANCE i) It helps in holding the parental character togetherii) It checks the appearance of new recombination and helps in bringing the

hybrid population which resembles the original parents iii) Linked genes dilute the effects of undesirable traits

Subject Eng Literature (The Tempest ndash William Shakespeare) Topic Essay Questions (EQ-3)Question No 3

Give a character sketch of CalibanAnswer

The character of Caliban has been wonderfully conceived by Shakespeare as the manifestation of all that is gross and earthy ndash a sort of creature of the earth as Ariel is a sort of creature of the air

Calibanrsquos Physical Appearanceo Caliban is lsquofreckledrsquo a lsquomisshapen knaversquo not honoured with human shape

o Prospero calls him lsquothou tortoisersquo (Act I Sc 2 Line 317) Trinculo stumbling upon him describes him as ldquoA strange fish hellip Legged like a man And his fins like armsrdquo He ldquosmells like a fishrdquo (Act II Sc 2 Line 25)

o Prospero also calls him a ldquobeastrdquo (Act IV Sc 1 Line 140) and ldquoThis misshapen knaverdquo (Act V Sc 1 Line 268)

o Further it appears that in addition to his physical deformity his spiritual inferiority is also suggested by Prosperorsquos claim that his birth resulted from the union between his mother the witch Sycorax and the devil

Calibanrsquos ParentageWhen the play opens Caliban is twenty four years of age having been born on the island twelve years before the coming of Prospero His mother was the foul witch Sycorax who was banished from Algiers for ldquomischiefs manifold and sorceries terrible to enter human hearingrdquo (Act I Sc 2 Line 264) and the father was the Devil himself Thus

Caliban is a monster of evil and brute nature ugly deformed and stinking

Calibanrsquos Savage and Malignant Natureo Caliban is entirely a creature of the earth ndash gross brutal and savage He regards himself as the rightful possessor

of the island and Prospero as a usurper

o In his young age he was on good terms with Prospero He had consented to be received by Prospero at his house and to be educated by him He has learnt human language only to curse his master whom he abhors

o His beastly nature soon breaks out and ends in a vicious attack on Miranda This opens the eye of Prospero who becomes severe to him and enforces his service by threats and violence

o Prospero uses him to make dams for fish to fetch firewood scraper trenches wash dishes and keep his cell clean

Calibanrsquos Hatred for ProsperoA profound hatred for Prospero has taken hold of Caliban It springs from a sense of his being dispossessed and ill-treated He would kill Prospero if he could but he knows the power of Prosperorsquos lsquobookrsquo Hence he transfers his allegiance to Stephano who seems like a god to him He also incites the two drunken associates to batter the skull of Prospero when he sleeps in the afternoon

Caliban Shows Considerable Intelligenceo He has learnt Prosperorsquos language

ldquoYou taught me language and my profit onrsquot (Act II Sc 2 Lines 86-89)Is I know how to curserdquo

o He is well aware of the futility of arguing with one who has more power than he has

ldquoI must obey his art is such power (Act I Sc 2 Lines 373-376)It would control my damrsquos god SetebosAnd make a vassal of himrdquo

o He realizes the importance of Prosperorsquos books

ldquoRemember (Act III Sc 2 Lines 89-92)First to possess his books for without themHersquos but a sot as I am nor hath notOne spirit to commandrdquo

o He knows the value of stealth when attacking the enemy

ldquoPray you tread softly that the blind mole may not (Act IV Sc 1 Lines 194-195)Hear a foot fall we now are near his cellrdquo

o Caliban has a better set of values than Stephano and Trinculo They are distracted from their plan by their greed for Prosperorsquos rich garments Only Caliban realizes that such a finery is unimportant

ldquoLeave it alone thou fool it is but trashrdquo (Act IV Sc 1 Lines 224)

Caliban is not a good judge of characterCaliban is not a good judge of character He decides for example that Stephano is a god because he dispenses lsquocelestial liquorrsquo (Act II Sc 2 Line 115) but then it must be remembered that he has only known his mother Sycorax Prospero Miranda and the spirits that torture him However he quickly discovers his error of judgementrdquo

ldquoWhat a thrice-double ass (Act V Sc 1 Lines 295-297)Was I to take this drunkard for a godAnd worship this dull foolrdquo

Calibanrsquos Imaginative NatureIf Caliban is sub-human in what has been said above he is human in the respect of the poetic side of his character He listens to music with rapture He tells of the beautiful dreams in which heaven rains treasures upon him and which upon waking he yearns to renew One of the most poetic passages in whole play is Calibanrsquos description of the island

to Stephano and Trinculo

ldquoBe not afeard The isle is full of noises (Act III Sc 2 Lines 135-143)Sounds and sweet airs that give delight and hurt notSometimes a thousand twangling instrumentsWill hum about mine ears and sometime voicesThat if I then had waked after long sleepWill make me sleep again and then in dreamingThe clouds methought would open and show richesReady to drop upon me that when I wakedI cried to dream againrdquo

Caliban - Less Ignoble Than Some OthersCalibanrsquos motive for murder is less dishonourable than that of Antonio and Sebastian They plan to kill Alonso to gain his power and wealth Caliban merely wants revenge and the return of lsquohisrsquo island

Conclusiono Calibanrsquos character is not portrayed very clearly in the play and hence we cannot decide whether he is a poor

savage being grossly maltreated by Prospero or whether he is evil and must therefore be kept in bondage or enslavement

o Caliban is contrasted with Ariel who is a spirit and thus swift and uninterested in physical activitieso Caliban is also contrasted with Prospero who is the all-powerful master of the island and of the destiny of all

those on the islando Caliban is also contrasted with civilized man showing him to be less evil than Antonio and Stephano and less

materialistic than Stephano and Trinculoo Caliban has suffered at the hands of Prospero and he has learnt to curse by listening to Prosperorsquos abuse He

certainly believes that Prospero has deprived him of his birthrighto Finally the character Caliban is thought to be one of Shakespearersquos masterpieces The complexity of the character

is reflected in the large volume of critical discussion that has grown around it

ECO ndash12 Topic-Forms of market

MonopolyMonopoly is a market structure in which there is a single seller there are no close substitutes for the commodity produced by the firm and there are barriers to entry Example Indian Railways which is operated under government of India Monopoly also implies absence of competitionFeatures of Monopoly Monopoly is characterized by1 Single Seller In monopoly there is only one firm producing the product The whole industry consists of this single firm Thus under monopoly there is no distinction between firm and industry Being the only firm there is significant control of the firm over supply and price Thus under monopoly buyers do not have the option of buying the commodity from any other seller They have to buy the product from the firm or they can go without the commodity This fact gives immense control to the monopolist over the market

2No Close Substitute There are no close substitutes of the product produced by the monopolist firm If there are close substitutes of the product in the market it implies presence of more than one firm and hence no monopoly In order to ensure a total of control over the market by the monopolist firm it is assumed that there are no close substitutes of the product

3 No Entry amp Exit Monopoly can only exist when there is strong barriers before a new firm to enter the market In fact once a monopoly firm starts producing the product no other firm can produce the same One reason for this is the ability of the

monopolist to produce the product at a lower cost than any new firm who thinks to enter the market If a new firm who knows that it cannot produce at a lower cost than the monopolist then that firm will never enter the market for fear of losing out in competition Similarly the monopolist who is operating for a long time may be enjoying reputation among its customers and is in a better position to use the situation in its own benefit A new firm has to take long time to achieve this and so may not be interested to enter the market

4 Price Maker Being the single seller of the product the monopolist has full control over the pricing of the product On the other hand if there is a large number of buyers in the market so no single buyer exercises any significant influence over price determination Thus it is a sellerrsquos market So monopoly firm is a price maker

5 Price Discrimination Having considerable control over the market on account of being single seller with no entry of other firms the monopolist can exercise policy of price discrimination it means that the monopolist can sell different quantities of the same product to a consumer at different price or same quantity to different consumers at different prices by adjudging the standard of living of the consumer

6 Shape of Demand Curve Since a monopolist has full control over the price therefore he can sell more by lowering the price This makes the demand curve downward sloping

Subject Ac-12 290620 Topic- retirement Model sumThe Balance Sheet of Rohit Nisha and Sunil who are partners in a firm sharing profits according to their capitals as on 31st March 2014 was as under

Liabilities Amount Assets Amount (Rs) (` Rs)

Creditors 25000 Machinery 40000Bills Payable 13000 Building 90000General Reserve 22000 Debtors 30000Capital Less Provision for Rohit 60000 Bad debts 1000

29000 Nisha 40000 Stocks 23000 Sunil 40000 140000 Cash at Bank 18000

200000 200000

On the date of Balance Sheet Nisha retired from the firm and following adjustments were made(i) Building is appreciated by 20(ii) Provision for bad debts is increased to 5 on Debtors(iii) Machinery is depreciated by 10(iv) Goodwill of the firm is valued at Rs 56000 and the retiring partnerrsquos share is adjusted

(v) The capital of the new firm is fixed at Rs120000 Prepare Revaluation Account Capital Accounts of the partner and Balance Sheet of the new firm after Nisharsquos retirement Revaluation AccountDr Cr

Particulars Amount Particulars Amount (`Rs) (Rs`)

Provision for Bad debt Ac 500 Building Ac 18000Machinery Ac 4000Profit transferred toCapital Accounts (3 2 2)Rohit 5786Nisha 3857Sunil 3857

13500

18000 18000

Capital Account

Dr Cr

Particulars Rohit Nisha Sunil Particulars Rohit Nisha Sunil (Rs`) (Rs`) (`Rs) (Rs`) (Rs`) (Rs`)

Sunilrsquos Capital ac 9600 mdash 6400 Balance bd 60000 40000 40000Bank - 66143 - General Reserve 9428 6286 6286Balance cd 72000 mdash 48000 Revaluation (Profi 5786 3857 3857 Rohitrsquos Capital Ac mdash 9600 mdash

Sunilrsquos Capital Ac 6400 Bank 6386 - 4257

81600 66143 54400 81600 66143 54400

Balance Sheet as at 31st March 2014

Liabilities Amount Assets Amount (Rs`) (Rs`)

Creditors 25000 Building 108000Bank overdraft 37500 Machinery 36000

Bills Payable 13000 Debtors 30000Capital Less ProvisionRohit 72000 for Bad debts 1500 28500Sunil 48000 120000 Stock 23000

195500 195500

Working Notes (i) (a) Profit sharing ratio is 60000 40000 40000 ie = 3 2 2(b) Gaining Ratio Rohit = 35 ndash 37 = 2135 ndash 1535 = 635Sunil = 25-27 = 1435 ndash 1035 = 435= 635 435= 6 4 = 3 2(c) Nisha Share of Goodwill = Rs 56000 times 27 = Rs16000Share of Goodwill in the gaining ratio by the existing partner ieRohit = Rs16000 times 35 = Rs 9600Sunil = Rs 16000 times 25 = Rs 6400

The journal entry isRohitrsquos Capital Ac Dr 9600Sunilrsquos Capital Ac Dr 6400 To Nisharsquos Capital Ac 16000(Share of Goodwill divided into gaining ratio)

  • 1 Static Friction
  • The frictional force that acts between the surfaces when they are at rest with respect to each other is called Static Friction
    • Static Friction Examples
      • 2 Sliding Friction
        • Examples Of Sliding Friction
          • 3 Rolling Friction
            • Examples Of Rolling Friction
              • Objects and Reasons of the Forest Conservation Act
Page 39:  · Web viewSubject . Topic . Summary . Execution . English 1 . Sounds of animals . Hens –cackle Horses –neigh Lions –roar Owls –hoots Snake –hiss. English 2 . Mother’s

individually

Features of RTGS1It is the continuous settlement of

funds transfer individually on an order by order basis

2RTGS facility is provided only by CBS core banking solution enabled Bank branches

3Amount charged from the customer for RTGS transactions vary from bank to bank

2) Explain the term NEFT Write the features of NEFT

Answer) National electronic funds transfer may be defined as a nationwide system that facilitates individuals Farms and copper operates to electronically transfer funds from any bank branch to any individual farm or corporate having an account with any other bank branch in the country

Features of NEFT2 Transfer can be made 7 times on

weekdays and 6 times on Saturday

3 NEFT cannot be used to receive foreign remittances

4 NEFT transaction takes place in batches

5 A bank branch must be NEFT enabled to become a part of NEFT fund transfer network

6 There is no maximum or minimum amount that can be transferred through NEFT when one bank has a bank account

English Language

CompositionEssay

A composition is an art of creating a piece of writing on any topic or subject It is the writing correctly beautifully and clearly in order to make some interesting reading Structure of the composition

Introduction ( you lay the foundation for your composition)

Body (it constitutes the main part of the essay)

Conclusion (final statement that leaves a lasting impression)

Kinds of essays1 The Narrative essay2 The descriptive essay3 The reflective essay4 The argumentative essay

Write a composition on any one of the following topics (350- 400 words)

1 Friendship Or2 The first day of your school

Subject Eng Literature (The Merchant of Venice ndash William Shakespeare)Topic Act V Scene 1 Lines 127 to 158 (Nerissa helliphellip The clerk will nersquoer wear hair onrsquos face that had it) [Students should read the original play and also the paraphrase given in the school prescribed textbook]

Summary Revision Questions o Soon thereafter Bassanio Gratiano

and Antonio arrive

o Bassanio tells Portia that he is feeling as if it is morning because of the presence of Portia who is shining like the sun When Antonio is introduced by Bassanio to Portia she tells Bassanio that he should be grateful to Antonio who took so much trouble on his account even to the extent of risking his life

o Nerissa starts quarrelling with Gratiano and demands that he show her the ring she had presented to him and which she had warned him not to lose She suspects that Gratiano must have presented the ring to some young woman and not to the lawyerrsquos clerk as he repeatedly says and assures

Answer the following questions to check your preparation of Act IV Scenes 1 and 2

You must attempt only after you have completed your preparation of Act IV The answers must be in complete sentences using textual evidence (with citation) when necessary

[It would be in your own interest to attempt the above questions honestly totally refraining from consulting your textbook or your notes during answering After completion you should correct the paper yourself consulting the textbooknotes etc and award marks as specified Please let me know the marks you scored through WhatsApp in the group or to my personal WhatsApp]

Act IV Scene 1 (each question carries 2 marks)

1 What did the Duke try to do for Antonio

2 Why does Shylock refuse to show mercy How does he justify his stance

3 Why does Antonio say he is ready to die 4 What information is contained in Bellariorsquos letter

5 Why does Portia (as Balthazar) assert that Shylock must show mercy How does he respond

6 What offers are made to Shylock to get him to spare Antonio How are they received

7 What does Antoniorsquos speech as he faces the prospect of Shylockrsquos knife tell you about his character

8 How do Bassanio and Gratiano react to the looming prospect of Antoniorsquos demise

9 How does Portia (as Balthazar) use the law to turn the tables on Shylock

10 What does the Duke decree should happen to Shylock Why What happens to Shylockrsquos estate

11 What does Portia ask Bassanio as payment for her ldquoservicesrdquo What is his initial response What makes him change his mind

Act IV Scene 2 (each question carries 1frac12 marks)

1 What does Gratiano bring to Portia (Balthazar)

2 What does Nerissa plan on getting from Gratiano What does Portiarsquos comment suggest about men

ECO-10 280620 Topic-Supply AnalysisSHIFTING OF SUPPLY

But if there is change in factors other than the price of the commodity then either more is supplied at the same price or less supplied at the same price In such cases the price of the commodity remains constant but there is a change in other factors like change in the price of inputs change in technology of production change in price of other related goods change in taxation policy of the government etc For example there is an improvement in the technology of production of the commodity in question It leads to decrease in per unit of cost production of the commodity The firm is willing to sell more quantity of the commodity at the same price So the supply other commodity increases at the same price This increase in supply is shown by rightward shift of supply curve On the other hand if the firm uses inferior technology of production the cost of production per unit of the commodity increases The firm is willing to sell less quantity at the same price So the supply of the commodity decreases at the same price This decrease in supply is shown by leftward shift of the supply curve The above cases of increase and decrease in supply can be shown with the help of the following figures

Y INCREASE OF SUPPLY Price (Rs) s

P A s1

B

s

X` O s1 X

q q1

Y` Quantity demanded (in units)

Y DECREASE IN SUPPLY s2

s

price (Rs)

C

p A

s2

s

X` o X

q2 q

Y` Quantity demanded ( in units)

Main factors causing increase in supply or rightward shift of supply Curve(i) Fall in the price of other related goods

(ii) Fall in the price of inputsfactors(iii) Use of better technology in production(iv) Decrease in the rate of excise duty by government(v) If the objective of producer changes from profit maximization to salesMaximization

Main factors causing decrease in supply or leftward shift of supply curve(i) Increase in the price of other related goods(ii) Rise in the price of inputsfactors(iii) Use of inferior technology in production(iv) Increase in the rate of excise duty by the government(v) If the objective

Subject - Biology Topic ndash Chapter mdash6 PhotosynthesisSummary Execution

Today we will know about photosynthesis and its stages

Q1 What do you mean by photosynthesis The process by which living plants containing chlorophyll produce food

substances from carbon-di- oxide and water by using light energy Sunlight

6CO2 +12 H2O----------------------- C6 H12O6 + 6H2O + 6O2

Chlorophyll

Q2 What are the importance of photosynthesis I) Food for all Green plants trap solar energy by photosynthesis

process and supply food and energy for all living organisms either directly or indirectly

Ii) Oxygen to breathe in by product of photosynthesis is oxygen which is essential for all living organisms respiration

Q3 Write about two main phases of photosynthesis A Light dependent phase This phase occur in grana of chloroplast I) The chlorophyll on exposure to light energy becomes activated by

absorbing photons Ii) The absorbed energy is used in splitting the water molecules (H2O)

into its two components (H+ and OH- ) and releasing electron s 2H2O------------------------- 4H+ + 4e- +O2

Energy of 4 photons This reaction is known as photolysis

End products are H+ and oxygen water

B Light independent (Dark ) phase The reactions in this phase require no light energy

Here CO2 combine with H+ and produce glucose

Class XI

Subject Topic Summary ExecutionEVS Chapter-4 Legal

regimes for sustainable development

Environmental legislationEnvironmental legislation is the collection of laws and regulations pertaining to air quality water quality the wilderness endangered wildlife and other environmental factors The act ensures that matters important to the environment are thoroughly

Learn -The Forest (Conservation) Act 1980

considered in any decisions made by federal agencies

The Forest (Conservation) Act 1980 The Forest (Conservation) Act 1980 an Act of the Parliament of India to provide for the conservation of forests and for matters connected therewith or ancillary or incidental thereto It was further amended in 1988 This law extends to the whole of IndiaObjects and Reasons of the Forest Conservation Act

Deforestation causes ecological imbalance and leads to environmental deterioration Deforestation had been taking place on a large scale in the country and it had caused widespread concern The act seeks to check upon deforestation and de-reservation of forests

Subject Eng Literature (The Tempest ndash William Shakespeare) Topic Act II Scene 1 Lines 314 to 329 (End of scene)

[Students should read the original play and also the paraphrase given in the school prescribed textbook]Summary Questions amp Answers

Conspiracy of Antonio and Sebastian (Contd)

o As they approach Ariel appears again and wakes up Gonzalo by singing a tune in his ear Alonso also wakes up and they see both Sebastian and Antonio with drawn swords On being caught off guard they make up a story saying that they had heard a bellowing of bulls or lions

o They then moved to another part of the island

o Ariel at once rushes to Prospero to inform him of this development

SUMMING-UP of ACT-2 SCENE-1

(i) Among the survivors Ferdinand is separated from the rest which results in the disconsolate grief of Alonso as he took him for dead

(ii) The villainy of Antonio is confirmed

(iii) The supremacy of Prosperorsquos magic which resulted in the failure of the human conspiracy

(1)

(Act II Sc 1 L 311-325)SEBASTIAN Whiles we stood here securing your repose

Even now we heard a hollow burst of bellowing Like bulls or rather lions Didt not wake youIt struck mine ear most terribly

ALONSO I heard nothingANTONIO O rsquotwas a din to fright a monsters ear

To make an earthquake Sure it was the roarOf a whole herd of lions

ALONSO Heard you this GonzaloGONZALO Upon mine honour sir I heard a humming

And that a strange one too which did awake meI shaked you sir and cried As mine eyes opened I saw their weapons drawn There was a noiseThats verily rsquoTis best we stand upon our guardOr that we quit this place Lets draw our weapons

(i) Why has Prospero sent Ariel to Gonzalo and Alonso What does Ariel do to awaken Gonzalo

Prospero has already come to know by his magic powers the danger which threatens Gonzalo who had been Prosperorsquos friend and so he sent Ariel to preserve the lives of both Gonzalo and Alonso Prospero does not want that his scheme should remain unfulfilled Ariel begins to sing a song in Gonzalorsquos ears to awaken him(ii) Who are ready to carry out their plan Who takes steps to stop them Why does Gonzalo feel surprised after being awakened

Sebastian and Antonio are ready to carry out their plans They are standing with their swords drawn to kill Alonso and

(iv) We see two sets of contrasting characters Gonzalo-Adrian against Antonio-Sebastian

(v) The grief that works in Alonso can be perceived to his repentance for his association in Antoniorsquos crime against Prospero

Gonzalo Ariel takes steps to stop them from carrying out their nefarious scheme When Gonzalo is awakened by the song sung by Ariel into his ears he (Gonzalo) feels surprised because he sees Sebastian and Antonio standing with their swords drawn(iii) What reason do Sebastian and Antonio tell of drawing their swords when they are suspected by Alonso and Gonzalo

When Sebastian and Antonio are seen with their swords drawn they are looked with suspicion by Gonzalo and Alonso At first Sebastian tells them that as they stood here to guard them during their sleep they heard only a little before a sudden loud noise very much like the roaring of bulls or more probably that of lions Then Antonio follows him saying that this was a noise so terrible as to frighten even a monsterrsquos ears and this noise could even have shaken the earth and it was surely like the roaring of a multitude of lions Then seeing the danger they have drawn their swords Perhaps after hearing the terrible noise they (Gonzalo and Alonso) woke up from their sound sleep

(iv) What does Gonzalo tell Alonso about the strange noise What did he see on opening his eyes Gonzalo tells Alonso that he did not hear the sound of roaring but he heard a humming sound which was strange and which woke him up After waking up he gave him (Alonso) a shaking and a loud cry On opening his eyes he saw these two gentlemen standing with their swords drawn(v) What does Gonzalo suggest

Gonzalo suggests that there was a noise indeed and of that he has no doubt at all and suggests that the best course for them would be to remain alert and vigilant against any possible danger to their lives or to leave this place and move to some other part of the island

Class XIISubject Topic Summary Execution

Commerce

Chapter- Management

Today we will discuss about LEVELS OF MANAGEMENT

Levels of management is a series or chain of managerial positions from top to bottom It helps individuals to know their authority responsibilities and superior-subordinate relations among themselves There are mainly three levels of Management TOP LEVEL MANAGEMENTMIDDLE LEVEL MANAGEMENTLOWER LEVEL MANAGEMENT

Top level managementIt consists of members at the highest level in the management hierarchy This level includes Board Of Directors Chief Executive Managing Directors Chairman President Vice President

Rolefunctions of the top levelmanagement1To analyse evaluate and deal

with theexternal environment2 To determine the objectives and

policies of the business3 To strive for welfare and survival

of business

4 To create an organisational Framework consisting of authority responsibility relationship

Middle level management Congress of members or groups who are concerned with implementation of the policies let down by the top managementThis level includes head of the department such as finance manager marketing manager branch and regional managers departmental and divisional heads plant superintendent etc

Role of functions of the middle level management

1 To interpret the policies framed by top management

2 To assign duties and responsibilities to lower level managers

3 To select and appoint employees for middle and supervisory level and evaluate their performance

4 To co-operate with other departments for smooth functioning

Operational or supervisory level managementIt refers to the group are members who are concerned with execution of the work They are also known as fast line managers This level includes supervisor 4 men Section Officer clerk Inspector etc

Role of functions of the lower level management1 To plan and execute day-to-

day operations2 To supervise and control the workers3 To arrange materials and

tools to start the process and make arrangements for training

4 Today present workers grievance and suggestions before the management and

ensure safe and proper working conditions in the factory

Business Studies

Staff Appraisal Chapter- 10 Today let us start with a new chapter

Staff Appraisal

Meaning of Performance Appraisal

Performance Appraisal is the systematic evaluation of the performance of employees and to understand the abilities of a person for further growth and developmentThe supervisors measure the pay of employees and compare it with targets and plansThe supervisor analyses the factors behind work performances of employeesThe employers are in position to guide the employees for a better performance

Objectives of Performance Appraisal

Following are the objectives of Performance Appraisal

To maintain records in order to determine compensation packages wage structure salaries raises etc

To identify the strengths and weaknesses of employees to place right men on right job

To maintain and assess the potential present in a person for further growth and development

To provide a feedback to employees regarding their performance and related status

To provide a feedback to employees regarding their performance and related status

Importance of Performance Appraisal

Performance appraisal provides important and useful information for the assessment of employees skill

knowledge ability and overall job performance The following are the points which indicate the importance of performance appraisal in an organization

1 Performance appraisal helps supervisors to assess the work performance of their subordinates

2 Performance appraisal helps to assess the training and development needs of employees

3 Performance appraisal provides grounds for employees to correct their mistakes and it also provides proper guidance and criticism for employees development4 Performance appraisal provides reward for better performance

5 Performance appraisal helps to improve the communication system of the organization

6 Performance appraisal evaluates whether human resource programs being implemented in the organization have been effective

7 Performance appraisal helps to prepare pay structure for each employee working in the organization

8 Performance appraisal helps to review the potentiality of employees so that their future capability is anticipated

Geography

DRIANAGE The SubarnarekhaThe Subarnarekha and the Brahmaniinterposed between the Ganga and the Mahanadi deltas drain an area of 19300 sq kmand 39033 sq km respectively The drainage basins of these streams are shared byJharkhand Odisha west Bengal and Chhattisgarh The Brahmani is known as southKoel in its upper reaches in Jharkhand

The NarmadaThe Narmada rises in the Amarkantak hills of MadhyaPradesh It flows towards the West in a rift valleyformed due to a geological fault The total length of it is 1300 km All the tributaries of the

Q1 Name the two westward flowing rivers in the peninsular plateauA1 Narmada and Tapi are the only westward flowing rivers of the peninsular plateau

Q2 Differentiate between east-flowing rivers and west-flowing riversA2

East-flowing rivers

West-flowing rivers

Narmada are very short inlength Most of its tributaries join the main streamright anglesThe Narmada basin covers parts of Madhya Pradesh and Gujarat

The Tapi The Tapi rises in the Satpura ranges in the Betul listrictof Madhya Pradesh It flows in a rift valley parallel tothe Narmada but it is much shorter in length It coversparts of Madhya Pradesh Gujarat and MaharashtraThe length is about 724 km

The Sabarmati and the MahiThe Sabarmati rises in the Aravali hills and flows south-south-westwards for a distance of 300 kilometres to the Arabian Sea The Sabarmatibasin extends over an area of 21674 sq km in Rajasthan and Gujarat The Mahi rises inthe east of Udaipur and drains an area of 34842 sq km lying in Madhya PradeshRajasthan and Gujarat It flows south-westwards for a distance of 533 km before it fallsinto the Gulf of Khambhat

The ChambalThe Chambal rises near Mhow in the Vindhya Range and flows towards the northgenerally in a gorge upto Kota Below Kota it turns to the north-east direction and afterreaching Pinahat it turns to the east and runs nearly parallel to the Yamuna beforejoining it in the southern part of the Etawah district in Uttar PradeshMajor Rivers of India with their basin area (Sqkm)

Himalayan System Indus 321290Ganga 861404

Brahmaputra 187110Indus System

Jhelum 34775Beas 20303

Ganga System Yamuna 366223Ghaghra 127950

Peninsular RiversNarmada 98796

Tapi 65145Mahanadi 141600

Subarnarekha 19300Sabarmati 21674

Mahi 34842Godavari 312812

Godavari Krishna Kaveri Mahanadi are the east-flowing rivers

Narmada Tapi west-flowing rivers

They fall into the Bay of Bengal

They fall into Arabian Sea

These rivers form big deltas

These rivers form comparativelysmall deltas

Catchment areas of these rivers are larger

Catchment areas of these rivers are smaller

Krishna 2589488Cauveri 87900

Subject ndashBiology Topic ndashChapter -5 Inheritance amp Variations Summary ExecutionToday we will discussabout linkage and its classification

LINKAGE The tendency of the genes located on the same chromosome to stay together is

hereditary transmission Linked genes the genes responsible for this Genes that exhibit the process of linkage locates in the same chromosome The distance between the linked genes in a chromosome determines the strength

of linkage i e genes that are located close to each other show stronger linkage than that are located far from each other

COMPLETE LINKAGE It is the type of linkage showed by the genes that are closely located or are tightly

linked with each other as they have no chance of separatingby crossing over These genes are always transmitted together to the same gamete and the same

offspring In such condition only parental or non cross over type of gametes are formedINCOMPLETE KINKAGE It is type of linkage showed by the genes that are distantly located orare loosely

linked with each other because they have chance of separating by crossing over

SIGNIFICANCE i) It helps in holding the parental character togetherii) It checks the appearance of new recombination and helps in bringing the

hybrid population which resembles the original parents iii) Linked genes dilute the effects of undesirable traits

Subject Eng Literature (The Tempest ndash William Shakespeare) Topic Essay Questions (EQ-3)Question No 3

Give a character sketch of CalibanAnswer

The character of Caliban has been wonderfully conceived by Shakespeare as the manifestation of all that is gross and earthy ndash a sort of creature of the earth as Ariel is a sort of creature of the air

Calibanrsquos Physical Appearanceo Caliban is lsquofreckledrsquo a lsquomisshapen knaversquo not honoured with human shape

o Prospero calls him lsquothou tortoisersquo (Act I Sc 2 Line 317) Trinculo stumbling upon him describes him as ldquoA strange fish hellip Legged like a man And his fins like armsrdquo He ldquosmells like a fishrdquo (Act II Sc 2 Line 25)

o Prospero also calls him a ldquobeastrdquo (Act IV Sc 1 Line 140) and ldquoThis misshapen knaverdquo (Act V Sc 1 Line 268)

o Further it appears that in addition to his physical deformity his spiritual inferiority is also suggested by Prosperorsquos claim that his birth resulted from the union between his mother the witch Sycorax and the devil

Calibanrsquos ParentageWhen the play opens Caliban is twenty four years of age having been born on the island twelve years before the coming of Prospero His mother was the foul witch Sycorax who was banished from Algiers for ldquomischiefs manifold and sorceries terrible to enter human hearingrdquo (Act I Sc 2 Line 264) and the father was the Devil himself Thus

Caliban is a monster of evil and brute nature ugly deformed and stinking

Calibanrsquos Savage and Malignant Natureo Caliban is entirely a creature of the earth ndash gross brutal and savage He regards himself as the rightful possessor

of the island and Prospero as a usurper

o In his young age he was on good terms with Prospero He had consented to be received by Prospero at his house and to be educated by him He has learnt human language only to curse his master whom he abhors

o His beastly nature soon breaks out and ends in a vicious attack on Miranda This opens the eye of Prospero who becomes severe to him and enforces his service by threats and violence

o Prospero uses him to make dams for fish to fetch firewood scraper trenches wash dishes and keep his cell clean

Calibanrsquos Hatred for ProsperoA profound hatred for Prospero has taken hold of Caliban It springs from a sense of his being dispossessed and ill-treated He would kill Prospero if he could but he knows the power of Prosperorsquos lsquobookrsquo Hence he transfers his allegiance to Stephano who seems like a god to him He also incites the two drunken associates to batter the skull of Prospero when he sleeps in the afternoon

Caliban Shows Considerable Intelligenceo He has learnt Prosperorsquos language

ldquoYou taught me language and my profit onrsquot (Act II Sc 2 Lines 86-89)Is I know how to curserdquo

o He is well aware of the futility of arguing with one who has more power than he has

ldquoI must obey his art is such power (Act I Sc 2 Lines 373-376)It would control my damrsquos god SetebosAnd make a vassal of himrdquo

o He realizes the importance of Prosperorsquos books

ldquoRemember (Act III Sc 2 Lines 89-92)First to possess his books for without themHersquos but a sot as I am nor hath notOne spirit to commandrdquo

o He knows the value of stealth when attacking the enemy

ldquoPray you tread softly that the blind mole may not (Act IV Sc 1 Lines 194-195)Hear a foot fall we now are near his cellrdquo

o Caliban has a better set of values than Stephano and Trinculo They are distracted from their plan by their greed for Prosperorsquos rich garments Only Caliban realizes that such a finery is unimportant

ldquoLeave it alone thou fool it is but trashrdquo (Act IV Sc 1 Lines 224)

Caliban is not a good judge of characterCaliban is not a good judge of character He decides for example that Stephano is a god because he dispenses lsquocelestial liquorrsquo (Act II Sc 2 Line 115) but then it must be remembered that he has only known his mother Sycorax Prospero Miranda and the spirits that torture him However he quickly discovers his error of judgementrdquo

ldquoWhat a thrice-double ass (Act V Sc 1 Lines 295-297)Was I to take this drunkard for a godAnd worship this dull foolrdquo

Calibanrsquos Imaginative NatureIf Caliban is sub-human in what has been said above he is human in the respect of the poetic side of his character He listens to music with rapture He tells of the beautiful dreams in which heaven rains treasures upon him and which upon waking he yearns to renew One of the most poetic passages in whole play is Calibanrsquos description of the island

to Stephano and Trinculo

ldquoBe not afeard The isle is full of noises (Act III Sc 2 Lines 135-143)Sounds and sweet airs that give delight and hurt notSometimes a thousand twangling instrumentsWill hum about mine ears and sometime voicesThat if I then had waked after long sleepWill make me sleep again and then in dreamingThe clouds methought would open and show richesReady to drop upon me that when I wakedI cried to dream againrdquo

Caliban - Less Ignoble Than Some OthersCalibanrsquos motive for murder is less dishonourable than that of Antonio and Sebastian They plan to kill Alonso to gain his power and wealth Caliban merely wants revenge and the return of lsquohisrsquo island

Conclusiono Calibanrsquos character is not portrayed very clearly in the play and hence we cannot decide whether he is a poor

savage being grossly maltreated by Prospero or whether he is evil and must therefore be kept in bondage or enslavement

o Caliban is contrasted with Ariel who is a spirit and thus swift and uninterested in physical activitieso Caliban is also contrasted with Prospero who is the all-powerful master of the island and of the destiny of all

those on the islando Caliban is also contrasted with civilized man showing him to be less evil than Antonio and Stephano and less

materialistic than Stephano and Trinculoo Caliban has suffered at the hands of Prospero and he has learnt to curse by listening to Prosperorsquos abuse He

certainly believes that Prospero has deprived him of his birthrighto Finally the character Caliban is thought to be one of Shakespearersquos masterpieces The complexity of the character

is reflected in the large volume of critical discussion that has grown around it

ECO ndash12 Topic-Forms of market

MonopolyMonopoly is a market structure in which there is a single seller there are no close substitutes for the commodity produced by the firm and there are barriers to entry Example Indian Railways which is operated under government of India Monopoly also implies absence of competitionFeatures of Monopoly Monopoly is characterized by1 Single Seller In monopoly there is only one firm producing the product The whole industry consists of this single firm Thus under monopoly there is no distinction between firm and industry Being the only firm there is significant control of the firm over supply and price Thus under monopoly buyers do not have the option of buying the commodity from any other seller They have to buy the product from the firm or they can go without the commodity This fact gives immense control to the monopolist over the market

2No Close Substitute There are no close substitutes of the product produced by the monopolist firm If there are close substitutes of the product in the market it implies presence of more than one firm and hence no monopoly In order to ensure a total of control over the market by the monopolist firm it is assumed that there are no close substitutes of the product

3 No Entry amp Exit Monopoly can only exist when there is strong barriers before a new firm to enter the market In fact once a monopoly firm starts producing the product no other firm can produce the same One reason for this is the ability of the

monopolist to produce the product at a lower cost than any new firm who thinks to enter the market If a new firm who knows that it cannot produce at a lower cost than the monopolist then that firm will never enter the market for fear of losing out in competition Similarly the monopolist who is operating for a long time may be enjoying reputation among its customers and is in a better position to use the situation in its own benefit A new firm has to take long time to achieve this and so may not be interested to enter the market

4 Price Maker Being the single seller of the product the monopolist has full control over the pricing of the product On the other hand if there is a large number of buyers in the market so no single buyer exercises any significant influence over price determination Thus it is a sellerrsquos market So monopoly firm is a price maker

5 Price Discrimination Having considerable control over the market on account of being single seller with no entry of other firms the monopolist can exercise policy of price discrimination it means that the monopolist can sell different quantities of the same product to a consumer at different price or same quantity to different consumers at different prices by adjudging the standard of living of the consumer

6 Shape of Demand Curve Since a monopolist has full control over the price therefore he can sell more by lowering the price This makes the demand curve downward sloping

Subject Ac-12 290620 Topic- retirement Model sumThe Balance Sheet of Rohit Nisha and Sunil who are partners in a firm sharing profits according to their capitals as on 31st March 2014 was as under

Liabilities Amount Assets Amount (Rs) (` Rs)

Creditors 25000 Machinery 40000Bills Payable 13000 Building 90000General Reserve 22000 Debtors 30000Capital Less Provision for Rohit 60000 Bad debts 1000

29000 Nisha 40000 Stocks 23000 Sunil 40000 140000 Cash at Bank 18000

200000 200000

On the date of Balance Sheet Nisha retired from the firm and following adjustments were made(i) Building is appreciated by 20(ii) Provision for bad debts is increased to 5 on Debtors(iii) Machinery is depreciated by 10(iv) Goodwill of the firm is valued at Rs 56000 and the retiring partnerrsquos share is adjusted

(v) The capital of the new firm is fixed at Rs120000 Prepare Revaluation Account Capital Accounts of the partner and Balance Sheet of the new firm after Nisharsquos retirement Revaluation AccountDr Cr

Particulars Amount Particulars Amount (`Rs) (Rs`)

Provision for Bad debt Ac 500 Building Ac 18000Machinery Ac 4000Profit transferred toCapital Accounts (3 2 2)Rohit 5786Nisha 3857Sunil 3857

13500

18000 18000

Capital Account

Dr Cr

Particulars Rohit Nisha Sunil Particulars Rohit Nisha Sunil (Rs`) (Rs`) (`Rs) (Rs`) (Rs`) (Rs`)

Sunilrsquos Capital ac 9600 mdash 6400 Balance bd 60000 40000 40000Bank - 66143 - General Reserve 9428 6286 6286Balance cd 72000 mdash 48000 Revaluation (Profi 5786 3857 3857 Rohitrsquos Capital Ac mdash 9600 mdash

Sunilrsquos Capital Ac 6400 Bank 6386 - 4257

81600 66143 54400 81600 66143 54400

Balance Sheet as at 31st March 2014

Liabilities Amount Assets Amount (Rs`) (Rs`)

Creditors 25000 Building 108000Bank overdraft 37500 Machinery 36000

Bills Payable 13000 Debtors 30000Capital Less ProvisionRohit 72000 for Bad debts 1500 28500Sunil 48000 120000 Stock 23000

195500 195500

Working Notes (i) (a) Profit sharing ratio is 60000 40000 40000 ie = 3 2 2(b) Gaining Ratio Rohit = 35 ndash 37 = 2135 ndash 1535 = 635Sunil = 25-27 = 1435 ndash 1035 = 435= 635 435= 6 4 = 3 2(c) Nisha Share of Goodwill = Rs 56000 times 27 = Rs16000Share of Goodwill in the gaining ratio by the existing partner ieRohit = Rs16000 times 35 = Rs 9600Sunil = Rs 16000 times 25 = Rs 6400

The journal entry isRohitrsquos Capital Ac Dr 9600Sunilrsquos Capital Ac Dr 6400 To Nisharsquos Capital Ac 16000(Share of Goodwill divided into gaining ratio)

  • 1 Static Friction
  • The frictional force that acts between the surfaces when they are at rest with respect to each other is called Static Friction
    • Static Friction Examples
      • 2 Sliding Friction
        • Examples Of Sliding Friction
          • 3 Rolling Friction
            • Examples Of Rolling Friction
              • Objects and Reasons of the Forest Conservation Act
Page 40:  · Web viewSubject . Topic . Summary . Execution . English 1 . Sounds of animals . Hens –cackle Horses –neigh Lions –roar Owls –hoots Snake –hiss. English 2 . Mother’s

Subject Eng Literature (The Merchant of Venice ndash William Shakespeare)Topic Act V Scene 1 Lines 127 to 158 (Nerissa helliphellip The clerk will nersquoer wear hair onrsquos face that had it) [Students should read the original play and also the paraphrase given in the school prescribed textbook]

Summary Revision Questions o Soon thereafter Bassanio Gratiano

and Antonio arrive

o Bassanio tells Portia that he is feeling as if it is morning because of the presence of Portia who is shining like the sun When Antonio is introduced by Bassanio to Portia she tells Bassanio that he should be grateful to Antonio who took so much trouble on his account even to the extent of risking his life

o Nerissa starts quarrelling with Gratiano and demands that he show her the ring she had presented to him and which she had warned him not to lose She suspects that Gratiano must have presented the ring to some young woman and not to the lawyerrsquos clerk as he repeatedly says and assures

Answer the following questions to check your preparation of Act IV Scenes 1 and 2

You must attempt only after you have completed your preparation of Act IV The answers must be in complete sentences using textual evidence (with citation) when necessary

[It would be in your own interest to attempt the above questions honestly totally refraining from consulting your textbook or your notes during answering After completion you should correct the paper yourself consulting the textbooknotes etc and award marks as specified Please let me know the marks you scored through WhatsApp in the group or to my personal WhatsApp]

Act IV Scene 1 (each question carries 2 marks)

1 What did the Duke try to do for Antonio

2 Why does Shylock refuse to show mercy How does he justify his stance

3 Why does Antonio say he is ready to die 4 What information is contained in Bellariorsquos letter

5 Why does Portia (as Balthazar) assert that Shylock must show mercy How does he respond

6 What offers are made to Shylock to get him to spare Antonio How are they received

7 What does Antoniorsquos speech as he faces the prospect of Shylockrsquos knife tell you about his character

8 How do Bassanio and Gratiano react to the looming prospect of Antoniorsquos demise

9 How does Portia (as Balthazar) use the law to turn the tables on Shylock

10 What does the Duke decree should happen to Shylock Why What happens to Shylockrsquos estate

11 What does Portia ask Bassanio as payment for her ldquoservicesrdquo What is his initial response What makes him change his mind

Act IV Scene 2 (each question carries 1frac12 marks)

1 What does Gratiano bring to Portia (Balthazar)

2 What does Nerissa plan on getting from Gratiano What does Portiarsquos comment suggest about men

ECO-10 280620 Topic-Supply AnalysisSHIFTING OF SUPPLY

But if there is change in factors other than the price of the commodity then either more is supplied at the same price or less supplied at the same price In such cases the price of the commodity remains constant but there is a change in other factors like change in the price of inputs change in technology of production change in price of other related goods change in taxation policy of the government etc For example there is an improvement in the technology of production of the commodity in question It leads to decrease in per unit of cost production of the commodity The firm is willing to sell more quantity of the commodity at the same price So the supply other commodity increases at the same price This increase in supply is shown by rightward shift of supply curve On the other hand if the firm uses inferior technology of production the cost of production per unit of the commodity increases The firm is willing to sell less quantity at the same price So the supply of the commodity decreases at the same price This decrease in supply is shown by leftward shift of the supply curve The above cases of increase and decrease in supply can be shown with the help of the following figures

Y INCREASE OF SUPPLY Price (Rs) s

P A s1

B

s

X` O s1 X

q q1

Y` Quantity demanded (in units)

Y DECREASE IN SUPPLY s2

s

price (Rs)

C

p A

s2

s

X` o X

q2 q

Y` Quantity demanded ( in units)

Main factors causing increase in supply or rightward shift of supply Curve(i) Fall in the price of other related goods

(ii) Fall in the price of inputsfactors(iii) Use of better technology in production(iv) Decrease in the rate of excise duty by government(v) If the objective of producer changes from profit maximization to salesMaximization

Main factors causing decrease in supply or leftward shift of supply curve(i) Increase in the price of other related goods(ii) Rise in the price of inputsfactors(iii) Use of inferior technology in production(iv) Increase in the rate of excise duty by the government(v) If the objective

Subject - Biology Topic ndash Chapter mdash6 PhotosynthesisSummary Execution

Today we will know about photosynthesis and its stages

Q1 What do you mean by photosynthesis The process by which living plants containing chlorophyll produce food

substances from carbon-di- oxide and water by using light energy Sunlight

6CO2 +12 H2O----------------------- C6 H12O6 + 6H2O + 6O2

Chlorophyll

Q2 What are the importance of photosynthesis I) Food for all Green plants trap solar energy by photosynthesis

process and supply food and energy for all living organisms either directly or indirectly

Ii) Oxygen to breathe in by product of photosynthesis is oxygen which is essential for all living organisms respiration

Q3 Write about two main phases of photosynthesis A Light dependent phase This phase occur in grana of chloroplast I) The chlorophyll on exposure to light energy becomes activated by

absorbing photons Ii) The absorbed energy is used in splitting the water molecules (H2O)

into its two components (H+ and OH- ) and releasing electron s 2H2O------------------------- 4H+ + 4e- +O2

Energy of 4 photons This reaction is known as photolysis

End products are H+ and oxygen water

B Light independent (Dark ) phase The reactions in this phase require no light energy

Here CO2 combine with H+ and produce glucose

Class XI

Subject Topic Summary ExecutionEVS Chapter-4 Legal

regimes for sustainable development

Environmental legislationEnvironmental legislation is the collection of laws and regulations pertaining to air quality water quality the wilderness endangered wildlife and other environmental factors The act ensures that matters important to the environment are thoroughly

Learn -The Forest (Conservation) Act 1980

considered in any decisions made by federal agencies

The Forest (Conservation) Act 1980 The Forest (Conservation) Act 1980 an Act of the Parliament of India to provide for the conservation of forests and for matters connected therewith or ancillary or incidental thereto It was further amended in 1988 This law extends to the whole of IndiaObjects and Reasons of the Forest Conservation Act

Deforestation causes ecological imbalance and leads to environmental deterioration Deforestation had been taking place on a large scale in the country and it had caused widespread concern The act seeks to check upon deforestation and de-reservation of forests

Subject Eng Literature (The Tempest ndash William Shakespeare) Topic Act II Scene 1 Lines 314 to 329 (End of scene)

[Students should read the original play and also the paraphrase given in the school prescribed textbook]Summary Questions amp Answers

Conspiracy of Antonio and Sebastian (Contd)

o As they approach Ariel appears again and wakes up Gonzalo by singing a tune in his ear Alonso also wakes up and they see both Sebastian and Antonio with drawn swords On being caught off guard they make up a story saying that they had heard a bellowing of bulls or lions

o They then moved to another part of the island

o Ariel at once rushes to Prospero to inform him of this development

SUMMING-UP of ACT-2 SCENE-1

(i) Among the survivors Ferdinand is separated from the rest which results in the disconsolate grief of Alonso as he took him for dead

(ii) The villainy of Antonio is confirmed

(iii) The supremacy of Prosperorsquos magic which resulted in the failure of the human conspiracy

(1)

(Act II Sc 1 L 311-325)SEBASTIAN Whiles we stood here securing your repose

Even now we heard a hollow burst of bellowing Like bulls or rather lions Didt not wake youIt struck mine ear most terribly

ALONSO I heard nothingANTONIO O rsquotwas a din to fright a monsters ear

To make an earthquake Sure it was the roarOf a whole herd of lions

ALONSO Heard you this GonzaloGONZALO Upon mine honour sir I heard a humming

And that a strange one too which did awake meI shaked you sir and cried As mine eyes opened I saw their weapons drawn There was a noiseThats verily rsquoTis best we stand upon our guardOr that we quit this place Lets draw our weapons

(i) Why has Prospero sent Ariel to Gonzalo and Alonso What does Ariel do to awaken Gonzalo

Prospero has already come to know by his magic powers the danger which threatens Gonzalo who had been Prosperorsquos friend and so he sent Ariel to preserve the lives of both Gonzalo and Alonso Prospero does not want that his scheme should remain unfulfilled Ariel begins to sing a song in Gonzalorsquos ears to awaken him(ii) Who are ready to carry out their plan Who takes steps to stop them Why does Gonzalo feel surprised after being awakened

Sebastian and Antonio are ready to carry out their plans They are standing with their swords drawn to kill Alonso and

(iv) We see two sets of contrasting characters Gonzalo-Adrian against Antonio-Sebastian

(v) The grief that works in Alonso can be perceived to his repentance for his association in Antoniorsquos crime against Prospero

Gonzalo Ariel takes steps to stop them from carrying out their nefarious scheme When Gonzalo is awakened by the song sung by Ariel into his ears he (Gonzalo) feels surprised because he sees Sebastian and Antonio standing with their swords drawn(iii) What reason do Sebastian and Antonio tell of drawing their swords when they are suspected by Alonso and Gonzalo

When Sebastian and Antonio are seen with their swords drawn they are looked with suspicion by Gonzalo and Alonso At first Sebastian tells them that as they stood here to guard them during their sleep they heard only a little before a sudden loud noise very much like the roaring of bulls or more probably that of lions Then Antonio follows him saying that this was a noise so terrible as to frighten even a monsterrsquos ears and this noise could even have shaken the earth and it was surely like the roaring of a multitude of lions Then seeing the danger they have drawn their swords Perhaps after hearing the terrible noise they (Gonzalo and Alonso) woke up from their sound sleep

(iv) What does Gonzalo tell Alonso about the strange noise What did he see on opening his eyes Gonzalo tells Alonso that he did not hear the sound of roaring but he heard a humming sound which was strange and which woke him up After waking up he gave him (Alonso) a shaking and a loud cry On opening his eyes he saw these two gentlemen standing with their swords drawn(v) What does Gonzalo suggest

Gonzalo suggests that there was a noise indeed and of that he has no doubt at all and suggests that the best course for them would be to remain alert and vigilant against any possible danger to their lives or to leave this place and move to some other part of the island

Class XIISubject Topic Summary Execution

Commerce

Chapter- Management

Today we will discuss about LEVELS OF MANAGEMENT

Levels of management is a series or chain of managerial positions from top to bottom It helps individuals to know their authority responsibilities and superior-subordinate relations among themselves There are mainly three levels of Management TOP LEVEL MANAGEMENTMIDDLE LEVEL MANAGEMENTLOWER LEVEL MANAGEMENT

Top level managementIt consists of members at the highest level in the management hierarchy This level includes Board Of Directors Chief Executive Managing Directors Chairman President Vice President

Rolefunctions of the top levelmanagement1To analyse evaluate and deal

with theexternal environment2 To determine the objectives and

policies of the business3 To strive for welfare and survival

of business

4 To create an organisational Framework consisting of authority responsibility relationship

Middle level management Congress of members or groups who are concerned with implementation of the policies let down by the top managementThis level includes head of the department such as finance manager marketing manager branch and regional managers departmental and divisional heads plant superintendent etc

Role of functions of the middle level management

1 To interpret the policies framed by top management

2 To assign duties and responsibilities to lower level managers

3 To select and appoint employees for middle and supervisory level and evaluate their performance

4 To co-operate with other departments for smooth functioning

Operational or supervisory level managementIt refers to the group are members who are concerned with execution of the work They are also known as fast line managers This level includes supervisor 4 men Section Officer clerk Inspector etc

Role of functions of the lower level management1 To plan and execute day-to-

day operations2 To supervise and control the workers3 To arrange materials and

tools to start the process and make arrangements for training

4 Today present workers grievance and suggestions before the management and

ensure safe and proper working conditions in the factory

Business Studies

Staff Appraisal Chapter- 10 Today let us start with a new chapter

Staff Appraisal

Meaning of Performance Appraisal

Performance Appraisal is the systematic evaluation of the performance of employees and to understand the abilities of a person for further growth and developmentThe supervisors measure the pay of employees and compare it with targets and plansThe supervisor analyses the factors behind work performances of employeesThe employers are in position to guide the employees for a better performance

Objectives of Performance Appraisal

Following are the objectives of Performance Appraisal

To maintain records in order to determine compensation packages wage structure salaries raises etc

To identify the strengths and weaknesses of employees to place right men on right job

To maintain and assess the potential present in a person for further growth and development

To provide a feedback to employees regarding their performance and related status

To provide a feedback to employees regarding their performance and related status

Importance of Performance Appraisal

Performance appraisal provides important and useful information for the assessment of employees skill

knowledge ability and overall job performance The following are the points which indicate the importance of performance appraisal in an organization

1 Performance appraisal helps supervisors to assess the work performance of their subordinates

2 Performance appraisal helps to assess the training and development needs of employees

3 Performance appraisal provides grounds for employees to correct their mistakes and it also provides proper guidance and criticism for employees development4 Performance appraisal provides reward for better performance

5 Performance appraisal helps to improve the communication system of the organization

6 Performance appraisal evaluates whether human resource programs being implemented in the organization have been effective

7 Performance appraisal helps to prepare pay structure for each employee working in the organization

8 Performance appraisal helps to review the potentiality of employees so that their future capability is anticipated

Geography

DRIANAGE The SubarnarekhaThe Subarnarekha and the Brahmaniinterposed between the Ganga and the Mahanadi deltas drain an area of 19300 sq kmand 39033 sq km respectively The drainage basins of these streams are shared byJharkhand Odisha west Bengal and Chhattisgarh The Brahmani is known as southKoel in its upper reaches in Jharkhand

The NarmadaThe Narmada rises in the Amarkantak hills of MadhyaPradesh It flows towards the West in a rift valleyformed due to a geological fault The total length of it is 1300 km All the tributaries of the

Q1 Name the two westward flowing rivers in the peninsular plateauA1 Narmada and Tapi are the only westward flowing rivers of the peninsular plateau

Q2 Differentiate between east-flowing rivers and west-flowing riversA2

East-flowing rivers

West-flowing rivers

Narmada are very short inlength Most of its tributaries join the main streamright anglesThe Narmada basin covers parts of Madhya Pradesh and Gujarat

The Tapi The Tapi rises in the Satpura ranges in the Betul listrictof Madhya Pradesh It flows in a rift valley parallel tothe Narmada but it is much shorter in length It coversparts of Madhya Pradesh Gujarat and MaharashtraThe length is about 724 km

The Sabarmati and the MahiThe Sabarmati rises in the Aravali hills and flows south-south-westwards for a distance of 300 kilometres to the Arabian Sea The Sabarmatibasin extends over an area of 21674 sq km in Rajasthan and Gujarat The Mahi rises inthe east of Udaipur and drains an area of 34842 sq km lying in Madhya PradeshRajasthan and Gujarat It flows south-westwards for a distance of 533 km before it fallsinto the Gulf of Khambhat

The ChambalThe Chambal rises near Mhow in the Vindhya Range and flows towards the northgenerally in a gorge upto Kota Below Kota it turns to the north-east direction and afterreaching Pinahat it turns to the east and runs nearly parallel to the Yamuna beforejoining it in the southern part of the Etawah district in Uttar PradeshMajor Rivers of India with their basin area (Sqkm)

Himalayan System Indus 321290Ganga 861404

Brahmaputra 187110Indus System

Jhelum 34775Beas 20303

Ganga System Yamuna 366223Ghaghra 127950

Peninsular RiversNarmada 98796

Tapi 65145Mahanadi 141600

Subarnarekha 19300Sabarmati 21674

Mahi 34842Godavari 312812

Godavari Krishna Kaveri Mahanadi are the east-flowing rivers

Narmada Tapi west-flowing rivers

They fall into the Bay of Bengal

They fall into Arabian Sea

These rivers form big deltas

These rivers form comparativelysmall deltas

Catchment areas of these rivers are larger

Catchment areas of these rivers are smaller

Krishna 2589488Cauveri 87900

Subject ndashBiology Topic ndashChapter -5 Inheritance amp Variations Summary ExecutionToday we will discussabout linkage and its classification

LINKAGE The tendency of the genes located on the same chromosome to stay together is

hereditary transmission Linked genes the genes responsible for this Genes that exhibit the process of linkage locates in the same chromosome The distance between the linked genes in a chromosome determines the strength

of linkage i e genes that are located close to each other show stronger linkage than that are located far from each other

COMPLETE LINKAGE It is the type of linkage showed by the genes that are closely located or are tightly

linked with each other as they have no chance of separatingby crossing over These genes are always transmitted together to the same gamete and the same

offspring In such condition only parental or non cross over type of gametes are formedINCOMPLETE KINKAGE It is type of linkage showed by the genes that are distantly located orare loosely

linked with each other because they have chance of separating by crossing over

SIGNIFICANCE i) It helps in holding the parental character togetherii) It checks the appearance of new recombination and helps in bringing the

hybrid population which resembles the original parents iii) Linked genes dilute the effects of undesirable traits

Subject Eng Literature (The Tempest ndash William Shakespeare) Topic Essay Questions (EQ-3)Question No 3

Give a character sketch of CalibanAnswer

The character of Caliban has been wonderfully conceived by Shakespeare as the manifestation of all that is gross and earthy ndash a sort of creature of the earth as Ariel is a sort of creature of the air

Calibanrsquos Physical Appearanceo Caliban is lsquofreckledrsquo a lsquomisshapen knaversquo not honoured with human shape

o Prospero calls him lsquothou tortoisersquo (Act I Sc 2 Line 317) Trinculo stumbling upon him describes him as ldquoA strange fish hellip Legged like a man And his fins like armsrdquo He ldquosmells like a fishrdquo (Act II Sc 2 Line 25)

o Prospero also calls him a ldquobeastrdquo (Act IV Sc 1 Line 140) and ldquoThis misshapen knaverdquo (Act V Sc 1 Line 268)

o Further it appears that in addition to his physical deformity his spiritual inferiority is also suggested by Prosperorsquos claim that his birth resulted from the union between his mother the witch Sycorax and the devil

Calibanrsquos ParentageWhen the play opens Caliban is twenty four years of age having been born on the island twelve years before the coming of Prospero His mother was the foul witch Sycorax who was banished from Algiers for ldquomischiefs manifold and sorceries terrible to enter human hearingrdquo (Act I Sc 2 Line 264) and the father was the Devil himself Thus

Caliban is a monster of evil and brute nature ugly deformed and stinking

Calibanrsquos Savage and Malignant Natureo Caliban is entirely a creature of the earth ndash gross brutal and savage He regards himself as the rightful possessor

of the island and Prospero as a usurper

o In his young age he was on good terms with Prospero He had consented to be received by Prospero at his house and to be educated by him He has learnt human language only to curse his master whom he abhors

o His beastly nature soon breaks out and ends in a vicious attack on Miranda This opens the eye of Prospero who becomes severe to him and enforces his service by threats and violence

o Prospero uses him to make dams for fish to fetch firewood scraper trenches wash dishes and keep his cell clean

Calibanrsquos Hatred for ProsperoA profound hatred for Prospero has taken hold of Caliban It springs from a sense of his being dispossessed and ill-treated He would kill Prospero if he could but he knows the power of Prosperorsquos lsquobookrsquo Hence he transfers his allegiance to Stephano who seems like a god to him He also incites the two drunken associates to batter the skull of Prospero when he sleeps in the afternoon

Caliban Shows Considerable Intelligenceo He has learnt Prosperorsquos language

ldquoYou taught me language and my profit onrsquot (Act II Sc 2 Lines 86-89)Is I know how to curserdquo

o He is well aware of the futility of arguing with one who has more power than he has

ldquoI must obey his art is such power (Act I Sc 2 Lines 373-376)It would control my damrsquos god SetebosAnd make a vassal of himrdquo

o He realizes the importance of Prosperorsquos books

ldquoRemember (Act III Sc 2 Lines 89-92)First to possess his books for without themHersquos but a sot as I am nor hath notOne spirit to commandrdquo

o He knows the value of stealth when attacking the enemy

ldquoPray you tread softly that the blind mole may not (Act IV Sc 1 Lines 194-195)Hear a foot fall we now are near his cellrdquo

o Caliban has a better set of values than Stephano and Trinculo They are distracted from their plan by their greed for Prosperorsquos rich garments Only Caliban realizes that such a finery is unimportant

ldquoLeave it alone thou fool it is but trashrdquo (Act IV Sc 1 Lines 224)

Caliban is not a good judge of characterCaliban is not a good judge of character He decides for example that Stephano is a god because he dispenses lsquocelestial liquorrsquo (Act II Sc 2 Line 115) but then it must be remembered that he has only known his mother Sycorax Prospero Miranda and the spirits that torture him However he quickly discovers his error of judgementrdquo

ldquoWhat a thrice-double ass (Act V Sc 1 Lines 295-297)Was I to take this drunkard for a godAnd worship this dull foolrdquo

Calibanrsquos Imaginative NatureIf Caliban is sub-human in what has been said above he is human in the respect of the poetic side of his character He listens to music with rapture He tells of the beautiful dreams in which heaven rains treasures upon him and which upon waking he yearns to renew One of the most poetic passages in whole play is Calibanrsquos description of the island

to Stephano and Trinculo

ldquoBe not afeard The isle is full of noises (Act III Sc 2 Lines 135-143)Sounds and sweet airs that give delight and hurt notSometimes a thousand twangling instrumentsWill hum about mine ears and sometime voicesThat if I then had waked after long sleepWill make me sleep again and then in dreamingThe clouds methought would open and show richesReady to drop upon me that when I wakedI cried to dream againrdquo

Caliban - Less Ignoble Than Some OthersCalibanrsquos motive for murder is less dishonourable than that of Antonio and Sebastian They plan to kill Alonso to gain his power and wealth Caliban merely wants revenge and the return of lsquohisrsquo island

Conclusiono Calibanrsquos character is not portrayed very clearly in the play and hence we cannot decide whether he is a poor

savage being grossly maltreated by Prospero or whether he is evil and must therefore be kept in bondage or enslavement

o Caliban is contrasted with Ariel who is a spirit and thus swift and uninterested in physical activitieso Caliban is also contrasted with Prospero who is the all-powerful master of the island and of the destiny of all

those on the islando Caliban is also contrasted with civilized man showing him to be less evil than Antonio and Stephano and less

materialistic than Stephano and Trinculoo Caliban has suffered at the hands of Prospero and he has learnt to curse by listening to Prosperorsquos abuse He

certainly believes that Prospero has deprived him of his birthrighto Finally the character Caliban is thought to be one of Shakespearersquos masterpieces The complexity of the character

is reflected in the large volume of critical discussion that has grown around it

ECO ndash12 Topic-Forms of market

MonopolyMonopoly is a market structure in which there is a single seller there are no close substitutes for the commodity produced by the firm and there are barriers to entry Example Indian Railways which is operated under government of India Monopoly also implies absence of competitionFeatures of Monopoly Monopoly is characterized by1 Single Seller In monopoly there is only one firm producing the product The whole industry consists of this single firm Thus under monopoly there is no distinction between firm and industry Being the only firm there is significant control of the firm over supply and price Thus under monopoly buyers do not have the option of buying the commodity from any other seller They have to buy the product from the firm or they can go without the commodity This fact gives immense control to the monopolist over the market

2No Close Substitute There are no close substitutes of the product produced by the monopolist firm If there are close substitutes of the product in the market it implies presence of more than one firm and hence no monopoly In order to ensure a total of control over the market by the monopolist firm it is assumed that there are no close substitutes of the product

3 No Entry amp Exit Monopoly can only exist when there is strong barriers before a new firm to enter the market In fact once a monopoly firm starts producing the product no other firm can produce the same One reason for this is the ability of the

monopolist to produce the product at a lower cost than any new firm who thinks to enter the market If a new firm who knows that it cannot produce at a lower cost than the monopolist then that firm will never enter the market for fear of losing out in competition Similarly the monopolist who is operating for a long time may be enjoying reputation among its customers and is in a better position to use the situation in its own benefit A new firm has to take long time to achieve this and so may not be interested to enter the market

4 Price Maker Being the single seller of the product the monopolist has full control over the pricing of the product On the other hand if there is a large number of buyers in the market so no single buyer exercises any significant influence over price determination Thus it is a sellerrsquos market So monopoly firm is a price maker

5 Price Discrimination Having considerable control over the market on account of being single seller with no entry of other firms the monopolist can exercise policy of price discrimination it means that the monopolist can sell different quantities of the same product to a consumer at different price or same quantity to different consumers at different prices by adjudging the standard of living of the consumer

6 Shape of Demand Curve Since a monopolist has full control over the price therefore he can sell more by lowering the price This makes the demand curve downward sloping

Subject Ac-12 290620 Topic- retirement Model sumThe Balance Sheet of Rohit Nisha and Sunil who are partners in a firm sharing profits according to their capitals as on 31st March 2014 was as under

Liabilities Amount Assets Amount (Rs) (` Rs)

Creditors 25000 Machinery 40000Bills Payable 13000 Building 90000General Reserve 22000 Debtors 30000Capital Less Provision for Rohit 60000 Bad debts 1000

29000 Nisha 40000 Stocks 23000 Sunil 40000 140000 Cash at Bank 18000

200000 200000

On the date of Balance Sheet Nisha retired from the firm and following adjustments were made(i) Building is appreciated by 20(ii) Provision for bad debts is increased to 5 on Debtors(iii) Machinery is depreciated by 10(iv) Goodwill of the firm is valued at Rs 56000 and the retiring partnerrsquos share is adjusted

(v) The capital of the new firm is fixed at Rs120000 Prepare Revaluation Account Capital Accounts of the partner and Balance Sheet of the new firm after Nisharsquos retirement Revaluation AccountDr Cr

Particulars Amount Particulars Amount (`Rs) (Rs`)

Provision for Bad debt Ac 500 Building Ac 18000Machinery Ac 4000Profit transferred toCapital Accounts (3 2 2)Rohit 5786Nisha 3857Sunil 3857

13500

18000 18000

Capital Account

Dr Cr

Particulars Rohit Nisha Sunil Particulars Rohit Nisha Sunil (Rs`) (Rs`) (`Rs) (Rs`) (Rs`) (Rs`)

Sunilrsquos Capital ac 9600 mdash 6400 Balance bd 60000 40000 40000Bank - 66143 - General Reserve 9428 6286 6286Balance cd 72000 mdash 48000 Revaluation (Profi 5786 3857 3857 Rohitrsquos Capital Ac mdash 9600 mdash

Sunilrsquos Capital Ac 6400 Bank 6386 - 4257

81600 66143 54400 81600 66143 54400

Balance Sheet as at 31st March 2014

Liabilities Amount Assets Amount (Rs`) (Rs`)

Creditors 25000 Building 108000Bank overdraft 37500 Machinery 36000

Bills Payable 13000 Debtors 30000Capital Less ProvisionRohit 72000 for Bad debts 1500 28500Sunil 48000 120000 Stock 23000

195500 195500

Working Notes (i) (a) Profit sharing ratio is 60000 40000 40000 ie = 3 2 2(b) Gaining Ratio Rohit = 35 ndash 37 = 2135 ndash 1535 = 635Sunil = 25-27 = 1435 ndash 1035 = 435= 635 435= 6 4 = 3 2(c) Nisha Share of Goodwill = Rs 56000 times 27 = Rs16000Share of Goodwill in the gaining ratio by the existing partner ieRohit = Rs16000 times 35 = Rs 9600Sunil = Rs 16000 times 25 = Rs 6400

The journal entry isRohitrsquos Capital Ac Dr 9600Sunilrsquos Capital Ac Dr 6400 To Nisharsquos Capital Ac 16000(Share of Goodwill divided into gaining ratio)

  • 1 Static Friction
  • The frictional force that acts between the surfaces when they are at rest with respect to each other is called Static Friction
    • Static Friction Examples
      • 2 Sliding Friction
        • Examples Of Sliding Friction
          • 3 Rolling Friction
            • Examples Of Rolling Friction
              • Objects and Reasons of the Forest Conservation Act
Page 41:  · Web viewSubject . Topic . Summary . Execution . English 1 . Sounds of animals . Hens –cackle Horses –neigh Lions –roar Owls –hoots Snake –hiss. English 2 . Mother’s

But if there is change in factors other than the price of the commodity then either more is supplied at the same price or less supplied at the same price In such cases the price of the commodity remains constant but there is a change in other factors like change in the price of inputs change in technology of production change in price of other related goods change in taxation policy of the government etc For example there is an improvement in the technology of production of the commodity in question It leads to decrease in per unit of cost production of the commodity The firm is willing to sell more quantity of the commodity at the same price So the supply other commodity increases at the same price This increase in supply is shown by rightward shift of supply curve On the other hand if the firm uses inferior technology of production the cost of production per unit of the commodity increases The firm is willing to sell less quantity at the same price So the supply of the commodity decreases at the same price This decrease in supply is shown by leftward shift of the supply curve The above cases of increase and decrease in supply can be shown with the help of the following figures

Y INCREASE OF SUPPLY Price (Rs) s

P A s1

B

s

X` O s1 X

q q1

Y` Quantity demanded (in units)

Y DECREASE IN SUPPLY s2

s

price (Rs)

C

p A

s2

s

X` o X

q2 q

Y` Quantity demanded ( in units)

Main factors causing increase in supply or rightward shift of supply Curve(i) Fall in the price of other related goods

(ii) Fall in the price of inputsfactors(iii) Use of better technology in production(iv) Decrease in the rate of excise duty by government(v) If the objective of producer changes from profit maximization to salesMaximization

Main factors causing decrease in supply or leftward shift of supply curve(i) Increase in the price of other related goods(ii) Rise in the price of inputsfactors(iii) Use of inferior technology in production(iv) Increase in the rate of excise duty by the government(v) If the objective

Subject - Biology Topic ndash Chapter mdash6 PhotosynthesisSummary Execution

Today we will know about photosynthesis and its stages

Q1 What do you mean by photosynthesis The process by which living plants containing chlorophyll produce food

substances from carbon-di- oxide and water by using light energy Sunlight

6CO2 +12 H2O----------------------- C6 H12O6 + 6H2O + 6O2

Chlorophyll

Q2 What are the importance of photosynthesis I) Food for all Green plants trap solar energy by photosynthesis

process and supply food and energy for all living organisms either directly or indirectly

Ii) Oxygen to breathe in by product of photosynthesis is oxygen which is essential for all living organisms respiration

Q3 Write about two main phases of photosynthesis A Light dependent phase This phase occur in grana of chloroplast I) The chlorophyll on exposure to light energy becomes activated by

absorbing photons Ii) The absorbed energy is used in splitting the water molecules (H2O)

into its two components (H+ and OH- ) and releasing electron s 2H2O------------------------- 4H+ + 4e- +O2

Energy of 4 photons This reaction is known as photolysis

End products are H+ and oxygen water

B Light independent (Dark ) phase The reactions in this phase require no light energy

Here CO2 combine with H+ and produce glucose

Class XI

Subject Topic Summary ExecutionEVS Chapter-4 Legal

regimes for sustainable development

Environmental legislationEnvironmental legislation is the collection of laws and regulations pertaining to air quality water quality the wilderness endangered wildlife and other environmental factors The act ensures that matters important to the environment are thoroughly

Learn -The Forest (Conservation) Act 1980

considered in any decisions made by federal agencies

The Forest (Conservation) Act 1980 The Forest (Conservation) Act 1980 an Act of the Parliament of India to provide for the conservation of forests and for matters connected therewith or ancillary or incidental thereto It was further amended in 1988 This law extends to the whole of IndiaObjects and Reasons of the Forest Conservation Act

Deforestation causes ecological imbalance and leads to environmental deterioration Deforestation had been taking place on a large scale in the country and it had caused widespread concern The act seeks to check upon deforestation and de-reservation of forests

Subject Eng Literature (The Tempest ndash William Shakespeare) Topic Act II Scene 1 Lines 314 to 329 (End of scene)

[Students should read the original play and also the paraphrase given in the school prescribed textbook]Summary Questions amp Answers

Conspiracy of Antonio and Sebastian (Contd)

o As they approach Ariel appears again and wakes up Gonzalo by singing a tune in his ear Alonso also wakes up and they see both Sebastian and Antonio with drawn swords On being caught off guard they make up a story saying that they had heard a bellowing of bulls or lions

o They then moved to another part of the island

o Ariel at once rushes to Prospero to inform him of this development

SUMMING-UP of ACT-2 SCENE-1

(i) Among the survivors Ferdinand is separated from the rest which results in the disconsolate grief of Alonso as he took him for dead

(ii) The villainy of Antonio is confirmed

(iii) The supremacy of Prosperorsquos magic which resulted in the failure of the human conspiracy

(1)

(Act II Sc 1 L 311-325)SEBASTIAN Whiles we stood here securing your repose

Even now we heard a hollow burst of bellowing Like bulls or rather lions Didt not wake youIt struck mine ear most terribly

ALONSO I heard nothingANTONIO O rsquotwas a din to fright a monsters ear

To make an earthquake Sure it was the roarOf a whole herd of lions

ALONSO Heard you this GonzaloGONZALO Upon mine honour sir I heard a humming

And that a strange one too which did awake meI shaked you sir and cried As mine eyes opened I saw their weapons drawn There was a noiseThats verily rsquoTis best we stand upon our guardOr that we quit this place Lets draw our weapons

(i) Why has Prospero sent Ariel to Gonzalo and Alonso What does Ariel do to awaken Gonzalo

Prospero has already come to know by his magic powers the danger which threatens Gonzalo who had been Prosperorsquos friend and so he sent Ariel to preserve the lives of both Gonzalo and Alonso Prospero does not want that his scheme should remain unfulfilled Ariel begins to sing a song in Gonzalorsquos ears to awaken him(ii) Who are ready to carry out their plan Who takes steps to stop them Why does Gonzalo feel surprised after being awakened

Sebastian and Antonio are ready to carry out their plans They are standing with their swords drawn to kill Alonso and

(iv) We see two sets of contrasting characters Gonzalo-Adrian against Antonio-Sebastian

(v) The grief that works in Alonso can be perceived to his repentance for his association in Antoniorsquos crime against Prospero

Gonzalo Ariel takes steps to stop them from carrying out their nefarious scheme When Gonzalo is awakened by the song sung by Ariel into his ears he (Gonzalo) feels surprised because he sees Sebastian and Antonio standing with their swords drawn(iii) What reason do Sebastian and Antonio tell of drawing their swords when they are suspected by Alonso and Gonzalo

When Sebastian and Antonio are seen with their swords drawn they are looked with suspicion by Gonzalo and Alonso At first Sebastian tells them that as they stood here to guard them during their sleep they heard only a little before a sudden loud noise very much like the roaring of bulls or more probably that of lions Then Antonio follows him saying that this was a noise so terrible as to frighten even a monsterrsquos ears and this noise could even have shaken the earth and it was surely like the roaring of a multitude of lions Then seeing the danger they have drawn their swords Perhaps after hearing the terrible noise they (Gonzalo and Alonso) woke up from their sound sleep

(iv) What does Gonzalo tell Alonso about the strange noise What did he see on opening his eyes Gonzalo tells Alonso that he did not hear the sound of roaring but he heard a humming sound which was strange and which woke him up After waking up he gave him (Alonso) a shaking and a loud cry On opening his eyes he saw these two gentlemen standing with their swords drawn(v) What does Gonzalo suggest

Gonzalo suggests that there was a noise indeed and of that he has no doubt at all and suggests that the best course for them would be to remain alert and vigilant against any possible danger to their lives or to leave this place and move to some other part of the island

Class XIISubject Topic Summary Execution

Commerce

Chapter- Management

Today we will discuss about LEVELS OF MANAGEMENT

Levels of management is a series or chain of managerial positions from top to bottom It helps individuals to know their authority responsibilities and superior-subordinate relations among themselves There are mainly three levels of Management TOP LEVEL MANAGEMENTMIDDLE LEVEL MANAGEMENTLOWER LEVEL MANAGEMENT

Top level managementIt consists of members at the highest level in the management hierarchy This level includes Board Of Directors Chief Executive Managing Directors Chairman President Vice President

Rolefunctions of the top levelmanagement1To analyse evaluate and deal

with theexternal environment2 To determine the objectives and

policies of the business3 To strive for welfare and survival

of business

4 To create an organisational Framework consisting of authority responsibility relationship

Middle level management Congress of members or groups who are concerned with implementation of the policies let down by the top managementThis level includes head of the department such as finance manager marketing manager branch and regional managers departmental and divisional heads plant superintendent etc

Role of functions of the middle level management

1 To interpret the policies framed by top management

2 To assign duties and responsibilities to lower level managers

3 To select and appoint employees for middle and supervisory level and evaluate their performance

4 To co-operate with other departments for smooth functioning

Operational or supervisory level managementIt refers to the group are members who are concerned with execution of the work They are also known as fast line managers This level includes supervisor 4 men Section Officer clerk Inspector etc

Role of functions of the lower level management1 To plan and execute day-to-

day operations2 To supervise and control the workers3 To arrange materials and

tools to start the process and make arrangements for training

4 Today present workers grievance and suggestions before the management and

ensure safe and proper working conditions in the factory

Business Studies

Staff Appraisal Chapter- 10 Today let us start with a new chapter

Staff Appraisal

Meaning of Performance Appraisal

Performance Appraisal is the systematic evaluation of the performance of employees and to understand the abilities of a person for further growth and developmentThe supervisors measure the pay of employees and compare it with targets and plansThe supervisor analyses the factors behind work performances of employeesThe employers are in position to guide the employees for a better performance

Objectives of Performance Appraisal

Following are the objectives of Performance Appraisal

To maintain records in order to determine compensation packages wage structure salaries raises etc

To identify the strengths and weaknesses of employees to place right men on right job

To maintain and assess the potential present in a person for further growth and development

To provide a feedback to employees regarding their performance and related status

To provide a feedback to employees regarding their performance and related status

Importance of Performance Appraisal

Performance appraisal provides important and useful information for the assessment of employees skill

knowledge ability and overall job performance The following are the points which indicate the importance of performance appraisal in an organization

1 Performance appraisal helps supervisors to assess the work performance of their subordinates

2 Performance appraisal helps to assess the training and development needs of employees

3 Performance appraisal provides grounds for employees to correct their mistakes and it also provides proper guidance and criticism for employees development4 Performance appraisal provides reward for better performance

5 Performance appraisal helps to improve the communication system of the organization

6 Performance appraisal evaluates whether human resource programs being implemented in the organization have been effective

7 Performance appraisal helps to prepare pay structure for each employee working in the organization

8 Performance appraisal helps to review the potentiality of employees so that their future capability is anticipated

Geography

DRIANAGE The SubarnarekhaThe Subarnarekha and the Brahmaniinterposed between the Ganga and the Mahanadi deltas drain an area of 19300 sq kmand 39033 sq km respectively The drainage basins of these streams are shared byJharkhand Odisha west Bengal and Chhattisgarh The Brahmani is known as southKoel in its upper reaches in Jharkhand

The NarmadaThe Narmada rises in the Amarkantak hills of MadhyaPradesh It flows towards the West in a rift valleyformed due to a geological fault The total length of it is 1300 km All the tributaries of the

Q1 Name the two westward flowing rivers in the peninsular plateauA1 Narmada and Tapi are the only westward flowing rivers of the peninsular plateau

Q2 Differentiate between east-flowing rivers and west-flowing riversA2

East-flowing rivers

West-flowing rivers

Narmada are very short inlength Most of its tributaries join the main streamright anglesThe Narmada basin covers parts of Madhya Pradesh and Gujarat

The Tapi The Tapi rises in the Satpura ranges in the Betul listrictof Madhya Pradesh It flows in a rift valley parallel tothe Narmada but it is much shorter in length It coversparts of Madhya Pradesh Gujarat and MaharashtraThe length is about 724 km

The Sabarmati and the MahiThe Sabarmati rises in the Aravali hills and flows south-south-westwards for a distance of 300 kilometres to the Arabian Sea The Sabarmatibasin extends over an area of 21674 sq km in Rajasthan and Gujarat The Mahi rises inthe east of Udaipur and drains an area of 34842 sq km lying in Madhya PradeshRajasthan and Gujarat It flows south-westwards for a distance of 533 km before it fallsinto the Gulf of Khambhat

The ChambalThe Chambal rises near Mhow in the Vindhya Range and flows towards the northgenerally in a gorge upto Kota Below Kota it turns to the north-east direction and afterreaching Pinahat it turns to the east and runs nearly parallel to the Yamuna beforejoining it in the southern part of the Etawah district in Uttar PradeshMajor Rivers of India with their basin area (Sqkm)

Himalayan System Indus 321290Ganga 861404

Brahmaputra 187110Indus System

Jhelum 34775Beas 20303

Ganga System Yamuna 366223Ghaghra 127950

Peninsular RiversNarmada 98796

Tapi 65145Mahanadi 141600

Subarnarekha 19300Sabarmati 21674

Mahi 34842Godavari 312812

Godavari Krishna Kaveri Mahanadi are the east-flowing rivers

Narmada Tapi west-flowing rivers

They fall into the Bay of Bengal

They fall into Arabian Sea

These rivers form big deltas

These rivers form comparativelysmall deltas

Catchment areas of these rivers are larger

Catchment areas of these rivers are smaller

Krishna 2589488Cauveri 87900

Subject ndashBiology Topic ndashChapter -5 Inheritance amp Variations Summary ExecutionToday we will discussabout linkage and its classification

LINKAGE The tendency of the genes located on the same chromosome to stay together is

hereditary transmission Linked genes the genes responsible for this Genes that exhibit the process of linkage locates in the same chromosome The distance between the linked genes in a chromosome determines the strength

of linkage i e genes that are located close to each other show stronger linkage than that are located far from each other

COMPLETE LINKAGE It is the type of linkage showed by the genes that are closely located or are tightly

linked with each other as they have no chance of separatingby crossing over These genes are always transmitted together to the same gamete and the same

offspring In such condition only parental or non cross over type of gametes are formedINCOMPLETE KINKAGE It is type of linkage showed by the genes that are distantly located orare loosely

linked with each other because they have chance of separating by crossing over

SIGNIFICANCE i) It helps in holding the parental character togetherii) It checks the appearance of new recombination and helps in bringing the

hybrid population which resembles the original parents iii) Linked genes dilute the effects of undesirable traits

Subject Eng Literature (The Tempest ndash William Shakespeare) Topic Essay Questions (EQ-3)Question No 3

Give a character sketch of CalibanAnswer

The character of Caliban has been wonderfully conceived by Shakespeare as the manifestation of all that is gross and earthy ndash a sort of creature of the earth as Ariel is a sort of creature of the air

Calibanrsquos Physical Appearanceo Caliban is lsquofreckledrsquo a lsquomisshapen knaversquo not honoured with human shape

o Prospero calls him lsquothou tortoisersquo (Act I Sc 2 Line 317) Trinculo stumbling upon him describes him as ldquoA strange fish hellip Legged like a man And his fins like armsrdquo He ldquosmells like a fishrdquo (Act II Sc 2 Line 25)

o Prospero also calls him a ldquobeastrdquo (Act IV Sc 1 Line 140) and ldquoThis misshapen knaverdquo (Act V Sc 1 Line 268)

o Further it appears that in addition to his physical deformity his spiritual inferiority is also suggested by Prosperorsquos claim that his birth resulted from the union between his mother the witch Sycorax and the devil

Calibanrsquos ParentageWhen the play opens Caliban is twenty four years of age having been born on the island twelve years before the coming of Prospero His mother was the foul witch Sycorax who was banished from Algiers for ldquomischiefs manifold and sorceries terrible to enter human hearingrdquo (Act I Sc 2 Line 264) and the father was the Devil himself Thus

Caliban is a monster of evil and brute nature ugly deformed and stinking

Calibanrsquos Savage and Malignant Natureo Caliban is entirely a creature of the earth ndash gross brutal and savage He regards himself as the rightful possessor

of the island and Prospero as a usurper

o In his young age he was on good terms with Prospero He had consented to be received by Prospero at his house and to be educated by him He has learnt human language only to curse his master whom he abhors

o His beastly nature soon breaks out and ends in a vicious attack on Miranda This opens the eye of Prospero who becomes severe to him and enforces his service by threats and violence

o Prospero uses him to make dams for fish to fetch firewood scraper trenches wash dishes and keep his cell clean

Calibanrsquos Hatred for ProsperoA profound hatred for Prospero has taken hold of Caliban It springs from a sense of his being dispossessed and ill-treated He would kill Prospero if he could but he knows the power of Prosperorsquos lsquobookrsquo Hence he transfers his allegiance to Stephano who seems like a god to him He also incites the two drunken associates to batter the skull of Prospero when he sleeps in the afternoon

Caliban Shows Considerable Intelligenceo He has learnt Prosperorsquos language

ldquoYou taught me language and my profit onrsquot (Act II Sc 2 Lines 86-89)Is I know how to curserdquo

o He is well aware of the futility of arguing with one who has more power than he has

ldquoI must obey his art is such power (Act I Sc 2 Lines 373-376)It would control my damrsquos god SetebosAnd make a vassal of himrdquo

o He realizes the importance of Prosperorsquos books

ldquoRemember (Act III Sc 2 Lines 89-92)First to possess his books for without themHersquos but a sot as I am nor hath notOne spirit to commandrdquo

o He knows the value of stealth when attacking the enemy

ldquoPray you tread softly that the blind mole may not (Act IV Sc 1 Lines 194-195)Hear a foot fall we now are near his cellrdquo

o Caliban has a better set of values than Stephano and Trinculo They are distracted from their plan by their greed for Prosperorsquos rich garments Only Caliban realizes that such a finery is unimportant

ldquoLeave it alone thou fool it is but trashrdquo (Act IV Sc 1 Lines 224)

Caliban is not a good judge of characterCaliban is not a good judge of character He decides for example that Stephano is a god because he dispenses lsquocelestial liquorrsquo (Act II Sc 2 Line 115) but then it must be remembered that he has only known his mother Sycorax Prospero Miranda and the spirits that torture him However he quickly discovers his error of judgementrdquo

ldquoWhat a thrice-double ass (Act V Sc 1 Lines 295-297)Was I to take this drunkard for a godAnd worship this dull foolrdquo

Calibanrsquos Imaginative NatureIf Caliban is sub-human in what has been said above he is human in the respect of the poetic side of his character He listens to music with rapture He tells of the beautiful dreams in which heaven rains treasures upon him and which upon waking he yearns to renew One of the most poetic passages in whole play is Calibanrsquos description of the island

to Stephano and Trinculo

ldquoBe not afeard The isle is full of noises (Act III Sc 2 Lines 135-143)Sounds and sweet airs that give delight and hurt notSometimes a thousand twangling instrumentsWill hum about mine ears and sometime voicesThat if I then had waked after long sleepWill make me sleep again and then in dreamingThe clouds methought would open and show richesReady to drop upon me that when I wakedI cried to dream againrdquo

Caliban - Less Ignoble Than Some OthersCalibanrsquos motive for murder is less dishonourable than that of Antonio and Sebastian They plan to kill Alonso to gain his power and wealth Caliban merely wants revenge and the return of lsquohisrsquo island

Conclusiono Calibanrsquos character is not portrayed very clearly in the play and hence we cannot decide whether he is a poor

savage being grossly maltreated by Prospero or whether he is evil and must therefore be kept in bondage or enslavement

o Caliban is contrasted with Ariel who is a spirit and thus swift and uninterested in physical activitieso Caliban is also contrasted with Prospero who is the all-powerful master of the island and of the destiny of all

those on the islando Caliban is also contrasted with civilized man showing him to be less evil than Antonio and Stephano and less

materialistic than Stephano and Trinculoo Caliban has suffered at the hands of Prospero and he has learnt to curse by listening to Prosperorsquos abuse He

certainly believes that Prospero has deprived him of his birthrighto Finally the character Caliban is thought to be one of Shakespearersquos masterpieces The complexity of the character

is reflected in the large volume of critical discussion that has grown around it

ECO ndash12 Topic-Forms of market

MonopolyMonopoly is a market structure in which there is a single seller there are no close substitutes for the commodity produced by the firm and there are barriers to entry Example Indian Railways which is operated under government of India Monopoly also implies absence of competitionFeatures of Monopoly Monopoly is characterized by1 Single Seller In monopoly there is only one firm producing the product The whole industry consists of this single firm Thus under monopoly there is no distinction between firm and industry Being the only firm there is significant control of the firm over supply and price Thus under monopoly buyers do not have the option of buying the commodity from any other seller They have to buy the product from the firm or they can go without the commodity This fact gives immense control to the monopolist over the market

2No Close Substitute There are no close substitutes of the product produced by the monopolist firm If there are close substitutes of the product in the market it implies presence of more than one firm and hence no monopoly In order to ensure a total of control over the market by the monopolist firm it is assumed that there are no close substitutes of the product

3 No Entry amp Exit Monopoly can only exist when there is strong barriers before a new firm to enter the market In fact once a monopoly firm starts producing the product no other firm can produce the same One reason for this is the ability of the

monopolist to produce the product at a lower cost than any new firm who thinks to enter the market If a new firm who knows that it cannot produce at a lower cost than the monopolist then that firm will never enter the market for fear of losing out in competition Similarly the monopolist who is operating for a long time may be enjoying reputation among its customers and is in a better position to use the situation in its own benefit A new firm has to take long time to achieve this and so may not be interested to enter the market

4 Price Maker Being the single seller of the product the monopolist has full control over the pricing of the product On the other hand if there is a large number of buyers in the market so no single buyer exercises any significant influence over price determination Thus it is a sellerrsquos market So monopoly firm is a price maker

5 Price Discrimination Having considerable control over the market on account of being single seller with no entry of other firms the monopolist can exercise policy of price discrimination it means that the monopolist can sell different quantities of the same product to a consumer at different price or same quantity to different consumers at different prices by adjudging the standard of living of the consumer

6 Shape of Demand Curve Since a monopolist has full control over the price therefore he can sell more by lowering the price This makes the demand curve downward sloping

Subject Ac-12 290620 Topic- retirement Model sumThe Balance Sheet of Rohit Nisha and Sunil who are partners in a firm sharing profits according to their capitals as on 31st March 2014 was as under

Liabilities Amount Assets Amount (Rs) (` Rs)

Creditors 25000 Machinery 40000Bills Payable 13000 Building 90000General Reserve 22000 Debtors 30000Capital Less Provision for Rohit 60000 Bad debts 1000

29000 Nisha 40000 Stocks 23000 Sunil 40000 140000 Cash at Bank 18000

200000 200000

On the date of Balance Sheet Nisha retired from the firm and following adjustments were made(i) Building is appreciated by 20(ii) Provision for bad debts is increased to 5 on Debtors(iii) Machinery is depreciated by 10(iv) Goodwill of the firm is valued at Rs 56000 and the retiring partnerrsquos share is adjusted

(v) The capital of the new firm is fixed at Rs120000 Prepare Revaluation Account Capital Accounts of the partner and Balance Sheet of the new firm after Nisharsquos retirement Revaluation AccountDr Cr

Particulars Amount Particulars Amount (`Rs) (Rs`)

Provision for Bad debt Ac 500 Building Ac 18000Machinery Ac 4000Profit transferred toCapital Accounts (3 2 2)Rohit 5786Nisha 3857Sunil 3857

13500

18000 18000

Capital Account

Dr Cr

Particulars Rohit Nisha Sunil Particulars Rohit Nisha Sunil (Rs`) (Rs`) (`Rs) (Rs`) (Rs`) (Rs`)

Sunilrsquos Capital ac 9600 mdash 6400 Balance bd 60000 40000 40000Bank - 66143 - General Reserve 9428 6286 6286Balance cd 72000 mdash 48000 Revaluation (Profi 5786 3857 3857 Rohitrsquos Capital Ac mdash 9600 mdash

Sunilrsquos Capital Ac 6400 Bank 6386 - 4257

81600 66143 54400 81600 66143 54400

Balance Sheet as at 31st March 2014

Liabilities Amount Assets Amount (Rs`) (Rs`)

Creditors 25000 Building 108000Bank overdraft 37500 Machinery 36000

Bills Payable 13000 Debtors 30000Capital Less ProvisionRohit 72000 for Bad debts 1500 28500Sunil 48000 120000 Stock 23000

195500 195500

Working Notes (i) (a) Profit sharing ratio is 60000 40000 40000 ie = 3 2 2(b) Gaining Ratio Rohit = 35 ndash 37 = 2135 ndash 1535 = 635Sunil = 25-27 = 1435 ndash 1035 = 435= 635 435= 6 4 = 3 2(c) Nisha Share of Goodwill = Rs 56000 times 27 = Rs16000Share of Goodwill in the gaining ratio by the existing partner ieRohit = Rs16000 times 35 = Rs 9600Sunil = Rs 16000 times 25 = Rs 6400

The journal entry isRohitrsquos Capital Ac Dr 9600Sunilrsquos Capital Ac Dr 6400 To Nisharsquos Capital Ac 16000(Share of Goodwill divided into gaining ratio)

  • 1 Static Friction
  • The frictional force that acts between the surfaces when they are at rest with respect to each other is called Static Friction
    • Static Friction Examples
      • 2 Sliding Friction
        • Examples Of Sliding Friction
          • 3 Rolling Friction
            • Examples Of Rolling Friction
              • Objects and Reasons of the Forest Conservation Act
Page 42:  · Web viewSubject . Topic . Summary . Execution . English 1 . Sounds of animals . Hens –cackle Horses –neigh Lions –roar Owls –hoots Snake –hiss. English 2 . Mother’s

(ii) Fall in the price of inputsfactors(iii) Use of better technology in production(iv) Decrease in the rate of excise duty by government(v) If the objective of producer changes from profit maximization to salesMaximization

Main factors causing decrease in supply or leftward shift of supply curve(i) Increase in the price of other related goods(ii) Rise in the price of inputsfactors(iii) Use of inferior technology in production(iv) Increase in the rate of excise duty by the government(v) If the objective

Subject - Biology Topic ndash Chapter mdash6 PhotosynthesisSummary Execution

Today we will know about photosynthesis and its stages

Q1 What do you mean by photosynthesis The process by which living plants containing chlorophyll produce food

substances from carbon-di- oxide and water by using light energy Sunlight

6CO2 +12 H2O----------------------- C6 H12O6 + 6H2O + 6O2

Chlorophyll

Q2 What are the importance of photosynthesis I) Food for all Green plants trap solar energy by photosynthesis

process and supply food and energy for all living organisms either directly or indirectly

Ii) Oxygen to breathe in by product of photosynthesis is oxygen which is essential for all living organisms respiration

Q3 Write about two main phases of photosynthesis A Light dependent phase This phase occur in grana of chloroplast I) The chlorophyll on exposure to light energy becomes activated by

absorbing photons Ii) The absorbed energy is used in splitting the water molecules (H2O)

into its two components (H+ and OH- ) and releasing electron s 2H2O------------------------- 4H+ + 4e- +O2

Energy of 4 photons This reaction is known as photolysis

End products are H+ and oxygen water

B Light independent (Dark ) phase The reactions in this phase require no light energy

Here CO2 combine with H+ and produce glucose

Class XI

Subject Topic Summary ExecutionEVS Chapter-4 Legal

regimes for sustainable development

Environmental legislationEnvironmental legislation is the collection of laws and regulations pertaining to air quality water quality the wilderness endangered wildlife and other environmental factors The act ensures that matters important to the environment are thoroughly

Learn -The Forest (Conservation) Act 1980

considered in any decisions made by federal agencies

The Forest (Conservation) Act 1980 The Forest (Conservation) Act 1980 an Act of the Parliament of India to provide for the conservation of forests and for matters connected therewith or ancillary or incidental thereto It was further amended in 1988 This law extends to the whole of IndiaObjects and Reasons of the Forest Conservation Act

Deforestation causes ecological imbalance and leads to environmental deterioration Deforestation had been taking place on a large scale in the country and it had caused widespread concern The act seeks to check upon deforestation and de-reservation of forests

Subject Eng Literature (The Tempest ndash William Shakespeare) Topic Act II Scene 1 Lines 314 to 329 (End of scene)

[Students should read the original play and also the paraphrase given in the school prescribed textbook]Summary Questions amp Answers

Conspiracy of Antonio and Sebastian (Contd)

o As they approach Ariel appears again and wakes up Gonzalo by singing a tune in his ear Alonso also wakes up and they see both Sebastian and Antonio with drawn swords On being caught off guard they make up a story saying that they had heard a bellowing of bulls or lions

o They then moved to another part of the island

o Ariel at once rushes to Prospero to inform him of this development

SUMMING-UP of ACT-2 SCENE-1

(i) Among the survivors Ferdinand is separated from the rest which results in the disconsolate grief of Alonso as he took him for dead

(ii) The villainy of Antonio is confirmed

(iii) The supremacy of Prosperorsquos magic which resulted in the failure of the human conspiracy

(1)

(Act II Sc 1 L 311-325)SEBASTIAN Whiles we stood here securing your repose

Even now we heard a hollow burst of bellowing Like bulls or rather lions Didt not wake youIt struck mine ear most terribly

ALONSO I heard nothingANTONIO O rsquotwas a din to fright a monsters ear

To make an earthquake Sure it was the roarOf a whole herd of lions

ALONSO Heard you this GonzaloGONZALO Upon mine honour sir I heard a humming

And that a strange one too which did awake meI shaked you sir and cried As mine eyes opened I saw their weapons drawn There was a noiseThats verily rsquoTis best we stand upon our guardOr that we quit this place Lets draw our weapons

(i) Why has Prospero sent Ariel to Gonzalo and Alonso What does Ariel do to awaken Gonzalo

Prospero has already come to know by his magic powers the danger which threatens Gonzalo who had been Prosperorsquos friend and so he sent Ariel to preserve the lives of both Gonzalo and Alonso Prospero does not want that his scheme should remain unfulfilled Ariel begins to sing a song in Gonzalorsquos ears to awaken him(ii) Who are ready to carry out their plan Who takes steps to stop them Why does Gonzalo feel surprised after being awakened

Sebastian and Antonio are ready to carry out their plans They are standing with their swords drawn to kill Alonso and

(iv) We see two sets of contrasting characters Gonzalo-Adrian against Antonio-Sebastian

(v) The grief that works in Alonso can be perceived to his repentance for his association in Antoniorsquos crime against Prospero

Gonzalo Ariel takes steps to stop them from carrying out their nefarious scheme When Gonzalo is awakened by the song sung by Ariel into his ears he (Gonzalo) feels surprised because he sees Sebastian and Antonio standing with their swords drawn(iii) What reason do Sebastian and Antonio tell of drawing their swords when they are suspected by Alonso and Gonzalo

When Sebastian and Antonio are seen with their swords drawn they are looked with suspicion by Gonzalo and Alonso At first Sebastian tells them that as they stood here to guard them during their sleep they heard only a little before a sudden loud noise very much like the roaring of bulls or more probably that of lions Then Antonio follows him saying that this was a noise so terrible as to frighten even a monsterrsquos ears and this noise could even have shaken the earth and it was surely like the roaring of a multitude of lions Then seeing the danger they have drawn their swords Perhaps after hearing the terrible noise they (Gonzalo and Alonso) woke up from their sound sleep

(iv) What does Gonzalo tell Alonso about the strange noise What did he see on opening his eyes Gonzalo tells Alonso that he did not hear the sound of roaring but he heard a humming sound which was strange and which woke him up After waking up he gave him (Alonso) a shaking and a loud cry On opening his eyes he saw these two gentlemen standing with their swords drawn(v) What does Gonzalo suggest

Gonzalo suggests that there was a noise indeed and of that he has no doubt at all and suggests that the best course for them would be to remain alert and vigilant against any possible danger to their lives or to leave this place and move to some other part of the island

Class XIISubject Topic Summary Execution

Commerce

Chapter- Management

Today we will discuss about LEVELS OF MANAGEMENT

Levels of management is a series or chain of managerial positions from top to bottom It helps individuals to know their authority responsibilities and superior-subordinate relations among themselves There are mainly three levels of Management TOP LEVEL MANAGEMENTMIDDLE LEVEL MANAGEMENTLOWER LEVEL MANAGEMENT

Top level managementIt consists of members at the highest level in the management hierarchy This level includes Board Of Directors Chief Executive Managing Directors Chairman President Vice President

Rolefunctions of the top levelmanagement1To analyse evaluate and deal

with theexternal environment2 To determine the objectives and

policies of the business3 To strive for welfare and survival

of business

4 To create an organisational Framework consisting of authority responsibility relationship

Middle level management Congress of members or groups who are concerned with implementation of the policies let down by the top managementThis level includes head of the department such as finance manager marketing manager branch and regional managers departmental and divisional heads plant superintendent etc

Role of functions of the middle level management

1 To interpret the policies framed by top management

2 To assign duties and responsibilities to lower level managers

3 To select and appoint employees for middle and supervisory level and evaluate their performance

4 To co-operate with other departments for smooth functioning

Operational or supervisory level managementIt refers to the group are members who are concerned with execution of the work They are also known as fast line managers This level includes supervisor 4 men Section Officer clerk Inspector etc

Role of functions of the lower level management1 To plan and execute day-to-

day operations2 To supervise and control the workers3 To arrange materials and

tools to start the process and make arrangements for training

4 Today present workers grievance and suggestions before the management and

ensure safe and proper working conditions in the factory

Business Studies

Staff Appraisal Chapter- 10 Today let us start with a new chapter

Staff Appraisal

Meaning of Performance Appraisal

Performance Appraisal is the systematic evaluation of the performance of employees and to understand the abilities of a person for further growth and developmentThe supervisors measure the pay of employees and compare it with targets and plansThe supervisor analyses the factors behind work performances of employeesThe employers are in position to guide the employees for a better performance

Objectives of Performance Appraisal

Following are the objectives of Performance Appraisal

To maintain records in order to determine compensation packages wage structure salaries raises etc

To identify the strengths and weaknesses of employees to place right men on right job

To maintain and assess the potential present in a person for further growth and development

To provide a feedback to employees regarding their performance and related status

To provide a feedback to employees regarding their performance and related status

Importance of Performance Appraisal

Performance appraisal provides important and useful information for the assessment of employees skill

knowledge ability and overall job performance The following are the points which indicate the importance of performance appraisal in an organization

1 Performance appraisal helps supervisors to assess the work performance of their subordinates

2 Performance appraisal helps to assess the training and development needs of employees

3 Performance appraisal provides grounds for employees to correct their mistakes and it also provides proper guidance and criticism for employees development4 Performance appraisal provides reward for better performance

5 Performance appraisal helps to improve the communication system of the organization

6 Performance appraisal evaluates whether human resource programs being implemented in the organization have been effective

7 Performance appraisal helps to prepare pay structure for each employee working in the organization

8 Performance appraisal helps to review the potentiality of employees so that their future capability is anticipated

Geography

DRIANAGE The SubarnarekhaThe Subarnarekha and the Brahmaniinterposed between the Ganga and the Mahanadi deltas drain an area of 19300 sq kmand 39033 sq km respectively The drainage basins of these streams are shared byJharkhand Odisha west Bengal and Chhattisgarh The Brahmani is known as southKoel in its upper reaches in Jharkhand

The NarmadaThe Narmada rises in the Amarkantak hills of MadhyaPradesh It flows towards the West in a rift valleyformed due to a geological fault The total length of it is 1300 km All the tributaries of the

Q1 Name the two westward flowing rivers in the peninsular plateauA1 Narmada and Tapi are the only westward flowing rivers of the peninsular plateau

Q2 Differentiate between east-flowing rivers and west-flowing riversA2

East-flowing rivers

West-flowing rivers

Narmada are very short inlength Most of its tributaries join the main streamright anglesThe Narmada basin covers parts of Madhya Pradesh and Gujarat

The Tapi The Tapi rises in the Satpura ranges in the Betul listrictof Madhya Pradesh It flows in a rift valley parallel tothe Narmada but it is much shorter in length It coversparts of Madhya Pradesh Gujarat and MaharashtraThe length is about 724 km

The Sabarmati and the MahiThe Sabarmati rises in the Aravali hills and flows south-south-westwards for a distance of 300 kilometres to the Arabian Sea The Sabarmatibasin extends over an area of 21674 sq km in Rajasthan and Gujarat The Mahi rises inthe east of Udaipur and drains an area of 34842 sq km lying in Madhya PradeshRajasthan and Gujarat It flows south-westwards for a distance of 533 km before it fallsinto the Gulf of Khambhat

The ChambalThe Chambal rises near Mhow in the Vindhya Range and flows towards the northgenerally in a gorge upto Kota Below Kota it turns to the north-east direction and afterreaching Pinahat it turns to the east and runs nearly parallel to the Yamuna beforejoining it in the southern part of the Etawah district in Uttar PradeshMajor Rivers of India with their basin area (Sqkm)

Himalayan System Indus 321290Ganga 861404

Brahmaputra 187110Indus System

Jhelum 34775Beas 20303

Ganga System Yamuna 366223Ghaghra 127950

Peninsular RiversNarmada 98796

Tapi 65145Mahanadi 141600

Subarnarekha 19300Sabarmati 21674

Mahi 34842Godavari 312812

Godavari Krishna Kaveri Mahanadi are the east-flowing rivers

Narmada Tapi west-flowing rivers

They fall into the Bay of Bengal

They fall into Arabian Sea

These rivers form big deltas

These rivers form comparativelysmall deltas

Catchment areas of these rivers are larger

Catchment areas of these rivers are smaller

Krishna 2589488Cauveri 87900

Subject ndashBiology Topic ndashChapter -5 Inheritance amp Variations Summary ExecutionToday we will discussabout linkage and its classification

LINKAGE The tendency of the genes located on the same chromosome to stay together is

hereditary transmission Linked genes the genes responsible for this Genes that exhibit the process of linkage locates in the same chromosome The distance between the linked genes in a chromosome determines the strength

of linkage i e genes that are located close to each other show stronger linkage than that are located far from each other

COMPLETE LINKAGE It is the type of linkage showed by the genes that are closely located or are tightly

linked with each other as they have no chance of separatingby crossing over These genes are always transmitted together to the same gamete and the same

offspring In such condition only parental or non cross over type of gametes are formedINCOMPLETE KINKAGE It is type of linkage showed by the genes that are distantly located orare loosely

linked with each other because they have chance of separating by crossing over

SIGNIFICANCE i) It helps in holding the parental character togetherii) It checks the appearance of new recombination and helps in bringing the

hybrid population which resembles the original parents iii) Linked genes dilute the effects of undesirable traits

Subject Eng Literature (The Tempest ndash William Shakespeare) Topic Essay Questions (EQ-3)Question No 3

Give a character sketch of CalibanAnswer

The character of Caliban has been wonderfully conceived by Shakespeare as the manifestation of all that is gross and earthy ndash a sort of creature of the earth as Ariel is a sort of creature of the air

Calibanrsquos Physical Appearanceo Caliban is lsquofreckledrsquo a lsquomisshapen knaversquo not honoured with human shape

o Prospero calls him lsquothou tortoisersquo (Act I Sc 2 Line 317) Trinculo stumbling upon him describes him as ldquoA strange fish hellip Legged like a man And his fins like armsrdquo He ldquosmells like a fishrdquo (Act II Sc 2 Line 25)

o Prospero also calls him a ldquobeastrdquo (Act IV Sc 1 Line 140) and ldquoThis misshapen knaverdquo (Act V Sc 1 Line 268)

o Further it appears that in addition to his physical deformity his spiritual inferiority is also suggested by Prosperorsquos claim that his birth resulted from the union between his mother the witch Sycorax and the devil

Calibanrsquos ParentageWhen the play opens Caliban is twenty four years of age having been born on the island twelve years before the coming of Prospero His mother was the foul witch Sycorax who was banished from Algiers for ldquomischiefs manifold and sorceries terrible to enter human hearingrdquo (Act I Sc 2 Line 264) and the father was the Devil himself Thus

Caliban is a monster of evil and brute nature ugly deformed and stinking

Calibanrsquos Savage and Malignant Natureo Caliban is entirely a creature of the earth ndash gross brutal and savage He regards himself as the rightful possessor

of the island and Prospero as a usurper

o In his young age he was on good terms with Prospero He had consented to be received by Prospero at his house and to be educated by him He has learnt human language only to curse his master whom he abhors

o His beastly nature soon breaks out and ends in a vicious attack on Miranda This opens the eye of Prospero who becomes severe to him and enforces his service by threats and violence

o Prospero uses him to make dams for fish to fetch firewood scraper trenches wash dishes and keep his cell clean

Calibanrsquos Hatred for ProsperoA profound hatred for Prospero has taken hold of Caliban It springs from a sense of his being dispossessed and ill-treated He would kill Prospero if he could but he knows the power of Prosperorsquos lsquobookrsquo Hence he transfers his allegiance to Stephano who seems like a god to him He also incites the two drunken associates to batter the skull of Prospero when he sleeps in the afternoon

Caliban Shows Considerable Intelligenceo He has learnt Prosperorsquos language

ldquoYou taught me language and my profit onrsquot (Act II Sc 2 Lines 86-89)Is I know how to curserdquo

o He is well aware of the futility of arguing with one who has more power than he has

ldquoI must obey his art is such power (Act I Sc 2 Lines 373-376)It would control my damrsquos god SetebosAnd make a vassal of himrdquo

o He realizes the importance of Prosperorsquos books

ldquoRemember (Act III Sc 2 Lines 89-92)First to possess his books for without themHersquos but a sot as I am nor hath notOne spirit to commandrdquo

o He knows the value of stealth when attacking the enemy

ldquoPray you tread softly that the blind mole may not (Act IV Sc 1 Lines 194-195)Hear a foot fall we now are near his cellrdquo

o Caliban has a better set of values than Stephano and Trinculo They are distracted from their plan by their greed for Prosperorsquos rich garments Only Caliban realizes that such a finery is unimportant

ldquoLeave it alone thou fool it is but trashrdquo (Act IV Sc 1 Lines 224)

Caliban is not a good judge of characterCaliban is not a good judge of character He decides for example that Stephano is a god because he dispenses lsquocelestial liquorrsquo (Act II Sc 2 Line 115) but then it must be remembered that he has only known his mother Sycorax Prospero Miranda and the spirits that torture him However he quickly discovers his error of judgementrdquo

ldquoWhat a thrice-double ass (Act V Sc 1 Lines 295-297)Was I to take this drunkard for a godAnd worship this dull foolrdquo

Calibanrsquos Imaginative NatureIf Caliban is sub-human in what has been said above he is human in the respect of the poetic side of his character He listens to music with rapture He tells of the beautiful dreams in which heaven rains treasures upon him and which upon waking he yearns to renew One of the most poetic passages in whole play is Calibanrsquos description of the island

to Stephano and Trinculo

ldquoBe not afeard The isle is full of noises (Act III Sc 2 Lines 135-143)Sounds and sweet airs that give delight and hurt notSometimes a thousand twangling instrumentsWill hum about mine ears and sometime voicesThat if I then had waked after long sleepWill make me sleep again and then in dreamingThe clouds methought would open and show richesReady to drop upon me that when I wakedI cried to dream againrdquo

Caliban - Less Ignoble Than Some OthersCalibanrsquos motive for murder is less dishonourable than that of Antonio and Sebastian They plan to kill Alonso to gain his power and wealth Caliban merely wants revenge and the return of lsquohisrsquo island

Conclusiono Calibanrsquos character is not portrayed very clearly in the play and hence we cannot decide whether he is a poor

savage being grossly maltreated by Prospero or whether he is evil and must therefore be kept in bondage or enslavement

o Caliban is contrasted with Ariel who is a spirit and thus swift and uninterested in physical activitieso Caliban is also contrasted with Prospero who is the all-powerful master of the island and of the destiny of all

those on the islando Caliban is also contrasted with civilized man showing him to be less evil than Antonio and Stephano and less

materialistic than Stephano and Trinculoo Caliban has suffered at the hands of Prospero and he has learnt to curse by listening to Prosperorsquos abuse He

certainly believes that Prospero has deprived him of his birthrighto Finally the character Caliban is thought to be one of Shakespearersquos masterpieces The complexity of the character

is reflected in the large volume of critical discussion that has grown around it

ECO ndash12 Topic-Forms of market

MonopolyMonopoly is a market structure in which there is a single seller there are no close substitutes for the commodity produced by the firm and there are barriers to entry Example Indian Railways which is operated under government of India Monopoly also implies absence of competitionFeatures of Monopoly Monopoly is characterized by1 Single Seller In monopoly there is only one firm producing the product The whole industry consists of this single firm Thus under monopoly there is no distinction between firm and industry Being the only firm there is significant control of the firm over supply and price Thus under monopoly buyers do not have the option of buying the commodity from any other seller They have to buy the product from the firm or they can go without the commodity This fact gives immense control to the monopolist over the market

2No Close Substitute There are no close substitutes of the product produced by the monopolist firm If there are close substitutes of the product in the market it implies presence of more than one firm and hence no monopoly In order to ensure a total of control over the market by the monopolist firm it is assumed that there are no close substitutes of the product

3 No Entry amp Exit Monopoly can only exist when there is strong barriers before a new firm to enter the market In fact once a monopoly firm starts producing the product no other firm can produce the same One reason for this is the ability of the

monopolist to produce the product at a lower cost than any new firm who thinks to enter the market If a new firm who knows that it cannot produce at a lower cost than the monopolist then that firm will never enter the market for fear of losing out in competition Similarly the monopolist who is operating for a long time may be enjoying reputation among its customers and is in a better position to use the situation in its own benefit A new firm has to take long time to achieve this and so may not be interested to enter the market

4 Price Maker Being the single seller of the product the monopolist has full control over the pricing of the product On the other hand if there is a large number of buyers in the market so no single buyer exercises any significant influence over price determination Thus it is a sellerrsquos market So monopoly firm is a price maker

5 Price Discrimination Having considerable control over the market on account of being single seller with no entry of other firms the monopolist can exercise policy of price discrimination it means that the monopolist can sell different quantities of the same product to a consumer at different price or same quantity to different consumers at different prices by adjudging the standard of living of the consumer

6 Shape of Demand Curve Since a monopolist has full control over the price therefore he can sell more by lowering the price This makes the demand curve downward sloping

Subject Ac-12 290620 Topic- retirement Model sumThe Balance Sheet of Rohit Nisha and Sunil who are partners in a firm sharing profits according to their capitals as on 31st March 2014 was as under

Liabilities Amount Assets Amount (Rs) (` Rs)

Creditors 25000 Machinery 40000Bills Payable 13000 Building 90000General Reserve 22000 Debtors 30000Capital Less Provision for Rohit 60000 Bad debts 1000

29000 Nisha 40000 Stocks 23000 Sunil 40000 140000 Cash at Bank 18000

200000 200000

On the date of Balance Sheet Nisha retired from the firm and following adjustments were made(i) Building is appreciated by 20(ii) Provision for bad debts is increased to 5 on Debtors(iii) Machinery is depreciated by 10(iv) Goodwill of the firm is valued at Rs 56000 and the retiring partnerrsquos share is adjusted

(v) The capital of the new firm is fixed at Rs120000 Prepare Revaluation Account Capital Accounts of the partner and Balance Sheet of the new firm after Nisharsquos retirement Revaluation AccountDr Cr

Particulars Amount Particulars Amount (`Rs) (Rs`)

Provision for Bad debt Ac 500 Building Ac 18000Machinery Ac 4000Profit transferred toCapital Accounts (3 2 2)Rohit 5786Nisha 3857Sunil 3857

13500

18000 18000

Capital Account

Dr Cr

Particulars Rohit Nisha Sunil Particulars Rohit Nisha Sunil (Rs`) (Rs`) (`Rs) (Rs`) (Rs`) (Rs`)

Sunilrsquos Capital ac 9600 mdash 6400 Balance bd 60000 40000 40000Bank - 66143 - General Reserve 9428 6286 6286Balance cd 72000 mdash 48000 Revaluation (Profi 5786 3857 3857 Rohitrsquos Capital Ac mdash 9600 mdash

Sunilrsquos Capital Ac 6400 Bank 6386 - 4257

81600 66143 54400 81600 66143 54400

Balance Sheet as at 31st March 2014

Liabilities Amount Assets Amount (Rs`) (Rs`)

Creditors 25000 Building 108000Bank overdraft 37500 Machinery 36000

Bills Payable 13000 Debtors 30000Capital Less ProvisionRohit 72000 for Bad debts 1500 28500Sunil 48000 120000 Stock 23000

195500 195500

Working Notes (i) (a) Profit sharing ratio is 60000 40000 40000 ie = 3 2 2(b) Gaining Ratio Rohit = 35 ndash 37 = 2135 ndash 1535 = 635Sunil = 25-27 = 1435 ndash 1035 = 435= 635 435= 6 4 = 3 2(c) Nisha Share of Goodwill = Rs 56000 times 27 = Rs16000Share of Goodwill in the gaining ratio by the existing partner ieRohit = Rs16000 times 35 = Rs 9600Sunil = Rs 16000 times 25 = Rs 6400

The journal entry isRohitrsquos Capital Ac Dr 9600Sunilrsquos Capital Ac Dr 6400 To Nisharsquos Capital Ac 16000(Share of Goodwill divided into gaining ratio)

  • 1 Static Friction
  • The frictional force that acts between the surfaces when they are at rest with respect to each other is called Static Friction
    • Static Friction Examples
      • 2 Sliding Friction
        • Examples Of Sliding Friction
          • 3 Rolling Friction
            • Examples Of Rolling Friction
              • Objects and Reasons of the Forest Conservation Act
Page 43:  · Web viewSubject . Topic . Summary . Execution . English 1 . Sounds of animals . Hens –cackle Horses –neigh Lions –roar Owls –hoots Snake –hiss. English 2 . Mother’s

considered in any decisions made by federal agencies

The Forest (Conservation) Act 1980 The Forest (Conservation) Act 1980 an Act of the Parliament of India to provide for the conservation of forests and for matters connected therewith or ancillary or incidental thereto It was further amended in 1988 This law extends to the whole of IndiaObjects and Reasons of the Forest Conservation Act

Deforestation causes ecological imbalance and leads to environmental deterioration Deforestation had been taking place on a large scale in the country and it had caused widespread concern The act seeks to check upon deforestation and de-reservation of forests

Subject Eng Literature (The Tempest ndash William Shakespeare) Topic Act II Scene 1 Lines 314 to 329 (End of scene)

[Students should read the original play and also the paraphrase given in the school prescribed textbook]Summary Questions amp Answers

Conspiracy of Antonio and Sebastian (Contd)

o As they approach Ariel appears again and wakes up Gonzalo by singing a tune in his ear Alonso also wakes up and they see both Sebastian and Antonio with drawn swords On being caught off guard they make up a story saying that they had heard a bellowing of bulls or lions

o They then moved to another part of the island

o Ariel at once rushes to Prospero to inform him of this development

SUMMING-UP of ACT-2 SCENE-1

(i) Among the survivors Ferdinand is separated from the rest which results in the disconsolate grief of Alonso as he took him for dead

(ii) The villainy of Antonio is confirmed

(iii) The supremacy of Prosperorsquos magic which resulted in the failure of the human conspiracy

(1)

(Act II Sc 1 L 311-325)SEBASTIAN Whiles we stood here securing your repose

Even now we heard a hollow burst of bellowing Like bulls or rather lions Didt not wake youIt struck mine ear most terribly

ALONSO I heard nothingANTONIO O rsquotwas a din to fright a monsters ear

To make an earthquake Sure it was the roarOf a whole herd of lions

ALONSO Heard you this GonzaloGONZALO Upon mine honour sir I heard a humming

And that a strange one too which did awake meI shaked you sir and cried As mine eyes opened I saw their weapons drawn There was a noiseThats verily rsquoTis best we stand upon our guardOr that we quit this place Lets draw our weapons

(i) Why has Prospero sent Ariel to Gonzalo and Alonso What does Ariel do to awaken Gonzalo

Prospero has already come to know by his magic powers the danger which threatens Gonzalo who had been Prosperorsquos friend and so he sent Ariel to preserve the lives of both Gonzalo and Alonso Prospero does not want that his scheme should remain unfulfilled Ariel begins to sing a song in Gonzalorsquos ears to awaken him(ii) Who are ready to carry out their plan Who takes steps to stop them Why does Gonzalo feel surprised after being awakened

Sebastian and Antonio are ready to carry out their plans They are standing with their swords drawn to kill Alonso and

(iv) We see two sets of contrasting characters Gonzalo-Adrian against Antonio-Sebastian

(v) The grief that works in Alonso can be perceived to his repentance for his association in Antoniorsquos crime against Prospero

Gonzalo Ariel takes steps to stop them from carrying out their nefarious scheme When Gonzalo is awakened by the song sung by Ariel into his ears he (Gonzalo) feels surprised because he sees Sebastian and Antonio standing with their swords drawn(iii) What reason do Sebastian and Antonio tell of drawing their swords when they are suspected by Alonso and Gonzalo

When Sebastian and Antonio are seen with their swords drawn they are looked with suspicion by Gonzalo and Alonso At first Sebastian tells them that as they stood here to guard them during their sleep they heard only a little before a sudden loud noise very much like the roaring of bulls or more probably that of lions Then Antonio follows him saying that this was a noise so terrible as to frighten even a monsterrsquos ears and this noise could even have shaken the earth and it was surely like the roaring of a multitude of lions Then seeing the danger they have drawn their swords Perhaps after hearing the terrible noise they (Gonzalo and Alonso) woke up from their sound sleep

(iv) What does Gonzalo tell Alonso about the strange noise What did he see on opening his eyes Gonzalo tells Alonso that he did not hear the sound of roaring but he heard a humming sound which was strange and which woke him up After waking up he gave him (Alonso) a shaking and a loud cry On opening his eyes he saw these two gentlemen standing with their swords drawn(v) What does Gonzalo suggest

Gonzalo suggests that there was a noise indeed and of that he has no doubt at all and suggests that the best course for them would be to remain alert and vigilant against any possible danger to their lives or to leave this place and move to some other part of the island

Class XIISubject Topic Summary Execution

Commerce

Chapter- Management

Today we will discuss about LEVELS OF MANAGEMENT

Levels of management is a series or chain of managerial positions from top to bottom It helps individuals to know their authority responsibilities and superior-subordinate relations among themselves There are mainly three levels of Management TOP LEVEL MANAGEMENTMIDDLE LEVEL MANAGEMENTLOWER LEVEL MANAGEMENT

Top level managementIt consists of members at the highest level in the management hierarchy This level includes Board Of Directors Chief Executive Managing Directors Chairman President Vice President

Rolefunctions of the top levelmanagement1To analyse evaluate and deal

with theexternal environment2 To determine the objectives and

policies of the business3 To strive for welfare and survival

of business

4 To create an organisational Framework consisting of authority responsibility relationship

Middle level management Congress of members or groups who are concerned with implementation of the policies let down by the top managementThis level includes head of the department such as finance manager marketing manager branch and regional managers departmental and divisional heads plant superintendent etc

Role of functions of the middle level management

1 To interpret the policies framed by top management

2 To assign duties and responsibilities to lower level managers

3 To select and appoint employees for middle and supervisory level and evaluate their performance

4 To co-operate with other departments for smooth functioning

Operational or supervisory level managementIt refers to the group are members who are concerned with execution of the work They are also known as fast line managers This level includes supervisor 4 men Section Officer clerk Inspector etc

Role of functions of the lower level management1 To plan and execute day-to-

day operations2 To supervise and control the workers3 To arrange materials and

tools to start the process and make arrangements for training

4 Today present workers grievance and suggestions before the management and

ensure safe and proper working conditions in the factory

Business Studies

Staff Appraisal Chapter- 10 Today let us start with a new chapter

Staff Appraisal

Meaning of Performance Appraisal

Performance Appraisal is the systematic evaluation of the performance of employees and to understand the abilities of a person for further growth and developmentThe supervisors measure the pay of employees and compare it with targets and plansThe supervisor analyses the factors behind work performances of employeesThe employers are in position to guide the employees for a better performance

Objectives of Performance Appraisal

Following are the objectives of Performance Appraisal

To maintain records in order to determine compensation packages wage structure salaries raises etc

To identify the strengths and weaknesses of employees to place right men on right job

To maintain and assess the potential present in a person for further growth and development

To provide a feedback to employees regarding their performance and related status

To provide a feedback to employees regarding their performance and related status

Importance of Performance Appraisal

Performance appraisal provides important and useful information for the assessment of employees skill

knowledge ability and overall job performance The following are the points which indicate the importance of performance appraisal in an organization

1 Performance appraisal helps supervisors to assess the work performance of their subordinates

2 Performance appraisal helps to assess the training and development needs of employees

3 Performance appraisal provides grounds for employees to correct their mistakes and it also provides proper guidance and criticism for employees development4 Performance appraisal provides reward for better performance

5 Performance appraisal helps to improve the communication system of the organization

6 Performance appraisal evaluates whether human resource programs being implemented in the organization have been effective

7 Performance appraisal helps to prepare pay structure for each employee working in the organization

8 Performance appraisal helps to review the potentiality of employees so that their future capability is anticipated

Geography

DRIANAGE The SubarnarekhaThe Subarnarekha and the Brahmaniinterposed between the Ganga and the Mahanadi deltas drain an area of 19300 sq kmand 39033 sq km respectively The drainage basins of these streams are shared byJharkhand Odisha west Bengal and Chhattisgarh The Brahmani is known as southKoel in its upper reaches in Jharkhand

The NarmadaThe Narmada rises in the Amarkantak hills of MadhyaPradesh It flows towards the West in a rift valleyformed due to a geological fault The total length of it is 1300 km All the tributaries of the

Q1 Name the two westward flowing rivers in the peninsular plateauA1 Narmada and Tapi are the only westward flowing rivers of the peninsular plateau

Q2 Differentiate between east-flowing rivers and west-flowing riversA2

East-flowing rivers

West-flowing rivers

Narmada are very short inlength Most of its tributaries join the main streamright anglesThe Narmada basin covers parts of Madhya Pradesh and Gujarat

The Tapi The Tapi rises in the Satpura ranges in the Betul listrictof Madhya Pradesh It flows in a rift valley parallel tothe Narmada but it is much shorter in length It coversparts of Madhya Pradesh Gujarat and MaharashtraThe length is about 724 km

The Sabarmati and the MahiThe Sabarmati rises in the Aravali hills and flows south-south-westwards for a distance of 300 kilometres to the Arabian Sea The Sabarmatibasin extends over an area of 21674 sq km in Rajasthan and Gujarat The Mahi rises inthe east of Udaipur and drains an area of 34842 sq km lying in Madhya PradeshRajasthan and Gujarat It flows south-westwards for a distance of 533 km before it fallsinto the Gulf of Khambhat

The ChambalThe Chambal rises near Mhow in the Vindhya Range and flows towards the northgenerally in a gorge upto Kota Below Kota it turns to the north-east direction and afterreaching Pinahat it turns to the east and runs nearly parallel to the Yamuna beforejoining it in the southern part of the Etawah district in Uttar PradeshMajor Rivers of India with their basin area (Sqkm)

Himalayan System Indus 321290Ganga 861404

Brahmaputra 187110Indus System

Jhelum 34775Beas 20303

Ganga System Yamuna 366223Ghaghra 127950

Peninsular RiversNarmada 98796

Tapi 65145Mahanadi 141600

Subarnarekha 19300Sabarmati 21674

Mahi 34842Godavari 312812

Godavari Krishna Kaveri Mahanadi are the east-flowing rivers

Narmada Tapi west-flowing rivers

They fall into the Bay of Bengal

They fall into Arabian Sea

These rivers form big deltas

These rivers form comparativelysmall deltas

Catchment areas of these rivers are larger

Catchment areas of these rivers are smaller

Krishna 2589488Cauveri 87900

Subject ndashBiology Topic ndashChapter -5 Inheritance amp Variations Summary ExecutionToday we will discussabout linkage and its classification

LINKAGE The tendency of the genes located on the same chromosome to stay together is

hereditary transmission Linked genes the genes responsible for this Genes that exhibit the process of linkage locates in the same chromosome The distance between the linked genes in a chromosome determines the strength

of linkage i e genes that are located close to each other show stronger linkage than that are located far from each other

COMPLETE LINKAGE It is the type of linkage showed by the genes that are closely located or are tightly

linked with each other as they have no chance of separatingby crossing over These genes are always transmitted together to the same gamete and the same

offspring In such condition only parental or non cross over type of gametes are formedINCOMPLETE KINKAGE It is type of linkage showed by the genes that are distantly located orare loosely

linked with each other because they have chance of separating by crossing over

SIGNIFICANCE i) It helps in holding the parental character togetherii) It checks the appearance of new recombination and helps in bringing the

hybrid population which resembles the original parents iii) Linked genes dilute the effects of undesirable traits

Subject Eng Literature (The Tempest ndash William Shakespeare) Topic Essay Questions (EQ-3)Question No 3

Give a character sketch of CalibanAnswer

The character of Caliban has been wonderfully conceived by Shakespeare as the manifestation of all that is gross and earthy ndash a sort of creature of the earth as Ariel is a sort of creature of the air

Calibanrsquos Physical Appearanceo Caliban is lsquofreckledrsquo a lsquomisshapen knaversquo not honoured with human shape

o Prospero calls him lsquothou tortoisersquo (Act I Sc 2 Line 317) Trinculo stumbling upon him describes him as ldquoA strange fish hellip Legged like a man And his fins like armsrdquo He ldquosmells like a fishrdquo (Act II Sc 2 Line 25)

o Prospero also calls him a ldquobeastrdquo (Act IV Sc 1 Line 140) and ldquoThis misshapen knaverdquo (Act V Sc 1 Line 268)

o Further it appears that in addition to his physical deformity his spiritual inferiority is also suggested by Prosperorsquos claim that his birth resulted from the union between his mother the witch Sycorax and the devil

Calibanrsquos ParentageWhen the play opens Caliban is twenty four years of age having been born on the island twelve years before the coming of Prospero His mother was the foul witch Sycorax who was banished from Algiers for ldquomischiefs manifold and sorceries terrible to enter human hearingrdquo (Act I Sc 2 Line 264) and the father was the Devil himself Thus

Caliban is a monster of evil and brute nature ugly deformed and stinking

Calibanrsquos Savage and Malignant Natureo Caliban is entirely a creature of the earth ndash gross brutal and savage He regards himself as the rightful possessor

of the island and Prospero as a usurper

o In his young age he was on good terms with Prospero He had consented to be received by Prospero at his house and to be educated by him He has learnt human language only to curse his master whom he abhors

o His beastly nature soon breaks out and ends in a vicious attack on Miranda This opens the eye of Prospero who becomes severe to him and enforces his service by threats and violence

o Prospero uses him to make dams for fish to fetch firewood scraper trenches wash dishes and keep his cell clean

Calibanrsquos Hatred for ProsperoA profound hatred for Prospero has taken hold of Caliban It springs from a sense of his being dispossessed and ill-treated He would kill Prospero if he could but he knows the power of Prosperorsquos lsquobookrsquo Hence he transfers his allegiance to Stephano who seems like a god to him He also incites the two drunken associates to batter the skull of Prospero when he sleeps in the afternoon

Caliban Shows Considerable Intelligenceo He has learnt Prosperorsquos language

ldquoYou taught me language and my profit onrsquot (Act II Sc 2 Lines 86-89)Is I know how to curserdquo

o He is well aware of the futility of arguing with one who has more power than he has

ldquoI must obey his art is such power (Act I Sc 2 Lines 373-376)It would control my damrsquos god SetebosAnd make a vassal of himrdquo

o He realizes the importance of Prosperorsquos books

ldquoRemember (Act III Sc 2 Lines 89-92)First to possess his books for without themHersquos but a sot as I am nor hath notOne spirit to commandrdquo

o He knows the value of stealth when attacking the enemy

ldquoPray you tread softly that the blind mole may not (Act IV Sc 1 Lines 194-195)Hear a foot fall we now are near his cellrdquo

o Caliban has a better set of values than Stephano and Trinculo They are distracted from their plan by their greed for Prosperorsquos rich garments Only Caliban realizes that such a finery is unimportant

ldquoLeave it alone thou fool it is but trashrdquo (Act IV Sc 1 Lines 224)

Caliban is not a good judge of characterCaliban is not a good judge of character He decides for example that Stephano is a god because he dispenses lsquocelestial liquorrsquo (Act II Sc 2 Line 115) but then it must be remembered that he has only known his mother Sycorax Prospero Miranda and the spirits that torture him However he quickly discovers his error of judgementrdquo

ldquoWhat a thrice-double ass (Act V Sc 1 Lines 295-297)Was I to take this drunkard for a godAnd worship this dull foolrdquo

Calibanrsquos Imaginative NatureIf Caliban is sub-human in what has been said above he is human in the respect of the poetic side of his character He listens to music with rapture He tells of the beautiful dreams in which heaven rains treasures upon him and which upon waking he yearns to renew One of the most poetic passages in whole play is Calibanrsquos description of the island

to Stephano and Trinculo

ldquoBe not afeard The isle is full of noises (Act III Sc 2 Lines 135-143)Sounds and sweet airs that give delight and hurt notSometimes a thousand twangling instrumentsWill hum about mine ears and sometime voicesThat if I then had waked after long sleepWill make me sleep again and then in dreamingThe clouds methought would open and show richesReady to drop upon me that when I wakedI cried to dream againrdquo

Caliban - Less Ignoble Than Some OthersCalibanrsquos motive for murder is less dishonourable than that of Antonio and Sebastian They plan to kill Alonso to gain his power and wealth Caliban merely wants revenge and the return of lsquohisrsquo island

Conclusiono Calibanrsquos character is not portrayed very clearly in the play and hence we cannot decide whether he is a poor

savage being grossly maltreated by Prospero or whether he is evil and must therefore be kept in bondage or enslavement

o Caliban is contrasted with Ariel who is a spirit and thus swift and uninterested in physical activitieso Caliban is also contrasted with Prospero who is the all-powerful master of the island and of the destiny of all

those on the islando Caliban is also contrasted with civilized man showing him to be less evil than Antonio and Stephano and less

materialistic than Stephano and Trinculoo Caliban has suffered at the hands of Prospero and he has learnt to curse by listening to Prosperorsquos abuse He

certainly believes that Prospero has deprived him of his birthrighto Finally the character Caliban is thought to be one of Shakespearersquos masterpieces The complexity of the character

is reflected in the large volume of critical discussion that has grown around it

ECO ndash12 Topic-Forms of market

MonopolyMonopoly is a market structure in which there is a single seller there are no close substitutes for the commodity produced by the firm and there are barriers to entry Example Indian Railways which is operated under government of India Monopoly also implies absence of competitionFeatures of Monopoly Monopoly is characterized by1 Single Seller In monopoly there is only one firm producing the product The whole industry consists of this single firm Thus under monopoly there is no distinction between firm and industry Being the only firm there is significant control of the firm over supply and price Thus under monopoly buyers do not have the option of buying the commodity from any other seller They have to buy the product from the firm or they can go without the commodity This fact gives immense control to the monopolist over the market

2No Close Substitute There are no close substitutes of the product produced by the monopolist firm If there are close substitutes of the product in the market it implies presence of more than one firm and hence no monopoly In order to ensure a total of control over the market by the monopolist firm it is assumed that there are no close substitutes of the product

3 No Entry amp Exit Monopoly can only exist when there is strong barriers before a new firm to enter the market In fact once a monopoly firm starts producing the product no other firm can produce the same One reason for this is the ability of the

monopolist to produce the product at a lower cost than any new firm who thinks to enter the market If a new firm who knows that it cannot produce at a lower cost than the monopolist then that firm will never enter the market for fear of losing out in competition Similarly the monopolist who is operating for a long time may be enjoying reputation among its customers and is in a better position to use the situation in its own benefit A new firm has to take long time to achieve this and so may not be interested to enter the market

4 Price Maker Being the single seller of the product the monopolist has full control over the pricing of the product On the other hand if there is a large number of buyers in the market so no single buyer exercises any significant influence over price determination Thus it is a sellerrsquos market So monopoly firm is a price maker

5 Price Discrimination Having considerable control over the market on account of being single seller with no entry of other firms the monopolist can exercise policy of price discrimination it means that the monopolist can sell different quantities of the same product to a consumer at different price or same quantity to different consumers at different prices by adjudging the standard of living of the consumer

6 Shape of Demand Curve Since a monopolist has full control over the price therefore he can sell more by lowering the price This makes the demand curve downward sloping

Subject Ac-12 290620 Topic- retirement Model sumThe Balance Sheet of Rohit Nisha and Sunil who are partners in a firm sharing profits according to their capitals as on 31st March 2014 was as under

Liabilities Amount Assets Amount (Rs) (` Rs)

Creditors 25000 Machinery 40000Bills Payable 13000 Building 90000General Reserve 22000 Debtors 30000Capital Less Provision for Rohit 60000 Bad debts 1000

29000 Nisha 40000 Stocks 23000 Sunil 40000 140000 Cash at Bank 18000

200000 200000

On the date of Balance Sheet Nisha retired from the firm and following adjustments were made(i) Building is appreciated by 20(ii) Provision for bad debts is increased to 5 on Debtors(iii) Machinery is depreciated by 10(iv) Goodwill of the firm is valued at Rs 56000 and the retiring partnerrsquos share is adjusted

(v) The capital of the new firm is fixed at Rs120000 Prepare Revaluation Account Capital Accounts of the partner and Balance Sheet of the new firm after Nisharsquos retirement Revaluation AccountDr Cr

Particulars Amount Particulars Amount (`Rs) (Rs`)

Provision for Bad debt Ac 500 Building Ac 18000Machinery Ac 4000Profit transferred toCapital Accounts (3 2 2)Rohit 5786Nisha 3857Sunil 3857

13500

18000 18000

Capital Account

Dr Cr

Particulars Rohit Nisha Sunil Particulars Rohit Nisha Sunil (Rs`) (Rs`) (`Rs) (Rs`) (Rs`) (Rs`)

Sunilrsquos Capital ac 9600 mdash 6400 Balance bd 60000 40000 40000Bank - 66143 - General Reserve 9428 6286 6286Balance cd 72000 mdash 48000 Revaluation (Profi 5786 3857 3857 Rohitrsquos Capital Ac mdash 9600 mdash

Sunilrsquos Capital Ac 6400 Bank 6386 - 4257

81600 66143 54400 81600 66143 54400

Balance Sheet as at 31st March 2014

Liabilities Amount Assets Amount (Rs`) (Rs`)

Creditors 25000 Building 108000Bank overdraft 37500 Machinery 36000

Bills Payable 13000 Debtors 30000Capital Less ProvisionRohit 72000 for Bad debts 1500 28500Sunil 48000 120000 Stock 23000

195500 195500

Working Notes (i) (a) Profit sharing ratio is 60000 40000 40000 ie = 3 2 2(b) Gaining Ratio Rohit = 35 ndash 37 = 2135 ndash 1535 = 635Sunil = 25-27 = 1435 ndash 1035 = 435= 635 435= 6 4 = 3 2(c) Nisha Share of Goodwill = Rs 56000 times 27 = Rs16000Share of Goodwill in the gaining ratio by the existing partner ieRohit = Rs16000 times 35 = Rs 9600Sunil = Rs 16000 times 25 = Rs 6400

The journal entry isRohitrsquos Capital Ac Dr 9600Sunilrsquos Capital Ac Dr 6400 To Nisharsquos Capital Ac 16000(Share of Goodwill divided into gaining ratio)

  • 1 Static Friction
  • The frictional force that acts between the surfaces when they are at rest with respect to each other is called Static Friction
    • Static Friction Examples
      • 2 Sliding Friction
        • Examples Of Sliding Friction
          • 3 Rolling Friction
            • Examples Of Rolling Friction
              • Objects and Reasons of the Forest Conservation Act
Page 44:  · Web viewSubject . Topic . Summary . Execution . English 1 . Sounds of animals . Hens –cackle Horses –neigh Lions –roar Owls –hoots Snake –hiss. English 2 . Mother’s

(iv) We see two sets of contrasting characters Gonzalo-Adrian against Antonio-Sebastian

(v) The grief that works in Alonso can be perceived to his repentance for his association in Antoniorsquos crime against Prospero

Gonzalo Ariel takes steps to stop them from carrying out their nefarious scheme When Gonzalo is awakened by the song sung by Ariel into his ears he (Gonzalo) feels surprised because he sees Sebastian and Antonio standing with their swords drawn(iii) What reason do Sebastian and Antonio tell of drawing their swords when they are suspected by Alonso and Gonzalo

When Sebastian and Antonio are seen with their swords drawn they are looked with suspicion by Gonzalo and Alonso At first Sebastian tells them that as they stood here to guard them during their sleep they heard only a little before a sudden loud noise very much like the roaring of bulls or more probably that of lions Then Antonio follows him saying that this was a noise so terrible as to frighten even a monsterrsquos ears and this noise could even have shaken the earth and it was surely like the roaring of a multitude of lions Then seeing the danger they have drawn their swords Perhaps after hearing the terrible noise they (Gonzalo and Alonso) woke up from their sound sleep

(iv) What does Gonzalo tell Alonso about the strange noise What did he see on opening his eyes Gonzalo tells Alonso that he did not hear the sound of roaring but he heard a humming sound which was strange and which woke him up After waking up he gave him (Alonso) a shaking and a loud cry On opening his eyes he saw these two gentlemen standing with their swords drawn(v) What does Gonzalo suggest

Gonzalo suggests that there was a noise indeed and of that he has no doubt at all and suggests that the best course for them would be to remain alert and vigilant against any possible danger to their lives or to leave this place and move to some other part of the island

Class XIISubject Topic Summary Execution

Commerce

Chapter- Management

Today we will discuss about LEVELS OF MANAGEMENT

Levels of management is a series or chain of managerial positions from top to bottom It helps individuals to know their authority responsibilities and superior-subordinate relations among themselves There are mainly three levels of Management TOP LEVEL MANAGEMENTMIDDLE LEVEL MANAGEMENTLOWER LEVEL MANAGEMENT

Top level managementIt consists of members at the highest level in the management hierarchy This level includes Board Of Directors Chief Executive Managing Directors Chairman President Vice President

Rolefunctions of the top levelmanagement1To analyse evaluate and deal

with theexternal environment2 To determine the objectives and

policies of the business3 To strive for welfare and survival

of business

4 To create an organisational Framework consisting of authority responsibility relationship

Middle level management Congress of members or groups who are concerned with implementation of the policies let down by the top managementThis level includes head of the department such as finance manager marketing manager branch and regional managers departmental and divisional heads plant superintendent etc

Role of functions of the middle level management

1 To interpret the policies framed by top management

2 To assign duties and responsibilities to lower level managers

3 To select and appoint employees for middle and supervisory level and evaluate their performance

4 To co-operate with other departments for smooth functioning

Operational or supervisory level managementIt refers to the group are members who are concerned with execution of the work They are also known as fast line managers This level includes supervisor 4 men Section Officer clerk Inspector etc

Role of functions of the lower level management1 To plan and execute day-to-

day operations2 To supervise and control the workers3 To arrange materials and

tools to start the process and make arrangements for training

4 Today present workers grievance and suggestions before the management and

ensure safe and proper working conditions in the factory

Business Studies

Staff Appraisal Chapter- 10 Today let us start with a new chapter

Staff Appraisal

Meaning of Performance Appraisal

Performance Appraisal is the systematic evaluation of the performance of employees and to understand the abilities of a person for further growth and developmentThe supervisors measure the pay of employees and compare it with targets and plansThe supervisor analyses the factors behind work performances of employeesThe employers are in position to guide the employees for a better performance

Objectives of Performance Appraisal

Following are the objectives of Performance Appraisal

To maintain records in order to determine compensation packages wage structure salaries raises etc

To identify the strengths and weaknesses of employees to place right men on right job

To maintain and assess the potential present in a person for further growth and development

To provide a feedback to employees regarding their performance and related status

To provide a feedback to employees regarding their performance and related status

Importance of Performance Appraisal

Performance appraisal provides important and useful information for the assessment of employees skill

knowledge ability and overall job performance The following are the points which indicate the importance of performance appraisal in an organization

1 Performance appraisal helps supervisors to assess the work performance of their subordinates

2 Performance appraisal helps to assess the training and development needs of employees

3 Performance appraisal provides grounds for employees to correct their mistakes and it also provides proper guidance and criticism for employees development4 Performance appraisal provides reward for better performance

5 Performance appraisal helps to improve the communication system of the organization

6 Performance appraisal evaluates whether human resource programs being implemented in the organization have been effective

7 Performance appraisal helps to prepare pay structure for each employee working in the organization

8 Performance appraisal helps to review the potentiality of employees so that their future capability is anticipated

Geography

DRIANAGE The SubarnarekhaThe Subarnarekha and the Brahmaniinterposed between the Ganga and the Mahanadi deltas drain an area of 19300 sq kmand 39033 sq km respectively The drainage basins of these streams are shared byJharkhand Odisha west Bengal and Chhattisgarh The Brahmani is known as southKoel in its upper reaches in Jharkhand

The NarmadaThe Narmada rises in the Amarkantak hills of MadhyaPradesh It flows towards the West in a rift valleyformed due to a geological fault The total length of it is 1300 km All the tributaries of the

Q1 Name the two westward flowing rivers in the peninsular plateauA1 Narmada and Tapi are the only westward flowing rivers of the peninsular plateau

Q2 Differentiate between east-flowing rivers and west-flowing riversA2

East-flowing rivers

West-flowing rivers

Narmada are very short inlength Most of its tributaries join the main streamright anglesThe Narmada basin covers parts of Madhya Pradesh and Gujarat

The Tapi The Tapi rises in the Satpura ranges in the Betul listrictof Madhya Pradesh It flows in a rift valley parallel tothe Narmada but it is much shorter in length It coversparts of Madhya Pradesh Gujarat and MaharashtraThe length is about 724 km

The Sabarmati and the MahiThe Sabarmati rises in the Aravali hills and flows south-south-westwards for a distance of 300 kilometres to the Arabian Sea The Sabarmatibasin extends over an area of 21674 sq km in Rajasthan and Gujarat The Mahi rises inthe east of Udaipur and drains an area of 34842 sq km lying in Madhya PradeshRajasthan and Gujarat It flows south-westwards for a distance of 533 km before it fallsinto the Gulf of Khambhat

The ChambalThe Chambal rises near Mhow in the Vindhya Range and flows towards the northgenerally in a gorge upto Kota Below Kota it turns to the north-east direction and afterreaching Pinahat it turns to the east and runs nearly parallel to the Yamuna beforejoining it in the southern part of the Etawah district in Uttar PradeshMajor Rivers of India with their basin area (Sqkm)

Himalayan System Indus 321290Ganga 861404

Brahmaputra 187110Indus System

Jhelum 34775Beas 20303

Ganga System Yamuna 366223Ghaghra 127950

Peninsular RiversNarmada 98796

Tapi 65145Mahanadi 141600

Subarnarekha 19300Sabarmati 21674

Mahi 34842Godavari 312812

Godavari Krishna Kaveri Mahanadi are the east-flowing rivers

Narmada Tapi west-flowing rivers

They fall into the Bay of Bengal

They fall into Arabian Sea

These rivers form big deltas

These rivers form comparativelysmall deltas

Catchment areas of these rivers are larger

Catchment areas of these rivers are smaller

Krishna 2589488Cauveri 87900

Subject ndashBiology Topic ndashChapter -5 Inheritance amp Variations Summary ExecutionToday we will discussabout linkage and its classification

LINKAGE The tendency of the genes located on the same chromosome to stay together is

hereditary transmission Linked genes the genes responsible for this Genes that exhibit the process of linkage locates in the same chromosome The distance between the linked genes in a chromosome determines the strength

of linkage i e genes that are located close to each other show stronger linkage than that are located far from each other

COMPLETE LINKAGE It is the type of linkage showed by the genes that are closely located or are tightly

linked with each other as they have no chance of separatingby crossing over These genes are always transmitted together to the same gamete and the same

offspring In such condition only parental or non cross over type of gametes are formedINCOMPLETE KINKAGE It is type of linkage showed by the genes that are distantly located orare loosely

linked with each other because they have chance of separating by crossing over

SIGNIFICANCE i) It helps in holding the parental character togetherii) It checks the appearance of new recombination and helps in bringing the

hybrid population which resembles the original parents iii) Linked genes dilute the effects of undesirable traits

Subject Eng Literature (The Tempest ndash William Shakespeare) Topic Essay Questions (EQ-3)Question No 3

Give a character sketch of CalibanAnswer

The character of Caliban has been wonderfully conceived by Shakespeare as the manifestation of all that is gross and earthy ndash a sort of creature of the earth as Ariel is a sort of creature of the air

Calibanrsquos Physical Appearanceo Caliban is lsquofreckledrsquo a lsquomisshapen knaversquo not honoured with human shape

o Prospero calls him lsquothou tortoisersquo (Act I Sc 2 Line 317) Trinculo stumbling upon him describes him as ldquoA strange fish hellip Legged like a man And his fins like armsrdquo He ldquosmells like a fishrdquo (Act II Sc 2 Line 25)

o Prospero also calls him a ldquobeastrdquo (Act IV Sc 1 Line 140) and ldquoThis misshapen knaverdquo (Act V Sc 1 Line 268)

o Further it appears that in addition to his physical deformity his spiritual inferiority is also suggested by Prosperorsquos claim that his birth resulted from the union between his mother the witch Sycorax and the devil

Calibanrsquos ParentageWhen the play opens Caliban is twenty four years of age having been born on the island twelve years before the coming of Prospero His mother was the foul witch Sycorax who was banished from Algiers for ldquomischiefs manifold and sorceries terrible to enter human hearingrdquo (Act I Sc 2 Line 264) and the father was the Devil himself Thus

Caliban is a monster of evil and brute nature ugly deformed and stinking

Calibanrsquos Savage and Malignant Natureo Caliban is entirely a creature of the earth ndash gross brutal and savage He regards himself as the rightful possessor

of the island and Prospero as a usurper

o In his young age he was on good terms with Prospero He had consented to be received by Prospero at his house and to be educated by him He has learnt human language only to curse his master whom he abhors

o His beastly nature soon breaks out and ends in a vicious attack on Miranda This opens the eye of Prospero who becomes severe to him and enforces his service by threats and violence

o Prospero uses him to make dams for fish to fetch firewood scraper trenches wash dishes and keep his cell clean

Calibanrsquos Hatred for ProsperoA profound hatred for Prospero has taken hold of Caliban It springs from a sense of his being dispossessed and ill-treated He would kill Prospero if he could but he knows the power of Prosperorsquos lsquobookrsquo Hence he transfers his allegiance to Stephano who seems like a god to him He also incites the two drunken associates to batter the skull of Prospero when he sleeps in the afternoon

Caliban Shows Considerable Intelligenceo He has learnt Prosperorsquos language

ldquoYou taught me language and my profit onrsquot (Act II Sc 2 Lines 86-89)Is I know how to curserdquo

o He is well aware of the futility of arguing with one who has more power than he has

ldquoI must obey his art is such power (Act I Sc 2 Lines 373-376)It would control my damrsquos god SetebosAnd make a vassal of himrdquo

o He realizes the importance of Prosperorsquos books

ldquoRemember (Act III Sc 2 Lines 89-92)First to possess his books for without themHersquos but a sot as I am nor hath notOne spirit to commandrdquo

o He knows the value of stealth when attacking the enemy

ldquoPray you tread softly that the blind mole may not (Act IV Sc 1 Lines 194-195)Hear a foot fall we now are near his cellrdquo

o Caliban has a better set of values than Stephano and Trinculo They are distracted from their plan by their greed for Prosperorsquos rich garments Only Caliban realizes that such a finery is unimportant

ldquoLeave it alone thou fool it is but trashrdquo (Act IV Sc 1 Lines 224)

Caliban is not a good judge of characterCaliban is not a good judge of character He decides for example that Stephano is a god because he dispenses lsquocelestial liquorrsquo (Act II Sc 2 Line 115) but then it must be remembered that he has only known his mother Sycorax Prospero Miranda and the spirits that torture him However he quickly discovers his error of judgementrdquo

ldquoWhat a thrice-double ass (Act V Sc 1 Lines 295-297)Was I to take this drunkard for a godAnd worship this dull foolrdquo

Calibanrsquos Imaginative NatureIf Caliban is sub-human in what has been said above he is human in the respect of the poetic side of his character He listens to music with rapture He tells of the beautiful dreams in which heaven rains treasures upon him and which upon waking he yearns to renew One of the most poetic passages in whole play is Calibanrsquos description of the island

to Stephano and Trinculo

ldquoBe not afeard The isle is full of noises (Act III Sc 2 Lines 135-143)Sounds and sweet airs that give delight and hurt notSometimes a thousand twangling instrumentsWill hum about mine ears and sometime voicesThat if I then had waked after long sleepWill make me sleep again and then in dreamingThe clouds methought would open and show richesReady to drop upon me that when I wakedI cried to dream againrdquo

Caliban - Less Ignoble Than Some OthersCalibanrsquos motive for murder is less dishonourable than that of Antonio and Sebastian They plan to kill Alonso to gain his power and wealth Caliban merely wants revenge and the return of lsquohisrsquo island

Conclusiono Calibanrsquos character is not portrayed very clearly in the play and hence we cannot decide whether he is a poor

savage being grossly maltreated by Prospero or whether he is evil and must therefore be kept in bondage or enslavement

o Caliban is contrasted with Ariel who is a spirit and thus swift and uninterested in physical activitieso Caliban is also contrasted with Prospero who is the all-powerful master of the island and of the destiny of all

those on the islando Caliban is also contrasted with civilized man showing him to be less evil than Antonio and Stephano and less

materialistic than Stephano and Trinculoo Caliban has suffered at the hands of Prospero and he has learnt to curse by listening to Prosperorsquos abuse He

certainly believes that Prospero has deprived him of his birthrighto Finally the character Caliban is thought to be one of Shakespearersquos masterpieces The complexity of the character

is reflected in the large volume of critical discussion that has grown around it

ECO ndash12 Topic-Forms of market

MonopolyMonopoly is a market structure in which there is a single seller there are no close substitutes for the commodity produced by the firm and there are barriers to entry Example Indian Railways which is operated under government of India Monopoly also implies absence of competitionFeatures of Monopoly Monopoly is characterized by1 Single Seller In monopoly there is only one firm producing the product The whole industry consists of this single firm Thus under monopoly there is no distinction between firm and industry Being the only firm there is significant control of the firm over supply and price Thus under monopoly buyers do not have the option of buying the commodity from any other seller They have to buy the product from the firm or they can go without the commodity This fact gives immense control to the monopolist over the market

2No Close Substitute There are no close substitutes of the product produced by the monopolist firm If there are close substitutes of the product in the market it implies presence of more than one firm and hence no monopoly In order to ensure a total of control over the market by the monopolist firm it is assumed that there are no close substitutes of the product

3 No Entry amp Exit Monopoly can only exist when there is strong barriers before a new firm to enter the market In fact once a monopoly firm starts producing the product no other firm can produce the same One reason for this is the ability of the

monopolist to produce the product at a lower cost than any new firm who thinks to enter the market If a new firm who knows that it cannot produce at a lower cost than the monopolist then that firm will never enter the market for fear of losing out in competition Similarly the monopolist who is operating for a long time may be enjoying reputation among its customers and is in a better position to use the situation in its own benefit A new firm has to take long time to achieve this and so may not be interested to enter the market

4 Price Maker Being the single seller of the product the monopolist has full control over the pricing of the product On the other hand if there is a large number of buyers in the market so no single buyer exercises any significant influence over price determination Thus it is a sellerrsquos market So monopoly firm is a price maker

5 Price Discrimination Having considerable control over the market on account of being single seller with no entry of other firms the monopolist can exercise policy of price discrimination it means that the monopolist can sell different quantities of the same product to a consumer at different price or same quantity to different consumers at different prices by adjudging the standard of living of the consumer

6 Shape of Demand Curve Since a monopolist has full control over the price therefore he can sell more by lowering the price This makes the demand curve downward sloping

Subject Ac-12 290620 Topic- retirement Model sumThe Balance Sheet of Rohit Nisha and Sunil who are partners in a firm sharing profits according to their capitals as on 31st March 2014 was as under

Liabilities Amount Assets Amount (Rs) (` Rs)

Creditors 25000 Machinery 40000Bills Payable 13000 Building 90000General Reserve 22000 Debtors 30000Capital Less Provision for Rohit 60000 Bad debts 1000

29000 Nisha 40000 Stocks 23000 Sunil 40000 140000 Cash at Bank 18000

200000 200000

On the date of Balance Sheet Nisha retired from the firm and following adjustments were made(i) Building is appreciated by 20(ii) Provision for bad debts is increased to 5 on Debtors(iii) Machinery is depreciated by 10(iv) Goodwill of the firm is valued at Rs 56000 and the retiring partnerrsquos share is adjusted

(v) The capital of the new firm is fixed at Rs120000 Prepare Revaluation Account Capital Accounts of the partner and Balance Sheet of the new firm after Nisharsquos retirement Revaluation AccountDr Cr

Particulars Amount Particulars Amount (`Rs) (Rs`)

Provision for Bad debt Ac 500 Building Ac 18000Machinery Ac 4000Profit transferred toCapital Accounts (3 2 2)Rohit 5786Nisha 3857Sunil 3857

13500

18000 18000

Capital Account

Dr Cr

Particulars Rohit Nisha Sunil Particulars Rohit Nisha Sunil (Rs`) (Rs`) (`Rs) (Rs`) (Rs`) (Rs`)

Sunilrsquos Capital ac 9600 mdash 6400 Balance bd 60000 40000 40000Bank - 66143 - General Reserve 9428 6286 6286Balance cd 72000 mdash 48000 Revaluation (Profi 5786 3857 3857 Rohitrsquos Capital Ac mdash 9600 mdash

Sunilrsquos Capital Ac 6400 Bank 6386 - 4257

81600 66143 54400 81600 66143 54400

Balance Sheet as at 31st March 2014

Liabilities Amount Assets Amount (Rs`) (Rs`)

Creditors 25000 Building 108000Bank overdraft 37500 Machinery 36000

Bills Payable 13000 Debtors 30000Capital Less ProvisionRohit 72000 for Bad debts 1500 28500Sunil 48000 120000 Stock 23000

195500 195500

Working Notes (i) (a) Profit sharing ratio is 60000 40000 40000 ie = 3 2 2(b) Gaining Ratio Rohit = 35 ndash 37 = 2135 ndash 1535 = 635Sunil = 25-27 = 1435 ndash 1035 = 435= 635 435= 6 4 = 3 2(c) Nisha Share of Goodwill = Rs 56000 times 27 = Rs16000Share of Goodwill in the gaining ratio by the existing partner ieRohit = Rs16000 times 35 = Rs 9600Sunil = Rs 16000 times 25 = Rs 6400

The journal entry isRohitrsquos Capital Ac Dr 9600Sunilrsquos Capital Ac Dr 6400 To Nisharsquos Capital Ac 16000(Share of Goodwill divided into gaining ratio)

  • 1 Static Friction
  • The frictional force that acts between the surfaces when they are at rest with respect to each other is called Static Friction
    • Static Friction Examples
      • 2 Sliding Friction
        • Examples Of Sliding Friction
          • 3 Rolling Friction
            • Examples Of Rolling Friction
              • Objects and Reasons of the Forest Conservation Act
Page 45:  · Web viewSubject . Topic . Summary . Execution . English 1 . Sounds of animals . Hens –cackle Horses –neigh Lions –roar Owls –hoots Snake –hiss. English 2 . Mother’s

4 To create an organisational Framework consisting of authority responsibility relationship

Middle level management Congress of members or groups who are concerned with implementation of the policies let down by the top managementThis level includes head of the department such as finance manager marketing manager branch and regional managers departmental and divisional heads plant superintendent etc

Role of functions of the middle level management

1 To interpret the policies framed by top management

2 To assign duties and responsibilities to lower level managers

3 To select and appoint employees for middle and supervisory level and evaluate their performance

4 To co-operate with other departments for smooth functioning

Operational or supervisory level managementIt refers to the group are members who are concerned with execution of the work They are also known as fast line managers This level includes supervisor 4 men Section Officer clerk Inspector etc

Role of functions of the lower level management1 To plan and execute day-to-

day operations2 To supervise and control the workers3 To arrange materials and

tools to start the process and make arrangements for training

4 Today present workers grievance and suggestions before the management and

ensure safe and proper working conditions in the factory

Business Studies

Staff Appraisal Chapter- 10 Today let us start with a new chapter

Staff Appraisal

Meaning of Performance Appraisal

Performance Appraisal is the systematic evaluation of the performance of employees and to understand the abilities of a person for further growth and developmentThe supervisors measure the pay of employees and compare it with targets and plansThe supervisor analyses the factors behind work performances of employeesThe employers are in position to guide the employees for a better performance

Objectives of Performance Appraisal

Following are the objectives of Performance Appraisal

To maintain records in order to determine compensation packages wage structure salaries raises etc

To identify the strengths and weaknesses of employees to place right men on right job

To maintain and assess the potential present in a person for further growth and development

To provide a feedback to employees regarding their performance and related status

To provide a feedback to employees regarding their performance and related status

Importance of Performance Appraisal

Performance appraisal provides important and useful information for the assessment of employees skill

knowledge ability and overall job performance The following are the points which indicate the importance of performance appraisal in an organization

1 Performance appraisal helps supervisors to assess the work performance of their subordinates

2 Performance appraisal helps to assess the training and development needs of employees

3 Performance appraisal provides grounds for employees to correct their mistakes and it also provides proper guidance and criticism for employees development4 Performance appraisal provides reward for better performance

5 Performance appraisal helps to improve the communication system of the organization

6 Performance appraisal evaluates whether human resource programs being implemented in the organization have been effective

7 Performance appraisal helps to prepare pay structure for each employee working in the organization

8 Performance appraisal helps to review the potentiality of employees so that their future capability is anticipated

Geography

DRIANAGE The SubarnarekhaThe Subarnarekha and the Brahmaniinterposed between the Ganga and the Mahanadi deltas drain an area of 19300 sq kmand 39033 sq km respectively The drainage basins of these streams are shared byJharkhand Odisha west Bengal and Chhattisgarh The Brahmani is known as southKoel in its upper reaches in Jharkhand

The NarmadaThe Narmada rises in the Amarkantak hills of MadhyaPradesh It flows towards the West in a rift valleyformed due to a geological fault The total length of it is 1300 km All the tributaries of the

Q1 Name the two westward flowing rivers in the peninsular plateauA1 Narmada and Tapi are the only westward flowing rivers of the peninsular plateau

Q2 Differentiate between east-flowing rivers and west-flowing riversA2

East-flowing rivers

West-flowing rivers

Narmada are very short inlength Most of its tributaries join the main streamright anglesThe Narmada basin covers parts of Madhya Pradesh and Gujarat

The Tapi The Tapi rises in the Satpura ranges in the Betul listrictof Madhya Pradesh It flows in a rift valley parallel tothe Narmada but it is much shorter in length It coversparts of Madhya Pradesh Gujarat and MaharashtraThe length is about 724 km

The Sabarmati and the MahiThe Sabarmati rises in the Aravali hills and flows south-south-westwards for a distance of 300 kilometres to the Arabian Sea The Sabarmatibasin extends over an area of 21674 sq km in Rajasthan and Gujarat The Mahi rises inthe east of Udaipur and drains an area of 34842 sq km lying in Madhya PradeshRajasthan and Gujarat It flows south-westwards for a distance of 533 km before it fallsinto the Gulf of Khambhat

The ChambalThe Chambal rises near Mhow in the Vindhya Range and flows towards the northgenerally in a gorge upto Kota Below Kota it turns to the north-east direction and afterreaching Pinahat it turns to the east and runs nearly parallel to the Yamuna beforejoining it in the southern part of the Etawah district in Uttar PradeshMajor Rivers of India with their basin area (Sqkm)

Himalayan System Indus 321290Ganga 861404

Brahmaputra 187110Indus System

Jhelum 34775Beas 20303

Ganga System Yamuna 366223Ghaghra 127950

Peninsular RiversNarmada 98796

Tapi 65145Mahanadi 141600

Subarnarekha 19300Sabarmati 21674

Mahi 34842Godavari 312812

Godavari Krishna Kaveri Mahanadi are the east-flowing rivers

Narmada Tapi west-flowing rivers

They fall into the Bay of Bengal

They fall into Arabian Sea

These rivers form big deltas

These rivers form comparativelysmall deltas

Catchment areas of these rivers are larger

Catchment areas of these rivers are smaller

Krishna 2589488Cauveri 87900

Subject ndashBiology Topic ndashChapter -5 Inheritance amp Variations Summary ExecutionToday we will discussabout linkage and its classification

LINKAGE The tendency of the genes located on the same chromosome to stay together is

hereditary transmission Linked genes the genes responsible for this Genes that exhibit the process of linkage locates in the same chromosome The distance between the linked genes in a chromosome determines the strength

of linkage i e genes that are located close to each other show stronger linkage than that are located far from each other

COMPLETE LINKAGE It is the type of linkage showed by the genes that are closely located or are tightly

linked with each other as they have no chance of separatingby crossing over These genes are always transmitted together to the same gamete and the same

offspring In such condition only parental or non cross over type of gametes are formedINCOMPLETE KINKAGE It is type of linkage showed by the genes that are distantly located orare loosely

linked with each other because they have chance of separating by crossing over

SIGNIFICANCE i) It helps in holding the parental character togetherii) It checks the appearance of new recombination and helps in bringing the

hybrid population which resembles the original parents iii) Linked genes dilute the effects of undesirable traits

Subject Eng Literature (The Tempest ndash William Shakespeare) Topic Essay Questions (EQ-3)Question No 3

Give a character sketch of CalibanAnswer

The character of Caliban has been wonderfully conceived by Shakespeare as the manifestation of all that is gross and earthy ndash a sort of creature of the earth as Ariel is a sort of creature of the air

Calibanrsquos Physical Appearanceo Caliban is lsquofreckledrsquo a lsquomisshapen knaversquo not honoured with human shape

o Prospero calls him lsquothou tortoisersquo (Act I Sc 2 Line 317) Trinculo stumbling upon him describes him as ldquoA strange fish hellip Legged like a man And his fins like armsrdquo He ldquosmells like a fishrdquo (Act II Sc 2 Line 25)

o Prospero also calls him a ldquobeastrdquo (Act IV Sc 1 Line 140) and ldquoThis misshapen knaverdquo (Act V Sc 1 Line 268)

o Further it appears that in addition to his physical deformity his spiritual inferiority is also suggested by Prosperorsquos claim that his birth resulted from the union between his mother the witch Sycorax and the devil

Calibanrsquos ParentageWhen the play opens Caliban is twenty four years of age having been born on the island twelve years before the coming of Prospero His mother was the foul witch Sycorax who was banished from Algiers for ldquomischiefs manifold and sorceries terrible to enter human hearingrdquo (Act I Sc 2 Line 264) and the father was the Devil himself Thus

Caliban is a monster of evil and brute nature ugly deformed and stinking

Calibanrsquos Savage and Malignant Natureo Caliban is entirely a creature of the earth ndash gross brutal and savage He regards himself as the rightful possessor

of the island and Prospero as a usurper

o In his young age he was on good terms with Prospero He had consented to be received by Prospero at his house and to be educated by him He has learnt human language only to curse his master whom he abhors

o His beastly nature soon breaks out and ends in a vicious attack on Miranda This opens the eye of Prospero who becomes severe to him and enforces his service by threats and violence

o Prospero uses him to make dams for fish to fetch firewood scraper trenches wash dishes and keep his cell clean

Calibanrsquos Hatred for ProsperoA profound hatred for Prospero has taken hold of Caliban It springs from a sense of his being dispossessed and ill-treated He would kill Prospero if he could but he knows the power of Prosperorsquos lsquobookrsquo Hence he transfers his allegiance to Stephano who seems like a god to him He also incites the two drunken associates to batter the skull of Prospero when he sleeps in the afternoon

Caliban Shows Considerable Intelligenceo He has learnt Prosperorsquos language

ldquoYou taught me language and my profit onrsquot (Act II Sc 2 Lines 86-89)Is I know how to curserdquo

o He is well aware of the futility of arguing with one who has more power than he has

ldquoI must obey his art is such power (Act I Sc 2 Lines 373-376)It would control my damrsquos god SetebosAnd make a vassal of himrdquo

o He realizes the importance of Prosperorsquos books

ldquoRemember (Act III Sc 2 Lines 89-92)First to possess his books for without themHersquos but a sot as I am nor hath notOne spirit to commandrdquo

o He knows the value of stealth when attacking the enemy

ldquoPray you tread softly that the blind mole may not (Act IV Sc 1 Lines 194-195)Hear a foot fall we now are near his cellrdquo

o Caliban has a better set of values than Stephano and Trinculo They are distracted from their plan by their greed for Prosperorsquos rich garments Only Caliban realizes that such a finery is unimportant

ldquoLeave it alone thou fool it is but trashrdquo (Act IV Sc 1 Lines 224)

Caliban is not a good judge of characterCaliban is not a good judge of character He decides for example that Stephano is a god because he dispenses lsquocelestial liquorrsquo (Act II Sc 2 Line 115) but then it must be remembered that he has only known his mother Sycorax Prospero Miranda and the spirits that torture him However he quickly discovers his error of judgementrdquo

ldquoWhat a thrice-double ass (Act V Sc 1 Lines 295-297)Was I to take this drunkard for a godAnd worship this dull foolrdquo

Calibanrsquos Imaginative NatureIf Caliban is sub-human in what has been said above he is human in the respect of the poetic side of his character He listens to music with rapture He tells of the beautiful dreams in which heaven rains treasures upon him and which upon waking he yearns to renew One of the most poetic passages in whole play is Calibanrsquos description of the island

to Stephano and Trinculo

ldquoBe not afeard The isle is full of noises (Act III Sc 2 Lines 135-143)Sounds and sweet airs that give delight and hurt notSometimes a thousand twangling instrumentsWill hum about mine ears and sometime voicesThat if I then had waked after long sleepWill make me sleep again and then in dreamingThe clouds methought would open and show richesReady to drop upon me that when I wakedI cried to dream againrdquo

Caliban - Less Ignoble Than Some OthersCalibanrsquos motive for murder is less dishonourable than that of Antonio and Sebastian They plan to kill Alonso to gain his power and wealth Caliban merely wants revenge and the return of lsquohisrsquo island

Conclusiono Calibanrsquos character is not portrayed very clearly in the play and hence we cannot decide whether he is a poor

savage being grossly maltreated by Prospero or whether he is evil and must therefore be kept in bondage or enslavement

o Caliban is contrasted with Ariel who is a spirit and thus swift and uninterested in physical activitieso Caliban is also contrasted with Prospero who is the all-powerful master of the island and of the destiny of all

those on the islando Caliban is also contrasted with civilized man showing him to be less evil than Antonio and Stephano and less

materialistic than Stephano and Trinculoo Caliban has suffered at the hands of Prospero and he has learnt to curse by listening to Prosperorsquos abuse He

certainly believes that Prospero has deprived him of his birthrighto Finally the character Caliban is thought to be one of Shakespearersquos masterpieces The complexity of the character

is reflected in the large volume of critical discussion that has grown around it

ECO ndash12 Topic-Forms of market

MonopolyMonopoly is a market structure in which there is a single seller there are no close substitutes for the commodity produced by the firm and there are barriers to entry Example Indian Railways which is operated under government of India Monopoly also implies absence of competitionFeatures of Monopoly Monopoly is characterized by1 Single Seller In monopoly there is only one firm producing the product The whole industry consists of this single firm Thus under monopoly there is no distinction between firm and industry Being the only firm there is significant control of the firm over supply and price Thus under monopoly buyers do not have the option of buying the commodity from any other seller They have to buy the product from the firm or they can go without the commodity This fact gives immense control to the monopolist over the market

2No Close Substitute There are no close substitutes of the product produced by the monopolist firm If there are close substitutes of the product in the market it implies presence of more than one firm and hence no monopoly In order to ensure a total of control over the market by the monopolist firm it is assumed that there are no close substitutes of the product

3 No Entry amp Exit Monopoly can only exist when there is strong barriers before a new firm to enter the market In fact once a monopoly firm starts producing the product no other firm can produce the same One reason for this is the ability of the

monopolist to produce the product at a lower cost than any new firm who thinks to enter the market If a new firm who knows that it cannot produce at a lower cost than the monopolist then that firm will never enter the market for fear of losing out in competition Similarly the monopolist who is operating for a long time may be enjoying reputation among its customers and is in a better position to use the situation in its own benefit A new firm has to take long time to achieve this and so may not be interested to enter the market

4 Price Maker Being the single seller of the product the monopolist has full control over the pricing of the product On the other hand if there is a large number of buyers in the market so no single buyer exercises any significant influence over price determination Thus it is a sellerrsquos market So monopoly firm is a price maker

5 Price Discrimination Having considerable control over the market on account of being single seller with no entry of other firms the monopolist can exercise policy of price discrimination it means that the monopolist can sell different quantities of the same product to a consumer at different price or same quantity to different consumers at different prices by adjudging the standard of living of the consumer

6 Shape of Demand Curve Since a monopolist has full control over the price therefore he can sell more by lowering the price This makes the demand curve downward sloping

Subject Ac-12 290620 Topic- retirement Model sumThe Balance Sheet of Rohit Nisha and Sunil who are partners in a firm sharing profits according to their capitals as on 31st March 2014 was as under

Liabilities Amount Assets Amount (Rs) (` Rs)

Creditors 25000 Machinery 40000Bills Payable 13000 Building 90000General Reserve 22000 Debtors 30000Capital Less Provision for Rohit 60000 Bad debts 1000

29000 Nisha 40000 Stocks 23000 Sunil 40000 140000 Cash at Bank 18000

200000 200000

On the date of Balance Sheet Nisha retired from the firm and following adjustments were made(i) Building is appreciated by 20(ii) Provision for bad debts is increased to 5 on Debtors(iii) Machinery is depreciated by 10(iv) Goodwill of the firm is valued at Rs 56000 and the retiring partnerrsquos share is adjusted

(v) The capital of the new firm is fixed at Rs120000 Prepare Revaluation Account Capital Accounts of the partner and Balance Sheet of the new firm after Nisharsquos retirement Revaluation AccountDr Cr

Particulars Amount Particulars Amount (`Rs) (Rs`)

Provision for Bad debt Ac 500 Building Ac 18000Machinery Ac 4000Profit transferred toCapital Accounts (3 2 2)Rohit 5786Nisha 3857Sunil 3857

13500

18000 18000

Capital Account

Dr Cr

Particulars Rohit Nisha Sunil Particulars Rohit Nisha Sunil (Rs`) (Rs`) (`Rs) (Rs`) (Rs`) (Rs`)

Sunilrsquos Capital ac 9600 mdash 6400 Balance bd 60000 40000 40000Bank - 66143 - General Reserve 9428 6286 6286Balance cd 72000 mdash 48000 Revaluation (Profi 5786 3857 3857 Rohitrsquos Capital Ac mdash 9600 mdash

Sunilrsquos Capital Ac 6400 Bank 6386 - 4257

81600 66143 54400 81600 66143 54400

Balance Sheet as at 31st March 2014

Liabilities Amount Assets Amount (Rs`) (Rs`)

Creditors 25000 Building 108000Bank overdraft 37500 Machinery 36000

Bills Payable 13000 Debtors 30000Capital Less ProvisionRohit 72000 for Bad debts 1500 28500Sunil 48000 120000 Stock 23000

195500 195500

Working Notes (i) (a) Profit sharing ratio is 60000 40000 40000 ie = 3 2 2(b) Gaining Ratio Rohit = 35 ndash 37 = 2135 ndash 1535 = 635Sunil = 25-27 = 1435 ndash 1035 = 435= 635 435= 6 4 = 3 2(c) Nisha Share of Goodwill = Rs 56000 times 27 = Rs16000Share of Goodwill in the gaining ratio by the existing partner ieRohit = Rs16000 times 35 = Rs 9600Sunil = Rs 16000 times 25 = Rs 6400

The journal entry isRohitrsquos Capital Ac Dr 9600Sunilrsquos Capital Ac Dr 6400 To Nisharsquos Capital Ac 16000(Share of Goodwill divided into gaining ratio)

  • 1 Static Friction
  • The frictional force that acts between the surfaces when they are at rest with respect to each other is called Static Friction
    • Static Friction Examples
      • 2 Sliding Friction
        • Examples Of Sliding Friction
          • 3 Rolling Friction
            • Examples Of Rolling Friction
              • Objects and Reasons of the Forest Conservation Act
Page 46:  · Web viewSubject . Topic . Summary . Execution . English 1 . Sounds of animals . Hens –cackle Horses –neigh Lions –roar Owls –hoots Snake –hiss. English 2 . Mother’s

ensure safe and proper working conditions in the factory

Business Studies

Staff Appraisal Chapter- 10 Today let us start with a new chapter

Staff Appraisal

Meaning of Performance Appraisal

Performance Appraisal is the systematic evaluation of the performance of employees and to understand the abilities of a person for further growth and developmentThe supervisors measure the pay of employees and compare it with targets and plansThe supervisor analyses the factors behind work performances of employeesThe employers are in position to guide the employees for a better performance

Objectives of Performance Appraisal

Following are the objectives of Performance Appraisal

To maintain records in order to determine compensation packages wage structure salaries raises etc

To identify the strengths and weaknesses of employees to place right men on right job

To maintain and assess the potential present in a person for further growth and development

To provide a feedback to employees regarding their performance and related status

To provide a feedback to employees regarding their performance and related status

Importance of Performance Appraisal

Performance appraisal provides important and useful information for the assessment of employees skill

knowledge ability and overall job performance The following are the points which indicate the importance of performance appraisal in an organization

1 Performance appraisal helps supervisors to assess the work performance of their subordinates

2 Performance appraisal helps to assess the training and development needs of employees

3 Performance appraisal provides grounds for employees to correct their mistakes and it also provides proper guidance and criticism for employees development4 Performance appraisal provides reward for better performance

5 Performance appraisal helps to improve the communication system of the organization

6 Performance appraisal evaluates whether human resource programs being implemented in the organization have been effective

7 Performance appraisal helps to prepare pay structure for each employee working in the organization

8 Performance appraisal helps to review the potentiality of employees so that their future capability is anticipated

Geography

DRIANAGE The SubarnarekhaThe Subarnarekha and the Brahmaniinterposed between the Ganga and the Mahanadi deltas drain an area of 19300 sq kmand 39033 sq km respectively The drainage basins of these streams are shared byJharkhand Odisha west Bengal and Chhattisgarh The Brahmani is known as southKoel in its upper reaches in Jharkhand

The NarmadaThe Narmada rises in the Amarkantak hills of MadhyaPradesh It flows towards the West in a rift valleyformed due to a geological fault The total length of it is 1300 km All the tributaries of the

Q1 Name the two westward flowing rivers in the peninsular plateauA1 Narmada and Tapi are the only westward flowing rivers of the peninsular plateau

Q2 Differentiate between east-flowing rivers and west-flowing riversA2

East-flowing rivers

West-flowing rivers

Narmada are very short inlength Most of its tributaries join the main streamright anglesThe Narmada basin covers parts of Madhya Pradesh and Gujarat

The Tapi The Tapi rises in the Satpura ranges in the Betul listrictof Madhya Pradesh It flows in a rift valley parallel tothe Narmada but it is much shorter in length It coversparts of Madhya Pradesh Gujarat and MaharashtraThe length is about 724 km

The Sabarmati and the MahiThe Sabarmati rises in the Aravali hills and flows south-south-westwards for a distance of 300 kilometres to the Arabian Sea The Sabarmatibasin extends over an area of 21674 sq km in Rajasthan and Gujarat The Mahi rises inthe east of Udaipur and drains an area of 34842 sq km lying in Madhya PradeshRajasthan and Gujarat It flows south-westwards for a distance of 533 km before it fallsinto the Gulf of Khambhat

The ChambalThe Chambal rises near Mhow in the Vindhya Range and flows towards the northgenerally in a gorge upto Kota Below Kota it turns to the north-east direction and afterreaching Pinahat it turns to the east and runs nearly parallel to the Yamuna beforejoining it in the southern part of the Etawah district in Uttar PradeshMajor Rivers of India with their basin area (Sqkm)

Himalayan System Indus 321290Ganga 861404

Brahmaputra 187110Indus System

Jhelum 34775Beas 20303

Ganga System Yamuna 366223Ghaghra 127950

Peninsular RiversNarmada 98796

Tapi 65145Mahanadi 141600

Subarnarekha 19300Sabarmati 21674

Mahi 34842Godavari 312812

Godavari Krishna Kaveri Mahanadi are the east-flowing rivers

Narmada Tapi west-flowing rivers

They fall into the Bay of Bengal

They fall into Arabian Sea

These rivers form big deltas

These rivers form comparativelysmall deltas

Catchment areas of these rivers are larger

Catchment areas of these rivers are smaller

Krishna 2589488Cauveri 87900

Subject ndashBiology Topic ndashChapter -5 Inheritance amp Variations Summary ExecutionToday we will discussabout linkage and its classification

LINKAGE The tendency of the genes located on the same chromosome to stay together is

hereditary transmission Linked genes the genes responsible for this Genes that exhibit the process of linkage locates in the same chromosome The distance between the linked genes in a chromosome determines the strength

of linkage i e genes that are located close to each other show stronger linkage than that are located far from each other

COMPLETE LINKAGE It is the type of linkage showed by the genes that are closely located or are tightly

linked with each other as they have no chance of separatingby crossing over These genes are always transmitted together to the same gamete and the same

offspring In such condition only parental or non cross over type of gametes are formedINCOMPLETE KINKAGE It is type of linkage showed by the genes that are distantly located orare loosely

linked with each other because they have chance of separating by crossing over

SIGNIFICANCE i) It helps in holding the parental character togetherii) It checks the appearance of new recombination and helps in bringing the

hybrid population which resembles the original parents iii) Linked genes dilute the effects of undesirable traits

Subject Eng Literature (The Tempest ndash William Shakespeare) Topic Essay Questions (EQ-3)Question No 3

Give a character sketch of CalibanAnswer

The character of Caliban has been wonderfully conceived by Shakespeare as the manifestation of all that is gross and earthy ndash a sort of creature of the earth as Ariel is a sort of creature of the air

Calibanrsquos Physical Appearanceo Caliban is lsquofreckledrsquo a lsquomisshapen knaversquo not honoured with human shape

o Prospero calls him lsquothou tortoisersquo (Act I Sc 2 Line 317) Trinculo stumbling upon him describes him as ldquoA strange fish hellip Legged like a man And his fins like armsrdquo He ldquosmells like a fishrdquo (Act II Sc 2 Line 25)

o Prospero also calls him a ldquobeastrdquo (Act IV Sc 1 Line 140) and ldquoThis misshapen knaverdquo (Act V Sc 1 Line 268)

o Further it appears that in addition to his physical deformity his spiritual inferiority is also suggested by Prosperorsquos claim that his birth resulted from the union between his mother the witch Sycorax and the devil

Calibanrsquos ParentageWhen the play opens Caliban is twenty four years of age having been born on the island twelve years before the coming of Prospero His mother was the foul witch Sycorax who was banished from Algiers for ldquomischiefs manifold and sorceries terrible to enter human hearingrdquo (Act I Sc 2 Line 264) and the father was the Devil himself Thus

Caliban is a monster of evil and brute nature ugly deformed and stinking

Calibanrsquos Savage and Malignant Natureo Caliban is entirely a creature of the earth ndash gross brutal and savage He regards himself as the rightful possessor

of the island and Prospero as a usurper

o In his young age he was on good terms with Prospero He had consented to be received by Prospero at his house and to be educated by him He has learnt human language only to curse his master whom he abhors

o His beastly nature soon breaks out and ends in a vicious attack on Miranda This opens the eye of Prospero who becomes severe to him and enforces his service by threats and violence

o Prospero uses him to make dams for fish to fetch firewood scraper trenches wash dishes and keep his cell clean

Calibanrsquos Hatred for ProsperoA profound hatred for Prospero has taken hold of Caliban It springs from a sense of his being dispossessed and ill-treated He would kill Prospero if he could but he knows the power of Prosperorsquos lsquobookrsquo Hence he transfers his allegiance to Stephano who seems like a god to him He also incites the two drunken associates to batter the skull of Prospero when he sleeps in the afternoon

Caliban Shows Considerable Intelligenceo He has learnt Prosperorsquos language

ldquoYou taught me language and my profit onrsquot (Act II Sc 2 Lines 86-89)Is I know how to curserdquo

o He is well aware of the futility of arguing with one who has more power than he has

ldquoI must obey his art is such power (Act I Sc 2 Lines 373-376)It would control my damrsquos god SetebosAnd make a vassal of himrdquo

o He realizes the importance of Prosperorsquos books

ldquoRemember (Act III Sc 2 Lines 89-92)First to possess his books for without themHersquos but a sot as I am nor hath notOne spirit to commandrdquo

o He knows the value of stealth when attacking the enemy

ldquoPray you tread softly that the blind mole may not (Act IV Sc 1 Lines 194-195)Hear a foot fall we now are near his cellrdquo

o Caliban has a better set of values than Stephano and Trinculo They are distracted from their plan by their greed for Prosperorsquos rich garments Only Caliban realizes that such a finery is unimportant

ldquoLeave it alone thou fool it is but trashrdquo (Act IV Sc 1 Lines 224)

Caliban is not a good judge of characterCaliban is not a good judge of character He decides for example that Stephano is a god because he dispenses lsquocelestial liquorrsquo (Act II Sc 2 Line 115) but then it must be remembered that he has only known his mother Sycorax Prospero Miranda and the spirits that torture him However he quickly discovers his error of judgementrdquo

ldquoWhat a thrice-double ass (Act V Sc 1 Lines 295-297)Was I to take this drunkard for a godAnd worship this dull foolrdquo

Calibanrsquos Imaginative NatureIf Caliban is sub-human in what has been said above he is human in the respect of the poetic side of his character He listens to music with rapture He tells of the beautiful dreams in which heaven rains treasures upon him and which upon waking he yearns to renew One of the most poetic passages in whole play is Calibanrsquos description of the island

to Stephano and Trinculo

ldquoBe not afeard The isle is full of noises (Act III Sc 2 Lines 135-143)Sounds and sweet airs that give delight and hurt notSometimes a thousand twangling instrumentsWill hum about mine ears and sometime voicesThat if I then had waked after long sleepWill make me sleep again and then in dreamingThe clouds methought would open and show richesReady to drop upon me that when I wakedI cried to dream againrdquo

Caliban - Less Ignoble Than Some OthersCalibanrsquos motive for murder is less dishonourable than that of Antonio and Sebastian They plan to kill Alonso to gain his power and wealth Caliban merely wants revenge and the return of lsquohisrsquo island

Conclusiono Calibanrsquos character is not portrayed very clearly in the play and hence we cannot decide whether he is a poor

savage being grossly maltreated by Prospero or whether he is evil and must therefore be kept in bondage or enslavement

o Caliban is contrasted with Ariel who is a spirit and thus swift and uninterested in physical activitieso Caliban is also contrasted with Prospero who is the all-powerful master of the island and of the destiny of all

those on the islando Caliban is also contrasted with civilized man showing him to be less evil than Antonio and Stephano and less

materialistic than Stephano and Trinculoo Caliban has suffered at the hands of Prospero and he has learnt to curse by listening to Prosperorsquos abuse He

certainly believes that Prospero has deprived him of his birthrighto Finally the character Caliban is thought to be one of Shakespearersquos masterpieces The complexity of the character

is reflected in the large volume of critical discussion that has grown around it

ECO ndash12 Topic-Forms of market

MonopolyMonopoly is a market structure in which there is a single seller there are no close substitutes for the commodity produced by the firm and there are barriers to entry Example Indian Railways which is operated under government of India Monopoly also implies absence of competitionFeatures of Monopoly Monopoly is characterized by1 Single Seller In monopoly there is only one firm producing the product The whole industry consists of this single firm Thus under monopoly there is no distinction between firm and industry Being the only firm there is significant control of the firm over supply and price Thus under monopoly buyers do not have the option of buying the commodity from any other seller They have to buy the product from the firm or they can go without the commodity This fact gives immense control to the monopolist over the market

2No Close Substitute There are no close substitutes of the product produced by the monopolist firm If there are close substitutes of the product in the market it implies presence of more than one firm and hence no monopoly In order to ensure a total of control over the market by the monopolist firm it is assumed that there are no close substitutes of the product

3 No Entry amp Exit Monopoly can only exist when there is strong barriers before a new firm to enter the market In fact once a monopoly firm starts producing the product no other firm can produce the same One reason for this is the ability of the

monopolist to produce the product at a lower cost than any new firm who thinks to enter the market If a new firm who knows that it cannot produce at a lower cost than the monopolist then that firm will never enter the market for fear of losing out in competition Similarly the monopolist who is operating for a long time may be enjoying reputation among its customers and is in a better position to use the situation in its own benefit A new firm has to take long time to achieve this and so may not be interested to enter the market

4 Price Maker Being the single seller of the product the monopolist has full control over the pricing of the product On the other hand if there is a large number of buyers in the market so no single buyer exercises any significant influence over price determination Thus it is a sellerrsquos market So monopoly firm is a price maker

5 Price Discrimination Having considerable control over the market on account of being single seller with no entry of other firms the monopolist can exercise policy of price discrimination it means that the monopolist can sell different quantities of the same product to a consumer at different price or same quantity to different consumers at different prices by adjudging the standard of living of the consumer

6 Shape of Demand Curve Since a monopolist has full control over the price therefore he can sell more by lowering the price This makes the demand curve downward sloping

Subject Ac-12 290620 Topic- retirement Model sumThe Balance Sheet of Rohit Nisha and Sunil who are partners in a firm sharing profits according to their capitals as on 31st March 2014 was as under

Liabilities Amount Assets Amount (Rs) (` Rs)

Creditors 25000 Machinery 40000Bills Payable 13000 Building 90000General Reserve 22000 Debtors 30000Capital Less Provision for Rohit 60000 Bad debts 1000

29000 Nisha 40000 Stocks 23000 Sunil 40000 140000 Cash at Bank 18000

200000 200000

On the date of Balance Sheet Nisha retired from the firm and following adjustments were made(i) Building is appreciated by 20(ii) Provision for bad debts is increased to 5 on Debtors(iii) Machinery is depreciated by 10(iv) Goodwill of the firm is valued at Rs 56000 and the retiring partnerrsquos share is adjusted

(v) The capital of the new firm is fixed at Rs120000 Prepare Revaluation Account Capital Accounts of the partner and Balance Sheet of the new firm after Nisharsquos retirement Revaluation AccountDr Cr

Particulars Amount Particulars Amount (`Rs) (Rs`)

Provision for Bad debt Ac 500 Building Ac 18000Machinery Ac 4000Profit transferred toCapital Accounts (3 2 2)Rohit 5786Nisha 3857Sunil 3857

13500

18000 18000

Capital Account

Dr Cr

Particulars Rohit Nisha Sunil Particulars Rohit Nisha Sunil (Rs`) (Rs`) (`Rs) (Rs`) (Rs`) (Rs`)

Sunilrsquos Capital ac 9600 mdash 6400 Balance bd 60000 40000 40000Bank - 66143 - General Reserve 9428 6286 6286Balance cd 72000 mdash 48000 Revaluation (Profi 5786 3857 3857 Rohitrsquos Capital Ac mdash 9600 mdash

Sunilrsquos Capital Ac 6400 Bank 6386 - 4257

81600 66143 54400 81600 66143 54400

Balance Sheet as at 31st March 2014

Liabilities Amount Assets Amount (Rs`) (Rs`)

Creditors 25000 Building 108000Bank overdraft 37500 Machinery 36000

Bills Payable 13000 Debtors 30000Capital Less ProvisionRohit 72000 for Bad debts 1500 28500Sunil 48000 120000 Stock 23000

195500 195500

Working Notes (i) (a) Profit sharing ratio is 60000 40000 40000 ie = 3 2 2(b) Gaining Ratio Rohit = 35 ndash 37 = 2135 ndash 1535 = 635Sunil = 25-27 = 1435 ndash 1035 = 435= 635 435= 6 4 = 3 2(c) Nisha Share of Goodwill = Rs 56000 times 27 = Rs16000Share of Goodwill in the gaining ratio by the existing partner ieRohit = Rs16000 times 35 = Rs 9600Sunil = Rs 16000 times 25 = Rs 6400

The journal entry isRohitrsquos Capital Ac Dr 9600Sunilrsquos Capital Ac Dr 6400 To Nisharsquos Capital Ac 16000(Share of Goodwill divided into gaining ratio)

  • 1 Static Friction
  • The frictional force that acts between the surfaces when they are at rest with respect to each other is called Static Friction
    • Static Friction Examples
      • 2 Sliding Friction
        • Examples Of Sliding Friction
          • 3 Rolling Friction
            • Examples Of Rolling Friction
              • Objects and Reasons of the Forest Conservation Act
Page 47:  · Web viewSubject . Topic . Summary . Execution . English 1 . Sounds of animals . Hens –cackle Horses –neigh Lions –roar Owls –hoots Snake –hiss. English 2 . Mother’s

knowledge ability and overall job performance The following are the points which indicate the importance of performance appraisal in an organization

1 Performance appraisal helps supervisors to assess the work performance of their subordinates

2 Performance appraisal helps to assess the training and development needs of employees

3 Performance appraisal provides grounds for employees to correct their mistakes and it also provides proper guidance and criticism for employees development4 Performance appraisal provides reward for better performance

5 Performance appraisal helps to improve the communication system of the organization

6 Performance appraisal evaluates whether human resource programs being implemented in the organization have been effective

7 Performance appraisal helps to prepare pay structure for each employee working in the organization

8 Performance appraisal helps to review the potentiality of employees so that their future capability is anticipated

Geography

DRIANAGE The SubarnarekhaThe Subarnarekha and the Brahmaniinterposed between the Ganga and the Mahanadi deltas drain an area of 19300 sq kmand 39033 sq km respectively The drainage basins of these streams are shared byJharkhand Odisha west Bengal and Chhattisgarh The Brahmani is known as southKoel in its upper reaches in Jharkhand

The NarmadaThe Narmada rises in the Amarkantak hills of MadhyaPradesh It flows towards the West in a rift valleyformed due to a geological fault The total length of it is 1300 km All the tributaries of the

Q1 Name the two westward flowing rivers in the peninsular plateauA1 Narmada and Tapi are the only westward flowing rivers of the peninsular plateau

Q2 Differentiate between east-flowing rivers and west-flowing riversA2

East-flowing rivers

West-flowing rivers

Narmada are very short inlength Most of its tributaries join the main streamright anglesThe Narmada basin covers parts of Madhya Pradesh and Gujarat

The Tapi The Tapi rises in the Satpura ranges in the Betul listrictof Madhya Pradesh It flows in a rift valley parallel tothe Narmada but it is much shorter in length It coversparts of Madhya Pradesh Gujarat and MaharashtraThe length is about 724 km

The Sabarmati and the MahiThe Sabarmati rises in the Aravali hills and flows south-south-westwards for a distance of 300 kilometres to the Arabian Sea The Sabarmatibasin extends over an area of 21674 sq km in Rajasthan and Gujarat The Mahi rises inthe east of Udaipur and drains an area of 34842 sq km lying in Madhya PradeshRajasthan and Gujarat It flows south-westwards for a distance of 533 km before it fallsinto the Gulf of Khambhat

The ChambalThe Chambal rises near Mhow in the Vindhya Range and flows towards the northgenerally in a gorge upto Kota Below Kota it turns to the north-east direction and afterreaching Pinahat it turns to the east and runs nearly parallel to the Yamuna beforejoining it in the southern part of the Etawah district in Uttar PradeshMajor Rivers of India with their basin area (Sqkm)

Himalayan System Indus 321290Ganga 861404

Brahmaputra 187110Indus System

Jhelum 34775Beas 20303

Ganga System Yamuna 366223Ghaghra 127950

Peninsular RiversNarmada 98796

Tapi 65145Mahanadi 141600

Subarnarekha 19300Sabarmati 21674

Mahi 34842Godavari 312812

Godavari Krishna Kaveri Mahanadi are the east-flowing rivers

Narmada Tapi west-flowing rivers

They fall into the Bay of Bengal

They fall into Arabian Sea

These rivers form big deltas

These rivers form comparativelysmall deltas

Catchment areas of these rivers are larger

Catchment areas of these rivers are smaller

Krishna 2589488Cauveri 87900

Subject ndashBiology Topic ndashChapter -5 Inheritance amp Variations Summary ExecutionToday we will discussabout linkage and its classification

LINKAGE The tendency of the genes located on the same chromosome to stay together is

hereditary transmission Linked genes the genes responsible for this Genes that exhibit the process of linkage locates in the same chromosome The distance between the linked genes in a chromosome determines the strength

of linkage i e genes that are located close to each other show stronger linkage than that are located far from each other

COMPLETE LINKAGE It is the type of linkage showed by the genes that are closely located or are tightly

linked with each other as they have no chance of separatingby crossing over These genes are always transmitted together to the same gamete and the same

offspring In such condition only parental or non cross over type of gametes are formedINCOMPLETE KINKAGE It is type of linkage showed by the genes that are distantly located orare loosely

linked with each other because they have chance of separating by crossing over

SIGNIFICANCE i) It helps in holding the parental character togetherii) It checks the appearance of new recombination and helps in bringing the

hybrid population which resembles the original parents iii) Linked genes dilute the effects of undesirable traits

Subject Eng Literature (The Tempest ndash William Shakespeare) Topic Essay Questions (EQ-3)Question No 3

Give a character sketch of CalibanAnswer

The character of Caliban has been wonderfully conceived by Shakespeare as the manifestation of all that is gross and earthy ndash a sort of creature of the earth as Ariel is a sort of creature of the air

Calibanrsquos Physical Appearanceo Caliban is lsquofreckledrsquo a lsquomisshapen knaversquo not honoured with human shape

o Prospero calls him lsquothou tortoisersquo (Act I Sc 2 Line 317) Trinculo stumbling upon him describes him as ldquoA strange fish hellip Legged like a man And his fins like armsrdquo He ldquosmells like a fishrdquo (Act II Sc 2 Line 25)

o Prospero also calls him a ldquobeastrdquo (Act IV Sc 1 Line 140) and ldquoThis misshapen knaverdquo (Act V Sc 1 Line 268)

o Further it appears that in addition to his physical deformity his spiritual inferiority is also suggested by Prosperorsquos claim that his birth resulted from the union between his mother the witch Sycorax and the devil

Calibanrsquos ParentageWhen the play opens Caliban is twenty four years of age having been born on the island twelve years before the coming of Prospero His mother was the foul witch Sycorax who was banished from Algiers for ldquomischiefs manifold and sorceries terrible to enter human hearingrdquo (Act I Sc 2 Line 264) and the father was the Devil himself Thus

Caliban is a monster of evil and brute nature ugly deformed and stinking

Calibanrsquos Savage and Malignant Natureo Caliban is entirely a creature of the earth ndash gross brutal and savage He regards himself as the rightful possessor

of the island and Prospero as a usurper

o In his young age he was on good terms with Prospero He had consented to be received by Prospero at his house and to be educated by him He has learnt human language only to curse his master whom he abhors

o His beastly nature soon breaks out and ends in a vicious attack on Miranda This opens the eye of Prospero who becomes severe to him and enforces his service by threats and violence

o Prospero uses him to make dams for fish to fetch firewood scraper trenches wash dishes and keep his cell clean

Calibanrsquos Hatred for ProsperoA profound hatred for Prospero has taken hold of Caliban It springs from a sense of his being dispossessed and ill-treated He would kill Prospero if he could but he knows the power of Prosperorsquos lsquobookrsquo Hence he transfers his allegiance to Stephano who seems like a god to him He also incites the two drunken associates to batter the skull of Prospero when he sleeps in the afternoon

Caliban Shows Considerable Intelligenceo He has learnt Prosperorsquos language

ldquoYou taught me language and my profit onrsquot (Act II Sc 2 Lines 86-89)Is I know how to curserdquo

o He is well aware of the futility of arguing with one who has more power than he has

ldquoI must obey his art is such power (Act I Sc 2 Lines 373-376)It would control my damrsquos god SetebosAnd make a vassal of himrdquo

o He realizes the importance of Prosperorsquos books

ldquoRemember (Act III Sc 2 Lines 89-92)First to possess his books for without themHersquos but a sot as I am nor hath notOne spirit to commandrdquo

o He knows the value of stealth when attacking the enemy

ldquoPray you tread softly that the blind mole may not (Act IV Sc 1 Lines 194-195)Hear a foot fall we now are near his cellrdquo

o Caliban has a better set of values than Stephano and Trinculo They are distracted from their plan by their greed for Prosperorsquos rich garments Only Caliban realizes that such a finery is unimportant

ldquoLeave it alone thou fool it is but trashrdquo (Act IV Sc 1 Lines 224)

Caliban is not a good judge of characterCaliban is not a good judge of character He decides for example that Stephano is a god because he dispenses lsquocelestial liquorrsquo (Act II Sc 2 Line 115) but then it must be remembered that he has only known his mother Sycorax Prospero Miranda and the spirits that torture him However he quickly discovers his error of judgementrdquo

ldquoWhat a thrice-double ass (Act V Sc 1 Lines 295-297)Was I to take this drunkard for a godAnd worship this dull foolrdquo

Calibanrsquos Imaginative NatureIf Caliban is sub-human in what has been said above he is human in the respect of the poetic side of his character He listens to music with rapture He tells of the beautiful dreams in which heaven rains treasures upon him and which upon waking he yearns to renew One of the most poetic passages in whole play is Calibanrsquos description of the island

to Stephano and Trinculo

ldquoBe not afeard The isle is full of noises (Act III Sc 2 Lines 135-143)Sounds and sweet airs that give delight and hurt notSometimes a thousand twangling instrumentsWill hum about mine ears and sometime voicesThat if I then had waked after long sleepWill make me sleep again and then in dreamingThe clouds methought would open and show richesReady to drop upon me that when I wakedI cried to dream againrdquo

Caliban - Less Ignoble Than Some OthersCalibanrsquos motive for murder is less dishonourable than that of Antonio and Sebastian They plan to kill Alonso to gain his power and wealth Caliban merely wants revenge and the return of lsquohisrsquo island

Conclusiono Calibanrsquos character is not portrayed very clearly in the play and hence we cannot decide whether he is a poor

savage being grossly maltreated by Prospero or whether he is evil and must therefore be kept in bondage or enslavement

o Caliban is contrasted with Ariel who is a spirit and thus swift and uninterested in physical activitieso Caliban is also contrasted with Prospero who is the all-powerful master of the island and of the destiny of all

those on the islando Caliban is also contrasted with civilized man showing him to be less evil than Antonio and Stephano and less

materialistic than Stephano and Trinculoo Caliban has suffered at the hands of Prospero and he has learnt to curse by listening to Prosperorsquos abuse He

certainly believes that Prospero has deprived him of his birthrighto Finally the character Caliban is thought to be one of Shakespearersquos masterpieces The complexity of the character

is reflected in the large volume of critical discussion that has grown around it

ECO ndash12 Topic-Forms of market

MonopolyMonopoly is a market structure in which there is a single seller there are no close substitutes for the commodity produced by the firm and there are barriers to entry Example Indian Railways which is operated under government of India Monopoly also implies absence of competitionFeatures of Monopoly Monopoly is characterized by1 Single Seller In monopoly there is only one firm producing the product The whole industry consists of this single firm Thus under monopoly there is no distinction between firm and industry Being the only firm there is significant control of the firm over supply and price Thus under monopoly buyers do not have the option of buying the commodity from any other seller They have to buy the product from the firm or they can go without the commodity This fact gives immense control to the monopolist over the market

2No Close Substitute There are no close substitutes of the product produced by the monopolist firm If there are close substitutes of the product in the market it implies presence of more than one firm and hence no monopoly In order to ensure a total of control over the market by the monopolist firm it is assumed that there are no close substitutes of the product

3 No Entry amp Exit Monopoly can only exist when there is strong barriers before a new firm to enter the market In fact once a monopoly firm starts producing the product no other firm can produce the same One reason for this is the ability of the

monopolist to produce the product at a lower cost than any new firm who thinks to enter the market If a new firm who knows that it cannot produce at a lower cost than the monopolist then that firm will never enter the market for fear of losing out in competition Similarly the monopolist who is operating for a long time may be enjoying reputation among its customers and is in a better position to use the situation in its own benefit A new firm has to take long time to achieve this and so may not be interested to enter the market

4 Price Maker Being the single seller of the product the monopolist has full control over the pricing of the product On the other hand if there is a large number of buyers in the market so no single buyer exercises any significant influence over price determination Thus it is a sellerrsquos market So monopoly firm is a price maker

5 Price Discrimination Having considerable control over the market on account of being single seller with no entry of other firms the monopolist can exercise policy of price discrimination it means that the monopolist can sell different quantities of the same product to a consumer at different price or same quantity to different consumers at different prices by adjudging the standard of living of the consumer

6 Shape of Demand Curve Since a monopolist has full control over the price therefore he can sell more by lowering the price This makes the demand curve downward sloping

Subject Ac-12 290620 Topic- retirement Model sumThe Balance Sheet of Rohit Nisha and Sunil who are partners in a firm sharing profits according to their capitals as on 31st March 2014 was as under

Liabilities Amount Assets Amount (Rs) (` Rs)

Creditors 25000 Machinery 40000Bills Payable 13000 Building 90000General Reserve 22000 Debtors 30000Capital Less Provision for Rohit 60000 Bad debts 1000

29000 Nisha 40000 Stocks 23000 Sunil 40000 140000 Cash at Bank 18000

200000 200000

On the date of Balance Sheet Nisha retired from the firm and following adjustments were made(i) Building is appreciated by 20(ii) Provision for bad debts is increased to 5 on Debtors(iii) Machinery is depreciated by 10(iv) Goodwill of the firm is valued at Rs 56000 and the retiring partnerrsquos share is adjusted

(v) The capital of the new firm is fixed at Rs120000 Prepare Revaluation Account Capital Accounts of the partner and Balance Sheet of the new firm after Nisharsquos retirement Revaluation AccountDr Cr

Particulars Amount Particulars Amount (`Rs) (Rs`)

Provision for Bad debt Ac 500 Building Ac 18000Machinery Ac 4000Profit transferred toCapital Accounts (3 2 2)Rohit 5786Nisha 3857Sunil 3857

13500

18000 18000

Capital Account

Dr Cr

Particulars Rohit Nisha Sunil Particulars Rohit Nisha Sunil (Rs`) (Rs`) (`Rs) (Rs`) (Rs`) (Rs`)

Sunilrsquos Capital ac 9600 mdash 6400 Balance bd 60000 40000 40000Bank - 66143 - General Reserve 9428 6286 6286Balance cd 72000 mdash 48000 Revaluation (Profi 5786 3857 3857 Rohitrsquos Capital Ac mdash 9600 mdash

Sunilrsquos Capital Ac 6400 Bank 6386 - 4257

81600 66143 54400 81600 66143 54400

Balance Sheet as at 31st March 2014

Liabilities Amount Assets Amount (Rs`) (Rs`)

Creditors 25000 Building 108000Bank overdraft 37500 Machinery 36000

Bills Payable 13000 Debtors 30000Capital Less ProvisionRohit 72000 for Bad debts 1500 28500Sunil 48000 120000 Stock 23000

195500 195500

Working Notes (i) (a) Profit sharing ratio is 60000 40000 40000 ie = 3 2 2(b) Gaining Ratio Rohit = 35 ndash 37 = 2135 ndash 1535 = 635Sunil = 25-27 = 1435 ndash 1035 = 435= 635 435= 6 4 = 3 2(c) Nisha Share of Goodwill = Rs 56000 times 27 = Rs16000Share of Goodwill in the gaining ratio by the existing partner ieRohit = Rs16000 times 35 = Rs 9600Sunil = Rs 16000 times 25 = Rs 6400

The journal entry isRohitrsquos Capital Ac Dr 9600Sunilrsquos Capital Ac Dr 6400 To Nisharsquos Capital Ac 16000(Share of Goodwill divided into gaining ratio)

  • 1 Static Friction
  • The frictional force that acts between the surfaces when they are at rest with respect to each other is called Static Friction
    • Static Friction Examples
      • 2 Sliding Friction
        • Examples Of Sliding Friction
          • 3 Rolling Friction
            • Examples Of Rolling Friction
              • Objects and Reasons of the Forest Conservation Act
Page 48:  · Web viewSubject . Topic . Summary . Execution . English 1 . Sounds of animals . Hens –cackle Horses –neigh Lions –roar Owls –hoots Snake –hiss. English 2 . Mother’s

Narmada are very short inlength Most of its tributaries join the main streamright anglesThe Narmada basin covers parts of Madhya Pradesh and Gujarat

The Tapi The Tapi rises in the Satpura ranges in the Betul listrictof Madhya Pradesh It flows in a rift valley parallel tothe Narmada but it is much shorter in length It coversparts of Madhya Pradesh Gujarat and MaharashtraThe length is about 724 km

The Sabarmati and the MahiThe Sabarmati rises in the Aravali hills and flows south-south-westwards for a distance of 300 kilometres to the Arabian Sea The Sabarmatibasin extends over an area of 21674 sq km in Rajasthan and Gujarat The Mahi rises inthe east of Udaipur and drains an area of 34842 sq km lying in Madhya PradeshRajasthan and Gujarat It flows south-westwards for a distance of 533 km before it fallsinto the Gulf of Khambhat

The ChambalThe Chambal rises near Mhow in the Vindhya Range and flows towards the northgenerally in a gorge upto Kota Below Kota it turns to the north-east direction and afterreaching Pinahat it turns to the east and runs nearly parallel to the Yamuna beforejoining it in the southern part of the Etawah district in Uttar PradeshMajor Rivers of India with their basin area (Sqkm)

Himalayan System Indus 321290Ganga 861404

Brahmaputra 187110Indus System

Jhelum 34775Beas 20303

Ganga System Yamuna 366223Ghaghra 127950

Peninsular RiversNarmada 98796

Tapi 65145Mahanadi 141600

Subarnarekha 19300Sabarmati 21674

Mahi 34842Godavari 312812

Godavari Krishna Kaveri Mahanadi are the east-flowing rivers

Narmada Tapi west-flowing rivers

They fall into the Bay of Bengal

They fall into Arabian Sea

These rivers form big deltas

These rivers form comparativelysmall deltas

Catchment areas of these rivers are larger

Catchment areas of these rivers are smaller

Krishna 2589488Cauveri 87900

Subject ndashBiology Topic ndashChapter -5 Inheritance amp Variations Summary ExecutionToday we will discussabout linkage and its classification

LINKAGE The tendency of the genes located on the same chromosome to stay together is

hereditary transmission Linked genes the genes responsible for this Genes that exhibit the process of linkage locates in the same chromosome The distance between the linked genes in a chromosome determines the strength

of linkage i e genes that are located close to each other show stronger linkage than that are located far from each other

COMPLETE LINKAGE It is the type of linkage showed by the genes that are closely located or are tightly

linked with each other as they have no chance of separatingby crossing over These genes are always transmitted together to the same gamete and the same

offspring In such condition only parental or non cross over type of gametes are formedINCOMPLETE KINKAGE It is type of linkage showed by the genes that are distantly located orare loosely

linked with each other because they have chance of separating by crossing over

SIGNIFICANCE i) It helps in holding the parental character togetherii) It checks the appearance of new recombination and helps in bringing the

hybrid population which resembles the original parents iii) Linked genes dilute the effects of undesirable traits

Subject Eng Literature (The Tempest ndash William Shakespeare) Topic Essay Questions (EQ-3)Question No 3

Give a character sketch of CalibanAnswer

The character of Caliban has been wonderfully conceived by Shakespeare as the manifestation of all that is gross and earthy ndash a sort of creature of the earth as Ariel is a sort of creature of the air

Calibanrsquos Physical Appearanceo Caliban is lsquofreckledrsquo a lsquomisshapen knaversquo not honoured with human shape

o Prospero calls him lsquothou tortoisersquo (Act I Sc 2 Line 317) Trinculo stumbling upon him describes him as ldquoA strange fish hellip Legged like a man And his fins like armsrdquo He ldquosmells like a fishrdquo (Act II Sc 2 Line 25)

o Prospero also calls him a ldquobeastrdquo (Act IV Sc 1 Line 140) and ldquoThis misshapen knaverdquo (Act V Sc 1 Line 268)

o Further it appears that in addition to his physical deformity his spiritual inferiority is also suggested by Prosperorsquos claim that his birth resulted from the union between his mother the witch Sycorax and the devil

Calibanrsquos ParentageWhen the play opens Caliban is twenty four years of age having been born on the island twelve years before the coming of Prospero His mother was the foul witch Sycorax who was banished from Algiers for ldquomischiefs manifold and sorceries terrible to enter human hearingrdquo (Act I Sc 2 Line 264) and the father was the Devil himself Thus

Caliban is a monster of evil and brute nature ugly deformed and stinking

Calibanrsquos Savage and Malignant Natureo Caliban is entirely a creature of the earth ndash gross brutal and savage He regards himself as the rightful possessor

of the island and Prospero as a usurper

o In his young age he was on good terms with Prospero He had consented to be received by Prospero at his house and to be educated by him He has learnt human language only to curse his master whom he abhors

o His beastly nature soon breaks out and ends in a vicious attack on Miranda This opens the eye of Prospero who becomes severe to him and enforces his service by threats and violence

o Prospero uses him to make dams for fish to fetch firewood scraper trenches wash dishes and keep his cell clean

Calibanrsquos Hatred for ProsperoA profound hatred for Prospero has taken hold of Caliban It springs from a sense of his being dispossessed and ill-treated He would kill Prospero if he could but he knows the power of Prosperorsquos lsquobookrsquo Hence he transfers his allegiance to Stephano who seems like a god to him He also incites the two drunken associates to batter the skull of Prospero when he sleeps in the afternoon

Caliban Shows Considerable Intelligenceo He has learnt Prosperorsquos language

ldquoYou taught me language and my profit onrsquot (Act II Sc 2 Lines 86-89)Is I know how to curserdquo

o He is well aware of the futility of arguing with one who has more power than he has

ldquoI must obey his art is such power (Act I Sc 2 Lines 373-376)It would control my damrsquos god SetebosAnd make a vassal of himrdquo

o He realizes the importance of Prosperorsquos books

ldquoRemember (Act III Sc 2 Lines 89-92)First to possess his books for without themHersquos but a sot as I am nor hath notOne spirit to commandrdquo

o He knows the value of stealth when attacking the enemy

ldquoPray you tread softly that the blind mole may not (Act IV Sc 1 Lines 194-195)Hear a foot fall we now are near his cellrdquo

o Caliban has a better set of values than Stephano and Trinculo They are distracted from their plan by their greed for Prosperorsquos rich garments Only Caliban realizes that such a finery is unimportant

ldquoLeave it alone thou fool it is but trashrdquo (Act IV Sc 1 Lines 224)

Caliban is not a good judge of characterCaliban is not a good judge of character He decides for example that Stephano is a god because he dispenses lsquocelestial liquorrsquo (Act II Sc 2 Line 115) but then it must be remembered that he has only known his mother Sycorax Prospero Miranda and the spirits that torture him However he quickly discovers his error of judgementrdquo

ldquoWhat a thrice-double ass (Act V Sc 1 Lines 295-297)Was I to take this drunkard for a godAnd worship this dull foolrdquo

Calibanrsquos Imaginative NatureIf Caliban is sub-human in what has been said above he is human in the respect of the poetic side of his character He listens to music with rapture He tells of the beautiful dreams in which heaven rains treasures upon him and which upon waking he yearns to renew One of the most poetic passages in whole play is Calibanrsquos description of the island

to Stephano and Trinculo

ldquoBe not afeard The isle is full of noises (Act III Sc 2 Lines 135-143)Sounds and sweet airs that give delight and hurt notSometimes a thousand twangling instrumentsWill hum about mine ears and sometime voicesThat if I then had waked after long sleepWill make me sleep again and then in dreamingThe clouds methought would open and show richesReady to drop upon me that when I wakedI cried to dream againrdquo

Caliban - Less Ignoble Than Some OthersCalibanrsquos motive for murder is less dishonourable than that of Antonio and Sebastian They plan to kill Alonso to gain his power and wealth Caliban merely wants revenge and the return of lsquohisrsquo island

Conclusiono Calibanrsquos character is not portrayed very clearly in the play and hence we cannot decide whether he is a poor

savage being grossly maltreated by Prospero or whether he is evil and must therefore be kept in bondage or enslavement

o Caliban is contrasted with Ariel who is a spirit and thus swift and uninterested in physical activitieso Caliban is also contrasted with Prospero who is the all-powerful master of the island and of the destiny of all

those on the islando Caliban is also contrasted with civilized man showing him to be less evil than Antonio and Stephano and less

materialistic than Stephano and Trinculoo Caliban has suffered at the hands of Prospero and he has learnt to curse by listening to Prosperorsquos abuse He

certainly believes that Prospero has deprived him of his birthrighto Finally the character Caliban is thought to be one of Shakespearersquos masterpieces The complexity of the character

is reflected in the large volume of critical discussion that has grown around it

ECO ndash12 Topic-Forms of market

MonopolyMonopoly is a market structure in which there is a single seller there are no close substitutes for the commodity produced by the firm and there are barriers to entry Example Indian Railways which is operated under government of India Monopoly also implies absence of competitionFeatures of Monopoly Monopoly is characterized by1 Single Seller In monopoly there is only one firm producing the product The whole industry consists of this single firm Thus under monopoly there is no distinction between firm and industry Being the only firm there is significant control of the firm over supply and price Thus under monopoly buyers do not have the option of buying the commodity from any other seller They have to buy the product from the firm or they can go without the commodity This fact gives immense control to the monopolist over the market

2No Close Substitute There are no close substitutes of the product produced by the monopolist firm If there are close substitutes of the product in the market it implies presence of more than one firm and hence no monopoly In order to ensure a total of control over the market by the monopolist firm it is assumed that there are no close substitutes of the product

3 No Entry amp Exit Monopoly can only exist when there is strong barriers before a new firm to enter the market In fact once a monopoly firm starts producing the product no other firm can produce the same One reason for this is the ability of the

monopolist to produce the product at a lower cost than any new firm who thinks to enter the market If a new firm who knows that it cannot produce at a lower cost than the monopolist then that firm will never enter the market for fear of losing out in competition Similarly the monopolist who is operating for a long time may be enjoying reputation among its customers and is in a better position to use the situation in its own benefit A new firm has to take long time to achieve this and so may not be interested to enter the market

4 Price Maker Being the single seller of the product the monopolist has full control over the pricing of the product On the other hand if there is a large number of buyers in the market so no single buyer exercises any significant influence over price determination Thus it is a sellerrsquos market So monopoly firm is a price maker

5 Price Discrimination Having considerable control over the market on account of being single seller with no entry of other firms the monopolist can exercise policy of price discrimination it means that the monopolist can sell different quantities of the same product to a consumer at different price or same quantity to different consumers at different prices by adjudging the standard of living of the consumer

6 Shape of Demand Curve Since a monopolist has full control over the price therefore he can sell more by lowering the price This makes the demand curve downward sloping

Subject Ac-12 290620 Topic- retirement Model sumThe Balance Sheet of Rohit Nisha and Sunil who are partners in a firm sharing profits according to their capitals as on 31st March 2014 was as under

Liabilities Amount Assets Amount (Rs) (` Rs)

Creditors 25000 Machinery 40000Bills Payable 13000 Building 90000General Reserve 22000 Debtors 30000Capital Less Provision for Rohit 60000 Bad debts 1000

29000 Nisha 40000 Stocks 23000 Sunil 40000 140000 Cash at Bank 18000

200000 200000

On the date of Balance Sheet Nisha retired from the firm and following adjustments were made(i) Building is appreciated by 20(ii) Provision for bad debts is increased to 5 on Debtors(iii) Machinery is depreciated by 10(iv) Goodwill of the firm is valued at Rs 56000 and the retiring partnerrsquos share is adjusted

(v) The capital of the new firm is fixed at Rs120000 Prepare Revaluation Account Capital Accounts of the partner and Balance Sheet of the new firm after Nisharsquos retirement Revaluation AccountDr Cr

Particulars Amount Particulars Amount (`Rs) (Rs`)

Provision for Bad debt Ac 500 Building Ac 18000Machinery Ac 4000Profit transferred toCapital Accounts (3 2 2)Rohit 5786Nisha 3857Sunil 3857

13500

18000 18000

Capital Account

Dr Cr

Particulars Rohit Nisha Sunil Particulars Rohit Nisha Sunil (Rs`) (Rs`) (`Rs) (Rs`) (Rs`) (Rs`)

Sunilrsquos Capital ac 9600 mdash 6400 Balance bd 60000 40000 40000Bank - 66143 - General Reserve 9428 6286 6286Balance cd 72000 mdash 48000 Revaluation (Profi 5786 3857 3857 Rohitrsquos Capital Ac mdash 9600 mdash

Sunilrsquos Capital Ac 6400 Bank 6386 - 4257

81600 66143 54400 81600 66143 54400

Balance Sheet as at 31st March 2014

Liabilities Amount Assets Amount (Rs`) (Rs`)

Creditors 25000 Building 108000Bank overdraft 37500 Machinery 36000

Bills Payable 13000 Debtors 30000Capital Less ProvisionRohit 72000 for Bad debts 1500 28500Sunil 48000 120000 Stock 23000

195500 195500

Working Notes (i) (a) Profit sharing ratio is 60000 40000 40000 ie = 3 2 2(b) Gaining Ratio Rohit = 35 ndash 37 = 2135 ndash 1535 = 635Sunil = 25-27 = 1435 ndash 1035 = 435= 635 435= 6 4 = 3 2(c) Nisha Share of Goodwill = Rs 56000 times 27 = Rs16000Share of Goodwill in the gaining ratio by the existing partner ieRohit = Rs16000 times 35 = Rs 9600Sunil = Rs 16000 times 25 = Rs 6400

The journal entry isRohitrsquos Capital Ac Dr 9600Sunilrsquos Capital Ac Dr 6400 To Nisharsquos Capital Ac 16000(Share of Goodwill divided into gaining ratio)

  • 1 Static Friction
  • The frictional force that acts between the surfaces when they are at rest with respect to each other is called Static Friction
    • Static Friction Examples
      • 2 Sliding Friction
        • Examples Of Sliding Friction
          • 3 Rolling Friction
            • Examples Of Rolling Friction
              • Objects and Reasons of the Forest Conservation Act
Page 49:  · Web viewSubject . Topic . Summary . Execution . English 1 . Sounds of animals . Hens –cackle Horses –neigh Lions –roar Owls –hoots Snake –hiss. English 2 . Mother’s

Krishna 2589488Cauveri 87900

Subject ndashBiology Topic ndashChapter -5 Inheritance amp Variations Summary ExecutionToday we will discussabout linkage and its classification

LINKAGE The tendency of the genes located on the same chromosome to stay together is

hereditary transmission Linked genes the genes responsible for this Genes that exhibit the process of linkage locates in the same chromosome The distance between the linked genes in a chromosome determines the strength

of linkage i e genes that are located close to each other show stronger linkage than that are located far from each other

COMPLETE LINKAGE It is the type of linkage showed by the genes that are closely located or are tightly

linked with each other as they have no chance of separatingby crossing over These genes are always transmitted together to the same gamete and the same

offspring In such condition only parental or non cross over type of gametes are formedINCOMPLETE KINKAGE It is type of linkage showed by the genes that are distantly located orare loosely

linked with each other because they have chance of separating by crossing over

SIGNIFICANCE i) It helps in holding the parental character togetherii) It checks the appearance of new recombination and helps in bringing the

hybrid population which resembles the original parents iii) Linked genes dilute the effects of undesirable traits

Subject Eng Literature (The Tempest ndash William Shakespeare) Topic Essay Questions (EQ-3)Question No 3

Give a character sketch of CalibanAnswer

The character of Caliban has been wonderfully conceived by Shakespeare as the manifestation of all that is gross and earthy ndash a sort of creature of the earth as Ariel is a sort of creature of the air

Calibanrsquos Physical Appearanceo Caliban is lsquofreckledrsquo a lsquomisshapen knaversquo not honoured with human shape

o Prospero calls him lsquothou tortoisersquo (Act I Sc 2 Line 317) Trinculo stumbling upon him describes him as ldquoA strange fish hellip Legged like a man And his fins like armsrdquo He ldquosmells like a fishrdquo (Act II Sc 2 Line 25)

o Prospero also calls him a ldquobeastrdquo (Act IV Sc 1 Line 140) and ldquoThis misshapen knaverdquo (Act V Sc 1 Line 268)

o Further it appears that in addition to his physical deformity his spiritual inferiority is also suggested by Prosperorsquos claim that his birth resulted from the union between his mother the witch Sycorax and the devil

Calibanrsquos ParentageWhen the play opens Caliban is twenty four years of age having been born on the island twelve years before the coming of Prospero His mother was the foul witch Sycorax who was banished from Algiers for ldquomischiefs manifold and sorceries terrible to enter human hearingrdquo (Act I Sc 2 Line 264) and the father was the Devil himself Thus

Caliban is a monster of evil and brute nature ugly deformed and stinking

Calibanrsquos Savage and Malignant Natureo Caliban is entirely a creature of the earth ndash gross brutal and savage He regards himself as the rightful possessor

of the island and Prospero as a usurper

o In his young age he was on good terms with Prospero He had consented to be received by Prospero at his house and to be educated by him He has learnt human language only to curse his master whom he abhors

o His beastly nature soon breaks out and ends in a vicious attack on Miranda This opens the eye of Prospero who becomes severe to him and enforces his service by threats and violence

o Prospero uses him to make dams for fish to fetch firewood scraper trenches wash dishes and keep his cell clean

Calibanrsquos Hatred for ProsperoA profound hatred for Prospero has taken hold of Caliban It springs from a sense of his being dispossessed and ill-treated He would kill Prospero if he could but he knows the power of Prosperorsquos lsquobookrsquo Hence he transfers his allegiance to Stephano who seems like a god to him He also incites the two drunken associates to batter the skull of Prospero when he sleeps in the afternoon

Caliban Shows Considerable Intelligenceo He has learnt Prosperorsquos language

ldquoYou taught me language and my profit onrsquot (Act II Sc 2 Lines 86-89)Is I know how to curserdquo

o He is well aware of the futility of arguing with one who has more power than he has

ldquoI must obey his art is such power (Act I Sc 2 Lines 373-376)It would control my damrsquos god SetebosAnd make a vassal of himrdquo

o He realizes the importance of Prosperorsquos books

ldquoRemember (Act III Sc 2 Lines 89-92)First to possess his books for without themHersquos but a sot as I am nor hath notOne spirit to commandrdquo

o He knows the value of stealth when attacking the enemy

ldquoPray you tread softly that the blind mole may not (Act IV Sc 1 Lines 194-195)Hear a foot fall we now are near his cellrdquo

o Caliban has a better set of values than Stephano and Trinculo They are distracted from their plan by their greed for Prosperorsquos rich garments Only Caliban realizes that such a finery is unimportant

ldquoLeave it alone thou fool it is but trashrdquo (Act IV Sc 1 Lines 224)

Caliban is not a good judge of characterCaliban is not a good judge of character He decides for example that Stephano is a god because he dispenses lsquocelestial liquorrsquo (Act II Sc 2 Line 115) but then it must be remembered that he has only known his mother Sycorax Prospero Miranda and the spirits that torture him However he quickly discovers his error of judgementrdquo

ldquoWhat a thrice-double ass (Act V Sc 1 Lines 295-297)Was I to take this drunkard for a godAnd worship this dull foolrdquo

Calibanrsquos Imaginative NatureIf Caliban is sub-human in what has been said above he is human in the respect of the poetic side of his character He listens to music with rapture He tells of the beautiful dreams in which heaven rains treasures upon him and which upon waking he yearns to renew One of the most poetic passages in whole play is Calibanrsquos description of the island

to Stephano and Trinculo

ldquoBe not afeard The isle is full of noises (Act III Sc 2 Lines 135-143)Sounds and sweet airs that give delight and hurt notSometimes a thousand twangling instrumentsWill hum about mine ears and sometime voicesThat if I then had waked after long sleepWill make me sleep again and then in dreamingThe clouds methought would open and show richesReady to drop upon me that when I wakedI cried to dream againrdquo

Caliban - Less Ignoble Than Some OthersCalibanrsquos motive for murder is less dishonourable than that of Antonio and Sebastian They plan to kill Alonso to gain his power and wealth Caliban merely wants revenge and the return of lsquohisrsquo island

Conclusiono Calibanrsquos character is not portrayed very clearly in the play and hence we cannot decide whether he is a poor

savage being grossly maltreated by Prospero or whether he is evil and must therefore be kept in bondage or enslavement

o Caliban is contrasted with Ariel who is a spirit and thus swift and uninterested in physical activitieso Caliban is also contrasted with Prospero who is the all-powerful master of the island and of the destiny of all

those on the islando Caliban is also contrasted with civilized man showing him to be less evil than Antonio and Stephano and less

materialistic than Stephano and Trinculoo Caliban has suffered at the hands of Prospero and he has learnt to curse by listening to Prosperorsquos abuse He

certainly believes that Prospero has deprived him of his birthrighto Finally the character Caliban is thought to be one of Shakespearersquos masterpieces The complexity of the character

is reflected in the large volume of critical discussion that has grown around it

ECO ndash12 Topic-Forms of market

MonopolyMonopoly is a market structure in which there is a single seller there are no close substitutes for the commodity produced by the firm and there are barriers to entry Example Indian Railways which is operated under government of India Monopoly also implies absence of competitionFeatures of Monopoly Monopoly is characterized by1 Single Seller In monopoly there is only one firm producing the product The whole industry consists of this single firm Thus under monopoly there is no distinction between firm and industry Being the only firm there is significant control of the firm over supply and price Thus under monopoly buyers do not have the option of buying the commodity from any other seller They have to buy the product from the firm or they can go without the commodity This fact gives immense control to the monopolist over the market

2No Close Substitute There are no close substitutes of the product produced by the monopolist firm If there are close substitutes of the product in the market it implies presence of more than one firm and hence no monopoly In order to ensure a total of control over the market by the monopolist firm it is assumed that there are no close substitutes of the product

3 No Entry amp Exit Monopoly can only exist when there is strong barriers before a new firm to enter the market In fact once a monopoly firm starts producing the product no other firm can produce the same One reason for this is the ability of the

monopolist to produce the product at a lower cost than any new firm who thinks to enter the market If a new firm who knows that it cannot produce at a lower cost than the monopolist then that firm will never enter the market for fear of losing out in competition Similarly the monopolist who is operating for a long time may be enjoying reputation among its customers and is in a better position to use the situation in its own benefit A new firm has to take long time to achieve this and so may not be interested to enter the market

4 Price Maker Being the single seller of the product the monopolist has full control over the pricing of the product On the other hand if there is a large number of buyers in the market so no single buyer exercises any significant influence over price determination Thus it is a sellerrsquos market So monopoly firm is a price maker

5 Price Discrimination Having considerable control over the market on account of being single seller with no entry of other firms the monopolist can exercise policy of price discrimination it means that the monopolist can sell different quantities of the same product to a consumer at different price or same quantity to different consumers at different prices by adjudging the standard of living of the consumer

6 Shape of Demand Curve Since a monopolist has full control over the price therefore he can sell more by lowering the price This makes the demand curve downward sloping

Subject Ac-12 290620 Topic- retirement Model sumThe Balance Sheet of Rohit Nisha and Sunil who are partners in a firm sharing profits according to their capitals as on 31st March 2014 was as under

Liabilities Amount Assets Amount (Rs) (` Rs)

Creditors 25000 Machinery 40000Bills Payable 13000 Building 90000General Reserve 22000 Debtors 30000Capital Less Provision for Rohit 60000 Bad debts 1000

29000 Nisha 40000 Stocks 23000 Sunil 40000 140000 Cash at Bank 18000

200000 200000

On the date of Balance Sheet Nisha retired from the firm and following adjustments were made(i) Building is appreciated by 20(ii) Provision for bad debts is increased to 5 on Debtors(iii) Machinery is depreciated by 10(iv) Goodwill of the firm is valued at Rs 56000 and the retiring partnerrsquos share is adjusted

(v) The capital of the new firm is fixed at Rs120000 Prepare Revaluation Account Capital Accounts of the partner and Balance Sheet of the new firm after Nisharsquos retirement Revaluation AccountDr Cr

Particulars Amount Particulars Amount (`Rs) (Rs`)

Provision for Bad debt Ac 500 Building Ac 18000Machinery Ac 4000Profit transferred toCapital Accounts (3 2 2)Rohit 5786Nisha 3857Sunil 3857

13500

18000 18000

Capital Account

Dr Cr

Particulars Rohit Nisha Sunil Particulars Rohit Nisha Sunil (Rs`) (Rs`) (`Rs) (Rs`) (Rs`) (Rs`)

Sunilrsquos Capital ac 9600 mdash 6400 Balance bd 60000 40000 40000Bank - 66143 - General Reserve 9428 6286 6286Balance cd 72000 mdash 48000 Revaluation (Profi 5786 3857 3857 Rohitrsquos Capital Ac mdash 9600 mdash

Sunilrsquos Capital Ac 6400 Bank 6386 - 4257

81600 66143 54400 81600 66143 54400

Balance Sheet as at 31st March 2014

Liabilities Amount Assets Amount (Rs`) (Rs`)

Creditors 25000 Building 108000Bank overdraft 37500 Machinery 36000

Bills Payable 13000 Debtors 30000Capital Less ProvisionRohit 72000 for Bad debts 1500 28500Sunil 48000 120000 Stock 23000

195500 195500

Working Notes (i) (a) Profit sharing ratio is 60000 40000 40000 ie = 3 2 2(b) Gaining Ratio Rohit = 35 ndash 37 = 2135 ndash 1535 = 635Sunil = 25-27 = 1435 ndash 1035 = 435= 635 435= 6 4 = 3 2(c) Nisha Share of Goodwill = Rs 56000 times 27 = Rs16000Share of Goodwill in the gaining ratio by the existing partner ieRohit = Rs16000 times 35 = Rs 9600Sunil = Rs 16000 times 25 = Rs 6400

The journal entry isRohitrsquos Capital Ac Dr 9600Sunilrsquos Capital Ac Dr 6400 To Nisharsquos Capital Ac 16000(Share of Goodwill divided into gaining ratio)

  • 1 Static Friction
  • The frictional force that acts between the surfaces when they are at rest with respect to each other is called Static Friction
    • Static Friction Examples
      • 2 Sliding Friction
        • Examples Of Sliding Friction
          • 3 Rolling Friction
            • Examples Of Rolling Friction
              • Objects and Reasons of the Forest Conservation Act
Page 50:  · Web viewSubject . Topic . Summary . Execution . English 1 . Sounds of animals . Hens –cackle Horses –neigh Lions –roar Owls –hoots Snake –hiss. English 2 . Mother’s

Caliban is a monster of evil and brute nature ugly deformed and stinking

Calibanrsquos Savage and Malignant Natureo Caliban is entirely a creature of the earth ndash gross brutal and savage He regards himself as the rightful possessor

of the island and Prospero as a usurper

o In his young age he was on good terms with Prospero He had consented to be received by Prospero at his house and to be educated by him He has learnt human language only to curse his master whom he abhors

o His beastly nature soon breaks out and ends in a vicious attack on Miranda This opens the eye of Prospero who becomes severe to him and enforces his service by threats and violence

o Prospero uses him to make dams for fish to fetch firewood scraper trenches wash dishes and keep his cell clean

Calibanrsquos Hatred for ProsperoA profound hatred for Prospero has taken hold of Caliban It springs from a sense of his being dispossessed and ill-treated He would kill Prospero if he could but he knows the power of Prosperorsquos lsquobookrsquo Hence he transfers his allegiance to Stephano who seems like a god to him He also incites the two drunken associates to batter the skull of Prospero when he sleeps in the afternoon

Caliban Shows Considerable Intelligenceo He has learnt Prosperorsquos language

ldquoYou taught me language and my profit onrsquot (Act II Sc 2 Lines 86-89)Is I know how to curserdquo

o He is well aware of the futility of arguing with one who has more power than he has

ldquoI must obey his art is such power (Act I Sc 2 Lines 373-376)It would control my damrsquos god SetebosAnd make a vassal of himrdquo

o He realizes the importance of Prosperorsquos books

ldquoRemember (Act III Sc 2 Lines 89-92)First to possess his books for without themHersquos but a sot as I am nor hath notOne spirit to commandrdquo

o He knows the value of stealth when attacking the enemy

ldquoPray you tread softly that the blind mole may not (Act IV Sc 1 Lines 194-195)Hear a foot fall we now are near his cellrdquo

o Caliban has a better set of values than Stephano and Trinculo They are distracted from their plan by their greed for Prosperorsquos rich garments Only Caliban realizes that such a finery is unimportant

ldquoLeave it alone thou fool it is but trashrdquo (Act IV Sc 1 Lines 224)

Caliban is not a good judge of characterCaliban is not a good judge of character He decides for example that Stephano is a god because he dispenses lsquocelestial liquorrsquo (Act II Sc 2 Line 115) but then it must be remembered that he has only known his mother Sycorax Prospero Miranda and the spirits that torture him However he quickly discovers his error of judgementrdquo

ldquoWhat a thrice-double ass (Act V Sc 1 Lines 295-297)Was I to take this drunkard for a godAnd worship this dull foolrdquo

Calibanrsquos Imaginative NatureIf Caliban is sub-human in what has been said above he is human in the respect of the poetic side of his character He listens to music with rapture He tells of the beautiful dreams in which heaven rains treasures upon him and which upon waking he yearns to renew One of the most poetic passages in whole play is Calibanrsquos description of the island

to Stephano and Trinculo

ldquoBe not afeard The isle is full of noises (Act III Sc 2 Lines 135-143)Sounds and sweet airs that give delight and hurt notSometimes a thousand twangling instrumentsWill hum about mine ears and sometime voicesThat if I then had waked after long sleepWill make me sleep again and then in dreamingThe clouds methought would open and show richesReady to drop upon me that when I wakedI cried to dream againrdquo

Caliban - Less Ignoble Than Some OthersCalibanrsquos motive for murder is less dishonourable than that of Antonio and Sebastian They plan to kill Alonso to gain his power and wealth Caliban merely wants revenge and the return of lsquohisrsquo island

Conclusiono Calibanrsquos character is not portrayed very clearly in the play and hence we cannot decide whether he is a poor

savage being grossly maltreated by Prospero or whether he is evil and must therefore be kept in bondage or enslavement

o Caliban is contrasted with Ariel who is a spirit and thus swift and uninterested in physical activitieso Caliban is also contrasted with Prospero who is the all-powerful master of the island and of the destiny of all

those on the islando Caliban is also contrasted with civilized man showing him to be less evil than Antonio and Stephano and less

materialistic than Stephano and Trinculoo Caliban has suffered at the hands of Prospero and he has learnt to curse by listening to Prosperorsquos abuse He

certainly believes that Prospero has deprived him of his birthrighto Finally the character Caliban is thought to be one of Shakespearersquos masterpieces The complexity of the character

is reflected in the large volume of critical discussion that has grown around it

ECO ndash12 Topic-Forms of market

MonopolyMonopoly is a market structure in which there is a single seller there are no close substitutes for the commodity produced by the firm and there are barriers to entry Example Indian Railways which is operated under government of India Monopoly also implies absence of competitionFeatures of Monopoly Monopoly is characterized by1 Single Seller In monopoly there is only one firm producing the product The whole industry consists of this single firm Thus under monopoly there is no distinction between firm and industry Being the only firm there is significant control of the firm over supply and price Thus under monopoly buyers do not have the option of buying the commodity from any other seller They have to buy the product from the firm or they can go without the commodity This fact gives immense control to the monopolist over the market

2No Close Substitute There are no close substitutes of the product produced by the monopolist firm If there are close substitutes of the product in the market it implies presence of more than one firm and hence no monopoly In order to ensure a total of control over the market by the monopolist firm it is assumed that there are no close substitutes of the product

3 No Entry amp Exit Monopoly can only exist when there is strong barriers before a new firm to enter the market In fact once a monopoly firm starts producing the product no other firm can produce the same One reason for this is the ability of the

monopolist to produce the product at a lower cost than any new firm who thinks to enter the market If a new firm who knows that it cannot produce at a lower cost than the monopolist then that firm will never enter the market for fear of losing out in competition Similarly the monopolist who is operating for a long time may be enjoying reputation among its customers and is in a better position to use the situation in its own benefit A new firm has to take long time to achieve this and so may not be interested to enter the market

4 Price Maker Being the single seller of the product the monopolist has full control over the pricing of the product On the other hand if there is a large number of buyers in the market so no single buyer exercises any significant influence over price determination Thus it is a sellerrsquos market So monopoly firm is a price maker

5 Price Discrimination Having considerable control over the market on account of being single seller with no entry of other firms the monopolist can exercise policy of price discrimination it means that the monopolist can sell different quantities of the same product to a consumer at different price or same quantity to different consumers at different prices by adjudging the standard of living of the consumer

6 Shape of Demand Curve Since a monopolist has full control over the price therefore he can sell more by lowering the price This makes the demand curve downward sloping

Subject Ac-12 290620 Topic- retirement Model sumThe Balance Sheet of Rohit Nisha and Sunil who are partners in a firm sharing profits according to their capitals as on 31st March 2014 was as under

Liabilities Amount Assets Amount (Rs) (` Rs)

Creditors 25000 Machinery 40000Bills Payable 13000 Building 90000General Reserve 22000 Debtors 30000Capital Less Provision for Rohit 60000 Bad debts 1000

29000 Nisha 40000 Stocks 23000 Sunil 40000 140000 Cash at Bank 18000

200000 200000

On the date of Balance Sheet Nisha retired from the firm and following adjustments were made(i) Building is appreciated by 20(ii) Provision for bad debts is increased to 5 on Debtors(iii) Machinery is depreciated by 10(iv) Goodwill of the firm is valued at Rs 56000 and the retiring partnerrsquos share is adjusted

(v) The capital of the new firm is fixed at Rs120000 Prepare Revaluation Account Capital Accounts of the partner and Balance Sheet of the new firm after Nisharsquos retirement Revaluation AccountDr Cr

Particulars Amount Particulars Amount (`Rs) (Rs`)

Provision for Bad debt Ac 500 Building Ac 18000Machinery Ac 4000Profit transferred toCapital Accounts (3 2 2)Rohit 5786Nisha 3857Sunil 3857

13500

18000 18000

Capital Account

Dr Cr

Particulars Rohit Nisha Sunil Particulars Rohit Nisha Sunil (Rs`) (Rs`) (`Rs) (Rs`) (Rs`) (Rs`)

Sunilrsquos Capital ac 9600 mdash 6400 Balance bd 60000 40000 40000Bank - 66143 - General Reserve 9428 6286 6286Balance cd 72000 mdash 48000 Revaluation (Profi 5786 3857 3857 Rohitrsquos Capital Ac mdash 9600 mdash

Sunilrsquos Capital Ac 6400 Bank 6386 - 4257

81600 66143 54400 81600 66143 54400

Balance Sheet as at 31st March 2014

Liabilities Amount Assets Amount (Rs`) (Rs`)

Creditors 25000 Building 108000Bank overdraft 37500 Machinery 36000

Bills Payable 13000 Debtors 30000Capital Less ProvisionRohit 72000 for Bad debts 1500 28500Sunil 48000 120000 Stock 23000

195500 195500

Working Notes (i) (a) Profit sharing ratio is 60000 40000 40000 ie = 3 2 2(b) Gaining Ratio Rohit = 35 ndash 37 = 2135 ndash 1535 = 635Sunil = 25-27 = 1435 ndash 1035 = 435= 635 435= 6 4 = 3 2(c) Nisha Share of Goodwill = Rs 56000 times 27 = Rs16000Share of Goodwill in the gaining ratio by the existing partner ieRohit = Rs16000 times 35 = Rs 9600Sunil = Rs 16000 times 25 = Rs 6400

The journal entry isRohitrsquos Capital Ac Dr 9600Sunilrsquos Capital Ac Dr 6400 To Nisharsquos Capital Ac 16000(Share of Goodwill divided into gaining ratio)

  • 1 Static Friction
  • The frictional force that acts between the surfaces when they are at rest with respect to each other is called Static Friction
    • Static Friction Examples
      • 2 Sliding Friction
        • Examples Of Sliding Friction
          • 3 Rolling Friction
            • Examples Of Rolling Friction
              • Objects and Reasons of the Forest Conservation Act
Page 51:  · Web viewSubject . Topic . Summary . Execution . English 1 . Sounds of animals . Hens –cackle Horses –neigh Lions –roar Owls –hoots Snake –hiss. English 2 . Mother’s

to Stephano and Trinculo

ldquoBe not afeard The isle is full of noises (Act III Sc 2 Lines 135-143)Sounds and sweet airs that give delight and hurt notSometimes a thousand twangling instrumentsWill hum about mine ears and sometime voicesThat if I then had waked after long sleepWill make me sleep again and then in dreamingThe clouds methought would open and show richesReady to drop upon me that when I wakedI cried to dream againrdquo

Caliban - Less Ignoble Than Some OthersCalibanrsquos motive for murder is less dishonourable than that of Antonio and Sebastian They plan to kill Alonso to gain his power and wealth Caliban merely wants revenge and the return of lsquohisrsquo island

Conclusiono Calibanrsquos character is not portrayed very clearly in the play and hence we cannot decide whether he is a poor

savage being grossly maltreated by Prospero or whether he is evil and must therefore be kept in bondage or enslavement

o Caliban is contrasted with Ariel who is a spirit and thus swift and uninterested in physical activitieso Caliban is also contrasted with Prospero who is the all-powerful master of the island and of the destiny of all

those on the islando Caliban is also contrasted with civilized man showing him to be less evil than Antonio and Stephano and less

materialistic than Stephano and Trinculoo Caliban has suffered at the hands of Prospero and he has learnt to curse by listening to Prosperorsquos abuse He

certainly believes that Prospero has deprived him of his birthrighto Finally the character Caliban is thought to be one of Shakespearersquos masterpieces The complexity of the character

is reflected in the large volume of critical discussion that has grown around it

ECO ndash12 Topic-Forms of market

MonopolyMonopoly is a market structure in which there is a single seller there are no close substitutes for the commodity produced by the firm and there are barriers to entry Example Indian Railways which is operated under government of India Monopoly also implies absence of competitionFeatures of Monopoly Monopoly is characterized by1 Single Seller In monopoly there is only one firm producing the product The whole industry consists of this single firm Thus under monopoly there is no distinction between firm and industry Being the only firm there is significant control of the firm over supply and price Thus under monopoly buyers do not have the option of buying the commodity from any other seller They have to buy the product from the firm or they can go without the commodity This fact gives immense control to the monopolist over the market

2No Close Substitute There are no close substitutes of the product produced by the monopolist firm If there are close substitutes of the product in the market it implies presence of more than one firm and hence no monopoly In order to ensure a total of control over the market by the monopolist firm it is assumed that there are no close substitutes of the product

3 No Entry amp Exit Monopoly can only exist when there is strong barriers before a new firm to enter the market In fact once a monopoly firm starts producing the product no other firm can produce the same One reason for this is the ability of the

monopolist to produce the product at a lower cost than any new firm who thinks to enter the market If a new firm who knows that it cannot produce at a lower cost than the monopolist then that firm will never enter the market for fear of losing out in competition Similarly the monopolist who is operating for a long time may be enjoying reputation among its customers and is in a better position to use the situation in its own benefit A new firm has to take long time to achieve this and so may not be interested to enter the market

4 Price Maker Being the single seller of the product the monopolist has full control over the pricing of the product On the other hand if there is a large number of buyers in the market so no single buyer exercises any significant influence over price determination Thus it is a sellerrsquos market So monopoly firm is a price maker

5 Price Discrimination Having considerable control over the market on account of being single seller with no entry of other firms the monopolist can exercise policy of price discrimination it means that the monopolist can sell different quantities of the same product to a consumer at different price or same quantity to different consumers at different prices by adjudging the standard of living of the consumer

6 Shape of Demand Curve Since a monopolist has full control over the price therefore he can sell more by lowering the price This makes the demand curve downward sloping

Subject Ac-12 290620 Topic- retirement Model sumThe Balance Sheet of Rohit Nisha and Sunil who are partners in a firm sharing profits according to their capitals as on 31st March 2014 was as under

Liabilities Amount Assets Amount (Rs) (` Rs)

Creditors 25000 Machinery 40000Bills Payable 13000 Building 90000General Reserve 22000 Debtors 30000Capital Less Provision for Rohit 60000 Bad debts 1000

29000 Nisha 40000 Stocks 23000 Sunil 40000 140000 Cash at Bank 18000

200000 200000

On the date of Balance Sheet Nisha retired from the firm and following adjustments were made(i) Building is appreciated by 20(ii) Provision for bad debts is increased to 5 on Debtors(iii) Machinery is depreciated by 10(iv) Goodwill of the firm is valued at Rs 56000 and the retiring partnerrsquos share is adjusted

(v) The capital of the new firm is fixed at Rs120000 Prepare Revaluation Account Capital Accounts of the partner and Balance Sheet of the new firm after Nisharsquos retirement Revaluation AccountDr Cr

Particulars Amount Particulars Amount (`Rs) (Rs`)

Provision for Bad debt Ac 500 Building Ac 18000Machinery Ac 4000Profit transferred toCapital Accounts (3 2 2)Rohit 5786Nisha 3857Sunil 3857

13500

18000 18000

Capital Account

Dr Cr

Particulars Rohit Nisha Sunil Particulars Rohit Nisha Sunil (Rs`) (Rs`) (`Rs) (Rs`) (Rs`) (Rs`)

Sunilrsquos Capital ac 9600 mdash 6400 Balance bd 60000 40000 40000Bank - 66143 - General Reserve 9428 6286 6286Balance cd 72000 mdash 48000 Revaluation (Profi 5786 3857 3857 Rohitrsquos Capital Ac mdash 9600 mdash

Sunilrsquos Capital Ac 6400 Bank 6386 - 4257

81600 66143 54400 81600 66143 54400

Balance Sheet as at 31st March 2014

Liabilities Amount Assets Amount (Rs`) (Rs`)

Creditors 25000 Building 108000Bank overdraft 37500 Machinery 36000

Bills Payable 13000 Debtors 30000Capital Less ProvisionRohit 72000 for Bad debts 1500 28500Sunil 48000 120000 Stock 23000

195500 195500

Working Notes (i) (a) Profit sharing ratio is 60000 40000 40000 ie = 3 2 2(b) Gaining Ratio Rohit = 35 ndash 37 = 2135 ndash 1535 = 635Sunil = 25-27 = 1435 ndash 1035 = 435= 635 435= 6 4 = 3 2(c) Nisha Share of Goodwill = Rs 56000 times 27 = Rs16000Share of Goodwill in the gaining ratio by the existing partner ieRohit = Rs16000 times 35 = Rs 9600Sunil = Rs 16000 times 25 = Rs 6400

The journal entry isRohitrsquos Capital Ac Dr 9600Sunilrsquos Capital Ac Dr 6400 To Nisharsquos Capital Ac 16000(Share of Goodwill divided into gaining ratio)

  • 1 Static Friction
  • The frictional force that acts between the surfaces when they are at rest with respect to each other is called Static Friction
    • Static Friction Examples
      • 2 Sliding Friction
        • Examples Of Sliding Friction
          • 3 Rolling Friction
            • Examples Of Rolling Friction
              • Objects and Reasons of the Forest Conservation Act
Page 52:  · Web viewSubject . Topic . Summary . Execution . English 1 . Sounds of animals . Hens –cackle Horses –neigh Lions –roar Owls –hoots Snake –hiss. English 2 . Mother’s

monopolist to produce the product at a lower cost than any new firm who thinks to enter the market If a new firm who knows that it cannot produce at a lower cost than the monopolist then that firm will never enter the market for fear of losing out in competition Similarly the monopolist who is operating for a long time may be enjoying reputation among its customers and is in a better position to use the situation in its own benefit A new firm has to take long time to achieve this and so may not be interested to enter the market

4 Price Maker Being the single seller of the product the monopolist has full control over the pricing of the product On the other hand if there is a large number of buyers in the market so no single buyer exercises any significant influence over price determination Thus it is a sellerrsquos market So monopoly firm is a price maker

5 Price Discrimination Having considerable control over the market on account of being single seller with no entry of other firms the monopolist can exercise policy of price discrimination it means that the monopolist can sell different quantities of the same product to a consumer at different price or same quantity to different consumers at different prices by adjudging the standard of living of the consumer

6 Shape of Demand Curve Since a monopolist has full control over the price therefore he can sell more by lowering the price This makes the demand curve downward sloping

Subject Ac-12 290620 Topic- retirement Model sumThe Balance Sheet of Rohit Nisha and Sunil who are partners in a firm sharing profits according to their capitals as on 31st March 2014 was as under

Liabilities Amount Assets Amount (Rs) (` Rs)

Creditors 25000 Machinery 40000Bills Payable 13000 Building 90000General Reserve 22000 Debtors 30000Capital Less Provision for Rohit 60000 Bad debts 1000

29000 Nisha 40000 Stocks 23000 Sunil 40000 140000 Cash at Bank 18000

200000 200000

On the date of Balance Sheet Nisha retired from the firm and following adjustments were made(i) Building is appreciated by 20(ii) Provision for bad debts is increased to 5 on Debtors(iii) Machinery is depreciated by 10(iv) Goodwill of the firm is valued at Rs 56000 and the retiring partnerrsquos share is adjusted

(v) The capital of the new firm is fixed at Rs120000 Prepare Revaluation Account Capital Accounts of the partner and Balance Sheet of the new firm after Nisharsquos retirement Revaluation AccountDr Cr

Particulars Amount Particulars Amount (`Rs) (Rs`)

Provision for Bad debt Ac 500 Building Ac 18000Machinery Ac 4000Profit transferred toCapital Accounts (3 2 2)Rohit 5786Nisha 3857Sunil 3857

13500

18000 18000

Capital Account

Dr Cr

Particulars Rohit Nisha Sunil Particulars Rohit Nisha Sunil (Rs`) (Rs`) (`Rs) (Rs`) (Rs`) (Rs`)

Sunilrsquos Capital ac 9600 mdash 6400 Balance bd 60000 40000 40000Bank - 66143 - General Reserve 9428 6286 6286Balance cd 72000 mdash 48000 Revaluation (Profi 5786 3857 3857 Rohitrsquos Capital Ac mdash 9600 mdash

Sunilrsquos Capital Ac 6400 Bank 6386 - 4257

81600 66143 54400 81600 66143 54400

Balance Sheet as at 31st March 2014

Liabilities Amount Assets Amount (Rs`) (Rs`)

Creditors 25000 Building 108000Bank overdraft 37500 Machinery 36000

Bills Payable 13000 Debtors 30000Capital Less ProvisionRohit 72000 for Bad debts 1500 28500Sunil 48000 120000 Stock 23000

195500 195500

Working Notes (i) (a) Profit sharing ratio is 60000 40000 40000 ie = 3 2 2(b) Gaining Ratio Rohit = 35 ndash 37 = 2135 ndash 1535 = 635Sunil = 25-27 = 1435 ndash 1035 = 435= 635 435= 6 4 = 3 2(c) Nisha Share of Goodwill = Rs 56000 times 27 = Rs16000Share of Goodwill in the gaining ratio by the existing partner ieRohit = Rs16000 times 35 = Rs 9600Sunil = Rs 16000 times 25 = Rs 6400

The journal entry isRohitrsquos Capital Ac Dr 9600Sunilrsquos Capital Ac Dr 6400 To Nisharsquos Capital Ac 16000(Share of Goodwill divided into gaining ratio)

  • 1 Static Friction
  • The frictional force that acts between the surfaces when they are at rest with respect to each other is called Static Friction
    • Static Friction Examples
      • 2 Sliding Friction
        • Examples Of Sliding Friction
          • 3 Rolling Friction
            • Examples Of Rolling Friction
              • Objects and Reasons of the Forest Conservation Act
Page 53:  · Web viewSubject . Topic . Summary . Execution . English 1 . Sounds of animals . Hens –cackle Horses –neigh Lions –roar Owls –hoots Snake –hiss. English 2 . Mother’s

(v) The capital of the new firm is fixed at Rs120000 Prepare Revaluation Account Capital Accounts of the partner and Balance Sheet of the new firm after Nisharsquos retirement Revaluation AccountDr Cr

Particulars Amount Particulars Amount (`Rs) (Rs`)

Provision for Bad debt Ac 500 Building Ac 18000Machinery Ac 4000Profit transferred toCapital Accounts (3 2 2)Rohit 5786Nisha 3857Sunil 3857

13500

18000 18000

Capital Account

Dr Cr

Particulars Rohit Nisha Sunil Particulars Rohit Nisha Sunil (Rs`) (Rs`) (`Rs) (Rs`) (Rs`) (Rs`)

Sunilrsquos Capital ac 9600 mdash 6400 Balance bd 60000 40000 40000Bank - 66143 - General Reserve 9428 6286 6286Balance cd 72000 mdash 48000 Revaluation (Profi 5786 3857 3857 Rohitrsquos Capital Ac mdash 9600 mdash

Sunilrsquos Capital Ac 6400 Bank 6386 - 4257

81600 66143 54400 81600 66143 54400

Balance Sheet as at 31st March 2014

Liabilities Amount Assets Amount (Rs`) (Rs`)

Creditors 25000 Building 108000Bank overdraft 37500 Machinery 36000

Bills Payable 13000 Debtors 30000Capital Less ProvisionRohit 72000 for Bad debts 1500 28500Sunil 48000 120000 Stock 23000

195500 195500

Working Notes (i) (a) Profit sharing ratio is 60000 40000 40000 ie = 3 2 2(b) Gaining Ratio Rohit = 35 ndash 37 = 2135 ndash 1535 = 635Sunil = 25-27 = 1435 ndash 1035 = 435= 635 435= 6 4 = 3 2(c) Nisha Share of Goodwill = Rs 56000 times 27 = Rs16000Share of Goodwill in the gaining ratio by the existing partner ieRohit = Rs16000 times 35 = Rs 9600Sunil = Rs 16000 times 25 = Rs 6400

The journal entry isRohitrsquos Capital Ac Dr 9600Sunilrsquos Capital Ac Dr 6400 To Nisharsquos Capital Ac 16000(Share of Goodwill divided into gaining ratio)

  • 1 Static Friction
  • The frictional force that acts between the surfaces when they are at rest with respect to each other is called Static Friction
    • Static Friction Examples
      • 2 Sliding Friction
        • Examples Of Sliding Friction
          • 3 Rolling Friction
            • Examples Of Rolling Friction
              • Objects and Reasons of the Forest Conservation Act
Page 54:  · Web viewSubject . Topic . Summary . Execution . English 1 . Sounds of animals . Hens –cackle Horses –neigh Lions –roar Owls –hoots Snake –hiss. English 2 . Mother’s

Bills Payable 13000 Debtors 30000Capital Less ProvisionRohit 72000 for Bad debts 1500 28500Sunil 48000 120000 Stock 23000

195500 195500

Working Notes (i) (a) Profit sharing ratio is 60000 40000 40000 ie = 3 2 2(b) Gaining Ratio Rohit = 35 ndash 37 = 2135 ndash 1535 = 635Sunil = 25-27 = 1435 ndash 1035 = 435= 635 435= 6 4 = 3 2(c) Nisha Share of Goodwill = Rs 56000 times 27 = Rs16000Share of Goodwill in the gaining ratio by the existing partner ieRohit = Rs16000 times 35 = Rs 9600Sunil = Rs 16000 times 25 = Rs 6400

The journal entry isRohitrsquos Capital Ac Dr 9600Sunilrsquos Capital Ac Dr 6400 To Nisharsquos Capital Ac 16000(Share of Goodwill divided into gaining ratio)

  • 1 Static Friction
  • The frictional force that acts between the surfaces when they are at rest with respect to each other is called Static Friction
    • Static Friction Examples
      • 2 Sliding Friction
        • Examples Of Sliding Friction
          • 3 Rolling Friction
            • Examples Of Rolling Friction
              • Objects and Reasons of the Forest Conservation Act